You are on page 1of 937

Fourth Edition Fourth Edition

Khattar
The Pearson Guide to
The Pearson Guide to
Quantitative Aptitude
Quantitative

Quantitative Aptitude
for
MBA Entrance Examinations

MBA Entrance Examinations


for

Aptitude for

The Pearson Guide to


Dinesh Khattar
This edition is designed to cater needs of candidates appearing for various MBA entrance examinations. It is a
perfect blend of brand new and previous years’ questions along with their explanatory answers. Each chapter

MBA Entrance
contains concise definitions and explanation of concepts which are further aided by illustrative examples, and
tips and tricks for a quick revision. These tools will aid aspirants to analyse their level of understanding of subjects
that are covered in these examinations.

Salient Features
 Comprehensive coverage of topics for CAT, MAT, XAT, SNAP, IIFT and other entrance examinations.

Examinations
 More than 5,000 multiple-choice questions for practice with varying levels of difficulty.
 Topic-wise presentation of questions with solutions from last 10 years’ papers.
 Concise definitions and explanations of basic principles supported by ample number of examples.
 Special section on short-cut methods in each chapter.
Questions from last 10 years’ papers of
CAT, MAT, NMAT, XAT, SNAP, FMS, IIFT and
Dinesh Khattar, M.Sc. (Mathematics), M.Phil., Ph.D., has impeccable academic credentials. He is a gold medalist
and topper in B.Sc. (Mathematics) and M.Sc. (Mathematics), University of Delhi. Besides teaching mathematics to
undergraduate and postgraduate students for more than 25 years, he is actively involved in many research Fourth other MBA Entrance Examinations
projects as well. He has also presented numerous research papers in prestigious international conferences across Edition
several countries. He has authored 17 books on mathematics with renowned publications.
More than 5,000 MCQs for practice with
varying levels of difficulty

Dinesh Khattar

Size :254x203mm Spine : 37mm ISBN : 9789332546981 Title Sub Title Edition Authors / Editors Name With CD Red Band Territory line URL Price mQuest
The Pearson Guide to

Quantitative Aptitude
for
MBA Entrance Examinations

Fourth Edition

Dinesh Khattar
Head
Department of Mathematics
Kirori Mal College, University of Delhi

FM.indd 1 6/10/2015 2:36:16 PM


No part of this eBook may be used or reproduced in any manner whatsoever without the
publisher’s prior written consent.

Copyright © 2014 Dorling Kindersley (India) Pvt. Ltd.

This eBook may or may not include all assets that were part of the print version. The publisher
reserves the right to remove any material in this eBook at any time.

ISBN: 9789332546981

e-ISBN: 9789332558854

First Impression

Head Office: 7th Floor, Knowledge Boulevard, A-8(A) Sector 62, Noida 201 309, India.
Registered Office: 11 Community Centre, Panchsheel Park, New Delhi 110 017, India.

FM.indd 2 6/10/2015 2:36:16 PM


TABLE OF CONTENTS
Preface to the First Edition iv
Preface to the Fourth Edition v
Chapter 1 Numbers 1.1–1.33
Chapter 2 H.C.F. and L.C.M. of Number 2.1–2.13
Chapter 3 Square Root and Cube Root 3.1–3.11
Chapter 4 Simplification 4.1–4.28
Chapter 5 Percentage 5.1–5.32
Chapter 6 Average 6.1–6.20
Chapter 7 Ratio and Proportion 7.1–7.34
Chapter 8 Partnership 8.1–8.11
Chapter 9 Profit and Loss 9.1–9.36
Chapter 10 Time, Work and Wages 10.1–10.36
Chapter 11 Pipes and Cisterns 11.1–11.22
Chapter 12 Time and Distance 12.1–12.55
Chapter 13 Boats and Streams 13.1–13.14
Chapter 14 Races and Games of Skill 14.1–14.9
Chapter 15 Alligation or Mixture 15.1–15.28
Chapter 16 Problems on Ages 16.1–16.16
Chapter 17 Simple Interest 17.1–17.24
Chapter 18 Compound Interest 18.1–18.22
Chapter 19 Logarithms 19.1–19.18
Chapter 20 Stocks, Shares and Debentures 20.1–20.16
Chapter 21 Discount: True and Banker’s 21.1–21.12
Chapter 22 Binary Number System 22.1–22.7
Chapter 23 Series 23.1–23.28
Chapter 24 Clocks and Calendar 24.1–24.12
Chapter 25 Polynomials 25.1–25.10
Chapter 26 H.C.F. and L.C.M. of Polynomials 26.1–26.8
Chapter 27 Linear Equations 27.1–27.22
Chapter 28 Quadratic Equations 28.1–28.18
Chapter 29 Progressions 29.1–29.32
Chapter 30 Set Theory 30.1–30.22
Chapter 31 Permutations and Combinations 31.1–31.30
Chapter 32 Probability 32.1–32.36
Chapter 33 Mensuration I: Area and Perimeter 33.1–33.66
Chapter 34 Mensuration II: Volume and Surface Area 34.1–34.38
Chapter 35 Trigonometric Ratios 35.1–35.17
Chapter 36 Heights and Distances 36.1–36.22
Chapter 37 Plane Geometry 37.1–37.52
Chapter 38 Co-ordinate Geometry 38.1–38.12

FM.indd 3 6/10/2015 2:36:16 PM


PREFACE TO THE FIRST EDITION
It gives me immense pleasure to present The Pearson Guide to Quantitative Aptitude for MBA Entrance Examinations to
readers. The quantitative aptitude tests are an important part of various competitive examinations, including those for MBA
and MCA courses and for recruitment in banks, railways, defence, and other such services. I have presented the concepts in
a concise manner and covered numerous questions and problems that have appeared in recent competitive examinations. I
have approached each of these questions and problems in logical sequence and provided explanatory answers that would
help to drive home core concepts. This book aims at helping students to enhance their knowledge of mathematics and
equipping them with skills that are required to succeed in any competitive examination.
The book is noteworthy of the following aspects:
☞ Spread across 38 chapters, it sets forth the various patterns followed in competitive examinations.
☞ It contains concise definitions and explanations of basic principles, well augmented with illustrative examples.
☞ It devotes an exclusive section on short-cut methods in each chapter to enable students to deduce the correct
answer with ease
☞ It is replete with questions that provide rigorous practice at varying levels of difficulty.
I am indebted to many teachers at various coaching centres as well as to my students throughout the country, who contributed
to this edition through their constructive criticism and valuable suggestions. I shall appreciate and acknowledge suggestions
and comments, from readers, to enrich the contents of the book. I am thankful to the editorial team at Pearson Education
for their unfailing cooperation.

 Dinesh Khattar

FM.indd 4 6/10/2015 2:36:16 PM


PREFACE TO THE FOURTH EDITION
Gaining entry into India’s premier business schools remains as competitive as ever. It is hoped that The Pearson Guide to
Quantitative Aptitude for MBA Entrance Examinations which you have bought to prepare for the Quantitative Aptitude
Section of these examinations will help you to meet the challenge squarely.
The overwhelming response to the third edition of this book from students and coaches in various preparatory institutions
has not only boosted my confidence but has also become a shaping force for this fourth edition.
In this new edition, we have categorised the problems into two types—‘Difficulty Level-1’ and ‘Difficulty Level-2’
keeping in mind the varying requirements of the average as well as the bright student. We have also added questions from
the latest competitive examinations.
Problem solving trick(s) are included to enhance and sharpen the student’s acumen. These tricks will also serve as useful
aids to arrive at the answers in the quickest possible time and thus help to conquer the examinations without sacrificing
mathematical accuracy. We wish to place on record our sincere thanks to the editorial team at Pearson Education for their
help and suggestions at the time of planning and preparing the manuscript of this book.
Thank you for choosing this book and wish you best of luck.

¾ Dinesh Khattar

FM.indd 5 6/10/2015 2:36:16 PM


This page is intentionally left blank

FM.indd 6 6/10/2015 2:36:16 PM


CHAPTER

Numbers 1
Introduction The set of all natural numbers can be represented by
In Hindu Arabic System, we use ten symbols 0, 1, 2, 3, 4, 5, N = {1, 2, 3, 4, 5, . . .}.
6, 7, 8, 9 called digits to represent any number. This is the
decimal system where we use the numbers 0 to 9. 0 is called Whole Numbers
insignificant digit whereas 1, 2, 3, 4, 5, 6, 7, 8, 9 are called If we include 0 among the natural numbers, then the
significant digits. numbers 0, 1, 2, 3, 4, 5, ... are called whole numbers.
A group of figures denoting a number is called a The set of whole numbers can be represented by
numeral. For a given numeral, we start from extreme right
W = {0, 1, 2, 3, 4, 5, ...}
as unit’s place, ten’s place, hundred’s place and so on.
Clearly, every natural number is a whole number but 0
Illustration 1  We represent the number 309872546 as is a whole number which is not a natural number.
shown below:
Integers
All counting numbers and their negatives including zero are
known as integers.
The set of integers can be represented by
Z or, I = {... – 4, –3, –2, –1, 0, 1, 2, 3, 4, ...}

Positive Integers
We read it as The set I + = {1, 2, 3, 4, ...} is the set of all positive
‘Thirty crores, ninety-eight lakhs, seventy-two integers. Clearly, positive integers and natural numbers
thousands five hundred and forty-six.’ are synonyms.
In this numeral:
The place value of 6 is 6 × 1 = 6. Negative Integers
The place value of 4 is 4 × 10 = 40. The set I – = {–1, –2, –3, ...} is the set of all negative integers.
The place value of 5 is 5 × 100 = 500. 0 is neither positive nor negative.
The place value of 2 is 2 × 1000 = 2000 and so on.
Non-negative Integers
The face value of a digit in a number is the value itself
wherever it may be. The set {0, 1, 2, 3, ... } is the set of all non-negative integers.
Thus, the face value of 7 in the above numeral is 7. The
face value of 6 in the above numeral is 6 and so on. Rational Numbers
p
The numbers of the form , where p and q are integers and
Number System q
4 3 5 0
Natural Numbers q ≠ 0, are known as rational numbers, e.g., , ,  , ,
7 2 8 1
2
Counting numbers 1, 2, 3, 4, 5, . . . are known as natural  , etc.
3
numbers.

Chapter 01.indd 1 6/3/2015 6:21:30 PM


1.2 Chapter 1

The set of all rational numbers is denoted by Q. Note that the sum, difference or, product of a rational

{ p
i.e., Q = x:x = ; p, q ≠ I, q ≠ 0
q } and irrational number is irrational, e.g., 3 + 2 , 4 – 3 ,
2
– 5 , 4 3 , – 7 5 are all irrational.
a 3
Since every natural number ‘a’ can be written as s,
1 Even Numbers
0
so it is a rational number. Since 0 can be written as and All those numbers which are exactly divisible by 2 are called
a 1
even numbers, e.g., 2, 6, 8, 10, etc., are even numbers.
every non-zero integer ‘a’ can be written as , so it is also
1
a rational number.
Odd Numbers
Every rational number has a peculiar characteristic that
when expressed in decimal form is expressible either in All those numbers which are not exactly divisible by 2 are
terminating decimals or, non-terminating repeating decimals. called odd numbers, e.g., 1, 3, 5, 7, etc., are odd numbers.
1 1 22
For example, = 0.2, = 0.333 ..., = 3.1428714287, Prime Numbers
5 3 7
8 A natural number other than 1 is a prime number if it is
= 0.181818 ..., etc. divisible by 1 and itself only.
44
For example, each of the numbers 2, 3, 5, 7, etc., are
The recurring decimals have been given a short prime numbers.
notation as
Composite Numbers
0.333... = 0.3
Natural numbers greater than 1 which are not prime are
4.1555...= 4.05 known as composite numbers.
0.323232... = 0.32. For example, each of the numbers 4, 6, 8, 9, 12, etc., are
Irrational Numbers composite numbers.
Those numbers which when expressed in decimal form are
neither terminating nor repeating decimals are known as Notes:

irrational numbers, e.g., 2, 3, 5, p, etc. 1. The number 1 is neither a prime number nor a composite
number.
22 22
Note that the exact value of p is not . is rational 2. The number 2 is the only even number which is prime.
7 7
22 3. Prime numbers up to 100 are:
while p is irrational number. is approximate value of p.
7 2, 3, 5, 7, 11, 13, 17, 19, 23, 29, 31, 37, 41, 43, 47, 53,
Similarly, 3.14 is not an exact value of it. 59, 61, 67, 71, 73, 79, 83, 89, 97, i.e., 25 prime numbers
between 1 and 100.
Real Numbers 4. Two numbers which have only 1 as the common factor
The rational and irrational numbers combined together are are called co-primes or, relatively prime to each other,
13 2 3 e.g., 3 and 5 are co-primes.
called real numbers, e.g., , ,  , 3, 4  2, etc. are
21 5 7 Note that the numbers which are relatively prime
real numbers. need not necessarily be prime numbers, e.g., 16 and 17 are
The set of all real numbers is denoted by R. relatively prime although 16 is not a prime number.

Addition and Subtraction (Short-cut Methods)

The method is best illustrated with the help of following To obtain 1 at unit’s place add 9 to 5 4321
examples: make 31. In the answer, write 9 at 9 87 6
Illustration 2 54321 – (9876 + 8967 + 7689) = ? unit’s place and carry over 3. 8 96 7
Step 1 Add 1st column: Step 2 Add 2nd column: 7689
6 + 7 + 9 = 22 3 + 7 + 6 + 8 = 24 2 7 7 8 9

Chapter 01.indd 2 6/3/2015 6:21:30 PM


Numbers 1.3

To obtain 2 at ten’s place, add 8 to make 32. In the To obtain 4 at thousand’s place add 7 to make 34.
answer, write 8 at ten’s place and carry over 3. In the answer, write 7 at thousand’s place and carry
Step 3 Add 3rd column: over 3.
3 + 8 + 9 + 6 = 26
Step 5 Add 5th column:
To obtain 3 at hundred’s place, add 7 to make 33.
In the answer, write 7 at hundred’s place and carry To obtain 5 at ten-thousand’s place, add 2 to
over 3. make 5. In the answer, write 2 at ten-thousand’s
place.
Step 4 Add 4th column:
3 + 9 + 8 + 7 = 27 \ 54321 – (9876 + 8967 + 7689) = 27789

MultiPlicAtion (Short-cut MethodS)

1. Multiplication of a given number by 9, 99, 999, 3. Multiplication of a given number by 15, 25, 35,
etc., that is by 10n – 1. etc.
Method: Put as many zeros to the right of the Method: Double the multiplier and then multiply the
multiplicant as there are nines in the multiplier and multiplicant by this new number and finally divide
from the result subtract the multiplicant and get the the product by 2.
answer.
Illustration 5 Multiply
Illustration 3 Multiply (a) 7054 × 15 (b) 3897 × 25
(a) 3893 by 99 (b) 4327 by 999 (c) 4563 × 35
(c) 5863 by 9999 Solution:
1
(a) 7054 × 15 = (7054 × 30)
Solution: 2
(a) 3893 × 99 = 389300 – 3893 = 385407 1
= (211620) = 105810
(b) 4327 × 999 = 4327000 – 4327 = 4322673 2
(c) 5863 × 9999 = 58630000 – 5863 = 58624137 1 1
(b) 3897 × 25 = (3897 × 50) = (194850)
2 2
= 97425
2. Multiplication of a given number by 11, 101,
1 1
(c) 4536 × 35 = (4563 × 70) = (319410)
1001, etc., that is, by 10n + 1. 2 2
Method: Place n zeros to the right of the multiplicant = 159705
and then add the multiplicant to the number so
obtained. 4. Multiplication of a given number by 5, 25, 125,
625, etc., that is, by a number which is some
power of 5.
Illustration 4 Multiply
Method: Place as many zeros to the right of the
(a) 4782 × 11 (b) 9836 × 101 multiplicant as is the power of 5 in the multiplier,
(c) 6538 × 1001 then divide the number so obtained by 2 raised to the
same power as is the power of 5.
Solution:
(a) 4782 × 11 = 47820 + 4782 = 52602
Illustration 6 Multiply
(b) 9836 × 101 = 983600 + 9836 = 993436 (a) 3982 × 5 (b) 4739 × 25
(c) 6538 × 1001 = 6538000 + 6538 = 6544538 (c) 7894 × 125 (d) 4863 × 625

Chapter 01.indd 3 6/3/2015 6:21:31 PM


1.4 Chapter 1

Solution: dIStrIbutIVe LAWS


(a) 3982 × 2 =
39820
= 19910 For any three numbers a, b, c, we have
2 (a) a × b + a × c = a × (b + c)
473900 473900 (b) a × b – a × c = a × (b – c)
(b) 4739 × 25 = = = 118475
2 2
4 Illustration 7 438 × 637 + 438 × 367 = ?
7894000 7894000 Solution: 438 × 637 + 438 × 367 = 438 × (637 + 367)
(c) 7894 × 125 = =
2 3
8 = 438 × 1000 = 438000
= 986750
Illustration 8 674 × 832 – 674 × 632 = ?
48630000 48630000
(d) 4863 × 625 = =
24 16 Solution: 674 × 832 – 674 × 632 = 674 × (832 – 632)
= 3039375 = 674 × 200 = 134800

SQuAreS (Short-cut MethodS)

(d) (1005)2 = (1005 – 5) (1005 + 5) + 52


1. To square any number ending with 5. = 1010000 + 25 = 1010025
Method: (A5)2 = A(A + 1)/25 dIVISIon
Division is repeated subtraction.
Illustration 9
For example, when we divide 63289 by 43, it means 43
(a) (25)2 = 2 (2 + 1)/25 = 6/25 = 625
can be repeatedly subtracted 1471 times from 63289 and the
(b) (45)2 = 4 (4 + 1)/25 = 20/25 = 2025 remainder 36 is left.
((c)) (85)2 = 8 (8 + 1)/25 = 72/25 = 7225

2. To square a number in which every digit is one.


Method: Count the number of digits in the given
number and start writing numbers in ascending order
from one to this number and then in descending
order up to one.

Illustration 10
(a) 112 = 121 (b) 1112 = 12321
(c) 11112 = 1234321
Dividend = (Divisor × Quotient ) + Remainder
(d) 2222 = 22 (111)2 = 4 (12321) = 49284
( ) 33332 = 32 (1111)2 = 9 (1234321) = 11108889
(e) Dividend  Remainder
or, Divisor =
Quotient
3. To square a number which is nearer to 10x.
Method: Use the formula:
Illustration 12 On dividing 7865321 by a certain number, the
x2 = (x2 – y2) + y2 = (x + y)(x – y) + y2
quotient is 33612 and the remainder is 113. Find the divisor.
Illustration 11 Dividend  Remainder
(a) (97)2 = (97 + 3) (97 – 3) + 32 Solution: Divisor =
Quotient
= 9400 + 9 = 9409
(b) (102) = (102 – 2) (102 + 2) + 22
2 7865321  113 7865208
= = = 234
= 10400 + 4 = 10404 33612 33612
(c) (994)2 = (994 + 6) (994 – 6) + 62 Illustration 13 A number when divided by 315 leaves
= 988000 + 36 = 988036 remainder 46 and the value of quotient is 7. Find the number.

Chapter 01.indd 4 6/3/2015 6:21:32 PM


Numbers 1.5

Solution: Number = (Divisor × Quotient) + Remainder


3. Divisibility by 4: A number is divisible by 4 if the
= (315 × 7) + 46 = 2205 + 46 number formed by the last two digits (ten’s digit
= 2251 and unit’s digit) is divisible by 4 or are both zero.
Illustration 14 Find the least number of 5 digits which is For example, the number 2616 is divisible by 4
exactly divisible by 632. since 16 is divisible by 4.
4. Divisibility by 5: A number is divisible by 5 if
Solution: The least number of 5 digits is 10000. Dividing
the unit’s digit in the number is 0 or 5.
this number by 632, the remainder is 520. So, the required
For example, 13520, 7805, 640, 745, etc., are all
number = 10000 + (632 + 520) = 10112.
divisible by 5.
5. Divisibility by 6: A number is divisible by 6 if the
number is even and sum of its digits is divisible by 3.
For example, the number 4518 is divisible by 6
since it is even and sum of its digits 4 + 5 + 1 + 8
= 18 is divisible by 3.
Illustration 15 Find the greatest number of 5 digits which 6. Divisibility by 7: The unit digit of the given
is exactly divisible by 463. number is doubled and then it is subtracted from
Solution: The greatest number of 5 digits is 99999. Dividing the number obtained after omitting the unit digit.
this number by 463, the remainder is 454. So, the required If the remainder is divisible by 7, then the given
number = 99999 – 454 = 99545. number is also divisible by 7.
For example, consider the number 448. On doubling
the unit digit 8 of 448 we get 16. Then, 44 – 16 = 28.
Since 28 is divisible by 7, 448 is divisible by 7.
7. Divisibility by 8: A number is divisible by 8, if the
number formed by the last 3 digits is divisible by 8.
For example, the number 41784 is divisible by 8
as the number formed by last three digits, i.e., 784
Illustration 16 Find the number nearest to 13700 which is is divisible by 8.
exactly divisible by 235. 8. Divisibility by 9: A number is divisible by 9 if
Solution: On dividing the number 13700 by 235, the
the sum of its digits is divisible by 9.
remainder is 70. Therefore, the nearest number to 13700,
which is exactly divisible by 235 = 13700 – 70 = 13630. For example, the number 19044 is divisible by
9 as the sum of its digits 1 + 9 + 0 + 4 + 4 = 18
is divisible by 9.
9. Divisibility by 10: A number is divisible by 10, if
it ends in zero.
For example, the last digit of 580 is zero, therefore,
580 is divisible by 10.
10. Divisibility by 11: A number is divisible by 11, if
TESTS OF DIVISIBILITY
the difference of the sum of the digits at odd places
1. Divisibility by 2: A number is divisible by 2 if the and sum of the digits at even places is either zero or
unit’s digit is zero or divisible by 2. divisible by 11.
For example, 4, 12, 30, 18, 102, etc., are all For example, in the number 38797, the sum of the
divisible by 2. digits at odd places is 3 + 7 + 7 = 17 and the sum of
2. Divisibility by 3: A number is divisible by 3 if the the digits at even places is 8 + 9 = 17. The difference
sum of digits in the number is divisible by 3. is 17 – 17 = 0, so the number is divisible by 11.
For example, the number 3792 is divisible by 3 11. Divisibility by 12: A number is divisible by 12 if
since 3 + 7 + 9 + 2 = 21, which is divisible by 3. it is divisible by 3 and 4.

Chapter 01.indd 5 6/3/2015 6:21:33 PM


1.6 Chapter 1

12. Divisibility by 18: An even number satisfying 14. Divisibility by 88: A number is divisible by 88 if
the divisibility test of 9 is divisible by 18. it is divisible by 11 and 8.
13. Divisibility by 25: A number is divisible by 25 15. Divisibility by 125: A number is divisible by 125
if the number formed by the last two digits is if the number formed by the last three digits is
divisible by 25 or the last two digits are zero. divisible by 125 or the last three digits are zero.
For example, the number 13675 is divisible by 25 For example, the number 5250 is divisible by 125
as the number formed by the last two digits is 75, as 250 is divisible by 125.
which is divisible by 25.

SoMe uSeful Short-cut MethodS

Illustration 20 A number on being divided by 10 and 11


1. Test to find whether a given number is a prime successively leaves the remainders 5 and 7, respectively.
Step 1 Select a least positive integer n such that Find the remainder when the same number is divided by 110.
n2 > given number. Solution: The required remainder = d1 × r2 + r1
Step 2 Test the divisibility of given number by = 10 × 7 + 5 = 75
every prime number less than n.
Step 3 The given number is prime only if it is not 4. To find the number of numbers divisible by a
divisible by any of these primes. certain integer.
The method is best illustrated with the help of
Illustration 17 Investigate whether 571 is a prime number. following example.
Solution: Since (23)2 = 529 < 571 and (24)2 = 576 > 571
\ n = 24 Illustration 21 How many numbers up to 532 are divisible
Prime numbers less than 24 are 2, 3, 5, 7, 11, 13, 17, by 15?
19, 23. Since 24 is divisible by 2, 571 is not a prime number
Solution: We divide 532 by 15.
Illustration 18 Investigate whether 923 is a prime number.
532 = 35 × 15 + 7
Solution: Since (30)2 = 900 < 923 and (31)2 = 961 > 923
The quotient obtained is the required number of
\ n = 31
numbers. Thus, there are 35 such numbers
Prime numbers less than 31 are 2, 3, 5, 7, 11, 13, 17,
19, 23, 29. Since 923 is not divisible by any of these primes, Illustration 22 How many numbers up to 300 are divisible
therefore 923 is a prime number by 5 and 7 together?
Solution: L.C.M. of 5 and 7 = 35
2. The least number which when divided by d1, d2 and We divide 300 by 35
d3 leaves the remainders r1, r2 and r3, respectively,
300 = 8 × 35 + 20
such that (d1 – r1) = (d2 – r2) = (d3 – r3) is (LCM of
Thus, there are 8 such numbers.
d1, d2 and d3) – (d1 – r1) or (d2 – r2) or (d3 – r3).

Illustration 19 Find the least number which when divided by 5. Two numbers when divided by a certain divisor
9, 10 and 15 leaves the remainders 4, 5 and 10, respectively. give remainders r1 and r2. When their sum is
Solution: Here 9 – 4 = 10 – 5 = 15 – 10 = 5 divided by the same divisor, the remainder is r3.
Also, L.C.M. (9, 10, 15) = 90 The divisor is given by r1 + r2 – r3.
\ the required least number = 90 – 5 = 85
Illustration 23 Two numbers when divided by a certain
3. A number on being divided by d1 and d2 divisor give remainders 473 and 298, respectively. When
successively leaves the remainders r1 and r2, their sum is divided by the same divisor, the remainder is
respectively. If the number is divided by d1 × d2, 236. Find the divisor.
then the remainder is (d1 × r2 + r1).
Solution: The required divisor = 437 + 298 – 236 = 499

Chapter 01.indd 6 6/3/2015 6:21:34 PM


Numbers 1.7

Practice Exercises

Difficulty Level-1
(Based on Memory)

1. A student was asked to divide a number by 6 and add 12 (a) 248 (b) 348
to the quotient. He, however, first added 12 to the number (c) 148 (d) 448
and then divided it by 6, getting 112 as the answer. The [Based on MAT, 2002]
correct answer should have been:
10.
For every positive real number:
(a) 122 (b) 118
(c) 114 (d) 124  x   x + 1  = ...
2 +  2 
[Based on MAT, 2004]    
2. Which of the following integers is the square of an integer where ([]) is the greatest integer function.
for every integer n? (a) 0 (b) 1
(a) n2 + l (b) n2 + n (c) [x + 1] (d) [x]
(c) n + 2n (d) n2 + 2n + 1
2
[Based on MAT, 2002]
[Based on MAT, 2004]
11. How many 5-digit multiples of 11 are there, if the five
3. Given that N = (521)125 × (125)521, find the last two digits digits are 3, 4, 5, 6 and 7 in the same order?
of N.
(a) 12 (b) 13
(a) 75 (b) 25
(c) 10 (d) None of these
(c) 45 (d) None of these
[Based on MAT, 2002]
4. The sum of the digits of a 3-digit number is subtracted
12. The smallest number by which 3600 can be divided to
from the number. The resulting number is always:
make it a perfect cube is:
(a) Divisible by 6 (b) Not divisible by 6
(a) 9 (b) 50
(c) Divisible by 9 (d) Not divisible by 9
(c) 300 (d) 450
[Based on MAT, 2004]
[Based on MAT, 2002]
5. The least number that must be subtracted from each of the
numbers 14, 17, 34 and 42, so that the remainders may be 13. The least number having four digits which is a perfect
proportional, is: square is:
(a) 0 (b) 1 (a) 1004 (b) 1016
(c) 2 (d) 7 (c) 1036 (d) None of these
[Based on MAT, 2003] [Based on MAT, 2002]
84
6. The highest power of 5 that is contained in 125 125
– 14.
The remainder when 7 is divided by 342 is:
2525 is: (a) 0 (b) 1
(a) 25 (b) 50 (c) 49 (d) 341
[Based on MAT, 2001]
(c) 75 (d) 125
7. Of the 120 people in the room, three-fifths are women. If two- 15.
A 2-digit number is such that the product of the digits
thirds of the people are married, then what is the maximum is 14. When 45 is added to the number, then the digits
number of women in the room who could be unmarried? interchange their places. Find the number.
(a) 40 (b) 20 (a) 72 (b) 27
(c) 30 (d) 60 (c) 37 (d) 14
[Based on MAT, 2003] [Based on MAT, 2001]
2/3 1/3 3 2
8. If x = 2 + 2 +2 , then the value of x – 6x + 6x is: 16. In a division sum, the divisor is 12 times the quotient and
(a) 3 (b) 2 5 times the remainder. If the remainder is 48, then what is
(c) 1 (d) None of these the dividend?
[Based on MAT, 2002] (a) 240 (b) 576
9.
A number of three digits in scale 7 when expressed in (c) 4800 (d) 4848
scale 9 has its digits reversed in order. The number is: [Based on IIFT, 2003]

Chapter 01.indd 7 6/3/2015 6:21:34 PM


1.8 Chapter 1

17. Which of the following integers has the most divisors? 26.
One of a group of swans, 7/2 times the square root of the
(a) 88 (b) 91 number are playing on the shore of the pond. The two
remaining are inside the pond. What is the total number of
(c) 99 (d) 101
swans?
[Based on SCMHRD Ent. Exam., 2003]
(a) 10 (b) 14
18. In which of the following pairs of numbers, it is true that (c) 12 (d) 16
their sum is 11 times their product?
[Based on MAT (Dec), 2008]
(a) 1, 1/11 (b) 1, 1/10 27.
A girl counted in the following way on the fingers of her
(c) 1, 1/12 (d) 1, 10 left hand; she started by calling the thumb 1, the index
 [Based on SCMHRD, 2002] finger 2, the middle finger 3, the ring finger 4, the little
finger 5 and then reversed direction calling the ring figure
19. If m, n, o, p and q are integers, then m (n + o) (p – q) must 6, the middle finger 7 and so on. She counted upto 1994.
be even when which of the following is even? She ended counting on which finger?
(a) m + n (b) n + p (a) The middle finger (b) The index finger
(c) m (d) p (c) The thumb (d) The ring finger
[Based on REC Tiruchirapalli, 2002] [Based on MAT (Sept), 2008]
20.
Which of the following numbers is exactly divisible by 28.
An Army Commander wishing to draw up his 5180 men
99? in the form of a solid square found that he had 4 men less.
(a) 114345 (b) 135792 If he could get four more men and form the solid square,
(c) 3572404 (d) 913464 the number of men in the front row is:
[Based on MAT, 2005] (a) 72 (b) 68
(c) 78 (d) 82
21.
Of the three numbers, the sum of the first two is 45; the
sum of the second and the third is 55 and the sum of the [Based on MAT (Feb), 2008]
third and thrice the first is 90. The third number is: 29.
To win an election, a candidate needs three-fourths of
(a) 20 (b) 25 the votes cast. If after two-thirds of the votes have been
(c) 30 (d) 35 counted, a candidate has 5/6 of what he needs, then what
part of the remaining votes does he still need?
[Based on MAT, 2005]
(a) 1/8 (b) 7/12
22.
Two times a 2-digit number is 9 times the number
(c) 1/4 (d) 3/8
obtained by reversing the digits and sum of the digits is 9.
[Based on MAT (Feb), 2008]
The number is:
(a) 72 (b) 54 30.
The sum of the place values of 3 in the number 503535 is:
(c) 63 (d) 81 (a) 3300 (b) 0.6
[Based on MAT (Feb), 2010] (c) 60 (d) 3030
[Based on MAT (Feb), 2008]
23.
The sum of two numbers, one of which is one-third of the
other is 36. The smaller number is: 31.
Find the whole number which when increased by 20 is
equal to one-sixth times the new number:
(a) 6 (b) 7
(a) 7 (b) 5
(c) 8 (d) 9
(c) 3 (d) 4
[Based on MAT (Sept), 2009] [Based on MAT (Sept), 2007]
24.
If such numbers which are divisible by 5 and also those 32.
A number when divided by 765 leaves a remainder 42.
which have 5 as one of the digits are eliminated from the What will be the remainder if the number is divided by 17?
numbers 1 to 60, how many numbers would remain? (a) 8 (b) 5
(a) 40 (b) 47 (c) 7 (d) 6
(c) 53 (d) 45 [Based on MAT (Sept), 2007]
[Based on MAT (May), 2009] 33.
After being set up, a company manufactured 6000 scooters
in the third year and 7000 scooters in the seventh year.
25.
How many numbers are there between 500 and 600 in
Assuming that the production increases uniformly by a fixed
which 9 occurs only once?
number every year, what is the production in the tenth year?
(a) 19 (b) 18 (a) 7850 (b) 7650
(c) 20 (d) 21 (c) 7750 (d) 7950
[Based on MAT (Feb), 2009] [Based on MAT (May), 2006]

Chapter 01.indd 8 6/3/2015 6:21:34 PM


Numbers 1.9

34.
In a class, the number of girls is one less than the number 42.
The smallest perfect square that is divisible by 7! is:
of the boys. If the product of the number of boys and that (a) 44100 (b) 176400
of girls is 272, then the number of girls in the class is:
(c) 705600 (d) 19600
(a) 15 (b) 14 [Based on IIFT, 2010]
(c) 16 (d) 17
43.
How many two-digit numbers have their square as 1 more
[Based on MAT (Feb), 2011]
than a multiple of 24?
35.
A number of friends decided to go on a picnic and planned
(a) 30 (b) 31
to spend `96 on eatables. Four of them, did not turn up. As a
consequence, the remaining ones had to contribute `4 each (c) 32 (d) 29
extra. The number of those who attended the picnic was: 44. If x, y and z are consecutive negative integers, and if x >
(a) 8 (b) 16 y > z, which of the following must be a positive integer?
(c) 12 (d) 24 (a) x – yz (b) xyz
[Based on MAT (Feb), 2006] (c) x + y + z (d) (x – y)(y – z)
36.
A box of light bulbs contains 24 bulbs. A worker replaces
 [Based on MHT-CET, MBA, 2010]
17 bulbs in the shipping department and 13 bulbs in the
accounting department. How many boxes of bulbs did the 45. Sum of three numbers is 132. First number is twice the
worker use? second and third number is one-third of the first. Find the
1 second number:
(a) 1 (b) 1
4 (a) 18 (b) 36
3 (c) 20 (d) 16
(c) 1 (d) 2
4
[Based on MAT (Sept), 2003] 46. The number obtained by interchanging the two digits of a
37.
Of the numbers 7, 9, 11, 13, 29, 33 how many are prime two digit number is less than the original number by 27. If
numbers? the difference between the two digits of the number is 3,
what is the original number?
(a) 3 (b) 4
(c) 5 (d) 6 (a) 74 (b) 63
[Based on MAT, 1998] (c) 85 (d) Cannot be determined
38. A three-digit number is selected such that it contains no  [Based on IRMA, 2009]
zeros. Now this three-digit number is written beside itself
47. In certain games, each player scores either 2 points or 5
to form the six-digit number. Its factor is:
points. If n players score 2 points and m players score 5
(a) 5 (b) 11 points and the total number of points scored is 50, what is
(c) 4 (d) None of these the least possible positive difference between n and m?
39. The sum of the digits of a three-digit number is 16. If the (a) 5 (b) 3
ten’s digit of the number is three times the unit’s digit and
(c) 1 (d) 7
the unit’s digit is one-fourth of the hundredth digit, then
[Based on NMAT, 2005]
what is the number?
(a) 446 (b) 561 48. What least number must be added to 7231 so that
the resulting number is exactly divisible by 5 and 9
(c) 682 (d) 862
together?
[Based on MAT, 1998]
40.
If one-third of a number is 3 more than one-fourth of the (a) 20 (b) 18
number, then what is the number? (c) 14 (d) 16
(a) 18 (b) 24 49. If the digit in the unit’s place of a two-digit number is
(c) 30 (d) 36 halved and the digit in the ten’s place is doubled, the
[Based on MAT, 1998] number thus obtained is equal to the number obtained
41.
What is the least fraction which when added to or  sub­ by interchanging the digits. Which of the following is
29 15 definitely true?
tracted from + will make the result a whole number?
12 16 (a) Digit in the unit’s place and the ten’s place are equal.
21 31 (b) Digit in the unit’s place is twice the digit in the ten’s
(a) (b)
38 38 place.
31 17 (c) Sum of the digits is a two-digit number.
(c) (d)
48 48 (d) Digit in the unit’s place is half of the digit in the ten’s
[Based on MAT, 1999] place. [Based on NMAT, 2005]

Chapter 01.indd 9 6/3/2015 6:21:35 PM


1.10 Chapter 1

50. If m and n are two integers such that m × n = 64, which of If a is a positive integer and if the unit’s digit of a2 is 9 and
57.
the following cannot be the value of m + n? (a + 1)2 is 4, what is the unit’s digit of (a + 2)2?
(a) 20 (b) 65 (a) 1 (b) 3
(c) 16 (d) 35 [Based on ATMA, 2005] (c) 5 (d) 14
51. If the quotient is positive, which of the following must be [Based on ATMA, 2008]
true?
58.
When 10 is divided by the positive integer n, the remainder
(a) a > 0 (b) b > 0
is n – 4. Which of the following could be the value of n?
(c) a – b > 0 (d) ab > 0
(a) 3 (b) 4
[Based on ATMA, 2006]
(c) 7 (d) 12
52.
If a positive integers n is divisible by both 5 and 7, n must
also be divisible by which of the following? [Based on ATMA, 2008]
109 109
I. 12 II. 35 59. 4 +6 is divided by 25, the remainder is:
III. 70 (a) 20 (b) 10
(a) None (b) II only (c) 5 (d) 0
(c) I and II (d) II and III [Based on JMET, 2006]
[Based on ATMA, 2006]
60. What is the digit in the units place of 10251?
53.
A number consists of there digits whose sum is 10. The
(a) 2 (b) 4
middle digit is equal to sum of the other two and the
number will be increased by 99, if the final digit and the (c) 6 (d) 8
third digit are interchanged. The digit in the hundreds [Based on JMET, 2006]
place is: 61. Find the least number which must be subtracted from 9269
(a) 3 (b) 5 so that resulting number is exactly divisible by 73:
(c) 4 (d) 2 (a) 17 (b) 57
[Based on ATMA, 2006] (c) 71 (d) 63
54.
If x and y are any natural numbers, then which of the
following is an odd number? 62. Find the least number which must be added to 15463 so
that the resulting number is exactly divisible by 107?
(a) xy + yx (x – y) (xy + x) (b) xy (x + y) (xy + x)
(a) 52 (b) 71
(c) yx (x2 – y) (xy – x) (d) None of these
(c) 55 (d) 19
[Based on ATMA, 2008]
63. If a, b, c, d and e are real numbers such that a + b < c + d,
55.
a, b and c are positive integers divisible by 5, 3 and 12
b + c < d + e, c + d < e + a and d + e < a + b, then:
respectively and p is a two-digit prime number, then
which of the following statement (s) is/are TRUE? (a) the largest number is a and the smallest is b.
I. Product of abcp is zero. (b) the largest number is a and the smallest is c.
a + b + c + p is odd.
II. (c) the largest number is e and the smallest is c.
III. (b2 + c2) – (p2 – a2) is odd. (d) the largest number is c and the smallest is b.
a (p – c) + a (c + b) is divisible by 5.
IV. [Based on GBO, Delhi University, 2011]
x+y y+z
64.
Let 2 = 10, 2 = 20 and 2z + x = 30 where x, y and z
(a) I and IV only (b) II and III only
are any three real numbers. The value of 2x is:
(c) II and IV only (d) IV only
3
[Based on ATMA, 2008] (a) (b) 15
2
56.
If x, y and z and positive integers such that x is a factor of
y and x is a multiple of z, which of the following is NOT 6
(c) (d) 15
necessarily an integer? 2
xy y+z [Based on GBO, Delhi University, 2011]
(a) (b)
x 65. What is the number just more than 5000 which is exactly
yz x+ y divisible by 73?
(c) (d)
x z (a) 5001 (b) 5009
[Based on ATMA, 2008] (c) 5037 (d) 5027

Chapter 01.indd 10 6/3/2015 6:21:35 PM


Numbers 1.11

66. The sum of two numbers is 100 and their difference is 37. 69. The number of five figures to be added to a number of
The difference of their squares is: four fives to obtain the least number of six figures exactly
(a) 37 (b) 100 divisible by 357 is:
(c) 63 (d) 3700 (a) 94762 (b) 94802
(c) 94485 (d) None of these
67.
The number of times 79 be subtracted from 50000, so that
the remainder be 43759 is:
70. The least value to be given to * so that the number
(a) 69 (b) 79 5 * 3457 is divisible by 11 is:
(c) 59 (d) None of these (a) 2 (b) 3
68. The nearest figure to 58701 which is divisible by 567 is: (c) 0 (d) 4
(a) 58968 (b) 58434
(c) 58401 (d) None of these

Difficulty Level-2
(Based on Memory)

1.
Let a, b, c, d be the four integers such that a + b + c + d = 5.
If a, a + 2 and a + 4 are prime numbers, then the number
4m + 1, where m is a positive integer. Given m, which one of possible solutions for a is:
of the following is necessarily true? (a) One (b) Two
(a) The minimum possible value of (c) Three (d) None of these
a2 + b2 + c2 + d 2 is 4m2 – 2m + 1 [Based on CAT, 2004]
(b) The minimum possible value of What is the remainder when 496 is divided by 6?
6.
a2 + b2 + c2 + d 2 is 4m2 + 2m + 1 (a) 0 (b) 2
(c) The maximum possible value of (c) 3 (d) 4
a2 + b2 + c2 + d 2 is 4m2 – 2m + 1 [Based on CAT, 2004]
163
(d) The maximum possible value of 7. The remainder when 5 is divided by 1000 is:
2 2 2 2 2 (a) 125 (b) 625
a + b + c + d  is 4m + 2m + 1.
[Based on CAT, 2003] (c) 25 (d) None of these
2.
How many three-digit positive integers with digits x, y
and z in the hundred’s, ten’s and unit’s place, respectively, 8.
If the sum of n consecutive integers is 0, which of the
exist such that x < y, z < y and x ≠ 0? following must be true?
(a) 245 (b) 285 I. n is an even number.
(c) 240 (d) 320 II. n is an odd number.
[Based on CAT, 2003] III. The average of the n integers is 0.
3.
The number of positive integers n in the range 12 ≤ (a) I only (b) II only
n ≤ 40 such that the product (n – 1) (n – 2) . . . 3 × 2 × 1 is
(c) III only (d) II and III
not divisible by n is:
(a) 5 (b) 7 9. A player holds 13 cards of four suits of which seven are
black and six are red. There are twice as many diamonds
(c) 13 (d) 14
as spades and twice as many hearts as diamonds. How
4.
Let x and y be positive integers such that x is prime and y many clubs does he hold?
is composite. Then,
(a) 4 (b) 5
(a) y – x cannot be an even integer.
(c) 6 (d) 7
(b) xy cannot be an even integer.
[Based on FMS Delhi, 2004]
( x  y)
(c) cannot be an even integer. 10. In three coloured boxes: red, green and blue, 108 balls
x are placed. There are twice as many in the green and red
(d) None of these [Based on CAT, 2004] boxes combined as they are in the blue box and twice

Chapter 01.indd 11 6/3/2015 6:21:35 PM


1.12 Chapter 1

as many in the blue box as they are in the red box. How divide evenly all the nine numbers, i.e., 111, 222, 333,
many balls are there in the green box? 444, 555, 666, 777, 888, 999?’ Tanu immediately gave the
(a) 18 (b) 36 desired answer. It was:
(c) 45 (d) None of these (a) 7, 37, 111 (b) 3, 37, 111
[Based on FMS Delhi, 2004] (c) 9, 37, 111 (d) 9, 13, 111

11. If a = 1 , b = 2 , c = 3 ... z = 2627. In the product of all the


2 3 4 18. The smallest prime number that is the fifth term of
alphabets, how many zeros exist in the end? an increasing arithmetic sequence for which all four
(a) 100 (b) 104 preceding terms are also prime:
(a) 17 (b) 37
(c) 80 (d) 106
[Based on FMS Delhi, 2004] (c) 29 (d) 53

12. The unit’s digit of a two-digit number is one more than the 19. When 1012 – 1 is divided by 111, the quotient is:
digit at ten’s place. If the number is more than five times (a) 9009009 (b) 9000009
of the sum of the digits of the number, then find the sum (c) 9009009009 (d) 9000000009
of all such possible numbers:
20. A number N is defined as the addition of 4 different
(a) 246 (b) 275 integers. Each of the four numbers gives a remainder
(c) 290 (d) 301 zero when divided by four. The first of the four numbers
[Based on FMS Delhi, 2004] defined as A is known to be as 461. The other three
numbers arranged in the increasing order and defined
13. Let 20 × 21 × 22 × ... × 30 = A. If A is divisible by 10x, as B, C and D are each 4 times more than the previous
then find the maximum value of x: number. Thus, the number B = 4 × A, similarly C = 4 × B
(a) 3 (b) 4 and also D = 4 × C. Thus the number N so formed is
(c) 5 (d) 6 perfectly divisible by:
[Based on FMS Delhi, 2004] (a) 11 (b) 10
(c) 3 (d) 13
14. A student was asked to find the sum of all the prime
numbers between 10 to 40. He found the sum as 180. 21. Which of the following is a prime number?
Which of the following statements is true? (a) 889 (b) 997
(a) He missed one prime number between 10 and 20. (c) 899 (d) 1147
(b) He missed one prime number between 20 and 30. [Based on FMS Delhi, 2004]
(c) He added one extra prime number between 10 and 20. 22. A cube is cut into n identical pieces. If it can be done so
(d) None of these in only one way, then which of the following could be the
 [Based on FMS Delhi, 2004] value of n?
(a) 179 (b) 203
15. −1 is not defined but it is denoted by i. Clearly, i is not
a real number, so it is called imaginary number. Now (c) 143 (d) 267
100 [Based on IIT Joint Man. Ent. Test, 2004]
find ∑ (i)n : 23. A gardener has to plant trees in rows containing equal
n =1
number of trees. If he plants in rows of 6, 8, 10 or 12, then
(a) i (b) 1 five trees are left unplanted. But if he plants in rows of 13
(c) –1 (d) 0 trees each, then no tree is left. What is the number of trees
[Based on FMS Delhi, 2004] that the gardener plants?
16. (a + b + c + d + e)/(v + w + x + y + z) = N, where a, b, c, (a) 485 (b) 725
d, e are five consecutive even integers and v, w, x, y, z are (c) 845 (d) None of these
five consecutive odd integers. If v = a + 1 and n represent [Based on IIT Joint Man. Ent. Test, 2004]
natural numbers, then which of the following is the most 24. I think of a number. I double the number, add 6 and
suitable value of N? multiply the result by 10. I now divide by 20 and subtract
(a) (n + 4)/(n + 5) (b) (n + 3)/(n + 4) the number I first thought of. The result is:
(c) (n + 2)/(n + 3) (d) (n + 2)/(n + 2.5) (a) Depends upon the number thought
[Based on FMS Delhi, 2004] (b) 1
17. Manu and Tanu are playing mathematical puzzles. Manu (c) 2
asks Tanu: ‘which whole numbers, greater than one, (d) 3

Chapter 01.indd 12 6/3/2015 6:21:36 PM


Numbers 1.13

25. Consider a 99-digit number created by writing side by 32. Number S is obtained by squaring the sum of digits of a
side the first fifty four natural numbers as follows: two-digit number D. If difference between S and D is 27,
1 2 3 4 5 6 7 8 9 10 11 12 13 _ _ _ _ 53 54 then the two digit number D is:
the above number when divided by 8 will leave a (a) 24 (b) 54
remainder: (c) 34 (d) 45
(a) 6 (b) 4 [Based on CAT, 2002]
(c) 2 (d) 0 33. The owner of a local jewellery store hired 3 watchmen to
guard his diamonds, but a thief still got in and stole some
26. The denominator of a rational number is 3 more than
diamonds. On the way out, the thief met each watchman,
its numerator. If the numerator is increased by 7 and the
one at a time. To each he gave half of the diamonds he had
denominator is decreased by 2, we obtain 2. The rational
then, and 2 more besides. He escaped with one diamond.
number is:
How many did he steal originally?

1
(a) (b)
5 (a) 40 (b) 36
4 8 (c) 25 (d) None of these
7 8 [Based on CAT, 2002]
(c) (d)
10 11 34.
A rich merchant had collected many gold coins. He did
[Based on FMS Delhi, 2003] not want anybody to know about him. One day, his wife
asked, ‘how many gold coins do we have?’ After pausing
27. A teacher gave the simple multiplication exercise to the
a moment, he replied, ‘well! if I divide the coins into two
kids. But one kid reversed the digits of both the numbers
unequal numbers, then 48 times the difference between
and carried out the multiplication and found that the
the two numbers equals the difference between the
product was exactly the same as the one expected by
squares of the two numbers. ‘The wife looked puzzled.
the teacher. Only one of the following pairs of numbers
Can you help the merchant’s wife by finding out how
will fit in the description of the exercise. Which one
many gold coins the merchant has?
is that?
(a) 96 (b) 53
(a) 14, 42 (b) 42, 28
(c) 43 (d) None of these
(c) 19, 63 (d) 13, 62
[Based on CAT, 2002]

28. Find the remainder when (11 1715 1115


+ 13 )  is divided by 7: 35. A child was asked to add first few natural numbers (that
is 1 + 2 + 3 + ...) so long his patience permitted. As he
(a) 0 (b) 1 stopped, he gave the sum as 575. When the teacher
declared the result wrong, the child discovered he had
(c) 2 (d) 3
missed one number in the sequence during addition. The
29. What is the smallest value of n for which (n13 – n) (52n – 1) number he missed was:
is divisible by 169? (a) Less than 10 (b) 10
(a) 5 (b) 4 (c) 15 (d) More than 15
[Based on CAT, 2002]
(c) 1 (d) 2
256
36. When 2 is divided by 17, the remainder would be:
30. If a number is divided by 2 the remainder is 1. If it is
(a) 1 (b) 16
divided by 3 the remainder is 2. What is the remainder
when the number is divided by 6? (c) 14 (d) None of these
[Based on CAT, 2002]
(a) 0 (b) 1
(c) 4 (d) 5 37. After the division of a number successively by 3, 4 and 7,
the remainders obtained are 2, 1 and 4, respectively. What
31. If there are 10 positive real numbers n1 < n2 < n3 will be the remainder if 84 divides the same number?
... < n10 . . . . How many triplets of these numbers (a) 80 (b) 75
(n1, n2, n3), (n2, n3, n4) . . . can be generated such that in
(c) 41 (d) 53
each triplet the first number is always less than the second
number, and the second number is always less than the [Based on CAT, 2002]
third number? 6n 6n
38. 7 – 6 , where n is an integer > 0, is divisible by:
(a) 45 (b) 90 (a) 13 (b) 127
(c) 120 (d) 180 (c) 559 (d) None of these
[Based on CAT, 2002] [Based on CAT, 2002]

Chapter 01.indd 13 6/3/2015 6:21:36 PM


1.14 Chapter 1

39. If x2 < 51 and y2 < 21 and x and y are integers, then which 46. In a number system, the product of 44 and 11 is 1034.
of the following is the least number which when divided The number 3111 of this system, when converted to the
by the least value of x and least value of y gives a negative decimal number system, becomes:
quotient? (a) 406 (b) 1086
(a) 28 (b) 56 (c) 213 (d) 691
(c) –28 (d) –56  [Based on CAT, 2001]
40. What is the product of remainders when 64 is divided by 47. A set of consecutive positive integers beginning with 1
24 and 75 is divided by 142? is written on the blackboard. A student came along and
erased one number. The average of the remaining numbers
(a) 7 (b) 5 7
is 35 . What was the number erased?
(c) 0 (d) 4 77
41. Of 128 boxes of oranges, each box contains at least 120 (a) 7 (b) 8
and at most 144 oranges. The number of boxes containing (c) 9 (d) None of these
the same number of oranges is at least: [Based on CAT, 2001]
(a) 5 (b) 103 48. Let D be a recurring decimal of the form D = 0. ala2ala2ala2
(c) 6 (d) None of these ..., where digits a1 and a2 lie between 0 and 9. Further, at
most one of them is zero. Which of the following numbers
[Based on CAT, 2001]
necessarily produces an integer, when multiplied by D?
42.
In a four-digit number, the sum of the first two digits is (a) 18 (b) 108
equal to that of the last two digits. The sum of the first and (c) 198 (d) 288
last digits is equal to the third digit. Finally, the sum of the  [Based on CAT, 2000]
second and fourth digits is twice the sum of the other two
49. What is the value of the following expression?
digits. What is the third digit of the number?
 1   1   1   1 
(a) 5 (b) 8  2  +  2  +  2  +  
(c) 1 (d) 4  (2 − 1)   (4 − 1)   (6 − 1)   (20 2
− 1) 

[Based on CAT, 2001] 9 10
(a) (b)
43. Anita had to do a multiplication. Instead of taking 35 as 19 19
one of the multipliers, she took 53. As a result, the product 10 11
went up by 540. What is the new product? (c) (d)
21 21
(a) 1050 (b) 540 [Based on CAT, 2000]
(c) 1440 (d) 1590 50. Consider a sequence of seven consecutive integers. The
[Based on CAT, 2001] average of the first five integers is n. The average of all the
seven integers is:
44. m is the smallest positive integer such that for any integer
(a) n
n ≤ m, the quantity n3 – 7n2 + 11n – 5 is positive. What is
the value of m? (b) n + 1
(a) 4 (b) 5 (c) k × n, where k is a function of n
(c) 8 (d) None of these (d) n +   
2
[Based on CAT, 2000]
[Based on CAT, 2001]  
7

45. Three friends, returning from a movie, stopped to eat at a 51.


Let N = 1421 × 1423 × 1425. What is the remainder when
restaurant. After dinner, they paid their bill and noticed a N is divided by 12?
bowl of mints at the front counter. Sita took one-third of (a) 0 (b) 9
the mints, but returned four because she had a monetary (c) 3 (d) 6
pang of guilt. Fatima then took one-fourth of what was
[Based on CAT, 2000]
left but returned three for similar reasons. Eswari then
took half of the remainder but threw two back into the 52.
The integers 34041 and 32506, when divided by a three-
bowl. The bowl had only 17 mints left when the raid was digit integer n, leave the same remainder. What is the
over. How many mints were originally in the bowl? value of n?
(a) 38 (b) 31 (a) 289 (b) 367
(c) 41 (d) None of these (c) 453 (d) 307
[Based on CAT, 2001] [Based on CAT, 2000]

Chapter 01.indd 14 6/3/2015 6:21:36 PM


Numbers I 1.15

53. f (a, b, c) = a + b + c and g (a, b, c) = a × b × c. I. x2 – 2y + 2


Then, how many such integer triplets a, b, c are there for II. y2 – 2x + 3
which f (a, b, c) = g (a, b, c)? (a, b, c are all distinct). III. 4x2 – y – 1
(a) 0 (b) Only 1 (a) I only (b) II only
(c) 2 (d) More than 2 (c) I and II (d) II and III
Let N = 553 + 173 – 723. N is divisible by:
54. 63. Which of the following statements is/are true?
(a) Both 7 and 13 (b) Both 3 and 13 I. n p – n is divisible by p where n and p are integers.
(c) Both 17 and 7 (d) Both 3 and 17 II. n p – n is divisible by p where n is a whole number and
[Based on CAT, 2000] p is a natural number.
55. Which of the following numbers has maximum factors? III. n p – n is divisible by p where n is an integer and p is a
(a) 36 (b) 76 prime number.
(c) 82 (d) 191 (a) Only I (b) Only II
56. Which of the following numbers has minimum factors? (c) Only III (d) I and III
(a) 58 (b) 88 64.
A two-digit number is four times the sum of the two digits.
(c) 137 (d) 184 If the digits are reversed, the number so obtained is 18
more than the original number. What is the original
57. From 1–90 how many numbers end in 4? number?
(a) 25 per cent (b) 30 per cent
(a) 36 (b) 24
(c) 20 per cent (d) 10 per cent (c) 48 (d) None of these
58. From 10–99 both inclusive how many numbers have their ....( n terms)
 1
unit digit smaller than the other digit? 1 1− 4 
 1−
(a) 90 (b) 45    1− 1  3  
(c) 32 (d) 26  2 2   
65. is equal to
     
 
   

 
5 13 5 13
59. If x =  6   6 ... to infinite terms, 1
2 4 2 4
(a) (2) n1 (b) 2n
then x =
n

3 2 3 5 (c) 2 n1 (d) 2log n


(a) (b)
2 2 66.
The number 311311311311311311311 is:
2 5 3 5 1 (a) Divisible by 3 but not by 11
(c) (d)
3 2 (b) Divisible by 11 but not by 3
60. The number of people in a row is equal to the number (c) Divisible by both 3 and 11
of rows in a playground. If total number of people in the (d) Neither divisible by 3 nor by 11
playground is 19044, find the number of rows: [Based on SNAP, 2007]
(a) 128 (b) 138
If p = 23n + 1, then which of the following is correct
67.
(c) 148 (d) 158 about p?
61. Let R be the remainder when 35n + 1 is divided by 7. (a) p is always divisible by 24.
Which of the following statements are true? (b) p is never divisible by 24.
I. R = 4, when n is even. (c) p is always divisible by 22.
II. R = 5, when n is even. (d) p is never divisible by 22.
III. R = 6, when n is odd.
68.
A three-digit number 4a3 is added to another three-digit
IV. R = 3, when n is odd.
number 984 to give the four-digit number 13b7, which is
(a) I and III (b) II and III divisible by 11. Then, (a + b) is:
(c) II and IV (d) I and IV (a) 10 (b) 11
62. If 2x – 1 is an odd number and 3y – 1 is an even number, (c) 12 (d) 15
which of the following is/are necessarily even? [Based on FMS (MS), 2006]

M01_KHAT6981_C01.indd 15 6/16/2015 5:15:23 PM


1.16 Chapter 1

69.
A three-digit number has, from left to right, the digits (a) 0 (b) 1
h, t and u with h > u. When the number with the digits (c) 2 (d) 3
reversed is subtracted from the original number, the
units’ digit in the difference is 4. The next two digits,  [Based on XAT, 2007]
from right to left, are:
76.
Let S be the set of rational numbers with the following
(a) 5 and 9 (b) 9 and 5 properties:
(c) 5 and 4 (d) 4 and 5
1
[Based on FMS, 2011] I. ∈ S;
2
70.
In our number system the base is ten. If the base were 1 x
II. If x ≠ S, then both ∈ S and ∈S
changed to four, you would count as follows: x +1 x +1
1, 2, 3, 10, 11, 12, 13, 20, 21, 22, 23, 30,
Which of the following is true?
The twentieth number would be:
(a) S contains all rational numbers in the interval 0 < x
(a) 110 (b) 104
< 1.
(c) 44 (d) 38 (b) S contains all rational numbers in the interval –1 < x
 [Based on FMS, 2011] < 1.
(c) S contains all rational numbers in the interval –1 < x
71.
If the square of a number of two digits is decreased by the < 0.
square of the number formed by reversing the digits, then (d) S contains all rational numbers in the interval –1 < x
the result is not always divisible by: < ∞.
(a) 9
[Based on XAT, 2007]
(b) the product of the digits.
(c) the sum of the digits. 77.
We define a function f on the integers f (x) = x/10, if x is
(d) the difference of the digits. divisible by 10, and f (x) = x + 1 if x is not divisible by 10.
 [Based on FMS, 2011] If A0 = 1994 and An+1 = f (An), what is the smallest n such
that An = 2?
72.
If the digit 1 is placed after a two-digit number whose (a) 9 (b) 18
ten’s digit is t, and unit’s digit is u, the new number is:
(c) 128 (d) 1993
(a) 10t + u + 1
 [Based on XAT, 2007]
(b) 100t + 10u + 1
(c) 1000t + 10u + 1 78.
Four digits of the number 29138576 are omitted so that
(d) t + u + 1 the result is as large as possible. The largest omitted digit
[Based on FMS, 2011] is:
(a) 9 (b) 8
73.
A number n is said to be perfect, if the sum of all its
(c) 7 (d) 5
divisors (excluding n itself) is equal to n. An example of
perfect number is:  [Based on XAT, 2008]

(a) 9 (b) 15
irections (Q. 79): The question given below is followed by
D
(c) 21 (d) 6
two statements labelled as I and II. You have to decide if these
[Based on XAT, 2006] statements are sufficient to conclusively answer the question.
Choose the appropriate answer from options given below:
n
74.
For how many integers n, is the square of an
20 − n (a) If Statement I alone is sufficient to answer the question.
integer? (b) If Statement II alone is sufficient to answer the question.
(a) 0 (b) 1 (c) If Statement I and Statement II together are sufficient
(c) 2 (d) 3 but neither of the two alone is sufficient to answer the
question.
 [Based on XAT, 2007]
(d) If either Statement I or Statement II alone is sufficient to
75.
Let p be any positive integer and 2x + p = 2y, p + y = x answer the question.
and x + y = z. For what value of p would x + y + z attain its (e) Both Statement I and Statement II are insufficient to
maximum value? answer the question.

Chapter 01.indd 16 6/3/2015 6:21:37 PM


Numbers 1.17

79.
A, B, C, D, E and F are six integers such that E < F, B > A, 85. What is the remainder when 774 − 574 is divided by 4?
A < D < B. C is the greatest integer? (a) 0 (b) 1
I. E + B < A + D (c) 2 (d) None of these

II. D < F [Based on CAT, 2009]
 [Based on XAT, 2008]
86. The sum of the number of factors of the number N and
N2 is 34. How many such distinct numbers N < 150
80.
If x and y are real numbers, then the minimum value of
exist?
x2 + 4xy + 6y2 – 4y + 4 is:
(a) 6 (b) 2
(a) –4 (b) 0 (c) 4 (d) 3
(c) 2 (d) 4 [Based on CAT, 2010]
 [Based on XAT, 2010]
87. The values of the numbers 22004 and 52004 are written one
81.
Let X be a four-digit positive integer such that the unit after another. How many digits are there in all?
digit of X is prime and the product of all digits of X is also (a) 4008 (b) 2003
prime. How many such integers are possible? (c) 2004 (d) None of these
(a) 4 (b) 8 [Based on CAT, 2011]

(c) 12 (d) 24
88. Let P = {2, 3, 4, …, 100} and Q = {101, 102, 103, ….,
[Based on XAT, 2010] 200}. How many elements of Q are there such that they do
not have element of P as a factor?
82.
The micro manometer in a certain factory can measure (a) 20 (b) 24
the pressure inside the gas chamber from 1 unit to 999999
(c) 23 (d) 21
units. Lately this instrument has not been working
[Based on CAT, 2012]
properly. The problem with the instrument is that it always
skips the digit 5 and moves directly from 4 to 6. What is
89. What is the sum of all the two-digit numbers that leave a
the actual pressure inside the gas chamber if the micro
remainder of 6 when divided by 8?
manometer displays 003016?
(a) 612 (b) 594
(a) 2201 (b) 2202
(c) 324 (d) 872
(c) 2600 (d) 2960 [Based on CAT, 2012]
 [Based on XAT, 2011]
90. Find the remainder of 21040 divided by 131.
83.
Let an = 1111111 ... 1, where 1 occurs n number of times. (a) 1 (b) 3
Then, (c) 5 (d) 7
I. a741 is not a prime. [Based on CAT, 2012]

II. a534 is not a prime. 91. If p is a prime number, p > 3, and let x be the product
III. a123 is not a prime. of positive numbers 1, 2, 3,..., (p − 1), then consider the
IV. a77 is not a prime. following statements:
I. x is a composite number divisible by p.
(a) (I) is correct
II. x is a composite number not divisible by p but some
(b) (I) and (II) are correct prime number greater than p may divide x.
(c) (II) and (III) are correct III. x is not divisible by any prime number (p − 2).
(d) All of these are correct IV. All prime numbers less than (p − 1) dividex
    which of the following statement(s) is/are correct.
 [Based on XAT, 2011]
(a) I and II are correct
84. What is the least value of K so that the number 6735K1 is (b) II and III are correct
divisible by 9? (c) III and IV are correct
(a) 5 (b) 7 (d) IV alone is correct
(c) 4 (d) 3 [Based on CAT, 2013]

Chapter 01.indd 17 6/3/2015 6:21:37 PM


1.18 Chapter 1

92. A three-digit number which on being subtracted from 98. Three friends, returning from a movie, stopped to eat at a
another three-digit number consisting of the same digits restaurant. After dinner, they paid bill and noticed a bowl
in reverse order gives 594. The minimum possible sum of of mints at the front counter. Sita took 1/3 of the mints, but
all the three digits of this number is: returned four because she had a monetary pang of guilt.
(a) 6 (b) 7 Fatima then took 1/4 of what was left but returned three
(c) 8 (d) Cannot be determined for similar reasons. Eswari then took half of the remainder
but threw two back into the bowl. The bowl had only 17
[Based on CAT, 2013] mints left when the raid was over. How many mints were
originally in the bowl?
93. If 223 + 233 + 243 + … + 873 + 883 is divided by 110, (a) 38 (b) 31
then the remainder will be:
(c) 41 (d) None of these
(a) 55 (b) 1
[Based on CAT, 2001]
(c) 0 (d) 44
[Based on CAT, 2013] 99. ABCDEFGH is a regular octagon. A and E are opposite
vertices of octagon. A frog starts jumping from vertex to
94. If p, q, r and s are positive real numbers such that p + q + vertex, beginning from A. From any vertex of the octagon
r + s = 2, then m = (p + q) (r + s) satisfies the relation: except E, it may jump to either of the two adjacent
vertices. When it reaches E, the frog stops and stays there.
(a) 0 ≤ m ≤ 1
Let, an be the number of distinct paths of exactly n jumps
(b) 1 ≤ m ≤ 2 ending in E. Then, what is the value of a2n – 1?
(c) 2 ≤ m ≤ 3 (a) 0 (b) 4
(d) 3 ≤ m ≤ 4 (c) 2n – 1 (d) None of these
[Based on CAT, 2000]
95. Consider four digit numbers for which the first two digits
and the last two digits are equal. How many such numbers 100. Let N = 553 + 173 – 723. N is divisible by:
are perfect squares? (a) Both 7 and 13 (b) Both 3 and 13
(a) 2 (b) 4 (c) Both 17 and 7 (d) Both 3 and 17
(c) 0 (d) 1 [Based on CAT, 2000]
[Based on CAT, 2007]
101. The integers 34041 and 32506, when divided by a three-
digit integer n, leave the same remainder. What is the value
96. A confused bank teller transposed the rupees and paise of n?
when he cashed a cheque for Shailaja, giving her rupees
(a) 289 (b) 367
instead of paise and paise instead of rupees. After buying
a toffee for 50 paise, Shailaja noticed that she was left (c) 453 (d) 307
with exactly three times as much as the amount on the [Based on CAT, 2000]
cheque. Which of the following is a valid statement about
the cheque amount? 102. Let D be a recurring decimal of the form D = 0. a1 a2
a1 a2 a1 a2 . . ., where digits a1and a2 lie between 0 and
(a) Over `7 but less than `8
9. Further, at most one of them is zero. Which of the
(b) Over `22 but less than `23 following numbers necessarily produces an integer, when
(c) Over `18 but less than `19 multiplied by D?
(d) Over `4 but less than `5 (a) 18 (b) 108
(c) 198 (d) 288
[Based on CAT, 2007]
[Based on CAT, 2000]
97. A set of consecutive positive integers beginning with 1
103. If n = 1 + x, where x is the product of four consecutive
is written on the blackboard. A student came along and
positive integers, then which of the following is/are true?
erased one number. The average of the remaining numbers
A. n is odd
7
is 35 . What was the number erased? B. n is prime
17
C. n is a perfect square
(a) 7 (b) 8 (a) A and C only (b) A and B only
(c) 9 (d) None of these (c) A only (d) None of these
[Based on CAT, 2001] [Based on CAT, 1999]

Chapter 01.indd 18 6/3/2015 6:21:37 PM


Numbers 1.19

104. The remainder when 784 is divided by 342 is: 112. P and Q are two positive integers such that PQ = 64.
(a) 0 (b) 1 Which of the following cannot be the value of P + Q?
(c) 49 (d) 341 (a) 20 (b) 65
[Based on CAT, 1999] (c) 16 (d) 35
[Based on CAT, 1997]
105. Let a, b, c be distinct digits. Consider a two-digit number
‘ab’ and a three-digit number ‘ccb’, defined under the
usual decimal number system, if ab2 = ccb > 300, then the 113. If n is an integer, how many values of n will give an
value of b is: (16n 2 + 7 n + 6)
integral value of  ?
(a) 1 (b) 0 n
(c) 5 (d) 6 (a) 2 (b) 3
[Based on CAT, 1999] (c) 4 (d) None of these
[Based on CAT, 1997]
106. A hundred digit number is formed by writing first 54 natural
numbers in front of each other as 12345678910111213 . . . .
Find the remainder when this number is divided by 8. 114. If n is any odd number greater than 1, then n(n2 − 1) is:
(a) 1 (b) 7 (a) divisible by 96 always
(c) 2 (d) 0 (b) divisible by 48 always
[Based on CAT, 1998] (c) divisible by 24 always
(d) None of these
107. A certain number when divided by 899 leaves the [Based on CAT, 1996]
remainder 63. Find the remainder when the same is
divided by 29. 115. If a number 774958A96B is to be divisible by 8 and 9, the
(a) 5 (b) 4 respective values of A and B will be:
(c) 1 (d) Cannot be determined (a) 7 and 8 (b) 8 and 0
[Based on CAT, 1998] (c) 5 and 8 (d) None of these
[Based on CAT, 1996]
108. What is the digit in the unit’s place of 251?
(a) 2 (b) 8
116. Three consecutive positive even numbers are such that
(c) 1 (d) 4 thrice the first number exceeds double the third by 2, the
[Based on CAT, 1998] third number is:
(a) 10 (b) 14
109. n3 is odd. Which of the following statement(s) is (are)
(c) 16 (d) 12
true?
A. n is odd [Based on CAT, 1995]
B. n2 is odd
C. n2 is even 117. The remainder obtained when a prime number greater
than 6 is divided by 6 is:
(a) A only (b) B only
(a) 1 or 3 (b) 1 or 5
(c) A and B only (d) A and C only
(c) 3 or 5 (d) 4 or 5
[Based on CAT, 1998]
[Based on CAT, 1995]
110. Which of the following is true?
553 + 453
(a) 73 = (73 ) 2 (b) 73 > (73 ) 2
2
118. The value of is :
2

55 − 55 × 45 + 452
2
(c) 73 < (73 ) 2 (d) None of these
2

(a) 100 (b) 105


[Based on CAT, 1997]
(c) 125 (d) 75
111. If m and n are integers divisible by 5, which of the [Based on CAT, 1995]
following is not necessarily true?
(a) (m – n) is divisible by 5 119. The number of positive integers not greater than 100,
(b) (m2 – n2) is divisible by 25 which are not divisible by 2, 3 or 5, is:
(c) (m + n) is divisible by 10 (a) 26 (b) 18
(d) None of these (c) 31 (d) None of these
[Based on CAT, 1997] [Based on CAT, 1993]

Chapter 01.indd 19 6/3/2015 6:21:38 PM


1.20 Chapter 1

120. What are the last two digits of 72008? (a) Statement I
(a) 21 (b) 61 (b) Statement II
(c) 01 (d) 41 (c) Statement I and Statement II
[Based on CAT, 2008] (e) Either Statement I or Statement II
121.
After distributing the sweets equally among 25
123. How many whole numbers between 100 and 800 contain
children, 8 sweets remain. Had the number of children
the digit 2?
been 28, 22 sweets would have been left after equal
distribution. What was the total number of sweets? (a) 200 (b) 214
(a) 328 (b) 348 (c) 220 (d) 240
(c) 358 (d) Data inadequate (e) 248
[Based on SNAP, 2013]  [Based on XAT, 2013]
122. Consider four natural numbers: x, y, x + y and x – y. Two
statements are provided below: 124. p, q and r are three non negative integers such that p + q +
r = 10. The maximum value of pq + qr + pr + pqr is:
I. All four numbers are prime numbers.
(a) ≥ 40 and < 50 (b) ≥ 50 and < 60
II. The arithmetic mean of the numbers is greater than 4.
Which of the following statements would be sufficient to (c) ≥ 60 and < 70 (d) ≥ 70 and < 80
examine the sum of the four numbers?  [Based on XAT, 2013]

Answer Keys
Difficulty Level-1

1. (a) 2. (d) 3. (b) 4. (c) 5. (c) 6. (b) 7. (a) 8. (d) 9. (a) 10. (d ) 11. (a) 12. (d) 13. (d)
14. (b) 15. (b) 16. (d) 17. (a) 18. (b) 19. (c) 20. (a) 21. (c) 22. (d) 23. (d) 24. (a) 25. (b) 26. (d)
27. (d) 28. (a) 29. (b) 30. (d ) 31. (d) 32. (a) 33. (c ) 34. (c) 35. (a) 36. (d) 37. (b) 38. (b ) 39. (d)
40. (d ) 41. (d) 42. (b) 43. (a ) 44. (d) 45. (b) 46. (d) 47. (b ) 48. (c ) 49. (b) 50. (d) 51. (d) 52. (b)
53. (d) 54. (a) 55. (d) 56. (b) 57. (c) 58. (c) 59. (c) 60. (d) 61. (c) 62. (a) 63. (a ) 64. (b) 65. (c)
66. (d) 67. (b) 68. (a) 69. (a) 70. (a)

Difficulty Level-2

1. (b) 2. (c) 3. (b) 4. (d) 5. (a) 6. (d) 7. (a) 8. (d) 9. (c) 10. (d) 11. (d) 12. (c) 13. (b)
14. (d ) 15. (d) 16. (d) 17. (b) 18. (c) 19. (c) 20. (b) 21. (b) 22. (a) 23. (c) 24. (d ) 25. (c) 26. (b)
27. (d) 28. (b) 29. (d) 30. (d) 31. (c ) 32. (b) 33. (b) 34. (d) 35. (d ) 36. (a) 37. (d ) 38. (b) 39. (a)
40. (c) 41. (a) 42. (a) 43. (d) 44. (d) 45. (d) 46. (a ) 47. (a) 48. (c) 49. (c) 50. (b) 51. (c) 52. (d)
53. (d) 54. (d ) 55. (a) 56. (c ) 57. (d) 58. (b) 59. (b) 60. (b) 61. (a) 62. (d) 63. (c ) 64. (b) 65. (a)
66. (d) 67. (d) 68. (a) 69. (b) 70. (a) 71. (b) 72. (b) 73. (d) 74. (c) 75. (a) 76. (a ) 77. (a) 78. (d )
79. (a) 80. (c) 81. (a) 82. (a) 83. (d) 84. (a) 85. (a) 86. (b) 87. (d) 88. (d) 89. (b ) 90. (a) 91. (d )
­ 92. (c) 93. (a) 94. (a) 95. (d) 96. (c) 97. (a) 98. (d) 99. (d) 100. (d) 101. (d) 102. (c ) 103. (a) 104. (b )
105. (a) 106. (a) 107. (a) 108. (b) 109. (c) 110. (b) 111. (c) 112. (d) 113. (c) 114. (c) 115. (b ) 116. (b) 117. (b )
118. (a) 119. (a) 120. (c) 121. (c) 122. (a) 123. (b) 124. (c)­­­

Chapter 01.indd 20 6/3/2015 6:21:38 PM


Numbers 1.21

Explanatory Answers

Difficulty Level-1

1.
(a) Let x be the number, No. of unmarried people = 40
x  12 No. of men in the room = 48
\ (x + 12) ÷ 6 = 112 ⇒ = 112
6 If all the men are supposed to be married, then number
⇒ x = 112 × 6 – 12 of married women could be 80 – 48 = 32
⇒ x = 672 – 12 = 660 \ Maximum number of unmarried women could be
x  12 72 – 32 = 40.
\ Correct answer =
6 8.
(d) x = 2 + 22/3 + 21/3 ⇒ x > 2
660 For, x = 2, x3 – 6x2 + 6x = –4
=  12 = 110 + 12 = 122.
6 For, x = 3, x3 – 6x2 + 6x = –9
\ x3 – 6x2 + 6x < 0.
(d) (n + 1)2 = n2 + 2n + 1.
2.
9.
(a) 248 in the scale of 7 is written as 503. In scale 9, it is
(b) Last 2 digits of (125)521 will be 25.
3.
written as 305.
To find the last two digits of (521)125, we need to
x x +1
consider (21)125 only. 10.
(d) Given expression = +
2 2
The last 2 digits for different powers of 21 are:
2x  1 1
(21)1 → 21 = = x  = [x].
2
 2
(21) 2 → 41 11.
(a) 5 3 6 4 7 is a multiple of 11 because the difference of
 the sum of the digits in the odd places and the sum of
(21)3 → 61
the digits in the even places is divisible by 11, because
 It is a cycle of 5 for the last two digits.
(21) 4 → 81 (5 + 6 + 7) – (3 + 4) = 11
 \ Total number of five-digit multiples of 11
(21)5 → 01
(21)6 → 21 = 3! (Permutation of 5, 6 and 7 in the odd places)
  × 2! (Permutation of 3 and 4 in the even places)
So, 125 being divisible by 5, the last 2 digits of
= 6 × 2 = 12.
(521)125 will be 01.
Thus, 25 × 01 = 25 3600
12.
(d) = 8 = 23.
Therefore, last two digits of N are 25. 450
4.
(c) Let the three-digit number be 100x + 10y + z 13.
(d) 1024.
\ (100x + 10y + z) – (x + y + z) = 99x + 9y (b) 73 = 343, when divided by 342, leaves a remainder
14.
= 9 (11x + y) of 1
which is always divisible by 9. 74 = 2401, when divided by 342, leaves a remainder
of 7
5.
(c) Let x must be subtracted from 14, 17, 34 and 42 such 75 = 16807, when divided by 342, leaves a remainder
that of 49
(14 – x) (42 – x) = (17 – x) (34 – x) ⇒ x = 2. 76 = 117649, when divided by 342, leaves a remainder
of 1
6. (b) 125125 – 2525 = 5375 – 550 = 550(5325 – 1). And so on.
Now 5325 – 1 is not divisible by 5. Hence, the highest \ 784, when divided by 342, will leave a remainder
power of 5 that is contained in the given expression of 1.
is 50. 15.
(b) Let the digits be a and b such that the number is
10a + b
2
7.
(a) No. of women in the room = × 120 = 72 \ ab = 14 and 10a + b + 45
5
= 10b + a
2 i.e., 9a – 9b = – 45
No. of married people = × 120 = 80
5 i.e., a – b = – 5

Chapter 01.indd 21 6/3/2015 6:21:39 PM


1.22 Chapter 1

\ (a + b)2 = (a – b)2 + 4ab = 81 24.


(a) Eliminated numbers are 5, 10, 15, 20, 25, 30, 35, 40,
⇒ a + b = 9 45, 50, 51, ..., 60
So, total eliminated numbers are 20
⇒ a = 2, b = 7
\ 40 numbers would remain.
\ The number is 27.
25.
(b) Required numbers are 509, 519, 529, 539, 549, 559,
16.
(d) Divisor = 12 × Quotient 569, 579, 589, 590, 591, 592, ..., 1598.
Divisor = 5 × Remainder 26.
(d) Let the total number of swans be x.
7
Remainder = 48 The number of swans playing on shore = x
2
⇒ Divisor = 240, Number of remaining swans = 2
\ Quotient = 20 \ x =
7
x+2
Hence, 2
Dividend = 240 × 20 + 48 = 4848. 7
⇒ (x – 2) = x
2
17.
(a) Divisors of 88 are 2, 4, 8, 11, 22, 44
27. (d)
Divisors of 91 are 7 and 13
Thumb Index Middle Ring Little
Divisors of 99 are 3, 9, 11, 33.
1 2 3 4 6
1 11 9← 8 7 6
18.
(b) 1  =
10 10 10 11 12 13
1 1 17 ← 16 15 14
1× = 18 19 20 21
10 10
25 ← 24 23 22
11 1 26 27 28 29
\ Sum = = 11 × = 11 × Product.
10 10 33 ← 32 31 30
19.
(c) m(n + 0)( p – q) is even ⇒ m must be even. From the above counting pattern, we find that every
20.
(a) A number divisible by 99 must be divisible by 9 as multiple of 8 comes on index finger and moves
well as 11. towards thumb therefore, the last multiple of 8 which
1994
\ 114345 is divisible by both. appears on index finger will be ⇒ 1992
8
21.
(c) Let the numbers be x, y and z. Hence, 1994 will be on ring finger.
x + y = 45, y + z = 55 and 3x + z = 90 28.
(a) Total number of men = 5180 + 4 = 5184
y = 45 – x, \ Number of men in first row = 5184 = 72.
z = 55 – y = 55 – (45 – x) = 10 + x 29.
(b) Let total number of votes cast be x.
\ 3x + 10 + x = 90 2
Total number of counted votes = x
or,   x = 20 3
y = 45 – 20 = 25 5 2 5
Votes that candidate got = × x = x
z = 10 + 20 = 30 6 3 9
\ Third number is 30. 3 5 7
Votes still need to win = x − x =x
22.
(d) Let the two-digit number = xy 4 9 36
\ 2(10x + y) = 9(10y + x) 1
Remaining uncounted votes = x
⇒ 88y – 11x = 0 (1) 3
7 3 7
Also, x + y = 9 (2) \ Required part =
× =.
36 1 12
Solving Eqs. (1) and (2), we get
30.
(d) Required sum = 3000 + 30 = 3030.
x = 8 and y = 1
So, the number is 81. 31.
(d) Let the whole number be x.
1
23.
(d) Let the numbers be 3x and x. \  x = ( x + 20)
6
3x + x = 36 ⇒ 6x = x + 20
⇒ 4x = 36 ⇒ 5x = 20
⇒ x = 9. ⇒ x = 4.

Chapter 01.indd 22 6/3/2015 6:21:39 PM


Numbers 1.23

32.
(a) Let the number be (765x + 42) Using (3) and (4) in (2), we get
When this number is divided by 17, then quotient will
3 1
be (45x + 2) and remainder will be 8 x+ x + x = 16
4 4
33.
(c) Production in third year = 6000
Production in seventh year = 7000 ⇒ x = 8
\ Production in fourth year = 1000 \ y = 6, z = 2
i.e., Production increases @ 250 scooters every year. Hence, the number is 862.
\ Production in tenth year
= (7000 + 250 × 3) = 7750. 1 1
40.
(d) = K K +3
3 4
34.
(c) Let the number of girls and boys be x and y.
Then, x – 1 = y ⇒ K = 36.
and xy = 272
⇒ x(x – 1) = 272 29 15 116 + 45 161 17
41.
(d) + = = = 3
⇒ x2 – x – 272 = 0 12 16 48 48 48
⇒ (x + 17)(x – 16) = 0
42.
(b) 7! = 7 × 6 × 5 × 4 × 3 × 2 × 1
⇒ x = 16.
= 24 × 32 × 51 × 71
35.
(a) Let there were x friends, then contribution of one Thus, the least perfect square which is divisible by 7!
friend =
96 should be (24 × 32 × 51 × 71)(51 × 71)
x i.e., 5040 × 35 = 176400.
If four friends have left, then contribution of each
43.
(a) If the square of any natural number n leaves a
friend =
96 remainder of 1 when divided by 24, that natural
x−4 number must be of the form 6p ± 1 (since n must
96 96 be divisible by neither 2 nor 3) where p is a natural
\ − = 4 ⇒ x = 12 number.
x−4 x
\ the two digit numbers must be of the form 6p ± 1,
Hence, number of friends who attended the picnic There are 15 two-digit numbers in the form 6p + 1 and
= 12 – 4 = 8. the same number of two digit numbers in the form
36.
(d) Number of boxes used 6p – 1.
17 + 13 30 5 1 \ a total of 30 two-digit numbers satisfy the given
= = = = 1 condition.
24 24 4 4
Since, the number of boxes used should be a whole 44. (d) If we put consecutive negative integers as x = –1,
number, hence the number of boxes used is 2. y = –2 and z = –3, then from option (d),
37.
(b) There are four prime numbers, viz., 7, 1 1, 13, 29. (–1 + 2)(–2 + 3) = 1 × 1
38.
(b) Let the number be abc; so the six-digit number is = 1 (Positive integer).
abcabc. Now, the sum of alternate digits is:
45. (b) Let the second number be 3x, so that the first number
(i) a + c + b is 6x and the third one is 2x.
(ii) b + a + c
\ 6x + 3x + 2x = 132
Both being equal, the six-digit number is definitely
divisible by 11. ⇒ 11x = 132 or, x = 12

39.
(d) Let x, y and z be the digits at the hundredth place, ten’s Second number = 3x = 3 × 12 = 36.
place and unit’s place respectively.
46. (d) Let the unit digit be y and tens digit be x.
\ x + y + z = 16 (1)
\ The number = 10x + y
y = 3z (2)
On interchanging the digits, the number = 10y + x
1
z = x (3) \ 10x + y – 10y – x = 27
4
3 ⇒ x – y = 3
\ (2) ⇒ y = x (4)
4 (already given in the question)

Chapter 01.indd 23 6/3/2015 6:21:39 PM


1.24 Chapter 1

Now, y ≠ 0 and the set of digits satisfying the condition xy


are (9, 6), (8, 5), (7, 4), (6, 3), (5, 2), (4, 1) (a) = by, it is an integer
z
\ We can not reach on a distinct answer. y + z ax + z
(b) = , it is not an integer
47. (b) 2n + 5m = 50 x x
\ Possible value of n and m are: yz
(c) = az, it is an integer
(25, 0), (10, 6), (20, 5), (15, 4), (5, 8) x

Hence, least difference between 5 and 8 is 3. x + y bz + abz


(d) = = (b + ab), it is an integer.
z z
48. (c) Divide 7231 by 45, the remainder is 31
57. (c) Given that unit digit of a2 = 9
\ Required number = 45 – 31 = 14. and, (a + 1)2 = 4

49. (b) Let a two-digit number = 48 i.e., unit digit of a must be 3

When unit digit is halved = 4 ∴ Unit digit of (a + 2)2
⇒ (3 + 2)2 = 52
Ten’s digit is doubled = 8
⇒ = 25
\ Number = 84
i.e., 5.
Hence, digit in the unit’s place is twice the digit in the
58.
(c) After dividing 10 by 7,
ten’s place.
we get remainder n – 4
50. (d) According to question
i.e., 7 – 4 = 3.
=
16 × 4 64 =
64 × 1 64 = 8 × 8 64 = 7 × 5 35
59. (c) We see that 42 + 62 = 52 when divided by 25, remainder
(16 +=4) 20 (64 +=1) 65 (8 +=8) 16 (7 +=5) 12
is 2.
51.
(d) ab > 0 because a and b both are positive. 43 + 63 = 280, divide by 25, remainder is 5
52.
(b) n must be divisible by 35. 44 + 64 = 1552, divide by 25, remainder is 2
53.
(d) Let the number be 253 When taking m odd, the remainder is 5
Which unit place is 2 When taking m even, the remainder is 2
\ Digit at 100 place of original number is 2. Hence, remainder = 5.

54.
(a) x and y are natural numbers 60. (d) Unit’s digit in 102 is 2.
We know that for any natural number p, The digit in the unit’s place of 10251 will be same as in
pn + p is even 251 or, 23 = 8. [ 51 = 4.12 + 3]
and, pn – p is even 61. (c) Divide 9269 by 73, the remainder is 71
When, we multiply an even number to any natural \ 71 is the required least number.
number the resultent number is even.
62.
(a) Divide 15463 by 107, the remainder is 55, therefore,
55.
(d) (I) Product of 4 positive numbers cannot be zero. the number to be added = 107 – 55 = 52.
(II) a can be odd or even, b can be odd or even, c is even, 63.
(a) a + b < c + d (1)
p is odd. We cannot definitely say that a + b +
b + c < d + e (2)
c + p is odd.
c + d < e + a (3)
(III) (b2 + c2) – (p2 – a2), here b2 + c2 can be odd or
d + e < a + b (4)
even, (p2 – a2) can be odd or even.
From (1) and (4),
(IV) a (p – c) + a (c + b) = a [p – c + c + b]
a + b + d + e < c + d + a + b
Where a is divisible by 5
⇒ e < c
So, a (p – c) + a (c + b) will be divisible by 5
From (2) and (4),
So, only (IV) is correct.
b + c + d + e < d + e + a + b
56.
(b) x is a factor of y
⇒ c < a
∴ y = ax (Suppose)
From (1) and (3),
x is a multiple of z a + b + c + d < c + d + e + a
∴ x = bz (Suppose) ⇒ b < e.

Chapter 01.indd 24 6/3/2015 6:21:40 PM


Numbers 1.25

64.
(b) zx + y = 10, zy + z = 20 67.
(b) 50000 = 79 × quotient + 43759
x+z
2 = 30 \ 50000 – 43759 = 79 × quotient
⇒ 2z + y × 2y + z × 2z + x = 10 × 20 × 30 = 6000 or, 6241 = 79 × quotient
2 (x + y + z)
⇒ 2 = 6000 6241
\ Required number of times = = 79.
⇒ 22(y + z) = 400 79
6000 68. (a) On dividing 58701 by 567
⇒ 22 (y + y + z – y – z) = = 15 1
400 Remainder = 300 > (567)
⇒ 22x = 5 2
\ Integer nearest to 58701 and divisible by 567
⇒ 2x = 15
= 58701 + (567 – 300)
65. (c) Dividing 5000 by 73, the remainder is 36. The number = 58701+ 267 = 58968.
greater than 5000 is obtained by adding to 5000 the 69. (a) The least no. of six figures is 100000.
difference of divisor and the remainder.
On dividing 100000 by 357, remainder = 40
\ The required number
\ Least number of six figures which is divisible by
= 5000 + (73 – 36)
357 = 100000 + (357 – 40)
= 5037. = 100317
66. (d) Let the numbers be a and b. \ Required number = 100317 – 5555 = 94762.
Then, a + b = 100 and, a – b = 37 70. (a) Let the least value to be given to * be x
\ a – b2 = (a + b) (a – b)
2
Then, x + 4 + 7 = 5 + 3 + 5
= 100 × 37 = 3700. x = 2

Difficulty Level-2

1.
(b) a, b, c and d are four integers such that a + b + c + d = \ Total no. of possible three-digit numbers
4m + 1. = (1 × 2) + (2 × 3) + (3 × 4) + (4 × 5)
Minimum possible value of a2 + b2 + c2 + d 2 is when
+ ... + (7 × 8) + (8 × 9) = 240.
a, b, c and d are as close to each other as possible.
Since RHS is not the multiple of 4, as, b, c and d can 3. (b) Product (n – 1) (n – 2) ... 3 × 2 × 1 is not divisible by
not be equal to m. n if n is 4 or a prime number.
Hence the numbers may be of the form, m, m, m and We have to find the number of primes in
m + 1. 12 ≤ n ≤ 40.
\ a2 + b2 + c2 + d2 = 4m2 + 2m + 1. i.e., {13, 17, 19, 23, 29, 31, 37}
2.
(c) We have to find the number of three-digit numbers in \ No. of positive integers in the range
which the digit at ten’s place is greater than the digit 12 ≤ n ≤ 40 is 7.
at unit’s and hundred’s places. That is, 4. (d) Take any arbitrary value of x and y
Hundred Ten Unit Let, x = 2 (prime number)
y = 50 (composite number)
x y z
Going through the options,
x<y>z x y
(a), (b) and (c) are wrong because y – x, xy and
The following chart shows the number of ways in x
which it can be formed. are even integers for x = 2 and y = 50
Number of ways \ None of the statements are true.
Number of ways in in which unit’s 5. (a) The set of prime numbers 3, 5, 7 is the only set which
which unit’s place, Digit at ten’s place, i.e., y can satisfies the given condition.
i.e., x can be filled place, i.e., y be filled (d) If 42 is divided by 6, remainder is 4
6.
1 (i.e., 1) 2 2 (i.e., 0, 1) If 43 is divided by 6, remainder is 4
2 (i.e., 1, 2) 3 3 (i.e., 0, 1, 2) If 44 is divided by 6, remainder is 4
... ... ... ... ... ...
8 (i.e., 1, 2, 3, ... 8) 9 9 (i.e., 0, 1, 2, ...9) If 496 is divided by 6, remainder is 4.

Chapter 01.indd 25 6/3/2015 6:21:40 PM


1.26 Chapter 1

7. (a) After 54, the remainder left when 5n is divided by of a number should be divisible by 9. Hence options
1000 is 125 when n is odd and 625 when n is even. (c) and (d) are ruled out as all the given numbers are
Hence, the remainder is 125. not divisible by 9 (because the sum of their digits is
8.
(d) For every integer a, a + (– a) = 0. Therefore, by not divisible by 9). Option (b) is the answer as 3 and
pairing 1 with –1, 2 with –2, and so on, one can 37 are factors of 111 and 111 is the divisor of all the
see that in order for the sum to be zero, a list of given numbers.
consecutive integers must contain the same number 18.
(c) The prime numbers are 2, 3, 5, 7, 11, 13, 17, 19, 23,
of positive integers as negative integers, in addition 29, etc.
to the integer ‘0’. Therefore, the list has an odd The numbers 5, 11, 17, 23, 29 form an increasing
number of consecutive integers and their average sequence for which 29 is the fifth term.
will also be 0.
19.
(c) 1012 – 1 = (106 – 1) (106 + 1)
      
9.
(c) No. of Spades = 1
= (103 – 1) (103 + 1) (106 + 1)
No. of Diamonds = 2
= 999 × 1001 × 1000001
No. of Hearts = 4
No. of Clubs = 6. 10 − 1 999 × 1001 × 1000001
12
 Therefore, =
=
111 111
10.
(d) No. of balls in Red Box = 18
= 9 × 1001 × 1000001
No. of balls in Blue Box = 36
= 9009009009.
No. of balls in Green Box = 54.
61 62
20.
(b) N = 4 +4 + 4 + 464
63
(d) The given product contains 5106 and 2x
11.
= 4 (1 + 4 + 16 + 64) = 461 × 85
61
where, x > 106
= 461 × 5 × 17 = 460 × 4 × 5 × 17
\ There will be 106 zeroes in the product, because zero
will come only by multiplying 2 and 5. = 460 × 2 × 17 × 10
Hence, it is divisible by 10.
12.
(c) Such numbers are 56, 67, 78 and 89
21.
(b) 889 = 7 × 127
Sum of these numbers = 290.
899 = 29 × 31
13.
(b) 20 × 21 × 22 × 23 × 24 = 5100480
1147 = 31 × 437.
25 × 26 × 27 × 28 = 491400
22.
(a) 179 is a prime number.
29 × 30 = 870.
203 = 7 × 29
14.
(d) Sum of the prime numbers between 10 and 40 = 11 +
143 = 11 × 13
13 + 17 + 19 + 23 + 29 + 31 + 37 = 180.
267 = 3 × 89.
100
= (i + i2 + i3 + i4) + (i5 + i6 + i7 + i8) + 23.
(c) Multiple of 120 + 5, which is divisible by 13.
15.
(d) ∑ in
n =1 24.
(d) Let n be the number.
... + (i97 + i98 + i99 + i100) Then, the result is
= (i – 1 – i + 1) + (i – 1 – i + 1) + ...
{(2n + 6)10}  2n + 6 − 2n = 3.
= 0 + 0 + ... + 0 = 0. =   –x=
 20  2
16.
(d) Let the five consecutive even numbers 2n, 2n + 2, 25.
(c) By the rules of divisibility, we know that any number
2n + 4, 2n + 6, 2n + 8 be respectively equal to a, b, c, is divisible 8, if the last three digits of the number is
d and e, where n is a natural number. also divisible by 8.
Then, v, w, x, y and z are equal to 2n + 1, 2n + 3, In the given number last three digits are 354. So, the
2n + 5, 2n + 7, 2n + 9. remainder is 2.
2n  2n  2  2n  4  2n  6  2n  8 p
⇒ N = 26.
(b) Let the rational number be
2n  1  2n  3  2n  5  2n  7  2n  9 q
\ q = p + 3
10n  20 n2
= = . p+7
10n  25 n  2.5 \ = 2 ⇒ p + 7 = 2p + 2
p +3−2
17.
(b) For the number to be divisible by 3, the sum of the ⇒ p = 5
digits of a number should be divisible by 3. Also, for ⇒ Given rational number = .
5
the number to be divisible by 9, the sum of the digits 8

Chapter 01.indd 26 6/3/2015 6:21:40 PM


Numbers 1.27

27.
(d) Let the two numbers be ab and xy. 33.
(b) Escaped with 1
\ (100a + b) × (100x + y) = (100b + a) + (100y + x) Before 3rd watchman, he had (1 + 2) × 2 = 6
⇒ 10000ax + 100ay + 100bx + by Before 2nd watchman, he had (6 + 2) × 2 = 16
Before 1st watchman, he had (16 + 2) × 2 = 36.
= 10000by + 100bx + 100ay + ax
⇒ 9999ax = 9999by 34.
(d) Let the no. of gold coins = x + y
48 (x – y) = x2 – y2
⇒ ax = by
⇒ 48 (x – y) = (x –­ y) (x + y)
Now, check from the options ⇒ x + y = 48.
For option (d): a = 1, b = 3, x = 6, y = 2
n2 + n
\ ax = 1 × 6 = 6 and by = 3 × 2 = 6 35.
(d) 575 = −x
2
Hence, ax = by.
⇒ 1150 = n2 + n – 2x
15
28.
(b) When 17 is divided by 6 For, n = 34,
(18 − 1)15 , remainder = 5 40 = 2x

6 \ x = 20.

\ 1715 can be written as 6K + 5 (a) (24)64 = (17 – 1)64 = 17n + (–­1)64


36.
= 17n + 1
116K+5 (7+4)6K+5 46K+5 Hence, remainder = 1.
\ = =
7 7 7 37.
(d) 3 {4 (7x + 4) + 1} + 2 = 84x + 53
Therefore, remainder is 53.
16 × (43 ) 2K+1 16 × (63 + 1) 2K+1
 = = (b) 76n – 66n
38.
7 7
Put n = 1
Remainder = 2 × 1 = 2
76 – 66 = (73 – 63) (73 + 63)
This is a multiple of 73 – 63 = 127.
15
1311 (14 – 1)odd
    = ⇒ Remainder = 6
7 7
39. (a) Here the least value of x = 49 = –7
\ Remainder when 111715 + 131715 is divided by
7 is 1. and the least value of y = 16 = – 4

(d) n13 – n is divisible by 13 for all n ∈ whole numbers


29. So, the least number here which when divided by –7
52n – 1 is divisible by 13 for even n. and – 4 gives a negative quotient in each case is 28
28 28
The smallest even number is 2. since = – 4 and = –7.
–7 –4
\ When n = 2, the expression is divisible by 169.
30.
(d) When a number is divided by 6 possible remainders 40. (c) Since 64 ÷ 24 = 1296 ÷ 16 = 81 and remainder 0.
are 1, 2, 3, 4, 5 (x = 6y + remainder). But only odd So, we need not calculate the remainder in second
numbers are possible as with even numbers the case as the product will be 0.
remainder when divided by 2 would be 0. 41.
(a) Since he has to put minimum 120 oranges and
Of 1, 3, 5 only for 5, division by 3 has remainder 2. maximum 144 oranges, i.e., 25 oranges need to be
\ Remainder when divided by 6 = 5. filled in 128 boxes with same number of oranges in
the boxes.
31.
(c) Total possible arrangements = 10 × 9 × 8
Therefore, total 125 = 25 × 5 oranges could be filled in
Now three numbers can be arranged among themselves the boxes, i.e., 25 in each of the 5 boxes which would
in = 3! ways = 6 ways. be the minimum and have the same number of oranges.
Given condition is satisfied by only 1 out of 6 ways. Hence, the answer is 5.
Hence, required number of arrangements.
42.
(a) Let the 4-digit number be abcd.
10 × 9 × 8 Then, a + b = c + d (1)
= = 120.
6 b + d = 2 (a + c) (2)
32.
(b) Check choices and, a + d = c (3)
54 ⇒ S = (5 + 4)2 = 81 From Eqs. (1) and (3), b = 2d
⇒ S – D = 81 – 54 = 27. From Eqs. (1) and (2), 3b = 4c + d

Chapter 01.indd 27 6/3/2015 6:21:41 PM


1.28 Chapter 1

⇒ 3(2d) = 4c + d 1 1 1 1
⇒ 5d = 4c 49.
(c)    ... 
1.3 3.5 5.7 19.21
5 1 1 11 1 11 1 1 1 1 
⇒ c = d =
4 1 −  +  −  +  −  + ...  − 
2 3 2 3 5 2 5 7  2  19 21 
Now d can be 4 or 8. But if d = 8, then c = 10 is not
1 1 (21 − 1) 20 10 10
possible. So, d = 4, which gives c = 5. = − = = = = .
2 42 42 42 21 21
43.
(d) Let the number be x
Increase in product = 53x – 35x = 18x 50.
(b) Use any 7 consecutive numbers to check the answers.
⇒ 18x = 540 ⇒ x = 30 (1  2  3  4  5)
n = = 3, average of 7 integers is
Raised product = 53 × 30 = 1590. 5
(1  2  3  4  5)
(d) Let,   y = n3 – 7n2 + 11n – 5
44. k = = 4. So k = n + 1.
7
At   n = 1, y = 0
\ (n – 1) (n2 – 6n + 5) = (n – 1)2 (n – 5) 51.
(c) N = 1421 × 1423 × 1425. When divided by 12, it shall
Now, (n – 1)2 is always positive. [(1416 + 5) × (1416 + 7) × (1416 + 9)]
look like .
Also, for n < 5, the expression gives a negative 12
quantity. Therefore, the least value of n will be 6. Now the remainder will be governed by the term 5 ×
Hence, m = 6. 7 × 9, which when divided by 12 leaves the remainder 3.
45.
(d) Let there be x mints originally in the bowl. 52.
(d) Let r be the remainder. Then, 34041 – r and 32506 –
1 r are perfectly divisible by n. Hence, their difference
Sita took , but returned 4 should also be divisible by the same.
3
2 (34041 – r) – (32506 – r) = 1535
So, now the bowl has x  4 mints.
3 which is divisible by only 307.
1
Fatima took of remainder, but returned 3 53.
(d) Any triplet of the form (– n, 0, n) satisfies the given
4
condition, e.g., (–2, 0, 2).
3 2 
So, the bowl has x + 4  + 3 mints. (d) N can be written either (54 + 1)3 + (18 – 1)3 – 723 or
54.
4  3 
(51 + 4)3 + 173 – (68 + 4)3.
Eswari took half of remainder that is The first form is divisible by 3, and the second by 17.
1 3 2  
55.
(a) 36 = 2 × 2 × 3 × 3
  x + 4  + 3 . She returns 2, so the bowl now
2 4  3   Hence, divisors of
1 3 2   36 = 1, 2, 3, 4, 6, 9, 12, 18, 36, i.e., 9 in all.
has  x + 4  + 3 + 2 = 17 ⇒ x = 48.
2  4  3   76 = 2 × 2 × 19
Hence, divisors of
46.
(a) The product of 44 and 11 is 484
76 = 1, 2, 4, 19, 38, 76, i.e., 6 in all.
Here, 3x3 + 4x2 + 1x1 + 4 × x0 = 484
82 = 2 × 41
⇒ 3x3 + 4x2 + x = 480
Hence, divisors of
This equation is satisfied only when x = 5. 82 = 1, 2, 41, 82, i.e., 4 in all
In decimal system, the number 3111 can be written 191 = 1 × 191
as 406.
Hence, divisors of
47.
(a) Let the highest number be n. 191 = 1, 191, i.e., 2 in all.
n (n + 1)
−x 56.
(c) 58 = 2 × 29
2 7 602
Then, = 35 = ,
(n − 1) 77 17 Hence, divisors of
where x is the number erased. 58 = 1, 2, 29, 58, i.e., 4 in all
Hence, n = 69 and x = 7 satisfy the above conditions. 88 = 2 × 2 × 2 × 11
a1a2 Hence, divisors of
48.
(c) 99 × D = a1a2. Hence, D = . So, D must be
99 88 = 1, 2, 4, 8, 11, 22, 44, 88, i.e., 8 in all
multiplied by 198 as 198 is a multiple of 99. 137 = 1 × 137

Chapter 01.indd 28 6/3/2015 6:21:41 PM


Numbers 1.29

Hence, divisors of 64.


(b) All the options satisfy the first condition. So, testing
137 = 1, 137, i.e., 2 in all the options for second condition, only option (b), i.e.,
24 satisfies the second condition, i.e., 24 + 18 = 42.
Hence, divisors of
1 2 n 1
× ×...
184 = 1, 2, 4, 8, 23, 46, 92, 184, i.e., 8 in all. 65.
(a) The given expression = 2 2 3 n +1 = (2) n  1
57.
(d) Total number of numbers, which end with 4 = 9 66.
(d) The number is neither divisible by 3 nor by 11.
Total numbers from 1 to 90 = 90 67.
(d) p may or may not be divisible by 24. But, p is never
9 divisible by 22 because
Therefore, required percentage = = 10%
90 23n + 1 = (22 + 1)n + 1 = 22k + 2.
58.
(b) There are 99 – 10 + 1 = 90 two digit numbers in all. 68.
(a) 4 a 3
We can have 0–9 digits at unit’s place. For 0 in unit’s
place we can have 1–9 digits at tens place, i.e., we 984
13 b 7
have 9 choices. For 1 in unit’s place we have 8 choices
and so on. Hence, total numbers satisfying given As 13 b 7 is divisible by 11
condition = 9 + 8 + . . . + 1 = 45.
\ a = 1 and, b = 9
59.
(b) Squaring both sides of the given equation \ a + b = 1 + 9 = 10.

x2 =
5

13
 6x 69.
(b) Given h t u
2 4 − u t h
5 13 − − 4
⇒ x2 − =  6x
2 4 The difference between a three-digit number and its
2 reverse is always a multiple of 99. The only multiple
⇒  2 5  = 13 of 99 and less than 1000 that ends in 4 is 594. Thus,
x −   6x
 2 4 the remaining two-digits in that order are 9 and 5.
(Squaring both sides again) 70.
(a) In base 4, the 20th number will be
Going by the choices, only x = 3  5 satisfies the = 42 (1) + 41 (1) + 40 (0) = 110.
2 71.
(b) Let the two-digit numbers = xy
equation above.
The square of xy = (10x + y)2
60. (b) Assume the number of rows be n.
The square of the number formed by reversing the
Then, n × n = 19044 digits of xy = (10y + x)2
or, n = 138. (10x + y)2 – (10y + x)2 = 99 (x2 – y2)

0
61. (a) n = 0 ⇒ 35 + 1 = 4, n = 1 ⇒ 35 + 1 = 244
1
= 99 (x – y) (x + y)
The remainders can be seen to be R = 4, when n = 0, Thus, it will always be divisible by 9, the sum of the
i.e., even and R = 6 when n = 1, i.e., odd. Therefore, digits as well as the difference of the digits. But, it is
I and III are true. not divisible by their product xy.
62. (d) 2x – 1 is an odd number. 72.
(b) The value of the three-digit number tu 1
⇒ x can be either odd or even. = 100t + 10u + 1.
73.
(d) Factors of 6 are 1, 2, 3. Now 1 + 2 + 3 = 6.
3y – 1 is an even number.
74.
(c) By hit and trial, we see that n = 10 and n = 16 satisfy
⇒ y is an odd number.
the conditions.
I. In x2 – 2y + 2, 2y is even, but x2 can be either odd or
75.
(a) Given, 2x + p = 2y, p + y = x
even, so we can not say whether
and, x + y = z
x2 – 2y + 2 is odd or even.
⇒ x + y + z = 2z = 2(x + y)
II. In y2 – 2x + 3, y2 is odd, 2x is even and 3 is odd
So, p = x – y = 2(y – x)
⇒ y2 – 2x + 3 is even.
The condition is satisfied only when x = y
III. In 4x2 – y – 1, 4x2 is even, y is odd and 1 is odd
Then,   p = 0
⇒ 4x2 – y – 1 is even.
76.
(a) We know that for any rational number,
(c) For any integer n, n3 – n is divisible by 3, n5 – n is
63.
divisible by 5, n11 – n is divisible by 11 but n4 – n is not 1
< 1
necessarily divisible by 4. Thus, statement III is true. x 1

Chapter 01.indd 29 6/3/2015 6:21:42 PM


1.30 I Chapter 1

x 84. (a) 6 + 7 + 3 + 5 + K + 1 = 22 + K
and, < 1 The least number greater than 22 and divisible by 9
x 1
is 27.
Hence, (a) is the correct answer.
\ 27 = 22 + K
77.
(a) A0 = 1994, which is not divisible by 10.
⇒ K = 5.
Hence, f (A0) = A0 + 1 = 1995. Since,
85. (a) We know that,
Am+1 = f (Am) ⇒ A1 = f (A0) = 1995, similarly A2 = 1996,
A3 = 1997, A4 = 1998, A5 = 1999, A6 = 2000, which a n − b n is divisible by (a + b) and (a − b) if n is even.
2000 Therefore, 774 − 574 is divisible by 12 and 2, and as a
is divisible by 10. Hence, f (A6) = = 200 = A7 result by 4.
10
similarly A8 = 20 and A9 = 2. 86. (b) We know that number of factors in a perfect square is
78.
(d) Four digits of the number 29138576 are omitted so always even. So factors in N2 is an even number.
that result is large. Now, factors of N2 = 34 – an even number
 = even number
\ Omitted digits are 1, 2, 3, 5
So, N is a perfect square too.
Hence, the largest omitted digit is 5.
Now to find out the actual values of N, we consider
79.
(a) A < B that if the factors of N are of form a x × b y then factors
A < D < B, C is the greatest integer. of N2 will be a 2 x × b 2 y
\ With the help of 1st statement E + B < A + D, the
So, sum of factors will be (x + 1)(y + 1) + (2x + 1)
result can be obtained. (2y + 1) = 34. Only value which satisfies this equation
(c) x2 + 4xy + 6y2 – 4y + 4
80. is x = 2, y = 2.
(x, y are obviously positive integers)
= ( x) 2  2 2 x 2 y  (2 y ) 2  ( 2 y ) 2
For N < 150, we have only N = 36 and N = 100 (check
− 2 2 y 2 + ( 2) 2 + 2 for perfect squares)

So, the answer is 2.
= ( x + 2 y ) 2 + ( 2 y − 2) 2 + 2
87. (d) Total number of digits when 21 and 51 are written side
Now, on putting the value of x = – 2 and y = 1, we get by side
the minimum value of expression. (25) = (1 + 1)
81.
(a) Numbers can be 1112, 1113, 1115, 1117. Total numbers of digits when 22 and 52 are written one
after another
82.
(a) The metre skips all the numbers in which there is a 5.
From 0000 to 0099, 5 occurs 10 times in the tens place (425) = (2 + 1)
and 10 times in the units place, (which includes the Total number of digits when 23 and 53 are written one
number 55). after another
\ It occurs in a total of 10 + 10 – 1 numbers, i.e., 19
(8125) = (3 + 1) and so on
numbers. Similarly, from 0100 to 0199, from 0200 Therefore, the total number of digits when 22004 and
to 0299, 0300 to 0399 from 0400 to 0499, 0600 to 0699, 52004 are written one after another
... 0900 to 0999. It occurs in 8 (19) numbers. From 0500
2004 + 1 = 2005
to 0599, there are 100 numbers. The micromanometer
reading could change from 0499 to 0600. 88. (d) We have to find the number of prime numbers from
101 to 200, which is 21.
Total number of numbers skipped from 0000 to 0999
89. (b) The numbers are of the from 8n + 6
= 19 (9) + 100 = 271
Therefore, sum of all such numbers is
Similarly, from 1000 to 1999 and from 2000 to 2999, 11
 11 
271 + 271 numbers are skipped. Finally, 3005 and ∑ (8n + 6) = 6(11) + 8   (12) = 594.
2
3015 are also skipped. n =1

\ Total number of skips = 271 (3) + 2 = 815 90. (a) We have


\ Actual pressure = 3016 – 815 = 2201. 21040 ( 28 )130 ( 256)130
= =
83.
(d) I has 741 ones, II has 534 ones and III has 123 ones. Sum 131 131 131
of the digits of each of I, II, and III is divisible by 3. The remainder is 1.
\ I, II, III are all divisible by 3 and hence not prime.
Shortcut: Whenever an is divided by (n + 1), where
Choice (d) follows, since only this supports that all n + 1 is prime and relatively prime to a, the remainder
these three are not prime. is always 1.

Chapter 01.indd 30 6/3/2015 6:21:42 PM


Numbers I 1.31

91. (d) Clearly Statements I and II are wrong, since when Now, the value of Y should be an integer. For X = 18,
p is prime number so it does not have any factor. Y is an integer 56. Hence, option (c) is the correct
Therefore, x = 1.2….(p − 1) is not divisible by p or choice.
any prime number greater than p. Statement III is 97. (a) Let the last number of the series be n and number
wrong, as 1.2.3.4.5.6 is divisible by 5. erased be x, then
But Statement IV is correct. n( n + 1)
−x
92. (c) Let x, y and z be the hundredth, tens and unit digits of 2 602
=
the original number. n −1 17
We are given, n( n − 1) − 2 x 602
⇒ =
(100z + 10y + x) – (100x + 10y + z) = 594 2( n − 1) 17
⇒ 99(z − x) = 594 ⇒ (z − x) = 6
From the options, we find that x = 7, n is an integer
So, the possible values of (x, z) are (1, 7), (2, 8) and (3, 9) i.e., 69.
As the ten digits can have any values from 0, 1, 2,…, 9, 98. (d) Let x mints were originally in the bowl.
∴ Minimum values for their sum = x + y + z = 1 + 0  x
+ 7 = 8. Number of mints before Eswari took =  x −  + 2 = 17
 2 
93. (a) We have ⇒ x = 30
223 + 233 + 243 + + 873 + 8833  x
Number of mint before Fatima took =  x −  + 3 = 30
= (223 + 883) + (233 + 873) + (243 + 863) + + (543 +  4
⇒ x = 36
563) + 553
 x
Now, all the terms except 553 is divisible by 110 Number of mint before Sita took =  x −  + 4 = 36
 3
[Shortcut: an + bn is divisible by (a + b) when n is an ⇒ x = 48
odd number.] Hence, there were 48 mints originally.
Therefore, the required remainder when the given 99. (d) The frog can move either clockwise or anticlockwise
expression is divided by 110 is 55. in order to reach point E. In any case number of jumps
94. (a) Let p + q = α and r + s = β required is 4.
Given: p + q + r + s = 2 For, n = 4, a2n −1 = a8 −1 = 7.
So, α + β = 2 and α β > 0
100. (d) N = 553 + 173 − 723 = (54 + 1)3 + (18 − 1)3 − 723
Since AM ≥ GM
= (51 + 4)3 + 173 − (68 + 4)3
α +β
⇒ ≥ αβ These two different forms of given expressions are
2
divisible by 3 and 17 both.
⇒ 1 ≥ αβ
101. (d) Let the common remainder be x.
On squaring both sides, we get Then, the numbers (34041 – x) and (32506 – x) would
1 ≥ αβ be completely divisible by n. Hence, the difference of

the numbers i.e., (34041 – x) and (32506 – x) will also
⇒ αβ = m ≤ 1
be divisible by n or (34041 – x – 32506 + x) = 1535
∴0≤m≤1 will also be divisible by n.
95. (d) Any four digit number in which the first two digits and Now, using options, we find that 1535 is divisible
last two digits are equal will be of the form 11 × (100 + by 307.
b) i.e., it will be a multiple of 11 like 1122, 3366, 2244,
. . .  Now, let the required, number be aabb. Since, aabb 102. (c) D = 0.a1a2
is a perfect square, the only pairs, of values a and b, 100 D = a1a2 ⋅ a1a2
that satisfy the above mentioned condition is a = 7 and
a1a2
b = 4. Hence, 7744 is a perfect square. ∴ 99 D = a1a2 ⇒ D= .
99
96. (c) Suppose the cheque for Shailaja is of ` X and Y paise
It is given that Required number should be the multiple of 99.
Hence, 198 is the required number.
3 × (100 X + Y ) = (100Y + X ) − 50
300 X + 3Y = 100Y + X − 50 103. (a) We have,
299 X = 97Y − 50 x = 1 × 2 × 3 × 4 = 24
299 X + 50 (We have taken the four consecutive integers to be 1,
∴Y= .
97 2, 3 and 4)

Chapter 01.indd 31 6/3/2015 6:21:44 PM


1.32 Chapter 1

∴ n = 1 + 24 = 25, we find that n is odd and a perfect 114. (c) Let us solve the question for any two odd numbers
square. This is true for any set of four consecutive greater then 1 i.e., 3 and 5 then
positive integers.
n( n2 − 1) for n = 3 = 3 × 8 = 24
104. (b) According to the remainder theorem, the following
expression will have the same remainder, n( n2 − 1) for n = 5 = 5 × 24 = 120
(7)84 (73 ) 28 (343) 28 From the options, we find that both the numbers are
or or ⇒ Remainder = 1.

342 342 (342) divisible by 24.

105. (a) (ab)2 = ccb. The greatest possible value of ‘ab’ to be 115. (b) The number 77958A96B is divisible by 8 if 96B
31. Since (31)2 = 961 and ccb > 300, 300 < ccb < 961, divisible by 8. And 96B is divisible by 8 if B is either
0 or 8
so 18 < ab < 31. So, the possible value of ab that
satisfies (ab)2 = ccb is 21. So, (21)2 = 441 Now to make the same number divisible by 9, sum of
all the digits should be divisible by 9. Hence, (55 +
∴ a = 2, b = 1 and c = 4. A + B) is divisible by 9 if (A + B) is either 0 or 8

⇒ either A = 0 and B = 8 or A = 8 and B = 0
106. (a) We know that any number is divisible by 8, if the
number formed by the last three digits is divisible Since, the number is divisible by both A and B. Hence,
by 8. And the same rule will be applicable to find the A and B may take either values i.e., 8 or 0
remainder.
  Now, the last three digits in the hundred digit 116. (b) Let the three even numbers be ( x − 2), x, ( x + 2)
number of the form 1234567891011121314 . . . is 545. Then, we are given
Therefore, the remainder when 545 is divided by 8 is 1.
3( x − 2) − 2( x + 2) = 2
107. (a) Dividend = Divisor × Quotient + Remainder
⇒ 3x − 6 − 2 x − 4 = 2
= 899Q + 63
⇒ x = 12
Dividend = 29 × 31Q + 29 × 2 + 5 = 29(31Q + 2) + 5.
∴ The third number = (12 + 2) = 14.
108. (b) Unit digit in= , ( 2)8 6, ( 2)16 = 6. Hence, 2 has
( 2) 4 6=
a cyclicity of four. Hence, unit digit in (2)48 = 6 117. (b) Let us solve the question for some prime numbers
Therefore, unit digit in (2) 51
= (2) 48 3
× (2) = 6 × greater than 6 i.e., 7, 11, 13 and 7. If these numbers
8  ⇒ 8 are divided by 6, the remainder is always either 1 or 5.

109. (c) If n3 is odd, then n and n2 will be odds. It can be 118. (a) 553 + 453
checked for any odd integer. =
If n 3=
, n2 9, n3 = 27. 55 − 55 × 45 + 452
2

(55 + 45)(552 − 55 × 45 + 452 )


=
110. (b) 73 = 79 and (73 ) 2 = 76. Hence, clearly 79 > 76. (552 − 55 × 45 + 452 )
2

= (55 + 45)
111. (c) Let m = 10 and n = 5, then (m – n) = (10 – 5) = 5,
which is divisible by 5 = 100.

( m 2 − n2 ) = (100 − 25) = 75, which is divisible by 5 119. (a) There are 50 odd numbers less than 100 which are not
divisible by 2.
( m + n) = (10 + 5) = 15, which is divisible by 10.
Out of these 50, 17 numbers are divisible by 3
112. (d) (P, Q) may be any of the following: Out of remaining, 7 numbers are divisible by 5
(1, 64), ( 2, 32), ( 4, 16), (8, 8). Hence, P + Q cannot Hence, numbers which are not divisible by 2, 3, and 5
be 35. = (50 – 17 – 7) = 26
(16 n2 + 7n + 6) 6
113. (c) = 16 n + 7 +   . 120. (c) The last two digits of the number in the expansion is
n n (7)4 = 01(2401) and if the power of 7 is any multiple
Since n is an integer, hence for the entire expression of 4, the last two digits will not change
6 i.e., (7)4 = 2401 ⇒ 01
to become an integer   should be an integer. And
n
 6  can be integer for n = 1, 2, 3, 6. Hence, n will (7)8 = 5764801 ⇒ 01
n
  Since, power of 7, i.e., 2008 is a multiple of 4, the last
have four values. two digits of (7)2008 will be 01.

Chapter 01.indd 32 6/3/2015 6:21:45 PM


Numbers 1.33

121. (c) Let the total number of sweets be ( 25 x + 8) Since no unique solution is possible, Statement II is
not sufficient.
Then (25x + 8) − 22 is divisible by 28
⇔ ( 25 x − 14) is divisible by 28 ⇔ 28x − (3x + 14) is 123. (b) Between 100 and 199, there will be 19 numbers which
contain ‘2’. They are as follows:
divisible by 28
100, 112, 120–129 (10 numbers), 132, 142, 152, 162,
⇔ (3x + 14) is divisible by 28 ⇔ x = 14
172, 182, 192
∴ Total number of sweets = ( 25 × 14 + 8) = 358
Similar would be the case for 300 – 339, 400 – 499,
122. (a) Considering Statement I: 500 – 599, 600 – 699
For 200–299, all 100 numbers will have 2
x y x+y x−y
∴  Total number of numbers containing ‘2’ = 19 × 6 +
5 2 7 3 100 = 114 + 100 = 214.
Since this the only possible solution, Statement I is 124. (c) As p, q, r are non-negative integers, the maximum
sufficient. will be achieved when the value of each variable is
Considering Statement II: close to each other.
i.e., p, q are 3, 3, 4 (not necessarily in the same order).
x y x+y x−y
Hence the value of
5 2 8 2
pq + qr + pr + pqr = 3 × 3 + 3 × 4 + 3 × 4 + 3 × 3 × 4
6 4 10 2 = 9 + 12 + 12 + 36 = 69.

Chapter 01.indd 33 6/3/2015 6:21:45 PM


This page is intentionally left blank

FM.indd 6 6/10/2015 2:36:16 PM


H.C.F. and L.C.M.
CHAPTER

of Numbers 2
COMMON FACTOR Illustration 3 Find the H.C.F. of 360 and 132
A common factor of two or more numbers is a number Solution: 360 = 23 × 32 × 5
which divides each of them exactly. 132 = 22 × 31 × 11
For example, 4 is a common factor of 8 and 12.
\ H.C.F. = 22 × 31 = 12.
HIGHEST COMMON FACTOR
Illustration 4 If x = 23 × 35 × 59 and y = 25 × 37 × 511, fi nd
Highest common factor (H.F.C.) of two or more numbers is H.C.F. of x and y
the greatest number that divides each one of them exactly.
Solution: The factors common to both x and y are 23, 35
For example, 6 is the highest common factor of 12, 18 and
and 59
24. Highest common factor is also called Greatest Common
Divisor or Greatest Common Measure. \ H.C.F. = 23 × 35 × 59
Symbolically, these can be written as H.C.F. or G.C.D. or
G.C.M., respectively. II. Method of Division
A. For two numbers:
MetHoDs of finDinG H.c.f.
Step 1 Greater number is divided by the smaller one.

I. Method of Prime Factors Step 2 Divisor of (1) is divided by its remainder.


Step 1 Express each one of the given numbers as the Step 3 Divisor of (2) is divided by its remainder.
product of prime factors.
This is continued until no remainder is left.
[A number is said to be a prime number if it is exactly
divisible by 1 and itself but not by any other number, H.C.F. is the divisor of last step.
e.g. 2, 3, 5, 7, etc. are prime numbers]
Step 2 Choose common factors.
Step 3 Find the product of these common factors. Illustration 5 Find the H.C.F. of 3556 and 3444
This is the required H.C.F. of given numbers.
3444 ) 3556 ( 1
Illustration 1 Find the H.C.F. of 70 and 90 3444
Solution: 70 = 2 × 5 × 7 112 ) 3444 ( 30
90 = 2 × 5 × 9 3360
Common factors are 2 and 5 84 ) 112 ( 1
\ H.C.F. = 2 × 5 = 10 84
Illustration 2 Find the H.C.F. of 3332, 3724 and 4508
28 ) 84 ( 3
Solution: 3332 = 2 × 2 × 7 × 7 × 17
3724 = 2 × 2 × 7 × 7 × 19 84
4508 = 2 × 2 × 7 × 7 × 23 ×
\ H.C.F. = 2 × 2 × 7 × 7 = 196 \ H.C.F. = 28

Chapter 02.indd 1 6/4/2015 11:31:38 AM


2.2 Chapter 2

COMMON MULTIPLE
B. For more than two numbers:
A common multiple of two or more numbers is a number
Step 1 Any two numbers are chosen and their H.C.F. which is exactly divisible by each one of them.
is obtained.
For example, 32 is a common multiple of 8 and 16.
Step 2 H.C.F. of H.C.F. (of (1)) and any other 18 × 4 = 32
number is obtained. 16 × 2 = 32.
Step 3 H.C.F. of H.C.F. (of (2)) and any other LEAST COMMON MULTIPLE
number (not chosen earlier) is obtained. The least common multiple of two or more given numbers
This process is continued until all numbers have been is the least or lowest number which is exactly divisible by
chosen. H.C.F. of last step is the required H.C.F. each of them.
For example, consider the two numbers 12 and 18.
Multiples of 12 are 12, 24, 36, 48, 72, ...
Illustration 6 Find the H.C.F. of 13915, 9499 and 2553 by Multiples of 18 are 18, 36, 54, 72, ...
division method Common multiples are 36, 72, ...
Solution: \ Least common multiple, i.e., L.C.M. of
12 and 18 is 36.

MetHoDs of finDinG l.c.M.

A. Method of Prime Factors


Step 1 Resolve each given number into prime factors.
Step 2 Take out all factors with highest powers that
occur in given numbers.

Step 3 Find the product of these factors. This


product will be the L.C.M.

Illustration 8 Find the L.C.M. of 32, 48, 60 and 320


Solution: 32 = 25 × 1
48 = 24 × 3
60 = 22 × 3 × 5
Now, in the next step, we will fi nd the H.C.F. of 23 and 320 = 26 × 6
2553 \ L.C.M. = 26 × 3 × 5 = 960

B. Method of Division
Step 1 The given numbers are written in a line
separated by common.
Step 2 Divide by any one of the prime numbers 2, 3, 5,
7, 11, . . . which will divide at least any two of the given
Thus, H.C.F. of 13915, 9499 and 2553 = 23 numbers exactly. The quotients and the undivided
numbers are written in a line below the fi rst.
Illustration 7 Find the greatest possible length which can
be used to measure exactly the lengths 7 m, 3 m 85 cm, Step 3 Step 2 is repeated until a line of numbers
12 m 95 cm. (prime to each other) appears.
Solution: Required length Step 4 Find the product of all divisors and numbers
in the last line which is the required L.C.M.
= (H.C.F. of 700, 385, 1295) cm = 35 cm

Chapter 02.indd 2 6/4/2015 11:31:39 AM


H.C.F. and L.C.M. of Numbers 2.3

Illustration 9 Find the L.C.M. of 12, 15, 20 and 54.


Note:
Solution: 2 12, 15, 20, 54
2 6, 15, 10, 27 Before fi nding the L.C.M. or H.C.F., we must ensure that
all quantities are expressed in the same unit.
3 3, 15, 5, 27
5 1, 5, 5, 9
1, 1, 1, 9

L.C.M. = 2 × 2 × 3 × 5 × 1 × 1 × 1 × 9 = 540

soMe useful sHort-cut MetHoDs

\ H.C.F. of 616 and 1300 is 4


1. H.C.F. and L.C.M. of Decimals
Thus, the required H.C.F. = 0.04
Step 1 Make the same number of decimal places in all
the given numbers by suffi xing zero(s) if necessary. 2. L.C.M. and H.C.F. of Fractions
Step 2 Find the H.C.F./L.C.M. of these numbers L.C.M. of the numbers in numerators
without decimal. L.C.M =
H.C.F. of the numbers in denominators
Step 3 Put the decimal point (in the H.C.F./L.C.M. H.C.F. of the numbers in numerators
of step 2) leaving as many digits on its right as there H.C.F. =
are in each of the numbers. L.C.M. of the numbers in denominators

Illustration 10 Find the L.C.M. of 1.2, 0.24 and 6 2 3 6


Illustration 12 Find the L.C.M. of , and
Solution: The given numbers can be written as 5 10 25
Solution: L.C.M. of numerators 2, 3 and 6 is 6
1.20, 0.24 and 6.00
H.C.F. of denominators 5, 10 and 25 is 5
Now, ignoring the decimal we fi nd the L.C.M. of L.C.M. of numerators
120, 24 and 600 \ Required L.C.M. =
H.C.F. of denominators
2 120, 24, 600 6
=
2 60, 12, 300 5
2 30, 6, 150 4 10 20
Illustration 13 Find the H.C.F. of , and
3 15, 3, 75 9 21 63
Solution: H.C.F. of numerators 4, 10 and 20 is 2
5 5, 1, 25
L.C.M. of denominators 9, 21 and 63 is 63
1, 1, 5
H.C.F. of numerators
\ Required H.C.F. =
\ L.C.M. = 2 × 2 × 2 × 3 × 5 × 1 × 5 = 600 L.C.M. of denominators
Thus, the required L.C.M. = 6.00, i.e., 6 2
=
63
Illustration 11 Find the H.C.F. of 6×16 and 13
Solution: The given numbers can be written as Notes:
6.16 and 13.00
Now, ignoring the decimals we fi nd the H.C.F. of 616 1. If the given set of numbers includes fractions as well
and 1300 as whole numbers, treat whole number too as fraction
with 1 in its denominator.
2. The H.C.F. of a number of fractions is always a fraction,
but the L.C.M. may be a fraction or an integer.
3. Product of two numbers
= L.C.M. of the numbers × H.C.F. of the numbers

Chapter 02.indd 3 6/4/2015 11:31:39 AM


2.4 Chapter 2

Illustration 14 The H.C.F. and the L.C.M. of any two


8. To fi nd the least number which when divided by
numbers are 63 and 1260, respectively. If one of the two
x, y and z leaves the same remainder r in each case.
numbers is 315, fi nd the other number
Required number = (L.C.M. of x, y and z) + r.
Solution: The required number
L.C.M.  H.C.F. 1260  63 Illustration 19 Find the least number which when divided
= = = 252
First number 315 by 12, 16 and 18, will leave in each case a remainder 5

4. To fi nd the greatest number that will exactly Solution: The required smallest number
divide x, y and z. = (L.C.M. of 12, 16 and 18) + 5
Required number = H.C.F. of x, y and z. = 144 + 5 = 149

Illustration 15 Find the greatest number that will exactly 9. To fi nd the greatest number that will divide x, y
divide 200 and 320 and z leaving the same remainder in each case.
Solution: The required greatest number (A) When the value of remainder r is given:
= H.C.F. of 200 and 320 = 40
Required number = H.C.F. of (x – r), (y – r)
5. To fi nd the greatest number that will divide x, y and (z – r).
and z leaving remainders a, b and c, respectively. (B) When the value of remainder is not given:
Required number = H.C.F. of (x – a), (y – b) and Required number = H.C.F. of | (x – y) |, | (y – z) |
(z – c). and | (z – x) |

Illustration 16 Find the greatest number that will Illustration 20 Find the greatest number which will divide
divide 148, 246 and 623 leaving remainders 4, 6 and 11, 772 and 2778 so as to leave the remainder 5 in each case
respectively
Solution: The required greatest number
Solution: The required greatest number = H.C.F. of (772 – 5) and (2778 – 5)
= H.C.F. of (148 – 4), (246 – 6) and (623 – 11), = H.C.F. of 767 and 2773
i.e., H.C.F. of 144, 240 and 612 = 12 = 59
Illustration 21 Find the greatest number which on dividing
6. To fi nd the least number which is exactly
152, 277 and 427 leaves equal remainder.
divisible by x, y and z.
Solution: The required greatest number
Required number = L.C.M. of x, y and z.
= H.C.F. of |(x – y)|, |(y – z)| and |(z – x)|
Illustration 17 What is the smallest number which is = H.C.F. of |(152 – 277)|, |(277 – 427)|
exactly divisible by 36, 45, 63 and 80? and |(427 – 152)|
Solution: The required smallest number = H.C.F. of 125, 275 and 150
= L.C.M. of 36, 45, 63 and 80 = 25
= 5040
10. To fi nd the n-digit greatest number which, when
7. To fi nd the least number which when divided divided by x, y and z,
by x, y and z leaves the remainders a, b and c,
(A) leaves no remainder (i.e., exactly divisible)
respectively. It is always observed that (x – a) =
(y – b) = (z – c) = k (say) Step 1 L.C.M. of x, y and z = L
\ Required number = (L.C.M. of x, y and z) – k. Step 2 L ) n-digit greatest number (

Illustration 18 Find the least number which when divided Remainder = R


by 36, 48 and 64 leaves the remainders 25, 37 and 53, Step 3 Required number = n-digit greatest number
respectively –R
Solution: Since (36 – 25) = (48 – 37) = (64 – 53) = 11, (B) leaves remainder K in each case.
therefore, the required smallest number Required number = (n-digit greatest number
= (L.C.M. of 36, 48 and 64) – 11 – R) + K.
= 576 – 11 = 565

Chapter 02.indd 4 6/4/2015 11:31:41 AM


H.C.F. and L.C.M. of Numbers 2.5

Illustration 22 Find the greatest number of 4 digits which,


when divided by 12, 18, 21 and 28, leaves 3 as a remainder (B) leaves remainder K in each case.
in each case. Required number = n-digit smallest number
+ (L – R) + k.
Solution: L.C.M. of 12, 18, 21 and 28 = 252
252 ) 9999 ( 39
9828 Illustration 24 Find the least number of four digits which
171 is divisible by 4, 6, 8 and 10.
\ The required number = (9999 – 171) + 3 = 9931 Solution: L.C.M. of 4, 6, 8 and 10 = 120.
Illustration 23 Find the greatest number of 4 digits which,
120 ) 1000 ( 8
when divided by 12, 15, 20 and 35 leaves no remainder
Solution: L.C.M. of 12, 15, 20 and 35 = 420 960

420 ) 9999 ( 23 40
9660 \ The required number = 1000 + (120 – 40) = 1080.
339
Illustration 25 Find the smallest 4-digit number, such that
\ The required number = 9999 – 339 = 9663 when divided by 12, 18, 21 and 28, it leaves remainder 3 in
each case
11. To fi nd the n-digit smallest number which when
divided by x, y and z Solution: L.C.M. of 12, 18, 21 and 28 = 252
(A) leaves no remainder (i.e., exactly divisible) 252 ) 1000 ( 3
Step 1 L.C.M. of x, y and z = L 756
Step 2 L ) n-digit smallest number ( 244
Remainder = R
\ The required number
Step 3 Required number = n-digit smallest number
+ (L – R). = 1000 + (252 – 244) + 3
= 
Practice Exercises

Difficulty level-1
(BaseD on MeMory)

1. The L.C.M. of two numbers is 4800 and their HCF is 160. 4. Find the number of four-digit numbers that are divisible
If one of the numbers is 480, then the second number is: by 30 and 35 but not by 140.
(a) 16 (b) 16000 (a) 21 (b) 22
(c) 160 (d) 1600 (c) 43 (d) 44
[Based on MAT, 2004] 7 33 28 15
5. Find the L.C.M. of , , and .
2. What is the least number which when divided by 12, 18, 14 21 3 63
36, and 45 leaves remainders 8, 14, 32 and 41, respectively? (a) 220 (b) 4260
(a) 176 (b) 88 (c) 356 (d) None of these
(c) 98 (d) 42 6. Find the least common multiple of 6804 and 9828 given
their highest common factor is 756.
3. An electronic device makes a beep after every 60 s.
(a) 84852 (b) 88452
Another device makes a beep after every 62 s. They
beeped together at 10 a.m. The time when they will next (c) 85482 (d) 88542
make a beep together at the earliest is: 7. H.C.F. of two numbers is 43 and their sum is 430. Total
(a) 10.30 a.m. (b) 10.31 a.m. number of distinct pairs of two such numbers is:
(c) 10.59 a.m. (d) 11 a.m. (a) 5 (b) 2
[Based on MAT, 2005] (c) 6 (d) Data insuffi cient

Chapter 02.indd 5 6/4/2015 11:31:41 AM


2.6 Chapter 2

8. Find the greatest possible length of the planks, if three 17.


The smallest perfect square number which is divisible by
pieces of timber 42 m, 49 m and 63 m long have to be 8, 12 and 15 is:
divided into planks of the same length? (a) 2,500 (b) 3,600
(a) 8 m (b) 49 m (c) 3,721 (d) 6,400
(c) 7 m (d) 63 m [Based on MAT, 2000]
9. A rectangular floor in my office has its area equal to 56
m2. The minimum number of tiles required, if all the tiles 18.
The greatest common divisor of 123456789 and
are in square shape is: 987654321 is:
(a) 15 (b) 9 (a) 1 (b) 3
(c) 14 (d) Cannot be determined (c) 9 (d) Greater than 9
10. H.C.F. and L.C.M. of 24, 82, 162, 203 are: [Based on JMET, 2011]
(a) 23; 32000 (b) 24; 32000
(c) 24; 25600 (d) 22; 3200 19. What is the least number of cut pieces of equal length that
can be cut out of two lengths 10 m 857 mm and 15 m
11.
L.C.M. of 35,85 and a number k is 7,735. The H.C.F. is 5. 87 mm?
What is the least possible value of k?
(a) 174 (b) 172
(a) 65 (b) 25
(c) 325 (d) 13 (c) 164 (d) 184

12.
HCF and LCM of two numbers are 21 and 4641, 20. L.C.M. of two numbers is 12 times their H.C.F. The sum
respectively. If one of the numbers lies between 200 and of H.C.F. and L.C.M. is 403. If one number is 93, find the
300, then the two numbers are: other.
(a) 273, 357 (b) 273, 361 (a) 134 (b) 124
(c) 273, 359 (d) 273, 363 (c) 128 (d) None of these
[Based on MAT (May), 2006]
[Based on NMAT, 2006]
13.
LCM and HCF of two numbers are 84 and 21, respectively.
If the ratio of the two numbers is 1:4, then the larger of the 21. Let x denote the greatest 4-digit number which when
two numbers is: divided by 6, 7, 8, 9 and 10 leaves a remainder of 4, 5,
6, 7 and 8, respectively. Then, the sum of the four-digits
(a) 12 (b) 48
of x is:
(c) 84 (d) 108
(a) 25 (b) 18
[Based on MAT, 1997]
(c) 20 (d) 22
14.
About the number of pairs which have 16 as their HCF
and 136 as their LCM, we can definitely say that: [Based on JMET, 2009]
(a) only one such pair exists 22. Find the greatest number of five digits which when
(b) only two such pairs exist divided by 8, 9 and 10 leaves 3 as remainder in each case.
(c) many such pairs exist (a) 99996 (b) 99723
(d) no such pair exists (c) 99983 (d) None of these
[Based on MAT, 2000]
23. A heap of stones can be made up into groups of 21. When
15.
L.C.M. of first 100 natural numbers is N. What is the
made up into groups of 16, 20, 25 and 45, there are 3
L.C.M. of first 105 natural numbers?
stones left in each case. How many stones at least can
(a) 5! × N (b) 10403N there be in the heap?
(c) 105N/103 (d) 4N
(a) 7203 (b) 2403
16.
The traffic lights at three different road crossings change (c) 3603 (d) 4803
after every 48 seconds, 72 seconds and 108 seconds,
respectively. If they all change simultaneously at 8:20:00 24.
Find the greatest number of four digits which must be
hrs, then they will again change simultaneously at: added to 5231 so that the final number becomes exactly
(a) 8:27:12 hrs (b) 8:27:24 hrs divisible by 12, 15, 27, 32 and 40.
(c) 8:27:36 hrs (d) 8:27:48 hrs (a) 7929 (b) 7829
[Based on MAT, 2000] (c) 9729 (d) 7729

Chapter 02.indd 6 6/4/2015 11:31:41 AM


H.C.F. and L.C.M. of Numbers 2.7

25. Find the greatest number of four digits which is exactly 28. A, B and C are running on a circular track of 120 m at a
divisible by 24, 28, 30 and 35. speed of 5 m/s, 8 m/s, and 10 /s respectively. A, B and C
(a) 9225 (b) 9240 all three are moving in the same direction. When will all
the three meet again at the starting point?
(c) 9250 (d) 9260
(a) After 55 seconds (b) After 2 minutes
26. Find the least number which when decreased by 11 is
divisible by 14, 15, 21, 32 and 60. (c) After 5 minutes (d) After 40 seconds
(a) 4371 (b) 3271 [Based on MAT, 2013]
(c) 3371 (d) 3360
29. Let x denote the greatest 4-digit number which when
27. In a morning walk, three person step off together, their divided by 6, 7, 8, 9 and 10 leaves a remainder of 4, 5,
steps measure 80 cm, 85 cm and 90 cm respectively. What 6, 7 and 8 respectively. Then, the sum of the four-digits
is the minimum distance each should walk so that they can of x is:
cover the distance in complete steps?
(a) 25 (b) 18
(a) 122 m 40 cm (b) 123 m 45 cm
(c) 122 m 45 cm (d) 122 m 50 cm (c) 20 (d) 22
[Based on SNAP, 2013]
[Based on MAT, 2011]

Difficulty Level-2
(Based on Memory)

1. A simple mechanical device has four gears A, B, C and D (a) 4 (b) 10


such that they mesh in the order A—B—C—D. The number (c) 15 (d) 16
of teeth in these gears are 10, 21, 12, 17, respectively.
[Based on FMS (MS), 2006]
If we start the mechanism from rest, how may revolutions
will the largest gear have to turn before one full cycle is 5.
21 mango trees, 42 apple trees and 56 orange trees have to
completed and the gears are in the position from where be planted in rows such that each row contains the same
they started? number of trees of one variety only. Minimum number of
(a) 270 (b) 340 rows in which the trees may be planted is:
(c) 7140 (d) 285 (a) 20 (b) 17
(c) 15 (d) 3
2. A stamp collector likes to arrange and rearrange his [Based on FMS, 2005]
collection in a symmetrical order. Sometimes he lines 6.
Five bells begin to toll together and toll respectively at
his stamps up in pairs, sometimes in threes, sometimes in intervals of 6 seconds, 7 seconds, 8 seconds, 9 seconds
fours, occasionally in fives, and once in a while in groups and 12 seconds. How many times they will toll together in
of six. After arranging them in any of these groups, he one hour, excluding the one at the start?
invariably has one stamp left over. When he arranges
(a) 3 (b) 5
his stamps in groups of seven, he finds that there is not a
single stamp to spare. How many stamps will be left over (c) 7 (d) 9
[Based on FMS, 2005]
if he arranges them in groups of 8?
(a) 5 (b) 7 7.
A number which when divided by 10 leaves a remainder of
9, when divided by 9 leaves a remainder of 8, by 8 leaves
(c) 1 (d) 0
a remainder of 7, etc., down to where, when divided by 2,
3. Three rings complete 60, 36 and 24 revolutions in a minute. it leaves a remainder of 1, is:
They start from a certain point in their circumference (a) 59 (b) 419
downwards. By what time they come together again in the
(c) 1259 (d) 2519
same position?
[Based on FMS, 2011]
(a) 5 seconds (b) 6 seconds
(c) 8 seconds (d) 1 seconds 8.
The smallest positive number x, which leaves a remainder
1 when divided by 2, 3, 4 and 5, is:
4.
Six bells commence tolling together and toll at intervals
of 2 seconds, 4 seconds, 6 seconds, 8 seconds, 10 seconds (a) Greater than 75 (b) Divisible by 7
and 12 seconds, respectively. In 30 minutes, how many (c) A prime number (d) None of these
times do they toll together? [Based on IIFT, 2005]

Chapter 02.indd 7 6/4/2015 11:31:41 AM


2.8 Chapter 2

9. Five bells first begin to toll together and then at intervals of (a) 16 (b) 12
3 seconds, 5 seconds, 7 seconds, 8 seconds and 10 seconds. (c) 11 (d) 13
Find after what interval they will again toll together. How  [Based on CAT, 2000]
many times do they toll together in one hour?
15. Let N = 1421 × 1423 × 1425. What is the remainder when
(a) 14 minutes, 3 times (b) 12 minutes, 4 times N is divided by 12?
(c) 14 minutes, 4 times (d) 12 minutes, 3 times (a) 0 (b) 9
(c) 3 (d) 6
10.
A chocolate dealer has to send chocolates of three
brands to a shopkeeper. All the brands are packed in  [Based on CAT, 2000]
boxes of same size. The number of boxes to be sent 16. For two positive integers a and b, define the function h(a,
is 96 of brand A, 240 of brand B and 336 of brand C. b) as the greatest common factor (GCF) of a, b. Let A be
These boxes are to be packed in cartons of same size a set of n positive integers. G(A) the GCF of the elements
containing equal number of boxes. Each-carton should of set A is computed by repeatedly using the function h.
contain boxes of same brand of chocolates. What could The minimum number of times h is required to be used to
be the minimum number of cartons that the dealer has compute G is:
to send? 1
(a) n (b) (n − 1)
(a) 20 (b) 48 2
(c) n (d) None of these
(c) 42 (d) 38
 [Based on CAT, 1999]
 [Based on XAT, 2010]
17. A is set of positive integers such that when divided by 2,
11. When asked for his taxi number, the driver replied, ‘If you 3, 4, 5, 6 leaves the remainder 1, 2, 3, 4, 5, respectively.
divided the number of my taxi by 2, 3, 4, 5 and 6 each time How many integers between 0 and 100 belong to set A?
you will find a remainder of one. But, if you divide it by (a) 0 (b) 1
11 the remainder is zero. You will also not find any other
(c) 2 (d) None of these
driver with a taxi having a lower number who can say the
same’. What is the taxi number?  [Based on CAT, 1998]

(a) 121 (b) 1001 18. Three wheels can complete 60, 36, 24 revolutions per
minute, respectively. There is a red spot on each wheel
(c) 1881 (d) 781
that touches the ground at time zero. After how much
 [Based on CAT, 2012] time, all these spots will simultaneously touch the ground
again:
12. A red light flashes 3 times per minute and a green light
5
flashes 5 times in two minutes at regular intervals. If both (a) 5 seconds (b) seconds
lights start flashing at the same time, how many times do 3 3
they flash together in each hour?
(c) 6 seconds (d) 7.5 seconds
(a) 30 (b) 24
 [Based on CAT, 1998]
(c) 20 (d) 60
 [Based on CAT, 2001]
19. Number of students who have opted for the subjects A, B
and C are 60, 84 and 108, respectively. The examination
3 1 is to be conducted for these students such that only the
1
13. Three pieces of cakes of weights 4 1bs, 6 1bs and 7 students of the same subject are allowed in one room.
2 4 5
Also, the number of students in each room must be same.
1bs, respectively, are to be divided into parts of equal What is the minimum number of rooms that should be
weights. Further, each part must be as heavy as possible. arranged to meet all these conditions?
If one part is served to each guest, then what is the (a) 28 (b) 60
maximum number of guests that could be entertained?
(c) 12 (d) 21
(a) 54 (b) 72
 [Based on CAT, 1998]
(c) 20 (d) None of these
20. Three bells chime at intervals of 18 minutes, 24 minutes
 [Based on CAT, 2001]
and 32 minutes, respectively. At a certain time, they begin
14. Let S be the set of integers x such that: to chime together. What length of time will elapse before
I. 100 ≤ x ≤ 200 they chime together again?
II. x is odd (a) 2 hrs and 24 minutes (b) 4 hrs and 48 minutes
III. x is divisible by 3 but not by 7 (c) 1 hr and 36 minutes (d) 5 hrs
How many elements does S contain?  [Based on CAT, 1995]

Chapter 02.indd 8 6/4/2015 11:31:42 AM


H.C.F. and L.C.M. of Numbers 2.9

21. Which is the least number that must be subtracted from 23. The smallest number which, when divided by 4, 6 or 7
1856 so that the remainder when divided by 7, 12, and 16 leaves a remainder of 2, is:
is 4? (a) 44 (b) 62
(a) 137 (b) 1361 (c) 80 (d) 86
(c) 140 (d) 172
 [Based on CAT, 1993]
 [Based on CAT, 1994]
24. The number of common terms in the sequences 17, 21, 25,
22. What is the smallest number which when increased by 5 is
…, 417 and 16, 21, 26, …, 466 is:
completely divisible by 8, 11 and 24?
(a) 264 (b) 259 (a) 78 (b) 19
(c) 269 (d) None of these (c) 20 (d) 77
 [Based on CAT, 1994]  [Based on CAT, 2008]

Answer Keys
Difficulty Level-1

1. (d) 2. (a) 3. (b) 4. (b) 5. (d) 6. (b) 7. (b) 8. (c) 9. (b) 10. (b) 11. (a) 12. (a) 13. (c)
14. (d) 15. (b) 16. (a) 17. (b) 18. (c) 19. (d) 20. (b) 21. (a) 22. (b) 23. (a) 24. (d) 25. (b) 26. (c)
27. (a) 28. (b) 29. (a)

Difficulty Level-2

1. (b) 2. (a) 3. (a) 4. (c) 5. (b) 6. (c) 7. (d) 8. (c) 9. (c) 10. (b) 11. (a) 12. (a) 13. (d)
14. (d ) 15. (c) 16. (b) 17. (b) 18. (c) 19. (d) 20. (b) 21. (d) 22. (b) 23. (d) 24. (c )­­­

Explanatory Answers

Difficulty Level-1

HCF × LCM number is divisible by 210 but not by 420. The least
1.
(d) Second number = and the greatest four digit multiples of 210 are 1050
First number
and 9870, respectively
4800 × 160
  = = 1600. \ The number of multiples of four-digit multiples of
480
9870 – 1050 8820
2. (a) L.C.M. of 12, 18, 36 and 45 210 are + 1= + 1 = 43
210 210
⇒ L.C.M. = 5 × 3 × 3 × 2 × 2 = 180
⇒  Required answer = 180 – 4 = 176 Similarly, the number of four-digit multiples of 420
Note:   Here, 12 – 8 = 18 – 14 are 9660  1260  1 21
420
 = 36 – 32 = 45 – 41 = 4.
\ The number of multiples of 210 which are not
3. (b) L.C.M. of 60 seconds and 62 seconds is 1860 seconds
divisible by 420 are 43 – 21 i.e., 22.
= 31 minutes
\ They will beep together at 10.31 a.m.
 7 23 28 15 
5. (d) L.C.M. of  , , , 
4. (b) If a number is divisible by 30 and 35, it is divisible by  14 21 3 63 
L.C.M. (30, 35) i.e., 210
If a number is divisible by 210 but not by 140, then it L.C.M. of (7, 23, 28, 15)
=
is not divisible by L.C.M. (210, 140) i.e., 420. So, the H.C.F. of (14, 21, 3, 63)

Chapter 02.indd 9 6/4/2015 11:31:42 AM


2.10 Chapter 2

15 × 28 × 11 14. (d) Using the relation,


= = 4620. First No. × Second No. = H.C.F. × L.C.M.
1
= 16 × 136 = 16 × (8 × 17)
6. (b) We use the fact that, for any two numbers, a and b, It is clear from here that no such pair is pos­sible.
a × b = H.C.F. × L.C.M 15.
(b) If we look at the numbers 100 < N ≤ 105, we see only
6804 × 9828 101 and 103 do not have their factors in N (because
⇒ L.C.M. =
756 these are primes). So, obviously the new L.C.M. will
be 101 × 103 × N.
= 9 × 9828 = 88452.
16.
(a) Let us first calculate L.C.M. of 48, 72, 108
7. (b) 43k + 43l = 430
2 48, 72, 108
⇒ 43(k + l) = 430
2 24, 36, 54
⇒  k + l = 10
2 12, 18, 27
But k, l must be co-primes.
3 6, 9, 27
So (k, l) = (1, 9) and (3, 7).
3 2, 3, 9
8. (c) Required length = H.C.F. of (42, 49, 63) = 7 meters. 2, 1, 3
9. (b) 56 = 1 × 56 \ L.C.M. of 48 seconds, 72 seconds, 108 seconds
⇒ H.C.F. is 1, hence number of tiles = 56 = 2 × 2 × 2 × 3 ×3 × 2 × 3 = 432 seconds
Thus, the second time the three lights will change
2 × 28 ⇒ H.C.F. is 2, hence number of tiles
after 432 seconds = 7 minutes 12 seconds
= 1 + 14 = 15 Hence, next time the three lights will change
4 × 14 ⇒ H.C.F. is 2, therefore number of tiles simultaneously at
= 2 + 7 = 9 8:20:00 + 0:7:12 = 8:27:12 hrs.
7 × 8 ⇒ H.C.F. is 1, therefore number of tiles 17. (b) Let us first calculate L.C.M. of 8, 12, 15
= 7 + 8 = 15 \ L.C.M. of 8, 12, 15 = 2 × 2 × 3 × 2 × 5 = 120
Hence, the minimum number of tiles can be 9 and the Hence, the smallest perfect square number di­visible
dimension of a tile is 2 meters each side. Thus, (b) is by 8, 12, 15 i.e., by 120 is 3600.
correct option. 18.
(c) We notice that 9 divides both numbers. We can use
4 Euler’s algorithm to obtain the GCD
10. (b) H.C.F. of (24, 82, 162, 203) = 2
L.C.M. of (24, 82, 162, 203) = 28 × 125 = 32000. 123456789 ) 987654321 ( 8
    
11. (a) Prime factors of 35 = 5 × 7 987456321
85 = 5 × 17 9
 L.C.M. = 7735 = 5 × 7 × 17 × 13
 A
If x divides A and x divides B, x divides the Rem  
Since, H.C.F. = 5, Let k = 5a B
where (A > B)
\ 35 × 85 × 5a = 5 × 7 × 17 × 13
⇒  a = 13 \ The GCD is 9.
\ least possible value of k = 5 × 13 = 65. 19. (d) H.C.F. of 10857 and 15087 is 141
12.
(a) Product of the numbers = H.C.F. × L.C.M. = 21 × 4641 The least number of cut pieces
= (10857 + 15087) ÷ 141 = 184.
= 21 × 3 × 7 × 13 × 17 = 3 × 7 × 3 × 7 × 13 × 17
\ The numbers can be 3 × 7 × 13 and 3 × 7 × 17, i.e., 20. (b) Let H.C.F. be x, then, L.C.M. = 12x
273 and 357. \ 12x + x = 403 ⇒ x = 31
13.
(c) Let the numbers be x and y \ L.C.M. = 12 × 31 = 372
x 1 Other number = 31 × 272 ÷ 93 = 124.
\ xy = 84 × 21 and, = ⇒ 4x = y
y 4
21. (a) The number will be a multiple of 6, 7, 8, 9, 10
Now, putting the value of y in the above equa­tion, we
have L.C.M. of 6, 7, 8, 9, 10 = 2520
x × 4x = 84 × 21 \ Largest 4-digit number divided by this = 7560
\ x = 21 and, y = 21 × 4 = 84 \ Required number = 7558
Hence, the larger number = 84. Sum of the digits of this number = 25.

Chapter 02.indd 10 6/4/2015 11:31:43 AM


H.C.F. and L.C.M. of Numbers I 2.11

22.
(b) L.C.M. of 8, 9, 10 = 360 Greatest number of four digits = 9999
360 ) 99999 ( 277 Quotient when 9999 is divided by 840 is 11 and
.llL remainder is 759
2799 \ Greatest number of four digits in this case
2520 = 9999 – 759 = 9240.
2799
2520 26. (c) Required number
279 = (L.C.M. of 14, 15, 21, 32, 60) + 11
Greatest number of 5 digits which is divisible by 360 = 3360 + 11 = 3371
= 99999 – 279 = 99720 27. (a) The minimum distance each person should walk =
\ Required number = 99729 + 3 = 99723. LCM of (80 cm, 85 cm and 90 cm)
23.
(a) L.C.M. of 16, 20, 25, 45 = 3600 = 12240 cm = 122 m 40 cm.

1st number = 3600 × 1 + 3 = 3603 which is not 28. (b) Time taken by A to complete one round,
divisible by 21
Distance 120
= = 24 seconds
2nd number = 3600 × 2 + 3 = 7203 which is divisible Speed 5
by 21. 120
Similarly, time taken by B = = 15 seconds
24.
(d) L.C.M. of 12, 15, 27, 32, 40 = 4320. Let us add 5231 8
to the greatest number of four digits and then divide 120
Time taken by C = = 12 seconds
by 4320 to find the remainder 10
Time taken or the time after which all meet at a
4320 ) 15230 (3 starting point.
12960 = LCM of 24 seconds, 15 seconds and 12 seconds
2270 2 24, 15, 12
Required greatest number of four digits 2 12, 15, 6
= 9999 – 2270 = 7729. 2 6, 15, 3
25.
(b) L.C.M. of 24, 28, 30 and 35 3 3, 15, 3
2 24, 28, 30, 35 1, 5, 1
2 12, 14, 15, 35 = 2 × 2 × 2 × 3 × 5 = 120 seconds = 2 minutes.
3 6, 7, 15, 35
29. (a) The number will be a multiple of 6, 7, 8, 9, 10
5 2, 7, 5, 35
LCM of 6, 7, 8, 9 annd 10 = 2520
7 2, 7, 1, 7 ∴ Largest 4-digit number divisible by 2520 = 7560
2, 1, 1, 1 ∴ Required number = 7558

= 2 × 2 × 2 × 3 × 5 × 7 = 840 Sum of the digits of this number = 25.

Difficulty Level-2

1. (b) Let n be the minimum number of teeth advanced Since it must be exactly divisible by 7, so the collector
before all the wheels are in starting position again has 301 stamps and when stamps are arranged in
Hence, n must be every divisible by 10, 21 12 and 17 groups of 8, he will have 5 stamps left over.
60
Hence, n = L.C.M. = 7140 3. (a) Time taken by each ring in one revolution are
60 60
60
So, the required number of revolutions in largest gear seconds, seconds and seconds respectively.
= 7140/21 = 340. 36 24
2. (a) Number of tickets with the collector 5 5
i.e., 1, and seconds
= L.C.M. of (3, 4, 5, 6) + 1, i.e., 60 + 1 = 61 3 2
or one more than the multiple of 60 i.e., 61, 121, 181, 5 5
Requiring time = L.C.M. of 1, , = 5 seconds.
241, 301 etc 3 2

Chapter 02.indd 11 6/4/2015 11:31:43 AM


2.12 Chapter 2

4.
(c) L.C.M. of 2, 4, 6, 8, 10, 12 = 120  9 27 36  HCF of (9, 27, 36) 9
30 × 60  2 , 4 , 5  = LCM of ( 2, 4, 5) = 20 lbs
Total no. of times = = 15  
120 18.45 18.45 × 20
∴ Numbber of guests = = = 41.
5.
(b) H.C.F. of 21, 42, 56 = 7 9 9
119 20
\ Number of rows = = 17
7 14. (d) Numbers between 100 and 200, which are divisible
6.
(c) L.C.M. of 6, 7, 8, 9, 12 = 504 seconds by 3 are 102, 106, 109, . . . , 198 = 102 + (n − 1) × 3
⇒ n = 33
1 hr = 1 × 60 × 60 seconds = 3600
3600 Out of these 33 numbers, 17 are even and 16 are odd
\ Number of times = ≈ 7.
504 Out of these 16 odd numbers, there are three numbers
(= 105, 147, 189), which are divided by the LCM of
7.
(d) Every time the difference between divisor and (7, 3), i.e., 21
remainder is 1. So, the number is of the form L.C.M.
(10, 9, 8, 7, 6, 5, 4, 3, 2) K – 1 Hence, in all (16 − 3) = 13 numbers are contained in S.

= 2520 K – 1 15. (c) According to the remainder theorem, the remainders


for the following expressions will be same
when K = 1,
1421 × 1423 × 1425 5 × 7 × 9 315
2520 – 1 = 2519. or or
12 12 12
8.
(c) L.C.M. of  2, 3, 4, 5 = 60 ⇒ Remainder = 3 .
\ Smallest number will be 60 + 1 = 61 16. (b) It is clear that for n positive integers function h(a, b)
and 61 is a prime number. has to be used one time less than number of integers
i.e., (n−1) times.
9.
(c) Required time interval
17. (b) Required number of the set is calculated by the LCM
= L.C.M. of (3, 5, 7, 8 and 10) of (2, 3, 4, 5, 6)−(common difference)
= 840 seconds = 14 minutes In this case, common difference
Number of times they will toll together in one hour = (2 − 1) = (3 − 2) = (4 − 3) = (5 − 4) = (6 − 5) = 1.
60 ∴ All integers of the set will be given by (60n − 1)
= = 4 times (ignoring the fraction part).
14
if n = 1, (60n − 1) = 59
10.
(b) For minimum number of cartoons, there should be if n = 2, (60n − 1) = 119
maximum number of chocolates in a cartoon that is
Since, range of the set A is between 0 and 100, hence
H.C.F. of 96, 240 and 336, which is 48.
there will exist only one number i.e., 59.
11. (a) The smallest number divisible by 2, 3, 4, 5 and 6 is 18. (c) Time taken by red spot on all the three wheels to
their LCM, i.e., LCM (2, 3, 4, 5, 6) = 60 touch the ground again simultaneously will be equal
∴ The taxi number is of the form (60n + 1). The to the LCM of the time taken by the three wheels to
options that satisfy the condition are (a) and (d). But complete one revolution
the number has to be the smallest. The first wheel completes 60 revolutions per minute.
Therefore option (a). Therefore, to complete one revolution it takes
12. (a) First light blinks, after 20 seconds, second light blinks  60 
 60  = 1 seconds
after 24 seconds  
Now, they blink together after LCM of 20 and Time taken by the second wheel to complete one
24 seconds = 120 seconds = 2 minutes 36 3
revolution
= = seconds
Hence, the number of times they blink together in an 60 5
hour = 30. And the time taken by the third wheel to complete one
24 2
13. (d) Total, the weight of three pieces revolution
= = seconds
60 5
 9 27 36  369 Hence, LCM of
= + + = = 18.45 lbs
2 4 5  20 3 2 LCM(1, 3, 2) 6
=
1, , = = 6 seconds
Required weight of a single piece is HCF of 5 5 HCF(1, 5, 5) 1

Chapter 02.indd 12 6/4/2015 11:31:44 AM


H.C.F. and L.C.M. of Numbers 2.13

19. (d) Number of students that should be seated in each 22.


(b) Required number = LCM of (8, 11, 24) − 5
room is the HCF of 60, 84 and 108 that is 12. = (264 − 5) = 259.
∴ Number of rooms required for subjects A, B and 23. (d) Required number = LCM of (4, 6, 7) + 2
60 84 108 ⇒ 84 + 2 = 86.
C= = 5 rooms,  = 7 rooms and = 9 rooms,
12 12 12 24. (c) Both the sequences (17, 21, 25,…) and (16, 21, 26,…)
respectively are arithmetic progressions with common difference
Hence, minimum number of rooms required to satisfy of 4 and 5, respectively
all the conditions
In both the sequences, first common term is 20. Hence,
= (5 + 7 + 9) = 21. a new arithmetic sequence containing the common
20. (b) The bells will chime together again after a time that terms of both the series can be formed with a common
is equal to the LCM of 18, 24 and 32 = 288 minutes = difference of LCM of (4, 5) i.e., 20
4 hrs and 48 minutes. ∴ New sequence will be 21, 41, 61,…, 401
21. (d) LCM of (7, 12, 16) = 336 nth term = a + (n − 1) d
401 = 21 + (n − 1) 20
If we divide 1856 by 336, then remainder is 176.
Since, it is given that remainder in this condition 401 − 21
∴ ( n − 1) = = 19
is 4. Hence, the least number to be subtracted 20
= (176 − 4) = 172. ∴ n = 20.

Chapter 02.indd 13 6/4/2015 11:31:44 AM


This page is intentionally left blank

FM.indd 6 6/10/2015 2:36:16 PM


Square Root
CHAPTER

and Cube Root 3


SQUARE II. Method of Division
A number multiplied by itself is known as the square of the This method is used when the number is large and the
given number. For example, square of 6 is 6 × 6 = 36. factors cannot be easily determined.
Square Root The working rule is explained with the help of following
Square root of a given number is that number which when example:
multiplied by itself is equal to the given number.
For example, square root of 81 is 9 because 92 Step 1 The digits of a number, whose square
= 9 × 9 = 81. root is required, are separated into periods
The square root of a number is denoted by the symbol of two beginning from the right. The last
√ or , called the radical sign. period may be either single digit or a pair.
Thus, 81 = 9, 64 = 8 and so on. 476

Note that 1 = 1. 8 22 65 76
87 16
Methods of Finding the Square Root 665
I. Prime Factorization Method 609
1. Find the prime factors of the given number. 946 5676
2. Group the factors in pairs. 5676
3. Take one number from each pair of factors and then
×
multiply them together.
This product is the square root of the given number. Step 2 Find a number (here, 4) whose square may
be equal to or less than the first period
Illustration 1 Find the square root of
(here, 22).
(i) 4761 (ii) 207025
Step 3 Find out the remainder (here, 6) and bring
Solution: (i) 4761 = 23  23  3
3 down the next period (here, 65).

Step 4 Double the quotient (here, 4) and write to
\ 4761 = 23 × 3 = 69 the left (here, 8).
 5 × 7
(ii) 207025 = 5  7 × 13  13 Step 5 The divisor of this stage will be equal to

the above sum (here, 8) with the quotient
of this stage (here, 7) suffixed to it (here,
\ 207025 = 5 × 7 × 13 = 455
87).
Note: Step 6 Repeat this process (step 4 and step 5) till
all the periods get exhausted.
The above method is used when the given number is a
The quotient (here, 476) is equal to the square
perfect square or when every prime factor of that number
root of the given number (here, 226576).
is repeated twice.

Chapter 03.indd 1 6/4/2015 10:53:10 AM


3.2 Chapter 3

Illustration 2  Find the square root of (ii) 11.27


(i) 180625 (ii) 1498176 1 1 27. 01 29
Solution: (i) 425 1 .
8 18 06 25 21 27
21
82 16
222 601
206
444
164
2447 15729
845 4225 15729
4225                ×
        × \ 127.0129 = 11.27
Thus, 180625 = 425 (iii) Since the number of decimal places is odd, we
make it even by affixing one 0 to the right.
(ii)   1224 0.423
1 1 49 81 76 4 0. 17 90 13 60
1 16
22 49 82 190
44 164
242 581 843 2613
484 2529
2444 9776 846 8460
9776
In the above solution, after bringing down the last
              × period, we note that the remainder is not zero. So, a pair
Thus, 1478176 = 1224 of zeros can be annexed and process can be continued to
find the square root up to 4 places of decimals. The above
process can be continued still further and square root up to
Square Root of a Decimal required number of decimal places can be obtained.
If the given number is having decimal, we separate the
digits of that number into periods of two to the right and left Note:
starting from the decimal point and then proceed as in the If a decimal has an odd number of decimal places, its square
following illustration: root cannot be found exactly.
Illustration 3  Find the square root of (iv) .025
(i) 12.1801 (ii) 127.0129 0. 00 06 25
(iii) 0.1790136 (iv) 0.000625 00
   02   06
Solution:  (i) 3.49   04
3 12. 18 01    45    225
16    225
64 665       ×
609 \   0.000625 = 0.025
689 6201
6201 Square Root of a Fraction

        × (a) If the denominator is a perfect square:  The square


root is found by taking the square root of the numerator
\   12.1801 = 3.49 and denominator separately.

Chapter 03.indd 2 6/4/2015 10:53:10 AM


Square Root and Cube Root 3.3

(b) If the denominator is not a perfect square: The Illustration 5 Find the cube root of 42875.
fraction is converted into decimal and then square root Solution: Resolving 42875 into prime factors, we get
is obtained or the denominator is made perfect square 42875 = 5× 5 × 5 × 7 × 7
×7
by multiplying and dividing by a suitable number and
then its square root is obtained. 3
\ 42875 = 5 × 7 = 35
Illustration 4 Find the square root of
2704 44 II. Short-cut Method to Find Cube Roots of Exact
(i) (ii) Cubes Consisting of up to 6 Digits:
49 25
Before we discuss the method to find the cube
354 461
(iii) (iv) roots of exact cubes, the following two remarks
43 32 are very useful and must be remembered by heart.
2704 2704 52 × 52 52 1. 13 = 1; 23 = 9; 33 = 27; 43 = 64; 53 = 125; 63
Solution: (i) = = =
49 49 7×7 7 = 216; 73 = 343; 83 = 512;
3 93 = 729; 103 = 1000.
= 7 .
7 2. If the cube ends in 1, then its cube root ends in 1
If the cube ends in 2, then its cube root ends in 8
44 44 44 44 6.6332
(ii) = = = = If the cube ends in 3, then its cube root ends in 7
25 25 5×5 5 5 If the cube ends in 4, then its cube root ends in 4
= 1.3266 (nearly) If the cube ends in 5, then its cube root ends in 5
If the cube ends in 6, then its cube root ends in 6
354
(iii) = 8.2325 = 2.8692 (nearly) If the cube ends in 7, then its cube root ends in 3
43
If the cube ends in 8, then its cube root ends in 2
461 461× 2 922 30.3644 If the cube ends in 9, then its cube root ends in 9
(iv) = = =
32 32 × 2 64 8 If the cube ends in 0, then its cube root ends in 0
= 3.7955 (nearly) Clearly from above
1 ↔ 1, 4 ↔ 4, 5 ↔ 5, 6 ↔ 6, 9 ↔ 9, 0 ↔ 0
Cube
2 ↔ 8, 3 ↔ 7.
Cube of a number is obtained by multiplying the number
itself thrice.
For example, 27 is the cube of 3 as 27 = 3 × 3 × 3. The method of finding the cube root of a number up
to 6 digits which is actually a cube of some number
Cube Root
consisting of 2 digits is best illustrated with the help of
Cube root of a given number is that number which when following examples:
raised to the third power produces the given number, that is
Illustration 6 Find the cube roots of the following:
the cube root of a number x is the number whose cube is x.
(i) 2744 (ii) 9261
The cube root of x is written as 3 x .
(iii) 19684 (iv) 54872
For example, cube root of 64 is 4 as 4 × 4 × 4 = 64. (v) 614125
Solution: (i) Make groups of 3 digits from the right side.
Methods to Find Cube Root
2 744
2 lies between 13 and 23, so left digit is 1
I. Method of Factorization
744 ends in 4, so right digit is 4
1. Write the given number as product of prime factors. Thus, cube root of 2744 is 14
2. Take the product of prime numbers, choosing (ii) 9 261
one out of three of each type.
9 lies between 23 and 33, so left digit is 2
This product gives the cube root of the given 261 ends in 1, so right digit is 1
number. Thus, cube root of 9261 is 21

Chapter 03.indd 3 6/4/2015 10:53:11 AM


3.4 Chapter 3

(iii)
19 683 872 ends in 2, so right digit is 8
19 lies between 23 and 33, so left digit is 2 Thus, cube root of 19683 is 38
683 ends in 3, so right digit is 7 (iv) 614 125
Thus, cube root of 19683 is 27 614 lies between 83 and 93, so left digit is 8
(iv) 54 872 125 ends in 5, so right digit is 5
54 lies between 33 and 43, so left digit is 3 Thus, cube root of 614125 is 85

Practice Exercises

Difficulty Level-1
(Based on Memory)

8. What least number should be subtracted from the square


1. Taking 2 = 1.4l4, 3 = 1.732, 5 = 2.236 and 6 15
root of 21 so that the result is a whole number?
9+ 2 6− 2 289
= 2.449, then the value of + to three
5+ 3 5− 3 (a) 15/289 (b) 7/17
places of decimals is:
(c) 10/17 (d) 5/17
(a) 9.231 (b) 13.716
(c) 11.723 (d) 15.892 9. A general wishing to draw up his 16160 men in the form
of a solid square found that he had 31 men over. The
[Based on MAT, 2002]
2
number of men in the front row is:
2. If n = 12345678987654321, what is n?
(a) 127 (b) 123
(a) 12344321 (b) 1235789
(c) 137 (d) 129
(c) 111111111 (d) 11111111
5 +1 5 −1
3. One-fourth of a herd of camels was seen in the forest. 10. If a = and b = , then the value of
Twice the square root of the herd had gone to mountains 5 −1 5 +1
and the remaining 15 camels were seen on the bank of a a 2 + ab + b 2
is:
river. Find the total number of camels. a 2 − ab + b 2
(a) 32 (b) 34
3 4
(c) 35 (d) 36 (a) (b)
4 3
[Based on MAT, 2005]
3 5
4. The smallest number by which 3600 must be multiplied to (c) (d)
make it a perfect cube is: 5 3
(a) 40 (b) 60
3 3 3 3 3
(c) 20 (d) 15 11. 3 =?
5. By what least number, 2450 be multiplied, so that the (a) 331/64 (b) 331/32
resulting number is perfect square? (c) 31/64 (d) None of these
(a) 8 (b) 10
12. A general wishing to draw up his 16160 men in the form
(c) 5 (d) 2 of a solid square, found that he had 31 men left over. Find
6. The largest number of five digits which is a perfect square the number of men in the front.
is: (a) 126 (b) 125
(a) 97344 (b) 98596 (c) 128 (d) 127
(c) 99856 (d) None of these
13. A man plants his orchard with 5625 trees and arranges
7. The smallest number which when subtracted from the them so that there are as many rows as there are trees in a
number 62512 makes it a perfect square is: row. How many rows are there?
(a) 22 (b) 32 (a) 125 (b) 25
(c) 12 (d) 2 (c) 75 (d) None of these

Chapter 03.indd 4 6/4/2015 10:53:12 AM


Square Root and Cube Root 3.5

14. Find the number whose square is equal to the difference 2



between the squares of 75.15 and 60.12.  −1  3
23.
The value of   is:
(a) 225.9 (b) 67.635  216 
(c) 45.09 (d) 15.03 1 1
(a) (b) –
36 36
2176 x
15. If 1+ 1− = 1  , value of x is: (c) – 36 (d) 36
2401 7
[Based on MAT, 1998]
(a) 3 (b) 1
5 10
(c) 5 (d) 7 24.
If 5 = 2.236, then the value of –  + 125 is
2   5
16. The areas of two square fields are 420.25 m2 and 441 m2 equal to:
respectively. The ratio of their sides is: (a) 7.826 (b) 8.944
(a) 20:21 (b) 40:41 (c) 5.59 (d) 10.062
(c) 41:42 (d) 40:42 [Based on MAT, 1998]
2
1 1 1
3 4 25.
If a =   , b = and c = , then which of the
17. If 12 = 3.464, value of + 2 is:  10  5 100
4 3 following statements is correct?
(a) 3.17 (b) 3.464 (a) a < b < c (b) a < c < b
(c) 3.1753 (d) None of these (c) b < c < a (d) c < a < b
[Based on MAT, 1998]
18. What will come in place of the question mark (?) in the
following equation: 26.
Which is the smallest of the following numbers?
257.5 × 52.5 ÷ 1251.5 = 5 ? (a) 7 (b) 1/ 7
(a) 16 (b) 17.5 (c) 7 /7 (d) 1/7
(c) 8.5 (d) 13 [Based on MAT, 1999]
[Based on PNB Management Trainee Exam, 2003]
27.
The value of 3 0.000064 is:
19. The least number by which 14175 be divided to make it a (a) 0.02 (b) 0.2
perfect square is: (c) 2.0 (d) None
(a) 3 (b) 5 [Based on MAT, 1999]
(c) 7 (d) 15 28.
Square root of 117649 is:
20. Multiply the difference between the two lowest numbers (a) 347 (b) 343
with the difference between the two highest numbers in (c) 353 (d) 357
the following sequences: [Based on MAT, 2000]
89, 7, 91, 72, 31, 25, 18, 89, 16, 58, 38, 42, 86 29.
Cube root of 658503 is:
(a) 18 (b) 77 (a) 83 (b) 77
(c) 81 (d) 16 (c) 87 (d) 97
 [Based on NABARD, 1999] [Based on MAT, 2000]
21.
One-fifth of a number is equal to five-eighths of the second
30. 110.25 × 0.01 ÷ 0.0025 − 420.25 equals to:
number. If 35 is added to the first number it becomes four
times of second number. What is the value of the second (a) 0.75 (b) 0.50
number? (c) 0.64 (d) 0.73
(a) 125 (b) 70 [Based on SNAP, 2010]
(c) 40 (d) 25 31. In a class of 40 students and 5 teachers, each student got
 [Based on NABARD, 1999] sweets that are 25 per cent of the total number of students
and each teacher got sweets that are 15 per cent of the
22.
If x + 49 = 8.2, then the value of x is equal to: total number of students. How many sweets were there?
(a) 1.20 (b) 1.40 (a) 480 (b) 440
(c) 1.44 (d) 1.89 (c) 430 (d) 450
[Based on MAT, 1998]  [Based on IRMA, 2008]

Chapter 03.indd 5 6/4/2015 10:53:12 AM


3.6 I Chapter 3

32. 915849 + 795664 = (?)2 : If x = 3 + 2 2, then the value of (x1/2 – x– 1/2) is:
37.
(a) 1849 (b) 79 (a) 2 (b) 2
(c) 33 (d) None of these (c) 2 − 1 (d) 2 + 1
 [Based on IRMA, 2009] [Based on ATMA, 2008]
2 2
38. (32) + ? – (23) = 536:
33. 956240 = ?
(a) 1764 (b) 1849
(a) 979 (b) 864
(c) 1521 (d) 1681
(c) 1009 (d) 647
 [Based on NMAT, 2008]
 [Based on IRMA, 2009]
39. 1562000 = ?
5 3 (a) 1175 (b) 1200
34. Simplify :
5 3 (c) 1250 (d) 1325
(a) 1 (b) 2  [Based on NMAT, 2008]
(c) 3 (d) None of these 3 3
40.
The value of .j1.001001001 − .j1.001001 is closest to:
[Based on NMAT, 2006]
–6
(a) 0 (b) 10
35. If x = 0.888, y = 0.888 and z = (0.888)2, then which of (c) 3.10– 9 (d) (3.3).10– 10
the following is true?
[Based on GBO Delhi University, 2011]
(a) y < x < z
41.
If the square root of a number is between 6 and 7, then the
(b) y < z < x
cube root of the number will be between:
(c) x < z < y
(a) 1 and 2 (b) 2 and 3
(d) z < x < y
(c) 3 and 4 (d) 4 and 5
[Based on ATMA, 2005]
[Based on GBO Delhi University, 2011]
36.
The positive integer n is divisible by 25. If n is greater 42.
V is inversely proportional to the square root of m and m is
than 25, which of the following could be the value of inversely proportional to the square of t. The relationship
n
? between V and t is:
25
(a) V ∝ t2 (b) V ∝ t
(a) 26 (b) 23 1
(c) 25 (d) 22 (c) V ∝ (d) V ∝ t
t
[Based on ATMA, 2008] [Based on GBO Delhi University, 2011]

Difficulty Level-2
(Based on Memory)

1. Let n (> 1) be a composite integer such that 2. An operation ‘$’ is defined as follows:
n is not an
integer. Consider the following statements: For any two positive integers x and y,

A: n has a perfect integer–valued divisor which is greater


 x y
x$y =
 +  then which of the following is an
than 1 and less than n  y x
 
B: n has a perfect integer–valued divisor which is greater
integer?
than n but less than n (a) 4$9 (b) 4$16
(c) 4$1 (d) None of the above
(a) Both A and B are false
3. A certain number of people agree to subscribe as
(b) A is true but B is false many rupees each as there are subscribers. The whole
(c) A is false but B is true subscription is `2582449. Find the number of subscribers.
(d) Both A and B are true (a) 1607 (b) 1291225
(c) 1503 (d) 1603

Chapter 03.indd 6 6/4/2015 10:53:13 AM


Square Root and Cube Root 3.7

4. Which of the following numbers, wherein some of the 11.


The number 8  3 7 is equal to:
digits have been suppressed by symbols, can possibly be
the perfect square of a three-digit odd number?
8+ 7 
(a) 65 ××× 1 (a)   (b) 8 − 3 7
 2 
(b) 9 ×× 1
(c) 10 ××× 4 8− 7 
(c) 2 2  3 7 (d)  
(d) 9 ×××××× 5  2 
[Based on FMS, 2009]
5.
Three cubes of iron whose edges are 6 cm, 8 cm and
10 cm, respectively, are melted and formed into a single 4 3
12. The expression − is equal to:
cube. The edge of the new cube formed is: 3 4
(a) 12 cm (b) 14 cm
(c) 16 cm (d) 18 cm 3 3
(a) (b) –
[Based on FMS (MS), 2006] 6 6

6.
If 841 = 29, then the value of: 3 5 3
(c) (d)
 
6 6
841  8.41  0.0841  0.000841 is [Based on FMS, 2010]
(a) 322.19 (b) 32.219
6407522209
(c) 34.179 (d) 31.129 13.
2− = is equal to:
3600840049
[Based on FMS (MS), 2006]
(a) 0.666039 (b) 0.666029
a
7.
If (0.05 × 0.5 × a) = 0.5 × 0.05 × b , then is equal to: (c) 0.666009 (d) None of these
b
(a) 0.0025 (b) 0.025 [Based on IIFT, 2008]

(c) 0.25 (d) None of these 2


14. If n = 12345678987654321, what is n?
[Based on FMS (MS), 2006] (a) 12344321
a (b) 1235789
8.
If 0.04 × 0.4 × a = 0.4 × 0.04 × b , than is :
b (c) 111111111
(a) 0.016 (b) 0.16 (d) 11111111
(c) 1 (d) 16 [Based on CAT, 1999]
[Based on FMS, 2005]
15. Find the sum:
9.
A gardener plants 17956 trees in such a way that there are
as many rows as there are trees in a row. The number of 1 1 1 1 1 1
trees in a row are: 1+ 2 + 2 + 1+ 2 + 2 + + 1+ 2
+ 2
1 1 2 3 2007 2008
(a) 136 (b) 134
(c) 144 (d) 154 1 1
[Based on FMS, 2006] (a) 2008 − 2007 −
(b)
2008 2007
[1/( 9 − 8) − 1 / ( 8 − 7) + 1/( 7 − 6)
10.
1 1
(c) 2007 − 2008 −
(d)
−1 / ( 6 − 5) + 1 / ( 5 − 4)] = ? 2008 2007
(a) 0 (b) 1 1
(c) 5 (d) 1/3 (e) 2008 −
[Based on FMS, 2006]
2009 [Based on CAT, 2008]

Chapter 03.indd 7 6/4/2015 10:53:14 AM


3.8 Chapter 3

Answer Keys
Difficulty Level-1

1. (c) 2. (d ) 3. (d ) 4. (b) 5. (d ) 6. (c) 7. (c) 8. (c) 9. (a) 10. (b) 11. (b) 12. (d ) 13. (c)
14. (c) 15. (b) 16. (c) 17. (c) 18. (d ) 19. (c) 20. (a) 21. (c) 22. (c) 23. (d ) 24. (a) 25. (b) 26. (d )
27. (b) 28. (b) 29. (c) 30. (b) 31. (c) 32. (d ) 33. (a) 34. (d ) 35. (d ) 36. (a) 37. (a) 38. (d ) 39. (c)
40. (a) 41. (c) 42. (b)

Difficulty Level-2

1. (d ) 2. (d ) 3. (a) 4. (a) 5. (a) 6. (b) 7. (b) 8. (a) 9. (b) 10. (c) 11. (c) 12. (a) 13. (a)
14. (d ) 15. (a)­­­

Explanatory Answers

Difficulty Level-1

9+ 2 6− 2 6.
(c) Largest number of 5 digits = 99999
1.
(c) + 316
5+ 3 5− 3
3 9 99 99
9 5 − 9 3 + 10 − 6 + 6 5 + 6 3 − 10 − 6 9
=
2 61 99
15 5 − 3 3 − 2 6 61
= 625 3899
2
3756
15 × 2.236 − 3 × 1.732 − 2 × 2.449
= 143
2
Required number = (316)2 = 99856.
33.54 − 5.196 − 4.898 7.
(c)
= = 11.723. 250
2
2 62 5 12
2. (d) Square root of 12345678987654321 is 1111111.
4
36 45 225
3.
(d) No. of camels seen in the forest = =9
4 225
50 12
No. of camels gone to the mountains = 2 36 = 12
No. of camels seen on the bank of river = 15 So, 12 is the smallest number which when subtracted
from 62512 makes it a perfect square
\ Total number of camels = 9 + 12 + 15 = 36.
62500 = 250.
4.
(b) 3600 = 2 × 2 × 2 × 2 × 3 × 3 × 5 × 5 2
= 22 × 22 × 32 × 52 15 6084  78 
8.
(c) 21 = =  
\ 3600 should be multiplied by 2 × 2 × 3 × 5. That is 289 289  17 
60 to make it a perfect cube. 78 10
\ Square root = =4
17 17
5. (d) 2450 = 5 × 5 × 7 × 7 × 2 10
\ Least fraction to be subtracted = .
\ 2450 must be multiplied by 2. 17

Chapter 03.indd 8 6/4/2015 10:53:14 AM


Square Root and Cube Root I 3.9

9. (a) The number of men in the front row is the square root
of 16160 – 31, that is 16129 which is 127. f f
(c) 3 + 2 4 =
17.
4 3
1
,12J 4-+-3 
2

3.464 × 11
-r5 + 1 × -r5 + 1 = (-r5 + 1) 2
= = 3.1753
10. (b) a =
r- r-
5 −1 5 +1 ---i=-r~
( 5) − (1)
2 2 12

=
5 +1+ 2 5 -r 18.
(d) 257.5 × 52.5 ÷ 1251.5 = 5?
5 −1 or, 52×7.5 × 52.5 ÷ 53×1.5 = 5?
=
62 5
=
3 5 ~-
-r -r or, 515 × 52.5 ÷ 54.5 = 5?
4 2 1
or, 517.5 × 4.5 = 5?  or,  513 = 5?
-r5 − 1 × -r5 − 1 = (-r5 − 1) 2
5
b =
r- r-
5 +1 5 −1 ---i=-r~
( 5) 2 − (1) 2 or, ? = 13.

19.
(c) 14175 = 5 × 5 × 3 × 3 × 3 × 3 × 7
= _--"----r
5 +1− 2 5 6 + 2 5 = _-r
= ~-r 3− 5
= 52 × 34 × 7
5 −1 4 2
It must be multiplied by 7.
(3 + 5) + (3 − 5)
2 2
a2 + b2 =
4 20.
(a) Difference between two lowest numbers
9+5+6 5 +9+5−6 5 = 16 – 7 = 9
= = 28 = 7
5 4 Difference between two highest numbers
ab = 1 = 91 – 89 = 2
a 2 + ab + b 2 7 + 1 8 4 \ Product of these two numbers = 9 × 2 = 18.
\ = = = .
a − ab + b
2 2 7 − 1 6 3 1 5
21.
(c) I = II
5 8
3 3 3 3 3 3
11. (b) 3.31/2 = 3.33/4 = 3.37/8 I 25
\ =
II 8
 = 3.315/16 = 331/32.
I + 35 = 4 II
12. (d) 16160 – 31 = 16129; 16129 = 127. 25
or,   II + 35 = 4 II
13.
(c) Let the number of trees be n 8
\ n2 = 5625 \ II = 40.

⇒ n = 5625 = 75. 22.


(c) x = 8.2 – 7 = 1.2 ⇒ x = 1.44.
2 2 2
14.
(c) x = (75.15) – (60.12) 2 2
− − −2
= 5647.52 – 3614.41  −1  3  −1  3  1
23. (d)   =  3  = − 
= 2033.11  216  6   6
\ x = 45.09.
1 1
= = = 36.

~r=
2176 x  1
2 1
15. (b) 1 + 1 − =1+ − 
2401 7  6 36

⇒ 1+
- fF
x
7
= 1
225
2401
=
r=
1
15
49 24.
(a) -r25 − -r
10
5
+ ,J
125

\
F-
x = 1.
64
49
=
8
7
=1+
1
7 = -r,J = ~~
5 − 20 + 2 5 125
-r 2 5
5 − 20 + 60
2 5

16.
(c) Ratio of their sides is the ratio of their square roots 35 × -r5
=
2-r-r
5× 5
= 420.25 : 441
= 20.5:21 7 7
= - × -r5 = - × 2.236 = 7 × 1.118 = 7.826.
= 41:42. 2 2

Chapter 03.indd 9 6/4/2015 10:53:16 AM


3.10 Chapter 3

2 33. (a)
 1 1
25. (b) a =  = = 0.1
 100 5− 3 5− 3 8 − 2 15
10
  34. (d) × =
5+ 3 5− 3 2
1
b = = .2    = 4 – 15 .
5
35. (d) x = 0.888,
1
c = = .01 .1
= y = 0.888 = 0.942
100
z = (0.888)2 = 0.78
\ a < c < b.
\ z < x < y.
26.
(d)
27.
(b) Given expression = 3 0.008 = 0.2. 36.
(a) Since, n > 25 and also n is divisible by 25 then the
number could be 650
28.
(b) 117649 = 7 × 7 × 7 × 7 × 7 × 7 n 650
\ = = 26.
\ 117649 = 7 × 7 × 7 = 343. 25 25
29.
(c) 658503 = 3 × 3 × 3 × 24389 37.
(a) x = 3 + 2 2 = (1 + 2 )2
= 3 × 3 × 3 × 29 × 29 × 29 Now, taking (x1/2 – x– 1/2)
3
\ 658503 = 3 × 29 = 87.  1 
= (1 + 2) −  
30.
(b) 110.25 =10.5 1+ 2 

0.01 = 0.1  2 −1


= (1 + 2) −  
0.0025 = 0.05  2 −1 

420.25 = 20.5 = 1 + 2 – 2 +1
= 2.
\ 110.25 × 0.01 ÷ 0.0025 × 420.25 38. (d)
0.1 39. (c) 1250 × 1250 = 1562500 (1562000 approx.)
= 10.5 × – 20.5
0.05 40. (a)
1.05 41. (c)
= − 20.5
0.05 a b
42. (b) V = ,m= 2,
= 21 – 20.5 = 0.5 m t
where a and b are constants
Hence, option (b).
a a a
40 × 25 40 × 15 V = = = ×t
31. (c) 40 × + 5× = 400 + 30 = 430. m b b
100 100 c2
32. (d) It will be square of 43. ⇒ V ∝ t.

Difficulty Level-2

1. (d) Consider a number n = 6 2.


(d) By direct substitution.

n = 245 3.
(a) Let the number of subscribers be n
A: We have a divisor 2 which is greater than 1 and ⇒ n2 = 258249
less than 6 . \ n = 2582449 = 1607.
B: We have a divisor 3 which is greater than but less 4. (a) The square of an odd number cannot have 4 as the unit
than 6. digit. The square of a 3-digit number will have at least
Thus, both statements are true. 5 digits and at the most 6 digits.

Chapter 03.indd 10 6/15/2015 2:23:01 PM


Square Root and Cube Root 3.11

(a) 63 + 83 +103 = New cube


5. 11. (c) 8 + 3 7 = 2 2 + 3 7 .
216 + 512 + 1000 = New cube
⇒ 3
1728 = 12 cm. 4 3 4× 4 − 3× 3
12. (a) − =
3 4 3× 4
29 29 29
6.
(b) 29    4−3 1
10 100 1000 = =
12 12
= 29 + 2.9 + 0.29 + 0.029
= 32.219. 12 2 3
= 
7.
(b) On squaring both the sides, we get 12 12
0.05 × 0.5 × a = (0.5)2 × (0.05)2 × b 3
= .
a 6
⇒ = 0.025.
b
6407522209 80047
13. (a) 2− = 2−
8.
(a) 0.04 × 0.4 × a = 0.4 × 0.04 × b 3600840049 60007
 = 2 – 1.3339610
On squaring both the sides, we get
 = 0.666039.
0.04 × 0.4 × a = (0. 4)2 × (0.04)2 × b 14. (d) Square root of 12345678987654321 is 11111111.
a 15. (a) First term is
\      = 0.016.
b
1 3 1
1+1+ = =2−
4 2 2
9.
(b) 17956 = 134.
Sum of first two terms is
1 1 1 3 1 1
10.
(c) − + + 1+ +
9− 8 8− 7 7− 6 2 4 9
1 1 3 36 + 9 + 4 3 7 16 8 1
− + = + = + = = = 3−
6− 5 5− 4 2 36 2 6 6 3 3
Sum of first three terms is
= ( 9 − 8) − ( 8 + 7) + ( 7 + 6)
8 169 8 13 45 15 1
= = + = = =4−
−( 6 + 5) + ( 5 + 4) 3 144 3 12 12 4 4
1
= ( 9  4) = 3 + 2 = 5. Similarly, sum of the given terms is 2008 − .
2008

Chapter 03.indd 11 6/4/2015 10:53:17 AM


This page is intentionally left blank

FM.indd 6 6/10/2015 2:36:16 PM


CHAPTER

Simplification 4
Simple Arithmetic Operations 17 16 9 16 69 
=    
It is a common need to simplify the expressions formulated 2  5 2 3 8 
according to the statements of the problems relating to the
17 16 24 69 
practical life. To do this, it is essential to follow in sequence =    
the mathematical operations given by the term “BODMAS”. 2  5 1 8
17 16 1 69 
BODMAS =   
2  5 24 8 
Each letter of the word BODMAS stands as follows:
B for bracket : [{(–)}] 17  16 69 
=  
There are four brackets, namely, – bar, ( ), { } and [ ]. 2 120 8 
They are removed, strictly in the order –, ( ), { } and [ ].
17 16  1035 
O for of : of = 
2  120 
D for division : ÷
M for multiplication : × 17 1051
= 
A for addition : + 2 120
S for subtraction : – 1020  1051 31
The order of various operations in exercises involving = = −
120 120
brackets and fractions must be performed strictly
according to the order of the letters of the word BODMAS. Illustration 2  Simplify
1  1  1 1 1  
Note: 5  4   3  2   
3  3  3 3 3  
Here, – 5  8 = – (–3) = 3.
Solution: Given expression
Illustration 1  Simplify
16 13  10 7 1  
1  1 1 1   1 5   =     
8  3  4 of 5  11   3  1    3  3  3 3 3  
2  5 2 3   4 8  
Solution: Given expression 16 13  10 6  
=      
17 16 9 16   5 5   3  3  3 3 
=    of  11   3    
2  5 2 3   4 8   16 13 4 
=   
17 16 9 16   5   3  3 3
=    of  11   3   
2 5 2 3   8   16  9  16 9 7 1
=   =  = = 2
17 16 9 16  19  3 3 3 3 3 3
=    of  11  
2 5 2 3  8  Use of Algebraic Formulae
17 16 9 16 69  The following important formulae are sometimes found
=    of 
2 5 2 3 8  useful in dealing with the simplifications:

Chapter 04.indd 1 6/4/2015 3:52:02 PM


4.2 Chapter 4

1. (a + b)2 = a2 + 2ab + b2 4  0.345  0.255


= [Q (a + b)2 – (a – b)2 = 4ab]
2 2 2 4  0.345  0.255
2. (a – b) = a – 2ab + b
=1
3. (a + b)2 + (a – b)2 = 2(a2 + b2)
0.682  0.682  0.318  0.318
4. (a + b)2 – (a – b)2 = 4ab (v)
0.682  0.318
5. a2 – b2 = (a + b) (a – b)
Solution: Given expression
6. (a + b)3 = a3 + 3a2b + 3ab2 + b3
(0.682) 2  (0.318) 2
= a3 + b3 + 3ab(a + b) =
0.682  0.318
7. (a – b)3 = a3 – 3a2b + 3ab2 – b3
= a3 – b3 – 3ab(a – b)  a 2  b2 
= (0.682 + 0.318)   a  b
8. a3 + b3 = (a + b) (a2 – ab + b2)  a b 
9. a3 – b3 = (a – b) (a2 + ab + b2) =1
a3  b3  c3  3abc (3.29) 2  (0.81) 2
10. = (a + b + c) (vi)
a 2  b 2  c 2  ab  bc  ca 4
11. a4 – a4 = (a2 + b2) (a + b) (a – b). Solution: Given expression
(3.29) 2  (0.81) 2
=
Illustration 3 Simplify the following: 3.29  0.81
(i) 0.32 × 0.32 + 0.64 × 0.68 + 0.68 × 0.68  a 2  b2 
= (3.29 – 0.81)   a  b
Solution: Given expression a b
 
= 0.32 × 0.32 + 2 × 0.32 × 0.68 + 0.68 × 0.68 = 2.48
= (0.32)2 + 2 × 0.32 × 0.68 + (0.68)2 (vii) (2.35)3 + 1.95 × (2.35)2 + 7.05 × (0.65)2 + (0.65)3
= (0.32 + 0.68)2 [Q a2 + 2ab + b2 = (a + b)2] Solution: Given expression
= 12 = 1 = (2.35)3 + 3 × 0.65 × (2.35)2
(ii) 2.45 × 2.45 – 0.9 × 2.45 + 0.45 × 0.45 + 3 × 2.35 × (0.65)2 + (0.65)3
Solution: Given expression = (2.35 + 0.65)3
= 2.45 × 2.45 – 2 × 2.45 × 0.45 + 0.45 × 0.45 [Q a3 + 3a2b + 3ab2 + b3 = (a + b)3]
= (2.45)2 – 2 × 2.45 × 0.45 + (0.45)2 = (3)3 = 27
= (2.45 – 0.45)2 [Q a2 – 2ab + b2 = (a – b)2] (4.32)3  0.96  (4.32) 2  12.96  (0.32) 2  (0.32)3
2 (viii)
= (2) = 4 444
7  {(146  92) 2  (146  92) 2 } Solution: Given expression
(iii) (4.32)3  3  0.32  (4.32) 2  3  4.32  (0.32) 2  (0.32)3
(146) 2  (92) 2 =
444
Solution: Given expression
(4.32  0.32)3
7  2{(146) 2  (92) 2 } = [Q a3 – 3a2b + 3ab2 – b3 = (a – b)3]
= 43
(146) 2  (92) 2 3
 4
[Q (a + b)2 + (a – b)2 = 2(a2 + b2)] =   =1
= 14  4

(0.345  0.255) 2  (0.345  0.255) 2 885  885  885  115  115  115
(iv) (ix)
0.345  1.02 885  885  115  115  885  115
Solution: Given expression
Solution: Given expression
(885)3  (115)3
(0.345  0.255) 2  (0.345  0.255) 2 =
= (885) 2  (115) 2  885  115
4  0.345  0.255

Chapter 04.indd 2 6/4/2015 3:52:03 PM


Simplification 4.3

(xi)
 a 3  b3 
= (885 + 115)  2 2
 a  b (2.3)3  (1.5)3  (1.2)3  3  2.3  1.5  1.2
 a  ab  b 
2.3  2.3  1.5  1.5  1.2  1.2  2.3  1.5  2.3  1.2  1.5  1.2
= 1000
Solution: Given expression
0.62  0.62  0.62  0.41  0.41  0.41
(x) (2.3)3  (1.5)3  (1.2)3  3  2.3  1.5  1.2
0.62  0.62  0.62  0.41  0.41  0.41 =
(2.3) 2  (1.5) 2  (1.2) 2  2.3  1.5  2.3  1.2  1.5  1.2
Solution: Given expression
= (2.3 + 1.5 + 1.2)
(0.62)3  (0.41)3  a3  b3  c3  3abc 
=  2  a  b  c
(0.62) 2  0.62  0.41  (0.41) 2 2 2
 a  b  c  ab  ac  bc 
=5
 a 3  b3 
= (0.62 – 0.41)  2 2
 a  b
 a  ab  b  Surds and Indices
= 0.21 an is called a surd if n is a fraction and an is called an index
if n is an integer. a is called the base.

soMe useful forMulae

2
1. am × an = am+n 14. ( a  b ) = a + b – 2 ab
2. am ÷ an = am–n 15. a + b =c+ d ⇒ a = c and b = d
3. (am)n = (an)m = amn 1 a b a b
m m 16. = =
 a b ( a  b )( a  b ) a b
 a n  b n
4.   =  
 b  a 1 a b a b
17. = =
m
5. a ÷ b = a × b –n m n a b ( a  b )( a  b ) a b
18. If x = n(n + 1), then
6. ( n a ) n = a, where ‘n’ is a +ve integer and ‘a’ a
+ve rational number (a) x  x  x  ...  = n
n
7. a n b = n ab , where ‘n’ is a +ve integer and
‘a’, ‘b’ are rational numbers (b) x  x  x  ...  = (n + 1).
n
a a
8. n
= n , where ‘n’ is a +ve integer and ‘a’,
b b
Illustration 4 Find the value of (243)0.8 ÷ (243)0.4.
‘b’ are rational numbers
mn mn nm
Solution: (243)0.8 ÷ (243)0.4 = (243)0.8–0.4
9. a = a = a , where ‘m’, ‘n’ are +ve
[Q am ÷ an = am–n]
integers and ‘a’ is a +ve rational number
= (243)0.4
nm k m n mn
10. (a ) = ak = a km , where ‘m’, ‘n’, 2
5 5
‘k’ are +ve integers and ‘a’ is a +ve rational = (3 ) = 32 = 9
number
Illustration 5 Find the value of (27)2/3 ÷ (64)–4/3
11. a a =a Solution: (27)2/3 ÷ (64)–4/3 = (33)2/3 × (64)4/3
12. a b = ab [Q am ÷ b–n = am × bn]
2 = 32 × (43)4/3
13. ( a  b ) = a + b + 2 ab
= 9 × (44) = 9 × 256 = 2304

Chapter 04.indd 3 6/4/2015 3:52:05 PM


4.4 Chapter 4

( 2)( 4)
Illustration 6  Find the value of ( 3)( 2) 2 3
=
( 2)( 4) (2) 2  ( 3) 2
( 4) (2) ( 2)( 4)
( 2)( 2)  1  1
Solution: ( 3) =   =   =
2 3
=2– 3
 3  9
43
( 4)
(2)
= (9) 4
1 7 3 5 2 7 3 5 2
(ii) =
= (81)–4 =   48  18
 81 4  3  32  2
2

4
1 1
=  4  = =
7 3 5 2
=
7 3 5 2 4 3 3 2

3  3 4 3 3 2 4 3 3 2 4 3 3 2
5
Illustration 7  Find the value of x if 2x  7 – 3 = 0 (7 3  5 2)(4 3  3 2)
Solution: We have =
(4 3  3 2)(4 3  3 2)
5
2 x  7 – 3 = 0
5 7 34 3 7 33 2 5 2 4 3 5 2 3 2
⇒ 2 x  7 = 3 =
(4 3) 2  (3 2) 2
⇒ ( 5 2 x  7)5 = 35
⇒ 2x – 7 = 243 [Q ( n a )n = a] 28 3  3  21 3  2  20 2  3  15 2  2
=
⇒ 2x = 250 or, x = 125 16  3  9  2

Illustration 8  Find the value of 5


64  5 512 28  3  21  6  20 6  15  2
=
48  18
Solution: 5 64  5 512
84  (21  20) 6  30 114 − 41 6
n = =
= 5
64  512  [ anb 
n
ab ] 30 30
5 2 5 5 n
= 8  83 = 8 = 8 [Q a n = a] Illustration 13  If a and b are rational numbers, find the
values of a and b in the following equation:
Illustration 9  Find the value of 3 2
729
3 2
6
= a + b 6.
Solution:   3 2 729 = 729  [ mn
a  mn a ] 3 2
6 6 n n
= 3 = 3 [ a  a] 3 2 3 2 3 2
Solution: = 
7 5
(21 ) 7 5 3 2 3 2 3 2
Illustration 10  Find the value of .
( 3  2) 2
5 3
(75 )3 =
( 3) 2  ( 2) 2
Solution: Given expression
322 3  2 52 6
7
(21)7  nm p m n  p
= =
=   (a )  a  32 1
5 (7)5  
= 5 + 2 6
21 n
= = 3[Q a n = a] 3 2
7 \ = a + b 6
Illustration 11  Find the value of 5 × 125 3 2

⇒ 5 + 2 6 = a + b 6
Solution: 5  125 = 625  [ a  b  ab ]
= 25 On equating rational and irrational parts, we get
a = 5 and b = 2.
Illustration 12  Simplify each of the following by rational-
ising the denominator. Illustration 14  Find the value of
1 7 3 5 2 ( 72  72  72  ...  ) ÷ ( 12  12  12  ...  )
(i) (ii)
2 3 48  18
Solution: Since 72 = 9 × 8
1 1 2 3
Solution: (i) =  therefore, 72 + 72  72  ...  = 9
2 3 2 3 2 3

Chapter 04.indd 4 6/4/2015 3:52:08 PM


Simplification 4.5

Also, since 12 = 4 × 3 2. Comparison of Fractions


The following points are found useful while comparing two
therefore, 12  12  12  ... = 3
or more fractions:

Thus, the given expression = 9 = 3 (a) If the denominators of the fractions are same,
3 the largest is one whose numerator is the largest.
Fractions
Illustration 16 Which is the largest fraction among
1. Continued Fraction
3 7 5
, and ?
2 8 8 8
Fractions of the form 7 
3 7
5 Solution:
2
4 8
1
3 (b) If the numerators of the fractions are same, the
4
are called continued fractions. largest is one whose denominator is the smallest.
To simplify a continued fraction, we start from the
bottom and work upwards Illustration 17 Which is the largest fraction among
5 5 5
Illustration 15 Simplify 3 
1 , and ?
. 2 7 9
7
4 5
5 Solution:
9
2 2
6
3
(c) If neither the numerators nor denominators
1 1 of the fractions are same then they are
Solution: 3  = 3
7 7 converted into equivalent fractions of the
4 4
5 15 same denominator by taking the L.C.M. of
9 9
2 20 the denominators of the given fractions. Then,
6
3 the fractions are compared according to (1).
 Multiply the numerator and denominator of the lowest 
 5 15 
 term by 3 to get .  Illustration 18 Which is the largest fraction among
  2 20 
  6    1 2 4 5
, , and ?
 5  2 3 5 8
1 1
= 3 = 3 Solution: L.C.M. of 2, 3, 5 and 8 = 120
7 28
4 4 1
3 33 1  60 60
9 Then, = =
4 2 2  60 120
 Multiply the numerator and denominator  2 2  40
 = = 80
 of the lowest term 7 28  3 3  40 120
by 4 to get
  3 33  4 = 4  24
= 96
9 

  4 
 5 5  24 120
5 5  15
= 3
33 and, = = 75
8 8  15 120
160
Now, the denominator of these fractions are same and
 Multiply the numerator and denominator of the 
  the largest numerator is 96. Hence, the largest fraction is
 term 1 33  4
by 33 to get 96
  28  160  , that is,
 4+  120 5
 33 
 
480  33 447 127 (d) Two fractions can also be compared by cross
= = = 2 multiplication method.
160 160 160

Chapter 04.indd 5 6/4/2015 3:52:10 PM


4.6 Chapter 4

Illustration 19 Which is greater


6 5
or ? Illustration 22 Which of the following fractions is the least?
13 7
2 4 6 8
Solution: Step 1. Cross multiply the two given fractions , , ,
5 11 17 23
6 5
 , Increase in numerator 1
13 7 Solution: Since = 2 = is less
Increase in denominator 6 3
we get 6 × 7 = 42 and 13 × 5 = 65
2 8
Step 2. Since 65 is greater than 42 and in 65, the numerator than the first fraction , therefore, the last fraction is
6 5 23
of 5 is included, \ 5 is greater than the least
7 7 13

(e) If the difference of the numerator and 3. Inserting a fraction between two given fractions
denominator of each of the given fractions To insert a fraction between two given fractions
be same then the fraction of the largest a1 a
and 2 , the following steps may be useful:
numerator is the smallest. b1 b2

Illustration 20 Which one of the following fractions is the Step 1 The numerators of the two given
largest? fractions are added to get the numerator of the
2 3 5 9 resulting fraction, that is, a1 + a2.
, , and
3 4 6 10 Step 2 The denominators of the two given
Solution: Since in each of the given fractions the difference fractions are added to get the denominator of the
between the numerator and denominator is same and the resulting fraction, that is, b1 + b2.
9
largest numerator is 9, therefore, the largest fraction is a1  a2
10 Step 3 Resulting fraction = .
b1  b2

x x  a x  2a
(f) In the given fractions, , , , ..., 4
x  na , where a < b y y  b y  2b Illustration 23 Insert one fraction between 2 and .
y  nb 5 7
Solution: Using the above method,
Increase in numerator
(a) If > first fraction,
Increase in denominator 2 24 4 2 6 4 2 1 4
, , = , , or, , ,
the last value is the greatest 5 57 7 5 12 7 5 2 7
Increase in numerator
(b) If < first fraction, 9
Increase in denominator Illustration 24 Insert three fractions between 5 and .
the last value is the least. 7 11
Solution: Using the above method,
Increase in numerator
(c) If = first fraction,
5 59 9 5 14 9
Increase in denominator
, , = , , or, 5 , 7 , 9
all values are equal. 7 7  11 11 7 18 11 7 9 11
Further,
Illustration 21 Which one the following fractions is the 5 57 7 79 9 5 12 7 16 9
greatest? , , , , = , , , ,
7 7  9 9 9  11 11 7 16 9 20 11
3 4 5 6 7
, , , , . 5 3 7 4 9
8 11 14 17 20 or , , , ,
7 4 9 5 11
Increase in numerator 1
Solutoin: Since, = is less than
9
Increase in denominator 3 Thus, the three fractions inserted between 5 and are
3 3 7 11
the first fraction , therefore, the first fraction is the
8 8 3 7 4
greatest , and
4 9 5

Chapter 04.indd 6 6/4/2015 3:52:11 PM


Simplification 4.7

Practice Exercises

Difficulty Level-1
(Based on Memory)

2 (a) 22 × 53 × 72 (b) 22 × 35 × 72
 1 3 1
1. If  a   = 3, then what is the value of a + ? (c) 23 × 35 × 72 (d) 25 × 52 × 72
 a a3
[Based on MAT, 2001]
10 3
(a) (b) 0 21/2 31/3 41/4 34/3 57/5
3 8.
The value of  is:
1/5 3/5
(c) 3 3 (d) 6 3 10 5 4 3/5 6
 [Based on MAT, 2003] (a) 5 (b) 6
y ab (c) 10 (d) 15
2.
If a = and b = then is equal to:
x y x y ab  2.75  2.75  2.75  2.25  2.25  2.25 
9.
The value of  is:
xy 2
x y 2  2.75  2.75  2.75  2.25  2.25  2.25 
(a) (b)
x2  y 2 xy (a) 0.30 (b) 0.50
2 (c) 3 (d) 5
x  y 
(c) (d)   [Based on MAT, 2005]
x y  x  y 
[Based on MAT, 2003]  1   1  1   1 x
If 1   1   1   ... 1   =
10. , then what is
 2   3   4   70  70
4ab x  2a x  2b
3. If x = , then the value of  is equal the value of x?
ab x  2a x  2b
to: (a) 69 (b) 35
1
(a) 0 (b) 1
9 (c) 20 (d) 1
1
(c) 2 (d) None of these (0.6)0  (0.1) 1
9 11. is equal to:
1 3 1
[Based on MAT, 2003]  3  3  1
 3      
2 2 3
x  4  x  10 5
4.
Given  . The value of x is: 3 1
x  4  x  10 2 (a)  (b) 
2 2
331 3
(a) 1 (b) 2
5 (c) (d)
3 2
263 17 [Based on MAT, 2005]
(c) (d)
20 21
12.
What is the missing figure in the expression given below?
 [Based on MAT, 2002]
16 16 * 9 9 9
1.073  1.073  0.927 (34 ) 4  96      1
5.
The value of + is: 7 7 7 7 7 7
1.073  0.927 (27)7  (3)9
(a) 1 (b) 7
1 1
(a) 2 (b) 2 (c) 4.57 (d) 32
3 5 [Based on MAT, 2000]
1
(c) 2 (d) 3 13.
If the numerator and the denominator of a proper fraction are
9
increased by the same quantity, then the resulting fraction is:
6. 96 +7, when divided by 8, would have a remainder of: (a) Always greater than the original fraction.
(a) 0 (b) 6
(b) Always less than the original fraction.
(c) 5 (d) None of these
(c) Always equal to the original fraction.
[Based on MAT, 2001]
(d) None of these.
7. H.C.F. of 3240, 3600 and a third number is 36 and their
L.C.M. is 24 × 35 × 52× 72. The third number is:  [Based on MAT, 2001]

Chapter 04.indd 7 6/4/2015 3:52:16 PM


4.8 Chapter 4

2 2 cd cd
14.
A man completes th of his journey by aeroplane, th (a) (b)
15 5 cd cd
by train and the rest by taxi. What part of his journey does
cd cd
he complete by taxi? (c) (d)
cd cd
8 7
(a) (b)  1  3  5  999 
15 15 23.
The value of is  2 −  2 −  2 −  ...  2 − :
 3  5  7  1001 
9
(c) (d) None of these 1003 1003
15 (a) (b)
15. The unit digit of 36 × 47 × 63 × 74 × 82 × 95 is: 3 1001
(a) 6 (b) 5 1
(c) (d) None of these
(c) 4 (d) 2 1001
16. If a man spends five-sixths part of money and then again 24. The sum of the two digits of a number is 13 and the
earns part of the remaining money, what half part of his difference between the number (x) and that formed by
money is with him now? reversing the digits (y) is 27, i.e., (x – y) is 27. Find the
(a) 1/2 (b) 1/4 number.
(c) 2/3 (d) 3/4 (a) 72 (b) 58
(c) 27 (d) 85
17. The difference between the squares of two consecutive
odd integers is always divisible by: [Bases on IMT Ghaziabad, 2002]

(a) 8 (b) 7 25. Find a positive number which when increased by 17 is


(c) 6 (d) 3 equal to 60 times the reciprocal of the number.
[Based on Narsee Manjee Inst. of Man. Studies, 2003] (a) 20 (b) 10

18.
What is the difference between the largest and the smallest (c) 3 (d) 17
5 21 9 6 [Based on IMT Ghaziabad, 2002]
fractions , , and ?
8 35 16 7 26. Four of the following five parts numbered (1), (2), (3), (4)
33 11 and (5) are exactly equal. The number of the part which is
(a) (b) not equal to the remaining four parts will be your answer.
112 37
(a) 36 × 15 + 27 × 13
13 9
(c) (d) (b) 53 × 4 + 64 ÷ 16 × 7
41 35
(c) 328 ÷ 41 × 21 + 9 × 23
19.
When one-fourth of a number is subtracted from one-third
of the same number, the remainder obtained is 12. The (d) 1024  11  16  7
number is: [Based on IRMA, 2002]
(a) 144 (b) 72
27. What approximate value should come in place of question
(c) 120 (d) 63
mark (?) in the following equation?
0
 1
Simplify    (64) 1/2  ( 32) 4/5:
20. 95.9753.5 ÷ 16.0013.5 × 6.0021.5 ÷ 35.992 = ?
 64 
(a) 36 (2) 16
1 3 (c) 96 (d) 6
(a) 17 (b) 17
8 8
 [Based on IRMA, 2002]
7 7
(c) 11 (d) 17
8 8 If x * y = (x + 2)2(y – 2) then 7 * 5 = ?
28.
2
21.
(1.06 + 0.04) – ? = 4 × 1.06 × 0.04: (a) 234 (b) 243
(a) 1.04 (b) 1.4 (c) 343 (d) 423
(c) 1.5 (d) Cannot be determined If m and n are whole numbers such that mn = 121, then
29.
2
a b 2 ab ab (m – 1)n+1 = ?
22.
If = , then find the value of in terms
c2  d 2 cd ab (a) 10 (b) 102
of c and d only. (c) 1000 (d) 104

Chapter 04.indd 8 6/4/2015 3:52:17 PM


Simplification 4.9

30.
If we multiply a fraction by itself and divide the product 1
38.
What fraction must be subtracted from the sum of and
by the square of its reciprocal, the fraction so obtained is 4
13 1 1
3 . The original fraction is: to have an average of of all the three fractions?
81 6 12
16 8 1 1
(a) (b) (a) (b)
9 9 2 3
4 1 1 1
(c) (d) (c) (d)
3 3 4 6
31. Suppose a = 2/3 b, b = 2/3 c, and c = 2/3 d. What would 39.
If we multiply a fraction by itself and divide the product
be the value of b as a fraction of d? 26
by its reciprocal, the fraction thus obtained is 18 . The
(a) 2/3 (b) 4/3 27
(c) 4/9 (d) 8/27 fraction is:
[Based on I.P. Univ., 2002] 8 2
(a) (b) 2
27 3
32. Find x and y:
1
x2 – xy = 4 and y2 – xy = –3 (c) 1 (d) None of these
3
(a) (4, 3) or (–4, –3) 1 1
2 2
(b) (4, 1) or (–4, –1) 40.
Simplify: 7 2 1 .
(c) (3, 4) or (–3, –4) 1 1 1
2 1 2
4 7 1
(d) x = y = 0 2
1
[Based on SCMHRD, 2002] 2
2
ab 1
33.
If a * b = , find 3 * (3 * –1): 1
ab (a) (b) –
2 8
(a) –3 (b) –1.5 1 1
(c) – (d) –
(c) 2/3 (d) 3 6 4
[Based on SCMHRD, 2002]
7
x x1
41.
Which of the following fractions is less than and
 9  8  2 8
34.
Solve     = : 1
 4   27  3 greater than ?
(a) 1 (b) 2 3
(c) 3 (d) 4 1 23
(a) (b)
[Based on SCMHRD, 2002] 4 24
11 17
 3   1  = 7 3 , find the values of x and y. (c) (d)
35.
 2    y  12 24
x 2 4
42.
In a college, one-fifths of the girls and one-eighths of the
(a) (3, 19) (b) (3, 14)
boys took part in a social camp. What of the total number
(c) (14, 3) (d) (24, 6) of students in the college took part in the camp?
36. Which of the given numbers is the greatest? 13 13
(a) (b)
40 80
(a) 6 3 5 (b) 8 3 2
2
(c) (d) Data inadequate
(c) 2 3 130 (d) 3 900 13
[Based on REC Tiruchirapalli, 2002] 43.
Which of the following is true?
37. If x = 2 + 2 2/3 1/3 3 2
+ 2 , then the value of x – 6x + 6x is: A. 99/101 < 97/99 < 95/97
(a) 3 (b) 2 B. 95/97 < 97/99 < 99/101
(c) 1 (d) None of these C. (95/97)2 > (97/99)2 > (99/101)2
[Based on REC Tiruchirapalli, 2002] D. (99/101)2 > (97/99)2 > (95/97)2

Chapter 04.indd 9 6/4/2015 3:52:19 PM


4.10 Chapter 4

(a) Only A (b) Only B 2 2 2


53. If x = y and y = z and z = w,  what fraction of w
(c) B and C (d) B and D 3 3 3
[Based on MAT (Dec), 2006] is y?
4 8 4
44.
If G = H + , then L equals: (a) (b)
L 27 9
(a) 4/(G – H)2 (b) 4(G – H)2 2 4
(c) (d)
2 2
(c) 4/(G – H ) (d) 4(G – H ) 2 2 3 3
[Based on MAT, 1997] [Based on MAT, 1998]

1 1 1 54.
How many multiples of 9 can be found which are less than
45.
If = + , then z equals: 9999 and are perfect cubes?
x y z
(a) xy/(x – y) (b) x – y (a) 5 (b) 6
(c) xy/(y – x) (d) (x – y)/xy (c) 7 (d) 8
[Based on MAT, 1997]
(3 2 − 3)
55.
If 6 = 2.45, then the value of  is:
46.
If (x – 3)(2x + 1) = 0, then possible values of 2x + 1 are: (3 2 + 2)
(a) 0 only (b) 0 and 3 (a) 0.40 (b) 0.41
1 (c) 0.42 (d) 0.44
(c) − and 3 (d) 0 and 7
2 [Based on MAT, 1998]
[Based on MAT, 1997]
56.
Three-fourths of 68 is less than two-thirds of 114 by:
47.
Simplify 2 ÷ [2 + 2 ÷ {2 + 2 ÷ (2 + 2 ÷ 3)}]: (a) 12 (b) 25
(a) 13/15 (b) 17/15 (c) 35 (d) 48
(c) 11/15 (d) None of these [Based on MAT, 1998]

3 3  3 3 1 2 2 The remainder when 87185 is divided by 7 is:


57.
48.
Simplify  of  2  2  + ÷1 
10 7  10 5 5 5 7 (a) 5 (b) 1
(a) 1 (b) 2 (c) 6 (d) 4
(c) 0 (d) 3
The value of (1/x2) + (1/y2), where x = 2 +  3 and
58.
  1  y = 2 –  3 , is:
49.
1  1  1  1  1    = ?
  3  (a) 14 (b) 12
7 2 (c) 10 (d) 16
(a) (b)
5 3 [Based on MAT, 1999]

4 9
(c) (d) None of these If 9 x =
59. then x is:
5 3x
(a) 1/3 (b) 2/3
9 4 3 2 (c) 3 (d) 4/3
50.
48  12   of  of  = ?
8 3 4 3 [Based on MAT, 1999]
(a) 9 (b) 12 5a + 3b 23
60.
If = , then the value of a:b is:
(c) 15 (d) None of these 2a − 3b 5
If ax = b, by = c and cz = a, then the value of xyz = ?
51. (a) 2:1 (b) 1:4
(a) 0 (b) 1 (c) 1:2 (d) 4:1
(c) –1 (d) 2 [Based on MAT, 1999]
 [Based on MAT, 1997] 1
61. Find the value of x4 + , if x = 3 + 2 2 :
52.
If ax = b, by = c and cz = a, then xyz is equal to: x4
(a) 0 (b) –1 (a) 1154 (b) 1024
(c) 1 (d) a + b + c (c) 1734 (d) None of these
[Based on MAT, 1998] [Based on NMAT, 2006]

Chapter 04.indd 10 6/4/2015 4:07:19 PM


Simplification 4.11

62. The value of expression: 63. When simplified, the product:


 xb 
 c
b+c−a
 xc 
 a
c+a−b
 xa 
 b
a+b−c 2−
3
1
2−{ }{ }{ } {
3
5
5
2 − ... 2 −
7
999
1001 }
x  x  x  is equal to:
1003 1001
(a) (b)
(a) xab + bc + ca (b) 1 13 13
991
(c) xabc (d) xa + b + c (c) (d) None of these
1001
[Based on NMAT, 2005] [Based on NMAT, 2005]

Difficulty Level-2
(Based on Memory)

5. The unit digit of the product of all the prime numbers


a 3  b3 13 ab
1.
If 3 3
= , then find : between 1 and (11)11 is:
a b 14 ab
(a) 6 (b) 5
3
(a) (b) 1 (c) 4 (d) 0
2
(c) 2 (d) None of these 6. Find the remainder when 721 + 722 + 723 + 724 is divided
[Based on FMS (Delhi), 2004] by 25.
(a) 0 (b) 2
2. Arrange the following in ascending order of values:
(c) 4 (d) 6
2 2 [Based on FMS (Delhi), 2004]
87  87  87  87 
,  , , 
83  83  89  89  7. A four-digit number is formed, using digits 1, 2, 3 and 4,
without repeating any one of them. What is the sum of all
2 2 such possible numbers?
87 87  87   87 
(a) , ,  ,  (a) 66600 (b) 66660
89 89  89   83 
(c) 66666 (d) 60000
2 2
87 87  87   87  [Based on FMS (Delhi), 2004]
(b) , ,  , 
83 89  83   89 
8. How many multiples of 9 can be found which are less than
2 2 9999 and are perfect cubes?
 87  87 87  87 
(c)   , , ,   (a) 5 (b) 6
 89  89 83  83 
(c) 7 (d) 8
2 2
 87  87 87  87  [Based on FMS (Delhi), 2004]
(d)   , , ,  
 83  83 89  89 
b
9. If a 5b  a 2 then (a, b) could be:
[Based on FMS (Delhi), 2004]
(a) (3, 4) (b) (2, 12)
3.
If two-thirds part of a number is 96, what is the value of
(c) (4, 18) (d) (6, 4)
three-fourths part of the same number?
[Based on IIT Joint Man. Ent. Test, 2004]
(a) 48 (b) 192
(c) 108 (d) 72 10. Manmohan spends one-fifth part of his money as pocket
money and four-fifths of the remainder in other affairs.
4.
If the difference between four-fifths part and three-fourths If he is left with `48 per month, what is the monthly
part of a number is 4, what is the number? income?
(a) 60 (b) 100 (a) `360 (b) `400
(c) 80 (d) 40 (c) `320 (d) `300

Chapter 04.indd 11 6/4/2015 3:52:20 PM


4.12 Chapter 4

11. What is the smallest number with which 1800 must be (a) 162 (b) 152
multiplied to make it a perfect cube? (c) 142 (d) 132
(a) 12 (b) 5
885  885  885  115  115  115
(c) 18 (d) 15 19. =?
885  885  115  115  885  115
[Based on IIT Joint Man. Ent. Test, 2004]
(a) 1000 (b) 770
12. The number 39 + 312 + 315 + 3n is a perfect cube of an
(c) 885 (d) 115
integer for natural number n equalling:
[Based on FMS (Delhi), 2003]
(a) 12 (b) 13
(c) 14 (d) 15  1  6 1
20. If  x   = 3, then the value of  x  6  is:
[Based on IITTM, Gwalior, 2003]  x x
(a) 927 (b) 414
13. The number of ways, in which 8064 can be resolved as the
product of two factors, is: (c) 364 (d) 322
(a) 10 (b) 16 [Based on FMS (Delhi), 2003]

(c) 24 (d) 48 (2.3)3  0.027


[Based on IITTM, Gwalior, 2003] 21. The value of is:
(2.3) 2  0.69  0.09
14. Find the positive integer, which when added to the (a) 2 (c) 3
2
numerator and denominator of , will result in a fraction (d) 2.327 (d) 2.273
13 3
[Based on FMS (Delhi), 2003]
nearest to .
15
 1  1 
(a) 6 (b) 5 22.
The value of 1   1  :
 x  1  x  2
(c) 4 (d) 3
[Based on IITTM, Gwalior, 2003]  1  1 
× 1   1    is:
 x  3  x  4
 
15. If x = 5  2 6, then  x  1 is equal to:
 x  1 1
(a) 1 + (b)
x5 x5
(a) 2 3 (b) 3
1 x5
(c) 2 2 (d) None of these (c) x + (d)
x5 x 1
[Based on IITTM, Gwalior, 2003]
[Based on FMS (Delhi), 2003]
1 1 1
16. The largest number in the sequence 1, 2 2 , 33 , 44 is:
1  x4 1  x2 1
1 23.
The value of   =?
1 x x x (1  x)
(a) 1 (b) 22
1 1 1
(a) (b) 1 + x
(c) 33 (c) 4 4 x
[Based on IITTM, Gwalior, 2003] (c) 1 – x2 (d) 1
[Based on FMS (Delhi), 2003]
s
17. The term (r2 + s)1/2 is approximately equal to r  .
2r If (a/b)x–1 = (b/a)x–3, then x is equal to:
24.
Which of the following is the closest approximation to (a) 1 (b) 1/2
(85)1/2?
(c) 7/2 (d) 2
(a) 9.06 (b) 9.34
[Based on FMS (Delhi), 2003]
(c) 9.22 (d) 9.28
[Based on REC Tiruchiraplli, 2003] 1 1
25.
The expression +
18.
The highest score in an inning was two-ninths of the 1  x (b  a )  x ( c  a ) 1  x( a b)  x(c b)
total score and the next highest was two-ninths of the
remainder. These scores differ by 8 runs. What was the 1
+ (b  c )
:
total score in the innings? 1 x  x(a  c)

Chapter 04.indd 12 6/4/2015 3:52:21 PM


Simplification 4.13

(a) xa–b–c (b) 1


(a) 38 (b) 40
(c) 0 (d) None of these (c) 36 (d) 41
[Based on FMS (Delhi), 2003] [Based on SCMHRD Ent. Exam., 2003]

1 1 33. How much more is 1/2 of 2/3 than 3/4 of 1/3?


26. n  m  is equal to: (a) 1/4 (b) 1/3
1 a 1  amn
(c) 1/12 (d) 7/12
(a) 0 (b) 1
[Based on SCMHRD Ent. Exam., 2003]
(c) 1/2 (d) am+n
[Based on FMS (Delhi), 2003]
34. Assuming that in Harappan era, rocks, stones and pebbles
were used for money. The following used to be the
27. Consider the following statements: currency valuations:1 rock = 7 stones, 1 stone = 7 pebbles.
A. If a x= b, b y = c, c z = a, then xyz = 0. If a person used 6 rocks to purchase a cave that costs 5
rocks, 2 stones and 3 pebbles, then how much would the
B. If p = a x, q = a y, (p yq x) z = a2, then xyz = 1 change be?
C. x a = y b = z c, ab + bc + ca = 0, then xyz = 1. (a) 4 stones, 4 pebbles
(b) 5 stones, 4 pebbles
Of these statements:
(c) 1 rock, 5 stones, 4 pebbles
(a) A and B are correct
(d) 5 stones, 5 pebbles
(b) B and C are correct [Based on SCMHRD Ent. Exam., 2003]
(c) Only A is correct
35.
In an examination, a student was asked to find
(d) A and C are correct three-fourths of a certain number. By mistake he found
[Based on FMS (Delhi), 2003] three-fourths of it. His answer was 150 more than the
28.
Simplify: correct answer. The given number is:
(a) 180 (b) 240
a1/2  a 1/2 1  a 1/2 (c) 280 (d) 290

1 a 1 a
a 17 ab
36.
If = , what is equal to?
a a 1 ab 23 ab
(a) (b)
a 1 2
11 17
(a) (b)
2 2 23 32
(c) (d)
a 1 1 a 23
23
(c) (d)
11 17
29. Let (a/b) – (b/a) = x:y and (x – y) = (a/b) + (b/a), then
x is equal to: 37.
If we multiply a fraction by itself and divide the product
(a) (a + b)/a (b) (a + b)/b 26
by its reciprocal, the fraction thus obtained is 18 . The
(c) (a – b)/a (d) None of these 27
original fraction is:
[Based on IIFT, 2003]
8 2
30. If (3) (x+y)
= 81 and 81 (x–y)
= 3, then the values of x and y are: (a) (b) 2
27 3
(a) 17/8, 9/8 (b) 17/8, 15/8
1
(c) 17/8, 11/8 (d) 15/8, 11/8 (c) 1 (d) None of these
3
 [Based on IIFT, 2003]
8
Given that x and y are real numbers, let P (x, y) = x2 – y2.
31. 38.
A boy was asked to multiply a given number by .
17
Then, P (3, P (3,4)) = ? 8
Instead, he divided the given number by and got the
(a) –40 (b) –7 17
(c) 40 (d) 7 result 225 more than what he should have got if he had
[Based on IIFT, 2003] 8
multiplied the number by . The given number was:
17
32. Of the two-digit numbers (those from 10 to 99, both
inclusive), how many have a second digit greater than the (a) 8 (b) 17
first digit? (c) 64 (d) 136

Chapter 04.indd 13 6/4/2015 3:52:22 PM


4.14 Chapter 4

5 1 5 1 1 1
39. If a = and b = , then the value of (a) 1 (b)
1
5 1 5 1 3 4
1
 a 2  ab  b 2  (c) 1 (d) None of these
 2 would be: 7
2
 a  ab  b  [Based on FMS (Delhi), 2006]
(a) 3/4 (b) 4/3
1 1 1 1
(c) 3/5 (d) 5/3 46.
The value of    correct to three
2 2.3 2.3.4 2.3.4.5
[Based on FMS (Delhi), 2002]
places of decimal is:
40. If (x + y):(x – y) = 4:1, then (x2 + y2):(x2 – y2) = ? (a) 0.713 (b) 0.715
(a) 25:9 (b) 16:1 (c) 0.717 (d) 0.718
(c) 8:17 (d) 17:8
[Based on FMS (Delhi), 2002]
6
47.
A person was to multiply a fraction by . Instead, he
7
41. A number consists of two digits whose sum is 7. If the divided and got an answer which exceeds the correct
digits are reversed, then the number is increased by 27. 1
answer by . The correct answer was:
The number is: 7
(a) 25 (b) 34 6 36
(a) (b)
(c) 16 (d) 52 13 91
[Based on FMS (Delhi), 2002]
7
(c) (d) None of these
42. The expression: 13
(2a – 3b) (4a + 0.5b) – a (8a – 11b)
48.
In a certain college, the number of girls is twice the
(a) Is always positive for all a and b number of boys. One-fifth of the girls and one-eighth of
(b) Is always negative for all a and b the boys took part in a social camp. What part of the total
(c) Is zero number of students took part in the camp?
(d) Depends on the values of a and b 7 7
(a) (b)
[Based on SCMHRD, 2002] 40 80

3 1 5 1 2 1
43.
The difference between the sum of 1 ,2 ,3 ,5 (c) (d)
4 3 12 5 12 24
1
and 2 and the nearest whole number is: 1 1
6 49.
How many s are there in 37 ?
8 2
2 13
(a) (b) (a) 300 (b) 400
15 15
(c) 500 (d) Cannot be determined
11
(c) (d) None of these
60
 1 1 1 1 2 1 7  2 1 
50.
7   of   2  1 of 1  1   = ?
44. At the first stop on his route, a driver unloaded two-fifths  2 2 2 4 5 3 8  5 3 
of the packages in his van. After he unloaded another three
packages at his next stop, half of the original number of 1 1
(a) 3 (b) 2
packages in the van remained. How many packages were 5 24
in the van before the first delivery? 1
(a) 10 (b) 25 (c) 4 (d) None of these
30
(c) 30 (d) 36
[Based on REC Tiruchirapalli, 2002]
1 1 1
51.
The value of 1 +   up to four places
4  3 4  32 4  33
 1  4 of decimals is:
45. 1   1 is equal to:
1  7 (a) 1.1202 (b) 1.1203
 1 
 1
1  (c) 1.1204 (d) None of these
 3 

Chapter 04.indd 14 6/4/2015 3:52:23 PM


Simplification 4.15

61.
The value of:
1  1  1  1 1 1  
7   2  1  ? 1     = 3:
52.
2  4  4  2 3 6   ( x a /xb )( a  b )  ( xb /x c )(b  c )  ( x c /x a )( c  a ) is :

1 3 (a) 0 (b) x abc
(a) (b)
4 4
(c) xa + b + c (d) 1
4
(c) (d) None of these [Based on FMS, 2006]
3
62.
The value of [1/(216) – 213 + 1/(256) – 314 + 1/(243)
Find the remainder when 721 + 722 + 723 + 724 is divided
53. – 1/5] is:
by 25.
(a) 107 (b) 105
(a) 0 (b) 2
(c) 103 (d) None of these
(c) 4 (d) 6 [Based on FMS, 2006]
54.
A four-digit number is formed, using digits 1, 2, 3 and 4,
1 17
without repeating any one of them. What is the sum of all 63.
If x is an integer such that x   , then the value of
such possible numbers? x 4
1
x– is:
(a) 66600 (b) 66660 x
(c) 66666 (d) 60000 13
(a) 4 (b)
55.
What is the smallest number with which 1800 must be 4
multiplied to make it a perfect cube? 15 1
(c) (d)
(a) 12 (b) 5 4 4
(c) 18 (d) 15 [Based on FMS, 2006]

The number 39 + 312 + 315 + 3n is a perfect cube of an


56. x 1 x 1
64.
If  2 , then the value of x is:

integer for natural number n equalling: 1 x x 6
(a) 12 (b) 13 6 4 3 2
(a) or (b) or
(c) 14 (d) 15 13 13 2 3
57.
The rank of 2/9 in the following fraction when expressed 5 2 9 4
(c) or (d) or
in ascending order is –2/3, 1/7, 0, 4/9, 2/9, 14/15, 9/11. 2 3 13 13
(a) 4 (b) 5 [Based on FMS, 2009]
(c) 6 (d) 9 ab cd
65.
If  , then:
bc d a
58.
The sum of the squares of two numbers is 3341 and the
difference of their squares is 891. The numbers are: (a) a must equal c
(a) 25, 46 (b) 35, 46 (b) a + b + c + d must equal zero
(c) 25, 36 (d) None of these (c) either a = c or a + b + c + d = 0, or both
[Based on FMS (MS), 2006] (d) a(b + c + d) = c(a + b + d)
[Based on FMS, 2009]
 1  1  1  1 66.
Let r be the result of doubling both the base and the
59.
1   1   1   ... 1   is equal to:
3 4  5  n exponent of ab, b ¹ 0. If r equals the product of ab by xb,
1 2 then x equals:
(a) (b) (a) 2a (b) 4a
n n
(c) 2 (d) 4
2 2(n  1)
(c) (d) [Based on FMS, 2010]
n(n  1) n
[Based on FMS (MS), 2006]
m 4 r 9 3mr  nt
67.
If  and  , the value of is:
n 3 t 14 4nt  7 mr
 1  1
If  x 4  4  322, the value of  x   is :
60.
 1 11
 x   x (a) –5 (b) 
2 14
(a) 4 (b) 6
1 11
(c) 8 (d) 3 2 (c) 1 (d)
4 14
[Based on FMS, 2005] [Based on FMS, 2011]

Chapter 04.indd 15 6/4/2015 3:52:25 PM


4.16 Chapter 4

4 4 (a) I and II (b) I Only


Simplify  3 6 a9   6 3 a9  ; the result is:
68. (c) I and III (d) II and III
(a) a16 (b) a12 [Based on ATMA, 2005]
8 4
(c) a (d) a a − ab + b 2
1 2
a
75. If   = , then find .
[Based on FMS, 2011] a 2 + ab + b 2 3 b
1 1 (a) 1 (b) 2
69.
The expression 1   equals: (c) 3 (d) 4
1 3 1 3
[Based on CAT, 2009]
(a) 1  3 (b) 1
76. Which one of the terms 21/ 3, 31/ 4, 41/ 6, 61/ 8 and 101/12 is the
(c)  3 (d) 3 largest?
[Based on FMS, 2011] (a) 21/ 3 (b) 31/ 4

1 1 (c) 41/ 6 (d) 101/12


70.
The expression 2  2   equals: [Based on CAT, 2012]
2 2 22
x y z
(a) 2 (b) 2  2 77. If = = , then xy + yz + zx is equal to:
a b c
(c) 2  2 (d) 2 2
(a + b + c) 2
[Based on FMS, 2011] (a)
x2 + y 2 + z 2
2n  4  2(2n ) x 2 (a + b + c) 2 − a 2 ( x 2 + y 2 + z 2 )
71. when simplified is: (b)
2(2n  3 ) 2a 2
ax + by + cz
1 (c )
(a) 2n1  (b) –2n1 (a + b + c) 2
8
ax 2 + by 2 + cz 2
7 (d )
(c) 1 – 2n (d) (a + b + c)
2

8
[Based on FMS, 2011] [Based on CAT, 2013]

1 1
= 1 and p = x 4000 + 4000 and q be the digit at units
78. If x +
72.
If u1
 3, u2
 3 , u3
3 3 3 3 etc, u10 : u9 is : x n
x
place in the number 22 + 1, n being a natural number
1 greater than 1, then (p + q) is equal to:
(a) 3 (b) 310 (a) 8 (b) 6
1 (c) 4 (d) 2
(c) 3 20 (d) None of these [Based on CAT, 2013]
[Based on IIFT, 2007] 1 3
79. Find the value of + :
1 4
73.
What is the value of x that would satisfy 1+ 3−
4 1
3− 3+
( 3 + 2) x + ( 3 − 2) x = 10: 2+
1
2−
1
1 2
1 3−
(a) ± 2 (b) − 2
3
13 15
(c) 4 (d) – 4 (a) (b)
[Based on ATMA, 2008]
7 7
11 17
74. Which of the following is/are true? (c) (d)
21 28 [Based on CAT, 1996]
I. (xb–c)a (xc–a)b (xa–b)c = 1 6
80. 5 – 1 is divisible by:
3 19 16 (a) 13 (b) 31
II. Fractions , of are in descending order.
18 20 19 (c) 5 (d) None of these
III. If log (x + 1) – log (x – 1) = log 2, then x = 3 [Based on CAT, 1995]

Chapter 04.indd 16 6/4/2015 3:52:33 PM


Simplification 4.17

81. For the product n(n + 1)(2n + 1), n Î N, which one of the grade in the course if the average of her scores is more
following is not necessarily true? than or equal to 90. Grade B is awarded to a student
(a) It is even if the average of her score is between 87 and 89 (both
included). If the average is below 87, the student gets
(b) Divisible by 3 a C grade. Ramesh is preparing for the last quiz and he
(c) Divisible by the sum of the squares of first n natural realizes that he must score a minimum of 97 to get an A
numbers grade. After the quiz, he realizes that he will score70,
(d) Never divisible by 237 and he will just manage a B. How many quizzes did
Prof. Suman take?
[Based on CAT, 1995]
(a) 6 (b) 7
82. Prof. Suman takes number of quizzes for a course. (c) 8 (d) 9
All the quizzes are out of 100. A student can get an A [Based on XAT, 2014]

Answer Keys
Difficulty Level-1

1. (b) 2. (a) 3. (a) 4. (c) 5. (c) 6. (a) 7. (b) 8. (c) 9. (b) 10. (d ) 11. (a) 12. (d ) 13. (a)
14. (b) 15. (a) 16. (b) 17. (a) 18. (a) 19. (a) 20. (a) 21. (a) 22. (d ) 23. (a) 24. (d ) 25. (c) 26. (a)
27. (d ) 28. (b) 29. (c) 30. (c) 31. (c) 32. (a) 33. (a) 34. (d ) 35. (c) 36. (a) 37. (b) 38. (d ) 39. (b)
40. (d ) 41. (d ) 42. (c) 43. (d ) 44. (a) 45. (c) 46. (d ) 47. (c) 48. (c) 49. (a) 50. (b) 51. (b) 52. (c)
53. (b) 54. (c) 55. (d) 56. (b) 57. (b) 58. (a) 59. (b) 60. (d ) 61. (a) 62. (b) 63. (d )

Difficulty Level-2

1. (c) 2. (c) 3. (c) 4. (c) 5. (d) 6. (a) 7. (b) 8. (b) 9. (b) 10. (d ) 11. (d ) 12. (c) 13. (c)
14. (b) 15. (d ) 16. (c) 17. (c) 18. (a) 19. (a) 20. (d ) 21. (a) 22. (d ) 23. (d ) 24. (d ) 25. (b) 26. (b)
27. (b) 28. (d ) 29. (d ) 30. (b) 31. (a) 32. (c) 33. (c) 34. (a) 35. (c) 36. (c) 37. (b) 38. (d ) 39. (b)
­­­ 40. (d ) 41. (a) 42. (b) 43. (a) 44. (c) 45. (d ) 46. (c) 47. (b) 48. (a) 49. (a) 50. (c) 51. (b) 52. (b)
53. (a) 54. (b) 55. (d ) 56. (c) 57. (a) 58. (b) 59. (b) 60. (a) 61. (d ) 62. (c) 63. (c) 64. (d ) 65. (c)
66. (b) 67. (b) 68. (d ) 69. (a) 70. (a) 71. (d ) 72. (d ) 73. (a) 74. (c) 75. (a) 76. (d ) 77. (b) 78. (b)
79. (d ) 80. (b) 81. (d ) 82. (d )

Explanatory Answers

Difficulty Level-1

2
 1 1 x y
(b)  a   = 3 ⇒ a  =
1. 3 2. (a) a = ,b=
 a a x y x y
3
 1 1  1
\  a   = a3  3  3  a  
 a a  a xy x2  y 2
\ ab = , a+b=
1 x2  y 2
x2  y 2
⇒      3 3  a3  3  3 3
a
1 ab xy
⇒ a3  3 = 0. \ = .
a a  b x2  y 2

Chapter 04.indd 17 6/4/2015 3:52:33 PM


4.18 Chapter 4

6ab  2a 2 6b 2  2ab (22  32 )  24  35  52  7 2


x=

x  2a x  2b ab ab 23  34  5  24  32  52
3.
(a)  = 
x  2a x  2b 2ab  2a 2 2ab  2b 2 26  37  52  7 2
ab ab =
27  36  53
2a (3b  a ) 2b (3b  a ) = 22 × 35 × 72.
= 
2a (b  a ) 2b (a  b)
21/2  31/3  41/4 34/3  57/5
3b  a 3b  a 8.
(c) 
=  = 0. 10 1/5  53/5 4 3/5  6
ba ba
21/2  31/3  (22 )1/4  101/5 34/3  43/5
4.
(c) Given expression = 
53/5 57/5  6
2
( x  4  x  10) 25
⇒ = 21/2  31/3  21/2  21/5  51/5 57/5  2  3
( x  4)  ( x  10) 4 = 3/5

5 34/3  26/5
2
x  4  x  10  2 x  6 x  40 5 1 1 1 6 1 4 1 3 7
⇒ =    1  1  
14 2 = 2 2 2 5 5  33 3  55 5 5

⇒   2 x  2 x 2  6 x  40 = 41 = 21 × 30 × 51 = 2 × 5 = 10

⇒   (2x – 41)2  = [ 2 x 2  6 x  40]2 a 3  b3


9.
(b) By using =a–b
a 2  ab  b 2
⇒ 4x2 + 1681 – 164x = 4 (x2 – 6x – 40)
 = 4x2 – 24x – 160 (2.75)3  (2.25)3
= 2.75 – 2.25 = 0.50.
(2.75)  2.75  2.25  (2.25) 2
2
⇒ 140x = 1841
263 1 1 1 1 x
⇒ x= . (d) 1   1   1   ... 1   =
10.
20  2   3  4   70  70

1.073  1.073  0.927  0.927 (34 ) 4  (9)6 1 2 3 69 x


5.
(c) + \    ... =
2 3 4 70 70
1.073  0.927 (27)7  (3)9
1 x
(1.073) 2  (0.927) 2 (34 ) 4  (32 )6 \ =
=  3 7 70 70
1.073  0.927 (3 )  (3)9 \ x = 1.
1
(1.073  0.927)(1.073  0.927) 328 1
=  30 1  
1.073  0.927 3  10  1  10
11.
(a) = 3 3
1  2   33
3 2 3
1 1  3
= 2 + =2+ =2 .  3   2   ( 3) 3 23
32 9 9  

9 3
(a) Consider 9, 92, 93, 94, 96. Each of the these numbers,
6. = =  .
93 2
when divided by 8, will leave a remainder of 1.
\ 96 + 7, when divided by 8, will leave a remainder * 9 256 81
12.
(d)   =   1
of 0. 7 7 49 49
7.
(b) The product of the numbers = H.C.F × L.C.M. ⇒ – * × 9 = – 256 – 81 + 49
Let the third number be x. ⇒ * = 32.
\ 3240 × 3600 × x = 36 × 24 × 35 × 52 × 72 1 11 1 2
13.
(a) < ⇒ <
4 5 2 2 2 2 1 2 3
36  2  3  5  7
⇒ x = 2 2 1 2 3
(2  34  5)  (24  32  52 )
3
< ⇒ <
3 3 1 3 4
 3240  23  34  5 
  2 2 1 2 1
< ⇒ < .
 3600  24  32  52  5 5 1 5 2

Chapter 04.indd 18 6/4/2015 3:52:36 PM


Simplification 4.19

14. (b) Journey completed by aeroplane and train 19.


(a) Let the number be 1
2 2 26 8 1 1 1 1
=  = = \ of 1 =  and,  of 1 =
15 5 15 15 3 3 4 4
8 7 1 1 43 1
\ Remaining journey = 1 – = \  = =
15 15 3 4 12 12
7 1
\ He completed th part of his journey by taxi. \ Number 12 ÷ = 144.
15 12
(a) The unit digit of 36 is 9
15. 0
The unit digit of 47 is 4 (a)  1   (64)  1/2  (  32) 4/5
20.
  64 
The unit digit of 63 is 6
The unit digit of 74 is 1 = 1 + (82)–1/2 + (–1 × 32)4/5
The unit digit of 82 is 4 = 1 + 8–1 + [(–1)4/5 × (32)4/5]
The unit digit of 15 is 9 = 1 + 8–1 + [((–1)2)2/5 × (25)4/5]
Therefore the unit digit of the given expression is 6
(Since 9 × 4 × 6 × 1 × 4 × 9 = 7776). 1 1
= 1 + + [1 × 16] = 17 .
8 8
16.
(b) Let the money with the man at first be `1
21. (a) Putting x for ? and solving
5 5
\ Money spent = of 1 = ` (1.06 + 0.04)2 – x = 4 × 1.06 × 0.04
6 6
Here, 1.06 = a and 0.04 = b
5 1
\ Remaining money = 1 – =` \ (a + b)2 – x = 4ab
6 6
\  x = (a + b)2 – 4ab = (a – b)2
1 1 1 = (1.06 – 0.04)2
and money earned = of ` = `
2 6 12 = (1.02)2 = 1.0404.
\ Total money with him now
a 2  b2 ab a 2  b2 2ab
1 1 3 1 22.
(d) = or, 2 =
= + =` =` c2  d 2 cd c  d2 2cd
6 12 12 4
1 a 2  b 2  2ab c 2  d 2  2cd
\ th part of his money is with him now. or, =
2 2
4 a  b  2ab c 2  d 2  2cd

(a) (2x + 3)2 – (2x + 1)2 = 4x2 + 12x + 9 – (4x2 + 4x + 1)


17. [by componendo and dividendo]
= 8x + 8 = 8 (x + 1). a  b
2
cd
2
or,  = 
18.
(a) L.C.M. of 7, 8, 16 and 35 = 560  a  b   c  d 
5 5 × 70 350 ab cd
\ = = \ = .
8 8 × 70 560 ab cd
21 21 × 16 336
= = 23.
(a) Given product
35 35 × 16 560
5 7 9 1003 1003
=    ...  =
9 9 × 35 315 3 5 7 1001 3
= =
16 16 × 35 560
24. (d) Let the number be 10p + q
6 6 × 80 480 \ p + q = 13 and (10p + q) – (10q + p) = 27
and, = =
7 7 × 80 560 (x = 10p + q, y = 10q + p)
\ Difference between the largest and the smallest i.e., 9p – 9q = 27,  i.e.,   p – q = 3
fractions \ p = 8, q = 5
6 9 480 315 Hence the required number = 85.
= − = −
7 16 560 560 25. (c) Let the number be x
165 33 60
= = . \ x + 17 = ⇒ x2 + 17x – 60 = 0
560 112 x

Chapter 04.indd 19 6/4/2015 3:52:38 PM


4.20 Chapter 4

⇒ (x + 20) (x – 3) = 0 33.
(a) Given expression
⇒ x = 3 (x > 0).  3  ( 1)   3 
= 3 *   3* 
 3  ( 1)   2
26.
(a) (1) = 36 × 15 ÷ 27 × 13
15  3  9
= 36 × × 13 = 260 3 
27  2
= = 2 = –3.
(2) = 53 × 5 + 64 ÷ 16 × 7  3  3
3 
64  2 2
= 53  4  7
16
x x1
= 212 + 28 = 240  9  8 2
34.
(d)      =
4 27 3
328
 21  9  23
(3) = 2x 3( x  1)
41  3  2 2
⇒     =
= 168 + 9 × 8 2  3 3

= 168 + 72 = 240 2 x 3 x 3 1
 2  2  2
(4) = 1024  11  16  7 ⇒     =  
3  3  3
= 32 × 11 – 112
⇒ – 2x + (3x – 3) = 1 ⇒ x = 4.
= 352 – 112 = 240
(5) = 17  18  121  6 35.
(c) Taking the quotient 2, y and 7, we get 2y = 7, which
= 17 × 18 – 11 × 6 gives the quotient as 3
= 306 – 66 = 240. \ y = 3. Substituting the value of y, we get
3 1 3
963.5  61.5 1 2  3 = 7
27.
(d) Given expression =  x 2 4
163.5 362
3
7
65 4 = 2 3 ⇒ 2 3 = 2 3 .
 = = 6. Now,
64 1 x 14 x
3
2
28.
(b) Substituting x = 7 and y = 5, we get
\ x = 14, y = 3.
7 * 5 = (7 + 2)2 (5 – 2) = (9)2 × 3 = 243.
(a) (6  3 5)3 = 216 × 5 = 1080
36.
(c) Given that mn = 121 ⇒ mn = 112
29.
Hence, m = 11, n = 2. Substituting these values (8 × 3 2)3 = 512 × 2 = 1024
(m – 1)n+1 = (11 – 1)2+1 = 103 = 1000.
(2  3 130)3  = 8 × 130 = 1040
30.
(c) Let x be the fraction
2
( 3 900)3 = 900.
4
 1 13 256 4
x × x ÷   = 3 ⇒ x4 = =   37. (b) x = 2 + 22/3 + 21/3
 x 81 81  3
⇒ (x – 2) = 22/3 + 21/3
4
\ x = . ⇒ (x – 2)3 = (22/3 + 21/3)3
3
= 4 + 2 + 3 × 22/3 × 21/3
2 2
31.
(c) b = c, c = d [22/3 + 21/3]
3 3
= 6 + 3 × 2 (x – 2)
2 2 4
⇒ b = × d = d. ⇒ (x – 2)3 = 6 + 6x – 12 = 6x – 6
3 3 9 3
⇒ x – 8 – 6x (x – 2) = 6x – 6
32.
(a) x2 – xy = 4 ⇒ x (x – y) = 4 (1) ⇒ x3 – 6x2 + 6x = 2.
y2 – xy = –3 ⇒ y (y – x) = –3 (2)
1 1 1
38.
(d) Let   x = 3 ×
Equations (1) and (2) 4 6 12
4 3 x 4 1 1 1 1
⇒          ⇒  . Then,  x = or, x = .
x y y 3 4 6 4 6

Chapter 04.indd 20 6/4/2015 3:52:40 PM


Simplification 4.21

a 4
39.
(b) Let the fraction be , then 44. (a) G = H +
b L
4
 a a b 26 512 ⇒ (G – H)2 =
    = 18 27 = 27 L
b b a 4
⇒ L = .
 a
3
 8
3 (G − H ) 2
or,   =  3 
b 1 1 1 y−x
45.
(c) = − =
z x y xy
a 8 2
\ = =2 . xy
b 3 3 ⇒ z = .
y−x
40.
(d) Given expression 46.
(d) (x – 3) (2x + 1) = 0
15 5 ⇒ x – 3 = 0 or, 2x + 1 = 0

1 If x = 3, 2x + 1 = 7
= 7 2 
9 8 1 Hence, possible values of 2x + 1 are 0 and 7.
 2
4 7 2 47.
(c) Given expression
2
3    2  
= 2 ÷  2  2  2  2   2   
5 28 1    3  
=  
14 95 2  3
  3 
8 = 2 ÷  2  2  2  2  
  8 
2 8 2 19 1
=  =  = .  11   4
19 19 19 8 4 = 2 ÷ 2  2   = 2 ÷ 2  2  
 4  11 
1 7 30 11 11
41.
(d) = 0.33 and = 0.875 =2÷ =2× = .
3 8 11 30 15
1 48.
(c) Given expression
= 0.25 does not lie between 0.33 and 0.875 3 3
4  23 13  1 5 2
=  of      
10 7  10 5  5 7 7
23
= 0.96 which exceeds 0.875 3 3 49 1 2 3 21 1
24 =  of   =  
10 7 10 7 7 10 10 7
11
= 0.92 which exceeds 0.875 3 10 1 1 1
12 =   =  = 0.
10 21 7 7 7
17
= 0.708 which lies between 0.33 and 0.875. 49.
(a) Given expression
24   2 
42.
(c) Out of the 5 girls, 1 took part in the camp. = 1 + 1 ÷ 1  1    
  3 
Out of the 8 boys, 1 took part in the camp.  3
Out of the 13 students, 2 took part in the camp. = 1 + 1 ÷ 1  1  
 2
2  3 5
\ th of total number of students took part in the = 1 + 1 ÷ 1   = 1 + 1 ÷
13  2 2
camp.
2 7
2 = 1 + 1 × =1+ = .
99  99  5 5
43.
(d) = 0.9802,   = 0.9607
101  101  50.
(b) Given expression
2 9 4 3 2
97  97  = 48 ÷ 12 ×  of  of 
= 0.9797,   = 0.9600 8 3 4 3
99  99 
48  9 4   3 2  
2 =         
95  95  12  8 3   4 3  
= 0.9793,   = 0.9592
97  97  48  3 
=    2 = 4 × 3 = 12.
Hence, only B and D are correct. 12  2 

Chapter 04.indd 21 6/4/2015 3:52:43 PM


4.22 Chapter 4

51. (b) ax = b, by = c, cz = a 1 1 1 1
On multiplying, we get 58. (a) 2
+ 2
= 2
+
x y (2 + 3) (2 − 3) 2
ax × by × cz = a × b × c
⇒ (abc)xyz = (abc)1 1 1
= + = 14.
⇒ xyz = 1 = 1 7+4 3 7−4 3
52. (c) Here, a = cz = (by)z = byz = (ax)yz = axyz 9 32
\   a1 = axyz 59.
(b) 9x = ⇒ 32x =
3x 3x
\  xyz = 1.
2 2 2 ⇒ 3 = 32–x = 2x = 2 – x
2x
53. (b) Given x = y, y = z , z = w ⇒ x = 2/3.
3 3 3 a
3 2 23 5 +3
5a + 3b 23
\ y =z= w
2 3 60. (d) = ⇒ b =
5 a
2 2 4
2a − 3b 2 −3 5
or, y = × w = w b
3 3 9 a a
⇒ 25 + 15 = 46 − 69
54. (c) 27, 216, 729, 1728, 3375, 5832, 9261 b b

3 2− 3 3 2− 3 a a 4
55.
(d) Given expression = = ⇒ 21 = 84 ⇒ = .
3 2+ 2 4 2 b b 1
3 1 3 61. (a) x = 3 + 2 2
 = −
4 4 2 = 3 + 2 × 1.414
3 1 6 3 1 2.45 = 5.828, i.e., less than 6
 = − = − × Number must be close and less than 64 = 1296
4 4 2 4 4 2
\ Required number = 1154.
3
 = – 0.31 = 0.75 – 0.31 = 0.44.
4 62. (b) x(b – c)(b + c – a) × x(c – a)(c + a – b) × x(a – b)(a + b – c)

2 3 = x0 = 1.
56.
(b) of 114 – of 68 = 76 – 51 = 25.
3 4 63. (d) When simplified

(b) Since (a + 1)
57.
a
n
leaves always remainder 1. { }{ }{ } {
2−
1
3
2−
3
5
5
2 − ... 2 −
7
999
1001 }
81785 (7 + 1)1785 5 7 9 11 1003 1003
\ = gives the remainder 1. = × × × × ... × = .
7 7 3 5 7 9 1001 3

Difficulty Level-2

If x < 1 ⇒ x2 < x
a 3  b3 13
1. (c) 3 3
= If x > 1 ⇒ x2 > x.
a b 14
⇒ 14a3 – 14b3 = 13a3 + 13b3 2
3. (c) Q part = 96
a 3
⇒ a3 = 27b3 ⇒ =3
b 3 3 3
a \ part = 96 × × = 108.
1
ab b 4 4 2 4
\ = = = 2.
ab a 2
1 4. (c) Let the number be 1
b 4 4
2 3 3
 87  \ of 1 =  and,  of 1 =
2. (c)   = 0.9555 5 5 4 4
 89 
87 4 3 1
= 0.9775 \ Difference = – =
89 5 4 20
87 1
= 1.0481 \ Number = 4 ÷ = 80.
83 20
2
 87  5.
(d) The set of prime numbers.
  = 1.0985
 83  S = {2, 3, 5, 7, 11, 13, ...}.

Chapter 04.indd 22 6/4/2015 3:52:46 PM


Simplification 4.23

Since there is one 5 and one 2 which gives 10 after


⇒ x = 3 2
multiplying mutually, it means the unit digit will be zero.
(a) Given expression = 721 (1 + 7 + 72 + 73)
6. x 1 3  2 1 3 2
\ = 
= 721 × 400, x 3 2 3 2
which is completely divisible by 25 = 1  3  2 = 1  ( 3  2).
Hence, remainder is zero.
16.
(c) 11/2 = 1, (21/2)12 = 26 = 64
7.
(b) Sum of the digits in the thousand’s place
(3 ) = 34 = 81, (41/4)12 = 43 = 64.
1/3 12
= 6000 + 12000 + 18000 + 24000 = 60000
17.
(c) r = 9, s = 4
Sum of the digits in the hundred’s place
= 600 + 1200 + 1800 + 2400 = 6000 s 4 2
\ 851/2 = r  = 9 = 9 = 9.22.
Sum of the digits in the ten’s place 2r 18 9
= 60 + 120 + 180 + 240 = 600 18.
(a) Let the total score be x runs, such that
Sum of the digits in the unit’s place 2 2  2  2 2 7
= 6 + 12 + 18 + 24 = 60. x    x  x = 8 or, x   x = 8
9 9  9  9 9 9
8.
(b) 27, 216, 729, 1728, 3375, 5832, 9261. 2 2
or x  = 8 or, x = 162.
9 9
9.
(b) Put a = 2, b = 12 in ab = 5b + a2
(a) a3 + b3 = (a + b) (a2 + b2 – ab)
19.
\ 212 = 5 × 12 + 4 = 64 Put, a = 885, b = 115, we have
⇒ 26 = 64, which is true.
a 3  b3
? = = a + b = 1000.
10.
(d) Let the monthly income of Manmohan be `1 a  b 2  ab
2
1 1
\ Pocket money = of  `1 = ` 1  
5 5 20.
(d) x  = 3 ⇒   = 27
x 
1 4
and remainder = 1 – =  ` 1 1 1
5 5 ⇒ x3  3 + 3 × x ×  x   = 27
x x x
4 4 16
\ Other expenses = of  ` =  ` 1
5 5 25 ⇒ x3  3 = 18
x
4 16 4 2
\ Saving = – =  `  1
5 25 25 ⇒  x3  3  = 324
 x 
4 1
\ Monthly income = 48, = `300. ⇒ x 6  6 + 2 = 324
25 x
11. (d) 1800 = 2 × 2 × 2 × 3 × 3 × 5 × 5 1
⇒ x 6  6 = 322.
\ 1800 must be multiplied by 3 × 5 = 15 so that the x
resulting number becomes a perfect cube.
(2.3)3  0.027
12.
(c) For, n = 14, 39 (1 + 33 + 36 + 35) 21.
(a)
(2.3) 2  0.69  0.09
= 39 (1 + 27 + 729 + 243)
(2.3) 2 − (.3)3
= 39 × 103. =
(2.3) + 2.3 × .3 + (.3) 2
2
13.
(c) 8064 is divisible by 1, 2, 3, 4, 6, 7, 8, 9, 12, 14, 16, 18,
= 2.3 – .3 = 2
21, 24, 28, 32, 36, 42, 48, 56, 63, 64, 72, 84.
[Q a3 – b3 = (a – b) (a2 + ab + b2)].
2  K 13
14.
(b) Let  =
⇒ 2K = 9 ⇒ K = 4.5 ≈ 5. 22.
(d) Given expression
3  K 15
x2 x3 x4 x5
=   
15.
(d) x = 5  2 6 x 1 x  2 x  3 x  4
⇒ x = 52 6 = A  B , say x5
= .
x 1
\ x = 5  2 6 = A + B + 2 AB
⇒ A + B = 5, AB = 6 (1  x 2 )(1  x)(1  x) x 1
23.
(d) ? = × 2
 = 1.
⇒ A = 3, B = 2 (1  x) 1 x x (1  x)

Chapter 04.indd 23 6/4/2015 3:52:48 PM


4.24 I Chapter 4

x 3
 a  b
x 1
 a
 x 3
a 2  b2 (a 2  b 2 )
24. (d)   =   =   ⇒ x =  2 .
 b  a  b ab a  b 2  ab
\ x – 1 = –x + 3 ⇒ x = 2.
30.
(b) 3x+y = 81 = 34, 81x–y = 3
25.
(b) Given expression
1 1 1 ⇒ x + y = 4, (34)(x–y) = 31
= +  = 1.
xb x c xa xc xb x a ⇒ x + y = 4, 4x – 4y = 1
1  1  1 
xa xa xb xb xc xc x =
17
, y=
15
.
8 8
1

1 am an
26.
(b) n m =  = 1. 31. (a) P (x, y) = x2 – y2
a a am  an am  an
1 1 \ P (3, 4) = 32 – 42 = 9 – 16 = –7
am an
⇒ P (3, P (3, 4)) = P (3, –7)
27.
(b) (B) is correct. = (3)2 – (–7)2 = 9 – 49
(p yq x)z = a2 ⇒ (axy × ayx)z = a2 = ­–40.
⇒ (a2xy)z = a2 32.
(c) 12 to 19:8, 23 to 29 : 7,
⇒ a2xyz = a2 34 to 39:6, 45 to 49 : 5,
⇒ 2xyz = 2 ⇒ xyz = 1 56 to 59:4, 67 to 69 : 3,
(C) is correct. 78 to 79:2, 89 : 1
xa = yb = zc Total : 36.
⇒ a log x = b log y = c log z
1 2 3 1 1 1 1
\ ab + bc + ca = 0 33.
(c) of  of =   .
2 3 4 3 3 4 12
log z log z log z log z
⇒ c c + c ccc = 0. 34.
(a) Cost of Cave
log x log y log y log x
= 5 rocks + 2 stones + 3 pebbles
z log z + log x log z + log y log z = 0
⇒ log 
= 35 stones + 14 pebbles + 3 pebbles
⇒ log z + log x + log y = 0
= 245 pebbles + 14 pebbles + 3 pebbles
⇒ log (xyz) = 0 = log 1 ⇒ xyz = 1.
= 262 pebbles
1/2 1/2 1/2    1 rock = 49 pebbles
(d) a
28. a

1 a
1 a 1 a \ To use 6 rocks, it requires 294 pebbles
\ Change required = 294 – 262 = 32 pebbles
a1/2  a 1/2 1  a 1/2
= + = 4 stones and 4 pebbles.
(1  a1/2 )(1  a1/2 ) 1  a1/2
3 3 15
35.
(c) x  x = 150 or, x = 150
a1/2  a 1/2  (1  a 1/2 )(1  a1/2 ) 4 14 28
=
(1  a1/2 )(1  a1/2 ) 150  28
\ x = = 280.
15
= a
1/2
a 1/2
1 a 1/2
a 1/2
1 = 2 .
1 a 1 a a 17
36.
(c) Given that =
ab 23
x a b a 2  b2
29.
(d) =  = i.e., if a = 17, then a + b = 23 or, b = 6
y b a ab
a – b = 17 – 6 = 11
a b a 2  b2
x – y =  = ab 23
b a ab \ = .
ab 11
a 2  b2 a 2  b2 abx
⇒ x = y =  2 1 26 512
ab ab a  b2 37.
(b) x × x ÷ = 18 or, x3 =
x 27 27
 ab  a 2  b2  8
3
8 2
⇒ x 1  2 2
= \ x3 =   and so x = = 2 .
 a  b  ab  3 3 3

Chapter 04.indd 24 6/4/2015 3:52:50 PM


Simplification 4.25

17 8 225 44.
(c) Suppose there were x packages in the van before
38.
(d) x × –x× = 225 or, x = 225 delivery.
8 17 136
\ x = 136. \ After first delivery, the number of packages in the van
2 3
= x  x  x
5  1 ( 5  1) 2 62 5 3 5 5 5
39.
(b) a =  = 
5 1 4 4 2 After second delivery, the number of packages in the
van
5  1 ( 5  1) 2 62 5 3 5 3 3 x  15
b =  =  = x  3  .
5 1 4 4 2 5 5
2 3 x  15 x
3 5 14  6 5 \       5  2 (Given)
a2 =   
 2  4
⇒ x = 30.
2
3 5 14  6 5
b2 =   1  4
  45.
(d) 1  1
 2  4 1  7
 1 
 1
 5  1  5  1 4 1 
ab =    = 1 =1  3 
 5  1  5  1
 1  11  4  11
a 2  ab  b 2 32 4 = 1   = 1  
\  = . 3 7  7  7
 1 
a 2  ab  b 2 24 3  4
11 11 22
x y 4 5 =  = .
40.
(d) =  ⇒ x = y 7 7 7
x y 1 3
25 2 34 y 2 1
\ x2 + y2 = y  y2  46.
(c) = 0.50000 (1)
9 9 2
25 2 16 y 2
x2 –­ y2 = y  y2  = 0.16667 (2)  (divide (1) by 3)
9 9 2.3
x 2  y 2 34 17 1
\ 2  = . = 0.04167 (3)  (divide (2) by 4)
x  y 2 16 8 2.3.4

41.
(a) Let the number be 10x + y. 1
= 0.00833 (4)  (divide (3) by 5)
\ x + y = 7 2.3.4.5
Adding, we have 0.71667 or 0.717 up to three places.
and, 10y + x = 10x + y + 27
47.
(b) Let x be the fraction
⇒ y = 5, x = 2
7 6 1 6
\ The number = 10x + y = 25. x  x = ⇒x=
6 7 7 13
3 6 6 6 36
(b) Given expression =  b 2 . Since b2 is always positive,
42. Correct answer = x =  = .
2 7 7 13 91
3
therefore,  b 2 is always negative. 48. (a) Let total number of students = x
2
2x
7 7 41 26 13 Number of girl students =
43.
(a) Sum =     3
4 3 12 5 6 x
Number of boy students =
105  140  205  312  130 3
=
60 Number of girls who took part in camp
892 13 1  2x  2
= = 14 =   = x
60 15 5 3  15
which is nearer to 15 than 14 Number of boys who took part in camp
13 2 1  x x
Difference = 15 – 14 = . =   =
15 15 8  3 24

Chapter 04.indd 25 6/4/2015 4:07:20 PM


4.26 Chapter 4

Total number of students who took part in camp 55.


(d) 1800 = 2 × 2 × 2 × 3 × 3 × 5 × 5
2 x  16  5  \ 1800 must be multiplied by 3 × 5 = 15 so that the
= x =  x
15 24  120  resulting number becomes a perfect cube.
7 56.
(c) For n = 14, 39 (1 + 33 + 36 + 35)
= x.
40 = 39 (1 + 27 + 729 + 243)
1 75 1 75 = 39 × 103.
49.
(a) Number of ’s =  =  8 = 300.
8 2 8 2 2 1 2 4 9 14
50.
(c) Given expression 57.
(a)  , 0, , , , , .
3 7 9 9 11 15
15 1 1 2 7 15  7 4 58.
(b) x2 + y2 = 3341
=      of   
2 2 8 5 3 8  5 3
x2  y 2  891
15 2 7 15 1
= 4   of 2 x 2  4232
2 5 3 8 15  
⇒ x = 46, y = 35.
15 2 7 1
= 4   2 3 4 5 n 1 2
2 5 3 8 59.
(b)     ...  = .
3 4 5 6 n n
15 2 7 8
= 4  
2 5 3 1 1
60.
(a) Given x4  = 322
15 112 23 112 x4
= 4 = 
2 15 2 15  2 1
2

121 1
⇒  x  2   2 = 322
= = 4 . x
30 30 2
 2 1
51.
(b) Given expression ⇒  x  2  = 324
x
108  9  3  1 121
= = = 1.1203. 2 1
108 108 ∴ x  2 = 18
52.
(b) Let missing figure = x x
2
15  9  5  3 1 1    1
     x      = 3 ⇒  x    2 = 18
2 4 4  2 3 6    x
1
15  9  5  ⇒ x  = 16 = 4.
    x  = 3 x
2  4  4 
15 9/4 (d) (x)(a – b) (a + b) + (b – c) (b + c) + (c – a) (c + a)
61.
 3 = = x0 = 1.
2 5/4  x
9 62.
(c) The given expression is
9
= 62 + 43 + 3 = 36 + 64 + 3 = 103.
2 5  4x
5 – 4x = 2 1 17 1
63.
(c) x = , x– =?
3 x 4 x
or, x =
4 2 2
 1  1 1
53. 21 2
(a) Given expression = 7 (1 + 7 + 7 + 7 ) 3 Now,  x   = x    4  x
 x  x x
 = 721 × 400
Putting the value, we get
which is completely divisible by 25
Hence, remainder is zero. 1 15
x = .
x 4
54.
(b) Sum of the digits in the thousand’s place
= 6000 + 12000 + 18000 + 24000 = 60000 x 1 x 13
64.
(d) Given  =
Sum of the digits in the hundred’s place 1 x x 6

= 600 + 1200 + 1800 + 2400 = 6000
x 1 x x(1  x) 169
Sum of the digits in the ten’s place ⇒  2 =
1 x x x(1  x) 36
= 60 + 120 + 180 + 240 = 600
2 2
x  (1  x) 169
Sum of the digits in the unit’s place ⇒  2 =
= 6 + 12 + 18 + 24 = 60. x(1  x) 36

Chapter 04.indd 26 6/4/2015 3:52:55 PM


Simplification I 4.27

x2  1  x2  2 x  2 x  2 x2 169 2− 2 −2− 2
⇒ = = 2 + 2 +
x(1  x) 36 (2 + 2) (2 − 2)
1 169 2 2
⇒ = = 2 + 2 −
x(1  x) 36 2

9 4 2  2  2 = 2.
⇒ x = or .
13 13
2n + 4 − 2 (2n ) 2n + 4 − 2n + 1
ab cd 71.
(d) =
65.
(c) = 2 (2n + 3 ) 2n + 4
bc ad

⇒ a2 + ad + ab + bd = bc + bd + c2 + cd 2n + 4 2n + 1 1 7
n + 4 − n + 4 = 1 − =.
⇒ a2 + ad + ab = c2 + bc + cd 2 2 8 8
⇒ ad + ab – bc – cd = c2 – a2
(d) u1 = 31/2
72.  , u2 3 3 33/4
  , u3 3 3 37/8
3
⇒ a(b + d) – c(b + d) = (c – a) (a + c)
⇒ (a – c) (b + d) = – (a – c) (a + c) 2n 1
⇒ un = 3 2n
⇒ (a – c) (b + d) + (a – c) (a + c) = 0
⇒ (b + d + a + c) (a – c) = 0 210 1 29 1
So, either a = c or a + b + c + d = 0 or both. ⇒ u10 = 3 210 , u9  3 29
66.
(b) According to the question, 210 1 29 1
r = (2a)2b = 22b × a2b = (4)b × (ab)2 u10 10

\ = 3 2 29
Also, r = ab × xb u9
⇒ a × xb = 4b × (ab)2
b 210 − 1 − 2(29 − 1)
= 3 210
⇒ xb = 4b × ab
⇒ xb = (4a)b 210 1  210  2 1

\ x = 4a. 210 210


= 3 2 .
m 4 r 9 73.
(a) By putting x = 2, we have
67.
(b) Given,
  and
n 3 t 14
Put the values and calculate
.r.r .r.r
( 3 + 2) 2 + ( 3 − 2) 2

3mr  nt 3  4  9  3  14 = 3 + 2 .r6 + 2 + 3 − 2 6 + 2 = 10
=
4nt  7 mr 4  3  14  7  4  9 Again putting x = –2 we get

108  42 66 11 (.r.r
3 + 2) + (.r.r −2
3 − 2) −2
= =   .
168  252 84 14 1 1
= +
4
3 6  6 3 
4 (.r.r
3 + 2) (.r.r
3 − 2) 2 2
(d)  a9   a9  = (((a9)1/6)1/3)4 (((a9)1/3)1/6)4
68.
    (.r.r
3 − 2) + (.r.r3 + 2) 2 2
=
  = (a2) (a2) = a4. (.r.r
3 + 2) (.r.r3 − 2) 2 2

(a) 1 −
69.
1
+
1
=
1+ 3 −1.r
+
1
=
10
=
10
.r .r
1+ 3 1− 3 1+ 3 .r .r
1− 3 {(.r3) − (.r2) } 2
(3 − 2) 2 2 2

=
.r + 1
3 = 10/1 = 10.
1-----r
+ 3 1-----r− 3 So, that x = ± 2 will satisfy the equation.

=
.r3 (1 − .r3) + 1 + .r3 74. (c) xab–ac+bc–ab+ac–cb = 1 (Statement I)
1 − (.r3) 2 According to statement III,
log (4) – log (2) = log 2
=
.r3 − 3 + 1 + .r.r
3
=
2 32
4
−2 2 log   = log 2
2
= 1 − .r 3.
2 = 2.
1 1
70.
.r -----r
(a) 2 + 2 + +
2 + 2 r- 2−2
75. (a) Given,
1 1 a 2 − ab + b 2 1
= 2 + 2 +
.r -----r
2 + 2 -----r

2− 2
2
a + ab + b
2
=
3

Chapter 04.indd 27 6/4/2015 3:52:58 PM


4.28 Chapter 4

By componendo–dividendo, 78. (b) We have


1
2(a 2 + b 2 ) 4 x+ =1
= x
−2(ab) −2
⇒ x2 − x + 1 = 0
Therefore, Since x ≠ −1, therefore
a 2 + b2 (x + 1) (x2 – x + 1) = 0
=2
ab ⇒ x3 + 1 = 0
So, ⇒ x3 = − 1
a 2 + b 2 = 2ab ⇒ (x3)1333x = −x
⇒ x4000 = −x
⇒ a 2 − 2ab + b 2 = 0
1 1 1
⇒ ( a − b) 2 = 0 Now, p = x 4000 + = −x + = −( x + ) = −1
x 4000 −x x
⇒a=b Let, n = 2
a 2

⇒ =1 Then, q = 22 + 1 = 16 + 1 = 17
b
Units place digit = 7
76. (d) We have ∴ p + q = − 1 + 7 = 6.
1 1
2 3 = (28 ) 24 79. (d) Given expression is equal to 4 + 11 = 15 .
1 1 7 7 7
3 4 = (36 ) 24
1 1 1
80. (b) (56 − 1) = (53 ) 2 − (1) 2 = (125) 2 − (1) 2
4= 2= (2 )
6 3 8 24
= (125 + 1)(125 − 1) = 126 × 124 = 31 × 4 ×1126
1 1
It is therefore clear that the expression is divisible
6 8 = (63 ) 24
by 31.
1 1
1012 = (102 ) 24 81. (d) It is clear that for n = 237, the expression n (n + 1)
1 (2n + 1) is divisible by 237
Clearly, 10 12 is the largest term. Hence, option (d) is not necessarily true.
x y z 82. (d) Let the total number of quizzes = x:
77. (b) Let, = = =k
a b c And score of previous quizzes = y
Then, x= ak, y = bk and z = ck
(x + y + z) = k(a + b + c) y + 97 y + 70
∴ = 90, = 87
On squaring both sides, we get x x
y 97 y 70
( x + y + z ) 2 = k 2 (a + b + c) 2 ⇒ + = 90, + = 87
x x x x
⇒ 2( xy + yz + zx) = k 2 (a + b + c) 2 − ( x 2 + y 2 + z 2 ) y 97
⇒ = 90 − (1)
k 2 (a + b + c) 2 − ( x 2 + y 2 + z 2 ) x x
∴ xy + yz + zx = y 70
2 and = 87 − (2)
x x x
Also, k = From (1) and (2), we get
a
97 70
x 2 90 − = 87 −
⇒ k2 = 2 x x
a −70 97 27
⇒3= + =
x 2 (a + b + c) 2 − ( x 2 + y 2 + z 2 )a 2 x x x
∴ xy + yz + zx =
2a 2 ∴
x = 9 .

Chapter 04.indd 28 6/4/2015 3:53:05 PM


CHAPTER

Percentage 5
INTRODUCTION Thus, 40 per cent will be written as 40%
The term per cent means per hundred or for every hundred. A fraction whose denominator is 100 is called a percentage
It is the abbreviation of the Latin phrase per centum. and the numerator of the fraction is called rate per cent, e.g.
Scoring 60 per cent marks means out of every 100 marks 5
and 5 % means the same thing, i.e., 5 parts out of every
the candidate scored 60 marks. 100
The term per cent is sometimes abbreviated as p.c. The hundred parts.
symbol % is often used for the term per cent.

soMe Basic forMulae

1. To convert a fraction into a per cent: 3. To find a percentage of a given number:


l x
To convert any fraction to rate per cent, x % of given number (N) = × N.
m l 100
multiply it by 100 and put % sign, i.e., × 100%
m
Illustration 3 75% of 400 = ?
75
Illustration 1 What percentage is equivalent to
3
? Solution: 75% of 400 = × 400 = 300
100
5
3 Illustration 4 Find a number whose 4% is 72
Solution: × 100 = 60%
5 Solution: Let the required number be x
Then, 4% of x = 72
2. To convert a per cent into a fraction:
4 100
To convert a per cent into a fraction, drop the ⇒ × x = 72 ⇒ x = × 72 = 1800
100 4
per cent sign and divide the number by 100.
Illustration 5 What per cent of 25 kg is 3.5 kg?
2 Solution: Let x% of 25 kg be 3.5 kg.
Illustration 2 What fraction is 16 %?
3 Then, x% of 25 kg = 3.5 kg
 50  x 3.5  100
  ⇒ × 25 = 3.5 ⇒ x = = 14
2 3  50 1  1 100 25
Solution: 16 % = =   = .
3 100  3 100  6 Hence, 3.5 kg is 14% of 25 kg

Chapter 05.indd 1 6/4/2015 2:47:21 PM


5.2 Chapter 5

SOME USEFUL SHORT-CUT METHODS

Solution: Here x = 40
1. (a) If A is x% more than that of B, then B is less
 x 
than that of A by ∴ Required answer =   100 %
 100  x 
 x 
100  x  100  %
%.  40
= 

 100 %
 100  40 
(b) If A is x% less than that of B, then B is more
2
than that of A by = 66 %
3
 x 
100  x  100  %
%. 2. If A is x% of C and B is y% of C, then
x
A= × 100% of B.
y
Explanation
x 100  x Explanation
Given A=B+ B= B x A
100 100 Given A= C ⇒ C = 100
100 x
100  x y B
∴ A–B= B–B and, B= C ⇒ C = 100
100 100 y
 100  x  x A B x
=   1 B = B ∴ C = 100 = 100 ⇒ A = B
 100  100 x y y
x x
B or, × 100% of B
A B 100 x y
So, = =
A 100  x 100  x
B Illustration 8 If A is 20% of C and B is 25% of C, then
100 what percentage is A of B?
 x  Solution: Here x = 20 and y = 25
⇒ A–B=   100 % of A
 100  x 
x
A= × 100% of B
Therefore, B is less than that of A by y
 x  20
  100 %. = × 100% of B, i.e., 80% of B
100  x  25
Similarly, (b) can be proved. 3. (a) If two numbers are, respectively, x% and y%
Illustration 6 If Mohan’s salary is 10% more than that of more than a third number, then the first number
Sohan, then how much per cent is Sohan’s salary less than  100  x 
that of Mohan? is   100 % of the second and the
 100  y 
Solution: Here x = 10
 100  y 
 x  second is   100 % of the first.
∴ Required answer =   100 %  100  x 
 100  x 
(b) If two numbers are, respectively, x% and y%
 10 
=   100 % less than a third number, then the first number
 100  10 
 100  x 
1 is   100 % of the second and the
= 11 %.  100  y 
9
 100  y 
Illustration 7 If A’s income is 40% less than B’s
’s income, second is   100 % of the first.
 100  x 
then how much per cent is B’s income more than A’s
’s income?

Chapter 05.indd 2 6/4/2015 2:56:44 PM


Percentage 5.3

Explanation Explanation
Let A, B and C be the three numbers. Let the original price of the commodity be ` 100.
Given P
Then, the increased price = 100 +  100
x  100  x   100  100
A=C+ C=  C ⇒ C = A 
100  100   100  x  = ` (100 + P)
y  100  y   100  Therefore, to keep the price unchanged, there should
and, B=C+ C=   C ⇒ C =B  be a reduction in the consumption of the commodity by ` P.
100  100  100  y 
 100   Decrease in ` (100 + P) = ` P
 100 
\ A =B 
 100  x   100  y  \ Decrease in ` 100 =
P
 100
100  P
 100  x   100  x 
⇒ A=   B or  × 100% of B \ Required reduction in consumption is
 100  y   100  y 
 P 
 100  y   100  y    100 %.
and, B=  A or  × 100% of A 100  P 
 100  x   100  x 
Similarly, (b) part can be proved.
Similarly, (b) can be proved.
Illustration 11 If the price of sugar increases by 25%, find
Illustration 9 Two numbers are respectively 20% and 50% how much per cent its consumption be reduced so as not to
more than a third number. What per cent is the first of the increase the expenditure
second? Solution: Reduction in consumption
Solution: Here x = 20 and y = 50.
 P 
 100  x  =   100 %
\ First number =  × 100% of the second  100  P 
 100  y 
 25 
 100  20  =   100 % or 20%
=  × 100% of the second  100  25 
 100  50 
i.e., 80% of the second Illustration 12 If the price of a commodity decreases by
Illustration 10 Two numbers are, respectively, 32% and 25%, find how much per cent its consumption be increased
20% less than a third number. What per cent is the first of so as not to decrease the expenditure
the second? Solution: Increase in consumption
Solution: Here x = 32 and y = 20.  P 
=   100 %
 100  x   100  P 
\ First number =   100 % of the second
 100  y   25  1
=   100 % or 33 %
 100  32   100  25  3
=   100 % of the second
 100  20 
i.e., 85% of the second 5. If a number is changed (increased/decreased)
successively by x% and y%, then net % change
 xy 
4. (a) If the price of a commodity increases by is given by  x  y   % which represents
 100 
P%, then the reduction in consumption so as increase or decrease in value according as the sign
not to increase the expenditure is is +ve or –ve.
 P  If x or y indicates decrease in percentage, then put
  100 %.
100  P  –ve sign before x or y, otherwise +ve sign.
(b) If the price of a commodity decreases by P%,
then the increase in consumption so as not to Explanation
decrease the expenditure is Let the given number be N.
 P  If it is increased by x%, then it becomes
  100 %. Nx N ( x  100)
100  P  N + x% of N = N + = .
100 100

Chapter 05.indd 3 6/4/2015 2:47:23 PM


5.4 Chapter 5

If it is further increased by y%, then it becomes Illustration 15 If the side of a square is increased by 20%,
N ( x  100) y N ( x  100) its area is increased by k% Find the value of k
  Solution: Since side × side = area
100 100 100
∴ Net % change in area
N ( x  100) ( y  100)
=  xy   20  20 
(100) 2 = x y  % =  20  20  %
 100 100 
N ( x  100) ( y  100)
∴ Net change = –N [Here x = 20 and y = 20]
(100) 2 = 44%
N (100 x  100 y  xy ) Therefore, the area is increased by 44%
=
(100) 2 Here k = 44
 xy  1 100 Illustration 16 The radius of a circle is increased by 2%
∴ % change = N  x  y   
 100  100 N Find the percentage increase in its area
 xy  Solution: Since π × radius × radius = area
= x y  %.
 100  ∴ Net % change in area
Illustration 13 If salary of a person is first increased by  xy   2  2
= x y  % =  2  2  %
15% and thereafter decreased by 12%, what is the net  100 100 
change in his salary? [Here x = 2 and y = 2]
Solution: Here x = 15 and y = –12 1
= 4 %
∴ The net change in the salary 25
 xy   15  12  1
= x y Therefore, the percentage increase in area is 4 %.
  % = 15  12   % or 1.2% 25
100  100 
Since the sign is +ve, the salary of the person increases Illustration 17 The tax on a commodity is diminished by
by 1.2% 15% and its consumption increases by 10% Find the effect
on revenue
Illustration 14 The population of a town is decreased
Solution: Since tax × consumption = revenue
by 25% and 40% in two successive years. What per cent
population is decreased after two years? ∴ Net % change in revenue
Solution: Here x = – 25 and y = – 40.  xy   15  10 
= x y  % =  15  10  %
 100  100 
∴ The net % change in population
 xy  [Here x = – 15 and y = 10]
= x y %
 100  = – 6.5%
 25  40  ∴ The revenue decreases by 6.5%
=  25  40   % or – 55%
 100 
7. If the present population of a town (or value
Since the sign is –ve, there is decrease in population
of an item) be P and the population (or value of
after two years by 55%
item) changes at r% per annum, then
6. If two parameters A and B are multiplied to (a) Population (or value of item) after n years
get a product and if A is changed (increased/  r 
n
decreased) by x% and another parameter B is = P 1 
 100 
changed (increased/decreased) by y%, then the
net % change in the product (A × B) is given (b) Population (or value of item) n years ago
 xy  P
 x  y   % which represents increase or = n
.
100   r 
decrease in value according as the sign in +ve or 1  
100 
–ve. where r is +ve or –ve according as the
If x or y indicates decrease in percentage, then put population (or value of item) increases or
–ve sign before x or y, otherwise +ve sign. decreases.

Chapter 05.indd 4 6/4/2015 3:02:45 PM


Percentage 5.5

Explanation Solution: Here P = 144000, x = 5, y = –10 and z = 15


Population at the end of first year \ Population after 3 years
r  r   x  y  z 
= A 1  1 1
=P+
100
P = P 1 
 100 
.  100   100   100 
Now, the population at the beginning of second year  5  10   15 
= 144000 1  1 1
 r   100   100   100 
= P 1  .
 100  144000  105  90  115
= = 156492
\ Population at the end of second year 100  100  100
2
 r  r  r   r  9. In an examination, the minimum pass percentage
= P 1    P 1   = P 1 
 100  100  100   100  is x% If a student secures y marks and fails
by z marks, then the maximum marks in the
   100( y  z )
n examination is .
 r  x
Population at the end of n years = P 1  .
 100 
Explanation
Illustration 18 The population of a town increases 5% Let the maximum marks be m
annually. If its present population is 84000, what will it be
in 2 years time? Given x% of m = y + z
Solution: Here P = 84000, r = 5 and n = 2 x 100( y  z )
⇒ × m = y + z or m = .
\ Population after 2 years 100 x
Illustration 21 In an examination, a student must get 60%
n 2
 r   5  marks to pass. If a student who gets 120 marks, fails by 60
= P 1   = 84000 1 
 100   100  marks, find the maximum marks
Solution: Here x = 60, y = 120 and z = 60.
105 105
= 84000 ×  = 92610 \ Maximum marks
100 100
100( y  z ) 100(120  60) 100  180
Illustration 19 The population of a town increases at the = = = = 300
x 60 60
rate of 5% annually. If the present population is 4410, what
it was 2 years ago?
10. In an examination x% and y% students respectively
Solution: Here P = 4410, r = 5 and n = 2. fail in two different subjects while z% students
\ Population of the town 2 years ago fail in both the subjects, then the percentage of
students who pass in both the subjects will be
P 4410 4410 (100 – (x + y – z))%
= n
= = = 4000
 r   5 
2 105 105

1   1   100 100 Explanation
100  100 
% of students who failed in one subject only = (x – z)%
8. If a number A is increased successively by x% % of students who failed in other subject only = (y – z)%
followed by y% and then by z%, then the final % of students who failed in both the subjects = z%
value of A will be \ % of students who passed in both the subjects
x  y  z  = [100 – [(x – z) + (y – z) + z]]%

A 1  1 1 = (100 – (x + y – z))%
 100   100   100 
Illustration 22 In an examination, 42% students failed in
In case a given value decreases by any percentage, Mathematics and 52% failed in Science. If 17% failed in
we will use a negative sign before that. both the subjects, find the percentage of those who passed
in both the subjects
Illustration 20 The population of a town is 144000. Solution: Here x = 42, y = 52 and z = 17
It increases by 5% during the first year. During the second \ Percentage of students passing both the subjects.
year, it decreases by 10% and increases by 15% during the = (100 – (x + y – z))%
third year. What is the population after 3 years? = (100 – (42 + 52 – 17))% or, 23%

Chapter 05.indd 5 6/4/2015 2:47:25 PM


5.6 Chapter 5

Practice Exercises

Difficulty Level-1
(Based on Memory)

1.
The population of a city increases at the rate of 4% per (a) `10,000 (b) `80,000
annum. There is an additional annual increase of 1% in the (c) `40,000 (d) `54,000
population due to the influx of job seekers. Therefore, the
8.
A reduction of 20% in the price of sugar enables a
per cent increase in the population after 2 years will be:
1
(a) 10 (b) 10.25 purchaser to obtain 2 kg more for `160. Find the
2
(c) 10.50 (d) 10.75 original price per kg of sugar.
[Based on MAT, 2004] (a) `12 (b) `15
2.
Three papers were set in an examination and the maximum (c) `16 (d) `18
marks per paper were in the ratio of 1:2:2, respectively. If a [Based on MAT, 2001]
student obtained 50% in the first paper, 60% in the second,
and 65% in the third, what per cent did he obtain overall? 9.
Successive discounts of 20% and 15% are equivalent to a
single discount of:
(a) 58.3% (b) 66.66%
(a) 35% (b) 32%
(c) 33.33% (d) 60%
(c) 17.5% (d) 17%
3.
A recipe gives directions to mix 4 parts of substance
[Based on MAT, 2001]
A with 7 parts of substance B. These substances are to
be taken by weight, but by mistake they were taken by 10.
X’s salary is increased by 20% On the increase, the tax
volume. Find the error in the percentage of the weight of rate is 10% higher. Find the percentage of increase in the
A in the mixture, if 117 cm3 of the substance A weighs as tax liability.
much as 151 cm3 of the substance B. (a) 20 (b) 22
(a) 5.05% (b) 6.00% (c) 23 (d) Indeterminate
(c) 7.05% (d) None of these [Based on MAT, 2001]
[Based on MAT, 2003] 11.
Vishal goes to a shop to buy a radio costing `2568. The
4.
A manufacturer offers a 20% rebate on the marked price rate of sales tax is 7% He tells the shopkeeper to reduce
of a product. The retailer offers another 30% rebate on the the price of the radio to such an extent that he has to pay
reduced price. The two reductions are equal to a single `2568, inclusive of sales tax. Find the reduction needed in
reduction of: the price of the radio.
(a) 50% (b) 44% (a) `179.76 (b) `170
(c) 46% (d) 40% (c) `168 (d) `169
[Based on MAT, 2002] [Based on MAT, 2001]
5.
Ram’s weight is 140% of Manu’s weight. Tanu’s weight is 12.
Sunder purchased an office bag with a price tag of `600
90% of Mahesh’s weight. Mahesh weighs twice as much in a sale where 25% discount was being offered on the
as Manu. What percentage of Ram’s weight is Tanu’s tag price. He was given a further discount of 10% on the
weight? amount arrived at after giving usual 25% discount. What
(a) 64.3% (b) 77.8% was the final amount paid by Sunder?
(c) 90% (d) 128.6% (a) `210 (b) `540
(c) `405 (d) `450
6.
If A’s salary is 25% higher than B’s salary, then how much
[Based on MAT, 2001]
per cent is B’s salary lower than A’s?
(a) 15% (b) 20% 13.
A shopkeeper has certain number of eggs of which 5% are
1 found to be broken. He sells 93% of the remainder and
(c) 25% (d) 33 % still has 266 eggs left. How many eggs did he originally
3
have?
[Based on MAT, 2001]
(a) 3800 (b) 4000
7.
A businessman’s earning increases by 25% in one year
(c) 4200 (d) None of these
but decreases by 4% in the next. After 5 years his total
earnings would be `72,000. What is his present earning? [Based on IIT Joint Main. Ent. Test, 2004]

Chapter 05.indd 6 6/4/2015 2:47:25 PM


5.7
Percentage

14.
If 3/5 of a number is 23 more than 50% of the same 22.
In a group of 20 singers and 40 dancers, 20% of the singers are
number, then what will be 80% of the number? less than 25 years old and 40% of the entire group are under
(a) 92 (b) 184 25 years. What percentage of dancers are under 25 years?
(c) 180 (d) 186 (a) 20% (b) 40%
[Based on IIT Joint Man. Ent. Test, 2004] (c) 50% (d) 60%
15.
One kg of tea and one kg of sugar together cost `95. If the  [Based on IIFT, 2003]
price of tea falls by 10% and that of sugar rises by 20%, 23.
Tom’s salary is 125% of Tina’s salary. Tito’s salary is
then the price of one kg of each combined comes to `90. 80% of Tina’s salary. The total of all the three salaries is
The original price of tea in `per kg is: `61,000. What is Tito’s salary?
(a) `72 (b) `55 (a) `16,000 (b) `16,500
(c) `60 (d) `80 (c) `15,500 (d) `15,000
[Based on IIT Joint Man. Ent. Test, 2004]  [Based on IIFT, 2003]
16.
Two numbers are respectively 30% and 40% less than a 24.
Mayur weighs twice as much as Shweta. Shweta’s weight
third number. What is the second number as a percentage is 60% of Deepika’s weight. Rakesh weighs 50% of
of the first? Vikas’s weight. Vikas weighs 190% of Mayur’s weight.
(a) 70% (b) 75% Which of these people weighs the least?
5 2 (a) Mayur (b) Deepika
(c) 85 % (d) 116 %
7 3 (c) Shweta (d) Rakesh
[Based on Narsee Monjee Inst. of Man. Studies, 2003] [Based on SCMHRD Ent. Exam., 2003]
25.
What percentage of numbers from 1 to 70 have squares
17.
If 1 micron = 10,000 angstroms, then 100 angstroms is that end in the digit 1?
what per cent of 10 microns?
(a) 20 (b) 14
(a) 0.0001% (b) 0.001%
(c) 1 (d) 21
(c) 0.01% (d) 0.1%
[Based on SCMHRD Ent. Exam., 2003]
[Based on REC Tiruchirapalli, 2003]
26.
If X is increased to 23 from 20, then Y should increase
18.
A man walked diagonally across a square lot. Approx.,
from 100 to:
what was the per cent saved by not walking along the
edges? (a) 103 (b) 106
(a) 30 (b) 20 (c) 109 (d) 112
(c) 33 (d) 24 [Based on IMT Ghaziabad, 2002]
[Based on REC Tiruchirapalli, 2003] 27.
Salary of an officer increases every year by 20% His salary
19.
If 1/4th of x equals 30% of y which is intern equal to 45% in the year 2001 was `26640. What was his salary in 1999?
of z, then x as a percentage of z is: (a) `20000 (b) `19028
(a) 160% (b) 170% (c) `18500 (d) `18840
(c) 180% (d) 190%  [Based on IRMA, 2002]
28.
In a certain shop, which stocks four types of caps, there
20.
If the numerator of a fraction is increased by 20% and the
are one-third as many red caps as blue and half as many
denominator is diminished by 10%, then the value of the
green caps as red caps. There are equal numbers of green
fraction is 16/21. The original fraction is:
caps and yellow caps. If there are 42 blue caps, then what
3 4 per cent of the total caps in the shop are blue?
(a) (b)
5 7 (a) 70% (b) 28%
2 5 (c) 60% (d) 14%
(c) (d)
3 7 [Based on Narsee Monjee Inst. of Man. Studies, 2002]
[Based on FMS (Delhi), 2003] 29.
A bag contains 600 pens of brand A and 1200 pens of
21.
At what percentage above the cost price must an article be brand B. If 12% of brand A pens and 25% of brand B pens
marked so as to gain 33% after allowing the customer a are removed, then what is the approximate percentage of
discount of 5%? total pens removed from the bag?
(a) 48% (b) 43% (a) 37% (b) 36%
(c) 40% (d) 38% (c) 22% (d) 18%
[Based on IIFT, 2003] [Based on Narsee Monjee Inst. of Man. Studies, 2002]

Chapter 05.indd 7 6/4/2015 2:47:25 PM


5.8 Chapter 5

30.
A invested 125% as much money as B. C invested 80% 38.
A rainy day occurs once in every 10 days. Half of the rainy
as much money as B. The total of all the three is 61,000. days produce rainbows. What per cent of all the days do
How much did C invest? not produce rainbow?
(a) 25000 (b) 16000 (a) 95% (b) 10%
(c) 20000 (d) 45000 (c) 50% (d) 5%
[Based on Narsee Monjee Inst. of Man. Studies, 2002] [Based on SNAP, 2007]
39.
A rabbit on a controlled diet is fed daily 300 g of a mixture
31.
In a municipal election, there were two candidates. One
of two foods, food X and food Y. Food X contains 10%
who got 30% of the votes polled was defeated by 16000
protein and food Y contains 15% protein. If the rabbits
votes. Calculate the total number of votes polled.
diet provides exactly 38 g of protein daily, how many
(a) 24000 (b) 28000 grams of food X are in the mixture?
(c) 30000 (d) 40000 (a) 100 (b) 150
[Based on I.P. Univ., 2002] (c) 200 (d) 140
32.
A person usually spent `48 to buy groundnuts for roasting [Based on ATMA, 2008]
and resale. On one occasion he could buy 1.5 kg of 40.
In an examination paper of 5 questions, 5% of the
groundnuts less for `48 as the price had gone up by 25% candidates answered all of them and 5% none. Of the rest,
What was the earlier price of groundnut per kg? 25% answered only 1 question and 20% answered only 4
(a) `8 (b) `6.40 1
questions. If 24 % of the entire candidates answered only
(c) `7.20 (d) None of these 2
[Based on I.P. Univ., 2002] 2 questions and 200 candidates answered only 3 questions,
how many candidates appeared at the examination?
33.
X, Y and Z shared `7400 so that X received 25% more than (a) 1000 (b) 800
Y, and Y received 20% more than Z. What amount did Z (c) 600 (d) 400
receive?
[Based on ATMA, 2008]
(a) `3500 (b) `3000
41.
The contents of a certain box consist of 14 apples and 23
(c) `2400 (d) `2000 oranges. How many oranges must be removed from the
[Based on I.P. Univ., 2002] box so that 70% of the pieces of fruit in the box will be
34.
What would be the value of 20% of m as a percentage of apples?
p, if 8% of m = 4% of p? (a) 12 (b) 6
(a) 80% (b) 16% (c) 17 (d) 36
(c) 10% (d) None of these [Based on ATMA, 2005]
[Based on I.P. Univ., 2002] 42.
Income tax is raised from 4 paise in a rupee but the
revenue is increased by 10% only. Find the decrease per
35.
If S is 150% of T, then T is what per cent of S + T ? cent in the amount taxed.
1 (a) 12 (b) 14
(a) 33 % (b) 40%
3 (c) 16 (d) None of these
(c) 75% (d) 80% [Based on NMAT, 2006]
[Based on REC Tiruchirapalli, 2002]
43.
When income tax is 3 paise in a rupee, a person’s net
36.
At a school, 20% of the students are seniors. If all of income is `23700. What will it be when the income tax is
the seniors attended the school play, and 60% of all the raised to 7 paise?
students attended the play, then what per cent of the non- (a) `38 (b) `2330
seniors attended the play?
(c) `3460 (d) None of these
(a) 20% (b) 40%
[Based on NMAT, 2006]
(c) 50% (d) 100%
44. A clothing supplier stores 800 coats in a warehouse of
[Based on REC Tiruchirapalli, 2002]
which 15% are full length coats. If 500 of the shorter
37.
In an examination a candidate got 30% marks and failed length coats are removed from the warehouse, what per
by 30 marks. If the passing marks are 60% of the total cent of the remaining coats is full-length?
marks, then the maximium marks will be: (a) 35% (b) 9.37%
(a) 450 (b) 600 (c) 5.62% (d) 40%
(c) 300 (d) 100 [Based on NMAT, 2005]

Chapter 05.indd 8 6/4/2015 2:47:26 PM


5.9
Percentage

45.
Wheat is now being sold at `27 per kg. During last month pays `325 per year as taxes and realizes 5.5% annually on
its cost was `24 per kg. Find by how much per cent a family his investment. Find the monthly rent.
reduces its consumption so as to keep the expenditure fixed? (a) `634.76 (b) `654.76
(a) 10.2% (b) 12.1% (c) `554.76 (d) `456.32
(c) 12.3% (d) 11.1% [Based on MAT (Dec 2010, May), 2009]
[Based on SNAP, 2009] 52.
A person spends 40% of his salary on his educational
46. The total tractor production in a state is 294000, out of expenses. He spends 60% of it in purchasing books and
which 150000 are made by Mahindra and Mahindra. Out one-half of the remaining in purchasing stationery items.
of every 1000 Mahindra tractors, 98 are red in colour, but If he saves `160 every month, which is one-fourth of the
only 53% of the total tractor production is red. Find the balance after spending over books and stationery items,
percentage of non-Mahindra tractors that are red out of what is his monthly salary?
total non-Mahindra tractors. (a) `8000 (b) `4800
(a) 5.025% (b) 5.130% (c) `9600 (d) Data inadequate
(c) 0.6125% (d) 6.140% [Based on MAT (Feb), 2010]
[Based on MAT (Dec), 2008] 53.
The tax on a commodity is diminished by 10% and its
consumption increased by 10% The effect on the revenue
47.
In an examination, 40% marks are required to pass. A
derived from it is K% change. Find the value of K.
obtains 10% less than the number of marks required to
(a) 1 (b) 2
1 3
pass. B obtains 11 % less than A and C obtained 41 % (c) –1 (d) –2
9 17
less than the number of marks obtained by A and B [Based on MAT (Sept), 2009]
together. What marks did C get? 54.
A salesman’s terms were changed from a flat commission
(a) 50 (b) 40 of 5% on all his sales to a fixed salary of `1000 plus
2.5% commission on all sales exceeding `4000. If his
(c) 35 (d) 45 remuneration as per the new scheme was `600 more than
[Based on MAT (Feb), 2011] by the first scheme, what were his sales worth?
48.
Mohan spends 40% of his salary on food items, 50% of (a) `11000 (b) `17000
the remaining on transport, 30% of the remaining, after (c) `16000 (d) `12000
spending on food and transport, he spends on clothes and [Based on MAT (Sept), 2009]
saves the balance. If he saves `630 every month, what is 55.
If a bucket is 80% full, then it contains 2 litres more water
his monthly salary?
2
(a) `1500 (b) `3000 than when it is 66 % full. What is the capacity of the
3
(c) `5000 (d) `6500 bucket?
[Based on MAT (Feb), 2011] (a) 10 l (b) 15 l
49.
When the price of sugar was increased by 32%, a family 2
(c) 16 l (d) 20 l
reduced its consumption in such a way that the expenditure 3
on sugar was only 10% more than before. If 30 kg per month [Based on MAT (Sept 2009, Dec), 2007]
were consumed before, find the new monthly consumption. 56. A salesman averages `240 during a normal 40-hour week.
(a) 42 kg (b) 35 kg During a sale, his rates are increased by 50% What is his
(c) 25 kg (d) 16 kg commission if he puts in 60 hours during the sale?
[Based on MAT (Dec), 2010] (a) 390 (b) 540
50.
A man’s income is increased by `1200 and at the same (c) 600 (d) 640
time, the rate of tax to be paid is reduced from 12% to [Based on MAT, 2000]
10% He now pays the same amount of tax as before. What 57.
In an examination, it is required to get 296 of the total
is his increased income, if 20% of his income is exempted maximum aggregate marks to pass. A student gets 259
from tax in both cases? marks and is declared failed. The difference of marks
(a) `6300 (b) `7200 obtained by the student and that required to pass is 5%
(c) `4500 (d) `6500 What are the maximum aggregate marks a student can get?
[Based on MAT (Dec), 2010] (a) 690 (b) 780
51.
Vellu buys a generator for `100000 and rents it. He puts (c) 740 (d) Cannot be determined
12.5% of each month’s rent aside for upkeep and repairs, [Based on MAT (May), 2009]

Chapter 05.indd 9 6/4/2015 2:47:26 PM


5.10 Chapter 5

58.
In an examination, Mohit obtained 20% more than (a) 500 million tonnes (b) 400 million tonnes
Sushant but 10% less than Rajesh. If the marks obtained (c) 600 million tonnes (d) 700 million tonnes
by Sushant are 1080, find the percentage marks obtained [Based on MAT (Dec), 2008]
by Rajesh if the full marks are 2000.
65.
Water tax is increased by 20% but its consump­tion is
(a) 72% (b) 86.66%
decreased by 20% Then, the increase or decrease in the
(c) 78.33% (d) None of these expenditure of the money is
[Based on MAT (Feb), 2009]
(a) no change (b) 5% decrease
59.
Ram spends 20% of his monthly income on his household
(c) 4% increase (d) 4% decrease
expenditure, 15% of the rest on books, 30% of the rest on
clothes and saves the rest. On counting, he comes to know [Based on MAT, 2000]
that he has finally saved `9520. Find his monthly income. 66.
In a co-educational school there are 15 more girls than
(a) `15000 (b) `10000 boys. If the number of girls is increased by 10% and the
number of boys is also increased by 16%, there would be
(c) `20000 (d) None of these
nine more girls than boys. What is the number of students
[Based on MAT (Feb), 2009]
in the school?
60.
Out of the total production of iron from haematite, an ore
(a) 140 (b) 125
of iron, 20% of the ore gets wasted. Out of the remaining
iron, only 25% is pure iron. If the pure iron obtained in (c) 265 (d) 255
a year from a mine of haematite was 80000 kg, then the [Based on MAT, 1999]
quantity of haematite mined in the year is
67.
The digit at unit’s place of a two-digit number is increased
(a) 400000 kg (b) 500000 kg by 100% and the ten’s digit of the same number is increased
(c) 450000 kg (d) 600000 kg by 50% The new number thus formed is 19 more than the
[Based on MAT (Feb), 2009] original num­ber. What is the original number?
61.
There are two candidates Bhiku and Mhatre for an (a) 22 (b) 63
election. Bhiku gets 65% of the total valid votes. If the (c) 44 (d) None of these
total votes were 6000, what is the number of valid votes
[Based on MAT, 1999]
that the other candidate Mhatre gets if 25% of the total
votes were declared invalid? 68.
At a college entrance examination, each candidate is
(a) 1575 (b) 1625 admitted or rejected according to whether he has passed
or failed the test. Of the candidates who are really
(c) 1675 (d) 1525 capable, 80% pass the tests and of the incapable, 25%
[Based on MAT (Feb), 2009] pass the tests. Given that 40% of the candidates are really
62.
In a medical certificate by mistake a candidate gave his capable, then the proportion of capable college students
height as 25% more than actual. In the interview panel, is about:
he clarified that his height was 5 ft 5 inches. Find the (a) 73% (b) 70%
percentage correction made by the candidate from his (c) 68% (d) 75%
stated height to his actual height. [Based on MAT (May), 2007]
(a) 28.56 (b) 20 69.
In an examination, there were 2000 candidates, out of
(c) 25 (d) None of these which 900 candidates were boys and the rest were girls. If
[Based on MAT (Feb), 2009] 32% of the boys and 38% of the girls passed, then the total
percentage of failed candidates is:
63.
In a Mathematics exam, a student scored 30% in the first
(a) 68.5% (b) 64.7%
paper out of a total of 180. How much should he score in
the second paper (out of 150) if he is to get at least 50% (c) 35.3% (d) 70%
marks overall? [Based on MAT (May), 2007]

(a) 75% (b) 80% 70.


If the price of gold increases by 30%, find by how much
the quantity of ornaments must be reduced so that the
(c) 74% (d) 84%
expenditure may remain the same as before?
[Based on MAT (Dec 2008, Sept), 2007]
1
64.
7% of the total quantity of wheat is lost in grinding when (a) 30% (b) 23 %
a country has to import 6 million tonnes, but when only 13
1 2
5 % is lost, it can import 3 million tonnes. Find the (c) 27 % (d) 19%
5 13
quantity of wheat grown in the country. [Based on MAT (May), 2007]

Chapter 05.indd 10 6/4/2015 2:47:26 PM


5.11
Percentage

71.
In a company, there are 75% skilled workers and remaining (a) 10 (b) 20
ones are unskilled. 80% of skilled workers and 20% of (c) 30 (d) 40
unskilled workers are permanent. If number of temporary [Based on MAT, 1998]
workers is 126, then what is the number of total workers?
79.
Given two positive integers x and y with x < y. The per
(a) 480 (b) 510
cent that x is less than y is
(c) 360 (d) 377
[Based on MAT (Dec), 2006] 100( y  x) 100( x  y )
(a) (b)
72.
An MBA student gets a fellowship from which he spends x x
70% on personal expenses and 20% on books, fees, etc. 100( y  x)
The remaining amount is saved and it amounts to `4800 (c) (d) 100(y – x)
y
in a year. The value of the monthly fellowship is: [Based on FMS, 2011]
(a) `3000 (b) `3500 80. The price of petrol is increased by 25% How much percent
(c) `5000 (d) `4000 must a car owner to reduce his consumption of petrol so as
[Based on MAT, 1998] not to increase his expenditure on petrol?
73.
Population of a district is 296000 out of which 166000 are (a) 50% (b) 30%
males. 50% of the population is literate. If 70% males are (c) 25% (d) 20%
literate, then the number of women, who are literate, is: [Based on MAT, 2011]
(a) 32200 (b) 31800 81. When 5% is lost in grinding wheat, a country has to
(c) 66400 (d) 48000 import 20 million bags to make up for the loss. But when
[Based on MAT (Feb), 2006] only 2% is lost, it has to import 15 million bags. What
74.
Sharma’s expenditure and savings are in the ratio of is the quantity of wheat, which grows in the country in
3:2. Her income increases by 10% Her expenditure also million bags?
increases by 12% How much per cent does her savings 1
increase? (a) 133 (b) 150
3
(a) 7% (b) 6%
2 2
(c) 13% (d) 11% (c) 106 (d) 166
3 3
[Based on MAT (May), 2010]
75.
There are four friends. The average score in unit test of the [Based on MAT, 2011]
first three is 15 and that of the last three is 16. If the score 82. In a consumer preferences survey, 20% respondents opted
of the last friend is 19, then first friend’s score is what per for product A whereas 60% opted the product B. The
cent of average of the last three? remaining individuals were undecided. If the difference
2 between those who opted for product B and those who
(a) 66 % (b) 300% were undecided is 720, how many individuals had been
3
interviewed for the survery?
1
(c) 33 % (d) None of these (a) 1440 (b) 1800
3
[Based on MAT (Sept), 2010] (c) 3600 (d) Data inadequate
76.
A monthly return railway ticket costs 25% more than a [Based on MAT, 2012]
single ticket. A week’s extension can be had for the former
83. Gauri went to the stationers and bought items worth
by paying 5% of the monthly ticket’s cost. If the money
`25, out of which 30 paise went on sales tax on taxable
paid for the monthly ticket (with extension) is `84, the
purchases. If the tax rate was 6%, then what was the cost
price of the single ticket is
of the tax-free items?
(a) `64 (b) `80
(a) `15 (b) `15.70
(c) `48 (d) `72
[Based on MAT (May), 2007]
(c) `19.70 (d) `20
[Based on MAT, 2012]
77.
When the price of an audio system was reduced by 20%,
the sale increased by 80% What was the net effect on the 84. Shyam had three notebooks X, Y and Z. Of these, X had
sale? 120 pages, Y had 10% more and Z had 10% less. If
(a) 44% increase (b) 44% decrease he tore out 5%, 10% and 15% of pages in X, Y and Z
(c) 66% increase (d) 60% increase. respectively, then what percent of total pages did he tear
out?
[Based on MAT, 1998]
78.
If two numbers are respectively 20% and 50% of a third (a) 8% (b) 15%
number, what is the percentage of the first number to the (c) 7% (d) None of these
second? [Based on MAT, 2012]

Chapter 05.indd 11 6/4/2015 2:47:26 PM


5.12 Chapter 5

85. In a class of 90 students, amongst 50% of the students, are 4:1, 2:3, 4:3 in the respective vessels. If all three
each student got number of sweets that are 20% of the vessels are emptied into a single large vessel, then
total number of students and amongst the remaining what will be the ratio of water to milk in the resultant
50% of the students, each students got number of sweets mixture?
that are 10% of the total number of students. How many (a) 43:62
sweets were distributed among 90 students?
(b) 197:214
(a) 960 (b) 1015
(c) 219:117
(c) 1215 (d) 1620
(d) 179:234
[Based on MAT, 2012]
[Based on MAT, 2013]
86. In a test consisting of 150 questions carrying 1 mark each,
Saurabh answered 80% of the first 75 questions correctly. 92. 400 students took the mock test 60% of the boys and
What per cent of the other 75 questions does he need to 80% of the girls cleared the cut off in the test. If the total
answer correctly to score 60% overall? percentage of students qualifying is 65%, how many girls
appeared in the test?
(a) 20 (b) 40
(c) 50 (d) 60 (a) 100 (b) 120
[Based on MAT, 2012] (c) 150 (d) 300
[Based on MAT, 2014]
87. Nilam’s Mathematics Test had 75 problems, i.e., 10
arithmetic, 30 algebra and 35 geometry problems. 93. After receiving two successive raises, Ajitha’s salary
Although she answered 70% of the arithmetic, 40% of the became equal to 15/8 times of her initial salary. By how
algebra and 60% of the geometry problems correctly, she much per cent was the salary raised the first time if the
did not pass the test because she got less than 60% of the second raise was twice as high (in per cent) as the first?
problems right. How many more questions would she have (a) 15 (b) 20
needed to answer correctly to earn 60% passing grade? (c) 25 (d) 30
(a) 3 (b) 5 [Based on MAT, 2014]
(c) 7 (d) 10
94. A person saves 6% of his income. 2 year later, his income
[Based on MAT, 2012]
shoots up by 15% but his savings remain the same. Find
88. 900 kg of mortar consists of 45% sand, 144 kg of lime the hike in his expenditure (in approx percent).
and the rest cement. What percentage of cement is there (a) 13.65% (b) 12.45%
in mortar?
(c) 14.85% (d) 15.95%
(a) 27% (b) 45%
[Based on MAT, 2014]
(c) 39% (d) 21%
[Based on MAT, 2013]
95. A reduction in the price of petrol by 10% enables a
motorist to buy 5 gallons more for `180. Find the original
89. A vendor sells 60% of apples he had and throws away 15% price of petrol (in ` per gallon).
of the remainder. Next day he sells 50% of the remainder (a) 20 (b) 30
and throws away the rest. What per cent of his apples does
the vendor throw: (c) 40 (d) 50
(a) 17% (b) 23% [Based on MAT, 2014]

(c) 42% (d) 15% 96. In an examination, Raman scored 25 marks less than
[Based on MAT, 2013] Rohit. Rohit scored 45 more marks than Sonia. Rohan
scored 75 marks which is 10 more than Sonia. Ravi’s
90. In a supermarket a 50 g tube of toothpaste costs `299 and score is 50 less than, maximum marks of the test. What
a 100 g tube costs `509. Approximately what percentage approximate percentage of marks did Ravi score in the
do you pay more if you buy two 50 g tubes instead of one examination, if he gets 34 marks more than Raman?
100 g tube?
(a) 90 (b) 70
[Based on MAT, 2013]
(c) 80 (d) 60
(a) 20% (b) 18%
[Based on SNAP, 2013]
(c) 15% (d) 10%
97. Mr. Giridhar spends 50% of his monthly income on
91. In three vessels, each of 100 L capacity, mixture of household items and out of the remaining he spends
milk and water is filled. The ratio of milk and water 50% on transport, 25% on entertainment, 10% on sports

Chapter 05.indd 12 6/4/2015 2:47:26 PM


5.13
Percentage

and the remaining amount of `900 is saved. What is tax for each slab and adding them up. The rates applicable
Mr. Giridhar’s monthly income? are as follows:
(a) `6000
(b) `12000 Annual income Tax rate
(c) `9000 0 – 50,000 0%
(d) Cannot be determined 50,001 – 60,000 10%
[Based on SNAP, 2013]
60,001 – 1,50,000 20%
98. Last year there were 610 boys in a school. The number
decreased by 20 percent this year. How many girls are > 1,50,000 30%
there in the school if the number of girls is 175 percent of
the total number of boys in the school this year? If my income is `1,7,000, then what is the tax payable by
me?
(a) 854 (b) 848
(a) `51,000
(c) 798 (d) 782
(b) `17,000
[Based on SNAP, 2013]
(c) `34,000
99. A country follows a progressive taxation system under (d) `25,000
which the income tax rates applicable varies for different
slabs of income. Total tax is computed by calculating the [Based on SNAP, 2012]

Difficulty Level-2
(Based on Memory)

1.
A person has some amount with him. 25% of it is stolen savings fell to 50% of the amount saved last month.
in a bus, 10% is lost through a hole in the pocket, 50% Salary of the last month was `10,000. This month there
of remainder is spent on food. He then purchases a book was an increase of 15% in the salary. How much did the
worth `26 from the remainder. He walks back home family spend this month?
because all his money is over. What was the initial (a) `667 (b) `11,167
amount?
(c) `10,833 (d) `9,833
(a) `160 (b) `1,230
(c) `90 (d) `80 5.
500 kg of ore contained a certain amount of iron. After
the first blast furnace process, 200 kg of slag containing
2.
Of the total number of candidates in an examination 40% 12.5% of iron was removed. The percentage of iron in
fail in first subject, of the rest 10% fail in the second and the remaining ore was found to be 20% more than the
of those that pass in these two subjects, only 75% pass percentage in the original ore. How many kg of iron were
in the remaining subject. The number of unsuccessful there in the original 500 kg ore?
candidates exceeds that of the successful ones by 570. (a) 212 (b) 89.2
What is the total number of candidates?
(c) 85 (d) 145
(a) 2,800 (b) 8,400
(c) 3,000 (d) 1,200 6.
If a > b, then by what percentage is less then its reciprocal?

3.
A man invests `1,200 at 10% p.a. At the end of the year  b 1  1 a2 
he withdraws 30% of total amount and pays `24 as (a) 100  2   (b) 100   
a b a b 
transaction fee. At the end of 2nd year he withdraws 30%
of the amount and pays `93 as transaction fee. What is the
balance at the end of the third year?  b2   a2 
(c) 100 1   (d) 100   1
2 2
(a) `660 (b) `825  a  b 
(c) `500 (d) `770
7.
In a society, there are 100 members. Each of them has
4.
A family’s ratio of savings to expenditure for last month been allotted membership number from 1 to 100. They
was 2:13. This month, due to unforeseen expenditure, started a business in which the nth member contributed

Chapter 05.indd 13 6/4/2015 2:47:27 PM


5.14 Chapter 5

`(10 × 2n – 5). After one year, 4th member gets `62 as his (a) 5.62% (b) 34%
share. Find the total profit in the business after one year. (c) 40% (d) 48%
(a) `8 [2100 – 26] (b) `4 [299 – 26] [Based on REC Tiruchirapalli, 2003]
(c) `2 [2100 – 26] (d) None of these 13.
5% income of A is equal to 15% income of B and 10%
[Based on FMS (Delhi), 2004] income of B is equal to 20% income of C. If the income of
C is `2000, then the total income of A, B and C (in `) is:
8.
In a school, 60% of the students of Class X were boys.
75% of the boys passed the Class X exams. 40% of the (a) 6000 (b) 9000
passed boys got first division. 80% of the total students (c) 12000 (d) 18000
passed the exam and 50% of the passed students got first [Based on FMS (Delhi), 2003]
division. Which of the following conclusion is not correct? 14.
Ajay and Vikas are sharing a flat in Delhi, with an
(a) 75% of the failed students are boys. arrangement of equally dividing the household expenses.
(b) 55% of the first-divisioners are girls. Ajay went to Pune, where a sale was going on and bought
(c) Number of passed girls is more than that of the boys. batteries for the house, worth `150 on 20% discount. But
he lost them on his way back and had to buy new ones,
(d) If x students failed, 2x got first division.
after he reached Delhi. How much did he end up spending
[Based on FMS (Delhi), 2004] on the batteries?
9.
A trader sells soaps in economy packs of four soaps (a) `280 (b) `195
per pack, each pack being charged at the listed price of (c) `270 (d) `75
three soaps. For every set of five such packs bought by [Based on SCMHRD Ent. Exam., 2003]
a customer, the trader gives him one extra soap as a free
gift. If a customer buys 12 economy packs, what is the Directions (Q 15 to 17): It is given that 5% increase in X always
effective percentage of discount that he gets? means 3% increase in Y and 5% increase in Y always implies 2.5%
4 increase in Z.
(a) 28% (b) 28 %
7 15.
Relationship between X and Z could be:
5 (a) 1.05X = 1.025Z (b) 0.5X = 0.25Z
(c) 4% (d) 35 %
7 (c) 25X = 7.5Z (d) 250X = 213Z
10.
The normal dosage of a particular medicine is t tablets per [Based on IMT Ghaziabad, 2002]
day for each patient. A hospital’s current supply of these If Y is increased by 30%, then Z2 should increase by:
16.
tablets will last p patients for d days. If the recommended (a) 32.25% (b) 60%
dosage increases by 20% and the number of patients
decreases by one-third, then for how many days will the (c) 69% (d) 90%
hospital’s supply last? [Based on IMT Ghaziabad, 2002]

5d 4d 17.
If X is increased from 10 to 15, then Z must increase from
(a) (b) 30 to:
4 5
4 pt (a) 35 (b) 45
(c) (d) None of these (c) 60 (d) 75
5
[Based on REC Tiruchirapalli, 2003] [Based on IMT Ghaziabad, 2002]

11.
In a town, 60% of the adult population is male. a% of 18.
In a market survey, 20% opted for product A, whereas
the adult males and b% adult females are educated. The 60% opted for product B. The remaining individuals were
total number of educated adult males and uneducated not certain. If the difference between those who opted for
adult females is equal in number to the total number of product B and those who were uncertain was 720, how
uneducated adult males and educated adult females. If many individuals were covered in the survey?
a and b are both integers, which of the following could be (a) 3,600 (b) 1,440
the set of values (a, b)? (c) 1,800 (d) Data inadequate
(a) (20, 30) (b) (20, 10)
19.
If the charges per hour of internet surfing are increased by
(c) (30, 15) (d) (30, 20) 25%, then find the percentage decrease in the time period
of surfing of a user (a net savvy) who can afford only a
12.
A clothing supplier stores 800 coats in a warehouse, of
10% increase in expenditure:
which 15 per cent are full-length-coats. If 500 of the
shorter length coats are removed from the warehouse, (a) 22% (b) 12%
then what per cent of the remaining coats are full-length? (c) 15% (d) 9.09%

Chapter 05.indd 14 6/4/2015 2:47:27 PM


5.15
Percentage

20.
Lagaan is levied on 60% of the cultivated land. The 28.
The price of LPG increases by 20% By what per cent
revenue department collected total `3,84,000 through the must a family reduce the consumption of LPG, so that the
lagaan from the village of Sukhiya. Sukhiya, a very rich expenditure on gas is the same as before:
farmer, paid only `480 as lagaan. The percentage of total (a) 83.33 (b) 80
land of Sukhiya in the total cultivated land of the village is:
(c) 20 (d) 16.66
(a) 0.15% (b) 15% [Based on FMS (Delhi), 2006]
(c) 0.125% (d) None of these
29.
In a certain water body 50 fish were caught, tagged and
21.
The price of cooking oil has increased by 25% The released to the same water body. A few days later, 50 fish
percentage of reduction that a family should effect in the were caught again, of which 2 were found to have been
use of cooking oil so as not to increase the expenditure on tagged on previous occasion. If the per cent of tagged fish
this account is: in the second catch approximates the per cent of tagged
(a) 25% (b) 30% fish in the water body, what is the approximate number of
(c) 20% (d) 15% fish in the water body?
[Based on FMS (Delhi), 2002] (a) 10000 (b) 625
22.
A flat costs `10 lakhs. Incidental expenses and taxes (c) 1250 (d) 2500
amount to `36000 a year. What rent per month must the [Based on ATMA, 2008]
owner receive to earn at least 6 per cent on his investment? 30.
Of the 50 researchers in a workgroup, 40% will be assigned
(a) `6000 (b) `6500 to team A and the remaining 60% to team B. However,
(c) `8000 (d) `8500 70% of the researchers prefer team A and 30% prefer team
[Based on FMS (Delhi), 2002] B. What is the least possible number of researchers who
will not be assigned to the team they prefer?
23.
A businessman charges 20% over cost price. He allows
his customers 10% off on their bills for cash payment. His (a) 15 (b) 20
net gain per cent is: (c) 35 (d) 30
(a) 10 (b) 8 [Based on ATMA, 2005]
(c) 12 (d) 15 31.
In a city, 35% of the population is composed of migrants,
[Based on FMS (Delhi), 2002] 20% of whom are from rural area. Of the local population,
24.
X gets 25% more than Y and Y gets 20% more than Z. The 48% is female while this figure for rural and urban migrants
share of Z out of a sum of `740 is: is 30% and 40% respectively. If the total population of the
city is 728400 what is its female population?
(a) `200 (b) `300
(a) 509940 (b) 349680
(c) `250 (d) `350
[Based on FMS (Delhi), 2002] (c) 324138 (d) None of these
[Based on NMAT, 2005]
25.
In my office there are 30% female employees and 30% of
these earn greater than `8,000 per month and 80% of male 32. A company received two shipments of ball bearings. In
employees earn less than `8,000 per month. What is the the first shipment, 1% of the ball bearings were defective.
percentage of employees who earn more than `8,000 per In the second shipment, which was twice as large as the
month? first, 4.5% of the ball bearings were defective. If the
(a) 30% (b) 23% company received a total of 100 defective ball bearings,
how many ball bearings were in the first shipment?
(c) 60% (d) Cannot be determined
(a) 2000 (b) 1000
26.
A house-owner was having his house painted. He was
advised that he would require 25 kg of paint. Allowing for (c) 990 (d) 3000
15% wastage and assuming that the paint is available in 2 [Based on NMAT, 2005]
kg cans, what would be the cost of paint purchased, if one 33.
There are two types of employees in Sun Metals, general
can costs `16? graduates and engineers. 40% of the employees in Sun
(a) `240 (b) `180 Metals are general graduates and 75% of the engineers
(c) `160 (d) `360 earn more than `5 lakhs/year. If 50% of the organization’s
employees earn more than `5 lakhs/year, what proportion
27.
On April 1, 2005 my salary increased from `10,000 to
of the general graduates employed by the organization
`16,000. Simultaneously the rate of income tax decreased
earn `5 lakhs or less?
by 37.5% If so the amount of income tax paid by me
remains constant, what is the value of income tax I pay. (a) 3/5 (b) 7/8
(a) `3,000 (b) `6,000 (c) 1/2 (d) None of these
(c) `1,600 (d) Cannot be determined [Based on XAT, 2010]

Chapter 05.indd 15 6/4/2015 2:47:27 PM


5.16 Chapter 5

34.
A survey shows that 61%, 46% and 29% of the people 39.
A 14.4 kg gas cylinder runs for 104 h when the smaller
watched ‘3 idiots’, ‘Rajneeti’ and ‘Avtaar’ respectively. burner on the gas stove is fully opened while it runs for 80
25% people watched exactly two of the three movies and h when the larger burner on the gas stove is fully opened.
3% watched none. What percentage of people watched all Which of these values are the closest to the percentage
the three movies? difference in the usage of gas per hour, of the smaller
(a) 39% (b) 11% burner over the larger burner?
(c) 14% (d) 7% (a) 26.23% (b) 30%
[Based on IIFT, 2010] (c) 32.23% (d) 23.07%
35.
Bennett distribution company, a subsidiary of a major [Based on SNAP, 2008]
cosmetics manufacturer Bavlon, is forecasting the zonal 40. What is the present worth of a house that would worth
sales for the next year. Zone I with current yearly sales ` 50000 after 3 years if it depreciates at the rate of 10 %?
to `193.8 lakhs is expected to achieve a sales growth (a) `35765.74
of 7.25%; Zone II with current sales of `79.3 lakhs is
(b) `67560.74
expected to grow by 8.2% and Zone III with sales of
`57.5 lakhs is expected to increase sales by 7.15% What (c) `67655.74
is the Bennett’s expected sales growth for the next year ? (d) `68587.10
(a) 7.46% (b) 7.53% [Based on CAT, 2009]

(c) 7.88% (d) 7.41% 41. What percentage of the viewers responded to all three?
[Based on IIFT, 2009] (a) 10 (b) 12
36.
In 2006, Raveendra was allotted 650 shares of Sun (c) 14 (d) None of these
Systems Ltd in the initiail public offer, at the face value of [Based on CAT, 2010]
`10 per share. In 2007, Sun Systems declared a bonus at 42. Assuming 20% respond to Ahead and Bong, and 16%
the rate of 3:13. In 2008, the company again declared the respond to Bong and Luck, what is the percentage of
bonus at the rate of 2:4. In 2009, the company declared a viewers who watch only Luck?
dividend of 12.5% How much dividend does Raveendra
(a) 20 (b) 10
get in 2009 as the percentage of his initial investment?
(c) 16 (d) None of these
(a) 24.5% (b) 23.9%
[Based on CAT, 2010]
(c) 24.1% (d) 23%
43. A piece of paper is in the shape of a right-angled triangle
[Based on IIFT, 2009]
and is cut along a line that is parallel to the hypotenuse,
37.
In view of the present global financial crisis, the leaving a smaller triangle. There was a 35% reduction in
Finance Minister decided to slash the excise duties to the length of the hypotenuse of the triangle. If the area of
boost demand and propel economic growth. The excise the original triangle was 34 cm2 before the cut, what is the
duty on cement was reduced by 30% of its present area of the smaller triangle (in cm2) formed after the cut.
amount to boost the spending in the infrastructure. What (a) 16.565 (b) 15.465
should be the percentage increase in the consumption of
(c) 16.665 (d) 14.365
cement so that the revenue of the government remains
unchanged? [Based on CAT, 2013]
44. Fresh grapes contain 90% water by weight while dried
5 6
(a) 42 % (b) 42 % grapes contain 20% water by weight. What is the weight
7 7 of dry grapes available from 20 kg of fresh grapes?
6 5 (a) 2 kg (b) 2.4 kg
(c) 34 % (d) 34 %
7 7 (c) 2.5 kg (d) None of these
[Based on FMS, 2009] [Based on CAT, 1997, 2001]
38.
In a public school, one-fifth of girls and one-fourth of boys Directions (Q 45–46): Answer the questions based on the
are under 12 year of age. If the total strength of the school following information.
is 1000 and number of girls is two-fifths of the total, what
part of the total strength of the school is accounted for by A company purchases components A and B from Germany and
those which are 12 year or more of age? USA, respectively. A and B form 30% and 50% of total production
cost. Current gain is 20% Due to change in the international
(a) 23% (b) 45%
scenario, cost of the German mark increased by 30% and that
(c) 55% (d) 77% of USA dollar increased by 22% Due to market conditions, the
[Based on FMS (MS), 2006] selling price cannot be increased beyond 10%

Chapter 05.indd 16 6/4/2015 2:47:27 PM


5.17
Percentage

45. What is the maximum current gain possible? from a mine of hematite was 8000 kg then the quantity of
(a) 10% (b) 12.5% hematite mined in the year is:
(c) 0% (d) 7.5% (a) 400000 kg (b) 500000 kg
[Based on CAT, 1998] (c) 450000 kg (d) 600000 kg
46. If the USA dollar becomes cheap by 12% over its original [Based on MAT, 1998]
cost and the cost of German mark increased by 20%, what 49. In a medical certificate, by mistake a candidate gave his
will be the gain? (The selling price is not altered) height as 25% more than actual. In the interview panel,
(a) 10% (b) 20% he clarified that his height was 55 ft 5 inches. What is
(c) 15% (d) 7.5% the percentage correction made by the candidate from his
[Based on CAT, 1998] stated height to his actual height?
47. Suppose that a maximum of 25g salt dissolves in 100 g of (a) 28.56% (b) 20%
water. Any more salt, if added, remains undissolved and (c) 25% (d) None of these
a sediment falls at the bottom. Now, water is evaporated [Based on MAT, 2013]
from 1 kg of a 4% solution at the rate of 28 g hour. After 50. The total tractor population in a State is 294000, out of
how long will it start sedimenting, approximately? which 150000 are made by Mahindra and Mahindra. Out
(a) 29 h (b) 31 h of every 1000 Mahindra tractors, 98 are red in colour.
(c) 35 h (d) 23 h But only 5.3% of the total tractor population is red. Find
[Based on MAT, 2013] percentage of non-Mahindra tractors that are red.
48. Out of the total production of iron hematite, an ore of iron, (a) 6.125% (b) 5.025%
20% of the ore gets wasted. Out of the remaining iron, (c) 5.130% (d) 6.140%
only 25% is pure iron, If the pure iron obtained in a year [Based on MAT, 2013]

Answer Keys
Difficulty Level-1

1. (b) 2. (d ) 3. (d ) 4. (b) 5. (b) 6. (b) 7. (c) 8. (c) 9. (b) 10. (b) 11. (c) 12. (c) 13. (b)
14. (b) 15. (d ) 16. (c) 17. (d ) 18. (a) 19. (c) 20. (b) 21. (c) 22. (c) 23. (a) 24. (c) 25. (a) 26. (c)
27. (c) 28. (c) 29. (c) 30. (b) 31. (d ) 32. (b) 33. (d ) 34. (c) 35. (b) 36. (b) 37. (d ) 38. (a) 39. (d )
40. (b) 41. (c) 42. (d ) 43. (d ) 44. (d ) 45. (d ) 46. (c) 47. (b) 48. (b) 49. (c) 50. (b) 51. (c) 52. (a)
53. (c) 54. (d ) 55. (b) 56. (b) 57. (c) 58. (a) 59. (c) 60. (a) 61. (a) 62. (b) 63. (c) 64. (a) 65. (d )
66. (c) 67. (d ) 68. (c) 69. (b) 70. (b) 71. (c) 72. (d ) 73. (b) 74. (a) 75. (c) 76. (a) 77. (a) 78. (d )
79. (c) 80. (d ) 81. (d ) 82. (b) 83. (d ) 84. (a) 85. (c) 86. (b) 87. (b) 88. (c) 89. (b) 90. (b) 91. (a)
92. (a) 93. (d) 94. (d ) 95. (c ) 96. (b) 97. (b) 98. (a) 99. (d)

Difficulty Level-2

1. (d ) 2. (c) 3. (a) 4. (c) 5. (b) 6. (c) 7. (a) 8. (c) 9. (a) 10. (a) 11. (d ) 12. (c) 13. (d )
14. (b) 15. (c) 16. (a) 17. (a) 18. (c) 19. (b) 20. (d ) 21. (c) 22. (b) 23. (b) 24. (a) 25. (d ) 26. (a)
27. (d ) 28. (d ) 29. (c ) 30. (d) 31. (c) 32. (b) 33. (b) 34. (d ) 35. (a) 36. (d ) 37. (b) 38. (d ) 39. (b)
­­­ 40. (d ) 41. (a) 42. (d ) 43. (d ) 44. (c) 45. (a) 46. (b) 47. (b) 48. (a) 49. (b) 50. (a)

Chapter 05.indd 17 6/4/2015 2:59:28 PM


5.18 Chapter 5

Explanatory Answers

Difficulty Level-1

1.
(b) 100 → 105 → 110.25, i.e., 10.25% 8. (c) Let the original price be `x per kg
2. (d) Ratio of maximum marks = 1:2:2 4
\ Reduced price = ` x per kg
Ratio of marks obtained 5
= (0.5 × 1):(0.6 × 2):(0.65 × 2) 5 160 1
= 0.5:1.2:1.3 ⇒ × 160 = +2
4x x 2
⇒ Overall percentage 200 160 5
0.5  1.2  1.3 ⇒ = +
= × 100 x x 2
1 2  2 40 5
= 60% ⇒ = ⇒ x = 16.
x 2
3.
(d) Required ratio = 4VA dA:7VBdB
(−20) × (−15) −32
4VAd A 9.
(b) (–20) + (–15) + = .
= :7VB, 100 100
dB
where d is density of the substance 10.
(b) Let X’s salary = `100
Given 117dA = 151dB \ Salary after increase = `120, i.e., `20 is the increase
dA 151 in salary on `100.
\ =
dB 117
Let the tax on the original salary was 30%
Now with 7VB of substance B, 4VA of substance A is
151 Hence the tax on the increased salary (`20) will be
used in place of 4VA × 33%, i.e., `6.60.
117
34 117 6.60
⇒ % error = × × 100 ≈ 22% \ Increase in tax liability = × 100 = 22%
117 151 30
4. (b) 100 – 20% = 80 11. (c) Let reduced price of the radio be `x.
80 – 30% = 56
\ x + 7% of x = 2568
\ Single discount = 44%
⇒ 107x = 256800
5. (b) If the weight of Manu is 50 kg then Ram’s weight will
be 70 kg. So, Mahesh’s weight is 100 kg and Tanu’s ⇒ x = 2400
weight is 90 kg. Hence, percentage of Ram’s weight \ Reduction needed in the price of radio = `168.
to Tanu’s weight
12.
(c) 600 – 25% = 450
70
= × 100 = 77.8% 450 – 10% = 405.
90
6.
(b) A = B + 25% of B 13.
(b) Suppose the shopkeeper had x eggs, originally.
B 5B x
⇒ A = B + = No. of broken eggs = 5% of x =
4 4 20
4 1
⇒ B = A = A − A = A – 20% of A. x 19 x
5 5 Balance = x − =
20 20
7.
(c) The businessman’s earning after five years
= `72,000 19 x
\ 7% of = 266 ⇒ x = 4000.
Let his earnings be `100 20
After 1st year →  125  (25% increase)
14.
(b) Let the number be K
After 2nd year →  120  (4% decrease)
After 3rd year →  150  (25% increase) 3 1
After 4th year →  144  (4% decrease) \ K = K + 23
5 2
After 5th year →  180  (25% increase)
100 ⇒ K = 230
∴ Earning today =  72,000 = `40,000
180 ⇒ 80% of K = 80% of 230 = 184.

Chapter 05.indd 18 6/4/2015 2:47:28 PM


5.19
Percentage

15.
(d) Let C.P. of 1 kg of tea be `x and 1 kg of sugar be `y. p
20. (b) Let the fraction be
\ x + y = 95 (1) q

 x  y p + 20% of p 16
Also,  x −  +  y +  = 90 \ =
 10   5 q − 10% of q 21
⇒ 3x + 4y = 300 (2)
p
From (1) and (2) we, get p+
⇒ 5 = 16
x = 80, y = 15. q 21
q−
10
16.
(c) Let the third number be x.
3x 7x 6 p 10 16
\ 1st number = x – 30% of x = x − = ⇒ × =
10 10 5 9q 21
4x 6x
2nd number = x – 40% of x = x − = p 16 9 144 4
10 10 ⇒ = × = = .
q 21 12 252 7
Suppose 2nd number = K% of 1st number
6x K 7x 21.
(c) Let the marked price be `x
\ = ×
10 100 10
Let the C.P. be `y
600 5
⇒ K = = 85 .
7 7 95 x 19 x
S.P. = x – 5% of x = =
100 20
17. (d) Let 100 angstroms = x% of 10 microns
19 x
⇒ 100 angstroms = x% of 100000 angstroms y + 33% of y =
20
100 × 100 1
⇒ x = = = 0.1. ⇒ 7y = 5x
100000 10
18.
(a) Suppose side of the square = 1 metre 7 2
⇒  x = y = y+ y
\ Diagonal = 2m 5 5
= y + 40 % of y.

22.
(c) No. of singers = 20
No. of dancers = 40
\ 4 singers are less than 25 years old.
24 out of 60 members of the group are less than 25
years old.
\ 20 dancers are less than 25 years old, i.e., 50% of
Distance saved by not walking along the edges the dancers are less than 25 years old.
= 2 − 2
23.
(a) Suppose Tito’s salary = x,
2− 2 
i.e.,  × 100  % i.e., 29.3% ≈ 30%
Tom’s salary = y and Tina’s salary = z
 2 
5z
19.
(c) 0.25x = 0.3y = 0.45z \ y = 125% of z =
4
x  x y
Now, × 100 =    × 100 4 5
z  y z x = 80% of z = z ⇒z= x
5 4
0.3 0.45
=  × 100
0.25 0.3 5z 5 5 25
\ y = = × x = x
9 4 4 4 16
= × 100 = 180%
5 Also x + y + z = 61000

Chapter 05.indd 19 6/4/2015 2:47:29 PM


5.20 Chapter 5

25 5 30.
(b) A = 125% of B, C = 80% of B
⇒ x + x + x = 61000
16 4 5 5 4 4 16
61x ⇒ A = B,C= B = × A = A
⇒ = 61000 4 4 5 5 25
16
⇒ x = 16000 A + B + C = 61000
Also, y = 25000, z = 20000. 4 16
⇒ A + A+ A = 61000
24.
(c) Mayur = 2 (Shweta) 5 25
3 ⇒ A = 25000
Shweta = 60% of Deepika = of Deepika
5
\ B = 20000, C = 16000.
1
Rakesh = 50% of Vikas = of Vikas
2 31.
(d) Let the total number of votes polled = x
19 \ The winning candidate got 70% of the votes
Vikas = 190% of Mayur = of Mayur
10 polled
\ M > D, S < D, R < V, V > M Hence, 40% of x = 16000 ⇒ x = 40000.
⇒ V > M > D > S, V > R
32. (b) Let the earlier price of groundnut be ` x/kg
19
Rakesh = of Mayur 48
10 \ kg of groundnuts could be purchased for `48
x
20
⇒  Mayur = of Rakesh
19  48 3  5x
⇒  − × = 48 ⇒ x = 6.40.
10  x 2 4
Shweta = of Rakesh
19 33.
(d) X + Y + Z = 7400 (1)
⇒ S < R ⇒ R > S 5Y
\ Shweta weighs the least. X = Y + 25% of Y = (2)
4
25.
(a) 1, 9, 11, 19, 21, 29, 31, 39, 41, 49, 51, 59, 61, 69:
6Z
These numbers have their squares ending in digit 1, Y = Z + 20% of Z = (3)
5
i.e., 14 out of 70, i.e., 20% \ Eq. (1) gives
26.
(c) X is increased from 20 to 23, i.e., there is 15% increase 5Y 6 Z
+ + Z = 7400
in X. 4 5
⇒ There is 9% increase in Y i.e., Y will increase from 3Z 6 Z
100 to 109. ⇒ + + Z = 7400
2 5
⇒ 37Z = 74000 ⇒ Z = 2000
27.
(c) Suppose salary in 1999 was `x
\ Y = 2400, X = 3000.
2
 20 
\ x 1 +  = 26640 34. (c) 8% of m = 4% of p ⇒ 2m = p
 100 
m p
\ 20% of m = = = 10% of p.
26640 × 25 5 10
⇒ x= = `18500. 35. (b) S = 150% of T
36
150T 3
1 1 ⇒ S = ⇒ S =  T
28.
(c) R = B,G= R =G=Y 100 2
3 2
3 5T
Since, B = 42, R = 14, G = 7 and Y = 7, ⇒ S + T = T +T =
2 2
42
\ Percentage of blue caps = × 100 = 60. 2
70 (S + T )
⇒ T =
5
29.
(c) No. of pens removed = 40% of (S + T).
= 12% of 600 + 25% of 1200
36. (b) Suppose total number of students = 100
= 72 + 300 = 372 \ No. of seniors who attended the play = 20
\ Percentage of total pens removed Total number of students who attended the play = 60
372 \ No. of non-seniors who attended the play
= × 100 = 20.67 = 22.
1800 = 60 – 20 = 40 i.e., 40%

Chapter 05.indd 20 6/4/2015 2:47:30 PM


5.21
Percentage

37.
(d) Passing marks are 0.6x 41.
(c) Total fruit = 14 + 23 = 37
So, 0.3x + 30 = 0.6x x × 70
Again, = 14
100
⇒ x = 100 x = 20
38.
(a) Rainbow will occur once in 20 days Reduce oranges = 37 – 20 = 17
Rest 19 days will not producer rainbow 4 × (− x)
42.
(d) 4 – x + = 10
19 100
% = × 100 = 95% 4x
20 ⇒ –x – =6
100
39.
(d) Let there be a gm of food X and (300 – a) g of food Y.
⇒ –104x = 600
Then, a × 10% + (300 – a) 15% = 38 −600
⇒  x = %
10a 15 104
⇒  + (33 − a )  = 38
100 100
43.
(d) 97% = 23700
⇒ 10a + 4500 – 15a = 3800 23700
\ 93% = × 93 = `22722.679
97
⇒  – 5a = – 700
44.
(d) Number of full length coast = 15% of 800 = 120
700
\  a = Remaining coats = 800 – 500 = 300
5
120
= 140 g of food X \ Required percentage = × 100% = 40%
300
40.
(b) Suppose that total candidates appeared be x. 45.
(d) Let earlier there was consumption of 1 kg wheat
Then, number of candidates who answered all \ Expenditure = 24
5 Now, 27 is price of 1 kg
= x × 1
100 \ 1 is price of kg
27
Number of candidates who answered none
24 8
5 \ 24 is price of = kg
= x × 27 9
100 1
\ Consumption should be reduced by or 11.1%
 5  9
\ Remaining = x − 2  x × 
 100  46.
(c) Total production of red tractors
x 9x = 294000 × 53% = 15582
= x − =
10 10 Number of red tractors of Mahindra make
Number of candidates answered only 1 question
150000
= × 98 = 14700
9x 25 9 x 1000
= 25% of = ×
10 100 10 Number of non-Mahindra tractors
9x = 294000 – 150000 =144000
=
40
\ Number of Mahindra red tractors
Number of candidates answered 4 questions
= 15582 – 14700 = 882
9x 9x
= × 20% = Hence, required percentage
10 50
Candidates who answered 2 questions 882
= × 100 = 0.6125%
49 49 144000
= % of x = x
2 200 47.
(b) Let total marks be 100.
 5  9 x 9 x 49 x Then, minimum marks to be passed 40.
Now 2  x × + + + + 200 = x
 100  40 50 200
10
\ A obtained marks = 40 – 40 ×
By solving this, we get x = 800. Number of candidates 100
appeared = 800. = 36 marks

Chapter 05.indd 21 6/4/2015 2:47:31 PM


5.22 Chapter 5

100 36 xy
\ B obtained marks = 36 – × 53. (c) Using, formula K % = – x + y –
9 100 100
= 36 – 4 10 × 10
 = – 10 + 10 – = – 1%
= 32 marks 100
\ C obtained marks 54.
(d) Let his sales were x.
700 Then,
= (36 + 32) – (36 + 32) ×
17 × 100 2.5 5
= 68 – 28 1000 + ( x − 4000) = x + 600
1000 100
= 40 marks.
⇒  100000 + 2.5x – 10000 = 5x + 60000
48.
(b) Let monthly salary be `x. ⇒ 2.5x = 30000
According to the given condition, ⇒  x = `12000
4 x 6 x 50 3 x 30 55.
(b) Let the capacity of bucket be x L.
+ × + × + saving money = x
10 10 100 10 100 Then, 0.80x = 0.667x + 2
4 x 3x 9 x
⇒  + + + 630 = x 2
10 10 100 ⇒  x = = 15 L
0.133
100 x − 70 x − 9 x
⇒ 630 =
100 240
56.
(b) In normal time rate = = `6 per hour
21x 40
⇒ 630 =
100
⇒  x = `3000 During sale rates are increased by 50% i.e.,

49.
(c) Let the price of sugar be `x per kg. Rates 40-hour week = 240 + 50% of 240
\ Initial expenditure = `30x = `360
New expenditure = `33x 360
\ Rate per hour = = `9 per hour
\ New monthly consumption 40
33 x Now, according to the question,
= = 25 kg
1.32 x Required commission = 9 × 60 = `540
50.
(b) Let his increased income be x. 57.
(c) Let the maximum marks be x.
80 12 80 10 Then, 296 – 259 = 5% of x
( x − 12000) × + = x× ×
100 100 100 100 5
⇒ 12x – 14400 = 10x ⇒  x = 37
100
⇒ x = `7200
⇒  x = 740
51.
(c) Let his monthly rent be x.
58.
(a) Marks obtained by Sushant = 1080
12.5 5.5
12 x − 12 x × − 325 = 100000 × Marks obtained by Mohit
100 100
⇒ 12x – 1.5x – 325 = 5500 = 1.2 × 1080 = 1296
5500 + 325 1296
⇒ x = ≈ `554.76 Marks obtained by Rajesh = = 1440
10.5 0.9
52.
(a) Let his monthly salary be `x. So, percentage of marks obtained by Rajesh
He spends `0.4x on educational expenses, `0.24x on 1440
purchasing books and `0.08x on purchasing stationary = × 100 = 72%
2000
items.
59.
(c) Let Ram’s monthly income be x.
Remaining amount = 0.4x – (0.24x + 0.08x)
80 85 70
= `0.08x Total savings = x × × ×
100 100 100
1
Also, × 0.08x = 160
4 100 100 100
⇒  x = 9520 × × ×
160 × 4 80 85 70
\ x = = `8000
0.08 = `20000

Chapter 05.indd 22 6/4/2015 2:47:32 PM


5.23
Percentage

60.
(a) Let the quantity of haematite mined be x kg. \ New expenditure on water
80 25 6 4 24
Then, x × × = 80000 = ` x × y = ` xy
100 100 5 5 25
100 100 \ Decrease in expenditure on water
⇒  x = 80000 × ×
80 25  24  1
= `  xy − xy  = ` xy
= 400000 kg  25  25
Hence, percentage decrease
61.
(a) Total number of votes = 6000
1
Total number of valid votes xy
100
= 25 × 100 = = 4%
= 6000 × 0.75 = 4500 xy 25
Total valid votes that Bhiku gets Quicker Method: If the value is first increased by x%
= 4500 × 0.65 = 2925  xy 
and then decreased by y% then there is  x − y − %
 100 
Total valid voted that Mhatre gets increase or decrease, according to the +ve or –ve sign
= 4500 – 2925 = 1575 respectively.
62.
(b) Required percentage correction Here, x = 20% and y = 20%

1.25 − 1 20 × 20
= × 100 = 20% \ % effect = 20 – 20 – =–4
1.25 100
Note: This question can be solved, even if his height
Thus, there is 4% decrease in the expenditure of the
is not given because there is no need of his height. money.

63.
(c) Required marks i.e., 50% of (180 + 150) = 165 66.
(c) G = B + 15 (1)

Marks scored in first paper = 54 G + 10% of G = B + 16% of B + 9


11G 116 B
Marks required to be scored in second paper = 111 i.e., = +9
10 100
111
\ Required percentage = × 100 = 74% i.e., 110G – 116B = 900 (2)
150
Using (2) in (1), we get
64.
(a) Difference in quantity (percentage) lost B = 125, G = 140
= 7% – 5.2% = 1.8% \ Total number of students = B + G = 265.
Let the total quantity of wheat grown be x million
67.
(d) Let the original number be 10x + y
tonnes.
Here unit’s digit is y.
Then,
According to the question, unit’s digit of new number
1.8% x = (6 + 3)
= y + 100% of y = 3y
9 × 100
⇒ x = = 500 million tonnes Ten’s digit of new number
1.8
= 10x + 50% of 10x = 15x
65.
(d) Suppose water tax = `x
Now, 15x + 3y – 10x – y = 19
Consumption of water = y litres
or, 5x + 2y = 19
\ Original expenditure on water = `xy By trial and error method, we have x = 3 and y = 2
Increased water tax = `(x + 20% of x)
Since 5 × 3 + 2 × 2 = 19
6 \ Original number = 10 × 3 + 2 = 32
=` x
5
68.
(c) Suppose there are 100 candidates for entrance.
Decreased consumption of water
\ No. of capable candidates = 40
4 and no. of incapable candidates
= y – 20% of y = y litre
5 = 100 – 40 = 60

Chapter 05.indd 23 6/4/2015 3:00:54 PM


5.24 Chapter 5

Now, no. of capable candidates who pass the test Amount spent on books
= 80% of 40 = 32 = `20
No. of incapable candidates who pass the test \ Monthly amount saved
= 25% of 60 = 15
= `100 – (70 + 20) = `10
(Since these successful candidates become college
students.) Actual yearly savings
Thus, there are 32 + 15 = 47 colleges students in all, = `4800
of which 32 are capable. Actual monthly savings
Hence, proportion of capable college students 4800
= ` = `400
32 12
= × 100 ≈ 68%
47 If the amount saved is `10, monthly fellowship
= `100
69.
(b) Total candidates = 2000 \ If the amount saved is `400 monthly fellowship
No. of boys = 900 100
= ` × 400 = `4000.
No. of girls = 1100 10
No. of students who passed 73.
(b) Number of literate women
32 × 900 38 × 1100 50 70
= + = 296000 × – 166000 ×
100 100 100 100
= 288 + 418 = 706 = 148000 – 116200
= 31800
No. of students who failed = 2000 – 706 = 1294
74.
(a) Let initial expenditure and savings be 3x and 2x
1294 respectively.
Required percentage = × 100 = 64.7 %
2000
So, initial income = 3x + 2x = 5x
70.
(b) Required per cent decrease New income = 5.5x
30 300 1 New expenditure = 3x × 1.12 = 3.36x
= × 100 = = 23 %
130 13 13
New savings = 5.5x – 3.36x = 2.14x
71.
(c) Let the total number of workers be 100. Percentage increase in savings
Number of skilled workers = 75% of 100 = 75 2.14 x − 2 x
= × 100 = 7%
Number of unskilled workers = 100 – 75 = 25 2x
Number of permanent workers = 80% of 75 + 20% of 25
75.
(c) Total score of first three friends = 15 × 3 = 45
80 20 and total score of last three friends = 16 × 3 = 48
= × 75 + × 25
100 100 \ Total score of four friends = 45 + 19 = 64
= 60 + 5 = 65 \ Score of first friend = 64 – 48 = 16
16 1
Number of temporary workers = 100 – 65 = 35 \ Required percentage = × 100% = 33 %
48 3
The number of temporary workers are 35, then total
workers = 100 76.
(a) Required price of the single ticket
⇒ Number of total workers when number of 84 100 100 100 100
= × × = 84 × × = `64
temporary workers is 126 105 1 125 105 125
100 77.
(a) Suppose original price = `100
= × 126 =
360.
35
Reduction = 20%
72.
(d) Suppose monthly fellowship 100 − 20
\ Reduced price = `100 ×
= `100 100
Amount spent on monthly expenses 80
= `100 ×  = `80
= `70 100

Chapter 05.indd 24 6/4/2015 2:47:32 PM


5.25
Percentage I
Increased in sale = 80% 84. (a) Number of pages in notebook X = 120
100 + 80 Number of pages in notebook Y = 110% of 120 = 132
\ Increased sale = `80 ×
180 Number of pages in notebook Z = 90% of 120 =108
Total number of pages in all the notebooks = 120+
180
= `80 × 132+ 108 = 360
  100 Number of pages torn by Shyam
= `144 in notebook X = 5% of 120 = 6
  in notebook Y = 10% of 132 = 13.2
Net increase in sale = `144 – `100 in notebook Z = 15% of 108 = 10.8
= `44 on `100 Total number of pages torn = 6 + 13.2 + 10.8 = 30
30
= 44% ∴Required percentage = × 100%  8%
360
78.
(d) Suppose the numbers are x, y and z.
85. (c) Total number of students = 90
\ x = 20% of z, y = 50% of z Now, each of 50% of students get 20% of the total
2 1 number of students i.e., 20% of 90 = 18
\ x = y ⇒ y = z Also, each of remaining 50% of students get 10% of
5 2
the total number of students i.e., 10% of 90 = 9
2 x y Hence, total number of sweets distributed
⇒ x = y ⇒ = =K
5 2 5 = 45 × 18 + 45 × 9
⇒ x = 2K, y = 5K = 45 × (18 + 9)
\ Percentage of x to y is = 45 × 27
2K
×100 = 40 = 1215.
5K 86. (b) Let Saurabh should answer x% of 75 questions.
79.
(c) Given, x < y Then, 80% of 75 + x% of 75 = 60% of 150
80 × 75 + x × 75 = 60 × 150
The per cent by which x is less than y
⇒ 80 + x = 60 × 2
y−x ⇒ 80 + x = 120
= × 100
y
⇒ x = 40
80. (d) r = 25% 87. (b) Number of questions Nilam did
r = 70% of 10 + 40% of 30 + 60% of 35 = 7 + 12 + 21
Required reduction of petrol = × 100%
100 + r = 40
25 To get 60%, Nilam should have done = 60% of 75 = 45
= × 100% = 20%
125 Hence, she would have solved 5 more questions
correctly.
81. (d) Let X million be the required quantity of wheat bags. 88. (c) Content of sand = 45% of 900 kg
Then, given, 45 × 900
95% of x + 20 = 98% of x + 15 = = 450 kg
100
⇒  3% of x = 5
Content of cement = 900 − (405 + 144)
5 500 2
⇒  x = × 100 = = 166 = 900 − 549 = 351
3 3 3
351
82. (b) If number of individuals be X, then ∴ Required percentage = × 100%
900
60% of x – 20% of x = 720 = 39%

⇒  40% of x = 720
89. (b) Let the total number of apples be 100, the vendor sold
720 60%, then per centage of apples remained
⇒  x = × 100 = 1800
40 = (100 − 60) = 40 %
83. (d) Let cost of the tax free items be x 40 × 100
Now, 40 % of 100 = = 40
Then 6% of x = 30 paise 100
30 He throws 15 % of the remaining apples
⇒ x = × 100 paise = 500 paise= `5 15 × 40
6 = 15% of 40 = =6
∴ Cost of tax free items is 25 – 5 = `20 100

M05_KHAT6981_C05.indd 25 6/16/2015 3:48:22 PM


5.26 I Chapter 5

Apples left with vendor = 40−6 = 34 92. (a) Total number of students qualifying the test
On 2nd day, he sold 50% of apples and throws the 400 × 65
= = 260
remaining. 100
50 × 34 Let number of girls = x
∴ 50% of 34 = = 17
100 and number of boys = (400 − x)
∴ Total apples which the vendor throws = (17 + 6) x × 80 4 x
= 23 Now, cut off cleared by girls = =
100 5
23 × 100 and cut off cleared by boys
Therefore, required percentage = = 233%
100 (400 − x) × 60 1200 − 3 x
=
90. (b) Cost, when two 50 g toothpastes are purchased 100 5
= 2 × 299 = `598 4 x 1200 − 3 x
Now, + = 260
Cost, when one 100 g toothpaste is purchased = ` 509 5 5
4 x + 1200 − 3 x
Then, required percentage ⇒ = 260
598 − 509 5
× 100 % ⇒ x + 1200 = 1300
509
89 ⇒ x = 1300 − 1200 = 100
= × 100 % = 17.48 ≈ 18%
509 ∴ x = 100

Hence, one hundred girlls appeared in the test.
91. (a) Amount of water in 1st vessel
1 93. (d) Let Ajitha’s initial salary = `100
= × 100 = 20 L
5 15 375
Now, after raises the salary = 100 × =`
∴Amount of milk in1st vessel 8 2
4 375 175
= × 100 = 80 L So, Raises in the salary = − 100 = `
5 2 2
Similarly, amount of water in 2nd vessel Let, first raise in salary = x%
3 and second raise in salary = 2x%
= × 100 = 60 L 175
5 ∴ x + 2x =
Amount of milk in 2nd vessel 2
175 175 175
2 ⇒ 3x = ⇒x= =
= × 100 = 40 L 2 2×3 6
5
⇒ x = 29.16667 ≈ 30%
Amount of water in 3rd vessel ∴ x = 30% (approox)

3 300
= × 100 = 94. (d) Let, the person’s income = `100
7 7
and savings = 6% of `100 = `6
Amount of milk in 3rd vessel
∴ Expenditure = 100 – 6 = `94
4 400 2 yr later, his income = 100 + 15% of 100
= × 100 =
7 7 = 100 + 15 = `115
∴ Required ratio Now, savings = `6
300 ∴ Expenditure = 115 −6 = `109
20 + 60 +
= 7 109 − 94
400 So, percentage hike in expenditure = × 100%
80 + 40 + 94
7 15
140 + 420 + 300 = × 100%
94
= 7
= 15.95%
560 + 280 + 400
7 95. (c) Let original price of petrol = ` x per gallon
860 43 90 9x
= = After deduction, price of petrol = ` x × =`
1240 622 100 10

M05_KHAT6981_C05.indd 26 6/16/2015 4:31:13 PM


5.27
Percentage I
We are given: 97. (b) Let total monthly income of Mr. Girdhar be `x.
1800 1800 We are given,
− =5
9x x 50 15
x× × = 900
10 100 100
2000 1800 200 x = `1200
⇒ − =5 ⇒ =5
x x x Hence, monthly income of Mr. Giridhar = `12000.
200 98. (a) No. of boys, last year = 610
⇒ x=
5 20% of 610 = 122
∴ x = ‘40 No. of boys, current year = 610 – 122 = 488
Hence, original price of petrol = `40 per gallon No. of girls = 175% of 488
96. (b) Rohan’s marks = 75
175 × 488
Sonia’s marks = 65 = = 854 girls
100
Rohit’s marks = 65 + 45 = 110
Raman’s marks = 110 − 25 = 85 99. (d) Total income can be divided as,
Ravi got marks = 85 + 34 = 119
`170,000 = 50,000 + 10,000 + 90,000 + 20,000
Total maximum marks = 119 + 50 + 169
119 ∴ Total tax payable on `170000.= 50,000 × 0% +
Percentage of Ravi’s marks = × 100% = 70.4% 10000 × 10% + 90,000 × 20% + 20,000 × 30% = 1000
169
= 70% + 18000 + 6000= `25000

Difficulty Level-2

1. (d) Let total money be `X Number of students failing – Number of students


Then, X = 0.25X + 0.1X + 0.5 [1 – 0.25 – 0.1] X + 26 passing = 570 (Given)
⇒ X = `80  119 81
i.e.,     % of X = 570 ⇒ 19% of X = 570
 2 2
2.
(c) Let the total number of students = X
570  100
Number of students failing in first subject = 40% of X ⇒  Thus, X = = 3000
19
Number of students failing in second subject Hence, the total number of students are 3,000
= 10% of rest = 10% (60%) of X = 6% of X
Therefore, total number of students failing in both the 3.
(a) The man invests `1,200 at 10% p.a.
subjects At the end of 1st year the amount = `1,320
= (40 + 6)% of X = 46% of X (1) 30
Withdrawal  1320  24 = `420
Therefore, students passing in two subjects 100
= 54% of X Amount at the end of second year
  The students passing in remaining subject = 900 × 1.1 = `990
81 30
= 75% (54% of X) = % of X Withdrawal =  990  93 = `390
2 100
Hence students failing in remaining subject ∴ Amount at the end of 3 years
= 600 × 1.1 = `660
 81 27
=  54   % of X = % of X (2) 4.
(c) When expressed as a fraction, savings last month
 2 2
2
Therefore, total number of students failing in all the were of the salary. Reduced this month by 50%
15
subjects equation (1) + (2) 1
savings are of last month’s salary, which is
15
 27  119 `667. New salary is `11,500. Expenditure this month is
=  46   % X = % of X
 2 2 11,500 – `667 = `10,833.

M05_KHAT6981_C05.indd 27 6/16/2015 3:48:30 PM


5.28 I Chapter 5

5.
(b) Let the amount of iron be x kg 4 9 7K
K –
No. of girls passed = K =
 x   x – 25  5 20 20
∴ 1.2  = ⇒ 3.6x = 5x – 125
 500   300  No. of girls who got 1st division
or 1.4x = 125 2 9 11K
= K − K =
5 50 50
∴ x = 89.28 kg
2K 7 K K
a b a 2  b2 No. of girls failed = − = .
6.
(c) Here the difference = – = 5 20 20
b a ab
9.
(a) Number of packs bought by customer = 12
 a 2  b2 
 ab  Number of gift soaps received
b a
Now, < by   100 %  12 
a b  a  = Integer part of   = 2
 b   5
Total number of soaps received by the customer
a 2  b2  b2 
= × 100 = 100 1  2  = (4 × 12) + 2 = 50
a2  a 
Total money paid by the customer = 12 × 3 × s, where
n s is the listed sale price of each soap
7.
(a) nth member contributed `(10 × 2 – 5)
⇒ 1st member contributed `15 For 50 soaps, the listed sale price = 50s
2nd member contributed `35 Actual amount paid is 36s
3rd member contributed `75 Hence, discount is 14s
4th member contributed `155 14 s 
  Discount percentage =   100 = 28%
And so on.  50 s 
Since 4th member gets `62 as his share in the profit,
therefore we conlcude that 40% profit is earned by ptd ptd 5
10.
(a) = = d.
each member.  p 6t 2 p 4
(t + 20% of t )  p −  ×
\ Total profit earned  3 5 3
= 40% of [15 + 35 + 75 + ... + upto 100 terms] 11.
(d) Let total adult population of town = T
= 2 [3 + 7 + 15 + 31 + ... + upto 100 terms] ⇒ Adult population that is male = 0.6T
= 2 [(4 + 8 + 16 + 32 + ... + upto 100 terms] Adult population that is female = 0.4T
= 8 [(1 + 2 + 4 + 8 + ... + upto 100 terms) – 25] a% of adult male population and b% of adult
 2100 − 1   female population is educated.
= 8   − 25
 2 − 1 Given, educated adult males + uneducated adult
  
females = Sum of uneducated adult males
= 8 (2100 – 1 – 25)
+ Educated adult females
= 8 (2100 – 26).
a (100  b)
8.
(c) Total no. of students = K, say ⇒  0.6 T   0.4T 
100 100
3
\ No. of boys = K 0.6T (100  a ) 0.4 Tb
5 = 
3 9 100 100
No. of boys who passed = 75% of K = K
5 20 ⇒ 3a – 2b = 50
No. of boys who got 1st Division
Only choice (d) satisfies the above equation.
9 9
= 40% of K = K
20 50 12. (c) 120 coats for full length. 500 shorter length coats are
4 removed.
No. of students passed = K
5 \ Percentage of full length coats out of the remaining
No. of students who get 1st division 300 coats
2 120
= K = × 100 = 40.
5 300

Chapter 05.indd 28 6/4/2015 2:47:39 PM


5.29
Percentage

13. (d) 5% of A = 15% of B ⇒ 5A = 15B ⇒ A = 3B In order that after increase in price, the total expenditure
10% of B = 20% of C ⇒ 10B = 20C ⇒ B = 2C of the family remains unaffected, therefore
If C = 2000, then B = 4000 125x = 10000 ⇒ x = 80
\ A = 12000 \ Reduction in consumption of units
= 20, i.e., 20%
Hence, the total income of A, B and C = 18000.
6% of 1000000
14.
(b) Expenditure by Ajay on batteries 22.
(c) = `5000/month
12
150 Incidental Expenses + Taxes = `3000/month
= 80% of 150 +
2 Total Rent per month = `8000.
= 120 + 75 = `195.
23.
(b) 100 + 20% = 120
15.
(c) 5% increase in X ⇒ 3% increase in Y 120 – 10% = 108
5% increase in Y ⇒ 2.5% increase in Z
\ Gain % = 8.
\ 3% increase in Y ⇒ 1.5% increase in Z
\ 5% increase in X 5Y
24.
(a) X = Y + 25% of Y =
⇒ 1.5% increase in Z i.e., 5X = 1.5Z 4
⇒ 25X = 7.5Z. 6Z
Y = Z + 20% of Z =
5
16.
(a) 30% of increase in Y
X Y Z
⇒ 15% of increase in Z i.e., 32.25% increase in Z2. \ 4X = 5Y = 6Z ⇒ = =
15 12 10
17. (a) X is increased from 10 to 15, i.e., there is 50% increase i.e., X, Y, Z share their profit in the ratio 15:12:10.
in X.
10
\ There must be 15% increase in Z, i.e., Z must \ Z’s share = × 740 = `200.
increase from 30 to 34.5, i.e., 35 approx. 37

18. (c) Percentage of those who were not certain 25.


(d) Cannot be determined. We do not know whether there
are some male employees who have exactly `8,000
= 100 – (20 + 60) = 20%
per month as their salary or not.
Now, let the number of persons involved in the survey
be x 26.
(a) Let the quantity of paint purchased be x kg
Then x × 60% – x × 20% = 720 then  (x – 15% of x) = 25
⇒ x × 40% = 720 ⇒  x = 29.41 or 30 kg
∴ x = 1,800 So, he must purchase 15 cans
19. (b) Time × Rate = Total charges Total cost = (16 × 15) = `240
1 × 1 = 1 27.
(d) Since we do not have sufficient data. Further any
x × 1.25 = 1.1 value is possible as the required income tax.
1.1
∴ x =  × 100 = 88%
1.25  P 
28.
(d) Required % =  × 100  %
Thus, decrease in time = 12%  100 + P 

480 x  20 
20.
(d) Total land of Sukhiya =  = 800 x  =  × 100  % = 16.66%
0.6  120 
∴ Cultivated land of village = 384000 x
800 x 29.
(c) Let the number of fish be x then,
∴ Required percentage = × 100
384000 50 × 100 48 × 100
=
             = 0.20833 x ( x − 50)
21. (c) Let the price of the cooking oil
50 48
= `100/unit ⇒ =
x x − 50
Let the family consumes 100 units
\ Total expenditure on this account = `10000 ⇒ 50x – 2500 = 48x
If the price becomes `125 and the family consumes, ⇒ 50x – 48x = 2500
say x units, then the total expenditure will become ⇒ 2x = 2500
`125x. \ x = 1250

Chapter 05.indd 29 6/4/2015 2:47:39 PM


5.30 Chapter 5

30.
(d) Number of researchers prefer team A 34.
(d) We know that,
70 Exactly 1 + 2 Exactly 2 + 3 Exactly 3
50 × = 35
100 = 61 + 46 + 29
40 = 136% (1)
Researchers assigned to A = 50 × = 20 Exactly 1 + Exactly 2 + Exactly 3
100
Difference = 15 = 100 – 3 = 97% (2)
30 Adding Eqs. (1) and (2),
Prefer team B = 50 × = 15
100 Exactly 2 + 2 Exactly 3 = 39
60 Exactly 2 = 25
Assigned to B = 50 × = 30
100 ⇒ 25 + 2 Exactly 3 = 39
Difference = 15 Exactly 3 = 7%
Hence, least possible number of researchers who will So, 7% of people watched all the movies.
not be assigned to the team they prefer = 15 + 15 = 30

35 35.
(a) Total growth in sales
31.
(c) Total migrants population = × 728400 = 254940
100 = (193.8 × 7.25%) + (79.3 × 8.2%) + (57.5 × 7.19%)
= 14.05 + 6.5 + 4.11 = 24.66
65
Local population = × 728400 = 473460 Total sales from all the zones = 330.6
100
\ Overall percentage growth
20
Total rural migrants = × 254940 = 50988 24.66
100 =  100 
7.46%
330.6
80
Total urban migrants = × 254940 = 203952 36.
(d) Bonus share of Sun Systems Limited received by
100
3
Raveendra in 2007 = 650 × = 150 and in 2008
\ Population of females 13
1
it is 800  = 400
48 30 40 2
= × 473460 + × 50988 + × 203952
100 100 100 Hence, in 2009, when bonus share were announced he
has 550 shares additionally.
= 227260.8 + 15296.4 + 81580.8 = 324138
1200  12.5
32.
(b) Let company received x ball bearing in first shipment \ Percentage =  100 = 23%
and 2x ball bearing in 2nd shipment. 650

\ 1% of x + 4.5% of 2x = 100 37.


(b) Let earlier `100 was cost of 1 kg cement.
Now it is `70.
x 9x
⇒ + = 100
100 100 Now `70 for 1 kg cement.
⇒ x = 1000 1
\ `1 for kg cement.
70
33.
(b) Let there are 100 employees in Sun Metals. 100
\ General graduates = 40 and engineers = 60 \ `100 for kg cement.
70
No. of engineers having salary more than 5 lakhs/year 100 30
\ Increased cement = 1 =
 75  70 70
=  60  
45
 100  30 6
No. of employees having salary more than 5 lakhs/year \ Percentage increase =  100 = 42 %
70 7
= 50
\ No of general graduates having salary less than 5 2
38.
(d) Number of girls = × 1000 = 400
lakhs/year = 35. 5
This is 7/8th of the number of general graduates. Boys = 600

Chapter 05.indd 30 6/4/2015 2:47:40 PM


5.31
Percentage I
According to question, production cost `100.
1 1 Since, profit is 20% Hence, selling price = `120.
 400   600 = 230 Now, new cost price of component A = `39
5 4
\ 12 year above = 1000 – 230 = 770 New cost price of component B = `61
New production cost (other expenses do not change)
770
\ Percentage = × 100 = 77% = (39 + 61 + 20) = `120
1000
Since, new SP = 120 × 1.1 = 132
39.
(b) Smaller burner burns for 24 more hours. 132 − 120
∴ New profit% = × 100 = 10.
24 120
\ Percentage difference = × 100 = 30%
80 46. (b) New cost of component A = 30 × 1.2 = `36
40. (d) The value of the house after 3 years will be `. 50000 New cost of component B = 50 × 0.88 = `44
At present, its worth is, New production cost = `(36 + 44 + 20) = `100
New selling price is same.
50000 50000
= Hence, profit = 120 − 10 = 20 or 20%
 10  (0.9)3
1 −  4
 100  47. (b) Amount of salt in given solution = 1000 × = 40 g
100
≈ ` 68587.
∴ Minimum amount of water required after which
41. (a) The % of respondents who watch all 3 channels sedimentation starts
[30 + 20 + 85 − 20 − (100 − 5)] =
40
× 100 = 160 g
= = 10
2 25
∴ Minimum 840 g needs to evaporated.
42. (d) Those watching L and B only ( = 16 − 10) = 6, while
Time required to evaporate 840 g of water
thosewatching A and B only (= 20 −10) = 10.
840
Those watching L and A only (20 − 6 −10) = 4. = = 30 hrs
28
Those watching L 20 − (6 + 10 +4) = 0, which is not
among the choices given. ∴ After 31 h the given solution starts sedimenting.
43. (d) As the length of hypotenuse is decreased by 35%, the 48.
(a) Let the total production of hematite be x kg
total reduction in the area of the triangle is Amount of ore gets wasted = 20% of x
 Remaining amount = 80% of x
n2 
 2n −  % (here, n = 35%) 25 80
 100  Amount of pure iron obtained = × ×x
100 100
 (35) 2 
=  2 × 35 − % 1
= x kg
 100  5
= (70 − 12.25)% = 57.75% We are given,

Amount of pure iron obtained in a year = 80000 kg
Therefore, the new triangle will have area equal to
(100 – 57.75)% of the area of the bigger triangle, i.e., 1
∴ x = 80000
42.25% of 34 5
42.25 × 34 or x = 400000kg
= = 14.365 cm 2
100 49. (b) Let the actual height be x ft.
44. (c) Let x kg of dry grapes be obtained Then, height given in medical certificate = 125x ft
Then, solid part in fresh grapes = solid part in dry 1.25 x − x
Therefore, per cent correction = × 100
grapes 1.25
i.e., 0.10 × 20 = 0.8 × x ⇒ x = 2.5 kg. 0.25
= × 100 = 20%
45. (a) Let the cost of component A and B be `30 and 50, 1.25
respectively. 50. (a) Total tractor population in state = 294000
Then cost of production = `(30 + 50 + 20), where Tractor manufactured by Mahindra and Mahindra
`20 contributes to the other expenses, assuming total = 150000

Chapter 05.indd 31 6/4/2015 2:47:42 PM


5.32 Chapter 5

Out of every 1000 Mahindra tractor 98 are red. ∴ Percentage of non-Mahindra tractor that are red
∴  Number of mahindra tractors that are red
882
=
98
× 150000 = 14700 = × 100%
1000 (294000 − 150000)
53 882
Total number of red tractors = 294000 × = 15582 = × 100%
100 144000
Number of non-Mahindra tractors that are red
882
= %
= 15582 − 14700 144
= 882 = 6.125%

Chapter 05.indd 32 6/4/2015 2:47:42 PM


CHAPTER

Average 6
INTRODUCTION The average or mean or arithmetic mean of a number of
Whenever we are asked the marks scored by us in any quantities of the same kind is equal to their sum divided by
examination, we usually tell the marks in percentage, the number of those quantities. For example, the average of
taking the percentage of total marks of all subjects. This 3, 9, 11, 15, 18, 19 and 23 is
percentage is called average percentage. Also, in a class, 3  9  11  15  18  19  23 98
= = 14.
if there are 100 students, instead of knowing the age of 7 7
individual student, we usually talk about average age.

soMe Basic forMulae

1000 + 1500 + 2700


Sum of quantities Average price of 1 toy =
1. Average = 20
Number of quantities
5200
= = `260
2. Sum of quantities = Average × Number of quantities 20
Sum of quantities Illustration 2 The average marks obtained by 200 students
3. Number of quantities = in a certain examination is 45. Find the total marks.
Average
Solution: Total marks
= Average marks × Number of students
Illustration 1 A man purchased 5 toys at the rate of `200 = 200 × 45 = 900
each, 6 toys at the rate of `250 each and 9 toys at the rate of Illustration 3 Total temperature for the month of September
`300 each. Calculate the average cost of one toy. is 840°C. If the average temperature of that month is 28°C,
Solution: Price of 5 toys = 200 × 5 = `1000 find of how many days is the month of September.
Price of 6 toys = 250 × 6 = `1500 Solution: Number of days in the month of September
Price of 9 toys = 300 × 9 = `2700 Total temperature 840
= = = 30 days
Total number of toys = 5 + 6 + 9 = 20 Average temperature 28

soMe useful sHort-cut MetHoDs

1. Average of two or more groups taken together respectively, the combined average (average of
(a) If the number of quantities in two groups all of them put together) is
be n and n and their average is x and y, n1 x  n2 y
1 2
n1  n2

Chapter 06.indd 1 6/4/2015 4:03:00 PM


6.2 Chapter 6

Explanation Illustration 5 Average salary of all the 50 employees


No. of quantities in first group = n1 including 5 officers of a company is `850. If the average
Their average = x salary of the officers is `2500, find the average salary of the
\ Sum = n1 × x remaining staff of the company.
No. of quantities in second group = n2 Solution: Here n1 = 50, n2 = 5, x = 850 and y = 2500
Their average = y \ Average salary of the remaining staff
\ Sum = n2 × y n x  n2 y 50  850  5  2500
= 1 =
No. of quantities in the combined group = n1 + n2 n1  n2 50  5
Total sum (sum of quantities of first group and second 42500  12500 30000
group) = =
45 45
= n1x + n2y. = `667 (approx.)
\ Average of the two groups
n1 x  n2 y
= 2. If x is the average of x1, x2, ..., xn, then
n1  n2
(a) The average of x1 + a, x2 + a, ..., xn + a is x
+ a.
(b) If the average of n1 quantities is x and the
average of n2 quantities out of them is y, (b) The average of x1 – a, x2 – a, ... xn – a is x – a.
the average of remaining group (rest of the (c) The average of ax1, ax2, ..., axn is a x ,
quantities) is provided a ≠ 0.
n1 x  n2 y x x x x
. (d) The average of 1 , 2 , ..., n is , provided
n1  n2 a ≠ 0. a a a a

Explanation Illustration 6 The average value of six numbers 7, 12, 17,


No. of quantities = n1 24, 26 and 28 is 19. If 8 is added to each number, what will
Their average = x be the new average?
\ Sum = n1 x Solution: The new average = x + a
No. of quantities taken out = n2 = 19 + 8 = 27
Their average = y Illustration 7 The average of x numbers is 5x. If x – 2 is
\ Sum = n2y subtracted from each given number, what will be the new
Sum of remaining quantities = n1x – n2y average?
No. of remaining quantities = n1 – n2 Solution: The new average x = – a
n x  n2 y = 5x – (x – 2) = 4x + 2
\ Average of remaining group = 1
n1  n2 Illustration 8 The average of 8 numbers is 21. If each of
the numbers is multiplied by 8, find the average of a new
Illustration 4 The average weight of 24 students of section
set of numbers.
A of a class is 58 kg whereas the average weight of 26
students of section B of the same class is 60.5 kg. Find the Solution: The average of a new set of numbers
average weight of all the 50 students of the class. = a x = 8 × 21 = 168
Solution: Here n1 = 24, n2 = 26, x = 58 and y = 60.5.
3. The average of n quantities is equal to x. If one of
\ Average weight of all the 50 students
the given quantities whose value is p, is replaced
n x  n2 y by a new quantity having value q, the average
= 1
n1  n2 becomes y, then q = p + n(y – x)
24  58  26  60.5
=
24  26 Illustration 9 The average weight of 25 persons is increased
1392  1573 2965 by 2 kg when one of them whose weight is 60 kg is replaced
= = = 59.3 kg
50 50 by a new person. What is the weight of the new person?

Chapter 06.indd 2 6/4/2015 4:03:01 PM


Average 6.3

Solution: The weight of the new person Illustration 13 What is the average of squares of the
= p + n(y – x) natural numbers from 1 to 41?
= 60 + 25(2) = 110 kg Solution: The required average
(n  1)(2n  1) (41  1)(2  41  1) 42  83
4. (a) The average of n quantities is equal to x. When = = =
a quantity is removed, the average becomes 6 6 6
y. The value of the removed quantity is 3486
= = 581.
n(x – y) + y. 6
(b) The average of n quantities is equal to y. When Illustration 14 Find the average of cubes of natural
a quantity is added, the average becomes y. numbers from 1 to 27.
The value of the new quantity is n(y – x) + y. Solution: The required average
n(n  1) 2 27  (27  1) 2 27  28  28
Illustration 10 The average age of 24 students and the lass = = =
4 4 4
teacher is 16 years. If the class teacher’s age is excluded,
the average age reduces by 1 year. What is the age of the 21168
= = 5292
class teacher? 4
Solution: The age of class teacher Illustration 15 What is the average of odd numbers from
= n(x – y) + y 1 to 40?
= 25(16 – 15) + 15 Solution: The required average
= 40 years last odd number + 1 39 + 1
= = = 20
Illustration 11 The average age of 30 children in a class 2 2
is 9 years. If the teacher’s age be included, the average age Illustration 16 What is the average of even numbers from
becomes 10 years. Find the teacher’s age. 1 to 81?
Solution: The teacher’s age Solution: The required average
= n(y – x) + y last even number + 2 80 + 2
= 30(10 – 9) + 10 = = = 41
2 2
= 40 years
6. (a) If n is odd: The average of n consecutive
5. (a) The average of first n natural numbers is numbers, consecutive even numbers or
n 1 consecutive odd numbers is always the
.
2 middle number.
(b) The average of square of natural numbers till (b) If n is even: The average of n consecutive
(n  1)(2n  1) numbers, consecutive even numbers or
n is . consecutive odd numbers is always the
6
(c) The average of cubes of natural numbers till average of the middle two numbers.
n(n  1) 2 (c) The average of first n consecutive even
n is . numbers is (n + 1).
4
(d) The average of first n consecutive odd
(d) The average of odd numbers from 1 to n is
numbers is n.
last odd number + 1
. (e) The average of squares of first n consecutive
2
(e) The average of even numbers from 1 to n is 2(n +1)(2n + 1)
even numbers is .
3
last even number + 2
. (f) The average of squares of consecutive even
2
(n + 1)(n + 2)
numbers till n is .
Illustration 12 Find the average of first 81 natural numbers.. 3
(g) The average of squares of consecutive odd
Solution: The required average
n(n + 2)
n  1 81  1 numbers till n is .
= = = 41. 3
2 2

Chapter 06.indd 3 6/4/2015 4:03:02 PM


6.4 Chapter 6

Solution: The required difference


(h) If the average of n consecutive numbers is
= 2(n – 1) = 2(6 – 1) = 10
m, then the difference between the smallest
and the largest number is 2(n – 1).
7. Geometric mean or geometric average
Geometric mean of x1, x2, ..., xn is denoted by
Illustration 17 Find the average of 7 consecutive numbers
3, 4, 5, 6, 7, 8, 9. G.M. = n x1  x2  ...  xn
Solution: The required average = middle number = 6 Geometric mean is useful in calculating averages
Illustration 18 Find the average of consecutive odd of ratios such as average population growth rate,
numbers 21, 23, 25, 27, 29, 31, 33, 35. average percentage increase and so on.
Solution: The required average
Illustration 25 The production of a company for three
= average of middle two numbers
successive years has increased by 10%, 20% and 40%,
= average of 27 and 29 respectively. What is the average annual increase of
27  29 production?
= = 28
2 Solution: Geometric mean of x, y and z = (x × y × z)1/3
Illustration 19 Find the average of first 31 consecutive \ Average increase = (10 × 20 × 40)1/3% = 20%
even numbers.
Solution: The required average = (n + 1) = 31 + 1 = 32. Illustration 26 The population of a city in two successive
years increases at the rates of 16% and 4%, respectively.
Illustration 20 Find the average of first 50 consecutive Find the average increase of two years.
odd numbers. Solution: In case of population increase, the geometric
Solution: The required average = n = 50. mean is required.
Illustration 21 Find the average of squares of first 19 \ Geometric mean of 16% and 4% is
consecutive even numbers. = (16 × 4)1/2 %, i.e., 8%
Solution: The required average
8. Harmonic mean or harmonic average
2(n  1)(2n  1) 2(19  1)(2  19  1)
= = Harmonic means of x1, x2, ..., xn is denoted by
3 3
2  20  39 1560 1
= = = 520. H.M. =
3 3 1 1 1 1
   ...  
n  x1 x2 xn 
Illustration 22 Find the average of squares of consecutive
even numbers from 1 to 25. Harmonic mean is useful for finding out average
Solution: The required average speed of a vehicle, average production per day
(n  1)(n  2) (25  1)(25  2) and so on.
= =
3 3
Illustration 27 A man runs 1 Km at 15 Km per hour and
26  27 702 another 1 Km he walks at 5 Km per hour. Find his average
= =
3 3 speed for the whole distance in covering 2 Km.
= 234
Solution: When the distance is constant and the speed
Illustration 23 Find the average of squares of consecutive varies, harmonic mean is used. Harmonic mean of x and y
odd numbers from 1 to 31. 2xy
2
Solution: The required average is or .
1 1 x y
n(n  2) 31  (31  2) 31  33 
= = = = 341 x y
3 3 3
\ Average speed for the whole distance
Illustration 24 If the average of 6 consecutive numbers
is 48, what is the difference between the smallest and the 2  15  5
= = 7.5 Km/h
largest number? 15  5

Chapter 06.indd 4 6/4/2015 4:03:03 PM


Average 6.5

Illustration 31 A person covers 9 Km at a speed of


9. If a certain distance is covered at a speed of x 3 Km/h, 25 Km at a speed of 5 Km/h and 30 Km at a speed
Km/h and the same distance is covered at a speed of 10 Km/h. Find the average speed for the entire journey.
of y Km/h, the average speed during the entire
 A BC 
journey is Solution: The average speed =  A B C 
 2xy     
 x  y  Km/h  x y z
 9  25  30 
Illustration 28 If half of the journey is travelled at a speed =  9 25 30 
   
of 15 Km/h and the next half at a speed of 12 Km/h, find the 3 5 10 
average speed during the entire journey. 64 9
Solution: The average speed = = 5 Km/h.
11 11
 2xy   2  15  12 
=  =   12. If a person covers Ath part of the distance at x
 x  y   15  12 
Km/h, Bth part of the distance at y Km/h and the
360 1 remaining Cth part at z Km/h, then the average
= = 13 Km/h
27 3 speed during the entire journey is
Illustration 29 A man goes to a certain place at a speed of 30  1 
Km/h and returns to the original place at a speed of 20 Km/h,  A B C  Km/h.
find the average speed during up and down journey.    
 x y z
Solution: The average speed
 2xy   2  30  20  1200
=  =   = = 24 Km/h Illustration 32 A person covers the first one-forth of the
 x  y   30  20  50 distance at 8 Km/h, the next three-fifths at 6 Km/h and the
10. If a person or a motor car covers three equal remaining distance at 15 Km/h. Find the average speed
distances at the speed of x Km/h, y Km/h and z during the entire journey.
Km/h, respectively, then for the entire journey Solution: The average speed
average speed of the person or motor car is 1  1 
 3xzy  = =  1/4 3/5 3/20 
 A B C
 xy  yz  zx  Km/h.  x  y  z     
8 6 15 

Illustration 30 A train covers the first 160 Kms at a speed  1 3  1 3 3 


 Here, A 4 , B  5 and C = 1   4  5   20 
of 120 Km/h, another 160 Kms at 140 Km/h and the last  
160 Kms at 80 Km/h. Find the average speed of the train for
1 3200 9
the entire journey. = 1 1 1 = 452 = 7 113 Km/h
Solution: Average speed  
32 10 100
3xyz 3  120  140  80
= = Illustration 33 A train covers 50% of the journey at
xy  yz  zx 120  140  140  80  80  120
30 Km/h, 25% of the journey at 25 Km/h and the remaining
360  140  80
= at 20 Km/h. Find the average speed of the train during the
16800  11200  9600 entire journey.
4032000 11
= = 107 Km/h Solution: The average speed
37600 47
 100   100 
11. If a person covers A Km at a speed of x Km/h, B =  A B C  =  50 25 25 
Km at a speed of y Km/h and C Km at a speed of z        
 x y z 30 25 20 
Km/h, the average speed during the entire journey is
 A BC  [Here A = 50, B = 25 and C = 25]
 A B C  Km/h. 100 1200 25
    = = = 25 Km/h
 x y z
47/12 47 47

Chapter 06.indd 5 6/4/2015 4:03:04 PM


6.6 Chapter 6

Practice Exercises

Difficulty Level-1
(Based on Memory)

1. The average of 11 numbers is 10.9. If the average of the 8. The average of three numbers is 135. The largest number
first six numbers is 10.5 and that of the last six numbers is is 180 and the difference of the other numbers is 25. The
11.4, then the middle number is: smallest number is:
(a) 11.5 (b) 11.4 (a) 130 (b) 125
(c) 11.3 (d) 11.0 (c) 120 (d) 100
[Based on MAT, 2003] [Based on IIFT, 2003]
2. A car owner buys petrol at `7.50, `8.00 and `8.50 per 9. The ratio of the arithmetic mean of two numbers to one of
litre for three successive years. What approximately is the the numbers is 3:5. What is the ratio of the smaller number
average cost per litre of petrol if he spends `4000 each to the larger one?
year? (a) 1:5 (b) 1:4
(a) `8 (b) `9 (c) 1:3 (d) 1:2
(c) `7.98 (d) `8.50 [Based on IIFT, 2003]
[Based on MAT, 2001] 10. A person travels from X to Y at a speed of 40 Km/h
3. There are 197 boys and 591 girls in a college. If the and returns by increasing his speed by 50% What is his
average weight of boys is 63 kg and average weight of average speed for both the trips?
girls is 51 kg, then what is the average weight of the (a) 36 Km/h (b) 45 Km/h
students in the college? (c) 48 Km/h (d) 50 Km/h
(a) 56 kg (b) 57 kg [Based on IIFT, 2003]
(c) 54 kg (d) 60 kg
11.
The average of 5 consecutive numbers is n. If the next two
[Based on IIT Joint Man. Ent. Test, 2004]
numbers are also included,. the average will:
4. A student on his birthday distributed on an average 5 (a) Increase by 1 (b) Remain the same
chocolates per student. If on the arrival of the teacher and the (c) Increase by 1.4 (d) Increase by 2
headmaster to whom the student gives 10 and 15 chocolates
[Based on MAT, 2005]
respectively, the average chocolate distributed per head
increases to 5.5, then what is the strength of the class? 12. For 10 hrs, a train travels at a constant speed of 20 miles
(a) 28 (b) 30 per hour and during the next 15 hrs, it travels 240 miles.
(c) 32 (d) None of these What is the average speed of the train for the whole
[Based on IIT Joint Man. Ent. Test, 2004]
journey?
(a) 17.6 miles/hr (b) 20.8 miles/hr
5. If the mean of a, b, c is M and ab + bc + ca = 0, then the
(c) 130 miles/hr (d) 176 miles/hr
mean of a2, b2, c2 is:
[Based on IMT Ghaziabad, 2002]
(a) M2 (b) 3M2
13. Average of 10 positive numbers is X . If each number
(c) 6M2 (d) 9M2
[Based on IITTM, Gwalior, 2003]
increases by 10%, then X :
(a) Remains unchanged
6.
A is the set of first ten consecutive natural numbers.
(b) Is increased by 10%
Find the number of ways in which a subset B can be
formed out of set A, such that the sum of all the elements (c) May decrease
in B is odd. (d) May either increase or decrease
(a) 761 (b) 763 [Based on IMT Ghaziabad, 2002]
(c) 765 (d) 767 14.
On an 800 miles trip, car W travelled half the distance at
7. Three years ago, the average age of A and B was 18 years. 80 miles per hour and the other half at 100 miles per hour.
With C joining them, the average age becomes 22 years. What was the average speed of the car?
How old is C now? (a) 18.00 (b) 180.00
(a) 24 years (b) 27 years 8
(c) 90.00 (d) 88
(c) 28 years (d) 30 years 9
[Based on FMS (Delhi), 2003] [Based on Narsee Monjee Inst. of Man. Studies, 2002]

Chapter 06.indd 6 6/4/2015 4:03:04 PM


6.7
Average

15. Having scored 98 runs in the 19th innings, a cricketer (a) 8,000 (b) 18,000
increases his average score by 4. What will be his average (c) 13,500 (d) 9,000
score after the 19th innings?
23.
While calculating the average of a batsman as 36 in 100
(a) 28 (b) 26 matches that he played, one of the score 90 was incorrectly
(c) 24 (d) 22 noted as 40. The percentage error is:
[Based on I.P. Univ., 2002] (a) 0.6% (b) 1.36%
16. Angad was conducting an experiment in which the average (c) 1.34% (d) 1.21%
of 11 observations came to be 90, while the average of [Based on MAT (Sept), 2010]
first five observations was 87, and that of the last five was
84. What was the measure of the 6th observation? 24.
The average sale of a car dealership was 15 cars per week.
After a promotional scheme, the average sale increased to
(a) 165 (b) 150 21 cars per week. The percentage increase in the sale of
(c) 145 (d) 135 cars was:
[Based on I.P. Univ., 2002] (a) 40% (b) 140%
17. A student who scored 30% marks in the first paper of 6
Physics out of 180 marks, has to get an overall score of (c) 42 % (d) 39.33%
7
at least 50% in two papers, the second paper carrying 150
[Based on MAT (Feb), 2010]
marks. What percentage of marks should he score in the
second paper to get the overall average score? 25.
In a class with a certain number of students, if one
(a) 80% (b) 76% new student weighing 50 kg is added, then the average
weight of the class increased by 1 kg. If one more student
(c) 74% (d) 70%
weighing 50 kg is added, then the average weight of the
[Based on I.P. Univ., 2002] class increases by 1.5 kg over the original average. What
18.
The captain of a cricket team of 11 players is 25 years is the original weight (in kg) of the class?
old and the wicket keeper is 3 years elder to the captain. (a) 46 (b) 42
If the ages of these two are excluded, the average age of
(c) 27 (d) 47
the remaining players is 1 year less than the average age
of the whole team. What is the average age of the whole [Based on MAT (Sept), 2009]
team? 26. The average marks of a student in 8 subjects is 87. Of
(a) 21.5 years (b) 22 years these, the highest marks is 2 more than the one next in
value. If these two subjects are eliminated, the average
(c) 22.5 years (d) 32 years
marks of the remaining subjects is 85. What is the highest
19. A batsman has a certain average of runs for 12 innings. marks obtained by him?
In the 13th innings he scores 96 runs, thereby increasing (a) 94 (b) 91
his average by 5 runs. What is his average after the 13th
(c) 89 (d) 96
innings?
[Based on MAT (Sept), 2009]
(a) 64 (b) 48
27.
The average of 5 consecutive odd numbers A, B, C, D and
(c) 36 (d) 72
E is 41. What is the product of A and E?
[Based on I.P. Univ., 2002]
(a) 1677 (b) 1517
20.
Average of n numbers is 36 where n is a multiple of 4.
If three-fourth of the numbers are increased by 4 and the (c) 1665 (d) 1591
remaining decreased by 4, what is the new average? [Based on MAT (May), 2009]
(a) 40 (b) 37.5 28.
The average age of a woman and her daughter is 42 year.
(c) 38 (d) None of these The ratio of their ages is 2:1 respectively. What is the
daughter’s age?
21.
The average age of a group of 14 persons is 27 years and
9 months. Two persons, each 42 years old, left the group. (a) 28 years (b) 48 years
What will be the average age of the remaining persons in (c) 52 years (d) 32 years
the group? [Based on MAT (May), 2009]
(a) 26.875 years (b) 26.25 years 29.
The average weight of 29 students in a class is 48 kg. If
(c) 25.375 years (d) 25 years the weight of the teacher is included, the average weight
rises by 500 g. Find the weight of the teacher.
22.
The average income of A, B and C is `12,000 per month
and the average income of B, C and D is `15,000 per (a) 57 kg (b) 60 kg
month. If the average salary of D be twice that of A, then (c) 65 kg (d) 63 kg
the average salary of B and C is (in `) [Based on MAT (Feb), 2009]

Chapter 06.indd 7 6/4/2015 4:03:04 PM


6.8 Chapter 6

30.
The average of marks obtained by 120 candidates was 38.
The average age of a family of 6 members is 22 years.
35. If the average of the passed candidates was 39 and If the age of the youngest member be 7 years, what was
that of the failed candidates was 15, then the number of the average age of the family at the birth of the youngest
candidates who passed the examination was: members?
(a) 100 (b) 110 (a) 15 (b) 18
(c) 120 (d) 150 (c) 21 (d) 22
[Based on MAT (May), 2008]
39.
The average score of boys in an examination in a school
31.
The mean of five observations is 4 and their variance 5.2. is 71 and that of the girls is 73. The average score of the
If three of these observations are 1, 2 and 6, then the other school is 71.8. The ratio of the number of boys to that of
two are: the girls that appeared in the examination is:
(a) 2 and 9 (b) 3 and 8 (a) 1:2 (b) 3:2
(c) 4 and 7 (d) 5 and 6 (c) 2:3 (d) 4:2
[Based on MAT (Feb), 2008, (Sept), 2007] [Based on MAT (May), 2006]
32.
The average of 11 results in 50. If the average of first six 40.
The mean monthly salary paid to 75 workers in a factory
results is 49 and that of last six 52, the sixth result is: is `5680. The mean salary of 25 of them is `5400 and that
(a) 60 (b) 56 of 30 others is `5700. The mean salary of the remaining
(c) 64 (d) 70 workers is:
[Based on MAT (Feb), 2008] (a) `5000 (b) `7000
33.
The average temperature from Monday to Thursday is 48° (c) `6000 (d) `8000
and from Tuesday to Friday is 52°. If the temperature on [Based on MAT (May), 2006]
Monday is 42°, what was it on Friday? 41. The average monthly expenditure of a family was `2200
(a) 55° (b) 52° during the first 3 months; `2250 during the next 4 months
(c) 58° (d) 51° and `3120 during the last 5 months of a year. If the total
[Based on MAT (Feb), 2008] savings during the year were `1260, then the average
monthly income was:
34. An investor earns 3% returns on one-fourth of his capital,
5% on two-thirds and 11% on the remainder. What is the (a) `2605 (b) `2805
average rate of return he earns on his total capital? (c) `2705 (d) `2905
(a) 10% (b) 5% [Based on MAT (May), 2006]
(c) 5.5% (d) 10.5% 42.
Ram spends `3620 for buying pants at the rate of
[Based on MAT (Feb), 2008] `480 each and shirts at the rate of `130 each. What will
be the ratio of pants to shirts when maximum number of
35.
A batsman has a certain average of runs for 12 innings. In the
pants are to be bought?
13th inning, he scores 96 runs thereby increasing his average
by 5 runs. What is his average after the 13th innings? (a) 7:2 (b) 7:3
(a) 64 (b) 48 (c) 2:7 (d) None of these
(c) 36 (d) 72 [Based on MAT, 1999]
[Based on MAT (Feb), 2008] 43.
The average weight of 45 students in a class is 52 kg. 5 of
36.
There was one mess for 30 boarders in a certain hostel. If them whose average weight is 48 kg leave the class and
the number of boarders was increased by 10, the expenses other 5 students whose average weight is 54 kg join the
of the mess increased by `40 per month, while the average class. What is the new average weight (in kg) of the class?
expenditure per head diminished by `2. Find the original 2
(a) 52.6 (b) 52
monthly expenses. 3
(a) `390 (b) `360 1
(c) 52 (d) None of these
(c) `410 (d) `480 3
[Based on MAT, 1999]
[Based on MAT (Dec), 2007]
37.
The mean of 25 observations was found to be 78.4. But 44.
Of the three numbers, the first is twice the sec­ond and
later on it was found that 96 was misread as 69. The the second is twice the third. The average of these three
correct mean is: numbers is 21. Find the largest number.
(a) 79.48 (b) 76.54 (a) 36 (b) 38
(c) 81.32 (d) 78.4 (c) 47 (d) 48
[Based on MAT (Sept), 2007] [Based on MAT, 1999]

Chapter 06.indd 8 6/4/2015 4:03:04 PM


6.9
Average

45.
The average of three numbers is 135. The largest number 53.
If the algebraic sum of deviations of 20 observations
is 180 and the difference of the others is 25. The smallest measured from 23 is 70, mean of these observations
number is x: would be:
(a) 130 (b) 125 (a) 24 (b) 25
(c) 120 (d) 100 (c) 26 (d) None of these
[Based on MAT, 1999] [Based on SNAP, 2010]
46.
The batting average for 40 innings of a cricket player is 54.
A cricketer has completed 20 innings and his average
50 runs. His highest score exceeds his lowest score by 172 is 44.5 runs. How many runs must be make in his next
runs. If these two innings are excluded, the average of the innings so as to raise his average to 45?
remaining 38 innings is 48. His highest score was: (a) 45 (b) 60
(a) 172 (b) 173 (c) 40 (d) 55
(c) 174 (d) 176
[Based on MAT, 1999] 55.
The average of 6 numbers is 30. If the average of first four
is 25 and that of the last three is 35, the fourth number is:
47.
Nine persons went to a hotel for taking their meals. Eight (a) 35 (b) 30
of them spent `12 each over their meals and the ninth
(c) 25 (d) 20
spent `8 more than the average expenditure of all the nine.
[Based on FMS, 2006]
Total money spent by them was:
(a) `104 (b) `105 56. The average marks of the students in four sections A, B,
(c) `116 (d) `117 C, and D of a school is 60% The average marks of the
[Based on MAT, 1999] students of the A, B, C and D individually are 45%, 50%,
72%, and 80%, respectively. If the average marks of the
48.
Out of three numbers, the first is twice the sec­ond and is
students of sections A and B together is 48% and that of
half of the third. If the average of the three numbers is 56,
the students of B and C together is 60%, What is the ratio
the three numbers in order are:
of the number of students in sections A and D?
(a) 48, 96, 24 (b) 48, 24, 96
(a) 2:3 (b) 4:3
(c) 96, 24, 48 (d) 96, 48, 24
(c) 5:3 (d) 3:5
[Based on MAT, 2000]
[Based on MAT, 2012]
49.
If a, b, c, d, e are five consecutive odd numbers, their
average is: 57. The average monthly salary of employees, consisting
(a) 5 (a + 4) (b) a b c d e/5 of officers and workers, of an organization is `3000.
The average salary of an officer is `10000 while that
(c) 5 (a + b + c + d + e) (d) None of these
of a worker is `2000 per month. If there are a total
[Based on MAT, 2000]
400 employees in the organisation, find the number of
50.
The average weight of three men A, B and C is 84 kg. officers.
Another man D joins the group and the average now (a) 50 (b) 60
becomes 80 kg. If another man E, whose weight is 3 kg
(c) 80 (d) 40
more than that of D, replaces A, then the average weight
[Based on MAT, 2012]
of B, C, D and E becomes 79 kg. The weight of A is:
(a) 70 kg (b) 72 kg 58. The average marks of a student in 8 subjects is 87. Of
(c) 75 kg (d) 80 kg. these, the highest marks are 2 more than the next in value.
[Based on MAT, 2000] If these two subjects are eliminated, the average marks of
the remaining subjects is 85. What is the highest score?
51.
In a mathematics exam, a student scored 30% marks in the
first paper out of a total of 180. How much should he score (a) 91 (b) 94
in the second paper out of a total of 150, if he is to get an (c) 89 (d) 96
overall average of at least 50%? [Based on MAT, 2012]
(a) 74% (b) 76% 59. The average monthly salary of employees, consisting
(c) 70% (d) 80% of officers and workers, of an organization is `3000.
[Based on MAT, 2000] The average salary of an officer is `10000 while that
52.
The average of nine numbers is M and the average of three of a worker is `2000 per month. If there are a total 400
of these is P. If the average of remaining numbers is N, then: employees in the organisation, find the number of officers.
(a) M = N + P (b) 2M = N + P (a) 50 (b) 60
(c) 3M = 2N + P (d) 3M = 2 P + N (c) 80 (d) 40
[Based on SNAP, 2007] [Based on MAT, 2012]

Chapter 06.indd 9 6/4/2015 4:03:05 PM


6.10 Chapter 6

60. The average marks of a student in 8 subjects is 87. Of (a) 31 (b) 29


these, the highest marks are 2 more than the next in value. (c) 25 (d) 35
If these two subjects are eliminated, the average marks of [Based on SNAP, 2013]
the remaining subjects is 85. What is the highest score?
63. Average weight of 19 men is 74 kg, and the average
(a) 91 (b) 94 weight of 38 women is 63 kgs. What is the average weight
(c) 89 (d) 96 (rounded off to the nearest integer) of all the men and the
[Based on MAT, 2012] women together?
61. The average age of all the students of a class is 18 yr. The (a) 59 kg (b) 65 kg
average age of boys of the class is 20 yr and that of the (c) 69 kg (d) 67 kg
girls is 15 yr. If the number of girls in the class is 20, then [Based on SNAP, 2013]
what is the number of boys in the class? 64. The average marks in English subject of a class of 24
(a) 30 (b) 15 students is 56. If the marks of three students were misread
(c) 45 (d) 50 as 44, 45 and 61 of the actual marks 48, 59 and 67
[Based on MAT, 2013 ] respectively, then what would be the correct average?
62. Average of five numbers is 61. If the average of first and (a) 56.5 (b) 59
third number is 69 and the average of second and fourth (c) 57.5 (d) None of these
number is 69, what is the fifth number? [Based on SNAP, 2013]

Difficulty Level-2
(Based on Memory)

1.
An officer’s penson on retirement from service is equal to 5. In a one-day cricket match, Agarkar, Sehwag, Sachin,
half the average salary during last 36 months of his service. Dravid and Ganguly scored an average of 39 runs. Dravid
His salary from 1 January, 1954 is `380 per month with scored 7 more than Ganguly. Ganguly scored 9 fewer than
increment of `40 on 1 October, 1954, 1 October, 1955 and Agarkar. Sehwag scored as many as Dravid and Ganguly
1 October, 1956. If he retires on 1 January, 1957, what combined; and Sehwag and Sachin scored 110 runs
pension does he draw? between them. How many runs did Sachin score?
(a) `210 (b) `215 (a) 47 (b) 51
(c) `220 (d) `225 (c) 53 (d) None of the above
[Based on FMS (Delhi), 2004]
2.
A hiker walked for two days. On the second day, the hiker
walked 2 hrs longer and at an average speed of 1 Km per 6. There is a sequence of 11 consecutive odd numbers. If the
hour faster than he walked on the first day. If during the average of first 7 numbers is X, then find the average of all
two days he walked a total of 64 Km and spent a total of 18 the 11 integers.
hrs walking, what was his average speed on the first day? (a) X + 3 (b) X + 4
(a) 2 Km/h (b) 3 Km/h (c) X + 5 (d) X + 7
(c) 4 Km/h (d) 5 Km/h [Based on FMS (Delhi), 2004]

3.
In a class of 100 students the average marks obtained by 7. The average temperature of the town in the first four
a student in Maths is 44. If we remove the highest and days of a month was 58 degrees. The average for the
the lowest the average of the class becomes 43.92. A total second, third, fourth and fifth days was 60 degrees. If the
of three students get either highest or lowest. What is the temperatures of the first and fifth days were in the ratio
average of these three students? 7:8, then what is the temperature on the fifth day?
(a) 73.33 (b) 46.66 (a) 240 degrees (b) 232 degrees
(c) 59.99 (d) Cannot be determined (c) 64 degrees (d) None of these
[Based on Narsee Manjee Inst. of Man. Studies, 2003]
4.
Neeta’s attendance for first two semesters out of four
was 60% and 70%, respectively. What is the minimum 8. An aeroplane flies along the four sides of a square field at
attendance required in third semester so that her average the speeds of 200, 400, 600 and 800 Km/h. The average
attendance will be 80% throughout four semesters? speed of the plane around the field in Km/h is:
(Assume equal number of days among the four semesters) (a) 384 (b) 400
(a) 70% (b) 80% (c) 500 (d) 284
(c) 90% (d) None of these [Based on FMS (Delhi), 2003]

Chapter 06.indd 10 6/4/2015 4:03:05 PM


6.11
Average

9. The average of marks obtained by 120 candidates was 35. (a) 55 (b) 60
If the average of the passed candidates was 39 and that (c) 62 (d) Cannot be determined
of the failed candidates was 15, then the number of those
candidates, who passed the examination, was: 17.
The average age of Sachin and Ganguly is 35 years. If Kaif
(a) 100 (b) 110 replaces Sachin, the average age becomes 32 years and
if Kaif replaces Ganguly, then the average age becomes
(c) 120 (d) 150
38 years. If the average age of Dhoni and Irfan be half
[Based on FMS (Delhi), 2003] of the average age of Sachin, Ganguly and Kaif, then the
10.
A painter is paid x rupees for painting every 10 metres of average age of all the five people is:
a wall and y rupees for painting every extra metre. During (a) 28 years (b) 32 years
one week, he painted 10 metres on Monday, 13 metres on
(c) 25 years (d) None of these
Tuesday, 12 metres on Wednesday, 11 metres on Thursday
and 12 metres on Friday. What is his average daily earning 18.
The average of any 5 consecutive odd natural numbers is
in rupees for the five day week? k. If two more such numbers, just next to the previous 5
(a) x + (8/5) y (b) (5x + 9y)/5 numbers are added, the new average becomes:
(c) 10x + (8/5)y (d) 5x + 8y 2
(a) (k  1) (b) 2k – 3
[Based on SCMHRD Ent. Exam., 2003] 7
(c) 2k + 1 (d) k + 2
11.
There were 35 students in a hostel. If the number of students
increases by 7, the expenses of mess increase by `42 per 19.
The average age of A and B is 20 years. If C were to replace
day while the average expenditure per head diminshes by A, the average would be 19 and if C were to replace B, the
Re 1. Find the original expenditure of the mess. average would be 21. What are the ages of A, B and C?
(a) `320 (b) `420 (a) 22, 18, 20 (b) 18, 19, 20
(c) `160 (d) `158 (c) 22, 20, 17 (d) Cannot be determined
12.
Manmohan calculated the average of 10, ‘three digit
numbers’. But due to mistake he reversed the digits of 20.
Given the set of n numbers, n > 1, of which one is 1 –
a number and thus his average increased by 19.8. The (1/n), and all the others are 1. The arithmetic mean of the
difference between the unit digit and hundreds digit of n numbers is:
that number is: 1
(a) 1 (b) n 
(a) 8 (b) 4 n
(c) 2 (d) Cannot be determined 1 1
(c) n  2 (d) 1  2
13.
The average age of a husband and wife was 23 when n n
they were married 5 years ago. The average age of the [Based on FMS, 2010]
husband, the wife and a child who was born during the
21.
In a B-School, there are three levels of faculty positions,
interval, is 20 years now. How old is the child now?
i.e., Professor, Associate Professor and Assistant
(a) 9 months (b) 1 year Professor. It is found that the sum of the ages of all faculty
(c) 3 years (d) 4 years present is 2160, their average age is 36; the average age
14.
There are twice the number of two wheelers as there are of the Professor and Associate Professor is 39; of the
three wheelers and the number of 4 wheelers are equal 8
to the number of two wheelers. The average number of Associate Professor and Assistant Professor is 32 ; of
11
wheels per vehicle is:
2
(a) 3 (b) 4 the Professor and Assistant Professor is 36 . Had each
3
(c) 5 (d) None of these
Professor been 1 year older, each Associate Professor 6
15.
If p, q, r be three positive numbers such that p > q > r year older and each Assistant Professor 7 year older, then
when the smallest number is added to the difference of the their average age would increase by 5 years. What will be
rest two numbers, then the average of the resultant number the number of faculty at each level and their average ages?
and the original numbers except to the smallest number is
21 more than the average of all the three original numbers. (a) (16, 24, 20:45, 35, 30 year)
The value of (p – q) is: (b) (18, 24, 20:42, 38, 30 year)
(a) 7 (b) 14 (c) (16, 20, 24:50, 30, 30 year)
(c) 63 (d) 42 (d) None of these
[Based on IIFT, 2010]
16.
The average marks of a students in 10 papers are 80. If
the highest and the lowest scores are not considered, the 22. a, b, c, d and e be non-negative real numbers such that a +
average is 81. If the highest score is 92, find the lowest? b + c + d + e = 10. Let, X be the maximum of the numbers

Chapter 06.indd 11 6/4/2015 4:03:05 PM


6.12 Chapter 6

a + b, b + c, c + d and d + e. The least possible value of X of food in 2008, the percentage expenditure on food
lies in the ‘interval’: items, calculated on total annual gross salary in 2008, is
(a) [0, 2] (b) [2, 3] approximately:
(c) [3, 4] (d) [4, 5] (a) 43% (b) 45%
[Based on XAT, 2007] (c) 47% (d) 49%
23.
Professor Bee noticed something peculiar while entering [Based on JMET, 2009]
the quiz marks of his five students into a spreadsheet. The 25. Consider a sequence of seven consecutive integers. The
spreadsheet was programmed to calculate the average average of the first five integers is n. The average of all the
after each score was entered. Professor Bee entered the seven integers is:
marks in a random order and noticed that after each (a) n
mark was entered, the average was always an integer. In
(b) n + 1
ascending order, the marks of the students were 71, 76,
(c) k × n, where k is a function of n
80, 82 and 91. What were the fourth and fifth marks that
Professor Bee entered? 2
(d) n +  
(a) 71 and 82 (b) 71 and 76 7
(c) 71 and 80 (d) 76 and 80 [Based on CAT, 2000]
[Based on XAT, 2011] 26. Let x < 0 50, 0 < y < 1, z > 1. Given a set of numbers,
24. ‘Mr. Haque’s total annual gross salary, which was `10 the middle number, when they are arranged in ascending
lakhs per year in 2007, has been reduced by 10% in 2008. order, is called the median. So the median of the numbers
In 2007 his family expenditure for food items was 40% x, y and z would be:
of the total annual gross salary. The prices of average (a) Less than one (b) Between 0 and 1
food items have increased by 5% between 2007 and 2008. (c) Greater than one (d) Cannot say
Assuming that the family consumed the same amount [Based on CAT, 1993]

Answer Keys
Difficulty Level-1

1. (a) 2. (c) 3. (c) 4. (a) 5. (b) 6. (b) 7. (a) 8. (d) 9. (a) 10. (c) 11. (a) 12. (a) 13. (b)
14. (d ) 15. (b) 16. (d ) 17. (c) 18. (b) 19. (c) 20. (c) 21. (c) 22. (c) 23. (b) 24. (a) 25. (d ) 26. (a)
27. (a) 28. (a) 29. (d ) 30. (a) 31. (c) 32. (b) 33. (c) 34. (b) 35. (c) 36. (b) 37. (a) 38. (b) 39. (b)
40. (c) 41. (c) 42. (a) 43. (b) 44. (a) 45. (d ) 46. (c) 47. (d ) 48. (b) 49. (d ) 50. (c) 51. (a) 52. (c)
53. (d ) 54. (d ) 55. (c) 56. (b) 57. (a) 58. (b) 59. (a) 60. (b) 61. (a) 62. (b) 63. (d ) 64. (d )

Difficulty Level-2

1. (b) 2. (b) 3. (b) 4. (c) 5. (d ) 6. (b) 7. (c) 8. (a) 9. (a) 10. (a) 11. (b) 12. (c) 13. (d )
14. (a) 15. (c) 16. (b) 17. (a) 18. (d ) 19. (a) 20. (d ) 21. (a) 22. (a) 23. (c) 24. (c) 25. (b) 26. (b)

Chapter 06.indd 12 6/4/2015 4:03:05 PM


6.13
Average

Explanatory Answers

Difficulty Level-1

1.
(a) 6 × 10.5 + 6 × 11.4 – 11 × 10.9 8. (d) Let the three numbers be X, Y and Z.
= 63 + 68.4 – 119.9 X +Y +Z
\ = 135 ⇒ X + Y + Z = 405
= 131.4 – 119.9 = 11.5. 3
2.
(c) Let average cost of petrol per litre be `x. Let X be the largest number
12000 \ X = 180 ⇒ Y + Z = 225
\ x = = 7.98.
4000 4000 4000 Y – Z = 25
+ +
7.50 8 8.50 \ Y = 125,
197 × 63 + 591 × 51 197 × 63 + 591 × 51 Z = 100 (smallest number).
3.
(c) =
197 + 591 788 9.
(a) Let X and Y be the two numbers
63 3 63 + 153 216 X +Y
 = + × 51 = = 2 = 3
4 4 4 4 \
  = 54 kg. X 5
4.
(a) Suppose strength of the class = x X +Y 3
⇒ =
\ 5x + 10 + 15 = 5.5 (x + 2) 2 X 5
⇒  0.5x = 14 ⇒ x = 28. ⇒ 5X + 5Y = 6X ⇒ X = 5Y
X 1
5.
(b) a + b + c = 3M ⇒ = .
Y 5
(a + b + c)2 = a2 + b2 + c2 + 2 (ab + bc + ca)
= a2 + b2 + c2 10.
(c) Let the distance between X and Y be x Km.
2 2 2 2 2 x
⇒ a + b + c = (3M) = 9M \ Time taken from X to Y @ 40 Km/h = hrs
9M 2 40
⇒ Mean of a2, b2 and c2 = = 3M2. x
3 Time taken from Y to X @ 60 Km/h = hrs
60
6. (b) The given sets are formed by taking 1, 3, 5, ...... 2x
odd numbers starting from the second odd number \ Average speed = = 48 Km/h.
x x
onwards (i.e., 3 onwards), in the successive sets. So, +
40 60
there are (2n – 1) successive odd numbers in An
∴ Total number of odd numbers in all the sets from 11. (a) Let the consecutive numbers be x, x + 1, x + 2, x + 3, x
A1 to An (both inclusive) +4
5 x + 10
n n n Average = =x+2
=  (2i – 1) = 2 i – 1 5
5 x + 10 + x + 5 + x + 6
1 1 1 Average of 7 numbers =
7
= n(n + 1) –n = n2 7 x + 21
∴ The number of odd numbers in all the sets from = =x+3
7
A1 to A20 is = 400(starting from 3). Therefore, the \ The average increased by 1.
400th odd number (starting from 3) is 2(400) + 1 =
801. This is the last number in A20 20 × 10 + 240 200 + 240 440
Similarly, the last odd number in A19 is the 361st odd 12.
(a) = =
25 25 25
number starting from 3 = (361) (2) + 1 = 723 = 17.6 miles/hr.
∴ The first odd number in A20 is 725
∴ The average of the numbers of 13.
(b)
725  801
A20 = = 763. 14. (d) The distance of 800 miles was covered in 5 + 4 = 9
2 hrs.
7.
(a) (A – 3) + (B – 3) = 36 ⇒ A + B = 42 800 8
\ Average speed of W = = 88 miles per hour.
Also A + B + C = 66 ⇒ C = 24. 9 9

Chapter 06.indd 13 6/4/2015 4:03:06 PM


6.14 Chapter 6

15. (b) Let the average score of the 1st 18 innings be x 23.
(b) The total score by a batsman = 100 × 36
\ 18x + 98 = 19(x + 4)  = 3600
⇒ x = 22 \ The correct total score = 3600 – 40 + 90 = 3650
Average score after 19th innings = x + 4 = 26. 3650 − 3600 5000
\ Percentage error = × 100 =
3650 3650
16.
(d) 11 × 90 – 5 × 87 – 5 × 84
 = 1.36%
= 990 – 435 – 420 = 135.
21 − 15
17. (c) Out of 330 marks, the student is required to obtain 165 24.
(a) \ Required percentage =
× 100
15
marks, i.e., 50%
600
\ In first paper, because of scoring 54 marks only, = = 40%
he has to score 111 marks out of 150 in the 2nd paper, 15
i.e., 74% 25.
(d) Let number of students be n and average weight w.
nw + 50
18. (b) Let the average age of the whole team be x According to the given condition, =w+1
n +1
Total age = 11x
⇒ n + w = 49 (1)
Total age of 9 players = 11x – (28 + 25)
11x  3 nw + 50 + 50
Average of 9 players = =x–1 and = w + 1.5
9 n+2
⇒             x = 22 years. ⇒ 1.5n + 2w = 97 (2)
On solving Eqs. (1) and (2), we get
19.
(c) Suppose his average after 12 inning = x w = 47.
12 + 96 26.
(a) Total marks of a student in 8 subjects
Then,  = x + 5
13 = 8 × 87 = 696
x = 31
Total marks of a student in 6 subjects = 6 × 85 = 510
\ Required average = x + 5 = 31 + 5 = 36.
\ Remaining marks of 2 subjects =186
3n n Let the second highest marks be x, then highest marks
20.
(c) For numbers average is 36 + 4 and for numbers
4 4 is x + 2.
average is 36 – 4 \ x + x + 2 = 186
3n n ⇒ 2x = 184
 40   32
New average = 4 4 = 30 + 8 = 38. ⇒ x = 92
n \ Highest marks is x + 2 = 94.

111 27.
(a) Let first number be x, then
21.
(c) Total age of 14 persons =  × 14 years
4 x + ( x + 1) + ( x + 2) + ( x + 3) + ( x + 4)
= 41
Total age of 12 persons       5
 1554  ⇒ 5x + 10 = 205
1218
 
 84 =  = 304.5 ⇒ 5x = 195
 4  4 ⇒ x = 39
Average age of 12 persons \ Product of A and E = x × (x + 4)
304.5 = 39 × 43 = 1677.
  = 25.375 years.
12 28.
(a) Let woman and her daughter ages be 2x and x.
2x + x
22.
(c) A + B + C = 12,000 × 3 Also, = 42
2
B + C + D = 15,000 × 3 \ = 3x – 84
⇒ D – A = 3000 × 3 ⇒ x = 28
D – A = 9000 Hence, daughter’s age be 28 years.
also, D = 2A 29.
(d) Total weight of 29 students = 29 × 48  = 1392 kg
⇒ D = 18,000 and A = 9,000 If teachers weight is included, then total weight
= 30 × 48.5 = 1455 kg
Therefore average salary of B and C \ Weight of teacher
(45,000  18,000) = 1455 – 1392
= = 13,500.
2 = 63 kg.

Chapter 06.indd 14 6/4/2015 4:03:07 PM


6.15
Average

30.
(a) Total marks obtained by the candidates 35.
(c) Let the average of 12 innings be x.
= 120 × 35 = 4200 12 x + 96
Also, =x+5
Let passed students be x, then 13
4200 = x × 39 + (120 – x)15
⇒ 12x + 96 = 13x + 65
⇒ 4200 = 24x + 1800
⇒ 2400 = 24x ⇒ x = 31
⇒ x = 100. 12 × 31 + 96
\ Required average =
31.
(c) Let other two observations be x1 and x2. 13
1 + 2 + 6 + x1 + x2 468
\ 4 = = = 36.
5 13
⇒ x1 + x2 = 11
(4 − 1) 2 + (4 − 2) 2 + (4 − 6) 2  36.
(b) Let the original average expenditure be `x.
 
 + (4 − x1 ) 2 + (4 − x2 ) 2  Then,
and 5.2 = 40(x – 2) – 30x = 40
5
⇒ 26 = 9 + 4 + 4 + (x2 – 7)2 + (4 – x2)2 ⇒  10 x = 120 ⇒ x = 12
⇒ 9 = 2x22 – 22x2 + 49 + 16 \  Original expenditure = 30 × 12 = `360.

2
⇒ x 2 – 11x2 + 28 = 0
⇒ (x2 – 4) (x2 – 7) = 0 78.4 × 25 + 96 − 69
37.
(a) Required correct mean =
⇒ x2 = 4, 7 ⇒ x1 = 7, 4 25
Hence, other observations are 4, 7. 1960 + 27 1987
     = = = 79.48.
32.
(b) Total of eleven results = 11 × 50 = 550 25 25
Total of first six results = 49 × 6 38. (b) Total present age of the family of 6 members
= 294 = 6 × 22 = 132 years
Total of last six results = 52 × 6
Total age of the family of 6 members 7 years ago
= 312
= (132 – 7 × 6) = 90 years
\ Required six results = 294 + 312 – 550
= 56. ∴  Average age of the family at the birth of the
90
Mon + Tue + Wed + Thu youngest member =  = 18 years.
33.
(c) Given = 48° 5
4
\ 42° + Tue + Wed +Thu = 192° 39.
(b) Let the total number of boys and girls be B and G
respectively.
⇒ Tue + Wed + Thu = 150° (1)
\ Total score of boys = 71 B
Tue + Wed + Thu + Fri
and, = 52º Total score of girls = 73 G
4
⇒ 150° + Fri = 208° Total score of the class = 71.8(B + G)
 [from Eq. (1)] \ 71B + 73G = 71.8(B + G)
⇒ Fri = 58°.
⇒ 0.8B = 1.2G
34.
(b) Let total capital be `x. B 1.2 3
\ Required average ⇒   = = .
G 0.8 2
x 2x x
× 3% + × 5% + × 11% 40.
(c) Total salary of 75 workers = `426000
= 4 3 12
x Total salary of 25 workers = `135000
 3 10 11  Total salary of 30 workers = `171000
=  + + %
 4 3 12  \ Total salary of remaining 20 workers
 9 + 40 + 11  = 426000 – (135000 + 171000)
=  %
 12  = 120000
 60  120000
=   % = 5% \ Mean salary of 20 workers = = `6000.
 12  20

Chapter 06.indd 15 6/4/2015 4:03:07 PM


6.16 Chapter 6

41. (c) Total expenditure for the year 46.


(c) Let x be the highest score and y be the lowest score.
= [2200 × 3 + 2250 × 4 + 3120 × 5] \ x + y = 40 × 50 – 38 × 48
= 6600 + 9000 + 15600 = `31200 = 2000 – 1824 = 176
  Total saving = `1260 x – y = 172
Total income = expenses + savings \ x = 174, y = 2.
= 31200 +1260 = `32460 47.
(d) Suppose ninth person spent `x.
Average income Total money spent by nine persons = 96 + x
32460
= = `2705. 96 + x
12 \ x = +8
9
42.
(a) 480 × 7 = 3360 ⇒ 9x = 96 + x + 72
480 × 8 = 3840 ⇒ x = 21
\ Maximum number of pants that can be purchased \ Total money spent = 96 + x = 117.
is 7.
Balance Amount = 3620 – 3360 = 260 48.
(b) Suppose the third number is x.
x
In `260, two shirts can be purchased. \ First number =
2
43.
(b) Total weight of 45 students = 45 × 52 = 2340 1 x x
and second number =   =
2 2 4
Total weight of 5 students who leave
According to the question,
= 5 × 48 = 240
x x
Total weight of 5 students who join + +x
2 4 x x
= 5 × 54 = 270 = 56 or, + + x = 168
3 2 2
Hence new total weight of 45 students or,  7x = 672
= 2340 – 240 + 270 = 2370 \   x = 96
2370 2 96 96
\ Average = = 52 kg . Hence, the numbers are , , 96.
45 3 2 4
49.
(d) Suppose
44.
(a) Let the numbers be F, S and T.
a = 2n – 5, b = 2n – 3, c = 2n – 1, d = 2n + 1,
According to the question, e = 2n + 3
F + S +T
F = 2S, S = 2T and = 21 \ Their average
3
(2n − 5) + (2n − 3) + (2n − 1)
or, F + S + T = 63 (1)
Now, putting the value of F and T in (1), we have + (2n + 1) + (2n + 3)
=
S 5
28 + S + = 63
2 10n − 5
63 × 2 = = 2n – 1 = c.
or, S = = 18 5
7
50.
(c) According to the question,
\ F = 2S
= 2 × 18 = 36 A+ B+C
= 84
3
S 18 or, A + B + C = 252 (1)
and, T = = = 9
2 2 A+ B+C + D
\ largest number is 36. = 80
4
45.
(d) Let the three numbers be x, y and z. or, A + B + C + D = 320 (2)
x+ y+z B + C + D + ( D + 3)
\ = 135 and, = 79
3 4
or, B + C + 2D + 3 = 316
i.e., x + y + z = 405 (1) or, B + C + 2D = 313 (3)
Let z be the largest number Subtracting (1) from (2),
\ z = 180 D = 320 – 252 = 68 kg
⇒ x + y = 225 (2) Subtracting (3) for (2),
Also, x – y = 25 (3) A – D = 7
Solving (2) and (3), we get x = 125, y = 100. A = D + 7 = 68 + 7 = 75 kg.

Chapter 06.indd 16 6/4/2015 4:03:08 PM


6.17
Average

51.
(a) Max. marks = 180 + 150 = 330 Total marks in section C = 72c
If the required percentage of marks is x% in the Total marks in section D = 80d
second paper, then according to the question, 30% of Now, average marks of the students of sections A, B,
180 + x% of 150 = 50% of 330 C and D = 60
30 x 50 45a + 50b + 72c + 80d
or × 180 + × 150 = × 330 = 165 ∴ = 60
100 100 100 a+b+c+d
3 ⇒ 45a + 50b + 72c + 80d = 60a + 60b + 60c + 60d
or 54 + x = 165
2 ⇒ 15a + 10b − 12c − 20d = 0
3 ⇒ 15a + 10b = 12c + 20d (1)
or x = 111
2 Also,avarage marks of thhe students in sections A and
x = 74 %
B = 48
52.
(c) Let A1, A2, …, A9 be the numbers.
45a + 50b
Let A1 + A2 + … + A9 = 9 M ⇒ = 48
A1 + A2 + A3 = 3P a+b
A4 + A5 + … + A9 = 6 N ⇒ 45a + 50b = 48a + 48bb
Then, 9M = 3P + 6N ⇒ 3a = 2b ( 2)
3M = P + 2N. and avarage marks of students in sections B and
53.
(d) Let a, b, c, d, e and f be six numbers, such that C = 60
a = e + m
50b + 72c
b = e – k ⇒ = 60
c = e – l b+c
d = e – j ⇒ 50b + 72c = 60b + 60c
f = e + p ⇒ 12c = 10b (3)
Then, the sum of deviations of a, b, c, d, e and f from Putting the value of 12c from Eq. (3) in Eq. (1),
e is (m – k + l – j + p)
we get
The mean of a, b, c, d, e and f is given by
15a + 10 = 10b + 20d
abcd e f
⇒ 15a = 20d
6
emek el e j ee p ⇒ a:d = 20:15 = 4:3
=
6 57. (a) Let there be ‘n’ number of officers in the organisation.
6e  (m  k  l  j  p )
= Then,
6 n × 10000 + (400 − n)2000 = 400 × 3000
6e  Sum of deviations 10n + 800 − 2n = 1200
=
6
⇒ 8n = 400
Using the similar logic, the required mean
20  23  70 ⇒ n = 50
= = 26.5
20 58. (b) Total score of 8 subjects = 87 × 8 = 696
Hence, option (d). Total score of 6 subjects = 85 × 6 = 510
54. (d) Total runs upto 20 innings = 20 × 44.5 = 890 runs ∴ Score of remaining two subjects = 696 – 510 = 186
Let he scores x runs in the 21st innings then Now, let the highest and the next highest score are x.
and x – 2, then
890  x
Average after 21 innings = ( x) + ( x − 2) = 186
21
890  x ⇒ 2 x = 188 ⇒ x = 94,
⇒ 45 = ⇒ 945 = 890 + x
21 which is the highest score.
or x = 55 runs. 59. (a) Let there be ‘n’ number of officers in the organisation.
Then,
55.
(c) Total 6 numbers → 6 × 30 = 180
n × 10000 + (400 − n)2000 = 400 × 3000
1st 4 numbers → 4 × 25 = 100
10n + 800 − 2n = 1200
last 3 numbers → 3 × 35 = 105
4th number = 205 – 180 = 25. ⇒ 8n = 400
⇒ n = 50
56. (b) Let a, b, c and d be the number of students in class A,
B, C, and D, respectively. 60. (b) Total score of 8 subjects = 87 × 8 = 696
Then, total marks in section A = 45a Total score of 6 subjects = 85 × 6 = 510
Total marks in section B = 50b
∴ Score of remaining two subjects = 696 – 510 = 186

Chapter 06.indd 17 6/4/2015 4:07:55 PM


6.18 I Chapter 6

Now, let the highest and the next highest score are x x2 + x4
and x – 2, then And = 69
2
( x) + ( x − 2) = 186 ⇒ x2 + x4 = 138
⇒ 2 x = 188 ⇒ x = 94, Now , x1 + x3 + x2 + x4 + x5 = 305

⇒ 138 + 138 + x5 = 305
which is the highest score.
⇒ x5 = 305 − 276
61. (a) Let the number of boys in the class be x.
∴ x5 = 29
and number of girls in the class = 20
We are given, 63. (d ) Average weight of 19 men = 74 kg
Total weight of 19 men = 74 × 19 = 1406 kg
18( x + 20) = 20 × x + 20 × 15
Average weight of 38 women = 63
⇒ 18 x + 360 = 20 x + 300
Total weight of 38 women = 38 × 63 = 2394
⇒ 20 x = 60
Average weight of men and women together
∴ x = 30

2394 + 1406
=
62. (b) Let the five numbers be x1, x2, x3, x4, x5. 38 + 19
Average of 5 numbers = 61 3800
= = 66.66  67 kg .
x1 + x2 + x3 + x4 + x5 57
⇒ = 61
5 64. (d) Correct average
⇒ x1 + x2 + x3 + x4 + x5 = 305 (24 × 56) + (48 + 59 + 67) − (44 + 45 + 61)
x +x =
Now, 1 3 = 69 24
2 1344 + 174 − 150 1368
⇒ x1 + x3 = 138 = = = 57
24 24

Difficulty Level-2

1.
(b) For first nine months his salary is `380 per month. For 3. (b) Total marks of those 3 students must be
next 12 months, it was `420. For next 12 months it (44 × 100 – 97 × 43.92) = 139.96.
was `460 and for the last three months it was `500. 139.76
∴ Average of those 3 students = = 46.6.
∴ His average salary of last 36 months 3

[9(380) + 12(420 + 460) + 3(500)] 4.


(c) Since, we want to find the minimum attendance in
= = `430 third semester, we will take the attendance in fourth
36
semester 100%
∴ His pension is `215 per month. Let the required % of attendance be x

2.
(b) If t is the number of hrs the hiker walked on the first 60  70  x  100
= 80
day, then t + 2 is the number of hrs he walked on the 4
second day. ⇒ x = 320 – 230 = 90.

∴ t + t + 2 = 18, or t = 8. If s was the hiker’s average 5.


(d) (Agarkar + Sehwag + Sachin + Dravid + Ganguly)
speed in Km/h on the first day, then s + 1 was his make = 39 × 5 = 195 runs.
average speed on the second day. So, the total With respect to scoring runs
distance hiked in 2 days was (8) × (s) + (10)
 Dravid = Ganguly + 7
× (s + 1).
Ganguly = Agarkar – 9
Therefore, 8s + 10 (s + 1) = 64
Sehwag = Dravid + Ganguly
⇒ 18s = 54  or  s = 3 Km/h. Sehwag + Sachin = 110

Chapter 06.indd 18 6/4/2015 4:03:10 PM


6.19
Average I
⇒ Agarkar, Dravid, Ganguly, Sehwag and Sachin New total expenses = `(35x + 42)
scored 32, 30, 23, 53 and 57 runs respectively
35 x  42
∴ = (x – 1)
6.
(b) Average of first 7 numbers will be the 4th number = X 42
(Given)
⇒ 35x + 42 = 42x – 42
Average of all the 11 numbers will be the 6th number,
i.e., X + 4. or x = 12, therefore original expenditure of the mess
7.
(c) Suppose temperature on 1st day = 7K = 35 × 12 = `420.

Suppose temperature on 5th day = 8K abc


– cba
12.
(c) Remember
\ Temperature on 99(a  c)
M + T + W + Th = 232 where abc and cba are the three digit numbers and
Average temperature on (a, c) ≠ 0
T + W + Th + F = 240 Again since the difference in average = 19.8
\ 232 – 7K = 240 – 8K Therefore, the difference in total = 19.8 × 10 = 198

⇒ K = 8 Thus, 99 × (a – c) = 198

\ Temperature on 5th day = 64º. ⇒ (a – c) = 2.

8. (a) Let each side of the square field be x Km 13.


(d) Present total age of husband and wife
\ Average speed of the plane = (23 × 2 + 5 × 2) = 56 years
Present total age of husband, wife and child
4x
= = 20 × 3 = 60 years
x x x x
+ + + ∴ age of child = (60 – 56) = 4 years.
800 600 400 200
14.
(a)
4 × 2000 9600
= = No. of 2 No. of 3 No. of 4
3 + 4 + 6 + 12 25
wheelers wheelers wheelers
= 384 Km/h. 2x x 2x
9.
(a) Suppose the number of candidates passed =  x No. of wheels 2 × 2x 3×x 2x × 4
\ 39x + 15 (120 – x) = 120 × 15 = 4x  
= 3x  
= 8x
⇒ 24x = 120 × 35 – 120 × 35 Therefore average number of wheels
= 120 (35 – 15) = 120 × 20 4 x  3x  8 x
=  = 3.
⇒ x = 100. 5x
10. (a) On Monday, the payment = `x
[r  ( p  q )]  p  q pqr
On Tuesday, the payment = `(x + 3y) 15.
(c) = 21 
3 3
On Wednesday, the payment = `(x + 2y)
2p  r pqr
On Thursday, the payment = `(x + y) ⇒  21 =
3 3
On Friday, the payment = `(x + 2y)
pq
\ Average daily earnings during the five-day week ⇒ = 21
3
5x + 8 y 8y ⇒ p – q = 63.
= = x+ .
5 5
16.
(b) Total marks in 10 papers = 800
11.
(b) Let the average expenditure per student be `x
Total marks in 8 papers = 648
Therefore, original total expenses = `35x
Total of highest and lowest marks = 152
New average expenditure per student = `(x – 1) lowest marks = 152 – 92
New average expenditure per student = `(x – 1) = 60.

Chapter 06.indd 19 6/4/2015 4:03:10 PM


6.20 Chapter 6

17.
(a) Average Total Solving these,
S + G → 35 70 a = 16, b =24, c =20, p = 45, q = 35, r = 30.
K + G → 32 64
S + K → 38 76 22.
(a) a + b + c + d + e = 10
S + K + G → 35 105 10
Average = =2
35 5
D + I → 35
2 a = 0 to 4
Least value of x:0 ≤ x ≤ 2
S  K  G  D  I 105  35
∴     28. b = 0 to 4 [Q average = 2]
S 5
c = 0 to 4
18.
(d) The 5 consevtice odd numbers whose average is k are d = 0 to 4
(k – 4) (k – 2), k, (k + 2) (k + 4)
e = 0 to 4.
Again the average of (k – 4), (k – 2), (k), (k + 2),
(k + 4), (k + 6), (k + 8) is (k + 2) 23.
(c) The marks of the 5 students and the remainder that
Alternatively: Consider some appropriate numbers. they leave when divided by 3 are tabulated below.

19.
(a) Total age of A and B = 20 × 2 = 40 years Marks 71 76 80 82 91
Total age of B and C = 19 × 2 = 38 years Remainder 2 1 2 1 1

Total age of A and C = 21 × 2 = 42 years
After the first 3 marks are entered, the total has to be
40  38  42 a multiple of 3. The remainders can only be 1, 1, 1.
Total age of (A + B + C) =
2 (2, 1, 1 or 2, 2, 1 would not produce a multiple of 3)
= 60 years.
\ The fourth and fifth marks to be entered were 71
Therefore, ages of A, B and C = 22 years, 18 years and
and 80.
20 years, respectively.
24. (c) Haque’s salary in 2008 = 900000 rupees
 1 Expenditure on food in 2007 = 400000 rupees
20.
(d) Sum of numbers = 1   + 1 + 1 + 1 ... (n – 1) times
 n
Expenditure on food in 2008
1 1 5
= 1   (n  1)  n  = 400000 + 400000 ×
n n 100
\ Arithmetic mean of n numbers = 420000 rupees
\ Percentage expenditure on food items in 2008
1
n 2 420000
= n  n  1 1  1 . = × 100
n n2 n2 900000
= 46.67 ≈ 47%
21.
(a) Let the number of professors, associates and assistant
professors be a, b and c, respectively. 25. (b) Average of first five integers
Let their average ages = p, q and r, respectively. 1 + 2 + 3 + 4 + 5 15
= = = 3 = n (given)
So, ap + bq + cr = 2160 (1) 5 5
and, average of first seven integers
ap  bq
= 39 (2)
ab 1+ 2 + 3 + 4 + 5 + 6 + 7
=
7
bq  cr 328
= (3) 28
bc 11 = = 4 = (3 + 1) = (n + 1).
7
ap  cr 362
  = (4) 26. (b) Given x < 0.50, 0 < y < 1, z > 1. It is therefore clear
ac 3
that, value of x and y range between 0 and 1, hence
   a(p + l) + b(q + 6) + c(r + 7) = 2460 (5) median will also lie between 0 and 1.

Chapter 06.indd 20 6/4/2015 4:03:11 PM


CHAPTER

Ratio and Proportion 7


Ratio 2. Triplicate Ratio  The ratio of the cubes of two numbers
A ratio is a comparison of two quantities by division. It is is called the triplicate ratio of the two numbers.
a relation that one quantity bears to another with respect 33 27 3
to magnitude. In other words, ratio means what part one For example, or is triplicate ratio of .
quantity is of another. The quantities may be of same kind 43 64 4
or different kinds. For example, when we consider the ratio 3. Sub-duplicate Ratio  The ratio of the square roots of
of the weight 45 kg of a bag of rice to the weight 29 kg of a two numbers is called the sub-duplicate ratio of two
bag of sugar we are considering the quantities of same kind numbers.
but when we talk of allotting 2 cricket bats to 5 sportsmen, 3 9
For example, is the sub-duplicate ratio of .
we are considering quantities of different kinds. Normally, 4 16
we consider the ratio between quantities of the same kind.
4. Sub-triplicate Ratio  The ratio of the cube roots of
a two numbers is called the sub-triplicate ratio of two
If a and b are two numbers, the ratio of a to b is or
b numbers.
a ÷ b and is denoted by a:b. The two quantities that are being 2 8
For example, is the sub-triplicate ratio of .
compared are called terms. The first is called antecedent 3 27
and the second term is called consequent. 5. Inverse Ratio or Reciprocal Ratio  If the antecedent
3 and consequent of a ratio interchange their places, the
For example, the ratio 3:5 represents with antecedent new ratio is called the inverse ratio of the first. Thus,
5
3 and consequent 5. 1 1
if a:b be the given ratio, then : or b:a is its inverse
a b
Notes: ratio.
3 5
For example, is the inverse ratio of .
1. A ratio is a number, so to find the ratio of two quantities, 5 3
they must be expressed in the same units.
2. A ratio does not change if both of its terms are multiplied 6. Compound Ratio  The ratio of the product of the
or divided by the same number. Thus, antecedents to that of the consequents of two or more
4 given ratios is called the compound ratio. Thus, if
2 6
= = etc. a:b and c:d are two given ratios, then ac:bd is the
3 6 9 compound ratio of the given ratios.
Types of Ratios
3 4 5
1. Duplicate Ratio  The ratio of the squares of two For example, if , and be the given ratios, then
4 5 7
numbers is called the duplicate ratio of the two
numbers. 3× 4 × 5 3
their compound ratio is , that is, .
32 9 4× 5× 7 7
For example, 2 or is called the duplicate ratio
4 16
3 Proportion
of .
4 The equality of two ratios is called proportion.

Chapter 07.indd 1 6/4/2015 5:30:24 PM


7.2 Chapter 7

a c a c
If = , then a, b, c and d are said to be in proportion Each term of the ratio and is called a proportional. a,
b d b d
and we write a:b::c:d. This is read as “a is to b as c is to d”. b, c and d are, respectively, the first, second, third and fourth
3 6 roportionals.
For example, since = , we write 3:4::6:8 and say
4 8 Here a, d are known as extremes and b, c are known
3, 4, 6 and 8 are in proportion. as means.

soMe Basic forMulae

Illustration 2 Find a third proportional to the numbers


1. If four quantities are in proportion, then 2.5, 1.5.
Product of means = Product of extremes Solution: Let x be the third proportional, then
For example, in the proportion a:b::c:d, we have 2 ⋅ 5 1⋅ 5
bc = ad. 2×5:1×5:: 1×5:x or, =
1⋅ 5 x
1⋅ 5 ×1⋅ 5
\ x = = 0.9
2⋅5

4. Mean Proportional If a:x::x:b, x is called the


mean or second proportional of a, b.
From this relation, we see that if any three of a x
the four quantities are given, the fourth can be We have = or, x2 = ab or, x = ab .
x b
determined.
\ Mean proportional of a and b is ab
2. Fourth proportional If a:b::c:x, x is called the
fourth proportional of a, b, c. We also say that a, x, b are in continued proportion.
a c b×c
We have = or, x =
b x a Illustration 3 Find the mean proportional between 48
b×c and 12.
Thus, fourth proportional of a, b, c is .
a Solution: Let x be the mean proportional. Then,
48 x
Illustration 1 Find a fourth proportional to the numbers 48:x::x:12 or, =
x 12
2, 5, 4.
or, x2 = 576 or, x = 24
Solution: Let x be the fourth proportional, then
2 4
2:5::4:x or, = a c
5 x 5. If = , then
b d
5× 4
\ x = = 10 a+b c+d
2 (i) = (Componendo)
b d
3. Third Proportional If a:b::b:x, x is called the a −b c−d
third proportional of a, b. (ii) = (Dividendo)
b d
a b b2 a+b c+d
We have = or, x = . (iii) = (Componendo and dividendo)
b x a a −b c−d
b2 a a+c a−c
Thus, third proportional of a, b is . (iv) = = .
a b b+d b−d

Chapter 07.indd 2 6/4/2015 5:30:26 PM


Ratio and Proportion 7.3

Illustration 4 The sum of two numbers is c and their x p


p \ =
quotient is . Find the numbers. x+ y p+q
q
x p
Solution: Let the numbers be x, y. ⇒ = [Using (1)]
c p+q
Given x+y=c (1)
pc
x p ⇒ x=
and, = (2) p+q
y q

soMe useful sHort-cut MetHoDs

Illustration 5 Two numbers are in the ratio of 4:5 and the


1. (a) If two numbers are in the ratio of a:b and the
sum of these numbers is 27. Find the two numbers
sum of these numbers is x, then these numbers
ax bx Solution: Here a = 4, b = 5 and x = 27
will be and , respectively. ax 4 × 27
a+b a+b \ The first number = = = 12
or a+b 4+5
If in a mixture of x litres, two liquids A and B bx 5 × 27
and, the second number = = = 15
are in the ratio of a:b, then the quantities of a+b 4+5
ax
liquids A and B in the mixture will be Illustration 6 Three numbers are in the ratio of 3:4:8 and
a+b the sum of these numbers is 975. Find the three numbers
bx
litres and litres, respectively. Here a = 3, b = 4, c = 8 and x = 975
a+b
ax 3 × 975
\ The first number = = = 195
(b) If three numbers are in the ratio of a:b:c and a+b+c 3+ 4+8
the sum of these numbers is x, then these bx 4 × 975
ax bx The second number = = = 260
numbers will be , and a+b+c 3+ 4+8
a+b+c a+b+c cx 8 × 975
cx and, the third number = = = 520
, respectively. a+b+c 3+ 4+8
a+b+c

2. If two numbers are in the ratio of a:b and


Explanation difference between these numbers is x, then these
Let the three numbers in the ratio a:b:c be A, B and C. numbers will be
ax bx
Then, (a) and , respectively (where a > b)
a −b a −b
A = ka, B = kb, C = kc
ax bx
and, A + B + C = ka + kb + kc = x (b) and , respectively (where a < b).
b−a b−a
x
⇒ k(a + b + c) = x ⇒ k =
a+b+c Explanation
ax Let the two numbers be ak and bk.
\ A = ka = Let a > b.
a+b+c
Given ak – bk = x
bx x
B = kb = ⇒ (a – b)k = x or, k =
a+b+c a −b
cx ax bx
C = kc = . Therefore, the two numbers are and .
a+b+c a −b a −b

Chapter 07.indd 3 6/4/2015 5:30:27 PM


7.4 Chapter 7

Illustration 7 Two numbers are in the ratio of 4:5. If


the difference between these numbers is 24, then find the Therefore, the two numbers are ax(c − d ) and
ad − bc
numbers. bx(c − d )
Solution: Here a = 4, b = 5 and x = 24 ad − bc
ax 4 × 24
\ The first number = = = 96
b−a 5−4 (b) The ratio between two numbers is a:b. If x
is subtracted from each of these numbers, the
bx 5 × 24
and the second number = = = 120 ratio becomes c:d.
b−a 5−4
The two numbers are given as:
ax(d − c) bx(d − c)
3. (a) If a:b = n1:d1 and b:c = n2:d2, then and .
ad − bc ad − bc
a:b:c = (n1 × n2):(d1 × n2):(d1 × d2).
(b) If a:b = n1:d1, b:c = n2:d2
Explanation
and c:d = n3:d3, then
Let the two numbers be ak and bk.
a:b:c:d = (n1 × n2 × n3):(d1 × n2 × n3) ak − x c
: (d1 × d2 × n3):(d1 × d2 × d3). Given = ⇒ akd – xd = bck – xc
bk − x d
⇒ k(ad – bc) = x(d – c)
Illustration 8 If A:B = 3:4 and B:C = 8:9, find A:B:C. x(d − c)
⇒k=
Solution: Here n1 = 3, n2 = 8, d1 = 4 and d2 = 9 ad − bc
\ a : b : c = (n1 × n2):(d1 × n2):(d1 × d2) ax(d − c)
Therefore, the two numbers are and
= (3 × 8):(4 × 8):(4 × 9) ad − bc
= 24:32:36 or, 6:8:9 bx(d − c)
Illustration 9 If A:B = 2:3, B:C = 4:5 and C:D = 6:7, find A:D. ad − bc
Solution: Here n1 = 2, n2 = 4, n3 = 6, d1 = 3, d2 = 5 and Illustration 10 Given two numbers which are in the ratio
d3 = 7 of 3:4. If 8 is added to each of them, their ratio is changed
\ A:B:C:D = (n1 × n2 × n3):(d1 × n2 × n3) to 5:6. Find the two numbers.
:(d1 × d2:n3):(d1 × d2 × d3) Solution: We have
= (2 × 4 × 6):(3 × 4 × 6):(3 × 5 × 6) a:b = 3:4, c:d = 5:6 and x = 8
:(3 × 5 × 7) ax(c − d )
\ The first number =
= 48:72:90:105 or, 16:24:30:35 ad − bc
Thus, A:D = 16:35 3 × 8 × (5 − 6)
= = 12
(3 × 6 − 4 × 5)
4. (a) The ratio between two numbers is a:b. If x
bx(c − d )
is added to each of these numbers, the ratio and, the second number =
becomes c:d. The two numbers are given as: ad − bc
ax(c − d ) bx(c − d ) 4 × 8 × (5 − 6)
and . = = 16
ad − bc ad − bc (3 × 6 − 4 × 5)

Illustration 11 The ratio of two numbers is 5:9. If each


Explanation number is decreased by 5, the ratio becomes 5:11. Find the
Let two numbers be ak and bk. numbers.
ak + x c Solution: We have a:b = 5:9, c:d = 5:11 and x = 5
Given = ⇒ akd + dx = cbk + cx ax(d − c)
bk + x d \ The first number =
⇒ k(ad – bc) = x(c – d) ad − bc
x (c − d ) 5 × 5 × (11 − 5)
⇒k= = = 15
ad − bc (5 × 11 − 9 × 5)

Chapter 07.indd 4 6/4/2015 5:30:28 PM


Ratio and Proportion 7.5

bx(d − c)
and the second number = 6. There are four numbers a, b, c and d.
ad − bc
(i) The number that should be subtracted from
9 × 5 × (11 − 5)
= = 27 each of these numbers so that the remaining
(5 × 11 − 9 × 5) numbers may be proportional is given by
ad − bc
5. (a) If the ratio of two numbers is a:b, then the .
(a + d ) − (b + c)
numbers that should be added to each of the
numbers in order to make this ratio c:d is
given by
Explanation
ad − bc
. Let x be subtracted from each of the numbers.
c−d
The remainders are a – x, b – x, c – x and d – x
a−x c−x
Explanation Given =
b−x d−x
Let the required number be x
a+x c ⇒ (a – x) (d – x) = (b – x) (c – x)
Given = ⇒ ad + xd = bc + xc
b+ x d ⇒ ad – x(a + d) + x2 = bc – x(b + c) + x2
⇒ x(d – c) = bc – ad ⇒ (b + c)x – (a + d)x = bc – ad
ad − bc
or x= . bc − ad bc − ad
c−d ∴ x= or,
(b + c) − (a + d ) (b + c) − (a + d )
(b) If the ratio of two numbers is a:b, then the
number that should be subtracted from each (ii) The number that should be added to each of
of the numbers in order to make this ratio c:d these numbers so that the new numbers may
is given by be proportional is given by
bc − ad bc − ad
. .
c−d (a + d ) − (b + c)

Explanation Explanation
Let the required number be x Let x be added to each of the numbers.
a−x c The new numbers are a + x, b + x, c + x and d + x
Given, = ⇒ ad – xd = bc – xc
b−x d a+x c+x
⇒ x(c – d) = bc – ad Given, = .
b+ x d+x
bc − ad ⇒ (a + x) (d + x) = (b + x) (c + x)
or, x= .
c−d ⇒ ad + x(a + d) + x2 = bc + x(b + c) + x2
Illustration 12 Find the number that must be subtracted ⇒ (a + d)x – (b + c)x = bc – ad.
from the terms of the ratio 5:6 to make it equal to 2:3. bc − ad
Solution: We have a:b = 5:6 and c:d = 2:3 ∴ x=
(a + d ) − (b + c)
∴ The required number
Illustration 14 Find the number subtracted from each of
bc − ad 6× 2 − 5× 3
= = =3 the numbers 54, 71, 75 and 99 leaves the remainders which
c−d 2−3 are proportional.
Illustration 13 Find the number that must be added to the
Solution: We have a = 54, b = 71, c = 75 and d = 99
terms of the ratio 11:29 to make it equal to 11:20
ad − bc
Solution: We have a:b = 11:29 and c:d = 11:20 The required number =
∴ The required number (a + d ) − (b + c)
ad − bc 11× 20 − 29 × 11 54 × 99 − 71× 75
= = = 11 = =3
c−d 11 − 20 (54 + 99) − (71 + 75)

Chapter 07.indd 5 6/4/2015 5:40:20 PM


7.6 Chapter 7

Illustration 16 The incomes of Mohan and Sohan are in


7. The incomes of two persons are in the ratio of
the ratio 7:2 and their expenditures are in the ratio 4:1. If
a:b and their expenditures are in the ratio of c:d.
each saves `1000, find their expenditures.
If the saving of each person be `S, then their
Solution: We have a:b = 7:2, c:d = 4:1 and S = 1000
incomes are given by
cS (b − a ) 4 × 1000 × (2 − 7)
aS (d − c) bS (d − c) \ A’s expenditure = =
` and ` ad − bc (7 × 1 − 2 × 4)
ad − bc ad − bc
= `20000
and their expenditures are given by
dS (b − a ) 1× 1000 × (2 − 7)
cS (b − a ) dS (b − a ) B’s expenditure = =
` and ` . ad − bc (7 × 1 − 2 × 4)
ad − bc ad − bc
= `5000
Explanation 8. (a) If in a mixture of x litres of two liquids A and
Let their incomes be `ak and `bk, respectively. Since B, the ratio of liquids A and B is a:b, then the
each person saves `S, quantity of liquid B to be added in order to
\ expenditure of first person = `(ak – S) make this ratio.
and expenditure of second person = `(bk – S) x(ad − bc)
ak − S c c:d is .
Given, = c ( a + b)
bk − S d
⇒ akd – Sd = bkc – Sc Explanation
(d − c) S
⇒ k(ad – bc) = (d – c)S or, k = ax
ad − bc Quantity of liquid A in the mixture =
a+b
Therefore, the incomes of two persons are
a(d − c) S b( d − c ) S bx
and Quantity of liquid B in the mixture =
ad − bc ad − bc a+b
and, their expenditures are Let litres of liquid B to be added in order to make this
ak – S and bk – S ratio as c:d.
a(d − c) S b( d − c ) S ax bx
that is, – S and –S Then, : + l = c:d
ad − bc ad − bc a+b a+b
cS (b − a ) dS (b − a ) ax bx + l (a + b)
or, and . or, : = c:d
ad − bc ad − bc a+b a+b
Illustration 15 Annual income of A and B is in the ratio of ax c
5:4 and their annual expenses bear a ratio of 4 : 3. If each or, =
bx + l (a + b) d
of them saves `500 at the end of the year, then find their
annual income. or, axd = bcx + cl(a + b)
Solution: We have a:b = 5:4, c:d = 4:3 and S = 500 x(ad − bc)
aS (d − c) or, l= .
\ Annual income of A = ( a + b )c
ad − bc
5 × 500 × (3 − 4)
= (b) In a mixture of two liquids A and B, the ratio
(5 × 3 − 4 × 4) of liquids A and B is a:b. If on adding x litres
= `2500. of liquid B to the mixture, the ratio of A to B
bS (d − c) becomes a:c, then in the beginning the quantity
and, annual income of B =
ad − bc ax
of liquid A in the mixture was litres and
4 × 500 × (3 − 4) c−b
= bx
(5 × 3 − 4 × 4) that of liquid B was litres.
= `2000 c−b

Chapter 07.indd 6 6/4/2015 5:30:31 PM


Ratio and Proportion 7.7

Explanation Illustration 19 In what ratio the two kinds of tea must be


Let the quantity of mixture be M litres. mixed together into one at `9 per kg and another at `15 per
aM kg, so that mixture may cost `10.2 per kg?
Then, the quantity of liquid A = litres Solution: We have c1 = 9, c2 = 15, cm = 10.2
a+b
bM q1 c −c
and, the quantity of liquid B= litres \ = 2 m
a+b q2 cm − c1
If x litres of liquid B is added, then 15 − 10.2 4.8
= =
a M bM 10.2 − 9 1.2
: + x = a:c
a+b a+b 4
=
a M bM + x(a + b) 1
or, : = a:c
a+b a+b Thus, the two kinds of tea are mixed in the ratio 4:1.
aM a Illustration 20 In a mixture of two types of oils O1 and
or, =
bM + x(a + b) c O1, the ratio O1:O2 is 3:2. If the cost of oil O1 is `4 per litre
or, cM = bM + x(a + b) and that of O2 is `9 per litre, then find the cost/litre of the
x ( a + b) resulting mixture
or, M=
c−b We have q1 = q2 = 2, c1 = 4 and c2 = 9
\ Quantity of liquid \ The cost of resulting mixture
ax(a + b) ax c1 × q1 + c2 × q2
A= = litres =
(c − b)(a + b) c−b q1 + q2
and quantity of liquid 4×3 + 9× 2 30
= = = `6
bx(a + b) bx 3+ 2 5
B= = litres
(c − b)(a + b) c−b
10. (a) If a mixture contains two ingredients A and
Illustration 17 729 ml of a mixture contains milk and water B in the ratio a:b, then
in the ratio 7:2. How much more water is to be added to get
a
a new mixture containing milk and water in the ratio of 7:3. percentage of A in the mixture = ×
a+b
Solution: Here x = 729, a : b = 7 : 2 and c:d = 7:3 100% and percentage of B in the mixture =
\ The quantity of water to be added b
× 100%
x(ad − bc) 729 × (7 × 3 − 2 × 7) a+b
= = = 81 ml
c ( a + b) 7(7 + 2)
(b) If two mixtures M1 and M2 contain ingredients
Illustration 18 A mixture contains alcohol and water in A and B in the ratios a:b and c:d, respectively,
the ratio of 6:1. On adding 8 litres of water, the ratio of then a third mixture M3 obtained by mixing
alcohol to water becomes 6:5. Find the quantity of water in M1 and M2 in the ratio x:y will contain
the mixture.
Solution: We have a:b = 6:1, a:c = 6:5 and x = 8.  a× x c× y 
a+b + c+d 
\ The quantity of water in the mixture   × 100% ingredient A, and
bx 1× 8  x+ y 
= = = 2 litres
c−b 5 −1   ax cy 
 a+b + c+d 
9. When two ingredients A and B of quantities q1 and 100% −  
  x+ y 
q2 and cost price/unit c1 and c2 are mixed to get a   
mixture c having cost price/unit cm, then
 bx dy 
q c −c c ×q +c ×q a+b + c+d 
(a) 1 = 2 m and (b) cm = 1 1 2 2 or,   × 100% ingredient B.
q2 cm − c1 q1 + q2  x+ y 

Chapter 07.indd 7 6/4/2015 5:30:32 PM


7.8 Chapter 7

Illustration 21  If a mixture contains water and alcohol in Then, percentage quantity of sliver in the new alloy
the ratio 2:3, what is the percentage quantity of water in the
mixture?  ax cy   3x 5 y 
a+b + c+d   4 + 8 
Solution: Here a = 2, b = 3 =   × 100% =   × 100%
\ Percentage quantity of water in the mixture  x+ y   x+ y 
a 6x + 5 y
= × 100% = × 100% (1)
a+b 8( x + y )
2 Since the ratio of silver and copper in the new alloys
= × 100% is 2:1
2+3
\ Percentage quantity of sliver in the new alloy
2
= × 100% 2 200
5 = × 100% = %(2)
2 +1 3
200 From (1) and (2), we get
= or, 40%
5
6x + 5 y 2
=
Illustration 22  Two alloys contain silver and copper in the 8( x + y ) 3
ratio 3:1 and 5:3. In what ratio the two alloys should be
added together to get a new alloy having silver and copper or, 18x + 15y = 16x + 16y
in the ratio of 2:1? or, 2x = y
Solution: We have a:b = 3:1, c:d = 5:3 or, x:y = 1:2
Let the two alloys be mixed in the ratio x:y Hence, the two alloys should be mixed in the ratio 1:2.

Practice Exercises

Difficulty Level-1
(Based on Memory)

1. The ratio of the rate of flow of water in pipes varies (a) 18 (b) 24
inversely as the square of the radius of the pipes. What (c) 12 (d) 20
is the ratio of the rates of flow in two pipes of diameters  [Based on MAT, 2004]
2 cm and 4 cm?
4. A box filled with paper bundles weighs 36 kilograms. If the
(a) 1:2 (b) 2:1
weight of the box and paper bundles, respectively, are in the
(c) 1:8 (d) 4:1 ratio of 3:22, then find the weight of the papers in grams.
[Based on MAT, 2004] (a) 30,680 (b) 30,710
2. Half the girls and one-third of the boys of a college reside (c) 31,500 (d) 31,680
in the hostel. What fractional part of the student body is 5. A, B and C scored 581 runs such that four times A’s runs
hostel dwellers if the total number of girls in the college is are equal to 5 times B’s runs which are equal to seven
100 and is one-fourth of the total strength? times C’s runs. Determine the difference between A’s runs
(a) Two-fifths (b) Five-twelfths and C’s runs.
(c) One-fifth (d) Three-eighths (a) 125 (b) 120
[Based on MAT, 2004] (c) 105 (d) 90
3. In a class room, three-fourths of the boys are above 160 6. A person spends one-third of the money with him on clothes,
cm in height and they are 18 in number. Also out of the one-fifth of the remaining on food and one-fourth of the
total strength, the boys form only two-thirds and the rest remaining on travel. Now, he is left with `100. How much
are girls. The total number of girls in the class is: did he have with him in the beginning?

Chapter 07.indd 8 6/4/2015 5:30:32 PM


Ratio and Proportion 7.9

(a) `200 (b) `250 (a) 15 and 20 (b) 16 and 24


(c) `300 (d) `450 (c) 13 and 17 (d) 17 and 9
[Based on MAT, 2003] [Based on MAT, 1999]
7. One-third of the contents of a container evaporated on the 15.
`770 have been divided among A, B and C such that A
1st day. three-fourths of the remaining evaporated on the receives two-ninths of what B and C together receive.
second day. What part of the contents of the container is Then, A’s share is:
left at the end of the second day? (a) `140 (b) `154
(a) One-fourth (b) One-half (c) `165 (d) `170
(c) One-eighteenths (d) One-sixth [Based on MAT, 2001]
[Based on MAT, 2003]
16.
What least number must be subtracted from each of the
8. The monthly incomes of two persons are in the ratio of numbers 14, 17, 34 and 42 so that the remainders are
4:5 and their monthly expenditures are in the ratio of 7:9. proportional?
If each saves `50 a month, then what are their monthly (a) 0 (b) 1
incomes?
(c) 2 (d) 7
(a) `100, `125 (b) `200, `250 [Based on MAT, 2001]
(c) `300, `375 (d) `400, `500
[Based on MAT, 2002] 17.
The mean proportional between 45 and a certain number
is three times the mean proportional between 5 and 22.
9. When a bus started from the first stop, the number of The number is:
male passengers to the number of female passengers was (a) 24 (b) 49
3:1. At the first stop, 16 passengers got down and 6 more
(c) 22 (d) 9
female passengers got in. The ratio of the male to female
passengers now became 2:1. What was the total number 18.
If the mean proportional between X and Y is n times the
of passengers in the bus when it started from the first stop? third proportional, then the ratio X:Y will be:
(a) 64 (b) 48 (a) n2:1 (b) 1:n2
(c) 54 (d) 72 (c) n2/3:1 (d) 1:n2/3
[Based on MAT, 2002]
If x varies inversely as (y2 – 1) and is equal to 24 when
19.
10. If the ratio of boys to girls in a class is B and the ratio of y = 10, the value of x when y = 5 will be:
girls to boys is G, then 3 (B + G) is: (a) 100 (b) 101
(a) Equal to 3 (b) Less than 3 (c) 99 (d) None of the above
(c) More than 3 (d) Less than one-third 20. Which of the following is the ratio between a number and
[Based on MAT, 2001] the number obtained by adding one-fifth of that number
11.
If a:b = 2:5, then the value of (2a + 3b):(7a + 5b) is: to it?
19 99 (a) 6:5 (b) 5:6
(a) (b) (c) 5:4 (d) 4:5
39 13
[Based on Narsee Monjee Inst. of Man. Studies, 2003]
31 19
(c) (d) 21. The ratio of the age of a man and his wife is 4:3. After 4
19 31
[Based on MAT, 1999] years, this ratio will be 9:7. If at the time of the marriage,
the ratio was 5:3, then how many years ago they were
12.
If two numbers are in the ratio 6:13 and their least common married ?
multiple is 312, the sum of the numbers is:
(a) 12 years (b) 8 years
(a) 75 (b) 57
(c) 10 years (d) 15 years
(c) 76 (d) 67 [Based on IIFT, 2003]
[Based on MAT, 1999]
22.
X’s income is three-fourths of Y’s income, and X’s
13. The ratio of boys to girls in an engineering college is expenditure is four-fifths of Y’s expenditure. If X’s income
20:1. How many girls need to be added to make this ratio is nine-tenths of Y’s expenditure, find the ratio of X’s
8:3? savings to Y’s savings.
(a) 26 (b) 43 (a) 1:2 (b) 2:1
(c) 20 (d) Cannot be determined (c) 1:4 (d) 2:3

14.
Two numbers are in the ratio 2:3. If eight is added to both 23.
If x:y = 1:2, y:z = 1:3, z:w = 3:8 and u:w = 2:5, find the
the numbers, the ratio becomes 3:4. The numbers are: value of (xyu):(w2z).

Chapter 07.indd 9 6/4/2015 5:30:33 PM


7.10 I Chapter 7

(a) 1:90 (b) 3:80 (a) One-eighth (b) One-tenth


(c) 1:120 (d) 3:160 (c) One-fourth (d) Three-eights
24. A man spends `500 in buying 12 tables and chairs. The [Based on MAT, 2008]
cost of one table is `50 and that of one chair is `40. What 32.
When 30 per cent of a number is added to another number
is the ratio of the numbers of the chairs and the tables the second number increases by its 20 per cent. What is
purchased? the ratio between the first and the second number?
(a) 5:4 (b) 3:2 (a) 3:2 (b) 2:3
(c) 1:5 (d) None of these (c) 2:5 (d) Data inadequate
[Based on IMT Ghaziabad, 2002] 33.
An amount of money is to be distributed among A, B and
25. If x varies directly as 3y + 1 and x = 9 when y = 1, then C in the ratio 5:8:12, respectively. If the total share of B
what is the value of x when y = 5? and C is our times that of A. What is A’s share?
(a) 11 (b) 10 (a) `3,000
(c) 20 (d) 36 (b) `5,000
[Based on IMT Ghaziabad, 2002] (c) Cannot be determined
26. Two numbers are in the ratio of 1:2. If 7 be added to both, (d) None of these
their ratio changes to 3:5. The greater number is: 34. A and B are two different alloys of gold and copper
(a) 20 (b) 24 prepared by mixing metals in the proportion 7:2 and 7:11,
respectively. If equal quantities of the alloys are melted to
(c) 28 (d) 32
form a third alloy C, find the ratio of gold and copper in C.
[Based on FMS (Delhi), 2002]
(a) 5:7 (b) 6:6
27.
The prices of a room air conditioner and an automatic
(c) 7:5 (d) 14:13
washing machine are in the ratio of 3:2. What would be
the price of the washing machine if it costs `6000 less 35. A sum of money is divided among A, B, C and D in the
than the air conditioner? ratio of 3:7:9:13 respectively. If the share of B is `9180
more than the share of A, then what is the total amount of
(a) `18000 (b) `10000
money of A and C together?
(c) `12000 (d) `6000
(a) `27540 (b) `27560
[Based on I.P. Univ., 2002]
(c) `26680 (d) `24740
28.
A dog takes 6 leaps for every 4 leaps of a hare and 2 leaps [Based on NMAT, 2008]
of the dog are qual to 5 leaps of the hare. What is the ratio
of their speed? 36. A shopkeeper mixes two kinds of flour, one costing `3.50
per kg and the other at `2.75 per kg. The ratio of first kind
(a) 8:13 (b) 24:15 of flour to that of the second is:
(c) 12:7 (d) 15:4 (a) 1:2 (b) 1:3
29. A jar contains black and white marbles. If there are ten (c) 3:4 (d) None of these
marbles in the jar, then which of the following could NOT [Based on NMAT, 2006]
be the ratio of black to white marbles?
37.
The numbers x, y, z are proportional to 2, 3, 5. The sum of
(a) 9:1 (b) 7:3 x, y and z is 100. The number y is given by the equation y
(c) 1:10 (d) 1:4 = ax – 10. Then, a is:
[Based on REC Tiruchirapalli, 2002] 3
(a) 2 (b)
2
30. Eight people are planning to share equally the cost of a
rental car. If one person withdraws from the arrangement 5
(c) 3 (d)
and the others share equally the entire cost of the car, then 2
the share of each of the remaining persons increased by: [Based on FMS, 2011]
(a) One-ninth (b) One-eighth 38.
If x varies as the cube of y, and y varies as the fifth root of
(c) One-seventh (d) Seven-eighths z, then x varies as the nth power of z, where n is:
[Based on REC Tiruchirapalli, 2002] 1 5
(a) (b)
15 3
31. To win an election, a candidate needs three-fourths of
the votes cast. If, after two-thirds of the votes have been 3
(c) (d) 15
counted, a candidates has five-sixths of what he needs, 5
then what part of the remaining ratio does he still need? [Based on FMS, 2011]

M07_KHAT6981_C07.indd 10 6/16/2015 5:26:40 PM


Ratio and Proportion 7.11

39.
Two persons are climbing up on two moving escalators 46.
The ratio of the prices of two houses A and B was 4:5 last
which have 120 steps. The ratio of 1st person’s speed year. This year, the price of A is increased by 25 per cent
to that of 1st escalator is 2:3 (steps). The ratio of 2nd and that of B by `50,000. If their prices are now in the
person’s speed to that of 2nd escalator is 3:5 (steps). Find ratio 9:10, the price of A last year was:
the total number of steps they both have taken together. (a) `3,60,000 (b) `4,50,000
(a) 85 (b) 93 (c) `4,80,000 (d) `5,00,000
(c) 80 (d) 75 [Based on MAT, 1998]
[Based on SNAP, 2007]
47.
Two numbers are in the ratio 5:4 and their difference is 10.
40.
A driver’s income consists of his salary and tips. During What is the larger number?
one week, his tips were five-fourths of his salary. What
(a) 30 (b) 40
fraction of his income came from tips?
(c) 50 (d) 60
(a) Four-ninths (b) Five-ninths
[Based on MAT, 1998]
(c) Five-eighths (d) Five-fourths
[Based on MAT, 2000] 48.
If A gets 25 per cent more than B and B gets 20 per cent
more than C, the share of C out of a sum of `7400 is:
41.
A man ordered 4 pairs of black socks and some pairs of (a) `3000 (b) `2000
brown socks. The price of a black pair is double that a
brown pair. While preparing the bill, the clerk interchanged (c) `2400 (d) `3500
the number of black and brown pairs by mistake which [Based on MAT, 1998]
increased the bill by 50 per cent. The ratio of the number
of black and brown pairs of socks in the original order was: 49.
A, B, C and D have `40, 50, 60 and 70 respectively when
they go to visiit a fair. A spends `18, B spends `21, C
(a) 4:1 (b) 2:1 spends `24, and D spends `27. Who has done the highest
(c) 1:4 (d) 1:2 expenditure proportionate to his resources?
[Based on MAT, 1999] (a) A (b) B
42.
If P varies as QR and the three corresponding values of (c) C (d) D
P, Q, R be 6 ,9, 10 respectively, then the value of P, when
[Based on MAT, 1998]
Q = 5 and R = 3, is:
(a) 3 (b) 2 50.
The total emoluments of A and B are equal. However, A
(c) 1 (d) 4 gets 65 per cent of his basic salary as allowances and B
[Based on MAT, 1999]
gets 80 per cent of his basic salary as allowances. What is
the ratio of the basic sala­ries of A and B?
43.
Two numbers are such as the square of one is 224 less than (a) 16:13 (b) 5:7
8 times the square of the other. If the numbers be in the
ratio of 3:4, their values are: (c) 12:11 (d) 7:9
[Based on MAT, 1997]
(a) 12, 16 (b) 6, 8
(c) 9, 12 (d) None of these 51.
A contractor employed 25 labourers on a job. He was
[Based on MAT, 1999] paid `275 for the work. After retaining 20 per cent of
this sum, he distributed the remaining amount amongst
x2 + 7 y 2 + 9 z 2 the labourers. If the number of men to women labourers
44.
If x:y:z::1:3:5, then the value of is: was in the ratio 2:3 and their wages in the ratio 5:4, what
x
wages did a woman labourer get?
(a) 7 (b) 17
(a) `10 (b) `8
(c) 13 (d) 1
(c) `12 (d) `15
[Based on MAT, 1999]
[Based on MAT, 1997]
45.
An amount of money is to be distributed among A, B and
C in the ratio 3:1:5. The difference between B’s and C’s 52.
Railway fares of 1st, 2nd and 3rd classes between two
shares is `3600. What is the total of A’s and B’s shares? stations were in the ratio of 8:6:3. The fares of 1st
and 2nd class were subsequently reduced by 1/6 and
(a) `5400 (b) `3600
1/12, respectively. If during a year, the ratio between
(c) `2700 (d) `1800 the passengers of 1st, 2nd and 3rd classes was 9:12:26
[Based on MAT, 1999] and total amount collected by the sale of tickets was

Chapter 07.indd 11 6/4/2015 5:30:33 PM


7.12 Chapter 7

`1088, then find the collection from the passengers of 59.


Men, women and children are employed to do a work
1st class. in the proportion of 1:2:3 and their wages are in the
(a) `260 (b) `280 proportion of 6:3:2. When 50 men are employ­ed, total
wages of all amount to `4500. What is the weekly wages
(c) `300 (d) `320
paid to a man, a woman and a child, in rupees?
[Based on MAT, (Dec), 2006]
(a) 210, 105, 80 (b) 210, 105, 70
53.
Ajay, Aman, Suman and Geeta rented a house and agreed (c) 210, 105, 90 (d) 200, 105, 70
to share the rent as follows: [Based on MAT (Feb), 2011]
Ajay:Aman = 8:15,
Aman:Suman = 5:8 and 60.
One year ago, the ratio between Mahesh’s and Suresh’s
Suman:Geeta = 4:5. salaries was 3:5. The ratio of their individual salaries of
The part of rent paid by Suman will be: last year and present year are 2:3 and 4:5 respectively. If
(a) 24/77 (b) 13/66 their total salaries for the present year are `43000, what is
the present salary of Mahesh?
(c) 12/55 (d) 13/77
[Based on MAT (Dec), 2007] (a) `19000 (b) `18000
54.
Mira’s expenditure and savings are in the ratio 3:2. Her (c) `16000 (d) `15500
income increases by 10% Her expenditure also increases [Based on MAT (Feb), 2011]
by 12 per cent. By how much per cent do her savings
61.
The first, second and third class fares between two stations
increase?
were 10:8:3 and the number of first, second and third class
(a) 7% (b) 9% passengers between the two stations in a day was 3:4:10.
(c) 10% (d) 13% The sale of tickets to passengers running between two
[Based on MAT (Dec), 2008] stations on that day was `8050. How much was realized
by the sale of second class tickets?
55.
The ratio between the number of passengers travelling by
I and II classes between the two railway stations is 1:50, (a) `3000 (b) `2800
whereas the ratio of I and II classes fares between the (c) `4500 (d) `3500
same stations is 3:1. If on a particular day, `1325 were [Based on MAT (Feb), 2011]
collected from the passengers travelling between these
stations, then what was the amount collected from the II 62.
A person distributes his pens among four friends A, B, C
class passengers? and D in the ratio 1/3:1/4:1/5:1/6. What is the minimum
number of pens that the person should have?
(a) `750 (b) `1000
(a) 65 (b) 55
(c) `850 (d) `1250
(c) 23 (d) 57
[Based on MAT (Dec), 2008, (May), 2007]
[Based on MAT (Dec), 2010]
56.
What should be subtracted from 15, 28, 20 and 38 so that
the remaining numbers may be proportional? 63.
At the start of a seminar, the ratio of the number of
male participants to the number of female participants
(a) 6 (b) 4
was 3:1. During the tea break, 16 male participants left
(c) 2 (d) None of these and 6 more female participants registered. The ratio
[Based on MAT (May), 2009, (Feb), 2008] of the male to the female participants became 2:1. The
57.
The sum of money is to be divided amongst A, B and C in total number of participants at the start of the seminar
the respective ratio of 3:4:5 and another sum of money is was:
to be divided between E and F equally. If F got `1050 less (a) 112 (b) 48
than A, how much amount did B receive? (c) 54 (d) 72
(a) `750 (b) `2000 [Based on MAT (Sept) 2009, (May) 2003, (Dec) 2002]
(c) `1500 (d) Cannot be determined
64.
The prime cost of an article is three times the value of the
[Based on MAT (May), 2009] raw material used. The cost of raw materials increases
58.
The number that must be added to each of the numbers 8, in the ratio of 5:12 and manufacturing expenses in the
21, 13 and 31 to make the ratio of first two numbers equal ratio 4:5. The article, which originally cost `6, will new
to the ratio of last two numbers is: cost:
(a) 7 (b) 5 (a) `10 (b) `17
(c) 9 (d) None of these (c) `20.50 (d) None of these
[Based on MAT (Sept), 2009] [Based on MAT (Dec), 2010]

Chapter 07.indd 12 6/4/2015 5:30:33 PM


Ratio and Proportion 7.13

65.
The sum of the reciprocals of the ages of two brothers is (a) 30 (b) 45
five times the difference of the reciprocals of their ages. (c) 25 (d) 55
If the ratio of the product of their ages to the sum of their [Based on MAT, 2011]
ages is 14.4:1, find their ages.
72. There are a total of 43800 students in 4 schools of a city.
(a) 36 and 24 years Half the number of students of the first school, two-third
(b) 24 and 20 years of the second, three-fourth of the third and four-fifth of
(c) 18 and 15 years the fourth are all equal. What is the ratio of the number of
students of A to D, if A, B, C, and D be the first, second,
(d) 12 and 9 years
third, and fourth schools, respectively?
[Based on MAT (Dec), 2010]
(a) 8:5 (b) 1:3
66.
Three numbers A, B and C are in the ratio of 12:15:25. If (c) 2:3 (d) 7:9
sum of these numbers is 312, ratio between the difference [Based on MAT, 2012]
of B and A and the difference of C and B is:
73. Two companies A and B quote for a tender. On the tender
(a) 3:7 (b) 10:3
opening day, A realizes that the two quotes are in the ratio
(c) 3:10 (d) 5:1 7:4 and hence decreases its price during negotiations
[Based on MAT (Sept), 2010] to make it `1 lakh lower than B’s quoted price. B then
67.
A man left one-half of the property for his wife. One- realises that the final quotes of the two were in the ratio
third to his son and the remainder to his daughter and her 3 : 4. By how much did A decrease its price in order to win
share was worth `45000, how much money did the man the bid?
leave? (a) `7 lakhs (b) `4 lakhs
(a) `245000 (b) `260000 (c) `9 lakhs (d) None of these
(c) `270000 (d) `275000 [Based on MAT, 2012]
[Based on MAT (Sept), 2009] 74. Ratio of the earnings (in `) of A and B is 4:7. If the
68.
A man ordered 4 pairs of black socks and some pairs of earnings of A increase by 50% and those of B decrease
brown socks. The price of a black pair is double that of by 25%, the new ratio of their earnings becomes 8:7. How
a brown pair. While preparing the bill, the clerk did a much is A earning?
mistake and interchanged the number of black and brown (a) `28000 (b) `21000
pairs. This increased the bill by 50 per cent. The ratio (c) `26000 (d) Data inadequate
of the number of black and brown pairs of socks in the [Based on MAT, 2012]
original order was:
75. One-fourth of sixty percent of a number is equal to two-
(a) 4:1 (b) 2:1 fifth of twenty percent of another number. What is the
(c) 1:4 (d) 1:2 respective ratio of the first number to the second number?
[Based on MAT (Feb), 2010, (Jan), 1999]
(a) 8:15 (b) 5:9
69.
Three friends Anita, Bindu and Champa divided `1105 (c) 8:13 (d) 4:7
amongst them in such a way that if `10, `20 and `15 are [Based on MAT, 2012]
removed from the sums that Anita, Bindu and Champa
76. A sum of `53 is divided among A, B and C in such a way
received respectively, then the share of the sums that
that A gets `7 more than what B gets and B gets `8 more
they got will be in the ratio of 11:18:24. How much did
than what C gets. The ratio of their shares is:
Champa receive?
(a) 15:8:30 (b) 18:25:10
(a) `495 (b) `510
(c) 25:18:10 (d) 16:18:10
(c) `480 (d) `375
[Based on MAT, 2012]
[Based on MAT (Feb), 2010]
77. The ratio of efficiency of A is to C is 5:3. The ratio of
70. A, B, C together earn `1450 and spend 60%, 65% and number of days taken by B is to C is 2:3. A takes 6 days
70% of their salaries respectively. If their saving are in the less than C, when A and C complete the work individually.
ratio 14:21:15, the salary of B is: B and C started the work and left after 2 days. The
(a) `500 (b) `600 number of days taken by A to finish the remaining work
(c) `450 (d) `750 is:
[Based on MAT, 2011] (a) 4.5 days (b) 5 days
71. `9000 were divided equally among a certain number of (c) 6 days
1
(d) 9 days
person. Had there been 20 more persons, each would have 3
got `160 less. The original number of persons were: [Based on MAT, 2013]

Chapter 07.indd 13 6/4/2015 5:30:33 PM


7.14 Chapter 7

78. A student is studying for a test from 11:00 am to 08:00 pm 83. `73,689 are divided between A and B in the ratio 4:7.
on weekdays and one-third of that on Saturdays. On What is the difference between thrice the share of A and
Sundays he takes a break from school and goes fishing. twice the share of B?
For what fractional part of the entire week is the student (a) `36,699 (b) `46,893
studying? (c) `20.097 (d) `13.398
(a) 2/5 (b) 3/7
[Based on SNAP, 2013]
(c) 2/7 (d) 3/8
[Based on CAT, 2013] 84. Profit earned by an organization is distributed among
the officers and clerks in the ratio of 5:3. If the number
79. The number of balls in three baskets are in the ratio of of officers is 45 and the number of clerks is 80 and the
3:4:5. In which ratio the number of balls in first two amount received by each officer is `25,000, what was the
baskets must be increased so that the new ratio becomes total amount of profit earned?
5:4:3? (a) `22 lakhs (b) `18.25 lakhs
(a) 1:3 (b) 2:1 (c) `18 lakhs (d) `23.25 lakhs
(c) 3:4 (d) 2:3 [Based on SNAP, 2013]
[Based on CAT, 2013]
85. Mrs. X spends `535 in purchasing some shirts and ties for
80. Two numbers are in the ratio P:Q. When 1 is added to her husband. If shirts cost `43 each and the ties cost `21
both the numerator and the denominator, the ratio gets each, then what is the ratio of the shirts to the ties, that are
changed to R/S. Again, when 1 is added to both the purchased?
1 (a) 1:2 (b) 2:1
numerator and the denominator, it becomes . Find the
2 (c) 2:3 (d) 3:4
sum of P and Q.
[Based on SNAP, 2013]
(a) 3 (b) 4
(c) 5 (d) 6 86. In an express train, the numbers of passengers travelling
in A.C. Sleepers class, 1st Class and Sleeper Class are
[Based on CAT, 2014]
in the ratio 1:2:3, and the fares to each of these classes
81. Kajal spends 55% of the monthly income on grocery, are in the ratio 5:4:2. If the total income from the train is
`54000, then the income from the A.C. sleeper class is:
clothes and education in the ratio of 4:2:5 respectively.
If the amount spent on clothes is `5540, what is Kajal’s (a) `8000 (b) `12000
monthly income? (c) `14210 (d) None of these
(a) `55,400 (b) `54,500 [Based on SNAP, 2012]
(c) `55,450 (d) `55,650 αω
87.
Consider the formula, S= , where all the
[Based on SNAP, 2013] τ + ρω
82. A certain amount was to be distributed among A, B and parameters are positive integers. If ω is increased and a , τ
C in the ratio 2:3:4 respectively, but was erroneously and ρ are kept constant, then S:
distributed in the ratio 7:2:5 respectively. As a result of (a) Increases
this, B got ` 40 less. What is the amount? (b) Decreases
(a) `210 (b) `270 (c) Increases and then decreases
(c) `230 (d) `280 (d) Decreases and then increases
[Based on SNAP, 2013] [Based on XAT, 2014]

Chapter 07.indd 14 6/4/2015 5:30:34 PM


Ratio and Proportion 7.15

Difficulty Level-2
(Based on Memory)

1.
Let a, b, c, d and e be integers such that a = 6b = 12c, and (a) `140 (b) `160
2b = 9d = 12e. Then, which of the following pairs contain (c) `144 (d) None of the above
a number that is not an integer?
7. A factory employs skilled workers, unskilled workers and
 a b  a c
(a)  ,  (b)  ,  clerks in the proportion 8:5:1 and the wage of a skilled
 27 e   36 e  worker, an unskilled worker and a clerk are in the ratio
5:2:3. When 20 unskilled workers are employed, the total
 a bd  a c daily wages of all amount to `3180. Find the daily wages
(c)  ,  (d)  , 
 12 18  6 d paid to each category of employees.
[Based on CAT, 2004] (a) 2100, 800, 280 (b) 2400, 480, 300
2. A man fell in love with a woman who lived 63 miles (c) 2400, 600, 180 (d) 2200, 560, 420
away. He decided to propose his beloved and invited her
to travel to his place and offered to meet her en route and 8.
The soldiers in two armies when they met in a battle were
bring her home. The man is able to cover 4 miles per hour in the ratio of 10:3. Their respective losses were as 20:3
to the woman’s 3 miles per hour. How far will each have and the survivors as 40:13. If the number of survivors in
travelled upon meeting? the larger army be 24,000, find the original number of
soldiers in army.
(a) Man = 27 miles; woman = 36 miles
(a) 28000, 8400 (b) 25000, 7500
(b) Man = 36 miles; woman = 27 miles
(c) 29000, 2750 (d) 26000, 7800
(c) Man = 40 miles; woman = 23 miles
(d) Man = 45 miles; woman = 18 miles 9.
What must be added to each of the numbers 7, 11 and
19, so that the resulting numbers may be in continued
3. The price of branded PC (personal computer) and
proportion?
assembled PC is in the ratio of 64:27. If from now on,
every year price of branded PC goes on decreasing by 10 (a) 3 (b) 5
per cent and price of assembled PC goes on increasing by (c) 4 (d) –3
20 per cent, how after many years the price of both will be
equal? 10. At Narmada Sarovar Bachao demonstration, supporters
of Ms. Patkar outnumbered the police by 9:1. The police
(a) 2 years (b) 3 years arrested 135 NSB supporters averaging 5 for every 3
1 1 policemen. How many supporters of NSB were there in
(c) 3 years (d) 2 years the demonstration?
3 2
4. A certain product C is made of two ingredients A and B in (a) 405 (b) 665
the proportion of 2:5. The price of A is three times that of (c) 1215 (d) None of the above
B. The overall cost of C is `5.20 per kg including labour [Based on FMS (Delhi), 2004]
charges of 80 paisa per kg. Find the cost of B per kg?
11. The intensity of illumination on a surface from a source of
(a) `8.40 (b) `4.20 light varies inversely as the square of the distance of the
(c) `4.80 (d) `2.80 surface from the source. The effect of moving a piece of
paper 3 times as far from the source is to:
5. A sum of `430 has been distributed among 45 people
consisting of men, women and children. The total (a) Divide the intensity by 3
amounts given to men, women and children are in the (b) Multiply the intensity by 3
ratio 12:15:16. But, the amounts received by each man, (c) Divide the intensity by 9
woman and child are in the ratio 6:5:4. Find, what each (d) Multiply the intensity by 9
man, woman and child receives (in `). [Based on REC Tiruchirapalli, 2003]
(a) 12, 10, 8 (b) 18, 15, 12
(c) 120, 150, 160 (d) 60, 75, 80 12.
Suppose y varies as the sum of two quantities of which
one varies directly, as x and the other varies inversely as
6. The total salary of A, B, C is `444. If they spend 80%, 1
x. If y = 6 when x = 4 and y = 3 when x = 3, then the
85%, 75% of their salaries, respectively, their savings are 3
as 7:6:9. The salary of B is: relation between x and y is:

Chapter 07.indd 15 6/4/2015 5:30:34 PM


7.16 Chapter 7

4 4 20.
The cost of a bat increased by 10 per cent and the cost of
(a) y = x + (b) y = –2x + a ball increased by 18 per cent. Before the price rise, the
x x
ratio of the cost of the bat to the cost of the ball was 9:2.
8 8 If the cost of 12 bats and 54 balls before the price rise was
(c) y = 2x + (d) y = 2x – `C, what is their cost (in `) now?
x x
[Based on FMS (Delhi), 2003] (a) 1.12 C (b) 1.13 C
(c) 1.14 C (d) 1.15 C
13. Pressure varies inversely with volume while temperature
varies directly with volume. At a time, Volume = 50 m3, 21.
Radhika purchased one dozen bangles. One day she
Temperature = 25º K and Pressure = 1 atmosphere. If the slipped on the floor fell down. What cannot be the ratio of
volume is increased to 200 m3, then the temperature will broken to unbroken bangles?
be: (a) 1:2 (b) 1:3
(a) 100º K (b) 50º K (c) 2:3 (d) 1:5

(c) 12 K (d) 0º K 22.
A precious stone worth `6,800 is accidently dropped and
2 breaks into three pieces. The weight of three pieces are in
[Based on IIFT, 2003]
the ratio 5:7:8. The value of the stone is proportional to
2 3
14. x varies directly as y and when x = 6, y = 3. Which of the the square of its weight. Find the loss.
following equations correctly represents the relationship (a) `4,260 (b) `4,273
between x and y?
(c) `4,454 (d) `3,250
(a) 6x2 = 3y3 (b) 3y2 = 6x3
23.
A man spends `8,100 in buying tables at `1,200 each
(c) 3x2 = 2y3 (d) 3x2 = 4y3
chairs at `300 each. The ratio of chairs to tables when the
[Based on IMT Ghaziabad, 2002]
maximum number of tables is purchased is:
1 1
It is given that y ∝
15. 3
. For x = 2, value of y is . If (a) 1:4 (b) 5:7
x −x 6
(c) 1:2 (d) 2:1
x = 1, then the value of y will be:
(a) 1 (b) 0 24. From a number of mangoes, a man sells half the number
of existing mangoes plus 1 to the first customer, then sells
(c) –1 (d) None of these
one-third of the remaining number of mangoes plus 1 to
[Based on IMT Ghaziabad, 2002]
the second customer, then one-fourth of the remaining
16.
Between two stations, the first, second and third class fares number of mangoes plus 1 to the third customer and
are in the ratio 9:7:2. The number of passengers travelling one-fifth of the remaining number of mangoes plus 1 to the
in a day are in the ratio 5:3:2, respectively, in the above fourth customer. He then finds that he does not have any
classes. If the sale of tickets generated revenue of `98,000 mango left. How many mangoes did he have originally?
that day and if 200 passengers travelled by third class, (a) 12 (b) 14
what was the fare for a first class ticket? (c) 15 (d) 13
(a) `84 (b) `92 [Based on FMS (Delhi), 2002]
(c) `106 (d) `126

17.
If (x – y + z):(y – z + 2w):(2x + z – w) = 2:3:5, find the 25. The ratio between the number of passengers travelling
value of S, where S is (3x + 3z – 2w):w. z:w = 3:8 and by I and II class between the two railway stations is 1:50,
u:w = 2:5, find the value (xyu):(w2z). whereas the ratio of I and II class fares between the same
stations is 3:1. If on a particular day, `1325 were collected
(a) 7:1 (b) 6:1
from the passengers travelling between these stations, then
(c) 13:2 (d) None of these what was the amount collected from the II class passengers?
18. How many boys are studying Science? (a) `750 (b) `850
(a) 52 (b) 65 (c) `1000 (d) `1250
(c) 115 (d) None of these [Based on I.P. Univ., 2002]
[Based on IRMA, 2002]
26.
A sporting goods store ordered an equal number of
19. What is the ratio between the girls studying Arts & white and yellow tennis balls. The tennis ball company
Science respectively? delivered 45 extra white balls, making the ratio of white
(a) 13:23 (b) 26:79 1 1
balls to yellow balls : . How may white tennis balls
(c) 8:13 (d) 23:36 5 6
[Based on IRMA, 2002] did the store originally order for?

Chapter 07.indd 16 6/4/2015 5:30:34 PM


Ratio and Proportion 7.17

(a) 450 (b) 270 33. In a certain company, the ratio of the number of managers
(c) 225 (d) None of these to the number of production-line workers is 5 to 72. If 8
additional production-line workers were to be hired, the
27. Determine the ratio of the number of people having ratio of the number of managers to the production-line
characteristic X to the number of people having workers would be 5 to 74. How many managers does the
characteristic Y in a population of 100 subjects from the company have?
following table:
(a) 10 (b) 20
Having X and Y 10
(c) 30 (d) 25
Having X but not Y 30 [Based on ATMA, 2005]
Having Y but not X 20
34.
In a cricket match, Team A scored 232 runs without losing
Having neither X nor Y 40 a wicket. The score consisted of byes, wides and runs
scored by two opening batsmen: Ram and Shyam. The
(a) 4:3 (b) 3:2
runs scored by the two batsmen are 26 times wides. There
(c) 1:2 (d) 2:3 are 8 more byes than wides. If the ratio of the runs scored
[Based on REC Tiruchirapalli, 2002] by Ram and Shyam is 6:7, then the runs scored by Ram is:
Directions (Questions 28 to 30): Answer the questions (a) 88 (b) 96
based on the following information. (c) 102 (d) 112
Alphonso, on his death bed, keeps half his property for his [Based on XAT, 2008]
wife and divides the rest equally among his three sons: Ben, 35.
The number of students in three rooms is 138. The ratio
Carl and Dave. Some years later, Ben dies leaving half his of the number of students in 1st and the 2nd room is 3:4.
property to his widow and half to his brothers Carl and Dave The ratio of the number of students in 2nd and 3rd room is
together, sharing equally. When Carl makes his will, he 7:5. The number of students in the 1st, 2nd and 3rd room
keeps half his property for his widow and rest he bequeaths respectively is:
to his younger brother Dave. When Dave dies some years
later, he keeps half his property for his widow and the (a) 56, 40, 42 (b) 42, 56, 40
remaining for his mother. The mother now has `15,75,000. (c) 40, 56, 42 (d) 56, 42, 40
28.
What was the worth of the total property? [Based on IIFT, 2005]
(a) `30 lakhs (b) `8 lakhs 36.
Indiacate in which one of the following equations y is
(c) `18 lakhs (d) `24 lakhs neither directly nor inversely proportional to x :
29.
What was Carl’s original share? (a) x + y = 0 (b) 3xy = 10
(a) `4 lakhs (b) `12 lakhs (c) x = 5y (d) 3x + y = 10
(c) `6 lakhs (d) `5 lakhs [Based on FMS, 2011]

30. What was the ratio of the property owned by the widows 37.
Instead of walking along two adjacent sides of a
of the three sons in the end? rectangular field, a boy took a short-cut along the diagonal
(a) 7:9:13 (b) 8:10:15 of the field and saved a distance equal to half of the longer
(c) 5:7:9 (d) 9:12:13 side. The ratio of the shorter side of the rectangle to the
31. Fresh grapes contain 90 per cent water by weight while longer side is:
dried grapes contain 20 per cent water by weight. What
1 2
is the weight of dry grapes contain 20 per cent water by (a) (b)
weight. What is the weight of dry grapes available from 2 3
20 kg of fresh grapes? 1 3
(c) (d)
(a) 2 kg (b) 2.4 kg 4 4
(c) 2.5 kg (d) None of these [Based on FMS, 2011]
32. One year ago, the ratio between A’s and B’s salary was 38.
In counting n coloured balls, some red and some black,
4:5. The ratio of their individual salaries of last year and it was found that 49 of the first 50 counted were red.
present year are 3:5 and 2:3 respectively. If their total Thereafter, 7 out of every 8 counted were red. If, in all,
salaries for the present year is `680, the present salary of 90 per cent or more of the balls counted were red, the
A is (`): maximum value of n is:
(a) 4080.00 (b) 3200.00 (a) 225 (b) 210
(c) 4533.40 (d) 2720.00 (c) 200 (d) 180
[Based on ATMA, 2008] [Based on FMS, 2010]

Chapter 07.indd 17 6/4/2015 5:30:34 PM


7.18 Chapter 7

39.
The income distribution in the villages of Delhi 1
is symmetrical. Two surveys estimated that the (a) (b) 2
2 3
implementation of Gramin Rozgar Yozna of central
government in a village of Delhi will increase the income 1
(c) (d)
3
of every villager either: 4 4
[Based on CAT, 2002]
(i) by a certain proportion or
45. Mayank, Mirza, Little and Jaspal bought a motorbike for
(ii) by `3650. `60,000. Mayank paid one half of the sum of the amounts
Will the symmetry of income distribution be affected? paid by the other boys, Mirza paid one third of the sum of
the amounts paid by the other boys. How much did Jaspal
(a) Change in income distribution in case of (i) has to pay?
(b) Change in income distribution in case of (ii) (a) `15,000
(c) Change in income distribution in both the cases (i) (b) `13,000
and (ii) (c) `17,000
(d) No change in income distribution in both the cases (i) (d) None of these
and (ii) [Based on CAT, 2002]
[Based on FMS, 2009]
46. A piece of string is 40 cm long. It is cut into three pieces.
40.
Four milkmen rented a pasture. A grazed 18 cows for 4 The longest piece is 3 times as long as the middle-sized
months, B 25 cows for 2 months, C 28 cows for 5 months and the shortest piece is 23 cm shorter than the longest
and D 21 cows for 3 months. If A’s share of rent is `360, piece. Find the length of the shortest piece (in cm).
the total rent of the field (in rupees) is:
(a) 27 (b) 5
(a) 1500 (b) 1600
(c) 4 (d) 9
(c) 1625 (d) 1650 [Based on CAT, 2002]
[Based on FMS, 2006]
47. You can collect rubies and emeralds as many as you can.
41.
What is the ratio whose terms differ by 40 and the measure Each ruby is of `4 crore and emerald is of `5 crore. Each
of which is two-sevenths? ruby weighs 0.3 kg and emerald weighs 0.4 kg. Your bag
(a) 6:56 (b) 14:56 can carry at the most 12 kg. What you should collect to get
(c) 16:56 (d) 16:72 the maximum wealth?
[Based on FMS, 2005] (a) 20 rubies and emeralds
(b) 40 rubies

1
42. p ∝ q, q ∝ , s ∝ .
1 (c) 28 rubies and 9 emeralds
s r (d) None of these
p = 1, when q = 2, q = 3; when s = 4, s = 4; when r = 5. [Based on CAT, 1998]
Find r , when p = 6.
48. I have one rupee coins, fifty paise coins and twenty five
(a) 35 (b) 30 paise coins. The number of coins is in the ratio 2:5:3:4.
(c) 20 (d) 16 If the total amount with me is Rs 210, find the number of
[Based on CAT, 2009] one rupee coins.
(a) 90 (b) 85
43. In a T-shirt stiching factory, the approved pieces were
95% of the total production on Friday and the rejected (c) 100 (d) 105
pieces were 10% of the total production on Saturday. The [Based on CAT, 1998]
overall rejection rate for the two days combined works
49. The cost of diamond varies directly as the square of its
out to be 8.33% What was the ratio of the production of
weight. Once, this diamond broke into four pieces with
Friday to the production of Saturday?
weights in the ratio 1:2:3:4. When the pieces were sold,
(a) 1:2 (b) 2:1 the merchant got `70,000 less. Find the original price of
(c) 1:3 (d) 1:1.75 the diamond.
[Based on CAT, 2009] (a) `1.4 lakhs
44. Instead of walking along two adjacent sides of a rectangular (b) `2 lakhs
field, a boy took a short cut along the diagonals and saved (c) `1 lakh
a distance equal to half the longer side. Then, the ratio of (d) `2.1 lakhs
the shorter side to the longer side is: [Based on CAT, 1996]

Chapter 07.indd 18 6/4/2015 5:30:35 PM


Ratio and Proportion 7.19

50. Two oranges, three bananas and four apples cost `15. 52. Four numbers are in proportion .The sum of the squares of
Three oranges, two bananas and one apple cost `10. I the four numbers is 50 and the sum of the mean is 5. The
bought 3 oranges, 3 bananas and 3 apples. How much did ratio of first two terms is 1:3. What is the average of the
I pay? four numbers?
(a) `10 (a) 1 (b) 3
(b) `8 (c) 5 (d) 6
(c) `15 [Based on MAT, 2012]
(d) Cannot be determined
[Based on CAT, 1993]
53. A and B quote for a tender. On the tender opening day,
A realizes that their quotes are in the ratio 7:4 and hence
51. From each of two given numbers, half the smaller number decreases its price during negotiations to make it `1 lakh
is subtracted. Of the resulting numbers, the larger one is lower than B’s quoted price. B then realizes that the final
three times as large as the smaller. What is the ratio of the quotes of the two were in the ratio 3:4. What was the price
two numbers? at which B won the bid?
(a) 2:1 (b) 3:1 (a) `7 lakhs (b) `4 akhs
(c) 3:2 (d) None of these (c) `3 lakhs (d) `1 lakh
[Based on CAT, 1993] [Based on MAT, 2013]

Answer Keys
Difficulty Level-1

1. (d ) 2. (d ) 3. (c) 4. (d ) 5. (c) 6. (b) 7. (d ) 8. (d ) 9. (a) 10. (c) 11. (a) 12. (c) 13. (d )
14. (b) 15. (a) 16. (c) 17. (c) 18. (c) 19. (c) 20. (b) 21. (a) 22. (a) 23. (c) 24. (d ) 25. (d ) 26. (c)
27. (c) 28. (d ) 29. (c) 30. (c) 31. (d ) 32. (b) 33. (c) 34. (c) 35. (a) 36. (d ) 37. (a) 38. (c) 39. (b)
40. (b) 41. (c) 42. (c) 43. (b) 44. (b) 45. (b) 46. (a) 47. (c) 48. (b) 49. (a) 50. (c) 51. (b) 52. (d)
53. (a) 54. (a) 55. (d ) 56. (c) 57. (d ) 58. (b) 59. (b) 60. (b) 61. (b) 62. (d ) 63. (a) 64. (d ) 65. (a)
66. (c) 67. (c) 68. (c) 69. (a) 70. (b) 71. (c) 72. (a) 73. (b) 74. (d ) 75. (a) 76. (c) 77. (c) 78. (c)
79. (b) 80. (c) 81. (a) 82. (a) 83. (d ) 84. (d ) 85. (b) 86. (c) 87. (a)

Difficulty Level-2

1. (d ) 2. (b) 3. (b) 4. (d ) 5. (a) 6. (b) 7. (c) 8. (a) 9. (d ) 10. (d ) 11. (c) 12. (d ) 13. (a)
14. (d ) 15. (d ) 16. (d ) 17. (a) 18. (d ) 19. (a) 20. (c) 21. (c) 22. (c) 23. (c) 24. (b) 25. (d ) 26. (c)
27. (a) 28. (d ) 29. (a) 30. (b) 31. (c) 32. (b) 33. (b) 34. (b) 35. (b) 36. (d ) 37. (d ) 38. (b) 39. (b)
40. (c) 41. (c) 42. (c) 43. (a) 44. (d ) 45. (b) 46. (c) 47. (b) 48. (d ) 49. (c) 50. (c) 51. (a) 52. (b)
53. (d )

Chapter 07.indd 19 6/4/2015 5:30:35 PM


7.20 Chapter 7

Explanatory Answers

Difficulty Level-1

1. (d) Radii of the two pipes are 1 cm and 2 cm. 2x


Squares of the radii of the two pipes are 1 cm and 4 cm. \ = 100
5
\ Rates of flow of the two pipes are in the ratio ⇒ x = 250.
1
1: , i.e., 4:1.
4 2
7.
(d) After first day, rd of the contents remain.
2. (d) Number of girls = 100 3
\ Number of boys = 300 2 3 2 2 1 1
After second day, −   = – = of the
\ Number of hostel dwellers = 50 + 100 = 150 3 4 3 3 2 6
⇒ Required ratio = 150:400 contents remain.
3
= 3:8 = . 8. (d) Let the incomes of the two persons be 4I and 5I
8 respectively.
3.
(c) Total number of boys in the class = 24 Let the expenditure of the two persons be 7E and 9E
Total number of girls in the class = 12 respectively.
Total strength of the class = 36. \ 4I – 7E = 50 and 5I – 9E = 50
⇒ 20I – 35E = 250
4.
(d) The box to paper ratio is 3:22. To arrives the weight
20I – 36E = 200
of the box, we first divide 36000 by 25 to get 1440.
⇒ E = 50 and I = 100
Multiplying this by 3 we get 4320 which is the weight
\ Monthly incomes of the two persons are `400 and
of the box. Therefore, weight of the paper is
`500 respectively.
36000 – 4320 = 31,680 kg.
5.
(c) Given, 4A = 5B = 7C 9. (a) Let, initially, the number of males and females in the
A B C bus be 3x and x respectively.
⇒ = = At the first stop, suppose m males and f females left
35 28 20
the bus.
So, A:B: = 35:28:20 \ At the first stop: No. of the Males is 3x – m
35 and No. of females
∴ A’s Runs = × 581 = 245
(35  28  20) (x – f) + 6
20 3x − m 2
and, C’s runs =
× 581 = 140 \ =
83 (x − f ) + 6 1
Thus, difference between A’s score and B’s score ⇒ 3x – m = 2x – 2f + 12
= (245 – 140) = 105. Also m + f = 16
6.
(b) Suppose the amount in the beginning was `x ⇒ x = 28 – 3f
1 \ f = 4, x = 16
Money spent on clothes = ` x \ Total number of passengers in the beginning
3
2 = 4x = 64
Balance = ` x
3 For no other value of f any of the other alternatives
1 2 2 holds good.
Money spent on food = of x = ` x
5 3 15 10. (c) Let number of boys = x
2 2 8x Let number of girls = y
Balance = x − x = `
3 15 15 x y
1 8x 2x \ = B and =G
Money spent on travel = of =` y x
4 15 15
x y
8x 2x \ 3 (B + G) = 3  + 
= − y x
15 15
6x 2x 3( x 2 + y 2 )
= =` = >3.
15 5 xy

Chapter 07.indd 20 6/4/2015 5:30:36 PM


Ratio and Proportion 7.21

a 2 19.
(c) Let, X = K/(Y2 – 1)
11.
(a) =
b 5 Now, 24 = K/(100 – 1)
a
2 +3 or, K = 24 × 99. X = K/(Y2 – 1)
2a + 3b b
= When, Y = 5, X = 24 × 99/(25 – 1) = 99.
7 a + 5b a
7 +5
b 20.
(b) Let the number be x
4
+3 x x 5
5 19 \ Required ratio = = = .
=
14 = . 1 6x 6
+5 39 x+ x
5 5 5
21.
(a) Man’s age = 4k, say
12. (c) Let the two numbers be 6K and 13K.
Wife’s age = 3k, say
L.C.M. of 6K and 13K = 78K
\ 78K = 312 4k + 4 9
\ = ⇒ k = 8.
⇒ K = 4 3k + 4 7
\ Sum of the numbers
\ Man’s age = 32 years
= 6K + 13K = 19K = 76.
Wife’s age = 24 years
13.
(d) Since the ratio is 20:1, the number of boys can be 20, Suppose they were married x years ago.
40, 60, ... and number of girls can be 1, 2, 3, . . .
32 − x 5
Thus, as the number of students is not known, we \ = ⇒ x = 12.
cannot say for sure the number of girls to be required. 24 − x 3
Hence, data is insufficient. 22.
(a) Let X’s income be 3k then Y’s income is 4k. Let, X’s
14.
(b) Let x and y be the two numbers expenditure be 4g then Y’s expenditure is 5g.
x 2 x+8 3 But 3k = 9/10 (5g) or, k = 3/2g
\ = , =
y 3 y +8 4 ⇒ X’s saving/Y’s saving = (3k – 4g)/(4k – 5g)
⇒ x = 16, y = 24.
3(3 / 2 g )  4 g 1
⇒ =
2 4(3 / 2 g )  5 g 2
15.
(a) A = (B + C )
9 ⇒ X’s saving:Y’s saving = 1:2.
A + B + C = 770
9A 23.
(c) Given x:y = 1:2,  therefore y = 2x
⇒ A + = 770
2  8
⇒ 11A = 770 × 2 Similarly, z = 3y = 6x and w =   z = 16x
 3
⇒ A = 140.
 2  32 
and, x =   w    x
14 − x 34 − x  5  5
16.
(c) =
17 − x 42 − x
Therefore,  xyu   1 .
⇒ 588 – 56x + x2 = 578 – 51x + x2  w2 z  120
⇒  x = 2.
24. (d) 50T + 40C = 500
17.
(c) If X be the required number, then T + C = 12
(45  X ) = 3  (5  22) ⇒ C = 10, T = 2
\ Ratio of the number of chairs and tables = 5:1.
or, 45X = 9 × 110 or, X = 22.
25.
(d) x ∝ 3y + 1
18.
(c) If A and B are mean & third proportional, then ⇒ x = K (3y + 1)
X/A = A/Y Put x = 9, y = 1,
or, A = ( XY ) and X/Y = Y/B 9
2
we get, K =
or, B = Y /X 4
Now ( XY ) = n × Y2/X 9
\ x =  (3y + 1)
3/2 4
or, (X/Y) = n/1
or, X/Y = n2/3:1. \ When, y = 5, x = 36.

Chapter 07.indd 21 6/4/2015 5:30:37 PM


7.22 Chapter 7

x 1 x+7 3 120
26.
(c) = , = 32.
(b) 30% of I + II = II ×
y 2 y+7 5 100

⇒ x = 14, y = 28. 3 12
or, I + II = II
AC 3 10 10
27.
(c) =
W 2
3 2
W = AC – 6000 or, I = II
10 10
3W
⇒ W = − 6000 ⇒ I:II = 2:3.
2
⇒ 2W = 3W – 12000 33.
(c) It cannot be determined because the total money to be
distributed is not given.
⇒ W = 12000.

28.
(d) 2 leaps of the dog = 5 leaps of the hare, or 1 leap of the 34.
(c) Suppose 18 kg each is melted. Ratio of gold and
dog = 2.5 leaps of hare copper in one alloy will be 14:4 and in another 7 : 11.
\ Ratio of gold and copper in the new alloys
∴ 6 leaps of dog = 15 leaps of hare. Hence, ratio of
leaps of dog to hare = 15:4. C = 14 + 7:4 + 11 = 21:15 = 7:5.

29.
(c) 1:10 ⇒ There are atleast 11 marbles in the jar. 35. (a) Total amount of A and C
30.
(c) When there are eight people, the share of each person 2580
= × (3 + 9)
1 (7 − 3)
is of the total cost.
8
2580
When there are seven people, the share of each person = × 12 = 27540.
4
1
is of the total cost.
7 36. (d) We do not know the average price of two flours.
\ Increase in the share of each person 37.
(a) Let the value of x, y and z be 2k, 3k and 5k respectively.
1 1 1 1 1 1 Sum of x, y and z will be
= − = , i.e., of , i.e.,
7 8 56 7 8 7 2k + 3k + 5k = 100
of the original share of each person. 10k = 100
⇒ k = 10
31.
(d) Suppose total votes = x
So, the numbers are 20, 30 and 50.
3 3
To win a candidate required = ×x= x y = ax – 10
4 4
⇒ 30 = a × 10 – 10
2 2
When × x = x votes were counted, a candidate has ⇒ 10a = 20
3 3
a = 2.
5 3 5
× x = x 38.
(c) X varies as the cube of Y and Y varies as the fifth
6 4 8
root of Z.
So now he needed
X ∝ Y3 and Y ∝ Z1/5
3 5 6x − 5x x
x − x = = X = K1 Y 3 and Y = K2 Z1/5
4 8 8 8
Votes out of remaining X = K1(K2 Z1/5)3

2 x ⇒ X = K1(K2)3 Z 3/5
x – x =
3 3 X = K3 Z 3/5
x 3 3 3
\ required ratio = × = . As X varies as the nth power of Z, so, n = .
8 8 8 5

Chapter 07.indd 22 6/4/2015 5:30:37 PM


Ratio and Proportion 7.23

Also y2 = 8x2 – 224


 2 120 
39.
(b) Steps for 1st person =  
3 1 2  ⇒ 16K2 = 72K2 – 224
 
 3 ⇒ K = 2
3 2
= 120   = 48 Therefore, the numbers are 6 and 8.
5 3
Steps for 2nd person x y z
44.
(b) = = = k, say
1 3 5
3 120
=  \ x = k, y = 3k, z = 5k
5 1 3
5
x2 + 7 y 2 + 9 z 2 k 2 + 7 × 9k 2 + 9 × 25k 2
\ =
3 5 x k
= × 120 × = 45
5 8
= 289 = 17 .

\ Total steps taken together = 48 + 45 = 93.
45.
(b) Suppose A, B and C get `3K, `K and `5K, respectively.
40.
(b) Let salary of the driver be `k. Therefore, income from
\ 5K – K = 3600
5
tips = ` k ⇒ K = 900
4
\ A’s share = `2,700
5 9
Therefore, total income = k + k= k B’s share = `900
4 4
C’s share = `4,500
5 9k 5k
⇒ of = \ Required total = `3,600.
9 4 4
46.
(a) Let the price of the two houses A and B be 4k and 5k,
5
⇒ of total income = Income from tips. respectively.
9
According to the question,
41.
(c) Let x pairs of brown socks were ordered.
4k + 25% of 4k 9
Let p be the price of a brown pair. =
50000 10
\ 2p is the price of a black pair.
5k 9
\ (2p × 4 + px) + 50% of (2p × 4 + px) ⇒ =
5k + 50000 10

= 2p × x + 4p
⇒ k = 90000
⇒ x = 16
\ Price of house A = `360000.
\ Required ratio = 1:4.
47.
(c) Let the number be 5k and 4k.
42.
(c) P a QR, P = 6, Q = 9, R = 10.
\ 5k – 4k = 10
⇒ P = KQR,
⇒ k = 10
where K is the constant of proportionality
⇒ Largest number = 50.
⇒ 6 = K × 9 × 10
48.
(b) Suppose the share of C = `100
1
⇒ K = 100 + 20
15 \ Share of B = `100 ×
120
⇒ 15P = QR
120
When Q = 5 and R = 3, then P = 1. = `100 × = `120
100
43.
(b) Let the numbers be x and y. 100 + 25
Share of A = `120 ×
x 3 x y 100
\ = ⇒ = = K, say
y 4 3 4 125
`120 × = `150
\ x = 3K, y = 4K. 100

Chapter 07.indd 23 6/4/2015 5:30:38 PM


7.24 Chapter 7

\ Ratio of shares of A, B, C = 150:120:100 ⇒ Ratio of amount collected


= 15:12:10 = 40 × 9:12 × 33:26 × 18
Sum of ratios = 15 + 12 + 10 = 37
= 90:99:117
Total amount = `7400
Amount collected from 1st class fares
7400
Hence, share of C = ` × 10 = `2000. 90
37 = × 1088
306
49.
(a) Percentage of A’s expenditure
18 = `320.
= × 100 =45%
40
Percentage of B’s expenditure 53.
(a) Ajay:Aman = 8:15

21 Aman:Suman = 5:8
= × 100 = 42%
50 Suman:Geeta = 4:5
Percentage of C’s expenditure \ Ajay:Aman:Suman:Geeta
24 = 8:15:24:30
= × 100 = 40%
60
\  Part of rent paid by Suman
Percentage of D’s expenditure
4 24 24
27 = = .
= × 100 = 38 % 8 + 15 + 24 + 30 77
70 7
Hence, A’s expenditure is the highest. 54.
(a) Let expenditure be `60 and savings be `40.
50.
(c) Suppose that the basic salaries of A and B be x and y Total income = `100
respectively. New income = `110
\ x + 65% of x = y + 80% of y New expenditure = `67.2
65 80 New savings = 110 – 67.2 = `42.8
⇒ x + x = y+ y
100 100
\ Percentage increase in savings

⇒ 165 x = 180 y
2.8
x 180 12 = × 100 =
7%
⇒ = = . 40
y 165 11
55.
(d) Let the number of passengers travelling by Class I and
51.
(b) Suppose the wages of each man = `5K and wages of Class II be x and 50 x respectively.
each woman = `4K Then, amount collected from Class I and Class II will
2 be `3x and `50 x respectively.
Number of men = × 25 = 10
5 Given, 3x + 50x = 1325
3 ⇒ 53x = 1325
Number of women = × 25 = 15
5 ⇒ x = 25
Now `220 are to be divided among 10 men and 15 \ Amount collected from Class II = 50 × 25 = `1250.
women.
56.
(c) Let x should be subtracted from each number.
\ 10 × 5K+ 15 × 4K = 220
15 − x 20 − x
⇒ 110K = 220 Then, =
28 − x 38 − x
⇒ K = 2.
⇒ 570 – 38x – 15x + x2 = 560 – 28x – 20x + x2
52.
(d) New ratio of fares (1st, 2nd and 3rd)
⇒ 570 – 53x = –48x + 560
5 11
= 8 × : 6 × : 3 × 1 ⇒ x = 2.
6 12
57.
(d) As the sum of money that are to be divided among A,
= 80:66:36 = 40:33:18
B and C and between E and F are not given. So, the
Ratio of passengers = 9:12: 26 amount that B receive cannot be determined.

Chapter 07.indd 24 6/4/2015 5:30:39 PM


Ratio and Proportion 7.25

58.
(b) Let that number be x. 1 1 1 1
62.
(d) : : : = 20:15:12:10
8+ x 13 + x 3 4 5 6
Then, =
21 + x 31 + x So, the minimum number of pens that the person
should have = 20 + 15 + 12 + 10 = 57.
⇒ 248 + 31x + 8x + x2
63.
(a) Let number of male and female participants at the
= 273 + 21x + 13x + x2 start of seminar be 3x and x, respectively.
⇒ 5x = 25 3 x − 16 2
Then, =
⇒ x = 5. x+6 1
⇒ 3x – 16 = 2x + 12
59.
(b) Since, the total number of men employed is 50, then ⇒ x = 28
women and children are employed 100 and 150 \ Total number of participants at the start of seminar
respectively. = 3x + x = 4 × 28 = 112.
Let the men, women and children wages are 6x, 3x
64.
(d) If the original cost of article is `6.
and 2x respectively.
Then, original raw material cost = `2
\ 50 × 6x + 100 × 3x + 150 × 2x = 4500 12
New cost of raw material = 2 × = `4.80
⇒ 900x = 4500 5
Original manufacturing expenses = (6 – 2) = `4
⇒ x = 5
5
\ Per day wages of men, women and children are New manufacturing expenses = 4 × = `5
4
`30, `15 and `10.
\ New cost of article = 4.80 + 5 = `9.80.
\ Weekly wages of men, women and children are
`210, `105 and `70. 65.
(a) Let their ages be x and y.
1 1 1 1
60.
(b) Let the present and last year salary of Mahesh’s and \ + = 5 − 
x y x y
Suresh’s be x, x′ and y, y′ respectively.
⇒ y + x = 5(y – x)
According to the given condition, ⇒ 6x = 4y
x′ 3 x′ 2 x 2
= , = ⇒ = (1)
y′ 5 x 3 y 3

xy 14.4
Now, =
y′ 4 x+ y 1
and, =
y 5 ⇒ xy = 14.4(x + y) (2)

x′/x 2/3 From Eqs. (1) and (2),
\ =
y ′ /y 4/5 x = 24 year and y = 36 year.

66.
(c) Let three numbers A, B and C are 12x, 15x and 25x
x′ y 10 3 y 10 respectively.
⇒ × = ⇒ × =
y′ x 12 5 x 12
\ 12x + 15x + 25x = 312
y 50 312
⇒ = ⇒ x = =6
x 36 52
15 × 6 − 12 × 6
Also, x + y = 43000 \ Required ratio =
25 × 6 – 15 × 6
50 3× 6 3
⇒ x + x = 43000 = =
36 10 × 6 10
43000 × 36 Short-cut method
⇒ x = = `18000.
86 There is no need to calculate the value of x.
15 x − 12 x
61.
(b) The sale of second class ticket Required ratio =
25 x − 15 x
8× 4 32 3x 3
= × 8050 = × 8050 = `2800. = = .
30 + 32 + 30 92 10 x 10

Chapter 07.indd 25 6/4/2015 5:30:40 PM


7.26 Chapter 7

67.
(c) Let the man left `x. 71. (c) Let the number of persons be x
x x 9000
\ Share of daughter = x − − Originally, each person gets `
2 3 x
9000
x In the second condition, each person gets = `
⇒ 45000 = x + 20
6 We are given,
⇒ x = `270000. 9000 9000
− = 160
x x + 20
68.
(c) Let he purchase x pairs of brown socks.
⇒ 225( x + 20 − x) = 4( x)( x + 20)
Price of black socks and brown socks be `2a and `a
⇒ x( x + 20) = 5 × 225 = 45 × 25
per pair respectively.
⇒ x = 25.
3
\ (4 × 2a + x × a) = x × 2a + 4 × a
2 72. (a) We are given,
3
⇒ 12a + xa = 2xa + 4a 1 2 3
A= B= C = D
4
2 2 3 4 5
3 A 4D
⇒ 12 + x = 2x + 4 ⇒ =
2 2 5
x A 8
⇒ =8 ⇒ =
2 D 5
⇒ x = 16
4 1 ⇒ A:D = 8:5.
\ Required ratio = = .
16 4 73. (b) Let the quotes of A and B be 7X and 4X, respectively.
After decreasing, quotes of A = 4X – 1.
69.
(a) 1105 =11x + 10 + 18x + 20 + 24x + 15 Then, we are given,
⇒ 1105 = 53x + 45 4X −1 3
⇒ x = 20 =
4X 4
\ Amount with Champa = 24x + 15 ⇒ 4 X − 1 = 3X
= 24 × 20 + 15 = `495. ⇒ X =1
70. (b) Lat salaries of A, B and C be x, y and (1450 – x − y), Then, decrement by A = 7 X − (4 X − 1)
respectively. = 3x + 1 = 3 + 1
Then, we have = ` 4 lakhs.

40% of x:35% of y:30% of (1450 − x − y) = 14:21:15
74. (d) Let earnings of A and B be 4x and 7x respectively.
Taking first two terms of the above ratio, we get Then, we are given,
40 x 14 4 x + 50% of 4 x 8
= =
35 y 21 7 x − 25% of 7 x 7
⇒ x= y
7 6x 8 24 x 8
⇒ = ⇒ =
12 21x 7 21x 7
Now, taking last two ratio, we get 4
35 y 21 From this equation, we cannot find the value of x.
=
30(1450 − x − y ) 15 Hence, data is inadequate.
7y 7
⇒ = 75. (a) Let the two numbers be x and y respectively.
6(1450 − x − y ) 5
⇒ 5 y = 8700 − 6 x − 6 y 1 2
Then, of 60% of x = of 20% of y
4 5
7
⇒ 11 y + 6 × y = 8700
12 1 60 2 20
⇒ × ×x= × ×y
29 4 100 5 100
⇒ y = 8700
2 6 4 x 16 18
⇒ x= y ⇒ = =
⇒ y = ` 600 4 5 y 30 15

Hence, salary of B is `600. ⇒ x : y = 8 : 15.

Chapter 07.indd 26 6/4/2015 5:30:42 PM


Ratio and Proportion 7.27

76. (c) Given, B = C + 8 79. (b) Suppose required ratio be K:1


and, A = B + 7 = C + 15 Then, 3 x + K : 4 x + 1 : 5 x = 5 : 4 : 3
and, A + B + C = 53
4x + 1 4
C + 15 + C + 8 + C = 53 Using II and III parts of the ratio, =
5x 3
3C = 59 − 23 ⇒ 12 x + 3 = 20 x
3C = 30 ⇒ 8x = 3
⇒ C = 10 3
∴ B = C + 8 = 10 + 8 = 18 ⇒ x=
8
and A = C + 15 = 10 + 15 = 25 3x + K 5
Using I and II parts of the ratio, =
Hence, required ratio A:B:C = 25:18:10. 4x + 1 4
⇒ 12 x + 4 K = 20 x + 5
1
77. (c) Efficiency ∝ ⇒ 4K = 8x + 5
Time taken
3
Now, efficiency of A and C = 5x and 3x. = 8× + 5 = 8
As A takes 6 days less, therefore 8
⇒ K =2
1 1
− =6 ∴ Required ratio = 2:1.
3x 5 x
5−3 1 15 × 6 1 P 1
⇒ =6⇒ = ⇒ = 45 80. (c) Let =
15 x x 2 x Q K
1 45
∴ Time taken by A = = = 9 days
5x 5 Then, on adding 1 to both numerator and denominator
1+1 R
and time taken by B =
1 45
= = 15 days the ratio changes as = .
3x 3 K +1 S
2 R
Ratio of number of days taken by B and C = 2 :3 ⇒ =
K +1 S
2
∴ Time taken by B = × 15 = 10 days
5 When 1 is again added the ratio changes as
Now, B and C’s one day work when worked together 2 +1 R +1 1  R +1 1 
= =  = ,given 
1 1 3+ 2 5 1 K +1+1 S +1 2  S +1 2 
= + = = =
10 15 30 30 6 2 +1 1 3 1
or = ⇒ =
1 1 K +1+1 2 K +2 2
Two days work of B and C = 2 × = ⇒ 6=K +2
6 3
1 2 ∴ K =4
Remaining work = 1 − =
3 3 So, Q = K = 4 and P =1
∴ Number of days taken by A to finish the work ∴ P :Q = 1:4 and P + Q = 1 + 4 = 5.

2
= × 9 = 6 days. 81. (a) Let monthly income be y
3
Let money spent on grocery, clothes and education be
78. (c) Student studies for 9 hrs from 11 am to 8 pm on 4x, 2x, 5x
Monday to Friday. Money spent of clothes = `5540 = 2x
Also, he studies 1/3rd of the time on Sunday, i.e., x = 2770
1
×9 = 3 h Now, 4x + 2x + 5x = 11x = 11 × 2770 = 30470 = 55% of y
3  
Total hours, he studied during the week 30470 × 100
⇒y=
= 5 × 9 + 3 = 45 + 3 = 48 h 55
∴ y = 55, 400.

Total hours, in a week = 24 × 7 = 168
∴ Required fraction 82. (a) Let amount of B = ` x
48 2 B’s ratio
= = . B’s share without error = × Total Amount
168 7 Total ratio

Chapter 07.indd 27 6/4/2015 5:30:46 PM


7.28 Chapter 7

3 Amount received by all the clerks


x = × Total amount (1) = 80 × 15000 = 12, 00, 000
9
B’s New ratio Total amount of profit earned = 11,25,000 – 12,00,000
B’s share after error  = × Total Amount
Total new ratio = `23.25 lakhs.
2 85. (b) Mrs. X spends = `353
⇒ x − 40 = × Total amount (2)
14 Total cost = 43 shirt + 21 ties = 535
From equations (1) and (2), we get By hit and trial, S = 10, T = 5
3x = 7(x − 40)
⟹ 3x − 7x = − 280 ⇒ Total cost = 43 × 10 + 21 × 5 = 535
\ x = 70 Hence, ratio of shirts to ties = 10:5 = 2:1.
Total amount = 7(70 40) = `210.
83. (d) A and B ratio is 4:7 86. (c) Let the passengers in A.C. Sleeper Class, 1st Class
and Sleeper Class be x, 2x and 3x respectively
⇒ 4 x + 7 x = 73689
and the fares in these classes be 5y, 4y and 2y
⇒ 11x = 73689 respectively.
⇒ x = 6699   Then, the income from these classes are 5xy, 8xy,

Share of A = `26796 6xy respectively.
5
Share of B = `46893 \ Required income = × 54000 = 14210.
19
Difference = twice of share B – thrice of share A
= 2 × 46893 − 3 × 26796 = `13398. 1 τ + ρω τ ρ k
87. (a) = = + = 1 + k2
84.
(d) Amount received by all the officers S αω αω α ω
= 45 × 25000 = 11, 25, 000 where k1 and k2 are constant
3 ⇒ 1/S decreases with ω increases.
Amount received by each clerk = × 25000 = 15000
5 ⇒ S increases with ω increases.

Difficulty Level-2

1.
(d) a = 6b = 12c 3.
(b) Let, the price of branded PC be = 64x and that of
1 1 assembled PC = 27x.
⇒ a:b:c ≡ 1: :
6 12 Let, after n years the prices of both will become equal.
⇒ a:b:c = 12:2:1 (1)
Then, (0, 9)n 64x = (1.2)n × 27x
It is also given
2b = 9d = 12e 64  4 
n
1 1 1 ⇒  
⇒ b:d:e ≡ : : 27  3 
2 9 12
⇒ b:d:e = 18:4:3 (2) Hence, n = 3 years.
From (1) and (2),
4.
(d) Let the price of B per kg be `X. Then, the price of A
a:b:c:d:e = 108:18:9:4:3
per kg = `3X
⇒ a = 108 K, b = 18K,
c = 9K, d = 4K, e = 3K 1kg of C contains 2/7 kg of A and 5/7 kg of B
Now, going through the options, the pair in option Price of 1 kg of C = (2/7) × 3X + (5/7) X = (11/7) X
(d) is not an integer:
By the given condition, 11X/7
a c
is an integer but is not an integer. = 5.20 – 0.80

6 d
= `4.40
2.
(b) Ratio of speed = Ratio of distance. Therefore, ratio of
distance = 4:3. Hence, the man travels 36 miles and ⇒ X = 4.40 × (7/11) = `2.80
the woman 27 miles. Hence the price of B per kg = `2.80.

Chapter 07.indd 28 6/4/2015 5:30:47 PM


Ratio and Proportion 7.29

5.
(a) Ratio of personal shares = 6:5:4 8.
(a) Let the soldiers in the two armies be 10X, 3X and
Ratio of the amounts = 12:15:16 losses be 20Y, 3Y, then we have
Ratio of men, women and children 10X – 20Y = 24000 (1)
12 15 16 and, 3X – 3Y = 24000 × 13/40 = 7800 (2)
= : :
6 5 4 Solving, we have
= 2:3:4 10X = 28000, 3X = 8400.
Sum of these ratios = (2 + 3 + 4) = 9
9.
(d) Let X be the required number, then
 45  2 
Number of men =  = 10
 9  (7 + X):(11 + X) = (11 + X):(19 + X)
⇒ (7 + X) (19 + X) = (11 + X)2
 49  3 
Number of women =  = 15 ⇒ X2 + 26X + 133 = X2 + 22X + 121
 9 
and, the number of children = 45 – (10 + 15) = 20 ⇒ 4X = –12 or, X = –3.
Now, dividing `430 in the ratio 12:15:16 3
10.
(d) No. of policemen = × 135 = 81
Total amount of men’s share 5
430  12  Since number of supporters: number of policemen =
= ` 
 9:1, therefore number of supporters must be 729.
 43 
1
= `120 11. (c) I a 2 .
Total amount of women’s share R
1
430  15  12. (d) Let y ∝ p + q, where p ∝ x and q ∝
= ` 

x
 43  \ y = K (p + q),
= `150 N
Total amount of children’s share p = Mx, q =
x
= `[430 – (120 + 150)] N
= `160 ⇒ y = K  Mx + 

 x
 120 
∴ Each man’s share = `  = `12 10
 10  y = ,
3
 150  when, x = 3
Each woman’s share = `  = `10
 15 
 N  10
 150  ⇒ K  3M +  =
Each child’s share = `  = `8.  3 3
 15 
\ MK = 2 and NK = –8
6. (b) Let their respective salaries be X, Y, Z.
2x −8
Then, A saves 0.2 X, B saves 0.15 Y, and C saves \ p = ,q=
K Kx
0.25 Z.
We have, X + Y + Z = 444 (1) 8
and so, y = 2x − .
0.2 X:0.15 Y:0.25 Z = 7:6:9 x
⇒ 4X:3Y:5Z = 7:6:9 13.
(a) Q Temperature ∝ Volume
or, X = 7Y/8, Z = 9Y/10
If Temperature = 25º K,
Putting in (1), we have Y = 160.
Volume = 50 m3
8 
(c) Skilled workers =   20 = 32
7.
5  If Volume = 200m3, then
1  Temperature = 100º K.
Number of clerks =   20 = 4
5 
14.
(d) x2 ∝ y3 ⇒ x2 = Ky3
Ratio of amount of 32 skilled workers, 20 unskilled
\ x = 6, y = 3
workers and 4 clerks
= 5 × 32:2 × 20:3 × 4 4
⇒ K =
= 160:40:12 or, 40:10:3 3
Now, divide `3,180 in the ratio 40:10:3. \ 3x2 = 4y3.

Chapter 07.indd 29 6/4/2015 5:30:48 PM


7.30 I Chapter 7

1 19.
(a) No. of girls studying Arts = 65
15.
(d) y ∝ 3 No. of girls studying Science = 115
x −x
\ Required ratio = 65:115 = 13:23.
K
⇒ y = 20. (c) Let the prices of a bat and a ball be 9x and 2x
x3 − x respectively.
1 ⇒ The new prices are 9.9x and 2.36x
When, x = 2, y =
6 Given: 12 × 9x + 54 × 2x = C.
⇒ 108x + 108x = C
1 K
\ = C
6 8−2 ⇒ 108x =
2
⇒ K = 1 C C
Hence, new price =  1.10   1.18
1 2 2
\ y =
x3 − x C
= (2.28) = 1.14 C .
2
\ x = 1
1 21.
(c) Since there are 12 bangles, then the number of broken
⇒ y = = ∞. to unbroken bangles can not be 2:3, since 5x = (2x
0
+ 3x) can not divide 12 for any integral value of x i.e.,
16.
(d) Ratio of number of passengers is 5:3:2 all the sum of ratios which are the factors of 12 can
possibly be the ratio of broken to unbroken.
∴ If 200 passengers travelled by third class, 500
must have travelled by first class. 22.
(c) Total weight of the stone
Sum of ratio of amount collected = (5 × 9 + 3 × 7 = 5 + 7 + 8 = 20
+ 2 × 2) = 45 + 21 + 4 = 70. Given value α (weight)2
∴ Amount collected from 1st class 6800 = K (20)2 ⇒ K = 17
Therefore, value of the pieces
45
=  98000 = `63,000 = K (5)2 + K (7)2 + K (8)2
70
= 17 × 25 + 17 × 49 + 17 × 64
∴ Fare for the first class
= `2,346
= ` 
63000  ∴ Loss in value = `(6800 – 2346) = `4,454.
= `126.
 500 
23.
(c) Number of maximum number of tables costing
`1200 each = 6 and from remaining money i.e., (8100
17.
(a) Let (x – y + z) = 2k, (y – z + 2w) = 3k
– 7200) = `900 he can buy 3 chairs costing `300 each.
and (2x + z – w) = 5k Hence ratio of chairs to tables = 3:6 = 1:2.
Then, (x – y + z) + (y – z + 2w)
= 2k + 3k 24.
(b) Let the number of mangoes that the man had
originally = x
= 5k = 2x + z –w
No. of Mangoes sold to Balance
or, x + z = 3w
x x−2
∴ S = (3x + 3z – 2w):w = 7:1. 1st customer = + 1
2 2
18.
(d) Number of boys = 210 x−2 x−5
2nd customer = +1
Number of girls = 180 6 3
Number of students studying Arts and Science are in x−5 x−9
3rd customer = +1
the ratio of 3:7 12 4
\ No. of student studying Arts = 117 x−9
4th customer = + 10
No. of students studying Science = 273 20
No. of boys studying Arts = 52 x−9 x−9
\ + 1 =
No. of girls studying Arts = 65 20 4
\ No. of boys studying Science = 210 – 52 = 158. ⇒ x = 14.

Chapter 07.indd 30 6/4/2015 5:30:49 PM


Ratio and Proportion I 7.31

25.
(d) Ratio of the amount collected from the 1st class and Now, given that
the 2nd class passengers = 3:50
 x 15 x 
  = 157500
\ Amount collected from the 2nd class passengers  2 96 
50
= × 1325 = `1250. ⇒  x = 24,00,000.
53
26.
(c) Ratio of white to yellow balls = 6:5  x  24 
29.
(a) Carl’s original share =   =   = `4 lakhs.
 6  6
Difference in the number of white and yellow balls
= 6x – 5x = x = 45 30.
(b) Ratio of property owned by the widows of three sons
Therefore, number of white balls now available
x 5 x 15 x
= 45 × 6 = : : = 8:10:15.
12 48 96
Number of white balls ordered
= (45 × 6) – 45 = 225. 31.
(c) Fresh grapes contain 10% pulp.
\ 20 kg fresh grapes contain 2 kg pulp.
27.
(a) No. of people having characteristic X
Dry grapes contain 80% pulp.
= 10 + 30 = 40
2 kg pulp would contain
No. of people having characteristic Y
2 20
= 10 + 20 = 30 = = = 2.5 kg dry grapes.
0.8 8
Required ratio = 40:30 = 4:3.

28.
(d) Let the total property of Alphonso be `x 32.
(b) Ratio of A’s last year and present year Salary = 3:5 let
salary be 3x and 5x.
After first distribution, money possessed by the family
members would be Ratio B’s is last year and present year. Salary = 2:3
i.e., salary be 2y and 3y respectively
 x  x  x  x
wife =    , Ben   , Carl   , Dave  
 2  6  6  6 3x 4
Given that, =
2y 5
After second distribution, money possessed by each
⇒ 15x = 8y (1)
 x
of them would be Alphonso’s wife =   , Ben (0), Also, given
 12 

 x x x  x x  5x + 3y = 6800 (2)
Ben’s wife   , Carl    , Dave   
 12   6 24   6 24  From Eq. (1) 15x = 8y

After third distribution, money possessed by them 8


\ 5x = y
 x 3
would be Alphonso’s wife   , Ben (0),
 2
Putting this value in Eq. (2), we have
 x  5x 
Ben’s wife   , Carl (0), Carl’s wife   ,
 12   48  + 3y = 6800

 x x 5x  15 x \ y = 1200
Dave     =
6 24 48  48 Put the value of y in Eq. (1), we get 5x = 3200
After last distribution, money possessed by them
\ A’s present salary = 5x = 3200.
 x 15 x 
Alphonso’s wife   , Ben (0),
 2 96  33. (b) According to question,

 x  5x  5x 5
=
Ben’s wife   , Carl (0), Carl’s wife   , 72 x + 8 74
 12   48 
⇒ x = 4
 15 x 
Dave (0), Dave’s wife 
 96  Managers = 5 × 4 = 20.

Chapter 07.indd 31 6/4/2015 5:30:49 PM


7.32 Chapter 7

34.
(b) 3
Q L ≠ 0, B = L
4
L 4
=
B 3
3
Thus, the ratio of the shorter side to the longer side = .
4
38.
(b) In the first 50 balls, 49 are red. If 8x balls are counted
after the first 50 balls, then 7x out of them are red.
7 x  49
So, the proportion of red balls is
\ x + 8 + x + 26x = 232 8 x  50
⇒ 28x = 232 – 8 = 224 Here, n = 8x + 50
x = 8
7 x  49 90
\ Ram runs scored = 12x = 96. ≥
8 x  50 100
1st 2nd 3rd
35.
(b) : :
3 4 5 \ 700x + 4900 ≥ 720x + 4500
\ 1st:2nd:3rd = 21:28:20 \ 20x ≤ 400
\ x ≤ 20
1st = 42 
 Q n = 8x + 50
2nd = 56  By option it can easily be found. 3rd.
\ n ≤ 210
3rd = 40 
The maximum value of n is 210.
36.
(d) Option (a) , y = – x. So, y is directly proportional to x. 39.
(b) If income is increased in proportion, then the similarity
10 will be unaltered but if it is increased by a fixed price
Option (b) , y = . So, y is inversely proportional to x.
3x it will change.
1
Option (c) , y = x . So, y is directly proportional to x. 40.
(c) Ratio of rent = 18 × 4:25 × 2:28 × 5:21 × 3
5
Option (d) , y varies with x, but it is neither directly nor 72x = 360
inversely proportional to x. x = 5

37.
(d) Let the longer and shorter side of the rectangle = L and Total rent = 325 × 5 = `1625.
B respectively. 41.
(c) By option method 16:56,
When the boy took the short cut, the distance covered Now, (56 – 16) = 40
by him was = L2  B 2
16 2
and = .
L 56 7
Given, boy covered L  B  as he saved a distance
2
42. (c) We know that,
equal to half the longer side.
L p∝
1
and s ∝
1
L2  B 2 = B s r
2
Therefore, p ∝ r
Taking the square on both sides,
So, p ∝ q and q ∝ r and therefore p ∝ r
L2 2 2 When p = 3, r = 5
L + B =  B 2  LB
4
10
3 2 3 ∴q = 2 ⇒ r =
L = LB ⇒ LB – L2  0 3
4 4 10
i.e., when p = 1, r =
 3  3
L  B  L = 0
 4  ⇒ r = 20 when p = 6.

Chapter 07.indd 32 6/4/2015 5:30:50 PM


Ratio and Proportion 7.33

43. (a) Let the production on Friday be X Let amount of Re 1 50 paise and 25 paise coins be
So, the approved pieces are 0.95X and the rejected 5 x, 3 x and 2 x, respectively.
pieces are 0.05X So, 5 x + 3 x + 2 x = 210 (given ) ⇒ x = 21
Let production on Saturday be Y
∴ Value of one rupee coins
So, the approved pieces are 0.9Y and the rejected
= number of one rupee coins = 21 × 5 = 105.
pieces are 0.1Y
49. (c) Let the weight of the diamond be 10x, then price of the
0.05 X + 0.1Y
∴ = 0.0833 diamond will be k (10x)2 = k 100x2 , k is a constant.
X +Y
Weight of each piece = x, 2x, 3x and 4x. Therefore
So, X :Y = 1:2.
their price will be kx2 , k4x2, k 9x2 and k16x2

3 Total price of pieces = kx 2 (1 + 4 + 9 + 16) = 30kx 2 .


44. (d)
4 We are given

k100 x − k 30 x = 70, 000 or kx = 1000.


2 2 2
y y
Given, ( x + y ) − x + y = ⇒ x + = x 2 + y 2 .
2 2

2 2 ∴ Original price of diamond = k100x2 = 1000 × 100 =


100,000.
From the options, we find that option (d) satisfies the
50. (c) We have, 2O + 3B + 4A = 15 (1)
above relation 3 + 2 = 9 + 16 ⇒ 5 = 5.
and 3O + 2B + A = 10 (2)

45. (b) Mayank paid


1
of the sum paid by other three. Let, the Adding Eqs. (1) and (2), we get
2 5O + 5B + 5A = 25 or, O + B + A = 5
x
other three paid ` x jointly, then Mayank paid .
2 ∴ 3O + 3B + 3A = 3 × 5 = 15.

x
So, x + = 60, 000 ⇒ x = 40, 000. 51. (a) Let the two numbers be x and y, and x < y. Then,
2
Hence, Mayank paid `20,000.  x  x  x  3x
 y −  = 3 x −  ⇒  y −  =
Likewise, Mirza and Little paid `15,000 and `12,000,  2  2  2 2
respectively. 4x
or y= ⇒ y = 2x
Hence, the amount paid by Jaspal is ` [60,000 − 2
(20,000 + 15,000 + 12,000)] = ` 13,000. ∴ y : x = 2 : 1.

46. (c) Let the longest piece be 3x, then the middle and the
shortest piece would be x and (3x − 23), respectively. 52. (b) Let the four numbers be a, b, c, and d.

We have 3 x + x + ( 3 x − 22 ) = 40 ⇒ x = 9. Then, a:b = c:d = 1:3



a c 1
Therefore, the shortest piece = ( 3 × 9 − 23) = 4 . ⇒ = =
b d 3
4 ⇒ b = 3a, d = 3c
47. (b) Value of 1 kg of ruby = crores = 13.33 crores (1)
0.3 Also, a 2 + b 2 + c 2 + d 2 = 50
5 ⇒ a 2 + 9a 2 + c 2 + 9c 2 = 50
Value of 1 kg of emerald = crores = 12.5 crores
0.4 ⇒ 10a 2 + 10c 2 = 50
To maximize wealth, maximum number of ruby and
minimum number of emerald be collected, as price ⇒ a2 + c2 = 5 (2)

per kg of ruby is more than that of emerald. From the
options, 40 > 28 + 9. Also, sum of means = 5

48. (d) Ratio of number of one rupee, fifty paise, and twenty b+c=5
five paise coins = 2.5:3:4 ⇒ b =5−c
3 4 5−c
∴ Ratio of value of coins = 2.5 × 1 : : = 5 : 3 : 2. ⇒ a= [using Eq.(1)]
2 4 3

Chapter 07.indd 33 6/4/2015 5:30:54 PM


7.34 Chapter 7

Putting this value of a in Eq. (2), we get Hence,required avearage


5−c
2
a + b + c + d 1 + 3 + 2 + 6 12
 +c =5 = = = = 3.
2

 3  4 4 4
25 + c 2 − 10c
⇒ + c2 = 5 53. (d) Using option (d),
9
⇒ 25 + c 2 − 10c + 9c 2 = 45 Bid price of B = `100000
⇒ 10c 2 − 10c − 20 = 0 Then, initially ratio of bid is 7:4,
⇒ c2 − c − 2 = 0 Bid price for A = `175000.
⇒ c=2 Then, A withdraws `100000
5−c 5−2
Therefore, a = = =1 Amount of bid left for A
3 3
b = 3a = 3 = `175000 – `100000 = `75000
c=2 \ Ratio of bid after withdrawal of `100000
d = 3c = 6 = 75000:100000 = 3:4.

Chapter 07.indd 34 6/4/2015 5:30:54 PM


CHAPTER

Partnership 8
INTRODUCTION Compound partnership is one in which the capitals of
In partnership, two or more persons carry on a business and partners are invested for different periods.
share the profits of the business at an agreed proportion. Again partner can be working partner or sleeping
Persons who have entered into partnership with one another partner.
are individually called partners and collectively called a Sleeping partner is one who invests the capital in the
firm and the name under which their business is carried on business but does not actively participate in the conduct of
is called the firm name. The partnership may be simple or business.
compound. Working partner besides investing capital, takes part
Simple partnership is one in which the capital of each in running the business. For his work, he is either paid some
partner is in the business for same time. salary or given a certain per cent of profit, in addition.

soMe Basic forMulae

Therefore, profit share of A


1. (a) If capitals of two partners be `C1 and C1 × P
`C2 for the same period and the total profit be `P, =
C1 + C2 + C3
then shares of the partners in the profits are
20000 × 12100
 C ×P   C2 × P  =
` 1 20000 + 50000 + 40000
 and `  
 C1 + C2   C1 + C2  2
(b) If the capitals of three partners be `C1, `C2 and = × 12100 = `2200
11
`C3 for the same period and the total profit be
Profit share of B
`P, then shares of the partners in the profits are
C2 × P
 C1 × P   C2 × P  =
` , `   C1 + C2 + C3
 C1 + C2 + C3   C1 + C2 + C3 
50000 × 12100
 C3 × P  =
and 20000 + 50000 + 40000
` .
 C1 + C2 + C3  5
= × 12100 = `5500
11
Illustration 1 A, B and C invested `20000, `50000 and and, profit share of C
`40000, respectively, in a business. The net profit for C3 × P
the year was `12100 which was divided in proportion to =
C1 + C2 + C3
investments. Find the amount of profit each partner earned.
Solution: We have, C1 = 20000, C2 = 50000, C3 = 40000 40000 × 12100 4
= = × 12100 = `4400
and P = 12100. 20000 + 50000 + 40000 11

Chapter 08.indd 1 6/4/2015 6:09:14 PM


8.2 Chapter 8

Illustration 2 A, B are two partners in a business.


2. (a) If the capitals of two partners be `C1 and `C2
A contributes `1200 for 5 months and B `750 for 4 months.
for the periods t1 and t2, respectively, and the
If total profit is `450, find their respective shares.
total profit be `P, then shares of the partners in
the profits are: Solution: We have C1 = 1200, C2 = 750, t1 = 5, t2 = 4 and
P = 450
 C1 × t1 × P   C2 × t 2 × P  ∴ Profit share of A
`  and `   C ×t × P 1200 × 5 × 450
 C1t1 + C2t2   C1t1 + C2t2  = 1 1 =
C1t1 + C2t2 1200 × 5 + 750 × 4
(b) If the capitals of three partners be `C1, `C2 and `C3
2700000
for the periods t1, t2 and t3, respectively, and the total = = ` 300
profit be `P, then shares of the partners in the profits 9000
are and profit share of B
C2 × t 2 × P
 C1 × t1 × P   C2 × t 2 × P  =
` , `   C1t1 + C2t2
C t +
 11 2 2C t + C t
33 C t +
 11 2 2C t + C t
33
750 × 4 × 450
=
 C3 × t3 × P  1200 × 5 + 750 × 4
and, `  
 C1t1 + C2t2 + C3t3  1350000
= = `150
9000

soMe useful sHort-cut MetHoDs

1. (a) If the capitals of two partners be `C1 and `C2 2. If the capitals of three partners are invested
for the periods t1 and t2 respectively, then in the ratio C1:C2:C3 and their profits are in the
Profit of A C ×t ratio P1:P2:P3, then the ratio of timing of their
= 1 1 .
Profit of B C2 × t 2 P1 P2 P3
investments = : : .
(b) If the capitals of three partners be `C1, `C2 and `C3 C1 C2 C3
for the periods t1, t2 and t3 respectively, then profit of
A:profit of B:profit of C = C1 × t1:C2 × t2:C3 × t3.
Illustration 4 Anu, Manu and Tanu invested capitals in the
Note: ratio 4:6:9. At the end of the business term, they received
the profits in the ratio 2:3:5. Find the ratio of time for which
If there is a loss in the business, then
they invested their capitals.
Loss of A:Loss of B:Loss of C
= C1 × t1:C2 × t2:C3 × t3. Solution: We have, C1:C2:C3 = 4:6:9
and, P1:P2:P3 = 2:3:5
Illustration 3 There are three partners A, B and C in
a certain business. A puts in `2000 for 5 months, B Therefore, the ratio of time for which Anu, Manu and
`1200 for 6 months and C `2500 for 3 months. Find the Tanu invested their capitals
ratio of their shares in the profit. P1 P2 P3 2 3 5
Solution: Here C1 × t1 = 2000 × 5 = 10000, C2 × t2 = 1200 = : : = : :
C1 C2 C3 4 6 9
× 6 = 7200 and C3 × t3 = 2500 × 3 = 7500
∴ Profit of A:Profit of B:Profit of C or,
1 1 5
: :
= C1 × t1:C2 × t2:C3 × t3 2 2 9
= 10000:7200:7500 or, 100:72:75 or, 9:9:10

Chapter 08.indd 2 6/4/2015 6:09:15 PM


Partnership 8.3

Solution: We have, P1:P2:P3 = 3:4:5


3. Three partners invested their capitals in a business.
If the timing of their investments is in the ratio and, t1:t2 : t3 = 1:2:5
t1:t2:t3 and their profits are in the ratio P1:P2:P3, then
P1 P2 P3
P1 P2 P3 ∴ Required ratio = : :
the ratio of their capitals invested is : : . t1 t2 t3
t1 t2 t3
3 4 5
Illustration 5 Gupta, Singhal and Kansal start a business. = : : or, 3:2:1
If the ratio of their periods of investments are 1:2:5 and their 1 2 5
profits are in the ratio of 3:4:5, find the ratio of capitals of Thus, Gupta, Singhal and Kansal invested their capitals
Gupta, Singhal and Kansal. in the ratio 3:2:1.

Practice Exercises

Difficulty level-1
(BaseD on MeMory)

1. A, B, C subscribe `50000 for business. A subscribes 4000 6. Sita and Gita enter into a partnership, Sita contributes
more than B and B `5000 more than C. Out of total profit `5000 while Gita contributes `4000. After 1 month, Gita
of `35000 A receives: withdraws one-fourth part of her contribution and after 3
(a) `11900 (b) `8400 months from the starting, Sita puts `2000 more. When Gita
(c) 14700 (d) 13600 withdraws her money Rita also joins them with `7000.
[Based on MAT, 2005] If at the end of 1 year, there is profit of `1218, what will
2. Rahul started a business with a capital of `8,000. After be the share of Rita in the profit?
six months, Sanjay joined him with an investment of (a) `844.37 (b) `488.47
some capital. If at the end of the year each of them gets (c) `588.47 (d) None of these
equal amount as profit, how much did Sanjay invest in the [Based on MAT (Feb), 2009]
business? 7. Three partners invested capital in the ratio 2:7:9. The time
(a) `17,500 (b) `18,000 period for which each of them invested was in the ratio of
(c) `16,000 (d) `16,500 the reciprocals of the amount invested. Find the share of
[Based on MAT, 2008] the partner who brought in the highest capital, if profit is
3. Anu is a working partner and Bimla is a sleeping partner `1080.
in a business. Anu puts in `5000 and Bimla puts in (a) `120 (b) `360
`6000. Anu receives 12.5 per cent of the profit for (c) `540 (d) `420
managing the business and the rest is divided in proportion [Based on MAT (Sept), 2008]
to their capital. What does each get out of a profit of `880? 8. A, B, C start a business. A invests three times as much as B
(a) `400 and `480 (b) `460 and `420 invests and B invests two-thirds of what C invests. Then,
(c) `450 and `430 (d) `470 and `410 the ratio of capitals of A, B and C is:
[Based on MAT (Sept), 2010, (Dec), 2009] (a) 3:9:2 (b) 6:10:15
4. X and Y are partners in a business. X contributed one-third (c) 5:3:2 (d) 6:2:3
of the capital for 9 months and Y received two-fifths of the 9. A, B, C enter into a partnership with shares in the ratio
profits. For how long was Y’s money used in the business? 7 4 6
(a) 4 months (b) 3 months : : . After 4 months, A increases his share by 50 per cent.
2 3 5
(c) 2 months (d) 5 months If the total profit at the end of one year be `21,600, the B’s
[Based on MAT (Sept), 2010, (Dec) 2009] share in the profit is:
5. X and Y entered into partnership with `700 and `600 (a) `2,100 (b) `2,400
respectively. After 3 months, X withdrew two-sevenths of (c) `3,600 (d) `4,000
his stock but after 3 months, he puts back three-fifths of 10. Three shepherds A, B and C rented a pasture for a year.
what he had withdrawn. The profit at the end of the year is A grazed 22 sheep for 4 months, B grazed 16 sheep for 8
`726. How much of this should X receive? months and C grazed 32 sheep for 6 months. If C’s share
(a) `336 (b) `366 of rent is `600, the total rent for the year was:
(c) `633 (d) `663 (a) `3,550 (b) `2,000
[Based on MAT (Sept), 2009, 2008)] (c) `1,275 (d) `675

Chapter 08.indd 3 6/4/2015 6:09:15 PM


8.4 I Chapter 8

11. `1950 is divided amongst three workers A, B and C such one partner received `30 more than the other, then the total
that 6 times A’s share is equal to 4 time B’s share which is profit is:
equal to 8 times C’s share. How much did A get? (a) `262.50 (b) `622.50
(a) `600 (b) `550 (c) `220.50 (d) `226.50
(c) `900 (d) `450 [Based on MAT, 2011]
[Based on MHT-CET MBA, 2010] 19. A, B and C invested capitals in the ratio 7:3:2. At the end
12. A, B and C enter into a partnership by investing ` 28000, of the business term, they received the profits in the ratio
` 32000 and ` 18000. A is a working partner and gets a 2:3:7. Find the ratio of time for which they contributed
fourth of the profit for this services and the remaining their capitals.
profit is divided amongst the three in the ratio of their (a) 4:14:49 (b) 49:14:41
investments. What is the amount of profit that B gets if A (c) 14:41:59 (d) 49:41:4
gets a total of ` 4995? [Based on MAT, 2011]
(a) `1665 (b) `2960 20. Three partners invested capital in the ratio 2:7:9. The time
(c) `2590 (d) Cannot be determined period for which each of them invested was in the ratio of
[Based on MHT-CET MBA, 2010] the reciprocals of the amount invested. Find the share of
the partner who brought in the highest capital, if the profit
13. Firoz invested `650000 to start a business. Dhruv joined is `1080.
him six months later by investing `800000. At the end of
two years from the commencement of the business, they (a) `120 (b) `360
earned a profit of `435000. What is Firoz’s share of the (c) `540 (d) `420
profit? [Based on MAT, 2011]
(a) `195000 (b) `185000 21. Arvind began a business with `550 and was joined
(c) `240000 (d) None of these afterwards by Brij with `330. When did Brij join, if the
 [Based on IRMA, 2006] profits at the end of the year were divided in the ratio 10:3?
14. A, B and C invested their capital in the ratio 5:6:8. At the (a) After 4 months (b) After 6 months
end of the business they received the profits in the ratio (c) After 4.5 months (d) None of these
5:3:1. Find the ratio of time for which they contributed [Based on MAT, 2012]
their capital. 22. Three partners A, B and C agree to divide the profits or
(a) 12:9:7 (b) 25:18:8 losses in the ratio 1.50:1.75:2.25. If in a particular year,
(c) 5:6:8 (d) 8:4:1 they earn a profit of `66000, find the share of B.
15. Three bachelors, Amar, Akbar and Anthony rented a (1) `21000 (2) `27000
house for a year. Amar left after 4 months, Akbar stayed (3) `18000 (4) `22000
for 8 months and only Anthony stayed for the entire year. [Based on MAT, 2012]
If the annual rent was `6,000, find the share of Akbar.
(a) `4,000 (b) `2,000 23. A, B and C are partners. A receives 9/10 of the profit and
B and C share the remaining profit equally. A’s income is
(c) `300 (d) `2,500
increased by `270 when the profit rises from 12% to 15%
16. A and B enter into partnership, investing `12,000 and Find the capital invested by B and C each:
`16,000, respectively. After 8 months, C joins them with
(a) `5000 (b) `1000
a capital of `15,000. The share of C in a profit of `45,600
after 2 years will be: (c) `500 (d) `1500
[Based on MAT, 2012]
(a) `21,200 (b) `19,200
(c) `14,400 (d) `12,000 24. In a business, A and C invested capitals in the ratio 2:1,
17. A, B and C enter into a partnership. A contributes `320 whereas the ratio between amounts invested by A and B
for 4 months, B contributes `510 for 3 months and C was 3:2. If `157300 was their profit, how much amount
contributes `270 for 5 months. If the total profit is `208, did B receive?
find the profit share of A, B and C. (a) `24200 (b) `36300
(a) `64, `76.5 and `67.5 (b) `46, `76.5 and `67 (c) `48400 (d) `72600
(c) `40, `50 and `65 (d) `62, `72 and `82 [Based on MAT, 2014]
[Based on MAT, 2011]
25. `5783 is divided among Ramesh, Shiv and Bhuwan in
18. Two partners invested `1250 and `850 respectively in a such a way that if `28, `37, and `18 be deducted from their
business. Both the partners shared 60% of the profit and respective shares, they have money in the ratio 4:6:9. Find
distributed the rest 40% as the interest on their capitals. If Ramesh’s share.

Chapter 08.indd 4 6/4/2015 6:09:15 PM


8.5
Partnership

(a) `1256 (b) `1084 If at the end of the year each of them gets equal amount as
(c) `1456 (d) `1228 profit, how much did Sanjay invest in the business?
[Based on MAT, 2014] (a) `17,500 (b) `18,000
26. Rahul started a business with a capital of `8,000. After six (c) `16,000 (d) `16,500
months, Sanjay joined him with investment of some capital. [Based on SNAP, 2012]

Difficulty Level-2
(Based on Memory)

1. A and B enter into a partnership with `50,000 and `60,000, share to the sum of A and B’s share. Then, A, B and C’s
respectively. C joins them after x months contributing share, respectively, is:
`70,000 and B leaves x months before the end of the year. (a) 150, 100, 150 (b) 100, 150, 50
If they share the profit in the ratio of 20:18:21, then find (c) 100, 100, 100, (d) 150, 50, 100
the value of x.
7.
A and B enter into a partnership. A puts in the whole
(a) 9 (b) 3 capital of `45,000 on the condition that the profits will be
(c) 6 (d) 8 equally divided after which B will pay A interest on half
[Based on IIT Joint Man. Ent. Test, 2004] the capital at 10 per cent p.a. and receive `60 per month
2. `120 are divided among X, Y and Z so that X’s share is from A for carrying on the concern. What is the yearly
`20 more than Y’s share and `20 less than Z’s share. What profit, if B’s income is half of A’s income?
is Y’s share? (a) `8,190 (b) `9,180
(a) `40 (b) `30 (c) `3,600 (d) `6,900
(c) `25 (d) `20 8.
A started a business with a capital of `2,100. After
[Based on IIFT, 2003] 4 months he admitted another partner B. What amount
should B put in so that the profit may be divided equally
3.
Surendra, Rajendra and Manindra invested some amount at the end of the year?
in a business in the ratio of 5:7:6, respectively. In the next
year, they increased their investments by 26 per cent, (a) `3,000 (b) `4,120
20 per cent and 15 per cent respectively. The profit earned (c) `3,150 (d) `3,600
during the second year should be distributed in what ratio 9.
A, B and C enter into partnership in a business with
among Surendra, Rajendra and Manindra. capitals of `5,000, `6,000 and `4,000 respectively. A gets
(a) 31:27:21 (b) 21:28:23 30 per cent of the profit for managing the business and
(c) 26:20:15 (d) Cannot be determined balance is divided in proportion to their capitals. At the
end of the year, A gets `200 more than B and C together.
[Based on IRMA, 2002]
Find the total profit.
4. X and Y start a business. X invests `3000 for 4 months and (a) `3,600 (b) `3,000
Y invests `2000 for 6 months. How much should X be paid (c) `2,875 (d) `2,550
out of a total profit of `500?
10.
A and B enter into partnership. A supplies whole of the
(a) `200 (b) `300
capital amounting to `45,000 with the condition that the
(c) `250 (d) `350 profit are to be equally divided and that B pays the interest
[Based on FMS (Delhi), 2002] on half the capital to A at 10 per cent per annum, but receives
5.
A and B started a business together, but they both were `120 per month for carrying on the concern. Find their total
working on different units. The ratio of investment of A yearly profit when B’s income is one half of A’s income.
and B is 7:5. If A has a loss which is 3 times the profit (a) `9,215 (b) `9,000
of B and they get total `6,000 amount (net profit) back, (c) `9,227.5 (d) `9,180
then how much money does both A and B invest if sum of 11.
A, B and C enter into partnership. A invests some money
investment of B and 5 times his profit is 9,500? at the beginning, B invests double the amount after 6
(a) `8,400 (b) `7,400 months and C invests thrice the amount after 8 months.
(c) `9,600 (d) None of these If the annual profit be `27000, C’s share is:
6.
`300 is divided among A, B and C. The ratio of A’s share (a) `9000 (b) `11250
to the sum of B and C’s share equals ratio of B’s share to (c) `10800 (d) `8625
the sum of A and C’s share and also equals the ratio of C’s [Based on FMS (MS), 2006]

Chapter 08.indd 5 6/4/2015 6:09:15 PM


8.6 Chapter 8

12.
John, Mona and Gordon, three US based business 16.
Ram and Shyam form a partnership (with Shyam as
partners, jointly invested in a business project to supply working partner) and start a business by investing `4,000
nuclear fuel to India. As per their share in the investment, and `6,000, respectively. The conditions of partnership
Gordon will receive 2/3 of the profits whereas John and are as follows:
Mona divide the remainder equally. It is estimated that the • In case of profits till `200,000 per annum, profit would
income of John will increase by $60 million when the rate be shared in ratio of the invested capital.
of profit rises from 4 per cent to 7 per cent. What is the
capital of Mona? • Profits from `200,001 till `. 400,000, Shyam would
take 20% out of the profit, before the division of
(a) $ 2000 million (b) $ 3000 million
remaining profits, which will then be based on ratio of
(c) $ 5000 million (d) $ 8000 million
invested capital.
[Based on FMS, 2009]
• Profits in excess of `400,000 Shyam would take 35%
13.
Amber Chew opened a departmental store at Great India out of the profits beyond `400,000, before the division
Palace in Noida by investing `20 million. After a few of remaining profits, which will then be based on ratio
months her brother Sheesh Chew joined the business and of invested capital.
invested `30 million. At the end of the year, the profit was
If Shyam’s share in a particular year was `367,000, which
shared in the ratio of 3:2. After how many months did
option indicates the total business profit (in `) for that
Amber’s brother join the business?
year?
(a) 4 months (b) 6 months
(a) 520,000 (b) 530,000
(c) 7 months (d) 8 months
(c) 540,000 (d) 550,000
[Based on FMS, 2009]
[Based on XAT, 2012]
14.
A and B invest `60,000 and `80,000 in a business. A
receives `100 per month out of profit for running the 17.
A, B and C enter into a partnership by making investments
business and the rest of the profit is divided in the ratio of in the ratio 3:5:7. After a year, C invests another `3,37,600
investments. If A receives `3,900 annually, B receives: while A withdraws `45,600. The ratio of investments then
(a) `3,200 (b) `2,700 changes to 24:59:167. How much did A invest initially?
(c) `3,600 (d) `2,925 (a) `45,600 (b) `96,000
(c) `1,41,600 (d) None of these
15. Sumant started a business investing `48000. After 6 months
Maurya joined him with a captial `56000. At the end of 18.
Eight people enter into a partnership; 6 of them bring in
the year the total profit was `24529. What is the difference `30 each. The seventh person brings in `10 more than the
between the share of profits of Sumant and Maurya? average of eight persons, and the eighth person brings in
(a) `6455 (b) `7775 `55. What is the total sum brought in?
(c) `5545 (d) `4875 (a) `40 (b) `240
[Based on NMAT, 2008] (c) `280 (d) `250

Answer Keys
Difficulty Level-1

1. (c) 2. (c) 3. (b) 4. (b) 5. (b) 6. (b) 7. (b) 8. (d ) 9. (d ) 10. (c) 11. (a) 12. (b) 13. (d )
14. (d ) 15. (b) 16. (d ) 17. (a) 18. (a) 19. (a) 20. (b) 21. (b) 22. (a) 23. (c) 24. (c) 25. (d ) 26. (c)

Difficulty Level-2

1. (b) 2. (d ) 3. (b) 4. (c) 5. (a) 6. (c) 7. (b) 8. (c) 9. (b) 10. (d ) 11. (a) 12. (a) 13. (b)
14. (c) 15. (a) 16. (d ) 17. (c) 18. (c)

Chapter 08.indd 6 6/4/2015 6:09:16 PM


8.7
Partnership I
Explanatory Answers

Difficulty Level-1

1. (c) Let C = x 7.
(b) Ratio of capital = 2:7:9
B = x + 5000, 1 1 1
Ratio of time = : :
A = x + 5000 + 4000 2 7 9
\ x + x + 5000 + x + 9000 \ Ratio of investment
= 50000 1 1 1
= 2 × : 7 × : 9 × = 1:1:1
\ x = 12000 2 7 9
\ A:B:C = 21000:17000:12000 \ Share of each partner
= 21:17:12 1
= × 1080 = `360.
21 3
A’s share = 35000 × = `14700.
50
8.
(d) Let C invests `x, then ratio of investments of A, B
8000 × 12 1 and C.
2.
(c) = ⇒ x = 16000. 2x
x×6 2 = 2x : : x  = 6:2:3.
3
12.5
3.
(b) 12.5% of profit = × 880 = `110 9.
(d) Ratio of investments of A, B and C
100
Remaining `770 is divided in the ratio 7 105  4 × 12 6 × 12
=  ×4+ × 8 : :
= 5000:6000 = 5:6  2 2  3 5
5 ⇒ 56:16:14.4. Therefore, B’ share
Profit of Anu = × 770 + 110 = `460
11 16
= × 21600 = `4,000.
6 86.4
Profit of Bimla = × 770 = `420.
11
10.
(c) A’s Monthly Equivalent Rent = 22 × 4
1 2
4.
(b) Ratio of capital = : = 1:2 B’s Monthly Equivalent Rent = 16 × 8
3 3
3 2 C’s Monthly Equivalent Rent = 32 × 6
Ratio of profit = : = 3:2
5 5 B’s Monthly Equivalent Rent
Let Y’s money was used for n months. C’s Monthly Equivalent Rent
\ (1  × 9):(2 × n) = 3:2 Rent paid by B
⇒ n = 3 months. =
Rent paid by C
5.
(b) X’s investment
16  8
 5   3 ∴ = Rent paid by B
= (700 × 3) +  700 × × 3  +  500 + 200 ×  × 6 32  6 600
 7   5 
16  8  600
 = `7320 ∴ Rent paid by B =  = `400
Y’s investment = 600 × 12 = `7200. 32  6
\ X’s share from profit Similarly, rent paid by
7320
= × 726 = `366. 22  4  60
(7320 + 7200) A =  = `275
32  6
6.
(b) Ratio of investment of Sita, Gita and Rita is (5000 × ∴ Total rent = 400 + 600 + 275 = `1,275.
3 + 7000 × 9):(4000 × 1 + 3000 × 11):(7000 × 11)
11. (a) Let, A × 6 = B × 4 = C × 8 = l
= 78000:37000:77000
λ λ λ
= 78:37:77 So, ,B= ,C=
A =
6 4 8
\ Share of Rita in profit
Amount ratio among them
77
= × 1218 λ λ λ
78 + 37 + 77 = : :
6 4 8
≈ `488.47.

Chapter 08.indd 7 6/4/2015 6:09:16 PM


8.8 Chapter 8

= 4:6:3 17. (a) Ratio of the profits of A, B and C


4 = Ratio of their partnership
Hence, A’s share = × `1950
(4 + 6 + 3) = 320 × 4:510 × 3:270 ×5
4
 = × `1950 = 128:153:135
13
Let the profits of A, B and C be 128x, 153x and 135x
 = `600.
respectively.
12. (b) Investment ratio among A, B and C Then, 128x + 153x + 135 = 208
       = 28000:32000:18000
       = 14:16:9 ⇒ 416 x = 208

Suppose total profit = `x 1
⇒ x=
1 2
A’s profit for his services = `x × =` 1
4 Hence, share of A = 128 × = ` 64
2
x 3x
Remaining profit = x – =` 1
4 4 of B 153
Share = = ` 76.5
A’s profit according to his investment 2
3x 14 1
= ` × Share of C = 135 × = ` 67.5 .
4 (14 + 16 + 9) 2
3 x 14 18. (a) Ratio of the profit = 1250:850 = 25:17
= ` ×
4 39 They shared 60% of the profit.
7x If the total profit is x, then
⇒          = `
26 25 − 17
 x 7x  60% of x × = 30
Then,  +  = ` 4995 25 + 27
 4 26 
6x 8
13 x + 14 x ⇒ × = 30 ⇒ x = ` 262.50.
⇒ = `4995
52 10 42  
4995 19. (a) Ratio of capitals of A, B and C = 7:3:2
⇒ x = 52 × = `9620
27 Ratio of profits of A, B and C = 2:3:7
3 x 16 2 3 7 2 7
Hence, B’s profit = ` × ∴ Ratio of time = : : = : 1 : = 4:14:49.
4 39 7 3 2 7 2
3 × 9620 16
=`
     × = `2960. 20. (b) Ratio of capitals = 2:7:9
4 39
1 1 1
Ratio of time period = : :
13. (d) Firoz Dhruv 2 7 9
650000 × 24 : 800000 × 18 1 1 1
∴ Ratio of profit = 2 × : 7 × : 9 × = 1 : 1 : 1
13 : 12 2 7 9
13 Hence, profit of all the three partners is same and
\ Share of Firoz = × 435000 1080
25 equal to = ` 360 .
= `226200. 3
21. (b) Let Brij joined after x months. Then,
14.
(d) Let they invest money for x, y and z months
then 5x:6y:8z = 5:3:1 550 × 12 : 330 × (12 − x) = 10 : 3
1 1 5 × 12 10
or, x:y:z = 1: :  = 8:4:1. =
2 8 3 × (12 − x) 3
6
15. (b) Rate in which the rent is to be divided = 4:8:12 ⇒ = 1 ⇒ 6 = 12 − x
12 − x
8 ⇒ x = 12 − 6 = 6
∴ Akbar’s share of rent =  6000  = `2,000.
24
Hence, Brij joined after 6 months.
16.
(d) Ratio of investments of A, B and C
= 12000 × 24:16000 × 24:15000 × 16 22. (a) Share of B =
1.75
× 66000
= 18:24:15 1.50 + 1.75 + 2.25
15 1.75
So share of C = × 45600 = `12,000. = × 66000 = ` 21000.
57 5.5

Chapter 08.indd 8 6/4/2015 6:09:19 PM


8.9
Partnership

23. (c) Ratio of investment of A, B and C = Ratio of profit of ⇒  19x + 83 = 5738  ⇒  19x = 5700
A, B and C
5700
9 1 1 ∴ x= = 300
= = : : 18 : 1 : 1 19
10 20 20
\ Ramesh share = 4x + 28
Also, given that A’s income is increased by `270,
when the profits rises 3% (from 12% to 15%) = 4 × 300 + 28  ⇒  `1228
270 P × R ×T
∴ Investment of A × 100 = ` 9000 ∴ SI =
3 100
If investment of A, B and C, 18x, x and x, then 10000 × 8 25
⇒ 10000 = × ⇒ 10000 = 10000
18x = 9000 100 2
x = `500 Hence, only Statement I alone is sufficient to answer
Hence, the capital invested by B and C each is `500. the question.
24. (c) Given, A:B = 3:2 and A:C = 2:1
26. (c) Rahul stated a business with a capital of `8000.
∴ A: B :C = 6:4:3 Let Sanjay invests `x
Total profit = `157300
Since, each of them gets equal amount as profit
4
∴ B’s share = × 157300 = ` 48400. \ Profit is divided in the ratio 1:1
13
Since, Sanjay joined Rahul after 6 months.
25. (d) Let the share of Ramesh, Shiv and Bhuwan be `4x,
\ We have
`6x and `9x after the reducation of money.
8000 × 12:x × (12 − 6) = 1:1
We are given
Ramesh’s actual share = ` (4x + 28) 8000 × 12 1
⇒ =
Shiv’s actual share = ` (6x + 37) 6x 1
Bhuwan’s actual share = ` (9x + 18) ⇒ 8000 × 12 = 6 x
And their sum = `5783
⇒ 16, 000 = x
\ 4x + 28 + 6x + 37 + 9x + 18 = 5783
⇒ 4x + 6x + 9x + 28 + 37 + 18 = 5738 Hence, Sanjay invests `16,000 in the business.

Difficulty Level-2

1. (b) Let B leaves x months before the end of the year. 3.


(b) 5K + 26% of 5K:7K + 20% of 7K:6K + 15% of 6K
\ B stays for (12 – x) months. 630 840 690
⇒     K: K: K
C joins after x months. 100 100 100
\ C also remains for (12 – x) months. ⇒     63:84:69  ⇒  21:28:23.
\ Profits are shared among A, B and C in the ratio. 4. (c) The ratio in which X and Y should be paid the profit
out of `500.
50000 × 12:60000 (12 – x):70000 (12 – x)
= 3000 × 4:2000 × 6 = 12:12 = 1:1
= 20:18:21 (Given)

\ Each of X and Y should be paid `250.
60 20
\ = ⇒ x = 3. 5.
(a) Let 7x and 5x be the investments of A and B,
6(12 − x) 18
respectively.
2.
(d) X + Y + Z = 120 Let 3y and y be the loss of A and profit of B,
X = Y + 20, X = Z – 20 respectively, then, 7x – 3y + 5x + y = 6000
⇒ (Y + 20) + Y + (X + 20) = 120 ⇒ 6x – y = 3000 (1)
⇒ X + 2Y = 80 and 5x + 5y = 9500
⇒ (Y + 20) + 2Y = 80 ⇒ x + y = 1900 (2)
⇒ 3Y = 60 (1) + (2) ⇒ x = 700
⇒ Y = 20. Hence, total money invested = 7x + 5x = `8,400.

Chapter 08.indd 9 6/4/2015 6:09:20 PM


8.10 Chapter 8

6.
(c) `300 is to be divided among A, B and C Total money recevied by B
Let A be A’s share, B be B’s share and C be C’s share   x  1440    x  3060 
= `     1440  2250  = `  
A B C   2    2
∴ = =
BC AC AB
1  x  3060   x  3060 
Given:   =  
ABC 1 2 2 2
= =
2( A  B  C ) 2 x  3060 x  3060
or, = =x
∴ A = B = C = `100. 4 2
7.
(b) Interest on `22,500 = 0.1 × 22,500 = `2,250 or, x = 9180
Charges for managing the concern = 60 × 12 Hence, the total profit = `9,180.
= `4,720
x 11.
(a) A B C
If yearly profit is `x, then B’s share and A’s share = 
2 x  12 : 2x  6 : 3x  4
x 1x   12 x : 12 x : 12 x
∴  – 2250 + 720 =   720  2250
2 2 2 
\ 3x = 27000  ⇒  x = `9000.
∴ x = `9,180.
12.
(a) Ratio of the shares of investment of Gordon, John and
8. (c) Initial Capital = `2,100. A’s capital continues for 12
months. 2 1 1
Mona = 4:1:1 = : :
B’s capital continues (12 – 4) = 8 months. Let, B puts in ` x 3 6 6
⇒ 2100 × 12 = x × 8 Let the capital be x.
⇒ x = 2100 × 12/8 = `3,150. (7  4) x 3 x
Total profit increased = 
9.
(b) Let the total profit be `x 100 100
 3x   7x  3x
Amount of profit = `  x   = `   \ John’s increased share =
 10   10  600
3x
Ratio of capitals = 5000:6000:4000 or, 5:6:4 \ = 60 million
600
 8x 
∴ A’s share = `   7x  5   3 x  = `  
⇒ x = 12000 million

 10 15  10   15 
7x 6  7x 12000
B’s share = `   = `   \ Capital of Mona = = $ 2000 million.
 10 15   25  6

7x 4  14 x  13.
(b) Let x month be the period of investment of Sheesh
C’s share = `   = ` 
 10 15   75  Chew.

7 x 14 x 8x \ Ratio of the profits of Amber Chew and Sheesh


∴   200 = or x = 3000
25 75 15 20  12
Chew =
Thus, the total profit is `3,000. 30  x
20  12 3
10.
(d) Let the yearly total profit be `x ⇒ =   ⇒  x  = 5.33 = 6 months.
30  x 2
Amount paid to B as salary = `(120 × 12) = `1,440
14.
(c) A’s profit + 1200 = 3900
 x – 1440 
Share of each = `   ⇒ A’s profit = `2,700
 2
B’s profit = (80000/60000) × 2,700
 22500  10  = (4/3) × 2,700 = `3,600.
Interest paid by B = `   = `2,250
 100
15. (a) Sumant : Maurya
Total money received by A 12 × 48000 : 6 × 56000
12 × 48 : 6 × 56
= `  x  1440  2250  x  3060 
`  576 : 336
 2   2
12 : 7

Chapter 08.indd 10 6/4/2015 6:09:21 PM


8.11
Partnership

\ Share of profit of Sumant or x = `47,200, therefore, investment of A = 3


12 × 47200 = `1,41,600.
= × 24529 = 12 × 1291 = `15492
19 18.
(c) Six people contribute a total of `180. Let, the seventh
Share of profit of Maurya = 24529 – 15492 = `9037 person contributes `x
Difference between the share of profit of Sumant and Eighth person contributes `55
Maurya = 15492 – 9037 = `6455. Total contributions of these eight persons = 235 + x
16.
(d) 235  x
Now x =  10
17.
(c) Let the initial investments of A, B and C be 3x, 5x and 8
7x respectively. Investment after one year.
7 1
= 3x – 45600, 5x, 7x + 337600 ⇒ x  (235) + 10 ⇒ x = 45
8 8
Given 3x – 45600:5x:7x + 337600
= 24:59:167 ∴ Total collection = 235 + 45 = `280.

Chapter 08.indd 11 6/4/2015 6:09:21 PM


This page is intentionally left blank

FM.indd 6 6/10/2015 2:36:16 PM


CHAPTER

Profit and Loss 9


INTRODUCTION Loss If the cost price of an article is greater than the selling
Business transactions have now-a-days become common price, the seller suffers a loss.
feature of life. When a person deals in the purchase and sale Thus, Loss = C.P. – S.P.
of any item, he either gains or loses some amount generally.
Note that profi t and loss are always calculated with
The aim of any business is to earn profi t. The commonly
respect to the cost price of the item.
used terms in dealing with questions involving sale and
purchase are:
Illustration 1
Cost price The cost price of an article is the price at which
(i) If C.P. = `235, S.P. = `240, then profi t = ?
an article has been purchased. It is abbreviated as C.P.
(ii) If C.P. = `116, S.P. = `107, then loss = ?
Selling price The selling price of an article is the price at
which an article has been sold. It is abbreviated as S.P.
Solution:
Profit or gain If the selling price of an article is more than
the cost price, there is a gain or profi t. (i) Profi t = S.P. – C.P. = 240 –235 = `5
Thus, Profi t or gain = S.P. – C.P. (ii) Loss = C.P. – S.P. = 116 – 107 = `9

soMe Basic forMulae

Illustration 3 Anu bought a necklace for `750 and sold it


1. Gain on `100 is gain per cent for `675. Find her percentage loss
Gain × 100 Solution: Here C.P. = `750, S.P. = `675.
Gain% =
C.P. Loss = C.P. – S.P. = 750 – 675 = `75
Loss on `100 is loss per cent Loss × 100
\ Loss% =
Loss × 100 C.P.
Loss% =
C.P. 75 × 100
= = 10%
750
Illustration 2 The cost price of a shirt is `200 and selling
price is `250. Calculate the % profi t 2. When the selling price and gain pre cent are given:
Solution: We have C.P. = `200, S.P. = `250.  100 
C.P. =   × S.P.
Profi t = S.P. – C.P = 250 – 200 = `50.  100 + Gain% 
Profit × 100 3. When the cost and gain per cent are given:
\ Profi t% =
C.P.
 100 + Gain% 
50 × 100 S.P. =   × C.P.
= = 25%  100 
200

Chapter 09.indd 1 6/4/2015 7:17:46 PM


9.2 Chapter 9

Explanation Illustration 4 Mr. Sharma buys a cooler for `4500. For how
Profit × 100 much should he sell so that there is a gain of 8%?
Since Profit% =
C.P. Solution: We have C.P. = `4500, gain% = 8%

 (S.P. − C.P.) × 100   100 + Gain% 


=  ∴ S.P. =   × C.P
 C.P.  100 
Profit% S.P.  100 + 8 
∴ = –1 =  × 4500
100 C.P.  100 
S.P. Profit%
or, =1+ 108
C.P. 100 = × 4500
100
 100 + Profit% 
∴ S.P. =   × C.P. = `4860
 100 
Illustration 5 By selling a fridge for `7200, Pankaj loses
 100 
and, C.P. =   × S.P. 10% Find the cost price of the fridge
 100 + Profit% 
Solution: We have, S.P. = `7200, loss = 10%
4. When the cost and loss per cent are given:  100 
∴ C.P. =   × S.P
 100 − Loss%   100 − Loss% 
S.P. =   × C.P.
 100 
 100 
5. When the selling price and loss per cent are given: =  × 7200
 100 − 10 
 100 
C.P. =   × S.P. 100
 100 − Loss%  = × 7200
90

Explanation = `8000

Loss × 100 Illustration 6 By selling a pen for `99, Mohan gains


Since Loss% =
C.P. 1
12 % Find the cost price of the pen
2
 (C.P. − S.P.) × 100 
= 
 C.P. 1 25
Solution: Here S.P. = `99, gain% = 12 % or, %
2 2
Loss% S.P.
∴ =1–  
100 C.P. 100
∴ C.P. =   × S.P.
 100 + Gain% 
S.P. Loss%
or =1–
C.P. 100  100 
= × 99
25 
 100 + 
 100 − Loss%   2 
∴ S.P. =   × C.P.
 100 
 100 × 2 
=  × 99
100  225 
and, C.P. =   × S.P.

 100 − Loss%  = `88.

Chapter 09.indd 2 6/15/2015 2:28:46 PM


Profi t and Loss 9.3

soMe useful sHort-cut MetHoDs

1. If a man buys x items for `y and sells z items for  xw 


% profi t =  − 1 × 100%
`w, then the gain or loss per cent made by him is  zy 
 xw   11 × 11 
 − 1 × 100% =  − 1 × 100%
 zy   10 × 10 
21
Explanation = × 100% = 21%
100
S.P. of z items = `w
w Illustration 8 A fruit seller buys apples at the rate of `12
S.P. of x items = ` x
z per dozen and sells them at the rate of 15 for `12. Find his
w percentage gain or loss.
Net profi t = x− y.
z Solution:
w Quantity Price
x− y
\ % Profi t = z × 100% 12 12
y
 xw 
i.e.,  − 1 × 100% , 15 12
 zy 
which represents loss, if the result is negative.  xw 
% gain or loss =  − 1 × 100%
 zy 
Note:
 12 × 12 
In the case of gain per cent the result obtained bears =  − 1 × 100%
 15 × 12 
positive sign whereas in the case of loss per cent the
result obtained bears negative sign. 36
= – × 100% = –20%
180
How to remember:
Since the sign is –ve, there is a loss of 20%
Quantity Price (C.P. or S.P.)
x y 2. If the cost price of m articles is equal to the

selling price of n articles, then


z w
m−n
% gain or loss =   × 100
1. Cross-multiply the numbers connected by the arrows  n 
(xw and zy)
[If m > n, it is % gain and if m < n, it is % loss]
2. Mark the directon of the arrows for cross-
multiplicaton. The arrow going down forms the Explanation
numerator while the arrow going up forms the
Let the C.P. of one article be `1
 xw 
denominator  . \ C.P. of m articles = `m × 1 = `m
 zy 
 S.P. of n articles = `m
Illustration 7 If 11 oranges are bought for `10 and sold at
m
10 for `11, what is the gain or loss%? \ S.P. of 1 article = `
n
Solution:
Quantity Price m  m−n
\ Profi t on 1 article = `  − 1 i.e., `  
 n   n 
11 10
m − n 100 m−n
\ % profi t = × i.e.,   × 100
10 11 n 1  n 

Chapter 09.indd 3 6/4/2015 7:17:47 PM


9.4 Chapter 9

Illustration 9 A shopkeeper professes to sell his goods on cost


price but uses 800 gm, instead of 1kg. What is his gain %? 4. If ‘A’ sells an article to ‘B’ at a gain/loss of m%
and ‘B’ sells it to ‘C’ at a gain/loss of n% If ‘C’
Solution: Here cost price of 1000 gm is equal to selling
pays `z for it to ‘B’ then the cost price for ‘A’ is
price of 800 gm
m−n  1002 z 
\ % gain =   × 100  
 n   (100 + m) (100 + n) 
 1000 − 800  where m or n is –ve, of it indicates a loss, otherwise
=   × 100 it is +ve.
 800 
200 Illustration 12 Mohit sells a bicycle to Rohit at a gain of 10%
= × 100 = 25%
800 and Rohit again sells it to Jyoti at a profi t of 5% If Jyoti pays
`462 to Rohit, what is the cost price of the bicycle for Mohit?
Illustration 10 If the selling price of 12 articles is equal to
the cost price of 18 articles, what is the profi t %? Solution: Here m = 10, n = 5, z = `462.
Solution: Here m = 18, n = 12 Using the formula,
m−n  18 − 12   1002 z 
\ Profi t% =   × 100 =   × 100 C.P. =  ,
 n   12   (100 + m) (100 + n) 
6  1002 × 462 
= × 100 = 50% we get C.P. for Mohit =  
12
 (100 + 10) (100 + 5) 
3. If an article is sold at a price S.P.1, then % gain or 462 × 10000
= = `400
% loss is x and if it is sold at a price S.P.2, then % 110 × 105
gain or % loss is y. If the cost price of the article is
C.P., then Illustration 13 ‘A’ sells a DVD to ‘B’ at a gain of 17% and
‘B’ again sells it to ‘C’ at a loss of 25% If ‘C’ pays `1053 to
S.P1 S.P2 C.P. S.P1 − S.P2 ‘B’, what is the cost price of the DVD to ‘A’?
= = = ,
100 + x 100 + y 100 x− y Solution: We have, m = 17, n = –25, z = `1053
where x or y is –ve, if it indicates a loss, otherwise
\ Cost price of DVD to ‘A’
it is +ve.
 1002 z 
=  
Illustration 11 By selling a radio for `1536, Suresh lost 20%  (100 + m) (100 + n) 
What per cent shall he gain or lose by selling it for `2000? 100 × 100 × 1053
Solution: Here S.P1 = 1536, x = – 20 =
(100 + 17) (100 − 25)
(–ve sign indicates loss)
100 × 100 × 1053
S.P2 = `2000, y = ? = = `1200
117 × 75
Using the formula,
S.P1 S.P2
= 5. If ‘A’ sells an article to ‘B’ at a gain/loss of m%
100 + x 100 + y
and ‘B’ sells it to ‘C’ at a gain/loss of n%, then the
1536 2000 resultant profi t/loss per cent is given by
we get =
100 − 20 100 + y  mn 
m + n +  (1)
2000 × 80 1  100 
⇒ 100 + y = = 104 where m or n is –ve, if it indicates a loss, otherwise
1536 6
it is +ve.
1
⇒ y = 4 %
6 Note:
1 The expression given by (1) represents resultant profi t
Thus, Suresh has a gain of 4 % by selling it for
6 or loss accordingly as it is +ve or –ve.
`2000.

Chapter 09.indd 4 6/4/2015 7:17:48 PM


Profi t and Loss 9.5

Illustration 14 ‘A’ sells a horse to ‘B’ at a profi t of 5% and Since gain/loss of x% is made on the fi rst, cost price of the
‘B’ sells it to ‘C’ at a profi t of 10% Find the resultant profi t fi rst article
per cent  100 
= `z  
Solution: We have, m = 5 and n = 10  100 + x 
 mn  Also, gain/loss of y% is made on the second, therefore
\ Resultant profi t% =  m + n + 
 100  cost price of the second article
 5 × 10   100 
=  5 + 10 +  = `z  
 100   100 + y
31 1  100   100 
= % or, 15 % \ Total C.P. = z   + z 
2 2  100 + x   100 + y
Illustration 15 Manoj sells a shirt to Yogesh at a profi t of 100 (100 + y ) + 100 (100 + x) 
15% and Yogesh sells it to Suresh at a loss of 10% Find the = z 
(100 + x) (100 + y ) 
 
resultant profi t or loss
Total S.P. = 2z
Solution: Here m = 15, n = –10 S.P. − C.P.
\ Overall % gain or loss = × 100
 mn  C.P.
\ Resultant profi t/loss% =  m + n + 
 100  100 z [100 + x + 100 + y ]
2z −
(100 + x)(100 + y )
 15 × − 10  = × 100
= 15 − 10 +  100 z[100 + x + 100 + y ]
 100 
(100 + x)(100 + y )
 150  2(100 + x)(100 + y ) − 100(200 + x + y )
= 15 − 10 −  =
× 100
 100  100(200 + x + y )
1 100 x + 100 y + 2 xy
= 7/2% or, 3 % =
%
2 (100 + x) + (100 + y )
which represents profi t as the sign is +ve  100( x + y ) + 2 xy 
=
 %
6. When two different articles are sold at the same  (100 + x) + (100 + y ) 
selling price, getting gain/loss of x% on the fi rst and Note:
gain/loss of y% on the second, then the overall%
gain or % loss in the transaction is given by In case y = –x, we have
 100( x + y ) + 2 xy  x2
 (100 + x) + (100 + y )  % Overall% gain or loss = − %
  100
Since the sign is –ve, there is always a loss.
The above expression represent overall gain or
loss accordingly as its sign is +ve or –ve. Illustration 16 Mahesh sold two scooters, each for `24000.
If he makes 20% profi t on the fi rst and 15% loss on the
7. When two different articles are sold at the same second, what is his gain or loss per cent in the transaction?
selling price getting a gain of x% on the fi rst and
Solution: Here x = 20 and y = –15
loss of x% on the second, then the overall% loss
in the transaction is given by \ Over all gain/loss%
 100( x + y ) + 2 xy 
 x
2 =  %
  %  (100 + x) + (100 + y ) 
 10 
100(20 − 15) + 2 × 20 × − 15 
Note that in such questions there is always a loss. =
 %
 (100 + 20) + (100 − 15) 
100 20
= – % = – %
Explanation 205 41
Let each article be sold at `z. which represents loss, being a –ve expression.

Chapter 09.indd 5 6/4/2015 7:17:49 PM


9.6 Chapter 9

Illustration 17 Rajesh sold two horses for `990 each; Illustration 19 A dis-honest shopkeeper professes to sell
gaining 10% on the one and losing 10% on the other. Find his goods at cost price but he uses a weight of 800 g for the
his total gain or loss per cent kg weight. Find his gain per cent
Solution: Here x = 10 Solution: True measure = 1000 g
2
 x False measure = 800 g
\ Overall loss% =   %
 10  Also, x = 0
2 \ Overall gain% is given by
 10 
=   % = 1% 100 + g True measure
 10  =
100 + x False measure
8. A merchant uses faulty measure and sells his 100 + g 1000
goods at gain/loss of x% The overall % gain/ ⇒ =
100 800
loss(g) is given by
1000 × 100
100 + g True measure ⇒ 100 + g =
= 800
100 + x Faulty measure
1000
Note: If the merchant sells his goods at cost price, ⇒ g = – 100 = 25%
then x = 0. 8
9. A merchant uses y% less weight/length and sells Illustration 20 A shopkeeper sells the goods at 44% loss on
his goods at gain/loss of x% The overall % gain/ cost price but uses 30% less weight. What is his percentage
loss is given by profi t or loss?
 y + x   Solution: Here x = –44 and y = 30
  × 100  %
 100 − y    y+x 
\ Overall gain/loss% =   × 100%
 100 − y 
Illustration 18 A dishonest shopkeeper professes to sell  30 − 44 
cloth at the cost price but he uses faulty metre rod. His =  × 100  %
metre rod measures 95 cm only. Find his gain per cent  100 − 30 

Solution: Here True measure = 100 cm  −14 


=  × 100  % = –20%
 70 
False measure = 95 cm.
which represents loss being a negative expression.
Since the shopkeeper sells the cloth at cost price,
\ x = 0. 10. A person buys two items for `A and sells one at a
loss of l% and other at a gain of g% If each item
\ Over all gain% is given by was sold at the same price, then
100 + g True measure (a) The cost price of the item sold at loss
=
100 + x Faulty measure A (100 + %gain)
=

(100 − %loss) + (100 + % gain)
100 + g 100
⇒ =
100 95 (b) The cost price of the item sold at gain
100 × 100 A (100 − % loss)
⇒ 100 + g = =

95 (100 − %loss) + (100 + % gain)
10000
⇒ g = – 100 Illustration 21 Ramesh buys two books for `410 and sells
95
one at a loss of 20% and the other at a gain of 25% If both
5 the books are sold at the same price, fi nd the cost price of
= 5 %
19 two books

Chapter 09.indd 6 6/4/2015 7:17:50 PM


Profi t and Loss 9.7

Solution: Cost price of the book sold at a loss of 20% 100 (100 − l ) − (100 − l ) m
= `
410 (100 + 25) 100
=
(100 − 20) + (100 + 25) (100 − l ) ⋅ (100 − m)
= `
410 × 125 100
=
= `250
80 + 125 (100 − l ) (100 − m)
Third discount at n% on `
Cost price of the book sold at a profi t of 25% 100
410 (100 − 20) 410 × 80 (100 − l ) (100 − m) n
= = = `
(100 − 20) + (100 + 25) 80 + 125 100 × 100

`160
= \ S.P. after third discount
(100 − l ) (100 − m) (100 − l ) (100 − m) n
11. If two successive discounts on an article are =
` –
100 100 × 100
m% and n%, respectively, then a single discount
equivalent to the two successive discounts will be (100 − l ) (100 − m) (100 − n)
=
`
 mn  100 × 100
m + n − %
 100   (lm + ln + mn) lmn 
=  l + m + n − + 
Explanation  100 (100) 2 
Let the marked price of the article be `100. \ Single equivalent discount
\ S.P. after the fi rst discount = `(100 – m) and  (lm + ln + mn) lmn 
(100 − m) × n  l + m + n −
= + %
discount at n% on `(100 – m) = `  100 (100) 2 
100
\ Single equivalent discount Illustration 22 Find a single discount equivalent to two
 (100 − m) × n  successive discounts of 10% and 20%
=  m +  %
 100 Solution: The equivalent single discount is given by
 100m + 100n − mn   10 × 20 
=
 % 10 + 20 −  % i.e., 28%
 100   100 
 mn  Illustration 23 Find a single discount equivalent to three
m + n −
= %
 100  successive discounts of 10%, 20% and 30%
Solution: The equivalent single discount is given by
12. If three successive discounts on an article are l%,
m% and n%, respectively, then a single discount  (10 × 20 + 10 × 30 + 20 × 30) 10 × 20 × 30 
equivalent to the three successive discounts will be 10 + 20 + 30 − + %
 100 1002 
 (lm + ln + mn) lmn 
l + m + n − + %  6 496
 100 1002  i.e.,  60 − 11 +  % = % or, 49.6%
 10  10

Explanation Illustration 24 Two shopkeepers sell machines at the same


Let the marked price of the article be `100 list price. The fi rst allows two successive discounts of 30%
and 16% and the second 20% and 26% Which discount
\ S.P. after the fi rst discount = `(100 – l)
series is more advantageous to the purchaser?
Second discount at m% on `(100 – l)
Solution: A single discount equivalent to the two successive
(100 − l ) × m
= `  30 × 16 
100 discounts of 30% and 16% is  30 + 16 − %
 100 
\ S.P. after second discount
(100 − l ) m  24  1
=
`(100 – l) – or,  46 −  % or, 41 %
100  5  5

Chapter 09.indd 7 6/4/2015 7:17:51 PM


9.8 Chapter 9

Also, a single discount equivalent to the two successive Illustration 25 A shopkeeper sold sarees at `266 each after
giving 5% discount on labelled price. Had he not given the
 20 × 26 
discounts of 20% and 26% is  20 + 26 − % discount, he would have earned a profi t of 12% on the cost
 100  price. What was the cost price of each saree?
 26  4 Solution: We have, labelled price z = `266, discount
or,  46 −  % or, 40 %
 5  5 d = 5% and profi t p = 12%
Clearly, the discount series being offered by the fi rst Using the formula:
shopkeeper is more advantageous to the purchaser.
 1002 z 
13. A shopkeeper sells an item at `z after giving a C.P. =  
discount of d% on labelled price. Had he not given  (100 − d ) (100 + p ) 
the discount, he would have earned a profi t of p% we get the cost price of each saree
on the cost price.
 100 × 100 × 266 
The cost price of each item is given by =
 
 (100 − 5) (100 + 12) 
 1002 z 
C.P. =   100 × 100 × 266
 (100 − d ) (100 + p )  =

95 × 112
= `250

Practice Exercises

Difficulty level-1
(BaseD on MeMory)

1. The cost price of 20 articles is the same as the selling price 5. A trader wants 10% profi t on the selling price of a product
of x articles. If the profi t is 25%, then the value of x is: whereas his expenses amount to 15% on sales. What
(a) 25 (b) 18 should his rate of mark up be on an article of `9?
(a) 20% (b) 25%
(c) 16 (d) 15
100
[Based on MAT, 2004] (c) 30% (d) %
3
2. A man sells an article at 5% profi t. If he had bought it at
[Based on MAT, 2003]
5% less and sold it for `1 less, he would have gained 10%
The cost price of the article is: 6. On selling a pen at 5% loss and a book at 15% gain, Karim
gains `7. If he sells the pen at 5% gain and the book at 10%
(a) `200 (b) `150 gain, then he gains `13. The actual price of the book is:
(c) `250 (d) `240 (a) `100 (b) `80
[Based on MAT, 2003] (c) `10 (d) `400
3. A shopkeeper sold a TV set for `17,940, with a discount of [Based on MAT, 2003]
8% and gained 19.6% If no discount is allowed, then what 7. By selling 33 metres of cloth, a shopkeeper gains the price
will be his gain per cent? of 11 metres of cloth. His gain per cent is:
(a) 25% (b) 26.4% (a) 7% (b) 50%
(c) 24.8% (d) None of these (c) 20% (d) 22%
[Based on MAT, 2003] [Based on MAT, 2002]
4. My friend collects antique stamps. She purchased two, but 8. If 7% of the sale price of an article is equivalent to 8% of
found that she needed to raise money urgently. So she sold its cost price and 9% of its sale price exceeds 10% of its
them for `8000 each. On one she made 20% and on the cost price by `1, then what is the cost price of the article?
other she lost 20% How much did she gain or lose in the (a) `400 (b) `350
entire transaction? (c) `300 (d) `280
(a) No loss/profi t (b) `667 loss [Based on MAT, 2002]
(c) `667 profi t (d) None of these 9. A cycle agent buys 30 bicycles, of which 8 are fi rst grade
[Based on MAT, 2003] and the rest are second grade, for `3150, Find at what

Chapter 09.indd 8 6/4/2015 7:17:52 PM


Profit and Loss 9.9

price he must sell the first grade bicycles so that if he sells 2


(a) 66 (b) 100
the second grade bicycles at three quarters of the price, he 3
may make a profit of 40% on his outlay?
1
(a) `200 (b) `240 (c) 105 (d) 120
3
(c) `180 (d) `210 [Based on MAT, 2001]
[Based on MAT, 2002] 18. Rawat sold a book at 10% loss. Had he sold it for `85
10. The retail price of a water geyser is `1265. If the more, then he would have made a profit of 7% What was
manufacturer gains 10%, the wholesale dealer gains 15% the cost price of the book?
and the retailer gains 25%, then the cost of the product is: (a) `500 (b) `850
(a) `800 (b) `900 (c) `615 (d) `585
(c) `700 (d) `600 [Based on IIT Joint Man. Ent. Test, 2004]
[Based on MAT, 2002]
19. What is the percentage profit made by selling an umbrella
11. If the cost of 12 pencils is equal to the selling price of 10 at a certain price, if by selling at two-thirds of that price,
pencils, the profit per cent in the transaction is: there would be a loss of 10%?
2 (a) 25% (b) 30%
(a) 16 % (b) 18%
3 (c) 35% (d) None of these
(c) 20% (d) 25% [Based on Narsee Monjee Inst. of Man. Studies, 2003]
[Based on MAT, 2001]
20. If 7 kg of tea of price `72/kg is blended with 33 kg of tea
12. Two motor cars were sold for `9,900 each, gaining 10% of `87/kg and 35 kg of `85/kg and the mixture is sold at
on one and losing 10% on the other. The gain or loss per 15% profit, then what is the selling price of the mixture?
cent in the whole transaction is:
(a) `80.66/kg (b) `102.33/kg
(a) Neither loss nor gain (b) 1% profit
(c) `91.22/kg (d) `97.37/kg
100
(c) % profit (d) 1% loss [Based on IIFT, 2003]
99
[Based on MAT, 2001] 21. A property dealer sells a house for `6,30,000 and
in the bargain makes a profit of 5% Had he sold it for
13. There would be 10% loss if a toy is sold at `10.80 per
`5,00,000, then what percentage of loss or gain he would
piece. At what price should it be sold to earn a profit of
have made?
20%?
(a) 15% loss (b) 15% gain
(a) `12 (b) `1296
(c) `14.40 (d) None of these 2 2
(c) 16 % gain (d) 16 % loss
[Based on MAT, 2005] 3 3
1 [Based on IIFT, 2003]
14. Ravi sells an article at a gain of 12 % . If he had sold it
2 22. Sita buys a fridge at 15/16 of its original value and sells it
at `22.50 more, he would have gained 25% The cost price for 10% more than its value. Then, the gain % is:
of the article is: (a) 15.55 (b) 11.67
(a) `162 (b) `140 (c) 16.67 (d) None of these
(c) `196 (d) `180 [Based on MAT, 2008]
[Based on MAT, 1999]
23. A cash payment that will settle a bill for 250 chairs are `50
15. A machine is sold at a profit of 10% Had it been sold for per chair less 20% and 15% with a further discount of 5%
`40 less, there would have been a loss of 10% What was on cash payment is:
the cost price?
(a) `8075
(a) `175 (b) `200
(b) 7025
(c) `225 (d) `250
[Based on MAT, 2000] (c) `8500
16. A house costs C rupees. Later it was sold for a profit of (d) None of these
25% What is the capital gains tax if it is 50% of the profit? [Based on MAT, 2008]
(a) C/24 (b) C/8 24. A shopkeeper is giving 6 kg of rice at the price of `5 per
(c) C/4 (d) C/2 kg. What should be the markup on cost price if he wants
[Based on MAT, 2000] to make a profit of 20 per cent?
17. If selling price is doubled, the profit triples. Find the profit (a) 25% (b) 50%
per cent. (c) 44% (d) 20%

Chapter 09.indd 9 6/4/2015 7:17:52 PM


9.10 I Chapter 9

25.
A sweet seller sells three-fifths part of sweets at a profit 33.
A man buys 6 dozen eggs for `10.80, and 12 eggs are
of 10% and remaining at a loss of 5% If the total profit is found rotten and the rest are sold at 5 eggs per rupee. Find
`1500, then what is the total cost price of sweets? his percentage gain or loss.
(a) `36,500 (b) `37,000 1 1
(a) 11 % gain (b) 11 % loss
(c) `37,500 (d) None of these 9 9
26. A sold an article to B at a profit of 20% B sold the same 1 1
article to C at a loss of 25% and C sold the same article to (c) 9 % gain (d) 9 % loss
11 11
D at a profit of 40% If D paid `252 for the article, then find
how much did A pay for it? 34.
If an article is sold at 8% profit instead of 8% loss, it would
(a) `175 (b) `200 have brought `12 more. Find out the cost price of the article.
(c) `180 (d) `210 (a) `75 (b) `72
27. If the absolute difference between the selling price of the (c) `60 (d) `70
article when there is 15% loss and 15% gain in selling a 35. A dishonest shopkeeper pretends to sell his goods at cost
article is `450, then what is the cost price of the article? 1
price but using false weights and gains 11 % . For a
(a) `1,200 (b) `1,500 9
(c) `2,000 (d) `2,200 weight of 1 kg he uses:
28. On selling an article at successive discounts of 20% and (a) A weight of 900 g
25%, a dealer makes a net profit of 20% Find the net profit (b) A weight of 950 g
per cent if the dealer sells the same article at a discount (c) A weight of 875 g
of 25%
(d) None of these
(a) 50% (b) 40%
[Based on NMAT, 2006]
(c) 66.66% (d) 60%
36. Rehaan purchased a bike for `54000. He sold it at a loss
29.
If the selling price of a mat is five times the discount of 8 per cent. With that money he again purchased another
offered and if the percentage of discount is equal to the bike and sold it at a profit of 10 per cent. What is his
percentage profit, find the ratio of the discount offered to overall loss/profit?
the cost price.
(a) Loss of `657 (b) Profit of `567
(a) 11:30 (b) 1:5
(c) Loss of `648 (d) Profit of `648
(c) 1:6 (d) 7:30
30.
A sells his house to B at a profit of 10% who in turn sells it 37.
A retailer bought a certain number of CDs for `1800.
to C at a profit of 15% who in turn sells it to D at a profit of Keeping one to himself, he sold the rest at a profit of `6
25% and D sells it to E at 35% profit. If cost price of E’s each. In total, he earned a profit of `114. The number of
house is `35,00,000, what is the approximate cost price of CDs he bought is:
A’s house? (a) 20 (b) 28
(a) `15,40,000 (c) 32 (d) 30
(b) `15,10,000 [Based on MAT (Sept), 2010]
(c) `15,00,000 38.
A garment company declared 15% discount for wholesale
(d) `16,40,000 buyers. Mr Sachdev bought garments from the company
31.
A book vendor sold a book at a loss of 10% Had he sold it for `25000 after getting discount. He fixed up the selling
for `108 more, he would have earned a profit of 10% Find price of garments in such a way that he earned a profit of
the cost price of the book. 8% on original company price. What is the approximate
total selling price?
(a) `432 (b) `540
(c) `648 (d) `740 (a) `28000 (b) `29000
(c) `31000 (d) `29500
32.
A dishonest dealer marks up the price of his goods by
20% and gives a discount of 10% to the customer. He also [Based on MAT (Sept), 2010]
uses a 900 g weight instead of 1 kilogram weight. Find his 39.
A person bought two tables for `2200. He sells one at 5%
profit percentage. loss and the other at 6% profit and thus on the whole he
(a) 8% neither gains nor loses. Find the cost price of each table.
(b) 12% (a) `1500, `700 (b) `2000, `200
(c) 20% (c) `1200, `1000 (d) `1100, `1100
(d) None of these [Based on MAT (Sept), 2010, (Dec), 2009]

M09_KHAT6981_C09.indd 10 6/16/2015 5:29:27 PM


Profit and Loss 9.11

40. A shopkeeper sold an article at a profit of 17.5% If he had (a) `800 (b) `900
bought it at 8% less and sold it at 30% profit, he would have (c) `700 (d) `600
earned `11.55 more as profit. Cost price of the article is: [Based on MAT (Sept), 2009]
(a) `550 (b) `675 48. A machine is sold at a profit of 10% Had it been sold for
(c) `750 (d) `1475 `40 less, there would have been a loss of 10% What was
[Based on MAT (Sept), 2010] the cost price?
41. A man invested `8000 for a year in the share market. (a) `175
At the end of the year, he gained 15% and he invested the (b) `200
amount with profit again for the second year. At the end of (c) `225
the second year, he suffered a loss of 15% Find the gain or (d) None of these
loss per cent in the investment after two years: [Based on MAT (Sept), 2009, (Dec), 2000, (Sept), 1999]
(a) Gain 3.25% (b) Loss 2.25% 49. What per cent profit would be if 34% of cost price is
(c) No loss no gain (d) Loss 5% 26% of the selling price?
[Based on MAT (May), 2010] (a) 30.77% (b) 74%
42. By selling an article at 80% of its marked price, a merchant (c) 25.16% (d) 88.40%
makes a loss of 12% What will be the per cent profit or [Based on MAT (Sept), 2009]
loss made by the merchant if he sells the article at 95% of
50. What per cent selling price would be 34% of cost price, if
its marked price?
gross profit is 26% of the selling price?
(a) 5.5% profit (b) 1% loss
(a) 25.16% (b) 74.00%
(c) 5% profit (d) 4.5% profit
[Based on MAT (May), 2010] (c) 17.16% (d) 88.40%
43. What is the maximum percentage discount that a merchant [Based on MAT (Sept), 2009]
can offer on her marked price so that she ends up selling at 51. A student purchased a computer system and a colour
no profit or loss, if she had initially marked her goods up printer. If he sold the computer system at 10% loss and the
by 50%? colour printer at 20% gain, he would not lose anything.
(a) 16.67% (b) 20% But if he sells the computer system at 5% gain and the
(c) 50% (d) 33.33% colour printer at 15% loss, he would lose `800 in the
bargain. How much did he pay for the colour printer?
[Based on MAT (May), 2010]
(a) `8000 (b) `16000
44. A tradesman gives 4% discount on the marked price and
1 article free with every 15 articles bought and still gains (c) `9000 (d) `5334
35% The marked price is more than the cost price by: [Based on MAT (Sept), 2009, 2008]
(a) 40% (b) 39% 52. A man sells a book at a profit of 20% If he had bought it
(c) 20% (d) 50% at 20% less and sold it for `18 less, he would have gained
[Based on MAT (Dec), 2009] 25% The cost price of the book is:
45. Even after reducing the marked price of a pen by `32, (a) `80 (b) `70
a shopkeeper makes a profit of 15% If the cost price be (c) `60 (d) `90
`320, what percentage of profit does he make if he sells
[Based on MAT (May), 2009]
the pen at the marked price?
53. Two-third of a consignment was sold at a profit of 5% and
(a) 25% (b) 20%
the remainder at a loss of 2% If the total profit was `400,
(c) 10% (d) 30% the value of the consignment was:
[Based on MAT (Dec), 2009]
(a) `15000 (b) `12000
46. Tarun got 30% concession on the labelled price of an
(c) `10000 (d) `20000
article and sold it at `8750 with 25% profit on the price he
bought. What was the labelled price? [Based on MAT (May), 2009]
(a) `16000 (b) `12000 54. Even after reducing the marked price of a transistor by
(c) `10000 (d) `14000 `32, a shopkeeper makes a profit of 15% If the cost price
be `320, what percentage of profit would he have made if
[Based on MAT (Dec), 2009]
he had sold the transistor at the marked price?
47. The retail price of a water geyser is `1265. If the
(a) 25% (b) 20%
manufacturer gain 10%, the wholesale dealer gains
15% and the retailer gains 25%, then the cost of the (c) 10% (d) None of these
product is: [Based on MAT (May), 2009]

Chapter 09.indd 11 6/4/2015 7:17:53 PM


9.12 Chapter 9

55. By selling 12 notebooks, the seller earns a profit equal to the 62. A manufacturer of a certain item can sell all he can
selling price of 2 notebooks. What is his percentage profit? produce at the selling price of `60 each. It costs him
(a) 25% (b) 20% `40 in materials and labour to produce each item and he
has overhead expenses of `3000 per week in order to
2 operate that plant. The number of units he should produce
(c) 16 % (d) Data inadequate
3 and sell in order to make a profit of at least `1000 per
[Based on MAT (Feb), 2009] week is:
56. A shopkeeper marks the prices of his goods at 25% higher (a) 250 (b) 300
than the original price. After that, he allows a discount of (c) 400 (d) 200
12% What profit or loss did he get? [Based on MAT (May), 2007]
(a) 15% profit (b) 10% profit 63. If the selling price of a product is increased by `162, then
(c) 10% loss (d) 15% loss the business would make a profit of 17% instead of a loss
[Based on MAT (Feb), 2009] of 19% What is the cost price of the product?
57. Ramesh purchased a bicycle for `5200 and spent `800 on (a) `540 (b) `450
its repairs. He had to sell it for `5500. Find his profit or (c) `360 (d) `600
loss per cent. [Based on MAT (May), 2007]
1 1 64. A firm of readymade garments makes both men’s and
(a) 7 % gain (b) 8 % loss
2 3 women’s shirts. Its average profit is 6% of the sales. Its
profit in men’s shirts average 8% of the sales and women’s
(c) 9% loss (d) None of these
shirts comprise 60% of the output. The average profit per
[Based on MAT (Feb), 2009]
shirt in women’s shirts is:
58. A milkman buys milk contained in 10 vessels of equal (a) 0.0466 (b) 0.0666
size. If he sells his milk at `5 a litre, he loses `200; if he
(c) 0.0166 (d) None of these
sells it at `6 a litre, he would gain `150 on the whole. Find
[Based on MAT (May), 2006]
the number of litres contained in each vessel:
65. A trader charges 20% over the cost price. He allowed a
(a) 20 L (b) 30 L
10% discount on account of Diwali on the listed price.
(c) 25 L (d) 35 L What is the net percentage gain?
[Based on MAT (Dec), 2008]
(a) 15% (b) 12%
59. A man buys apples at a certain price per dozen and sells
(c) 8% (d) 10%
them at eight times per hundred. What is his gain or loss
[Based on MAT, 1998]
per cent?
1 66. A shopkeeper sells two radios at `1540 each. On one he
(a) 4% loss (b) 8 % loss gains 12% and on the other he loses 12% What was the net
4
result of the sale of both the radios?
1 (a) No loss no profit
(c) 4% gain (d) 6 % gain
4 (b) Gain of `165
[Based on MAT (Dec), 2008] (c) Loss of `45
15 (d) Gain of `45
60. Sita buys a fridge at
of its original value and sells it
16 [Based on MAT, 1998]
for 10% more than its value. Then, gain per cent is: 67. By selling 12 marbles for a rupee, a shopkeeper loses 20%
(a) 15.55 (b) 11.67 In order to gain 20% in the transaction, he should sell the
(c) 16.67 (d) None of these marbles at the rate of how many marbles for a rupee?
[Based on MAT (Feb), 2008] (a) 8 (b) 6
61. A trader has 50 kg of rice, a part of which he sells at 10% (c) 4 (d) 3
profit and the rest at 5% loss. He gain 7% on the whole. [Based on MAT, 1998]
How much was sold at 10% gain and how much was sold 68. In a certain store, the profit is 320% of the cost. If the cost
at 5% loss? increases by 25% but the selling price remains constant,
(a) 40 kg and 15 kg approximately what per cent age of the selling price is the
(b) 30 kg and 10 kg profit?
(c) 35 kg and 40 kg (a) 30% (b) 70%
(d) 40 kg and 10 kg (c) 100% (d) 250%
[Based on MAT (Feb), 2008] [Based on MAT, 1998]

Chapter 09.indd 12 6/4/2015 7:17:53 PM


Profit and Loss 9.13

69. A loss of 19% gets converted into a profit of 17% when (a) 10% (b) 11%
the selling price is increased by `162. Find the cost price (c) 21% (d) 100%
of the article. [Based on MAT, 2000]
(a) `450 (b) `600
78. A owns a house worth `10,000. He sells it to B at a profit
(c) `360 (d) `540 of 10% based on the worth of the house. B sells the house
[Based on MAT, 1999] back to A at a loss of 10% In this transaction A gets:
70. When a commission of 36% is given on the retail price, (a) No profit no loss (b) Profit of `100
profit is 8.8% Find the profit when the commission is
decreased by 24% (c) Profit of `1,100 (d) Profit of `2,000
(a) 76% (b) 54% [Based on MAT, 2000]

(c) 58% (d) 49.6% 79. A horse and a cow were sold for `12,000 each. The horse
[Based on MAT, 1999] was sold at a loss of 20% and the cow at a gain of 20%
The entire transaction resulted in:
71. A horse and a carriage together cost `8,000. If by selling
the horse at a profit of 10%, and the carriage at a loss of (a) No loss or gain (b) Loss of `1,000
10%; a total profit of 2.5% is made, then what is the cost (c) Gain of `1,000 (d) Gain of `2,000
price of the horse? [Based on MAT, 2000]
(a) `3,000 (b) `3,500 80. A machine is sold at a profit of 10% Had it been
(c) `4,000 (d) `5,000 sold for `40 less, there would have been a loss of 10%
[Based on MAT, 1999] What was the cost price?
72. A machine is sold at a profit of 10% Had it been sold for (a) `175 (b) `200
`80 less, there would have been a loss of 10% The cost
(c) `225 (d) `250
price of the machine is:
[Based on MAT, 2000]
(a) `350 (b) `400
(c) `450 (d) `520 81. Ajay loses 20% of his money. After spending 80% of the
remainder, he is left with `4500. How much money (in
[Based on MAT, 1999]
rupees) did he have initially?
73. At what percentage above the cost price must an article
be marked so as to gain 33% after allow­ing a customer a (a) 16785 (b) 36165
discount of 5%? (c) 17165 (d) 28125
(a) 48% (b) 43% [Based on JMET, 2011]
(c) 40% (d) 38% 82. Even after reducing the marked price of a transistor by
[Based on MAT, 1999] `32, a shopkeeper makes a profit of 15% If the cost price
74. A man gains 10% by selling a certain article for a certain be `320, what percentage of profit would he have made if
price. If he sells it at double the price, then the profit he had sold the transistor at the marked price?
made is: (a) 25% (b) 20%
(a) 120% (b) 60% (c) 10% (d) None of these
(c) 100% (d) 80% [Based on MAT, 2011]
[Based on MAT, 1999] 83. A man bought two packets of toffees, the same number in
75. The sale price of an article including the Sales Tax is each. The first, he bought at 5 paise each, but the second
`616. The rate of Sales Tax is 10% If the shopkeeper has at 3 for 13 paise. He then mixed them all together and sold
made a profit of 12%, find the cost price. them at 70 paise a dozen. His gain is:
(a) `500 (b) `515 (a) 25% (b) 12%
(c) `550 (d) `600 (c) 24% (d) 16%
[Based on MAT, 1999] [Based on MAT, 2012]
84. A manufacturer sell a pair of glasses to a wholesale dealer
76. There would be 10% loss if rice is sold at `5.40 per kg. At
at a profit of 18% The wholesaler sells the same to a
what price per kg should it be sold to earn a profit of 20%?
retailer at a profit of 20% The retailer in turn sells them
(a) `12 (b) `12.96 to a customer for `30.09, thereby earning a profit of 25%
(c) `14.40 (d) `7.20 The cost price for the manufacture is:
[Based on MAT, 2000] (a) `15 (b) `16
77. If I purchased 11 books for `10 and sold all the books at (c) `17 (d) `18
the rate of 10 books for `11, the profit per cent is: [Based on MAT, 2012]

Chapter 09.indd 13 6/4/2015 7:17:53 PM


9.14 Chapter 9

85. A manufacturer of a certain item can sell all he can in materials and labour to produce each item and he has
produce at the selling price of `60 each. It cost him overhead expenses of `3000 per week in order to operate
`40 in materials and labour to produce each item and that plant. The number of units he should produce and sell
he has overhead expenses of `3000 per week in order in order to make a profit of at least `1000 per week is:
to operate the plant. The number of units he should (a) 300 (b) 400
produce and sell in order to make a profit of atleast (c) 250 (d) 200
`1000 per week, is:
[Based on MAT, 2013]
(a) 200 (b) 250
92. Aditi bought an article and spent `110 on its repairs.
(c) 300 (d) 400
She then sold it to Samir at a profit of 20% Samir
[Based on MAT, 2012]
sold it to vikas at a loss of 10% Vikash finally sold
86. A man bought two packets of toffees, the same number in
it for `1188 at a profit of 10% How much did Aditi
each. The first, he bought at 5 paise each, but the second
at 3 for 13 paise. He then mixed them all together and sold
pay for the article?
them at 70 paise a dozen. His gain is: (a) `890 (b) `1000
(a) 25% (b) 12% (c) `780 (d) `840
(c) 24% (d) 16% [Based on MAT, 2014]
[Based on MAT, 2012] 93. A shopkeeper labeled the price of his articles so as to earn
87. A manufacturer sell a pair of glasses to a wholesale dealer a profit of 30% on the cost price. He, then sold the articles
at a profit of 18% The wholesaler sells the same to a by offering a discount of 10% on the labeled price. What
retailer at a profit of 20% The retailer in turn sells them is the actual per cent in the deal?
to a customer for `30.09, thereby earning a profit of 25% (a) 18%
The cost price for the manufacture is: (b) 15%
(a) `15 (b) `16 (c) 20%
(c) `17 (d) `18 (d) None of these
[Based on MAT, 2012] [Based on SNAP, 2013]
88. A manufacturer of a certain item can sell all he can 94. A mixture of 12 kg of wheat flour costing 16 per kg and
produce at the selling price of `60 each. It cost him `40 4 kg of corn flour costing `2 per kg is sold at `16 per kg.
in materials and labour to produce each item and he has What is the profit made in selling 40 kg of the mixture?
overhead expenses of `3000 per week in order to operate
the plant. The number of units he should produce and sell (a) `140 (b) `280
in order to make a profit of atleast `1000 per week, is: (c) `300 (d) `420
(a) 200 (b) 250 [Based on SNAP, 2013]

(c) 300 (d) 400 95. A man buys a watch for `1950 in cash and sells it for
[Based on MAT, 2012] `2200 on credit of 1 year. If the rate of interest is 10% per
annum, then the man:
89. What per cent selling price would be 34% of the cost price
if the gross profit is 26% of the selling price? (a) Gains `55 (b) Gains `50
(a) 74.00% (b) 25.16% (c) loses `30 (d) Gains `30
[Based on SNAP, 2012]
(c) 17.16% (d) 88.40%
[Based on MAT, 2013] 96. A man buys apples at a certain price per dozen and sells
them at eight times per hundred. What is his gain or loss
90. A student purchased a computer system and a colour percent?
printer. If he sold the computer system at 10% loss and the
colour printer at 20% gain, he wold not lose anything. But (a) 4% loss (b) 8½ loss
if he sells the computer system at 5% gain and the colour (c) 4% gain (d) 6¼% gain
printer at 15% loss, he would lose `800 in the bargain. [Based on SNAP, 2012]
How much did he pay for the colour printer? 97. By selling 12 notebooks, the seller earns a profit equal to the
(a) `16000 (b) `8000 selling price of two notebooks. What is his percentage profit?
(c) `9000 (d) `5334 (a) 25% (b) 20%
[Based on MAT, 2013] 2
(c) 16 % (d) data inadequate
91. A manufacturer of a certain item can sell all he can 3
produce at the selling price of `60 each. It cost him `40 [Based on SNAP, 2012]

Chapter 09.indd 14 6/4/2015 7:17:54 PM


Profit and Loss 9.15

Difficulty Level-2
(Based on Memory)

1. The marked price for a pair of shoes is 60% above the cost (a) 16% (b) 15.48%
price. A shopkeeper sells the pair of shoes after giving a (c) 16.64% (d) 36%
discount of x% Now he raises the marked price by x%
[Based on FMS (Delhi), 2003]
and gives a discount of (x + 5)% If the profit earned in the
second case is twice that of the first case, find the value 8. In a game show, each participation costs `150. The 1st
of x. person to play wins `10, the second person wins `50, the
3rd person wins `100 and the 4th person wins `250. This
(a) 20 (b) 25
cycle is repeated with 5th person winning `10. After 83
(c) 10 (d) 15 people have played, how much profit has the game show
2. A showroom owner sells a leather jacket for `X and claims made for itself?
to make a profit of 10% He plans to have a stall in the (a) 4090 (b) 3990
trade fair and marks the same jacket at `2X. At the stall,
(c) 3900 (d) 8180
he allows a discount of 20% What will be the percentage
profit that he will make at the trade fair? [Based on SCMHRD En. Exam., 2003]
(a) 80% (b) 60% 9. A furniture store owner has determined that he can sell
(c) 76% (d) None of these 100 chairs a month at a selling price of `200 each. For
each rise of `4 in the selling price, he will sell 2 less
[Based on IIT Joint Man. Ent. Test, 2004]
chairs a month. If he sells the chairs for `Z each, then
3. If a person makes a profit of 10% on one-fourth of the how much money a month will he receive from the sale
quantity sold and a loss of 20% on the rest, then what is of chairs?
his average per cent profit or loss?
(a) 200 – Z/2 (b) 50Z + Z2/4
(a) 12.5% profit (b) 11.25% loss
(c) 150Z – Z2/4 (d) 200Z – Z2/2
(c) 11.75% profit (d) 12.5% loss
[Based on IIT Joint Man. Ent. Test, 2004] [Based on SCMHRD Ent. Exam., 2003]

4. The initial profit percentage for a shirt is 87.5%, which is 10. To make an article, it takes 40 hrs for a workman who is
sold after giving a discount of 20% Due to some reason paid `1.80 per hour. The material, 20 per cent of which is
the cost price of making a shirt increases by 25% Now a wasted in the course of working, costs `22.5 per kilogram.
shirt is sold after raising the marked price by `500 and The final weight of the article is 8 kg. At what price it
giving a discount of 25% If the percentage profit still must be sold so as to yield a profit of 33.33 per cent?
remains the same, find the new selling price of the shirt. (a) `360 (b) `404
(a) `800 (b) `1,000 (c) `396 (d) `384
(c) `1,200 (d) `1,500
11. A shopkeeper sold an article for `6,750 after giving a
5. A manufacturer of a certain item can sell all he can discount of 10% on the labelled price. He would have
produce at the selling price of `60 each. It costs him `40 earned a profit of 50%, had there been no discount. What
in materials and labour to produce each item and he has was the actual percentage of profit earned?
overhead expenses of `3000 per week in order to operate
that plant. The number of units he should produce and sell (a) 36 (b) 40
in order to make a profit of at least `1000 per week is: (c) 35 (d) None of these
(a) 400 (b) 300 [Based on IRMA, 2002]
(c) 250 (d) 200 12. The evergreen shrubs at Ravi’s nursery are planted in rows
[Based on FMS (Delhi), 2003] on a square plot of land measuring 2,401 square ft. The
shrubs are planted in such a manner that the centres of the
6. Profit after selling an article for `425 is the same as the
shrubs are 7 ft apart and the outer shrubs are planted along
loss after selling it for `355. The cost of the article is:
the edges of the plot, with a shrub at each corner. Ravi
(a) `385 (b) `390 spent `896 to cover all the costs necessary for raising this
(c) `395 (c) `400 crop of the evergreen shrubs. If Ravi succeeds in selling
[Based on FMS (Delhi), 2003] each shrub for `35, his profit will be what percentage of
his total cost?
7. A tradesman by means of false balance defrauds to the
extent of 8% in buying goods and also defrauds to the (a) 100% (b) 50%
extent of 8% in selling. His gain per cent is: (c) 125% (d) 150%

Chapter 09.indd 15 6/4/2015 7:17:54 PM


9.16 Chapter 9

13. A wealthy man bought two flats in a posh locality to have 19. The cost price of three varieties of apples namely A, B and
a fixed return. But a financial crunch at his home forced C is `20/kg, `40/kg and `50/kg. Find the selling price of
him to sell both the flats soon, and he sold both at the one kg of apple in which these three varieties of apples are
rate of `3,00,000 each. He thereby incurred a loss of 20% mixed in the ratio of 2:3:5 such that there is a net profit of
on one of the flats and a gain of 20% on the other. What 20%?
was the overall profit or loss he incurred in the entire (a) `48 (b) `48.6
transaction? (c) `49.2 (d) `49.8
(a) No gain, no loss (b) `25,000 profit
20. The market price of an article was 40% more than its cost
(c) `25,000 loss (d) `10,000 loss price. I was going to sell it at market price to a customer,
14. A shopkeeper marked price of an article as 150% of cost but he showed me some defects in the article, due to
price and he has two schemes which I gave him a discount of 28.57% Next day he came
(i) Scheme A: You can purchase one article at a discount again and showed me some more defects, hence I gave
of 20% him another discount that was equal to 12.5% of the cost
price. What was the approximate loss to me?
(ii) Scheme B: Purchase two articles at a discount of
(a) Loss of 10% (b) Loss of 12.5%
30% each.
(c) Loss of 15% (d) None of these
He sold 80% of articles under the scheme B and rest under
the scheme A. Then, his overall profit is: 21. A shopkeeper buys a toy at `100 and sells it at `120.
Another shopkeeper buys the same toy at `120 but sells it
(a) 8% (b) 12% at `100. What are the respective profit/loss per cent for the
(c) 16% (d) 6% two shopkeepers?
15. The amount of wheat at the rate of `610 per quintal which (a) 20%, 20% (b) 20%, 16.7%
should be added to 126 quintals of wheat costing `285 per (c) 16.7%, 16.7% (d) 16.7% 10%
quintal so that 20% may be gained by selling the mixture
at `480 per quintal will be: 22. Cost price of 12 oranges is equal to the selling price of
9 oranges and the discount on 10 oranges is equal to the
(a) 38 quintals profit on 5 oranges. What is the percentage point difference
(b) 49 quintals between the profit percentage and discount percentage?
(c) 69 quintals (a) 20 (b) 22.22
(d) None of the above (c) 16.66 (d) 15
16. Amit went to Mumbai and bought a pair of watches 23. If books bought at prices ranging from `200 to `350 are
costing `360 at 25% discount on each but on the way sold at prices ranging from `300 to `425. What is the
back he loses one of these watches and had to buy them greatest possible profit that might be made in selling eight
(pair) again home. How much did he totally spend on the books?
watches? (a) `800 (b) `1,800
(a) `620 (b) `720 (c) `1,800 (d) None of these
(c) `540 (d) `630
24. A businessman marked the price of his goods 30% more
17. The cost price of 16 apples is equal to the selling price 1
of 10 apples. The cost price of 12 oranges is equal to the than his C.P. He then sells th of his stock at a discount
4
selling price of 16 oranges and the cost price of 6 mangoes
of 15%, and half of the stock at the marked price, and the
is equal to the selling price of 4 mangoes. If the ratio of
rest at a discount of 30% Find his gain percentage.
the cost price of 1 apple, 1 orange and 1 mango is in the
ratio of 1:1:2, then find the net profit per cent on the sale (a) 16.5% (b) 15.375%
of 1 apple, 2 oranges and 2 mangoes. (c) 14.20% (d) 13.37%
(a) 25% (b) 30% 25. When a bicycle manufacturer reduced its selling price by
(c) 35% (d) 40% 50%, the number of bicycles sold radically increased by
600% Initially the manufacturer was getting only 140%
18. The market price of an article is `100. If it is sold at a
profit. What is the percentage increase of his profit?
discount of 10%, a profit of 35% is made. How much loss
of profit will be made if it is sold for ` less then the market (a) 10%
price? (b) 14%
(a) 5% loss (b) 8% gain (c) 0%
(c) 5% gain (d) 8% loss (d) Cannot be determined

Chapter 09.indd 16 6/4/2015 7:17:54 PM


Profit and Loss 9.17

26. The marked price of a watch is `1,600. The shopkeeper two and sold the entire quantity at the rate of `18.60 per
gives successive discount of 10%, r% to the customer. kg and made 20% overall profit. At what price per kg did
If the customer pays `1,224 for the watch, find the value he purchase the lot of another 30 kg rice?
of r. (a) `14.50 (b) `13.50
(a) 10% (b) 20% (c) `12.50 (d) `15.50
(c) 25% (d) 15% [Based on NMAT, 2005]
27. A trader sells goods to a customer at a profit of k% 33. A trader purchases a watch and a wall clock for `390.
over the cost price, besides it he cheats his customer by He sells them marking a profit of 10% on the watch and
giving 880g only instead of 1 kg. Thus his overall profit 15% on the wall clock. He earns a profit of `51.50. The
percentage is 25% Find the value of k. difference between the original prices of the wall clock
(a) 8.33% (b) 8.25% and the watch is equal to:
(c) 10% (d) 12.5% (a) `110 (b) `100
28. Ms. Priyanka sold two properties X and Y for `30000 (c) `80 (d) `120
each. She sold property X for 20% more than she paid for [Based on NMAT, 2005]
it and sold property Y for 20% less than she paid for it. If 34. A shopkeeper sold an air-conditioner for `25935 at a
the expenses are disregarded, what was her gain or loss, if discount of 9% and earned a profit of 3.74% What would
any, on the two properties? have been the percentage of profit earned if no discount
(a) Gain of `2500 had been offered?
(b) Loss of `2500 (a) 15.6% (b) 16%
(c) Gain of `1250 (c) 12.3% (d) None of these
(d) There was neither a net gain or a net loss  [Based on IRMA, 2005]
[Based on ATMA, 2005] 35. Pure Ghee costs `100 per kg. After adulterating it with
29. A farmer sold a cow and an ox for `800 and got a profit of vegetable oil costing `50 per kg, a shopkeeper sells the
20% on the cow and 25% on the ox. If he sells the cow and mixture at the rate of `96 per kg, thereby making a profit
the ox for `820 and gets a profit of 25% on the cow and of 20% . In what ratio does he mix the two?
20% on the ox, the individual cost price of the cow and the (a) 1:2 (b) 3:2
ox is:
(c) 3:1 (d) None of these
(a) `515.60, `115.60 (approx.) [Based on FMS (MS), 2006]
(b) `531.50, `135.50 (approx.)
36. A fruit vendor professes to sell fruits at the cost price, but
(c) `530.60, `131.60 (approx.) uses false weights. He gains 30% in this manner. What
(d) Cannot be determined weight does he substitute for one kilogram?
[Based on NMAT, 2005] 1
(a) 645 g (b) 750 g
30. A man sells two horses for `1485. The cost price of the 3
first is equal to the selling price of the second. If the first
is sold at 20% loss and the second at 25% gain, what is his 3
(c) 769 g (d) 800 g
total gain or loss (in rupees)? 13
(a) `80 gain (b) `60 gain [Based on JMET, 2011]
(c) `60 loss (d) Neither gain nor loss 37. A dishonest dealer sells his goods at the cost price and still
[Based on NMAT, 2005] earns a profit of 25% by under weighing. What weight
31. Vineet calculates his profit percentage on the selling price does he use for a kilogram?
whereas Roshan calculates his profit on the cost price. (a) 750 g (b) 800 g
They find that the difference of their profit is `275. If the (c) 825 g (d) 850 g
selling price of of both them are the same and Vineet gets
[Based on FMS, 2005]
25% profit and Roshan gets 15% profit, then find their
selling price.
Directions (Q. 38-39): Based on the information given below.
(a) `2100 (b) `2300
Books and More sells books, music CDs and film DVDs. In
(c) `2350 (d) `2250
December 2009, they earned 40% profit in music CDs and 25%
[Based on NMAT, 2005] profit in books. Music CDs contributed 35% towards their total
32. Padam purchased 30 kg of rice at the rate of `17.50 per kg sales in rupees. At the same time total sales in rupees from books
and another 30 kg of rice at a certain rate. He mixed the is 50% more than that of music CDs.

Chapter 09.indd 17 6/4/2015 7:17:54 PM


9.18 Chapter 9

38. If Books and More have earned 20% profit overall, then in (a) 30 (b) 20
film DVDs they made: (c) 25 (d) None of these
(a) 15.2% profit (b) 10.0% profit [Based on IIFT, 2007]
(c) 10.0% loss (d) 23.4% loss 45. A Techno Company has 14 machines of equal efficiency
[Based on XAT, 2010] in its factory. The annual manufacturing costs are `42000
39. If Books and More made 50% loss in film DVDs, then and establishment charges are `12000. The annual
overall they made: output of the company is `70000. The annual output
and manufacturing costs are directly proportional to the
(a) 12.3% profit (b) 8.7% profit
number of machines. The shareholders get 12.5% profit,
(c) 0.4% loss (d) 6.25% loss which is directly proportional to the annual output of the
[Based on XAT, 2010] company. If 7.14% machines remain closed throughout
40. A dealer sold a radio at a loss of 2.5% Had he sold it for the year, then the percentage decrease in the amount of
1 profit of the shareholders would be:
`100 more, he would have gained 7 % In order to gain
2 (a) 12% (b) 12.5%
1 (c) 13% (d) None of these
12 %, he should sell it for:
2 [Based on IIFT, 2010]
(a) `850 (b) `925
46. A small and medium enterprise imports two components A
(c) `1080 (d) `1125 and B from Taiwan and China respectively and assembles
[Based on FMS, 2006] them with other components to form a toy. Component
41. A space research company wants to sell its two products A A contributes to 10% of production cost. Component B
and B. If the product A is sold at 20% loss and the product contributes to 20% of the production cost. Usually, the
B at 30% gain, the company will not lose anything. If the company sells this toy at 20% above the production cost.
product A is sold at 15% loss and the product B at 15% Due to increase in the raw material and labour cost in
gain, the company will lose `6 million in the deal. What is both the countries, component A became 20% costlier
the cost of product B? and component B became 40% costlier. Owing to these
reasons the company increased its selling price by 15%
(a) `140 million (b) `120 million
Considering that cost of other components does not
(c) `100 million (d) `80 million change, what will be the profit percentage, if the toy is
[Based on FMS, 2009] sold at the new price?
1 (a) 15.5% (b) 25.5%
42. A jobber buys an article at “`24 less 12 % ”. He then
2 (c) 35.5% (d) 40%
1 [Based on IIFT, 2010]
wishes to sell the article at a gain of 33 % of his cost
3 47. A salesman sells two kinds of trousers—cotton and
after allowing a 20% discount on his marked price. At woollen. A pair of cotton trousers is sold at 30% profit
what price, in rupees, should the article be marked? and a pair of woollen trousers is sold at 50% profit.
The salesman has calculated that if he sells 100% more
(a) 30.00 (b) 33.60
woollen trousers than cotton trousers, his overall profit
(c) 40.00 (d) None of these will be 45% However he ends up selling 50% more cotton
[Based on FMS, 2010] trousers than woollen trousers. What will be his overall
43. A farmer bought 749 sheep. He sold 700 of them for the profit?
price paid for the 749 sheep. The remaining 49 sheep were (a) 37.5% (b) 40%
sold at the same price per head as the other 700. Based on (c) 41% (d) 42.33%
the cost, the per cent gain on the entire transaction is:
[Based on XAT, 2009]
(a) 6.5 (b) 6.75
48. A shopkeeper gives two successive discounts of 10 and
(c) 7.0 (d) 7.5
20 % on a marked price of `5000 for a bicycle. He had to
[Based on FMS, 2010] give a further discount of 20% of the cost price on his new
44. Sumit works as a state contractor for PWD and supplies selling price, as a result of which he made neither a profit
bitumen mix for road construction. He has two varieties nor a loss. Find the cost price of the bicycle.
of bitumen, one at `42 per kg and the other at `25 per kg.
(a) `2000 (b) `3000
How many kg of first variety must Sumit mix with 25 kg
of second variety, so that he may, on selling the mixture at (c) `3600 (d)
`4500
40 kg, gain 25% on the outlay? [Based on CAT, 2009]

Chapter 09.indd 18 6/4/2015 7:17:54 PM


Profit and Loss 9.19

Directions (Q. 49–50): Answer the questions based on the (a) 500 (b) 700
following information. (c) 800 (d) 1000
Mr. David manufactures and sells a single product at a fixed price [Based on CAT, 1996]
in a niche market. The selling price of each unit is `30. On the 55. Instead of metre scale, cloth merchant uses a 120 cm
otherhand, the cost, in rupees, of producing x units is 240 + bx scale while buying, but uses an 80 cm scale while selling
+ cx2, where b and c are some constants. Mr. David noticed that the same cloth. If he offers a discount of 20% on cash
doubling the daily production from 20 to 40 units increases the payment, what is his overall profit percentage?
2 (a) 20% (b) 25%
daily production cost by 16 % . However, an increase in daily
3 (c) 40% (d) 15%
production from 40 to 60 units results in an increase of only 50% [Based on CAT, 1996]
in the daily production cost. Assume that demand is unlimited and 56. A stockiest wants to make some profit by selling sugar.
that Mr. David can sell as much as he can produce. His objective He contemplates about various methods. Which of the
is to maximize the profit. following would maximize his profit?
49. How many units should Mr. David produce daily? I. Sell sugar at 10% profit
(a) 100 (b) 70 II. Use 900 g of weight instead of 1 kg.
(c) 150 (d) 130 III. Mix 10% impurities in sugar and selling sugar at cost
[Based on CAT, 2007] price.
50. What is the maximum daily profit, in rupees, that Mr. IV. Increase the price by 5% and reduce weights by 5%
David can realize from his business? (a) I or II (b) II
(a) 920 (b) 840 (c) II, III, and IV (d) Profit are same
(c) 760 (d) 620 [Based on CAT, 1995]
(e) Cannot be determined 57. A dealer offer a cash discount of 20% and still makes
[Based on CAT, 2007]
a profit of 20%, when he further allows 16 articles to
a dozen to a particularly sticky bargainer. How much
51. After allowing a discount of 11.11%, a trader still makes a percent above the cost price were his wares listed?
gain of 14.28% At what percent above the cost price does
(a) 100% (b) 80%
he mark in his goods?
2
(a) 28.56% (b) 35% (c) 75% (d) 66 %
3
(c) 22.22% (d) None of these [Based on CAT, 1994]
[Based on CAT, 1997]
58. A driver of an auto-rickshaw makes a profit of 20%
52. A dealer buys fruits at `100, 80 and 60 per kg. He mixes on every trip when he carries three passengers and
them in ratio 3: 4: 5 by weight and sells at a profit of 50% the price of petrol is `30/L. Find the percentage
At what price per kg does he sell the dry fruit?
profit for the same journey if he goes for four
(a) `80 (b) `100
passenger per trip and the price of petrol reduces to
(c) `95 (d) None of these `24/L? (Assume that revenue per passenger is the
[Based on CAT, 1997] same in both the cases).
(a) 33.33%
Directions (Q. 53–54): Answer the questions based on the
following information. A watch dealer incurs an expense of `150 (b) 65.66%
for producing every watch. He also incurs an additional expenditure (c) 100%
of `30,000 that is independent of the number of watches produced. (d) Data inadequate
If he is able to sell a watch during the season, he sells it for `250. If [Based on MAT, 2014]
he fails to do so, he has to sell each watch for `100.
59. After selling a watch, Sultan found that he had
53. If he is able to sell only 1200 out of 1500 watches he has made a loss of 10% He also found that had he sold
made in the season, then he has made a profit of: it for `27 more, he would have made a profit of
(a) `90,000 (b) `75,000 5% The actual initial loss was what percentage of
(c) `45,000 (d) `60,000 the profit earned, had he sold the watch for a 5%
[Based on CAT, 1996] profit?
54. If he produces 1500 watches, what is number of watches (a) 23% (b) 150%
that he must sell during the season in order to breakeven, (c) 180% (d) 200%
given that he is able to sell all the watches produced? [Based on MAT, 2014]

Chapter 09.indd 19 6/4/2015 7:17:54 PM


9.20 Chapter 9

60. A dealer buys dry fruit at the rate of `100, `80 and `60 (a) 20% (b) 24%
per kg. He bought them in the ratio 12 : 15 : 20 by weight. (c) 25% (d) 30%
He in total gets 20% profit by selling the first two and at [Based on SNAP, 2012]
last he finds he has no gain no less in selling the whole
quantity which he had. What was the percentage loss he 62. A manufacturer of a certain item can sell all he can
suffered for the third quantity? produce at the selling price of `60 each. It costs him `40
(a) 40% (b) 20% in materials and labour to produce each item and he has
overhead expenses of `3,000 per week in order to operate
(c) 30% (d) 50%
that plane. The number of units he should produce and
[Based on SNAP, 2012] sell in order to make a profit of at least `3,000 per week in
61. 11. The capital of company is made up of 50,000 order to operate that plant. The number of units he should
preferred shares with dividend of 20% and 20,000 produce and sell in order to make a profit of at least `1,000
common shares, the par value of each type of share per week is:
being 10. The company had a total profit of 1,80,000 out (a) 300 (b) 250
of which 30,000 was kept in reserve and the remaining
(c) 400 (d) 200
distributed to shareholders. Find the dividend percent to
the common shareholders. [Based on SNAP, 2012]

Answer Keys
Difficulty Level-1

1. (c) 2. (a) 3. (d ) 4. (b) 5. (d ) 6. (b) 7. (b) 8. (b) 9. (c) 10. (a) 11. (c) 12. (d ) 13. (c)
14. (d ) 15. (b) 16. (b) 17. (b) 18. (a) 19. (c) 20. (d ) 21. (d ) 22. (d ) 23. (a) 24. (c) 25. (c) 26. (b)
27. (b) 28. (a) 29. (d ) 30. (d ) 31. (b) 32. (c) 33. (a) 34. (a) 35. (a) 36. (d ) 37. (a) 38. (c) 39. (c)
40. (a) 41. (b) 42. (d ) 43. (d ) 44. (d ) 45. (a) 46. (c) 47. (a) 48. (b) 49. (a) 50. (a) 51. (b) 52. (d )
53. (a) 54. (a) 55. (b) 56. (b) 57. (b) 58. (d ) 59. (a) 60. (d ) 61. (d ) 62. (d ) 63. (b) 64. (a) 65. (c)
66. (c) 67. (a) 68. (b) 69. (a) 70. (d ) 71. (d ) 72. (b) 73. (c) 74. (a) 75. (a) 76. (d ) 77. (c) 78. (c)
79. (b) 80. (b) 81. (d ) 82. (a) 83. (a) 84. (c) 85. (a) 86. (a) 87. (c) 88. (a) 89. (b) 90. (a) 91. (d )
92. (a) 93. (d ) 94. (a) 95. (a) 96. (a) 97. (b)

Difficulty Level-2

1. (b) 2. (c) 3. (d ) 4. (d ) 5. (d ) 6. (b) 7. (c) 8. (a) 9. (d ) 10. (c) 11. (c) 12. (d ) 13. (c)
14. (a) 15. (c) 16. (c) 17. (b) 18. (c) 19. (c) 20. (b) 21. (b) 22. (b) 23. (b) 24. (b) 25. (c) 26. (d )
27. (c) 28. (b) 29. (c) 30. (d ) 31. (b) 32. (b) 33. (a) 34. (d ) 35. (b) 36. (c) 37. (b) 38. (d ) 39. (b)
40. (d ) 41. (d ) 42. (d ) 43. (c) 44. (d ) 45. (b) 46. (b) 47. (b) 48. (b) 49. (a) 50. (c) 51. (a) 52. (d )
53. (b) 54. (b) 55. (a) 56. (b) 57. (a) 58. (c) 59. (d ) 60. (a) 61. (c) 62. (d )

Chapter 09.indd 20 6/4/2015 7:17:55 PM


Profit and Loss 9.21

Explanatory Answers

Difficulty Level-1

1. (c) Let S.P. of x articles = `100 8000 × 100


= C.P. of 20 articles = = 10000
80
\ C.P. of one article = `5
50000
Profit = 25% \ C.P. of both the stamps =
\ S.P. of one article = 6.25 3
⇒ S.P. of x articles = 6.25x S.P. of both the stamps =16000
2000
\ 6.25x = 100 \ Loss =
⇒ x = 16. 3
= `666.67 ≈ `667.
2. (a) Let the C.P. of the article be `x.
5. (d) Let S.P. = `x
Profit = 5%
21x \ Expenses = 15% of x = 0.15x
\ S.P. = x + 5% of x = `
20 Profit = 10% of x = 0.10x
C.P. = `9 (Given)
19 x
If C.P. would have been (x – 5% of x), i.e., ` and \ 9 + 0.15x + 0.1x = x
20
⇒  x = 12
 21x 
S.P. would have been `  − 1 , then gain % = 10 \ The rate of mark up on the article should be
 20 
100
 21x  19 x %
 − 1 − 3
\  20  20 × 100 = 10
19 x 6. (b) Let actual price of the book = `x
20 Let actual price of the pen = `y
2 x − 20 \ (x + 15% of x) + (y – 5% of y) = x + y + 7
⇒ × 100 = 10
19 x ⇒ 15x – 5y = 700 (1)
⇒ 19x = 20x –­ 200 Also (x + 10% of x) + (y + 5% of y) = x + y + 13
⇒ x = 200. ⇒ 10x + 5y = 1300 (2)
Using (1) and (2), we get x = 80, y = 100
3. (d) S.P. = `17940
Discount = 8% \ Actual price of the book = `80.
17940 × 100 7. (b) Suppose S.P. of 33 metres of cloth = `33
\ Marked Price = = `19500
100 − 8 \ Gain = `11 ⇒ C.P. = `22
Gain = 19.6% ⇒ Gain = 50%
17940 × 100
C.P. = = `15000 8. (b) Suppose C.P. = x and S.P. = y
100 + 19.6
If no discount is allowed on the Marked Price, then ⇒ 7% of y = 8% of x
S.P. = `19500 and 9% of y = 10% of x + 1
4500 ⇒ 7y = 8x and 9y = 10x + 100
\ Gain % = × 100 = 30.
15000 8x
⇒ 9× = 10x + 100
7
8000 × 100 ⇒ x = 350.
4. (b) C.P. of one stamp =
100 + 20
9. (c) Suppose the price of first grade cycle = `x and the
8000 × 100 20000
= = price of second grade cycle = `y.
120 3 \ 8x + 22y = 3150
8000 × 100 Suppose he sells the first grade bicycles @ `z per
C.P. of another stamp =
100 − 20 bicycle

Chapter 09.indd 21 6/4/2015 7:17:55 PM


9.22 Chapter 9

3z 15. (b) Let the C.P. be `k


\ 8 z + 22 × = 3150 + 40% of 3150 11k
4 \ S.P. = k + 10% of k =
i.e., 98z = 17640 10
⇒ z = 180 11k
\ k – 10% of k = – 40
10
\ S.P. of the first grade bicycle = `180
9k 11k
S.P. of the second grade bicycle = `135. ⇒ = − 40
10 10
10. (a) Suppose the cost = `x 2k

= 40
\ S.P. of the manufacturer 10
11x
= x + 10% of x = ⇒ k = 200.
10
⇒ S.P. of the wholesale dealer
16. (b) C.P. = `C
11x 11x C
= + 15% of Profit = 25% of C =
10 10 4
11x 33 x 253 C 5C
= + = x S.P. = C + =
10 200 200 4 4
⇒ S.P. of the retailer C
If = 50% profit, then Capital Gains Tax = 50% of
253 253 4
= x 25% of x
200 200 C C
= .
253 253 1265 4 8
= x+ x = x
200 800 800
1265 17. (b) Let C.P. = `x
\ x = 1265 \ S.P. = `y.
800
⇒ x = 800. Profit = `(y – x)
If S.P = 2y, then profit = 3(y – x)
11. (c) Let C.P. of 12 pencils = `12 \ 2y – x = 3(y – x)
\ S.P. of 10 pencils = `12 ⇒ y = 2k
C.P. of 10 pencils = `10  \ Profit on `x = `x; i.e., 100%
\ Profit = 20%
18. (a)  Let C.P. of the book = `x
12. (d) C.P. of the 1st Motor Car \ x – 10% of x + 85 = x + 7% of x
9900 × 100 ⇒ 17% of x = 85
= = `9000
100 + 10 ⇒ x = 500.
C.P. of the 2nd Motor Car
9900 × 100 19. (c) Let C.P of the umbrella = `x
= = `11000
100 − 10 Let S.P. of the umbrella = `y
\ Total C.P. = `20000 2
x− y
Total S.P. = `19800 \ 3 × 100 = 10
200 x
\ Loss % = × 100 = 1. 3x − 2 y
20000 ⇒ × 100 = 10
3x
13. (c) 90:10.80 = 120:x
 2 y
90 120 ⇒ 1 − ×  × 100 = 10
=  3 x
10.80 x y
⇒ = 1.35
120 × 10.80 x
\ x =
90 y−x
= 14.40. \ Profit % = × 100
x
1 y
14. (d) 12 % = `22.50 = 100 − 100
2 x
⇒ C.P. = `180. = 135 – 100 = 35.

Chapter 09.indd 22 6/4/2015 7:17:56 PM


Profit and Loss 9.23

20. (d) C.P./kg of the mixture 27. (b) 115% of C.P. – 85% of C.P. = `450
7 × 72 + 33 × 87 + 35 × 85 ∴ C.P. = `1,500
=
75
28. (a) Let the cost price and market price of the article be ‘x’
504 + 2871 + 2975 and ‘y’ respectivley.
=
75 Case 1: Successive discounts of 20% and 25%
6350 Selling price of the aritcle = (0.6)0.75y) = 0.6y
= = `84.66
75 Therefore, 0.6y = 1.2x  or,  y = 2x
Profit = 15% Case 2: A single discount of 25%
Selling price of the article = 0.75y = 1.5x
\ S
.P. per kg = `84.66 + 15% of `84.66
 (1.5 x  x) 
= `97.37. Net profit per cent =  100 = 50%
 ( x) 
21. (d) S.P. = `630000
29. (d) Since S.P. = 5 (M.P. – S.P.)
Profit = 5%
⇒ 5 M.P. = 6 S.P.
\ C.P. = `600000 6
⇒ M.P. = S.P.
If S.P. would have been `500000, then there would 5
2
have been a loss of 16 % Since the percentage discount = Percentage profit,

3 6
S.P. – S.P. S. P . − C . P .
22. (d) Let original value = x
5 × 100 = × 100
15 15 6 C.P.
C.P. = ×x= x S.P.
16 16 5
1 S.P. 7
10 11 ∴ = – 1 ⇒ S.P. =  C.P.
S.P. = x + ×x= x 6 C.P. 6
100 10
11 15 6 6 7 7
x− x M.P. = S.P. = × C.P. =  C.P.
10 16 52 5 5 6 5
gain% = × 100% = = 17.33%
15 3
x ∴ Ratio of discount to C.P.
16
M.P. – S.P.
=
 80  85  95  C.P.
23. (a) `250 × 50     = `8075
 100  100  100 
24. (c) Let x be the cost price. 7 7
C.P. – C.P. 7
= 5 6 = .
∴ cost of 6 kgs of rice = 6x 30
C.P.
With 20% profit margin, selling price = 6x × 1.2
30. (d) Let the cost price of A’s house = `x
This is the price charged for 5 kg ∴ Cost price of E’s house
6 x  1.2 = 1.1 × 1.15 × 1.25 × 1.35 × x
∴ Selling price of 1 kg = = 1.44x
5 ∴ 3500000 = 1.1 × 1.15 × 1. 25 × 1.35 × x
∴ Markup = 44%
3500000
∴ x =
25. (c) Assume A be the cost price. 1.1  1.15  1.25  1.35
3 10 2 5 
∴   A   A = 1500
100 
3500000
5 100 5
≈ ≈ `16,47,059
1.26  1.7
or,  A = `37,500
The nearest option is `16,40,000.
26. (b) Let the article costs ‘x’ to A
Cost price of B = 1.2x
31. (b) Given: 108 = 10% loss + 10% profit
Cost price of C = 0.75(1.2x) = 0.9x = 20% extra
Cost price of D = 1.4(0.9x) = 1.26x = 252 Now, 20% = 108
Amount paid by A for the article = `200 ∴ 100% = 540

Chapter 09.indd 23 6/4/2015 7:17:56 PM


9.24 Chapter 9

32. (c) Let the C.P. of 1,000 gm of goods be `1,000 39. (c) Let the cost price of one table be x.
Marked price = `1,200 Then, cost price of other table will be (2200 – x).
Selling price = `1,200 × 0.9
95 106
= `1,080 x× + (2200 – x) × = 2200
100 100
(after a discount of 10%)
`1,080 is the selling price of 900 g of goods (as he ⇒ 95x + 233200 – 106x = 220000
cheats to the extent of 10% while selling) ⇒ 11x = 13200
C.P. of 900 g = `900
⇒ x = `1200
Profit = `180
and, 2200 – x = `1000
180
∴ Profit % =  100  = 20%
900 40. (a) Let cost price of article be `x.
33. (a) 6 dozens eggs cost = `10.80 Then,
Since one dozen is rotten, he sells only 5 dozen at 5
117.5 92 130
eggs per rupee. × + 11.55 = x × ×
100 100 100
Hence, S.P. = `12
(12  10.8) 1 11.55
this gain % =  100  = 11 % ⇒ x = × 100
10.8 9 (0.92 × 130 − 117.5)
34. (a) Let the C.P. of the article be x. = `550
1.08x – 0.92x = 12
12
⇒ 0.16x = 12 ⇒ x = = `75 41. (b) Total amount after one year
0.16
35. (a) Let gain = x 115
= 8000 × = `9200
100 x 100
% = × 100%
9 1000 −x 85
Total amount after 2nd year = 9200 × = 7820
⇒ x = 100 100
\ He uses 1000 – 100 = 900 g 8000 − 7820
\ Loss per cent = × 100 = 2.25%
8000
36. (d) Cost price = 54 000
Short cut method
(100 − 8)
Selling price = 54000 × When a value is increased and then decreased by same
100
= 49680 percentage, then the value is always decreased and it
Now, the cost price of another bike = 49680 x2
is decreased by %
110 100
Selling price of another bike = 49680 × = 54648
100 (15) 2
So, loss per cent = = 2.25%
Overall profit = 54648 – 54000 = 648 100
37. (a) Let he bought x number of CDs. 42. (d) Let the cost price of article be `x.
\ (x – 1) × 6 = 114 Then, selling price of article = 0.88x
⇒ x – 1 = 19 Marked price of article
⇒ x = 20 0.88
= × 100 × x = 1.1x
80
38. (c) Cost price of garments = `25000
New selling price of article = 1.045x
25000 1.045 x − x
Original company price = × 100 \ Profit per cent = × 100 =
4.5%
85 x
\ Selling price of garments
43. (d) Let cost price = `100
25000 108
= × 100 × Marked price = `150
85 100 50
\ Discount per cent = × 100 = 33.33%
= `31764.71 ≈ `31000 150

Chapter 09.indd 24 6/4/2015 7:17:57 PM


Profit and Loss 9.25

44. (d) Discount on articles 50. (a) Let the selling price be `100, then profit = `26
1 \ Cost price = 100 – 26 = `74
= × 100 = 6.25%
16 \ Required per cent = (34% of 74)%
Overall discount = 25.16%
4 × 6.25
= –4 – 6.25 + = –10%
100 51. (b) Let the cost price of colour printer and computer
Let cost price = `100, then
system be x and y respectively.
Selling price = `135
So, 90% of marked price = 135 120 90

+ y× =x+y
135 × 100 100 100
Marked price = = `150 ⇒ 0.2x = 0.1y (1)
90
Marked price is increased by 85 105
x× + y× = x + y – 800
150 − 100 100 100
= × 100 = 50%
100 ⇒ 0.05y = 0.15x – 800 (2)
From Eqs. (1) and (2),
115 115 x = `16000
45. (a) S.P. = C.P. = × 320 = `368
100 100
And S.P. = MP – 32 52. (d) Let the cost price of book be `x.
So, MP = S.P. + 32 = 368 + 32 = `400 Then, (1.2x – 18) – 0.8x = 0.25 × 0.8x
400 − 320 0.4x – 18 = 0.20x
Per cent profit = × 100 = 25%
320 18
⇒ x= = `90
0.20
70
46. (c) S.P. = MP
700 53. (a) Let the value of consignment be x.
S.P. of Tarun = `8750 2 1
125 70 Then, x × 1.05 + x × 0.98 = x + 400
Labelled price = × MP = 8750 3 3
100 100
1
⇒ MP = `10000 ⇒ x (3.08) = x + 400
3
47. (a) Let the cost of the product be `x. 0.08
⇒ x = 400
Then, 3
110 115 125 ⇒ x = `15000
x × × × = 1265
100 100 100
54. (a) Cost price of transistor = `320
100 100 100
⇒ x = 1265 × × × Selling price of transistor
110 115 125
= `800 = 320 × 1.15 = `368
Marked price of transistor
48. (b) Let the cost price of the machine be `x. = 368 + 32 = `400
Then,
110 90 \ Required percentage of profit
x× − 40 = x ×
100 100 400 − 320
= × 100 = 25%
100 320
⇒ x = 40 × = `200
20
55. (b) Let the selling price of a notebook be `x.
49. (a) Let cost price and selling price be x and y, respectively.
Then, cost price of 12 notebooks
34 26
\ x = y = 12x – 2x = 10x
100 100
2x
17 \ Profit per cent = × 100 = 20%
⇒ y = x 10 x
13
y−x
\ Profit per cent = × 100 56. (b) Profit per cent or loss per cent
x
17 25 × 12
x−x = + 25 – 12 – = + 10%
100
 = 13 × 100 ≈ 30.77%
x As the sign is + ve. So, there is a profit of 10%

Chapter 09.indd 25 6/4/2015 7:17:57 PM


9.26 Chapter 9

57. (b) Total cost price = 5200 + 800 = `6000 \ Quantity of rice sold at 10% gain
Selling price = `5500 12
= × 50 =40 kg
6000 − 5500 1 12 + 3
\ Loss per cent = × 100 =
8 %
6000 3 Quantity of rice sold at 50% loss
58. (d) Let the quantity of milk in each vessel be x L. 3
= × 50 =10 kg
12 + 3
\ Quantity of milk in 10 vessels will be 10x L.
Selling price of 10x L = 10x × 5 = `50x 62. (d) Required number of items
\ C.P. = `(50 x + 200) (3000 + 1000)
=
New selling price of 10x L (60 − 40)
=10x × 6 = `60x 4000
\ C.P. = `(60x – 150) = = 200
20
Now, (50x + 200) = (60x – 150)
63. (b) (17 + 19) = 36% of the cost price = `162
⇒ 10x = 350
162
\ x = 35 L \ 100% of the cost price = × 100 = `450
36
59. (a) Let the cost price of 12 apples be `100. 64. (a) Women’s shirts comprise 60% of the output.
Then, selling price of 100 apples will be `800. \ Men’s shirts comprise 40% of the output.
100 25 \ Average profit from men’s shirts = 8% of 40
\ C.P. of 1 apple = =` = 3.2 out of 40
12 3
Overall average profit = 6 out of 100
800
and S.P. of 1 apple = = `8 Average profit from women’s shirts = 2.8 out of 60,
100
i.e., 0.0466 out of each shirt.
 25  1
Loss =   8 = ` 65. (c) Suppose C.P. = `100
 3  3
1 Profit = 20%
Loss 100 + 20
Loss per cent = × 100 = 3 × 100 \ Listed price = ` 100 ×
CP 25 100
1 3 3 120
= × × 100 = ` 100 × = `120
3 25 100
= 4 % Discount = 10%
100 − 10
\ Discounted price = 120 ×
60. (d) Let the original value of fridge be `x. 100
15 90
Then, cost price = x = `120 × = `108
16 100

110 \ Gain = `108 – `100
Selling price = ×x
100 = `8 on `100 = 8%
110 15 \ Gain = `108 – `100 – 8 on `100
x− x
\ Gain per cent = 100 16 × 100 = 8%
15
x
16 66. (c) S.P. = `1540, Gain = 12%
= 17.33% SP × 100
C.P. =
100 + Gain%
61. (d) By the rule of allegation,
1540 × 100
= = `1375
112
S.P. = `1540, loss = 12%
SP × 100 1540 × 100
C.P. = =
100 − Loss% 88
= `1750

Chapter 09.indd 26 6/4/2015 7:17:58 PM


Profit and Loss 9.27

\ S.P. of both the radios = `3080 71. (d) Suppose C.P. of Horse = `x
C.P. of both the radios = `3125 \ C.P. of Carriage = `(8000 – x)
\ Net loss = `45 \ 10% of x – 10% of (8000 – x)
1 = 2.5% of 8000
67. (a) S.P. of each marble = Re ⇒ 20% of x = 1000
12
Loss = 20% ⇒ x = 5000
1 100 5
C.P. = × = 72. (b) Let C.P. = `x
12 80 48
11x
If gain = 20%, then S.P. per marble \ S.P. = x + 10% of x =
10
5 5 1 11x
= + 20% of = If S.P. = – 80, then loss would have been 10%
48 48 8 10
68. (b) Suppose C.P. = `100 11x
\ – 80 = x – 10% of x
Profit = `320 10
\ S.P. = `420 ⇒ 11x – 800 = 9x
If C.P. becomes `125, S.P. remaining the same, then ⇒ x = 400
profit = `295
295 73. (c) Let C.P. = `x
\ Profit % on S.P. = × 100 ≈ 70
420 Marked Price = `y
133
69. (a) Let the cost price of the article be `x. \ S.P. = x + 33% of x = x
100
At 19% loss, selling price
Now, according to the question,
 100 − 19 
= ` x   y – 5% of y =
133
x
 100 
100
81x x y
= ` ⇒ = = K, say
100 95 133
Now, according to the question,
⇒ x = 95K, y = 133K
 81   117 
  x + 162 = x   \ x + p% of x = y, say
 100   100  ⇒ 95K + p% of 95K = 133K
117 81 p
or, x− x = 162 ⇒ 95 + × 95 = 133
100 100 100
36 ⇒ 95p = 13300 – 9500 = 3800
or, x = 162
100 ⇒ p = 40
162 × 100
\ x = = `450 74. (a) Let the C.P. be `100, then
36
S.P. = `110
70. (d) Suppose retail price = `100 New S.P. = `220
Commission = 36% = `36
\ Profit % = 220 – 100 = 120
\ S.P. = `64
Profit = 8.8%
75. (a) Let the C.P. be `x
Let, C.P. = `K
112 56 28
\ K + 8.8% of K = 64 \ S.P. = x + 12% of x = = x = x x
100 50 25
64 × 100
⇒ K = = 58.8 28 28
108.8 \ x + 10% of x = 616
I f commission = 12% = `12, 25 25
S.P. becomes `88 208
⇒  x = 616
29.2 250
\ Profit % = × 100 = 49.6
58.8 ⇒  x = 500

Chapter 09.indd 27 6/4/2015 7:17:59 PM


9.28 Chapter 9

76. (d) .P. of 1 kg of rice = `5.40


S C.P. of cow, sold at gain of 20%

Loss = 10% 100
100 = `12000 ×
\ C.P. of 1 kg of rice = `5.40 × 100 + 20
100 − 10
100 100
= `5.40 × = `6 = `12000 × = `10000
90 120
In order to have a profit of 20% Total C.P. of horse and cow
S.P. of 1 kg of rice should be = `15000 + `10000 = `25000
100 × 20
= `6 × Hence, total loss = `25000 – `24000
100
120 = `1000
= `6 × = `7.20
100 Quicker Method: When each of the two commodities
is sold at the same price, and a profit of x% is made on
77. (c) C.P. of 11 books = `10 one and a loss of x% is made on the other, then there
S.P. of 10 books = `11 is always loss and the percentage value is given as
11
\ S.P. of 11 books = ` × 11 = `12.10 ( % value )2
10
\ Profit = `12.10 – `11 = `2.10 100
2.10 (20) 2
Hence, profit % = × 100 = 21% Here required x% loss = =4%
10 100
Quicker Method:
 100 
Cost price = 12000 × 2 ×  
 100 − 4 
100
= 24000 × = `25000
96
\ loss = `25000 – `24000 = `1000
11 × 11 − 10 × 10
% profit = × 100 = 21%
10 × 10
80. (b) Let the C.P. be `k
11k
78. (c) A’s Cost price of house = `10,000 \ S.P. = k + 10% of k =
10
100 + 10 Now, according to the question,
\ A’s Selling Price = `10,000 ×
100 11k
110 k – 10 % of k = − 40
= `10,000 ×
= `11,000 10
100 9k 11k
or, = − 40
or B’s Cost Price = `11,000 10 10
100 − 10
B’s Selling Price = `11,000 × 2k
100 or, = 40
10
90
= `11,000 × = `9,900 \ k = 200
100
= `9,900 81. (d) Let the amount after the loss = `x
or A’s second Cost Price = `9,900
So, x – 80% of x = 4500
Hence A’s profit in this transaction
20% of x = 4500
= `(11,000 – 9,900) = `1,100
4500  100
x = = `22500
79. (b) Total S.P. of horse and cow = `12000 × 2 20
= `24,000 This amount is after the 20% loss.
C.P. of horse, sold at loss of 20% So, initial amount will be
100 = 80% of initial amount
= `12000 ×
100 − 20 = 22500
100 22500  100
= `12000 × = `15,000 Initial amount = = `28125
80 80

Chapter 09.indd 28 6/4/2015 7:17:59 PM


Profit and Loss 9.29

82. (a) Given, CP = `320 89. (b) Let SP of the object be `100.
SP = 115% of 320 = `368 ∴ CP of the object
Then, MP = 368 + 32 = `400 26
If transistor is sold at MP, then = 100 − × 100 = ` 74
100
400 − 320 80 We are given,
Profit per cent = × 100% = × 100% = 25%
320 320 34% of CP of the object
83. (a) Let both the packets have x number of toffees. 34 × 74
= = ` 25.16
13 x 28 100
then, CP of 2x toffees = 5 x + = x = 25.16% of SP
3 3
70 35
And SP of 2x toffees = × 2x = x 90. (a) Let the cost of computer system be `x and the cost of
12 3 colour printer be `y.
 35 28  Then, it is given that,
 − x
∴ Gain percentage = 
3 3  90 120
× 100 x+ y= x+ y
28
x 100 100
3 ⇒ 10 x − 2 y = 0 (1)
7
= × 100 = 25% 105 85
and ( x + y ) − 800 = x+ y
28
100 100
84. (c) CP for the manufacturer −5 15
⇒ + y = 800
100 100 100 100 100
= 30.09 × × × = `17
118 120 125 ⇒ − x + 3 y = 16000 (2)

85. (a) Let the manufacturer should produce ‘x’ number of On solving Eqs. (1) and (2),.we get
articles per week.
=
x 32000
= , y 16000
Then for the profit of atleast `1000 per week,
SP − CP = 1000 91. (d) Let the number of units produced per week be x.
⇒ 60 x − (40 x + 3000) = 1000 Selling price of all units = `60x
⇒ 20 x = 4000 and CP of all units = ` 40x + 3000
⇒ x = 200 We are given, 40x + 3000 + 1000 = 60x
⇒ 20 x = 4000
86. (a) Let both the packets have x number of toffees.
∴ x = 200
13 x 28
then, CP of 2x toffees = 5 x + = x
3 3 Hence, he should produce at least 200 units to make a
70 35 profit of `2000 per week.
And SP of 2x toffees = × 2x = x
12 3 92. (a) Let Aditi pay for the article = `x
 35 28  She spent `110 on his repair, so its total cost =
 − x
 3 3  `(x + 110)
∴ Gain percentage = × 100
28
x Now, she sold it to Samir at a profit of 20%
3 20
7 So, sale price = ( x + 110) + ( x + 110) ×
= × 100 = 25% 100
28 ( x + 110)
= ( x + 110) +
87. (c) CP for the manufacturer 5
100 100 100 5 x + 550 + x + 110  6 x + 660 
= 30.09 × × × = `17 = = ` 
118 120 125 5  5 
88. (a) Let the manufacturer should produce ‘x’ number of Now, Samir sold it to Vikas at a loss of 10%
articles per week. 6 x × 660  6 x + 660 10
So, sale price =  − ×
Then for the profit of atleast `1000 per week,  5  5 100
SP − CP = 1000 6 x + 660 6 x + 660
= −
⇒ 60 x − (40 x + 3000) = 1000 5 50
⇒ 20 x = 4000 60 x + 6600 − 6 x − 660 54 x + 5940
= =`
⇒ x = 200 50 50

Chapter 09.indd 29 6/4/2015 7:18:02 PM


9.30 Chapter 9

Now, finally vikas sold it for `1188, at a profit of 94. (a)


10% C.P. of 16 kg of the mixture = ` (12 × 16 + 4 × 2) = ` 200
So, sale price S.P. of 16 kg of the mixture = 16 × 16 = ` 256
54 x + 5940 54 x + 5940 10 40
= + × ∴ Actual gain = × 56 = `140
50 50 100 16
54 x + 5940 54 x + 5940 95. (a) Money paid to buy watch = `1950.
⇒ 1188 = +
50 500 Money paid as interest of 10% on `1950 = `195
540 x + 59400 + 54 x + 5940
⇒ 1188 = \  Total money paid = `2145
500
Since S.P = `2200, therefore the man gained `55.
594 x + 65340
⇒ = 1188 96. (a) Let man buys apples at `x per dozen.
500
4(148.5 x + 16335) x
⇒ = 1188 \  Cost price =
500 12
⇒ 148.5 x + 16335 = 148500 8x
and selling price =
⇒ 148.5 x = 132165 100
132165 Here, S.P < C.P., therefore there is loss
⇒ x=
148.5 x 8 x 25 x − 24 x x
whichisequaltoCP–SP = − = =
∴ x = `890 12 100 300 300
Loss x 12
So, Aditi bought the article for `890. \ Loss per cent = × 100 = × 100 × = 4%
C.P 300 x
93. (d) Let the cost price of the articles be `100 97. (b) Let x be the selling price of 1 notebook.
Marked price = `130 \  Selling price of 2 note books = 2x = profit
After giving a discount of 10% the selling price of the As, we know, profit = selling price − cost price
articles = 0.9 × 130 = 117 \  Cost price of 12 note books = 12x − 2x = 10x
(117 − 100) 2x
So, actual profit per cent = × 100 = 17% \  Profit (percentage) = × 100 = 20%
100 10 x

Difficulty Level-2

1. (b) Let the C.P. be `100. 38 x


  (100  x)   = 55 × 100
Then, 2 160    100  10 x
  100   11
  (100  x)   (100  ( x  5))  
= 160     100  38 1
  100  100   = × × 100 = 76.
5 10
Now, solve for x, we get x = 25
2. (c) S.P. at the stall at the trade fair 3. (d) Suppose C.P. = `100
8x \ Gain on `25 = `2.50
= 2x – 20% of 2x =
5
⇒ S.P. = `27.50
x × 100 10 x
C.P. of the jacket = = Loss on `75 = `15
100 + 10 11
⇒ S.P. = `60
8 x 10 x 38 x
\ Profit = − =
5 11 55 \ Total S.P. = `87.50
\ Profit % made at the trade fair ⇒ Loss = `12.50.

Chapter 09.indd 30 6/4/2015 7:18:04 PM


Profit and Loss 9.31

4. (d) Let the C.P. be (d ) x 10. (c) Workman’s wages = 1.8 × 40 = `72
Then, initial profit percentage Weight of material taken before working
  15 x   4    5
  8   5   x  = 8 ×   = 10 kg
=   × 100 (1)  4
x
Cost of 10 kg of material = `225
In the second case, profit percentage
Therefore, C.P. of the material = 225 + 72 = `297
  15 x    3   5x  
    500        Hence, S.P. = 1.33 × 297 = `396
8 4 4 
=    (2)
 5x  11. (c) Marked Price = `x, say
 
4
9x
Equate (1) and (2) to find the value of x as 800 S.P. = x – 10% of x =
10
And the new selling price
9x
 5 \ = 6750
=   × (1500 + 500) 10
 4
⇒ x = 7500
= `1,500
If S.P. = `7500, then the C.P. would have been `5000,
5. (d) Suppose the manufacturer should produce x items
due to 50% profit earned by the shopkeeper .
\ 60x – [40x + 3000] = 1000
\ Actual percentage of profit by selling the article
⇒ 20x = 4000 for `6750
⇒ x = 200.
1750
= × 100 = 35%
6. (b) Suppose C.P. = `x 5000
\ 425 – x = x – 355
12. (d) By finding the square root of 2401, you can determine
⇒ 2x = 780 that the plot of land measures 49 ft × 49 ft.
⇒ x = 390.
With shrubs planted along the edges and at the corners
7. (c) The tradesman pays for 100 kg and 108 kg of goods of the plot, with 7 ft between each shrub, there is room
by means of false balance. for 8 rows, each with 8 shrubs, for a total of 64 shrubs.
Actually, he sells 108 kg of goods, but due to false So Ravi’s total selling price is 64 × `35 = `2,240
balance, he sells 116.64 kg of goods.
His profit equals the total selling price less the total
cost be produce the shrubs. So Ravi’s profit will be
8. (a) Profit per cycle = `190
`2240 – `896 = `1,344.
Total profit = 20 × 190 + 140 + 100 + 50 To write `1,344 (his profit) as a percentage of `896
= 3800 + 290 = `4090. (his total cost); you can write it first as a fraction or as
a decimal, and then multiply by 100.
9. (d) If S.P. of each chair is `Z = 200 + 4K, then number of 1344
chairs purchased = 1.5 or, 150%
896
2( Z − 200)
= 100 – 2K = 100 –
4 13. (c) C.P.1 = `3,75,000
and, C.P.2 = `2,50,000
Z
= 100 − + 100 ⇒ Σ C.P. – Σ S.P. = 625000 – 600000
2
= `25,000
Z
= 200 −
2
14. (a) Let number of articles = 100
\ Total money received from the sale of chairs
Let his C.P. = `x
2
 Z Z Then, he sold 80 articles under scheme B and 20 under
= Z  200 −  = 200 Z − .
 2 2 scheme A.

Chapter 09.indd 31 6/4/2015 7:18:04 PM


9.32 Chapter 9

80  30  20  20 19. (c) Cost price of one kg of apple in which the three


Hence, overall discount = = 28% varieties of apples are mixed in the ratio 2:3:5 is
100
equal to S where S = 0.2 × 20 + 0.3 × 40 + 0.5 × 50
Therefore, selling price = 1.5x × 0.72 = 1.08x, where = 4 + 12 + 25 = `41
x is his cost price.
Selling price per kg of apples to ensure there is a net
Hence, net profit is 8% proift of 20% = 1.2 × 41 = `49.2
15. (c) We have C.P. of mixture 20. (b) Let the cost price be `100
100 Then, market price is `140
=  S.P.
100  %profit 2
Now, the first discount is of 28.57% ≈ th of market
100 7
=  480 = 400 price.
120
5
Quantity of cheaper/Quantity of dearer Hence, its selling price = 140 × = `100
7
= (610 – 400)/(400 – 285) = 42:23
Now since you are selling at cost price, any further
If cheaper wheat is 42 quintals, dearer one = 23 quintals discount will be equal to loss percentage.
⇒ If cheaper wheat is 126 quintals, dearer one
120  100
= 23/42 × 126 = 69 quintals 21. (b) Percentage profit = × 100 = 20%
100
16. (c) After the discount of `25% each, the cost of watches
120  100
is `270. Percentage loss = × 100
120
He has to buy them again, hence total cost of the
20
watches is 270 + 270 = `540         = × 100 = 16.7%
120
17. (b) Let the cost price of 1 apple be ‘x’, therefore the cost 22. (b) C.P. : S.P.
price of 1 orange and 1 mango would be ‘x’ and ‘2x’
respectively. 3 : 4
Profit on 3 apples = `1 (consider C.P. = `1)
 16 x 
Selling price of 1 apple =   1.6 x Profit = 33.33%
 10 
and discount = 11.11%
 12 x 
Selling price of 1 orange =   0.75 x Since, C.P. S.P. M.P.
 16 
3 4 4.5
 12 x  (1) (0.5)
Selling price of 1 mango =   3x
 4  Profit is double that of discount
Total cost price of 1 apple, 2 oranges and 2 mangoes So, the percentage point difference
= 33.33% – 11.11% = 22.22% point
= x + 2x + 4x = 7x.
Total selling price of 1 apple, 2 oranges and 2 mangoes 23. (b) Profit would be maximum if books are bought for
`200 and sold for `425.
= 1.6x + 1.5x + 6x = 9.1x
Profit = `(425 – 200) = `225
Net Profit = 9.1x – 7x = 2.1x
Profit of 8 books = `225 × 8 = `1,800
 2.1x 
Net profit per cent =  100  30%
 7 x  24. (b) If C.P. = 100, M.P. = 130
1 1 1
200 S.P. =  110.5   130   91
18. (c) 135% of C.P. = `90 C.P. = 4 2 4
3
S.P. = 27.625 + 65 + 22.75
Profit on selling it for `70
= 115.375
 200  10
= `  70  ` Hence, profit = 15.375%
 3  3
10 25. (c) Let the C.P. of a bicycle = `100
3
Profit% = Now, since profit is 140%
200  100  5%
3 ∴ S.P. = `240

Chapter 09.indd 32 6/4/2015 7:18:05 PM


Profit and Loss 9.33

Now, 7 bicycles are being sold instead if 1 bicycle, but 31. (b) Let the S.P. of Vineet and Roshan be x.
the sale price of new bicycle = `120 300
15
Therefore total sale price of new sale of bicycles 15% profit on C.P. = × 100 = % profit on S.P.
115 23
= 7 × 120 = `840 and the C.P. = 7 × 100 = 700
x × 25 300 x
So the new profit = 840 – 700 = `140 \ − = 275
100 23 × 100
Since the initial profit is same as the new so there is 0
increase in percentage. On solving x = 2300

26. (d) Marked price of the article = `1,600 32. (b) Cost price of 30 kg of 1st rice = 30 × 17.5
∴ Selling price = (100 – 10)% of (100 – r%) 30 kg of 2nd rise = 30 × x
  of 1600 Total cost price = 30 × 17.5 + 30x
90 100  r
   1600 30 × 17.5 + 30 x 120
100 100 \ × = 18.6
60 100
9
⇒ 1224 =  (100  r )  16 On solving, we get x = `13.5
10
1224  10 33. (a) Let cost of the watch be `x.
⇒ = (100 – r) \ 10% of x + 15% of (390 – x) = 51.5
9  16
On solving, we get,
25 120  k (Profit) x = 140
27. (c) Profit % =  = ⇒ k = 100
100 880 (Sale) Hence, cost of clock = 390 – 140
100 = `250
Therefore, net profit % =  100 = 10%
1000 \ Difference = 250 – 140

28. (b) If selling price of 2 article is same and sold at x% = `110


more and x% less, then there will be always loss
25935 × 100
x 2 202 34. (d) Market price = = 28500 = S.P.
= = = 4% 91
100 100
25935 × 100
\ Gain, 96% = 60000 Cost price = = 25000
103.74
Loss, 4% = `2500
28500 − 25000
29. (c) Let C.P. cow be x and C.P. of ox be y. \ Profit percentage = × 100
25000
120 125  = 14%
\ x× + y× = 800
100 100
35. (b)
125 120
x× + y× = 820
100 100
On solving, we get x =`530.6
and y = `131.14

30. (d) 1st horse 2nd horse 96  100


\ C.P. = = 80
C.P. = 100 120

S.P. = 100 \ Ratio = 3:2


S.P. = 80 36. (c) Let the weight substituted for 1 kg = x g
100 × 100 100  x
C.P. = = 80 So,  100 = 30
125 x
Hence, total C.P. = Total S.P. 3
Solving x = 769 g
Hence, neither gain nor loss. 13

Chapter 09.indd 33 6/4/2015 7:18:05 PM


9.34 I Chapter 9

x Cost of second variety


37. (b) 25% = × 100%
1000  x = 25 × 25 = 625
Total cost price = 625 + 42x
⇒ x = 200
S.P. of both variety = (25 + x) × 40
Hence, he uses weight, 1000 – 200 = 800 g
= 1000 + 40x
38. (d) Let total sales be `100. Now, find the respective sales
Profit = 1000 + 40x – 625 – 42x
of Music CDs and Books and Rest will be DVDs.
Since, profit is given, cost price of CDs and Books can = 375 – 2x
be calculated. Required profit = 25%
Further from overall profit, overall cost price can be
375  2 x
calculated.  100 = 25
625  42 x
39. (b) Apply same concept as in the previous question.
⇒ 1500 – 8x = 625 + 42x
40. (d) Let C.P. of radio be x.
⇒ 875 = 50x
 1 1 
otal difference = x 107 %  97 %
T
 2 2  ⇒ x = 17.5 kg

 = x × 10% 45. (b) Original profit = 70000 – 42000 – 12000


x  10  = 16000
\ = 100
100 If 7.14% of 14 i.e., one of the machines remain closed
\ x = 1000 throughout the year, then change in profit will be
12.5% gain on `1000 = `1125 13
= (70000  42000)
41. (d) Let cost of A = x and B = y 14
4x i.e., 26000 – 12000 = 14000
S.P. of A =
5 Thus, the decrease in the profit %

13 y 2000
and B = =  100 
12.5%
10 16000

4 x 13 y 46. (b) Let the price of the product be 100. Then, the prices of
Now, x + y =  (because company does not
5 10 the components A and B will be 10 and 20 respectively.
lose anything) As the profit is 20%, the selling price = 120.
x 3y Due to increase in the price of raw material, the
⇒ = new costs of components A and B will be 12 and 28
5 10
respectively.
⇒ 2x = 3y
The new selling price = 115% of 120 = 138
Now, use this relation further.
As, there is no change in the price of the other
42. (d) Cost price = 24 × (100 – 12.5)% = `21 components, new cost of the product = 110
 1 28
Selling price = 21 × 100  33  = `28 Thus, the new profit % =  100 = 25.45%
 3 110
28
Marked price = × 100 = `35 47. (b) Let the cost of cotton trouser be x and woollen trouser
(100  20)
be y.
43. (c) Let the price of one sheep = `1 13 x
Selling price of cotton trouser be and woolen
So, cost price of 749 sheep = `749 10
Selling price of 700 sheep = `749 3y
trouser be .
Selling price of 1 sheep = `1.07 2
1.07  1 Earlier salesman sells 100 cotton and 200 woollen
\ Per cent gain =  100 
7%
1 trousers.
44. (d) Let the amount of first variety be x kg. \ C.P. = 100x + 200y
Cost of first variety = 42x and, S.P. = 130x + 300y

Chapter 09.indd 34 6/4/2015 7:18:06 PM


Profit and Loss I 9.35

Since, salesman gains profit of 45% 52. (d) Cost price of (3 + 4 + 5) = 12 kg of fruits = `(300 +
45 320 + 300) = `920.
\ S.P. = (100x + 200y) + (100x + 200y) ×
100 SP at a profit of 50% = `1380.
= 145x + 290 y ∴  SP of fruits per kg = 1380 = `115.
Now, 130x + 300y = 145x + 290y 12
⇒ 15x = 10y 53. (b) Production cost of 1500 watches

2y = (1500 × 150 + 30, 000) = ` 255, 000.


⇒ x =
3 Amount realized on the sale of 1500 watches
Use this relation for further calculation. = (1200 × 250 + 300 × 100) = ` 330, 000

48. (b) The marked price of bicycle is `5000. ∴  Profit earned
Let the cost price be p. = (330,000 − 255,000) = `75,000
Successive discounts are 10% and 20% 54. (b) Production cost of 1500 watches =`255,000
∴ 5000 × 0.9 × 0.8 − 0.2 p = p Let he sells x watches during the season, therefore
number of watches sold after the season = (1500 − x)
5000 × 0.72 watches.
⇒ p=
1.2 Amount realized on the sale of 1500 watches
⇒ p = ` 3000
= 250 × x + 100(1500 − x) = 150 x + 150, 000
New, break-even is achieved if production cost is
200
49. (a) Since the production increases by % when equal to the selling price.
3
production is increased from 20 to 40 units. ∴ 150 x + 150, 000 = 255, 000 ⇒ x = 700.

55. (a) Let the price of 100 cm of cloth be `100, but he gets
(240 + 40b + 402 c) − (240 + 20b + 202 c) 200 2 120 cm of cloth for `100. Hence, his actual cost for
∴ = = (1)
240 + 20b + 202 c 300 3 100 5
1=
cm = ` .
(240 + 60b + 602 c) − (240 + 40b + 402 c) 1
and = (2) 120 6
240 + 40b + 402 c 2
New, instead of selling 100 cm, by cheating he sells
Solving the equations (1) and (2), we get 80 cm of cloth for the cost price of 100 cm of cloth. To
calculate his profit, the cost price of 80 cm of cloth
1
= c = and b 10. 5
= × 80 = ` 66.66.
10
6
x2 Selling price of 80 cm of cloth (actually 100 cm for
So, cost of producing x units = 240 + 10 x + .
10 the buys) at a discount of 20%
Profit earned in producing x units = 100 × 0.8 = `80.

 x2  x2 80 − 66.66
= 30 x −  240 + 10 x +  = 20 x − − 240. ∴  Profit percentage = × 100 = 20.01%
 10  10 66.66
or 20% (approximately).
Clearly from the options, we find that profit is
maximum at x = 100. 56. (b) Let the CP of 1 kg of sugar be `100.
50. (c) For x = 100, profit Then, CP of 900 g of sugar =
100
× 900 = ` 90
100 × 100 1000
= 20 × 100 − − 240 = 760. Hence, profit percent in Case II
10
100 − 90
51. (a) Let the CP be `100, then SP = `114.28 as profit is = × 100 = 11.11%
14.28% 1000
This SP is arrived after giving a discount of 11.11% on If he adds 10% impurity then his CP for 1 kg
marked price, hence if marked price = x. 100
= × 1000 = ` 90.90
Then 1000
x × 0.8889 = 114.28 ⇒ x = `128.56 100 − 90 ⋅ 90
  Hence, profit percent in Case III = × 100 = 10.01%
which is 28.56% more than the CP. 90 ⋅ 90

Chapter 09.indd 35 6/4/2015 7:18:09 PM


9.36 Chapter 9

If reduces weight by 5% 59. (d) Let the cost price of watch = `x


Then, cost price of 950 g Now, loss = 10%

=
100
× 950 = ` 95 and SP = `105 x × 10 9x
∴  Sale price = x − =`
1000 100 10
Hence, profit percent in Case IV We are given,
105 − 95 9x x×5
= × 100 = 10.52%. + 27 = x +
95 10 100
Thus, method II maximizes his profit. 9x x
⇒ + 27 = x +
57. (a) Let the CP of the article be `x, since he earns a profit 10 20
of 20%, hence SP = 1.2x. x 9x
It is given that he incurs loss by selling 16 articles at ⇒ x+ − = 27
20 10
the cost of 12 articles 20 x + x − 18 x
16 − 12 ⇒ = 27
  20
 loss = × 100 = 25% 
 16  3x 20 × 27
⇒ = 27 ⇒ x =
∴  His selling price = SP × 0.75. 20 3
∴ x = `180
1.2
Now, SP × 0.75 = 1.2 x ⇒ SP = x = 1.6 x
0.75
∴ CP = `180, Loss = 10% of `180 = `18
This SP is arrived after giving a discount of 20% on and profit = 5% of `180 = `9
MP.
1.6 x 18
MP = 2 x. It means that article has been
Hence, = ∴  Required percentage = × 100% = 200%
0.8 9
marked 100% above the cost price. 60. (a) Let the quantity bought be 12x, 15x, and 20x kg
58. (c) Let the price paid per trip by a passenger be `x. respectively.
and auto rickshaw consumes y L petrol Total cost price = 100 × 12x + 80 × 15x + 60 × 20x
Total expenditure in a trip = 30 × y = `30y = 1200x + 1200x + 1200x = 3600x
Total earning in a trip = 30 × x = `3x Selling price of first two quantities at profit of 20%
We are given, 120
= × 2400 x = 2880 x
3 x − 30 y = 20% of 30 y 100
20 × 30
⇒ 3 x − 30 y = y Total selling price = 3600x (No profit no loss)
100
∴  Selling price of third quantity = 3600x − 2880x =
⇒ 3 x − 30 y = 6 y
720x
⇒ 3 x = 36 y
1200 x − 720 x
∴ x = 12, ∴  Loss percentage = × 100 = 40%
1200 x
Now, the price of petrol is reduced to `24 and
passengers taken by auto-rickshaw are four. 61. (c) Dividend for the preferred share
∴  Total expenditure in a trip = 24 × y = `24y 20
Total earning in a trip = 4 × x = `4x = 50000 × 10 × = 1, 00, 000
100
Total earning − Total expenditure
Profit per cent = × 100% Rest of the dividend = 180000 − 100000 – 30000 =
Total exp enses
`50000
4 x − 24 y
= × 100%
24 y 50000 × 100
% of dividend for general shares = = 25%
4 × 12 y − 24 y 20000 × 10
= × 100 [ x = 12 y ]
24 y 62. (d) Let x be no. of units.
48 y − 24 y ∴  Profit per unit x = (60 − 40)x = 20x.
= × 100
24 y Now, additional cost = 3000
24 y ∴  To make a profits of at least 1000, we have
= × 100 = 100%
24 y 20x − 3000 = 1000  ⇒ 20x = 4000 ⇒ x = 200

Chapter 09.indd 36 6/4/2015 7:18:11 PM


CHAPTER

Time, Work and Wages 10


INTRODUCTION
n
In our daily life, we come across situations where we need complete the full work, ‘A’ will take hrs.
m
to complete a particular job in a reasonable time. We have
to complete the project earlier or later depending upon the 4. If ‘A’ does three times faster work than ‘B’, then
needs. Accordingly, the men on duty have to be increased or ratio of work done by A and B is 3:1 and ratio of
decreased, i.e., the time allowed and the men engaged for a time taken by A and B is 1:3.
project are inversely proportional to each other, i.e., the more
5. A, B and C can do a piece of work in T1, T2 and T3
the number of men involved, the lesser is the time required
days, respectively. If they have worked for D1, D2
to finish a job. We also come across situations where time
and D3 days, respectively, then
and work or men and work are in direct proportion to each
other. D1
Amount of work done by A =
For solving problems on time and work, we follow the T1
following general rules:
D2
Amount of work done by B =
1. If ‘A’ can do a piece of work in n days, then at a T2
1
uniform rate of working ‘A’ will finish th work D3
n and, Amount of work done by C =
in one day. T3
1 Also, the amount of work done by A, B and C
2. If of a work is done by ‘A’ in one day, then ‘A’
n together
will take n days to complete the full work.
D1 D2 D3
1 = + +
3. If ‘A’ does th of a work in one hour then to T1 T2 T3
n
which will be equal to 1, if the work is complete.

soMe useful sHort-cut MetHoDs

1. If A can do a piece of work in X days and B can do 1


B’s 1 day’s work =
the same work in Y days, then both of them working Y
XY 1 1 X +Y
together will do the same work in days. Then, (A + B)’s 1 day’s work = + =
X +Y X Y XY
\ A and B together can complete the work in
Explanation
XY
1 = days
A’s 1 day’s work = X +Y
X

Chapter 10.indd 1 6/4/2015 11:06:21 AM


10.2 Chapter 10

Illustration 1 A can finish a piece of work by working alone Explanation


in 6 days and B, while working alone, can finish the same A and B together can complete the work in X days.
work in 12 days. If both of them work together, then in how
1
many days, the work will be finished? \ (A + B)’s 1 day’s work =
X
Solution: Here X = 6 and Y = 12
1
\ Working together, A and B will complete the work in Similarly, A’s 1 day’s work =
Y
XY 6 × 12
= days = days, i.e., 4 days. 1 1 Y−X
X +Y 6 + 12 Therefore, B’s 1 day’s work = – = .
X Y XY
2. If A, B and C, while working alone, can
complete a work in X, Y and Z days, respectively,  XY 
\ B alone can complete the work in   days
then they will together complete the work in Y − X 
XYZ \ B alone will complete the work in
days.
XY + YZ + ZX XY 15 × 20
= days = , i.e., 60 days
Y−X 20 − 15
Explanation
1 Illustration 3 A and B working together take 15 days to
A’s 1 day’s work =
X complete a piece of work. If A alone can do this work in 20
1 days, how long would B take to complete the same work?
B’s 1 day’s work = Solution: Here X = 15 and Y = 20.
Y
1
C’s 1 day’s work = 4. If A and B, working together, can finish a piece
Z of work in X days, B and C in Y days, C and A in
\ (A + B + C)’s 1 day’s work Z days, then
1 1 1 XY + YZ + ZX (a) A, B and C working together, will finish the
= + + =
X Y Z XYZ job in
So, A, B and C together can complete the work in  2XYZ 
 XYZ    days.
 XY + YZ + ZX 
=   days
 XY + YZ + ZX  (b) A alone will finish the job in
Illustration 2 A, B and C can complete a piece of work in  2XYZ 
10, 15 and 18 days, respectively. In how many days would   days.
 XY + YZ − ZX 
all of them complete the same work working together?
(c) B alone will finish the job in
Solution: Here X = 10, Y = 15 and Z = 18.
 2XYZ 
Therefore, the work will be completed in   days.
XYZ  YZ + ZX − XY 
= days
XY + YZ + ZX
Explanation
10 × 15 × 18
= days 1
10 × 15 + 15 × 18 + 18 × 10 (A + B)’s 1 day’s work =
X
2700 1
i.e., or, 4 days 1
600 2 (B + C)’s 1 day’s work =
Y
3. Two persons A and B, working together, can 1
complete a piece of work in X days. If A, working (C + A)’s 1 day’s work =
Z
alone, can complete the work in Y days, then B,
XY So, [(A + B) + (B + C) + (C + A)]’s 1 day’s work =
working alone, will complete the work in
Y−X 1 1 1
+ +
days. X Y Z

Chapter 10.indd 2 6/4/2015 11:06:22 AM


Time, Work and Wages 10. 3

 1 1 1 7200
or 2 (A + B + C)’s 1 day’s work =  + +  or , i.e., 20 days.
X Y Z 360

1 1 1 1 C alone can do the work in


or (A + B + C)’s 1 day’s work =  + +  2XYZ
2X Y Z = days
ZX + XY − YZ
 XY + YZ + ZX 
i.e.,   2 × 12 × 15 × 20
 2 XYZ  = days
20 × 12 + 12 × 15 − 15 × 20
\ A, B and C working together will complete the work
7200
 2XYZ  or , i.e., 60 days.
in   days. 120
 XY + YZ + ZX 
Also, A’s 1 day’s work = (A + B + C)’s
)’s 1 day’s work 5. (a) If A can finish a work in X days and B is k
– (B + C)’s 1 day’s work times efficient than A, then the time taken by
1 1 1 1 1 both A and B working together to complete
=  + + − x
2X Y Z Y the work is .
1+ k
1 1 1 1 (b) If A and B working together can finish a work
=  − + 
2X Y Z in X days and B is k times efficient than A,
XY + YZ − ZX then the time taken by
= (i) A, working alone, to complete the work is
2 XYZ
(k + 1) X.
 2XYZ  (ii) B, working alone, to complete the work
So, A alone can do the work in   days.
 XY + YZ − ZX   k + 1
is   X.
 2XYZ   k 
Similarly, B alone can do the work in  
 YZ + ZX − XY 
Illustration 5 Harbans Lal can do a piece of work in 24
 2XYZ  days. If Bansi Lal works twice as fast as Harbans Lal, how
days and C alone can do the work in   days.
 ZX + XY − YZ  long would they take to finish the work working together?
Illustration 4 A and B can do a piece of work in 12 days, Solution: Here X = 24 and k = 2
B and C in 15 days, C and A in 20 days. How long would \ Time taken by Harbans Lal and Bansi Lal, working
each take separately to do the same work? together to complete the work
Solution: Here X = 12, Y = 15 and Z = 20.  X 
=   days
\ A alone can do the work in 1 + k 
2XYZ
=
XY + YZ − ZX  24 
=   days, i.e., 8 days
2 × 12 × 15 × 20 1 + 2 
= days
12 × 15 + 15 × 20 − 20 × 12 Illustration 6 A and B together can do a piece of work in
7200 3 days. If A does thrice as much work as B in a given time,
or , i.e., 30 days. find how long A alone would take to do the work?
240
B alone can do the work in Solution: Here X = 3 and k = 3.
2XYZ \ Time taken by A, working alone, to complete the
= days work
YZ + ZX − XY
2 × 12 × 15 × 20  k + 1  3 + 1
= days =  X =   3 = 4 days
15 × 20 + 20 × 12 − 12 × 15  k   3 

Chapter 10.indd 3 6/4/2015 11:06:23 AM


10.4 Chapter 10

6. If A working alone takes a days more than A and a


B working alone takes b days more than A and B 8. If A can complete part of work in X days, then
b
together, then the number of days taken by A and
c b×c× X
B, working together, to finish a job is given by part of the work will be done in days.
d a×d
ab .

Illustration 9 A can do three-fourths of a work in 12 days.


Illustration 7 A alone would take 8 hrs more to complete
In how many days can he finish one-eighth of the work?
the job than if both A and B worked together. If B worked
1 Solution: Here a = 3, b = 4, X = 12, c = 1 and d = 8
alone, he took 4 hrs more to complete the job than Therefore, number of days required to finish one-eighth
2
of the work
A and B worked together. What time would they take if both
A and B worked together? b×c× X 4 × 1 × 12
= = = 2 days
9 a×d 3×8
Solution: Here a = 8 and b =
2
\ Time taken by A and B,, working together, to 9. (a) There are two groups of people with same
complete the job efficiency. In one M1 persons can do W1
= ab days works in D1 time and in the other M2 persons
can do W2 works in D2 time. The relationship
9 between the two groups is given by
= 8× or, 6 days
2 M1D1W2 = M2D2W1
(b) There are two groups of people with same
7. If A is k times more efficient than B and is therefore
efficiency. In one M1 persons can do W1
able to finish a work in l days less than B, then works in D1 time working t1 hrs a day and M2
(a) A and B, working together, can finish the persons can do W2 works in D2 time working
kl t2 hrs a day. The relationship between the two
work in 2 days. groups is given by
k −1
M1D1t1W2 = M2D2t2W1.
(b) A, working alone, can finish the work in
l
days.
k −1 Illustration 10 If 10 persons can complete two-fifths of a
(c) B, working alone, can finish the work in work in 8 days, then find the number of persons required to
kl complete the remaining work in 12 days
days.
k −1
2
Solution: We have M1 = 10, W2 = ,D =8
5 1
Illustration 8 A is thrice as good a workman as B and takes
10 days less to do a piece of work than B takes. Find the 3
time in which B alone can complete the work M2 = ?, W2 = , D = 12
5 2
Solution: Here k = 3 and l = 10.
\ M1D1W2 = M2D2W1
\ Time taken by B, working alone, to complete the work
kl 23 2
= days ⇒ 10 × 8 × = M2 × 12 ×
k −1 5 5

3 × 10 ⇒ M2 = 10
= days
3 −1
i.e., 15 days

Chapter 10.indd 4 6/4/2015 11:06:24 AM


Time, Work and Wages 10. 5

Illustration 11 If 10 persons can cut 20 trees in 3 days Illustration 12 12 men or 15 women can do a work in
working 12 hrs a day. Then, in how many days can 24 14 days. In how many days, 7 men and 5 women would
persons cut 32 trees working 4 hrs a day? complete the work?
Solution: We have, M1 = 10, W1 = 20, D1 = 3, t1 = 12 Solution: Here a = 12, b = 15, n = 14, c = 7 and d = 5.
M2 = 24, W2 = 32, D2 = ?, t2 = 4 Required number of days
\ M1D1t1W2 = M2D2t2W1 nab
=
⇒ 10 × 3 × 12 × 32 = 24 × D2 × 4 × 20 bc + ad
⇒ D2 = 6 days.
 14 × 12 × 15 
=   days
10. If a men and b women can do a piece of  15 × 7 + 12 × 5 
work in n days, then c men and d women can
168 3
 nab  = days or, 15 days.
do the work in   days 11 11
 bc + ad 

Practice Exercises

Difficulty level-1
(BaseD on MeMory)

1. Running at the same constant rate, 6 identical machines 3 1


can produce a total of 270 bottles per min. At this rate, (a) 10 (b) 11
4 4
how many bottles could 10 such machines produce in 4
mins? 1
(c) 12 (d) 13
(a) 648 (b) 1,800 2
[Based on MAT, 2003]
(c) 2,700 (d) 10,800
5. A and B require 10 days to complete a job. B and C require
[Based on MAT, 2004]
12 days to complete the same job. A and C require 15 days
2. Machine A produces bolts at a uniform rate of 120 every to complete the same job. The number of days required, if
40 s, and machine B produces bolts at a uniform rate of all are at work, to compete the job is:
100 every 20 s. If the two machines run simultaneously, (a) 8 days (b) 9 days
then how many seconds will it take them to produce a (c) 6 days (d) 7 days
total of 200 bolts?
(a) 22 (b) 25 6. Rohit, Harsha and Sanjeev are three typists who, working
simultaneously, can type 216 pages in four hrs. In one
(c) 28 (d) 32 hour, Sanjeev can type as many pages more than Harsha
[Based on MAT, 2004] as Harsha can type more than Rohit. During a period of
3. Two men undertake to do a piece of work for `200. One rive hrs, Sanjeev can type as many pages as Rohit can
alone can do it in 6 days and the other in 8 days. With the during seven hrs. How many pages does each of them
help of a boy, they finish it in 3 days. How much is the type per hour?
share of the boy? (a) 16, 18, 22 (b) 14, 17, 20
(a) `45 (b) `40 (c) 15, 17, 22 (d) 15, 18, 21
(c) `30 (d) `25 [Based on MAT, 2003]
[Based on MAT, 2003] 7. A can do a work in 9 days. If B is 50% more efficient to A,
then in how many days can B do the same work?
2
4. A worker makes a basket in of an hour. If he works for (a) 13.5 (b) 4.5
3
1 (c) 6 (d) 3
7 hrs, then how many baskets will he make?
2 [Based on MAT, 2003]

Chapter 10.indd 5 6/4/2015 11:06:25 AM


10.6 Chapter 10

8. Two men and 7 children complete a certain piece of work 15.


A can do piece of work in 14 days which B can do in
in 4 days while 4 men and 4 children complete the same 21 days. They begin together but 3 days before the
work in only 3 days. The number of days required by 1 completion of the work A leaves off. The total number of
man to complete the work is: days to complete the work is:
(a) 60 days (b) 15 days 3 1
(c) 6 days (d) 51 days (a) 6 (b) 8
5 2
[Based on MAT, 2003]
1 1
9. Shatabadi Express has a capacity of 500 seats of which (c) 10 (d) 13
5 2
10% are in the Executive Class and the rest being Chair
[Based on MAT, 2005]
cars. During one journey, the train was booked to 85%
of its capacity. If Executive Class was booked to 96% 16.
A contractor undertook to do a certain work in 75 days
of capacity, then how many Chair Car seats were empty and employed 60 men to do it. After 25 days he found
during that journey? that only one-fourth of the work was done. How many
(a) 75 (b) 73 more men must be employed in order that the work may
(c) 71 (d) None of these be finished in time?
[Based on MAT, 2002] (a) 34 (b) 38
10. ‘A’ can do a piece of work in 25 days and B in 20 days. (c) 35 (d) 30
They work together for 5 days and then ‘A’ goes away. In 17.
A contractor undertook to do a piece of work in 9 days.
how many days will ‘B’ finish the remaining work? He employed certain number of labourers but 6 of them
(a) 17 days (b) 11 days being absent from the very first day, the rest could finish
(c) 10 days (d) None of these the work in 15 days. Find the number of men originally
employed.
[Based on MAT, 2002]
(a) 15 (b) 6
11. If Ajit can do one-fourth of a work in 3 days and Sujit can
(c) 13 (d) 9
do one-sixth of the same work in 4 days, how much will
[Based on MAT, 2001]
Ajit get if both work together and are paid `180 in all?
(a) `120 (b) `108 18. A man can do a job in 5 hrs. After 2 hrs 20 mins, the man
stops working. He is replaced by a woman to complete the
(c) `60 (d) `36
job. She does the remainder of the work in 1 hr 40 mins. If
[Based on MAT, 2001]
the woman works alone, how much faster will she be than
12. A and B weave a carpet in 10 days and 15 days, respectively. the man?
They begin to work together but B leaves after 2 days. In (a) 1 hr 25 mins (b) 1 hr 55 mins
what time will A complete the remaining work?
(c) 2 hrs (d) 1 hr 17 mins
1 2
(a) 6 days (b) 6 days 19. 30 men can produce 1500 units in 24 days working 6 hrs
3 3 a day. In how many days, can 18 men produce 1800 units
(c) 7 days (d) 8 days working 8 hrs a day?
[Based on MAT, 2001] (a) 36 days (b) 45 days
13.
A can do a work in 18 days, B in 9 days and C in 6 days. (c) 18 days (d) None of these
A and B start working together and after 2 days C joins [Based on IIT Joint Man. Ent. Test, 2004]
them. What is the total number of days taken to finish the 20. If a man can reap 80 hectares in 24 days and a woman can
work? reap 60 hectares in 30 days, then how much time they will
(a) 4.330 take together to reap 150 hectares?
(b) 4.5 (a) 26.25 days (b) 32 days
(c) 4.66 (c) 28.125 days (d) 30.12 days
(d) None the these
21. Niki types 150 words in 4 mins and Nishu types 150
[Based on MAT, 2000]
words in 5 mins, then in how much time they can together
14.
If 18 persons can build a wall 140 m long in 42 days, the type 375 words?
number of days that 30 persons will take to complete a 50 7
similar wall 100 m long is: (a) mins (b) mins
9 2
(a) 18 (b) 21
(c) 24 (d) 28 7
(c) mins (d) None of these
[Based on MAT, 2005] 9

Chapter 10.indd 6 6/15/2015 2:25:46 PM


Time, Work and Wages 10. 7

22. 16 men and 12 women can complete a work in 20 days. 29. A group of men decided to do a job in 4 days. But since
18 women can complete the same work in 40 days. In how 20 men dropped out every day, the job completed at the
many days will 12 men and 27 women complete the same end of the 7th day. How many men were there at the
work? beginning?
(a) 12 (b) 16 (a) 240 (b) 140
(c) 18 (d) 24 (c) 280 (d) 150
23. Sam, Bob and Kirm can do a job alone in 15 days, 10 30. A group of workers decided to finish a work in 10 days but
days and 30 days respectively. Sam is helped by Bob and 5 of them could not join the team. If the rest of the crew
Kim every third day. In how many days will the job be completed the job in 12 days, the number of members
completed? present originally in the team were:
1 (a) 45 (b) 30
(a) 9 (b) 8
3 (c) 50 (d) 35
1 31. A, B, and C, working together complete a job in 18
(c) 8 (d) 6
3 days. A, and B, together work twice as C, A, and C,
[Based on I.P. Univ., 2002]
together work thrice as much as B, A, alone can finish the
work in:
24. Construction of a road was entrusted to a civil engineer. (a) 18 days (b) 43.2 days
He was to finish the work in 124 days for which he
employed 120 workmen. Two-thirds of the work was (c) 54 days (d) 72 days
completed in 64 days. How many workmen can be 32. A is twice as fast as B, and is therefore able to finish the
reduced now without affecting the completion of the work in 30 days less than B. Find the time in which they
work on time? can do it working together.
(a) 56 (b) 64 (a) 18 days (b) 20 days
(c) 80 (d) 24 (c) 24 days (d) 22 days
[Based on I.P. Univ., 2002] 33. 12 men can complete a piece of work in 36 days. 18
women can complete the same piece of work in 60 days.
25. Two workers earned `225. The first worked for 10 days
8 men and 20 women work together for 20 days. If only
and the second for 9 days. How much did each of them get
women were to complete the remaining piece of work in
daily if the first worker got `15 more for working 5 days
4 days, how many women would be required?
than the second worker got for working 3 days?
(a) 70 (b) 28
(a) `11.70; `12.00 (b) `10.80; `13.00
(c) 66 (d) 40
(c) `11.25; `12.50 (d) `12.60; `11.00
[Based on IRMA, 2009]
[Based on SCMHRD, 2002]
34.
A can do a piece of work in 40 days. He starts working,
26. Eight children and 12 men complete a certain piece of
but having some other engagements he drops out after 5
work in 9 days. If each child takes twice the time taken by
days. Thereafter, B completes this work in 21 days. How
a man to finish the work, in how many days will 12 men
many days would A and B take to complete this work
finish the same work?
working together?
(a) 15 (b) 8
(a) 16 days (b) 15 days
(c) 9 (d) 12
[Based on MAT, 2008] (c) 17 days (d) 11 days
[Based on MAT (Feb), 2011]
27. A and B working separately can complete a job in 20 days 35.
A and B together can do a piece of work in 30 days. A
and 12 days, respectively. If they work on alternate days and B worked for 16 days and B finished the remaining
in tandem with B alone starting the job on the first day, work alone in 44 days. In how many days will B finish the
then the job will get completed on: whole work alone?
(a) 14th day (b) 15th day
(a) 24 days (b) 10 days
(c) 16th day (d) 17th day
(c) 32 days (d) 60 days
28. A, B and C contract a work for `4,500. A and B together [Based on MAT (Feb), 2011]
complete four-fifths of the work and then C takes over and
36.
Manoj and Ajita can do a job alone in 10 days and 12 days
finishes it. What is the amount that should be received by C?
respectively. Manoj starts the work and after 6 days Ajita
(a) `600 (b) `900 also joins to finish the work together. For how many days
(c) `1,200 (d) Cannot be determined did Ajita actually work on the job?

Chapter 10.indd 7 6/4/2015 11:06:25 AM


10.8 Chapter 10

3 1 43.
A computer can perform 30 identical tasks in 6 h. At that
(a) 2 (b)
2 rate, what is the minimum number of computers that
11 11
should be assigned to complete 80 of the tasks within 3 h?
2 1
(c) 2 (d) 3 (a) 12 (b) 7
11 12
[Based on MAT (Dec), 2010] (c) 6 (d) 16
[Based on MAT (Dec), 2009]
37.
Ashokan is thrice as good a workman as Nitin and is
therefore able to finish a piece of work in 40 days less than 44.
Construction of a road was entrusted to a civil engineer. He
Nitin. Find the time in which they can do it working together. was to finish the work in 124 days for which he employed
(a) 15 days (b) 7 days 120 workmen. Two-thirds of the work was completed
in 64 days. How many workmen can be reduced now
(c) 16 days (d) 13 days
without affecting the completion of the work on time?
[Based on MAT (Dec), 2010]
(a) 80 (b) 64
38.
A alone would take 8 hrs more to complete the job than if (c) 56 (d) 24
both A and B worked together. If B worked alone, he took
[Based on MAT (Dec), 2009]
1
4 hrs more to complete the job than if A and B worked 45.
Anu can complete a work in 10 days. Manu is 25% more
2
efficient than Anu and Sonu is 60% more efficient than
together. What time would they take if both A and B worked
Manu. Working together, how long would they take to
together?
finish the job?
(a) 8 days (b) 5 days
6 6
(c) 2 days (d) 6 days (a) 2 days (b) 5 days
17 7
[Based on MAT (Dec), 2010]
5 5
39.
Madhu takes twice as much time as Uma to complete a work (c) 3 days (d) 4 days
and Ramesh does it in the same time as Madhu and Uma 8 8
together. If all three working together can finish the work in [Based on MAT (Dec), 2009]
6 days, then the time taken by Madhu to finish the work is:
46.
If 15 men or 24 women or 36 boys can do a piece of work
(a) 12 days (b) 14 days in 12 days, working 8 hrs a day, how many men must be
(c) 36 days (d) 40 days associated with 12 women and 6 boys to do another piece
[Based on MAT (Dec), 2010] of work 2.25 times in 30 days working 6 hrs in a day?
40.
A 12 m long road can be dug by 18 men in 20 days. What (a) 9 (b) 7
length of road can be dug by 12 men in 15 days? (c) 5 (d) 8
(a) 9 m (b) 8 m [Based on MAT (Dec), 2009]
(c) 6 m (d) 7 m 47.
Two cogged wheels of which one has 16 cogs and the
[Based on MAT (May), 2010] other 27, work into each other. If the latter turns 80 times
41.
A, B and C can do a work in 8, 16 and 24 days respectively. in three quarters of a min, how often does the other turn in
They all begin together. A continues to work till it is 8 s?
finished, C left after 2 days and B one day before its (a) 26 (b) 25
completion. In what time is the work finished? (c) 24 (d) 27
(a) 7 days (b) 5 days [Based on MAT (Dec), 2009, 2008]
(c) 6 days (d) 8 days
1
[Based on MAT (May), 2010] 48.
X runs 1 times as fast as Y. If X gives Y a start of 300 m,
2
42.
Working together, Asha and Sudha can complete an how far must X run before he catches up with Y?
assigned task in 20 days. However, if Asha worked alone
and complete half the work and then Sudha takes over the (a) 450 m (b) 400 m
task and completes the second half of the task, the task (c) 1 km (d) 900 m
will be completed in 45 days. How long will Asha take [Based on MAT (Dec), 2009]
to complete the task if she worked alone? Assume that
Sudha is more efficient than Asha. 49.
A team of workers was employed by a contractor who
undertook to finish 360 pieces of an article in a certain
(a) 60 days (b) 30 days
number of days. Making four more pieces per day that
(c) 25 days (d) 65 days was planned, they could complete the job a day ahead of
[Based on MAT (May), 2010] schedule. How many days did they take to complete the job?

Chapter 10.indd 8 6/4/2015 11:06:26 AM


Time, Work and Wages 10. 9

(a) 8 days (b) 10 days 56. 18 men can complete a piece of work in 63 days. 9 women
(c) 9 days (d) 12 days take 189 days to complete the same piece of work. How
many days will 4 men, 9 women and 12 children together
[Based on MAT (Sept), 2009]
take to complete the piece of work if 7 children alone can
50.
The work done by a woman in 8 hrs is equal to the work complete the piece of work in 486 days?
done by a man in 6 hrs and by a boy in 12 h. If working (a) 76 (b) 63
6 hrs per day 9 men can complete a work in 6 days, then
in how many days can 12 men, 12 women and 12 boys (c) 54 (d) 81
together finish the same working 8 hrs per day? [Based on IRMA, 2005]

1 1 57.
A certain length of pathway has to be constructed. It is
(a) 2 days (b) 1 days found that three men can construct 1 km less than one-
2 2
fifth of all in 2 days, while 18 men can construct 1 km
more than two-fifths in one day. The length of the path is:
1
(c) 3 days (d) None of these (a) 25 km (b) 10 km
2
(c) 15 km (d) 20 km
[Based on MAT (Sept), 2009]
[Based on MAT (Sept), 2008]
51.
A sum of `25 was paid for a work which A can do in 32
days, B in 20 days, B and C in 12 days and D in 24 days. 58.
Two coal loading machines each working 12 hrs per
How much did C receive if all the four work together? day for 8 days handle 9000 tonnes of coal with an
efficiency of 90%; while 3 other coal loading machines
(a) `15/3 (b) `14/3 at an efficiency of 80% are set to handle 12000 tonnes of
(c) `13/3 (d) `16/3 coal in 6 days. Find how many hrs per day each should
[Based on MAT (May), 2009] work?
52. Ten women can complete a piece of work in 15 days. Six (a) 20 h/day (b) 18 h/day
men can complete the same piece of work in 10 days. In (c) 16 h/day (d) 14 h/day
how many days can five women and six men together [Based on MAT (Sept), 2008]
complete the piece of work?
59.
If 36 men can dig a trench 200 m long, 3 m wide and 2 m
(a) 15 days (b) 7.5 days
deep in 6 days working 10 hrs a day, in how many days,
(c) 9 days (d) 12.5 days working 8 hrs a day will 10 men dig a trench 100 m long,
[Based on IRMA, 2006] 4 m wide and 3 m deep?
53.
A, B and C working together completed a job in 10 days. (a) 15 days (b) 27 days
The ratio of their efficiency is 2:3:6. How many days (c) 20 days (d) 54 days
would be required by the fastest worker to complete the [Based on MAT (Sept), 2008]
entire work?
(a) 15 days (b) 20 days 60.
Bhavika alone would take 8 hrs more to complete the
job than when Bhavika and Rita worked together. If Rita
(c) 30 days (d) 40 days 1
[Based on MAT (Feb), 2009] worked alone, she would take 4 h more to complete the
2
54.
8 children and 12 men complete a certain piece of work in job than when Bhavika and Rita worked together. What
9 days. If each child takes twice the time taken by a man time would they take if both Bhavika and Rita worked
to finish the work, 12 men will finish the same work in: together?
(a) 12 days (b) 15 days (a) 5 days (b) 4 days
(c) 14 days (d) 8 days (c) 3 days (d) 6 days
[Based on MAT (Feb), 2009, 2008] [Based on MAT (Sept), 2008]
55.
If 15 men or 24 women or 36 boys can do a piece of work 61.
Ten men can finish a piece of work in 10 days, whereas
in 12 days, working 8 hrs a day, how many men must be it takes 12 women to finish it in 10 days. If 15 men and 6
associated with 12 women and 6 boys to do another piece woman undertake the work, how any days will they take
1 to complete it?
of work 2 times as great in 30 days working 6 hrs a day?
4 1
(a) 4 days (b) 4 days
(a) 4 (b) 8 2
(c) 6 (d) 10 (c) 5 days (d) 6 days
[Based on MAT (Dec), 2008] [Based on MAT (Sept), 2008]

Chapter 10.indd 9 6/4/2015 11:06:26 AM


10.10 Chapter 10

62.
Five painters can paint a wall 100 m long in 10 days of 68.
C is twice efficient as A. B takes thrice as many days as C.
8 hrs each. How many days of 6 hrs each will it take for A takes 12 days to finish the work alone. If they work in
8 men to paint a wall 30 m long? pairs (i.e., AB, BC, CA) starting with AB on the first day,
BC on the second day and AC on the third day and so on,
1
(a) 2 (b) 1 then how many days are required to finish the work?
2
1
3 (a) 6 days (b) 4.5 days
(c) 3 (d) 1 5
4
[Based on MAT (May), 2008]
1
(c) 5 days (d) 8 days
9
63.
There are two types of workers—category I and category [Based on MAT (Dec), 2006]
II. A category I worker can finish a piece of work in 2m
per 3 hrs and a category II worker can finish in m h. If one 69.
A can build up a wall in 8 days while B can break it in 3
worker of category I and two workers of category II are days. A has worked for 4 days and then B joined to work
employed, the work can be finished in how many hrs? with A for another 2 days only. In how many days will A
(a) 7 m/2 (b) 9 m/2 alone build up the remaining part of the wall?
(c) 2 m/7 (d) 2 m/9 1 1
(a) 13 days (b) 6 days
[Based on MAT (May), 2008] 3 3

64.
Two men A and B working together complete a piece of 1
(c) 7 days (d) 7 days
work which it would have taken them 30 and 40 days 3
respectively to complete if they worked separately. If they [Based on MAT (May), 2006]
received a payment of `2100, B’s share is: 70.
Children were fallen-in for a drill. If each row contained
(a) `900 (b) `800 4 children less, 10 more rows would have been made. But
(c) `1200 (d) `1300 if 5 more children were accommodated in each row the
number of rows would have reduced by 5. The number of
[Based on MAT (Dec), 2007]
children in the school is:
65.
Two men undertake to do a piece of work for `600. One (a) 200 (b) 150
alone could do it in 6 days and the other in 8 days. With (c) 300 (d) 100
the assistance of a boy, they finish it in 3 days. The boy’s [Based on MAT (Feb), 2011]
share should be:
71.
A and B together can do a piece of work in 12 days. B and C
(a) `75 (b) `225
together can do the same work in 16 days. After A has been
(c) `300 (d) `100 working at it for 5 days and B for 7 days, C finishes it in 13
[Based on MAT (May), 2007] days. In how many days will C alone be able to do the work?
66.
15 men can complete a work in 210 days. They started the (a) 16 (b) 24
work but at the end of 10 days 15 additional men, with (c) 36 (d) 48
double efficiency, were inducted. How many days in all [Based on MAT, 1998]
did they take to finish the work? 72.
5 persons working eight hrs daily can complete a wall
1
1 3 in 10 days. When they have worked for 5 days, 5
(a) 72 days (b) 84 days 2
2 4
more persons are brought to work. The wall can now be
2 completed in
(c) 76 days (d) 70 days (a) one more day (b) two more days
3
[Based on MAT (May), 2007] (c) three more days (d) four more days
[Based on MAT, 1998]
67.
4 men and 10 women were put on a work. They completed
one-third of the work in 4 days. After this 2 men and 2 73.
Ramesh is twice as good workman as Sunil and finished
women were increased. They completed two-ninths more a piece of work in 3 hrs less than Sunil. In how many hrs,
of the work in 2 days. If the remaining work is to be they together could finish that piece of work?
completed in 3 days, then how many more women must 1
(a) 2 (b) 2
be increased? 3
(a) 32 (b) 8 2
(c) 1 (d) None of these
(c) 50 (d) 55 3
[Based on MAT (Dec), 2006] [Based on MAT, 1999]

Chapter 10.indd 10 6/4/2015 11:06:26 AM


Time, Work and Wages 10. 11

74.
A mother and a daughter working together can complete 82.
A is twice as good a workman as B and together they finish
a work in 4 days. But if the mother works alone, she can a piece of work in 14 days. The number of days taken by A
complete the work in 6 days. Both of them worked for one alone to finish the work is:
day and then the mother had to leave. How long will the (a) 11 (b) 21
daughter take to complete the remaining work? (c) 28 (d) 42
(a) 7 days (b) 8 days [Based on FMS (MS), 2006]
(c) 9 days (d) 10 days
1
[Based on MAT, 1999] 83.
400 persons working 9 hrs per day th of the work in 10
4
75.
If 15 women or 10 men can complete a project in 55 days, days. The number of additional persons, working 8 hrs per
in how many days will 5 women and 4 men working day required to complete the remaining work in 20 days,
together complete the same project? is:
(a) 75 (b) 95 (a) 675 (b) 275
(c) 55 (d) 85 (c) 250 (d) 225
[Based on MAT, 1999] [Based on FMS (MS), 2006]
76.
24 men working at 8 hrs a day can finish a work in 10 84.
A job is completed by 10 men in 20 days and by 20 women
days. Working at the rate of 10 hrs a day, the number of in 15 days. How many days will it take for 5 men and 10
men required to finish the same work in 6 days is: women to finish that work?
(a) 30 (b) 32 1 1
(c) 34 (d) 36 (a) 17 (b) 17
2 7
[Based on MAT, 1999]
1
77.
A certain job was assigned to a group of men to do it (c) 17 (d) 17
120
in 20 days. But 12 men did not turn up for the job and
the remaining men did the job in 32 days. The original [Based on FMS (MS), 2006]
number of men in the group was: 85.
A and B can do a piece of work in 12 days; B and C in 15
(a) 32 (b) 34 days; C and A in 20 days. In how many days can A alone
(c) 36 (d) 40 do it?
[Based on MAT, 1999] (a) 30 (b) 24
78.
12 men complete a work in 18 days. Six days af­ter they 2
had started working, 4 men joined them. How many days (c) 15 (d) None of these
3
will all of them take to complete the remaining work?
[Based on FMS, 2005]
(a) 10 days (b) 12 days
(c) 15 days (d) 9 days 86.
A certain job was assigned to a group of men to do in
[Based on MAT, 1999] 20 days. But 12 men did not turn up for the job and the
remaining men did the job in 32 days. The original number
79.
A, B and C can do a work in 20, 25 and 30 days of men in the group was:
respectively. They undertook to finish the work together
(a) 32 (b) 34
for `2,220, then the share of A exceeds that of B by:
(a) `120 (b) `180 (c) 36 (d) 40
[Based on FMS, 2005]
(c) `300 (d) `600
[Based on MAT, 2000] 87.
40 men can build a wall 20 m high in 15 days. The
80.
A and B can together do a piece of work in 15 days. B number of men required to build a similar wall of
alone can do it in 20 days. In how many days can A alone 25 m high in 6 days will be:
do it? (a) 100 (b) 125
(a) 30 days (b) 40 days (c) 150 (d) 200
(c) 45 days (d) 60 days [Based on FMS, 2006]
[Based on MAT, 2000] 88.
A can complete a job in 9 days, B in 10 days and C in
81.
A worker is paid `56 for 35-hour week. Upto 40 hrs, he 15 days. B and C start the work and are forced to leave
is paid at the normal rate and on overtime, 1.5 times the after 2 days. The number of days taken by A to finish the
normal. How many hrs did he work to get `88? remaining work is:
(a) 48 (b) 52 (a) 8 (b) 9
(c) 58 (d) None the these (c) 6 (d) 14
[Based on MAT, 2000] [Based on FMS, 2006]

Chapter 10.indd 11 6/4/2015 11:06:26 AM


10.12 I Chapter 10

89.
A and B together can do a job in 2 days; B and C can do (a) 9000 (b) 9500
2 (c) 10000 (d) 10500
it in 4 days; and A and C in 2 days. The number of days
5 [Based on MAT, 2012]
required for A to do the job alone is:
96. A hostel has provision for 250 students for 35 days. After
(a) 1 (b) 3 5 days, a fresh batch of 25 students was admitted to the
(c) 6 (d) 12 hostel. Again after 10 days, a batch of 25 students left the
[Based on FMS, 2011] hostel. How long will the remaining provisions survive?
90. Two men undertake to do a piece of work for `600. One (a) 18 days (b) 19 days
alone could do it in 6 days and the other in 8 days. With (c) 20 days (d) 17 days
the assistance of a boy, they finished it in 3 days. Boy’s [Based on MAT, 2012]
share should be: 97. If 9 engines consume 24 metric tons of coal, when each is
(a) `75 (b) `225 working 8 hrs a day, how much coal will be required for
(c) `300 (d) `100 8 engines, each running 13 hrs a day, it being given that 3
[Based on MAT, 2011] engines of former type consume as much as 4 engines of
latter type?
91. A and B can separately do a piece of work in 20 and 15
days, respectively. They worked together for 6 days, after (a) 23 metric tons (b) 24 metric tons
which B was replaced by C. If the work was finished in (c) 25 metric tons (d) 26 metric tons
the next 4 days, then the number of days in which C alone [Based on MAT, 2012]
could do the work will be: 98. During one year, the population of a town increases by
(a) 60 days (b) 40 days 5% and during the next year, the population decreased by
(c) 35 days (d) 30 days 5% If the total population is 9975 at the end of the second
[Based on MAT, 2012]
year, then what was the population size in the beginning
of the first year?
92. B can do a piece of work in 6 h, B and C together can do
2 (a) 9000 (b) 9500
it in 4 h, and A, B and C together in 2 h. In how many (c) 10000 (d) 10500
3
hours can A and B together do the same piece of work? [Based on MAT, 2012]
1 99. A hostel has provision for 250 students for 35 days. After
(a) 11 h (b) 6
7 5 days, a fresh batch of 25 students was admitted to the
3 3 hostel. Again after 10 days, a batch of 25 students left the
(c) 2 h (d) 3 h hostel. How long will the remaining provisions survive?
7 7
[Based on MAT, 2012] (a) 18 days (b) 19 days
(c) 20 days (d) 17 days
1
93. X can do of a work in 10 days. Y can do 40% of the [Based on MAT, 2012]
4
1 100. If 9 engines consume 24 metric tons of coal, when each is
work in 40 days and Z can do of the work in 13 days. working 8 hrs a day, how much coal will be required for
3
Who will complete the work first? 8 engines, each running 13 hrs a day, it being given that 3
(a) X (b) Y engines of former type consume as much as 4 engines of
latter type?
(c) Z (d) Both X and Z
(a) 23 metric tons (b) 24 metric tons
[Based on MAT, 2012]
(c) 25 metric tons (d) 26 metric tons
94. A can do a piece of work in 7 days of 9 hrs each whereas B [Based on MAT, 2012]
can do the same work in 6 days of 7 hrs each. How long will
2 101. A and B can complete a task in 30 days when working
they take to do the work together, working 8  hrs a day? together. After A and B have been working together for
5
(a) 2 days (b) 3 days 11 days, B is called away and A completes the remaining
task in the next 28 days. Had A been working alone, the
(c) 4 days (d) 5 days
number of days taken by him to complete the task would
[Based on MAT, 2012] have been:
95. During one year, the population of a town increases by 3 6
(a) 33 days (b) 19 days
5% and during the next year, the population decreased by 19 25
5% If the total population is 9975 at the end of the second 4 7
(c) 44 days (d) 33 days
year, then what was the population size in the beginning 19 19
of the first year? [Based on MAT, 2013]

M10_KHAT6981_C10.indd 12 6/16/2015 3:53:57 PM


Time, Work and Wages 10. 13

102. Efficiency of Asha is 25% more than Usha and Usha 107. A and B completed a work together in 5 days. Had A
takes 25 days to complete a piece of work. Asha started worked at twice the speed and B at half the speed, it would
the work alone and then Usha joined her 5 days before have taken them 4 days to complete the job. How much
completion of the work. For how many days Asha worked time would it take for A alone to do the work?
alone? (a) 10 days (b) 20 days
(a) 13 days (c) 15 days (d) 25 days
(b) 11 days [Based on MAT, 2014]
(c) 10 days 108. A cistern is normally filled in 6 hrs but takes 4 hrs longer
(d) 15 days to fill because of a leak in its bottom. If the cistern is full,
[Based on MAT, 2013] the leak will empty it in how much time?
103. If 15 men, 24 women and 36 boys can do a piece of work (a) 20 h (b) 15 h
in 12 days, working 8 hrs per day, how many men must be (c) 23 h (d) 17 h
associated with 12 women and 6 boys to do another piece of [Based on MAT, 2014]
1
work 2  times as large in 30 days working 6 hrs per day? 109. 8 men and 4 women together can complete a piece of
4 work in 6 days. The work done by a man in one day is
(a) 6 (b) 4 double the work done by a women in one day. If 8 men
(c) 8 (d) 10 and 4 women started working and after 2 days 4 men left
[Based on MAT, 2013] and 4 new women joined, in how many more days will the
104. Two coal loading machines, each working 12 hrs per day work be completed?
for 8 days, handle 9000 tonne coal with an efficiency of (a) 5 days (b) 8 days
90%, while 3 other coal loading machines at an efficiency (c) 6 days (d) 4 days
of 80% are set to handle 12000 tonne of coal in 6 days. [Based on SNAP, 2013]
How many hours per day should each machine work?
110. A and B together can complete a task in 20 days. B and
(a) 18 hrs per day
C together can complete the same task in 300 days. A and
(b) 16 hrs per day C together can complete the same work in 40 days. What
(c) 20 hrs per day is the respective ratio of the number of days taken by A
(d) 14 hrs per day when completing the same task alone to the number of
[Based on MAT, 2013] days taken by C when completing the same task alone?
(a) 2:5 (b) 2:7
105. A, B and C working together completed a job in 10 days. (c) 3:7 (d) 1:5
However C only worked for the first three days when [Based on SNAP, 2013]
37/100 of the job was done. Also the work done by A in 5
days is equal to the work done by B in 4 days. How many 111. A certain number of people were supposed to complete a
days would be required by the fastest worker to complete work in 24 days. The work, however, took 32 days, since
the entire work? 9 people were absent throughout. How many people were
supposed to be working originally?
(a) 25 days
(a) 32 (b) 27
(b) 20 days
(c) 36 (d) 30
(c) 30 days
[Based on SNAP, 2013]
(d) 40 days
[Based on MAT, 2013] 112. A contractor undertakes to built a walls in 50 days. He
employs 50 people for the same. However after 25 days he
106. In a company XYZ Ltd. a certain number of engineers finds that only 40% of the work is complete. How many more
man need to be employed to complete the work in time?
can develop a design in 40 days. If there were 5
(a) 25
more engineers, it could be finished in 10 days less.
How many engineers were there in the beginning? (b) 30
(a) 18 (b) 20 (c) 35
(c) 25 (d) 15 (d) 20
[Based on MAT, 2014] [Based on SNAP, 2012]

Chapter 10.indd 13 6/4/2015 11:06:28 AM


10.14 Chapter 10

Difficulty Level-2
(Based on Memory)

1. A group of workers can complete a certain job in 9 days. p.m., one engineer is removed from the group every hour,
But it so happens that every alternate day starting from what is the time when they will finish the work?
the second day, 2 workers are withdrawn from the job and (a) 6.00 p.m. (b) 7.00 p.m.
every alternate day starting from the third day, one worker (c) 4.00 p.m. (d) 8.00 p.m.
is added to the group. In such a way, the job is finished by
the time there is no worker left. If it takes the double time 8. If four boys consume x calories every y days, six girls
to finish the job now, find the number of workers who y
consume z calories every days. How many calories
started the job? 2
will 10 girls and 10 boys consume in 12 days?
(a) 5 (b) 10
(c) 15 (d) 20 10
(a) 3x + 4z (b) (3x + 4z)
y
2.
A can do a piece of work in 40 days and B in 60 days.
Both of them start working together and 4 days before the 1
(c) (30x + 4z) (d) y(3x + 4z)
scheduled completion, A drops out. By how many days is y
the work extended beyond the normal schedule?
9. A piece of work is done by three men A, B, C in 5 days in
(a) 10 days (b) 8 days the following manner:
(c) 8.33 days (d) 6 days
A works for the whole time, B only on the first two days
3. Lal Singh can eat 50 laddoos in 4 hrs and Pal Singh can and C only on the last three days. This work could have
eat 42 laddoos in 6 hrs. If both of them start together, then been done by B and C in 6 days without involving A. If B
what is the total time required by them to eat 507 laddoos? and C working together can do as much work in two days
(a) 20 hrs (b) 21 hrs as A can do in 3 days, then find, how long will it take for
(c) 26 hrs (d) 25 hrs each one to do this work alone?
(a) 10, 20, 10 days (b) 7, 9, 11 days
4.
The daily work of 2 men is equal to that of 3 women or
(c) 8, 12, 9 days (d) 9, 18, 9 days
that of 4 youngsters. By employing 14 men, 12 women
and 12 youngsters a certain work can be finished in 24 10. If four examiners can examine a certain number of answer
days. If it is required to finish it in 14 days and as an books in 8 days by working 5 hrs a day, then for how
additional labour, only men are available, how many of many hrs a day, would two examiners have to work in
them will be required? order to examine twice the number of answer books in 20
(a) 20 men (b) 18 men days?
(c) 48 men (d) 28 men 1
(a) 6 hrs (b) 7 hrs
5. A man can finish a work in 6 days, a woman in 10 days 2
and a child in 12 days. The man starts the work and (c) 8 hrs (d) 9 hrs
works for one-third of a day, then the woman works for
[Based on FMS (Delhi), 2003]
one-third of a day and finally the boy works for remaining
one-third of a day. This cycle is repeated till work finishes. 11. 15 men take 21 days of 8 hrs each to do a piece of work.
Who was working when the work finished? How many days of 6 hrs each would 21 women take, if 3
(a) Man (b) Woman women do as much work as 2 men?
(c) Child (d) None (a) 20 (b) 18
6. If a man and a half can build a wall and a half in a day (c) 25 (d) 30
and a half, then how many walls do six men build in six [Based on FMS (Delhi), 2003]
days?
12. A certain job was assigned to a group of men to do in
(a) 3 (b) 6 20 days. But 12 men did not turn up for the job and the
(c) 12 (d) None of these remaining men did the job in 32 days. The original number
[Based on FMS (Delhi), 2004] of men in the group was:
7. Five engineers A, B, C, D and E can complete a process (a) 32 (b) 34
in 8 hrs, assuming that every engineer works with same (c) 36 (d) 40
efficiency. They started working at 10.00 a.m. If after 4.00 [Based on FMS (Delhi), 2003]

Chapter 10.indd 14 6/4/2015 11:06:28 AM


Time, Work and Wages 10. 15

13. Ram finishes a work in 7 days. Rahim finishes the same and left it after 15 days, how much more time would
job in 8 days and Robert in 6 days. They take turns to Mohan have taken to complete the remaining work?
finish the work. Ram worked on the first day, Rahim on (a) 7.5 days (b) 11.5 days
the second day and Robert on the third and then again (c) 12 days (d) 15 days
Ram and so on. Who was working on the last day when
work got finished? Directions (Questions 20 and 21): Answer the questions based on
(a) Ram (b) Rahim the following information.
(c) Robert (d) Rahim and Robert    Rakesh, Shiv and Vijay working alone can complete fencing a
(e) Cannot be determined wall in 12, 18 and 27 hrs respectively. All of them started fencing
[Based on IIFT, 2003] the wall together as a team and after 2 hrs, Rakesh left the team.
Shiv and Vijay continued to fence until Vijay fell ill and hence he
14. Ritu and Somaya are working on an assignment. Ritu had to leave the team. Then, Shiv finished fencing the wall in the
takes 6 hrs to type 32 pages on computer, while Somaya last 5 hrs.
takes 5 hrs for typing 40 pages. How much time will they
take, working together on two different computers to type 20. How many hrs did it take for the team to finish fencing the
an assignment of 110 pages? wall?
(a) 8 hrs (b) 7 hrs 30 mins (a) 11 hrs (b) 13 hrs
(c) 8 hrs 15 mins (d) 8 hrs 25 mins (c) 15.66 hrs (d) None of these
[Based on SCMHRD Ent. Exam., 2003] 21.
Which pair among the following will be able to finish
15. A boy read three-eighths of the book on one day and fencing the wall in the least time?
four-fifths of the remainder on another day. If there were (a) Rakesh and Shiv (b) Shiv and Vijay
30 pages unread, then how many pages did the book (c) Rakesh and Vijay (d) Either (b) or (c)
contain?
22. Robin, the cook, can cut either 120 carrots or 72 potatoes
(a) 600 (b) 300
or 60 beetroots in t mins. One day, during the first K mins,
(c) 240 (d) None of these he spent an equal amount of time cutting each of the three
[Based on IMT Ghaziabad, 2002] types of vegetables. However, in the next K mins, he cut
16. Twenty men were employed to do some work in a exactly n carrots, n beetroots and n potatoes. If he cut a
certain time. But when one-third of the scheduled time total of 282 vegetables during the first 2K mins on that
was over, it was found that only one-quarter of the total day, what is the value of n?
work was completed. How many more men should now (a) 225 (b) 75
be employed to complete the work in three-fourths of the (c) 45 (d) 135
originally scheduled time?
23. A team of 30 men is supposed to do a work in 38 days.
(a) 20 (b) 40 After 25 days, 5 more men were employed and the work
(c) 48 (d) 28 was finished one day earlier. How many days would it
17. Three workers, working all days, can do a work in 10 days, have been delayed if 5 more men were not employed?
but one of them having other employment can work only (a) 1 day (b) 4 days
half time. In how many days the work can be finished? (c) 3 days (d) 5 days
(a) 15 days (b) 16 days 24. If 15 men or 24 women or 36 boys can do a work in
(c) 12 days (d) 12.5 days 12 days working 8 hrs a day, how many men must be
[Based on FMS (Delhi), 2002] associated with 12 women and 6 boys to do another work 
18. Imran and Irfan are two tailors. Imran takes three hrs to 1
2  times as great in 30 days working 6 hrs per day ?
stitch 10 shirts and four hrs to stitch 12 pants. Irfan can 4
stitch 12 pants in three hrs and 10 shirts in four hrs. They (a) 10 (b) 15
get an order for the delivery of 200 shirts and 200 pants. (c) 8 (d) None of these
What is the quickest time in which they can deliver the
25. A contractor undertook to complete the work in 40 days
order?
and he deployed 20 men for his work 8 days before the
(a) 59 hrs 6 mins (b) 66 hrs 6 mins scheduled time he realised that one-third of the work was
(c) 63 hrs 12 mins (d) 55 hrs 43 mins still to be done. How many more men were required to
complete the work in stipulated time?
19. Mohan can complete a work in 25 days. He worked for
5 days and left the work; and then Bhim completed the (a) 16 (b) 15
remaining work in 30 days. Had Bhim started the work (c) 20 (d) 25

Chapter 10.indd 15 6/4/2015 11:06:28 AM


10.16 Chapter 10

26. A contractor undertook a work to complete in 60 days. But and 100 pages of Engineering Drawing. Alternatively,
just after 20 days he observed that only one-fifth of the he can also read 50 pages of Engineering Mathematics
project work had been completed. To complete the work and 250 pages of Engineering Drawing. Assume that the
in time (i.e., in rest days) minimum how many workers amount of time it takes to read one page of the textbooks
he had to increase, if there were initially 75 workers were of either subject is constant. Ramesh is confident about
deployed for the task? Engineering Drawing and wants to devote full time
(a) 25 (b) 50 to reading Engineering Mathematics. The number of
Engineering Mathematics text book pages he can read on
(c) 75 (d) Cannot be determined
Tuesday is:
Directions (Questions 27 and 28): Answer the questions based on
(a) 500  (b) 300  (c) 100  (d) 60
the following information.
[Based on JMET, 2006]
  The Weirodo Holiday Resort follows a particular system of
holidays for its employees. People are given holidays on the days 33. A started a work and left after working for 2 days. Then, B
where the first letter of the day of the week is the same as the first was called and he finished the work in 9 days. Had A left
letter of their names. All employees work at the same rate. the work after working for 3 days, B would have finished
the remaining work in 6 days. In how many days can each
27. Raja starts working on February 25, 1996 and finishes the of them, working alone, finish the whole work?
job on March 2, 1996. How much time would T and I take (a) 5 days, 8.5 days (b) 2.5 days, 7.5 days
to finish the same job if both start on the same day as Raja?
(c) 5 days, 15 days (d) None of these
(a) 4 days (b) 5 days [Based on NMAT, 2005]
(c) Either 4 or 5 days (d) Cannot be determined 34.
If a person makes ‘J’ jobs in ‘M’ mins, how many jobs will
28. Starting on February 25, 1996, if Raja had finished his he make in 2/3 hr?
job on April 2, 1996, when would T and S likely to have 2 J 2M
completed the job, had they started on the same day as Raja? (a) (b)
3M 3 J
(a) March 15, 1996 (b) March 14, 1996
J M
(c) March 22, 1996 (d) Data insufficient (c) 40 (c) 40
M J
Directions (Questions 29 and 30): Answer the questions on the [Based on XAT, 2006]
basis of the information given below. 35.
Three professors Dr. Gupta, Dr. Sharma and Dr. Singh
  Ram and Shyam run a race between points A and B, 5 km are evaluating answer scripts of a subject. Dr. Gupta is
apart. Ram starts at 9 a.m. from A at a speed of 5 km/h, reaches 40% more efficient than Dr. Sharma, who is 20% more
B, and returns to A at the same speed. Shyam starts at 9:45 a.m. efficient than Dr. Singh. Dr. Gupta takes 10 days less than
from A at a speed of 10 km/h, reaches B and comes back to A at Dr. Sharma to complete the evaluation work. Dr. Gupta
the same speed. starts the evaluation work and works for 10 days and then
Dr. Sharma takes over. Dr. Sharma evaluates for next 15
29. At what time do Ram and Shyam first meet each other? days and then stops. In how many days, Dr. Singh can
(a) 10.00 a.m. (b) 10.10 a.m. complete the remaining evaluation work?
(c) 10.20 a.m. (d) 10.30 a.m. (a) 7.2 days (b) 9.5 days
(c) 11.5 days (d) None of these
30. At what time does Shyam overtake Ram?
[Based on IIFT, 2010]
(a) 10.20 a.m. (b) 10.30 a.m.
36.
Aditya, Vedus and Yuvraj alone can do a job in 6 weeks, 9
(c) 10.40 a.m. (d) 10.50 a.m.
weeks and 12 weeks respectively. They work together for
31. Cloth markers Inc. has p spindles, each of which can 2 weeks. Then, Aditya leaves the the job. Vedus leaves the
produce q metres of cloth on an average in r mins. If job a week earlier to the completion of the work. The job
the spindles are made to run with no interruption, then would be completed in:
how many hrs will it take for 20,000 m of cloth to be (a) 4 weeks (b) 5 weeks
produced?
(c) 7 weeks (d) None of these
(a) 20,000 (pq)/r (b) 20,000 (rq)/p [Based on ITFT, 2009]
(c) 20,000 (pr)/(pq) (d) 20,000 r/60 (pq) 37.
The digging work of the DMRC on the Adchini-
32. Ramesh has two examinations on Wednesday-Engineering Andheriamore stretch requires twenty-four men to
Mathematics in the morning and Engineering Drawing in complete the work in sixteen days. As a part of the task if
the afternoon. He has a fixed amount of time to read the DMRC were to hire thirty-two women, they can complete
textbooks of both these subjects on Tuesday. During this the same work in twenty-four days. Sixteen men and
time he can read 80 pages of Engineering Mathematics sixteen wkomen started working the worked for twelve

Chapter 10.indd 16 6/4/2015 11:06:28 AM


Time, Work and Wages 10. 17

days. Due to time bound schedule the work had to be 43.


A contractor take up an assignment that 20 men can
completed in remaining 2 days, for which how many more complete in 10 days. The same assignment could be
men are to be employed? finished by 15 women in 20 days. The contractor decides
(a) 48  (b) 24  (c) 36  (d) 16 to employ 10 men and 10 women for the project. Given
[Based on IIFT, 2007] this, mark all the correct options.
38.
Due to the recent global financial crisis, many companies (a) If the wage rate for men and women are `50 and `45
in the field of IT Services offer jobs on contractual basis respectively, the total wage bill for the project will be
with a clause of fine for the leave taken by the employee. `11400.
An ITS company employs an engineer for 290 days on a (b) If the wage rate for men and women are `45 and `40
salary of `500 for eight hrs work per day, and the engineer respectively, the total wage bill for the project will be
has to pay a fine of `50 for each hour of his absence. `10200.
The engineer may compensate his one-day absence by (c) If the wage rate for men and women are equal at `40,
working 4 hrs extra for two days. If the engineer receives the total wage bill for the project will be `9100.
`132400 at the end of the contract, how many hrs was he (d) If the contractor decides to employ 20 men and 30
absent from his job? women for the project and the wage rate for men and
(a) 110 hrs  (b) 112 hrs   (c) 114 hrs  (d)  118 hrs women are `40 and `35 respectively, the total wage
[Based on FMS, 2009] bill for the project will be `9250.
39.
Three machines P, Q and R, working together, can do a job [Based on IIFT, 2006]
in x h. When working alone, P needs an additional 6 hrs to 44. Rohan and Sohan took a vacation and went to their
do the job; Q, one additional hour; and R, x additional hrs. grandparents’ house. During the vacation, they did several
The value of x is: activities together. They either played tennis in the evening
2 11
(a)   (b)    (c) 3   (d) 2 or practiced yoga in the morning, ensuring that they did not
3 12 2 undertake both the activities on any single day. There were
[Based on FMS, 2010] some days when they did nothing. Out of the days they
40.
Two candles of the same height are lighted at the same stayed at their grandparents’ house, they were involved
time. The first is consumed in 4 hrs and the second in 3 h. in one of the two activities on 22 days. However, their
Assuming that each candle burns at a constant rate, in how grandmother sent an end-of-vacation report to their parents
many hrs after being lighted was the first candle twice the stating that they did not do anything on 24 mornings and
height of the second? 12 evenings. How long was their vacation?
3 1 (a) 36 days (b) 14 days
(a) h (b) 1 h (c) 29 days (d) Cannot be determined
4 2
[Based on CAT, 2010]
(c) 2 h (d) h 45. Ram starts working on a job and continues for 12 days
[Based on FMS, 2011] completing 40 % of the work. To complete the work, he
employs Ravi. Together they work for another 12 days and
41.
A manufacturer builds a machine which will address 500 completed it. How much more efficient is Ram than Ravi?
envelopes in 8 min. He wishes to build another machine
so that when both are operating together they will address (a) 50  %  (b) 200  %  (c) 60  %  (d)  100 %
500 envelopes in 2 min. The equation used to find how [Based on CAT, 2010]
many mintues x it would required the second machine to 46. A, B and C can independently do a work in 15 days, 20
address 500 envelopes alone, is: days and 30 days, respectively. They work together for
1 1 1 some time after which C leaves. A total of `18000 is paid
(a) 8 – x = 2 (b)  =
8 x 2 for the work and B gets `6000 more than C. For how many
500 500 x x days did A work?
(c)  = 500 (d)   1 (a) 2  (b) 4  (c) 6  (d) 8
8 x 2 8
[Based on CAT, 2012]
[Based on FMS, 2011]
47. The amount of work in a steel plant increased by 50%
42.
Ramesh takes twice as much time as Mahesh and thrice By what percent is it necessary to increase the number
as much time as Suresh to complete a job. If working of workers to complete the new amount of work in
together, they can complete the job in 4 days, then the time previously planned time, if the productivity of the new
taken by each of them separately to complete the work is: labour is 25% more.
(a) 36, 24 and 16 days (b) 20, 16 and 12 days (a) 60% (b) 66.66%
(c) 24, 12 and 8 days (d) None of these (c) 40% (d) 33.33%
[Based on IIFT, 2005] [Based on CAT, 2013]

Chapter 10.indd 17 6/4/2015 11:06:29 AM


10.18 Chapter 10

Answer Keys
Difficulty Level-1

1. (b) 2. (b) 3. (d ) 4. (b) 5. (a) 6. (d ) 7. (c) 8. (b) 9. (b) 10. (b) 11. (a) 12. (b) 13. (d)
14. (a) 15. (c) 16. (d ) 17. (a) 18. (b) 19. (a) 20. (c) 21. (a) 22. (b) 23. (a) 24. (a) 25. (b) 26. (d)
27. (b) 28. (b) 29. (b) 30. (b) 31. (b) 32. (b) 33. (a) 34. (b) 35. (d ) 36. (c) 37. (a) 38. (d ) 39. (c)
40. (c) 41. (b) 42. (a) 43. (c) 44. (c) 45. (a) 46. (d ) 47. (c) 48. (d ) 49. (b) 50. (b) 51. (d ) 52. (b)
53. (b) 54. (a) 55. (b) 56. (d ) 57. (d ) 58. (c) 59. (b) 60. (d ) 61. (c) 62. (a) 63. (c) 64. (a) 65. (a)
66. (c) 67. (b) 68. (c) 69. (c) 70. (b) 71. (b) 72. (c) 73. (b) 74. (c) 75. (a) 76. (b) 77. (a) 78. (d )
79. (b) 80. (d ) 81. (d ) 82. (b) 83. (a) 84. (b) 85. (a) 86. (a) 87. (b) 88. (c) 89. (b) 90. (a) 91. (b)
92. (d ) 93. (c) 94. (b) 95. (c) 96. (b) 97. (d ) 98. (c) 99. (b) 100. (d ) 101. (c) 102. (c) 103. (c) 104. (b)
105. (c) 106. (d ) 107. (a) 108. (b) 109. (a) 110. (d ) 111. (c ) 112. (a)

Difficulty Level-2

1. (b) 2. (d ) 3. (c) 4. (a) 5. (b) 6. (b) 7. (d ) 8. (b) 9. (d ) 10. (c) 11. (d ) 12. (a) 13. (a)
14. (c) 15. (c) 16. (d ) 17. (c) 18. (d ) 19. (d ) 20. (a) 21. (a) 22. (c) 23. (a) 24. (c) 25. (c) 26. (c)
27. (c) 28. (c) 29. (d ) 30. (b) 31. (d ) 32. (c) 33. (c) 34. (c) 35. (a) 36. (d ) 37. (b) 38. (b) 39. (a)
40. (d ) 41. (b) 42. (c) 43. (a, b, c, d ) 44. (c) 45. (d ) 46. (d ) 47. (c)

Explanatory Answers

Difficulty Level-1

1.
(b) 6 machines in 4 mins can produce 1080 bottles. 1 15
7
10 machines in 4 mins can produce 4. (b) 2 = 2
2 2
1080 3 3
× 10 = 1800 bottles.
6
15 3 45
= × =
2.
(b) 160 bolts/20 s, i.e., 8 bolts/second, i.e., 200 bolts/25 s. 2 2 4

1
3.
(d) Suppose A and B are two men who can finish a certain = 11 .
work in 6 days and 8 days, respectively. Let, the boy 4
C can finish the same work in x days.
1 1 1
5.
(a) Given A + B = ,B+C= and C + A =
1 1 1 1 10 12 15
\ + + =
6 8 x 3
1 1 1 654
⇒ x = 24 ∴ 2(A + B + C) =   =
10 12 15 60
⇒ `200 must be distributed among A, B and C in the
15 1
1 1 1 = =
ratio : : , i.e., 4:3:1 60 4
6 8 24
1
1 ∴ A + B + C =
\ C’s share = × 200 8
8 Hence if A, B and C all work together, they will need
= `25. total 8 days to complete the work.

Chapter 10.indd 18 6/9/2015 1:16:30 PM


Time, Work and Wages 10. 19

6. (d) Suppose Rohit types x pages per hr. 1 1 , i.e., 2:1.


Harsha types y pages per hr :
12 24
Sanjeev types z pages per hr 2
\ Ajit’s share = × 180 = `120.
\ z – y = y – x 3
x+z
⇒ y = (1)
2 1 1 1
⇒ 5z = 7x (2) 12.
(b) (A + B)’s one day’s work = + =
10 15 6
4 (x + y + z) = 216
⇒ x + y + z = 54 (3) 1
\ (A + B)’s two day’s work =
Solving Eqs. (1), (2) and (3), we get 3
2
y = 18, x = 15, z = 21. Remaining of the work is done by A alone in
3
7.
(c) If A can do a work in 9 days then B can do the same 2 20 2
work in 6 days. (B being 50% more efficient than A). 10 × = = 6 days.
3 3 3
1
8.
(b) (2M + 7C)’s one day’s work =
4 13.
(d) (A + B + C)’s one day’s work
\ 8M + 28C can finish the whole work. 1 1 1 1+ 2 +3 1
= + + = =
1 18 9 6 18 3
Also, (4M + 4C)’s one day’s work =
3 (A + B)’s one day’s work
\ (12M + 12C) can finish the whole work 1 1 1+ 2 1
= + = =
\ 8M + 28C = 12M + 12C 18 9 18 6
⇒ M = 4C 1
(A + B)’s 2 days’ work =
\ 4M + 4C = 5M 3
Since 5 Men can complete the work in 3 days Remaining 2/3 of the work is finished by A, B and C
⇒ 1 Man can complete the work in 15 days. together in 2 days.
\ Total number of days taken to finish the work = 4.
9. (b) Total number of seats = 500
No. of seats in Executive Class = 50 14.
(a) 140 × 40 × x = 100 × 18 × 42
No. of seats in Chair Car = 450 100 × 18 × 42
⇒ x = = 18
Total seats occupied = 85% of 500 = 425 140 × 30
Total seats occupied in Executive Class \ Required no. of days = 18.
= 96% of 50 = 48
3 1
Total seats occupied in the Chair Car 15.
(c) B’s 3 days work = =
21 7
= 425 – 48 = 377
1 6
\ No. of seats vacant in the Chair Car Remaining work = 1 – =
7 7
= 450 – 377 = 73.
1 1 5
(A + B)’s 2 days’ work = + =
1 14 14 42
10.
(b) A’s one day’s work =
25 5
work is done by A and B in 1 day
1 42
B’s one day’s work = 6 42 6 36
20 \ work is done by A and B in × = days
1 1 9 7 5 7 5
(A + B)’s one day’s work = + = 36 1
25 20 100 \ Total time taken = 3 + = 10 days.
5 5
45 9
(A + B)’s five day’s work = = 1 3
100 20 16.
(d) 60 men in 25 days can do of work. For th of
11 4 4
Remaining of the work is to be finished by B in 3
20 work in 50 days, men required = 60 × = 90 men.
11 days. 2
Additional men = 90 – 60 = 30.
1
11.
(a) Ajit’s one day’s work = 17. (a) Let, the number of men originally employed = x.
12 \ (x – 6) men could finish the work in 15 days and x
1 men could finish the work in 9 days.
Sujit’s one day’s work =
24 \ 9x = 15(x ­– 6)
\ `180 are divided among them in the ratio ⇒ x = 15.

Chapter 10.indd 19 6/4/2015 11:06:30 AM


10.20 Chapter 10

1 therefore 1 woman can complete the work in 40 × 18


18.
(b) In 1 h, the man does th of the work. After 2 hrs he days = 720 days.
5
2 1
has completed th of the work. In 20 min he does ∴ 1 woman in 1 day will complete work
5 (40  18)
1 12
th of the work so when he stops working the man Now 12 women in 1 day complete work
15 (40  18)
2 1  7 12  20 1
has done    of the work, or in other words 12 women in 20 days complete  work
 5 15  15 (40  18) 3
th of the work. Balance work remaining to be done is 1 2
∴ 16 men in 20 days will finish = 1   work
7 8 3 3
1– =
15 15 2
∴ 1 man in 1 day can do work
8 (3  16  20)
The woman completes of the work in 100 mins.
15 12 men and 27 women in 1 day can complete
   She will complete the whole work on her own (or   27 12  2  

 
15
of the work) in 15 ×
100
mins ≅ 185 mins.   (40  18) (3  16  20)   work
15 8 5
  Man does it in 300 mins; hence the woman is = work in 1 day.
80
faster by 115 mins or 1 hour 55 mins.
80
19.
(a) 30 men can produce 1500 units in 144 hrs. ∴ Time taken to complete the work =  16 days.
5
30 men can produce 1800 units in
3 1 1 6 + 3 +1 1
144 864 23.
(a) In 3 days, + + = = of the work
× 1800 = hrs 15 10 30 30 3
1500 5
is done by Sam, Bob and Kim.
18 men can produce 1800 units in
\ The whole job will be finished by them in 9 days.
864 30
× hrs = 288 hrs 2
5 18 24.
(a) rd of the work was completed in 64 days by
3
= 36 days of 8 hrs each.
120 men.
80
20.
(c) A man can reap hectares in one day. A woman can 1
24 rd of the work was completed in 32 days by
60 3
reap hectares in one day. Together they can reap 120 men.
30
1
Also rd of the work is to be completed in 60 days by
80 60 16 3
  =  hectares in one day.
24 30 3 (120 – x) men, where x is the number of men reduced
3 in order to finish the work on schedule.
So, they will reap 150 hectares in  150 \ (120 – x) × 60 = 32 × 120 ⇒ x = 56
16
M received 5% of `7400 extra for being the working
= 28.125 days.
member, i.e., `370.
21.
(a) Niki can do a job in 4 mins, while Nishu can do the 25. (b) Let, A got `x per day and B got `y per day.
same job in 5 mins. \ 10x + 9y = 225 and 5x = 3y + 15
Then, in one min they can together do ⇒ x = 10.80, y = 13.
1 1 9 26.
(d) Time taken by child is twice the time taken by man
 = of the work.
4 5 20 4 + 12 = 16 men do the work in 9 days
5 5 16
Now, 375 = of 150. Hence, they can together do \ 12 men can do it in = 9 × = 12 days
2 2 12
27.
(b) Percentage of job completed by A in 1 day
50
of the work in mins. 1
9 = = 5%
20
22.
(b) Since 18 women can complete the work in 40 days,

Chapter 10.indd 20 6/4/2015 11:06:31 AM


Time, Work and Wages 10. 21
8 × 20 10
Percentage of job completed by B in 1 day \ 8 men in 20 days can do = work
12 × 36 27
1
= = 8.33% Similarly we find that 20 women in 20 days can do
12
10
Percentage of work completed in first two days work.
27
= 5 + 8.33 = 13.33%
7
Work completed in 14 days (7 for each A and B) \ Remaining work is .
27
= 13.33 × 7 = 93.31% Now because in 60 days a work is done by 20 women.
On 15th day (B will work) = 93.33 + 8.33 > 100 \ in 1 day a work is done by 20 × 60 women.
So, the work will be completed by B on 15th day.
20 × 60 × 7
\ in 4 days 7/27 work is done by
4 1 27 × 4
28.
(b) Work done by C = 1  =
5 5 = 70 women
1 1
∴ C’s amount = × 4500 = `900 34.
(b) A’s one day work =
5 40
29. (b) Go through option
140 × 4 = (140 + 120 + 100 +...+ 200) 5 1
A’s five days work = =
560 = 560 40 8
Alternatively: Let, n be the initial number of workers
then n + 4 = n + (n – 20) + (n – 40) + ... + (n – 120) 1 7
\ Remaining work = 1 − =
4n = 7n – 420 8 8
⇒ 3n = 420 \ B’s completed the remaining work in 21 days
⇒ n = 140 workers 7
=
30. (b) Amount of work to be done = 10n, where n = Number 8
of workers originally available.
7 1
Now 10n = 12(n – 5) ⇒ 2n = 60 1 day work of B = =
8 × 21 24
Therefore, n = 30
\ Both (A + B)’s one day work
1 1 1 1
31.
(b)   = (1) 1 1 3+5
a b c 18 = + =
40 24 120
1 1 2
 = (2) 8 1
a b c = =
120 15
1 1 3 Hence, both complete the work in 15 days.
 = (3)
a c b
35.
(d) Let, A can finish the work alone in x days.
1 1 2 1
(1) and (2)  ⇒    or c = 54 days But work of (A + B)’s =
18 c c 30
1 1 3 1 1
(1) and (3)  ⇒    or b = 72 days \ One day work of B alone = −
18 b b 30 x
1
∴ a = 43.2 days So, one day work of A =
x
32. (b) Let, B completes 1 work in 2x days then A will
complete the work in x days given, 2x – x = 30 ⇒ x = 16
16 days work of A =
30 days x
∴ work done by both of them working together 16
\ Remaining work = 1 –
1 1 1 x
=    work.
30 60 20
So, they can do it working together in 20 days.  1 1
44 days work of B = 44  − 
 30 x 
33. (a) Q 12 men in 36 days can do a work.
1 16  1 1
\ 1 man in a day can do work \ 1− = 44  − 
12 × 36 x  30 x 

Chapter 10.indd 21 6/4/2015 11:06:32 AM


10.22 Chapter 10

16 44 44  2x × x  2
⇒ 1− = −   days = x days
x 30 x  2 x + x  3

28 14 2
⇒ = So, Ramesh can complete the work in x days.
x 30 3

28 × 30 1 1 3 1
⇒ x = Now, + + =
14 2x x 2x 6
⇒ x = 60 days 6 1
⇒ =
1 1 1 2x 6
\ One day work of B alone = − =
30 60 60 ⇒ 2x = 36
So, Madhu can complete the work in 36 days.
Hence, B can do this work in 60 days.
6 3 M1D1 M D
36.
(c) Work done by Manoj for 6 days = = 40.
(c) = 2 2
10 5 W1 W2

3 2
Remaining work = 1 − = 18 × 20 12 × 15
5 5 ⇒ =
12 W2
2 2  10 × 12 
Manoj and Ajita can finish work in ×   12 × 15 × 12
5 5  10 + 12  ⇒ W2 = =6m
days. 18 × 20

2  10 × 12  41.
(b) Let, the work be finished in x days.
So, Ajita actually work for ×  days
5  10 + 12 
x ( x − 1) 2
Then, 1 = + +
2 120 2 8 16 24
= × 2 days
=
5 22 11
11 2x + x − 1
37. (a) Let, Ashokan can finish the work in x days. Then, Nitin ⇒ =
12 16
can finish the work in 3x days.
3x – x = 40 16 × 11
⇒ 3x – 1 =
⇒ x = 20 days 12
and 3x = 60 days
47
 20 × 60  ⇒ x = ≈ 5 days
So, together they can finish the work in  9

 20 + 60  42.
(a) Let, Asha can complete the work in x days and Sudha
days =15 days
can complete in y days.
38.
(d) (A + B)’s complete the work in x days. x× y
But A complete the work in (x + 8) days. Then, = 20
x+ y

9
But B complete the work in  x +  days. xy
2 ⇒ x + y = (1)
 20
 9 1 1
( x + 8)  x + 
 2 and x + y = 45
Using formula, =x 2 2
25
2x +
2 ⇒ x + y = 90 (2)

9 25 From Eqs. (1) and (2),
⇒ x2 + 8x + x + 36 = 2x2 + x xy = 1800
2 2
⇒ x2 = 36 ⇒ x = 6 days Now, go through options
xy = 60 × 30
39.
(c) Let, Madhu can complete the work in 2x days. Then,
Uma can complete the work in x days. Together they As Sudha is more efficient than Asha.
can complete the work in So, Asha can complete the work in 60 days.

Chapter 10.indd 22 6/9/2015 1:22:44 PM


Time, Work and Wages 10. 23

43.
(c) Let, number of computers required = N Total women and boys in terms of men
Tasks done by the computers = 15 + 5 = 20 = 10 men
6 h = 30 tasks 2 2 2
1 h = 5 tasks Let, number of men required = x
3 h = 15 tasks 15 × 12 × 8 × 2.25
Then, (x + 10) = = 18
So, 15N = 80 30 × 6
⇒ N = 5.33 ≈ 6 ⇒ x + 10 = 18
⇒ x = 8 men
44.
(c) Total work = 124 × 120 = 14880 men-days
Work completed in 64 days 3
47. × 60 min = 45 s
(c) 27 cogs turns 80 times in
4
2
= × 14880 men-days 27 cogs – 45 s = 80 times
3
27 × 80 × 8
= 9920 men-days \ 16 cogs – 8 s = = 24 times
16 × 45
Remaining work for 60 days
3
1 48.
(d) Speed of X =
Y
= × 14880 2
3 Distance before catching = D m
= 4960 men- days 3
D = D + 300
M1D1 M D 2
Q = 2 2
W1 W2 1
D = 300
2
120 × 64 M × 60 ⇒ D = 600 m
\ = 2
9920 4960 Total distance = 600 + 300 = 900 m
⇒ M2 = 64
49.
(b) Let, they make x pieces per day.
\ Workmen to be reduced
360 360
= 120 – 64 = 56 Then, − =1
x x+4

1
45.
(a) Anu’s 1 day work =  4 
10 ⇒ 360   =1
 x( x + 4) 
125 1 1 ⇒ x(x + 4) = 1440 = 36 × 40
Manu’s 1 day work = × =
100 10 8 ⇒ x = 36
360
160 1 1 \ Required number of days = =10 days
Sonu’s 1 day work = × = 36
100 8 5
50.
(b) 8 women = 6 men =12 boys
1 1 1 17 6
Total work = + + = \ 12 women = × 12 = 9 men
10 8 5 40 8
6
40 6 12 boys = × 12 = 6 men
Total days = = 2 days 12
17 17 \ M1 D1 T1 = M2 D2 T2
46.
(d) Men:Women:Boys = 15:24:36 \ 9 × 6 × 6 = (12 + 9 + 6) × D2 × 8
= 5:8:12 9×6×6 1
Convert women and boys in terms of men ⇒ D2 = = 1 days
27 × 8 2
8 women = 5 men
1 1 1
5 15 51.
(d) C’s 1 day’s work = − =
12 women = × 12 = men 12 20 30
8 2 Ratio of their efficiencies
12 boys = 5 men 1 1 1 1
= : : :
5 5 32 20 30 24
6 boys = ×6= men
12 2 = 15:24:16:20

Chapter 10.indd 23 6/9/2015 1:16:33 PM


10.24 Chapter 10

16 16 57.
(d) Let, the length of the path be x km.
\ C’s share = × 25 =
`
15 + 24 + 16 + 20 3 x 
Work done by 3 men in 2 days =  − 1 km
52. (b) 10 W × 15 = 6 M × 10  5 
15 5  x − 5
\ 1M = = W \ Work done by 1 man in 1 day = 
6 2  km
 30 
Now, 5 W + 6 M = (5 + 15) W = 20 W
10 × 15 = 20 × x day  2x 
And work done by 18 men in 1 day =  + 1 km
\ x = 7.5 days  5 
53.
(b) Ratio of efficiency = 2:3:6  2x + 5 
\ Work done by 1 man in 1 day =   km
1 1 1  90 
Ratio of time required = : : = 3 : 2 :1
2 3 6
 x − 5  2x − 5 
Let, A, B and C can complete the work in 3x, 2x and x Given,   =  
 30   90 
days respectively.
⇒ 3(x – 5) = 2x + 5
3x × 2 x × x
Then, = 10 ⇒ x = 20 km
3x × 2 x + 3x × x + 2 x × x
58.
(c) Q M1 × D1 × T1 × W2 = M2 × D2 × T2 × W1
6 x3 \ 2 × 12 × 8 × 12000 × 0.9
⇒ = 10
11x 2 = 3 × 6 × x × 9000 × 0.8
⇒ x ≈ 20 days
2 × 12 × 8 × 12000 × 0.9
54.
(a) 2 children = 1 man ⇒ x =
3 × 6 × 9000 × 0.8
\ (4 + 12) men can complete the work in 9 days.
= 16 h/day
9 × 16
\ 12 men can complete the work in
59.
(b) Q M2 × D1 × T1 × W2 = M2 × D2 × T2 × W1
12
= 12 days. \ 36 × 6 × 10 × 1200 = 10 × d2 × 8 × 1200

55.
(b) 15 M = 24 W = 36 B 36 × 6 × 10 × 1200
\ d2 =
5 5 10 × 8 × 1200
⇒ 1 W = M and 1 B = M
8 12 = 27 days
5 5
\ 12 W + 6 B = × 12 + × 6 = 10 M 60.
(d) Let, Bhavika and Rita working together can finish a
8 12 job in x days.
Now, m1 × d1 × t1 × w2 = m2 × d2 × t2 × w1 1
Then, in 1 day they will complete = work
Let, the number of additional men required be x. x
9 Bhavika working alone will do in 1 day
15 × 12 × 8 × = (10 + x) × 30 × 6
4 1
= work
15 × 3 × 8 × 9 ( x + 8)
⇒ (10 + x) =
30 × 6 Rita working alone will do in 1 day
⇒ x = 18 – 10 = 8 1
= work
56. (d)  9
 x + 
Men Women Children  2
18 × 63 = 9 × 189 =7 × 486
1 2 1
14 M
= =21 W 42 Children \ ( x + 8) + (2 x + 9) =
x
\ 1 M = 3 Children
(2 x + 9) + 2( x + 8) 1
1 W = 2 Children ⇒ =
( x + 8)(2 x + 9) x
(4M + 9W + 12C) = (12 + 18 + 12) Children
= 42 Children 4 x + 25 1
⇒ =
7 × 486 = 42 × x ( x + 8) (2 x + 9) x

⇒ x = 81 days ⇒ x = 6 days

Chapter 10.indd 24 6/9/2015 1:16:34 PM


Time, Work and Wages 10. 25

1 4×4 10 × 4 1 6 × 2 12 × 2 2
61.
(c) Work done by 1 man in 1 day = Then, = and + =
100 x y 3 x y 9
1
Work done by 1 woman in 1 day = Solving above equations, we get
120
x = 108, y = 216
Work done by 15 men and 6 women
Let, z women be added to complete the work in 3 days.
15 6
= +
100 120 6 × 3 3(12 + z ) 1 2 4
Then, + = 1−  + =
3 1 4 1 108 216 3 9 9
= + = = work
20 20 20 5 216 × 4
⇒ 36 + 36 + 3z = = 96
\ 15 men and 6 women will take 5 days to complete 9
the work. ⇒ 3z = 96 – 72 = 24
M1 D1 T1 M D T ⇒ z = 8
62.
(a) Q = 2 2 2
W1 W2 68.
(c) A B C

5 × 10 × 8 8 × D2 × 6 Efficiency 3 : 2 : 6
\ =
100 300 No. of days 2 : 3 : 1
5 × 10 × 8 × 30 1 ⇒ Number of days taken by A = 12,
⇒ D2 = = 2 days
100 × 8 × 6 2 Number of days taken by B = 18 and
   Number of days taken by C = 6
2m
63.
(c) One worker of category I can finish the work in h. 5
3 1 day’s work of (A + B) =
Two workers of category II can finish the work in 36
m 8
h. 1 day’s work of (B + C) =
2 36
2m m 9
× 2 1 day’s work of (C + A) =
So, required time = 3 2 = 2m = 2 m h 36
2m m 7m 7
+
3 2 Day 1 2 3 4 5 6
]
64. (a) A and B can complete the work in 30 and 40 days 5/36 8/36 9/36 5/36 8/36 1/36
respectively. 
35/36
1 1
So, ratio of their wages = : = 40:30
30 40

30 35
\ Share of B = × 2100 = `900 In 5 days total work done =
40 + 30 36
65.
(a) Ratio of efficiencies of the three persons 1 

24 34 24  24 24  Now, the rest of the work I  i.e.,  is done by AC
= : : − +  36 

6 8 3  6 8  Number of days taken by AC for the rest of the work
= 4:3:1
1/36 1
1 = =
\ Boy’s share = × 600 = `75 9/36 9
(4 + 3 + 1)
Therefore, total time taken to complete the work
66.
(c) Total time taken to finish the work
1 1
15(210 − 10) 15 × 200 = 5 + 5 days
=
= 10 + = 10 + 9 9
(15 + 15 × 2) 45
200 2 4 1
10 + = 76 days 69.
(c) Work done by A in 4 days = =
3 3 8 2

67.
(b) Let, one man takes x days to complete the work  1 1  −5
Net work done by (A + B) in 1 day  −  =
and one woman takes y days to complete the work  8 3  24
independently.

Chapter 10.indd 25 6/9/2015 1:16:35 PM


10.26 Chapter 10
1
72.
(c) 5 persons’ one day job =
10
−5 −5
Work done by (A + B) in 2 days = ×2= 1 1 11 11
24 12 5 persons’ 5 days job = × =
2 10 2 20
1 5 1
\ Work done in 6 days = − = 9 1
2 12 12 Remaining of the job is done by 10 persons. of
20 5
11
\ Remaining of the wall is built by A in 9
12 the job is done by 10 persons in 1 day. of the job is
20
8 × 11 88 22 1 1
= = = 7 days done by 10 persons in 2 days.
12 12 3 3 4
70.
(b) Let, the number of rows and columns be x and y. 73.
(b) Let, Sunil finish the job in x hrs
\ (y – 4)(x + 10) = xy x
Then, Ramesh will finish the job in hour
2
⇒ 10y – 4x = 40 (1) x
Given, =3⇒x=6
x−
and (y + 5) (x – 5) = xy 2
⇒ 5x – 5y = 25 \ Sunil finishes the job in 6 hrs and Ramesh does it in 3
⇒ x – y = 5 (2) hrs.
⇒ x = 15, y = 10 Work done by both of them in 1 hour
1 1 1
\ Total number of students = 15 × 10 = 150 = + =
6 3 2
1 \ They together could finish the job in 2 hrs.
71.
(b) (A + B)’s one day’s work =
12
1 1
(B + C)’s one day’s work = 74.
(c) (M + D)’s one day’s work =
16 4
Suppose C alone can finish the work in k days.
3
1 Remaining work =
⇒ C’s one day’s work = 4
k
1 1 k − 16 1
\ B’s one day’s work = − = M’s one day’s work =
16 k 16k 6
1 k − 16 k + 48 1 1 1
⇒ A’s one day s work = − = D’s one day’s work = − =
12 16k 48k 4 6 12
5(k + 48) 1
\    A’s 5 days’ work = \ th of the work is done by the daughter in one day.
48k 12
7(k − 16) 3
   B’s 7 days’ work = th of the work is done by the daughter in
16k 4
\ Remaining work to be done by C 12 × 3
= 9 days.
 5(k + 48) 7(k − 16)  11k + 48 4
= 1−  + =
 48k 16k  24k
75.
(a) 15W = 10M ⇒ 3W = 2M
which is finished by him in 13 days. 4 M = 6W
1
Now in one day, C does of the work. \ 5W + 4M = 11W
k
If 15W can do the job in 55 days, 11W will do the
13
In 13 days, C does of the work. 55 × 15
k same job in = 75 days.
11
13 11k + 48
\ = ⇒ k = 24 76.
(b)
k 24k

Chapter 10.indd 26 6/4/2015 11:06:36 AM


Time, Work and Wages 10. 27

81.
(d) Let, the worker worked for x hrs.
Now, according to the question,
56 56 3
6 :10  \
× 40 + (x – 40) × × = 88
 : 24:x 35 35 2
10 : 8 
56  3 
or, 6 × 10 × x = 10 × 8 × 24 ⇒
40 + x − 60  = 88
35  2 
10 × 8 × 24
\ x = = 32 men 3 88 × 35
6 × 10 ⇒
x = + 20 = 75
2 56
77. (a) Suppose x = original number of men in the group ⇒
x = 50
\ (x – 12) men did the job in 32 days.
\ 20x = 32(x – 12) 82.
(b) Work → A : B
i.e., x = 32 2x : x
1 1
6 1 Time → :
78. (d) 12 men’s 6 days’ work = = 2x x
18 3
2 1 1 1
Remaining of the work is done by 16 men. \  =
3 x 2x 14
⇒ x = 21 days
16 men can complete the work in
3 1
12 × 18 1 27 83.
(a) 400 × 9 × 10 × = 8 × 20 × ×x
= 13 = days 4 4
16 2 2
⇒ x = 675
2
i.e., 16 men can do of the work in 9 days
3 84.
(b) M × 10 × 20 = 20 × W × 15
2M = 2W
1
79.
(b) A’s 1 day’s work = 3 15
20 \ 5M = ×5=
2 2
1 From question,
B’s 1 day’s work =
25
1  15 
x   10 = 20 × 15
C’s 1 day’s work = 2 
30
A, B, C’s ratio of work 120
⇒ x =
1 1 1 7
= : :
20 25 30 1
⇒ x = 17 days
= 30:24:20 7
= 15:12:10
Sum of ratio = 15 + 12 + 10 = 37 1 1 1
85.
(a) 2( A  B  C ) =  
12 15 20
Hence, difference between A and B’s shares is
15 12 1
= ` × 2200 – ` × 2220 ⇒ A + B + C =
37 37 10
3
= × 2220 = `180 1 1 1
37 \ A can do alone =  =
1 10 15 30
80. (d) (A + B)’s 1 day’s work =
15 Hence A can alone complete the work in 30 days.
1 86.
(a) Let, original number of men = x
B’s 1 day’s work =
20 \ x × 20 = (x – 12) × 32
1 1 1 ⇒ 20x = 32x – 384
\ A’s 1 day’s work = − =
15 20 60 ⇒ 12x = 384
Hence, A alone can finish the work in 60 days. ⇒ x = 32

Chapter 10.indd 27 6/9/2015 1:16:37 PM


10.28 Chapter 10

40  15 6x 1 1 7
87. (b) = \  Workdone by A and B in 1 hr = + =
20 25 8 6 24
⇒ x = 125 24
Hence, A and B can complete the piece of work in
7
x x 2 3
i.e., in 3 days .
88.
(c)   =1
9 10 5 7
⇒ x = 6
93. (c) Time taken by X to complete the work = 4 × 10 =
89.
(b) Let, the total work = 12 units 40 days
Time taken by Y to complete the work =
A+ B B+C A+C
100
Out-put 12 12 12 40 × = 100 days
= 6= units 3=
units 5 units 40
perday 2 4 12 Time taken by Z to complete the work = 13 × 3 = 39
5 days.
Thus, Z will complete the work first.
Now, if we add (A + B), (B + C) and (A + C), we get
the work output for A, B and C each working for 2 94. (b) Time taken by A to complete the work = 7 × 9 = 63 h
days (6 + 3 + 5) = 14 units. Time taken by B to complete the work = 6 × 7 = 42 h
Thus, in 1 day A, B and C will do 7 units. Since, B Time taken by A and B together to complete the work
and C can do 3 units in 1 day, A alone in a day can 1 21 × 3 × 2 126
complete = (7 – 3) = 4 units. = = = h
1
+
1 3+ 2 5
Now in order to do the whole job, i.e., 12 units at the
rate of 4 units/day. 63 42
2
12 But if A and B work for 8 h daily, then number of
So, A will take = = 3 days. 5
4 days taken by them
90. (a) If the boy can do the work in x days, then 126 42 126
= ÷ = =3
1 1 1 1 1 1 1 1 1 1 5 5 42
+ + = ⇒ = − − ⇒ =
6 8 x 3 x 3 6 8 x 24
95. (c) Let, required population size be N. Then,
⇒ x = 24
 5  5 
\ Boy can do the work in 24 days. N 1 +  1 −  = 9975
1 1 1  100   100 
Therefore,the ratio of their share is : : i.e., 4:31
:  1  1 
6 8 24 N 1 +  1 −  = 9975
1  20   20 
Hence, boy’s share = × 600 = ` 75
8
⇒ N× ×
21 19
= 9975
20 20
91. (b) Workdone by A and B in 6days
⇒ N = 10000

 1 1
= 6× + 
 20 15  96. (b) 250 students have provisions for 35 days, ∴ 1
7 7 students has provision for 250 × 35 = 8750 days
= 6× =
60 10 250 students used the provisions for 5 days.
7 3 i.e., 1 student used the provisions for 250 × 5 = 1250
∴ Remaining work = 1 − = days
10 10
If C is able to do the work in x days, then Now, remaining provisions is sufficient for 1 student
4 4 3 for 8750 – 1250 = 7500 days
+ =
20 x 10 Now, (250 + 25) students used the remaining provision
4 3 1 3−2 1 for 10 days.
⇒ = − = =
x 10 5 10 10 i.e., 1 students used the provisions for 275 × 10 = 2750
⇒ x= 40days days
Again, remaining provisions is sufficient for 1 student
92. (d) Workdone by A in 1 hr = workdone by A, B and C in 1 hr for 7500 – 2750 = 4750
−workdone by B and C in 1 hr Therefore, (275 – 25) = 250 students can use the
3 1 1 4750
= − = provisions for = 19 days.
8 4 8 250

Chapter 10.indd 28 6/4/2015 11:06:38 AM


Time, Work and Wages 10. 29

97. (d) Since, 4 engines of latter type = 3 engines of former 11 19


type. Work remained = 1 − =
30 30
∴  8 engines of latter type = 6 engines of former type
Now, let A alone can finish the work in x days.
Engines Coal (Metric tons) Time (Hours)
We are given,
9 24 8
1 19
6 X 13 × 28 =
x 6 13 x 30
We have,= × 28 × 30 840
24 9 8 x= =
6 13 19 19
⇒ x = × × 24 = 26 metric tons 4
9 8 = 44 days.
19
98. (c) Let, required population size be N. Then,
 5  5  102. (c) Usha takes 25 days to complete a work.
N 1 +  1 −  = 9975 The time taken by Asha will be 25 − 25% of 25
 100   100 
25 × 25
 1  1  = 25 −
N 1 +  1 −  = 9975 100
 20   20  25 75
21 19 = 25 − = days
⇒ N× × = 9975 4 4
20 20 Let the total time taken to fiinish the work be x days
⇒ N = 10000
Then, we are given,
99. (b) 250 students have provisions for 35 days, ∴ 1 student 4 5 4 x + 15
x+ =1 ⇒ =1
has provision for 250 × 35 = 8750 days 75 25 75
250 students used the provisions for 5 days. ⇒ 4 x = 75 − 15
i.e., 1 student used the provisions for 250 × 5 = 1250
60
days ⇒x= = 15 days
Now, remaining provisions is sufficient for 1 student 4
for 8750 – 1250 = 7500 days ∴ Number of days for whichh Asha worked alone =
Now, (250 + 25) students used the remaining provision (15 − 5) = 10 days
for 10 days.
i.e., 1 students used the provisions for 275 × 10 = 2750 103. (c) We have, 15 men = 24 women = 36 boys
days 15 15
12 women = men and 6 boys = men
Again, remaining provisions is sufficient for 1 student 2 6
for 7500 – 2750 = 4750 ∴  12 women + 6 boys
Therefore, (275 - 25) = 250 students can use the  15 15 
4750 =  +  men
provisions for = 19 days  2 6
250
 45 + 15 
100. (d) Since, 4 engines of latter type = 3 engines of former =  men
 6 
type
= 10 men
∴  8 engines of latter type = 6 engines of former type
Semi-circular ∴ 1 work working 8 hrs per day is completed in 12
path days by 15 men.
1 9
∴ 2 = work working 6 hrs per day, to be completed
r r 4 4
Straight path 9 8 12
(via diameter) in 30 days will be completed by × × × 15 = 18 men
4 6 30
x 6 13 ∴  Number of extra men required = 18 – 10 = 8 men
We have,= ×
24 9 8
6 13 104. (b) It is given that two machines with 90% efficiency,
⇒ x = × × 24 = 26 metric tons. handles 9000 tonnes of coal in 8 days working 12 hrs
9 8 per day.
1
101. (c) Work done by A and B in 1 day working together =
30 ∴  3 machines with 80% efficiency, handling 12000
Now, they worked for 11 days. tonnes of coal in 6 days require
11 2 × 90 × 12000 × 8 × 12
∴ 11 day’s work = = 16 h per day
30 3 × 80 × 900 × 6

Chapter 10.indd 29 6/4/2015 11:06:40 AM


10.30 Chapter 10

105. (c) Let, A, B and C independently completes a work in x,


y and z days.
3 5−2 3 3
5 4 A= ⇒ A=
∴ = 2 20 2 20
x y
A 1
1 4 1 ⇒ =
⇒ = × (i) 2 20
x 5 y ∴ A = 10
37
Since, work is completed in 3 days by all of them. Hence, A alone would do that work in 10 days.
100
63
∴ work will be completed in 7 days by A and B. 108. (b) Let, cistern wil be empty in x h.
100
7 7 63 Then, due to leak time taken,
∴ + =
x y 100 1 1 1
− = ⇒
1 1 1
− =
1 1 9 (6 + 4) x 6 10 x 6
⇒ + =
x y 100 1 1 1 1 10 − 6 4
⇒ = − ⇒ = =
4 1 9 x 6 10 x 60 60
⇒ + = 1 4 1 1 1
5 y y 100 ⇒ = = ⇒ =
x 60 15 x 15
9 9
⇒ = ⇒ y = 20 ∴ x = 15 h
5 y 100
∴ x = 25 109. (a) 1M = 2W
3 3 3 37 (8M + 4W) (6 days − 2 days) = (4M + 8W) × x days
Also + + =
25 20 z 100 (8 × 2W + 4W) × (6 − 2) days
3 37 − 12 − 15 1
⇒ = = = (4 × 2W + 8W) × x days
z 100 10
(16 + 4)W × 4 days = 16W × x days
∴ z = 30 days 20 × 4
Hence, the fastest worker will complete the work in ∴ x= = 5 days [M 1D1 =M 2 D 2 ]
16
30 days.
106. (d) Let, there be x engineers in the beginning. 110. (d) A + B 1 day’s work =
1
 (i)
Now, 20
Days Number of Engineers 1
30 X+5
B + C 1 day’s work = (ii)
30
40 X 1
C + A 1 day’s work = (iii)
⇒ 40 : 30 = ( x + 5) : x 40
Adding Eqs. (1), (2) and (3), we get
⇒ 40 × x = 30 × ( x + 5)
⇒ 40 x = 30 x + 150 1 1 1
2( A + B + C ) = + +
⇒ 10 x = 150 20 30 40
6+4+3
∴ x = 15 ⇒ 2( A + B + C ) =
120
13
1 ⇒ A + B + C 1 day work toggether =
107. (a) (A + B)’s one day work = 240
5
1 (1) A’ Alone 1 day’s work
⇒ ( A + B) = =(A + B + C) 12 day’s work
5
We are given, = (A + B+ C) 12 day’s work − (B + C) 1 day’s work
1 1 13 1 13 − 8 5
2 A + B = (2) A= − ⇒ =
2 4 240 30 240 240
1
On multiplying Eq. (1) by we get 240
2 Number of days taken by A = days
5
1 1 1
⋅ A+ ⋅B = (3) C’ Alone 1 day’s work
2 2 10
On subtracting Eq. (3) from Eq. (2), we get = (A + B + C) 12 day’s work - (A + B) 1’ day’s work

Chapter 10.indd 30 6/4/2015 11:06:43 AM


Time, Work and Wages 10. 31

13 1 13 − 12 1 111. (c) Let, x people were supposed to be working originally.


⇒ − ⇒ = Therefore, 24x = 33 (x − 9) or 24x = 32x – 288 or x =
240 20 240 240
36 people
Number of day’s taken by C =
240
days 112. (a) 50 men complete 0.4 work in 25 days.
1 Applying the work rule, m1 × d1 × w2 = m2 × d2 × w1
We have, 50 × 25 × 0.6 = m2 × 25 × 0.4
240 240
Required Ratio : 50 × 25 × 0.6
5 1 or m2 = = 75 men
25 × 0.4
⇒ 1: 5
Number of additional men required = (75 − 50) = 25

Difficulty Level-2

1. (b) Let, the number of workers be x. 2 1


Then, total work = 9x man days. = 14 + × 12 + × 12
3 2
As per the given condition, number of workers starting = 28 men
with the first day will be
⇒ 28 × 24 = M × 14 ⇒ M = 48
x, x – 2, x – 1, x – 3, x – 2, x – 4, x – 3 ... and so on till
18 days. Therefore, additional labour = 48 – 28 = 20 men
So, separating the odd and even terms and adding
5.
(b) A man can finish a work in 6 days.
(9x – 36) + (9x – 54) = 9x
Work done by the man, woman and child in one day is
⇒ x = 10
1 1 1
2. (d) A and B together complete the work in , , respectively.
6 10 12
1
= 24 days
1 1 ∴ Work done in 1 day if each works for one-third of

40 60 1 1 1 1 7
the day =     =
Work completed in 20 days (4 days before scheduled 3  6 10 12  60
completion) 60 4
∴ Number of days required = = 8
5
= 
1 1 7 7
 × 20 = th
 40 60  6 56
In 8 days amount of work done =
1 60
To complete the remaining work, 56 1
6 ∴ Work left = 1  =
1 60 15
1 1
B will take 6 = 10 days In day a man will finish th of the work.
1 3 18
60 1 1 1
∴ Work left =  =
∴ Number of days more to complete the work 15 18 90
= 30 – 24 = 6 days. ∴ Woman would be working at that time.
3. (c) Lal Singh can eat 12.5 laddoos in 1 hour and Pal Singh 3 3 3
can eat 7 laddoos in 1 hr 6.
(b) Men can build wall in days.
2 2 2
⇒ Together they eat 19.5 laddoos in 1 hr
⇒ 507 laddoos will be eaten in 26 hrs. 3
Men can build 6 walls in 6 days.
2
4.
(a) Let Men = M, Women = W, Youngster = Y 6 Men can build 24 walls in 6 days.
Then, 2M = 3W = 4Y 1 1
7.
(d) Each engineer does = th of the process in 1
2 1 58 40
⇒ W = M and Y = M hour.
3 2 30
Till 4.00 pm, they have done of the process.
Given 14M + 12W + 12Y 40

Chapter 10.indd 31 6/4/2015 11:06:45 AM


10.32 Chapter 10

4 3 2 1 12. (a) Suppose the original number of men was x.


In next each hour,    . \ x men could do the job in 20 days and (x – 12) men
40 40 40 40
could do the same job in 32 days.
30 4 3 2 1
Hence, by 8.00 pm + + + + =1 \ 20x = 32 (x – 12)
40 40 40 40 40 ⇒ 12x = 32 × 12
Hence, they finish the work by 8.00 pm. ⇒ x = 32.
x 1 1 1 73
8.
(b) 1 boy consumes calories in y days. 13.
(a) Three day’s work = + + =
4 7 8 6 168
z y 73
1 girl consumes calories in days. Six day’s work =
6 2 84
x 1
∴ In 1 day, 1 boy consumes calories. Seventh day work = , done by Ram
4y 7
2z 73 1 85
∴ In 1 day, 1 girl consumes calories. Since = + > 1, therefore Ram was working on
6y 84 7 84
120 x 30x the last day.
In 12 days, 10 boys consume =
4y y
32 40 16 40
120 z 40x 14. (c) In one hour, + = +8 = of the pages
In 12 days, 10 girls consume = 6 5 3 3
3y y are typed.
30x 40z 10
\ To type 110 pages, it will take
∴ Total = + = (3x + 4z) calories.
y y y 3 33
× 110 =
40 4
9. (d) To complete the work individually, let A, B and C,
1
respectively, take a, b, and c days. = 8 hrs.
4
5 2 3 1 1 1  1 1 3
∴    1 ,   and 2    
a b c b c 6  b c a 3
15. (c) Book read on the first day =
⇒ a = 9 8
2 3 5 1 1 1 5
Now,  = 1 – and  = Remaining book to be read =
b c 9 b c 6 8
2 3 4 1 1 1 4 5 1
⇒  = and  = Book read on the second day =× =
b c 9 b c 6 5 8 2
1 1 5 1 1
⇒ c = 9 and = ⇒ b = 18 Remaining book to be read = − =
b 18 8 2 8
∴ A, B and C, respectively, can complete the work in 1
\ of the book contains 30 pages.
9, 18 and 9 days. 8
⇒ The book contains 240 pages.
10. (c) 4 examiners can examine certain number of Answer
Books in 40 hrs. 16.
(d) 20 men were employed to complete the work in say N
Suppose for x hrs a day, 2 examiners examine the days. Therefore, the estimated work is 20N man days.
same number of Answer Books in 10 days. N
⇒ 4 examiners examine the same number of answer Work completed in days is 5N man days.
3
books in 5x hrs.
Remaining time according to revised schedule
\ 5x = 40 ⇒ x = 8.
3N N 9N – 4N 5N
= – = =
11. (d) 15 men can do a piece of work in 168 hrs. 4 3 12 12
3 women = 2 men Remaining work = 15N man days.
⇒ 21 women = 14 men 20 Men in N/3 days do 5N mandays of work.
Let, 14 men can do the same piece of work in x days Number of men needed in 5N/12 days, to do 15N
of 6 hrs each, i.e., 14 men can do the same piece of
work in 6x hrs.  N   12   15 
mandays of work is (20)    = 48
\ 6x × 14 = 15 × 168  3   5 N   5 
⇒ x = 30. ⇒ 28 additional men are needed.

Chapter 10.indd 32 6/4/2015 11:06:46 AM


Time, Work and Wages 10. 33

17. (c) (A + B + C)’s one day’s work K


mins was spent cutting 6x potatoes
1 1 1 5 1 3
= + + = =
30 30 60 60 12 K
and mins was spent cutting 5x beetroots.
\ Work can be finished in 12 days. 3
During the next K mins, the time distribution was in
18.
(d) Imran can stitch 10/3 shirts/hr. 1 1 1
the ratio : : i.e., 6:10:12 = 3:5:6
Irfan can stitch 10/4 shirts/hr. 10 6 5
Imran can stitch 3 pants/hr.
Irfan can stitch 4 pants/hr. 3 5 6
Hence K, K and K
Now the work will be done fastest when Irfan stitches 14 14 14
the pants whereas Imran stitches the shirts. But Irfan 3 
is finished with 200 pants in 50 hrs, while Imran has  K  90 x
14
stitched only (10/3) (50) = 500/3 shirts by then. So the ⇒ Carrots cut  10 x = 3 carrots
K 14
remaining 100/3 shirts are stitched by both in  
3
 
100  1  40 5 90 x 90 x
  = =5 Similarly potatoes and beetroots are cut.
10
3    10 7 7 14 14
 3 4 Total number of vegetables cut in the first 2K mins
= 5 hrs 43 mins
5  90 x 
= 10x + 6x + 5x +   13 = 282
∴ Total time = 50 + 5  14 
7
= 55 hrs 43 mins. 90 x
⇒ x = 7 and = 45
14
19.
(d) Mohan’s 20 days work = Bhim’s 30 days work
30 23. (a) 35 men do the rest of the job in 12 days (37 – 25 = 12)
⇒ entire work can be finished by Bhim in 25  =
20 12  35
37.5 days. ∴ 30 men can do the rest of the job in
30
Bhim left after 15 days = 14 days.
⇒ Work of 22.5 days was remaining. Thus the work would have been finished in (25 + 14)
30 = 39 days.
Mohan can complete it in  22.5 = 15 days.
20 That is, (39 – 38) = 1 day after the scheduled time.
Questions 20 and 21
Let, fencing the entire wall be equivalent to 108 units 24.
(c) 15m = 24w = 36b
of work. x men must be associated.
⇒ Rakesh does 9 units per hour, Shiv does 86 units
∴ (x)m + 12w + 6b =  x  18  6 boys.
36
per hour and Ramu does 4 units per hour.
 15 
In 2 hrs (9 + 6 + 4) × 2 = 38 units were done.
Remaining = 70 units. 2
25.
(c) Work done =
In the last 5 hrs, Shiv has done = 5 × 6 = 30 units. 3
The other 40 units were done by Shiv and Vijay 1 2
Remaining work = , which is half of
40 3 3
together in = 4 hrs.
64 ∴
1
 × (20 × 32) = 8 × x
2
20.
(a) Total number of hrs taken = 2 + 5 + 4 = 11 hrs. ⇒ x = 40 men
21. (a) The pair which does the work in least time, will be Therefore, 20 more men were required.
able to fence the wall quicker than any other pair 1
26.
(c) Work done =
⇒ Rakesh and Shiv 5

22. (c) Given the ratio of working rates on the three types of 4
Remaining work =
vegetables as 120:72:60 i.e., 10:6:5. 5
Suppose, he cuts N vegetables in the first K mins ∴ 4 (20 × 75) = 40 × x
K x = 150
⇒ mins was spent cutting say 10x carrots.
3 Therefore 75 men should be increased.

Chapter 10.indd 33 6/4/2015 11:06:46 AM


10.34 I Chapter 10

27. (c) 1996 is a leap year. Hence Raja takes 7 days to \ p spindles can produce 20000 metres cloth in
complete a work. He completes 1 unit work per day, r r  2000
hence work completed in 7 days = 7 units. (Because × 20000 = hours
pq 60  pq
he does not have any holiday). Now T will have two
holidays in a week i.e., Tuesday and Thursday and S
will not have any holiday. Hence their arrangement 32. (c) Let, Ramesh read x pages/none of engineering Math
of work will follow the following pattern depending and y pages/none of engineering drawing.
upon which day 25 Feb 1996 falls on A/Q in a day he can read
Sun Mon Tues Wed Thurs Fri Sat 80 100 50 250
Total time = + = +
2 units 2 units 1 unit 2 units 1 unit 2 units 2 units x y x y
Hence both of them will take either 4 or 5 days to 30 150
⇒ =
complete the same work. x y
28. (c) Raja has worked 38 days. (Feb. = 5 days, March = ⇒ y = 5x
31 days, April = 2 days), Therefore, he completes Since, he has to read only engineering maths in a day
38 units work in 38 days. In a week T takes holidays putting y = 5x in above equation, we get
on Tuesday and Thursday, while S takes holiday on 50 250 100
Saturday and Sunday. Hence, their work arrangement + =
x 5x x
will follow the following pattern:
He can read 100 pages of maths in total time.
Sun Mon Tues Wed Thurs Fri Sat
33. (c) Suppose A can finish the work in x days and B can
1 unit 2 units 1 unit 2 units 1 unit 2 units 1 unit finish it in y days.
Therefore, in a week they work 10 units work. Hence, 2 9 3 6
in 3 weeks, they would complete 30 units work. Now \ + = 1 and + = 1
x y x y
8 units work can be completed either on 5th or 6th day
depending on which day the work begins. Hence, total On solving, we get x = 5 and y = 15
number of days taken by T and S to complete the job 34.
(c) In M mins the person makes J jobs.
= (21 + 5) = 26 days or (21 + 6) = 27 days.
M
29. (d) Ram In h the person makes J jobs
60
5km1h
• J  60
A_------====~B
In 1 hr the person makes jobs
M
Sbyam \0 kmIh

J  60 2
10 In h the person makes  jobs
In 1 hour Ram is at B, in that time Shyam covers M 3
4
= 2.5 km 2 40J
In h the person makes = jobs.
Remaining distance = 2.5 km 3 M
2.5 35.
(a) As per the given information, the ratio of the
Time =  60 = 10 mins
5  10 efficiencies of Dr. Gupta, Dr. Sharma and Dr. Singh
Therefore, they meet first time at 10.30 am. are = 42:30:25. Hence, the ratio of time taken by Dr.
Gupta and Dr. Sharma is 5:7. As, Dr. Gupta takes 10
30. (b) At the time when Shyam overtakes Ram, let Ram days less than Dr. Sharma, time taken by them will
travels for t mins, Shyam till that time travel for t – 45 be 25 days and 35 days respectively. Hence, the time
mins and both travel same distance. taken by Dr. Singh will be 42 days.
⇒ 5 × t = 10 (t – 45) 2
Part of the work completed by Dr. Gupta =
⇒ t = 90 mins 5
Hence, Shyam overtake Ram at 10.30 am. 3
Part of the work completed by Dr. Sharma =
31. (d) One spindle can produce 9 metres cloth in r mins 7
r 6
One spindle can produce 1 metres cloth in min The remaining work, i.e., will be completed by
q 35
Dr. Singh in
r
On spindle can produce 20000 metres cloth in × 42  6
q = 7.2 days.
20000 min. 35

Chapter 10.indd 34 6/4/2015 11:06:47 AM


Time, Work and Wages I 10. 35
36.
(d) 2 weeks work of Aditya, Vedus and Yuvraj ⇒ 3x2 + 7x – 6 = 0
 1 1 1  26 ⇒ 3x2 + 9x – 2x – 6 = 0
= 2      ⇒ 3x (x + 3) – 2 (x + 3) = 0
 6 9 12  36
⇒ (x + 3) (3x – 2) = 0
10
\ work left = 2
36 \ x= or, x = – 3
3
Let, Vedus works for x weeks with Yuvraj after Aditya
2
As x cannot be negative, x =
1 1  7x 3
\ x weeks’s work of both = x    , the
 9 12  36
remaining work is done by Yuvraj in 1 week. 40.
(d) Let, the initial length of the two candles = L
1 The first one burns in 4 hrs whereas the second one
\ One week’s work of Yuvraj = burns in 3 h.
12
Lt
26 7 x 1 Length of the first candle after t hr = L −
\   =1 4
36 36 12
⇒ 29 + 7x = 36 ⇒ x = l Lt
Length of the second candle after t hr = L −
\ Vedus worked for 1 week with Yuvraj. 3
Lt  Lt 
37.
(b) 16 men and 16 women together in 12 days did the So, L− = 2  L − 
4  3
1 3
work =  12 work 2t t
16 4 − = 1
3 4
3 1
Remaining work = 1   12 2
4 4 ⇒ t = = 2 h
5 5
16 men and 16 women in two days will do the work
1 1
=  2  work 41.
(b) The first machine addresses 500 envelopes in 8 min.
16 8
500
1 1 1 In 1 min, it will address = envelopes
Men needed to do remaining work =   work 8
4 8 8
24 × 16 Let, the second machine address 500 envelopes in x
Required men = = 24. min 500
2×8
500
In 1 mm, it will address envelopes
38.
(b) Total working hrs = 290 × 8 = 2320 x
500 Thus, both the machines together in 1 min will address
Wages for one hour work = `
8 500 500
Let, he was absent for x hrs. + envelopes
8 x
500
\ His wages = (2320 – x) – 50 × x Given, both the machines together will address 500
8 envelopes in 2 min.
1160000  900 x 500
⇒   132400 = Thus, in 1 min, both together will address
8 2
⇒   x = 112 h envelopes
39.
(a) P takes (x + 6) hrs to do the job alone. 500 500 500 1 1 1
+ = ⇒ + = .
Q takes (x + 1) hrs to do the job alone. 8 x 2 8 x 2
R takes (2 x) hrs to do the job alone.
The three take x hrs to do the job, when working 42.
(c) By option we can see only option (c) or (d) will be our
together. choice.
1 1 1 1 So, first we check option (c)
\   =
x  6 x  1 2x x 24 × 12 × 8 24 × 12 × 8
=
2x  7 1 24 × 12 + 12 × 8 + 24 × 8 12 (24 + 8 + 16)
⇒ =
x2  7 x  6 2x 24 × 8
= = 4 days.
⇒ 4x2 + 14x = x2 + 7x + 6 48

Chapter 10.indd 35 6/4/2015 11:06:48 AM


10.36 Chapter 10

43.
(a, b, c, d) 20 × 10 M = 15 × 20 W 12
\ 2 M = 3 M × 100 = 30 days.
40
\ 10 M = 15 M
Working alone, we know Ram takes 30 days to complete
⇒ 20 × l5 = (15 W + 10 W) × x = 25 × x the entire work. Let, us assume that Ravi takes x days
⇒ x = 12 to complete the entire work, if he works alone. And
Option (a), total wage of men together, they complete the entire work in 20 days.
= 12 × 10 × 50 Therefore,
= `6000 1 1 1
+ =
30 x 20
Total wage for women = 12 × 10 × 45 = `5400
1 1 1
Total = `11400 ⇒ = −
x 20 30
Hence, option (a) is correct. 1 1
⇒ =
Option (b), total wage for men x 60
= 12 × 10 × 45 = `5400 ⇒ x = 60
Total wage for women = 12 × 10 × 40 = `4800 Therefore, Ravi will take 60 days to complete the
Total = `10200 work.
Hence, option (b) is correct. Hence, Ram is 100% more efficient than Ravi.
46. (d)  Let, A and B work for m days and C for n days to
Option (c), 12 × 10 × 40 + 12 × 10 × 40 = `9600
complete the work.
Hence, (c) is correct.
Option (d), 5 × 20 × 40 + 5 × 30 × 35 = `9250 Therefore, m + m + n = 1 (1)
15 20 30
Hence, option (d) is correct.
Out of the total of `18000, B gets `6000 more than C.
44. (c) Let, the number of holidays in the vacation be x.
m n 6000
Then, they practiced yoga on (x − 24) mornings and i.e., − = (2)
played tennis on (x − 12) evenings. 20 30 18000
As they did not do both the activities together on any On adding (1) and (2), we get
single day, m 2m 4
+ =
 Days on which they had any activity = Number 15 20 5
of days they practiced Yoga+ Number of days they ⇒ m = 8.

played tennis.
47. (c) Let, the number of men be 100.
i.e., 22 = x- 24 + x- 12
Then, Men × Time = Work
or,  22 +24 +12 = 2x
100 × 1 = 100 unit
or,  58 = 2x
Amount of work increased by 50%
Hence, x = 29.
∴  New Work = 150 unit.
45. (d) Ram completes 40 % of the work in 12 days, so the
remaining 60 % of the work has to be completed by As the planned time remains same, i.e., 1, so the
Ram and Ravi. They have taken 12 days to complete number of men required will be 150, i.e., 50 more
60 % of the work. workers.But the new workers are 25% efficient, i.e.,
5
Therefore, Ram and Ravi, working together, would times efficient as existing workers.
have completed the work in 4
12 50
× 100 = 20 days. ∴  Actual number of workers = = 40 men
5/4
60
As Ram completes 40 % of the work in 12 days, he ∴  Required percent =
40
× 100 = 40%
would have completed the work in 100

Chapter 10.indd 36 6/4/2015 11:06:49 AM


CHAPTER

Pipes and Cisterns 11


INTRODUCTION
Pipes are connected to a tank or cistern and are used to fill terms of filling or emptying a cistern and the time taken is
or empty the tank. the time taken by a pipe or leak (crack) to fill or empty a
cistern, respectively.
Inlet A pipe connected with a tank or a cistern that fills it is
Generally, the time taken to fill a cistern is taken as
known as inlet.
positive and the time taken to empty a cistern is taken as
Outlet A pipe connected with a tank or cistern emptying it negative. The amount of work done, i.e., filling or emptying
is known as outlet. a cistern is generally taken as unity, unless otherwise
Pipes and Cistern problems are similar to those on time specified.
and work. The only difference here is the work done is in

SOME BASIC FORMULAE

Solution: We have, 1/40 part of the cistern is emptied in


1. If an inlet can completely fill the empty tank in X = 1 min.
1 ∴ 3/4 part of the cistern is emptied in
hrs, the part of the tank filled in 1 hr = .
X 3
2. If an outlet can empty the full tank in Y hrs, the = 40 × = 30 minutes
4
1 Illustration 4 A pipe can empty a cistern in 12 hrs. Find the
part of the tank emptied in 1 hr = .
Y part of the cistern emptied in 4 hrs
3. If both inlet and outlet are open, net part of the Solution: We have, part of the cistern emptied in 1 hr
1 1 = 1/12,
tank filled in 1 hr = − . 1 1
X Y ∴ part of the cistern emptied in 4 hrs = ×4 =
12 3
Illustration 5 A tap can fill a cistern is 8 hrs and another can
Illustration 1 A pipe can fill a tank in 5 hrs. Find the part of
empty it in 16 hrs. If both the taps are opened simultaneously,
tank filled in one hour
find the time (in hrs) to fill the cistern
Solution: The part of the tank filled in 1 hr = 1/5.
Solution: Here X = 8 and Y = 16
Illustration 2 A pipe can fill a tank in 28 minutes. Find the
∴ Part of the cistern filled in 1 hr
time in which 1/7 part of the tank will be filled
1 1
Solution: We have, 1/28 part of the tank is filled in 1 min. = −
X Y
∴ 1/7 part of the tank is filled in 28/7 minutes
1 1 1
= 4 minutes. = − =
Illustration 3 A pipe can empty a cistern in 40 minutes. 8 16 16
Find the time in which 3/4 part of the cistern will be emptied ∴ Total time taken to fill the cistern = 16 hrs

Chapter 11.indd 1 6/5/2015 1:12:01 AM


11.2 Chapter 11

SOME USEFUL SHORT-CUT METHODS

1
Part filled by C alone in 1 hr =
1. Two pipes A and B can fill (or empty) a cistern Z
in X and Y hrs, respectively, while working alone. All the three pipes are opened.
If both the pipes are opened together, then the ∴ Part filled in 1 hr
time taken to fill (or empty) the cistern is given by 1 1 1 XY + YZ + ZX
= + + =
 XY  X Y Z XYZ
  hrs. XYZ
 X +Y  ∴ Cistern will be filled in hrs
XY + YZ + ZX
Explanation
Note:
Part of the cistern filled (or emptied) by pipe A alone
1 We can generate more formulae like above by replacing
in 1 hr = .
X negative sign wherever a pipe starts emptying a cistern
Part of the cistern filled (or emptied) by pipe B alone instead of the standard positive sign.
1 Illustration 7 Two pipes A and B can separately fill a cistern
in 1 hr = .
Y is 8 hrs and 12 hrs, respectively, while a third pipe C can
∴ Part filled (or emptied) by (A + B) in 1 hr empty it in 6 hrs. In what time will the cistern be full, if all
the pipes are opened together?
1 1 X +Y
= + = . Solution: Here X = 8, Y = 12 and Z = – 6
X Y XY
∴ The cistern will be full in
Therefore, both the pipes A and B together will fill (or
 8 × 12 × −6 
 XY  =   hrs
empty) the cistern in   hrs.  8 × 12 − 12 × 6 − 6 × 8 
 X +Y 
Illustration 6 Two pipes A and B can fill a cistern in 20  576 
=   hrs or, 24 hrs.
and 30 minutes, respectively. If both the pipes are opened  24 
simultaneously, how long will it take to fill the cistern?
Solution: Here X = 20 and Y = 30 3. Two pipes A and B can fill a cistern in X hrs and
Y hrs, respectively. There is also an outlet C. If
∴ Part of the cistern filled by (A + B) in 1 min
all the three pipes are opened together, the tank is
1 1 1 1 5 1 full in Z hrs. The time taken by C to empty the full
=+ = + = =
X Y 20 30 60 12 tank is given by
∴ Both the pipes A and B together will fill the cistern
 XYZ 
in 12 minutes.   hrs.
 XZ + YZ − XY 
2. Three pipes A, B and C can fill a cistern in X, Y
and Z hrs, respectively, while working alone. If all
the three pipes are opened together, the time taken Explanation
to fill the cistern is given by Part of the tank emptied by C in 1 hr
 X ×Y × Z   1 1 1
=  + − 
  hrs. X Y Z
 XY +YZ +ZX 
 XYZ 
Explanation ∴ C can empty the full tank in   hrs.
 XZ +YZ − XY 
1
Part of the cistern filled by A alone in 1 hr = Illustration 8 Two taps A and B can fill a cistern in 30
X
1 minutes and 60 minutes, respectively. There is third exhaust
Part filled by B alone in 1 hr =
Y tap C at the bottom of the tank. If all taps are opened at the

Chapter 11.indd 2 6/9/2015 1:33:34 PM


Pipes and Cisterns 11.3

same time, the cistern will be full in 45 minutes. In what (b) If A can fi ll a cistern in y hrs, then the time in
time can exhaust tap C empty the cistern when full? which the cistern will be full, if both the fi ll
Solution: Here X = 30, Y = 60 and Z = 45  k 
pipes are opened together, is   y hrs.
\ Exhaust tap C can empty the cistern in  k +1
 XYZ 
=   minutes
 XZ +YZ − XY  Illustration 11 One fi ll pipe A is 10 times faster than second
 30 × 60 × 45  fi ll pipe B. If B can fi ll a cistern in 55 minutes, then fi nd
=   minutes the time when the cistern will be full if both fi ll pipes are
 30 × 45 × 60 × 45 − 30 × 60 
opened together
= 36 minutes.
Solution: Here k = 10 and x = 55.
4. A tank takes X hrs to be fi lled by a pipe. But due \ Cistern will be full in
to a leak, it is fi lled in Y hrs. The amount of time
 x 
in which the leak can empty the full tank =   minutes
 k +1
 XY 
=   hrs.  55 
Y − X  =   minutes or 5 minutes.
 10 + 1 
Illustration 9 A pipe can fi ll a tank in 12 hrs. Due to leakage
Illustration 12 One fi ll pipe A is 4 times faster than second
in the bottom, it is fi lled in 24 hrs. If the tank is full, how
fi ll pipe B. If A can fi ll a cistern in 15 minutes, then fi nd
much time will the leak take to empty it?
the time when the cistern will be full if both fi ll pipes are
Solution: Here X = 12 and Y = 24. opened together
\ The time taken by the leak to empty the full tank
Solution: Here k = 4 and y = 15
 XY   12 × 24  \ Cistern will be full in
=   hrs =   hrs or 24 hrs:
Y − X   24 − 12 
 k   4 
=   y minutes =   15 minutes
5. A cistern has a leak which can empty it in X hrs.  k +1  4 +1
A pipe which admits Y litres of water per hour = 12 minutes.
into the cistern is turned on and now the cistern is
emptied in Z hrs. The capacity of the cistern is
7. If one fi ll pipe A is k times faster and takes x
 XYZ  minutes less time than the other fi ll pipe B, then
  litres.
Z−X  (a) the time taken to fi ll a cistern, if both the pipes
 kx 
Illustration 10 A leak in the bottom of a tank can empty the
A leak in the bottom of a tank can empty the are opened together is  2  minutes.
full tank in 6 hrs. An inlet pipe fi lls water at the rate of 4 litres  (k − 1) 
per minute. When the tank is full, the inlet is opened and due to  x 
(b) A will fi ll the cistern in   minutes.
leak, the tank is empty in 8 hrs. Find the capacity of the tank.  k −1 
Solution: Here X = 6, Y = 4 × 60 = 240 and Z = 8  kx 
(c) B will fi ll the cistern in   minutes.
\ The capacity of the tank is  k −1 
 XYZ   6 × 240 × 8 
=   litres =   litres
Z−X   8−6  Illustration 13 One fi ll pipe A is 5 times faster than second
fi ll pipe B and takes 32 minutes less than the fi ll pipe B. When
= 5760 litres.
will the cistern be full if both fi ll pipes are opened together?
6. One fi ll pipe A is k times faster than the other fi ll Solution: Here k = 5 and x = 32
pipe B.
\ Cistern will be full in
(a) If B can fi ll a cistern in x hrs, then the time in
which the cistern will be full, if both the fi ll kx 5 × 32
= minutes = minutes
 x  (k − 1) 2
(5 − 1) 2
pipes are opened together, is   hrs.
 k +1 = 10 minutes.

Chapter 11.indd 3 6/5/2015 1:02:41 AM


11.4 Chapter 11

Practice Exercises

Difficulty Level-1
(Based on Memory)

1. Three-fourths of a tank is full of water. If 5 litres are added 8. A tin of oil was four-fifths full. When six bottles of oil
to it, then four-fifths of the tank becomes full. What is the were taken out and four bottles of oil were poured into it,
capacity of the tank? it was three-fourths full. How many bottles of oil can it
(a) 75 litres (b) 80 litres contain?
(c) 100 litres (d) 120 litres (a) 20 (b) 10
[Based on MAT, 2004] (c) 30 (d) 40
2. A vessel is fully filled with a special liquid. Four litres [Based on MAT (Feb), 2008]
of liquid is drawn out of this vessel and is replaced 9. A certain tank can be filled by pipes A and B separately in
with water. If the ratio of the special liquid to the water 4 and 5 minutes respectively, whereas pipe C can empty in
becomes 1:2, then what is the capacity of the vessel? 3 minutes. How long will it take to fill or empty the four-
(a) 8 litres (b) 10 litres fifths of the full tank, if all the three pipes start together?
(c) 12 litres (d) 14 litres 5 2
(a) 2 minutes to fill (c) 6 minutes to fill
[Based on MAT, 2002] 7 7
3. A pipe can fill a cistern in 6 hrs. Due to a leak in its bottom, 6 5
(c) 6 minutes to fill (d) 1 minutes to fill
it is filled in 7 hrs. When the cistern is full, in how much 7 7
time will it be emptied by the leak? [Based on MAT (Feb), 2011]
(a) 42 hrs (b) 40 hrs 10. When the waste pipe is closed, two taps can separately
(c) 43 hrs (d) 45 hrs fill a cistern in 10 and 12 minutes respectively. When the
[Based on MAT, 2002] waste pipe is opened they together fill it in 15 minutes.
4. 12 buckets of water fill a tank when the capacity of each How long does it take the waste pipe to empty the cistern,
bucket is 13.5 litres. How many buckets will be needed to when the taps are closed?
fill the same tank, if the capacity of each bucket is 9 litres? (a) 7 minutes 10 seconds (c) 8 minutes 34 seconds
(a) 8 (b) 16 (c) 12 minutes (d) 10 minutes
[Based on MAT (Feb), 2011]
(c) 15 (d) 18
11. A tank is connected with four pipes A, B, C and D of
5. A leak in the bottom of a tank can empty the full tank in 8 which two are filling the tank and other two are emptying
hrs. An inlet pipe fills water at the rate of 6 litres a minute. it. The time taken by A, B, C and D to finish their jobs are
When the tank is full, the inlet is opened and due to the 10 hrs, 15 hrs, 20 hrs and 30 hrs, respectively. All four
leak, the tank is empty in 12 hrs. How many litres does the pipes are opened. When the tank was empty, it took 12 hrs
cistern hold? to fill it completely. Which two are the outlet pipes?
(a) 7580 (b) 7960 (a) A and B (b) C and D
(c) 8290 (d) 8640 (c) A and C (d) B and D
[Based on MAT, 2005]
12. Two pipes can fill a tank in 8 hrs and 12 hrs, respectively
6. Pipes A and B running together can fill a cistern in whereas an escape pipe can empty it in 6 hrs. If the three
6 minutes. If B takes 5 minutes more than A to fill the pipes are opened at 1 pm, 2 pm and 3 pm respectively, at
cistern, then the time in which A and B will fill the cistern what time will the tank be filled?
separately will be respectively: (a) 8 am (c) 7 am
(a) 15 minutes, 20 minutes (b) 15 minutes, 10 minutes (c) 5 am (d) 7.30 am
(c) 10 minutes, 15 minutes (d) 25 minutes, 20 minutes [Based on MAT (Feb), 2011]
 [Based on MAT, 2001] 13. Two pipes P and Q can fill a cistern in 3 and 6 minutes,
7. A tap can fill a tank in 48 minutes, whereas another tap respectively, while an empty pipe R can empty the cistern
can empty it in 2 hrs. If both the taps are opened at 11:40 in 4 min. All the three pipes are opened together and after
a.m., then the tank will be filled at: 2 minutes pipe R is closed. Find when the tank will be full:
(a) 12:40 p.m. (b) 1:30 p.m. (a) 3 minutes (c) 6 minutes
(c) 1:00 p.m. (d) 1:20 p.m. (c) 5 minutes (d) 8 minutes
[Based on IIT Joint Man. Ent. Test, 2004] [Based on MAT (Dec), 2010]

Chapter 11.indd 4 6/5/2015 1:02:41 AM


Pipes and Cisterns 11.5

14. There is a leak in the bottom of a cistern. Before the leak, 20. Two pipes can fill a cistern in 15 minutes and 18 minutes
1 1 respectively. Both the pipes are operating together but
it could be filled in 4 hrs. It now takes hhrs longer. If
2 2 3 minutes before the cistern is full, one of the pipes is
the cistern is full, in how much time would the leakage closed. The cistern will be filled now in:
empty the full cistern? 1 3
(a) 9 minutes (c) 3 minutes
(a) 23 hrs (c) 35 hrs 7 11
(c) 52 hrs (d) 45 hrs 3
(c) 7 minutes (d) None of these
[Based on MAT (Dec), 2010, 2009] 11
[Based on MAT (May), 2010]
15. Two filling pipes A and B can fill a tank in 30 hrs and
20 hrs respectively. Pipe B alone is kept open for half the 21. Two taps A and B can fill a cistern in 12 minutes and
time and both pipes are kept open for the remaining time. 15 minutes respectively. They are opened together but
In how many hrs, will the tank be completely full? after a few minutes, A is turned off and the rest of the
(a) 25 hrs (c) 40 hrs cistern is filled by B in 5 minutes. After how many minutes
was A turned off ?
(c) 15 hrs (d) 28 hrs
[Based on MAT (Dec), 2010]
(a) 4 minutes (c) 7 minutes
(c) 6 minutes (d) None of these
16. Two pipes A and B can fill a tank in 20 and 30 hrs, [Based on MAT (May), 2010]
respectively. Both the pipes are opened to fill the tank but
when the tank is one-third full, a leak develops in the tank 22. Three pipes A, B and C can fill a tank in 20 minutes, 10
through which one-fourth water supplied by both pipes minutes and 30 minutes, respectively. When the tank is
goes out. What is the total time taken to fill the tank? empty, all the three pipes are opened. A, B and C discharge
chemical solutions x, y and z respectively. The proportion
2
(a) 14 hrs
h (c) 15 hrs of solution y in the liquid in the tank after 3 minutes is:
3
(a) 6/11 (c) 7/11
1 1 (c) 8/11 (d) 5/11
(c) 12 hrs
h (d) 9 hrs
h [Based on MAT (Feb), 2010]
2 2
[Based on MAT (Dec), 2010]
23. Three taps A, B and C can fill a tank in 12, 15 and 20 hrs,
respectively. If A is open all the time and B and C are open
17. A tank is filled by three pipes with uniform flow. The first for one hour each alternatively, the tank will be filled in:
two pipes operating simultaneously fill the tank in the (a) 6 hrs (c) 7 hrs
same time during which the tank is filled by the third pipe
(c) 5 hrs (d) None of these
alone. The second pipe fills the tank 5 hrs faster than the
[Based on MAT (Feb), 2010]
first pipe and 4 hrs slower than the third pipe. The time
required by the first pipe is: 24. In what time would a cistern be filled by three pipes whose
diameters are 2 cm, 3 cm and 4 cm running together, when
(a) 6 hrs (c) 10 hrs
the largest alone can fill it is 58 minutes? The amount of
(c) 15 hrs (d) 30 hrs water flowing in each pipe is proportional to the square of
[Based on MAT (Sept), 2010] its diameter:
18. A booster pump can be used for filling as well as for (a) 36 minutes (c) 32 minutes
emptying a tank. The capacity of the tank is 2400 m3. The (c) 23 minutes (d) 28 minutes
emptying capacity of the tank is 10 m3 per minute higher [Based on MAT (Dec), 2009]
than its filling capacity and the pump needs 8 minutes 25. A cistern has 3 pipes A, B and C. A and B running alone
lesser to empty the tank than it needs to fill it. What is the can fill it up in 2 hrs and 3 hrs respectively, while C is a
filling capacity of the pump? waste pipe. If all the 3 pipes be opened at once, 7.24 of the
(a) 50 m3/minute (c) 60 m3/minute cistern will be filled up in 30 minutes. In what time can C
3
(c) 72 m /minute (d) 36 m3/minute empty the full cistern?
[Based on MAT (Sept, Dec), 2010] (a) 5 hrs (c) 4 hrs
(c) 3 hrs (d) 6 hrs
19. Two pipes can fill a tank in 10 hrs and 15 hrs, respectively.
However, leakage at the bottom of the tank delays the [Based on MAT (Dec), 2009]
filling of the tank by 3 hrs when both the pipes are open 26. One fill pipe A is 3 times faster than second fill pipe B and
simultaneously. How much time would the leak take to takes 32 minutes less than the fill pipe B. When will the
empty the full cistern? cistern be full if both pipes are opened together?
(a) 22 hrs (b) 18 hrs (a) 12 minutes (c) 24 minutes
(c) 12 hrs (d) 21 hrs (c) 30 minutes (d) Data inadequate
[Based on MAT (May), 2010] [Based on MAT (Sept), 2009]

Chapter 11.indd 5 6/5/2015 1:02:42 AM


11.6 Chapter 11

27. A cistern can be filled by two pipes filling separately in through the former and only 5/6 through the latter pipe.
12 and 16 minutes, respectively. Both pipes are opened The obstructions, however, being suddenly removed, the
together for a certain time but being clogged, only seven- cistern is filled in 3 minutes from that moment. How long
eighths of full quantity water flows through the former and was it before the full flow began?
only five-sixths through the latter pipe. The obstructions, (a) 2.5 minutes (c) 4.5 minutes
however, being suddenly removed, the cistern is filled in (c) 3.5 minutes (d) 5.5 minutes
3 minutes from that moment. How long was it before the [Based on MAT (May), 2006]
full flow began?
(a) 4.5 minutes (c) 2.5 minutes 34. There are two indentical vessels X and Y. Y is filled
(c) 3.5 minutes (d) 5.5 minutes with water to the brim and X is empty. There are two pails
A and B, such that B can hold half as much water as A. One
[Based on MAT (May), 2009]
operation is said to be executed when water is transferred
28. A cistern has a leak which would empty it in 8 hrs. A tap
from Y to X using A once and water is transferred to Y
is turned on which admits 6 litres a minute into the cistern
from X using B once. If A can hold half a litre of water and
and it is now emptied in 12 hrs. The cistern can hold:
it takes 40 operations to equate the water level in X and Y,
(a) 7860 litres (c) 6840 litres what is the total volume of water in the system?
(c) 8640 litres (d) None of these
(a) 10 litres (b) 20 litres
[Based on MAT (Feb), 2009]
3
29. A bath tube can be filled by a cold water pipe in 20 minutes (c) 40 litres (d) 20 litres
and by a hot water pipe in 30 minutes. A person leaves the 4
[Based on MAT (Sept), 2009]
bathroom after turning on both pipes simultaneously and
returns at the moment when the bath tub should be full. 35. Pipe A can fill a tank in 3 hrs and 45 minutes. 2 hrs after
Finding however, that the waste pipe has been open, he the pipe started filling the empty tank the motor stopped
now closes it. In 3 minutes more the bath tub is full. In working. What per cent of the tank was left empty?
what time would the waste pipe empty it? 2
(a) 58% (b) 46 %
(a) 38 minutes (c) 45 minutes 3
(c) 43 minutes (d) 48 minutes
1 1
[Based on MAT (Dec, May), 2008] (c) 33 % (d) 53 %
3 3
30. Six pipes are fitted to a water tank. Some of these are inlet
pipes and the others outlet pipes. Each inlet pipe can fill 36. Pipes A and B can fill a tank in 5 and 6 hrs, respectively.
the tank in 9 hrs and each outlet pipe can empty the tank Pipe C can empty it in 12 hrs. The tank is half full. All
in 6 hrs. On opening all the pipes, an empty tank is filled the three pipes are in operation simultaneously. After how
in 9 hrs. The number of inlet pipes is: much time the tank will be full?
(a) 2 (b) 4 9
(a) 3 hrs (b) 11 hrs
(c) 3 (d) 5 17
[Based on MAT (Sept), 2008]
8 13
31. A leak in the bottom of a cistern can empty the tank in (c) 2 hrs (d) 1 hrs
11 17
12 hrs. An inlet pipe fills water at the rate of 5 litres a
[Based on MAT, 1999]
minute. When the tank is full, the inlet is opened and due
to the leak, the tank is emptied in 15 hrs. How many litres 37. A cistern has two taps (which fill it in 12 minutes and
does the cistern hold? 15 minutes, respectively) and an exhaust tap. When all
the three taps are opened together, it takes 20 minutes. to
(a) 8260 (b) 12000
fill an empty cistern. How long will the exhaust tap take
(c) 15000 (d) 18000 to empty it?
32. Two taps can fill a tank in 20 minutes and 30 minutes, (a) 20 minutes (b) 16 minutes
respectively. There is an outlet tap at exactly half level of (c) 12 minutes (d) 10 minutes
that rectangular tank which can pump out 100 litres of water
[Base d on MAT, 1999]
per minute. If the outlet tap is open, then it takes 24 minutes
to fill an empty tank. What is the volume of the tank? 38. A cistern is normally filled in 8 hrs, but takes two hrs
(a) 1800 litres (c) 1500 litres longer to fill because of a leak in its bottom. If the cistern
(c) 1200 litres (d) 2400 litres is full, the leak will empty it in:
[Based on MAT (May), 2007] (a) 16 hrs (b) 25 hrs
33.
A cistern can be filled by two pipes filling separately (c) 40 hrs (d) 20 hrs
in 12 and 16 minutes, respectively. Both pipes are [Based on MAT, 2000]
opened together for a certain time but being clogged, 39. Two taps can fill a tank in 12 minutes and 18 minutes,
only seven-eighths of the full quantity of water flows respectively. Both the taps are kept open for 2 minutes and

Chapter 11.indd 6 6/5/2015 1:02:42 AM


Pipes and Cisterns 11.7

then the tap that fills the tank in 12 minutes is turned off. (a) 39 minutes (b) 47 minutes
In how many more minutes will the tank be filled? (c) 45 minutes (d) 25 minutes
(a) 9 (b) 10  [Based on MAT, 2011]
(c) 12 (d) 13 46. To fill a certain tank, pipes A, B and C take 20 minutes, 15
40. Pipe A and Pipe B can completely fill a cistern in 8 and minutes and 12 minutes, respectively. If the three pipes are
12 hrs, respectively. The two pipes are simultaneously opened in every alternate minute, how long will it take to
opened but due to a leak at the bottom of the cistem it fill the tank?
takes 6 hrs extra to fill the cistern. Find the time in which (a) 5 minutes (b) 10 minutes
the leak can empty the full cistern. (c) 12 minutes (d) 15 minutes
120  [Based on MAT, 2011]
(a) 10 hrs (b) hrs
13 47. A bath can be filled by the cold water and hot water pipes in
123 10 minutes and 15 minutes, respectively. A person leaves
(c) hrs (d) none of these the bathroom after turning on both pipes simultaneously
15
and returns at the moment when the bath should be full.
41. A vessel has three pipes connected with it, two to supply
Finding, however, that the waste pipe has been open, he
liquid and one of draw liquid. The first alone can fill
1 then closes it. In exactly four minutes more the bath is fill.
the vessel in 4 hrs, the second in 3 hrs and the third In how much time would the waste pipe empty the full
2 1
can empty it in 1 h.hrs. If all the pipes are opened bath, if it alone opened?
2 (a) 9 minutes (b) 10 minutes
simultaneously when the vessel is half full, how soon will
it be emptied? (c) 12 minutes (d) None of these
1 1  [Based on MAT, 2012]
(a) 5 hhrs (b) 3 hrs
h
2 8 48. A, B and C are three pipes attached to a cistern. A and B
can fill it in 20 minutes and 30 minutes respectively, while
1 C can empty in 15 minutes. If A, B and C be kept open
(c) 4 hrs
h (d) None of these
3 successively for 1 minute each, how soon will the cistern
[Based on NMAT, 2006]
be filled?
42. One filling pipe A is 6 times faster than second filling pipe
(a) 180 minutes (b) 60 minutes
B. If B can fill a cistern in 28 minutes, then find the time
when the cistern will be full if both the pipes are opened (c) 157 minutes (d) 155 minutes
together.  [Based on MAT, 2012]
(a) 6 minutes (b) 8 minutes 49. Two taps can separately fill a cistern in 10 minutes and 15
(c) 4 minutes (d) 7 minutes minutes, respectively. If these two pipes and a waste pipe
are kept open simultaneously, the cistern gets filled in 18
43. A, B, C are pipes attached to a cistern. A and B can fill it minutes. the waste pipe can empty the full cistern in:
in 20 and 30 minutes respectively, while C can empty it in (a) 7 minutes (b) 13 minutes
15 minutes. If A, B and C are kept in operation successively (c) 23 minutes (d) 9 minutes
for one minute each, how soon will the cistern be filled?
 [Based on MAT, 2012]
(a) 167 minutes (b) 160 minutes
50. A tank is fitted with 8 pipes, some of them that fill the tank
(c) 166 minutes (d) 164 minutes and other pipes are meant to empty the tank. Each pipe
44. A pump can be operated both for filling a tank and for that fills the tank can fill it in 8 h, while each of those that
emptying it. The capacity of tank is 2400 m3. The empty the tank can empty it in 6 h. All the pipes are kept
emptying capacity of the pump is 10 m3 per minute higher open. When tank is full, it will take exactly 6 hrs for the
than its filling capacity. Consequently, the pump needs tank to empty. How many pipes are filing the tank?
8 minutes less to empty the tank than to fill it. Find the (a) 5 (b) 7
filling capacity of the pump. (c) 8 (d) 4
(a) 45 m3/minute (b) 30 m3/minute  [Based on MAT (Feb), 2012]
(c) 50 m3/minute (d) 55 m3/minute 51. A leak in the bottom of a tank can empty the full tank in
 [Based on MAT, 2011] 6 h. An inlet pipe fills water at the rate of 4 L per minute.
When the tank is full, the inlet is opened and due to the
45. Two pipes A and B can fill a cistern in 12 minutes and 15
leak the tank is emptied in 8 h. The capacity of the tank is:
minutes respectively, while a third pipe C can empty it in
6 minutes. Both A and B pipes are opened together for 5 (a) 9600 L (b) 5760 L
minutes and then the third pipe C is opened. In what time (c) 2880 L (d) None of these
will the cistern be empted?  [Based on MAT (Feb), 2012]

Chapter 11.indd 7 6/5/2015 1:02:42 AM


11.8 Chapter 11

52. Two pipes A and B can fill a cistern in 12 minutes and 15 58. One inlet pipe A is 3 times faster than second inlet pipe B
minutes, respectively, while a third pipe C can empty it in and take 20 minutes less than inlet pipe B. When will the
6 minutes. Both A and B pipes are opened together for 5 cistern be full if both pipes are opened together?
minutes in the beginning and then the third pipe C is also (a) 16 minutes (b) 15/2 minutes
opened. In what time will the cistern be emptied? (c) 15 minutes (d) None of these
(a) 30 minutes (b) 33 minutes 59. There are three pipes, A, B and C, opening into a tank.
(c) 37.5 minutes (d) 45 minutes Each pipe can be used to fill or empty the tank at the same
 [Based on MAT (Feb), 2012] respective rate. The ratio of the rates of the three pipes at,
53. A tank is fitted with 8 pipes, some of them that fill the tank which they either fill or empty the tank 2:3:4. Pipes A and
and other pipes are meant to empty the tank. Each pipe B, working together, take 3 hrs to fill the tank when both
that fills the tank can fill it in 8 h, while each of those that are used as inlet pipes. In the first, second and third hours
empty the tank can empty it in 6 h. All the pipes are kept of operation, the pipes, A, B and C, respectively are used
open. When tank is full, it will take exactly 6 hrs for the as emptying pipes, while the other two pipes are used as
tank to empty. How many pipes are filing the tank? filling pipes. What is the fraction of the tank to be filled at
(a) 5 (b) 7 the end of the three hours?
(c) 8 (d) 4 1 3
(a) (b)
 [Based on MAT (Feb), 2012] 5 5
2 4
54. A leak in the bottom of a tank can empty the full tank in (c) (d)
6­h. An inlet pipe fills water at the rate of 4 L per minute. 5 5
When the tank is full, the inlet is opened and due to the  [Based on MAT, 2013]
leak the tank is emptied in 8 h. The capacity of the tank is: 60. A booster pump can be used for filling as-well-as for
(a) 9600 L (b) 5760 L emptying a tank. The capacity of the tank is 2400 m3. The
emptying capacity of the tank is 10 m3 per minute higher
(c) 2880 L (d) None of these
than its filling capacity and the pump needs 8 minutes
 [Based on MAT (Feb), 2012] lesser to empty the tank than it need to fill it. What is the
55. Two pipes A and B can fill a cistern in 12 minutes and 15 filling capacity of the pump?
minutes, respectively, while a third pipe C can empty it in (a) 60 m3/minu (b) 50 m3/minu
6 minutes. Both A and B pipes are opened together for 5 3
(c) 72 m /minu (d) None of these
minutes in the beginning and then the third pipe C is also
opened. In what time will the cistern be emptied?  [Based on MAT, 2013]
(a) 30 minutes (b) 33 minutes 61. A cistern, open at the top, is to be lined with sheet lead
which weights 27 kg/m3. The cistern is 4.5 m long and 3
(c) 37.5 minutes (d) 45 minutes
m wide and holds 50 m3. The weight of lead required is:
 [Based on MAT (Feb), 2012]
(a) 1764.60 kg (b) 1864.62 kg
56. Pipe A can fill an empty tank in 30 h, while pipe B
(c) 1660.62 kg (d) 1860.62 kg
can fill it in 45 h. Pipe A and B are opened and closed
alternatively, i.e., first pipe A is opened, then B, again A  [Based on MAT, 2013]
and then B and so on for 1 hr each time without any time 62. Six pipes are fitted to a water tank. Some of these are inlet
gaps. In how many hours will the tank be filled when it pipes and the others outlet pipes. Each inlet pipe can fill
was empty, initially? the tank in 9 hrs and each outlet pipe can empty the tank
(a) 36 hrs (b) 54 hrs in 6 hrs. On opening all the pipes, an empty tank is filled
(c) 48 hrs (d) 60 hrs in 9 hrs. The number of inlet pipes is:
 [Based on MAT, 2013] (a) 4 (b) 3
(c) 2 (d) 5
57. Two pipes A and B can fill a tank in 30 minutes and
40 minutes respectively. If both the pipes are opened  [Based on MAT, 2013]
simultaneously, after how much time should B be closed 1
so that the tank gets filled in 20 minutes? 63. The fuel indicator in a car shows th of the fuel tank as
5
1 full. When 22 more liters of fuel are poured into the tank,
(a) 12 minutes (b) 13 minutes the indicator rests at three-fourth of the full mark. What is
3 the capacity of the fuel tank?
1 1 (a) 40 litres (b) 30 litres
(c) 11 minutes (d) 15 minutes
3 2 (c) 25 litres (d) 35 litres
 [Based on MAT, 2013]  [Based on MAT, 2013]

Chapter 11.indd 8 6/5/2015 1:02:44 AM


Pipes and Cisterns 11.9

64. A tank of capacity 25 L has an inlet and an outlet tap. 8 minutes less to empty the tank than to fill it. Find the
If both are opened simultaneously, the tank is filed filling capacity of the pump.
in 5 minutes. But if the outlet flow rate is doubled and (a) 45 m3/min (b) 30 m3/min
taps opened the tank never gets filled up. Which of the 3
(c) 50 m /min (d) 55 m3/min
following can be outlet flow rate?
 [Based on SNAP, 2013]
(a) 3 L/min (b) 4 L/min
(c) 5 L/min (d) None of these 68. A ship, 40 Km from the shore, spring a leak which admits
3
 [Based on MAT, 2014] 3 tonnes of water in 12 minutes 60 tonnes would suffice
4
65. A tank of 4800 m3 capacity is full of water. The
to sink her, but the ship’s pumps can throw out 12 tonnes
discharging capacity of the pump is 10 m3/minute higher
than its filling capacity. As a result the pump needs 16 of water in one hour. Find the average rate of sailing, so
minutes less to discharge the fuel then to fill up the tank. that it may reach the shore just as it begins to sink.
Find the filling capacity of the pump.
1 1
(a) 50 m3/minu (b) 25 m3/minu (a) 1 Km/h (b) 2 Km/h
2 2
(c) 55 m3/minu (d) None of these
1 1
 [Based on MAT, 2014] (c) 3 Km/h (d) 4 Km/h
2 2
66. A cistern contains 50 Litres of water 5 Litres of water
is taken out of it and replaced with wine. The process is  [Based on SNAP, 2013]
repeated again. Find the proportion’ of wine and water in 69. A leak was found in a ship when it was 77 Km from the
the resulting mixture. shore. It was found that the leak admits 2.25 tonnes of
(a) 1:4 (b) 41:50 water in 5.5 minutes. 92 tonnes will suffice to sink the
ship. But the pumps can throw out the water @ 12 tonnes
(c) 19:81 (d) 81:19
an hour. Find the average rate of sailing at which the ship
 [Based on MAT, 2014] may reach the shore as it begins to sink.
67. A pump can be operated both for filling a tank and for (a) 9.75 Km/h (b) 13 Km/h
emptying it. The capacity of tank is 2400 m3. The
(c) 14.5 Km/h (d) 10.5 Km/h
emptying capacity of the pump is 10 m3 per minute higher
than its filling capacity. Consequently, the pump needs  [Based on SNAP 2012]

Difficulty Level-2
(Based on Memory)

1. Two pipes A and B can fill a tank in 6 hrs and 9 hrs 3. Two taps can fill a tank in 20 minutes and 30 minutes
respectively when working alone. A third pipe C can respectively. There is an outlet tap at exactly half level
empty the same tank in 8 hrs. The pipes are operated such of that rectangular tank which can pump out 50 litres of
that A and C are open for the first couple of hrs, then again water per minute. If the outlet tap is open, then it takes
B and C for the next 2 hrs, again A and C for the next 2 24 minutes to fill an empty tank. What is the volume of
hrs, and so on. Operating in such a manner, if 68 litres of the tank?
water is filled in 30 hrs, then find the volume of the tank. (a) 1200 litres (b) 1500 litres
(a) 144 litres (b) 108 litres (c) 1800 litres (d) 2400 litres
(c) 72 litres (d) 216 litres [Based on IIT Joint Man. Ent. Test, 2004]
2. A tank is filled with water through five pipes. The first 4. A cistern can be filled by pipes A and B in 4 hrs and 6 hrs
pipe can fill it in 40 minutes. The second, the third and the respectively. When full, the tank can be emptied by pipe
fourth together can fill it in 10 minutes; the second, the C in 8 hrs. If all the taps were turned on at the same time,
third and the fifth fill it in 20 minutes; the fourth and the then the cistern will be full in:
fifth together in 30 minutes. In what time will the tank be (a) 3 hrs 18 minutes (b) 3 hrs 26 minutes
filled if all the five pipes work simultaneously?
(c) 3 hrs 42 minutes (d) 3 hrs 48 minutes
2 3
(a) 8 minutes (b) 7 minutes [Based on FMS (Delhi), 2003]
5 4
5.
Two pipes A and B can fill a cistern in 12 minutes and
4 1 15 minutes respectively but a third pipe C can empty the
(c) 8 minutes (d) 8 minutes
7 7 full tank in 6 minutes. A and B are kept open for 5 minutes

Chapter 11.indd 9 6/5/2015 1:02:46 AM


11.10 Chapter 11

in the beginning and then C is also opened. In what time tank in 12 h. There is a pipe P3 which can empty the tank
will the cistern be emptied? in 8 hrs. Both the pumps are opened simultaneously. The
(a) 30 minutes. (b) 33 minutes. supervisor of the tank, before going out on a work, sets a
timer to open P3 when the tank is half filled so that tank is
(c) 37.5 minutes. (d) 45 minutes.
exactly filled up by the time he is back. Due to technical
[Based on FMS (Delhi), 2002]
fault P3 opens when the tank is one third filled. If the
6. A steady stream flows into a cistern partly full which has supervisor comes back as per the plan what per cent of the
a number of equal holes at the bottom. If 12 holes are tank is still empty?
opened, the cistern is emptied in 4 hrs and if 10 holes are
(a) 25% tank (b) 12% tank
opened the cistern is emptied in 8 hrs. How many holes
should be opened so as to empty the cistern in 2 hrs? (c) 10% tank (d) None of these
[Based on ITFT, 2009]
(a) 14 (b) 16
(c) 15 (d) 12 12. Three pipes A, B and C are connected to a tank. These
pipes can fill the tank separately in 5 hrs, 10 hrs and
7. Two pipes A and B fill a swimming pool at constant rates
15 hrs, respectively. When all the three pipes were opened
of 10 gallons per minute and 15 gallons per minute,
simultaneously, it was observed that pipes A and B were
respectively. The pool can be filled in 60 hrs, 40 hrs or
supplying water at three-fourths of their normal rates for the
24 hrs depending on, whether pipe A alone, pipe B alone,
1st hrs after which they supplied water at the normal rate.
or both pipes are used. If the pool is filled by using pipe
Pipe C supplied water at two-thirds of its normal rate for
B alone for half the time and using both pipes for half the
first 2 hrs, after which it supplied at its normal rate. In how
time, how many hrs does it take to fill the pool?
much time, tank would be filled?
(a) 15 hrs (b) 30 hrs
(a) 1.05 hrs (b) 2.05 hrs
(c) 38.7 hrs (d) 42 hrs
(c) 3.05 hrs (d) None of these
8. A tank can be filled by a tap in 20 minutes and by another
[Based on IIFT, 2010]
tap in 60 minutes. Both the taps are kept open for 10
minutes and then the first tap is shut of. After this, the tank 13. A bath can be filled by the cold water pipe in 10 minutes
will be completely filled in: and by hot water pipe in 15 minutes (independently each).
(a) 10 minutes (b) 12 minutes A person leaves the bathroom after turning on both pipes
(c) 15 minutes (d) 20 minutes simultaneously and returns at the moment when the bath
should be full. Finding, however, that the waste pipe has
[Based on FMS (MS), 2006]
been opened he now closes it. In 4 minutes more, bath is
9. Two pipes can fill a cistern in 14 hrs and 16 hrs respectively, full. In what time would be the waste pipe empty it?
the pipes are opened simultaneously and it is found that (a) 6 minutes (b) 9 minutes
due to leakage in the bottom, 32 minutes extra are taken
(c) 3 minutes (d) 15 minutes
for the cistern to be filled up. If the cistern is full, in what
time would the leak empty it? [Based on ATMA, 2008]
(a) 96 hrs (b) 102 hrs 14. Pipe A can fill an empty tank in 30 hrs while B can fill it in
(c) 106 hrs (d) 112 hrs 45 hrs. Pipes A and B are opened and closed alternatively
[Based on IIFT, 2005] i.e., first pipe A is opened, then B, again A and then B
10. Pavan builds an overhead tank in his house, which has and so on for 1 hr each time without any time lapse. In
three taps attached to it. While the first tap can fill the how many hrs the tank will be filled when it was empty,
tank in 12 hrs, the second one takes one and a half times initially?
more than the first one to fill it completely. A third tap is (a) 36 (b) 54
attached to the tank which empties it in 36 hrs. Now, one (c) 48 (d) 60
day, in order to fill the tank. Pavan opens the first tap and
after two hrs opens the second tap as well. However, at 15. An inlet pipe can fill a tank in 5 hrs and an outlet pipe
the end of the sixth hour, he realizes that the third tap has can empty the same tank in 36 hrs, working individually.
been kept open right from the beginning and promptly How many additional number of outlet pipes of the same
closes it. What will be the total time required to fill the capacity are required to be opened, so that the tank never
tank? over flows?
(a) 8 hrs 48 minutes (b) 9 hrs 12 minutes (a) 3 (b) 6
(c) 9 hrs 36 minutes (d) 8 hrs 30 minutes (c) 8 (d) 7
[Based on IIFT, 2006]
16. Three pipes A, B and C are attached to a cistern.
11.
A cylindrical overhead tank is filled by two pumps–P1 A can fill it in 10 minutes B in 15 minutes, C is a waste
and P2. P1 can fill the tank in 8 hrs while P2 can fill the pipe for emptying it. After opening both the pipes A and

Chapter 11.indd 10 6/5/2015 1:02:46 AM


Pipes and Cisterns 11.11

B, a man leaves the cistern and returns when the cistern (a) 50 m3/minute (b) 60 m3/minute
should have been just full. Finding, however, that the (c) 45 m3/minute (d) 90 m3/minute
waste pipe had been left open, he closes it and the cistern
 [Based on CAT, 2013]
now gets filled in 2 minutes. In how much time the pipe C,
if opened alone, empty the full cistern. 18. A tank connected with 15 pipes. Some of them are inlet
(a) 12 minutes pipes, and the rest work as outlets pipes. Each of the inlet
(b) 16 minutes pipe can fill the tank in 8  h individually, while each of
(c) 18 minutes those that drain the tank, i.e., output pipe, can drain it in
6 h individually. If all the pipes are kept open when the
(d) 15 minutes tank is full, it will take exactly 6 h for the tank to empty.
17. A pump can be used to either fill or drain a tank. The How many of these are inlet pipes?
capacity of the tank is 3600 m3. The draining capacity of (a) 2 (b) 8
the pump is 10 m3/minute higher than its filling capacity.
(c) 5 (d) 6
What is the draining capacity of the pump if it takes
12 minutes more to fill the tank than to drain it?  [Based on CAT, 2013]

Answer Keys
Difficulty Level-1

1.  (c) 2.  (c) 3.  (a) 4.  (d ) 5.  (d ) 6.  (c) 7.  (c) 8.  (d ) 9.  (c) 10.  (b) 11.  (b) 12.  (c) 13.  (a)
14.  (d ) 15.  (c) 16.  (a) 17.  (c) 18.  (a) 19.  (b) 20.  (d ) 21.  (d ) 22.  (a) 23.  (b) 24.  (b) 25.  (b) 26.  (a)
27.  (a) 28.  (c) 29.  (d ) 30.  (b) 31.  (d ) 32.  (a) 33.  (b) 34.  (b) 35.  (b) 36.  (d ) 37.  (d ) 38.  (c) 39.  (d )
40.  (d ) 41.  (d ) 42.  (c) 43.  (a) 44.  (c) 45.  (c) 46.  (d ) 47.  (d ) 48.  (a) 49.  (d ) 50.  (d ) 51.  (b) 52.  (d )
53.  (d ) 54.  (b) 55.  (d ) 56.  (a) 57.  (b) 58.  (b) 59.  (c) 60.  (b) 61.  (b) 62.  (a) 63.  (a) 64.  (c) 65.  (a)
66.  (c) 67.  (c) 68.  (d ) 69.  (d )

Difficulty Level-2

1.  (a) 2.  (c) 3.  (a) 4.  (b) 5.  (d ) 6.  (b) 7.  (b) 8.  (d ) 9.  (d ) 10.  (b) 11.  (c) 12.  (c) 13.  (d)
14.  (a) 15.  (d ) 16.  (c) 17.  (b) 18.  (b)

Explanatory Answers

Difficulty Level-1

1.
(c) Let, the capacity of the tank be x litres. After 2nd operation
3 4 Quantity of special liquid
\ x + 5 =
4 5 6 24
= ( x − 4) − ( x − 4) i.e., x + − 10
⇒ x = 100. x x

2. (c) Suppose the volume of the vessel be x litres. 6 24


Water = 4 − × 4 + 6 = 10 −
x x
Therefore, the quantity of special liquid in the vessel
= x litres. 24
x+ − 10 1
After 1st operation x
\ =
Quantity of special liquid = x – 4 24 2
10 −
Water = 4 x

Chapter 11.indd 11 6/5/2015 1:02:46 AM


11.12 Chapter 11

48 24 1 1 1
⇒ 2x + − 20 = 10 − 9. (c) Net part filled in 1 minutes = + −
x x 4 5 3
72 15 + 12 − 20 7
⇒ 2x + − 30 = 0 = =
x 60 60
⇒ 2x2 – 30x + 72 = 0 7
Q part is filled = 1 minutes
⇒ x2 – 15x + 36 = 0 60
⇒ x = 12. 60
⇒ 1 part is filled = min
1 7
3.
(a) In one hour, of the cistern can be filled.
6 4 60 4 48
⇒ th part is filled = × =
1 5 7 5 7
In one hour, only of the cistern can be filled due to
7 6
= 6 minutes to fill.
leak in its bottom. 7
1 1 1
\ In one hour, − = of the cistern is empty. 10.
(b) Work done by the waste pipe in 1 minutes
6 7 42
1 1 1
\ The whose cistern will be emptied in 42 hrs. = − + 
15  10 12 
4. (d) Capacity of the tank = 12 × 13.5 = 162 litres 8 − (12 + 10) 14
= =–
Number of buckets required, if the capacity of the 120 120
162 \ Waste pipe will empty the full cistern
bucket is 9 litres =  = 18.
9 120 60
= =
14 7
1 1 1 = 8 minutes 34 seconds.
5.
(d) Work done by inlet in 1 hr = − =
8 12 24
11. (b) Efficiency of A = 10%
1 1 1
Work done by inlet in 1 minute = × = Efficiency of B = 6.66%
24 60 1440
1 Efficiency of C = 5%
Q Volume of part = 6 litres Efficiency of D = 3.33%
1440
\ Volume of whole = 1440 × 6 = 8640 litres. Efficiency of A + B + C + D = 8.33
(time = 12 hous)
6.
(c) Suppose pipe A fills the cistern in x minutes. Now, go through options and consider A and B as inlet
Therefore pipe B will fill the cistern in (x + 5) minutes. pipes and C and D as outlet pipes, then
\ In one minute, pipes A and B together can fill (10 + 6.66) – (5 + 3.33) = 8.33
1 1  Which is required, hence, it is certain that
 x + x + 5  of the cistern. C and D are outlet pipes.
 
1 1 1 12.
(c) Upto 3 pm both pipe fill the tank
⇒ + = 
x x+5   6
2 1 1 1 4 1
⇒ x = 10 = + = + = = part
8 12 4 12 12 3
\ Pipe A and B can fill the cistern in 10 minutes and
15 minutes respectively. 1 2
\ Remaining part = 1 − =
3 3
1 1 1
7.
(c) In one minute, − = of tank can be filled.
48 120 80 1 1 1 2
\  + − x =
\ The whole tank can be filled in 80 minutes.  8 12 6  3
(6 + 4 − 8) x 2
8.
(d) Let, the tin oil contain x number of bottles. ⇒ =
4 3 48 3
x − 6 + 4 = x 2x 3
5 4 ⇒ =
1 48 2
⇒ x = 2
20 ⇒ x = 16 hrs
\ x = 40 bottles. Hence, the cistern can be filled at 1 p.m + 16 hrs = 5 a.m.

Chapter 11.indd 12 6/5/2015 1:02:47 AM


Pipes and Cisterns 11.13

2 2 2 ⇒ x2 – 5x = 2x2 – 23x + 45
13. (a) Part of the tank filled in 2 minutes =  + −  =
1 3 6 4 ⇒ x2 – 18x + 45 = 0
2 ⇒ (x – 15)(x – 3) = 0
1
P and Q can fill part of tank in
2 ⇒ x = 15 hrs as x = 3 hrs is not possible.
1  3× 6  (a) Let, the filled capacity of the tank be x m3/minute.
18.
×   = 1 minutes
2 3+ 6
Then, emptied capacity of the tank
So, tank will be full in 3 minutes.
= (x + 10) m3/minute
14.
(d) Let, leakage can empty the full cistern in x hrs.
2400 2400
9 \ − =8
×x x x + 10

Then, 2 = 5
9  10 
x− ⇒ 2400   =8
2  x( x + 10) 
9 45 ⇒ x(x + 10) = 3000
⇒ x = 5x −
2 2 \ x = 50 m3/minute.
1 45
⇒ x =  10 × 15 
2 2 19.
(b) Two pipes can fill the tank in   hhrs = 6 hrs
⇒ x = 45 hrs.  10 + 15 

15.
(c) Let, the tank be completely full in x hrs. Due to leakage, the tank is filled in 9 hrs.
x
Pipe A is open for hrs and pipe B is open for x hrs. Let full tank can be empty in x hrs.
2
6× x
So, 9 =
x 1 1 x−6
\ × + x× =1
2 30 20
x  3x ⇒ 9x – 54 = 6x
⇒  =1
60 ⇒ x = 18 hrs.
⇒  x = 15 hrs
20.
(d) Let, the cistern will be filled in x minutes.
 20 × 30 
16.
(a) Together both pipes can fill the tank in   hrs Case I First pipe was closed
 20 + 30  x−3 x
= 12 hrs + =1
15 18
One-third tank can be filled in 4 hrs.
6 x − 18 + 5 x
Now, there is a leak which can empty the tank in ⇒ =1
90
(12 × 4) hrs = 48 hrs
⇒ 11x – 18 = 90
So, two-thirds tank can be filled in
9
⇒ x = 9 mins
2  12 × 48  2 11
×  hhrs = 10 hhrs
3  48 − 12  3
Case II Second pipe was closed.
So, total time to fill the tank
2 x x 3
= 4 + 10  =1
3 15 18
2
= 14 hhrs. 6 x  5 x  15
3 =1
90
17.
(c) Let, first pipe can fill the tank in x h.
Second pipe can fill the tank in (x – 5) hrs. 11x – 150 = 90

Third pipe can fill the tank in (x – 9) hrs. 6


x = 9 minutes
11

According to the question,
x × ( x − 5) Note: It is not given that which pipe was closed.
=x–9
x+ x−5 So, you have to calculate both the cases.

Chapter 11.indd 13 6/5/2015 1:02:48 AM


11.14 Chapter 11

21.
(d) Let, A was turned off after x minutes. \ Time required by 2 cm diameter pipe to fill the
x x+5 x
Then, + =1 cistern = × 928 = 232 minutes
12 15 4x
5 x + 4 x + 120 \ Time required by all the three pipes working
⇒ =1
60 1
58 × 103 × 232
⇒ 9x + 20 = 60 9
together =
4 1 1
⇒ x = 4 minutes 58 × 103 + 58 × 232 + 103 × 232
9 9 9
58 × 928 × 232
22.
(a) Filling done by all 3 pipes in 3 minutes =
58 × 928 + 58 × 232 × 9 + 928 × 232
3 3 3 11
= + + = = 32 minutes.
20 10 30 20
3 1
Filling done by 2nd pipe in 3 minutes = 25. (b) A’s work in 1 hr =
10 2
3 / 10 6 1
So, required ratio = = B’s work in 1 hr =
11 / 20 11 3
1
23. (b) Filling done by pipe A and B in 1 hr C’s work in 1 hr =
x
1 1 3 7
= + = Work done by all 3 pipes in 30 minutes = full
12 15 20 24
Filling done by pipe A and C in 1 hr 7 60 7
1 1 2 Work done by all 3 pipes in 1 hrs = × = full
= + = 24 30 12
12 20 15 1 1 1 7
3 2 17 so, + + =
Filling done in first 2 hrs = + = 2 3 x 12
20 15 60 5 1 7
17 51 ⇒ + =
Filling done in 6 hrs = ×3= 6 x 12
60 60 1 1
⇒ =–
51 3 x 4
Remaining filling = 1 −
=
60 20 ⇒ x = – 4 hrs
Now in 7th hour, filling done by pipes A and B, so Hence, C can empty in 4 hrs.
time
3 26.
(a) Let, pipe A takes x minutes to fill the cistern.
= 20 = 1 hhr Then, pipe B takes 3x minutes to fill the cistern.
3
\ 3x – x = 32
20
So, total time = 7 hrs. ⇒ x = 16 minutes and 3x = 48 minutes
 48 × 16 
24. (b) Pipe whose diameter is 4 cm fills the cistern in 58 So, both pipes can fill the cistern in   minutes
 48 + 16 
minutes. Let, the capacity of cistern be x l. = 12 minutes.
x
Then, flow rate of 4 cm diameter pipe = l/minute 27.
(a) Part of the cistern filled in 3 minutes
58
x 3 3 21 7
\ ∝ (4)2 = + = =
58 12 16 48 16
x x 9
⇒ = K × 16 ⇒ K = Let, remaining part was filled in x minutes.
58 928 16
x x 7 x 5 9
Flow rate of 3 cm diameter pipe = × (3)2 l/minute Then, × + × =
928 12 8 16 6 16
\ Time required by 3 cm diameter pipe to fill the
x 1 7 + 5 9
⇒ x  =
cistern = × 928 = 103 minutes  96  16
9x 9
x 9 96
Flow rate of 2 cm diameter pipe = × (2)2 l/minute ⇒ x= × 4.5 minutes.
=
928 16 12

Chapter 11.indd 14 6/9/2015 1:37:21 PM


Pipes and Cisterns 11.15

28.
(c) Let the tap can fill the cistern in x hrs. Thus, the capacity of the tank
x×8 = 100 × 9 × 2
\ = 12
x −8 = 1800 litres

⇒ 8x = 12x – 96 1 1 7
33.
(b) Both the pipes A and B can fill + = of the
⇒ x = 24 hrs 12 16 48
\ Capacity of cistern cistern in one minute, when there is no obstruction
1 7
= 24 × 60 × 6 = 8640 litres with obstruction, both the pipes can fill ×   + 
12 8
1 5 7 5 1
 ×  = + = of the cistern in one minute
29.
(d) The usual time required to fill the tank when both the 16 6 96 96 8
 1 1  1 Let, the obstruction were remained after x minutes.
pipes are opened =  + = , i.e., 12 minutes.
 20 30  12 x
It means work done by all the three pipes for 12 \ With obstruction, of the cistern could be filled in
8 x 8− x
minutes + work done by both the pipes for 3 minutes x minutes and so the remaining 1 – = of
= 1. Let, the waste pipe takes x minutes to empty the 8 8
tank. Then, the cistern was filled without obstruction is 3 minutes,
1 1 1 1 1 8− x
12  + −  + 3 +  = 1 i.e., in 1 minute of the cistern was filled with
24
 20 30 x   20 30  obstruction
 1 1 1 8− x 7
⇒   12  −  + 3   = 1 ⇒ =
12 x  12  24 48
⇒ 16 – 2x = 7
 x − 12  3
⇒   12  = ⇒ x = 4.5 minutes.
 12 x  4
⇒   16x – 192 = 12x 1
34. (b) In one operation, litre water is transferred from Y
⇒   4(x – 12) = 3x 4
to X.
⇒   x = 48 minutes. In 40 operations, total quantity of water transferred
1
30.
(b) Let, the number of inlet pipes be x, then number of = × 40
outlet pipes will be (6 – x). 4
= 10 litre
x (6 − x)
Then, − = \ Total volume of water in the system
9 6
= 10 × 2 = 20 litres.
2 x − 18 + 3 x 1
⇒   = 35. (b) The tank is filled in 3 hrs and 45 minutes = 225 minutes.
18 9
⇒ 5x = 20 120 8
∴ In 120 minutes, the pipe fills =
\ x=4 225 15
Hence, number of inlet pipes is 4. 7 2
Required percentage = × 100 = 46 %
15 3
1 1 1 36.
(d) In one hour,
31. (d) Part of tank filled in one hour  
12 15 60 1 1 1 12 + 10 − 5
So, the inlet can fill the tank in 60 hrs. + − =
5 6 12 60
∴ Capacity of the tank = 60 × 60 × 5 = 18000.
17
= of the tank is filled
32.
(a) The two filler taps can fill the tank in 60
1
 20 × 30  Therefore, of the tank is filled in
  = 12 minutes 2
(30 + 20) 

60 1 30 13
hrs = 1 hrs.
× = hours hours
The above information implies that half of the tank 17 2 17 17
will be filled in 6 minutes.
37. (d) Let, Tap A can fill the cistern in 12 minutes.
Hence, it took (24 – 6 = 18) minutes to fill the
Let, Tap B can fill the cistern in 15 minutes.
remaining half of the tank when the outlet pump was
open. Thus, the total time required to empty half of the Let, Tap C can empty the cistern in x minutes.
cistern 1 1 1 1
\ + − =
18 × 6 18 × 6 12 15 x 20
0 = = = 9 minutes
18 – 6 12 ⇒ x = 10.

Chapter 11.indd 15 6/5/2015 1:02:50 AM


11.16 Chapter 11

1 1
38. (c) Cistern’s 1 hr filling capacity = Now, its A’s turn. part of the tank is filled by A
8 20
1 1 1 1
Cistern’s 1 hr filling capacity due to leak = in 1 minutes, Since there is still     tank
10  12 20  30
\ Cistern’s 1 hr emptying capacity due to leak to be filled, which will be filled by B in 1 minutes.
1 1 1
= − = Therefore, required time = (165 + 2) = 167 minutesutes.
8 10 40
Hence, when the cistern is full, it will get empty in 44. (c) Let, filling capacity = x m3/min
40 hrs. And emptying capacity = (x + 10) m3/min
1 1 5 Then, according to the question,
39. (d) In 2 minutes, the taps fills 2    or of the 2400 2400
 12 18  18 − =8
tank. x x + 10
13
Therefore, of the tank is to be filled by the second 1 1 
18 ⇒ 2400  −  =8
1  x x + 10 
tap at the rate of of the tank per minute. This will
18 1 + 10 − x 
take another 13 minutes. ⇒ 2400   =8
 x( x + 10) 
2400 × 10
40. (d) Time taken to fill the cistern when pipe and pipe B are ⇒ x( x + 10) =
8
(8  12) 2
opened simultaneously =  = 4.8 hrs ⇒ x + 10 x − 3000 =0
(8  12) 2
⇒ x + 60 x − 50 x − 3000 = 0
Time taken due to the leek to fill the cistem
= 4.8 + 6 = 10.8 hrs ⇒ ( x − 50)( x + 60) = 0
Time taken by the leak alone to empty the cistern ⇒ x = 50 m3 / min.

(10.8  4.8) 1 1
= = 8.64 hrs. 45. (c) Part filled by (A + B) in 1 minute = +
(10.8  4.8) 12 15
1 1
Part filled by (A + B) in 5 minutes = 5  + 
9
41. (d) 1st pipe can fill in hrs  12 15 
2 9×5 3
2nd pipe can fill in 3 hrs = =
3 60 4

And 3rd pipe can empty in  h.
hrs Part emptied by (A + B + C) in 1 minutes
2
The vessel in emptied is 1 1 1 1
= − − =
6 12 15 60
2 1 2 1
+ − = − Thus, time taken to empty the full tank = 60 minutes
9 3 3 9 1
\ Then, vessel full emptied in 9 hrs Hence, time taken to empty the  tank
3 3
9 1 = 60 × = 45min
\ Then, vessel half emptied in hhrs is 4 h.
hrs.
2 2 4
1 1 1 1
46. (d) Part of tank filled in 3 minutes = + + +=
1 20 15 12 5
42. (c) In one minute, B will fill cistern. Thus, tank will be filled in 3 × 5 = 15 minutes.
28
6 47. (d) Time taken by cold water and hot water pipes to fill
Hence, A will fill cistern. Work done by both the
28 the both
 1 6 1 10 × 15 10 × 15
pipes in 1 minute =    cistern. Hence,
 28 28  4 = 10 + 15 = 25 = 6 minutes
they will fill 1 cistern in 4 minutes. Thus, when the waste pipe is also open, then time
taken to fill the bath = 6 + 4 = 10 minutes
 1 1 1 1 Now, let waste pipe can empty the full bath in x
43. (a) Work done in 3 minutes =    .
 20 30 15  60 minutes Then,
55 1 1 1 1
∴ Work done in 3 × 55 = 165 minutes = + − =
60 10 15 x 10
 55  5 1 1 1
Remaining tank = 1   = = ⇒ = ⇒ x = 15 minutes
 60  60 12 x 15

Chapter 11.indd 16 6/5/2015 1:02:57 AM


Pipes and Cisterns 11.17

1 1 1 1 54. (b) Let, the inlet pipe can fill the tank in x hrs. Then,
48. (a) Part of cistern filled in 3 minutes = + − =
20 30 15 60 1 1 1
Thus, time taken to fill the cistern = 3 × 60 = 180 − =
6 x 8
minutes 1 1 1 1
49. (d) Let, waste pipe can empty the full cistern in x minutes. ⇒ x = 6 − 8 = 24
Then, Therefore, inlet pipe will take 24 hrs to fill the tank.
1 1 1 1 Now,
+ − =
10 15 x 18 In 1 minute inlet pipe fill = 4 litres
1 1 1 1 9+6−5 1 ∴ In 24 hrs, inlet pipe will fill = 4 × 60 × 24
⇒ = + − = =
x 10 15 18 90 9 = 5760 litres
⇒ x = 9 minutes Which is the capacity to tank.

50. (d) Let, the number of pipes that fill and empty the tank 55. (d) Part of cistern that is filled by A and B in 5 minutes
be x and 8 – x respectively. Then, 1 1 3
8− x x 1 = 5 +  =
− =  12 15  4
6 8 6 Now, the time taken to empty the cistern, when all the
4(8 − x) − 3 x 1 pipes are opened
⇒ =
24 6 1 1
⇒ 32 − 7 x = 4 = = = 60 minutes
1 1 1 10 − 5 − 4
− +
⇒ 7 x = 28 ⇒ x = 4 6 12 15 60

51. (b) Let, the inlet pipe can fill the tank in x hrs. Then, ∴ Time taken to empty 3 part of the cistern
3 4
1 1 1 = × 60 = 45 min.
− =
6 x 8 4
1 1 1 1
⇒ = − = 56. (a) Part of tank filled by pipe A in 1 hr = 1/30
x 6 8 24
Part of tank filled by pipe B in 1 hr = 1/45
Therefore, inlet pipe will take 24 hrs to fill the tank.
Now, Then, part of tank filled in 2 hrs when one pipe is
In 1 minute inlet pipe fill = 4 litres opened at a time
∴  In 24 h, inlet pipe will fill = 4 × 60 × 24 1 1 3+ 2
= + =
 = 5760 litres 30 45 90
Which is the capacity to tank. 5 1
= =
90 18
52. (d) Part of cistern that is filled by A and B in 5 minutes
Now, let the tank takes x hrs to be filled. Then,
1 1 3 1
= 5 +  =
 12 15  4 × x = 1 ⇒ x = 18 hrs
18
Now, the time taken to empty the cistern, when all the
1
pipes are opened Also, part is filled in 2 hrs
1 1 18
= = = 60 minutes ∴  Tank is filled in 2 × h = 2 × 18 = 36 hrs
1 1 1 10 − 5 − 4
− +
6 12 15 60
57. (b) Let, the pipe B be closed x minutes before it gets
3
∴ Time taken to empty    part of the cistern completely filled.
3 4 Now, pipe A is open for 20 minutes.
= × 60 = 45 minutes
4 Similarly, pipe B is closed after (20 − x) minutes
53. (d) Let, the number of pipes that fill and empty the tank Then, we are given,
be x and 8 – x respectively. Then, 1 1
× 20 + (20 − x) = 1
8− x x 1 30 40
− =
6 8 6 2 20 − x
⇒ + =1
4(8 − x) − 3 x 1 3 40
⇒ =
24 6 80 + 60 − 3 x
⇒ =1
⇒ 32 − 7 x = 4 120
⇒ 7 x = 28 ⇒ x = 4 ⇒ 140 − 3 x = 120

Chapter 11.indd 17 6/5/2015 1:03:04 AM


11.18 Chapter 11

⇒ 140 − 120 = 3 x 2400 2400


− =8
20 x x + 10
⇒ 3 x = 20 ⇒ x =
3  10 
⇒ 2400   =8
∴ Pipe B is closed after (20 − x)minutes  x( x + 10) 
20 60 − 20 40 1 ⇒ x 2 + 10 x − 3000 = 0
= 20 − = =13 minutes
3 3 3 3 ⇒ ( x + 60)( x − 50) = 0
But x = −60 (not possible)
58. (b) Let, the time taken by pipe A = x minutes
∴ x = 50
∴ Time taken by pipe B = 3x minutes
Now, by given condition, Hence, the filling capacity of pump is 50 m3/minute
20
3 x − x = 20 ⇒ 2 x = 20 ⇒ x = = 10 minutes 61. (b) We have, length of cistern = 4.5 m
2
Breadth of cistern = 3 m
∴ Time taken by pipe A to fill the tank = 10 minutes Volume/capacity of cistern = 50 m3
Time taken by pipe B to fill the tank ∴ Height of cistern
= 3 x = 3 × 10 50 100
= = m
= 30 minutes 4.5 × 3 27

Now, time taken, when both are opened at the same Area to be lined with sheet lead
time, to fill the tank
100
1 1 3 +1 4 2 15 = 4.5 × 3 + 2 × (4.5 + 3)
+ = = = = minutes. 27
10 30 30 30 15 2 100
= 13.5 + 2 × × 7.5
27
59. (c) Let, the common ratio be x. 1500
= 13.5 +
Since, pipe A takes 2x hrs to fill or empty the tank. 27
Pipe B takes 3x hrs to fill or to empty the tank. 1864.5 3
= m
Pipe C takes 4x hrs to fill or to empty the tank. 27
Now, Pipe A and B can fill the tank in 3 hrs ∴ Weight of lead required

1 1 1 1864.5
∴ + = = × 27 = 1864.5 kg
2 x 3x 3 27
3+ 2 1 5×3 5 ≈ 1864.62 kg
⇒ = ⇒x= =
6x 3 6 2
62. (a) Let, the number of inlet pipes be x.
So, pipe A, B and C simultaneously take 5 hrs, 7.5 hrs
∴ Number of outlet pipes = 6 − x
and 10 hrs to fill or to empty the tank.
The fraction of the tank to be filled after three hours x (6 − x ) 1
It is given that, − =
9 6 9
1 1 1  1 1 1  1 1 1 
= + − + + − + + −  2 x − 3x × 6 + 3x 1
 5 7 . 5 10   7. 5 10 5   10 5 7.5  ⇒ =
18 9
 6 + 4 −3  4 + 3− 6  3+ 6 − 4 ⇒ 5 x − 18 = 2
= + + 
 30   30   30  ∴ x=4

7 1 5 13 2
= + + = 
30 30 30 30 5 63. (a) Let, the capacity of tank be x litres.
2 1 3
Hence, after 3 hrs tank is filled. ∴ x + 22 = x
5 5 4
3 1
60. (b) Let, the filling capacity of pump be x m3/min, ⇒ x − x = 22
4 5
Let, the emptying capacity of pump will be 11x
(x + 10) m3/min. ⇒ = 22
20
We are given ∴ x = 40 litres.

Chapter 11.indd 18 6/5/2015 1:03:09 AM


Pipes and Cisterns 11.19

64. (c) Capacity of tank = 25 litres And remaining water in final mixture = 40.5 litres
and time taken to fill the tank by both pipes = 5 ∴ Required ratio = 9.5:40.5 = 95:405 = 19:81.
minutes 67. (c) Capacity of tank = 2400 m3
Capacity Let, the empting capacity of pump = x m3/minutes
∴ Outlet flow rate =
Times taken
Then, filling capacity of pump = ( x − 10) m3 /min
25
= = 5 L/min We are given:
5
65. (a) Let, he filling capacity be x m3/min. 2400 2400
− =8
3
So, the discharging capacity = (x + 10) m /min x − 10 x
⇒ 2400 x − 2400( x − 10) = 8( x)( x − 10)
4800
Time taken to fill the tank = minutes and time ⇒ 2400 x − 2400 x + 24000 = 8 x 2 − 80 x
x
4800 ⇒ 8 x 2 − 80 x − 24000 = 0
taken to discharge the fuel from tank = minutes
x + 10 ⇒ x 2 − 10 x − 3000 = 0
We are given ⇒ ( x − 60)( x + 50) = 0
4800 4800 ⇒ x = 60 as x ≠ −50
− = 16
x x + 10
∴  Filling capacity of pump = 60 − 10 = 50 m3/min.
4800 x + 48000 − 4800 x
⇒ = 16
x( x + 0)
68. (d) Ship will get 3.75 × 5 − 12 = 6.75 tonnes of water in
⇒ 48000 = 16( x 2 + 10 x)
1 hr.
⇒ 3000 = x 2 + 10 x 60
Time to admit 60 tonnes of water = hrs
⇒ x 2 + 10 x − 3000 = 0 6.75
⇒ x 2 + 60 x − 50 x − 3000 = 0 40 × 6.75 1
∴ Required speed = = 4 Km/h
⇒ x( x + 60) − 50( x + 60) = 0 60 2
⇒ ( x − 50)( x + 60) = 0
69. (d) Capacity of water throwing pump
⇒ x = 50, − 60
12
∴ x = 50, because x cannot bee negative. = × 5.5 = 1.1 tonnes per 5.5 minutes
60
∴ Filling capacity of pump = 50 m3 /min.
Capacity of the leak to admit water = 2.25 tonnes per
66. (c) After 5 litres water taken out, remaining water = 50 – 5.5 minutes.
5 = 45 litres
In 5.5 minutes, net water accumulated by the leak
Since the water is replaced with wine.
= (2.25 − 1.1) = 1.15 tonnes
∴  Quantity of wine = 5 litres
Thus, to admit 92 tonnes of water, it will take
Now, in the next step same process is done and 10% of
5.5 440
water and 10% of wine is taken out from the mixture. = × 92 440
= min hrs
Therefore, in the mixture, remaining water = 45 – 4.5 = 1.15 60
40.5 litres and remaining wine = 5 – 0.5 = 4.5 litres Speed required for the ship to sail through safely
Now, 5 litres of wine is added. 77 × 60
= = 10.5 Km/h
So, total quantity of wine in final mixture 440
= 4.5 + 5 = 9.5 litres

Chapter 11.indd 19 6/5/2015 1:03:13 AM


11.20 I Chapter 11

Difficulty Level-2

1. (a) In the first 2 hrs, 1 1 1


 1 1 5. (d) In one minute, pipes A, B and C can fill + −
V 12 15 6
A and C can fill 2 ×    V = 1
 6 8 12 =– of the cistern, i.e., if all the three pipes are
60
 1 1 V
opened simultaneously, then the cistern can be
In 2 hrs, B and C can fill 2 ×    V =
 9 8 36 emptied in 60 minutes.
In 30 hrs, A and C will operate for 16 hrs and B and C 1 1 3
In one minute, pipes A and B can fill + =
for 14 hrs. 12 15 20
of the cistern.
16 V 14  V  15 3
⇒     = 68 In five minutes, = of the cistern will be filled.
2 12 2  36  20 4
⇒  V = 144 litres. 3
\ of the cistern can be emptied in
2. (c) Suppose that in one minute each pipe separately can 4
fill x, y, z, u, v parts of the tank. Then, 3
× 60 = 45 minutes.
40x = 1 4
10y + 10z + 10u = 1 6. (b) Let, the steady stream can fill the cistern in x hrs.
and   20y + 20z + 20v = 1 1
So, in one hour, stream can fill th of the cistern
30u + 30v = 1 x
1 1 Let, one (equal) hole can empty the cistern in y hrs.
⇒ y + z + u = ,y+z+v= 1
10 20 So, in one hour, one hole can empty th of the
1 y
and u + v = cistern.
30 12
Therefore, in one hour 12 holes can empty th of
11 y
⇒  2(u + v + z + y) = the cistern
60 1 12
Hence, portion emptied in one hour = 
x y
1
and  2x =   1 12 
20 Portion emptied in 4 hrs = 4    (1)
x y
1 11 14 7
∴ 2(x + y + z + u + v) =    Portion emptied by 10 holes in 8 hrs
20 60 60 30
7  1 10 
∴ x + y + z + u + v =  = 8    (2)
60 x y
60 4 x 1
∴ All together they take or 8 minutes to fill Equating equations (1) and (2) we get, 
7 7 y 8
the tank.
 1 n
1 1 1 Portion emitted by n holes in 2 hrs = 2    .
3.
(a) In one minute, + = of the tank can be  x y
20 30 12
filled. Now equating this equation with either equation (1) or
(2), we get n = 16.
The whole tank can be filled in 12 minutes.
Total time to fill the tank = 24 minutes (1 / 2) t (1 / 2) t
7. (b) Let, the time taken be t ⇒    = 1
\ Outlet tap is taking 12 minutes to empty half tank 40 24
at the speed of 50 litre/minute t t
⇒  = 1
80 48
\ Volume of half tank = 50 × 12 = 600 litres
80  48
\ Volume of full tank = 1200 litres. ⇒ t = = 30 hrs.
128
1 1 1 6+4−3 7 8. (d) Part filled in 10 minutes.
4.
(b) In one hour, + − = = of the
4 6 8 24 24 10 10 2
cistern can be filled.  =
24 20 60 3
\ Whole cistern could be filled in hrs, i.e., 3 hrs Remain = 1/3 filled by second
7
and 26 minutes appox. time = 1/ 3 × 60 = 20 minutes.

Chapter 11.indd 20 6/5/2015 1:03:15 AM


Pipes and Cisterns I 11.21

9. (d) The time taken by two pipes of fill the tank 139
1 112 112 Remaining part of the tank to be filled =
= = = 360
1 1 8 + 7 15
+
14 16 11
In 1 h, all the three pipes together will fill =
= 7 hrs 28 minutes 30
Now, total time taken = 7 hrs 28 minutes + 32 minutes Hence, the time taken to fill the remaining tank
= 8 hrs.  139   30 
=  = 1.0530 hrs.
Let, leakage can empty the tank in x hour.  360   11 
1 1 1 1
\ + − = Thus, the total time taken to fill the tank = 3.05 hrs.
14 16 x 8
⇒ x = 112 hrs. 13.
(d) Waste pipe alone empties the bath in

10.
(b) After 6 hrs remaining part was filled by 2 pipes. xy  xy 
1 +  minutes (1)
6 4 6 20 5 x + y  ( x + y) t 
\   = =
12 18 36 36 9 Here x = 10 minutes, y = 15 minutes and t = 4 minutes
4 Putting these values in Eq. (1), we get
\ Remaining part =
9
36 36 10 × 15  10 × 15 
A + B can fill the tank in = hhrs 1 + 
32 5 10 + 15  (10 + 15) × 4 
4
\ part filled in
9 10 × 15  10 × 15 
36 4 16 1
1 +  = 15 minutes.
25  25 × 4 
 = 3 h
5 9 5 5
1 1
\ Total time = 6 + 3 hhrs 14. (a) In one hour pipe A can fill =   part of the tank.
5 30

= 9 hrs 12 minutes. Therefore, in 36 hrs the tank will be completely filled.


24 Alternatively: Efficiency of pipe A = 3.33%
11.
(c) P1 and P2 can fill the tanks hhrs
5 Efficiency of pipe B = 2.22%
1 1  and Combined efficiency = 5.55%
[Q in 1 hrs these fill    part of tank]
 8 12 
12 Therefore in 2 hrs pipe A and B fill 5.55%
\ It takes hrs in filling half the tank
5 Thus to fill 100% tank, these pipe will take 36 hrs.
Far remaining half of the tanks P3 will open and this
will take 6 hrs 15. (d) Since, an inlet pipe is 7.2 times more efficient than an
 12  outlet pipe, therefore, in order to ensure that the tank
\ Supervisor has gone out for   6 hhrs
5  never overflows, we will need total 8 outlet pipes.
8
Now, l/3rd tank will fill in hhrs Thus, we need only 7 more (8 – 1 = 7) outlet pipes.
5
42 33
\ In remaining hhrs only th part of tank will 16. (c) Let, the pipe C alone empties the cistern in x minutes.
fill 5 60 10  15
A and B together can fill the cistern in 6
 1 33  6 10  15
\ empties part of tank = 1     
 3 60  60 minutes. Since, waste pipe was left open for 6
which is 10% of tank. 6
minutes, then in 6 minutes, part of the cistern
x
12.
(c) The part of the tank filled by A and B in first two hrs 6
will be emptied by waste pipe C. Now part of
31 1  1 1  x
=       the cistern would be filled by A and B together in 2
4  5 10   5 10 
x
The part of tank filled by C in first two hrs minutes. Therefore cistern will be filled in minutes.
3
 2  1  x
= 2     ∴   = 6 ⇒ x = 18 minutes.
 3   15  3

Chapter 11.indd 21 6/5/2015 1:03:16 AM


11.22 Chapter 11

17. (b) Let, x m3/minute be the filling capacity of the pump. So, we will accept the positive value of x (= 50).
Therefore, the draining capacity of the pump will be Therefore, draining capacity of the pump is
(x + 10) m3/minute. 50 + 10 = 60 m3/minute.
3600 18. (b) Suppose there are n inlet pipes and (15 − n) outlet
The time taken to fill the tank is .
x pipes.
3600 1 1 1
The time taken to drain the tank is . Therefore, (15 − n) − n × =
x + 10 6 8 6
We know that it takes extra 12 min to fill the tank than 15 − n n 1
⇒ − =
to drain it. 6 8 6
Therefore, 60 − 4n − 3n 1
⇒ =
24 6
3600 x + 36000 − 3600 x = 12( x 2 + 10 x)
24
⇒ 36000 = 12( x 2 + 10 x) ⇒ − 7 n + 60 =
6
⇒ 3000 = x 2 + 10 x ⇒ − 7 n = −60 + 4
⇒ x 2 + 10 x − 3000 = 0 ⇒ − 7 n = −56
⇒ ( x + 60)( x − 50) = 0 ∴ n=8
⇒ x = −60 or x = 50 ∴ Number of inlet pipes = 8.

Chapter 11.indd 22 6/5/2015 1:03:18 AM


CHAPTER

Time and Distance 12


INTRODUCTION Notes:

The terms ‘Time’ and ‘Distance’ are related to the speed of 1. If the time taken is constant, the distance travelled is
a moving object. proportional to the speed, that is, more the speed; more
the distance travelled in the same time.
Speed: We defi ne the speed of an object as the distance
covered by it in a unit time interval. It is obtained by 2. If the speed is constant, the distance travelled is
dividing the distance covered by the object, by the time it proportional to the time taken, that is, more the distance
takes to cover that distance. travelled; more the time taken at the same speed.
3. If the distance travelled is constant, the speed is inversely
Distance travelled proportional to the time taken, that is, more the speed;
Thus, Speed = .
Time taken less the time taken for the same distance travelled.

soMe Basic forMulae

Illustration 1 Calculate the speed of a train which covers a


Distance distance of 150 Km in 3 hrs
1. Speed =
Time Distance covered 150
2. Distance = Speed × Time Solution: Speed = = = 50 Km/h.
Time taken 3
Distance
3. Time = Illustration 2 How long does a train 100 m long running at
Speed the rate of 40 Km/h take to cross a telegraphic pole?
Units of Measurement Solution: In crossing the pole, the train must travel its own
Generally, if the distance is measured in kilometre, length.
we measure time in hrs and speed in kilometre per \ Distance travelled is 100 m.
hour and is written as km/h and if the distance is 40 × 1000 100
Speed = 40 Km/h = = m/s
measured in metre then time is taken in second and 60 × 60 9
speed in metre per second and is written as m/s. 100
\ Time taken to cross the pole = = 9 seconds
Conversion of Units 100/9
1000 m 5 Illustration 3 A train running at a speed of 90 Km/h passes
One kilometre/hr = = m/s. a pole on the platform in 20 s. Find the length of the train
60 × 60 s 18
in metres
18
\ One metre/s = km/h. Solution: Speed of the train = 90 Km/h
5
5
 5 = 90 × = 25 m/s.
Thus, x km/h =  x ×  m/s. 18
 18 
 18  \ Length of the train = Speed of the train
and, x m/s =  x ×  km/h. × time taken in crossing the pole
 5
= 25 × 20 = 500 m.

Chapter 12.indd 1 6/5/2015 3:10:45 PM


12.2 Chapter 12

soMe useful sHort-cut MetHoDs

1. (a) If A covers a distance d1 Km at s1 km/h 2. A person goes certain distance (A to B) at a speed


and then d2 Km at s2 km/h, then the average of s1 km/h and returns back (B to A) at a speed of
speed during the whole journey is given by s2 km/h. If he takes T hrs in all, the distance
s s (d + d ) between A and B is
Average speed = 1 2 1 2 km/h
s1d 2 + s2 d1  ss 
T 1 2 .
(b) If A goes from X to Y at s1 km/h and comes  s1 + s2 
back from Y to X at s2 km/h, then the average
speed during the whole journey is given by Explanation
2s1s2 Let the distance between A and B be d km.
Average speed =
s1 + s2
d
Time taken during onward journey = t1 = hrs.
s1
Explanation
(a) Time taken to travel d1 Km at s1 Km/h is d
Time taken during return journey = t2 = hrs.
d1 s2
t1 = hr \ Total time taken during the entire journey is
s1
d d d ( s1 + s2 )
Time taken to travel d2 Km at s2 Km/h is T = t1 + t2 = + =
s1 s2 s1s2
d2
t2 = hr
s2  ss 
\ d = T 1 2 .
d d   s1 + s2 
Total time taken = t1 + t2 =  1 + 2  hr Thus, the distance between A and B is
 s1 s2 
 ss 
s d +s d  = T 1 2 
=  1 2 2 1  hr  s1 + s2 
 s1s2 
Product of two speeds
Total distance covered = (d1 + d2)km. Therefore, = Total time taken × .
Sum of two speeds
Total distance covered
Average speed = Illustration 5 A boy goes to school with the speed of 3 Km
Total time taken an hour and returns with a speed of 2 Km/h. If he takes 5
s1s2 (d1 + d 2 ) hrs in all, fi nd the distance in Km between the village and
= Km/h (1)
( s1d 2 + s2 d1 ) the school
Solution: Here s1 = 3, s2 = 2 and T = 5.
(b) Let the distance from X to Y be d km
Take d1 = d2 = d in (1), we get \ The distance between the village and the school
2ds1s2 2s1s2  ss   3× 2 
Average speed = = . = T 1 2  = 5  = 6 km.
d ( s1 + s2 ) s1 + s2  s1 + s2   3+ 2 
Illustration 4 A ship sails to a certain city at the speed of
15 knots/hr and sails back to the same point at the rate of 3. If two persons A and B start at the same time
30 knots/hr. What is the average speed for the whole journey? from two points P and Q towards each other and
after crossing they take T1 and T2 hrs in reaching
Solution: Here s1 = 15 and s2 = 30.
Q and P, respectively, then
2s1s2 2 × 15 × 30
\ Average speed = = A's speed T2
s1 + s2 15 + 30 = .
B's speed T1
= 20 knots/hr

Chapter 12.indd 2 6/5/2015 3:10:47 PM


Time and Distance 12.3

Explanation Illustration 6 Nikita starts her journey from Delhi to


Let the total distance between P and Q be d km. Bhopal and simultaneously Nishita starts from Bhopal to
Delhi. After crossing each other they finish their remaining
Let the speed of A be s1 Km/h and that of B be s2 Km/h.
4
journey in 5 hrs and 9 hrs, respectively. What is Nishita’s
9
speed if Nikita’s speed is 36 Km/h?
Solution:
Since they are moving in opposite directions, their Nikita's speed T2 9 9
= = =
relative speed is (s1 + s2) Km/h. Nishita's speed T1 4 49
5
9 9
 d 
They will meet after   hr
 s1 + s2  81 9
= =
49 7
 d 
Distance travelled by A in   hr 7
 s1 + s2  ∴ Nishita’s speed = Nikita’s speed
9
 ds1  7
= PO =   km = × 36 = 28 Km/h.
 s1 + s2  9

 d  4. If a body travels d1, d2, d3, ..., dn metres with


Distance travelled by B in   hr different speeds s1, s2, s3, ... , sn m/s in time T1, T2, T3, ... ,
 s1 + s2 
Tn s, respectively, then the average speed of the
 ds2  body throughout the journey is given by
= QO =   km
 s1 + s2  Total distance travelled
Va =
Total time taken
Time taken by A to travel QO
d1 + d 2 + d3 + ... + d n
 ds2  =
T1 + T2 + T3 + ... + Tn
 
s +s
=  1 2 (If d1, d2, ..., dn and T1, T2, ..., Tn are known)
s1
s1T1 + s2T2 + s3T3 + ... + snTn
= T1 (given) (1) and, Va =
T1 + T2 + T3 + ... Tn
Time taken by B to travel PO
(If d1, d2, ..., dn and s1, s2, ..., sn are known)
 ds1 
 
s +s
=  1 2 Illustration 7 A car during its journey travels 40 minutes
s2 at a speed of 30 Km/h, another 50 minutes at a speed of
= T2 (given) (2) 60 Km/h and 1 hr at a speed of 30 Km/h. Find the average
speed of the car
Dividing Eq. (2) by Eq. (1), we get 40 50
Solution: Here T1 = ,T = , T = 1, s1 = 30,
s1/s2 T 60 2 60 3
= 2 s2 = 60, s3 = 30.
s2 /s1 T1
∴ Average speed of the car
2
 s1  T2 s T2 40 50
or,   = or, 1 = . 30 × + 60 × + 30 × 1
 s2  T1 s2 T1 sT +s T +s T
= 11 2 2 3 3 = 60 60
T1 + T2 + T3 40 50
A′s speed T2 + +1
∴ = . 60 60
B ′s speed T1 = 40 Km/h.

Chapter 12.indd 3 6/5/2015 6:21:37 PM


12.4 Chapter 12

a 7. A train travels a certain distance at a speed of


5. If the new speed is of the original speed, s1 km/h without stoppages and with stoppages, it
b
then the change in time taken to cover the same covers the same distance at a speed of s2 km/h.
The stoppage time per hour is given by
distance is given by
 s1 − s2   Difference of speed 
b    hr or,  
Change in time =  − 1 × original time.
a   s1   Speed without stoppages 

Illustration 8 By walking at four-fifths of his usual speed, Explanation


Mohan is 6 minutes late to his office. Find his usual time to Let the distance travelled be d km.
cover the distance
∴ Time taken by the train without stopping any where
a 4
Solution: Here change in time = 6 and = d
b 5 = hrs
s1
b  Also, time taken by the train with stoppages
We have change in time =  − 1 × original time
a  d
= hrs
Change in time s2
⇒ Original time =
b  d d s −s 
 − 1 Total stoppage time = − =  1 2  d hrs
a  s2 s1  s1s2 
6
= = 24 min  s1 − s2 
5 
 − 1  d
4   s1s2 
∴ Stoppage time per hour =
d
6. A body covers a distance d in time T1 with speed s2
s1, but when it travels with speed s2 covers the
same distance in time T2. s −s 
=  1 2  hrs
The following relations hold  s1 
Product of speed s s Difference of speed Illustration 10 Without stoppages, a train travels certain
= 1 = 2 =
d T2 T1 Difference of time distance with an average speed of 80 Km/h and with
Equating any two of the above, we can find the stoppages, it covers the same distance with an average
unknowns as per the given question. speed of 60 Km/h. How many minutes per hour the train
stops?
Illustration 9 Two bicyclists do the same journey by Solution: Here s1 = 80 and s2 = 60
travelling respectively, at the rates of 9 and 10 Km an hour. ∴ Stoppage time/hr
Find the length of the journey when one takes 32 minutes s −s 80 − 60 1
longer than the other = 1 2 = = hr
s1 80 4
Solution: Here change in speed = 10 – 9 = 1; product of
32 = 15 minutes
speed = 9 × 10 = 90 and difference of time =
60
Product of speed Difference of speed 8. (a) If a train overtakes a pole or a man or a
We have =
d Difference of time milestone, then the distance covered in
 Difference of time  overtaking is equal to the length of the train.
⇒ d = Product of speed ×  
 Difference of speed  (b) If a train overtakes a bridge or a tunnel or
a platform or another train, then the distance
32 covered is equal to the sum of the two lengths.
= 90 × = 48 km
60

Chapter 12.indd 4 6/5/2015 6:21:38 PM


Time and Distance 12.5

Illustration 11 A 600 m long train crosses a pole in 9 L1 + L2 135


seconds. What is the speed of the train in Km/h? ∴ The time taken = =
s1 + s2 5
Solution: Speed of the train 54 ×
135 × 18 18
Length of the train = = 9 s.
= 54 × 5
time taken in crossing the pole
Illustration 14 Two trains of length 110 metres and 90 m
600 600 18 are running on parallel lines in the same direction with a
= m/s = × = 240 Km/h.
9 9 5 speed of 35 Km/h and 40 Km/h, respectively. In what time
will they pass each other
Illustration 12 A train 130 m long passes a bridge in 21 Solution: Here L1 = 110 m, L2 = 90 m, s1 = 35 Km/h and
seconds moving with a speed of 90 Km/h. Find the length s2 = 40 Km/h
of the bridge 5
∴ s2 – s1 = 40 – 35 = 5 Km/h = 5 × m/s
Solution: We have speed of the train 18
L +L
length of the train + length of the bridge ∴ Time taken = 1 2
= s2 − s1
time taken in crossing the bridge
110 + 90 200 × 18
= =
5 130 + length of the bridge 5 5× 5
⇒ × 90 = 5×
18 21 18
= 144 seconds.
∴ Length of the bridge = 525 – 130 = 395 m
10. Two trains of lengths L1 m and L2 m run on
9. Relative Speed parallel tracks. When running in the same
(a) If two trains of lengths L1 Km and L2 km, direction, the faster train passes the slower one
respectively, are travelling in the same in T1 s, but when they are running in opposite
direction at s1 km/h and s2 km/h, respectively, directions with the same speeds as earlier, they
such that s1 > s2, then s1 – s2 is called their pass each other in T2 s.
relative speed and the time taken by the faster Then, the speed of the faster train
train to cross the slower train is given by L1 + L2  1 1 
=  +  m/s
2  T1 T2 
 L1 + L2 
  hr and the speed of the slower train
 s1 − s2 
L1 + L2  1 1 
(b) If two trains of length L1 Km and L2 km, =  −  m/s.
2  T1 T2 
respectively, are travelling in the opposite
directions at s1 km/h and s2 km/h, respectively,
then s1 + s2 is called their relative speed and Explanation
the time taken by the trains to cross each Let the speed of the faster train be s1 m/s and that of the
other is given by slower train be s2 m/s.
 L1 + L2  Total distance covered when the two trains cross each
  hr other = L1 + L2.
 s1 + s2 
When the two trains are running is the same direction,
their relative speed = (s1 – s2) m/s.
Illustrations 13 A train 135 m long is running with a speed L +L
∴ (s1 – s2) = 1 2 (1)
of 49 Km/h. In what time will it pass a man who is walking T1
at 5 Km/h in the direction opposite to that of the train? When the two trains are running in the opposite
Solution: Here L1 = 135, L2 = 0, s1 = 49 Km/h, s2 = 5 Km/h. directions, their relative speed = (s1 + s2) m/s.
5 L1 + L2
∴ s1 + s2 = 49 + 5 = 54 Km/h = 54 × m/s ∴ s1 + s 2 = (2)
18 T2

Chapter 12.indd 5 6/5/2015 6:21:39 PM


12.6 Chapter 12

Adding Eqs. (1) and (2), we get Therefore, speed of the faster train
L1 + L2 L1 + L2  75 15 
2s1 = + +
T1 T2  L1 + L2   T1 + T2   200 + 175   2 2 
=   =    75 15 
 2   T1T2   2  × 
1 1   2 2 
= (L1 + L2)  + 
 T1 T2  375 45 × 4
= × = 30 m/s.
2 75 × 15
 L + L  T + T 
or, s1 =  1 2   1 2  . Speed of slower train
 2   T1T2 
 75 15 
On subtracting Eq. (1) from Eq. (2), we get −
 L1 + L2   T1 − T2   200 + 175   2 2 
=   =
    75 15 
 1 1  2   T1T2   2  × 
2s2 = (L1 + L2)  −   2 2 
 T2 T1 
375 30 × 4
= × = 20 m/s
 L + L  T − T  2 75 × 15
or, s2 =  1 2   1 2  m/s.
 2   T1T2 
Therefore, 11. (a) A train starts from a place at s1 km/h and
another fast train starts from the same place
speed of the faster train after T hrs at s2 km/h in the same direction.
 L + L  T + T  Then, the distance from the starting place at
=  1 2   1 2  m/s. which both the trains will meet is given by
 2   T1T2 
and speed of the slower train  s1 × s2 × T 
  km.
 s2 − s1 
 L + L  T − T 
=  1 2   1 2  m/s. Also, the time after which the two trains will
 2   T1T2 
meet is given by
Notes:  s1T 
  hr.
If the two trains are of equal length that is, L1 = L2 = L (say),  s2 − s1 
then, (b) The distance between two stations A and B
T +T  is d km. A train starts from A to B at s1 km/h.
s1 = L  1 2  m/s and T hrs later another train starts from B to A at
 T1T2 
s2 km/h. Then, the distance from A, at which
both the trains will meet is given by
 T −T 
s2 = L  1 2  m/s.
 T1T2   d + s2T 
s1   km.
 s1 + s2 
Illustration 15 Two trains of lengths 200 m and 175 m run
on parallel tracks. When running in the same direction the Also, the time after which the two trains will
1 meet is given by
faster train crosses the slower one in 37 seconds. When
2
 d + s2T 
running in opposite directions at speeds same as their earlier   hr.
1  s1 + s2 
speeds, they pass each other completely in 7 seconds.
2
Find the speed of each train Illustration 16 A train starts from Mumbai at 10 a.m. with
75 a speed of 25 Km/h and another train starts from there on
Solution: We have L1 = 200, L2 = 175, T1 = and,
2 the same day at 3 p.m. in the same direction with a speed
15 of 35 Km/h. Find at what distance from Mumbai both the
T2 = trains will meet and fi nd also the time of their meeting
2

Chapter 12.indd 6 6/5/2015 3:10:50 PM


Time and Distance 12.7

Solution: Time from 10 a.m. to 3 p.m. = 5 hr.


12. Two trains start simultaneously from the stations
Distance of meeting point from Mumbai A and B towards each other with speeds s1 km/h
 s × s ×T  and s2 km/h, respectively. When they meet it is
=  1 2  Km. found that the second train had travelled d Km
 s2 − s1  more than the fi rst. Then, the distance between
 25 × 35 × 5  1 the two stations is given by
=   Km = 437 km
 35 − 25  2 s +s 
d  1 2  km.
Also, time of their meeting  s2 − s1 
 sT   25 × 5 
=  1  hr. =   hr Explanation
s −
 2 1s  35 − 25 
Let the distance between the two stations be x km. If
125 1 the fi rst train travels y Km then the second travels y +
= = 12 hr. after 3 P.M.
10 2 d km.
That is, 3.30 a.m. next day \ x = y + y + d = 2y + d.
Illustration 17 Chennai is at a distance of 560 Km from Since the time taken by both the trains is same
Mumbai. A train starts from Mumbai to Chennai at 6 a.m. y+d y
with a speed of 40 Km/h. Another train starts from Chennai \ =
s2 s1
to Mumbai at 7 a.m. with a speed of 60 Km/h. At what
distance from Mumbai and at what time will the two trains ⇒ s1 y + s1d = s2 y
be at the point of crossing? s1d
⇒ (s2 – s1)y = s1d or, y = .
Solution: Time from 6 a.m. to 7 a.m. = 1 hr. s2 − s1
Therefore, distance of meeting point from Mumbai  sd  d ( s1 + s2 )
\ x = 2 1 +d = km.
 d + s2T  s − s
 2 1 ( s2 − s1 )
= s1   km.
 s1 + s2  Illustration 18 Two trains start at the same time from Delhi
 560 + 60 × 1  and Rohtak and proceed towards each other at the rate of 75
= 40   = 248 km Km and 65 Km/h, respectively. When they meet, it is found
 40 + 60 
that one train has travelled 10 Km more than the other. Find
Also, time of their meeting
the distance between Delhi and Rohtak
 d + s2T  Solution: Distance between Delhi and Rohtak
=   hr
 s1 + s2 
s +s 
= d  1 2  km.
 560 + 60 × 1  31
 s1 − s2 
=   = hr
 40 + 60  5
 75 + 65 
= 6 hr 12 min. after 6 am = 10   km = 140 km.
 75 − 65 
That is, at 12.12 noon.

Practice Exercises

Difficulty level-1
(BaseD on MeMory)

1. The metro trains which travel at a uniform speed him, then what is the time-gap between one train that
between stations A and B run at a regular interval of overtakes him from behind and the immediately next
12 min. If Ajay, walking along the railway track at train overtaking him?
uniform speed, observes that every 10 minutes there is (a) 15 minutes (b) 16 minutes
a train coming from the opposite direction and passes (c) 13.5 minutes (d) None of these

Chapter 12.indd 7 6/5/2015 3:10:51 PM


12.8 Chapter 12

2. In covering a distance of 30 km, Amit takes 2 hrs more same time, then how many kilometres did the local train
than Suresh. If Amit doubles his speed, then he would travel in the time it took the express train to travel 600 km?
take 1 hour less than Suresh. Amit’s speed is: (a) 307.5 km (b) 900 km
(a) 5 Km/h (b) 7.5 Km/h (c) 1200 km (d) 1000 km
(c) 6 Km/h (d) 6.25 Km/h [Based on MAT, 2003]
[Based on MAT, 2003] 9. Two trains of equal length are running on parallel lines in
the same direction at 46 Km and 36 Km/h. The faster train
3. Two cars are running along the same road. The first one, passes the slower train in 36 s. The length of each train is:
which is running at the rate of 30 Km/h, starts 6 hrs ahead (a) 50 m (b) 80 m
of the second one, which is running at the rate of 50 Km/h. (c) 72 m (d) 82 m
How long will it take the second car to catch up with the [Based on MAT, 2003]
first one?
10. There are 20 poles with a constant distance between each
(a) 6 hrs (b) 9 hrs pole. A car takes 24 second to reach the 12th pole. How
(c) 12 hrs (d) 15 hrs much time will it take to reach the last pole?
[Based on MAT, 2003] (a) 25.25 seconds (b) 17.45 s
4. A train can travel 20% faster than a car. Both start from a (c) 35.75 s (d) 41.45 s
point A at the same time and reach point B 75 Km away [Based on MAT, 2003]
from A at the same time. On the way, however, the train 11. The jogging track in a sports complex is 726 m in
lost about 12.5 minutes while stopping at the stations. circumference. Suresh and his wife start from the same
Find the speed of the car in Km/h. point and walk in opposite directions at 4.5 Km/h and
(a) 50 Km/h (b) 55 Km/h 3.75 Km/h, respectively. They will meet for the first time in:
(c) 60 Km/h (d) 65 Km/h (a) 5.5 minutes (b) 6.0 minutes
[Based on MAT, 2003] (c) 5.28 minutes (d) 4.9 minutes
[Based on MAT, 2003]
5. A train can travel 20% faster than a car. Both start from the
point A at the same time and reach point B 75 Km away 12. A circular running path is 726 m in circumference. Two
from A at the same time. On the way, however, the train men start from the same point and walk in opposite
lost about 12.5 minutes while stopping at the stations. The directions @ 3.75 Km/h and 4.5 Km/h respectively. When
speed of the car is: will they meet for the first time?
(a) 50 Km/h (b) 55 Km/h (a) 5.5 minutes (b) 6.0 minutes
(c) 60 Km/h (d) 65 Km/h (c) 5.28 minutes (d) 4.9 minutes
[Based on MAT, 2003] [Based on MAT, 2002]
6. A starts 3 minutes after B for a place 4.5 Km distant B, 13.
It takes eight hrs for a 600 Km journey, if 120 Km is done
on reaching his destination, immediately returns and after by train and the rest by car. It takes 20 minutes more, if
walking a Km meets A. If A can walk 1 Km in 18 minutes, 200 Km is done by train and the rest by car. The ratio of
then what is B’s speed? the speed of the train to the speed of the car is:
(a) 5 Km/h (b) 4 Km/h (a) 4:3 (b) 3:4
(c) 6 Km/h (d) 3.5 Km/h (c) 3:2 (d) 2:3
[Based on MAT, 2003] [Based on MAT, 1999]
7. Wheels of diameters 7 cm and 14 cm start rolling 14.
Points A and B are 70 Km apart on a highway. One car
simultaneously from X and Y, which are 1980 cm apart, starts from A and another one from B at the same time. If
towards each other in opposite directions. Both of them they travel in the same direction, they meet in 7 hrs. But if
make same number of revolutions per second. If both of they travel towards each other, they meet in one hour. The
them meet after 10 s, the speed of the smaller wheel is: speeds of the two cars are:
(a) 22 cm/s (b) 44 cm/s (a) 45 and 25 Km/h (b) 70 and 10 Km/h
(c) 66 cm/s (d) 132 cm/s (c) 40 and 30 Km/h (d) 60 and 40 Km/h
[Based on MAT, 2003] [Based on MAT, 1999]
8. An express train travelled at an average speed of 100 15.
If train runs at 40 Km/h, it reaches its destination late by
Km/h, stopping for 3 minutes after every 75 Km. A local 11 minutes but if it runs at 50 Km/h it is late by 5 minutes
train travelled at a speed of 50 Km/h, stopping for 1 minute only. The correct time for the train to complete its
after every 25 Km. If the trains began travelling at the journey is

Chapter 12.indd 8 6/5/2015 3:10:52 PM


Time and Distance 12.9

(a) 13 minutes (b) 15 minutes 23. Dinesh travels 760 Km to his home, partly by train and
(c) 19 minutes (d) 21 minutes partly by car. He takes 8 hrs if he travels 160 Km by train
[Based on MAT, 2005] and the rest by car. He takes 12 minutes more if he travels
240 Km by train and the rest by car. The speeds of the
16.
A train of length 150 m takes 40.5 second to cross a tunnel train and the car respectively are:
of length 300 m. The speed of the train (in Km/h) is: (a) 80 Km/h, 100 Km/h (b) 100 Km/h, 80 Km/h
(a) 13.33 (b) 26.67 (c) 120 Km/h, 100 Km/h (d) 100 Km/h, 120 Km/h
(c) 40 (d) 400 [Based on MAT, 2001]
[Based on MAT, 2005] 24.
In a flight of 3000 km, an aircraft was slowed down by
17.
A train 100 m long passes a bridge at the rate of 72 Km/h bad weather. Its averae speed for the trip was reduced by
in 25 s. The length of the bridge is: 100 Km/h and the time increased by one hour. Find the
original duration of the flight.
(a) 150 m (b) 400 m
(a) 5 hrs (b) 6 hrs
(c) 300 m (d) 200 m
(c) 4 hrs (d) 10 hrs
[Based on MAT, 1999]
[Based on MAT, 2001]
18.
A train 110 m in length travels at 60 Km/h. How much 25.
The diameter of a cycle wheel is 70 cm. A cyclist takes
time does the train take in passing a man walking at 30 hrs to reach a destination at the speed of 22 Km/h.
6 Km/h against the train? How many revolutions will the wheel make during this
(a) 6 seconds (b) 12 seconds journey?
(a) 3 million (b) 3 lakhs
(c) 16 seconds (d) 18 seconds
(c) 4 lakhs (d) None of these
[Based on MAT, 1999]
[Based on MAT, 2001]
19.
Two rackets approach each other, one at 42,000 mph and 26. Walking at three-fourths of his usual pace, a man reaches
the other at 18,000 mph. They start 3256 miles apart. How his office 20 minutes late. Find out his usual time.
far are they apart (in miles) 1 minutes before impact?
(a) 1 hr (b) 2 hrs
(a) 1628 (b) 1000
(c) 3 hrs (d) 4 hrs
(c) 826 (d) 1200 [Based on IIFT, 2003]
[Based on MAT, 2000]
27. Two trains traveling in the same direction at 40 Km/h and
20.
Wheels of diameters 7 cm and 14 cm start rolling 22 Km/h completely pass each other in 1 min. If the length
simultaneously from X and Y, 1,980 cm apart, towards of the first train is 125 m, what is the length of the second
each other in opposite directions. Both of them make train?
same number of revolutions per second. If both of (a) 125 m (b) 150 m
them meet after 10 s, find the speed of the smaller (c) 175 m (d) 200 m
wheel.
28.
Raja was on a long distance trip. He travelled by air 2/5 of
(a) 22 cm/s (b) 44 cm/s the distance which was 1200 km. Then, he hired a car and
(c) 66 cm/s (d) 132 cm/s travelled 1/3 of the whole trip. Thereafter, he completed
[Based on MAT, 2001] the rest of the journey by train. Calculate the distance that
Raja travelled by train.
21.
Two trains of equal length are running on parallel lines (a) 480 km (b) 800 km
in the same direction @ 46 Km and 36 Km/h. The faster
(c) 1600 km (d) 1800 km
train passes the slower train in 36 s. The length of each
train is: [Based on I.P. Univ., 2002]

(a) 50 mts. (b) 80 mts. 29. A train T1 starts from Ahmedabad to Mumbai at 7 a.m.
and reaches at 12 noon. A second train T2 starts at 7 a.m.
(c) 72 mts. (d) 82 mts.
from Mumbai reaches Ahmedabad at 1 p.m. When did the
[Based on MAT, 2001]
two trains cross each other?
22. A car travelled 80 Km at 16 Km/h. What should be its (a) 10.13 a.m. (b) 10.00 a.m.
average speed for next 4 hrs so that the total average speed (c) 9.43 a.m. (d) 9.35 a.m.
becomes 14 Km/h?
30. A circular playground has an area of 616 sq. m. What time
(a) 15 Km/h (b) 11.5 Km/h will it take for a runner to run around the circular ground
(c) 15.4 Km/h (d) 12 Km/h at the speed of 22 Km/h?

Chapter 12.indd 9 6/5/2015 3:10:52 PM


12.10 Chapter 12

(a) 4 hrs (b) 3 hrs (a) 16:15 (b) 17:18


(c) 2 hrs (d) None of these (c) 19:20 (d) None of these
[Based on I.P. Univ., 2002] [Based on NMAT, 2006]
31. A bus covered a distance of 250 km, partly at an average 38. A and B walk around a circular course 35 Km in
speed of 40 Km/h and partly at 60 Km/h. If the total time circumference starting together from the same point. If
taken is 5 hrs, then the distance covered at 40 Km/h is: they walk at the speed of 4 Km/h and 5 Km/h, respectively
(a) 130 km (b) 120 km in the same direction, when will they meet?
(c) 100 km (d) None of these (a) 2 minutes (b) 1 minute
32. Two trains, 130 m 110 m long, are going in the same (c) 3 minutes (d) None of these
direction. The faster train takes one minute to pass [Based on NMAT (2006]
the other completely. If they are moving in opposite 39. A train running at the speed of 20 m/s crosses a pole in
directions, they pass each other completely in 3 s. Find 24 second less than the time it requires to cross a platform thrice
the speed of each train. its length at the same speed. What is the length of the train?
(a) 38 m/s, 36 m/s (b) 42 m/s, 38 s (a) 270 m (b) 340 m
(c) 36 m/s, 42 m/s (d) None of these (c) 180 m (d) None of these
[Based on MAT, 2008] [Based on IRMA, 2009]

33. Two identical trains A and B running is opposite directions 40.


Two cars, an Alto and a Swift, start at the same time in
at same speed take 2 minutes to cross each other opposite directions from two distinct points P and Q.
completely. The number of bogies of A are increased from Starting from P, the Alto reaches Q in 6 hrs 20 minutes
12 to 16. How much more time would they now require to and the Swift starting from Q, reaches Pin 4 hrs 45
cross each other? minutes. What is the speed of the Swift, if the speed of the
Alto is 60 Km/h?
(a) 40 s (b) 50 s
(a) 110 Km/h (b) 100 Km/h
(c) 60 s (d) 20 s
(c) 90 Km/h (d) 80 Km/h
[Based on SNAP, 2007] [Based on JMET, 2011]
34.
A hare pursued by a round is 30 m before the hound at 41.
Two boys A and B start at the same time to ride from Delhi
starting. Whilst the hare takes 4 leaps the hound takes 3. to Meerut, 60 Km away. A travels 4 Km an hour slower
1 1 than B. B reaches Meerut and at once turns back meeting
In one leap the hare goes 1 and the hound 2 m. How
2 2 A 12 Km from Meerut. The rate of A was:
far will the hare have gone when the hound will catch the
(a) 4 Km/h (b) 8 Km/h
hare?
(c) 12 Km/h (d) 16 Km/h
(a) 60 m (b) 120 m
[Based on FMS, 2011]
(c) 90 m (d) 100 m
[Based on ATMA, 2008] 42.
A train, an hour after starting, meets with an accident
which detains it for a half hour, after which it proceeds
35. A train 270 m long is moving at a speed of 25 Km/h. It
will cross a man coming from the opposite direction at a 3 1
at
of its former rate and arrives 3 hrs late. Had the
speed of 2 Km/h in: 4 2
(a) 13 seconds (b) 36 seconds accident happened 90 Km farther along the line, it would
(c) 41 seconds (d) 20 seconds have arrived only 3 hrs late. The length of the trip in
[Based on ATMA, 2005] kilometres was:
36. Two trains start simultaneously (with uniform speeds) (a) 400 (b) 465
from two stations 270 Km apart, each to the opposite (c) 600 (d) 640
1 [Based on FMS, 2011]
station; they reach their destinations in 6 hrs and 4 hrs
4 43.
A 500 g stone was dropped from the roof of a building.
after they meet. The rate at which the slower train travels
is (Km/h)? What is the height of the building if the stone reached the
ground in 4 s?
(a) 24 (b) 30
(a) 108.4 m (b) 98.4 m
(c) 25 (d) 16
(c) 88.4 m (d) 78.4 m
[Based on ATMA, 2005]
[Based on FMS, 2009]
37. A dog takes 4 leaps for every five leaps of a hare, but three
leaps of dog are equal to four leaps of hare. Compare their 44.
The radius of the wheel of a vehicle is 70 cm. The wheel
speeds. makes 10 revolutions in 5 s. The speed of the vehicle is:

Chapter 12.indd 10 6/5/2015 3:10:52 PM


Time and Distance 12.11

(a) 29.46 Km/h (b) 31.68 Km/h 52.


A train covers a distance in 50 minutes, if it runs at a speed
(c) 36.25 Km/h (d) 32.72 Km/h of 48 Km/h on an average. The speed at which the train must
[Based on FMS, 2006] run to reduce the time of journey to 40 minutes, will be:
45.
Two cyclists start from the same place in opposite (a) 50 Km/h (b) 55 Km/h
directions. One goes towards North at 18 Km/h and the (c) 60 Km/h (d) 70 Km/h
other goes towards South at 20 Km/h. What time will they [Based on MAT, 2000]
take to be 47.5 Km apart? 53.
Suresh travelled 1,200 Km by air which formed (two-
1 1 fifths) of his trip. One-third of the whole trip, he travelled
(a) 2 hhrs (b) 1 hhrs
4 4 by car and the rest of the journey he performed by train.
The distance travelled by train was:
1
(c) 2 hrs 23 minutes (d) 23 hhrs (a) 1, 600 km (b) 800 km
4
[Based on FMS, 2006]
(c) 1,800 km (d) 480 km
[Based on MAT, 2000]
46.
Two trains running in the same direction at 40 Km/h and
22 Km/h completely pass one another in 1 min. If the 54.
An express train travels 299 Km between two cities.
length of the first train is 125 m, the length of the second During the first 111 Km of the trip, the train traveled
train is: through mountainous terrain. The train traveled 10 Km/h
slower through mountainous terrain than through level
(a) 125 m (b) 150 m
terrain. If the total time to travel between two cities was
(c) 175 m (d) 200 m 7 hrs, what is the speed of the train on level terrain?
[Based on FMS, 2005]
(a) 56 Km/h (b) 55 Km/h
47.
A man covers a certain distance on scooter. Had he moved
(c) 47 Km/h (d) 88 Km/h
3 Km/h faster, he would have taken 40 minutes less. If
[Based on MAT (Feb), 2011]
he had moved 2 Km/h slower, he would have taken 40
minutes more. The distance (in km) is: 55.
Two goods trains each 500 m long are run­ning in opposite
(a) 20 (b) 36 directions, on parallel tracks. Their speeds are 45 Km/h
(c) 37.5 (d) 40 and 30 Km/h respectively. Find the time taken by the
slower train to pass the driver of the faster one.
[Based on FMS, 2005]
48.
If a student walks from his house to school at 5 Km/h, he (a) 60 s (b) 48 s
is late by 30 minutes. However, if he walks at 6 Km/h, he (c) 24 s (d) 12 s
is late by 5 minutes only. The distance of his school from [Based on MAT, 2000]
his house is:
56.
X express which goes from Hyderabad to Chennai, leaves
(a) 2.5 km (b) 3.6 km Hyderabad at 5:30 am and travels at a constant speed of
(c) 5.5 km (d) 12.5 km 50 Km/h towards Nalgonda which is 100 Km away. At
[Based on FMS (MS), 2006] 6:00 am, Y express leaves from Nalgonda for Hyderabad
49.
Two rockets approach each other, one at 42,000 mph and at a constant speed of 40 Km/h. At 6:30 am Mr Shah, the
the other at 18,000 mph. They start 3256 miles apart. How Control Officer realizes that both the trains are on the
far are they apart (in miles) 1 minute before impact? same track. How much time does Mr Shah have to avert
(a) 1628 (b) 1000 the accident?
(c) 826 (d) 1200 (a) 20 min (b) 30 min
[Based on MAT, 2000] (c) 25 min (d) 15 min
50.
A certain distance is covered at a certain speed. If half of [Based on MAT (Feb), 2011]
this distance is covered in double the time, the ratio of the 57.
A man covers a certain distance on a toy train. If the
two speeds is: train moved 4 Km/h faster, it would take 30 minutes
(a) 4:1 (b) 1:4 less. If it moved 2 Km/h slower, it would have taken
(c) 2:1 (d) 1:2 20 minutes more. Find the distance.
[Based on MAT, 2000] (a) 60 km (b) 45 km
51.
Ram travels a certain distance at 3 Km/h and reaches (c) 30 km (d) 20 km
15 minutes late. If he travels at 4 Km/h, he reaches 15 [Based on MAT (Dec), 2010, 2008]
minutes earlier. The distance he has to travel is:
58.
The average speed of a train is 20% less on the return
(a) 4.5 km (b) 6 km journey than on the onward journey. The train halts for
(c) 7.2 km (d) 12 km half an hour at the destination station before starting on
[Based on MAT, 2000] the return journey. If the total time taken for the to and

Chapter 12.indd 11 6/5/2015 3:10:53 PM


12.12 Chapter 12

fro journey is 23 hrs, covering a distance of 1000 km, the how much time would it take to cover a distance of 300
speed of the train on the return journey is: Km using this speed?
(a) 60 Km/h (b) 40 Km/h 1
(a) 5 hrs (b) 5 hrs
h
(c) 50 Km/h (d) 55 Km/h 2
[Based on MAT (Dec), 2010] 1
(c) 6 hrs (d) 6 hhrs
59.
Two trains move from station A and station B towards 2
each other at the speed of 50 Km/h and 60 Km/h. At the [Based on MAT (Sept), 2010]
meeting point, the driver of the second train felt that the 66.
Mohan and Puran are running towards each other, each
train has covered 120 Km more. What is the distance one from his own house. Mohan can reach Puran’s house
between A and B? in 25 minutes of running, which is half the time it takes
(a) 1320 km (b) 1100 km Puran to reach Mohan’s house. If the two started to run at
(c) 1200 km (d) 960 km the same time, how much more time will it take Puran to
[Based on MAT (Dec), 2010] reach the middle than Mohan?
60.
A pilot flies an aircraft at a certain speed for a distance of (a) 35 minutes (b) 25 minutes
800 km. He could have saved 40 minutes by increasing (c) 12.5 minutes (d) 50 minutes
the average speed of the plane by 40 Km/h. Find the [Based on MAT (May), 2010]
average speed of the aircraft. 67.
Two cars A and B are travelling on the same road towards
(a) 200 Km/h (b) 300 Km/h each other. If car A is travelling at a speed of 120 Km/h
(c) 240 Km/h (d) None of these and car B is travelling 15% slower than A, how much time
[Based on MAT (Dec), 2010] will it take the cars to meet, if the initial distance between
the two is 668.4 Km and car A started to drive one and a
61.
An Auto travels 10 Km/h faster than a Scooty for a journey
half hour before car B started?
of 1000 km. The Scooty takes 5 hrs more than the Auto.
Find the speed of the Scooty. (a) 2 hrs and 12 min (b) 2 hrs
(a) 40 Km/h (b) 70 Km/h (c) 1 hour and 30 min (d) 3 hrs and 15 min
[Based on MAT (May), 2010]
(c) 50 Km/h (d) None of these
68.
A train travelling at 100 Km/h overtakes a motorbike
[Based on MAT (Dec), 2010] travelling at 64 Km/h in 40 s. What is the length of the
62.
A train running at 7/11 of its own speed reached a place in train in metres?
22 hrs. How much time could be saved if the train runs at (a) 400 (b) 1822
its own speed? (c) 1777 (d) 1111
(a) 7 hrs (b) 8 hrs [Based on MAT (May), 2010]
(c) 14 hrs (d) 16 hrs 69.
A train travels a distance of 600 Km at a constant
speed. If the speed of the train is increased by
[Based on MAT (Sept), 2010]
5 Km/h, the journey would take 4 hrs less. Find the speed
63.
A train overtakes two persons walking along a railway of the train.
track. The first one walks at 4.5 Km/h. The other one walks (a) 100 Km/h (b) 25 Km/h
at 5.4 Km/h. The train needs 8.4 and 8.5 second respectively (c) 50 Km/h (d) None of these
to overtake them. What is the speed of the train if both the
[Based on MAT (May), 2010]
persons are walking in the same direction as the train?
70.
A plane left 30 minutes later than its scheduled time to
(a) 66 Km/h (b) 72 Km/h
reach its destination 1500 Km away. In order to reach
(c) 78 Km/h (d) 81 Km/h in time it increases its speed by 250 Km/h. What is its
[Based on MAT (Sept, May), 2010] original speed?
64.
A train travelling at 48 Km/h completely crosses another (a) 1000 Km/h (b) 750 Km/h
train having half its length and travelling in opposite (c) 600 Km/h (d) 800 Km/h
direction at 42 Km/h in 12 s. It also passes a railway [Based on MAT (Feb), 2010, 2006]
platform in 45 s. The length of the platform is: 71.
I have to reach a certain place at a certain time and find
(a) 400 m (b) 450 m that I shall be 15 minutes too late, if I walk at 4 Km an
(c) 560 m (d) 600 m hour and 10 minutes too soon, if I walk at 6 Km an hour.
How far have I to walk?
[Based on MAT (Sept), 2010 (May), 2008] (a) 25 km (b) 5 km
65.
A car takes 15 minutes less to cover a distance of 75 km. (c) 10 km (d) None of these
If it increases its speed by 10 Km/h from its usual speed, [Based on MAT (Feb), 2009]

Chapter 12.indd 12 6/5/2015 3:10:53 PM


Time and Distance 12.13

72.
A train travels a distance of 300 Km at a constant speed. 78.
A man sitting in a train travelling at the rate of 50 Km/h
If the speed of the train is increased by 5 Km an hour, the observes that it takes 9 second for a goods train travelling
journey would have taken 2 hrs less. The original speed of in the opposite direction to pass him. If the goods train is
the train was: 187.5 m long, find its speed.
(a) 25 Km/h (b) 20 Km/h (a) 25 Km/h (b) 40 Km/h
(c) 28 Km/h (d) 30 Km/h (c) 35 Km/h (d) 36 Km/h
[Based on MAT (Feb), 2009] [Based on MAT (Sept), 2009]
73.
A man takes 6 hrs 30 minutes in going by a cycle and 79.
Two cyclists start on a circular track from a given point
coming back by scooter. He would have lost 2 hrs 10 but in opposite directions with speeds of 7 m/s and 8 m/s
minutes by going on cycle both ways. How long would it respectively. If the circumference of the circle is 300 m,
take him to go by scooter both ways? after what time will they meet at the starting point?
1 (a) 20 s (b) 100 s
(a) 5 hrs (b) 5 hrs
2 (c) 300 s (d) 200 s
[Based on MAT (Sept), 2009]
1 1
(c) 4 hrs (d) 3 hrs 80.
A car travels a distance of 45 Km at the speed of 15 Km/h.
3 2
It covers the next 50 Km of its journey at the speed of 25
[Based on MAT (Dec), 2009] 1
Km/h and the last 25 Km of its journey at the speed of 8
74.
The distance between two stations A and B is 450 km. 3
Km/h. What is the average speed of the car?
A train starts from A and moves towards B at an average
speed of 15 Km/h. Another train starts from B, 20 minutes (a) 40 Km/h (b) 24 Km/h
earlier than the train at A and moves towards A at an (c) 15 Km/h (d) 18 Km/h
average speed of 20 Km/h. How far from A will the two [Based on MAT (May), 2009]
trains meet? 81.
A car travels a distance of 170 Km in 2 hrs partly at a
(a) 190 km (b) 320 km speed of 100 Km/h and partly at 50 Km/h. The distance
(c) 180 km (d) 260 km travelled at speed of 50 Km/h is:
[Based on MAT (Dec), 2009] (a) 50 Km (b) 40 Km
75.
The ratio between the speeds of Ramesh and Suresh is (c) 30 Km (d) 60 Km
3:4. If Ramesh takes 30 minutes more than Suresh to [Based on MAT (May), 2009]
cover a distance, then the actual times taken by Ramesh 82.
A train is scheduled to cover the distance between two
and Suresh respectively are: stations 46 Km apart in one hour. If it travels 25 Km at
1 1 1 a speed of 40 Km/h, find the speed for the remaining
(a) 2 hrs and 1 hhrs (b) 2 hhrs and 1 hrs
h
2 2 2 journey to complete it in the scheduled time.
(a) 66 Km/h (b) 56 Km/h
1
(c) 2 hrs and 1 hr (d) 2 hhrs and 1 hr (c) 46 Km/h (d) 36 Km/h
2 [Based on MAT (May), 2009]
[Based on MAT (Dec), 2009]
83.
A train started from station A and proceeded towards
76.
Ramu sees a train passing over a 1 Km long bridge. The station B at a speed of 48 Km/h. 45 minutes later, another
length of the train is half that of the bridge. If the train train started from station B and proceeded towards station
clears the bridge in 2 min, the speed of the train is: A at 50 Km/h. If the distance between the two stations is
(a) 50 Km/h (b) 43 Km/h 232 km, at what distance from station A will the trains
(c) 45 Km/h (d) None of these meet?
[Based on MAT (Dec), 2009] (a) 108 Km (b) 144 Km
77.
A motor cyclist goes from Mumbai to Pune, a distance of (c) 132 Km (d) None of these
192 km, at an average speed of 32 Km/h. Another man [Based on MAT (May), 2009]
1 84.
A railway passenger counts the telegraph poles on the
starts from Mumbai by car, 2 hhrs after the first and
2 rail road as he passes them. The telegraph poles are at a
reaches Pune half an hour earlier. What is the ratio of the distance of 50 m. What will be his count in 4 hrs if the
speeds of the motor cycle and the car? speed of the train is 45 Km/h?
(a) 10:27 (b) 1:3 (a) 2500 (b) 600
(c) 1:2 (d) 5:4 (c) 3600 (d) 5000
[Based on MAT (Dec), 2009] [Based on MAT (Feb), 2009]

Chapter 12.indd 13 6/5/2015 3:10:53 PM


12.14 Chapter 12

85.
A car driver, driving in a fog, passes a pedestrain who (a) 40 Km (b) 153 Km
was walking at the rate of 2 Km/h in the same direction. (c) 36 Km (d) 28 Km
The pedestrain could see the car for 6 minutes and it was [Based on MAT (Sept), 2008]
visible to him up to a distance of 0.6 km. What was the
speed of the car? 93.
A train consists of 12 bogies, each bogie is 15 m long.
(a) 15 Km/h (b) 30 Km/h The train crosses a telegraph post in 18 s. Due to some
problem, two bogies were detached. The train now crosses
(c) 20 Km/h (d) 8 Km/h
a telegraph post in:
[Based on MAT (Feb), 2009]
(a) 15 seconds (b) 12 seconds
86.
A cyclist moving on a circular track of radius 100 m
(c) 18 seconds (d) 20 seconds
completes one revolution in 2 min. What is the approxi­
mate speed of the cyclist? [Based on MAT (Sept), 2008]

(a) 200 m/min (b) 314 m/min 94.


Two buses, one moving towards North and the other
(c) 300 m/min (d) 900 m/min towards East, leave the same place at the same time. The
speed of one of them is greater than that of the other by 5
[Based on MAT (Feb), 2009]
Km/h. At the end of 2 hrs, they are at a distance of 50 Km
87.
A train 110 m in length travels at 60 Km/h. How much from each other. The speed of the bus going slower is
time does the train take in passing a man walking at
(a) 15 Km/h (b) 12 Km/h
6 Km/h against the train?
(c) 10 Km/h (d) 20 Km/h
(a) 6 seconds (b) 12 seconds
[Based on MAT (Sept), 2008]
(c) 16 seconds (d) 18 seconds
[Based on MAT, 1999] 95.
There is a ring road connecting points A, B, C and D.
88.
Alok walks to a viewpoint and returns to the starting point The road is in a complete circular form but having
by his car and thus takes a total time of 6 hrs 45 min. He several approach roads leading to the centre. Exactly in
would have gained 2 hrs by driving both ways. How long the center of the ring road there is a tree which is 20
would it have taken for him to walk both ways? Km from point A on the circular road. You have taken a
round of circular road starting from point A and finish
(a) 7 hrs 45 minutes (b) 8 hrs 45 minutes at the same point after touching points B, C and D. You,
(c) 5 hrs 30 minutes (d) None of these then drive 20 Km interior towards the tree from point A
[Based on MAT (Feb), 2009 (May), 2008] and from there, reach somewhere in between B and C on
89.
A train 100 m long passes a bridge at the rate of the ring road. How much distance do you have to travel
72 Km/h in 25 s. The length of the bridge is: from the tree to reach the point between B and C on the
ring road?
(a) 150 m (b) 400 m
(a) 80 Km (b) 15 m
(c) 300 m (d) 200 m
(c) 20 Km (d) 40 m
[Based on MAT, 1999]
[Based on MAT (Sept), 2008]
90.
A train passes a station platform in 36 second and a man
standing on the platform in 20 s. If the speed of the train is 96.
A passenger train takes two hrs less for a jour­ney of 300
54 Km/h, find the length of the platform. Km if its speed is increased by 5 Km/h from its normal
(a) 225 m (b) 235 m speed. The normal speed is:
(c) 230 m (d) 240 m (a) 35 Km/h (b) 50 Km/h
[Based on MAT (Dec), 2008] (c) 25 Km/h (d) 30 Km/h
91.
Two trains, 130 m and 110 m long, are going in the [Based on MAT, 1999]
same direction. The faster train takes one minute to pass 97.
A train 75 m long overtook a person who was walking at
the other completely. If they are moving in opposite 1
directions, they pass each other completely in 3 s. Find the rate of 6 Km/h and passes him in 7 s. Subsequently,
2
the speed of the faster train. 3
(a) 38 m/s (b) 46 m/s it overtook a second person and passes 3 him in 6 s. At
4
what rate was the second person travelling?
(c) 42 m/s (d) None of these
[Based on MAT (Dec, Sept, Feb), 2008] (a) 4 Km/h (b) 1 Km/h
(c) 2 Km/h (d) 5 Km/h
92.
A man walks half of the journey at 4 Km/h by cycle, does
[Based on MAT (May), 2008]
one-third of journey at 12 Km/h and rides the remainder
journey in a horse cart at 9 Km/h, thus completing the whole 98.
Train A leaves Ludhiana for Delhi at 11 am, running at
journey in 6 hrs and 12 min. The length of the journey is: the speed of 60 Km/h. Train B leaves Ludhiana for Delhi

Chapter 12.indd 14 6/5/2015 3:10:54 PM


Time and Distance 12.15

by the same route at 2 pm on the same day, running at the 1


speed of 72 Km/h. At what time will the two trains meet (a) 9 hrs (b) 9 hrs
2
each other?
1
(a) 5 a.m. on the next day (c) 8 hrs (d) 7 hrs
2
(b) 2 a.m. on the next day [Based on MAT (May), 2007]
(c) 5 p.m. on the next day
(d) None of the above 105.
A tower is 61.25 m high. A rigid body is dropped from
its top and at the same instant another body is thrown
[Based on MAT (May), 2008]
upwards from the bottom of the tower with such a velocity
99.
A person travels 285 Km in 6 hrs in two stages. In the that they meet in the middle of the tower. The velocity of
first part of the journey, he travels by bus at the speed of projection of the second body is:
40 km/ hr. In the second part of the journey, he travels by (a) 20 m/s (b) 25 m/s
train at the speed of 55 Km/h. How much distance did he (c) 24.5 m/s (d) None of these
travel by train?
[Based on MAT (Dec), 2006]
(a) 205 km (b) 165 km
106.
Two trains are 2 Km apart and their lengths are 200 m and
(c) 145 km (d) 185 km
300 m. They are approaching towards each other with a
[Based on MAT (Dec, May), 2007]
speed of 20 m/s and 30 m/s, respectively. After how much
100.
Two men A and B walk from P to Q at a distance of 21 Km time will they cross each other?
at the rates of 3 and 4 Km an hour, respectively. B reaches (a) 50 seconds (b) 100 seconds
Q and returns immediately and meets A at R. The distance (c) 25/3 seconds (d) 150 seconds
from P to R is:
[Based on MAT (Dec), 2006]
(a) 14 km (b) 16 km
107.
A jet plane is rising vertically with a velocity of 10 m/s. It
(c) 20 km (d) 18 km
has reached a certain height when the pilot drops a coin,
[Based on MAT (Dec), 2007]
which takes 4 second to hit the ground. Assuming that
101.
Two trains are moving in opposite directions at speeds there is no resistance to the motion of the coin, the height
of 60 Km/h and 90 Km/h their lengths are 1.10 Km and of the plane and the velocity of the coin on impact with
0.9 Km respectively, the time taken by the slower train to the ground are:
cross the faster train in seconds is: (a) 38.4 m, 29.2 m/s (b) 38.4 m, 28.7 m/s
(a) 36 (b) 49 (c) 26.5 m, 13.5 m/s (d) None of the above
(c) 45 (d) 48 [Based on MAT (Dec), 2006]
[Based on MAT (Sept), 2007]
108.
A train starts from Delhi at 6:00 am and reaches Ambala
102.
Excluding stoppages, the speed of a bus is 54 Km/h and Cantt at 10 a.m. The other train starts from Ambala Cantt
including stoppages, it is 45 Km/h. For how many minutes at 8 am and reaches Delhi at 11:30 a.m. If the distance
does the bus stop per hour? between Delhi and Ambala Cantt is 200 km, then at what
(a) 12 (b) 10 time did the two trains meet each other?
(c) 9 (d) 20 (a) 8:56 a.m. (b) 8:46 a.m.
[Based on MAT (May), 2007] (c) 7:56 a.m. (d) 8:30 a.m.
[Based on MAT (Dec), 2006]
103.
Rampur is 100 Km from Sitapur. At 3 pm, Bharat Express
leaves Rampur for Sitapur and travels at a constant speed 109.
A train with 90 Km/h crosses a bridge in 36 s. Another
of 30 Km/h. One hour later, Laxman Mail leaves Sitapur train 100 m shorter crosses the same bridge at 45 Km/h.
for Rampur and travels at a onstant speed of 40 Km/h. What is the time taken by the second train to cross the
Each train makes one stop only at a station 10 Km from its bridge?
starting point and remains there for 15 min. Which train is (a) 61 seconds (b) 63 seconds
nearer to Rampur when they meet?
(c) 62 seconds (d) 64 seconds
(a) Both are equidistant (b) Laxman Mail
[Based on MAT (May), 2006]
(c) Bharat Express (d) None of these
110.
Ramesh travels 760 Km to his home, partly by train and
[Based on MAT (May), 2007]
partly by car. He takes 8 hrs, if he travels 160 Km by train
104.
A car starts running with the initial speed of 40 Km/h, and the rest by car. He takes 12 minutes more, if he travels
with its speed increasing every hour by 5 Km/h. How 240 Km by train and the rest by car. What are the speeds
many hrs will it take to cover a distance of 385 km? of the train and the car?

Chapter 12.indd 15 6/5/2015 3:10:54 PM


12.16 Chapter 12

(a) Speed of car = 90 Km/h, Speed of train = 60 Km/h 117.


It takes eight hrs for a 600 Km journey, if 120 Km is done
(b) Speed of car = 100 Km/h, Speed of train = 80 Km/h by train and the rest by car. It takes 20 minutes more, if
200 Km is done by train and the rest by car. The ratio of
(c) Speed of car = 80 Km/h, Speed of train = 90 Km/h
the speed of the train to the speed of the car is:
(d) Speed of car = 100 Km/h, spped of train = 90 Km/h
(a) 4:3 (b) 3:4
[Based on MAT (May), 2006]
(c) 3:2 (d) 2:3
111.
A car travels 25 Km/h faster than a bus for a journey of [Based on MAT, 1999]
500 km. If the bus takes 10 hrs more than the car, then the
speeds of the bus and the car are: 118.
A plane left 30 minutes later than the scheduled time and
in order to reach its destination 1500 Km away in time,
(a) 25 Km/h and 40 Km/h respectively
it has to increase its speed by 250 Km/h from its usual
(b) 25 Km/h and 60 Km/h respectively speed. Find its usual speed.
(c) 25 Km/h and 50 Km/h respectively (a) 1000 Km/h (b) 750 Km/h
(d) None of the above
(c) 850 Km/h (d) 650 Km/h
[Based on MAT (May), 2006]
[Based on MAT, 1999]
112.
A train covered a certain distance at a uniform speed. If
the train had been 6 Km/h faster, it would have taken 4 hrs 119.
Two trains, each 120 m in length, run in opposite directions
less than the scheduled time. And, if the train were slower with velocities of 40 m/s and 20 m/s respectively.
by 6 Km/h, the train would have taken 6 hrs more than the How long will it take for the tail end of the two trains to
scheduled time. The length of the journey is: meet each other from the time their engines crossed each
other?
(a) 700 km (b) 740 km
(c) 720 km (d) 760 km (a) 2 seconds (b) 3 seconds
[Based on MAT (Feb), 2006] (c) 4 seconds (d) 5 seconds
113.
A small aeroplane can travel at 320 Km/h in still air. The [Based on MAT, 1997]
wind is blowing at a constant speed of 40 Km/h. The total 120.
A journey of 260 Km between two cities takes 3 hrs less
time for a journey against the wind is 135 minutes. What by train than by car. If the average speed of travel by car is
will be the time, in minutes, for the return journey with the 6 Km/h less than the speed of the train, the average speed
wind? (Ignore take off and landing times for the aeroplane). of the train is:
(a) 94.5 (b) 105 (a) 26 Km/h (b) 52 Km/h
(c) 108.125 (d) 120
(c) 20 Km/h (d) 13 Km/h
[Based on MAT, 1997]
[Based on MAT, 1998]
114. A journey of 192 Km between the two cities takes 2 hrs
less by a fast train than by a slow train. If the average 121.
A passenger train takes 2 hrs less for a jour­ney of 300 Km
speed of the slow train is 16 Km/h less than that of the fast if its speed is increased by 5 Km/h from its normal speed.
train, then the average speed of the fast train is: The normal speed is:
(a) 36 Km/h (b) 64 Km/h (a) 10 Km/h (b) 20 Km/h
(c) 32 Km/h (d) 48 Km/h (c) 25 Km/h (d) 30 Km/h
[Based on MAT, 1999] [Based on MAT, 1998]
115.
A certain distance is covered by a train with a certain 122.
Suresh travels 600 Km to his home partly by train and
speed. If half the distance is covered in double time, then partly by air. It takes him 8 hrs if he travels 120 Km by
the ratio of this speed to that of the original is: train and the rest by car. It takes him 20 minutes more if
(a) 1:4 (b) 4:1 he travels 200 Km by train and the rest by car. The speed
(c) 2:1 (d) 1:2 of the car is:
[Based on MAT, 1997] (a) 20 Km/h (b) 30 Km/h
116.
Points A and B are 70 Km apart on a highway. One car (c) 60 Km/h (d) 80 Km/h
starts from A and another one from B at the same time. If [Based on MAT, 1998]
they travel in the same direction, they meet in 7 hrs. But if
they travel towards each other they meet in one hour. The 123.
The time taken to run around a circular field of area 616 sq
speeds of the two cars are: Km at a speed of 22 Km/h is:
(a) 45 and 25 Km/h (b) 70 and 10 Km/h (a) 1 hr (b) 2 hrs
(c) 40 and 30 Km/h (d) 60 and 40 Km/h (c) 3 hrs (d) 4 hrs
[Based on MAT, 1999] [Based on MAT, 1998]

Chapter 12.indd 16 6/5/2015 3:10:54 PM


Time and Distance 12.17

124.
A man takes 50 minutes to cover a certain dis­tance at a 131. A train is moving at a speed of 132 Km/h. If the length of
speed of 6 Km/h. If he walks with a speed of 10 Km/h, he the train is 110 m, how long will it take to cross a railway
covers the same distance in: platform 165 m long?
(a) 30 minutes (b) 40 minutes (a) 5 s (b) 7.5 s
(c) 20 minutes (d) 10 minutes (c) 10 s (d) 15 s
[Based on MAT, 1998]
[Based on MAT, 1998]
125.
A train takes 50 minutes for a journey if it runs at 48 132.
A car can finish a certain journey in 10 hrs at the speed of
Km/h. The rate at which the train must run to reduce the 48 Km/h. In order to cover the distance in 8 hrs, the speed
time to 40 minutes will be: of the car must be increased by:
(a) 50 Km/h (b) 55 Km/h (a) 6 Km/h (b) 7.5 Km/h
(c) 12 Km/h (d) 15 Km/h
(c) 60 Km/h (d) 65 Km/h
 [Based on MAT, 1998]
[Based on MAT, 1998]
133. Anand travelled 300 Km by train and 20 Km by taxi. It
126.
A goods train in five successive minutes from its start runs
took him 5 hrs and 30 min. However, if he travels 2600
68 m, 127 m, 208 m, 312 m and 535 m and for the next
Km by train and 240 Km by taxi, he takes 6 minutes more.
5 minutes maintains an average speed of 33 Km/h. The
The speed of the train is:
average speed of the train for these 10 minutes is:
(a) 100 Km/h (b) 120 Km/h
(a) 23 Km/h (b) 24 Km/h
(c) 80 Km/h (d) 110 Km/h
(c) 25 Km/h (d) 26 Km/h
 [Based on MAT, 2011]
[Based on MAT, 1998]
127.
A certain distance is covered by a cyclist at a certain 134. The autorickshaw fare consists of a fixed charge together
speed. If a jogger covers half the distance in double the with the charge for the distance covered. For a journey of 10
time, the ratio of the speeds of the jogger to that of the km, the charge paid is `85 and for a journey of 15 km, the
cyclist is: charge paid is `120. The fare for a journey of 25 Km will be:
(a) 1:4 (b) 4:1 (a) `175 (b) `190
(c) 1:2 (d) 2:1 (c) `180 (d) `225
[Based on MAT, 1998]  [Based on MAT, 2011]

128.
In climbing a 21 m long round pole, a monkey climbs 6 m 135. A train after travelling 150 Km meets with an accident and
in the first minute and slips 3 m in the next minute. What then proceeds with 3/5 of its former speed and arrives at
time (in minutes) the monkey would take to reach the top its destination 8 hrs late. Had the accident occurred 360
of the pole? Km further, it would have reached the destination 4 hrs
late. What is the total distance travelled by the train?
(a) 11 (b) 14
(a) 840 Km (b) 960 Km
2
(c) 14 (d) 9 (c) 870 Km (d) 1100 Km
3
 [Based on MAT, 2011]
[Based on MAT, 1999]
136. A train is scheduled to cover the distance between two
129.
Two cyclists start on a circular track from a given point stations 46 Km apart in one hour. If it travels 25 Km at
but in opposite directions with speeds of 7 m/s and 8 m/s a speed of 40 Km/h, find the speed for the remaining
respectively. If the circumference of the circle is 300 m, journey to complete it in scheduled time.
after what time will they meet at the starting point? (a) 66 Km/h (b) 56 Km/h
(a) 20 seconds (b) 100 seconds (c) 46 Km/h (d) 36 Km/h
(c) 300 seconds (d) 200 seconds  [Based on MAT (Sept), 2011]
[Based on MAT, 1998] 137. A car covers a distance of 715 Km at a constant speed. If
130.
Two cars start together in the same direction from the same the speed of the car had been 10 Km/h more, than it would
place. The first goes with a uniform speed of 10  Km/h. have taken 2 hrs less to cover the same distance. What is
The second goes at a speed of 8 Km/h in the first hour the original speed of the car?
and increases the speed by half km, each succeeding hour. (a) 55 Km/h (b) 50 Km/h
After how many hrs will the second car overtake the first (c) 45 Km/h (d) 65 Km/h
if both go non-stop?  [Based on MAT, 2011]
(a) 9 hrs (b) 5 hrs 138. Two men starting from the same place walk at the rate of 5
(c) 7 hrs (d) 8 hrs Km/h and 5.5 Km/h respectively. What time will they take
[Based on MAT, 1999] to be 8.5 Km apart, if they walk in the same direction?

Chapter 12.indd 17 6/5/2015 3:10:54 PM


12.18 Chapter 12

(a) 16 hrs (b) 8 hrs 30 min 144. A manufacturer of fridges produced 600 units in the third
(c) 4 hrs 15 min (d) 17 hrs year and 700 units in the seventh year. Assuming that the
 [Based on MAT, 2011] production increases uniformly by a fixed number every
year, the total production in 7 yr will be:
2
139. Raghu travelled 1200 Km by air which formed th of his (a) 4537 (b) 4375
5
trip. One-third of the whole trip he travelled by car and the (c) 4753 (d) 4735
rest of the journey he did by train. The distance travelled  [Based on MAT, 2012]
by train is:
(a) 1600 Km (b) 800 Km 145. A motorboat covers a certain distance downstream in
6 hrs but takes 8 hrs to return upstream to the starting
(c) 1800 Km (d) 480 Km
point. If the speed of the stream be 6 Km/h, find the speed
 [Based on MAT, 2012] of the motor boat in still water.
140. A man has to cover a distance of 6 Km in 45 min. If the (a) 41 Km/h (b) 43 Km/h
2 (c) 42 Km/h (d) 44 Km/h
covers one-half of the distance in rd time, what should
3  [Based on MAT (Feb), 2012]
be his speed to cover the remaining distance in the
remaining time? 146. Rohit would like to attend his college day celebration. He
has to travel a certain distance from his home. He looks at
(a) 12 Km/h (b) 16 Km/h
his watch and he immediately does some calculation and
(c) 3 Km/h (d) 8 Km/h comes to know that if he drives at a speed of 40 Km/h he
 [Based on MAT, 2012] will reach his college 10 minutes late and if he drives at a
speed of 50 Km/h he will reach 5 minutes early. What is
141. A man starts cycling from A to B and, at the same time,
the distance that Rohit has to cover?
another man starts cycling from B to A along the same
2 2 (a) 40 Km (b) 50 Km
path. The completed their journeys in 1 h and 2 h, (c) 45 Km (d) 60 Km
3 5
respectively. At what speed has the second man cycled, if  [Based on MAT, 2013]
the first cycles at 16 Km/h?
147. A and B walk around a circular track. They start at 8 am
1 1 from the same point in the opposite directions. A and B
(a) 16 km/h (b) 18 km/h walk at a speed of 2 rounds per hour and 3 rounds per hour
2 2
respectively. How many times shall they cross each other
1 1
(c) 11 km/h (d) 13 km/h before 9:30 am?
9 3
(a) 6 (b) 5
 [Based on MAT, 2012]
(b) 7 (d) 8
142. Two man A and B start walking from a place X at  [Based on MAT, 2013]
1 3 2
4 km/h and 5 km/h, respectively. How many Km 148. A runs 1 times as fast as B. If A gives B a start of 80 m.
2 4 3
1 How far must the winning post be so that A and B reach it
apart will they be at end of 3 h, if they are walking in
2 at the same time?
the same direction?
(a) 300 m (b) 200 m
1 3
(a) 4 km (b) 5 km (c) 270 m (d) 160 m
3 4  [Based on MAT, 2013]
3 7
(c) 4 km (d) 35 km 149. A car driver, driving in a fog, passes a pedestrian who was
8 8 walking at the rate of 2 Km/h in the same direction. The
 [Based on MAT, 2012] pedestrian could see for 6 minutes and it was visible to him
143. A car travels 25 Km/h faster than a bus for a journey of up to a distance of 0.6 km. What was the speed of the car?
500 km. If the bus takes 10 hrs more than the car, then the (a) 15 Km/h (b) 30 Km/h
speeds of the car and the bus are: (c) 20 Km/h (d) 8 Km/h
(a) 25 Km/h and 40 Km/h  [Based on MAT, 2013]
(b) 50 Km/h and 25 Km/h
150. A railway passenger counts the telegraph poles on the
(c) 25 Km/h and 60 Km/h railroad as he passes them. The telegraph poles are at a
(d) None of the above distance of 50 m. What will be his count in 4 h, if the
 [Based on MAT, 2012] speed of the train is 45 Km/h?

Chapter 12.indd 18 6/5/2015 3:10:59 PM


Time and Distance I 12.19

(a) 600 (b) 2500 hour, and 10 minutes too soon, if I walk at 6 Km an hour.
(c) 3600 (d) 5000 How far have I to walk?
 [Based on MAT, 2014] (a) 25 Km (b) 5 Km
151. Two swimmers started simultaneously from the beach, (c) 10 Km (d) None of these
one to the South and the other to the East. Two hours later,  [Based on SNAP, 2013]
the distance between them turned out to be 100 km. Find
156. A train starts from Delhi at 6:00 AM and reaches Ambala
the speed of the faster swimmer, knowing that the speed
Cantt at 10 AM. The other train starts from Ambala Cantt
of one of them was 75% of the speed of the other.
at 8 Am and reaches Delhi at 11:30 PM. If the distance
(a) 30 Km/h (b) 40 Km/h between Delhi and Ambala Cantt. is 200 km, then at what
(c) 45 Km/h (d) 60 Km/h time did the two trains meet each other?
 [Based on MAT, 2014] (a) 8:56 AM (b) 8:46 AM
152. An aeroplane flies with an average speed of 756 Km/h. (c) 7:56 AM (d) 8:30 AM
A helicopter takes 48 hrs to cover twice the distance  [Based on SNAP, 2013]
covered by aeroplane in 9 hrs. How much distance will
157. Three persons start walking together and their steps
the helicopter cover in 18 hrs? (Assuming that flights are
measure 40 cm, 42 cm and 45 cm respectively. What is
non-stop and moving with uniform speed.)
the minimum distance each should walk so that each can
(a) 5010 Km (b) 4875 Km cover the same distance in complete steps?
(c) 5760 Km (d) 5103 Km (a) 25 m 20 cm
 [Based on SNAP, 2013]
(b) 52 m 40 cm
153. Anand travelled 300 Km by train and 200 Km by taxi. It (c) 75 m 60 cm
took him 5 hrs and 30 min. However, if he travels 260 Km
(d) 100 m 80 cm
by train and 240 Km by taxi, he takes 6 minutes more. The
speed of the train is:  [Based on SNAP, 2013]

(a) 100 Km/h (b) 120 Km/h 158. An airplane first flew with a speed of 440 Km/h and
(c) 80 Km/h (d) 110 Km/h covered a certain distance. It still had to cover 770 Km
less than what it had already covered, but it flew with a
 [Based on SNAP, 2013]
speed of 660 Km/h. The average speed for the entire flight
154. A plane left 30 minutes later than its scheduled time to was 500 Km/h. find the total distance covered.
reach its destination 1500 Km away. In order to reach in
(a) 3250 Km (b) 2750 Km
time it increases its speed by 250 Km/h. What is its original
speed? (c) 4400 Km (d) 1375 Km
(a) 1000 Km/h (b) 750 Km/h 159. A man in a train notices that he can count 21 telephone
(c) 600 Km/h (d) 800 Km/h posts in one minute. If they are known to be 50 metres
 [Based on SNAP, 2013] apart, then at what speed is the train travelling?
155. I have to reach a certain place at a certain time and I find (a) 45 Km/h (b) 60 Km/h
that I shall be 15 minutes too late, if I walk at 4 Km an (c) 63 Km/h (d) 65 Km/h

Difficulty Level-2
(Based on Memory)

1.
Two persons A and B are at two places P and Q, 2. Two champion swimmers start a two-length swimming
respectively. A walks at v Km/h and B is 2 Km/h faster race at the same time but from opposite ends of the pool.
than A, starting simultaneously from where they stand. If They swim in line and at constant but different speeds.
they walk towards each other, they meet in 72 min. If they They first pass at a point 18.5  m from the deep end and
walk in the same direction, the faster overtakes the slower having completed one length, each swimmer is allowed to
in 6 hrs. Find their respective speeds (in Km/h). rest on the edge of the pool for 45 s. After setting off on the
return length, the swimmers pass for the second time just
(a) 3 and 5 (b) 4 and 6
10.5 m from the shallow end. Thus, the length of the pool is
1 1 1 1 (a) 90 m (b) 45 m
(c) 2 and 4 (d) 3 and 5
2 2 2 2 (c) 26 m (d) 52 m

M12_KHAT6981_C12.indd 19 6/17/2015 12:03:20 PM


12.20 Chapter 12

3.
A student got down at a tram stop A and walked the 12VS 4VS
remaining distance to school. If he had stayed in the tram (a) (b)
V S 3S  V
until the next stop B and then walked to school he would
have taken a minute longer. If he had walked the entire VT
(c) (d) None of these
distance from A to school at twice his usual speed, he 3S
would have taken as much time as the tram would take for
travelling from A to B. If the school is 300 m from A, and Directions (Questions 9 and 10):  Refer to the following infor­
100 m from B, the walking speed of the student in Km/h is: mation and answer the questions that follow.
1    Two trains A and B, 100 m long are moving on parallel tracks
(a) 2 (b) 3 at speeds of 20 m/s and 30 m/s respectively. They are travelling in
2
opposite direction.
1
(c) 3 (d) 4 9. A boy in front of train A sees another boy in front of train
2 B when he is closest to high. He throws a ball at a speed
4.
In a shopping Mall, Raja decided to walk down the of 2 m/s which hits the tail of train B. What is the distance
escalator. He did some quick calculation in his mind. He between the two trains?
found that if he walks down thirty steps, he requires 18 (a) 0 m (b) 10 m
second to reach the bottom. However, if he is able to step (c) 4 m (d) 8 m
down thirty four stairs, he would only require 6 second to
10. If now the trains are travelling in the same direction
get to the bottom. If the time is measured from the moment
and the boy repeats his action according to the previous
the top step begins to descend to the time he steps off the
question where will the ball hit?
last step at the bottom, the height of the stair way in steps is:
(a) 46 (b) 38 (a) 10 m from the front of train B
(c) 36 (d) 32 (b) 20 m from the front of train B
(c) at the front of train A
5. The distance of P from Q is 7  km. An aeroplane flew (d) at the tail of train B
from P to Q against the wind and then comes back in
22 minutes. If its speed was decreased by 12½% flying 11. A train of 24 m length runs with a speed of 250 m/s. A
against the wind and increased by 5% when flying with man in the train at the tail end of the train runs with a
the wind, how long would the flight have taken, had there speed of 10 m/s. When he reaches the front end he turns
been no wind? back with a speed of 6 m/s and this process continues.
How many rounds (up and down) he will complete if the
(a) 15 minutes (b) 21 minutes
train runs 8 kms, providing that during running he will not
(c) 36 minutes (d) 60 minutes
loose contact with the train?
6.
Two trains 100 m and 80 m long, run at the rate of 30 (a) 3 (b) 4
Km/h and 50 Km/h respectively on parallel rails. How (c) 5 (d) 6
long will it take a man sitting in the second train to pass
12. Train X starts at 6.00 a.m. from a certain station with P
the first train if they run in opposite direction?
Km/h and train Y starts at 8.30 a.m. from the same station
(a) 2.4 seconds (b) 4.5 seconds at Q Km/h. If Q > P, then how many hrs will train Y take
(c) 8.3 seconds (d) 11.2 seconds to overtake train X?
7.
Two trains, 100 Km apart, travel towards each other on 5P 2P
(a) hrs (b) hrs
the same track. One train travels at 40 Km/h; the other 2(Q  P ) 5(Q  P )
travels at 60 Km/h. A bird starts flying at a speed of 90
Km/h, at the same location of the faster train. When it 2(Q  P ) 5(Q  P )
(c) hrs (d) hrs
reaches the slower train, it turns around and flies in the 5P 2P
opposite direction at the same speed. When it reaches the 13. A lives on 9th floor and B lives on 44th floor. A goes up at
faster train, again it turns around and so on. When the two a rate of 34 floors per minute and B comes down at a rate
trains collide, how far has the bird flown? of 36 floors per minute. At which floor they will meet?
(a) 90 km (b) 45 km (a) 27 (b) 26
(c) 180 km (d) 135 km (c) 18 (d) 32
8.
A car travelled 75% of the distance from town A to B by 14. Rashmi leaves office at 6.00 p.m. and catches a 6.30 p.m.
travelling at T hrs at an average speed of V Km/h. The car local train that arrives in her town at 7.00 p.m. Her father
travels at an average speed of S Km/h for the remaining leaves home to pick her up at 7.00 p.m. from the station
part of the trip. Which of the following expressions as she gets off the train. Yesterday, Rashmi left her office
represents the average speed for the entire trip? early and took a 6.00 pm train and arrived at 6.30 p.m. As

Chapter 12.indd 20 6/5/2015 3:11:00 PM


Time and Distance 12.21

her father was not there to pick her up, she started walking 21. Vaibhav can walk up a moving ‘up’ escalator in 30 s. He
towards home. Her father left home at the usual time, saw her can also walk down this moving ‘up’ escalator in 90 s. If
daughter walking, turned around, picked her up and drove his walking speed is same upwards and downwards, then
home, arriving there 10 minutes earlier than the usual. For how much time will he take to walk up the escalator, when
how long did Rashmi walk before her father picked her up? the escalator is stationary?
(a) 10 minutes (b) 15 minutes (a) 30 seconds (b) 45 seconds
(c) 20 minutes (d) 25 minutes (c) 60 seconds (d) 90 seconds
[Based on FMS (Delhi), 2004]
22.
Points A and B are 60 Km apart. A bus starts from A and
15. There are two friends A and B. A starts with his car at another from B at the same time. If they go in the same
the speed 40 Km/h. B starts one hour later from the same direction, then they meet in 6 hrs and if they go in opposite
place, in the same direction on his bike at the speed 50 directions, they meet in 2 hrs. The speed of the bus with
Km/h. After how many hrs they will meet? greater speed is:
(a) 12 hrs (b) 6 hrs (a) 10 Km/h (b) 20 Km/h
(c) 4 hrs (d) 11 hrs
(c) 30 Km/h (d) 40 Km/h
16. Ram travels from P to Q at 10 Km/h and returns at
[Based on FMS (Delhi), 2003]
15 Km/h. Sham travels from P to Q and returns at
12.5 Km/h. If he takes 12 minutes less than Ram, then 23. A train travelled 75% of the way from town X to town
what is the distance between P and Q? Y by travelling for A hrs at an average speed of B Km/h.
The train travels at an average speed of S Km/h for the
(a) 60 km (b) 45 Km
remaining part of the journey. Which of the following
(c) 36 km (d) 30 Km
expressions represents the average speed of the entire
[Based on IIT Joint Man. Ent. Test, 2004]
journey?
17. Two trains are 35 m apart and running in same direction (a) 0.75B + 0.25 (b) (4BS)/(3S + B)
with speeds 30 Km/h and 45 Km/h, respectively. If they
cross each other in 5 minutes then what is the total length (c) AB/3S (d) 0.75A + 0.25S
of both the trains? [Based on SCMHRD Ent. Exam., 2003]
(a) 354 m (b) 225 m 24. Plane A takes off at 4 pm and flies at a constant speed of
(c) 1215 m (d) 1322 m X mph. Plane B takes off at 5.30 p.m. and flies the same
route as that of A at a constant speed of Y mph. Assuming
18. Mir went on a ten-mile test drive of his new imported
that Y is greater than X, how many hrs after 5:30 p.m. will
bike. He started with a certain speed and after covering
the plane B overtake plane A?
each mile, his speed decreased by 20% for the next mile.
If he took 5 minutes to cover the first five miles of the (a) 3/2 X hrs (b) 3/(2Y) hrs
drive, what is the approximate time taken by him to cover (c) 3/2 (Y – X) hrs (d) 3X/2 (Y – X) hrs
the next five miles? [Based on SCMHRD Ent. Exam., 2003]
(a) 14 minutes and 14 s (b) 15 minutes and 15 s 25.
A 100 m long train passes a man, running in the same
(c) 16 minutes and 16 s (d) 17 minutes and 17 s direction at 6 Km/h, in 5 second and a car travelling in the
same direction in 6 s. At what speed is the car travelling
19. A motorcyclist goes from Delhi to Bharatpur, a distance
(length of both the man and car is negligible)?
of 192 km, at an average speed of 32 Km/h. Another man
starts from Delhi by car 2.5 hrs after the motorcyclist (a) 18 Km/h (b) 20 Km/h
started and reaches Bharatpur half an hour late. What is (c) 24 Km/h (d) 30 Km/h
the ratio of speed of the person on the motorcycle to the
26. Manu and Tanu run back and forth between the town hall
person going by the car?
and the county station at respective speeds of 12 Km/h
(a) 1:2 (b) 2:3 and 18 Km/h. They start simultaneously — Manu from
(c) 10:27 (d) 5:4 the town hall and Tanu from the county station. If they
cross each other for the first time 14 minutes from the
20. To get to a business meeting, John drove m miles in hrs
start, at what distance from the county station will they
hrs, and arrived 1/2 hour early. At what rate should he
cross each other for the fifth time?
have driven to arrive exactly on time?
m 2m (a) 4.2 km (b) 2.8 km
(a) (b) (c) 3.6 km (d) None of these
2h 2h + 1
2m 2m − h 27. Excluding stoppages, the speed of a train is 45 Km/h
(c) (d)
2h − 1 2h and including stoppages, it is 36 Km/h. For how many
[Based on REC Tiruchirapalli, 2003] minutes, does the train stop per hour?

Chapter 12.indd 21 6/5/2015 3:11:00 PM


12.22 Chapter 12

(a) 10 (b) 12 (a) 4 minutes (b) 2.5 minutes


(c) 15 (d) 18 (c) 1.5 minutes
28.
If the speed of a railway train is increased by 5 Km/h from (d) Patient died before reaching the hospital.
its normal speed, then it would have taken 2 hrs less for a [Based on CAT, 2002]
journey of 300 km, What is its normal speed? 33. At a bookstore, ‘MODERN BOOK STORE’ is flashed
(a) 10 Km/h (b) 25 Km/h using neon lights. The words are individually flashed at
(c) 20 Km/h (d) 30 Km/h 1 1 1
intervals of 2 , 4 , 5  second respectively, and each
[Based on I.P. Univ., 2002] 2 4 8
29.
An aircraft was to take off from a certain airport at 8 a.m. word is put off after a second. The least time after which
but it was delayed by 30 minutes. To make up for the lost the full name of the bookstore can be read again, is:
time, it was to increase its speed by 250 Km/h from the (a) 49.5 seconds (b) 73.5 seconds
normal speed to reach its destination 1500 Km away, on (c) 1744.5 seconds (d) 855 seconds
time. What was the normal speed of the aircraft?
[Based on CAT, 2002]
(a) 650 Km/h (b) 750 Km/h
34. A train approaches a tunnel AB. Inside the tunnel a cat
(c) 850 Km/h (d) 1000 Km/h
located at a point that is three-eighths of the distance AB
[Based on I.P. Univ., 2002] measured from the entrance A. When the train whistles,
30. If a child walks at the rate of 5 m/min from his home, he the cat runs. If the cat moves to the entrance of the tunnel,
is 6 minutes late for school; if he walks at the rate of 7 m/ A, the train catches the cat exactly at the entrance. If the
min, he reaches half an hour earlier. How far is his school cat moves to the exit B, the train catches the cat at exactly
from his home? the exit. The speed of the train is greater than the speed of
(a) 450 minutes (b) 540 minutes the cat by what order?
(c) 630 minutes (d) 360 minutes (a) 3:1 (b) 4:1
(c) 5:1 (d) None of these
31. Only a single rail track exists between stations A and B on a
[Based on CAT, 2002]
railway line. One hour after the north bound superfast train
N leaves station A for station B, a south bound passenger 35. Six technicians working at the same rate completely work
train S reaches station A from station B. The speed of the of one server in 10 hrs. If they start at 11: 00 a.m. and one
superfast train is twice that of a normal express train E, additional technician per hour being added beginning at
while the speed of a passenger train S is half that of E. On 5:00 p.m., at what time the server will be complete?
a particular day N leaves for station B from station A, 20 (a) 6:40 p.m. (b) 7.00 p.m.
minutes behind the normal schedule. In order to maintain
(c) 7:20 p.m. (d) 8:00 p.m
the schedule, both N and S increased their speeds. If the
[Based on CAT, 2002]
superfast train doubles its speed, what should be the ratio
(approximately) of the speed of passenger train to that of 36. A tiger is 50 of its own leaps behind a deer. The tiger takes
the superfast train so that passenger train S reaches exactly 5 leaps per minutes to the deer’s 4. If the tiger and the deer
at the scheduled time at station A on that day. cover 8 m and 5 m per leap respectively, what distance
(a) 1:3 (b) 1:4 will the tiger have to run before it catches the deer?
(c) 1:5 (d) 1:6 (a) 600 m (b) 700 m
[Based on CAT, 2002] (c) 800 m (d) 1000 m
32. On a 20 Km tunnel connecting two cities A and B, there 37. A can complete a piece of work in 4 days. B takes double
are three gutters. The distance between gutters 1 and 2 is the time taken by A, C takes double that of B, and D takes
half the distance between gutters 2 and 3. The distance double that of C to complete the same task. They are
from city A to its nearest gutter, gutter 1 is equal to the paired in groups of two each. One pair takes two-thirds
distance of city B from gutter 3. On a particular day, the the time needed by the second pair to complete the work.
hospital in city A receives information that an accident has Which is the first pair?
happened at the third gutter. The victim can be saved only (a) A, B (b) A, C
if an operation is stated within 40 minutes. An ambulance
(c) B, C (d) A, D
started from city A at 30 Km/h and crossed the first gutter
[Based on CAT, 2001]
after 5 minutes. If the driver had doubled the speed after
that, what is the maximum amount of time the doctor 38. Two men X and Y started working for a certain company
would get to attend the patient at the hospital. Assume at similar jobs on January 1, 1950. X asked for an initial
1 minute is elapsed for taking the patient into and out of salary of `300 with an annual increment of `30. Y asked
the ambulance. for an initial salary of `200 with a rise of `15 every six

Chapter 12.indd 22 6/5/2015 3:11:01 PM


Time and Distance 12.23

months. Assume that the arrangements remained unaltered (a) Increase the speed
till December 31, 1959. Salary is paid on the last day of (b) Decrease the speed
the month. What is the total amount paid to them as salary (c) Maintain the speed at 60 Km/h
during the period?
(d) None of these
(a) `93,300 (b) `93,200 [Based on CAT, 2001]
(c) `93,100 (d) None of these
[Based on CAT, 2001] Directions for questions (44 to 45): Answer the questions based
on the following in formation.
39.
At his usual rowing rate, Rahul can travel 12 miles
downstream in a certain river in six hrs less than it takes    There are five machines — A, B, C, D, and E — situated
him to travel the same distance upstream. But if he could on a straight line at distances of 10m, 20m, 30m, 40m and 50m
double his usual rowing rate for this 24 mile round trip, respectively from the origin of the line. A robot is stationed at the
the downstream 12 miles would then take only one hour origin of the line. The robot serves the machines with raw material
less than the upstream 12 miles. What is the speed of the whenever a machine becomes idle. All the raw materials are
current in miles per hour? located at the origin. The robot is in an idle state at the origin at
the beginning of a day. As soon as one or more machines become
(a) 7/3 (b) 4/3
idle, they send messages to the robot-station and the robot starts
(c) 5/3 (d) 8/3 and serves all the machines from which it received messages. If a
[Based on CAT, 2001] message is received at the station while the robot is away from it,
40. Shyam and Vyom walk up an escalator (moving stairway). the robot takes notice of the message only when it returns to the
The escalator moves at a constant speed. Shyama takes station. While moving, it serves the machines in the sequence in
three steps for every two of Vyom’s steps. Shyam gets to which they are encountered, and then returns to the origin. If any
the top of the escalator after having taken 25 steps, while messages are pending at the station when it returns, it repeats the
Vyom (because his slower pace lets the escalator do a little process again. Otherwise, it remains idle at the origin till the next
more of the work) takes only 20 steps to reach the top. If message(s) is (are) received.
the escalator were turned off, how many steps would they
44. Suppose on a certain day, machines A and D have sent
have to take to walk up?
the first two messages to the origin at the beginning of the
(a) 40 (b) 50 first second, C has sent a message at the beginning of the
(c) 60 (d) 80 5th second, B at the beginning of the 6th second and E at
[Based on CAT, 2001] the beginning of the 10th second. How much distance has
the robot travelled since the beginning of the day, when
41. There’s a lot of work in preparing a birthday dinner. Even it notices the message of E? Assume that the speed of
after the turkey is in the oven, there’s still the potatoes and movement of the robot is 10 m/s.
gravy, yams, salad, and cranberries, not to mention setting
(a) 140 m (b) 80 m
the table.
Three friends, Asit, Arnold, and Afzal, work together (c) 340 m (d) 360 m
to get all of these chores done. The time it takes them to [Based on CAT, 2000]
do the work together is six hrs less than Asit would have 45. Arun, Barun and Kiranmala start from the same place and
taken working alone, one hour less than Arnold would travel in the same diretion at speed of two hour after Arun.
have taken alone, and half the time Afzal would have If Barun and Kiranmala overtake Arun at the same instant,
taken working alone. how many hrs after Arun did Kiranmala starts?
How long did it take them to do these chores working (a) 3 (b) 3.5
together? (c) 4 (d) 4.5
(a) 20 minutes (b) 30 minutes [Based on CAT, 2006]
(c) 40 minutes (d) 80 minutes 46. Golu and Mayank start running simultaneously. Golu runs
[Based on CAT, 2001]
from point A to point B and Mayank from point B to point
42. Manasa makes the 200 Km trip from Mumbai to Pune at 6
A. Golu’s speed is of Mayank’s speed. After crossing
a steady speed of 60 Km/h. What is the amount of petrol 5
consumed for the journey? 1
Mayank, if Golu takes 2 h to reach B, how much time
(a) 12.5 litres (b) 13.33 litres 2
(c) 16 litres (d) 19.75 1itres does Mayank take to reach A after crossing Golu?
[Based on CAT, 2001] (a) 3 hrs 6 min (b) 3 hrs 16 min
(c) 3 hrs 26 min (d) 3 hrs 36 min
43. Manasa would like to minimize the fuel consumption for
the trip by driving at the appropriate speed. How should 47. Shruti and Archana start travelling together in the same
she change the speed? direction at 8 Km/h and 13 Km/h respectively. After 4 hrs

Chapter 12.indd 23 6/5/2015 3:11:01 PM


12.24 Chapter 12

Shruti doubles her speed and Archana reduces her speed respectively. There is a straight line path connecting B and
by 1 Km/h and reached the destination together. How long C. Then, Ram returns to point A after walking along the
the entire journey last? line segments BC and CA. Shyam also returns to A after
(a) 5 hrs (b) 9 hrs walking along line segments CB and BA. Their speeds
1 1 remain unchanged. The time interval (in hrs) between
(c) 7 hhrs (d) 9 hhrs Ram’s and Shyam’s return to the point A is
2 2
Directions (Questions 48 and 49): Answer the questions based on 10 19  26 2 19  10
(a) (b)
the following information. 3 3
   A thief, after committing the burglary, started fleeing at 12 19  26 19  10
(c) (d)
noon, at a speed of 60 Km/h. He was then chased by a policeman 3 3
X. X started the chase, 15 minutes after the thief has started, at a
speed of 65 Km/h. 54.
City Bus Corporation runs two buses from terminus A to
terminus B, each bus making 5 round trip in a day. There
48. At what time did X catch the thief? are no stops in between. These buses play back and forth
(a) 3.30 p.m. (b) 3 p.m. on the same route at different but uniform speeds. Each
(c) 3.15 p.m. (d) None of these morning the buses start at 7 a.m. from the respective
terminuses. They meet for the first time at a distance of 7
49. If another policeman had started the same chase along Km from terminus A. Their next meeting is at a distance
with X, but at a speed of 60 Km/h, then how far behind of 4 Km from terminus B, while travelling in opposite
was he when X caught the thief? directions. Assuming that the time taken by the buses at
(a) 18.75 km (b) 15 km terminuses is negligibly small, and the cost of running a
(c) 21 km (d) 37.5 km bus is `20 per km, find the daily cost of running the buses
(In `).
Directions (Questions 50 to 51): Answer the questions based on
the following information. (a) 3200 (b) 4000
   Boston is 4 hrs ahead of Frankfurt and 2 hrs behind India. X (c) 6400 (d) 6800
[Based on XAT, 2012]
leaves Frankfurt at 6 p.m. on Friday and reaches Boston the next
day. After waiting there for 2 h, he leaves exactly at noon and 55. Two cities A and B, at a distance of 50 km, are connected
reaches India at 1 a.m. On his return journey, he takes the same by two separate roads. The speed of any vehicle travelling
route as before, but halts at Boston for 1 hr less than his previous between the two cities on road 1 is 50 Km/h, while the
halt there. He then proceeds to Frankfurt.  80 
speed on road 2 is   Km/h, where n is the number of
50. If his journey, including stoppage, is covered at an average  n 
speed of 180 mph, what is the distance between Frankfurt vehicles (including the concerned vehicle.) If you travel
and India? in a vehicle from A to B on road 1 and come back from
(a) 3,600 miles (b) 4,500 miles B to A on road 2 (where there are already three vehicles
(c) 5,580 miles (d) Data insufficient playing), your approximate average speed is:
(a) 26 Km/h (b) 29 Km/h
51. If X had started the return journey from India at 2.55 a.m. (c) 32 Km/h (d) 35 Km/h
on the same day that he reached there, after how much
[Based on JMET, 2009]
time would he reach Frankfurt?
56. A man jogging inside a railway tunnel at a constant speed
(a) 24 hrs (b) 25 hrs
hears a train approaching the tunnel from behind at a
(c) 26 hrs (d) Data insufficient speed of 30 Km/h, when he is one third of the way inside
the tunnel. Whether he keeps running forward or turns
52. What is X’s average speed for the entire journey (to and
back, he will reach the end of the tunnel at the same time
fro)?
the train reaches that end. The speed at which the man is
(a) 176 m/h (b) 180 m/h running is:
(c) 165 m/h (d) Data insufficient (a) 6 Km/h (b) 8 Km/h
(c) 12 Km/h (d) 10 Km/h
53. Two straight roads R1 and R2 diverge from a point A at
[Based on JMET, 2009]
an angle of 120º. Ram starts walking from point A along
R1 at a uniform speed of 3 Km/h. Shyam starts walking 57. A father runs after his son, who is 1000 m ahead. The father
at the same time from A along R2 at a uniform speed runs at a speed of 1 Km every 8 minutes and the son runs at
of 2Km/h. They continue walking for 4 hrs along their a speed of 1 Km every 12 minutes. How much distance has
respective roads and reach points B and C on R1 and R2, the son covered at the point when the father overtakes him?

Chapter 12.indd 24 6/5/2015 3:11:01 PM


Time and Distance 12.25

(a) 2500 m (b) 2000 m 63.


Rajesh walks to and fro to a shopping mall. He spends
(c) 1500 m (d) 1000 m 30 minutes shopping. If he walks at speed of 10 Km an
[Based on JMET, 2009] hour, he returns to home at 19.00 hrs. If he walks at 15 Km
an hour, he returns to home at 18.30 hrs. How fast must he
A Mariti 800 crosses a lamp post in 4 × 4 2 s. If the speed
58. walk in order to return home at 18.15 hrs?
of the car is denoted by 2x m/s and length of the car is 23.25 (a) 17 Km/h (b) 17.5 Km/h
m, then actual speed of the car is: (c) 18 Km/h (d) None of these
(a) 3.25 m/s (b) 4 m/s [Based on XAT, 2009]
1
(c) 8 m/s (d) 1 m/s 64.
Mukesh, Suresh and Dinesh travel from Delhi to Mathura
4 to attend Janmashtmi Utsav. They have a bike which can
[Based on ATMA, 2008] carry only two riders at a time as per traffic rules. Bike
can be driven only by Mukesh. Mathura is 300 Km from
59.
A car driver driving in fog, passes a pedestrian who was
Delhi. All of them can walk at 15 Km/h. All of them start
walking at the rate of 2 Km/h in the same direction. The
their journey from Delhi simultaneously and are required
pedestrian could see the car for 6 minutes and it was visible to
to reach Mathura at the same time. If the speed of bike is
him upto a distance of 0.6 km. The speed of the car would be:
60 Km/h, then what is the shortest possible time in which
(a) 8 Km/h (b) 800 m/h all three can reach Mathura at the same time?
(c) 200 m/h (d) 15 Km/h 2 2
(a) 8 hhrs (b) 9 hhrs
[Based on ATMA, 2006] 7 7
60.
Two stations A and B are 110 Km apart on a straight line. (c) 10 hrs (d) None of these
One train train starts from ‘A’ at 7 am and travel towards [Based on IIFT, 2010]
‘B’ at 20 Km/h speed. Another train starts for ‘B’ at 8 am 65.
Two motorists Anil and Sunil are practicing with two different
and travel towards ‘A’ at 25 Km/h speed. At what time sports cars; Ferrari and Maclarun, on the circular racing track,
will they meet? for the car racing tournament to be held next month. Both
(a) 9 a.m. (b) 10 a.m. Anil and Sunil start from the same point on the circular track.
(c) 11 a.m. (d) None of these Anil completes one round of the track in 1 minutes and Sunil
[Based on NMAT, 2006] takes 2 minutes to complete a round. While Anil maintains
same speed for all the rounds, Sunil halves his speed after the
61. The speed of a train pulling out of a station is given by completion of each round. How many times Anil and Sunil
the equation s = t2 + t, where s is the speed in kilometers will meet between the 6th round and and 9th round of Sunil
per hour and t is the time is seconds from when the (6th and 9th round is excluded)? Assume that the speed of
train starts moving. The equation holds for all situations Sunil remains steady throughout each round and changes
where 0 ≤ t ≤ 4. In kilometers per hour, what is the only after the completion of that round.
difference in the speed of the train four seconds after it
starts moving, compared to the speed two seconds after (a) 260 (b) 347
it starts moving? (c) 382 (d) None of these
(a) 14 (b) 6 [Based on IIFT, 2009]
(c) 0 (d) 20 66.
Fortuner, the latest SUV by Toyota Motors, consumes
[Based on NMAT, 2005] 1  1000  
diesel at the rate of    x  L/km, when
62.
Amarendra and Dharmendra are brothers. One day they 400  x  
start at the same time from their home for Tatanagar driven at the speed of x Km/h. If the diesel is `35/L and
railway station in their respective cars. Amarendra took the driver is paid at the rate of `125 per hour, then find the
25 minutes to reach the station. After reaching the station approximate optimal speed (in Km/h) of Fortuner that will
Amarendra found that Dharmendra is 2500 m away from minimize the total cost of the round trip of 800 km.
the station. The distance of Tatanagar Station from their (a) 49 (b) 55
home is 15 km. Next day Dharmendra decided to start (c) 50 (d) 53
7 minutes early. If they drive at the speed same as the
[Based on IIFT, 2009]
previous day then Amarendra will reach the station:
(a) 120 seconds earlier than Dharmendra 67.
The Ghaziabad-Hapur-Meerut EMU and the Meerut-
(b) 120 seconds later than Dharmendra Hapur-Ghaziabad EMU start at the same time from
Ghaziabad and Meerut and proceed towards each other at
(c) 300 seconds earlier than Dharmendra
16 Km/h and 21 Km/h, respectively. When they meet, it
(d) 300 seconds later than Dharmendra is found that one train has travelled 60 Km more than the
 [Based on XAT, 2010)] other. The distance between two stations is:

Chapter 12.indd 25 6/5/2015 3:11:02 PM


12.26 Chapter 12

(a) 445 km (b) 444 km progression. If the men meet x Km nearer R than S in an
(c) 440 km (d) 450 km integral number of hrs, then x is:
[Based on IIFT, 2007] (a) 10 (b) 8
(c) 6 (d) 4
68.
Laxman and Bharat decide to go from Agra to Delhi for [Based on FMS, 2010]
watching a cricket match and board two different trains
for that purpose. While Laxman takes the first train that 72.
A flight of Jet Airways from Delhi to Mumbai has an
leaves for Delhi, Bharat decides to wait for some time average speed of 700 Km/h without any stoppage,
and take a faster train. On the way Laxman sitting by whereas a flight of Kingfisher from Delhi to Mumbai has
the window-seat noticed that the train boarded by Bharat an average speed of 560 Km/h with stoppage at Baroda.
crossed him in 12 s. Now, the faster train can travel What is the average stoppage time per hour of kingfisher
180 Km in 3 hrs, while the slower train takes twice as flight if both the planes fly at the same speed?
much time to do it. Given this, mark all the correct options. (a) 8 minutes (b) 12 minutes
(a) If the faster train has taken 30 seconds to cross the (c) 16 minutes (d) 24 minutes
entire length of the slower train, the difference [Based on FMS, 2009]
between the lengths of the two trains is 50 m. 73.
A boy is running at a speed of p Km/h to cover a distance of
(b) If the faster train had been running twice as much 1 km. But, due to slippery ground, his speed is reduced by q
faster, it would have taken 10 seconds to overtake the Km/h (p > q). If he takes r hour to cover the distance, then:
slower train. 1 1 1 1
(c) Had the faster train taken 24 seconds to cross the (a)   (b)  p  q
r p q r
entire length of the slower train, the length of the
slower train would have been 100 m. (c) r = p + q (d) r = p – q
(d) If the slower train had been running at one and a half [Based on FMS, 2006]
times of its current speed, the faster train would have 74.
The ratio between the rates of walking of A and B is 2:3
taken 24 seconds to overtake Laxman. and therefore A takes 10 minutes more than the time taken
[Based on IIFT, 2006] by B to reach the destination. If A had walked at double
69.
Two cyclist, k kilometres apart, and starting at the same the speed, he would have covered the distance in:
time, would be together in r hrs if they travelled in the (a) 15 minutes (b) 20 minutes
same direction, but would pass each other in t hrs if they (c) 25 minutes (d) 30 minutes
travelled in opposite direction. The ratio of the speed of [Based on FMS, 2005]
the faster cyclist to that of the slower is: 75.
A cyclist drove one kilometre, with the wind in his back,
r t r in three minutes and drove the same way back, against the
(a) (b)
r t r t wind in four minutes. If we assume that the cyclist always
puts constant force on the pedals, how much time would it
rt r
(c) (d) take him to drive one kilometer without wind?
r t
[Based on FMS, 2011]
1 3
(a) 2 minutes (b) 3 minutes
3 7
70.
A man drives 150 Km to the seashore in 3 hrs and 20
minutes. He returns from the shore to the starting point 3 7
(c) 2 minutes (d) 3 minutes
in 4 hrs and 10 minutes. Let, r be the average rate for 7 12
the entire trip. Then, the average rate for the trip going [Based on SNAP, 2008]
exceeds r, in kilometres per hour, by: 76.
Two identical trains A and B running in opposite directions
1 at same speed take 2 minutes to cross each other completely.
(a) 5 (b) 4
2 The number of bogies of A are increased from 12 to 16. How
(c) 4 (d) 2 much more time would they now require to cross each other?
[Based on FMS, 2010] (a) 40 seconds (b) 50 seconds
(c) 60 seconds (d) 20 seconds
71.
Two men at points R and S, 76 Km apart, set out at the [Based on SNAP, 2007]
same time to walk towards each other. The man at R
77. The difference between the time taken by two cars to travel
1
walks uniformly at the rate of 4 Km/h; the man at S a distance of 350 Km is 2 h 20 min. If the difference between
2 their speeds is 5 Km/h then the speed of the faster car is:
1
walks at the constant rate of 3 Km/h for the first hour (a) 30 Km/h (b) 35 Km/h
4
3 (c) 40 Km/h (d) 45 Km/h
at 3 Km/h for the second hour, and so on, in arithmetic
4 [Based on CAT, 2009]

Chapter 12.indd 26 6/5/2015 3:11:02 PM


Time and Distance 12.27

78. A car A starts from point P towards another point Q. 80. If Sanjit walks to school and rides his bicycle back home
Another car B starts (also from P) 1 hr after the first car it takes him 90 min. If he rides his bicycle both ways it
and overtakes it after covering 30% of the distance PQ. takes him 30 min. How many hours would it take him to
After that, the cars continue. On reaching Q, car B reverses make the trip to school and back by walking, assuming
1 that he walks at a constant speed and that he cycles at a
and meets car A, after covering 23 of the distance QP.
3 constant speed?
Find the time taken by car B to cover the distance PQ (in (a) 2 h (b) 2.5 h
hours). (c) 3 h (d) 3.5 h
(a) 3 (b) 4 [Based on MAT, 2013]
1 81. A man by walking diametrically across a circular grass
(c) 5 (d) 3
3 plot finds that it has taken him 45 seconds less than if he
[Based on CAT, 2012] had kept to the path round the outside. If he walks at 90 m/
79. Vidya leaves the studio every day at 6.00 pm to meet min, what is the circumference of the grass plot?
Salman at 7.00 pm at point X stopping anywhere along (a) 251.23 m (b) 371.25 m
the way. One day she left the studio at the usual time i.e., (b) 421.32 m (d) 125.12 m
6.00 pm and travelled the first half of the distance at 3/4th [Based on MAT, 2013]
of her original speed. At what speed must she travel the
82. Train A running at 60 Km/h leaves Mumbai for Delhi at 6
second half of the distance to reach the point X on time?
p.m. Train B running at 90 Km/h also leaves for Delhi at
3 9 p.m. if all the three trains C leaves Delhi for Mumbai at
(a) At times of her usual speed
2 9 p.m. if all the three trains meet at the same time between
(b) At 2 times of her usual speed Mumbai and Delhi, then what is the speed of train C, if
1 distance between Delhi and Mumbai is 1260 km?
(c) At times of her usual speed
6 (a) 60 Km/h (b) 90 Km/h
(d) None of these (c) 120 Km/h (d) 135 Km/h
[Based on MAT, 2013] [Based on, 2012]

Answer Keys
Difficulty Level-1

1. (a) 2. (a) 3. (b) 4. (c) 5. (c) 6. (a) 7. (c) 8. (a) 9. (a) 10. (d ) 11. (c) 12. (c) 13. (b)
14. (c) 15. (c) 16. (c) 17. (b) 18. (a) 19. (b) 20. (c) 21. (a) 22. (b) 23. (a) 24. (a) 25. (b) 26. (a)
27. (c) 28. (b) 29. (c) 30. (d ) 31. (c) 32. (a) 33. (d ) 34. (b) 35. (b) 36. (a) 37. (a) 38. (d ) 39. (d)
40. (d ) 41. (b) 42. (c) 43. (d ) 44. (b) 45. (b) 46. (c) 47. (d ) 48. (d ) 49. (b) 50. (a) 51. (b) 52. (c)
53. (b) 54. (c) 55. (b) 56. (a) 57. (a) 58. (b) 59. (a) 60. (a) 61. (b) 62. (c) 63. (d ) 64. (a) 65. (a)
66. (c) 67. (a) 68. (a) 69. (b) 70. (b) 71. (b) 72. (a) 73. (c) 74. (a) 75. (a) 76. (c) 77. (c) 78. (a)
79. (a) 80. (c) 81. (c) 82. (b) 83. (c) 84. (c) 85. (d ) 86. (b) 87. (a) 88. (b) 89. (b) 90. (d ) 91. (d )
92. (c) 93. (a) 94. (a) 95. (c) 96. (c) 97. (c) 98. (c) 99. (b) 100. (d ) 101. (c) 102. (b) 103. (a) 104. (d )
105. (c) 106. (a) 107. (a) 108. (a) 109. (d ) 110. (b) 111. (c) 112. (c) 113. (b) 114. (d) 115. (b) 116. (c) 117. (b)
118. (b) 119. (c) 120. (a) 121. (c) 122. (d ) 123. (d ) 124. (a) 125. (c) 126. (b) 127. (a) 128. (a) 129. (c) 130. (a)
131. (b) 132. (c) 133. (a) 134. (b) 135. (c) 136. (b) 137. (a) 138. (d ) 139. (b) 140. (a) 141. (c) 142. (c) 143. (b)
144. (b) 145. (c) 146. (b) 147. (c) 148. (b) 149. (d ) 150. (c) 151. (b) 152. (d ) 153. (a) 154. (b) 155. (b) 156. (a)
157. (a) 158. (b) 159. (b)

Difficulty Level-2
1. (b) 2. (b) 3. (b) 4. (c) 5. (b) 6. (b) 7. (a) 8. (b) 9. (c) 10. (b) 11. (c) 12. (a) 13. (b)
14. (d ) 15. (c) 16. (d) 17. (c) 18. (b) 19. (b) 20. (c) 21. (b) 22. (b) 23. (b) 24. (d ) 25. (a) 26. (a)
27. (b) 28. (b) 29. (b) 30. (c) 31. (d ) 32. (c) 33. (b) 34. (b) 35. (d ) 36. (c) 37. (d ) 38. (a) 39. (d)
40. (b) 41. (c) 42. (b) 43. (b) 44. (a) 45. (a) 46. (d ) 47. (b) 48. (c) 49. (b) 50. (b) 51. (a) 52. (a)
53. (b) 54. (d ) 55. (b) 56. (d ) 57. (b) 58. (b) 59. (a) 60. (b) 61. (a) 62. (b) 63. (d ) 64. (b) 65. (c)
66. (a) 67. (b) 68. (a, c, d) 69. (a) 70. (a) 71. (d ) 72. (b) 73. (b) 74. (a) 75. (b) 76. (d ) 77. (a)
78. (d ) 79. (a) 80. (b) 81. (b) 82. (c)

Chapter 12.indd 27 6/5/2015 3:11:04 PM


12.28 Chapter 12

Explanatory Answers

Difficulty Level-1

1.
(a) Let the speed of train be V and speed of Ajay be v. 5 25 75
Then, by relative speed \ × 75 +
= ⇒ x = 60.
6x 2 × 60 x
10(V + V) = 12V (1)
6. (a) A covers 3.5 Km before meeting B in (18 × 3.5) + 3 =
Similarly, 66 minutes
66
T(V – v) = 12V (2) B covers a distance of 5.5 Km in 66 minutes, i.e.,
60
From Eqs. (1) and (2), 11
hrs, i.e., hrs.
V v V v 12 12 10
 = 
V V 10 T 11 10
\ B’s speed = × = 5 Km/h.
10 2 11
⇒  T = 12 × = 15 minutes.
7.
(c) Wheel of diameter 7 cm starts from X.
2.
(a) Let Amit’s speed = x Km/h Its circumference = 2 × p × 3.5 = 22 m
Let Suresh’s speed = y Km/h Wheel of diameter 14 cm starts from Y.
30 30 Its circumference = 44 m
\
= + 2 (1)
x y Distance from X to Y = 1980 cm
If Amit’s speed becomes 2x Km/h, then Let both the wheels make x revolutions per second.
30 30
= – 1 (2) \ Distance covered by both the wheels in one second
2x y
= 22x + 44x = 66x cm
From Eqs. (1) and (2)
Distance covered by both the wheels in 10 seconds
30 30
⇒ – 2 = +1
x 2x = 660x cm
⇒ x = 5, y = 7.5. \ 660x = 1980 ⇒ x = 3
3.
(b) Suppose car 1 starts at 12 noon \ Speed of the smaller wheel = 22x/s
\ Car II starts at 6 p.m. = 66 cm/s.
Time Distance Covered Distance Covered 8.
(a) For express train, the time needed to travel 75 km
by Car I by Car II 60
= × 75
7 p.m. 210 km 50 km 100
12.00 night 360 km 300 km
= 45 (plus 3 min. stoppage) = 48 minutes.
3 a.m 450 km 450 mn
The time needed to travel 300 Km = 192 minutes
\ Car II will crosss Car I after 9 hrs. The time needed to travel 600 Km
4.
(c) Let the speed of the car be x Km/h. = 384 minutes – 3 minutes = 381 minutes
Speed of the train = x + 20% of x
\ For local train, time needed to travel 25 km
6x = 30 (plus 1 minute. stoppage) = 31 minutes
= Km/h
5 The time needed to travel 300 Km = 372 minutes
5 25 75
\ × 75 + = ⇒ x = 60. 50
6x 2 × 60 x In 9 min, the local train will cover × 9 = 7.5 Km
60
5.
(c) Let the speed of the car be x Km/h \ In 381 min, the local train will cover 307.5 Km.
Speed of the train = x + 20% of x
\ 9.
(a) Let the length of each train be x metres.
6x Relative motion of the faster train w.r.t the slower
= Km/h
5 train is the difference of their speeds i.e., 10 Km/h.

Chapter 12.indd 28 6/5/2015 3:11:05 PM


Time and Distance 12.29

\ The two trains will be clear of each other in 36 s y = 70 ­– x (If go in opposite direction)
which is the time taken in moving the distance of ⇒ x + y = 70 and x – y = 10
2x m @ 10 Km/h. ⇒ x = 40, y = 30.
i.e., 100 m/36 s
15.
(c) Let the correct time to complete the journey be x minutes.
⇒ 2x = 100
Distance covered in (x + 11) minutes at 40 Km/h.
⇒ x = 50
\ Length of each train = 50 m. = distance covered in (x + 5) minutes at 50 Km/h.

10.
(d) Suppose distance between each pole is 1 m. x + 11 x+5
\ × 40 = × 50
60 60
\ Total distance = 19 m
⇒ x = 19 minutes.
It takes 24 seconds to cover 11 m.
24 16.
(c) Total distance covered = 150 + 300 = 450 m
\ To cover 19 m, it will take × 19 = 41.45.
11 81
Time = seconds.
11.
(c) Suppose they meet after x minutes. 2
4500 meters are covered by Suresh in 60 minutes. 2 18
Speed = 450 × × = 40 Km/h.
81 5
4500
\ In x min, Suresh covers x metres . 17.
(b) Let the length of the bridge be x meters.
60
3750 meters are covered by Suresh’s wife in 60 minutes. 100 + x 25
\ = ⇒ x = 400.
72000 3600
3750
\ In x min, Suresh’s wife covers x metres
60 18.
(a) The relative speed of the train w.r.t. the speed of the
man
4500 3750
⇒ x+ x = 726 = speed of the train + speed of the man
60 60
⇒ x = 5.28. = 60 + 6 = 66 Km/h

12.
(c) The two men walk in opposite directions @ 62.5 m/ \ Time taken by the train in crossing the man = Time
min and 75 m/min respectively. taken in moving a distance equal to the length of
the train (110 m) @ 66 Km/h.

\ They will meet for the first time after
726 726 3600
= = 5.28 minutes. = × 100 = 6 seconds.
75 + 62.5 137.5 66000
13. (b) Let speed of the train be x Km/h 19.
(b) Because they cover 60000 miles in 60 minutes, i.e.,
Let speed of the car be y Km/h 1000 miles in one minute, therefore they should be
1000 miles apart, 1 minute before impact.
120 480
\ + =8 20.
(c) Let the speed on the smaller wheel be x cm/s.
x y
200 400 1 25 \ Distance covered by the smaller wheel in 10
+ = 8 = seconds = 10x cm.
x y 3 3
Distance covered by the smaller wheel in one
600 2400
⇒ + = 40 revolution
x y 22
= 2 × × 3.5 = 22 cm
600 1200 7
+ = 25
x y \ No. of revolutions made by the smaller wheel in
10 x 5x
1200 10 seconds = = (1)
⇒ = 15 ⇒ y = 80, x = 60 22 11
y
Distance covered by the bigger wheel in 10 seconds =
\ Required ratio = 60:80 = 3:4. (1980 – 10x) cm.
14.
(c) Let x Km/h and y Km/h be the speeds of the two cars \ No. of revolutions made by the bigger wheel in 10
respectively. 1980 − 10 x
seconds = (2)
7x – 70 = 7y (If go in same direction) 44

Chapter 12.indd 29 6/5/2015 3:11:06 PM


12.30 Chapter 12

5x 1980 − 10 x Sum of lengths of first and second train = 300 m


\ = ⇒ x = 66.
11 44 Length of the second train = 300 – 125 = 175 m.
21.
(a) Let the length of each train be x meters. 2
28.
(b) of the distance = 1200 km
Then the time in which the two trains are clear of each 5
other is the time taken by the faster train in moving a \ Whole distance = 3000 km
distance 2x m (sum of the lengths of the two trains)
w.r.t. the relative speed of 46 – 36 = 10 Km/h. Distance travelled by car = 1000 km

36 60 2 x \ Distance travelled by train = 800 km.


\ = × ⇒ x = 50.
60 10 1000
29. (c) Let the distance between Mumbai and Ahmedabad be
22.
(b) Distance Time x km.
80 km 5 hrs x
T1 takes 5 hrs, \ its speed = Km/h
d km 4 hrs 5
x
(80  d ) T2 takes 6 hrs, \ its speed = Km/h
Total average speed = = 14 6
(5  4)
⇒  d = 46 km They will meet after

⇒ Average speed for next 4 hrs = 11.5 Km/h. x 30


= hrs. i.e., 2 hrs 43 minutes approximately
x x 11
23.
(a) Let speed of the train be x Km/h and speed of the car 
5 6
be y Km/h.
i.e.,  9.43 a.m.
160 600
\ + =8
x y 30.
(d) pr2 = 616
616 × 7
240 520 1 41 ⇒ r2 = = 196
+ = 8 = 22
x y 5 5
⇒ r = 14
⇒ y = 100, x = 80.
22
24.
(a) Let original duration of the flight be x hrs. \ 2pr = Circumference = 2 × × 14 = 88 meters
7
3000 \ It will take 14.4 seconds for a runner to run around
\ Original speed =  Km/h
x a circular ground @ 22 Km/h.
3000 3000 31.
(c) Let distance travelled at 40 Km/h = x km
\ = – 100 ⇒ x = 5.
x +1 x
Then, distance travelled at 60 Km/h = (250 – x) km
25.
(b) Circumference of the wheel x 250  x
Given:  = 5
22 40 60
= 2 × × 35 = 220 cm
7 ⇒ 3x + 500 – 2x = 600
\ Number of revolutions made by the wheel during ⇒ x = 100 km.
the journey
2200000 × 30 32.
(a) Suppose the speeds of the trains are x m/s and y m/s
= = 300000 = 3 lakhs.
220 respectively.
26. (a) Suppose the distance covered = 4x km
110 + 130
Suppose the usual speed = 4x Km/h Then = 60 (1)
x−y
\ Time taken = 1 hr
Reduced speed = 3x Km/h 110 + 130
= 3 (2)
With this speed, the distance of 4x Km will be covered x−y
in 1 hour and 20 minutes. By Eqs. (1) and (2) we get
27.
(c) Relative speed = 18 Km/h = 5 m/s x = 42 m/s
Distance travelled in 60 seconds = 300 m and y = 38 m/s.

Chapter 12.indd 30 6/5/2015 3:11:07 PM


Time and Distance 12.31

33.
(d) Total bogies initially is 12 + 12 = 24 4x
\ Time taken in crossing the platform = seconds
Additional bogies = 16 – 12 = 4 20
24 bogies takes = 2 minutes x
Time taken in crossing the pole = seconds
2 20
1 bogy takes =
24 x 4x
\ + 24 =
2 1 20 20
4 bogies takes = ×4 = min ⇒ x = 160.
21 3
1 40.
(d)
= × 60 seconds = 20 s.
3
34.
(b) Let the hare take 4 leaps/m/min and hound 3 leaps/

min
1
1 Distance PQ = 60 × 6 = 380 km
\ Speed of hare = 4  1 = 6 m/min 3
2
380  4
1 1 Speed of Swift = = 80 Km/h
and speed of hound = 3 × 2 = 7 m/min 19
2 2
41.
(b) When the two boys meet for the first time, A covers
Initial distance = 30 m (60 – 2) = 48 Km and B covers (60 + 12) = 72 km
3
Difference in speed = m/min
2
Now, Q Speed × Time = Distance

3   As the speed of A is 4 Km/h slower than B, B


⇒ × t = 30 covers 4 Km more than A every one hour. Now, by the
2
time they meet, B has covered (72 – 48) = 24 Km more
⇒ t = 20 minutes than A. At the rate of 4 Km/h, B covers 24 Km more
Time taken by hound to catch hare is 20 minutes, hare in 6 hrs. Thus, they meet after 6 h. A covers 48 Km in
travels 20 × 6 = 120 m . 6 hrs. So, speed of A = 8 Km/h.
42.
(c) Let the journey be from A to B and the total distance
35. (b) According to question between A and B be D km.

270 18 Let the original speed of the train be V Km/h.


× = 36 seconds.
25 + 2 5

x 25/4 5
= 2
36. (a) Ratio of speeds= =
x1 4 4

By option method, we can conclude the speed of   Let the point where the accident occurred be C
slower train is 24 Km/h. and consider that the time the train would have taken
37. (a) to cover the distance CB at V Km/h be t hrs.
3
   Now, travelling at V, the time taken by the train
4
4
to cover the distance CB will be t , i.e., the train will
3
 t t
take  t   or extra time than what it would have
Ratio = 16:15  3 3
taken had no accident taken place.
38. (d) Relative speed = (5 – 4) Km = 1 km
1
35    The train was late by 3 hrs as hr
h was idle as it
Time taken = = 35 hrs. 2
1 was detained.
39. (d) Let the length of the train = x m t
= 3 hrs ⇒ t = 9 hrs
\ So, the length of the platform = 3x m 3

Chapter 12.indd 31 6/5/2015 3:11:08 PM


12.32 Chapter 12

Without any accident, the train would have taken 46.


(c) Relative speed = (40 – 22) Km/h
10 hrs for the entire journey, as accident occurred after
5
having travelled for an hour. = 18 × = 5 m/s
18
1 min = 300 m

⇒ L1 + L2 = 300
It is given that, had the accident occurred 90 Km
further, the train would have been late by 3 hrs. So, ⇒ 125 + L2 = 300
3
travelling at V over a distance of 90 km, train takes \ L2 = 175 m.
1 4
hr extra.
2 D D 40
47.
(d)  = (1)
V T = 90 (1) x x3 60
3 D D 40
and, V T1 = 90 (2) and,  = (2)
4 x2 x 60
T hrs and T1 hrs = The time taken to cover 90 Km \ x = 12 Km/h and D = 40 km.
3
respectively at V Km/h and V Km/h x x 25
4 48.
(d)  =  [Let x distance]
3 5 6 60
VT = VT1
4
6x  5x 25
4 1 =
T1 = T ⇒ T1 = T + T 30 60
3 3
x = 12.5 Km.
1 1 3 49.
(b) Relative speed of rockets
T = ⇒T= h
3 2 2
= (42000 + 18000) = 60000 miles per hour
3
Substituting T
= hrs in Eq. (1),   It means both of them together cover 60000
2
miles in 60 minutes, i.e., 1000 miles in one minute.
90 Therefore, they should be 1000 miles apart, 1 minute
    V= = 60 Km/h
3 before impact.
2
So, total distance between A and B = 60 × 10 = 600 km. 50.
(a) Suppose x Km is covered in t hrs

1 2 x
43.
(d) s = ut + at \ Original speed (s1) =
2 t
1 In second case when x/2 Km is covered in 2t hrs,
\ Distance (s) = 0  4   9.8  4  4
2 x /2 x
speed to (s2) = =
(\ a = g = 9.8; u = initial speed = 0) 25 4t
= 78.4 m . x /t
Hence, ratio of speeds = = 4 = 4:1.
x /4t
22
44.
(b) 2πr = 2   70 = 440 cm 51. (b) Suppose Ram has to travel 1 Km
7

Time taken to cover 1 Km in first case
Distance in 10 revolution = 440 × 10 cm
1
440  10 4400 18 = hr = 20 minutes
\ Speed =   3
5 5  100 5 Time taken to cover 1 Km in second case
Speed = 31.68 Km/h. 1
= hr = 15 minutes
4
45.
(b) Distance in one hour = 18 + 20 = 38 km
Difference in time to cover 1 Km = 20 minutes –
Relative speed (20 + 18) = 38 Km/h 15 minutes = 5 minutes
47.5 1 But Actual difference =15 minutes +15 minutes = 30
Time =  1 hhrs.
38 4 minutes

Chapter 12.indd 32 6/5/2015 3:11:10 PM


Time and Distance 12.33

If the difference is 5 minutes, distance = 1 km Hyderabad Nalgonda Chennai


50 km/h
1 X ← 100 km → Y 40 km/h
If the difference is 30 minutes, distance × 30 = 6 km.
5 At 6:30, distance between 2 trains = 30 km
52
(c) Distance covered by train in 50 minutes Time taken to travel this 30 km
48 30 1
= × 50 Km = 40 Km = = hhr
60 50 + 40 3
Now this distance has to be covered in 40 minutes.
1
40 = 20 min .
× 60 =
Hence, new speed of train = × 60 = 60 Km/h. 3
60
53.
(b) Suppose total journey = x km 57.
(a) Let the distance be d Km and normal speed be s Km/h.
According to the question, d
\ Normal time (t) = hhr (1)
2 s
x = 1200
5 1 d
\ t− = (2)
\ x = 3000 km 2 s+4
1
Distance covered by car = (3000) Km = 1000 km 1 d
3 and, t+ = (3)
3 s−4
Hence, the distance travelled by train
= 3000 – (1200 + 1000) Solving Eqs. (1), (2) and (3), we get

= 3000 – 2200 = 800 km. d = 60 km, s = 20 Km/h and t = 3 hrs.


58.
(b) Let the speed of train on onward journey be x Km/h.
54.
(c) Let the speed of train on level terrain = x Km/h
Then, the speed of train on return journey = 0.8 x Km/h
500 1 500
Total time = + +
x 2 0.8 x
1125 1
⇒ 23 = +
x 2
Then, the speed of train through mountainous 2
= (x – 10) Km/h ⇒ x = 1125 × = 50 Km/h
45
According to the question, \ Speed of train on return journey = 40 Km/h
188 111
+ =7 59.
(a) Let the distance covered by first train be x km.
x x − 10

Then, the distance covered by second train
188 x – 1880 + 111x
⇒ =7 = (x + 120) km
x( x − 10)

⇒ 7x2 – 369x + 1880 = 0 As both trains have travelled for same time.
⇒ x = 47 Km/h. x x + 120
\ =
50 60
55.
(b) Relative speed of the trains
= 45 + 30 = 75 Km/h ⇒ 60x = 50x + 6000

Distance to be covered = 500 + 500 = 1000 m ⇒ x = 600


Now 75 Km is covered in 1 hour = 3600 s \ Total distance = x + (x + 120) = 1320 km
3600 × 1000 60.
(a) Let the average speed be x Km/h.
\ 1000 m is covered in = = 48 s.
75 × 1000
800
Time taken by aircraft (t) = (1)
56.
(a) Distance travelled by X in 1 hr = 50 km x
1 40 80
Distance travelled by Y in hr = 20 km t− = (2)
2 60 x + 40

Chapter 12.indd 33 6/5/2015 3:11:11 PM


12.34 Chapter 12

From Eqs. (1) and (2),  10  1


⇒ 75   =
800 800 2  x ( x + 10)  4
= +
x x + 40 3 ⇒ x(x +10) = 3000 = 50 × 60
32000 2 \ x = 50 Km/h
⇒ =
x( x + 40) 3 300
\ Required time = = 5 hrs.
⇒ x (x + 40) = 48000 50 + 10
\ x = 200 Km/h. 66.
(c) Mohan can reach the middle in 12.5 minutes.
61.
(b) Let the speed of Scooty be x Km/h. Puran can reach the middle in 25 minutes.
1000 So, required time = 25 – 12.5 = 12.5 minutes.
Time taken by Scooty (t) = (1)
x 67.
(a) Speed of car A = 120 Km/h
Time taken by Auto
Speed of car B = 120 × 0.85 = 102 Km/h
1000
(t – 5) = (2) 1
x + 10 Distance traveled by car A in 1 hrs h
2
From Eqs. (1) and (2), 1
1000 1000 = 120 × 1 = 180 km
= +5 2
x x + 10 As the cars are travelling towards each other.
10000 668.4 − 180
⇒ =5 So, required time = = 2.2 hhrs
x( x + 10) 120 + 102

= 2 hrs 12 minutes.
⇒ x (x + 10) = 2000
5
\ x = 40 Km/h. 68.
(a) Length of train = 40 × (100 – 64) × = 400 m.
18
62.
(c) Actual time to reach that place
7 69.
(b) Let the speed of train be x Km/h.
= 22 × = 14 hrs 600 600
11 Then, = +4
So, total time saved = 22 – 14 = 8 hrs. x x+5

63.
(d) Let the speed of train be x Km/h.  5 
⇒ 600   =4
  As both the persons are walking in the same  x( x + 5) 
direction of train. ⇒ x(x + 5) = 750 = 25 × 30
⇒ x = 25 Km/h.
So, (x – 4.5) × 8.4 = (x – 5.4) × 8.5
⇒ 0.1 x = 8.1 70.
(b) Let the original time = T hrs
Let the original speed = x Km/h
⇒ x = 81 Km/h.
1500
64.
(a) Let the length of platform be x m, length of first train \ = T (1)
x
y
be y m and length of second train be m. As both 1500 30
2 and, =T– (2)
x + 250 60
trains are travelling in opposite direction.
y 5
Solving Eqs. (1) and (2),
So, y+ = (48 + 42) × × 12 Speed of plane = x = 250 – 1000
2 18
Discarding negative value,
3
⇒ y = 300
2 x = 750 Km/h
⇒ y = 200 m 71.
(b) Distance (D) = Speed (S) × Time (T)
5
Now, y + x = 48 × × 45 = 600  15 
18 \ D = 4 ×  T + 
⇒ x = 600 – 200 = 400 m.  60 
⇒ D = 4T + 1 (1)
65.
(a) Let the usual speed of car be x Km/h.
75 75 15  10 
Then, = + and, D = 6  T − 
x x + 10 60  60 

Chapter 12.indd 34 6/5/2015 3:11:12 PM


Time and Distance 12.35

D = 6T – 1 (2) ⇒ D = 6x
Solving Eqs (1) and (2), D 6x
Time taken by Ramesh = = = 2 hrs
T = 1 hr 3x 3x

D = 4 × 2 + l = 5 km. D 6x
Time taken by Suresh = = = 1.5 hrs
4x 4x
72.
(a) Let the normal speed of train = x Km/h
76.
(c) Length of bridge = 1000 m
Let the normal time of train = T hrs
Length of train = 500 m
300
Then, =T Total length = 1000 + 500 = 1500 m
x
300 1500 60
and, =T–2 Speed of train = × = 45 Km/h
x+5 1000 2

Solving Eqs (1) and (2), 192
77.
(c) Time taken by motorcyclist = = 6 hrs
x = 25 – 30 32

Discarding the negative value,



Speed of train = 25 Km/h.
Time taken by car = 6 – (2.5 + 0.5) = 3 hrs
73.
(c) Time taken by cycle = x minutes Ratio of the speeds of motor cycle:car
Time taken by scooter = y minutes = 3:6 = 1:2
So, x + y = 390 (1)
78.
(a) Let the speed of goods train be x Km/h.
and, 2x = 520 (2)
As the trains are running in opposite direction.
Solving Eqs. (1) and (2),
5
y = 130 minutes 187.5 = (50 + x) × ×9
18
Moving both ways by scooter = 2 × 130 = 260 minutes ⇒ 50 + x = 75
= 4 hrs 20 minutes. ⇒ x = 25 Km/h

74.
(a) Let the time of meet = t hrs 79.
(c) Time taken by 1st cyclist to complete a track
300
= seconds
7
Time taken by 2nd cyclist to complete a track

300
 20  = seconds
15  t −  + 20t = 450 8
 60 
300 300
⇒ t = 13 hrs \ Required time = LCM of and = 300 seconds

 1
Distance from A = 15 13 −  = 190 km 45 + 50 + 25
 3 80.
(c) Average speed of car =
45 50 25 × 3
+ +
75.
(a) Speed of Ramesh = 3x 15 25 25

Speed of Suresh = 4x 120


= = 15 Km/h
Let the distance = D 3+ 2+3

D D 1 81.
(c) Let the distance travelled at the speed of 50 Km/h be
− =
3x 4 x 2 x Km and (170 – x) Km at the speed of 100 Km/h.
D 1  1 x 170 − x
⇒   = Then, + =2
x  12  2 50 100

Chapter 12.indd 35 6/5/2015 3:11:13 PM


12.36 Chapter 12

⇒ 2x + 170 – x = 200 88.


(b) Time taken by car for one way
⇒ x = 30 km 6 h 45 min − 2 h 1
= = 2 h 22 min
25 5 2 2
82.
(b) Time taken to cover 25 Km = hrs = hrs Time taken in walking to go one way
40 8
Remaining distance 1
\ Remaining speed = = 6 hrs 45 minutes – 2h 22 min
Remaining time 2
1
    = 4 hrs 22 min
46 − 25 21 2
= = = 56 Km/h
5 3
1− \ Time taken in walking to go both ways
8 8
 1 
83.
(c) Let the trains will meet x hrs after the train from = 2  4 h 22 min 
 2 
station B started.
= 8 hrs 45 min
 45 
48 ×  x +  + 50 × x = 232
 60  89.
(b) Let length of the bridge be x m

⇒ 48x + 36 + 50x = 232 100 + x 25
\ = ⇒ x = 400
72000 3600
⇒ x = 2 hrs
Distance travelled by the train starting from station A 90.
(d) Train with a speed of 54 Km/h passes the man in
at the meeting point 20 seconds.

 45  5
= 48 ×  2 + 132 km \ Length of the train = 54 × × 20 = 300 m
= 18
 60 
Let the length of platform be x m.
84.
(c) Distance travelled by train in 4 hrs = 45 × 4 =180 km
5
Then, (300 + x) = 54 × × 36
180 × 1000 18
Number of telegraph poles = = 3600
50 \ x = 540 – 300 = 240 m
85.
(d) Let the speed of car be x Km/h. 91.
(d) Let the speed of the faster train be x Km/h and that of
As the pedestrain is walking in the same direction slower train be y Km/h.
0.6 Then, in first case, relative speed = (x – y) Km/h
\ (x – 2) = × 60
6 5
\ (130 + 110) = (x – y) × × 60
⇒ x = 6 + 2 = 8 Km/h 18
⇒ (x – y) = 14.4 (1)
22
86.
(b) Distance travelled = 2 × × 100
7 In second case, relative speed = (x + y) Km/h
5
22 \ (130 + 110) = (x + y) × × 30
2× × 100 18
\ Speed = 7 = 314.29 ≈ 314 m/min
2 ⇒ (x + y) = 288 (2)
From Eqs. (1) and (2),
87.
(a) The relative speed of the train with respect to the
speed of the man 2x = 302.4
= speed of the train + speed of the man 302.4
\ x = = 151.20 Km/h
= 60 + 6 = 66 Km/h 2
\ Time taken by the train in crossing the man 92.
(c) Let the distance of the journey be x km.
= Time taken in moving a distance equal to the x x
length of the train (110 m) at the rate of 66 Km/h. Then, time taken to cover Km = h
2 8
3600 x x
= × 100 = 6 seconds Time taken to cover km = h
66000 3 36

Chapter 12.indd 36 6/5/2015 3:11:14 PM


Time and Distance 12.37

x x 18
and, time taken to cover km = h ⇒ x – 6 = 10 ×
6 54 5
x x x 31 ⇒ x = 42 Km/h
Given, + + = Let the speed of second person be y Km/h.
8 36 54 5 5 75
⇒ x ≈ 36 km Then, (42 − y ) × = ×4
18 24
93.
(a) Length of train = 12 × 15 = 180 m 100 18
⇒ 42 – y = ×
Time = 18 s 9 5
180 ⇒ y = 42 – 40 = 2 Km/h
Speed = = 10 m/s 18
18 98.
(c) Distance travelled by train A in 3 hrs
New distance = 15 × 10 = 150 m = 3 × 60 = 180 km
150 Relative speed = 72 – 60 = 12 Km/h
\ Required time = = 15 s 180
10 Time taken to meet = = 15 hrs
12
94.
(a) So, two trains meet at (2 pm + 15 h) = 5 pm on next
day
99.
(b) Let he travel x Km by train.
x
\ Total time he travelled by train = h
55

285 − x
Total time he travelled by car = h
Distance travelled in 2 hrs will be 2x and 2(x + 5) 40
respectively. x 285 − x
\ + =6
\ 2 2
(50) = (2x) + [2(x + 5)]2 55 40
⇒ 8x + 3135 – 11x = 2640
2500 = 4x2 + 4x2 + 100 + 40x ⇒ 3x = 495
⇒ 8x2 + 40x – 2400 = 0 ⇒ x = 165 km
x2 + 5x – 300 = 0 100.
(d) Let the distance from P to R is x km.
(x – 15) (x + 20) = 0 ⇒ x = 15 Km/h As both A and B travel for same time.
95.
(c) x 21  (21  x)
\ =
3 4


⇒ 4x = 126 – 3x  ⇒  x = 18 km
101.
(c) As the trains are running in opposite directions.
 1.10 + 0.90 
\ Required time =   hhrs
 60 + 90 
2
= × 60 × 60 s = 45 s
Clearly, the radius of the circle is 20 km. Hence, one 150
has to cover a distance of 20 Km to reach a point 102.
(b) Suppose the distance is 270 km.
between B and C.
[Q L.C.M. of 54 and 45 = 270]
96.
(c) Let the normal speed be x Km/h. Without stoppage, time taken in the whole journey
300 300 270
\ = +2 = = 5 hhrs
x x+5 54
⇒ x = 25 With stoppage time taken in the whole journey
270
97.
(c) Let the speed of train be x Km/h. = 6 hrs
h
45
5 75 (6 − 5) 1
Then, ( x − 6) × = ×2 \ Stoppage per hour = = hrs = 10 minutes
18 15 6 6

Chapter 12.indd 37 6/9/2015 2:31:59 PM


12.38 Chapter 12

103.
(a) Wherever the two trains may meet, they will be Average speed of the train leaving
equidistant from a given place. 200 × 2 400
Ambala Cantt = = Km/h
n 7 7
104.
(d) We have [2 × 40 + (n – 1)5] = 385 By the time the other train starts from Ambala Cantt
2
the first train had travelled 100 km.
n
⇒ (80 + 5n – 5) = 385 Therefore, the trains meet after
2
⇒ 5n2 + 75n – 770 = 0 100 100 × 7 2×7 14
= = hrs = × 60 = 56 minutes
400 750 15 15
\ n = 7 50 +
7
105.
(c) Let the body moving downwards take ‘t’ s to reach Hence, they meet at 8:56 a.m.
half the height. 109.
(d) Let the length of first train be x m and that of bridge is
245 ym
⇒ = × 9.8 × t2 (g = 9.8)
90 × 5
Speed = 90 Km/h = = 25 m/s
18
245 × 2 25 5
⇒ t2 = = ⇒t= s Q D = S × T
8 × 9.8 4 2
\ (x + y) = 25 × 36 = 900 (1)
Again, assume that the second body is projected with
Length of another train = (x – 100)
velocity v upwards
45 × 5
245 5 245 Speed = m/s
⇒ = v× − 18
8 2 8 Q D = S × T
5 245 245 245 45 × 15
⇒ v× = + = \ (x – 100 + y) = ×t
2 8 8 4 18
25
245 2 49 ⇒ (900 – 100) = × t [from Eq. (1)]
= × = = 24.5 m/s 2
4 5 2
800 × 2
106.
(a) The trains will cross each other after ⇒ t = = 64 seconds
25
2000 + 200 + 300 2500 110.
(b) Let the speed of the train be x Km/h and the speed of
= = 50 seconds
20 + 30 50 car be y Km/h.
107.
(a) The coin will move up with a initial velocity of 10 m/s 160 600
Then, + = 8 (1)
till it comes to rest. Time taken is given by x y
10 240 520 1 41
0 = 10 – 9.8 t ⇒ t = s and, + = 8 = (2)
9.8 x y 5 5

Time taken to reach the ground from the highest point From Eqs. (1) and (2), x = 80 and y = 100
10 39.2 − 10 29.2 \ Speed of the train = 80 Km/h
= 4− = = s
9.8 9.8 9.8 Speed of car = 100 Km/h
Velocity of coin on impact 111.
(c) Let the speed of the bus be x Km/h.
29.2 \ Speed of the car = (x + 25) Km/h
=0+ × 9.8 = 29.2 m/s
9.8 500 500
\ = + 10
If ‘h’ is the height from which the coin was dropped, x x + 25
then (29.2)2 – (10)2 = 2 × 9.8 × hrs ⇒ hrs = 38.4 m ⇒ x2 + 25x – 1250 = 0
108.
(a) Average speed of the train leaving Delhi ⇒ x = 25
200 \ Speed of the bus = 25 Km/h
= = 50 Km/h
4 Speed of the car = 50 Km/h

Chapter 12.indd 38 6/9/2015 2:32:00 PM


Time and Distance 12.39

112.
(c) Let the length of the journey be x Km and speed of ⇒ 192 (x – 16) = 192x – 2x(x – 16)
train be V Km/h.
⇒ x = 48
x 115.
(b) Let the distance be x Km and time be t hrs.
Then, = t – 4 (1)
V +6 Now, according to the question,

x x
and, = t + 6 (2) x
V –6 v1 = and v2 = 2

t 2t
x x
\ − = 10 x x
V −6 V +6 or, v1:v2 =
: = 4:1
t 4t
⇒ 12x = 10(V2 – 36) (3) 116.
(c) Let x Km/h and y Km/h be the speeds of the two cars
From Eq. (1), respectively.
7x – 70 = 7y (If go in same direction)
x x
= −4 y = 70 – x
V +6 V
x(V + 6 – V )
⇒ 4 = (If go in opposite directions)
V (V + 6)
⇒ x + y = 70
4 x and, x – y = 10
⇒ =
6 V (V + 6)
x = 40, y = 30
2 117.
(b) Let speed of the train be x Km/h. Let, speed of the car
\ x = V(V + 6)
3 be y Km/h.
2 120 480
From Eq. (1), 12 × V(V + 6) = 10(V2 – 36) \ + =8
3 x y

⇒ V = 30 200 400 1 25
+ = 8 =
x y 3 3
\ x = 720 km
600 2400
⇒ + = 40
113.
(b) Speed of the aeroplane in still air = 320 Km/h Speed x y
of the wind = 40 Km/h
600 1200
\ Aeroplane will travel with the wind at + = 25
x y
(320 + 40) = 360 Km/h
1200
Aeroplane will travel against the wind at ⇒ = 15
y

(320 – 40) = 280 Km/h. ⇒ y = 80, x = 60
Suppose distance to be travelled = K km. \ Required ratio = 60:80 = 3:4
118.
(b) Suppose the usual speed = K Km/h
\ = (135 minutes)
280
1500 1 1500
− =
280 × 9 K 2 K + 250
i.e., K = = 630
⇒ K = 750 Km/h

630 119.
(c) Length of the trains = 120 m
Therefore, it takes hrs to cover a distance of
360 Relative velocity of the two trains running in opposite
K = 630 at 360 Km/h. i.e., 105 minutes directions = (40 + 20 =) 60 m/s
Time taken to cross each other
114.
(d) Let the speed of the fast train be x Km/h.
120  120
\ Speed of the slow train = (x – 16) Km/h = 4 seconds
60
192 192 120.
(a) Suppose the average speed of train = x Km/h
\ = −2
x x − 16 \ Average speed of car = (x – 6) Km/h

Chapter 12.indd 39 6/5/2015 3:11:18 PM


12.40 Chapter 12

260 Substituting the value of x in Eq. (1),


\ Time taken by car to cover 260 Km = hr
x−6 120 480
+ =8
260 0.75y y
Time taken by train to cover 260 Km = hr
x 160 480
or, + =8
According to the question, y y
260 260 260( x − x + 6) 640
− = 3 or, =3 or, y = = 80 Km/h.
x−6 x x( x − 6) 8

or, 520 = x2 – 6x (d) Area of circular field = pr2 = 616
123.
2
or, x – 6x – 520 = 0
616 616
or, x2 – 26x + 20x – 520 = 0 or, r2 == = 196
π 22 / 7
or, (x – 26)(x + 20) = 0
\ r = 14 km
\ x = 26 or, x = –20
Circumference of the circular field
Since x cannot be –ve,
\ x = 26 Km/h. 22
= 27pr = 2 × × 14 = 88 km
7
121.
(c) Suppose normal speed of train = x Km/h
Time taken to cover 22 Km = 1 hr
\ Increased speed of train = (x + 5) Km/h
1
According to the question, \ Time taken to cover 88 Km = × 88 = 4 hrs
22
300 300 300( x  5)  300 x
 = 2 or, =2 124.
(a) 1st speed = 6 Km/h
x ( x  5) x( x  5)
Distance covered by man in 50 @ 6 Km/h
or, 300x +1500 – 300x = 2x(x + 5)
50
or, 1500 = 2x2 + 10x = 6 × = 5 km
60
or, x2 + 5x – 750 = 0
2nd speed = 10 Km/h
or, x2 + 30x – 25x – 750 = 0
Time taken to cover 10 Km = 1 hour = 60 minutes
or, x(x + 30) – 25(x + 30) = 0
60
Time taken to cover 5 Km = × 5 = 30 minutes
or, (x – 25)(x + 30) = 0 10
or, x = 25 or, – 30
125.
(c) Speed of the train = 48 Km/h
\ x = 25 Km/h Time taken = 50 minutes
(neglecting the –ve sign). \ Distance covered by train in 50 minutes

122.
(d) Let speed of train = x Km/h and speed of car = y Km/h 48
= × 50 Km = 40 km
60
According to the question,
Now to cover the distance in 50 minutes speed
120 (600 − 120) = 48 Km/h
+ =8
x y Now to cover the distance in 40 minutes speed

or, 480x + 120y = 8xy (1)
48 × 50
= Km/h = 60 Km/h
200 400 40
and, + = 8.33
x y
126.
(b) Average speed for 1st 5 minutes
or, 384x + 192y = 8xy (2)
68 + 127 + 208 + 312 + 555
= m/mins
From Eqs. (1) and (2), 5
120y + 480x = 192y + 384x 1250
= m/mins = 250 m/mins
480x – 384x = 192y – 120y 5
or, 95x = 72y 250 × 60
= Km/h = 15 Km/h
\ x = 0.75y 1000

Chapter 12.indd 40 6/5/2015 3:11:18 PM


Time and Distance 12.41

Average speed for next 5 minutes = 33 Km/h n2 − n


or, 2n =
Hence, average speed for 4
15 + 33 48 \ n = 9 as n ≠ 0
10 minutes = = = 24 Km/h.
2 2
131.
(b) Speed in Km/h = 132 Km/h
127.
(a) Let the speed of the cyclist be k Km/h. 5 110
Speed in m/s = 132 × = m/s
1 18 3
\ Speed of the jogger = k km/2 hrs
2 Total distance to be covered = 110 + 165 = 275 m
1
= k km/hhrs 3 × 275
4 \ Time taken = = 7.5 ssec
110  
\ Ratio of the speeds of the jogger and the cyclist
132.
(c) 480 Km are to be covered in 8 hrs at the rate of
1
k 60 km/ hr. Hence increased speed
4 = 1
k 4 = 60 – 48 = 12Km/h.

128.
(a) Net distance travelled by monkey in 2 minutes = 3 minutes 133. (a) Let the speed of train and taxi be x Km/h and y Km/h
respectively.
\ In 10 minutes distance travelled = 15 minutes
Then, we have,
As it is given in the question that in first minute it
climbs 6 m and in other minute it slips down 3 m. And 300 200 11 3 2 11
+ = ⇒ + =
this happens al­ternatively. x y 2 x y 200
   Hence monkey will travel remaining 6 m in next 36 24 132 33
or, + = = (1)
minute. x y 200 50
\ Total time taken = (10 + 1) = 11 min. 260 240 28
and, + =
x y 5
129.
(c) When two or more persons start from the same place 26 24 28
at the same time and travel round a circle in the same ⇒ + = (2)
x y 50
direction or in opposite directions, then they will
be first together at the starting point again after an From equations (1) and (2), we get
interval of time which is the L.C.M. of the times in
which each of them makes one complete round. 10 33 28 5 1
= − = =
x 50 50 50 10
300 300
Here each one takes s and s in making one ⇒ x = 100 Km/h
7 8
complete round. 134. (b) Let the charge for the distance covered be ` x
300 300 and fixed charge be ` y
\ Required time = L.C.M. of and
7 8 Then, we are given,
10x + y = 85
L.C.M. of 300 and 300 300
= = s
= 300 sec. 15x + y = 120
H.C.F. of 7 and 8 1
Solving the above equations, we get x = Rs.7, y = `15
130.
(a) Let the second car overtakes the first car after n hrs Hence, fare for journey of 25 km
\ Distance covered by first car = Distance covered = ` (25x + y) = ` (25 × 7 + 15) = `190.
by second car
135. (c) Let the total distance travelled by the train be d km.
 1  n − 1
or, 10n = 8 +  8 +  + ... +  8 +  Speed of train = x
 2  2 
And time taken = t
1 Then, by the first condition, we have
or, 10n = 8n + [1 + 2 + ... + (n − 1)]
2
150 d − 150
1  n(n − 1)  + = t + 8 (1)
or, 10n = 8n +  v 3
v
2  2  5

Chapter 12.indd 41 6/5/2015 3:11:21 PM


12.42 Chapter 12

And by the second condition, we have 5


139. (b) Length of the journey = 1200 × = 3000 km
410 d − 410 2
+ =t+4
v 3 (2)
v Distance covered by air = 1200 Km
5
From equations (1) and (2) we have, 1
Distance covered by car = × 3000 = 1000 km
−360 360 3
+ =4
v 3 Thus, distance covered by train
−360 + 600 240 = 3000 – 1200 – 1000 = 800 Km
⇒ v= = = 60 km/h
4 4
d 140. (a) Remaining distance that has to to be covered
∴ Time t =
60 1
Now, from Eqq. (1), = 6 − × 6 = 3 km
2
150 d − 150 d 2
+ = +8 Remaining time = 45 − × 45
60 3
× 60 60 3
5 1
d − 150 d 5 = 45 − 30 = 15 min = h
⇒ − =8− 4
36 60 2 Thus, required speed to cover the remaining distance
5(d − 150) − 3d 11
⇒ = 3
36 × 5 2 in remaining time = = 12 km/h.
1/ 4
11
⇒ 2d − 750 = × 36 × 5 = 990
2
141. (c) Distance covered by first man = Distance covered by
1740
⇒ d= = 870 km. second man
2
136. (b) Total distance to be covered = 46 km 2 2
16 × 1 = speed of second man × 2
Total time to be taken = 1h 3 5
5 5
Time taken to cover 25= km 25 = h
5
h ⇒ Speed of second man = 16 × ×
40 8 3 12
Thus, remaining distance 21 Km is to be covered in 100 1
= = 11 km/h.
3 9 9
remaining time h.
8
21
Hence, = speed = km/h 56 km/h 142. (c) Distance covered by A in
3/8
137. (a) Let the original speed of the car be x Km/h. 1 1 1
3 h = 4 × 3 km
Then, we are given, 2 2 2
715 715 9 7 63
− =2 = × = km
x x + 10 2 2 4
⇒ 715( x + 10 − x) = 2 x ( x + 10) 1 3 1
Distance covered by B in 3 h = 5 × 3 km
⇒ 715 × 5 = x ( x + 10) 2 4 2
23 7 161
⇒ x 2 + 10 x − 3575 = 0 = × km = km
4 2 8
−10 ± 100 + 14300 1
⇒ x= Thus, distance between A and B at the end of 3 h
2 2
−10 + 120 161 63 161 − 126
⇒ x= = − =
2 8 4 8
110 36 3
⇒ x= = 55 km/h ( taking positive sign) = km = 4 km
2 8 8
138. (d) Let after time t h, the two men will be 8.5 Km apart.

Then, we have
5.5 × t − 5 × t = 8.5 143. (b) Let the speed of bus = x Km/h
⇒ 0.5t = 8.5 and speed of car = (x + 25) Km/h
⇒ t = 17 h Also, distance = 500 Km

Chapter 12.indd 42 6/5/2015 3:11:27 PM


Time and Distance 12.43

Then, we are given, ∴ They will meet 5 times in 1 hr and 2 times next 2 hrs
500 500 therefore, they will meet 7 times between 8 am to
− = 10 9:30 am
x x + 25
⇒ 500( x + 25 − x) = 10( x)( x + 25) 148. (b) Let the winning post be at a distance of x m.
⇒ x( x + 25) = 25 × 50
∴ If both of them reach at same time, then distance
⇒ x = 25 km/h covered by B = x − 80 m.
and ( x + 25) = 25 + 25 = 50 km/h.
2
Given speed of A = 1 Speed of B
Hence, speeds of the car and the bus are 50 Km/h and 3
25 Km/h respectively.
Speed of A 5
∴ =
144. (b) Let number of fridges produced in first year be ‘a’ and Speed of B 3
production increases by ‘d’ every year.
We have,
Clearly, production every year will form an AP.
Speed of A: speed of B = Distance covered by A:
Then, production in the third year, a + 2d = 600 (1)
Distance covered by B
and production in the seventh year, a + 6d = 700 (2)
On solving both equations, we get a = 550, d = 25 x 5
∴ =
Now, total production in 7 yr x − 80 3
⇒ 3 x = 5 x − 400
n ⇒ x = 200 m.
= [2a + (n − 1)d ]
2
7 149. (d) Let the speed of car be x Km/h.
= [2 × 550 + 6 × 25]
2
Speed of pedestrian = 2 Km/h
7
= × 1250 = 4375. 6
2 ∴ ( x − 2) × = 0.6
60
145. (c) Let speed of motorboat in still water be x Km/h. ⇒ x−2=6
Now, distance covered in downstream = distance ∴ x = 8 km/h.
covered in upstream
150. (c) Distance between telegraph poles = 50 m
( x + 6) × 6 = ( x − 6) × 8
⇒ 3 x + 18 = 4 x − 24 1000 m
Number of telegraph poles in 1 Km = = 20 poles
⇒ x = 18 + 24 50 m
Speed of train = 45 Km/h
= 42 km/h Distance covered by train in 4 hrs = 45 × 4 = 180 km

∴  Number of poles counted by a passenger
146. (b) Let the distance between college and Rohit home
= 180 × 20 = 3600.
= x km
Then, we are given, 151. (b) Let the speed of faster swimmer be x Km/h.
x x 10 + 5 75 3x
− = h Then, Speed of slower swimmer = ×x = km/h
40 50 60 100 4
5 x − 4 x 15 Now, distance covered by both in 2 h.
⇒ =
200 60 Faster swimmer = 2x Km and slower swimmer
200 × 15 3x 3x
x= = ×2 = km
60 4 2
= 50 km.
After, 2 hrs the distance between them is 100 km.
147. (c) A covers 2 rounds in 1 h, while B covers 3 rounds in 1 Semi-circular
hour of a circular track. path
Since, they are moving in the opposite direction
60
therefore minimum time when they first meet = r r
Straight path
5
= 12 min (via diameter)

Chapter 12.indd 43 6/5/2015 3:11:34 PM


12.44 Chapter 12

From the figure, 154. (b) Let the original time be T hours and original
(OE ) 2 + (OS) 2 = (ES) 2 speed be x km/min. We have
2 1500
 3x  =T (1)
=   + (2 x) 2 = (100) 2 x
 2 
1500 30
[Here, S = South direction, E = East direction, =T− (2)
x + 250 60
O = Origin]
Solving equations (1) and (2), we get
9x2 Speed of plane = x = 750 or − 1000 (not possible)
⇒ + 4 x 2 = 10000
4 ∴ x = 750 km/h
9 x 2 + 16 x 2
⇒ = 10000
4 155. (b) Distance (D) = Speed (S) × Time (T)
25 x 2 15 
⇒ = 10000 
4 D = 4×T + 
 60 
10000 × 4
⇒ x2 = D = 4T + 1 (1)
25
 10 
10000 × 4 D = 6 T − 
∴ x=  60 
25
D = 6T − 1 (2)
100 × 2
= = 40 km/h Solving equations (1) and (2), we get
5
∴ Speed of faster swimmer = 40 km/h
h. T =1 h

D = 4 ×1 + 1
152. (d) Distance covered by aeroplane in 9 h = 5 km.
= 9 × 756 = 6804 km
2 × 6804 156. (a) Speed of first train = 50 km/hr
Speed of helicopter = = 283.5 km/h
48 400
Speed of second train = km/hr.
Distance covered by helicopter in 18 hrs = 283.5 × 18 7
= 5103 km. At 8:00 am distance between two trains is 100 kms.
153. (a) Let the speed of train be v1 Km/h Relattive velocity
And speed of taxi be v2 Km/h. Then, we have 400
= 50 +
7
330 200 11 350 + 400
+ = =
v1 v2 2 7
3 2 11 750
⇒ + = (1) = km/h
v1 v2 200 7
260 240 336 100 × 7
+ = Time taken = × 60
v1 v2 60 7500
26 24 336 = 56 min.
⇒ + = (2)
v1 v2 600 Hence, the two trains meet each other at 8:56 am
m.
From Eqs. (1) and (2)
36 24 11 × 12
+ = 157. (a) The minimum distance is LCM of 40, 42, 45
v1 v2 200
36 24 336 = 23 × 32 × 51 × 71
⇒ + = = 2520 cm
v1 v2 600
10 132 336 = 25.2 m.
⇒ = −
v1 200 600 158. (b) Let the aeroplane covers x Km at a speed of 440 Km/h
396 − 336 1 and (x - 770) Km at a speed of 660 Km/h.
= =
600 10 Hence, it covers a total distance of (2x - 770) Km at a
∴ v1 = 100 km/h. speed of 500 Km/h.

Chapter 12.indd 44 6/5/2015 3:11:38 PM


Time and Distance 12.45

3/2x 2 x − 770 x x − 770


O E or, = +
500 440 660
or, x = 1760
2x 100 km
Therefore, the total distance covered
= 2x – 770
= 2 × 1760 − 770 = 2750 km.
S
Total distance 159. (b) Distance between the 1 st and 21 st post
Average speed = = (21 − 1) × 50 = 1000 m
Total time
2 x − 770 Therefore, the speed of train
⇒ 500 = = 1 km/min
x x − 770
+ = 60 Km/h.
440 660
2 x − 770 x x − 770
or = +
500 440 660
or x = 1760

Difficulty Level-2

1.
(b) Let d kilometres be the distance between A and B. 200 200
d or, = + 1 (1)
When A and B walk towards each other = u v
v  (v  2)
300 200
72 d = ,  i.e., 3v = 4u (2)
and if they walk in the same direction 2u v
60 (v  2)  v
Substitute in Eq. (1),
d
= 6, i.e., = 6 or d = 12 Km
2 200 200  3
= +1
d 72 u 4u
So, = and v = 4 Km/h 200 600
2v  2 60 ⇒ = +1
u 4u
Ratio of speeds = 4:6
200 50
⇒ = 1 ⇒ = 1,
2.
(b) When they pass for the first time, the combined 4u u
distance = Length of the pool. When they meet for i.e., u = 50 m/min = 3000 m/h = 3 Km/h
second time, the combined distance = 3 × length
of pool. As both have constant speeds, hence at the 4.
(c) Let the speed of the escalator be x steps per second.
second meeting each swimmer had covered 3 times
\ In 18 seconds the escalator will come down by 18x
as much distance as at the first meeting. Since the
steps and in 6 seconds it will come down by 6x
swimmer starting at the deep end had covered 18.5
steps. Since height of the stair way is same,
m when they first met, he covered 18.5 × 3 = 55.5 m
when they next met. It is clear than this distance is 30 + 18x = 34 + 6x \ x = 1/3
10.5 m more than the length of the pool which is \ height of the stair way = 30 + 18(1/3) = 36 steps
hence, 55.5 – 10.5 = 45 m.
5.
(b) Let the speed of plane be X Km/h. Then,
3.
(b)
7 7 22
 =
X  .125 X X  0.05 X 60

7 7 22
⇒  =
.875 X 1.05 X 60
Suppose the speed of the boy is u m/minute and the
speed of the tram is v m per minute. On solving it, we get X = 39.98 Km/h » 40 Km/h
300 200 100 7 7 14
= + +1 \  hour or 21 minutes.
u v u 40 40 40

Chapter 12.indd 45 6/5/2015 3:11:41 PM


12.46 I Chapter 12

6.
(b) For opposite direction \ For 1 round i.e., back and forth he will require
Relative speed = 80 Km/h 6.4 seconds.

Distance = 100 m 32
\ In 32 seconds he will complete = 5 rounds.
6.4
100  3600 9 12.
(a) Let after t hrs train Y overtakes train X, then by the
\ Required time = = seconds
80  1000 2 condition given in the problem
 = 4.5 seconds  5 5
Qt = P  t   ⇒ t (Q – P) = P
7.
(a) Since the trains are 100 Km apart, and the trains are  2 2
travelling towards each other at 40 and 60 Km/h, the
5P
trains will collide in one hour. The bird must be flying ⇒ t = hrs
at 90 Km/h during this time, so the bird travels 90 km. 2(Q – P )

Total distance 13.


(c) There are 35 floors between them.
8.
(b) Average speed =
Total time
Rate of descent of B = 36 floors per minute.
4 TV
Total distance = TV = TV + Rate of ascent of A = 34 floors per minute.
3 3
Thus, relative velocity = 34 + 36 = 70 floors per
= 75% of total distance minute.
 + 25% of total distance. Hence, B would have travelled
TV 1
Total time = T + 36  = 18 floors.
3S 2
Hence, average speed
14.
(d) As her father reached home 10 minutes earlier than
4 the usual time, therefore he saw his daughter five
TV mintues before the scheduled time and he turned back
4TVS 4VS
= 3 = = (saving 5 minutes each side). So he met her at 6.55
TV 3ST  TV 3S  V
T p.m., i.e., she walked for 25 minutes from the station
3S
towards her home before her father picked her up.
9.
(c) Since the trains are travelling in opposite direction
velocity for the boy in the faster train = 50 m/s. 15.
(c) Let the number of hrs be x.
Distance travelled = length of the train = 100 m Total distance travelled by A = 40 + 40 × x
Time taken by the ball from one train to the other Total distance travelled by B = 50 × x
100 40
= = 2 seconds 40 + 40 × x = 50 × x  ⇒  x =
50 10
Ball is thrown at 2 m/s, \ distance between the two ⇒ x = 4 hrs
trains = 2 × 2 = 4 m. 16.
(d) Let the distance between P and Q be x km.
10.
(b) If the trains are travelling in the same direction, speed x x x x 12
of faster train for the boy = 30 – 20 = 10 m/s. \ + = + + ⇒ x = 30.
10 15 12.5 12.5 60
\ In 2 seconds the front of the faster train travels 2 × 10
= 20 m 17.
(c) Respective speed of the trains = 15 Km/h

\ The balls hits at 20 m from the front of the train B. Let the total length of both the trains be x.
(35 + x) × 60
5 =
8000 15000
11.
(c) Train requires = 32 seconds to reach the target ⇒ x = 1,215 m
250
24 18.
(b) Let the speed of Mir for the first mile be v miles/min
Up time = = 2.4 seconds
10 ⇒ The speed of Mir for the 2nd, 3rd, 4th, . . . miles

24 4 16 64
Down time = = 4 seconds is v, v, v . . . respectively.
6 5 25 125

M12_KHAT6981_C12.indd 46 6/16/2015 3:56:30 PM


Time and Distance 12.47

Since the speed of Mir for the successive miles is in 1


K
4 1
geometric progression (with r = ), the speeds of Mir It takes 4 hrs to cover K km.
5 S 4
5
 4 3 1
for the 6th, 7th, 8th, 9th and 10th miles will be   K+ K
 5 4 4 K ( BS )
\ Average speed = =
times the speeds for the 1st, 2nd, 3rd, 4th and 5th 3 1 3 1
K K KS + KB
miles respectively. 4 +4 4 4
Hence the average speed of Mir of the last five miles B S
5 4 BS
 4 = Km/h.
is   times the average speed of the first five miles. 3S + B
 5
⇒ The time taken by Mir to cover the last five miles is 3
24.
(d) Upto 5.30 p.m., the plane A has covered X miles.
5 2
 4
  times the time taken by him to cover the first Suppose that the plane B overtakes plane A, T hrs after
5
5
5.30 p.m. Then,
 5 3125
five miles =    5 =  5 ≈ 3.05 × 5 ≈ 15.25 3 3X
 4 1024 TY =
X + TX ⇒ T = .
2 2(Y − X )
minutes ≈ 15 minutes and 15 seconds.
25.
(a) Since the train passes the man in 5 s, the relative speed
19.
(b) The motocyclist takes 6 hrs to complete the journey
form Delhi to Bharatpur and the person driving by car 100
of the train with respect to the man =  i.e., 20 m/s
will take 4 hrs. Ratio of the speed = 2:3 5
or 72 Km/h. So the speed of the train = 78 Km/h. The
1 2h − 1 train passes the car in 6 s, the relative speed of the
20.
(c) m miles in h − = hrs
2 2 train with respect to the car

2m  100  18
⇒ miles per hour. =   = 60 Km/h.
2h − 1  9  5
21.
(b) Relative speed of Vaibhav when he is moving up = \ Speed of the car = 78 – 60 = 18 Km/h
Vaibhav’s speed + speed of escalator = V + E
Relative speed of Vaibhav when he is moving down = 26.
(a) Relative speed of Manu with respect to Tanu
Vaibhav’s speed – speed of escalator = V – E = 12 + 18 Km/h = 30 Km/h
According to the question. Distance = Speed × Time
 1 1 x x 14
(V + E) =  6 − 2 +  , (V − E ) = = 30 × = 7 km
 2 2  30 90 60
where x is the total stationary length of the escalator. \ The distance covered by both of them, when they
cross each other for the first time is 7 km
 x x
Hence, Vaibhav’s speed, V = 0.5   
 30 90 

 4x  x
⇒        0.5   
 90  45

Hence he will take 45 seconds if the escalator is stationary.
Let the distance covered by Manu and Tanu when they
22.
(b) If buses I and II move in the same direction, then they cross each other for the first time be AB.
meet after covering 120 Km and 60 Km respectively    For such a situation, when the ratio of speeds = 3:2,
@ 20 Km/h and 10 Km/h respectively. If the buses it can be observed the total distance covered by both
move in opposite directions, then they meet after the runners put together, between two consecutive
covering 40 Km and 20 Km respectively. meetings is always twice the distance AB.
23.
(b) Suppose distance between X and Y be K Km X ® Y :    When they cross each other for the 2nd time, they
3 must have covered a distance of AB + 2AB
K
3
It takes 4 hrs to cover K km.    When they cross each other for the 3rd time, they
B 4 must have crossed AB + 4AB.

Chapter 12.indd 47 6/5/2015 3:11:43 PM


12.48 Chapter 12

\ Total distance covered by them when they cross 31.


(d) If speed of N = 4 units
each other for the fifth time is = AB + 4(2AB) = Speed of S = 1 unit
9AB = 7(9) = 63 km.
2 × 4 ×1
⇒ Average speed = = 1.6 units
63 1 4 +1
Time taken = =2 hrs
30 10 2 3
Because time available is , speed =
\ Distance from the county station 3 2
Now average speed = 2.4
 21
= 18   = 37.8 km Speed of N now = 8
 10 
Speed of S now = y
= 4 (7) + 9.8
2×8× y
= 7 + 2.8 = 2.8 Km from the town hall. = 2.4
8 + y
i.e.,  4.2 Km from the county station. ⇒ y = 1.3
Required ratio = 1.3:8 ≈ 1:6.
27.
(b) Let the distance be x. So, the train stops for
x x x 32.
(c) AG1 = 5 minutes at 30 Km/h = 2.5 Km
  hrs.
36 45 180 G1G3 = 15 Km
x x Time for AG1 = 5 minutes
i.e., in hrs stoppage is .
36 180 Time for G1G3 + G3A = 32.5 minutes

x  36 1 = total of 37.5 minutes
So, in 1 hour, stoppage will be  hour
180  x 5 1 minute is taken for transferring the patient into and
out of the ambulance.
= 12 minutes.
Hence (40 – 37.5 – 1) = 1.5 minutes are remaining.
28.
(b) Let the normal speed be a Km/h 33.
(b) Because each word is lit for a second
300 300 5 17 41   7 21 49 
\ = +2 LCM  + 1, + 1, + 1  = LCM  , , 
a a+5 2 4 8  2 4 8 

⇒ 300 (a + 5) = 300a + 2a (a + 5)
LCM (7, 21, 49)
⇒ 2a2 + 10a – 1500 = 0 =
HCF(2, 4, 8)
⇒ a2 + 5a – 750 = 0
49 × 3
= = 73.5 s.
⇒ (a + 30) (a – 25) = 0 2
⇒ a = 25. 34.
(b) Let tunnel = 8 Km and speed of cat = 8 Km/h

29.
(b) Let the normal speed of the aircraft be x Km/h. Time taken to reach entrance of tunnel by cat = 3 hr
Time taken to reach exit of tunnel by cat = 5 hr
1500 1 1500
\ − = Train will cover sum (length of tunnel) = 2 hr
x 2 250 + x
Therefore, ratio of speeds of train and cat = 4:1.
⇒ x2 + 250x – 750000 = 0
⇒ x = 750. 35.
(d) Total amount of work = 60 man hrs
From 11 a.m. to 5 p.m., 6 technicians = 36 man hrs
30.
(c) Let the distance be ‘d’, then we have
From 5 p.m. to 6 p.m., 7 technicians = 7 man hrs
d = 5(t + 6) = 7(t – 30);
From 6 p.m. to 7 p.m., 8 technicians = 8 man hrs
where t is usual time
From 7 p.m. to 8 p.m., 9 technicians = 9 man hrs
⇒ 5t + 30 = 7t – 210
Total = 60 man hrs.
⇒ 2t = 240
⇒ t = 120 min. 36.
(c) Speed of tiger = 40 m/min
Then, d = 7(120 – 30) = 7 × 90 = 630 min. Speed of deer = 20 m/min

Chapter 12.indd 48 6/5/2015 3:11:44 PM


Time and Distance 12.49

Relative speed = 40 – 20 = 20 m/min 4 x 2 − y2


⇒ y = (2)
Difference in distances = 50 × 8 = 400 m 4
\ Time taken in overtaking (or catching) Hence, from Eqs. (1) and (2), we have 2x2 = 5y2
400
= = 20 min 5 2
2 Putting x2 = y in (1), we get
2
\ Distance travelled in 20 minutes = 20 × 40 = 800 m.
3 2
1 1 1 y
8
37.
(d) Work done in one day by A, B, C and D are , , y = 2 ⇒y= .
4 8 16 4 3
1
and , respectively. Using answer choices, we note 40.
(b) If Shyam takes 1 minutes for every 3 steps, then he
32
1
3 takes minutes for every step.
that the pair of B and C does of work in one day; 3
16
1 1 9 25
the pair of A and D does + = of the work in For 25 steps, he takes minutes, i.e., 8.33 minutes.
4 32 32 3
one day.
1
So, Vyom takes minutes for every step.
32 2
Hence, A and D take days.
9
16 32 20
B and C take = days. Hence, the first pair For 20 steps, he takes minutes, i.e., 10 minutes.
3 6 2
must comprise of A and D. Difference between their times = 1.66 minutes.
38.
(a) Amount of money given to X Escalator takes 5 steps in 1.66 minutes.
= 12 × 300 + 12 × 330 + ... + 12 × 570 Speed of escalator is 1 step for 0.33 min. i.e., 3
= 12 [300 + 330 + ... + 540 + 570] steps per minute.
10 If escalator is moving, then Shyam takes 25 steps
= 12 × [600 + 9 × 30] = 52,200
2 and escalator also takes 25 steps.
Amount of money given to Y is Hence, total number of steps = 50.
6 × 200 + 6 × 215 + 6 × 230 + 6 × 245 + ... to 20 terms 41. (c) Let the time taken by Asit, Arnold and Afzal to do the
20 work alone be x, y and z hrs respectively.
= 6 × [400 + 19 × 15] = 6 [400 + 285]
2 \ Time taken to do the work together is
= 60 × 685 = 41,100 z
x – 6 = y – 1 =
\ Total amount paid = 52,200 + 41,100 = 93,300. 2
39.
(d) Let x be rowing rate of Rahul and y, be the rate of flow 1 1 1
In one hour, they can do + + of the work.
of current in mph. x y z
12 12 \ Total work can be completed by them in
− =6
x−y x+y xyz xyz
hrs, i.e., =x–6
y 1 xy + yz + zx xy + yz + zx
⇒ = Now, put y = x – 5 and z = 2 (x – 6) in the above
x2 − y 2 4
20
x 2 − y2 equation and solve for x. We get x = hrs
⇒ y = (1) 3
4
Hence, time taken by them to complete the work when
When Rahul dobles his rowing rate, then we have 20 2
working together = −6 = hour = 40 minutes.
12 12 3 3
− =1
2x − y 2x + y
42. (b) 60 Km/h is travelled in 4 litres petrol (from the graph)
2y 1 \ 1 litre is required for 15 km.
⇒ 2 2
=
4x − y 12 i.e., for 15 km, 1 litre petrol is required.

Chapter 12.indd 49 6/5/2015 3:11:45 PM


12.50 Chapter 12

They take same time to reach point C.


x Dx
⇒ = 
6 5
6
\ x =  D
11
5 5
Now Golu covers D in h.
11 2
5D 2D
So, VG =   km/h
5 11
For 200 km, = 13.33 litres is required. 11 
2
40 60
43.
(b) = 16.66 km; = 15 km 5 5 2 5D
2.4 4 ⇒ VM = × VG = × D =
6 6 11 33
80
= 10 Km approximately Mayank covers 5D in 33 hrs
7.9
\ Answer is 16.66 km, decrease the speed. 6 33 6
D in
So he will cover  = 3.6 hrs
44. (a) The robot begins to give material to machine A and 11 5 11
then to D, it thus covers 40 m in that time span and   = 3 hrs 36 min.
takes 4s. Also then it returns to the origin, and takes Alternative method 2:
4s, while covering 40 m again. When it arrives at the

origin, the messages of B and C are already there, thus
it moves to give the material to them, which takes it in VG tM
total 6s, and covers 30 + 30 = 60 m in total. Hence the =
VM tG
distance travelled by the robot will be 40 m + 40 m +
2
60 m = 140 m.  6 tM
⇒   =
45. (a) 5 tG
46. (d)
36 5
\ tm =  = 3 hrs 36 min.
25 2
Let x be the point where they meet on the way. 47.
(b) After 4 hrs distance between Shruti and Archana = (13
AX 6 × 4 – 8 × 4) = 20 km.
⇒      [As their speeds are in this ratio]
XB 5     Let both of them reach the destination C after x
t m d m / S m d m SG km, then time taken by A in travelling (20 + x) Km is
Now,          equal to time taken by B in travelling x Km as given
tG dG / SG dG Sm

d m AX 6 tm 6 6 36
⇒ Since        
dG XB 5 tG 5 5 25
2.5  36 20  x x  60 
      
tm = = 3.6 hr = 3 hr 36 min   4  ⇒ x = 60 km
25 
16 12  12 
Alternative method 1: \ Total time of the journey = = 9 hrs.
Conventional method of solving
48.
(c) Distance travelled by the thief in 15 min
15
= 60  = 15 km.
60
Hence, distance between police and thief when police
Let Golu and Mayank met at point C which is x started to chase = 15 km. Relative speed = (65 – 60) =
kilometres from A, and A and B are D kilometres 5 Km/h.
apart. Hence, time taken by police to catch the thief
V 6 15
The ratio of speeds = G  . = = 3 hrs.
VM 5 5

Chapter 12.indd 50 6/5/2015 3:11:47 PM


Time and Distance 12.51

Hence, required time = (12 hrs + 15 minutes + 3 hrs) 54.


(d)
= 3.15 p.m. Let the distance between A and B be P Km and speeds
49.
(b) Since the speed of the another policeman is same as of A and B be x Km/h and y Km/h, respectively
that of thief. Hence distance between thief and him When they meet first:
will be 15 km. And this is the required distance.
7 P−7
50.
(b) In all, X has travelled for 25 hrs (including stoppages) = (1)
x y
at an average speed of 180 miles per hour. Hence, the
When they met second
distance between Frankfurt and India = (25 × 180) =
4,500 miles. P+4 2P − 4
= (2)
51.
(a) For the return journey, X halts at Boston for one hour x y
less than his previous halt. Hence time taken by X for From (1) and (2)
his return journey is 24 hrs. 7 P−7
=
52.
(a) The distance between Frankfurt and India = 4,500 P+4 2P − 4
miles. Therefore, total distance travelled by him =
(4,500 + 4,500) = 9,000 miles. Time taken by him ⇒ 14P – 28 = P2 – 7P + 4P – 28

 11  ⇒ P2 = 17P
including halting time 1 h
 12  ⇒ P = 17 km.
 11  Total distance travelled
=  25  24  1  = 50.9 hrs
 12  = 34 × 10 = 340 km
Hence, average speed Total cost = 340 × 20 = `6800.
Total distance 9,000 50
= = = 176.81 m/hr. 55. (b) Time taken from road 1 = = 1 hr
Total time 50.9 50
53.
(b) Time Taken from road 2 is = 50/(80/4) = 2.5 hrs
\ Total time taken = 3.5 hrs
\ Total distance travelled = 100 km
100
\ Average speed = Km/h ≈ 29 Km/h
3.5

Applying cosine rule to find the third side 56. (d) Let train is at a distance y Km from the tunnel and the
length of the tunnel is x km. Man is at point ‘C’ which
BC2 = AB2 + AC2 – 2AB . AC cos 120º x
is Km away from B.
1 3
= 144  64  2  12  8 
2
= 144 + 64 + 96

⇒ BC = 304 = 4 19 Let M Km/h be the speed of man.

\ Time taken by Ram to travel to A Now, train is at A and man is at C and both will take
same time for reaching at B.
12  4 19  8 y x
= \ =
3 30 3( M )
Time taken by Shyam to go to A
10x
8  4 19  8 ⇒ M = (1)
= y
2
Also train and man will take same time for reaching
\ Required difference at ‘D’
24  12 19  36  24  8 19  16 y+x 2x
= \ =
6 30 3M
4 19  20 2 19  10 20x
= = \ M = (2)
6 3 y+x

Chapter 12.indd 51 6/5/2015 3:11:48 PM


12.52 Chapter 12

Solving equation (1) and (2), we get the value of 5 x


\ Time taken by him =  h
M = 10 Km/h. 10 10
57. (b) Speed of father = 7.5 Km/h Time taken by him with the speed of 15 Km/h
Speed of son = 5 Km/h
5 x
\ relative speed = 2.5 Km/h =  h
15 15
1
\ time taken to catch the son = h = 24 min 5 x 5 x 1
2.5 Now,    = [19:00 – 18:30]
10 10 15 15 2
\ in 24 minutes son would cover 2 Km = 2000 m.
⇒ x = 10
58.
(b) t = 4×42 Now, the total distance Rajesh has to travel = 15 km.
1 5
= 2 × 2 4 = 2 4 = 21.25 64.
(b) Mukesh starts from Delhi (say A). He has to take one
of the other two (say Dinesh) on his bike, take him
and, distance = 23.25 upto a certain point (say C) drop him there and return
Distance 23.25 for Suresh. Mean while Suresh starts walking. Suresh
\ Speed = = 1.25 = 22 = 4 m/s.
Time 2 and Mukesh meet at (say B) Mukesh picks up Suresh
at B and turns towards Mathura. All of them arrive
2 2 together at Mathura (say D)
59.
(a) Traveller distance in 6 minutes = ×6 = km
60 10
2 6 8
Total distance in 6 minutes = + = km
10 10 10
8
Speed = × 10 = 8 Km/h.
10
60.
(b)   As M drives at 60 Km/h and S (as well as D) walk
at 15 Km/h

AC + CB = 4(AB)
Distance travelled in 1 hr = 20 km
Remaining distance = 110 – 20 = 90 km BC + CB = 3(AB)
Time taken = Distance/Relative speed CB = 1.5(AB)
90 Let, AB = 2, BC = 3 (Also CD = 2)
= = 2 hrs
(20 + 25)
600 900 600
So, time = 8 am + 2 = 10 a.m. Actually, AB = , BC = , CD =
7 7 7
61. (a) s = t2 + t
600 00  600
If t = 2, Time taken = 
7  15 7  60
s = 22 + 2 = 6
If t = 4, 40 25 2
=  = 9 .
s = 42 + 4 = 20 7 7 7
Hence, required difference = 20 – 6 = 14.
65.
(c) Time taken by Sunil for 1st round = 2 min
15000 m 2nd round = 4 min, 3rd round = 8 min,
62.
(b) Speed of Amarendra = = 10 m/s.
1500 s
4th round = 16 min, 5th round = 32 min,
12500 m
Speed of Dharmandra = = 25/3 m/s 6th round = 64 min, 7th round = 128 min,
1500 s
8th round = 256 minutes
Consider the 7 minutes early departure of Dharmendra.
Q Anil takes one minutes for every round.
63.
(d) Rajesh has to walk 5 Km in mall.
\ He meets 127 times in 7th and 255 times in 8th
Let, the distance from mall to his home = x round with Sunil.

Chapter 12.indd 52 6/5/2015 3:11:49 PM


Time and Distance I 12.53

66.
(a) Expenditure of diesel for 1 km If they travel in the same direction, the faster one will
catch up the slower one in t hrs.
1  1000  
=    x (F – S) r = D (1)
400  x  
 1000   If they travel in opposite directions, together they
Expenditure of diesel for 800 Km = 2    x must cover the total distance of D Km in t hrs.
 x  
\ Expenditure on diesel (F + S) t = D (2)

 1000   From Eqs. (1) and (2),


= 35 × 2 
   x  rupees (F – S) r = (F + S) t
 x 
FS r
 800  =
Time of journey =   hrs FS t

By componendo and dividendo,
125  800 F r t
\  Driver’s pay = rupees =
x S r t

Total cost = 70 
 x
 
 1000  125  800
 
 x
70.
(a) Speed while going =
150
3
20
= 45 Km/h

60
Now if we go throw the options
Total distance
Then total cost at speed 49 Km/h = `6899.38 Average speed =
Total time
At speed 55 Km/h = `6940.90
300
At speed 50 Km/h = `6900.00 r =
7.5
At speed 53 Km/h = `6917.55
r = 40 Km/h
67.

Ghaziabad
--
(b) 16 kmJh
• • --
21 kmJh


Meerut

71.
\ Average rate for the trip going exceeds r by 45 –
40 = 5 Km/h.
(d) The two men meet when the sum of the distances
Suppose they meet after time t. covered by them is 76
Distance travelled by one train = 21t
Distance covered by
Sum of dis-
Distance travelled by second train = 16t No. of tances covered
21t – 16t = 60 hrs Man at R Man at S by the men
(in km)
⇒ 5t = 60
1 4.5 3.25 7.75
⇒ t = 12 hrs
2 4.5 + 4.5 = 9 3.25 + 3.75 = 7 16
Distance between them 3 9 + 4.5 = 13.5 7 + 4.25 = 11.25 24.75
4 13.5 + 4.5 = 18 11.25 + 4.75 = 16 34
= 21t + 16t
5 18 + 4.5 = 22.5 16 + 5.25 = 21.25 43.75
= 37t = 37 × 12 6 22.5 + 4.5 = 27 21.25 + 5.75 = 27 54
7 27 + 4.5 = 31.5 27 + 6.25 = 33.25 64.75
= 444 km
8 31.5 + 4.5 = 36 33.25 + 6.75 = 40 76
68.
(a, c, d)  Laxman takes the first train which is slower one.
Bharat takes the faster train. Let, the trains be A and The men meet 36 Km from R and 40 Km from S. They
B respectively. To overtake, train A; train B, has to meet 4 Km nearer to R than S.
cover its length. As we cannot determine the length of
the slower train, we cannot find the time taken in over 72.
(b) Let the distance between Delhi and Mumbai be x km.
take. Hence, option (b) is not correct. x
Time taken by Jet airways flight = hrs
69.
(a) Let the speeds of the faster cyclist and the slower 700
cyclist be F and S respectively. x
Time taken by Kingfisher flight = hrs
The distance between the two cyclists = D 560

Chapter 12.indd 53 6/5/2015 3:11:51 PM


12.54 I Chapter 12

x x 10 x  8 x According to the problem, the time taken by faster car


Stoppage time =  = - - -
560 700 5600 350
is  h.
x S
= hhrs
2800 Simultaneously, the time taken by slower car is
x / 2800
Average stoppage time per hour = 350
x / 560 h
( S − 5)
560
= hhrs = 12 minutes.
2800 350 350 1
− =2
S −5 S 3
73.
(b) D = S × T
 S − S + 5 7
1 350  =
\ = p – q.  S ( S − 5)  3
r
S ( S − 5) = 750
74.
(a) Ratio of speed = 2:3 By hit andd trial method,
Ratio of time = 3:2 S = 30 km/h.

\ 3x – 2x = 10
78. (d) Let PQ be x.
⇒ x = 10
When B overtakes A for the first time, both of them
A takes 3x = 30 min 3x
cover  .
B takes 2x = 20 min 10
30 7x 7x 28 x
When speed of A is double, then time = = 15 min. When B meets A after that, it covers + or ,
2 23 x 9 x 14 x 10 30 30
while A covers − or .
75.
(b) Let the speed of the cyclist be x Km/h and wind be y 30 30 30
Km/h. A starts 1 hour after B; it catches up within 1 hour.
\
1
=
3 Therefore, B covers 0.3x in 1 hour or x in 10 or 3 1 h.
x y 60 3 3
1 1 79. (a) First half of the distance is covered at 3/4th of its
⇒ = original speed.
x y 20
Then, Vidya taken 4/3 of its original time to cover half
⇒ 20 = x + y (1) of the distance.
1 4 So, Vidya takes 1/3 extra time during her first half
and =
x  y 60 journey.
⇒ x – y = 15 (2)  1
Now, she is left with 1 −  of the time to cover her
 3
On solving Eqs. (1) and (2), we get
remaining half distance = 2/3rd of the time.
35 5 So, she can cover the distance at 3/2 times of her usual
x = ,y=
2 2 speed.
2
Now, time taken by cyclist without wind =  60
35 80. (b) Let the time taken by Sanjit to walk to school = x min
3 Similarly, the time taken by Sanjit to ride bicycle to
= 3 min . school = y min
7
When Sanjit walks one way and rides the other way,
76.
(d) Total initially bogies is 12 + 12 = 24 we have
Additional bogies =16 – 12 = 4 x + y =90
When Sanjit rides both ways, we have
24 bogies take 2 minutes
y + y = 30
2  60 ⇒ 2 y = 30
4 bogies will take =  4 = 20 s
24
⇒ y = 15
77. (a) Let the speed of faster car be S Km/h. Now, x + y = 90
Then, the speed of slower car is (S − 5) Km/h. ⇒ x + 15 = 90 ⇒ x = 75

Chapter 12.indd 54 6/5/2015 3:11:56 PM


Time and Distance 12.55

∴ Time taken by Sanjit to walk both ways 135 135 135


= 75 + 75 = 150 minutes = 2.5 hrs. ⇒r= = = = 59.21 m
(2 × 3.14 − 4) (6.28 − 4) 228
81. (b) Let the radius of circular garden be r ∴ the circumferennce of grass plot = 2π r
Semi-circular = 2 × 3.14 × 59.21 = 371.25 m

path
82. (c) Let the speed of train C be x Km/h.
At 9 p.m. the train A will have covered a distance of
r r 180 km.
Straight path
(via diameter) For trains A and B relative speed = (90 − 60) = 30

Km/h
Then, distance covered by crossing the garden Distance between them = 180 Km/h
diagonally = r + r = 2r 180
Time after which they meet = = 6 hrs
Also, distance covered when man follows the circular 30
2π r
path = x km (x − 770) km
2

(∴ man has to cover half of the circumference of
circular = π r For train A and C relative speeds = (60 + x) Km/h
90 3 Distance between them 1080 km.
Speed of man in m/s = = m/s 1080
60 2 Time after which they meet = hrs
(60 + x)
We are given,
πr 2r As the time of meeting of all the three trains is the
− = 45 1080
3/ 2 3/ 2 same, we have =6
2 π r − 4r ( x + x)
⇒ = 45; (2π − 4r ) = 135
3 or x = 120 Km/h.
135 135 135
⇒r= = = = 59.21 m
(2 × 3.14 − 4) (6.28 − 4) 228
∴ the circumference of grass plot = 2π r

Chapter 12.indd 55 6/5/2015 3:12:01 PM


This page is intentionally left blank

FM.indd 6 6/10/2015 2:36:16 PM


CHAPTER

Boats and Streams 13


SOME IMPORTANT TERMS
Note:
1. Still Water If the speed of the water in the river is zero,
it is still water. When the speed of a boat or a swimmer is given, it usually
2. Stream If the water of the river is moving, it is called a means speed in the still water.
stream.
3. Upstream If a boat (or a swimmer) moves against
the stream, i.e., in the direction opposite to that of the
stream, it is called upstream.
4. Downstream If a boat (or a swimmer) moves with
the stream, i.e., along the direction of the stream, it is
called downstream.

soMe Basic forMulae

1. If the speed of a boat (or a swimmer) be x Km/h (b) Speed of the stream
and the speed of the stream or the current be y 1
Km/h, then = (Downstream Speed – Upstream Speed)
2
(a) speed of the boat (or swimmer) downstream
= (x + y) Km/h. Illustration 2 A boat is rowed down a river 40 Km in 5 hr
(b) speed of the boat (or swimmer) upstream and up a river 21 Km in 7 hr. Find the speed of the boat and
= (x – y) Km/h. the river
40
Solution: Speed of the boat downstream = = 8 Km/h
Illustration 1 The speed of a boat in still water is 20 Km/h. 5
If the speed of the stream be 4 Km/h, find its downstream 21
Speed of the boat upstream = = 3 Km/h
and upstream speeds 7
Solution: Speed of the boat (x) = 20 Km/h \ Speed of the boat
1
Speed of the stream ( y) = 4 Km/h = (Downstream Speed + Upstream Speed)
2
\ Downstream speed = x + y = (20 + 4) = 24 Km/h 1 11
= (8 + 3) = = or, 5 × 5 Km/h
and upstream speed = x – y = (20 – 4) = 16 Km/h 2 2
and speed of the river
2. (a) Speed of the boat (or swimmer) in still water 1
= (Downstream Speed – Upstream Speed)
2
1
= (Downstream Speed + Upstream Speed) 1 5
2 = (8 – 3) = or, 2.5 Km/h
2 2

Chapter 13.indd 1 6/5/2015 1:37:34 AM


13.2 Chapter 13

soMe useful sHort-cut MetHoDs

Explanation
1. If a man capable of rowing at the speed of x
Km/h in still water, rows the same distance up and Let the speed of the man in still water be x Km/h.
down a stream which flows at a rate of y Km/h, Then, downstream speed = (x + y) Km/h
then his average speed throughout the journey is and upstream speed = (x – y) Km/h.
Uptream × Downstream Since the distance covered downstream and upstream
=
Man's rate in still water are equal, we have
( x − y )( x + y ) (x + y)t1 = (x – y)t2
= Km/h.
x
or, xt1 + yt1 = xt2 – yt2
Illustration 3 A man rows at a speed of 8 Km/h in still
or, x(t2 – t1) = y(t2 + t1)
water to a certain distance upstream and back to the starting
point in a river which flows at 4 Km/h. Find his average t +t 
speed for total journey \ x = y  2 1  Km/h
Solution: Average speed  t2 − t1 

Upstream × Downstream Illustration 5 A motorboat covers a certain distance


=
Man's rate in still water downstream in 6 hrs but takes 8 hrs to return upstream to
the starting point. If the speed of the stream be 6 Km/h, find
(8 − 4)(8 + 4)
= = 6 Km/h the speed of the motor boat in still water.
8 Solution: Speed of the motorboat in still water
2. A man can row a boat in still water at x Km/h. In t +t 
a stream flowing at y Km/h, if it takes t hrs more = y  2 1  Km/h
in upstream than to go downstream for the same  t2 − t1 
distance, then the distance is given by 8+6
= 6  = 42 Km/h
2 2
( x − y )t 8−6
Km
2y
4. A man can row a boat in still water at x Km/h. In
Illustration 4 A man can row 7 Km/h in still water. If the a stream flowing at y Km/h if it takes him t hrs to
river is running at 3 Km/h, it takes 6 hrs more in upstream row to a place and come back, then the distance
than to go downstream for the same distance. How far is between the two places is
the place?
t ( x2 − y 2 )
Solution: The required distance Km.
2x
( x 2 − y 2 )t
=
2y Explanation
(49 − 9)6 Downstream speed = (x + y) Km/h
= = 40 Km
2×3 Upstream speed = (x – y) Km/h

3. A man rows a certain distance downstream in t1 Let, the distance between the two places be d Km. We
hrs and returns the same distance upstream in t2 have,
hrs. If the speed of the stream be y Km/h, then the Total time = Sum of time taken downstream and
speed of the man in still water is given by upstream
t +t 
y  2 1  Km/h. d d
 t2 − t1  ⇒ t= +
x+ y x− y

Chapter 13.indd 2 6/5/2015 1:37:35 AM


Boats and Streams 13.3

 ( x − y) + ( x + y)  5. A boat (or a swimmer) takes n times as long to


=d 
 ( x − y )( x + y )  row upstream as to row downstream the river. If
the speed of boat (or swimmer) be x Km/h and the
 2x  speed of stream be y Km/h, then
=d 2 2
x − y   n +1
x = y .
t ( x2 − y 2 )  n −1 
\ d= Km
2x Illustration 7 A man can row at the rate of 4 Km/h. in still
Illustration 6 A man can row 6 Km/h in the still water. If water. If the time taken to row a certain distance upstream
the river is running at 2 Km/h, it takes him 3 hrs to row to a is 3 times as much as to row the same distance downstream,
place and back. How far is the place? find the speed of the current
Solution: The required distance Solution: We have,
t ( x2 − y 2 )  n +1
Speed of the man =   speed of the current
= Km  n −1 
2x  3 +1
3(36 − 4) ⇒ 4=   speed of the current.
= = 8 Km  3 −1 
2×6 \ Speed of the current = 2 Km/h

Practice Exercises

Difficulty level-1
(BaseD on MeMory)

1. A man can row three-quarters of kilometer against the 5. A man can row 6 Km/h in still water. If it takes him twice
1 1 as long to row up, as to row down the river, then the rate
stream in 11 minutes and returns in 7 minutes. The
4 2 of current in the stream would be:
speed of the man in still water is: (a) 4 Km/h (b) 2 Km/h
(a) 2 Km/h (b) 3 Km/h (c) 3 Km/h (d) 8 Km/h
(c) 4 Km/h (d) 5 Km/h [Based on ATMA, 2006]
[Based on MAT, 2005] 6. A boat goes 30 Km upstream and 44 Km downstream in
1
2. A man swimming in a stream which flows 1 Km/h finds 10 hrs. In 13 hrs, it can go 40 Km upstream and 55 Km
2 down stream. The speed of the boat in still water is:
that in a given time he can swim twice as far with the
stream as he can against it. At what rate does he swim? (a) 3 Km/h (b) 4 Km/h
1 1 (c) 8 Km/h (d) None of these
(a) 5 Km/h (b) 4 Km/h
2 2 [Based on ITFT, 2008]
1 7. A motor boat can travel at 10 Km/h in still water. It
(c) 7 Km/h (d) None of these
2 traveled 91 Km downstream in river and then returned,
[Based on MAT, 2008]
3. A boat goes 24 Km upstream and 28 Km downstream in 6 taking altogether 20 hrs. Find the rate of flow of river.
hrs. If it goes 30 Km upstream and 21 Km downstream in (a) 3 Km/h (b) 5 Km/h
6 hrs and 30 minutes, find the speed of the stream. (c) 6 Km/h (d) 7 Km/h
(a) 10 Km/h (b) 5 Km/h
[Based on IIFT, 2005]
(c) 4 Km/h (d) 6 Km/h
[Based on MAT, 2001]
8. A boat goes 24 Km upstream and 28 Km downstream in
4. A person can row with the stream at 8 Km per hour and 6 hrs. It goes 30 Km upstream and 21 Km downstream
against the stream at 6 Km an hour. The speed of the in 6 hrs and 30 minutes. The speed of the boat in still
current is: water is:
(a) 1 Km/h (b) 2 Km/h (a) 10 Km/h (b) 4 Km/h
(c) 4 Km/h (d) 5 Km/h (c) 14 Km/h (d) 6 Km/h
[Based on FMS (Delhi), 2002] [Based on MAT, 1999]

Chapter 13.indd 3 6/5/2015 1:37:36 AM


13.4 Chapter 13

9.
A motor boat whose speed is 15 Km/h in still water goes (a) 30 m/min (b) 29 m/min
30 Km downstream and comes back in 4 hrs and 30 (c) 31 m/min (d) 32 m/min
minutes. Determine the speed of the stream.
[Based on MAT (Dec), 2008]
(a) 10 Km/h (b) 4 Km/h
17.
A boatman goes 2 Km against the current of the
(c) 5 Km/h (d) 6 Km/h stream in 1 hr and goes 1 Km along the current in
[Based on MAT, 1999] 10 minutes. How long will he take to go 5 Km in stationary
10.
A motor boat whose speed is 30 Km/h in still water goes water?
60 Km downstream, and comes back in 4 and a half hrs. (a) 1 hr 30 minutes (b) 1 hr 15 minutes
The speed of the stream is: (c) 1 hr (d) 40 minutes
(a) 5 Km/h (b) 10 Km/h [Based on MAT (Sept), 2010 (Dec), 2009, 2007]
(c) 15 Km/h (d) 20 Km/h 18.
A man rowed against a stream flowing 1.5 Km/h to a
[Based on MAT, 1998] certain point and then turned back, stopping 2 Km short
11.
A boat takes 90 minutes less to travel 36 miles downstream of the place from where he originally started. If the whole
than to travel the same distance upstream. If the speed of time occupied in rowing be 2 hrs 10 min and his uniform
the boat in still water is 10 m/h, the speed of the stream is: speed in still water be 4.5 Km/h, the man went up the
(a) 4 m/h (b) 3 m/h stream a distance of:
(c) 2.5 m/h (d) 2 m/h (a) 4 Km (b) 8 Km
[Based on MAT, 1997] (c) 7 Km (d) 5 Km
[Based on MAT (Sept), 2010 (Dec), 2009]
12.
Speed of a speed-boat when moving in the direction
perpendicular to the direction of the current is 19.
A man can row 30 Km upstream and 44 Km downstream
16 Km/h, speed of the current is 3 Km/h. So, the speed of in 10 hrs. Also, he can row 40 Km upstream and 55 Km
the boat against the current will be: downstream in 13 hrs. The rate of the current is:
(a) 22 Km/h (b) 9.5 Km/h (a) 3 Km/h (b) 3.5 Km/h
(c) 10 Km/h (d) None of these (c) 4 Km/h (d) 4.5 Km/h
[Based on MAT (Sept), 2009]
[Based on MAT (Feb), 2006]
20.
A man rows 8 Km/h in still water. If the river is running
13.
Twice the speed of a boat downstream is equal to thrice at 2 Km/h, it takes 32 minutes to row to a place and back.
the speed upstream. The ratio of its speed in still water to How far is the place?
its speed of current is:
(a) 1.5 Km (b) 2.5 Km
(a) 1:5 (b) 1:3 (c) 2 Km (d) 3 Km
(c) 5:1 (d) 2:3 [Based on MAT (Sept), 2009]
[Based on MAT (Dec), 2007] 1
21. A man swimming in a stream which flows 1 Km/h finds
2
14.
A man can row three-fourths of a Km against the stream that in a given time he can swim twice as far with the
1 1 stream as he can against it. At what rate does he swim?
in 11 minutes and return in 7 minutes. Find the speed
4 2 1 1
(a) 4 Km/h (b) 5 Km/h
of the man in still water. 2 2
(a) 4 Km/h (b) 3 Km/h 1
(c) 5 Km/h (d) 6 Km/h (c) 7 Km/h (d) None of these
2
[Based on MAT (May), 2008] [Based on MAT (Sept), 2009 (Feb), 2008]
22.
A boat travels upstream from B to A and downstream from
15.
A ship 55 Km from the shore springs a leak which admits
A to B in 3 hrs. If the speed of the boat in still water is 9
2 tonnes of water in 6 minutes, 80 tonnes would suffice to
Km/h and the speed of the current is 3 Km/h, the distance
sink her, but the pumps can throw out 12 tonnes an hour.
between A and B is:
The average rate of sailing that she may just reach the
shore as she begins to sink is: (a) 4 Km (b) 8 Km
(c) 6 Km (d) 12 Km
(a) 9.17 Km/h (b) 0.97 Km/h [Based on MAT (Dec), 2008]
(c) 55 Km/h (d) 5.5 Km/h 23.
A motor boat can travel at 10 Km/h in still water. It
[Based on MAT (Sept), 2008 (Dec), 2002, 2006 (Feb), 2004] travelled 91 Km downstream in a river and then returned,
16.
A man who can swim 48 m/minute in still water, swims taking altogether 20 hrs. Find the rate of flow of the river.
200 m against the current and 200 m with the current. If (a) 6 Km/h (b) 5 Km/h
the difference between those two times is 10 minutes, find (c) 8 Km/h (d) 3 Km/h
the speed of the current. [Based on MAT (Dec), 2008]

Chapter 13.indd 4 6/5/2015 1:37:36 AM


Boats and Streams 13.5

24. A man can row 40 Km upstream and 55 Km downstream (1) 11 Km/h (2) 10 Km/h
in 13 h. Also, he can row 30 Km upstream and 44 Km (3) 16 Km/h (4) None of these
downstream in 10 h. Find the speed of the man in still water.
[Based on MAT, 2011]
(1) 5 Km/h (2) 2 Km/h
28. The speed of a boat in still water is 6 Km/h and speed of
(3) 4 Km/h (4) None of these the stream is 1.5 Km/h. A man rows to place at a distance
[Based on MAT, 2011] of 22.5 Km and comes back to the starting point. Find the
25. Speed of a boat in standing water is 9 Km/h and the speed total time taken by him.
of the stream is 1.5 Km/h. A man rows to a place at a (1) 8 h (2) 4 h
distance of 105 Km and comes back to the starting point. (3) 7 h (4) 2 h
The total time taken by him is: [Based on MAT (Feb), 2012]
(1) 20 hrs (2) 18 hrs
29. A man who can swim 48 m/minute in still water swims
(3) 16 hrs (4) 24 hrs
200 m against the current and 200 m with the current.
[Based on MAT, 2011] If the difference between those two times is 10 minutes,
26. A boat takes 19 h for travelling downstream from point A what is the speed of the current?
to point B and coming back to point C midway between (1) 30 m/min (2) 31 m/min
A and B. If the velocity of the stream is 4 Km/h and the (3) 29 m/min (4) 32 m/min
speed of the boat in still water is 14 Km/h, what is the [Based on MAT (Feb), 2012]
distance between A and B?
(1) 200 Km (2) 180 Km 30. A boat travels upstream from B to A and downstream
from A to B in 3 h. If the speed of the boat in still water
(3) 160 Km (4) 220 Km is 9 Km/h and the speed of the current is 3 Km/h, what
[Based on MAT, 2011] distance between A and B?
27. A boat covers a certain distance downstream in 1 h, while (a) 6 Km (b) 4 Km
it comes back in 1.5 h. If the speed of the stream be 3 (d) 8 Km (d) 12 Km
Km/h, what is the speed of the boat in still water? [Based on MAT (Dec), 2012]

Difficulty Level-2
(Based on Memory)

1. A ship carrying 1,000 people moves 12 Km down stream 3. At his normal speed, Ramesh can travel 18 Km
and then 4 Km upstream. The river current is 1 Km/h. downstream in a fast flowing stream in 9 hrs less than
Within what limits must the speed of the ship (which is what he takes to travel the same distance upstream. The
denoted as V) lie for the entire trip to take not less than 3 downstream trip would take one hour less than what the
hrs and not more than 4 hrs? upstream trip would take, provided he doubles his rate of
1 rowing. What is the speed of the stream in Km/h?
(a)  V  2  3 (b) 2  3  V  2  3
3 2 1
(a) 6 (b) 8
1 3 3
(c)  V  5 (d) 2 ≤ V ≤ 5
3 8 10 950  960
(c) (d)
2. A and B in boat B1 challenge C and D in boat B2 in a race 3 1000
of 50 Km. A and B take turns to row the stretches of 6 Km 4. In a stream that is running at 2 Km/h, a man goes 10
and A begins. C and D take turns to row an hour and C Km upstream and comes back to the starting point in 55
begins. A and C can each row 5 Km/h, B and D each can minutes. Find the speed of the man in still water.
1 (a) 20 Km/h (b) 22 Km/h
row only 5 Km/h. Which boat wins and by what time?
2 (c) 24 Km/h (d) 28 Km/h
(a) B1 wins by 2 minutes 5.
A boat sails 15 Km of a river towards upstream in
7 5 hrs. How long will it take to cover the same distance
(b) B2 wins by 1 minutes
11 downstream, if the speed of current is one-fourth the
(c) Both reach the finishing spot together speed of the boat in still water:
4 (a) 1.8 hrs (b) 3 hrs
(d) B1 wins by 5 minutes
11 (c) 4 hrs (d) 5 hrs

Chapter 13.indd 5 6/5/2015 1:37:37 AM


13.6 Chapter 13

6.
A motorboat went the river for 14 Km and then up the 14. Two boats, travelling at 5 and 10 Km/h, head directly
river for 9 Km. It took a total of 5 hrs the entire journey. towards each other. They begin at a distance of 20 Km
Find the speed of the river flow if the speed of the boat in from each other. How far apart are they (in Kms) one
still water is 5 Km/h. minute before they collide?
(a) 1 Km/h (b) 1.5 Km/h (a) 1/12 (b) 1/6
(c) 2 Km/h (d) 3 Km/h (c) 1/4 (d) 1/3

7.
The different between downstream speed and upstream 15. At his usual rowing rate, Rahul can travel 12 miles
speed is 3 Km/h and the total time taken during upstream downstream in a certain river in 6 hrs less than it takes
and downstream is 3 hrs. What is the downstream him to travel the same distance upstream. But if he could
speed, if the downstream and upstream distance are double his usual rowing rate for this 24 mile round trip,
3 Km each? the downstream 12 miles would then take only one hour
less than the upstream 12 miles. What is the speed of the
(a) 2.5 Km/h (b) 4.3 Km/h
current in miles per hour?
(c) 4 Km/h (d) 3.3 Km/h
(a) 7/3 (b) 4/3
8.
The current of the stream is 1 Km/h. A boat goes 35 Km (c) 5/3 (d) 8/3
upstream and back to the starting point in 12 hrs. The 16. A man can row 30 Km upstream and 44 Km downstream
speed of the motorboat in still water is: in 10 hrs. Also, he can row 40 Km upstream and 55 Km
(a) 6 Km/h (b) 7 Km/h downstream in 13 hrs. Find the rate of the current and the
(c) 8.5 Km/h (d) 8 Km/h speed of the man in still water.
(a) 3 Km/h, 8 Km/h (b) 3 × 5 Km/h, 7 × 5 Km/h
9.
A boat takes 5 hrs more while going back in upstream than
(c) 4 Km/h, 7 Km/h (d) 4 × 5 Km/h, 6 × 5 Km/h
in downstream. If the distance between two places is 24
Km and the speed of boat in still water so that if can row 17. P, Q, R are three towns on a river which flows uniformly.
donwstream, 24 Km, in 4 hrs? Q is equidistant from P and R. A man rows from P to Q
(a) 1.5 Km/h (b) 3.5 Km/h and back in 10 hrs. He can row from P to R in 4 hrs. The
ratio of speed of man in still water to the speed of the
(c) 4.5 Km/h (d) 3 Km/h
current is:
10.
A man can row 30 Km upstream and 44 Km downstream (a) 5:3 (b) 3:5
in 10 hrs. It is also known that he can row 40 Km upstream (c) 2:5 (d) 1:2
and 55 Km downstream in 13 hrs. Find the speed of the
man in still water. 18. A boatman goes 2 Km against the current of the stream in
(a) 4 Km/h (b) 6 Km/h 1 hr and goes 1 Km along the current in 10 minutes. How
long will he take to go 5 Km in stationary water?
(c) 8 Km/h (d) 12 Km/h
(a) 1 hr (b) 1 hr 15 minutes
11. A boat, while going downstream in a river covered a 1
distance of 50 miles at an average speed of 60 miles per (c) 1 hrs (d) 40 minutes
2
hour. While returning because of the water resistance, it 19. A man can swim 3 Km/h in still water. If the velocity of
took 1 hr 15 minutes to cover the same distance. What was the stream be 2 Km/h, the time taken by him to swim to a
the average speed during the whole journey? place 10 Km upstream and back, is:
(a) 40 m/h (b) 48 m/h 1 1
(c) 50 m/h (d) 55 m/h (a) 8 hrs (b) 9 hrs
3 5
12. A boat takes 90 minutes less to travel 36 miles (c) 10 hrs (d) 12 hrs
downstream than to travel the same distance upstream. 20. Twice the speed downstream is equal to the thrice the
If the speed of the boat in still water is 10 m/h, the speed speed upstream, the ratio of speed in still water to the
of the stream is: speed of the current is:
(a) 4 (b) 3 (a) 1:5 (b) 5:1
(c) 2.5 (d) 2 (c) 1:3 (d) 2:3
21. A man rows upstream 12 Km and downstream 28 Km
13. Speed of a speed boat when moving in the direction
taking 5 hrs each time. The velocity of water current is:
perpendicular to the direction of the current is 16 Km/h,
speed of the current is 3 Km/h. So the speed of the boat 1 1
(a) 2 Km/h (b) 2 Km/h
against the current will be (in Km/h): 5 2
(a) 22 (b) 9.5 3
(c) 3 Km/h (d) 1 Km/h
(c) 10 (d) None of these 5

Chapter 13.indd 6 6/5/2015 1:37:37 AM


Boats and Streams 13.7

1 25. If a man’s rate with the current is 12 Km/h and the rate of the
22. A man swimming in a stream which flows 1 Km/h finds
2 1
current is 1 Km/h, then his rate against the current is:
that in a given time he can swim twice as far with the 2
stream as he can against it. At what rate does he swim? (a) 13 Km/h (b) 7 Km/h
1 1 (c) 9 Km/h (d) None of these
(a) 4 Km/h (b) 5 Km/h
2 2
26. A swimmer covers a distance of 28 Km against the current
1
(c) 7 Km/h (d) None of these and 40 Km in the direction of the current. If in each case
2 he takes 4 hrs, then the speed of the current is:
23. A boat travels upstream from B to A and downstream (a) 3.5 Km/h (b) 1.5 Km/h
from A to B in 3 hrs. If the speed of the boat in still water
(c) 2.5 Km/h (d) None of these
is 9 Km/h and the speed of the current is 3 Km/h, the
distance between A and B is: 27. Speed of a man is 10 Km/h in still water. If the rate of
(a) 8 Km current is 3 Km/h, then the effective speed of the man
(b) 16 Km upstream is:
(a) 7 Km/h (b) 8.5 Km/h
(c) 12 Km
(c) 9 Km/h (d) None of these
(d) None of these
28.
A man can row at 5 Km/h in still water. If the river is
24. In a river flowing at 2 Km/h, a boat travels 32 Km upstream running at 1 Km/h, it takes him 75 minutes to row to a
and then returns downstream to the starting point. If its place and back. How far is the place?
speed in still water be 6 Km/h, find the total journey time.
(a) 2.5 Km (b) 3 Km
(a) 16 hrs (b) 12 hrs (c) 4 Km (d) 5 Km
(c) 14 hrs (d) None of these [Based on FMS, 2005]

Answer Keys
Difficulty Level-1

1. (d ) 2. (b) 3. (c) 4. (a) 5. (b) 6. (c) 7. (a) 8. (a) 9. (c) 10. (b) 11. (d ) 12. (c) 13. (c)
14. (c) 15. (d ) 16. (d ) 17. (b) 18. (d ) 19. (a) 20. (c) 21. (a) 22. (d ) 23. (d ) 24. (d ) 25. (d ) 26. (b)
27. (d ) 28. (a) 29. (d ) 30. (d )

Difficulty Level-2

1. (a) 2. (b) 3. (a) 4. (b) 5. (b) 6. (c) 7. (b) 8. (a) 9. (b) 10. (c) 11. (b) 12. (d ) 13. (c)
14. (c) 15. (d ) 16. (a) 17. (a) 18. (b) 19. (d ) 20. (b) 21. (d ) 22. (a) 23. (c) 24. (b) 25. (c) 26. (b)
27. (a) 28. (b)­­­

Chapter 13.indd 7 6/5/2015 1:37:37 AM


13.8 Chapter 13

Explanatory Answers

Difficulty Level-1

3 4 30 km
1.
(d) Speed upstream = × × 60 = 4 Km/h. \ Upstream speed =
4 45 3x
3 2 30
Speed downstream = × = 6 Km/h.     = = 5 Km/h
4 15 3 2
1 44 km
Speed in still water = (6 + 4) = 5 Km/h. and, downstream speed =
2 4y
44
    = = 11 Km/h
3  3 4 1
2.
(b) x+ = 2  x − 
2  2 11  5
\ Speed of boat = = 8 Km/ hr.
1 2
\ x = 4 Km/h.
2
91 91
7.
(a) Since, + = 20
3.
(c) Let the speed of the stream be x Km per hour. x+ y x− y
Let speed of the boat in still water be y Km/h. 91 91
\      = + = 20
\ Boat will travel downstream @ (y + x) Km/h and 10 + y 10 − y
upstream @ (y – x) Km/h. By option, if y = 3
24 28 91 91
\ + = 6 + = 20.
y−x y+x 13 7
30 21 1 13
and, + = 6 =
y−x y+x 2 2 8.
(a) Let a Km/h be the speed of the boat in still water. Let
⇒ y + x = 14, y – x = 6 b Km/h be the speed of the cur­rent.
⇒ x = 4, y = 10. \ Boat will travel downstream @ (a + b) Km/h and
upstream @ (a – b) Km/h.
4.
(a) Let the speed of the current be x Km/h and speed of 28 24
the person in still water be y Km/h. \ + =6
a+b a−b
\ y + x = 8
21 30 1 13
y – x = 6 + = 6 =
⇒ y = 7, x = 1 a+b a−b 2 2
\ Speed of the current = 1 Km/h. 84 72
⇒ + = 18 (1)
a+b a−b
5.
(b) D = distance
84 120
x = speed of stream, + = 26 (2)
a+b a−b
2D 2D
Then, = Equation (1) and (2) give
6+ x 6−x
48
\ x = 2 Km/h. − =–8⇒a–b=6
a−b

6.
(c) Suppose 30 Km upstream is covered in 3x hrs and 40 28 24
Km upstream is covered in 4x hrs. \ Eq. (1) ⇒ + =6
a+b 6

  44 Km downstream is covered in 4y hrs and ⇒ a + b = 14
55 Km downstream is covered in 5y hrs. \ a = 10, b = 4
Then,
3x + 4y = 10 (1) 9.
(c) Let speed of the stream be K Km/h.
4x + 5y = 13 (2) Speed of the stream in still water = 15 Km/h
On solving, we get x = 2 and y = 1 \ Speed of the boat downstream = (15 + K) Km/h

Chapter 13.indd 8 6/5/2015 1:37:39 AM


Boats and Streams 13.9

\ Speed of the boat upstream = (15 – K) Km/h 14.


(c) Let the speed of man in still water be x Km/h and
30 30 9 speed of stream be y Km/h.
\ + =
15 + K 15 − K 2 45 3
\ ( x − y ) × =
⇒ K = 5 4 × 60 4

3 × 60
10. (b) Suppose speed of stream = x Km/h ⇒ x – y = = 4 (1)
45
\ Speed of boat upstream = (30 – x) Km/h
15 3
and speed of boat downstream = (30 + x) Km/h. and, (x + y) × =
2 × 60 4
\ Time taken to cover 60 Km upstream
3 × 2 × 60
60 ⇒ x + y = = 6 (2)
= hr 4 × 15
30 − x
and time taken to cover 60 Km upstream Solving Eqs. (1) and (2), x = 5 Km/h.

60 15.
(d) In 1 h water entered into ship = (20 – 12) = 8 tonnes
= hr
30 − x    Now, it will take 10 hrs to allow to enter 50 tonnes
According to the question, of water into ship and in this time ship has to cover
55 Km of distance.
60 60 1 9 Hence, required speed = 5.5 Km/h.
+ = 4 =
30 − x 30 + x 2 2

16.
(d) Let the speed of current be x m/minute. Then, speed
60(30 + x + 30 − x) 9 with current = (48 + x) m/minute and speed against
or, =
(30 − x)(30 + x) 2 current = (48 – x) m/minute

or, 60 × 60 × 2 = 9(900 – x2) 200 200
− = 10
or, 2
900 – x = 800 (48 − x) (48 + x)

or, x2 =100 ⇒ 40x = (48)2 – x2
\ x = 10 Km/h. ⇒ x2 + 40x – 2304 = 0

11.
(d) Suppose speed of the stream = K m/h ⇒ (x + 72)(x – 32) = 0
Speed of the boat in still water =10 m/h ⇒ x = 32 m/minute.
\ Boat will travel with the stream (downstream)
@ (10 + K) m/h and boat will travel against the 17.
(b) Upstream speed = 2 Km/h
stream (upstream) @ (10 – K) m/h.
1
Now, from the question, Downstream speed = × 60 = 6 Km/h
10
36 90 36 \ Speed in stationary water
+ =
10 + K 60 10 − K 2+6
= = 4 Km/h
K=2
⇒        2
5
12.
(c) Speed of speed boat = 16 – 3 = 13 Km/h \ Required time = = 1 hr 15 minute.
4
\ Speed of boat against the current
= 13 – 3 = 10 Km/h 18.
(d) Let the man went up the stream for x Km. Then, he
turned back for (x – 2) Km.
13.
(c) Let the speed in still water be x Km/h and speed of x x−2
current be y Km/h. \ + = 2 hrs 10 minutes
(4.5 − 1.5) (4.5 + 1.5)
Then, 2(x + y) = 3(x – y)
2x + x − 2 1
⇒ = 2
⇒ x = 5y 6 6
x 5 ⇒ 3x – 2 = 13
⇒ =
y 1 ⇒ x = 5 Km

Chapter 13.indd 9 6/5/2015 1:37:40 AM


13.10 I Chapter 13

19. (a) Let the speed of man and current be x Km/h and y 24. (d) Let the upstream speed be x Km/h
Km/h respectively. and the downstream speed be y Km/h
30 44 Then, according to the question,
Then, + = 10 (1)
x− y x+ y 40 55
+ = 13 (1)
x y
40 55
and, + = 13 (2) 30 44
x− y x+ y and, + = 10 (2)
x y
Solving Eqs. (1) and (2),
Solving the equations (1) and (2), we get x = 5 and y = 11
⇒ x + y = 11 (3)
Therefore, the speed of the man in still water
and, x – y = 5 (4) 1 1 16
Solving Eqs. (3) and (4), = ( x + y ) = (5 + 11) = = 8 km/h.
2 2 2
y = 3 Km/h.
25. (d) Speed of the boat in standing water = 9 Km/h
20.
(c) Let the place be x Km. Speed of the stream = 1.5 Km/h
∴ Speed downstream = 9 + 1.5 = 10.5 Km/h
x x 32 and speed upstream = 9 – 1.5 = 7.5 Km/h
Then, + =
8−2 8+2 60 Therefore, total time taken to row up and down
105 105
5 x + 3x 32 = + = 10 + 14 = 24 hrs
⇒ = 10.5 7.5
30 60
26. (b) Let the distance AB = x
32 30
⇒ x = × 2 km.
= Now, speed downstream = 14 + 4 = 18 Km/h
60 8 Speed upstream = 14 – 4 = 10 Km/h
21.
(a) Let the man was swimming at the rate of x Km/h and Distance
Since, = Time
he can swim D Km upstream. Speed
D 2D x x
\ = ∴ + = 19
1 1 18 2.10
x −1 x +1 x x
2 2 ⇒ + = 19
18 20
3
⇒ x + = 2x – 3 (10 + 9) x
2 ⇒ = 19
1 180
⇒ x = 4 Km/h. ⇒ x = 180 Km.
2
27. (d) Let the speed of boat in still water = x Km/h
22.
(d) Let the distance between A and B be x Km.
If distance covered by the boat in one side be d, then
Given, speed of boat in still water = 9 Km/h and speed d
= 1 (1)
of current = 3 Km/h x+3
\ Upward speed = (9 – 3) = 6 Km/h and downward [ speed downstream = x + 3 ]
speed = (9 + 3) = 12 Km/h d 3
and, = [ speed upstream = x − 3]
x−3 2
x x
\ + =3 ⇒ 2d = 3( x − 3)
6 12
⇒ 2( x + 3) = 3( x − 3) [using Eq.(1)]
⇒ x = 12 Km ⇒ 2 x + 6 = 3x − 9
23. (d) Let the rate of flow of river be x Km/h. ⇒ x = 15 Km/h.

Then, downward speed = (10 + x) Km/h
28. (a) Required time = Time spent in upstream + Time spent
and upward speed = (10 – x) Km/h in downstream
91 91 22.5 22.5
Given,   + = 20 = +
(10 + x) (10 – x) 6 − 1.5 6 + 1.5
22.5 22.5
For x = 3, the above expression holds true. = +
4.5 7.5
Hence, speed of flow of river is 3 Km/h. = 5 + 3 = 8 hrs.

Chapter 13.indd 10 6/5/2015 1:37:43 AM


Boats and Streams I 13.11

29. (d) Let speed of the current be x Km/h. 30. (d) Let the distance between A and B be x Km.
Here, speed of stream = 3 Km/h
Then, Speed of boat = 9 Km/h
According to the question,
200 200
− = 10 x x
48 − x 48 + x + =3
9−3 9+3
⇒ 20(48 + x − 48 + x) = (48 + x)(48 − x)
x x
⇒ 20 × 2 x = 2301 − x 2 ⇒ + =3
6 12
⇒ x 2 + 40 x − 2304 = 0 ⇒ 3 x = 36
⇒ x = 32 m/min ∴ x = 12 Km.

Difficulty Level-2

1.
(a) Let V be the speed of the ship. 126  4 4 4  126  524 29
=  =  1  9 hrs
12 4 55 11 11  5  55 55
Then, time t = 
V 1 V 1 For boat B2
12 4
\   3 ≤ 
V 1 V 1
≤4 I I I I I I IF
21 31! 42 47.5

2 2
⇒  3(V – 1) ≤ 12(V – 1) + 4 (V + 1 ≤ 4(V – 1)
In every 2 hrs, C and D cover 10.5 Km in 8 hrs, B2
⇒  (3V2 – 3) ≤ (16V – 8) ≤ (4V2 – 4)
covers 42 Km ⇒ in the next 1 hour, C covers 5.5 Km
⇒  3V2 – 16V + 5 ≤ 0 and 4V2 – 16V + 4 ≥ 0 1
1 2 1
1 To cover the last 2 Km, D takes 2 = hr
1st inequality is satisfied when  V  5 and the 2nd 2 5 2
3
inequality is satisfied for (2 – 3 ) ≤ V ≤ (2 + 3 ). 1 1
Total time required by boat B2 = 8 + 1 + = 9 hrs
2 2
1
\ we get ≤ V ≤ 2 + 3.
3 29 1 3 3 7
B2 wins by  = hour =  60 = 1
55 2 110 110 11
2.
(b) For boat B1 minutes.

3.
(a) Let Ramesh’s normal speed be u Km/h and the speed
of the current be v Km/h.

18 18
6 12 – = 9
To row 6 Km, A takes = hrs. u–v uv
1 11
5 18 18
2 and, – = 1
6 2u – v 2u  v
To row 6 Km, B takes hrs.
5 Let, u = kv
\ For each 12 Km, they take 1 1 v
– = (1)
12 6 126 k 1 k 1 2
 = hrs
11 5 55
1 1 v
and, – = (2)
At the end, there remains 2 Km, which is covered by 2k – 1 2k  1 18
2 4
A in = hrs Dividing Eqs. (1) by (2)
1 11
5
2 2 4k 2 – 1
2 =9
\ Total time required by boat B1 k –1 2

Chapter 13.indd 11 6/5/2015 1:37:45 AM


13.12 Chapter 13

⇒ 4k2 – 1 = 9k2 – 9 24 24
9.
(b)  =5
⇒ k = 85 (5.5  R ) (5.5  R )

4 4(5) 2 ⇒ R = 2.5 Km/h;  R ® Speed of river/current


From Eq. (1) ⇒  v = = = 6
k2 – 1 3 3 24
Again (B2 + R) = =6
4
4.
(b) Let the speed of the man in still water be x Km/h.
⇒ (B2 + 2.5) = 6 ⇒ B2 = 3.5 Km/h.
Then, 10.
(c) Let the speed of the man in still water be x Km/h and
speed of the stream be y Km/h then,
10 10 55
 = ⇒ x = 22 Km/h. 30 44
( x  2) ( x  2) 60  = 10 (1)
( x  y) ( x  y)
5.
(b) Upstream speed = B – S
40 55
Downstream speed = B + S  = 13 (2)
( x  y) ( x  y)
15
B – S = = 3 Km/h
5 Solving (1) and (2) x = 8 Km/h and y = 3 Km/h.
Again, B = 4S
11.
(b) Time taken by boat in down stream
\ B – S = 3 = 3S 50 5
= = hrs
⇒ S = 1 and B = 4 Km/h 60 6
\ B + S = 5 Km/h 5
Time taken by boat in upstream = hrs
15 4
\ Time during downstream = = 3 hrs.
2 2  50 100  24
Average speed = = = 48 m/h
6.
(c) Let the speed of the stream be x Km/h 5 5 50

6 4
Then, Upward speed = (5 – x) Km/h and Downward
speed = (5 + x) Km/h
12.
(d) Speed of boat in still water = 10 m/h
14 9
 = 5 ⇒ x = 2 Km/h. Let the speed of the steam = x m/h
(5  x) (5  x)
Then, speed of boat downward stream = (10 + x) m/h
7.
(b) Let x be the upstream speed, then the downstream Speed of boat upward steam = (10 – x) m/h
speed will be (x + 3)
36 90 36
3 3 \  =
\  = 3 (10  x) 60 (10  x)
x x3
⇒ x2 + x – 3 = 0 1  1 1 
⇒ = 4 
1 13 6  10  x 10  x 
⇒ x =
2
1  3.6 1  2x 
= = 1.3 Km/h ⇒ = 4 
2 6  100  x 2 
\ (x + 3) = 4.3 Km/h.
⇒ 100 – x2 = 48x
8. (a) Let the speed in the still water be x Km/h ⇒ x2 + 48x – 100 = 0
35 35
 = 12 ⇒ x = 2 because x ≠ –50.
( x  1) ( x  1)
or, 35 × 2x = 12(x2 – 1) 13.
(c) Speed of speed-boat = 16 – 3 = 13 Km/h.
2
⇒ 12x – 70x – 12 = 0 \ Speed of boat against the current
or, x = 6 Km/h = 13 – 3 = 10 Km/h.

Chapter 13.indd 12 6/5/2015 1:37:46 AM


Boats and Streams 13.13

14.
(c) In the final 1 minute before collision, the two boats a
1 1 1 \ = 2 (2)
travel 5  Km, and 10  Km i.e., Km and x y
1 60 60 12
Km. As they move in opposite directions, distance a
6 \ (1) ⇒ = 8 (3)
between the boats 1 minute before collision is x y

1 1 1 Dividing (2) and (3), we get


 =  Km.
12 6 4 x y 1
=
15.
(d) Let the speed of man in still water is x m/h and speed x y 4
of the current be y m/h. \ 4x – 4y = x + y
12 12 or, 3x = 5y
Then,  = 6 (1)
x y x y
x 5
or, = or, 5:3
12 12 y 3
and,  = 1 (2)
2x  y 2x  y
18.
(b) Upstream speed = 2 Km/h
Solving Eqs. (1) and (2), y = 8/3.
Downstream speed = 6 Km/h
16.
(a) Let upstream speed = x Km/h and downstream speed 26
\ Speed in still water = = 4 Km/h
= y Km/h 2
30 44 40 55 \ Time required to go 5 Km in still water
Then,  = 10, and  = 13
x y x y
5
or, 30u + 44v = 10, and 40u + 55v = 13 = hrs = 1 hr 15 minutes.
4
1 1
where, u = and v = 19.
(d) Speed upstream = (3 – 2) Km/h = 1 Km/h
x y
Speed downstream = (3 + 2) Km/h = 5 Km/h
1 1
Solving, we get u = and v =
5 11  10 10 
Total time taken =    hrs = 12 hrs
1 5
\ x = 5 and y = 11

5  11 20.
(b) Let speed in still water = x Km/h
\ Rate in still water = = 8 Km/h.
2 Speed of current = y Km/h
11  5 Speed downstream = (x + y) Km/h
Rate of current = = 3 Km/h
2 Speed upstream = (x – y) Km/h

17.
(a) Let the speed of man in still water = x Km/h \ 2(x + y) = 3(x – y)
Speed of the current = y Km/h \ x = 5y
Speed downstream = (x + y) Km/h x 5
Speed upstream = (x – y) Km/h or, = or, 5:1
y 1
Let river be flowing from P to R and PQ = QR = a
Then, PR = 2a 21.
(d) Let man’s rowing speed in still water = x Km/h and
speed of current = y Km/h
Speed upstream = (x – y) Km/h and speed downstream
= (x + y) Km/h
a a \ 5(x – y) = 12 and 5(x + y) = 28
\  = 10. (1)
x y x y subtracting 10y = 16
2a 8 3
and, = 4 \ y = = 1 Km/h
x y 5 5

Chapter 13.indd 13 6/5/2015 1:37:47 AM


13.14 Chapter 13

 n  1 3 1
22. (a) Speed of man =  speed of stream ⇒ = (12  x)
 n  1 2 2
 2  1 3 9 1 \ x = 12 – 3 = 9 Km/h
=   = or, 4 Km/h.
 2  1 2 2 2 26.
(b) Speed of the swimmer upstream
28
23.
(c) The distance between A and B is = = 7 Km/h
4
t ( x2  y 2 ) 3(81  9) Speed of the swimmer downstream
= Km = = 12 Km.
2x 29 40
= = 10 Km/h
24.
(b) Let the total journey time be t 4
\ Speed of the stream
t ( x2  y 2 )
Then, we have d = 1
2x = (Downstream Speed – Upstream Speed)
2
t (36  4) 3
⇒ 32 = = (10 – 7) = = 1.5 Km/h
26 2
\ t = 12 hrs
27.
(a) Speed of man in still water = 10 Km/h
25.
(c) Speed of the man downstream = 12 Km/h Speed of current = 3 Km/h
3 \ Speed of man upstream = 10 – 3 = 7 Km/h
Speed of the stream = Km/h
2
x x 75
Let the speed of the man upstream = x Km/h 28.
(b) Q  =
4 6 60
We have, Speed of the stream 10 x 5
\ =
1 24 4
= (Downstream Speed – Upstream Speed)
2 ⇒ x = 3 Km.

Chapter 13.indd 14 6/5/2015 1:37:48 AM


CHAPTER

Races and Games of Skill 14


INTRODUCTION Suppose A and B are two contestants in a race. We give below
certain statements and their corresponding mathematical
Race A contest of speed in running, driving, riding sailing meanings, which are frequently used:
or rowing over a specifi ed distance is called race.
Statements Mathematical Interpretation
Race Course The ground or path on which contests are
arranged is called a race course. 1. A beats B by t s A fi nishes the race t s before B
fi nishes.
Starting Point The point from where a race begins is called 2. A gives B a start of t A starts t s after B starts
the starting point. s from the same point.
Winning Post (or Goal) The point where the race fi nishes 3. A gives B a start of x While A starts at the starting
m point, B starts x m ahead from
is called the winning post or finishing point or goal.
the starting point at the same
Dead-heat Race If all the persons contesting a race reach time.
the fi nishing point exactly at the same time, then the race is 4. Game of 100 A game in which the participant
called a dead-heat race. scoring 100 points fi rst is the
winner.
Winner The person who fi rst reaches the fi nishing point is 5. In a game of 100, “A While A scores 100 points, B
the winner. can give B 20 points” scores only (100 – 20) or 80
points.

soMe useful sHort-cut MetHoDs

 5/3   5 
1. If A is n times as fast as B and A gives B a start = 60   = 60   = 150 m
of x m, then the length of the race course, so that  5/3 − 1  5−3
both A and B reach the winning post at the same
time, must be 2. If A can run xm race in t1 s and B in t2 s, where t1
 n  < t2, then A beats B by a distance
x  m.
 n −1  x
× (t2 − t1 )m .
2 t2
Illustration 1 A is 1 times as fast as B. If A gives B a start
3
of 60 m, how long should the race course be so that both of Illustration 2 A can run 100 m in 27 seconds and B in 30
them reach at the same time? seconds. By what distance A beats B?
5
Solution: Here n = and x = 60 Solution: A beats B by a distance
3
 n  x 100 300
\ Length of race course = x   = × (t2 − t1 ) = (30 − 27) = = 10 m
 n −1  t2 30 30

Chapter 14.indd 1 6/5/2015 1:35:40 AM


14.2 I Chapter 14

Practice Exercises

Difficulty Level-1
(Based on Memory)

1. Alok and Atul are competing in 800 m race. Alok gives 8. A and B run a 1,760 m race ending in a dead heat. At first
Atul a lead of 100  m. Initially, Alok runs at x times of A runs 20% faster than B. B then quickens his pace, and
Atul’s speed (x is an integer > 1), but after crossing 600 m, for the remaining distance runs 20% faster than A. When
1 B quickens his pace. A has already run:
he slows down to a speed which is time of Atul’s speed.
x (a) 800 m (b) 1,000 m
Atul continues to run at his original speed. If both of them (c) 790 m (d) 960 m
meet for the second time at a distance of 50 m from the
finish line, then who wins the race and by what distance? 9. In a running race, A gives a head start of 350 m to B. If the
(a) Atul, 33.33 m (b) Alok, 37.5 m ratio of speeds of A and B is 20:13, how far must the winning
(c) Atul, 37.5 m (d) Alok, 25 m post be so that A and B finish the race at the same time?
(a) 1 Km (b) 2 Km
2. A racecourse is 400 m long. A and B run a race and A wins (c) 3 Km (d) None of these
by 5 m. B and C run over the same course and B wins by
4 m. C and D run over it and D wins by 16 m. If A and D 10. A beats B by 100 m in a race of 1200 m and B beats C by
run over it, then who would win and by how much? 200 m in a race of 1600 m. Approximately by how many
(a) A by 8.4 m (b) D by 8.4 m metres can A beat C in a race of 9,600 m?
(c) D by 7.2 m (d) A by 7.2 m (a) 1,600 m (b) 1,800 m
[Based on MAT, 2002] (c) 1,900 m (d) 2,400 m
3. A and B start simultaneously from the same end of a 11. In an 800 m race around a stadium having the
50 m long swimming pool. The ratio of their speeds was circumference of 200 m, the top runner meets the last
2:1, respectively. If the race was one of 1,000  m, how runner on the 5th minute of the race. If the top runner runs
many times after the start would they have met at the end at twice the speed of the last runner, what is the time taken
of the race ? by the top runner to finish the race?
(a) 19 (b) 18 (a) 20 minutes (b) 15 minutes
(c) 16 (d) 15 (c) 10 minutes (d) 5 minutes
4. In a kilometre race, A beats B by 100 m and B beats C by 150
12. In a race, Guninder was not the first. Joginder came in
m. In the same race, by how many metres does A beat C?
after Harinder. Inderjeet was not ahead of Maninder.
(a) 250 m (b) 240 m
Guninder was not in front of Joginder. Inderjeet was not
(c) 225 m (d) 235 m fourth or fifth. Maninder was not the first. Who finished
[Based on IIT Joint Man. Ent. Test, 2004] first and second in the race?
5. In a 1,000 m long race, Karun gives Varun a head start of (a) Harinder followed by Maninder
40 m, and still beats Varun by 10 m. Find the distance by (b) Harinder followed by Joginder
which Karun will beat Varun, if Varun gives a start of 40 (c) Harinder followed by Guninder
m to Karun.
(d) Cannot be determined
(a) 88 m (b) 52 m
(c) 40 m (d) 50 m Directions (Q. 13–15): Answer the questions based on the following
information.
6. Abha beats Mansi in a kilometre race by 50 seconds and
   A certain race is made up of three stretches: A, B and C, each
Mansi beats Lata by 40 seconds in the same race. If Abha
2 Km long, and to be covered by a certain mode of transport. The
beats Lata by 450 m in a race of a 1 kilometre. How much
table given further gives these modes of transport for the stretches,
time Lata takes to run a kilometre?
and the minimum and the maximum possible speeds (in Km/h)
(a) 200 seconds (b) 150 seconds over these stretches. The speed over a particular stretch is assumed
(c) 110 seconds (d) 120 seconds to be constant. The previous record for the race is 10 minutes.
7. In a 100 m race, Sujit beats Rishi by 5 m and Rishi beats
Praveen by 5 m. By what distance does Sujit beats Praveen? A Car 40 60
(a) 10 m (b) 11 m B Motorcycle 30 50
(c) 9 m (d) 9.75 m I C I Bicycle I 10 I 20 I

Chapter 14.indd 2 6/5/2015 1:35:41 AM


Races and Games of Skill I 14.3

13. Anshuman travels at minimum speed by car over A and 21.


There are two concentric circular tracks of radii 100 m
completes stretch B at the fastest speed. At what speed should and 102 m, respectively. A runs on the inner track and
he cover stretch C in order to break the previous record? goes once round the track in 1 minute 30 seconds, while B
(a) Maximum speed for C (b)  Minimum speed for C runs on the other track in 1 minute 32 seconds. Who runs
(c) Cannot be determined  (d)  None of these faster?
(a) Both A and B are equal  (b)  A
14. Mr Hare completes the first stretch at the minimum speed
(c) B            (d)  None of the above
and takes the same time for stretch B. He takes 50% more
[Based on MAT (Sept), 2008]
time than the previous record to complete the race. What
is Mr Hare’s speed for the stretch C? 22.
A can run 22.5 m while B runs 25 m. In kilometre race B
(a) 10.9 Km/h (b) 13.3 Km/h beats A by:
1
(c) 17.1 Km/h (d) None of these (a) 100 m (b) 111 m
9
15. Mr Tortoise completes the race at an average speed of (c) 25 m (d) 50 m
20 Km/h. His average speed for the first two stretches is 4 [Based on FMS (MS), 2006]
times that for the last stretch. Find the speed over stretch C. 23.
In a 10 Km race, First beats Second by 2 Km and First
(a) 15 Km/h (b) 12 Km/h beats Third by 4 Km. If the runners maintain constant
(c) 10 Km/h (d) Cannot be determined speeds throughout the race, by how many kilometers does
Second beat Third?
16.
In a race of 600 m, A can beat B by 60 m and in a race of 1 1
500 m, B can beat C by 25 m. By how many metres will A (a) 2 (b) 2
4 2
beat C in a 400 m race?
3
(a) 56 m (c) 60 m (c) 2 (d) 3
4 [Based on FMS, 2010]
(c) 58 m (d) 54 m
[Based on MAT (Dec), 2009, 2008, 2007] 24.
Hari and Ravi started a race from opposite ends of the
pool. After a minute and a half, they passed each other in
17.
A can give B a start of 20 m and C a start of 39 m in a
the centre of the pool. If they lost no time in turning and
walking race of 400 m. How much start can B given C?
maintained their respective speeds, how many minutes
(a) 22 m (c) 20 m after starting did they pass each other the second time?
(c) 15 m (d) 26 m 1
[Based on MAT (Dec, Sept, May), 2009 (Dec), 2008]
(a) 3 (b) 4
2
18.
A and B run a 5 Km race on a round course of 400 m. If 1
their speeds be in the ratio 5:4, how often does the winner (c) 6 (d) 7
2
pass the other? [Based on FMS, 2011]
1 3 25. The respective ratio between the speeds of a car, a train
(a) 4 times (c) 2 times
2 4 and a bus is 5:9:4. The average speed of the car, the bus
and the train is 72 Km/h together. What is the average
1 1
(c) 3 times (d) 2 times speed of the car and the train together?
2 2
(a) 82 Km/h (b) 78 Km/h
[Based on MAT (Dec), 2008]
(c) 84 Km/h (d) Cannot be determined
19.
A long distance runner runs 9 laps of a 400 m track every
day. His timings (in minute) for four consecutive days are 26. In a thousand metre race A beats B by 100 m and B beats
88, 96, 89 and 87 respectively. On an average, how many C by 100 m. How many metres start can A give to C?
m/min does the runner cover? (a) 195 m (b) 200 m
(a) 17.78 (b) 90 (c) 205 m (d) 190 m
(c) 40 (d) None of these [Based on ATMA, 2005]
[Based on MAT (Sept) 2008] 27.
Four runners started running simultaneously from a point
20.
Muan and Sanjay ran a race which lasted a minute and a on a circular track. They took 200 seconds, 300 seconds,
half. Muan gave Sanjay a start of 9 m and beat him by 1 360 seconds and 450 seconds to complete one round.
m. Muan ran 40 m while Sanjay ran 39 m. The length of After how much time do they meet at the starting point for
the course is: the first time?
(a) 395 m (b) 355 m (a) 1800 seconds (b) 3600 seconds
(c) 400 m (d) 410 m (c) 2400 seconds (d) 4800 seconds
[Based on MAT (Sept), 2008] [Based on SSC (GL), 2011]

M14_KHAT6981_C14.indd 3 6/17/2015 12:42:19 PM


14.4 Chapter 14

28. In a 100 m race, Kamal defeats Bimal by 5 seconds. If the (c) Arjun beats Karan by 11 m
speed of Kamal is 18 Km/h, then the speed of Bimal is: (d) Karan beats Arjun by 1 m
(a) 15.4 Km/h (b) 14.5 Km/h 33. A sprinter starts running on a circular path of radius r m.
(c) 14.4 Km/h (d) 14 Km/h Her average speed (in m/min) is pr during the first 30 s,
[Based on SSC (GL), 2010] pr/2 during next 1 minute, pr/4 during next 2 minutes, pr/8
29. In a race of 200 m run, A beats S by 20 m and N by 40 during next 4 minutes, and so on. What is the ratio of the
m. If S and N are running a race of 100 m with exactly time taken for the nth round to that for the previous round?
same speed as before, then by how many metres will S (a) 4 (b) 8
beat N ? (c) 16 (d) 32
(a) 11.11 m (b) 10 m 34. A runs 1 times as fast as B. If A gives B a start of 120 m
(c) 12 m (d) 25 m and they reach the goal at the same time, the goal is at a
30. In a mile race, Akshay can be given a start of 128 m by distance of :
Bhairav. If Bhairav can give Chinmay a start of 4 m in a (a) 360 m (b) 440 m
100 m dash, then who out of Akshay and Chinmay will (c) 460 m (d) 380 m
win a race of one and half miles, and what will be the final 35. A can run 500 m in 30 seconds and B in 35 seconds. How
lead given by the winner to the loser? many metres start can A give to B in a Km race so that the
(One mile is 1,600 m) race may end in a dead-heat?
1 1 5 5
(a) Akshay, mile (b) Chinmay, mile (a) 139 (b) 138
2 32 7 7
6 5
1 1 (c) 142 (d) 140
(c) Akshay, mile (d) Chinmay, mile 7 7
24 16 36. Ram and Shyam run a race of 2000 m. First, Ram gives
31. Three runners A, B, and C run a race, with runner A finish Shyam a start of 200 m and beats him by 30 s. Next, Ram
12 m ahead of runner B and 18 m ahead of runners C, gives Shyam a start of 3 minutes and is beaten by 1000 m.
while runner B finishes 8 m ahead of runner C. Each Find the time in minutes in which Ram and Shyam can
runner travels the entire distance at a constant speed. run the race separately?
What was the length of the race? (a) 8 minutes, 10 minutes (b) 4 minutes, 5 minutes
(a) 36 m (b) 48 m (c) 5 minutes, 9 minutes (d) 6 minutes, 9 minutes
[Based on MAT, 2014]
(c) 60 m (d) 72 m
37. A race course is 400 metres long A and B run a race and A
32. Karan and Arjun run a 100 metre race, where Karan beats wins by 5 metres. B and C run over the same course and
Arjun by 10 m. To do a favour to Arjun, Karan starts 10 m B wins by 4 metres. C and D run over it and D wins by 16
behind the starting line in a second 100 metre race. They metres. If A and D run over it, then who would win and by
both run at their earlier speeds. Which of the following is how much?
true in connection with the second race? (a) A by 8.4 metres (b) D by 8.4 metres
(a) Karan and Arjun reach the finishing line simultaneously (c) D by 7.3 metres (d) A by 7.3 metres
(b) Arjun beats Karan by 1 m [Based on MAT, 2012]

Difficulty Level-2
(Based on Memory)

1. In a race of 200 m A beats S by 20 m and N by 40 m. If S (a) 11.11 m (b) 10 m


and N are running a race of 100 m with exactly the same (c) 12 m (d) 25 m
speed as before, then by how many metres will S beat N? [Based on CAT, 2010]

Chapter 14.indd 4 6/5/2015 1:35:41 AM


Races and Games of Skill I 14.5

Answer Keys
Difficulty Level-1

1.  (a) 2.  (c) 3.  (d ) 4.  (d ) 5.  (a) 6.  (a) 7.  (d ) 8.  (a) 9.  (a) 10.  (c) 11.  (c) 12.  (c) 13.  (c)
14.  (b) 15.  (c) 16.  (c) 17.  (b) 18.  (d ) 19.  (c) 20.  (c) 21.  (a) 22.  (a) 23.  (b) 24.  (b) 25.  (c) 26.  (b)
27.  (a) 28.  (c) 29.  (a) 30.  (d ) 31.  (b) 32.  (d ) 33.  (c) 34.  (b) 35.  (c) 36.  (b) 37.  (c)

Difficulty Level-2

1.  (a)

Explanatory Answers

Difficulty Level-1

1. (a) It is clear that Atul wins the race. 4.


(d) By the time A covers 1000 m, B has covered 900 m.
   So (a) and (c) are left. Since they meet at a distance \ By the time B covers 900 m,
of 50 m from the finish line, total distance covered by By the time B covers 1000 m, C has covered 850 m, C
Atul = 650 m and Alok = 750 m will cover
600
⇒ 100 + + 150x = 750 850
x × 900 = 765 m
(in the same time, distance is proportional to speed) 1000
⇒ (3x – 4) (x – 3) = 0 \ A beats C by 235 m.
4 5.
(a) As the race is of 1,000 m.
⇒ x = 3 or,
3    In first case, Karun runs 1,000 m while Varun runs
Since x is an integer, it has to be 3. (960 – 10) = 950 m
   So, when Atul covers the remaining 50 m, Alok
   In second case, Karun runs 960 m then Varun will
50
covers only , i.e., 16.67 m 950  960
3 run = 912 m.
1000
Therefore, Atul wins by 33.33 m.
\ Karun will beat Varun by 1000 – 912 = 88 m.
2. (c) When A covers 400 m, B covers 395 m
When B covers 400 m, C covers 396 m 6.
(a) In a race of 1,000 m if Abha takes t second, then
Mansi takes (t + 50) sec and Lata takes (t + 90) s
When B covers 395 m, C covers
396    Now, If Abha travels 1,000 m, Lata travels 550 m
× 395 = 391.05 m    It is thus clear that Lata travels 1,000 m in (t + 90)
400
\ When A covers 400 m, B covers 391.05 m second and 550 m in t second.
When D covers 400 m, C covers 384 m \ 1000:550 = (t + 90):t
When C covers 391.05 m, D covers 407.3 m ⇒ 100t = 55t + 4950 ⇒ t = 110 seconds
\ When A covers 400 m, D covers 407.3 m Therefore, Lata takes (110 + 90) = 200 seconds in
When D covers 400 m, A covers 392.8 m travelling 1,000 m.
\ D wins by 7.2 m, when D and A run the race.
7.
(d) When Sujit runs 100 m, Rishi runs 95 m.
3.
(d) After the start at the end of every 200  m of A they
would have met thrice and both would be at the When Rishi runs 100 m, Praveen runs 95 m.
starting point again. Hence at the end of the 1,000 m \ When Rishi runs 95 m, Praveen runs 90.25 m.
 1000  When Sujit runs 100 m, Praveen runs 90.25 m and is
race they would have met 3  = 15 times.
 200  beaten by 9.75 m.

Chapter 14.indd 5 6/5/2015 1:35:41 AM


14.6 I Chapter 14

Alternative method: 12.


(c)
The ratio of speeds of Sujit and Rishi 13.
(c) Total time taken to cover stretch A at a minimum
= 100:95  2 1
= 20:19. speed =    hrs = 3 minutes.
 40  20
Similarly, the ratio of speeds of Rishi and Parveen
Likewise total time taken to vover stretch B at a
= 20:19.
 2
\ The ratio of speeds of Sujit and Parveen maximum speed =   = 2.4 minutes. Total time
 50 
= 202 : 102 taken in covering these two stretches = (3 + 2.4) = 5.4
⇒ When Sujit goes 100 m, Parveen goes minutes.
361    To break the previous record the third stretch will
 100 = 90.25 m have to be covered in (10 – 5.4) = 4.6 minutes.
400
2
\ The lead that can be given is 100 – 90.25 Required speed =   = 0.434 Km/min = 26.08
4.6
= 9.75 m. Km/h. Since the maximum speed is 20 Km/h hence it
is not possible for C to break the previous record.
8.
(a) Let t1 be the time at which B switches the speed and
t1 + t2 be the total time between start and finish. Let x 14. (b) Time taken to cover the stretch A at minimum
be the speed of B initially. So A’s speed = 1.2x and B’s 2
final speed = 1.44x speed = = 3 minutes.
40
Now lag of B in time t1 Time taken to cover stretch B = 3 minutes. Time taken
= (1.2x – x)t1 = 0.2 × t1 (1) by him in covering the entire race = (1.5 × 10) = 15
minutes.
Also, gain of B in time t2
   Hence remaining time to cover stretch = (15 – 6) =
= (1.44x – 1.2x)t2 = 0.24 × t2 (2) 9 minutes.
Since both reach at the same time,  2
Therefore required speed =   = 0.22 Km/min.
\ lag = lead  9
⇒ 13.3 Km/h.
t1 0.24 6
⇒ = =
t2 0.20 5 15. (c) Let the average speed for the last stretch be x Km/h,
hence his average speed for the first two stretches
1760  t1 = 4x. So, total time taken to cover the three stretches
\ A covers of the distance
t1  t2 4 2
= 
1760  5 4x x
= = 800 m.
11 4 2 6
\    ⇒  x = 10 Km/h.
9.
(a) Let their speeds be 20x and 13x. then relative speed 4 x x 20
of both of them moving in the same direction is 16. (c) In 600 m race, Ratio of distances
(20x – 13x) = 7x
A : B
   Now B is 350 m ahead of A and this distance has
600 : 540
to be covered with a relative speed of 7x. Therefore,
10 : 9
350
time to cover this distance = In 500 m race, Ratio of distances
7x B : C
350 500 : 475
Distance covered by A in this time = 20 : 19
7x
350 A:B:C = (10 × 20):(9 × 20):(19 × 9)
Speed × time = 20x × = 1,000 m = 1 Km.
7x = 200:180:171
10.
(c) Ratio of speeds of A:B = 12:11 and ratio of speeds of So, when A runs 200 m → C runs 171 m
B:C = 8:7
171
Therefore ratio of speeds of A:B:C = 96:88:77 When A runs 1 m → C runs m
200
So in 9,600 m race A will beat C by 1,900 m. When A runs 400 m → C runs = 342 m
11.
(c) A can beat C by = 400 – 342 = 58 m.

Chapter 14.indd 6 6/5/2015 1:35:42 AM


Races and Games of Skill I 14.7

17.
(b) In a 400 m race — Ratio of distances Circumference of inner track = 2 × p × 100 = 200 pm
A : B : C Circumference of outer track = 2 × p × 102 = 204 pm
400 : 380 : 361 200
Speed of A = = 133.33 m/min
So, when B runs 380 m → C runs 361 m 1.5
361 × 400 204
When B runs 400 m → C runs = 380 m Speed of B = = 133.33 m/min.
380 1.53
The start B can give C = 400 – 380 = 20 m. 22.
(a) In 25 m race B beats A by 2.5 m
18.
(d) It is clear from the question that when A covers 500 m, 2.5
In 1000 m race B beats A by × 1000 = 100 m.
B covers 400 m, i.e., A takes a lead of 100 m in every 25
500 m of distance. Therefore, a lead of 400 m will be
taken in travelling a distance of 2000 m or in other 23.
(b) If the First runs 10 Km then the Second runs 8 Km and
words A passes B every after 2000 m. the Third runs 6 Km.
Hence, total number of such pass \ When the Second runs 8 Km the Third runs 6 Km.
5000 5 1 When the second runs 10 Km then the third runs
= = = 2 times. 6  10
2000 2 2 = 7.5 Km
8
88 + 96 + 89 + 87 \ The Second beats the Third by 2.5 Km in a 10 Km
19.
(c) = 90 mm
4 race.
It means on an average he runs (400 × 9 = 3600)
24. (b) To meet for the first time, both together cover the length
m distance in 90 minutes or in other words he runs of the pool whereas to meet for the second time, both
3600 together need to cover 3 times the length of the pool.
= 40 m/min. 1
90    If to cover the length of the pool, they take 1
40 4 2
20.
(c) Speed of Muan = = m/s minutes, then to cover 3 times the length of the pool,
90 9
1
39 13 they will take 4 minutes.
Speed of Sanjay = = m/s 2
90 30
1
   So, after 4 minutes, they will meet each other
~I'O------ 9 m ---+1'1 2
Muan oo~--------"o Sanjay for the second time.
Relative speed of Muan 25. (c) Total speed of car, bus and train
 4 13  40 − 39 1 = 72 × 3 = 216 Km
=  −  = = m/s
 9 30  90 90 5+9
Speed of car and train = × 216 =
168 km
Now, in order to cover a distance of 10 m, Muan has 5+9+4
1 168
to run at a speed of m/s Average = = 84 Km.
90 2
t = 900 seconds
26. (b) ____|______|________
It cover the distance of course, he covers it with a
4 C 100 m B 100 m A
speed of m/s in 900 seconds.
9
 A  1000m 
4 
\ D = × 900 = 400 m.  B 900m   A   C 200 .
9 C  800m 
21. (a)
27. (a) Required = L.C.M. of 200, 300, 350 and 450 seconds
= 1800 seconds.
28.
(c) Time taken by Kamal to run 100 m
100
= = 20 seconds
5
18 ×
18

Chapter 14.indd 7 6/5/2015 1:35:43 AM


14.8 Chapter 14


Therefore, time taken by Bimal to run 100 m 33.
(c) The radius of the track is r m
= 20 + 5 = 25 seconds \ the circumference is 2pr m
Hence, Bimal’s speed
πr πr πr
100 The average speed for successive πr , , , ,
= = 4 m/sec 2 4 8
25 time intervals of 1/2, 1, 2, 4 etc. minutes is
4 × 18 πr πr πr πr
= Km/h = 14.4 Km/h. , , , etc., m/min. Therefore in each
5 2 2 2 2
29.
(a) In the time when A runs 200 m, S runs 180 m and N interval (of increasing duration) the distances travelled
runs 160 m. In other words, in the time when S runs are etc., (i.e., exactly the same). For such intervals are
180 m, N runs 160 m. needed to cover one round. The next four intervals
Therefore, when S runs 100 m, N will run are needed for the next round. As each interval in the
 160  second group is 16 times the corresponding interval in
= 100   = 88.89 m. the previous group, the total time for each round is 16
 180 
times taken for the previous round.
Hence in a 100m race, S will beat N by (100 – 88.89) =
11.11 m. 34.
(b) The speed of A and B are in the ratio 11:8.
30. (d) When Bhairav covers 1,600 m, Akshay covers (1600 Let speeds be 11s and 8s (in m/sec)
– 128) m. So when Bhairav covers (1600/6) = 100 m, Let race be of x m
Akshay covers (128/16) m = 8 m less.
Then time taken by A to run xm is same as that of B to
   When Bhairav covers 100 m, Chinmay covers
run (x – 120) m.
(100 – 4) = 96 m.
   Thus, the ratio in which Akshay and Chinmay x x  120
\ =
cover distance is 92:96. In 96 m, Chinmay gains 11s 8s
(96 – 92) = 4 m over Akshay. So, in 1.5 miles, \ 3x = 11 × 120
1 \ x = 440.
Chinmay gains 100 m =   miles over Akshay.
 16 
35. (c) Time taken by A to run 1 Km
31. (b) Let the distance of race be x m which is covered by A = 30 × 2 = 60 seconds
in t second. Then, in the same time B covers (x – 12)
Time taken by B to run 1 Km = 35 × 2 = 70 seconds
m and C covers (x – 18) m.
x \ A can give B a start of (70 – 60) = 10 seconds
\ Speed of A = m/s, In 35 seconds B runs 500 m
t
( x  12) 500 1000
Speed of B = m/s . \ In 10 seconds B runs = × 10 =
t 35 7
( x  18) 6
and, Speed of C = m/s    = 142 m
t 7
Time taken by B to finish the race 6
  So, A can give B a start of 142 m in a Km race.
x x 7
= = t s.
( x  12) ( x  12)
t 36. (b) Let the time taken by Ram to finish the race = t minute
Now, distance travelled by C in this time In the first case:
x ( x  18)
= t =x–8 Ram gives Shyam a start of 200 m and beats him by
( x  12) t
1
x( x  18) 30 seconds, i.e., minute.
⇒ = x – 8 2
( x  12)
2000
⇒ x = 48 metre. ∴ Speed of Ram =
t
32. (d) In the first race when Karan runs 100 m, Arjun runs 1800
and Speed of Shyam =
only 90 m. Hence, the ratio of speeds of Arjun and 1
Karan is 90:100 = 9:10. In the second race, Karan has t+
2
to run 110 m. When he finishes the race, Arjun would
9 In the second case:
have run  110 = 99 m Ram gives Shyam a start of 3 minutes and losses to
10
(i.e., 1m less than 100 m) Shyam by 1000 m.
Hence Karan beats Arjun by 1 metre. \ Time taken by Shyam − Time taken by Ram = 3 m

Chapter 14.indd 8 6/5/2015 1:35:44 AM


Races and Games of Skill 14.9

2000 1000 2000 t 1800 1800


⇒ − =3 ⇒ − =3 = = × 2 = 400 m/min
1800 2000 1800 × 2 2 1 (8 + 1)
4+
1 t 2t + 1   2
t+
2 So, to cover 2000 m Shyam will take 5 minutes.
2000 × (2t + 1) t
⇒ − =3
1800 × 2 2 37. (c) The statements in the question can be reformulated as
5 t follows:
⇒ (2t + 1) − = 3
9 2 If A covers 400 m, B covers 395 m
10(2t + 1) − 9t If B covers 400 m, C covers 396 m
⇒ =3
18 If D covers 400 m, C covers 384 m
20t + 10 − 9t = 18 × 3 Therefore, if B covers 395 m, then C will cover,
⇒ 11t = 54 − 10
369
44 × 395 = 391.05 m
∴ t= = 4 minutes 400
11
Again, if C covers 391.05 m, then D will cover
So, time taken by Ram to finish the race is 4 minutes.
1800 400
Again, speed of Shyam = × 391.05 = 407.34 m
1 384
t+
2 Thus, if A and D run over 400 m, then D wins by 7.3 m.

Difficulty Level-2

1. (a) In the time that A takes to run 200 m, S runs 180 m and N runs 160 m. So in the time

 160 
S takes to run 200 m, N runs 200   = 177.77 m or is beaten by 22.22 m. So, in 100 m, N is beaten by 11.11 m.
 180 

Chapter 14.indd 9 6/5/2015 1:35:46 AM


This page is intentionally left blank

FM.indd 6 6/10/2015 2:36:16 PM


CHAPTER

Alligation or Mixture 15
INTRODUCTION
Quantity of cheaper quality d −m
Alligation literally means ‘linking’. It is a rule to fi nd:
Quantity of superior quality
=
m−c
(a)
) the ratio in which two or more ingredients at their
respective prices should be mixed to give a mixture at
a given price. Explanation
(b) The mean or average price of a mixture when the Suppose x kg of cheaper quality is mixed with y kg of
prices of two or more ingredients which may be mixed superior quality
together and the proportion in which they are mixed are
Price of cheaper ingredient = `cx
given.
Here cost price of a unit quantity of mixture is called Price of superior ingredient = `dy
the mean price. \ Price of mixture = `(cx + dy)
and quantity of mixture = (x + y ) kg.
Alligation Rule
Suppose `d per unit be the price of fi rst ingredient  cx + dy 
\ Price of mixture/kg = `  
(superior quality) mixed with another ingredient  x+ y 
(cheaper quality) of price `c per unit to form a cx + dy
mixture whose mean price is `m per unit, then the \ = m ⇒ cx + dy = mx + my
x+ y
two ingredients must be mixed in the ratio: ⇒ dy – my = mx – cx
Quantity of cheaper
⇒ y (d – m ) = x(m – c )
Quantity of superior
x d −m
C. P. superior − Mean price ⇒ =
= y m−c
Mean price − C. P. of cheaper
Illustration 1 In what ratio two varieties of tea one costing
i.e., the two ingredients are to be mixed in the inverse `27 per kg and the other costing `32 per kg should be
ratio of the differences of their prices and the mean blended to produce a blended variety of tea worth `30 per
price. kg. How much should be the quantity of second variety of
The above rule may be represented schematically as tea, if the fi rst variety is 60 kg
under : Solution:

Chapter_15.indd 1 6/5/2015 2:29:07 AM


15.2 Chapter 15

The required ratio of the two varieties of tea is 2:3,


i.e.,

Quantity of cheaper tea 2


=
Quantity of superior tea 3

60 × 3
\ Quantity of superior tea = = 90 kg
2

Thus, the second variety of tea is 90 kg. Quantity of cheaper sugar 20 − x


=
Quantity of dearer sugar x − 15
Illustration 2 Sugar at `15 per kg is mixed with sugar at 20 − x 2
`20 per kg in the ratio 2:3. Find the price per kg of the \ = ⇒ 60 – 3x = 2x – 30
x − 15 3
mixture
⇒ 5x = 90 or x = 18
Solution: Let the mean price of the mixture be `x Thus, the price per kg of the mixture is `18.

soMe useful MetHoDs

n
1. A vessel, full of wine, contains ‘a’ litres of Wine left after nth operation  b
\ = 1 −  .
which ‘b’ litres are withdrawn. The vessel is then Original quantity of wine  a
fi lled with water. Next, ‘b’ litres of the mixture
are withdrawn and again the vessel is fi lled with Illustration 3 A vessel contains 125 litres of wine. 25 litres
water. This process is repeated n times. Then, of wine was taken out of the vessel and replaced by water.
Wine left in the vessel after nth operation Then, 25 litres of mixture was withdrawn and again replaced
Original quantity of wine in the vessel by water. The operation was repeated for third time. How
much wine is now left in the vessel?
n
 a −b  Solution: Amount of wine left in the vessel
=   .
 a  3
 25  100 × 100 × 100 × 125
= 1 −  × 125 = = 64 litres
Explanation  125  125 × 125 × 125
Amount of wine after 1st operation
 b 2. There are n vessels of equal size fi lled with
= a – b = 1 −  × a
 a mixtures of liquids A and B in the ratio a1:b1:a2,
Ratio of wine and water after 1st operation is (a – b):b b2..., an:bn, respectively. If the contents of all the
\ In ‘b’ ’ litres of mixture withdrawn in 2nd operation, vessels are poured into a single large vessel, then
amount of wine withdrawn
a −b b Quantity of liquid A
= × b = (a – b)
( a − b) + b a Quantity of liquid B
\ Amount of wine left after 2nd operation
b  b
2  a1 a2 an 
= (a – b) – (a – b) × = a 1 −   + + ... + 
a  a  a1 + b1 a2 + b2 an + bn 
In general, quantity of wine left after nth operation =
 b1 b2 bn 
n  + + ... + 
 b  a1 + b1 a2 + b2 an + bn 
= 1 −  a
 a

Chapter_15.indd 2 6/5/2015 2:29:08 AM


Alligation or Mixture 15.3

Explanation Solution: 3 5 9
Let the capacity of each vessel be c litres. + +
Quantity of milk
Amount of liquid A in different vessels = 3 +1 5 + 3 9 + 7
Quantity of water 1 3 7
+ +
a1c a2c a3c an c 3 +1 5 + 3 9 + 7
= , , , ...,
a1 + b1 a2 + b2 a3 + b3 an + bn 31/16
= = 31:17.
Amount of liquid B in different vessels 17/17
b1c b2c b3c bn c
= , , , ..., 3. There are n vessels of sizes c1, c2, ... , cn fi lled
a1 + b1 a2 + b2 a3 + b3 an + bn with mixtures of liquids A and B in the ratio
So, in the resulting mixture, amount of liquid A a1 : b1, a2 : b2, ..., an:bn, respectively. If the
 a a2 an  contents of all the vessels are poured into a single
=  1 + + ... +  ×c large vessel, then
 a1 + b1 a2 + b2 an + bn  a1c1 ac a c
+ 2 2 + ... + n n
Amount of liquid B Quantity of Liquid A a + b a2 + b2 an + bn
= 1 1
 b b2 bn  Quantity of liquid B b1c1 b2c2 bc
=  1 + + + ... + n n
+ ... +  ×c a1 + b1 a2 + b2 an + bn
 a1 + b1 a2 + b2 an + bn 
Quantity of liquid A Illustration 5 Three glasses of sizes 3 litres, 4 litres and
Quantity of liquid B 5 litres contain mixture of milk and water in the ratio 2:3,
3:7 and 4:11, respectively. The contents of all the three
 a1 a2 an  glasses are poured into a single vessel. Find the ratio of milk
 + + ... + 
a + b a + b an + bn  to water in the resulting mixture
=  1 1 2 2
 b1 b2 bn  Solution:  2 × 3 3× 4 4 × 5 
 + + ... +   + + 
 a1 + b1 a2 + b2 an + bn  Quantity of milk  2 + 3 3 + 7 4 + 11 
=
Illustration 4 Three equal glasses are fi lled with mixture of Quantity of water  3 × 3 7 × 4 11× 5 
 + + 
milk and water. The proportion of milk and water in each  2 + 3 3 + 7 4 + 11 
glass is as follows: In the fi rst glass as 3:1, in the second 6 12 20
+ +
glass as 5:3 and in the third as 9:7. The contents of the
= 5 10 15 = 56:124 or, 14:31
three glasses are emptied into a single vessel. What is the 9 28 55
+ +
proportion of milk and water in it? 5 10 15

Practice Exercises

Difficulty level-1
(BaseD on MeMory)

1. Tea worth `126 per kg and `135 per kg are mixed with 3. In a mixture of 45 litres, the ratio of milk and water is 3:2.
a third variety in the ratio 1:1:2. If the mixture is worth How much water must be added to make the ratio 9:11?
`153 per kg, then the price of the third variety per kg will (a) 10 litres (b) 15 litres
be:
(c) 17 litres (d) 20 litres
(a) `169.50 (d) `170 [Based on MAT, 2002]
(c) `175.50 (d) `180
4. Two jars contain milk and water in the ratio 5:4 and 2:1,
[Based on MAT, 2001] respectively. What volume should be taken out from the
2. If the cost price of water is one-tenth that of milk and the fi rst jar if volumes have to be taken out from both jars so
milkman claims to sell the milk at a profi t of 20%, then as to fi ll up a third 30 litres jar with milk to water in the
what is his actual net profi t percentage? ratio 1:1?
(a) 72% (b) 82% (a) 7.5 litres (b) 15 litres
(c) 79% (d) 68% (c) 22.5 litres (d) It is impossible

Chapter_15.indd 3 6/5/2015 2:29:09 AM


15.4 Chapter 15

5. A dairy man pays `6.4 per litre of milk. He adds water and (a) 64:65 (b) 65:64
sells the mixture at `8 per litre, thereby making 37.5% (c) 19:65 (d) 65:19
profit. Find the proportion of the water to that of the milk
received by the customers. 13. In what ratio must a person mix three kinds of wheat
(a) 1:15 (b) 1:10 costing him `1.20, `1.44 and `1.74 per kg, so that the
mixture may be worth `1.41 per kg?
(c) 1:20 (d) 1:12
(a) 15:77:11 (b) 7:11:15
[Based on MAT, 2003]
(c) 11:77:15 (d) 11:77:7
6. Mr X mixed 10 kg of variety A rice with 15 kg of variety
B rice and sold the mixture at a price 40% more than that 14. The ratio of kerosene to petrol in 100 kg of mixed petrol
of A. He did not get any profit. What is the ratio of the cost normally used by three wheelers is 7:25. The amount of
price of variety A to that of B per kg? kerosene to be added to 100 kg of mixed petrol to make
(a) 2:5 (b) 3:5 the ratio 9:25 is:
(c) 4:5 (d) 5:8 (a) 5 kg (b) 6.625 kg
(c) 6.25 kg (d) 6.35 kg
7.
Several litres of acid were drawn off a 54-litre vessel full
of acid and an equal amount of water added. Again the 15. Two equal containers are filled with a mixture of water
same volume of the mixture was drawn off and replaced and alcohol. One of them contains three times as much
by water. As a result, the vessel contained 24 litres of pure alcohol as in the other. The mixtures in the two containers
acid. How much of the acid was drawn off initially? are then mixed and it is found that the ratio of water to
(a) 12 litres (b) 16 litres alcohol is 3:2. Find the ratio of water to alcohol in each of
(c) 18 litres (d) 24 litres the original containers.
[Based on MAT, 2001] (a) 2:1, 3:4 (b) 1:3, 1:2
8.
If 50% of the 2:3 solution of milk and water is replaced (c) 2:3, 4:1 (d) None of the above
with water, then the concentration of the solution is
reduced by: 16. I added 1 litre of water to 5 litres of a 20 per cent solution
(a) 25% (b) 33.33% of alcohol and water. The strength of alcohol is:
(c) 50% (d) 75% (a) unaltered (b) 5%
[Based on MAT, 2001] (c) 12.5% (d) 16.66%

9. Milk sold by a milkman contains 5% water. What quantity 17.


One type of liquid contains 25% of milk, the other contains
of pure milk should be added to 20 litres so that water 30% of milk. A can is filled with 6 parts of the first liquid
content comes down to 2%? and 4 parts of the second liquid. Find the percentage of
(a) 16 litres (b) 20 litres milk in the new mixture.
(c) 25 litres (d) None of these (a) 28% (b) 25%
[Based on IRMA, 2002] (c) 30% (d) 27%

10. Milk and water are in the ratio of 3:2 in a mixture of 80 18.
A jar contains a mixture of two liquids A and B in the ratio
litres. How much water should be added so that the ratio 4:1. When 10 litres of the mixture is taken out and 10
of the milk and water becomes 2:3? litres of liquid B is poured into the jar, the ratio becomes
(a) 25 litres (b) 40 litres 2:3. How many litres of liquid A was contained in the jar?
(c) 35 litres (d) 20 litres (a) 14 litres (b) 18 litres
[Based on FMS (Delhi), 2002] (c) 20 litres (d) 16 litres
11. How much water must be mixed in 300 ml of sugar 19.
How many litres of a 3% hydrogen peroxide solution
solution which contains 40% sugar by weight, such that it should be mixed with 6 litres of a 30% hydrogen peroxide
becomes a 30% sugar solution? solution so as to get a 12% solution?
(a) 50 ml (b) 75 ml (a) 3 litres (b) 6 litres
(c) 90 ml (d) 100 ml (c) 9 litres (d) 12 litres

12. Three vessels contain equal mixtures of milk and water in 20.
How much water must be added to 100 cc of 80 per cent
the ratio 6:1, 5:2 and 3:1 respectively. If all the solutions solution of boric acid to reduce it to a 50 per cent solution?
are mixed together, the ratio of milk to water in the final (a) 30 (b) 40
mixture will be: (c) 50 (d) 60

Chapter_15.indd 4 6/5/2015 2:29:09 AM


Alligation or Mixture 15.5

21. Milk and water are in the ratio of 3:2 in a mixture of 80 (a) 10 litres (b) 5 litres
litres. How much water should be added so that the ratio (c) 15 litres (d) 20 litres
of the milk and water becomes 2:3? [Based on MAT (Sept), 2009]
(a) 25 litres (b) 40 litres 30.
An 8-litre cylinder contains a mixture of oxygen and
(c) 35 litres (d) 20 litres nitrogen, the volume of oxygen being 16% of total
22. Milk sold by a milkman contains 5 per cent water. What volume. A few litres of the mixture is released and an
quantity of pure milk should be added to 20 litres so that equal amount of nitrogen is added. Then, the same
water content comes down to 2%? amount of the mixture as before is released and replaced
(a) 16 litres (b) 20 litres by nitrogen for the second time. As result, the oxygen
(c) 25 litres (d) None of these content becomes 9% of total volume. How many litres of
mixture is released each time?
23. A dairy man pays `6.4 per litre of milk. He adds water and
(a) 7 litres (b) 40 litres
sells the mixture at `8 per litre, thereby making 37.5%
profit. Find the proportion of the water to that of the milk (c) 2 litres (d) None of these
received by the customers. [Based on MAT (Sept), 2009, 2008 (Dec), 2005]

(a) 1:15 (b) 1:10 31.


Two gallons of a mixture of spirit and water contain 12%
of water. They are added to 3 gallons of another mixture,
(c) 1:20 (d) 1:12
containing 7% of water and half a gallon of water is then
24. In a mixture of 60 litres, the ratio of milk to water is 2:1. If added to the whole. Find the percentage of water in the
this ratio is to be 1:2, then the quantity of water (in litres) resulting mixture.
to be further added is:
3 12
(a) 20 (b) 30 (a) 17 % (b) 16 %
11 11
(c) 40 (d) 60
25. Mira’s expenditure and savings are in the ratio 3:2. Her 1
(c) 14 % (d) None of these
income increases by 10 per cent. Her expenditure also 11
increases by 12 per cent. By how much per cent does her [Based on MAT (Sept), 2009]
saving increase? 32.
A jar full of milk contains 40% water. A part of this milk
(a) 7% (b) 10% is replaced by another containing 19% water and now the
(c) 9% (d) 13% percentage of water is found to be 26% The quantity of
milk replaced is:
26. A trader has 50 kg of rice, a part of which he sells at 10 per
cent profit and the rest at 5 per cent loss. He gains 7 per (a) 2/3 (b) 1/3
cent on the whole. What is the quantity sold at 10 per cent (c) 3/7 (d) 4/7
gain and 5 per cent loss? [Based on MAT (Feb), 2009]
(a) 30 kg, 10 kg (b) 40 kg, 15 kg 33.
One litre of water is evaporated from 6 litres of a solution
(c) 35 kg, 40 kg (d) 40 kg, 10 kg containing 5% salt. Find the percentage of salt in the
remaining solution.
27.
The wheat sold by a grocer contained 10% low quality
wheat. What quantity of good quantity wheat should be (a) 8% (b) 10%
added to 150 kg of wheat so that the percentage of low (c) 6% (d) 4%
quality wheat becomes 5%? [Based on MAT (Feb), 2009]
(a) 85 kg (b) 50 kg 34.
When processing flower nectar into honey, a considerable
(c) 135 kg (d) 150 kg amount of water is added. How much flower nectar must
[Based on MAT (Feb), 2011] be processed to yield 1 kg of honey, if nectar contains 70%
28.
One type of liquid contains 25% of milk, the other contains water and the honey obtained from this nectar contains
30% of milk. A container is filled with 6 parts of the first 17% water?
liquid and 4 parts of the second liquid. The percentage of (a) 4.11 kg (b) 1.76 kg
milk in the mixture is: (c) 0.36 kg (d) 2.77 kg
(a) 27% (b) 31% [Based on MAT (Sept), 2008]
(c) 29% (d) 33% 35.
The amount of water (in mL) that should be added to
[Based on MAT (Dec), 2009 (Sept), 2009] reduce 9 mL lotion, containing 50% alcohol, to a lotion
29.
A solution of sugar syrup has 15% sugar. Another solution containing 30% alcohol, is:
has 5% sugar. How many litres of the second solution (a) 5 ml (b) 4 ml
must be added to 20 litres of the first solution to make a (c) 3 ml (d) 6 ml
solution of 10% sugar? [Based on MAT (May), 2007]

Chapter_15.indd 5 6/5/2015 2:38:16 AM


15.6 Chapter 15

36.
The ratio of milk to water in three containers of equal 43.
A mixture of 45 litres of spirit and water contains 20% of
capacity is 3:2, 7:3 and 11:4 respectively. The three water in it. How much water must be added to it make the
containers are mixed together. What is the ratio of milk to water 25% in the new mixture?
water after mixing? (a) 5 litres (b) 3 litres
(a) 38:8 (b) 21:9 (c) 4 litres (d) 6 litres
(c) 61:29 (d) 41:18 [Based on MAT (Dec), 2008]
[Based on MAT (May), 2010] 44.
A vessel contains liquid P and Q in the ratio 5:3. If 16
37.
There are 2 bottles containing a mixture of wine, water and litres of the mixture are removed and the same quantity
alcohol. The first bottle contains wine, water and alcohol of liquid Q is added, the ratio become 3:5. What quantity
in the ratio 3:5:2. The second bottle contains water and does the vessel hold?
wine in the ratio 5:4. 1 litre of the first and 2 litres of the (a) 35 litres (b) 45 litres
second are mixed together. What fraction of the mixture is (c) 40 litres (d) 50 litres
alcohol? [Based on MAT (Dec), 2007]
(a) 1/15 litres (b) 6/13 litres 45.
A bottle is full of dettol. One-third of it is taken out and
(c) 2/15 litres (d) 6/19 litres then an equal amount of water is poured into the bottle
[Based on MAT (May), 2010] to fill it. This operation is done four times. Find the final
38.
Two vessels contain spirit of 0.5 and 0.75 concentrations. ratio of dettol and water in the bottle.
If 2 litres from the first vessel and 3 litres from the second (a) 13:55 (b) 20:74
vessel are mixed, then what will be the ratio of the spirit (c) 16:65 (d) 10:48
and the water in the resultant solution? [Based on MAT (Dec), 2007]
(a) 15:17 (b) 7:17 46.
A person has a chemical of `25 per litre. In what ratio
(c) 13:7 (d) 17:15 should water be mixed in that chemical, so that after
[Based on MAT (Dec), 2009 (Feb), 2008] selling the mixture at `20 per litre he may get a profit of
39.
In a mixture of 60 litres, the ratio of milk and water is 2:1. 25%?
If the ratio of milk and water is to be 1:2, then the amount (a) 13:16 (b) 12:15
of water to be further added must be: (c) 16:9 (d) 19:22
(a) 40 litres (b) 30 litres [Based on MAT (Dec), 2007]

(c) 20 litres (d) 60 litres 47.


How many kg of sugar costing `5.75 per kg should be
[Based on MAT (Dec), 2009]
mixed with 75 kg of cheaper sugar costing `4.50 per kg so
that the mixture is worth `5.50 per kg?
40.
One test tube contains some acid and another test tube
(a) 350 kg (b) 300 kg
contains an equal quantity of water. To prepare a solution,
20 g of the acid is poured into the second test tube. Then, (c) 250 kg (d) 325 kg
two-thirds of the so-formed solution is poured from the [Based on MAT (May), 2007]
second tube into the first. If the fluid in the first test tube is 48.
5 litres of water is added to a certain quantity of pure milk
four times that in the second, what quantity of water was costing `3/litres. If by selling the mixture at the same
taken initially? price as before, a profit of 20% is made, then what is the
(a) 80 g (b) 60 g amount of pure milk in the mixture?
(a) 20 litres (b) 30 litres
(c) 40 g (d) None of these
[Based on MAT (May), 2009] (c) 25 litres (d) 35 litres
[Based on MAT (May), 2006]
41. A trader has 50 kg of pulses, part of which he sells at 8 per
cent profit and the rest at 18 per cent profit. He gains 14 49.
A sink contains exactly 12 litres of water. If water is
per cent on the whole. What is the quantity sold at 18 per drained from the sink until it holds exactly 6 litres of
cent profit? water less than the quantity drained away, how many litres
of water were drained away?
(a) 30 kg (b) 25 kg
(a) 2 litres (b) 6 litres
(c) 20 kg (d) 40 kg
(c) 3 litres (d) 9 litres
42. A bottle contains three-fourths of milk and the rest [Based on MAT (Feb), 2006]
water. How much of the mixture must be taken away and
50.
A chemist has 10 litres of a solution that is 10% nitric
replaced by an equal quantity of water so that the mixture
acid by volume. He wants to dilute the solution to 4%
has half milk and half water?
strength by adding water. How many litres of water must
1 be added?
(a) 25% (b) 33 %
3 (a) 15 (b) 20
(c) 45% (d) 50% (c) 18 (d) 25
[Based on MAT (Dec), 2008] [Based on MAT (Feb), 2006]

Chapter_15.indd 6 6/5/2015 2:29:10 AM


Alligation or Mixture 15.7

51.
Suppose, that a maximum of 25 g of salt dissolves in 100 g 59.
Two alloys contain zinc and copper in the ratio of 2:1 and
of water. Any more salt, if added, remains undissolved and 4:1. In what ratio the two alloys should be added together
a sediment falls at the bottom. Now water is evaporated to get as new alloy having zinc and copper in the ratio of
from 1 kg of a 4% solution at the rate of 28% g/h. After 3:1?
how long will it start sedimenting? (a) 7:5 (b) 5:7
(a) 35 hrs (approx) (b) 29 hrs (approx)
(c) 3:5 (d) None of these
(c) 31 hrs (approx) (d) 23 hrs (approx) [Based on IIFT, 2005]
[Based on MAT (Sept), 2008]
60.
In a glass of milk, the proportion of pure milk and water
52.
An alloy contains copper and zinc in the ratio 5:3 and
is 3:1, how much of the mixture must be withdrawn and
another alloy contains copper and tin in the ratio 8:5. If
substituted by water so that the resulting mixture may
equal weights of both the alloys are melted together, then
become half pure milk and half water?
the weight of tin in the resulting alloy per kg will be:
(a) 26/5 (b) 5/26 1 1
(a) unit (b) unit
(c) 7/31 (d) 31/7 4 3
[Based on MAT, 1997]
3 1
53.
Alok bought 25 kg of rice at the rate of `6 per kg and 35 (c) unit (d) unit
kg of rice at the rate of `7 per kg. He mixed the two and 4 2
[Based on ATMA, 2006]
sold the mixture at the rate of `6.75 per kg. What was his
gain or loss in the transaction? 61. A person has a chemical of `25 per litres. In what ratio
(a) `16 gain (b) `16 loss should water be mixed in that chemical so that after selling
(c) `20 gain (d) `10 gain the mixture at `20 per litres, he may get a profit of 25%?
[Based on MAT, 2000] (a) 12:15 (b) 16:9
54.
How much water must be added to 100 cc of 80% solution (c) 13:16 (d) 19:22
of boric acid to reduce it to a 50% solu­tion? [Based on MAT (Sept), 2011]
(a) 30 (b) 40 62. An oil refinery takes 100 lilters of crude oil as input and
(c) 50 (d) 60 after refining for 1 hr gives certain amount of output oil
[Based on MAT, 2000] X litres. This can be sold in the market at a profit of `30 per
55.
A bought 4 bottles of beer and B bought one bottle of lager. 1
litres. If this oil is further refined for h, it gives oil Y litres.
Lager per bottle costing twice that of the beer. C bought 2
nothing but paid `50 for his share of the drink which they This can be sold at a profit of `50 per litres. Output and
mixed together and shared equally. If C’s `50 covered his input ratio at both the stages is 90% The maximum amount
share, then what is the cost of the lager? that can be earned from 1000 lilters of crude input is:
(a) 50 (b) 75 (a) `40000 (b) `30000
(c) 30 (d) 46 (c) `27000 (d) `40500
[Based on MAT, 2000] [Based on MAT, 2011]
56.
An alloy of gold and silver weighs 50 g. It contains 80% 63. A and B are two alloys of gold and copper prepared by
gold. How much gold should be added to the alloy so that mixing metals in the ratio 7:2 and 7:11 respectively. If
percentage of gold is increased to 90? equal quantities of the alloys are melted to form a third
(a) 50 g (b) 60 g alloy C, the ratio of gold and copper in C will be:
(c) 30 g (d) 40 g (a) 7:5 (b) 5:9
[Based on SNAP, 2009]
(c) 5:7 (d) 9:5
57.
300 gm of salt solution has 40% salt in it. How much salt [Based on MAT, 2011]
should be added to make it 50% in the solution?
64. Two casks of 48 lilters and 42 lilters are filled with
(a) 40 gm (b) 60 gm
mixtures of wine and water, the proportions in the two
(c) 70 gm (d) 80 gm casks being respectively 13:7 and 18:17. If the contents of
[Based on SNAP, 2009] the two casks be mixed and 20 lilters of water is added to
58.
300 g of salt solution has 40% salt in it. How much salt the whole, what will be the proportion of wine to water in
should be added to make it 50% in the solution? the resultant solution?
(a) 40 g (b) 60 g (a) 21:31 (b) 12:13
(c) 70 g (d) 80 g (c) 13:12 (d) None of the above
[Based on SNAP, 2010] [Based on MAT, 2011]

Chapter_15.indd 7 6/5/2015 2:29:10 AM


15.8 Chapter 15

65. Three containers of capacity 20 litres, 5 liltres and 9 liltres 2 5 5 2


contain mixture of milk and water with milk concentrations (a) , (b) ,
7 7 7 7
90%, 80% and 70% respectively. The contents of three
containers are emptied into a large vessel. What is the 1 3
(c) 3 , 1 (d) ,
approximate ratio of milk to water in the resultant mixture? 4 4 4 4
(a) 3:1 (b) 4:1 [Based on MAT, 2013]
(c) 5:1 (d) 2:1 72. A merchant has six barrels with capacities of 15, 16, 18,
[Based on MAT, 2011] 19, 20 and 31 litres. One barrel contains wine and the other
66. The concentration of petrol in three different mixtures (of five contain oil. He keeps the barrel of wine for himself
petrol and kerosene) is 1/2, 3/5 and 4/5, respectively. If and sells the oil to two men in the ratio 1:2. The capacity
2 litres, 3 lilters and 1 lilters are taken from these three of the barrel containing the wine, in litres, is:
different vessels and mixed, what is the ratio of petrol and (a) 15 (b) 20
kerosene in the new mixture? (c) 19 (d) 18
(a) 4:5 (b) 3:2 [Based on MAT, 2013]
(c) 3:5 (d) 2:3 73. Two qualities of tea are mixed in the ratio 4:1 and the mixture
[Based on MAT, 2012] is sold at `72 per kg for a profit of 12.5% If the tea of the
second quality costs `3.25 more per kg then the tea of first
67. Vijay purchased two different kinds of alcohol. In the first
quality, what is the cost per kg of the tea of the first quality?
mixture, the ratio of alcohol to water is 3:4 and the second
mixture it is 5:6. If he mixes, the two given mixtures and (a) `63.35 (b) `23.65
makes a third mixture of 18 lilters in which the ratio of (c) `70.62 (d) `73.54
alcohol to water is 4:5, the quantity of the first mixture [Based on MAT, 2013]
(whose ratio is 3:4) that is required to make 18 lilters of 74. Three different containers contain different quantities of
the third kind of mixture is: a mixture of milk and water, whose measurements are
(a) 6 (b) 7 430 kg, 403 kg and 465 kg. What biggest measure must
(c) 8 (d) 9 be there to measure all the different quantities exactly?
[Based on MAT, 2012] (a) 70 kg (b) 51 kg
68. Two casks of 48 lilters and 42 lilters are filled with (c) 31 kg (d) 41 kg
mixtures of milk and water, the proportions in the two [Based on MAT, 2013]
casks being respectively, 13:7 and 18:17. If the contents
75. A jar full of milk contains 40% water. A part of this milk
of the two casks be mixed and 20 lilters of water be added
is replaced by another containing 19% water and now the
to the whole, what will be the proportion of milk and
percentage of water is found to be 26% The quantity of
water in the resulting mixture?
milk replaced is:
(a) 5:12 (b) 7:13
2 1
(c) 12:13 (d) 8:15 (a) (b)
3 3
[Based on MAT, 2012]
3 4
69. What amounts (in litres) of 90% and 97% pure acid solutions (c) (d)
are mixed to obtain 21 lilters of 95% pure acid solution? 7 7
[Based on MAT, 2013]
(a) 6 and 15 litres (b) 14 and 15 litres
76. Three containers A, B and C are having mixtures of milk
(c) 12 and 15 litres (d) 13 and 12 litres and water in the ratio 1:5, 3:5 and 5:7, respectively. If
[Based on MAT, 2012] the capacities of the containers are in the ratio 5:4:5, find
70. Two equal glasses are filled with mixture of alcohol and the ratio of milk to water, if the mixtures of all the three
water in the proportions of 2:1 and 1:1 respectively were containers are mixed together?
emptied into a third glass. What is the ratio of alcohol and (a) 53:115 (b) 23:123
water in the third glass?
(c) 11:4 (d) None of these
(a) 5:6 (b) 7:5
[Based on MAT, 2014]
(c) 4:2 (d) 8:1 77. What will be the ratio of petrol and kerosene in the final
[Based on MAT, 2012] solution formed by mixing petrol and kerosene that are present
in three vessels in the ratio 4:1, 5:2 and 6:1 respectively?
71. A litre of water weighs a kilogram and a litre of another
liquid weighs 1350 g. A mixture of the two weighs 1250 (a) 166:22 (b) 83:22
g/L. The volume of water and the liquid respectively, in a (c) 83:44 (d) None of these
litre of mixture is: [Based on MAT, 2014]

Chapter_15.indd 8 6/5/2015 2:29:13 AM


Alligation or Mixture 15.9

Difficulty Level-2
(Based on Memory)

1.
An empty container is filled with pure alcohol. The alcohol amount from B is transferred to C, from which again the
is slowly allowed to run out and when the container same amount is transferred to A. What is the proportion of
is one-fourth empty, it is replaced with water. Next, milk in container A at the end of the process?
when the container is half empty it is again filled with (a) 9/10 (b) 1/11
water. Finally, when it is three-fourths empty, it is again
(c) 1/121 (d) 2/13
filled with water. What percentage of container is alcohol
now? [Based on FMS (Delhi), 2002]

1 3 7. A beaker contains a 105 ml mixture of alcohol and water.


(a) 8 % (b) 11 % three-sevenths of the mixture is alcohol; the remainder
2 4
is water. An additional quantity of 105 ml of alcohol is
3 3
(c) 9 % (d) 14 % poured into the mixture. What is the ratio of the volume
8 8 of alcohol to water in the new mixture?
2.
Two solutions of milk and water are kept in two vessels A
(a) 3:7 (b) 3:4
and B. The ratio of quantity of milk in vessels A and B is
4:5 while the ratio of quantity of water in vessels A and B (c) 5:2 (d) 1:1
is 8:9. It is known that the concentration of milk in one of 8. Two vessels A and B contain mixtures of spirit and water.
these vessels in between 60% and 80% What could be the A mixture of 3 parts from A and 2 parts from B is found to
concentration of milk in the other vessel? contain 29% of spirit and a mixture of 1 part from A and 9
(a) 55% (b) 65% parts from B is found to contain 34 per cent of spirit. Find
(c) 75% (d) 85% the percentage of spirit in B and A.
(a) 35, 25 (b) 40, 20
3. Two vessels contain spirit of 0.5 and 0.75 concentrations.
(c) 25, 25 (d) 50, 50
If two litres from the first vessel and three litres from the
second vessel are mixed, then what will be the ratio of the 9. Three vessels whose capacities are as 5:3:2 are completely
spirit and the water in the resultant solution? filled with milk mixed with water. The ratio of milk to
(a) 13:7 (b) 7:17 water in the mixture of vessels are as 3:2, 2:1 and 3:1
respectively. Find the percentage of water in the new
(c) 15:17 (d) 17:15
mixture obtained when one-third of first, half of second
[Based on IIT Joint Man. Ent. Test, 2004] and two-thirds of the third vessel is taken out and mixed
together.
Directions (4 and 5): Refer to the data below and answer the
(a) 66.66% (b) 50%
questions that follow. Selling price of milk depends on the fat
content of milk. Normal fat content is 60% and normal selling (c) 16.66% (d) 33.33%
price is `20/litre. If the fat content falls below 55% then the 10. Two bottles A and B contain diluted sulphuric acid. In
selling price decreases by 20% A milkman procures milk at bottle A, the amount of water is double the amount of
`16/litre. acid while in bottle B, the amount of acid is 3 times that
4. How much water as per cent of pure milk can be added so of water. How much mixture should be taken from each
that selling price is not affected? bottle in order to prepare 5 litres of diluted sulphuric acid
containing equal amount of acid and water?
(a) 9% (b) 12.375%
(a) 1, 4 (b) 3, 2
(c) 10% (d) 15%
(c) 2, 3 (d) 4, 1
5. What per cent of the water should be added to pure milk 11. An alloy contains only zinc and copper. One such alloy
so that even if selling price decreases the profit percentage weighing 15 gm contains zinc and copper in the ratio of
is not affected? 2:3 by weight. If 10 gm of zinc is added then find what
(a) 10% (b) 30% amount of copper has to be removed from the alloy such
(c) 20% (d) 25% that the final alloy has zinc and copper in the ratio of 1:4
by weight?
6. There are 3 containers A, B and C which contain water,
(a) 5 gm (b) 5.5 gm
milk and acid respectively in equal quantities. 10% of the
content of A is taken out and poured into B. Then, the same (c) 6 gm (d) 4.8 gm

Chapter_15.indd 9 6/5/2015 2:29:13 AM


15.10 Chapter 15

12. Three beakers namely, A, B and C each contain 100 ml 18.


A housewife has 11 litre of solution that contains milk and
1 water in the ratio 3:1. She adds 250 ml of 3:2 solution of milk
of milk water solution. The ratio of milk to water in
m and water to it and then uses 250 ml of the combined mixture
the beakers A, B and C is 1:3, 1:4 and 2:3 respectively. to make curd How much of pure milk is she left with?
40 ml of solution is transferred from beaker A to beaker (a) 1,000 ml (b) 912.5 ml
C and than 28 ml of solution is transferred from beaker C (c) 750 ml (d) 720 ml
to beaker B. Find the final ratio of milk in the beakers A,
B and C. 19.
There are two alloys made up of copper and aluminium.
In the first alloy copper is half as much as aluminium and
(a) 3:6:8 (b) 6:15:20
in the second alloy copper is thrice as much as aluminium.
(c) 15:28:42 (d) None of these How many times the second alloy must be mixed with
first alloy to get the new alloy in which copper is twice as
13. Aqua regia is a mixture containing 50 per cent much as aluminium?
concentrated HCL and 70% concentrated HNO3 in the
(a) 2 (b) 3
ratio 1:3, respectively. Aqua regia is to be formed with
15 litres of HCL of 80% concentration and 56 litres of (c) 4 (d) 5
HNO3 of 90% concentration by adding water as a diluting 20.
A solution of sugar syrup has 15% sugar. Another solution
agent. Another mixture ‘X’ having 40% concentrated HCL has 5% sugar. How many litres of the second solution
and 30% concentrated H2SO4 in the ratio 5:7, respectively must be added to 20 litres of the first solution to make a
is added to the Aqua regia to form a solution of 111 litres. solution of 10% sugar?
Find the ratio of HCL to water in the solution. Only water
is used as a diluting agent. (a) 10 (b) 5
(c) 15 (d) 20
(a) 3:8 (b) 4:7
(c) 1:3 (d) 2:5 21. There are 3 containers A, B and C which contain water,
milk and acid respectively in equal quantities. 10% of the
14. Gold is 19 times as heavy as water and copper is 9 times content of A is taken out and poured into B. Then, the same
as heavy as water. In what ratio these two metals be mixed amount from B is transferred to C, from which again the
so that the alloy is 15 times as heavy as water is: same amount is transferred to A. What is the proportion of
milk in container A at the end of the process?
(a) 2:1 (b) 3:2
(a) 9/10 (b) 1/11
(c) 1:2 (d) 2:3 (c) 1/121 (d) 2/13
15. 25% of a solution containing 20% petrol, 50% diesel and 22.
If 50 per cent of the 2:3 solution of milk and water is
30% kerosene was replaced with kerosene. Now, two- replaced with water, then the concentration of the solution
thirds of the solution obtained in the previous step was is reduced by:
replaced with petrol. What is the percentage of diesel in
(a) 25% (b) 33.33%
this new solution?
(c) 50% (d) 75%
(a) 15% (b) 15.83%
(c) 12.5% (d) None of these 23.
Several litres of acid were drawn off a 54-litre vessel full
of acid and an equal amount of water was added. Again the
16. 2  l of Bournvita drink contains 236 calories and 2  l of same volume of the mixture was drawn off and replaced
complan drink contains 166 calories. If lilters mixture of by water. As a result, the vessel contained 24 litres of pure
both the drinks contains 88 calories, then the percentage acid. How much of the acid was drawn off initially?
of Bournvita drink in the mixture is: (a) 12 litres
(a) 25% (b) 16.66% (b) 16 litres
(c) 14.28% (d) 83.33% (c) 18 litres
17.
A vessel of capacity 2 litre has 25% alcohol and another (d) 24 litres
vessel of capacity 6 litre has 40% alcohol. The total liquid 24. A dairyman pays `8.0 per litre of milk. He adds water
of 8 litre is poured out in a vessel of capacity 10 litre and and sells the mixture at `9 per litre, thereby making 40%
the rest part of the vessel is filled with water. What is the profit. The proportion of water to milk received by the
new concentration of mixture? customers is:
(a) 31% (b) 71% (a) 45:11 (b) 11:45
(c) 49% (d) 29% (c) 2:45 (d) 45:2

Chapter_15.indd 10 6/5/2015 2:29:13 AM


Alligation or Mixture 15.11

25.
How many kg of tea worth `25 per kg must be blended with first glass as 3:4, in the second glass as 4:5 and in the third
30 kg of tea worth `30 per kg so that by selling the blended glass as 5:6. The contents of the three glasses are emptied
variety at `30 per kg there should be a gain of 10%? into a single vessel. What is the ratio of the spirit to water
(a) 36 kg (b) 40 kg in the mixture now?
(c) 32 kg (d) 42 kg (a) 920:1159 (b) 820:1149
[Based on SNAP, 2009] (c) 1120:1134 (d) 1010:1122
26. A person travels 285 km in 6 hrs in two stages. In the first [Based on IIFT, 2005]
part of the journey, he travels by bus at the speed of 40 33.
Ashok a master adulterator cum grosser sells haldi powder
Km/h. In the second part of the journey, he travels by train (turmeric powder), which contains five per cent saw dust.
at the speed of 55 Km/h. How much distance did he travel What quantity of pure haldi should be added to two kilos
by train? of haldi (containing five per cent saw dust) so that the
(a) 205 km (b) 145 km proportion of saw dust becomes four per cent?
(c) 165 km (d) 185 km (a) 1 kg (b) 2 kg
27. A person has a chemical of `25 per litre. In what ratio (c) 0.5 kg (d) None of these
should water be mixed in that chemical so that after [Based on IIFT, 2007]
selling the mixture at `20 per litre he may get a profit of
25 per cent? 34.
A manufacturer has 200 litres of acid solution which has
15% acid content. How many litres of solution with 30%
(a) 13:16 (b) 16:9 acid content may be added so that acid content in the
(c) 12:15 (d) 19:22 resulting mixture will be more than 20% but less than 25% ?
28.
A mixture contains milk and water in the ratio 5:1 or (a) More than 100 litres but less than 300 litres
adding 5 litres of water, the ratio of milk to water becomes (b) More than 120 litres but less than 400 litres
5:2. The quantity of milk in the original mixture is:
(c) More than 100 litres but less than 400 litres
(a) 16 litres (b) 25 litres
(d) More than 120 litres but less than 300 litres
(c) 22.75 litres (d) 32.5 litres
[Based on XAT, 2010]
[Based on FMS (MS), 2006]
35. There are two vessels of equal capacity, one full of milk,
29.
An alloy contains zinc and copper in the ratio 5:8 and and the second one-third full of water. The second vessel
another alloy contains zinc and copper in the ratio is, then filled up out of the first, the contents of the second
5:3. If equal amounts of both the alloys are melted are then poured back into the first till it is full and then
together, then the ratio of zinc and copper in the again the contents of the first are poured back into the
resulting alloy is: second till it is full. What is the proportion of milk in the
(a) 25:24 (b) 3:8 second vessel, if capacity of the vessel is 20 litres?
(c) 103:105 (d) 105:103 37 20
(a) (b)
[Based on FMS, 2006] 20 27
30.
Two vessels A and B contain milk and water mixed in the 20 27
ratio 5:3 and 2:3. When these mixtures are mixed to form (c) (d)
37 20
a new mixture containing half milk and half water, they
[Based on NMAT, 2005]
must be taken in the ratio:
36. Two barrels contain a mixture of ethanol and gasoline.
(a) 2:5 (b) 3:5 The content of ethanol is 60% in the first barrel and 30%
(c) 4:5 (d) 7:3 in the second barrel. In what ratio must the mixtures from
[Based on FMS, 2006] the first and the second barrels be taken to form a mixture
containing 50% ethanol?
31. The average weekly salary per head of all employees (a) 1:2 (b) 2:1
(supervisors and labourers) is `100. The average weekly (c) 2:3 (d) 3:2
salary per head of all the supervisors is `600 while the
[Based on JMET, 2006]
average weekly salary per head of all the labourers is `75.
Find the number of supervisors in the factory if there are 37. A milkman mixes 20 lilters of water with 80 lilters milk.
840 labourers in it. After selling one-fourth of this mixture, he adds water to
replenish the quantity that he has sold. What is the current
(a) 46 (b) 42
proportion of water to milk?
(c) 44 (d) 48 (a) 2:3 (b) 1:2
32.
Three equal glasses are filled with mixtures of spirit and (b) 1:3 (c) 3:4
water. The ratio of the spirit to water is as follows: in the [Based on CAT, 2004, 2010]

Chapter_15.indd 11 6/5/2015 2:29:14 AM


15.12 Chapter 15

38. Auto fare in Bombay is `2.40 for the first 1 km, `2.00 per km 42. There are two containers: the first contains 500 mL of
for the next 4 km and 1.20 for each additional km thereafter. alcohol, while the second contains 500 mL of water. Three
Find the fare in rupees for k km (k ≥ 5). cups of alcohol from the first container is taken out and is
(a) 2.4k + 1.2(2k- 3) mixed well in the second container. Then, three cups of
(b) 10.4 + 1.2(k- 5) this mixture is taken out and is mixed in the first container.
Let, A denote the proportion of water in the first container
(c) 2.4 + 2(k- 3) + 1.2(k- 5)
and B denote the proportion of alcohol in the second
(a) 10.4 + 1.2(k- 4) container. Then,
[Based on CAT, 2011]
(a) A > B (b) A < B
39. Half of the volume of milk and water mixture of ratio 7:5
(c) A = B (d) Cannot be determined
is converted into a mixture of ratio 3:1 by the substitution
[Based on CAT, 1998]
(or replacement) method. The mixture of ratio 7:5 was
formed from the mixture 7:3 by adding the water in it. 43. Two liquids A and B are in the ratio 5:1 in container 1 and
If 240  litres milk is required in the replacement method. 1:3 in container 2. In what ratio should the contents of the
What is the total amount of water added to prepare the two containers be mixed so as to obtain a mixture of A and
mixture in the ratio 7:5? B in the ratio 1:1?
(a) 100 litres (b) 400 litres (a) 2:3 (b) 4:3
(c) 50 litres (d) 200 litres (c) 3:2 (d) 3:4
[Based on CAT, 2013] [Based on CAT, 1996]
40. What is the minimum amount of sucrose (to the nearest 44. A man buys spirit at `60 per litre, adds water to it and then
gram) that must be added to one gram of saccharin to sells it at `75 per litre. What is the ratio of spirit to water
make mixture that will be at least 100 times as sweet as if his profit in the deal is 37.5%?
glucose? (a) 9:1 (b) 10:1
(a) 7 (b) 8 (c) 11:1 (d) None of these
(c) 9 (d) 100 [Based on CAT, 1994]
[Based on CAT, 1999]
45. Gopal has 73 lilters of wine a drum. He replaces 3.65
41. Approximately how many times sweeter than sucrose is a lilters of it with water and keeps doing so till the time
mixture consisting of glucose, sucrose and fructose in the the concentration of wine is less than 85% The minimum
ratio of 1:2:3? number of operations that Gopal has to perform is:
(a) 1.3 (b) 1.0 (a) 3 (b) 4
(c) 0.6 (d) 2.3 (c) 2 (d) None of these
[Based on CAT, 1999] [Based on MAT, 2013]

Answer Keys
Difficulty Level-1

1. (c) 2. (a) 3. (b) 4. (d ) 5. (b) 6. (b) 7. (c) 8. (c) 9. (d ) 10. (b) 11. (d ) 12. (d ) 13. (d )
14. (c) 15. (c) 16. (d ) 17. (d ) 18. (d ) 19. (d ) 20. (d ) 21. (b) 22. (d ) 23. (b) 24. (d ) 25. (a) 26. (d )
27. (d ) 28. (a) 29. (d ) 30. (c) 31. (a) 32. (a) 33. (c) 34. (d ) 35. (d ) 36. (c) 37. (a) 38. (c) 39. (d )
40. (d ) 41. (a) 42. (b) 43. (b) 44. (c) 45. (c) 46. (c) 47. (b) 48. (c) 49. (d ) 50. (a) 51. (b) 52. (b)
53. (d ) 54. (d ) 55. (a) 56. (a) 57. (b) 58. (b) 59. (c) 60. (d ) 61. (b) 62. (d ) 63. (a) 64. (b) 65. (c)
66. (b) 67. (b) 68. (c) 69. (a) 70. (b) 71. (a) 72. (b) 73. (a) 74. (c) 75. (a) 76. (a) 77. (b)

Difficulty Level-2

1. (c) 2. (b, c) 3. (a) 4. (a) 5. (d ) 6. (c) 7. (c) 8. (a) 9. (d ) 10. (b) 11. (a) 12. (a) 13. (c)
14. (b) 15. (c) 16. (c) 17. (d ) 18. (d ) 19. (c) 20. (d ) 21. (c) 22. (b) 23. (c) 24. (b) 25. (a) 26. (c)
27. (b) 28. (b) 29. (d ) 30. (c) 31. (b) 32. (a) 33. (c) 34. (c) 35. (b) 36. (b) 37. (a) 38. (b) 39. (d )
40. (c) 41. (a) 42. (c) 43. (d ) 44. (b) 45. (b)­­­­­

Chapter_15.indd 12 6/5/2015 2:29:14 AM


Alligation or Mixture 15.13

Explanatory Answers

Difficulty Level-1

1. (c) Suppose the quantities of Tea worth `126 per kg, Profit % = 37.5
`135 per kg and `x per kg purchased are y, y and S.P. × 100
2y kg respectively. \ C.P. =
100 + Gain%
126 y + 135 y + 2 xy (8 x + 8 y) × 100
\ = 153 ⇒ 6.4x =
4y 100 + 37.5

⇒ 261 + 2x = 612
⇒ 880x = 800x + 800y
351 ⇒ 80x = 800y ⇒ x = 10y
⇒ x = = 175.50
2
x y
⇒ =

\  Tea of the third variety is purchased @ `175.50 10 1
per kg.
\ Required ratio = 1:10.
1
2. (a) Since x + y = , he mixes milk and water in the ratio 6.
(b) Let, `X, `Y be the cost price per kg of variety A and
2
2:1. veriety B of rice
Let, us say, in 100 liltres milk, he mixes 50 liltres By data 10X + 15Y = 25 (1.4) X.
water. Since sale price of the mixture is 40% more than the
C.P. of milk = 10x per 100 l cost price of A.
x Therefore 10X + 15Y = 35X
C.P. of water = x per 100 l, i.e., per 50 l ⇒ 15Y = 25X
2
S.P. of milk = (1.2) × 15x = 18x Hence, X:Y = 15:25 = 3:5
7.
(c) Suppose x litres of acid was drawn off initially.
(18 x  10.5 x)
So, profit percentage = × 100 \ Remaining acid in the 54-litre vessel full of acid.
10.5 x
   = (54 – x) litres and water added = x litres.
= 72% (approximately)
Now out of the 54 litres of mixture of acid and water,
x litres of mixture is drawn off.
3.
(b) Let, M = 3K, W = 2K
 54 − x 
\ 3K + 2K = 45 ⇒ K = 9 ⇒ Quantity of acid drawn off =  × x  litres
 54 
⇒ Milk = 27 litres and Water = 18 litres x2
and quantity of water drawn off = litres.
Now suppose x litres of water is added to the mixture 54
such that   54 − x  
Now the vessel contains 54 − x −   x  litres
27 9   54  
= ⇒ 162 + 9x = 297
of acid.
18 + x 11
⇒ 9x = 135 ⇒ x = 15.  54 − x 
\ 54 − x −   x = 24
 54 
4.
(d) In both jars concentration of milk is more than 50%
Therefore, in jar three concentration of milk cannot be ⇒ x2 – 108x + 1620 = 0
50% Hence, we cannot decide the volumes. ⇒ x = 90, 18.
Since 90 > 54, therefore x = 90 is ruled out.
5. (b) Suppose the quantity of milk purchased = x litres
Hence x =18.
Suppose quantity of water mixed = y litres
8. (c) Milk : Water
\ Required ratio of the water and the milk in the 20 30
mixture = y:x
–10 –15
C.P. of x litres of milk = `6.4 x +25
S.P. of x litres of milk = `8 (x + y) 10 40

Chapter_15.indd 13 6/5/2015 2:29:15 AM


15.14 Chapter 15

Therefore, the concentration of the solution is reduced


from 20 to 10, i.e., reduced by 50% The dilution of
the solution increased from 30 to 40, i.e., increased
1
by 33 %
3

9.
(d) Total milk = 20 litres

Water Milk Total
1 litre 19 litres 20 By alligation rule:

1 19 + x 20 + x (Quantity of 1st kind of wheat) 33 11


= =
(Quantity of 3rd kind of wheat) 21 7
Now 1 = 2% of (20 + x), where x is the quantity of
i.e., they must be mixed in the ratio 11:7
pure milk added to the mixture.
⇒ x = 30 Step II: Mix wheat of 1st kind and 2nd kind to obtain
a mixture worth of `1.41 per kg.
where x = quantity of pure milk added to the mixture.

10.
(b) Quantity of milk in the mixture = 48 litres.
Quantity of water in the mixture = 32 litres
Let, x litres of water be added in the mixture so that

48 2
= ⇒ 2x + 64
    32 + x 3
      = 144 ⇒ x = 40. \ By alligation rule:
(Quantity of 1st kind of wheat) 3 1
11.
(d) Total quantity = 300 ml = =
(Quantity of 2nd kind of wheat) 21 7
Sugar by weight = 120 ml
i.e., they must be mixed in the ratio 1:7
Water = 180 ml
Let, x be the amount of water added to the solution. (Quantity of 2nd kind of wheat)
Thus,
(Quantity of 3rd kind of wheat)
120 30
Then,  =
300  x 100 (Quantity of 2nd kind of wheat)
   =
(Quantity of 3rd kind of wheat)
⇒ 1200 = 900 + 3x ⇒ 300 = 3x ⇒ x = 100 ml.
(Quantity of 1st kind of wheat)

12.
(d) Given that all vessels contain equal amount of mixture (Quantity of 3rd kind of wheat)
say V.
So in the first vessel → water:milk = 1/7:6/7  7 11 11
   =    = .
1 7  1
In the second vessel → 2/7:5/7
\ Quantities of wheat of (1st kind:2nd kind:3rd kind)
In the third vessel → 1/4:3/4.
 7
(6 / 7)  (5 / 7)  (3 / 4)    = 1: 7 :  = (11:77:7).
Hence, the final ratio is =  11
(1 / 7)  (2 / 7)  (1 / 4)
14.
(c) Kerosene in 100 kg = 7/32 × 100 = 21.875 kg
65
   = Petrol in 100 kg = 25/32 × 100 = 78.125 kg
19
Now 78.125 is 25 parts out of (25 + 9) = 34 parts
13.
(d) Step 1: Mixture of wheats of first and third kind to get Hence 9 parts = 9/25 × 78.125 = 28.125
a mixture worth `1.41 per kg? Amount to be added = 28.125 – 21.875 = 6.25 kg

Chapter_15.indd 14 6/5/2015 2:38:17 AM


Alligation or Mixture 15.15

15. (c) In the new mixture, water:alcohol = 3:2 = 12:8 21.


(b) Quantity of milk in the mixture = 48 litres
⇒  The capacity of each container = (12 + 8)/2 Quantity of water in the mixture = 32 litres
= 10 units Let, x litres of water be added in the mixture so that
⇒  The ratios of water to alcohol of the containers are
4:6 and 8:2 = 2:3 and 4:1 48 2
= ⇒ 2x + 64 = 144 ⇒ x = 40
32  x 3
16. (d) 5 litres of 20% of solution = 1 litre, i.e., the amount of
22.
(d) Total milk = 20 litres
1
alcohol. Now new strength of alcohol will be i.e., Water Milk Total
6
16.66% 1 litre 19 litres 20
1 19 + x 20 + x
30  x 6 3
17.
(d)   ⇒ 5x = 135 or x = 27 so, required    Now 1 = 2% of (20 + x), where x is the quantity of
x  25 4 2
pure milk added to the mixture.
percentage of milk = 27%
⇒ x = 30
18.
(d) % of liquid B in the original mixture. where, x = quantity of pure milk added to the
mixture.
1
= × 100 = 20% 23. (b) Suppose the quantity of milk purchased = x litres
5
Suppose quantity of water mixed = y litres
In the final mixture % of liquid B
\ Required ratio of the water and the milk in the
3 mixture = y:x
= × 100 = 60%
5 C.P. of x litres of milk = `6.4 x
Now using the rule of alligation
S.P. of x litres of milk = `8 (x + y)
Profit % = 37.5
S.P.  100
\ C.P. =
100  Gain%
(8 x  8 y )  100
⇒ 6.4 =
100  37.5

Hence reduced quantity of the first mixture and the ⇒ 880x = 800x + 800y
quantity of mixture B which is to be added are the same. ⇒ 80x = 800y ⇒ x = 10y
\ Total mxiture = 10 + 10 = 20 liters and quantity of x y
20 ⇒ =
liquid A = × 4 = 16 litres. 10 1
5 \ Required ratio = 1:10
19.
(d)
24. (d) Milk = 40 litres
Water = 20 litres
To make the ratio M:W = 1:2, 60 litres of water
should be added.
25.
(a)


i.e.,  2:1
Hence x litres must be 12 litres, i.e., (6 × 2).
20. (d) In 100 cc solution, quantity of boric acid = 80 cc and
quantity of water = 20 cc. Suppose x litres of water be
added to make the solution 50%
\ 50% (100 + x) = 80 ⇒ x = 60

Chapter_15.indd 15 6/5/2015 2:29:16 AM


15.16 Chapter 15

We get two values of x, 7 and 13. But to get a 31.


(a) Total quantity of mixture = 2 + 3 + 0.5 = 5.5 gallon
viable answer, we must keep in mind that the central Total quantity of water
value (10) must lie between x and 12. Thus, the value
of x should be 7 and not 13. 12 7
= 2 × +3× + 0.5
Required % increase = 7% 100 100

26.
(d) \ Ratio of quantities sold at 10 % profit and 5% loss = 0.95 gallon
= 12:3 = 4:1 \ Required percentage

0.95
= × 100
5.5

190 3
= = 17 %
11 11

32.
(a) Let, the quantity of milk replaced be x.
40 19 26
Then, (1 − x) + ×x = ×1
100 100 100

⇒ 40 – 40x + 19x = 26
\ The quantity sold at 10% profit ⇒ 21x = 14
50 2
= × 4 = 40 kg ⇒ x =
4+1 3
and, the quantity sold at 5% loss
33.
(c) Quantity of salt in 6 litres of solution
= 50 – 40 = 10 kg
5
= × 6 = 0.3 litres
27.
(d) Here 10% of 150 kg = 15 kg 100
\ Good quality of wheat is 135.
Percentage of salt in 5 litres of solution
To becomes the 5% low quality of wheat, we add 0.3
150 kg of more wheat. = × 100 = 6%
5
25 30 270
28.
(a) Milk in mixture = 6 × + 4× = 34.
(d) Given that nectar contains 70% water.
100 100 100
270 × 10 And honey obtained contains 17% water.
For 10 parts = = 27%
100 It means that out of 1 kg honey 17% is water, i.e., pure
honey = 0.83 kg
29.
(d) Let, x litres of second solution must be added.
Which is 20% of the flower nectar
Then,
[Q 70% is water].
15 × 20 + 5 × x
= 10 Now, let quantity of flower nectar processed be x kg.
20 + x

⇒ 300 + 5x = 200 + 10x Then,
100 0.3 × x = 0.83
⇒ x = = 20 litres
5 0.83
\ x = = 2.77 kg
30 (c) Go through options, options (a) and (b) are ruled out 0.3
as released amount cannot be either 7 or 8.
35.
(d) Required quantity of water
For option (c),
Oxygen = 1.28 and Nitrogen = 6.72 9 × (50 – 30) 9 × 20
= = = 6 ml
After first released oxygen 30 30
= 1.28 – 0.32 = 0.96 litres
36.
(c) LCM of (3 + 2), (7 + 3) and (11 + 4) is 30.
After second released oxygen
= 0.96 – 0.24 = 0.72 litres Let, the capacity of each container be 30 litres.
which is 9% of 8 litres. \ Quantity of milk after mixing

Chapter_15.indd 16 6/5/2015 2:29:17 AM


Alligation or Mixture 15.17

41.
(a) \ Ratio of quantities sold at 8% profit and 18% profit
 3 7 11 
=  + +  × 30 = 4:6 = 2:3
 5 10 15 
= 61 litres
Quantity of water after mixing

2 3 4
=  + +  × 30
 5 10 15 
= 29 litres
\ Required ratio = 61: 29

37.
(a) Quantity of alcohol in 1 litres mixture of first bottle

2 1 Therefore, the quantity sold at 18% profit


= × 1 = litres
10 5
50
As second bottle does not contains alcohol. = × 3 = 30 kg
2+3
1 1 1
So, required fraction = × = litres 3
3 5 15 42. (b) Milk = × 100 = 75
4
38.
(c) Spirit Water Ratio of milk and water = 3:1
Vessel 1 0.5 : 0.5 1
Water = × 100 = 25
1 : 1 4
Vessel 2 0.75 : 0.25 Let, x litres of mixture is taken away, then quantity of
3 : 1  3x 
milk left =  3 − 
 4 
1 3
×2+ ×3
Spirit 13
= 2 4 =  x
and water left = 1 −  + x
Water 1 × 2 + 1
×3 7  4
2 4

2 3x x
39.
(d) Quantity of milk = × 60 = 40 litres Given, 3 − = 1− + x
3 4 4

1 3x x
Quantity of water = × 60 = 20 litres ⇒ 3 – 1 = − +x
3 4 4

Quantity of water added = x litres (suppose) 6x


⇒ 2 =
4
40 1
Now, =
20 + x 2 4
⇒ x =
3
⇒ x = 60 litres
4
40.
(d) Initially, let xg of water was taken. 1
\ Required percentage = 3 × 100 =
33 %

According to the problem: 4 3
2 1
(x – 20) + (x + 20) = 4 × (x + 20) 43. (b) Quantity of water in the mixture
3 3
2 20
⇒ x – 20 = (x + 20) = × 45 =
9L
3 100
⇒ 3x – 60 = 2x + 40 Let, x litres of water must be added to make the water
⇒ x = 100 g 25% in the solution.

Chapter_15.indd 17 6/5/2015 2:29:18 AM


15.18 Chapter 15

9+ x 47.
(b) Using Alligation Method,
\ × 100 = 25
45 + x
⇒ 36 + 4x = 45 + x
⇒ 3x = 9
⇒ x = 3 litres

i.e., 4:1
44.
(c) Let, the quantity of liquid P and Q be 5x and 3x litres
respectively.
Hence, the required quantity of Sugar I
75
5 = × 4 = 300 kg
Quantity of P removed = × 16 = 10 litres 1
5+3
48.
(c) Let, the quantity of pure milk be x litres.
3 If 5 litres of water is added to it, then cost of (5 + x)
Quantity of Q removed = × 16 = 6 litres
5+3 litres = `(3x + 5)
\ Profit = `15
5 x − 10 3
Now, = Given, 20% of 3x =15
3 x − 6 + 16 5
3x
⇒ 25x – 50 = 9x + 30 ⇒ = 15
5
⇒ 16x = 80 ⇒ x = 25 litres
⇒ x = 5 \ The amount of pure milk in the mixture was
25 litres.
\ Quantity that vessel hold = 8 × 5 = 40 litres.
49. (d) Water (W) + Drained (D) = 12
45.
(c) Amount of dettol after
⇒ W + D = 12
2
First operation = Now, W = D – 6
3
D – 6 + D = 12
2 2 4
Second operation = × = ⇒ D = 9 litres
3 3 9
1
4 2 8 50.
(a) Quantity of nitric acid = 10 × = 1 litre
hird operation = × =
T 10
9 3 27 Water =10 – 1 = 9 litres
8 2 16 Let, x litres of water be added.
ourth operation =
F × =
27 3 81 4
Then,  (10 + x) × = 1 ⇒ x = 15 litres
Amount of water after fourth operation 100
16 65 51. (b) Given, that sedimenting starts when the ratio of salt to
= 1− = water is 1:4.
81 81
Now, 1000 g solution has 40 g salt and 960 g water.
16 65
\ Required ratio = : = 16 : 65 Process of sedimenting will start when the ratio of salt
81 81 to water is 1:4 i.e., 40:160.
46. (c) Selling price of mixture = `20 Hence, 800 g of water has to be evaporated.
100 Now, time required to evaporate 800 g of water
Cost price of mixture = × 20 = `16 800
125 = = 28.57 hrs ≈ 29 hrs
28
By the rule of alligation
5
52. (b) of the tin is in 1 kg of alloy. Since quantity of tin is
13
constant (because the other alloy does not contain tin),
5
therefore, there is of the tin in 2 kg of alloy.
13

5
So, required ratio = 16:9 Hence, there is of tin in 1 kg of alloy.
26

Chapter_15.indd 18 6/5/2015 2:29:18 AM


Alligation or Mixture I 15.19

53.
(d) CP of 25 kg of rice @ `6 = `25 × 6 = `150 \ 240 + 2x = 300 + x
CP of 35 kg of rice @ `7 = `35 × 7 = `245 x = 60 g
\ CP of 25 + 35, i.e., 60 kg of rice Hence, option (b).

= `(150 + 245) = `395 59. (c) 1st 2nd

SP of 1 kg of rice = `6.75 2/3~ /4/5


\ SP of 60 kg of rice = `6.75 × 60 = `405 314
Hence, total gain = `405 – `395 = `10. 1120/ ~1/12
54.
(d) Concentration of basic acid = 80% = 80 cc 1 1
\ :
Ratio = = 3:5
Quantity of water = 20 cc 20 12
60.
(d) Milk and water ratio = 3:1
Let, x cc of water be added to get the concentration of \ Total = 4
50%
\ We take half of pure milk and half of water, then
80 50
= 2 1
100 + x 100 total remainder = = unit.
4 2
80 1 61. (b) SP of mixture = `20 per L
⇒ =
100 + x 2 Profit = 25%

⇒ x = 60 cc 100
∴ CP of mixture = × 20 = `16
25
55.
(a) Let, Cost of beer = `x/ bottle
By the rule of allegation,
\ Cost of lager = `2x/ bottle
Chemical Water
\ A spent `4x and B spent `2x C paid `50 25 0
\ Cost of lager = `50 per bottle 16
16 9
Cost of beer = `12.50 per bottle
the ratio of chemical and water = 16:9.
80 62. (d) We have 1000 lilters of crude oil as input.
56. (a) Content of gold in alloy = 50 × = 40 g
100 After refining for 1 hr, we get
Let, x g of gold should be added to alloy and solve the Output = 90% of 1000 = 900 litres
equation for ‘x’.
Profit = `900 × 30 = 27000
40  x 1
 100 = 90 Further refining for hrs, we get
 x 50 2
57.
(b) Write now the amount of salt in the solutions Output = 90% of 900 = 8100 litres
Profit = `8100 × 50 = `40500
90
= 300  = 12 g Hence, maximum profit = `40500
100
7 7
63. (a) Quantity of gold in A = =
Let, x g salt is added to the solution 7+2 9
120  x 7 7
\ × 100 = 50 Quantity of gold in B = =
300  x 7 + 11 18
Let, quantity of gold in C = x
⇒ x = 60
According to the question,
58.
(b) 300 g solution has 40% salt in it. 7 7 7 7 21
x− = − x ⇒ 2x = + =
So, the amount of salt in it 18 9 9 18 18
7
40  300 ⇒ x=
=  120 g 12
100
Hence, ratio of gold and copper in C = 7:5.
For the salt percentage to be 50%,
64. (b) In first cask,
120  x 1 13
= Quantity of wine = × 48 = 31.2 litres
300  x 20
2

Chapter_15.indd 19 6/9/2015 2:52:59 PM


15.20 Chapter 15

7 3 5 4
Quantity of water = × 48 = 16.8 litres Then, × + (18 − X ) = (18)
20 7 11 9
In second cask, 3 X 90 5 X
⇒ + − =8
18 7 11 11
Quantity of wine = × 42 = 21.6 litres 5 X 3 X 90
35 ⇒ − = −8
11 7 11
17 2X 2
Quantity of water = × 42 = 20.4 litres ⇒ =
35 77 11
After mixing, ⇒ X =7L

Quantity of wine = 31.2 + 21.6 = 52.8 litres
68. (c) In the mixture of two casks,
Quantity of water = 16.8 + 20.4 + 20 = 57.2 litres
Quantity of milk
52.8 132 12 13 18
∴  Required ratio = = = = 12:13. = × 48 + × 42
57.2 143 13 20 35
65. (c) In first container, 156 108 264
= + = litres
Milk = 90% of 20 = 18 litres 5 5 5
7 17
∴  Water = 20 – 18 = 2 litres Quantity of water = × 48 + × 42
20 35
In second container,
84 102 186
Milk = 80% of 5 = 4 litres = + = litres
5 5 5
Water = 5 – 4 = 1 litres When 20 L of water is added to the miixture, then
In third container, 186
quantity of water = + 20
Milk = 70% of 9 = 6.3 litres 5
Water = 9 – 6.3 = 2.7 litres 186 + 100 286
= = litres
After mixing, 5 5
Total milk = 18 + 4 + 6.3 = 28.3 litres 264 286
Thus, required ratio = :
Total water = 2 + 1 + 2.7 = 5.7 litres 5 5
= 12:13.
∴  Required ratio = 28.3:5.7  5:1
66. (b) Quantity of petrol in the mixture 69. (a) Acid Solution Ι Acid Solution ΙΙ
90 97
1 3 4
= × 2 + × 3 + ×1 95
2 5 5
9 4 13 2 5
=1+ + =1+ ∴  Amounts of the solutions I and II are in the ratio
5 5 5
2:1.
18
= ∴  Amounts of the solutions I and II are
5
2 5
and quantity of kerosene in the mixture = 21 = 6 litres and × 21 = 15 litres, respectively.
7 7
1 2 1 70. (b) Let, both glasses have ‘x’ litres of mixture.
= × 2 + × 3 + ×1
2 5 5 2
6 1 7 In first glass, alcohol = x
=1+ + =1+ 3
5 5 5 1
12 Water = x
= 3
5 1
In second glass, alcohol = x
2
Thus, ratio of petrol and kerosene
1
Water = x
18 12 2
= =: 3: 2.
5 5 2 1
x+ x
Required ratio in the third glass = 3 2 = 7 = 7 : 5.
67. (b) Let, quantities of first and second kinds of alcohols be 1 1
x+ x 5
X L and (18 − X) L, respectively. 3 2

Chapter_15.indd 20 6/5/2015 2:29:27 AM


Alligation or Mixture 15.21

71. (a) 75. (a) Let, amount of solution be 100 ml


1000 1350    Amount of water = 40 ml
∴  Amount of milk = 60 ml
1250 Let, amount of solution replaced be x ml.
New solution that is added has 81% milk and 19%
100 250 water.
∴ Amount of water in new solution obtained = 26%
100 2 = 26 ml
Required ratio = =
250 5 40 x 19 x
∴ 40 − + = 26
100 100
2 2
∴  Volume of water = = 21x
2+5 7 ⇒ 40 − 26 =
100
5 5 21x
 Volume of liquid = = . ⇒ 14 =
2+5 7 100
14 2
72. (b) By using options, ⇒ x = × 100 = × 100
21 3
Let, the barrel which contain wine is 20 lilters barrel.
2
Then, he has to divide the remaining in ratio of 1:2 × 100
2
∴ Quantity of milk replaced = 3 = .
This can be divided as follows 100 3
15 + 18:19 + 16 + 31:66 = 1:2 76. (a) If we assume that quantity of mixture in all three
So, the barrel containing wine is 20 lilters barrel. containers are 5, 4 and 5, then total quantity of milk

73. (a) Let, the price/kg of tea of 1st kind be `x. 1 3 5


= ×5+ ×4 + ×5
Then, price/kg of tea of 2nd kind = `(x+325) 6 8 12
5 3 25
Now, quantity of two tea be 4a and a, respectively. = + +
6 2 12
We are given,
10 + 18 + 25 53
= = litres
112.5 12 12
[4a × x + a × ( x + 3.25)] × = (4a + a )72 53 115
100 and quantiity of water = (5 + 4 + 5) − = litres
5a × 72 × 100 12 12
⇒ 49 x + a × ( x + 3.25) = 53 115
112.5 ∴ Required ratio = : = 53:115.
5 × 72 × 100 12 12
⇒ 4a + x + 3.25 =
112.5
77. (b) Solution in first vessel = 4:1 = 5 × 7
⇒ 5 x + 3.25 = 320
320 − 3.25 316.75 Solution in second vessel = 5:2 = 7 × 5
⇒x= = = 63.35
5 5 Solution in third vessel = 6:1 = 7 × 5
∴ Price per kg of tea of 1st kind = ` 63.35. Now, ratio in first, second and third vessel are
respectively 28:7, 25:10 and 30:5.
74. (c) Quantity of milk to be measured is 403 kg, 434 kg and
465 kg. ∴  Required ratio 28:7
∴ Biggest measure to measure all the quantities 25:10

exactly 30:5
= HCF of 403, 434, 465 = 31 kg. _____
83: 22

Chapter_15.indd 21 6/9/2015 2:43:50 PM


15.22 Chapter 15

Difficulty Level-2

1. (c) Ratio of quantity of alcohol left to total quantity


3 1 1
1 4 2 4 3
=    
1 1 1 1 32
3 3
Alcohol percentage =  100  9 %
32 8

2. (b, c) Milk and water in A = 4x and 8y
1
Milk and water in B = 5x and 9y i.e., 11:1. Water as % of milk =  100 
9%
11
   If concentration of milk in A is between 60% 5. (d) Current selling price = `20
and 80%, then ratio of milk and water would be
Procurement price = `16
between 60:40 and 80 : 20
Profit % = 25
60 4 x 80
⇒          Decrease in selling price = 20%
40 8 y 20 \ Selling price if fat content is less than 55%
60 8 5 5 x 80 8 5 `16 if profit % is 25%
⇒         
40 4 9 9 y 20 4 9 16
\ cost price = `12.80
5 5 x 40 1.25
⇒           60% fat is available in 1 liter at `16.
3 9y 9
Cost price of the mixture should be `12.80.
⇒ Concentration of milk in B is between 62.5% and By alligation,
81% If concentration of milk in B is between 60%
\ Water as % of pure milk = 25%
and 80%, then ratio of milk and water would be
between 60:40 and 80:20. 6.
(c) Suppose there is 100  cc of liquid in each container
60 5 x 80 initially.
⇒          I Step: Container B has 10 cc water in 110 cc of
40 9 y 20
mixture.
60 9 4 4 x 80 9 4 II Step: 10 cc of this mixture contains.
⇒         
40 5 8 8 y 20 5 8 100 100 10
10 × =   cc of milk and  cc of water,
27 4 x 18 100 11 11
⇒          
20 8 y 5 which is transferred to container C.
⇒ Concentration of milk in A is between 57.44% and 100
Container C has 100 cc of acid,  cc of milk and
78.26% 11
10
 cc of water.
3. (a) From vessel A (0.5 concentration), 1 litre of spirit 11
and 1 litre of water is taken and from vessel B (0.75 Total quantity of mixture in container C = 110 cc.
concentration), 2.25 litres of spirit and 0.75 litre of
water is taken. III Step: 10  cc of this mixture is transferred to
A which contains.
\ Ratio of spirit to water in the resulting mixture
100 100
1 + 2.25 × 10 cc of milk =  cc of milk
= 1210 121
1 + 0.75
100
3.25 Thus, in 100 cc of mixture, there is  cc of milk of
= 121
1.75 1
the th proportion of the mixture.
13 121
= . 3
7 7.
(c) The amount of alcohol in the old mixture is × 105
7
4. (a) If selling price is not to be affected then fat content of = 45 ml. Further 105 ml alcohol is added.
the mixture should be 55% Fat content of milk is 60    Therefore, total alcohol in new mixture is 150 ml.
and of water is 0. Water content is (105 – 45) = 60 ml. Alcohol to water
By alligation ratio is 150:60 or 5:2.

Chapter_15.indd 22 6/5/2015 2:29:31 AM


Alligation or Mixture I 15.23

8.
(a) Let, X% be the percentage of spirit in A and Y% in B. Weight of copper in the alloy = 9 – x

3X 2Y Weight of zinc in the alloy = 6 + 10 = 16.


 = 29% of (3 + 2) Now, 16 = 4(9 – x)
100 100
Therefore, x = 5 gm
3X 2Y
 = 1.45 ⇒ 3x + 2Y = 145 (1)
100 100 12.
(a) Initial quantity of milk and water in the beakers.
Beaker A: Milk = 25 ml and Water = 75 ml
X 9Y 34
 =  10 = X + 9Y = 340 (2) Beaker B: Milk = 20 ml and Water = 80 ml
100 100 100
Beaker C: Milk = 40 ml and Water = 60 ml
Solving (1) and (2), we get X = 25 and Y = 35
   After 40 ml is transferred from beaker A to beaker
A contains 25% spirit and B contains 35% of spirit. C, the quantity of milk and water in the beakers is as
follows:
5X 3X 4 X
9.
(d) The quantity of new mixture =   Beakers A: Milk = 25 – 10 = 15 ml and
3 2 3
Water = 75 – 30 = 40 ml
27 X 9X
 = = Beaker B: Milk = 20 ml and Water = 80 ml
6 2
5X 3 3X 2 3X 2 Beaker C: Milk = 40 + 10 = 50 ml and
Percentage of milk =     
3 5 2 3 2 3 Water  = 60 + 30 = 90 ml
= 3X Now, Milk: Water in Beaker C = 5.9
5 X 2 3 X 1 1X 2 After 20 ml is transferred form beaker C to beaker B:
Percentage of water =     
3 5 2 3 2 3 Beaker A: Milk = 15 and Water = 40 ml
3X Beaker B: Milk = 20 + 10 = 30 ml and
=
2
3X / 2  Water = 80 + 18 = 98 ml
Percentage of water =  100
9X / 2 Beaker C: Milk = 50 – 10 = 40 ml and
Water = 90 – 18 = 72 ml
100 1
= = 33 % Required ratio = 15:30:40 = 3:6:8
3 3
13.
(c) 80% concentrated HCL is to be diluted to form a 50%
10. (b)
concentrated HCL solution. The quantity of water to
A B be added to the 15 litres solution is
Acid Water Acid Water
1 : 2 3 : 1
Acid = 1/3 Acid = 3/4
Mixture: Acid:Water = 1:1
Required acid = 1/2
Now
113
~ ~3/4 i.e., 5:3
~1/2~ \
3
× 15 = 9 l

II~ ~
5
\ 9 litres of water is added to HCL solution Similarly
116
16 litres of water is added to 90% concentrated
solution of HNO3 to dilute it to 70% concentration.
Required ratio = 1/4:1/6 = 3:2
\ The total quantity of HCL and HNO3 solutions are
So, the required quantity is 3 and 2 litres respectively.
24 litres and 72 litres respectively and that of the
11.
(a) Weight of zinc in the alloy = 6 gms and weight of aqua regia formed is 96 litres and that of water in
copper in the alloy = 9 gms. it is 33.6 litres.
Suppose ‘x’ gms of copper is removed. \ The quantity of mixture ‘X’ is 111 – 96 = 15

Chapter_15.indd 23 6/5/2015 2:29:31 AM


15.24 I Chapter 15

In the mixture X, the quantity of HCL is    A 250 ml of 3:2 solution contains 150 ml milk and
5 100 ml water.
× 0.4 × 15 = 2.5 litres
12 Total milk = 900 ml, total water = 350 ml
and the quantity of water is
After using 250 ml to make curd milk used
5 7 
12  0.6  12  0.7  × 15 @ (0.25 + 0.41) × 15 = 9.91 250
  = × 900 = 180 ml
12  2.5 14.5 1 1250
\ The required ratio is = =
33.6  9.9 43.5 3 Pure milk left = 900 – 180 = 720 ml
14. (b)
Gold Copper 19.
(c) First Alloy Second Alloy
19~ /19   C Al C Al
\ Required alloy
C Al


6/15~4 2 1

1
\ Copper in first alloy =
The alloy formed should contain gold and copper in 3
the ratio 6:4 i.e., 3:2
3
15.
(c) copper in second alloy =
4
Petrol Diesel Kerosene
Initially 2
20 50 30 copper in required alloy (mixture) =
3
Step 1 15 37.5 47.5

16.
(c)
I Step 2 I71.66 I 12.5 I
15.83 I Now, by alligation

2""
236

/88 /2
166


Boumvita
5
~Plan 30

(Q  IL mixture) 1 1
:
Bournvita 5 1 12 3
= =
Complan 30 6 ⇒ 1 : 4
1 20.
(d)
Bournvita = of the mixture.
7
1st type lind type
17.
(d) Amount of alcohol in first vessel = 0.25 × 2 = 0.5 litre
amount of alcohol in second vessel = 0.4 × 6 = 2.4 litre 15~ /5

   Total amount of alcohol out of 10 liters of mixture


10
is 0.5 + 2.4 = 2.9 litre
Hence, the concentration of the mixture is 29%


 2.9 
 100
5/ ~5
 10 
Hence both the types should be added in the ratio of
1:1 to obtain the required strength. Hence 20 litres of
18.
(d) In a mixture of 1,000 ml, milk:water = 3:1.
first type should be added to the 20 litres of the second
Hence, milk = 750 ml, water 250 ml type to get the desired solution.

Chapter_15.indd 24 6/5/2015 2:29:32 AM


Alligation or Mixture I 15.25

21.
(c) Suppose there is 100  cc of liquid in each container
 54  x 
initially. \ 54  x   x = 24
 54 
I Step: Container B has 10 cc water in 110 cc of
⇒ x2 – 108x + 1620 = 0
mixture.
⇒ x = 90, 18
II Step: 10 cc of this mixture contains
Since 90 > 54, therefore x = 90 is ruled out. Hence
100 100 10 x = 18.
   10  =  cc of milk and  cc of water,
100 11 11
24.
(b) Suppose the quantity of milk purchased = x litres
which is transferred to container C.
Suppose quantity of water mixed = y litres
100
  Container C has 100 cc of acid,  cc of milk \ Required ratio of the water and the milk in the
11 mixture = y:x
10
and  cc of water.
11 C.P. of x litres of milk = `8.0x
Total quantity of mixture in container C = 110 cc S.P of x litres of milk = 9 (x + y)
III Step: 10 cc of this mixture is transferred to A S.P.  100
which contains. \ C.P. =
100  Gain%
100 100
 10 cc of milk =  cc of milk (9 x  9 y )  100
1210 121 ⇒ 6.4 =
100  40
100 ⇒ 1120x = 900x + 900y
   Thus, in 100 cc of mixture, there is  cc of
121
1 ⇒ 220x = 900y
milk of the th proportion of the mixture.
121 y 220 11
⇒ = =
22.
(b) x 900 45
Milk : Water
20 30 25.
(a) Let, x kg tea is blended.
10  15 \ Total CP = `(25x + 900)
 25 \ Selling price on 10% profit
10 40 10
(25x + 900) + (25x + 900) × = `(27.5x + 990)
  Therefore, the concentration of the solution is 100
reduced from 20 to 10, i.e., reduced by 50% The Total weight of tea = (30 + x) kg
dilution of the solution increased from 30 to 40, i.e.,
increased by 33% \ Selling price at the rate of 30 kg = `(900 + 30x)
On comparing both the selling prices, we get ‘x’.
23.
(c) Suppose x litres of acid was drawn off initially.
\ Remaining acid in the 54-litre vessel full of acid. 26.
(c) In this question, the alligation method is applicable
for the speed.
= (54 – x) litres and water added = x litres.
Now out of the 54 litres of mixture of acid and Spcodofbus Speed of 1rain
water, x litres of mixture is drawn off. 40 55


 54  x
⇒ Quantity of acid drawn off = 
 54
2

 x litres

Average speed
/
x 285
and quantity of water drawn off = litres.
54 6




 54 
Now the vessel contains 54  x  
 54
x 
 x 

/
45 45
litres of acid. 6 6

Chapter_15.indd 25 6/5/2015 2:29:33 AM


15.26 I Chapter 15


\ Time spent in bus:time spent in train The required ratio is 500:25 or 20:1
45 45
= : = 1:1. Number of labourers 20
6 6 =
Number of supervisors 1
\ Distance travelled by train = 55 × 3 = 165 Km
27.
(b) In this question the alligation method is applicable on 840 20
prices, so we should get the average price of mixture. ⇒ =
Number of supervisors 1
S.P. of mixture = `20/litre, profit = 25%
100 840
\ A
verage price = 20 × = `16/litre \ Number of supervisors = = 42.
125 20
Chemical Water
32.
(a) Spirit:Water
25 0

~
3 4 5  4 5 6 

16
/ =  - + - + -  :  - + - + - 


 7 9 11   7 9 11 

= 920:1159.

16
/ ~ 9
33.
(c) Amount of saw dust in 2 kg of haldi

5
= 2000  = 100 g
\ Chemical:Water = 16:9. 100
5x 5 Required proportion = 4%
28.
(b) = ⇒ 10x = 5x + 25 ⇒ x = 5
x5 2
100 4
Milk = 25 litres. ⇒ =
2000  x 100

29.
(d)
Zinc Copper ⇒ x = 500 ⇒ x = 0.5 kg
5 8
34.
(c) Let, x litres of solution is mixed
13 13
5 3  3x 
\ Content of acid in new mixture =  30 +  litres
8 8  10 
\ Range of the new solution can be found by the
 5 5   8 3 300  3 x
\    :    = 105:103.
 13 8   13 8  equations × 100 = 20% and
10(200  x)
5 2 3 3
30.
(c) x  y = x  y 300  3 x
8 5 8 5  100 = 25%
10(200  x)
x 4
⇒ =
y 5 35. (b) Amount of liquid left after n operations, when the
⇒ Ratio = 4:5. container originally contain x units of liquid from
n
31.
(b)  y
which y units is taken out each time is x 1 − 
Average salary of Average salary of  x
labourers (l' 75) supervisors (l' 600) units.
2 40

~ / Here x = 20 litres, y = 20 × = ,n=3


3 3
Mean sa1ary of all 3 3
the staff (l' 100)  40/3   2
\ 20 1 −  = 20 1 − 
 20   3


/
500 25
1 1 1
= 20 × × × =
3 3 3
20
27

Chapter_15.indd 26 6/5/2015 2:29:34 AM


Alligation or Mixture 15.27

36. (b) Applying allegation 40. (c) Let, x g of sucrose be added with 1 g of saccharin to
obtain a mixture 100 times as sweet as glucose.

Then, 1.00 x + 1 × 675.00 = 0.74( x + 1) × 100

⇒ x + 675 = 74( x + 1)

⇒ x = 9.26 g or 9 g (approximately).

[(0.74) + (1.00)(2) + (1.7)(3)]
Hence, the required ratio is 2:1. 41. (a) = 1.3
6
37. (a) Out of total 100 lilters of mixture there is 20 lilters of
42. (c) Let, the capacity of each cup be 100 ml.
water and 80 lilters milk.
1 After first operation, first container will have 200 mL
He sells part of mixture that is 25 litres. of alcohol and second container will have 300 mL
4
alcohol and 500 mL water.
Now water will be 15 lilters and 60 lilters of milk
in total 75 litres of mixture. When he adds 25 lilters Ratio of water to alcohol in the second container
water in it now total water will be 25 + 15 = 40 lilters = 5: 3.
and milk 60 litres. So, the required ratio is 40:60 = 2:3. After second operation, the quantity of water and
Hence, answer is (a)  5
alcohol left would be  300 ×  = 187.5 mL and
 8 
38. (b) The fare in rupees for k km ( k ≥ 5 ) is
 3
2.4 + 4 × 2 + 1.2 ( k − 5 )  300 ×  = 112.5 mL, respectively and quantity of
 8
= 10.4 + 1.2 ( k − 5 ) water and alcohol in the first container is 187.5 mL

and (200 + 112.5) mL = 312.5 mL, hence, ratio of
39. (d) Let, the amount of milk and water be 7x and 3x L water and alcohol = 187.5:312.5 = 3: 5
Then, we have Hence, on comparing ratio of water and alcohol in
both the containers, we find that A = B.
1 5  240  3  240 
= 1 − ⇒ = 1 −  43. (d) Let, the ratio of contents of the two containers be x
4 2  12 x  5  12 x 
and y.
2 20
= ⇒ x = 50 5 1
5 x Then, quantity of a liquid A in the mixture = x + y
6 4
∴  Half of the initial amount = (350 + 150) L
1 3
Then, the required amount of water = 5x – 3x = 2x = And quantity of liquid B in the mixture = x+ y
6 4
2 × 50 = 100 L
We are given
But for the whole amount water required to be added =
2 × 100 = 200 L. 5x y
+
6 4 =1
Directions (Q 40–41): Answer the questions based on the following x 3y 1
information: +
6 4
The following table presents the sweetness of different items
relative to sucrose, whose sweetness is taken to be 1.00 5x x 3 y y
⇒ − = −
6 6 4 4
Lactose 0.16
4x 2 y
or =
Maltose 0.32 6 4
Glucose 0.74 x 3
⇒ = .
y 4
Sucrose 1.00
Fructose 1.70 44. (b) Selling price of the mixture at a profit of 37.5% is `75.
75
Saccharin 675.00 Hence, cost price = = ` 54.54.
1.375

Chapter_15.indd 27 6/5/2015 2:29:39 AM


15.28 Chapter 15

Assuming cost of water is 0. By allegation rule, we get n


85 × 73  3.65 
⇒ > 73 × 1 − 
Spirit Water 100  73 
60 0 85  73 − 3.65 
n

54.54 ⇒ > 
100  73 
54.54 5.454 n
17  69.35 
10 : 1 ⇒ > 
20  7300 
85 × 73 17  19 
n
45. (b) 85% of 73 lilters of wine = ⇒ > 
100 20  20 
Now, let the process be repeated n times. ⇒ 0.85 > (0.95) n
 n
 On solving, we get n > 3
Then, 85 × 73 > 73 1 − 3.65  
100   73   ∴ Minimum 4 opertions are requiried.

Chapter_15.indd 28 6/5/2015 2:29:41 AM


CHAPTER

Problems on Ages 16
INTRODUCTION
Problems based on ages are generally asked in most of (iii) Age some years hence
the competitive examinations. To solve these problems, Two of these situations are given and it is required to
the knowledge of linear equations is essential. In such find the third. The relation between the age of two persons
problems, there may be three situations: may also be given. Simple linear equations are framed and
(i) Age some years ago their solutions are obtained. Sometimes, short-cut methods
(ii) Present age given below are also helpful in solving such problems.

soMe useful sHort-cut MetHoDs

Illustration 1 The age of father is 4 times the age of his son.


1. If the age of A, t years ago, was n times the age If 5 years ago father’s age was 7 times the age of his son at
of B and at present A’s age is n2 times that of that time, what is father’s present age?
B, then
Solution: The father’s present age
 n −1 
A’s present age =  1  n2 t years  n −1 
 n1 − n2  =  1  n2t (Here n1 = 7, n2 = 4 and t = 5)
 n1 − n2 
 n −1   7 −1  6× 4×5
and B’s present age =  1  t years = 
 n1 − n2   4×5= = 40 years
7−4 3

Explanation 2. The present age of A is n1 times the present age


Let the present age of B be x years. of B. If t years hence, the age of A would be n2
Then, the present age of A = n2 x years times that of B, then
 n −1 
Given, t years ago, A’s present age =  2  n2 t years
 n1 − n2 
n1 (x - t) = n2 x - t or, (n1 – n2) x = (n1 - 1) t
 n −1 
 n −1  and B’s present age =  2  t years
or, x =  1  t.  n1 − n2 
 n1 − n2 
 n −1  Explanation
Therefore, B’s present age =  1  t years Let the present age of B be x years.
 n1 − n2 
Then, the present age of A = n1x
 n −1  Given, t years hence,
and A’s present age =  1  n2 t years.
 n1 − n2  (n1x + t) = n2 (x + t)

Chapter_16.indd 1 6/5/2015 2:05:02 AM


16.2 Chapter 16

or, (n1 - n2)x = (n2 - 1)t Solution: Present age of Anu


t (n − 1) + t1 (n1 − 1)
 n −1  = 2 2
or, x =  2 t n1 − n2
 n1 − n2  (Here n1 = 4, n2 = 2, t1 = 10 and t2 = 10)
 n −1  10(2 − 1) + 10(4 − 1) 10 + 30
Therefore, B’s present age =  2  n1t years = = = 20 years
4−2 2
 n1 − n2 
 n −1  4. The sum of present ages of A and B is S years. If, t
and A’s present age =  2  n1t years.
 n1 − n2  years ago, the age of A was n times the age of B,
then
Sn − t (n − 1)
Illustration 2 The age of Mr Gupta is four times the age Present age of A = years
of his son. After 10 years, the age of Mr Gupta will be only n +1
twice the age of his son. Find the present age of Mr Gupta’s S + t (n − 1)
and Present age of B = years.
son n +1
Solution: The present age of Mr Gupta’s son
Explanation
 n −1 
=  2 t Let the present ages of A and B be x and y years, respectively.
 n1 − n2  Given x+y =S (1)
 2 −1  and, x - t = n (y - t)
=   10 or, x - ny = (1 - n) t (2)
 4−2
(Here n1 = 4 , n2 = 2 and t = 10) Solving Eqs. (1) and (2), we get
= 5 years Sn − t (n − 1)
x=
n +1
3. The age of A, t1 years ago, was n1 times the age of S + t (n − 1)
and, y=
B. If t2 years hence A’s age would be n2 times that n +1
of B, then, Illustration 4 The sum of the ages of A and B is 42 years. 3
n (t + t )(n − 1) years back, the age of A was 5 times the age of B . Find the
A’s present age = 1 1 2 2 + t1 years
n1 − n2 difference between the present ages of A and B
t2 (n2 − 1) + t1 (n1 − 1) Solution: Here S = 42, n = 5 and t = 3
and B’s present age = years. \ Present age of A
n1 − n2
Sn − t (n − 1) 42 × 5 − 3(5 − 1)
= =
n +1 5 +1
Explanation
198
Let A’s present age = x years and B’s present age = y years. = = 33 years
6
Given x - t1 = n1 (y - t1) and x + t2 = n2 (y + t2) and present age of B
i.e., x - n1 y = (1 - n1) t1 (1)
5 + t (n + 1) 42 + 3(5 − 1)
and x - n2 y = (-1 + n2 ) t2 (2) = =
n +1 5 +1
Solving Eqs. (1) and (2), we get 54
= = 9 years
n1 (t1 + t2 )(n2 − 1) 6
x= + t1 \ Difference between the present ages of A and B =
n1 − n2
33 - 9 = 24 years.
t2 (n2 − 1) + t1 (n1 − 1)
and, y=
n1 − n2 Note:

Illustration 3 10 years ago Anu’s mother was 4 times older If, instead of sum (S), difference (D) of their ages is
than her daughter. After 10 years, the mother will be twice given, replace S by D and in the denominator (n + 1) by
older than the daughter. Find the present age of Anu ( n - 1) in the above formula.

Chapter_16.indd 2 6/5/2015 2:05:03 AM


Problems on Ages 16.3

5. The sum of present ages of A and B is S years. If, 6. If the ratio of the present ages of A and B is a:b and
t years hence, the age of A would be n times the t years hence, it will be c:d, then
age of B, then at (c − d )
Sn + t (n − 1) A’s present age =
present age of A = years ad − bc
n +1 bt (c − d )
S − t (n − 1) and, B’s present age = .
and present age of B = years. ad − bc
n +1
Illustration 6 The ratio of the age of father and son
Explanation at present is 6:1. After 5 years, the ratio will become
Let the present ages of A and B be x and y years, respectively 7:2. Find the present age of the son
Given x+y =S (1) bt (c − d )
Solution: The present age of the son =
and, x + t = n ( y + t) ad − bc
or, x - ny = t (n - 1) (2) (Here a = 6, b = 1, c = 7, d = 2 and t = 5)
1× 5(7 − 2)
Solving Eqs. (1) and (2), we get = = 5 years
6 × 2 − 1× 7
Sn + t (n − 1)
x = Note:
n +1
S − t (n − 1) If, with the ratio of present ages, the ratio of ages t years ago
and, y =
n +1 is given, then replace t by (-t) in the above formula.
Illustration 5 The sum of the ages of a son and father is 56 Illustration 7 Six years ago Mahesh was twice as old as
years. After four years, the age of the father will be three Suresh. If the ratio of their present ages is 9:5 respectively,
times that of the son. Find their respective ages what is the difference between their present ages?
Solution: The age of father Solution: Present age of Mahesh
Sn + t (n − 1) 56 × 3 + 4(3 − 1)
= = = −at (c − d )
n +1 3 +1 ad − bc
(Here S = 56, t = 4 and n = 3) −9 × 6(2 − 1)
=
176 1× 9 − 5 × 2
= = 44 years (Here a = 9, b = 5, c = 2, d = 1 and t = 6)
4
S − t (n − 1) = 54 years
The age of son = Present age of Suresh
n +1 −bt (c − d )
56 − 4(3 − 1) =
= ad − bc
3 +1 −5 × 6(2 − 1)
48 = = 30 years.
= = 12 years 1× 9 − 5 × 2
4 \ Difference of their ages = 54 - 30 = 24 years.

Practice Exercises

Difficulty level-1
(BaseD on MeMory)

1. A’s age is one-sixths of B’s age. B’s age will be twice of C’s 2. Sachin was twice as old as Ajay 10 years back. How
age after 10 years. If C’s eighth birthday was celebrated old is Ajay today if Sachin will be 40 years old 10 years
two years ago, then the present age of A must be: hence?
(a) 5 years (b) 10 years (a) 20 years (b) 10 years
(c) 15 years (d) 20 years (c) 30 years (d) None of these
[Based on MAT, 2002] [Based on MAT, 2005]

Chapter_16.indd 3 6/5/2015 2:05:04 AM


16.4 Chapter 16

3.
A demographic survey of 100 families in which two 10. A’s age is thrice that of B’s and four times that of C’s. Find
parents were present revealed that the average age A, of A:B:C.
the oldest child, is 20 years less than half the sum of the (a) 1:3:4 (b) 3:4:12
ages of the two parents. If F represents the age of one
(c) 2:3:8 (d) None of these
parent and M, the age of the other parent, then which of
the following equivalent to A? 11. Namrata’s father is now four times her age. In five years,
F + M − 20 F+M he will be three times her age. In how many years, will he
(a) (b) + 20 be twice her age?
2 2
F+M (a) 5 (b) 20
(c) − 20 (d) F + M – 10
2 (c) 25 (d) 15
[Based on MAT, 2001]
[Based on SCMHRD Ent. Exam., 2003]
4. Rohan is two years younger than Mohan who is three
12. A father is twice as old as his son. 20 years back, he was
years younger than Sohan who is four years older than
twelve times as old as the son. What are their present
Rohit who is two years older than Mohit who is three
ages?
years younger than Sohit. Thus:
(a) 24, 12 (b) 44, 22
(a) Sohan is 7 years older than Mohit
(c) 48, 24 (d) None of these
(b) Rohit is 2 years younger than Sohit
[Based on IMT Ghaziabad, 2002]
(c) Mohan is 3 years older than Mohit
(d) Rohit is 2 years older than Sohit 13. The ratio of ages of Rahul and Deepesh is 3:5. 10 years
later this ratio becomes 5:7. What is the present age of
5.
Two groups of student, whose average ages are 20 years Deepesh?
and 30 years, combine to form a third group whose (a) 20 years (b) 50 years
average age is 23 years. What is the ratio of the number of
(c) 25 years (d) 40 years
students in the first group to the number of students in the
second group? 14. The ratio of ages of A and B is 8:9 and the age of B is
(a) 5:2 (b) 2:5 two-thirds of C’s age and age of C is nine-thirteenths
(c) 7:3 (d) None of these times the age of D. If the age of B is 18 years, then the
[Based on IIT Joint Man. Ent. Test, 2004] age of C is:
(a) 36 years (b) 39 years
6. A years ago, a father was four times his son’s age. In six
years, his age will be 9 more than twice his son’s age. (c) 27 years (d) 54 years
What is the present age of the son?
15. If Dennis is one-third the age of his father Keith now, and
(a) 10 years (b) 9 years was one-fourth the age of his father 5 years ago, then how
(c) 20 years (d) None of these old will his father Keith be 5 years from now?
[Based on IIT Joint Man. Ent. Test, 2004] (a) 20 years (b) 45 years
7. 5 years ago his mother’s age was thrice that of Amit. (c) 40 years (d) 50 years
Amit’s present age is 20. What will be the ratio of their
ages 10 years from now? 16.
Fifteen years hence, a man will be four times as old as he
(a) 30:70 (b) 1:3 was fifteen years ago. His present age is:
(c) 5:2 (d) 1:2 (a) 25 years (b) 20 years
(c) 30 years (d) 45 years
8.
Sister’s age is 3 times that of her brother’s. After 5 years
the sister shall be twice as old as her brother. How many 17.
The ages of A, B and C together total 185 years. B is
years before, the sister’s age was 6 times of her brother’s twice as old as A and C is 17 years older than A. Then, the
age? respective ages of A, B and C are:
(a) 1 year (b) 3 years (a) 40, 86 and 59 years (b) 42, 84 and 59 years
(c) 5 years (d) 10 years (c) 40, 80 and 65 years (d) None of these

9. The average age of a class is 15.8 years. The average age 18. One years ago a father was four times as old as his son.
of the boys in the class is 16.4 years and that of the girls is In 6 years time his age exceeds twice his son’s age by 9
15.4 years. What is the ratio of boys to girls in the class? years. Ratio of their ages is:
(a) 1:2 (b) 3:4 (a) 13:4 (b) 12:5
(c) 2:3 (d) None of these (c) 11:3 (d) 9:2

Chapter_16.indd 4 6/5/2015 2:05:04 AM


Problems on Ages 16.5

19. The sum of ages of a father and son is 45 years. Five years P is added to thrice the age of Q and thrice the age of R,
ago, the product of their ages was four times the father’s the total becomes 108. What is the age of P?
age at that time. The present age of the father is: (a) 17 years (b) 19 years
(a) 39 years (b) 36 years (c) 15 years (d) 12 years
(c) 25 years (d) None of these [Based on MAT (Sept), 2008]
20. Jayesh is as much younger to Anil as he is older to 28.
A father’s age is three times the sum of the ages of his two
Prashant. If the sum of the ages of Anil and Prashant is 48 children, but 20 years hence his age will be equal to the
years, what is the age of Jayesh? sum of their ages. Then, the father’s age is:
(a) 20 years (b) 24 years (a) 30 years (b) 40 years
(c) 30 years (d) Cannot be determined (c) 35 years (d) 45 years
21.
The ratio of the ages of the father and the son at present is [Based on MAT (May), 2008 (Sept), 2007]
7:1. After 4 years, the ratio will become 4:1. What is the 29.
If 6 years are subtracted from the present age of Randheer
sum of the present ages of the father and the son? and the remainder is divided by 18, then the present age of
(a) 29 years (b) 35 years his grandson Anup is obtained. If Anup is 2 years younger
(c) 32 years (d) None of these to Mahesh, whose age is 5 years, then what is the age of
[Based on MAT (Sept), 2008] Randheer?
22.
The product of the present ages of Sarita and Gauri is 320. (a) 84 years (b) 96 years
Eight years from now, Sarita’s age will be three times the (c) 48 years (d) 60 years
age of Gauri. What was the age of Sarita when Gauri was [Based on MAT (Feb), 2008]
born? 30.
1 year ago, a mother was 4 times older to her son. After
(a) 40 years (b) 32 years 6 years, her age becomes more than double her son’s age
(c) 48 years (d) 36 years by 5 years. The present ratio of their age will be:
[Based on MAT (Feb), 2011] (a) 13:12 (b) 3:1
23.
Anil is at present one-fourth the age of his father. After (c) 11:3 (d) 25:7
16 years, he will be one-half age of his father. Find the [Based on MAT (Dec), 2007]
present age of Anil’s father. 31.
The average age of a husband, his wife and son 3 years
(a) 40 years (b) 36 years ago was 27 years and that of his wife and son 5 years ago
(c) 32 years (d) 28 years was 20 years. What is the husband’s present age?
[Based on MAT (Sept), 2009] (a) 35 years (b) 32 years
24.
Honey was twice as old as Vani 10 years ago. How old is (c) 37 years (d) 40 years
Vani today, if Honey will be 40 years old 10 years hence? [Based on MAT (Feb), 2008]
(a) 25 years (b) 20 years 32.
In a class, there are 20 boys whose average age is
decreased by 2 months, when one boy age 18 years is
(c) 15 years (d) 35 years
replaced by a new boy. The age of the new boy is:
[Based on MAT (Feb), 2009]
(a) 14 years 8 months (b) 16 years 4 months
25.
The age of the father 5 years ago was 5 times the age of
(c) 15 years (d) 17 years 10 months
his son. At present the father’s age is 3 times that of his
son. What is the present age of the father? [Based on MAT (Dec), 2007]

(a) 33 years (b) 30 years 33.


The age of the father of two children is twice that of the
elder one added to four times that of the younger one.
(c) 45 years (d) None of these
If the geometric mean of the ages of the two children
[Based on MAT (Feb), 2009]
is 4 3 and their harmonic mean is 6, then what is the
26.
Sonu is 4 years younger than Manu while dolly is 4 years father’s age?
younger than Sumit but one-fifth times as old as Sonu. If (a) 48 years (b) 32 years
Sumit is eight years old, how many times as old is Manu
(c) 40 years (d) 56 years
as Dolly?
[Based on MAT (Sept), 2008]
(a) 6 (b) 1/2
34.
The age of a man is 3 times that of his son. 15 years ago,
(c) 3 (d) None of these the man was 9 times as old as his son. What will be the age
[Based on MAT (Sept), 2008] of the man after 15 years?
27.
If the ages of P and R are added to twice the age of Q, (a) 45 years (b) 60 years
the total becomes 59. If the ages of Q and R are added to (c) 75 years (d) 65 years
thrice the age of P, the total becomes 68 and if the age of
[Based on MAT, 1999]

Chapter_16.indd 5 6/5/2015 2:05:04 AM


16.6 Chapter 16

35.
Ashu’s mother was three times as old as Ashu 5 years ago. (a) 18 years (b) 21 years
After 5 years, she will be twice as old as Ashu. How old is (c) 15 years (d) 24 years
Ashu today?
44.
The ratio of Laxmi’s age to the age of her mother is 3:11.
(a) 35 years (b) 10 years
The difference of their ages is 24 years. The ratio of their
(b) 20 years (d) 15 years ages after 3 years will be:
[Based on MAT, 1999] (a) 1:3 (b) 2:3
36.
Father is 5 years older than the mother and the mother’s (c) 3:5 (d) 2:5
age now is thrice the age of the daugh­ter. The daughter [Based on FMS, 2006]
is now 10 years old. What was the father’s age when the
daughter was born? 45. The ratio of A’s and B’s ages is 4:5. If the difference
between the present age of A and the age of B 5 years
(a) 20 years (b) 15 years
hence is 3 years, then what is the total of present ages of A
(c) 25 years (d) 30 years and B?
[Based on MAT, 1999] (a) 68 years (b) 72 years
37.
A father told his son, ‘I was as old as you are at present at (c) 76 years (d) 64 years
the time of your birth’. If the father is 38 years old now,
then what was the son’s age five years back? 46. If twice the son’s age in years be added to the father’s age,
the sum is 70 and if twice the father’s age is added to the
(a) 14 years (b) 19 years
son’s age, the sum is 95. Father’s age is:
(c) 38 years (d) 33 years (a) 40 years (b) 35 years
[Based on MAT, 1999]
(c) 42 years (d) 45 years
38.
Fifteen years hence, a man will be four times as old as he
was fifteen years ago. His present age is: 47.
Sneh’s age is one-sixth of her father’s age. Sneh’s father’s
(a) 25 years (b) 20 years age will be twice of Vimal’s age after 10 years. If Vimal’s
eighth birthday was celebrated 2 years before, then what
(c) 30 years (d) 45 years
is Sneh’s present age?
[Based on MAT, 1999]
(a) 30 years (b) 24 years
39.
The average age of an adult class is 40 years. 12 new
(c) 6 years (d) None of these
students with an average age of 32 years join the class,
[Based on NMAT, 2005]
thereby decreasing the average by 4 years. The original
strength of the class was: 48. A man’s age is 125% of what it was 10 years ago, but
(a) 10 (b) 11 1
83 % of what it will be after ten 10 years. What is his
(c) 12 (d) 15 3
[Based on MAT, 2000] present age?
40.
The father is five times older than his son. After 4 years, (a) 45 years (b) 50 years
the sum of their ages would be 44 years. Then the son’s (c) 55 years (d) 60 years
age at present is:
(a) 5 years (b) 6 years 49. The age of a person is equal to 4 times the sum of the
ages of her three daughters. 8 years hence her age will be
(c) 7 years (d) 8 years
double the sum of their ages. What is her age now?
[Based on MAT, 2000]
(a) 20 years (b) 40 years
41.
Father’s age is 4 times that of his son. 5 years back, it was
(c) 60 years (d) 80 years
7 times. His age now is:
[Based on ATMA, 2005]
(a) 30 (b) 35
(c) 40 (d) 45 1
50. The age of Mr. Chetan in 2002 was of his birth year.
[Based on MAT, 2000] 90
42.
Sushil was thrice as old as Snehal 6 years back. Sushil What is his age in 2006?
will be five-thirds times as old as Snehal 6 years hence. (a) 30 (b) 28
How old is Snehal today? (c) 26 (d) 22
(a) 18 years (b) 24 years [Based on JMET, 2006]
(c) 12 years (d) 15 years 51. 15 years hence, Rohit will be just four times as old as he
[Based on FMS (MS), 2006] was 15 years ago. How old is Rohit at present?
43. Ratio of Ashok’s age to Pradeep’s age is 4:3. Ashok will (a) 20 (b) 25
be 26 years old after 6 years. How old is Pradeep now? (c) 30 (d) 35

Chapter_16.indd 6 6/5/2015 2:05:04 AM


Problems on Ages 16.7

52. Ten years ago, Mohan was thrice as old as Ram was but (a) 48 years (b) 60 years
10 years hence, he will be only twice as old. Find Mohan’s (c) 84 years (d) 96 years
present age. [Based on MAT, 2013]
(a) 60 years (b) 80 years 55. The respective ratio between the present age of Manisha
(c) 70 years (d) 76 years and Deepali is 5:X . Manisha is one year younger than
53. If the ages of P and R are added to twice the age of Q, Parineeta. Parineetas age after 9 years will be 33 years. The
the total becomes 59. If the ages of Q and R are added to difference between Deepali’s and Manisha’s age is same as
thrice the age of P, the total becomes 68 and if the age of the present age of Parineeta. What will come in place of X?
P is added to thrice the age of Q and thrice the age of R, (a) 23 (b) 39
the total becomes 108. What is the age of P? (c) 15 (d) None of these
(a) 19 years (b) 15 years [Based on SNAP, 2013]
(c) 17 years (d) 12 years 56. The age of the father 5 years ago was 5 times the age of
[Based on MAT, 2013] his son. At present the father’s age is 3 times that of his
54. If 6 years are subtracted from the present age of Shyam and son. What is the present age of the father?
the remainder is divided by 18, then the present age of his (a) 33 years (b) 30 years
grandson Anup is obtained. If Anup is 2 years younger to (c) 45 years (d) none of these
Mahesh whose age is 5 years, then what is the age of Shyam? [Based on SNAP, 2012]

Difficulty Level-2
(Based on Memory)

1. There were 15 students in a class. When the ages of a (a) 13:29 (b) 11:27
teacher and a new boy are added, the average age of the (c) 29:17 (d) 13:25
class increases by 10 per cent while it remains the same
5.
Father is 5 years older than mother and mother’s age now
when only the age of a boy is added. If the teacher’s age
is thrice the age of the daughter. The daughter is now 10
is eight more than twice the age of the new boy, then find
years old. What was father’s age when the daughter was
the initial average age of the class.
born?
(a) 15.4 years (b) 16.5 years
(a) 20 years (b) 15 years
(c) 11.4 years (d) None of these (c) 25 years (d) 30 years
2. The age of a person k years ago was half of what his age 6. The average age of the mother and her 6 children is 12
would be k years from now. The age of the same person years which is reduced by 5 years if the age of the mother
p years from now would be thrice of what his age was is excluded. How old is the mother?
p years ago. What is the value of the ratio k:p?
(a) 42 years (b) 40 years
(a) 3:2 (b) 2:3
(c) 48 years (d) 50 years
(c) 1:4 (d) 4:1
7. A father’s age is three times the sum of the ages of his two
3. Ten years ago, the ages of the members of a joint family of children, but 20 years hence his age will be equal to the
eight people added up to 231 years. Three years later, one sum of their ages. Then the father’s age is:
member died at the age of 60 years and a child was born
(a) 30 years (b) 40 years
during the same years. After another three years, one more
member died, again at 60, and a child was born during the (c) 35 years (d) 45 years
same years. The current average age of this eight-member 8. Three times the present age of a father is equal to eight
joint family is nearest to: times the present age of his son. Eight years hence the
father will be twice as old as his son at that time. What are
(a) 21 years (b) 25 years
their present ages?
(c) 24 years (d) 23 years
(a) 35, 15 (b) 32, 12
[Based on CAT, 2007]
(c) 40, 15 (d) 27, 8
4. 10 years ago the age of Karisma was two-thirds of
the age of Babita. 14 years hence the ratio of ages of 9. Five years ago Mr Sohanlal was thrice as old as his son
Karishma and Babita will be 5:9. Find the ratio of their and ten years hence he will be twice as old as his son.
present ages. Mr Sohanlal’s present age (in years) is:

Chapter_16.indd 7 6/5/2015 2:05:04 AM


16.8 Chapter 16

(a) 35 (b) 45 (a) 1806 (b) 1836


(c) 50 (d) 55 (c) 1812 (d) 1825
[Based on FMS, 2011]
10.
If 6 years are subtracted from the present age of Randheer
and the remainder is divided by 18, then the present age 16. Five years ago, Bina’s age was three times that of Arti.
of his grandson Anup is obtained. If Anup is 2 years Ten years ago, Bina’s age was half that of Chitra. If c
younger to Mahesh whose age is 5 years, what is the age represents Chitra’s current age, which of the following
of Randheer? represents Arti’s current age?
(a) 84 years (b) 48 years (a) (c – 10)/3 (b) c/6 + 5
(c) 60 years (d) 96 years (c) 3c – 5 (d) 5c/3 – 10
[Based on MAT (Sept), 2008] [Based on MHT-CET MBA, 2010]

11.
If 1 is added to the age of the elder sister, then the ratio of 17.
If 1 is added to the age of the elder sister, then the ratio of
the ages of two sisters becomes 0.5:1, but if 2 is subtracted the ages of two sisters become 0.5:1, but if 2 is subtracted
from the age of the younger one, the ratio becomes 1:3. from the age of the younger one, the ratio becomes 1:3,
Find the age of the two sisters. the age of the younger sister will be?
(a) 8 and 5 years (b) 11 and 6 years (a) 9 years (b) 5 years
(c) 9 and 5 years (d) 8 and 6 years (c) 18 years (d) 15 years
[Based on MAT (Dec), 2010] [Based on ATMA, 2008]

12.
A boy was asked of his age by his friend. The boy said, 18.
The sum of the reciprocals of the ages of two colleagues
‘The number you get when you subtract 25 times my age is five times the difference of the reciprocals of their ages.
from twice the square of my age will be thrice your age.’ If the ratio of the product of their ages to the sum of their
If the friend’s age is 14, then the age of the boy is: ages is 14.4:1, the age (in years) of one of the colleagues
(a) 28 years (b) 21 years must be between (both inclusive):
(c) 14 years (d) 25 years (a) 20 and 23 (b) 23 and 26
[Based on MAT (Feb), 2011] (c) 26 and 30 (d) 30 and 35
[Based on ATMA, 2008]
13.
In a cricket 11, the average age of 11 players is 28
years. Out of these, the average ages of three groups of
19. The ages of Ram and Shyam differ by 16 years. Six years
three players each are 25 years, 28 years and 30 years,
ago, Mohan’s age was thrice as that of Ram’s, find their
respectively. If in these groups, the captain and the
present ages.
youngest player are not included and the captain is eleven
years older than the youngest player, what is the age of the (a) 14 years, 30 years
captain? (b) 12 years, 28 years
(a) 33 years (b) 34 years (c) 16 years, 34 years
(d) 18 years, 38 years
(c) 35 years (d) 36 years
[Based on FMS (MS), 2006]
20. Ten years ago, the ages of the members of a joint family of
14.
The average age of an adult class is 40 years. Twelve eight people added up to 231 years. Three years later, one
new students with an average age of 32 years join the member died at the age of 60 years and a child was born
class, thereby decreasing the average age of the class by during the same year. After another three years, one more
4 years. The original strength of the class was: member died, again at 60 a child was born during the
(a) 10 (b) 11 same year. The current average age of this eight-member
joint family is nearest to:
(c) 12 (d) 15
(a) 22 years (b) 21 years
[Based on FMS (MS), 2006]
(c) 45 years (d) 24 years
15.
A man born in the first half of the nineteenth century was (e) 23 years
x years old in the years x2. He was born in: [Based on CAT, 2007]

Chapter_16.indd 8 6/5/2015 2:05:04 AM


Problems on Ages 16.9

Answer Keys
Difficulty Level-1

1. (a) 2. (a) 3. (c) 4. (c) 5. (c) 6. (b) 7. (d ) 8. (b) 9. (c) 10. (d ) 11. (b) 12. (b) 13. (c)
14. (c) 15. (d ) 16. (a) 17. (b) 18. (c) 19. (b) 20. (b) 21. (c) 22. (b) 23. (c) 24. (b) 25. (b) 26. (a)
27. (d ) 28. (a) 29. (d ) 30. (d ) 31. (d ) 32. (a) 33. (c) 34. (c) 35. (d ) 36. (c) 37. (a) 38. (a) 39. (c)
40. (b) 41. (c) 42. (c) 43. (c) 44. (a) 45. (b) 46. (a) 47. (d ) 48. (b) 49. (d ) 50. (c) 51. (b) 52. (c)
53. (d ) 54. (b) 55. (d ) 56. (b)

Difficulty Level-2

1. (c) 2. (b) 3. (c) 4. (a) 5. (c) 6. (a) 7. (a) 8. (b) 9. (c) 10. (c) 11. (c) 12. (c) 13. (c)
14. (c) 15. (a) 16. (b) 17. (b) 18. (b) 19. (a) 20. (d )­­­

Explanatory Answers

Difficulty Level-1

1 8.
(b) Let the age of the sister = x years. So, the age of the
1.
(a) A = B, B + 10 = 2 (C + 10), C = 10 x
6 brother =
3
\  B = 30, A = 5 years. By the condition given in the problem,
2.
(a) Sachin’s age today = 30 years x 
x + 5 = 2   5   ⇒  x = 15 years
Sachin’s age 10 years back = 20 years 3 
Ajay’s age 10 years back = 10 years Again, let before t years sister’s age was 6 times that
Ajay’s age today = 20 years. of her brother’s.
\  15 – t = 6 (5 – t)  ⇒  t = 3.
F+M
3.
(c) A = – 20. 9. (c) Let number of boys = x
2
Let number of girls = y
4.
(c) Rohan = Mohan – 2 = (Sohan – 3) – 2 = (Rohit + 4) \ Total number of students = x + y
– 3 – 2 = (Mohit + 2) + 4 – 3 – 2 = (Sohit – 3) + 2 +
⇒ (x + y) × 15.8 = 16.4x + 15.4y ⇒ 0.6x = 0.4y
4 – 3 – 2 = (Sohit – 2).
x 0.4 2
5. (c) Let the number of students in the two groups be x and ⇒  = = .
y 0.6 3
y respectively.
x 7 10.
(d) Ratio of ages of A and B = 3:1
\ 20x + 30y = 23 (x + y) ⇒ 3x = 7y ⇒ = . Ratio of ages of A and C = 4:1
y 3
6.
(b) (F – 1) = 4 (S – 1) (1) A B C
3 1
where F and S are the Father’s and the Son’s ages
respectively at present. 4 1
\ (F + 6) = 2 (S + 6) + 9 (2) ------------------------
12 4 3
From Eqs. (1) and (2), S = 9.
11. (b) Let Namrata’s age = x years
7.
(d) Amit’s present age is 20. 5 years ago he was 15;
therefore his mother was 45. 10 years from now his Let Namrata’s father’s age = y years
mother will be 60; Amit will be 30. Hence the ratio of \ y = 4x and y + 5 = 3 (x + 5)
Amit’s age to mother’s is 1:2. \ x = 10, y = 10

Chapter_16.indd 9 6/5/2015 2:05:05 AM


16.10 I Chapter 16

Let, y + K = 2 (x + K) 18. (c) Let the present ages of father and son be x and y years,
(i.e., After K years, the father will be double of her age) respectively
Then, (x - 1) = 4(y - 1)
⇒ 40 + K = 2 (10 + K)  ⇒  K = 20
or 4y - x = 3 (1)
\ After 20 years, Namrata’s father will be double of
and (x + 6) - 2(y + 6) = 9
her age.
or -2y + x = 15 (2)
12.
(b) F = 2S, F – 20 = 12 (S – 20) Solving Eqs. (1) and (2), we get, x = 33, y = 9
⇒ 2S – 20 = 12S – 240 ⇒ S = 22 \ Ratio of their ages = 33:9 = 11:3
\    F = 44. 19.
(b) Let father’s present age = x years
R 3x Then, son’s present age = (45 - x) years
13.
(c) = Given: (x - 5)(45 - x - 5) = 4(x - 5)
D 5x
R 3 x  10 5 or,  x2 - 41x + 180 = 0  or,  (x - 36)(x - 5) = 0
⇒ 
D 5 x  10 7 \         x = 36 years.

⇒  x = 5 20.
(b)

R 15 21.
(c) Let the present age of father and son be 7x and x years,
Hence, =
D 25 respectively.
14.
(c) A:B = 8:9 After 4 years,
B:C = 2:3 age of father = (7x + 4) years
C:D = 9:13 age of son = (x + 4) years
A:B:C:D = 144x:162x:243x:351x 7x + 4 4
Given, =
But we need not solve this, since we already know x+4 1
that B:C = 2x:3x ⇒ 7x + 4 = 4x + 16
\ 2x:3x : : 18:K ⇒ K = 27 years
⇒ 3x = 12
y k 2
× = ⇒ k:p = 2:3 \ x = 4
p y 3
\ 7x + x = 28 + 4 = 32 years.
15.
(d) Let the present age of Dennis and his father be x and y
respectively. Then 22.
(b) Let the present ages of Sarita and Gauri are x and y.
1 Then, xy = 320
x = y (1)
3
and (x + 8) = 3(y + 8)
1
and (x – 5) = ( y  5) (2) ⇒  x – 3y = 16
4
From Eqs. (1) and (2), y = 45 years  320 
⇒  x – 3   = 16
Hence, required age = (x + 5) = 50 years  x 
16.
(a) Let the present age of the man = x years ⇒  x2 – 16x – 960 = 0
\ (x + 15) = 4 (x – 15) ⇒ (x – 40) (x + 24) = 0
⇒ 3x = 75  ⇒  x = 25 ⇒  x = 40 and y = 8
17.
(b) Let A’s age be x years At the time of Gauri born, the age of Sarita is 32 years.
B’s age be 2x years
C’s age = (x + 17) years 23.
(c) Let the present age of Anil’s father be x years.
According to the question, x
Then, Anil’s present age = years
x + 2x + (x + 17) = 185 4
\ 4x = 185 - 17 = 168  \  x = 42 x 1
\ + 16 = (x + 16)
\ A’s age = 42 years 4 2
B’s age = 84 years x x
⇒ + 16 = + 8
C’s age = 42 + 17 = 59 years 4 2

M16_KHAT6981_C16.indd 10 6/16/2015 3:58:41 PM


Problems on Ages I 16.11

x 29.
(d) Present age of Mahesh = 5 years
⇒ = 8
4 Present age of Anup = 3 years
⇒ x = 32 years.
Present age of Randheer = 3 × 18 + 6 = 60 years.
24.
(b) Present age of Honey = 30 years 30.
(d) Let present age of mother and son be x and y years
Honey’s age 10 years ago = 20 years respectively.
\ Vani’s age 10 years ago = 10 years Then, x – 1 = 4(y – 1)
\ Present age of Vani = 20 years. ⇒ x = 4y – 3 (1)
and, x + 6 = 2(y + 6) + 5
25.
(b) Let the present age of father be x years.
⇒ x = 2y + 11 (2)
x
\ Present age of son = years From Eqs. (1) and (2),
3
4y – 3 = 2y + 11
x 
x – 5 = 5 ×  − 5  14
3  ⇒ y = = 7 years
2
5x
⇒ x – 5 = − 25 and, x = 25 years
3
\ Required ratio = 25:7.
2x
⇒ = 20
3 31.
(d) Let present age of husband, his wife and son be x, y
and z respectively.
⇒ x = 30 years.
According to the given condition,
26.
(a) Sonu = Manu – 4
( x − 3) + ( y − 3) + ( z − 3)
Dolly = Sumit – 4 = 27
3
1
Dolly = Sonu ⇒        x + y + z = 90 (1)
5
( y − 5) + ( z − 5)
Sumit = 8 years, Dolly = 4 years, Sonu = 20 years and and,     = 20
Manu = 24 years
⇒          y + z = 50 (2)
Manu = 6 × Dolly.
From Eqs. (1) and (2), we get
27.
(d) 2Q + P + R = 59 (1) x = 90 – 50 = 40 years.
Q + R + 3P = 68 (2)
32.
(a) Total age decreases = 20 × 2 = 40 months
P + 3Q + 3R = 108 (3)
= 3 years 4 months
From Eqs. (2) and (3),
\ The age of new boy = 18 years – 3 years 4 months
3Q + 3R + 9P = 204
P + 3Q + 3R = 108 = 14 years 8 months.

8P = 96
⇒       33.
(c) F = 2E + 4Y (1)
P = 12 years.
\      
and, EY = 4 3
28.
(a) Let the father’s age be x years and age of his children ⇒ EY = 48 (2)
be a and b years.
2EY
x And, = 6 ⇒ E + Y = 16 (3)
\ (a + b) = E +Y
3
and (a + b) + 20 + 20 = x + 20 Now, (E – Y)2 = (E + Y)2 – 4EY

x = (16)2 – 4 × 48
⇒ + 20 = x
3 = 256 – 192 = 64
⇒ x = 30 years. ⇒ E – Y = 8 (4)

Chapter_16.indd 11 6/5/2015 2:05:06 AM


16.12 Chapter 16

From Eqs. (3) and (4), E =12 39.


(c) Suppose original strength = x
and, Y = 4 \ Total age of adult class = 40x years
From Eq. (1), Average age of 12 new students = 32 years
F = 2 × 12 + 4 × 4 = 40 years. \ Total age of 12 new students = 32 × 12
34.
(c) Let the age of man’s son be x years.         
= 384 years
\ age of the man = 3x years.
According to the question,
15 years ago, age of the son = x – 15 years and age of
the man = (3x – 15) years 40 x + 384
= 40 – 4 = 36
x + 12
Now, according to the question,
3 x − 15 or 40x + 384 = 36x + 432
=9
x − 15 or 4x = 48 or x = 12.

or, 3x – 15 = 9x – 135
40.
(b) Suppose the present ages of father and son are 5x,
or, 6x = 120 x years respectively.
\ x = 20 years According to the question,
\ age of the man is; 20 × 3 = 60 years
(5x + 4) + (x + 4) = 44
\ age of the man after 15 years
or, 6x + 8 = 44 or 6x = 36
= 60 + 15 = 75 years.
\ x = 6
35.
(d) Let the age of Ashu at present be x years and her
Hence, present age of son = x = 6 years.
mother be y years.
Now, according to the question, 41.
(c) Let the age of son be x years
3(x – 5) = (y – 5) \ Father’s age = 4x years
or, 3x – 15 = y – 5 5 years back age of son = x – 5 and age of father
or, 3x – y = 10 (1) = 4x – 5
Again, according to the question, Now, according to the question,
2(x + 5) = (y + 5) 4x − 5
=7
or, 2x + 10 = y + 5 x−5

or, 2x – y = – 5 (2) ⇒ x = 10 years
Subtracting Eq. (2) from (1), we get
\ Father’s age now is 10 × 4 = 40 years.
x = 15
Hence, Ashu’s today’s age is 15 years. 42.
(c) Sushil Snehal
36.
(c) Age of mother = 3 × 10 = 30 years 3 x x
Age of father = 30 + 5 = 35 years 5
(3 x  12)  ( x  12)
3
Age of father when the daughter was born
= 35 – 10 = 25 years. 4x = 24 ⇒ x = 6
Present age = 6 + 6 = 12 years.
37.
(a) Let the present age of the son = x years
Now, according to the question, 43.
(c) Let the present ages of Ashok and Pradeep be 4x
and 3x
x = 38 – x or, x = 19 years
So that 4x + 6 = 26 ⇒ x = 5
Five years back son’s age = 19 – 5 = 14 years. \ Present age of  Pradeep is 3x = 3 × 5, i.e., 15 years.
38.
(a) Let age of the man = x years 44.
(a) 11x – 3x = 24
\ x + 15 = 4(x – 15) ⇒ 8x = 24
⇒ x = 25. ⇒ x = 3

Chapter_16.indd 12 6/5/2015 2:05:06 AM


Problems on Ages 16.13

Present age = 9, 33 50. (c) Let age of Mr. Chetan in 2002 be x.


After 3 years = 12, 36 Then, his birth years = 2002 – x
\      
Ratio = 1:3. 2002 − x
According to question, x = ⇒ x = 22
90
A 4 5
45.
(b) Given =   or,  B = A So, his age in 2006 = 22 + 4 = 26.
B 5 4
and, B - (A + 5) = 3  or,  B = A + 8 51.
(b) Let the present age of Rohit be x years
5 Then, given: x + 15 = 4 (x - 15) ⇒ x = 25.
\ A = A + 8
4
52.
(c) Let Mohan’s present age be x years and Ram’s present
5 
or, A   1 = 8 age be y years.
4 
Then, according to the first condition,
\ A = 32 years
x - 10 = 3 ( y - 10)
5
and, B = × 32 = 40 years or, x - 3y = –20 (1)
4
Now, Mohan’s age after 10 years
\ A + B = 40 + 32 = 72 years.
= (x + 10) years
46.
(a) Let son’s age (in years) = x and father’s age (in years) Ram’s age after 10 years = (y + 10)
=y
\ (x + 10) = 2 (y + 10)
Given: 2x + y = 70  and, x + 2y = 95
or, x - 2y = 10 (2)
Solving for y, we get y = 40.
Solving Eqs. (1) and (2), we get
47. (d) Vimal’s present age = 8 + 2 = 10 x = 70 and y = 30
Father’s age after 10 years = (10 + 10) × 2 = 40 years \ Mohan’s age = 70 years and Ram’s age = 30 years.

Father’s present age = 40 – 10 = 30 years 53. (d) Let the ages of P, Q and R be x, y and z years, respectively.
\ Sneh’s present age = × 30 = 5 years. According to the question,
x + 2y + z = 59 (1)
48.
(b) Let the present age be x years. 3x + y + 3z = 68 (2)
Then, 125% of (x - 10) = x x + 3y + 3z = 108 (3)
1 On multiplying Eq. (2) by 3 and subtracting from Eq.
and, 83 % of (x + 10) = x (3) we get
3
1 8x = 96   ⇒  x =12 years
\ 125 % of (x - 10) = 83 % of (x + 10)
3 ∴  Age of P = 12 years.
5 5
or,  (x - 10) = (x + 10) 54.
(b) Let the present age of Shyam be x years.
4 6
x−6
5 5 50 50 ∴  Age of Shyam’s grandson Anup = years
or, x - x = + 18
4 6 6 4 According to the question,
5x 250 Age of Anup = Age of Mahesh -2 years = 5 - 2 = 3 years
or, =
12 12
x−6
or, x = 50 years. ⇒ =3
18
49. (d) Let age of 3 girls = x ∴ x = 60 years
\ Age of person = 4x + 8
Also 2(x + 24) = 4x + 8 55.
(d) According to the question,
\ x = 20 Present age of Parineeta = 33 – 9 = 24 years
\ Age of person = 4x = 4 × 20 = 80 years. Present age of Manisha = 24 – 9 = 15 years

Chapter_16.indd 13 6/5/2015 2:05:07 AM


16.14 I Chapter 16

Present age of Deepali = 24 + 15 = 39 years According to the question ,


  5:X = 15 : 39 x − 5 = 5(y − 5) (1)
5 × 39 and x = 3y(2)
∴  X = = 13
15 \  From Eqn (1) and (2), we have
56. (b) Let the present age of father = x year and Son’s present
y = 10 and x = 30 year.
age = y years.
Hence, father’s present age = 30 years.
5 year ago, father’s age = x − 5 and
Son’s age =y–5

Difficulty Level-2

1. (c) Let the initial average age of the class be x years. 7.


(a) Let the present age of father be x years and the present
Then, from the conditions given, the age of the new age of son be y years.
boy and the teacher is x and 8 + 2x years respectively. \ x = 3y(1)
15 x  8  3 x Also, (x + 20) = (y + 20 + 20) (2)
\  = 1.1x  ⇒  x = 11.4 years.
17
Solving Eqs. (1) and (2), we get
2.
(b) Let the present age of the persons be y years. The x = 30 years.
given data can be written as:
8.
(b)
1
(y + k) = (y + k) and (1)
2 9.
(c) Let Mr Sohanlal’s age (in years) = x
(y + p) = 3 (y – p) (2)
and his son’s age = y
yk 2
From Eq. (1), = Then, x - 5 = 3 (y – 5)
y–k 1
y 3 i.e., x - 3y + 10 = 0
⇒  = (3)
k 1 and, x + 10 = 2 ( y + 10)
(by componendo and dividendo) i.e., x - 2y - 10 = 0
y p 3
From Eq. (2), = ; Solving the two equations, we get
y–k 1
x = 50, y = 20
y 4
⇒    = = 2 (4)
p 2 R−6
10.
(c) =A
18
Dividing Eq. (4) by Eq. (3), x
Given Mahesh = 5 years
3.
(c) 10 years age, total age of 8 members = 231. After
there years, sum of the ages = 231 + 8 × 3 – 60 = 195 \ Anup = 3 years
There more years later sum of ages \ R = 18 × 3 + 6 = 60 years.
= 198 + 8 × 3 – 60 = 169
11.
(c) Let the ages of two sisters be x and y.
191
Current average age =   ≈ 24 years.
8 x 0.5
=
4.
(a) y +1 1

5. (c) Given F = M + 5, M = 3D, D = 10. ⇒ 2x = y + 1 (1)
\ M = 3 × 10 = 30 and F = 30 + 5 = 35. x−2 1
and, =
\ The father’s age when daughter was born = 35 – 10 y 3

= 25 years.
⇒ 3x – 6 = y (2)
6. (a) Total age of the mother and six children = 12 × 7 = 84
years. From Eqs. (1) and (2),
Total age of six children = 7 × 6 = 42 years. x = 5 and y = 9
\  Mother is 42 years old. So, their ages are 9 years and 5 years.

Chapter_16.indd 14 6/5/2015 2:05:08 AM


Problems on Ages 16.15

12.
(c) Let the age of boy be x years. c − 10 + 10
⇒        + 15  = 3y
\ 2x2 – 25x = 3 × 14 2
⇒ 2x2 – 25x – 42 = 0 c
⇒ + 15 = 3y
2
25 ± 625 + 336 c
\ x = ⇒ y = + 5.
2×2 6

25 ± 961 25 ± 31 17.
(b) Suppose that age of age of elder sister be x years and
= = younger sister be y years. Then,
4 4
y 0.5 1
25 + 31 56 = =
x = = = 14years.
yr x +1 1 2
4 4
⇒ 2y = x + 1
13.
(c) Total age, 28 × 11 = 308
⇒ x – 2y = – 1 (1)
3 × 25 = 75
3 × 28 = 84 y−2 1
Again given, =
3 × 30 = 90 x 3
—––
249 ⇒ 3y – 6 = x
Total age of a group = 75 + 84 + 90 = 249 ⇒ x – 3y = – 6 (2)
Difference of captain and youngest players = 308 – After subtracting Eq. (2) from Eq. (1), we get
249 = 59 y = 5
x – y = 11 Put this value in Eq. (1), we have
x + y = 59 x – 10 = – 1 ⇒ x = 10 – 1
—–———–
2x = 70 x = 9
x = 35 So, the age of younger sister is 5 years.
\ Captain age = 35 Years.
18.
(b) Suppose that age of two colleagues be x years and y
14.
(c) According to question years.
x  40  12  32 By question,
= 36
x  12 1 1 1 1
+ = 5  − 
⇒  x = 12. x y x y

15.
(a) The man was born between 1800 and 1850 and he was y+x  y− x
⇒ = 5  
x years old in the years x2. xy  xy 
Now, we can conclude that the years, when he was ⇒ y + x = 5y – 5x
x years old must be a perfect square. Now, the only
perfect square in between 1800 and 1900 is 1849, i.e., ⇒ 6x – 4y = 0
432. So, he was 43 years old in the years 1849, which ⇒ 3x – 2y = 0
means he was born in (1849 – 43) = 1806. 3
\ y = x (1)

16. (b) Let the Bina’s present age = x and Arti’s present 2
age = y Again by question
Then (x – 5) = 3(y – 5) xy 14.4
x – 5 = 3y – 15 (1) =
x+ y 1
1 144
Again, x – 10 = (c – 10) =
2 10
1 xy
x = (c – 10) + 10 (2) 72
2 ⇒ =
x+ y 5
On putting value of x in Eq. (1) ⇒ 5xy = 72 (x + y) (1)
1 3
(c – 10) + 10 – 5 = 3y – 15 By Eq. (1), put y = x , we have
2 2

Chapter_16.indd 15 6/5/2015 2:05:09 AM


16.16 Chapter 16

3 3 Also,  3 ( x - 6 ) = x + 16 - 6  or,  x = 14
5 x ⋅ x = 72 ( x + x)
2 2 \  Ram’s age = 14 years
and, Mohan’s age = 14 + 16 = 30 years.
15 2 5
⇒ x = 72 × x
2 2 20.
(d) Total age of eight people 10 years ago = 231 years

Total age of eight people 7 yr ago = 231 + 8 × 3 − 60 +
72 × 5 0 = 195.
⇒ x = = 24 years
15 Total age of eight people 4 yr ago = 195 + 3 × 8 − 60 +
i.e., age of one of colleagues lies between 23 and 26 0 = 159.
years. Current total age of eight people = 159 + 4 × 8 = 191
years
19.
(a) Let Ram’s age = x years
191
So, Mohan’s age = ( x + 16 ) years ∴  Average age = = 24 years (approximately).
8

Chapter_16.indd 16 6/5/2015 2:05:09 AM


CHAPTER

Simple Interest 17
INTRODUCTION
When A borrows some money from B, then A has to pay Interest can be of two types:
certain amount to B for the use of this money. This amount 1. Simple Interest
paid by A is called interest. The total amount of money
2. Compound Interest
borrowed by A from B is called the principal. The money
paid back to B, which comprises the principal and the
interest is called the amount. SIMPLE INTEREST
In other words, When the interest is payable on the principal only, it is
Amount = Principal + Interest called simple interest. For example, simple interest on `100
The interest is usually charged according to a specified at 5% per annum will be `5 each year, that is, at the end of
term, which is expressed as some per cent of the principal one year, total amount will be `105. At the end of second
and is called the rate of interest for the fixed period of year, it will be `110 and so on.
time. This fixed period may be a year, six months, three Thus, simple interest is the interest computed on the
months or a month and correspondingly the rate of interest principal for the entire period it is borrowed.
is charged annually, semi-annually, quarterly or monthly. In this chapter, we will limit ourselves to simple
For example, the rate of interest is 5% per annum means the interest. Compound interest will be discussed in the next
interest payable on `100 for one year is `5. chapter.

soMe Basic forMulae

If P stands for principal, R is the rate per cent 100 × Simple Interest
per annum, T is the number of years, I is the simple 2. Principal =
Rate × Time
interest and A is the amount, then
100 × I
Principal × Rate × Time or, P =
1. Simple Interest = R ×T
100
P × R ×T Illustration 2 A man earns `450 as an interest in 2 years
or, I =
100 on a certain sum invested with a company at the rate of 12
per cent per annum. Find the sum invested by the man in
Illustration 1 Find the simple interest on `5200 for 2 years the company.
at 6% per annum. Solution: We have I = `450, T = 2 years,
Solution: Here P = `5200, T = 2 years and R = 6% R = 12% per annum
P × R ×T 5200 × 6 × 2 I × 100 450 × 100
\ Simple interest = = \ P= = = `1875
100 100 R ×T 12 × 2
= `624 Thus, the money invested by the man was `1875

Chapter_17.indd 1 6/5/2015 2:40:54 AM


17.2 Chapter 17

Solution: Here P = `1200, I = `240, R = 5%


100 × Simple Interest 100 × I 100 × 240
3. Rate = \ T = = = 4 years
Principal × Time P×R 1200 × 5
100 × I
or, R =
P ×T
5. Amount = Principal + Simple Interest
Illustration 3 At what rate per annum will a sum of `5000 Principal × Rate × Time
= Principal +
amount to `6000 in 4 years? 100
Solution: Here P = `5000, A = `6000,  Rate × Time 
= Principal 1+ 
T = 4 years  100 
So, I = A – P = `(6000 – 5000) = `1000
 R ×T 
100 × I or, A = P 1+ 
\ R =  100 
P ×T
100 × 1000
= = 5% Illustration 5 Mahesh borrowed `3000 from his friend
5000 × 4
Suresh at 15% per annum for 3 years. Find the interest and
money returned by Mahesh to Suresh
100 × Simple Interest
4. Time = Solution: Here P = `3000, R = 15% per annum, T = 3 years
Rate × Principal
P × R ×T 3000 × 15 × 3
100 × I \ I = = = `1350
or, T = 100 100
R×P
\ A = P + I = `3000 + `1350 = `4350
Illustration 4 In what time will `1200 earn an interest of Thus, Mahesh paid `1350 as interest to Suresh and the
`240 at 5% per annum? amount returned by Mahesh to Suresh = `4350

soMe useful sHort-cut MetHoDs

Solution: We have, A = `570, R = 4% per annum,


1. If a certain sum in T years at R% per annum
7
amounts to `A, then the sum will be T= years
2
100 × A
P =
100 + R × T 100 × A 100 × 570
\ P = =
100 + R × T 100 + 4 × 7/2
Explanation
100 × 570
Let the principal be ` x = = `500
114
\ Simple interest = ` (A – x)
x × R ×T Thus, `500 will amount to `570 at 4% per annum in
\ A–x= 1
100 3 years
⇒ 100 A – 100 x = xRT 2
⇒ (100 + RT) x = 100 A
100 × A 2. The annual payment that will discharge a debt of
\ x= `A due in T years at R% per annum is
100 + R × T
 100 A 
Illustration 6 What principal will amount to `570 at 4% per Annual payment = ` 
RT (T − 1) 
1  100 T + 
annum in 3 years?  2 
2

Chapter_17.indd 2 6/5/2015 2:40:55 AM


Simple Interest 17.3

Explanation
3. If a certain sum is invested in n types of
Let the annual payment be `x. investments in such a manner that equal amount
Since the first instalment is paid at the end of first year, is obtained on each investment where interest
rates are R1, R2, R3, ..., Rn, respectively and time
\ Amount of first instalment at the end of t years
periods are T1, T2, T3, ..., Tn, respectively, then the
(T − 1) × R × x ratio in which the amounts are invested is
=x+
100 1 1
: :
Similarly, amount of second instalment at the end of t 100 + R1T1 100 + R2T2
years
1 1
(T − 2) × R × x : ... .
=x+ , and so on 100 + R3T3 100 + RnTn
100
Thus, total amount of T instalments
Explanation
 (T − 1) × R × x  Let P1, P2, ..., Pn be invested in n types of investments
A = x + 
 100 whose interest rates are R1, R2, ..., Rn and time periods are
(T − 2) × R × x  T1, T2, ..., Tn.

+ x +  + ... + x 100 × A
 100 Then, P1 =
100 + R1T1
Rx
= Tx + [(T − 1) + (T − 2) + ... + 1] 100 × A
100 P2 =
100 + R2T2
Rx  (T − 1) × T 
= Tx +   M 
100 2
100 × A
 T (T − 1)  Pn = .
or 100 Tx + Rx   = 100 A 100 + RnTn
 2
\ P1 : P2 : ... : Pn
 RT (T − 1) 
or x 100 T +  = 100 A 100 × A 100 × A 100 × A
 2 = : : ...
100 + R1T1 100 + R2T2 100 + RnTn
100 A
\ x= 1 1 1
RT (T − 1) = : : ...
100 T +
2 100 + R1T1 100 + R2T2 100 + RnTn
[ the amount A is same for all]
Illustration 7 Find the annual instalment that will discharge
a debt of `12900 due in 4 years at 5% per annum simple Illustration 8 A sum of `1586 is divided among three such
interest parts that amount obtained on these three parts of money
Solution: Here A = `12900, T = 4 years, R = 5% per annum after 2, 3 and 4 years, respectively at the rate of 5% per
annum remains equal. Find such three parts of the sum
100 × A Solution: Since the amount accured from each of the three
\ Annual instalment =
RT (T − 1) parts of `1586 at the rate of 5% p.a. in 2, 3 and 4 years,
100 × T +
2 respectively, remains equal, such three parts of `1586 will
100 × 12900 be in the ratio of
= 1 1 1
5(4 − 1) × 4 : :
(100 × 4) +
2 100 + R1T1 100 + R2T2 100 + R3T3
100 × 12900 100 × 12900 Hence, the ratio
= =
400 + 30 430 1 1 1
= : :
= `3000 100 + 5 × 2 100 + 5 × 3 100 + 5 × 4

Chapter_17.indd 3 6/5/2015 2:40:56 AM


17.4 Chapter 17

1 1 1 (2 − 1) × 100
= : : = years
110 115 120 8
1 × 30360 1 × 30360 1 × 30360 1
= : : = 12 years
110 115 120 2
( L.C.M. of 110, 115 and 120 is 30360)
\ ratio = 276:264:253 6. If a certain sum of money becomes n times itself
Sum of proportionals = 276 + 264 + 253 = 793 in T years at a simple interest, then the time T in
which it will become m times itself is given by
276
\ 1st part = × 1586 = `552  m −1
793 T’ =   × T years.
264  n −1 
2nd part = × 1586 = `528
793
253 Explanation
and, 3rd part = × 1586 = `506
793 Let the principal be `P.
Let it become m times in T ′ years.
4. If a certain sum of money becomes n times
Then, the amount in T years = `nP
itself in T years at simple interest, then the rate of
interest per annum is and the amount in T ′ years = `mP.
100(n − 1) P × R ×T
R= % \ nP – P =
T 100
P × R ×T
Explanation or, (n – 1) P = (1)
100
Let `P become `nP in t years
\ Simple interest I is given by P × R × T'
and, (m – 1) P = (2)
I = nP – P = (n – 1) P 100
\ Rate of interest R is given by (m − 1) P P × R × T' 100
\ = ×
100 × I 100 × (n − 1) P (n − 1) P 100 P × R ×T
R = =
P ×T P ×T m −1 T'
or, =
100 (n − 1) n −1 T
=
T
 m −1
Illustration 9 A certain sum of money trebles itself in \ T′ =   T years.
5 years simple interest. Find the rate per cent per annum  n −1 
Solution: Here n = 3, T = 5 years Illustration 11 A sum of money put out on simple interest
100 (n − 1) 100 (3 − 1) 1
\ R= %= % = 40% doubles itself in 12 years. In how many years would it
T 5 2
treble itself?
5. If a certain sum of money becomes n times itself
25
at R% per annum simple interest in T years, then Solution: Here, n = 2, m = 3, T = years.
2
 n −1 
T=   × 100 years.  m −1
 R  \ Required time (T ′) =   × T years
 n −1 
Illustration 10 In what time a sum of money will double
 3 − 1  25
itself at a rate of simple interest of 8% per annum? =  × years
(n − 1) × 100  2 −1 2
Solution: Required time (T) = years
R = 25 years

Chapter_17.indd 4 6/5/2015 2:40:57 AM


Simple Interest 17.5

7. Effect of change of P, R and T on simple interest P


We have 80 = × (4 × 4 –5 × 3)
is given by the following formula: 100
Change in Simple Interest ⇒ P = `8000
Product of fixed parameter
= 8. If a debt of `Z is paid in ‘n’ number of instalments
100 and if the value of each instalment is `a, then the
× [difference of product of variable parameters] borrowed (debt) amount is given by
For example, if rate (R) changes from R1 to R2 and Ra n (n − 1)
P, T are fixed, then Z = na + ×
100 × b 2
PT
Change in SI = × ( R1 − R2 ) where R = rate of interest per annum
100
b = no. of instalments/year
Similarly, if principal (P) changes from P1
to P2 and R, T are fixed, then change in SI b = 1, when each instalment is paid yearly
RT b = 2, when each instalment is paid half-yearly
= × ( P1 − P2 ) b = 4, when each instalment is paid quarterly
100
b = 12, when each instalment is paid monthly.
Also, if rate (R) changes from R1 to R2 and time
(T) changes from T1 to T2 but principal (P) is
P Illustration 15 A sum of `2 is lent to be paid back in
fixed, then change in SI = × ( R1T1 − R2T2 ). 3 equal monthly instalments of Re. 1 each. Find the rate
100
per cent
Solution: Here Z = `2, a = Re. 1, n = 3, b = 12, R = ?
Illustration 12 If simple interest on `600 increases by `30,
Using the formula
when the rate % increases by 4% per annum, find the time
Ra n (n − 1)
Solution: Here P = 600, change in SI = 30, R1 – R2 = 4, T Z = na + × ,
=? 100 × b 2
PT R ×1 3× 2
Using, change in SI = × ( R1 − R2 ) we have 2 = 3 × 1 + × ⇒ R = 400%
100 100 × 12 2
600 T 5 1 \ The rate % p.a. is 400%
We have, 30 = × 4 ⇒ T = , i.e., 1 years
100 4 4
9. If a certain sum of money P lent out at SI amounts
Illustration 13 If the simple interest on `1400 be more to A1 in T1 years and to A2 in T2 years, then
than the interest on `1000 by `60 in 5 years, find the rate
A1T2 − A2T1
per cent per annum P =
T2 − T1
Solution: Here change in SI = 60, P1 – P2 = 400, T = 5,
R=? A1 − A2
and R = × 100%
RT A1T2 − A2T1
Using change in SI = × ( P1 − P2 )
100
5R Illustration 16 If a certain sum of money at simple interest
We have 60 = × 400 ⇒ R = 3% amounts to `5184 in 2 years and to `5832 in 3 years, what
100
is the sum and the rate of interest?
Illustration 14 If the simple interest on a certain sum at A1T2 − A2T1
4% per annum for 4 years is `80 more than the interest on Solution: Principal =
T2 − T1
the same sum for 3 years at 5% per annum, find the sum
Solution: Here change in SI = 80, R1 = 4, R2 = 5, T1 = 4,  Here
= A1 5184,
= A2 5832 
T1 2,=
= T2 3 
T2 = 3, P = ?
P 5184 × 3 − 5832 × 2
Using change in SI = × ( R1T1 − R2T2 ) = = `3888
100 3−2

Chapter_17.indd 5 6/5/2015 2:40:58 AM


17.6 Chapter 17

(A2 − A1 ) × 100 (5832 − 5184) × 100 5000 × 2 + 2000 × 4 4


and, Rate = = We get, R = =2 %
T1 A2 − T2 A1 2 × 5832 − 3 × 5184 5000 + 2000 7
64800 2
= = 16 %
3888 3 12. If a certain sum of money is lent out in n parts
in such a manner that equal sum of money is
10. If a certain sum of money P lent out for a certain obtained as simple interest on each part where
time T amounts to A1 at R1% per annum and to A2 interest rates are R1, R2, ..., Rn, respectively and
at R2% per annum, then time periods are T1, T2, ..., Tn, respectively, then
the ratio in which the sum will be divided in n
A2 R1 − A1R2
P= parts is given by
R1 − R2
1 1 1
A1 − A2 : :...
and T= × 100 years R1T1 R2T2 RnTn
A2 R1 − A1R2
Explanation
Illustration 17 A certain sum is invested for certain time.
Let the n equal parts be P1, P2, ..., Pn and let I be the equal
It amounts to `450 at 7% per annum. But when invested at
interest earned on each part.
5% per annum, it amounts to `350. Find the sum and time
I × 100
Solution: Here A1 = 450, R1 = 7, A2 = 350, R2 = 5 Then, P1 =
R1T1
Using the formula,
A2 R1 − A1R2 I × 100
P = P2 =
R1 − R2 R2T2
350 × 7 − 450 × 5  
We get, P = = `100
7−5 I × 100
PN =
Also, using the formula, RnTn

 A1 − A2  I × 100 I × 100 I × 100


T=   × 100 ∴ P1 : P2 : ... : PN = : : ...
R1T1 R2T2 RnTn
 A2 R1 − A1R2 
 450 − 350  1 1 1
We get, T =  = : :...
 × 100 = 5 years R1T1 R2T2 RnTn
 350 × 7 − 450 × 5 
Illustration 19 If a sum of `1600 is divided into two such
11. If an amount P1 lent at simple interest rate of parts that the simple interest on the first part for two and half
R1% per annum and another amount P2 at simple years at the rate of 4% p.a. equals the simple interest on the
interest rate of R2% per annum, then the rate of second part for 5 years at the rate of 3% p.a., then find two
interest for the whole sum is such divisions of the sum
 P R + P2 R2  Solution: Ratio of one part to other part of `1600
R=  1 1 .
 P1 + P2  1 1
= :
R1T1 R2T2
Illustration 18 Mohan deposits `5000 in NSC at 2% per 1 1
annum and `2000 in mutual funds at 4% per annum. Find \ 1st part : 2nd part = :
4 × 5/2 3 × 5
the rate of interest for the whole sum
Solution: Here P1 = 5000, R1 = 2, P2 = 2000, R2 = 4 5
[Here R1 = 4% p.a., T1 = years, R2 = 3% p.a.,
Using the formula 2
T2 = 5 years]
 P R + P2 R2 
R =  1 1  1 1
or, 1st part : 2nd part = : =3:2
 P1 + P2  10 15

Chapter_17.indd 6 6/5/2015 2:40:59 AM


Simple Interest 17.7

Sum of proportionals = 3 + 2 = 5 1
14. Out of a certain sum P, part is invested at R1%
3 a
\ 1st part = × 1600 = `96 1  1 1
5 part at R2% and the remainder 1 − −  say
2 b  a b
and,   2nd part = × 1600 = `640 1
5 part at R3% If the annual income from all these
  13. When there is a change in principal (P), Rate (R) c
and Time (T), then the value of simple interest I also investments is `A, then the original sum is given by
changes and is given by  A × 100 
P=  
I1 P × R ×T R R R
= 1 1 1  1+ 2 + 3 
I2 P2 × R2 × T2  a b c 
A1 − P1 P × R ×T Illustration 21 Out of a certain sum, one-third is invested at
⇒ = 1 1 1
A2 − P2 P2 × R2 × T2 3% one-sixth at 6% and the rest at 8% If the annual income
is `300, then the original sum is
as, I1 = A1 – P1 and I2 = A2 – P2
1 1 1 1
Illustration 20 If `85 amounts to `95 in 3 years, what Solution: Here, = , = ,
a 3 b 6
`102 will amount to in 5 years at the same rate per cent?
Solution: Here P1 = `85, A1 = `95, T1 = 3 years, 1 1 1 1
= 1 –  +  = ,
P2 = `102, T2 = 5 years, R1 = R2 = R (say) c 3 6 2
Then, using the formula R1 = 3% R2 = 6% R3 = 8% A = `300.
A1 − P1 P × R ×T
= 1 1 1 A × 100
A2 − P2 P2 × R2 × T2 \ The original sum =
R1 R2 R3
+ +
95 − 85 85 × R × 3 a b c
We have, =
A2 − 102 102 × R × 5 300 × 100 30000
= =
⇒ A2 – 102 = 20 3 6 8 6
+ +
⇒ A2 = 122 3 6 2
\ The amount is `122 = `5000

Practice Exercises
Difficulty Level-1
(Based on Memory)

1. A sum of money invested at simple interest triples itself 1


in 8 years. How many times will it become in 20 years 3. Two equal sums of money were invested, one at 4 %
2
time?
and the other at 4% At the end of 7 years, the simple
(a) 8 times (b) 7 times interest received from the latter exceeded that received
(c) 6 times (d) None of these from the former by `31.50. Each sum was:
(a) `1,000 (b) `500
2. Ravi gave `1200 on loan. Some amount he gave at 4% per (c) `750 (d) `900
annum simple interest and remaining at 5% per annum [Based on MAT, 2001]
simple interest. After two years, he got `110 as interest.
Then the amounts given at 4% and 5% per annum simple 4.
I derive an annual income of `688.25 from `10,000
interest are respectively, invested partly at 8% p.a. and partly at 5% p.a. simple
interest. How much of my money is invested at 5%
(a) `500, `700 (b) `400, `800
(a) `3,725 (b) `4225
(c) `800, `300 (d) `1100, `1100 (c) `4,800 (d) `5,000
[Based on MAT, 2003] [Based on MAT, 2000]

Chapter_17.indd 7 6/5/2015 2:41:00 AM


17.8 Chapter 17

5. In 4 years, the simple interest on a certain sum of money 14.


`1,500 is invested at a rate of 10% simple interest and
is 7/25 of the principal. The annual rate of interest is: interest is added to the principal after every 5 years. In
(a) 4% (b) 4.5% how many years will it amount to `2,500?
(c) 7% (d) 9% 1 1
(a) 6 years (b) 6 years
[Based on SNAP, 2007] 9 4
6. Divide `6,000 into two parts so that simple interst on the (c) 7 years (d) None of these
first part for 2 years at 6% per annum may be equal to
simple interest on the second part for 3 years at 8% per 15.
If `1000 be invested at interest rate of 5% and the interest
annum. be added to the principal after 10 year, then the number of
(a) `4,000, `2,000 (b) `5,000, `1,000 years in which it will amount to `2000 is:
(c) `3,000, `3,000 (d) None of these 2 1
(a) 16 years (b) 16 years
3 4
7.
`25000 amount of `2600 in 5 years at simple interest. If
the interest rate were increased by 3% it would amount to (c) 16 years (d) 11 years
how much? [Based on MAT (Sept), 2010]
(a) `2900 (b) `3200 16.
A sum of `7700 is to be divided among three brothers
(c) `3600 (d) None of these Sunil, Sumant and Surat in such a way that simple interest
on each part at 5% per annum after 1, 2 and 3 year
[Based on I.P. Univ., 2002]
respectively remains equal. The share of Sunil is more
8.
A sum of `4000 is lent out in two parts, one at 8% simple than that of Surat by:
interest and other at 10% simple interest. If the annual
(a) `2800 (b) `2500
interest is `352, the sum lent at 8% is:
(a) `1600 (b) `2400 (c) `3000 (d) `2700
[Based on MAT (Sept), 2010]
(c) `1800 (d) `2800
[Based on MAT, 2005] 17.
A person invested some amount at the rate of 12% simple
interest and a certain amount at the rate of 10% simple
9. If the rate of simple interest is 12% per annum, the amount
interest. He received yearly interest of `130. But if he
that would fetch interest of `6,000 per annum is:
had interchanged the amounts invested, he would have
(a) `7,200 (b) `48,000 received `4 more as interest. How much did he invest at
(c) `50,000 (d) `72,000 12% simple interest?
10. A sum of `600 amounts to `720 in 4 years at simple (a) `700 (b) `500
interest. What will it amount to if the rate of interest is (c) `800 (d) `400
increased by 2%
[Based on MAT (Feb), 2010]
(a) `648 (b) `768
18.
A sum was put at simple interest at a certain rate for 3
(c) `726 (d) `792 years. Had it been put at 1% higher rate, it would have
11. A person takes a loan of `200 at 5% simple interest. He fetched `5100 more. The sum is:
returns `100 at the end of one year. In order to clear his (a) `170000 (b) `150000
dues at the end of 2 years, he would pay: (c) `125000 (d) `120000
(a) `125.50 (b) `110 [Based on MAT (Feb), 2010]
(c) `115.50 (d) None of these 19.
Prem invested a certain sum of money in a simple interest
12. `2,189 are divided into three parts such that interest on bond whose value grew to `300 at the end of 3 years and
them after 1, 2 and 3 years, respectively may be equal, the to `400 at the end of another 5 years. What was the rate of
rate of simple interest being 4% per annum in all cases. interest in which he invested his sum?
The smallest part is: (a) 12% (b) 12.5%
(a) `702 (b) `398 (c) 6.67% (d) 8.33%
(c) `756 (d) `1,093 [Based on MAT (Feb), 2010]

13. What should be the least number of years in which 20.


A money lender lent out `25000 in two parts, one at 8%
2 and the other at 8.5% If the total annual income on the
the simple interest on `2,600 at 6 % will be an exact amount is `2031.25, the money lent at 8% is:
3
number of rupees? (a) `12500 (b) `6250
(a) 2 (b) 3 (c) `10000 (d) `18750
(c) 4 (d) 5 [Based on MAT (Feb), 2010]

Chapter_17.indd 8 6/5/2015 2:41:00 AM


Simple Interest 17.9

21.
A man wants to sell his scooter. There are two offers, (a) I and III are correct (b) II alone is correct
one at `12000 cash and the other at a credit of `12880 to (c) III alone is correct (d) II and III are correct
be paid after 8 months, money being at 18% per annum.
[Based on MAT (Dec), 2006]
Which is the better offer?
(a) `12000 in cash (b) `12880 at credit 28.
A man divided his share to his sons A and B in such a
(c) Both are equal (d) None of these way that the interest received by A at 15% per annum for
3 years is double the interest received by B at 12% per
[Based on MAT (Feb), 2009]
annum for 5 years. At what ratio was his share divided?
22.
A trader owes a merchant `10028 due 1 year, hence
the trader wants to settle the account after 3 months. (a) 2/3 (b) 8/3
If the rate of interest is 12% per annum, how much cash (c) 3/8 (d) 3/2
should he pay? [Based on MAT (May), 2010]
(a) `9025 (b) `9200
29.
Divide `1586 in three parts in such a way that their
(c) `9600 (d) `9200
amounts at the end of 2, 3 and 4 years, respectively, at 5%
[Based on MAT (Feb), 2009]
per annum simple interest be equal:
23.
Asmita invests an amount of `9535 at the rate of 4% per
(a) `552, `528, `506 (b) `560, `520, `506
annum, for how many years did she invest the amount to
obtain the double her sum? (c) `556, `524, `506 (d) `548, `528, `510
(a) 10 years (b) 25 years [Based on MAT (Dec), 2006]
(c) 5 years (d) 4 years 30.
If a certain sum of money becomes double at simple
[Based on MAT (May), 2009] interest in 12 years, what would be the rate of interest per
24.
Anu owes Biresh `1120 payable 2 years hence, Biresh annum?
owes Anu `1081.50 payable 6 months. If they decide to 1
settle their accounts forthwith by payment of ready money (a) 8 (b) 10
2
and the rate of interest be 6% per annum, then who should
(c) 12 (d) 14
pay and how much?
[Based on MAT, 1998]
(a) Anu, `70 (b) Biresh, `50
(c) Anu, `50 (d) Biresh, `70 31.
Two equal sums were borrowed at 8% simple interest per
annum for 2 years and 3 years respectively. The difference
[Based on MAT (Sept), 2008]
in the interests was `56. The sums borrowed were:
25.
The present worth of bill due 7 months, hence is `1200.
(a) `690 (b) `700
If the bill were due at the end of two and half years its present
worth would be `1016. The rate per cent of the bill is: (c) `740 (d) `780
(a) 16% (b) 8% [Based on MAT, 1998]
(c) 10% (d) 18% 32.
What price should a shopkeeper mark on an article costing
[Based on MAT (Sept), 2008] him `153 to gain 20% after allowing a discount of 15%
26.
A owes B `1573, payable one and half years hence. Also (a) `162 (b) `184
B owes A `1444.50, payable 6 months hence. If they want (c) `216 (d) `224
to settle the account forthwith, keeping 14% as the rate of [Based on MAT, 1999]
interest, then who should pay whom and how much?
33.
A lent `600 to B for 2 years and `150 to C for 4 years and
(a) A to B, `28.50 (b) B to A, `37.50
received altogether from both `90 as simple interest. The
(c) A to B, `50 (d) B to A, `50 rate of interest is:
[Based on MAT (Dec), 2006]
(a) 4% (b) 5%
27.
Consider the following statements:
(c) 10% (d) 12%
If a sum of money is lent at simple interest, then the [Based on MAT, 2000]
I. money gets doubled in 5 years, if the rate of interest
34.
The rate of interest on a sum of money is 4% per annum
2 for the first 2 years, 6% per annum for the next 4 years and
is 16 %
3 8% per annum for the period beyond 6 years. If the simple
II. money gets doubled in 5 years, if the rate of interest is interest accrued by the sum for a total period of 9 years is
20% `1,120, what is the sum?
III. money becomes four times in 10 years, if it gets (a) `1,500 (b) `2,000
doubled in 5 years. (c) `2,500 (d) `4,000
Of these statements [Based on MAT, 2000]

Chapter_17.indd 9 6/5/2015 2:41:01 AM


17.10 Chapter 17

35.
A sum was put at simple interest at a certain rate for 2 42. An automobile financier claims to be lending money
years. Had it been put at 1% higher rate, it would have at simple interest but he includes the interest every six
fetched `24 more. The sum is: months for calculating the principal. If he is charging an
(a) `600 (b) `800 interest of 10% the effective rate of interest becomes:
(c) `1,200 (d) `480 (a) 10% (b) 10.25%
[Based on MAT, 2000] (c) 10.5% (d) None of these
36.
A sum of `2540 is lent out into two parts, one at 12% [Based on MAT, 2012]
and another one at 12.5% If the total annual income is 43. Harris invested `4000 in two different ventures A and B.
`311.60, the lent money at 12% The yearly return on B was 12% and the yearly return on
(a) `1,180 (b) `1,360 A was 8% If the total return was `4000, how much did
(c) `1,240 (d) `1,340 Harris invest in B?
[Based on MAT, 2000] (a) `8000 (b) `20000
(c) `14000 (d) `22000
37.
I derive an annual income of `688.25 from `10,000
invested partly at 8% pa and partly at 5% pa simple [Based on MAT, 2013]
interest. How much of my money is invested at 5% 44. The simple interest on a sum of money is 1/9 of the
(a) `3,725 (b) `4,225 principal and the number of years is equal to the rate
(c) `4,800 (d) `5,000 percent per annum. The rate percent per annum is:
[Based on MAT, 2000] (a) 3% (b) 0.33%
38.
A certain sum is invested for T years. It amounts to `400 (b) 3.33% (d) 2.3%
at 10% per annum. But when invested at 4% per annum, it [Based on MAT, 2013]
amounts to `200. Find the time (T). 45. Ashok borrows `1500 from two moneylenders. He pays
(a) 41 years (b) 39 years interest at the rate of 12% per annum for one loan and at
(c) 50 years (d) None of these the rate of 14% per annum for the other. The total interest
he pays for the entire year is `186. How much does he
39. Muan received `12000 as puja Bonus. He invested a
borrow at the rate of 12%?
part of it at 5% per annum and the remaining at 6% per
annum, simple interest being allowed in each case. The (a) `1200 (b) `1300
total interest earned by him in 4 years is `2580. The sum (c) `1400 (d) `1000
invested at 5% per annum is: [Based on MAT, 2014]
(a) `7500 (b) `4500 46. Divide `6000 into two parts, so that simple interest on the
(c) `4000 (d) `8000 first part for 2 years at 6% per annum may be equal to the
[Based on MAT, 2011] simple interest on the second part for 3 years at 8% per
annum?
40. A money-lender, lends a part of his money at 10%
per annum and the rest at 15% per annum. His annual (a) `4000, `2000
income is `1900. However, if he had interchanged the (b) `5000, `1000
rate of his interest on the two sums, he would have (c) `3000, `3000
earned `200 more. The amount lent will fetch what
amount at 15% (d) None of these
(a) `6000 (b) `4000 [Based on MAT, 2014]

(c) `10000 (d) `4400 47. A part of `38800 is lent out at 6% per six month. The rest
[Based on MAT (Feb), 2012] of the amount is lent out at 5% per annum after 1 years.
The ratio of interest after 3 years from the time when first
41. What will be the ratio of simple interest earned by a
amount was lent out is 5:4. Find the second part that was
cartain amount at the same rate of interest for 6 years and
lent out at 5%?
9 years?
(a) `26600 (b) `28800
(a) 1 : 3 (b) 1 : 4
(c) 2 : 3 (d) Data inadequate (c) `7500 (d) `28000
[Based on MAT, 2012] [Based on MAT, 2014]

Chapter_17.indd 10 6/5/2015 2:41:01 AM


Simple Interest 17.11

Difficulty Level-2
(Based on Memory)

1. A certain sum amounts to `2,300 in 3 years and `2,500 7. A bicycle can be purchased on cash payment of `1,500.
in 5 years at simple interest. Find the sum and the rate of The same bicycle can also be purchased at the down
interest. payment (initial payment, at the time of purchasing) of
(a) `1200, 6% (b) `1800, 5% `350 and rest can be paid in 3 equal installments of `400
for next 3 months. The rate of SI per annum charged by
(c) `2000, 5% (d) `1500, 6%
the dealer is:
[Based on IIT Joint Man. Ent. Test, 2004]
9 9
2.
Mr Anand deposited a total amount of `65000 in three (a) 23 % (b) 17 %
17 23
different schemes A, B and C with rates of interest
12% per annum, 16% per annum and 18% per annum, 9
(c) 13 % (d) None of these
respectively and earned a total interest of `10,180 in one 17
year. If the amount invested in Scheme A was 72% of the 8. An article costing `9,000 is sold at a discount which is
amount invested in Scheme C, then what was the amount equal to the simple interest on `3,000 for N months. Find
invested in Scheme B? N if the rate of discount is same as the rate of interest.
(a) `25000 (b) `22000
(a) 4 months (b) 6 months
(c) `18000 (d) Cannot be determined,
(c) 5 months (d) 36 months
[Based on Based on IRMA, 2002]
3. The simple interest on a certain sum at 5% for 9 months 9. Pratibha invests an amount of `15,860 in the names of her
is `10 greater than the simple interest on the same sum at three daughters A, B and C in such a way that they get the
the rate of 3% for 14 months. What is the sum of interest same interest after 2, 3 and 4 years, respectively. If the rate
in both the cases (i.e., total sum of interest)? of simple interest is 5% p.a., then the ratio of the amounts
invested among A, B and C will be:
(a) `130 (b) `290
(c) `120 (d) `330 1 1 1 1 1 1
(a) : : (b) : :
15 10 20 10 15 20
4. The simple interest on a sum of money is one-ninth of the
1 1
sum. The number of years is numerically equal to the rate (c) : :10 (d) None of these
per cent per annum. The rate per cent is: 15 20
1 10. If x is the simple interest on y and y is the simple interest
(a) 3 (b) 5
3 on z, the rate per cent and the time being the same in both
cases, what is the relation between x, y and z?
2
(c) 6 (d) 10 (a) x2 = yz (b) y2 = xz
3
(c) z2 = xy (d) xyz = 1
5. The rates of simple interest in two banks A and B are in
the ratio 5:4. A person wants to deposit his total savings in 11. A sum of `18,750 is left by will by a father to be divided
two banks in such a way that he recieves equal half yearly between two sons, 12 and 14 years of age, so that when
interest from both. He should deposit the savings in banks they attain maturity at 18, the amount (principal + interest)
A and B in the ratio: received by each at 5% simple interest will be the same.
(a) 2:5 (b) 4:5 Find the sum allotted at present to each son.
(c) 5:2 (d) 5:4 (a) `9,500, `9,250 (b) `8,000, `1,750
6. Some amount was lent at 6% per annum simple interest. (c) `9,000, `9,750 (d) None of these
After 1 year, `6,800 is repaid and the rest of the amount 12.
A sum of `1,440 is lent out in three parts in such a way
is repaid at 5% per annum. If the second year’s interest is that the interest on first part at 2% for 3 years, second part
11 at 3% for 4 years and third part at 4% for 5 years are equal.
of the first year’s interest, find what amount of money
20 Then the difference between the largest and the smallest
was lent out. sum is:
(a) `17,000 (b) `16,800
(a) `400 (b) `560
(c) `16,500 (d) `17,500
(c) `460 (d) `200

Chapter_17.indd 11 6/5/2015 2:41:01 AM


17.12 Chapter 17

13. Arun borrowed a sum of money from Jayant at the rate of interest accrued by the sum for a total period of 9 years is
8% per annum simple interest for the first four years, 10% `1,120, then the sum is:
per annum for the next 6 years and 12% per annum for the (a) `2,400 (b) `2,200
period beyond 10 years. If he pays a total of `12,160 as (c) `2,000 (d) None of these
interest only at the end of 15 years, how much money did
he borrow? 20.
Brinda borrowed `1,000 to build a hut. She pays 5%
(a) `12,000 (b) `10,000 simple interest. She lets the hut to Ramu and receives the
1
(c) `8,000 (d) `9,000 rent of ` 12 per month from Ramu. In how many years
2
14. Two equal sums of money are lent at the same time at Brinda ought to clear off the debt?
8% and 7% per annum simple interest. The former is
1
recovered 6 months earlier than the later and the amount (a) 10 years (b) 10 years
in each case is `2,560. The sum and the time for which the 4
sums of money are lent out are: 1
(c) 10 years (d) None of these
(a) `1,500, 3.5 years and 4 years 2
(b) `2,000, 3.5 years and 4 years
21.
Sumit lent some money to Mohit at 5% per annum simple
(c) `2,000, 4 years and 5.5 years interest. Mohit lent the entire amount to Birju on the same
(d) `3,000, 4 years and 4.5 years 1
day at 8 % per annum. In this transaction after a year
15.
Subbarao was approached by two neighbours for loan. He 2
had `2,540, a part of which he lent to one person at 12% Mohit earned a profit of `350. Find the sum of money lent
interest per annum, and the other part was lent to the by Sumit to Mohit.
second person at 12.5% At the end of a year, Subbarao (a) `9,000 (b) `10,000
received `311.60 as interest on the total loan. Calculate (c) `10,200 (d) None of these
the amount of money lent by him at 12% interest.
(a) `1,360 (b) `1,340 22.
The simple interest on a sum of money will be `600 after
10 years. If the principal is trebled after 5 years, what will
(c) `1,240 (d) `1,180
be the total interest at the end of the tenth year?
16.
Vikram borrowed `6,450 at 5 per cent simple interest (a) `1,200 (b) `1,190
repayable in 4 equal instalments. What will be the annual (c) `1,210 (d) None of these
instalment payable by him?
(a) `1,710 (b) `1,810 23.
A man purchased 40 fruits; apples and oranges for `17.
(c) `1,910 (d) `1,860 Had he purchased as many oranges as apples and as many
apples as oranges, he would have paid `15. Find the cost
17. A person closes his account in an investment scheme by of one pair of an apple and an orange,
withdrawing `10,000. One year ago, he had withdrawn (a) 70 paise (b) 60 paise
`6,000. Two years ago he had withdrawn `5,000. Three
(c) 80 paise (d) 1 rupee
years ago he had not withdrawn any money. How much
money had he deposited approximately at the time of [Based on SNAP, 2007]
opening the account 4 years ago, if the annual simple 24.
A man earns 6% SI on his deposits in Bank A while he
interest is 10% earns 8% simple interest on his deposits in the Bank.
(a) `15,600 (b) `16,500 If the total interest he earns is `1800 in three years on
(c) `17,280 (d) None of these an investment M of `9000, what is the amount invested
at 6%
18. Two equal sums of money were invested, one at 4%
1 (a) `3000 (b) `6000
and the other at 4 % At the end of 7 years, the simple
2 (c) `4000 (d) `4500
interest received from the latter exceeded that received [Based on SNAP, 2007]
from the former by `31.50. Each sum was: 25.
In 4 years, `6000 amounts to `8000. In what time at the
(a) `1,000 (b) `500 same rate will `525 amount to `700?
(c) `750 (d) `900 (a) 2 years (b) 3 years
19.
The rate of interest on a sum of money is 4% per annum (c) 4 years (d) 5 years
for the first 2 years, 6% per annum for the next 4 years and [Based on SNAP, 2009]
8% per annum for the period beyond 6 years. If the simple

Chapter_17.indd 12 6/5/2015 2:41:01 AM


Simple Interest 17.13

26.
A man invested one-third of his capital at 7%, one-fourth received by each at 5% simple interest will be the same.
at 8% and the remainder at 10% at simple interest. If his Find the sum allotted at present to each son.
annual income is `561, the capital is: (a) `9500, `9250 (b) `8000, `1750
(a) `5400 (b) `6000 (c) `9000, `9750 (d) None of these
(c) `6600 (d) `7200 [Based on NMAT, 2005]
[Based on FMS (MS), 2006] 33. Ms. Rao paid equated monthly installments (EMIs) of
`25000 each in January and February towards her home
27. A part of `9600 is invested at a 5% annual return, while
loan, whose outstanding principal amount was `1000000
the remainder is invested at a 3% annual return. If the
in December. Each EMI consists of interest of outstanding
annual income from both portions is the same, what is the
loan amount for the month and part payment of the loan
total income from the two investments?
amount. If the interest on the loan is 12% per annum
(a) `380 (b) `320 (interest is paid monthly) on the diminishing outstanding
(c) `440 (d) None of these in January and February was:
[Based on MHT-CET MBA, 2010] (a) `30150 (b) `20000
(c) `19850 (d) `19700
28.
Mr Mani invested an amount of `12,000 at the simple
interest rate of 10% per annum and another amount at [Based on JMET, 2006]
the simple interest rate of 20% per annum. The total 34. A certain sum of money is invested at an interest rate of
interest earned at the end of one year on the total amount 5% per annum and a second sum, twice as large as the
invested became 14% per annum. Find the total amount first, is invested at 5.5% per annum. The total amount
invested. of interest earned from the two investments together is
(a) `20,000 (b) `20,800 `1000 per year and the interest is withdrawn every year.
The second sum invested is:
(c) `21,000 (d) None of these
(a) `6250 (b) `10500
29.
A sum of `7,700 is to be divided among three brothers (c) `12500 (d) `15000
Vikas, Vijay and Viraj in such a way that simple interest [Based on JMET, 2006]
on each part at 5% per annum after 1, 2 and 3 years,
respectively remains equal. The share of Vikas is more 35.
A man invests `3,965 in the names of his three daughters
than that of Viraj by: Neeta, Sita and Gita in such a way that they get the same
amount after 2, 3 and 4 years, respectively. If the rate of
(a) `2,800 (b) `2,500
interest is 5% per annum, then the amount invested for
(c) `3,000 (d) None of these Neeta, Sita and Gita is:
30. Arun borrowed a sum of money from Jayant at the rate (a) `1,380, (b) `1,330,
of 8% per annum simple interest of the first 4 years, 10% `1,320, `1,360,
per annum for the next 6 years and 12% per annum for the `1,265 `1,380
period beyond 10 years. If he pays a total of `12160 as (c) `1,265, (d) None of these
interest only at the end of 15 years, how much money did `1,320,
he borrow?
`1,340
(a) `12000 (b) `10000
(c) `8000 (d) `9000 36.
If the amount obtained by Mahesh by investing `1,500 for
[Based on NMAT, 2005] two and half years at the rate of 8% per annum is equal to
the amount obtained by Suresh by investing a certain sum
31. A sum of `1440 is lent out in three parts in such a way that for 2 years at 5% per annum simple interest, then the sum
the interests on first part at 2% for 3 years, second part at invested by Suresh is:
3% for 4 years and third part at 4% for 5 years are equal. (a) `1636 (b) `1,636
Than the difference between the largest and the smallest
(c) `1636 (d) None of these
sum is:
(a) `400 (b) `560 1
37. A xerox machine is available for `78000 cash or for 33 %
(c) `460 (d) `200 3
cash down payment and 11 equal monthly installments of
[Based on NMAT, 2005] `4900 each. What is the rate of interest charged?
32. A sum of `18750 is left by will by a father to be divided (a) 2.43% (b) 8.54%
between two sons, 12 and 14 years of age, so that when (c) 9% (d) 6.5%
they attain maturity at 18, the amount (principal + interest) [Based on MAT, 2013]

Chapter_17.indd 13 6/5/2015 2:41:02 AM


17.14 Chapter 17

Answer Keys
Difficulty Level-1

1. (c) 2. (a) 3. (d ) 4. (a) 5. (c) 6. (a) 7. (a) 8. (b) 9. (c) 10. (b) 11. (c) 12. (b) 13. (b)
14. (a) 15. (a) 16. (a) 17. (b) 18. (a) 19. (d ) 20. (d ) 21. (a) 22. (b) 23. (b) 24. (b) 25. (c) 26. (d )
27. (b) 28. (b) 29. (a) 30. (a) 31. (b) 32. (c) 33. (b) 34. (b) 35. (c) 36. (a) 37. (a) 38. (c) 39. (a)
40. (a) 41. (c) 42. (b) 43. (b) 44. (c) 45. (a) 46. (a) 47. (b)

Difficulty Level-2

1. (c) 2. (b) 3. (b) 4. (a) 5. (b) 6. (a) 7. (d ) 8. (d ) 9. (b) 10. (b) 11. (c) 12. (b) 13. (c)
14. (b) 15. (d ) 16. (b) 17. (d ) 18. (d ) 19. (c) 20. (a) 21. (b) 22. (a) 23. (c) 24. (b) 25. (c) 26. (c)
27. (d ) 28. (a) 29. (a) 30. (c) 31. (b) 32. (c) 33. (c) 34. (c) 35. (a) 36. (a) 37. (a)­­­

Explanatory Answers

Difficulty Level-1

P×8× R ⇒
3k = 11175
1.
(c) 2P = ⇒ R = 25%
100 ⇒
k = 3725.

P × 25 × 20 P × R ×T
S.I. = = 5P 5.
(c) Interest =
100 100
7 x × r4 × r
\ Amount = (5P + P) = 6P x =
25 100
Therefore, it will become 6 times in 20 years time.
7 x × 100
r = = 7%
2.
(a) Let the amount given at 4% per annum be `x 25 × 400
\ Amount given at 5% per annum = ` (1200 – x) 6.
(a) Let one part be `x and other = `(6,000 – x)

x × 4 × 2 (1200 − x) × 5 × 2 x×2×6 (6000 − x) × 3 × 8


⇒   + = 110 =
100 100 100 100


⇒ x = 500. 12x = 144000 – 24x
x = `4,000 and other part
3. (d) Let each sum be `x = (6000 – 4000) = `2,000.
1 P × R ×T
x×4 ×7 7. (a) I =
2 x×4×7 100
\ − = 31.50
100 100 2000 × 5
600 =
7x 1 63 100
⇒ × = R = 6%
100 2 2
New, R = 6 + 3 = 9%
⇒ x = 900.
200 × 9 × 5
4. (a) Let money invested at 5% be `k I =
100
k × 1 × 5 (10000 − k ) × 1 × 8 = 900
\ + = 688.25
100 100 Amount = P = I = 2000 + 900
⇒ 5k – 8k + 80000 = 68825 = `2900.

Chapter_17.indd 14 6/5/2015 2:41:02 AM


Simple Interest 17.15

8.
(b) Total interest on `4000 in 1 year = `352 \ Simple interest = `2,500 – 2,250 = `250
352 × 100 250 × 100 10
Average rate = = 8.8% \ Time (T) = = years
4000 × 1 2250 × 10 9
1.2:0.8 = 3.2
10 55 1
First part = `2400. Hence, total time = 5 + =   or  6 years.
9 9 9
P × 12 × 1
9.
(c) 6000 = 
100 1000 × 5 × 10
15.
(a) SI for 10 year = = ` 500
⇒ P = `50,000. 100
600 × 4 × R Now, P = `1500,
10.
(b) 120 =
100 A = `2000
120 \ SI = `500
⇒ R =
%
24 1500 × 5 × T
= 5% new rate = (5 + 2) = 7% \ 500 =
100
600 × 4 × 7
\ S.I. at the new rate = = `168 500 × 100 2
100 ⇒ T = = 6 year
Therefore, amount = `(600 + 168) = `768. 1500 × 5 3

11.
(c) Amount to be paid in first year 2
\ Total time = 16 year.
200 × 5 × 1 3
= + 200 = 210
100 16.
(a) Let Sunil, Sumant and Surat get x, y and z amount,
Amount left as a principal for the second year respectively.
= 210 – 100 = 110 x × 5 ×1 y ×5× 2 z ×5×3
= =
\ Amount paid in second year 100 100 100
110 × 5 × 1 ⇒ x = 2y = 32
= 110 + = 115.5.
100 ⇒ x:y:z = 6:3:2
12.
(b) Let the amount invested for 1 year, 2 years and 3 years
be x, y and z, respectively. 6−2
\ Required amount = × 7700 = `2800.
x ×1× 4 y×2×4 z × 3× 4 6+3+ 2
then, = =
100 100 100 17.
(b) Amount invested at 12% = `x
25 25 Amount invested at 10% = `x
\ x = 25K,  y = K ,  z = K
2 3
x × 12 × 1 y × 10 × 1
25 25 130 = +
x : y : z = 25 : : = 6:3:2 100 100
2 3
⇒ 13000 = 12x + 10y (1)
2
Smallest part = × 2189 = `398.
11 x × 10 × 1 y × 12 × 1
and, 134 = +
100 100
2600 × 20 × T 250 × T
13.
(b) S.I. = =
3 × 100 3 ⇒ 13400 = 10x + 12y (2)
To make simple interest in exact number of rupees t Solving Eqs. (1) and (2), we get
should be 3.
x = `500
14.
(a) The simple interest on `1,500 invested at a rate of So, amount invested at 12% is `500.
10% p.a. for 5 years is
1500 × 10 × 5 5100
= = ` 750 18.
(a) Simple interest for 1 year = = `1700
100 3
Now, principal after 5 years = `1,500 + 750 1% of sum = 17000
= `2,250. 1700 × 100
\ Sum = = `170000.
Also, final amount = `2,500. 1

Chapter_17.indd 15 6/5/2015 2:41:03 AM


17.16 Chapter 17

19.
(d) Let principle be P and rate of interest is r. 23.
(b) Let she invest for x year
Then, A sum will be double when interest is equal to
principal
P×r×3
+ P = 300 (1) 9535 × 4 × x
100 \ 9535 =
100
P×r ×8
and, + P = 400 (2) ⇒ 4x = 100
100
Subtracting Eq. (1) from Eq. (2), we get ⇒ x = 25 years.
P×r×5 24.
(b) Present worth of money for Anu
= 100
100 P×6×2
1120 – P = ⇒ P = `1000
\ P × r = 2000 100
From Eq. (1),
Present worth of money for Biresh
2000 × 3
+ P = 300 ⇒ P = `240 P × 6 ×1
100 1081.50 – P =
\ 240 × r = 2000 2 × 100
⇒ r = 8.33% ⇒ 108150 – 100P = 3P

20.
(d) Let the amount in lented at 8% be `x. ⇒ P = `1050
Then, amount lented at 8.5% be (25000 – x). \ Biresh should pay `50.
x × 8 × 1 (25000 − x) × 8.5 × 1 25.
(c) Let the rate of interest be r %
\ + = 2031.25
100 100
Then,
⇒ 8x + 212500 – 8.5x = 203125 1200 × r × 7
A = 1200 +
⇒ – 0.5x = – 9375 12 × 100
⇒ x = `18750.
⇒ A = 1200 + 7r
21.
(a) In Ist case, the amount is `12000 1016 × r × 2.5
In IInd case, let the present value of money be x Again, 1200 + 7r = 1016 +
100
x × 18 × 8 ⇒ 1200 + 7r = 1016 + 25.4r
\ + x = 12880
12 × 100
⇒ 18.4r = 184
⇒ 0.12x + x = 12880
184
12880 \ r = = 10%
⇒ x = = `11500 18.4
1.12
\ `12000 in cash is the better offer. 26.
(d) Let the present value of what A owes B be `x.
22.
(b) Let the present value of money be x x × 14 × 3
Then, x + 1573
=
x × 12 2 × 100
Then, + x = 10028
100
⇒ 0.12x + x = 10028 21
⇒ x + x = 1573
10028 100
⇒ x =
1.12
121x
This amount after 3 months ⇒ = 1573
100
10028
× 12 × 3
10028
= 1.12 + ⇒ x =
1573 × 100
= `1300
12 × 100 1.12 121
10028 × 3 10028

= + Let y be the present value of what B owes A.
1.12 × 100 1.12
10028 × 103 1 14
= = 9222.17 ≈ ` 9200. Then, y + y × × = `1444.50
112 2 100

Chapter_17.indd 16 6/5/2015 2:41:04 AM


Simple Interest 17.17

7 276
⇒ y + y = 1444.50 y = × 1586 = `528
100 793
1444.50 × 100 253
⇒ y = = `1350 and, z = × 1586 = `506
107 793
Hence, B pay `50 to A.
Hence, the required three parts are `552, `528 and
27.
(b) Let the sum of money lent be `P. `506.
Then, 30.
(a) Suppose sum = P, SI = P
P × 50 × 5 5
SI received after 5 years = = P t = 12 years
3 × 100 6
P × 100 1
5 11P Rate % = =8 .
Amount after 5 years = P + P = P × 12 2
6 6
Therefore, I is not correct 31.
(b) Let S be the sum.

P × 20 × 5 S × 8(3 − 2)
II.  SI after 5 years =
=P \ = 56 ⇒ S = `700.
100 100

Amount after 5 years = P + P = 2P 32.


(c) CP = `153
SP = 153 + 20% of 153
Therefore, II is correct
= 153 + 30.60 = `183.60
III.  Amount after 10 years at the rate of 20% Let marked price be `x
\ x – 15% of x = 183.60
P × 20 × 10
= P+ = P + 2P = 3P 85 x
100 ⇒ = 183.60 ⇒ x = 216.
100
Therefore, III is not correct
33.
(b) Suppose rate = r % pa
Hence, (b) is the correct option. In 1st case:
28.
(b) Let A and B received x and y amount, respectively. Sum (P) = `600; Time (T) = 2 years
600 × 2 × r
x × 15 × 3 y × 12 × 5 \ Simple Interest = ` = `12r
Then, = 2× 100
100 100
In 2nd case:
x 2 × 12 × 5 8
⇒ = = . Sum (P) = `150; Time (T) = 4 years
y 15 × 3 3
150 × 4 × r
\ Simple Interest = ` = `6r
29.
(a) Let the three parts be `x, `y and `z. 100

According to question, According to the question,


12r + 6r = 90
x×2×5 y × 3× 5
x + =y+ or, 18r = 90
100 100
\ r = 5%
z×4×5
=z+ 34.
(b) Suppose sum = `P
100
Total interest earned in 9 years
⇒ 1.1x = 1.15 = 1.2z
P×4×2 P×6×4 P×8×3
x 1.15 23 = `  + `  + `
⇒ = = 100 100 100
y 1.1 22

According to the question,
y 1.2 24
and, = = 8 P 24 P 24 P
z 1.15 23 + + = 1120
100 100 100
⇒ x : y : z = 276 : 264 : 253 or, 14P = 1120 × 25
276 = 28000
⇒ x = × 1586 = `552,
793 or, P = `2000.

Chapter_17.indd 17 6/5/2015 2:41:05 AM


17.18 Chapter 17

35.
(c) Suppose sum = `P 400 − 200
Simple interest at certain rate r% for 2 years = × 100
200 × 10 − 400 × 4
P×r×2 Pr
= ` =` 20000
100 50 =
400
Also, simple interest at (r + 1)% for 2 years
= 50 years.
P × (r + 1) × 2 P (r + 1)
= ` =` 39. (a) According to the question,
100 50
According to the question, x × 5 × 4 (12000 − x) × 6 × 4
+ = 2580
P (r + 1) Pr 100 100
– = 24
50 50 (where x = sum investment at 5% per annum)
P ⇒ 5 x + 72000 − 6 x = 64500
or, = 24
50
⇒ x = ` 7500.
\ P = `1200.
36.
(a) Suppose first part = `x 40. (a) Let the parts of money invested at 10% and 15% per
\ Second part = `(2540 – x) annum be P1 and P2 respectively.
Time (t) = 1 year P1 × 10 × 1 P2 × 15 × 1
∴ + = 1900
In 1st case: 100 100
Rate (r) = 12.5% ⇒ 10 P1 + 15 P2 = 190000
x × 12.5 × 1 ⇒ 2 P1 + 3P2 = 38000 (1)
\ Simple Interest = `
100 Also,
In 2nd case:
Rate (r) = 12% P1 × 15 × 1 P2 × 10 × 1
+ = 2100
(2540 − x) × 12 × 1 100 100
\ Simple Interest = ` 15 P1 + 10 P2 = 210000
100
3P1 + 2 P2 = 42000
According to the question,
12.5 x (2540 − x)12 On solving Eqs. (1) and (2), we get P2 = `6000.
+ = 311.60
100 100 41. (c) Ratio of simple interests earned by a certain amount at
or, 0.5x + 30480 = 31160 the same rate of interest for different years is equal to
the ratio of years. Therefore, ratio of simple interests
or, 0.5x = 680 = 6:9 = 2:3.
\ x = 1360 42. (b) Let principal amount = `100
Hence, the money lent at 12% 100 × 5 × 1
In first six months, SI = = `5
= `(2540 – 1360) 100
\  Amount after six months = `105
= `1180.
105 × 5 × 1
Now, SI for next six months = =` 5.25
37.
(a) Let money invested at 5% be `k. 100
\  Amount = `110.25
k × 1 × 5 (10000 − k ) × 1 × 8
+ = 688.25 Effective rate = 110.25 - 100 = 10.25%
100 100
⇒ 5k – 8k + 80000 = 68825 43. (b) Let the money invested by Harris in B = `x

⇒ 3k = 11175 Then, money invested in A = `(40000 – x)

⇒ k = 3725. We are given,


x × 12 × 1 (40000 − x) × 8 × 1
38.
(c) We have A1 = `400, A2 = `200, R1 = 10% R2 = 4% + = ` 4000
100 100
A1 − A2 12 x 320000 − 8 x
\ Time (T) = × 100 ⇒ + = 4000
A2 R1 − A1R2 100 100

Chapter_17.indd 18 6/5/2015 2:41:08 AM


Simple Interest 17.19

12 x + 320000 − 8 x 46. (a) Let first part of money = `x


⇒ = 4000 and second part of money = `(6000 − x)
100
⇒ 4 x + 320000 = 400000 We are given,
4 x = 400000 − 320000 x × 6 × 2 (6000 − x) × 8 × 3
=
80000 100 100
x= = 20000 ⇒ x = 12000 − 2 x
4
∴ Harris invested ` 20000 in B ⇒ 3 x = 12000
∴ x = 4000
44. (c) Let rate of interest = number of years = x
So, first part of money = `4000 and second part of
Also, if principal = P
money = `2000.
Then, 47. (b) Let the first amount lent out @ 6% = `x
P
SI = and the second amount lent out @ 5% = `(38800 − x)
9
P×R ×T Ratio of interest = 5:4
SI =
100 x × 6 × 6 (38800 − x) × 5 × 2
∴ : = 5:4
P P× x× x 100 100
⇒ =
9 100
x×6×6
100 5
⇒ = x2 ⇒ 100 =
9 (38800 − x) × 5 × 2 4
100 10 100
⇒ x2 = = = 3, 33
9 3 36 x 5
⇒ =
45. (a) Let the loan @ 12% = `x (38800 − x) × 10 4
and the loan @ 14% = `(1500 − x)
18 x 5
Interest for 1 years = `186 ⇒ =
(38800 − x) × 5 4
We are given,
x × 12 × 1 (1500 − x) × 14 × 1 ⇒ 72 x = 970000 − 25 x
+ = 186
100 100 ⇒ 97 x = 970000
⇒ 12 x + 21000 − 14 x = 18600
∴ x = 10000
⇒ 2 x = 2400
2400 ∴ So, second part of money that was lent out @ 5%
⇒ x=
2
= 38800 − 10000
∴ x = `1200
Hence, the loan @12% = `1200 = ` 28800

Difficulty Level-2

1.
(c) Let P be the sum and R be the rate % 2.
(b) Suppose Amount invested in scheme A = ` x
P × 3× R Amount invested in scheme B = `y
P+
\ = 2300 (1)
100 Amount invested in scheme C = `z

P ×5× R \ x + y + z = 65000 (1)


and, P + = 2500 (2)
100 18
x = 72% of z = z (2)
25
2 PR
(1) – (2) gives = 2000 ⇒ PR = 10000, which is 12x + 16y + 18z = 1018000
100
satisfied only by alternative (c).  i.e.,  6x + 8y + 9z = 509000 (3)

Chapter_17.indd 19 6/5/2015 2:41:11 AM


17.20 Chapter 17

Using (2) in (1) and (3), we get 11 3P


It is given that second year interest = ×
20 50
43
z + y = 65000 (4) 33P 53P − 340000
25 \ =
1000 100
108 340000
z + 8 y + 9 z = 509000 ⇒ P = = `17,000.
25 20
333 P × r × 20
⇒ z + 8 y = 509000 (5) 7.
(d) P =
25 100
From (4), we have (Interest = Amount – Principal)
344 r = 5% p.a.
+ 8 y = 520000 (6)
5z 8.
(d) Let R be the rate of interest.

\ (6) – (5) 900 × R
Discount =
11z 100
⇒ = 11000
25 3000 × R × T
11z = 275000 Interest =
100
z = 25000
900 × R
\ x = 18000, By the given information;
100
y = 22000. 3000 × R × T
=
P × 5 × 9 P × 14 × 3 100
3.
(b) − = 10
100 × 12 100 × 12 T = 3 years = 36 months.
⇒ P = `4,000 P ×2×5 P × 3× 5 P ×4×5
9. (b) 1 = 2 = 3
4000 100 100 100
Now, [5 × 9 + 14 × 3] = `290.
100 × 12 ⇒ 10P1 = 15P2 = 20P3
4. (a) 1 1 1
⇒ P1 : P2 : P3 = 30 : 20 : 15 = : :
5.
(b) Rate of interest = 5x and 4x 10 15 20
Let he invests `P in bank A and Q in bank B y × R ×T 100x
10.
(b) x=   \ RT = (1)
P × 5x × 1 Q × 4x × 1 100 y
then, =
2 × 100 2 × 100 z × R ×T 100 y
and, y =   \ RT = (2)
5P = 4Q or, P:Q = 4:5. 100 z

6.
(a) Let the amount of money lent out be `P. Equations (1) and (2)
100 x 100 y
P×6 3P = ⇒ y2 = xz.
then first year interest = =` y z
100 50
x × 5 × (18 − 2)
3P 53P 11.
(c) x + = (18750 – x)
Amount to be returned = P + =` s 100
50 50
5× 4
Amount returned = `6,800, + (18750 – x) ×
100
 53P  ⇒ x = `9,000
Balance amount =  − 6800 
 50  \ Other son gets = `9,750.

 53P  x×2×3 y × 3× 4 z×4×5


 − 6800  × 5 12.
(b) = =
 50  100 100 100
Second year interest =
100 ⇒ 3x = 6y = 10z = k
53P − 340000 k k k
 = ` ⇒ x = , y = , z =  x + y + z = 1440
1000 3 6 10

Chapter_17.indd 20 6/5/2015 2:41:12 AM


Simple Interest 17.21

k k k 17. (d) Suppose the person had deposited `x at the time of


+ + = 1440  ⇒  k = 2400 opening the account.
3 6 10
\ After one year, he had
k k 7k 7
\ − = = × 2400 = 560
3 10 30 30  x × 10 × 1  11x
` x + = ` 
8× 4 10 × 6 12 × 5  100  10
13.
(c) x × + x× + x× = 12160
100 100 100 After two years, he had
⇒ x = `8,000.
11x 11x 10 × 1  121x
14. (b) Let the sum be `x ` + × = (1)
 10 10 100  100
\ `x are lent at 8% for t years and `x are lent at 7%
 1 121x
for  t +  years After withdrawing `5,000 from ` , the balance =
 2 100
x×t ×8 121x − 500000
\ + x = 2560 (1) `
100 100

x + (2t + 1) × 7 After 3 years, he had


and + x = 2560 (2)
2 × 100 121x − 500000 121x − 500000 10 × 1
+ ×
Solving Eqs. (1) and (2), we get 100 100 100
x = `2,000, t = 3.5 years.
11(121x − 500000)
15. (d) Let the amount of money lent at 12% interest be `x. = (2)
1000
\ Amount of money lent at 12.5% interest
= `(2540 – x) After withdrawing `6,000 from amount (2), the
balance
x × 12 × 1 (2540 − x) 12.5 × 1
\ + = 311.60 1331x 
100 100 = `  − 11500 
 1000 
−0.5 x 31750
⇒ + = 311.60 \ After 4 years, he had
100 100
⇒ 0.5x = 31750 – 31160 = 590 1331x − 5500000  1331x − 5500000 
`  + 10% of `  
⇒ x = `1,180  1000   1000 
16.
(b) Let the annual instalment be `x 11 1331x 
= ` − 11500  (3)
Amount of `100 after 4 years 10  1000 
 100 × 5 × 4 
= ` 100 +  = `120 [After withdrawing `10,000 from amount (3), the
 100  balance = 0]
\ Present Value (P.V.) of `120 due after 4 years 11 1331x 
\ − 11500  – 10,000 = 0
= `100 10  1000 
Present Value (P.V.) of `x due after 4 years ⇒ x = `15,470.
100 5 18. (d) Let each sum be `x
= x = x
120 6 1
20 x×4 ×7
Similarly, P.V. of `x due after 3 years = x 2 x×4×7 7x 1 63
23 \ − = 31.50 ⇒ × =
100 100 100 2 2
10
P.V. of `x due after 2 years = x ⇒ x = 900
11
20 19.
(c) Let the sum be `x.
P.V. of `x due after 1 year = x
21
Hence x× 4× 2 x×6× 4 x×8×3
Given: + + = 1120
5 20 10 20 100 100 100
x+ x+ x+ x = 6450
6 23 11 21 112000
⇒ 56x = 112000  or,  x = = `2,000
⇒ x ≈ `1,810 56

Chapter_17.indd 21 6/5/2015 2:43:49 AM


17.22 I Chapter 17

20.
(a) Simple interest paid by Brinda on `1,000 for 1 year = 24.
(b) Suppose, the man invests x in 6% and y in 8%
1000 × 5 × 1 x + y = 9000 (1)
= `50.
100  x×6 y ×8
and, 3 ×  +  = 1800
Rent received by Brinda from Ramu in 1 year  100 100 
1 ⇒ 3x + 4y = 30000 (2)
= 12 × 12 = `150.
2 From Eqs. (1) and (2), we get
\ Net savings = `100. \ x = `6000.
Thus, Brinda will clear the debt of `1,000 in 10 years. 25.
(c) Let the rate of interest is ‘r’
6000 × r × 4
21.
(b) Let the sum of money lent by Sumit to Mohit be `x. \ = 2000
100
Then, simple interest paid by Mohit after 1 year
25
x × 5 ×1 5x ⇒ r = %
= =` . 3
100 100
Now, calculate for `525.
Also, the simple interest received by Mohit from Birju
after 1 year 26.
(c) Let capital be 12x.
17 \ 4x × 7% + 3x × 8% + 5x × 10% = 561
x× ×1
2 17 x x = 550
= =` .
100 200 \ 550 × 12 = `6600
5x 17 x
Given: + 350 = 27. (d) Let the annual amount investment at 5% and 3% be
100 200 `x and `(9600 – x), respectively.
5 x + 35000 17 x x × 5 ×1 (9600 − x) × 3 × 1
⇒  = Then, =
100 200 100 100
⇒ 5x = 28800 – 3x
⇒  1700x – 1000x = 7000000
⇒ 8x = `28800
or, 700x = 7000000
28800
⇒ x = `
7000000 8
or, x = = `10,000.
700 = `3600
Thus, the sum of money lent by Sumit to Mohit is So the total income
`10,000. 3600 × 5 × 1 (9600 − 3600) × 3 × 1
= +
22.
(a) Interest for 5 years on the sum = `300. 100 100
When the principal is tripled, the interest is also = `180 + `180 = `360.
tripled.
28.
(a) Here P1 = `12,000, R1 = 10%, P2 = ?, R2 = 20%,
\ Interest for another 5 years on this increased sum = R = 14%
`(300 × 3) = `900.
Therefore, using the formula
\ Total interest = `300 + `900 = `1,200.
P1R1 + P2 R2
23.
(c) Man buys x apples at m price and y oranges at n price, R =
P1 + P2
then,
x + y = 40 12000 × 10 + P2 × 20
mx + ny = 17 (1) We get, 14 =
12000 + P2
Also, nx + my = 15 (2)
or, P2 = `8,000.
From Eqs. (1) and (2),
(m + n) (x + y) = 17 + 15 \ Total amount invested

32 = ` (12000 + 8000)
⇒  (m + n) = = `0.80 = 80 paise.
40 = `20,000.

Chapter_17.indd 22 6/5/2015 2:41:14 AM


Simple Interest I 17.23

29.
(a) Here T1 = 1, T2 = 2, T3 = 3, Total interest paid = `10000 + `9850
R1 = R2 = R3 = 5%  = `19850

The shares of Vikas, Vijay and Viraj will be in the 34. (c) Let the sum of money be x and 2x.
ratio Interest on both the sums
1 1 1 1 1 1
: : = : : x × 5 × 1 2 x × 5.5 × 1
R1T1 R2T2 R3T3 1× 5 2 × 5 3 × 5 = +
100 100
1 1 1
= : : = 6 : 3 : 2.
1 2 3 5 x 11x 16 x
= + =
Sum of proportionals = 6 + 3 + 2 = 11. 100 100 100
6 According to questions,
\ Share of Vikas = × 7700 = `4,200
11
16 x
3 = 1000
Share of Vijay = × 7700 = `2,100 100
11
2 1000 × 100
Share of Viraj = × 7700 = `1,400 ⇒ x = = `6250
11 16
Therefore, Vikas’s share is 4200 – 1400 = `2,800 Hence, second sum invested = 2x = `12500.
more than that of Viraj.
30. (c) Simple interest 35.
(a) We have T1 = 2 years, T2 = 3 years, T3 = 4 years.
= (8 × 4)% + (10 × 6)% + (12 × 5)% = 12160 R1 = R2 = R3 = 5% p.a.
⇒ 152% = 12160 \ The ratio in which the amount is invested
⇒ 100% = `8000.
1 1 1
= : :
31. (b) SI1 = SI2 = SI3 100 + R1T1 100 + R2T2 100 + R3T3
\ x × 6% = y × 12% = z × 20% = k 1 1 1
k k k i.e.,  : :
\ x:y:z = : : × 60 = 10:5:3 100 + 2 × 5 100 + 3 × 5 100 + 4 × 5
6 12 20
7 1 1 1
\ Required difference = × 1440 = `560. i.e.,  : :   or,  276 : 264 : 253.
18 110 115 120

32. (c) Let principal of two sons are x and y, respectively. Their sum = 276 + 264 + 253 = 793
x × 130 y × 120 x 12 \ The amount invested for
\ = ⇒ =
100 100 y 13 3965
Neeta = × 276 = `1,380
12 793
\ x = × 18750 = `9000
25 3965
Sita
= × 264 = `1,320
13 793
y = × 18750 = `9750.
25 3965
Gita = × 253 = `1,265.
33. (c) Outstanding amount in December = `1000000 793
Interest to be paid in January = 1% of `1000000
36.
(a) Let the sum of money invested by Suresh be `x.
= `10000 (1)
(Since, 12% per annum ⇒ 1% per month)   Since the amount obtained in both the cases is equal,
the ratio in which the sums are invested is
Past payment = `25000 – `10000
 = `15000 1 1
:
Outstanding amount in January 100 + R1T1 100 + R2T2

= `1000000 – `15000 = 985000 5
where R1 = 8%, T1 = years, R2 = 5%
Interest to be paid in February = 1% of 985000 2
= 9850 (2) T2 = 2 years.

Chapter_17.indd 23 6/5/2015 2:41:15 AM


17.24 Chapter 17

1 1 100 78000
That is, :  or,  1 : 1 = × = ` 26000
5 100 + 5 × 2 120 110 3 100
100 + 8 ×
2
Now, 11 monthly instalments of `4900 each.
1 1
Given 1500 : x : : :
120 110 Then, it is equal to 11 × 4900 = `53900
1500 × 1 1 Therefore, total cost of machine on instalment 26000
⇒ = ×x
110 120 + 53900 = `79900
1500 × 1 × 120 4 Now, interest = (79900 – 78000) = `1900
or, x = = 1636 .
110 11 Let the rate of interest by R.
4 Then,
\ The sum invested by Suresh is `1,636
11 P×R ×T
SI =
1 100
37. (a) Xerox machine is available at 33 % cash down
3 78000 × R × 1
⇒ 1900 =
payment and 11 equal monthly instalments of `4900 100
each. 1900 × 100
⇒R= = 2.43%
Then, 33 1 % of `78000 78000
3

Chapter_17.indd 24 6/5/2015 2:41:17 AM


CHAPTER

Compound Interest 18
INTRODUCTION
In Chapter 17, we discussed simple interest. A second to it. This method is used in investments such as savings
method of paying interest is the compound interest method, account and bonds. An understanding of compound interest
where the interest for each period is added to the principal is important not only for people planning careers with
before interest is calculated for the next period. With financial institutions but also for anyone planning to invest
this method the principal grows as the interest is added money.

SOME BASIC FORMULAE

15000 × 21 × 21
1. (a) The amount A due after t years, when a = = `16537.50.
20 × 20
principal P is given on compound interest at
the rate R% per annum is given by
Illustration 2 Find compound interest on `5000 for 2 years
t
 R  at 4% per annum
A = P 1 +  .
 100  Solution: Here P = 5000, R = 4 and, t = 2
(b) Compound interest (CI ) = A – P t
 R  
 t  ∴ CI = P 1 +  − 1
R   100  
= P  1 +  − 1
 100  
 2 
 A 1/t  4 
= 5000 1 +  − 1
(c) Rate of interest (R) =   − 1 % p.a.  100  
 P  
Note:  26 2 
= 5000   − 1 = 5000((1.04)2 – 1)
Simple interest and compound interest for 1 year at a  25  
given rate of interest per annum are always equal. = 5000(1.0816 – 1) = `408
Illustration 1 Mohan invested an amount of `15000 ∴ The compound interest is `408.
at compound interest rate 5% per annum for a period of
Illustration 3 Rashi invested `16000 for two years at
2 years. What amount will he receive at the end of 2 years?
compound interest and received an amount of `17640 on
Solution: Here P = 15000, R = 5 and, t = 2 maturity. What is the rate of interest?
t
 R  Solution: Here P = 16000, t = 2 and, A = 17640
∴ Amount = P 1 + 
 100 
2 2  A 1/t 
 5   1  ∴ R = 100   − 1 % p.a.
= 15000 1 +  = 15000 1 + 
 100   20   P  

Chapter 18.indd 1 6/5/2015 11:31:05 AM


18.2 Chapter 18

 17640 1/2 
= 100   − 1 % p.a.  1
×4 
 16000    A t
(c) Rate (R) = 4 × 100   − 1 % p.a.

 P  
 441 1/2 
= 100  In general, if the interest is compounded n
 − 1 % p.a.
 400  
times a year, then
 1  n×t
 21 
2×  R 

= 100  
2
− 1 % p.a. (a) Amount (A) = P 1 + 
 20    100 × n 
1 (b) Compound interest (CI)
= 100 × = 5% p.a. n×t
20  R  
= P  1 +  − 1
2. If the interest is compounded half-yearly, then  100 × n  
 
2t
 R  (c) Rate of interest (R)
(a) Amount (A) = P 1 +  .
 100 × 2   1
×n 
 A t
(b) Compound interest (CI) = n × 100   − 1 % p.a.

 P  
 2t 
R 

= P 1 +  − 1
 100 × 2   Illustration 5 Find the compound interest on `1000 at 40%
 
per annum compounded quarterly for 1 year
 1 
×2 Solution: Here P = 1000, R = 40 and, t = 1
 A t
(c) Rate (R) = 2 × 100   − 1 % p.a.

 P   \ Compound interest (CI)
 
 4×t 
R 
Illustration 4 Find the amount of `8000 in one and half 
= P 1 +  − 1
 100 × 4  
years at 5% per annum compound interest payable half-  
yearly 4×1
 40    11 4 
3 = 1000 1 + − 1 =1000   − 1
Solution: Here P = 8000, R = 5 and, t = 
2  100 × 4    10  
 
2t
 R  14641 − 10000 
\ Amount = P 1 + = 1000 

 100 × 2   10000  = `464.10.


3
3
Illustration 6 Find the compound interest on `4000 at 24%
 5  2  41  per annum for 3 months, compounded monthly
= 8000 1 +  = 8000  
 100 × 2   40  3
Solution: Here P = 4000, R = 24 and, t =
8000 × 41 × 41 × 41 12
= = `8615.13.
40 × 40 × 40  R 
12×t 
\ CI = P 1 +  − 1
 100 × 12  
3. If the interest is compounded quarterly, then  
4t 3
 P   12× 
(a) Amount (A) = P 1 +   24  12 
 100 ×4 = 4000 1 +  − 1
 100 × 12  
(b) Compound interest (CI)
 4t   51 3 
R  4000 × 7651
= P  1 +  − 1 = 4000   − 1 =
 100 × 4    50   50 × 50 × 50
 
= `244.83.

Chapter 18.indd 2 6/5/2015 11:21:19 AM


Compound Interest 18.3

soMe useful sHort-cut MetHoDs

Illustration 8 What will be the compound interest on


1. When the rates of interest are different for `15625 for two and half years at 4% per annum?
different years, say R1, R2, R3 per cent for first,
  1 
second and third year, respectively, then  2 4×  
4   2 −1
Solution: CI = 15625 1 +  1 +  
 R  R  R   100   100  
Amount = P 1 + 1 1 + 2 1 + 3  .
 100  100  100 
 26 26 51 
= 15625  × × −1
Explanation  25 25 50 
Let, the given sum of money be `P. Amount after first year 15625 × 3226
= = `1613
 R  31250
= P 1 + 1 
 100 
3. (a) The difference between the compound
This amount will be the principal for the second year. interest and the simple interest on a certain
sum of money for 2 years at R% per annum is
\ Amount after second year
given by
 R  R 
= P 1 + 1 1 + 2   R 
2
 100  100  CI – SI = P   [in terms of P and R]
 100 
This amount will be the principal for the third year.
R × SI
\ Amount after third year and, CI – SI = [in terms of SI and R]
2 × 100
 R  R  R 
= P 1 + 1 1 + 2 1 + 3  .
 100  100  100  Explanation
Illustration 7 Anu invests `5000 in a bond which gives Let, `P be given sum of money. Simple interest on `P for 2
interest at 4% per annum during the first year, 5% during years at R% per annum
the second year and 10% during the third year. How much P×R×2
=
does she get at the end of the third year 100
Solution: Here P = 5000, R1 = 4, R2 = 5 and, R3 = 10 and compound interest on `P for 2 years at R% per annum
\ Amount at the end of third year  2 
R 
= P  1 +  − 1
 R  R  R   100  
= P 1 + 1 1 + 2 1 + 3 
 100  100  100  2
 R   P×R×2
 4  5  10  \ CI – SI = P 1 +  − 1 −
= 5000 1 +
 100
1 +
 100
1 +
 100

  100   100

26 21 11  R2 2R 2R 
= 5000 × × × = `6006. = P 1 + + −1− 
25 20 10  10000 100 100 
2
2. When the time is given in the form of fraction, say  R 
= P 
3  100 
years, then,
4  R 
2
R  P× R×2
Also, CI – SI = P   = × 
 3   100  100 × 2  100 
3 R
 R   4 .
Amount = P 1 +  × 1 +  R × SI
 100   100  = .
2 × 100

Chapter 18.indd 3 6/5/2015 11:21:21 AM


18.4 Chapter 18

Illustration 10 If the difference between the compound


(b) The difference between the compound interest and simple interest on a certain sum of money for 3
interest and the simple interest on a certain years at 5% per annum is `61, find the sum
sum of money for 2 years at R% per annum is
given by Solution: Here CI – SI = 61 and, R = 5

  R 3 2   R 3  R  
2
 R   \ CI – SI = P   + 3   
CI – SI = P   + 3   
 100   100    100   100  

[in terms of P and R]   5 3  5  


2
⇒ 61 = P   + 3   
SI  R  2  R   100   100  
and, CI – SI =   + 3 
3  100   100     1 3 2
 1  
= P   + 3   
[in terms of SI and R]  20   20  
Explanation  1 + 3 × 20   61 
=P  = P 
Let, `P be the given sum of money. Simple interest on `P  20 × 20 × 20   20 × 20 × 20 
for 3 years at R% per annum
⇒ P = `8000.
P× R×3
=
100 4. If a certain sum becomes n times in t years at
and compound interest on `P for 3 years at R% per annum compound interest, then the same sum becomes
 3  nm times in mt years.
R 
= P  1 +  − 1
 100   Explanation
3
Let, `P be the given sum of money. We have,
 R   P × R ×3 t t
\ CI – SI = P 1 +  − 1 −  R   R 
 100   100 nP = P 1 +  ⇒n= 1 +  (1)
 100   100 
 R3 3R 2 3R 3R  Let, the sum become nm times in T years.
= P 1 + + + −1− 
 100000 10000 100 100 
 R 
T
Then, nm =  1 + 
 R3  100 
3R 2 
=P + 
1000000 1000   R 
T /m
or, n = 1 +  (2)
  R 3 2  100 
 R  
= P   + 3    On comparing Eqs. (1) and (2), we get
 100   100  
T/m = t or, T = mt years.
2
P × R × 3 1  R   R  Therefore, the sum becomes nm times in mt years.
= ×   + 3 
100 3  100   100   Illustration 11 A sum of money placed at compound
interest doubles itself in 3 years. In how many years will it
SI  R  2  R 
= amount to four times itself?
  + 3  .
3  100   100   Solution: Here n = 2, t = 3 and, m = 2
Illustration 9 What will be the difference between simple \ The given sum of money will become four times
and compound interest on a sum of `4500 put for 2 years at itself in mt, i.e., 2 × 3 = 6 years.
5% per annum?
5. If a certain sum becomes n times in t years, then
Here, P = 4500 and, R = 5.
2 2
the rate of compound interest is given by
 R   5  4500 R = 100[(n)1/t – 1].
\ CI – SI = P   = 4500   = = `11×25.
25.
 100   100  20 × 20

Chapter 18.indd 4 6/5/2015 11:21:22 AM


Compound Interest 18.5

Illustration 12 At what rate per cent compound interest


does a sum of money become four-fold in 2 years? 7. If a loan of `P at R% compound interest per
annum is to be repaid in n equal yearly instalments,
Solution: The required rate per cent is then the value of each instalment is given by
R = 100[(n)1/t – 1] = 100[(4)1/2 – 1]
P
= 100(2 – 1) = 100% ` 2 n
.
 100   100   100 
[Here n = 4 and, t = 2]  +  + ...  
 100 + R   100 + R   100 + R 
6. If a certain sum of money at compound interest
amounts to `x in A years and to `y in B years, then Explanation
the rate of interest per annum is Let, each instalment be of `X.
 y 1/B − A  \ Principal for the amount of `X due at end of first year
R =   − 1 × 100%
 x   at R% =
100 X
100 + R
Explanation Principal for the amount of `X due at the end of second
Let, the principal be `P and the rate of interest be R% per  100 
2
annum. year at R% =   X
A B
 100 + R 
 R   R    
Given x = P 1 +  and, y = P 1 + 
 100   100  Principal for the amount of `X due at the end of nth
B n
 R   100 
1 +  R 
B− A year at R% =   X
y 100  
\ =  A
=  1 +   100 + R 
x  R   100  2 n
 1 +  100 X  100   100 
 100  \ +  X + ...   X =P
1/B − A 100 + R  100 + R   100 + R 
 y R
\   =1+ P
x 100 or, X=` 2 n .
 100   100   100 
R  y
1/B − A  +  + ...  
or, =   –1  100 + R   100 + R   100 + R 
100  x 
Illustration 14 If a sum of `13040 is to be paid back in two
 y 1/B − A  3
or, R =   − 1 × 100. equal annual instalments at 3 % per annum, what is the
 x  
amount of each instalment?
4

Illustration 13 A sum of money at compound interest Solution: Each instalment


amounts to `4050 in one year and to `4723.92 in 3 years. P 13040
Find the rate of interest per annum = 2
= 2
 100   100   100   100 
Solution: Here x = 4050, y = 4723.92, A = 1 and, B = 3  +
    +
 100 + R   100 + R  15   15 
 100 +   100 + 
 y 1/B − A   4   4 
\ R =   − 1 × 100%  15 
 x    Here P = 13040 and R = 4 
13040 13040
 4723 ⋅ 92 1/2  = =
2 400  400 
=   − 1 × 100% 400  400 
 4050   +  1 + 
415  415  415  415 

 27  415 815
=  − 1 × 100% = 8% = 13040 × × = `6889.
 25  400 400

Chapter 18.indd 5 6/5/2015 11:21:23 AM


18.6 Chapter 18

Practice Exercises

Difficulty Level-1
(Based on Memory)

1. A scooter costs `25,000 when it is brand new. At the end 8. A sum is invested for 3 years compounded at 5%, 10%
of each year, its value is only 80% of what it was at the and 20% respectively. In three years, if the sum amounts
beginning of the year. What is the value of the scooter at to `16,632, then find the sum.
the end of 3 years? (a) `11000 (b) `12000
(a) `10,000 (b) `12,500 (c) `13000 (d) `14000
(c) `12,800 (d) `12,000 [Based on IIT Joint Man. Ent. Test, 2004]
[Based on MAT, 2004]
9. What will be the difference in the compound interest on
2. Sanjay borrowed a certain sum from Anil at a certain rate `50,000 at 12% for one year, when the interest is paid
of simple interest for 2 years. He lent this sum to Ram yearly and half-yearly?
at the same rate of interest compounded annually for the (a) `500 (b) `600
same period. At the end of two years, he received `4200 as (c) `180 (d) `360
compound interest but paid `4000 only as simple interest. [Based on IIT Joint Man. Ent. Test, 2004]
Find the rate of interest. 10. Akash borrows `65,000 at 10% per annum simple interest
(a) 15% (b) 20% for 3 years and lends it at 10% per annum, compound
(c) 35% (d) 10% interest for 3 years. Find his gain after three years.
[Based on MAT, 2002]
(a) `2015 (b) `1330
3. If the ratio of compound interest and simple interest for (c) `1300 (d) None of these
the same principal and rate of interest for two years is
[Based on IIT Joint Man. Ent. Test, 2004]
1.2% per annum, find the rate of interest.
(a) 3% (b) 15% 11. What is the compound interest on `5,000 for 4 years if the
(c) 140% (d) 300% rate of interest is 10% p.a. for the first 2 years and 20% per
annum for the next 2 years?
4. If the difference between the simple and the compound (a) `2,320.50 (b) `3,712
interests on some principal amount at 20% for 3 years is (c) `3,745 (d) None of these
`48, then the principal amount must be:
12. Rohit earns an interest of `1656 for the third year and
(a) `650 (b) `600
`1440 for the second year on the same sum. Find the rate
(c) `375 (d) `400
of interest if it is lent at compound interest.
[Based on MAT, 2001]
(a) 18% (b) 12%
5. A bank offers 10% interest rate compounded annually. A
person deposits `10,000 every year in his account. If he (c) 15% (d) None of these
does not withdraw any amount, then how much balance [Based on IIT Joint Man. Ent. Test, 2004]
will his account show after four years? 13. Divide `1301 between A and B, so that the amount of A
(a) `51051 (b) `45095 after 7 years is equal to the amount of B after 9 years, the
(c) `36410 (d) `51000 interest being compounded at 4% per annum.
[Based on FMS (Delhi), 2004]
(a) `676 and `625 (b) `650 and `651
6. Two customers borrowed the same amount of money, one (c) `670 and `631 (d) `660 and `630
at compound interest and the other at simple interest. If
[Based on IIFT, 2003]
after 2 years, the interest payable by one was `220 and by
the other `200, then, what was the principal money lent to 14.
Amit borrowed `800 at 10% rate of interest. He repaid
each one of them? `400 at the end of first year. What is the amount he is
(a) `450 (b) `500 required to pay at the end of second year to discharge his
(c) `550 (d) None of these loan which was calculated at compound interest?
[Based on IIT Joint Man. Ent. Test, 2004] (a) 420 (b) 440
(c) 450 (d) 528
7. What is the present value (in lakhs of `) of an income of
`2 lakhs to be received after 2 years if the rate of interest 15.
What is the net present value of stock valued at `54,880
is 5% after 3 years? (Rate of interest = 40% per annum and
(a) 1.81 (b) 1.67 interest in compounded annually)
(c) 2.2 (d) None of these (a) `20,000 (b) `30,000
[Based on IIT Joint Man. Ent. Test, 2004] (c) `40,000 (d) `50,000

Chapter 18.indd 6 6/9/2015 2:47:39 PM


Compound Interest 18.7

16.
A sum of `1,100 was taken as a loan. This is to be repaid repayment of loan and interest. How much does he still
in two equal instalments. If the rate of interest be 20% owe to the bank after three such instalments?
compounded annually, then the value of each instalment is: (a) `123.25 (b) `125
(a) `842 (b) `792 (c) `400 (d) `469.18
(c) `720 (d) `700 [Based on MAT (Sept), 2010]
17.
Ravi Shankar takes money from the employees 23.
Amit borrowed a certain sum of money for 2 years at 8%
cooperative society at lower rate of interest and saves in per annum on simple interest and immediately lent it to
a scheme, which gives him a compound interest of 20% Ravi but at compound interest and gained by `16. What
compounded annually. Find the least number of complete amount did Amit borrow?
years after which his sum will be more than double (a) `1600 (b) `2500
(a) 4 years (b) 2 years (c) `24000 (d) `1800
(c) 6 years (d) 8 years [Based on MAT (May), 2010]
[Based on MAT (Feb), 2011] 24.
Raju’s factory kept increasing its output by the same
18.
Equal sums of money are deposited in two different percentage every year. Find the percentage if it is known
banks by M/s Enterprises, one at compound interest, that his output is doubled after two years.
compounded annually and the other at simple interest, (a) 100 2% (b) 100( 2 + 1)%
both at 5% per annum. If after two years, the difference
in the amounts comes to `200, what are the amounts (c) 100( 2 − 1)% (d) 50( 3 − 1)%
deposited with each bank? [Based on MAT (May), 2010]
(a) `72000 (b) `64000 25.
The effective annual rate of interest corresponding to a
(c) `80000 (d) `8400 nominal rate of 8% per annum payable half yearly is:
[Based on MAT (Feb), 2011] (a) 8% (b) 8.01%
19.
Rohit took a loan of `20000 to purchase one LCD TV (c) 8.13% (d) 8.16%
set from a finance company. He promised to make [Based on MAT (Feb), 2010]
the payment after three years. The company charges 26.
The population of a colony was 3600 three years back. It
compound interest at the rate of 10% per annum for the is 4800 right now. What will be the population three years
same. But suddenly the company announces the rate of down the line, if the rate of growth of population has been
interest as 15% per annum for the last one year of the loan constant over the years and has been compounding annually?
period. What extra amount does Rohit have to pay due to (a) 6000 (b) 6400
this announcement of the new rate of interest?
(c) 7200 (d) 9600
(a) `7830 (b) `4410 [Based on MAT (Feb), 2010]
(c) `6620 (d) None of these 27.
`5887 is divided between Shyam and Ram, such that
[Based on MAT (Dec), 2011] Shyam’s share at the end of 9 years is equal to Ram’s
20.
A tree was planted three years ago. The rate of its growth share at the end of 11 years, compounded annually at the
is 30% per annum. If at present, the height of the tree is rate of 5% The share of Shyam is:
670 cm, what was it when the tree was planted? (a) `2088 (b) `2000
(a) 305 cm (b) 500 cm (c) `3087 (d) None of these
(c) 405 cm (d) 625 cm [Based on MAT (Feb), 2010]
[Based on MAT (Dec), 2010] 28.
The population of a village is 10000. If the population
21.
Sanju puts equal amounts of money, one at 10% per annum increases by 10% in the first year, by 20% in the second
compound interest payable half yearly and the second at year and due to mass exodus, it decreases by 5% in the
a certain per cent per annum compound interest payable third year, what will be its population after 3 years?
yearly. If he gets equal amounts after 3 years, what is the (a) 13860 (b) 11540
value of the second per cent? (c) 12860 (d) 12540
1 [Based on MAT (Feb), 2009]
(a) 10 % (b) 10%
4 29.
A sum of money is accumulating at compound interest
at a certain rate of interest. If simple interest instead of
1 1
(c) 9 % (d) 8 % compound were reckoned, the interest for the first two
2 4 years would be diminished by `20 and that for the first
[Based on MAT (Dec), 2010] three years by `61. Find the sum.
1 (a) `7000 (b) `8000
22.
A man borrows `4000 from a bank at 7 % compound
2 (c) `7500 (d) `6500
interest. At the end of every year, he pays `1500 as part [Based on MAT (Dec), 2008]

Chapter 18.indd 7 6/5/2015 11:24:50 AM


18.8 Chapter 18

30.
The difference between the compound interest and simple (a) 6 years (b) 4 years
interest earned at the end of second year on a sum of (c) 8 years (d) 5 years
money at 10% per annum is `20. The sum is: [Based on MAT, 1999]
(a) `4,000 (b) `2,000 38.
A sum of money is borrowed and paid back in two equal
(c) `1,500 (d) Data inadequate annual installments of `882 allowing 5% compound
31.
Ram takes a loan of `10000 and pays back `13310 after interest. The sum borrowed was:
3 years. The compound interest rate per annum will be (a) `1,620 (b) `1,600
approximately: (c) `1,680 (d) `1,700
(a) 8% (b) 9% [Based on MAT, 2000]
(c) 10% (d) 11% 39.
The difference between compound interest and simple
[Based on MAT (Sept), 2007] interest at the same rate `5000 for 2 years is `72. The rate
32.
The ratio of the amount for two years under CI annually of interest per annum is:
and for one year under SI is 6:5. When the ratio of interest (a) 6% (b) 8%
is same, then the value of the rate of interest is: (c) 10% (d) 12%
(a) 12.5% (b) 18% [Based on FMS, 2006]
(c) 20% (d) 16.66% 40.
The present value of an optical instrument is `20000. If its
[Based on MAT (Dec), 2006] value will depreciate 5% in the first year, 4% in the second
33.
A computer is available for `39000 cash or `17000 as cash year and 2% in the third year, what will be its value after
down payment followed by five monthly instalments of three years?
`4800 each. What is the rate of interest per annum under (a) `16534.5 (b) `16756.5
the instalment plan? (c) `17875.2 (d) `17556.8
(a) 35.71% (b) 37.71% [Based on FMS, 2009]
(c) 36.71% (d) 38.71% 41.
If the daily compounding rate of interest is 10% on an
[Based on MAT (May), 2006] investment, what is the present value of `50000 that is to
34.
Under the Rural Housing Scheme, the Delhi Development be received after two years?
Authority (DDA) allotted a house to Kamal Raj for (a) `40936.54 (b) `41037.33
`126100. This payment is to be made in three equal (c) `41322.31 (d) `40000
annual instalments. If the money is reckoned at 5% per [Based on FMS, 2009]
annum compound interest, then how much is to be paid 42. A loan was repaid in two annual instalments of `121 each.
by Kamal Raj in each instalment? If the rate of interest be 10% per annum, compounded
(a) `45205 (b) `47405 annually, the sum borrowed was:
(c) `46305 (d) `48505 (a) `200 (b) `210
[Based on MAT (May), 2006] (c) `217.80 (d) `216
35.
A sum of `30600 is divided between Anjali and Arun, who 43. The total compound interest earned on an amount @ 15%
are respectively 18 and 19 years old, in such a way that per annum at the end of three years is ` 9844.5375. What
if their shares are invested at 4% per annum compounded is the amount?
annually, they shall receive the same amount on reaching (a) `17500 (b) `18900
21 years of age. What is the share of Anjali?
(c) `19700 (d) `19400
(a) `16000 (b) `15000 [Based on IRMA, 2005]
(c) `15600 (d) `14600 44. The population of a city increases at a rate of 4% per
[Based on MAT (May), 2010] annum. There is an additional annual increase of 1% in
36.
Two friends A and B jointly lent out `81600 at 4% per the population due to the influx of job seekers. The per
annum compound interest. After 2 years, A gets the same cent increase in the population after 2 years is:
amount as B gets after 3 years. The investment made by B (a) 10.25 (b) 10
was: (c) 10.50 (d) 10.75
(a) `40000 (b) `30000
45.
A scooter costs `25,000 when it is brand new. At the end
(c) `45000 (d) `38000
of each year, its value is only 80% of what it was at the
[Based on MAT (Feb), 2008]
beginning of the year. What is the value of the scooter at
37.
A sum of money becomes eight times in 3 years if the the end of 3 years?
rate is compounded annually. In how much time the same (a) `10,000 (b) `12,500
amount at the same compound interest rate will become
(c) `12,800 (d) `12,000
sixteen times?

Chapter 18.indd 8 6/5/2015 11:21:24 AM


Compound Interest 18.9

46. Anuj has deposited certain amount in the bank to earn 52. Palvinder owes `1040000 to a bank, and he returns
compound interest at 10% per annum. The difference in `40000 quarterly to the bank. If the tax on the money
the interest on the amount between the 3rd and 2nd years is Palvinder owes is completed quarterly by 0.25% starting
`1,100. What amount has Anuj deposited? before Palvinder paid the first payment, how many months
(a) `10,000 (b) `11,000 would it take Palvinder to reach a point where he owns the
(c) Data inadequate (d) None of these bank not more than 1 million rupees?
47. The population of a town increases annually by 25% If the (a) 3 (b) 5
present population is one crore, then what is the difference (c) 9 (d) None of these
[Based on MAT, 2013]
between the population 3 years ago and 2 years ago?
(a) 25,00,000 (b) 12,80,000 53. The present worth of a bill due 7 months hence is `1200.
1
(c) 15,60,000 (d) None of these If the bill were due at the end of 2 years its present worth
2
48. Rohit earns an interest of `1,656 for the third year and would be `1016. What is the rate per cent of the bill?
`1,440 for the second year on the same sum. Find the rate (a) 8% (b) 10%
of interest if it is lent at compound interest. (c) 16% (d) 18%
(a) 18% (b) 12% [Based on MAT, 2013]
(c) 15% (d) None of these 54. Find the principal if compound interest is charged on the
49. A man borrows `6000 at 10% compound rate of interest. 2
principal at the rate of 16 % per annum for two years
He pays back `2000 at the end of each year to clear his 3
debt. The amount that he should pay to clear all his dues and the sum becomes `196.
at the end of third year is: (a) `140 (b) `154
(a) `6000 (b) `3366 (c) `150 (d) None of these
(c) `3060 (d) `3066 [Based on MAT, 2014]
[Based on MAT, 2011] 55. Mukesh borrows a certain sum of money from the
50. Sharma invested one-half of his savings in a bond that paid ABC Bank at 10% per annum at compound interest.
simple interest for 2 years and received `500 as interest. The entire debt is discharged in full by Mukesh on
He invested the remaining in a bond that paid compound payment of two equal amounts of `1000 each, one
interest, interest being compounded annually, for the same at the end of the first year and the other at the end of
2 years at the same rate of interest and received `605 as the second year. What is the approximate value of
interest. What was the value of his total saving before the amount borrowed to him?
investing in these two bonds?
(a) `1852 (b) `1694
(a) `22000 (b) `11000
(c) `1736 (d) `1792
(c) `5500 (d) `2750 [Based on MAT, 2014]
[Based on MAT, 2011]
56. The simple interest accrued on a sum of certain principal
51. A sum of money lent at compound interest for 2 years at 20% in 8 years at the rate of 13% per year is `6500. What
per annum would fetch `482 more, if the interest was payable would be the compound interest accrued on that principal
half yearly than if it was payable annually. The sum is: at the rate of 8% per year in 2 years?
(a) `10000 (b) `20000 (a) `1040 (b) `1020
(c) `40000 (d) `50000 (c) `1060 (d) `1200
[Based on MAT, 2012] [Based on SNAP, 2013]

Difficulty Level-2
(Based on Memory)

1. A and B each borrowed equal sums for 3 years at the 2. The value of a fixed asset depreciates at the rate of 10% on
rate of 5% simple and compound interest respectively. the value at the beginning of each year. If the value of the
At the time of repayment B has to pay `76.25 more asset, two years ago, was `12,000 more than the value of
than A. The sum borrowed and the interest paid by A the asset one year ago, then find the present value of the
(in `) is: asset, given that the asset was bought two years ago.
(a) `10,000, `1,500 (b) `11,000, `1,100 (a) `14520 (b) `96,000
(c) `97,200 (d) `17,520
(c) `10,000, `1,400 (d) `9,000, `200
[Based on IIT Joint Man. Ent. Test, 2004]

Chapter 18.indd 9 6/5/2015 11:21:25 AM


18.10 Chapter 18

3.
A father divides his property between his two sons A 10.
Equal amounts of each `43,892 is lend to two persons for
and B. A invests the amount at compound interest of 8% 3 years. One at the rate of 30% S.I. and second at the rate
per annum B invests the amount at 10% per annum simple of 30% C.I. annually. By how much per cent the C.I. is
interest. At the end of 2 years, the interest received by B is greater than the simple interest received in this 3 years
`1336 more than the interest received by A. Find the share duration?
of A in the father’s property of `25,000. (a) 23% (b) 33%
(a) `12,000 (b) `13,000 (c) 33.33% (d) None of these
(c) `12,500 (d) `10,000 11.
Mr Jeevan wanted to give some amount of money to his
[Based on IIT Joint Man. Ent. Test, 2004] two children, so that although today they may not be using
4. The population of a town increases annually by 25% it, in the future the money would be of use to them. He
If the present population is one crore, then what is the divides a sum of `18750 between his two sons of age
difference between the population 3 years ago and 2 years 10 years and 13 years respectively in such a way that
ago? each would receive the same amount at 3% per annum
compound interest when he attains the age of 30 years.
(a) 25,00,000 (b) 12,80,000
What would be the original share of the younger son?
(c) 15,60,000 (d) None of these
(a) 8959.80 (b) 8559.80
[Based on IIT Joint Man. Ent. Test, 2004]
(c) 8969.80 (d) 8995.80
5. Anuj has deposited certain amount in the bank to earn
[Based on IIFT, 2007]
compound interest at 10% per annum. The difference in
the interest on the amount between the 3rd and 2nd years 12.
To start a new enterprise, Mr Yogesh has borrowed a total
is `1,100. What amount has Anuj deposited? of 60000 from two money lenders with the interest being
compounded annually, to be repaid at the end of 2 years.
(a) `10000 (b) `11000 Mr Yogesh repaid `38800 more to the first money lender
(c) Data inadequate (d) None of these compared to the second money lender at the end of 2 years.
[Based on Narsee Manjee Inst. of Man. Studies, 2003] The first money lender charged an interest rate, which was
6.
The difference between compound interest and simple 10% more than what was charged by the second money
interest on a sum for 2 years at 10% per annum, when lender. If Mr Yogesh had instead borrowed `30000 from
the interest in compounded annually is `16. If the interest each at their respective initial rates for 2 years, he would
were compounded half yearly, the difference in two have paid `7500 more to the first money lender compared
interests will be: to the second. Then, money borrowed by Mr Yogesh from
(a) `24.81 (b) `31.61 first money lender is:
(c) `32.40 (d) `26.90 (a) 20000 (b) 35000
(c) 40000 (d) 42000
7.
The compound interest on a certain sum for 2 years
[Based on IIFT, 2010]
is `756 and S.I. (simple interest) is `720. If the sum is
13.
Mungeri Lai has two investment plans: A and B, to choose
invested such that the S.I. is `900 and the number of years
from plan A offers interest of 10% compounded annually
is equal to the rate per cent per annum, find the rate per
while plan B offers simple interest of 12% per annum. Till
cent.
how many years is plan B a better investment?
(a) 4 (b) 5/2
(a) 3 (b) 4
(c) 6 (d) 1.0
(c) 5 (d) 6
8.
A person invested a certain amount at simple interest at the [Based on XAT, 2009]
rate of 6% per annum earning `900 as an interest at the end 14.
Anu borrowed a sum of money and returned it in three
of three years. Had the interest been compounded every equal quarterly instalments of `17,576 each. Find the sum
year, how much more interest would he have earned on the borrowed, if the rate of interest charged was 16% per
same amount with the same interest rate after three years? annum compounded quarterly. Find also the total interest
(a) `38.13 (b) `25.33 charged.
(c) `55.08 (d) `35.30 (a) 46,900 and 4,700 (b) 48,775 and 3,953
(c) 68,320 and 1,200 (d) None of these
9.
The population of vultures in a particular locality decreases
by a certain rate of interest (compounded annually). If the 15.
A father divides his property between his two sons A and
current population of vultures be 29,160 and the ratio of B. A invests the amount at compound interest of 8% per
decrease in population for second year and 3rd year be annum and B invests the amount at 10% per annum simple
10:9. What was the population of vultures 3 years ago? interest. At the end of 2 years, the interest received by B
(a) 30,000 (b) 35,000 is `1,336 more than the interest received by A. Find the
share of A in the father’s property of `25,000.
(c) 40,000 (d) 50,000

Chapter 18.indd 10 6/5/2015 11:21:25 AM


Compound Interest 18.11

(a) `12,000 (b) `13,000 20. One can purchase a flat from a house building society for
(c) `12,500 (d) `10,000 `55000 cash or on the terms that he should pay `4275
16. The value of a fixed asset depreciates at the rate of 10% on as cash down payment and the rest in three equal yearly
the value at the beginning of each year. If the value of the instalments. The society charges interest at the rate of
asset, two years ago, was `12,000 more than the value of 16% per annum compounded half yearly. If the flat is
the asset one year ago, then find the present value of the purchased under instalments plan, find the value of each
asset, given that the asset was bought two years ago. instalments.
(a) `14520 (b) `96,000 (a) `39683 (b) `19683
(c) `97,200 (d) `17,520 (c) `29683 (d) `22803
[Based on NMAT, 2005]
17. Sanjay borrowed a certain sum from Anil at a certain rate
of simple interest for 2 years. He lent this sum to Ram 21. A man borrows a certain sum and pays it back in 2 years
at the same rate of interest compounded annually for the in two equal instalments. If the compound interest is
same period. At the end of two years, he received `4,200 reckoned at 4% and if he pays back annually `676 the
as compound interest but paid `4,000 only as simple sum borrowed is:
interest. Find the rate of interest. (a) `1275 (b) `1078
(a) 15% (b) 20% (c) `1870 (d) None of these
(c) 35% (d) 10% [Based on NMAT, 2006]
18. Shudhir invested `16000 in a scheme which earned him 22. Sultan took a loan from the bank at 8% per annum and was
simple interest @ 15% per annum. After two years he supposed to pay a sum of `2240 at the end of 4 years. If the
withdrew the principal amount plus interest and invested the same sum is cleared off in four equal annual installments
entire amount in amount scheme for two years, which earned at the same rate, the amount of annual installment will be:
him compound interest @ 12% per annum. What would be
(a) `500 (b) `550
the total interest earned by Sudhir at the end of 4 years?
(c) `600 (d) `1000
(a) `9792 (b) `10152.11
[Based on MAT, 2012]
(c) `9012.14 (d) None of these
[Based on IRMA, 2008] 23. A sum of money is accumulating at compound interest
19. A man borrows `4000 at 20% compound rate of interest. at a certain rate of interest. If simple interest instead of
At the end of each year he pays back `1500. How much compound were reckoned, the interest for the first two
amount should he pay at the end of the third year to clear years would be diminished by `20 and that for the first
all his dues? three years, by 61. What is the sum?
(a) `2952 (b) `2852 (a) `7500 (b) `7000
(c) `2592 (d) `2953 (c) `8000 (d) `6500
[Based on NMAT, 2005] [Based on MAT, 2013]

Answer Keys
Difficulty Level-1

1. (c) 2. (d ) 3. (c) 4. (c) 5. (a) 6. (b) 7. (a) 8. (b) 9. (c) 10. (a) 11. (b) 12. (c) 13. (a)
14. (d ) 15. (a) 16. (c) 17. (b) 18. (c) 19. (d ) 20. (a) 21. (a) 22. (a) 23. (b) 24. (c) 25. (d ) 26. (b)
27. (c) 28. (d ) 29. (b) 30. (b) 31. (c) 32. (c) 33. (d ) 34. (c) 35. (b) 36. (a) 37. (b) 38. (b) 39. (d )
40. (c) 41. (c) 42. (b) 43. (b) 44. (a) 45. (c) 46. (b) 47. (b) 48. (c) 49. (b) 50. (d ) 51. (b) 52. (d )
53. (b) 54. (d ) 55. (c) 56. (a)

Difficulty Level-2

1. (a) 2. (c) 3. (d ) 4. (b) 5. (b) 6. (a) 7. (b) 8. (c) 9. (c) 10. (b) 11. (a) 12. (c) 13. (b)
14. (b) 15. (d ) 16. (c) 17. (d ) 18. (d ) 19. (a) 20. (b) 21. (a) 22. (a) 23. (c)­­­

Chapter 18.indd 11 6/5/2015 11:21:26 AM


18.12 Chapter 18

Explanatory Answers

Difficulty Level-1

3 4
 20  4 4 4  216 − 125  364
(c) 25000 1 −
1.  = 25000 × 5 × 5 × 5
= 100   = 5 × 91 = 5
 100   125 
364 64
= 12800. \ C.I. – S.I. = − 60 =
5 5
2.
(d) Suppose the sum borrowed = `x 64
, principal = `100
If difference is `
Rate of interest = R% 5
Time = 2 years If difference is `48, principal
100 × 5
x ×R ×2 = × 48 = `375.
\ 4000 = 64
100
5.
(a) `10000 after 4 years become
⇒ Rx = 200000 (1) 4
 10  11 11 11 11
Now, = 10000 1 +  = 10000 × 10 × 10 × 10 × 10
R 
2  100 

x 1 +  = x + 4200 = 14641
 100 
`10000 after 3 years become
xR 2 2 RX 11 11 11
⇒ + = 4200 = 10000 × × × = 13310
10000 100 10 10 10
⇒ 20R + 4000 = 4200 `10000 after 2 years become
⇒ R = 10. 11 11
= 10000 × × = 12100
10 10
3.
(c) Let, P be the principal, r be the rate of interest. `10000 after 1 year become `11000
Pr \ Total amount after 4 years = `51051.
×r
Pr 100 Pr  r 
C.I. = + = 1 +  6. (b) S.I. for two years = `200
100 100 100  100 
S.I. for one years = `100
2 Pr
S.I. = C.I. for two years = `220
100
⇒ `20 is the interest on `100 for one year.
Pr  r 
1 +  If interest is `20, then amount = `100.
C.I. 100  100 
\ = 1.2 ⇒ If interest is `100, then amount
S.I. Pr
2 100
100 = × 100 = `500.
20
r r
⇒ 1 + = 2.4  or, = 1.4 7.
(a) P = Present value of `2 lakhs
100 100
2
r = 140%  5 
⇒ P 1 +  = 2 Lakhs
 100 
4.
(c) Let, the principal be `100.
2 × 20 × 20
⇒ P= ≈ 1.81.
100 × 20 × 3 21 × 21
\ S.I. = = `60
100
3 8.
(b) Let, P be the sum.
 20  21P
C.I. = 100 1 +  − 100 \ Amount after one year = P + 5% of P =
 100  20
6 6 6 21P 21P
= 100 × × × − 10C Amount after two years = + 10% of
5 5 5 20 20

Chapter 18.indd 12 6/5/2015 11:21:26 AM


Compound Interest 18.13

21P 21P 231P 13.


(a) Let, Amount of A = P
= + =
20 200 200 \ Amount of B = 1301 – P
Amount after three years 7 9
 4   4 
⇒ P 1 +  = (1301 – P)  1 + 
231P 231P  100   100 
= + 20% of
200 200
 26 26 
231P 231P 1386 P ⇒ P = (1301 – P)  × 
= + =  25 25 
200 1000 1000
⇒ 625P = 1301 × 676 – 676P
1386 P
\ = 16632 ⇒ P = 12000. ⇒ P = 676
1000
\ A gets `676 and B gets `625.
9.
(c) C.I. on `50000 @ 12% for one year, when the interest
14.
(d) Amount to be paid at the end of 2 years
is paid half yearly
800 × 10 × 2
 6 
2 = + 800 = 880
= 50000 1 + 100
 − 50000
 100 
Amount left as principal for the second year
53 53 = 480 = (880 – 400)
= 50000 × × − 50000
50 50 480 × 10
Amount to be paid after 2nd year = 480 +
= `6180 100
C.I. when the interest is paid yearly  = `528.

50000 × 12 × 1 (a) 54880 = P[1.4]3


15.
= = `6000  P = `20,000
100
Alternative method:
\ Required difference = C.I. – S.I.
Even if we assume S.I., there must be a growth of
= 6180 – 6000 = `180. 40% × 3 = 120% over initial amount.
Now work with answer choices.
10.
(a) S.I. on `65000 @ 10% for 3 years
If we increase (b), (c), (d ) by even 100% they would
65000 × 10 × 3 exceed 54880. Thus, option (a) is the answer.
= = `19500
100
16.
(c) Let, each instalment be `x
C.I. on `65000 @ 10% for 3 years Then, (present worth of `x due 1 year hence) +
3 (Present worth of `x due 2 years hence) = `1,100
 10 
= 65000 1 +  − 65000 x x
 100  \ + 2
= 1100
 20   20 
 1 +  1+
11 × 11 × 11 − 10 × 10 × 10   100   100 
= 65000  
 1000 
5 x 25 x
or, + = 1100
= `21515 6 36
\ Required gain = 21515 – 19500 = `2015. or, 55x = 36 × 1100
36 × 1100
2 2
\ x = = `720.
 10   20   55
11.
(b) C.I. = 5000 1 +  × 1 +  − 1 17.
(b) Let, the sum of money be P.
 100   100  
T
 20 
1856 \ P 1 +  ³ 2P
= 5000 × = `3,712.  100 
2500
T
6
12.
(c) Interest on `1440 = `216 for the third year ⇒   ³2
5
216 × 100 ⇒ (1.2)T ³ 2
⇒ Rate % = = 15.
1440 × 1 Hence, least value of T is 2 year.

Chapter 18.indd 13 6/5/2015 11:21:27 AM


18.14 Chapter 18

18.
(c) Let, P be the amount deposit in two different banks. 22.
(a) Amount remaining after
In 1st bank, 7.5 

 5 
2 1 year = 4000 1 +  − 1500 = `2800
CI = P 1 +  −P  100 
 100 
2  7.5 
 21  2 year = 2800 1 +  − 1500 = `1510
= P   − P  100 
 20 
441P  7.5 
= −P 3 year = 1510 1 +  − 1500 = `123.25
400  100 

41P
= 23.
(b) Let, Amit borrow `x.
400
 2  x×8× 2
In IInd bank, 8 
x  1 +  − 1 − = 16
P × R ×T P ×5× 2 P  100   100
SI = = = 
100 100 10
⇒ 0.1664x – 0.16x = 16
41P P
\ − = 200 16
400 10 ⇒x= = `2500.
0.0064
P
⇒ = 200
400 24.
(c) Let, the output be x and percentage be a.
⇒ P = `80000. Then,
19.
(d) Actual amount to be paid a 
2

3 \ x × 1 +  = 2x
 10   100 
= 20000 × 1 + 
 100  a
⇒ 1 + = 2
= `26620 100
\ Extra amount that Rohit have to pay
a
2 ⇒ = 2 – 1 ⇒ a = ( 2 – 1) × 100%
 10   15  100
= 20000 1 +  1 + =` 27830
 100   100 
25.
(d) Rate of 8% per annum payable half yearly.
\ Required amount = 27830 – 26620 = `1210.
So, effective rate = 4%
20.
(a) Let, the height of tree when it was planted be x cm.
4×4
3 Effective annual rate = 4 + 4 + = 8.16%
 30  100
\ x × 1 +  = 670
 100 
10 × 10 × 10 26.
(b) Let, rate of interest = R
⇒ x = 670 × ≈ 305 cm.
13 × 13 × 13 3
 R 
21.
(a) Let, the value of second rate of interest be x% and Then, 4800 = 3600 1 + 
 100 
equal amounts be P each.
6 3 3
 5   x  4  R 
\ P × 1 +  = P × 1 + ⇒ = 1 + (1)
 100   100 
 3  100 
2
 5   x  Now, the population after 3 years
⇒ 1 +  = 1 + 
 100   100  R 
3

= 4800 1 + 
105 × 105 100 + x  100 
⇒ =
100 × 100 100 From Eq. (1),

⇒ 110.25 = 100 + x 4
4800 × = 6400
⇒ x = 10.25% 3

Chapter 18.indd 14 6/5/2015 11:21:28 AM


Compound Interest 18.15

27.
(c) Let, Shyam’s share = x r 1
9 11
⇒ =
 5   5  100 10
Then, x 1 +  = (5887 – x) 1 + 100 
 100    \ r = 10%
2
x  5  2
⇒ = 1 +   r 
5887 − x  100  P 1 + 
 100  6
32.
(c) =
x  Pr  5
⇒ = 1.1025 P + 
5887 − x  100 
⇒ x = `3087
 r  6
⇒ 1 +  =
28.
(d) Population after 3 years  100  5
110 120 95 ⇒ r = 20%
= 10000 × × × = 12540
100 100 100
33.
(d) Cash price, CP = `39000
29.
(b) Let, the principal be P and rate of interest be r % Cash down payment, DP = `17000
Then, principal (when difference between CI and SI is Balance due, after Ist instalment, BD = `22000
for 2 years) is given by P = value of instalment = `4800
20 × (100) 2 n = no. of instalments = 5
P = (1)
r2 R = rate of interest
and principal (when difference between CI and SI is  nR   ( n − 1) R 
for 3 years) is given by \ 1 +  BD = 1 +  nP
 1200   2400 
61 × (100)3  5R   4R 
P = (2) ⇒ 1 +
r 2 (300 + r )
 22000 = 1 +  5 × 4800
 1200   2400 
From Eqs. (1) and (2),  5R  4R 

⇒ 1 + 11 = 1 + 12
20 × 10 4
61 × 10  1200   2400 
=
r2 r (300 r) 55 R 24 R
⇒ 11 + = 12 +
1200 1200
⇒ r = 305 – 300 = 5%
From Eq. (1), 55 R 24 R
⇒ − =1
4 1200 1200
20 × 10
P = 31R
25 ⇒ =1
= `8000 1200
2 1200
 100  ⇒ R = = 38.71%
30.
(b) Sum = Difference   31
 R 
34.
(c) Let, the amount of each instalment be `x.
2
 100  Amount of `100 for 3 years
= 20 ×   = 20 × 100
 10  3
 5 
= `2,000 = 100 1 + 
 100 
31.
(c) Let, the rate of compound interest be r.
21 21 21 9261
3 = 100 × × × =
`
 r  20 20 20 80
Then, 10000 × 1 +  = 13310
 100  9261
3 Present value of ` due after 3 years = `100
 r  1331 80
⇒  1 +  = 1000
 100  Present value of `x due after 3 years
r 11 100 × 80 8000 x
⇒ 1+ = = ×x=`
100 10 9261 9261

Chapter 18.indd 15 6/5/2015 11:21:29 AM


18.16 Chapter 18

Amount of `100 for 2 years 38.


(b) Suppose sum borrowed = `x
2 Amount paid, A = `(882 × 2) = `1764
 5  21 21 441
= 100 1 +  = 100 × × = Rate (r) = 5%, Time (t) = 2 years
 100  20 20 4
Thus,
400 2 2
Present value of `x due after 2 years = ` x  5   21  441
441 1764 = x 1 +  = x  = x
Similarly, present value of `x due after 1 year  100   20  400

20 1764 × 400
=` x x = = `1600.
21 441
8000 400 20
\ x+ x+ x = 126100 Pr 2
9261 441 21 39.
(d) Difference =
1002
⇒ 8000x + 8400x + 8820x = 126100 × 9261
⇒ 25220x = 126100 × 9261 Pr 2
⇒ 72 =
126100 × 9261 1002
⇒ x =
25220 72  100  100
⇒ r2 =  144
= `46305. 5000
\ r = 12%
35.
(b) Let, the share of Anjali be x.
40.
(c) Value after one year
\ Then, the share of Arun is (30600 – x).
5
3 2 = 20000 – 20000 × = `19000
 4   4  100
x × 1 +  = (30600 − x) 1 + 
 100   100  4
Value after 2 years = 19000 – 19000 ×
104 100
⇒ x × = 30600 – x = 19000 – 760
100
= `18240
204 2
⇒ x = 30600 Value after 3 years = 18240 – 18240 ×
100 100
= `17875.2.
⇒ x = `15000.

36.
(a) Let, the investment made by B = x 41.
(c) Let, the investment be `100.
2
Then, investment made by A = (81600 – x)  10 
\ After two years the sum = 100 1  = `121
 100 
2 3
 4   4 
\ (81600 − x) 1 +  = x 1 +  Q `121 is received when the investment was 100.
 100   100 
100
⇒ 81600 – x = 1.04x \ `1 is received when the investment was .
121
81600 \ `50000 is received when the investment was
x = = `40000.
2.04 100  50000
= = `41322.31.
121
37.
(b) Quicker Method: If a sum becomes ‘x’ times in ‘y’ years
at compound interest it will be (x)n times in ‘ny’ years. 42.
(b) Principal = (Present worth of `121 due 1 year hence)
Here x = 8 and (x)n = 16. Here we have to find the + (Present worth of `121 due 2 years hence)
value of n.
121 121
4 = +
(8)n = 16 ⇒ 23n = 24 ⇒ n =  10   10 
2
3 1 +  1 + 
y = 3 years.  100   100 
4
Hence the money will become sixteen times in ×3 121 121
3 = = `210.
= 4 years. 1.01 1.01 × 1.01

Chapter 18.indd 16 6/5/2015 11:21:30 AM


Compound Interest 18.17

3 For second year, p = `(6600 − 2000) = `4600


 r  
43. (b) CI = P 1 +  − 1 4600 × 10 × 1
 100   SI = ` = ` 460
100
  15 
3  At the end of second year, amount = `(4600 + 460) =
P  1 +  − 1 = 9844.5375 `5060
  100  
Return back = `2000
⇒ P[1.520875 – 1] = 9844.5375 For third year, p = `(5060 − 2000) = `3060
9844.5375 3060 × 10 × 1
\ P = = `18900. SI = ` = ` 306
0.520875 100
∴  Amount returned in third year = `(3060 + 306) =
5×5 `3366.
44.
(a) Required % = 5 + 5 + = 10.25%
100 50. (d) Let, the value of Sharma’s total saving = p
45.
(c) After first year, the value of the scooter = `20,000 Then, difference between CI and SI
After second year, the value of scooter = `16,000 SI × R
D=
After third year, the value of scooter = `12,800. 200
550 × R
46. (b) S.I. for one year = `1,100 ⇒ 605 − 550 =
200
\ Amount at the end of 2nd year
1100 × 100 55 R
= = `11,000. ⇒ 55 = ⇒ R = 20%
10 × 1 20
PR 2
47.
(b) Let, the population be x, three years ago Again, D =
1002
5x
\ Population two years ago = x + 25% of x = P × 20 × 20
4 ⇒ 55 =
100 × 100
5x 5x
Population one year ago = + 5% of ⇒ P = 55 × 25 = 1375
4 4
Hence, total saving = `1375 × 2 = `2750.
5x 5x 25 x
= + =
4 16 16 51. (b) If the sum be P, then we are given,
4 2
25 x 25x  20   20 
\ Present population = + 25% of P 1 +  − P 1 +  = 482
16 16  200   100 
4 2
125 x  11  6
= ⇒ P   − P   = 482
64  10  5
= 10000000 (Given) ⇒ P (1.4641 − 1.44) = 482
⇒ x = 5120000 482
⇒ P= = 20000.
5x x 0.0241
\ Required difference = −x=
4 4 52. (d) Tax for first 3 months
= 12,80,000. 1040000 × 0.25 × 3
= = ` 650
48.
(c) Interest on `1,440 = `216 for the third year 12 × 10
216 × 100 Then, Palvinder pays 40000 after 3 months
⇒ Rate % = = 15.
1440 × 1 ∴ Amount left = 1040650 − 40000
= `1000650
49. (b) For first year, p = `6000
Again the tax is calculated for next 3 month on
6000 × 10 × 1
SI = = ` 600 `1000650
100
At the end of first year, amount = `(6000 + 600) = `600 1000650 × 0.25 × 3
= = ` 625.40
Return back = `2000 12 × 100

Chapter 18.indd 17 6/5/2015 11:21:35 AM


18.18 Chapter 18

Now, amount left with Palvinder 2


7
= (1000650 + 625.40) – 40000 ⇒ 196 = P ×  
6
= 961275.4
49
So, it takes 6 months for Palvinder. ⇒ 196 = P ×
36
53. (b) Let, the rate per cent of bill be r% per annum.
196 × 36
Let, the amount of bill be `A. ∴ P= = `144.
49
Then, we are given,
 7  55. (c) Installment (I) = `1000, r = 10% amd t = 2,
1200 100 + r 
A =  12  = 12 100 + 7 r  (1) Pincipal = ?
 
100  12 
I I
1016  5   5  Principal = +
A= 100 + r  = 10.16 100 + r  (2) R  R 
2
100  2   2  1+
100 1 + 
 100 
From Eqs. (1) and (2), we get
1000 1000 1000 1000
 7   5  = + = +
12 100 + r  = 10.16 100 + r  10  1 
2
1  1
2

 12   2  1+ 1 + 1+ 1 +

100  100   
10  10  
⇒ 1200 + 7 r = 1016 + 25.4r
1000 1000 10000 1000 × 100
⇒ 18.44r = 184 = × 10 + × (10) 2 = +
11 (11) 2 11 121
∴ r = 10%
= 909.09 + 826.44 = `1736 (approx ).
Hence, rate cent of the bill is 10% per annum.

2 50
54. (d) We have, A = `196, r = 16 % = % per annum 56. (a) We have
3 3
n = 2 years
SI × 100 6500 × 100
 r 
n
P= = = 6250
∴ A = P 1 +  R ×T 8 × 13
 100  2
 8 
50 
n 2 CI = 6250 1 +  − 6250
  1  100 
⇒ 196 = P 1 +  = P 1 + 
 3 × 100   6 = `1040

Difficulty Level-2

1.
(a) Given Compound Interest – Simple Interest = `76.25 2.
(c) Suppose value of the asset two years ago = `x
n
 R  PRN
\ Value of the asset one year ago
⇒ P 1 +  −P− = 76.25
 100  100 9x
3
= x – 10% of x =
 5  P ×5×3 10
⇒ P 1 +  –P– = 76.25
 100  100 9x
\ x − = 12000 ⇒ x = 120000
10
 21 21 21  15 P
=P  × ×  –P–
  = 76.25
 20 20 20  100 \ Value of the asset at present
9261P − 8000 P − 1200 P 9x 9x 81x
=
  = 76.25 = − 10% of = = 81 × 1200
8000 10 10 100
⇒ 61P = 8000 × 76.25 ⇒ P = 10,000 = `97200.
PRN
Simple Interest paid by A =
100 3.
(d) Suppose A get `x

10,000 × 5 × 3 \ B gets `(25000 – x)


 = = `1500.
100 Interest received by A @ 8% p.a. C.I.

Chapter 18.indd 18 6/5/2015 11:21:39 AM


Compound Interest I 18.19

2 5
 8    5  
= x 1 +  −x = 1600 × 1 ×   = `1944.81
 100    100  
2
 27  104 x C.I. = (1944.81 – 1600) = `344.81
=   x − x =
 25  625
1600 × 10 × 2
Interest received by B @ 10% of p.a. S.I. S.I. = = `320
100
(25000 − x) × 10 × 2
= \ C.I. – S.I. = (344.81 – 320) = `24.81.
100

25000 − x 7.
(b) C.I. for 2 years = `756
=
5 S.I. for 2 years = `720
Given: It means the interest on the interest for the first year
25000 − x 104 x = `36 (=756 – 720)
= + 1336 ⇒ x = 10000.
5 625 This implies that the rate of interest is 5% as
36
× 100 = 5%
4.
(b) Let, the population be x, three years ago 720
5x It means the principal for first year was 14400
\ Population two years ago = x + 25% of x =
4 P × 5 ×1
Q = 720
5x 5x 100
Population one year ago = + 5% of
4 4 ⇒ P = 14400

5x 5x 25 x P×k ×k
= + = Now, = S.I.,  where r = t = k
4 16 16 100

14400 × k 2
25 x 25x ⇒ = 900
\ Present population = + 25% of 100
16 16
5
⇒ k=
125 x 2
= = 10000000
64
(Given)
900 × 100
⇒ x = 5120000 8.
(c) Sum = = `5,000
6×3
5x x \ Interest on `5,000 by C.I.
\ Required difference = −x =
4 4
3
= 1280000.  6 
= 5000 1 +  – 5000
 100 
5.
(b) S.I. for one year = `1100 = `955.08
1100 × 100
\ Amount at the end of 2nd year = \ More interest = `(955.08 – 900)
10 × 1
= `55.08.
   = `11000.

Decrease in second year 100 10


6.
(a) `16 is the S.I. on S.I. for 1 year 9.
(c) = =
Decrease in third year 100 – r 9

x × 10 × 1
\ 16 = or, x = `160 ⇒ r = 10%
100
Let, the population of vultures 3 years ago be P, then
Therefore, `160 is the simple interest for the first year.
3
 10 
 100 × 160 
P 1 −  = 29160
Now principal = `   = `1,600  100 
 10 × 1 
Amount for 2 years compounded half yearly ⇒ P = 40,000.

Chapter 18.indd 19 6/5/2015 11:21:40 AM


18.20 I Chapter 18

(b) 100(1.3)3 = 219.7 ⇒ C.I. = 119.7


10. Principal of all three instalments

100 × 3 × 30    25   25 2  25 3  
and, S.I. = = 90 = 17576    +   +    
100   26   
   26   26 
\ C.I. is greater than S.I. by `29.7  (119.7 – 90)
17576 × 25 × 1951
= = 48775
29.7 26 × 676
\ % increase = × 100 = 33.0%
90
Total amount paid = `17576 × 3 = `52,728
11.
(a) Let, the amount with the young son be x, time be 20 Interest charged = 52728 – 48775 = `3,953.
years rate 3%
   The amount with the elder son be (18750 – x) time 15.
(d) Suppose A gets `x
be 17 years, rate 3%
\ B gets `(25000 – x)
We know that
Interest received by A at the rate of 8% p.a. C.I.
t
 R  2
A = P 1 
 100  
= x 1 +
8 
 −x
 100 
Since, both receive the same amount.
2
 27 
20 17 =   x − x
 3   3   25 
\ x 1  = (18750  x) 1 
 100   100 
104 x
3 =
3  18750  x
⇒ 1   = 625
 100  x
Interest received by B at the rate of 10% of p.a. S.I.
3
 103  18750 (25000 − x) × 10 × 2
⇒  = 1
 100  x =
100
18750 25000 − x
⇒ 2.092727 = =
5
18750 Given:
⇒ x = = 8959.60
2.092727 25000 − x 104 x
= + 1336
       ≈ 8959.80. 5 625
12.
(c) If the rate of interest changed by the second part is r%,
then on the first it is (r + 10)% ⇒ x = 10,000
2 2 16. (c) Suppose value of the asset two years ago = `x
 r  10   r 
30000 1    30000 1  = 7500 \ Value of the asset one year ago
 100   100 
      r = 20% 9x
= x – 10% of x =
Let, the first part be x, then the second part will be 10
(60000 – x).
9x
2 2 \ x− = 12000
30  20 
x 1  
 (60000  x) 1  = 38800 10
 100   100  ⇒ x = 120000
  x = 40000
\ Value of the asset at present
13.
(b) Till forth year plan B will fetch `48 and plan A will
fetch `46. Fifth year interest from plan B will be `60 9x 9x
= − 10% of
and from plan A will be `61. 10 10
81x
14.
(b) Rate of interest = 16% annum = = 81 × 1200
100
Actual rate of interest = 4% per quarter = `97,200.

Chapter 18.indd 20 6/5/2015 11:21:41 AM


Compound Interest 18.21

17.
(d) Suppose the sum borrowed = `x 1
 8 
Rate of interest = R% x = P 1 + 
 100 
Time = 2 years
25
= ` x
x×R×2 75
\ 4000 = ⇒ Rx = 200000 (1)
100 2
 25 
R 
2 Similarly, principal for the 2nd instalment =   x
  27 
Now, x 1 +  = x + 4200
 100  3
 25 
2 Principal for 3rd year instalment = `   x
xR 2 RX  27 
⇒ + = 4200
10000 100 Total principal for the three instalments
⇒ 20R + 4000 = 4200
2 3
⇒ R = 10 25  25   25 
= x+  x+  x
27  27   27 
16000 × 30 = 50725
18. (d) SI = = 4800
100 On solving, we get x = 19683
\ Amount = 16000 + 4800 = 20800 \ Each instalment = `19683.
2
 12 
Now, CI = 20800 × 1 + 
 100  676 676
21.
(a) + 2
 4  
28 28 1 +  1 + 4 
= 20800 × ×  100   100 
25 25
= 26091.52 (Amount) 676 × 25 676 × 625
= +
\ A – P = 26091.52 – 20800 26 676

= 5291.52 = 650 + 625


After four years, interest = 5291.52 + 4800 = `1275.

 = 10091.52. 22. (a) Given A = `2240, t = 4 years r = 8%

120 Amount of annual installment


(a) At the end of 1st year = 4000 ×
19. = 4800
100  
But he pays back = 1500  100 A 
= 
At the end of 2nd year 100t + rt (t − 1) 
 2 
120
= 3300 × = 3960 – 1500 = 2460 100 × 2240
100 =
8× 4×3
120 100 × 4 +
At the end of 3rd year = 2460 × 2
100 100 × 2240
=
= `2952.
     400 + 48
100 × 2240
20.
(b) Cash price of the flat = `55000 =
448
In the instalment plan, cash down payment = `4275 = ` 500.

= 55000 – 4275 = 50725
23. (c) Let, the sum be P, rate r% per annum and time t years.
Let, each instalment be `x.
We are given,
Rate = 16% per annum = 8% half-yearly
t   r 
2
 P×r×2
 r   P 1 +  − P − = 20 (1)
\ A = P 1 −    100   100
 100 

Chapter 18.indd 21 6/5/2015 11:21:43 AM


18.22 Chapter 18

2 Pr Pr 2 2 Pr r Pr 2 Pr 2
⇒ P+ + −P− = 20 ⇒ × + 3 × = 61
100 (100) 2 100 100 (100) 2 (100) 2
Pr 2 r
∴ = 20 ⇒ × 20 + 3 × 20 = 61 [from Eq. (1))]
(100) 2 100

r
  3
 P×r ×3 ⇒ =1⇒ r = 5
r  5
and,  P 1 +  − P − = 61
  100   100 Pr 2 P × (5) 2
Now, 2
= 20 ⇒ = 20
Pr 3 3Pr 3Pr 2 3Pr (100) (100) 2
⇒ P+ + + −P− = 61
3
(100) 100 (100) 2 100 P × 25
⇒ = 20
Pr 3 3Pr 2 100 × 100
∴ + = 61 (2) ⇒ P = 400 × 20
(100) (100) 2
3

Using Eq. (1) in Eq. (2), we get = `8000.

Chapter 18.indd 22 6/5/2015 11:21:44 AM


CHAPTER

Logarithms 19
INTRODUCTION ‘log’ being the abbreviation of the word ‘logarithm’. Thus,
Logarithm, in mathematics, is the ‘exponent’ or ‘power’ to am = b ⇔ logab = m
which a stated number called the base is raised to yield a where, am = b is called the exponential form and log b = m
a
specifi c number. For example, in the expression 102 = 100, is called the logarithmic form.
the logarithm of 100 to the base 10 is 2. This is written
as log10 100 = 2. Logarithms were originally invented to Illustration 1 Refer to the following table
help simplify the arithmetical processes of multiplication,
division, expansion to a power and extraction of a ‘root’, Exponential form Logarithmic form
but they are nowadays used for a variety of purposes in pure
5
and applied mathematics. 3 = 243 log3 243 = 5
24 = 16 log2 16 = 4
Logarithm 0
3 = 1 log3 1 = 0
If for a positive real number (a ≠ 1), am = b, then the index
m is called the logarithm of b to the base a. We write this as 1
81/3 = 2 log8 2 =
logab = m 3

laWs of loGaritHMs

1. Product formula 4. Base changing formula


The logarithm of the product of two numbers is log a m log m
equal to the sum of their logarithms. lognm = . So, logn m =
log a n log n
i.e., loga (mn) = logam + logan
Generalisation: In general, we have where m, n, a are positived and n ≠ 1, a ≠ 1.
loga (mnpq...) = loga m + loga n + loga p + loga q +... 5. Reciprocal relation
2. Quotient formula logba × logab = 1
The logarithm of the quotient of two numbers is where a and b are positive and not equal to 1.
equal to the difference of their logarithms.
m 1
i.e., loga   = logam – logan 6. logba =
log a b
n
where a, m, n are positive and a ≠ 1. 7. aloga x = x, where a and x are positive, a ≠ 1.
3. Power formula 8. If a > 1 and x > 1, then loga x > 0.
The logarithm of a number raised to a power is equal
9. If 0 < a < 1 and 0 < x < 1, then loga x > 0.
to the power multiplied by logarithm of the number.
i.e., loga (mn) = n loga m, 10. If 0 < a < 1 and x > 1, then loga x > 0.
where a, m are positive and a ≠ 1. 11. If a > 1 and 0 < x < 1, then loga x < 0.

Chapter_19.indd 1 6/5/2015 12:07:48 PM


19.2 Chapter 19

soMe useful forMulae

natural logarithms. The logarithms to base 10 are called the


1. Logarithm of 1 to any base is equal to zero. common logarithms.
i.e., loga 1 = 0, where a > 0, a ≠ 1.
2. Logarithm of any number to the same base is 1. log1010 = 1, since 101 = 10.
i.e., loga a = 1, where a > 0, a ≠ 1. log10100 = 2, since 102 = 100.
log1010000 = 4, since 104 = 10000.
Common Logarithms log100.01 = –2, since 10–2 = 0.01.
There are two bases of logarithms that are extensively log100.001 = –3, since 10–3 = 0.001
used these days. One is base e (e = 2. 71828 approx.) and
2. 71828 approx.) and and, log101 = 0, since 10° = 1.
the other is base 10. The logarithms to base e are called

Practice Exercises

Difficulty level-1
(BaseD on MeMory)

1. The number of real solutions of the equation 7. The difference between the logarithms of sum of the
log (– x) = 2 log (x + 1) is: squares of two positive numbers A and B and the sum of
(a) One (b) Two logarithms of the individual numbers is a constant C. If
(c) Three (d) Four A = B, then C is:
[Based on MAT, 2005] (a) 2 (b) 1.3031
2 (c) log 2 (d) exp (2)
2. If log5 (x + x) – log5 (x + 1) = 2, then the value of x is:
[Based on MAT (Sept), 2003]
(a) 5 (b) 32 x/2
(c) 25 (d) 10 8. If logx a, a , and logb x are in GP, then x is:
[Based on FMS (Delhi), 2007] (a) loga(logb a)
3. If log12 27 = a, then log6 16 is: (b) loga(logea) + loga(loge b)
(a) (3 – a)/4 (3 + a) (b) (3 + a)/4 (3 – a) (c) – loga (loga b)
(c) 4(3 + a)/(3 – a) (d) 4(3 – a)/(3 + a) (d) loga (loge b) – loga (loge a)
[Based on MAT (Dec), 2006] [Based on MAT (Dec), 2002]
9. If loga (ab) = x, then logb (ab) is:
1 1 k
4. If logab = , logbc = and logca = , then the value of 1 x
2 3 5 (a) (b)
k is: x x +1
(a) 25 (b) 35 x x
(c) (d)
(c) 30 (d) 20 x −1 1− x
[Based on MAT (May), 2006]
[Based on MAT (May), 2002]
5. Which of the following is true? 1 1
(a) log17 275 = log19 375 10. If log8 x + log 8 = , then the value of x is:
(b) log17 275 < log19 375 6 3
(c) log17 275 > log19 375 (a) 18 (b) 24
(d) None of the above (c) 16 (d) 12
[Based on MAT (May), 2005] [Based on MAT (May), 1998]
1 1 1
6. log10 25 – 2 1og10 3 + log10 18 equals to: 11. If log 8 x + log 8 = , then the value of x is:
2 6 3
(a) 18 (b) 1 (a) 18 (b) 24
(c) log10 3 (d) None of these (c) 16 (d) 12
[Based on MAT (Sept), 2003] [Based on MAT, 1998]

Chapter_19.indd 2 6/5/2015 12:07:49 PM


Logarithms 19.3

12. If log10 (x2 – 6x + 45) = 2, then the value of x are: 23. Find log3/2 3.375
(a) 6, 9 (b) 9, –5 (a) 2 (b) 3
(c) 10, 5 (d) 11, –5 (c) 5/2 (d) 17/2
[Based on FMS, 2005]
16 25 81
13. If log10 125 + log10 8 = x, then x is equal to: 24. 7 log + 5 log + 3 log =
15 24 80
(a) –3 (b) 3 (a) log 2 (b) log 3
1
(c) (d) 0.064 (c) log 5 (d) None of these
3
[Based on FMS, 2005]
14. log (a2 / bc) + log (b2 / ac) + log (c2 / ab) is: log 49 7 + log 25 5 − log 4 2
25. =
(a) 1 (b) 0 log17.5
(c) 39 (d) abc (a) 5 (b) 2
[Based on FMS, 2006] (c) 5/2 (d) 3/2
15. What is the value of the following expression?
log (9/14) – log (15/16) + log (35/24) 26. log10 tan40° × log10 41° ... log10 tan 50° =
(a) 0 (b) 1 (a) 1 (b) 0
(c) 2 (d) 3 (c) –1 (d) None of these
[Based on IIFT, 2005]
1 1
16. The characteristics of the logarithm of 21735 is: 27. If y = ,z= and x = ak, then k =
a1− log a x a1− log a y
(a) 2 (b) 1
(c) 3 (d) 4 1 1
(a) 1− log a z
(b)
[Based on ATMA, 2005] a 1 − log a z
17. If loga 3 = 2 and logb 8 = 3, then loga b is: 1 1
(a) log3 2 (b) log2 3 (c) (d)
1 + log z a 1 − log z a
(c) log3 4 (d) log4 3
28. If loge 2 × logb 625 = log1016 × loge 10, then b =
18. If log7 2 = m, then log49 28 is equal to:
(a) 4 (b) 5
1 + 2m
(a) 2(1 + 2m) (b) (c) 1 (d) e
2
2
(c) (d) 1 + m log 7 log 7 5
1 + 2m 29. 5 5 − 7
If log10 x = y, then log1000 x2 is equal to:
19. (a) log 2 (b) 1
(a) y2 (b) 2y (c) 0 (d) None of these
3y 2y 30. 2log37 – 7log32
(c) (d)
2 3 (a) log2 7 (b) log 7
1 (c) log 2 (d) 0
20. log10 25 − 2log10 3 + log10 18 equals:
2
31. If log30 3 = a, log30 5 = b, then log30 8 =
(a) 18 (b) 1
(a) 3(1 – a – b) (b) a – b + 1
(c) log103 (d) None of these
(c) 1 – a – b (d) 1(a – b + 1)
21. If loga(ab) = x, then logb(ab) is:
32. log5 2 is:
1 x (a) an integer (b) a rational number
(a) (b)
x x +1 (c) an irrational number (d) a prime number
x x
(c) (d) log 27 9 × log16 64
x −1 1− x 33. What is the value of ?
log 4 2
22. A certain type of bacteria reproduces itself at the rate
1
of 10% every 5 minutes. In how many minutes will the (a) (b) 1
number double itself? 6 4
(a) 20 minutes (b) 30 minutes (c) 8 (d ) 4
(c) 40 minutes (d) None of these [Based on SNAP, 2013]

Chapter_19.indd 3 6/5/2015 12:07:50 PM


19.4 Chapter 19

Difficulty Level-2
(Based on Memory)

1. If log10 x − log10 x = 2 logx10, then a possible value of 1


7. If log x (0.1) = – , then the value of x is:
x is given by: 3
(a) 10 (b) 100
(a) 10 (b) 1/100
1
(c) 1/1000 (d) None of these (c) 1000 (d)
1000
[Based on CAT, 2004]
[Based on FMS (MS), 2006]
2.
What is the sum of ‘n’ terms in the series:
8. If log7 log5 ( x  5  x ) = 0, what is the value of x?
 m2   m3   m4  (a) 2 (b) 3
log m + log  + log  2  + log  3  + ...
 n  n  n  (c) 4 (d) 5
     
[Based on FMS (2009]
n/2 n/2 9. What is the value of x, if
n ( n −1)
 n m 
(a) log  ( n+1)  (b) log  n  1 1 1 1
 m   n  + + + ... +
log 442 x log 443 x log 444 x log 899 x
n/2 n/2 441 442 443 898
 m(1−n)   m( n+1) 
(c) log  (1+ n)  (d) log  ( n−1)  1
+ = 2?
 n   n  log 900 x
[Based on CAT, 2004] 899
2
3. The length of the circumference of a circle equals the (a) (b) 1
21
perimeter of a triangle of equal sides, and also the
perimeter of a square. The areas covered by the circle, 7 10
(c) (d)
triangle, and square are c, t and s respectively. Then, 10 7
(a) s > t > c (b) c > t > s 10. What is the value of [log10 (5 log10 100)]2?
(c) c > s > t (d) s > c > t (a) 25 (b) 10
[Based on CAT, 2004] (c) 2 (d) 1
4.
If logn 48 = a and logn  108 = b, what is the value of [Based on FMS, 2010]
logn 1296 in terms of a and b? 11.
If log10 2 = a and log10 3 = b, then log5 12 equals:
ab 2a  b
2(2a + b) (a + 3b) (a) (b)
(a) (b) 1 a 1 a
5 5
a  2b 2a  b
4(2a + b) 2(a + 3b) (c) (d)
(c) (d) 1 a 1 a
5 5
[Based on FMS, 2010]
n
1 12. If a = log8 225 and b = log2 15, then a in terms of b, is:

5.
log (a)
=
(a) b/2 (b) 2b/3
n =1 2n
(c) b (d) 3b/2
n (n + 1) n (n − 1) [Based on FMS, 2010]
(a) log a 2 (b) log 2 a
2 2 13. If log x – 5 log 3 = –2, then x equals:
(n + 1) 2 n 2 (a) 1.25 (b) 0.81
(c) log a 2 (d) None of these (c) 2.43 (d) 0.8
4
[Based on FMS, 2011]
a b (125) (625)
6. If log + log = log (a + b), then 14. The value of log5 is equal to:
b a 25
(a) a + b = 1 (b) a – b (a) 725 (b) 6
(c) a2 – b = 1 (d) a = b (c) 3125 (d) 5
[Based on FMS (Delhi), 2007] [Based on FMS, 2011]

Chapter_19.indd 4 6/5/2015 12:07:51 PM


Logarithms 19.5

15. The values of a in the equation: log10 (a2 – 15a) = 2 are: log a log b log c
15  233 25. If = = , then which of the following
(a) (b) 20, – 5 b−c c−a a−b
2 options holds true?
15  305 (a) ab . bc . ca = 1 (b) aabbcc = 1
(c) (d) ± 20
2 2a 2b 2c
(c) a b c = 1 (d) aabbbccac = 1
[Based on FMS, 2011]
26. If a, b, c are distinct positive numbers different from
log x log y log z 1 such that (logba . logca – logaa) + (logab . logcb – logbb)
16. If   , mark all the correct options.
bc ca ab + (logac . logbc – logcc) = 0 then abc =
(a) xyz = 1 (b) xa yb zc = 1 (a) 0 (b) e
b+c c+a a+b
(c) x y z = 1 (d) xb+c yc+a za+b = 0 (c) 1 (d) None of these
[Based on ITFT, 2006]
27. The value of 32log93 is:
17. If 2 log x = 5 log y + 3, then the relation between x and y
is: (a) 4 (b) 3
(a) x2 = 100y5 (b) x1/5 = 1000 y1/2 (c) 47 (d) 8
2 5
(c) x = 1000y (d) x2 = y5 + 1000 28. If log10 x − log10 x = 2 logx10, then a possible value of
a x is given by:
18.
What is the value of , if log 4 log 4 4a–b = 2 log4 (a) 10 (b) 1/100
b
( a  b )  1? (c) 1/1000 (d) None of these

5 29. The length of the circumference of a circle equals the


(a)  (b) 2 perimeter of a triangle of equal sides, and also the
3
perimeter of a square. The areas covered by the circle,
5 triangle, and square are c, t and s respectively. Then,
(c) (d) 1
3 [Based on IIFT, 2010] (a) s > t > c (b) c > t > s
(c) c > s > t (d) s > c > t
 1010
10

19. The value of log10log10 log10log10 1010  is: 30. The difference between the logarithms of sum of the
 
  squares of two positive numbers A and B and the sum
1 of logarithms of the individual numbers is a constant C.
(a) 1 (b) If A = B, then C is
10
(a) 2 (b) 1.3031
(c) 10 (d) Cannot be determined
(c) log 2 (d) exp (2)
20. log10(log2 3) + log10(log3 4) + … + log10(log1023 1024)
equals: 31. The number of real solutions of the equation
(a) 10 (b) e log (–­x) = 2 log (x + 1) is:
(c) 1 (d) 0 (a) One (b) Two
[Based on JMET, 2006] (c) Three (d) Four
21. log4 2 – log8 2 + log16 2 – ... to ∞ is: ( )
32. If log7 log5 ( x + 5 + x ) = 0, find the value of x.
(a) e2 (b) ln 2 + 1
(a) 1 (b) 0
(c) ln 2 – 1 (d) 1 – ln 2
(c) 2 (d) None of these
22. If log2x + log2 y ≥ 6, then the least value of xy is:
33. log2[log7 (x2 – x + 37)] = 1, then what could be the value
(a) 4 (b) 8 of x?
(c) 64 (d) 32 (a) 3 (b) 5
23.
Solve for x, log10 x + log x + log 3 100 x =
27 (c) 4 (d) None of these
10
1
(a) 1 (b) 106 34. If log3 M + 3 log3N = 1 + log50.008, then:
3
(c) 104 (d) 10
9 9
If y = 21/logx 4, then x is equal to:
24. (a) M9 = (b) N9 = 
N M
(a) y (b) y 3 3
2 (c) M3 = (d) N9 =
(c) y (d) y4 N N

Chapter_19.indd 5 6/5/2015 12:07:52 PM


19.6 Chapter 19

35. If log10x – log10 x = 2 logx 10, then a possible value of 46. xlog y–log z × ylog z–log x × zlog x–log y =
(a) 0 (b) 2
x is given by:
(c) 1 (d) None of these
1
(a) 10 (b)
100 47. If log10  [98 + x 2 − 12 x + 36] = 2, then x =
1
(c) (d) None of these (a) 4   (b)  8   (c)  12   (d)  4, 8
1000
48. If x = loga bc, y = logb ca, z = logc ab, then
36. Let, u = (log2 x)2 – 6 log2 x + 12, where x is a real number.
Then, the equation xn = 256, has: (a) xyz = x + y + z + 2 (b) xyz = x + y + z + 1
(a) no solution for x (c) x + y + z = 1 (d) xyz = 1
(b) exactly one solution for x 49. If ax = by = cz = dw, then loga (bcd) =
(c) exactly two distinct solutions for x
(d) exactly three distinct solutions for x 1 1 1 1  1 1 1
(a)  + +  (b) x  + + 
1 1 x y z w  y z w
37. If log8x + log8  = , then the value of x is:
6 3 y+z+w
(c) (d) None of these
(a) 18   (b)  24   (c)  16   (d)  12 x
38. Which of the following is true? 50. If log10 2 = 0.3010, then log10 (1/2) =
(a) log17275 = log19375 (b) log17275 < log19375 (a) –0.3010 (b) 0.6990
(c) log17275 > log19375 (d) None of these (c) 1 .6990 (d) 1 .3010
39. If x = log2a a, y = log3a 2a and z = log 4a 3a, find yz (2 – x).
51. If log2 (32x–2 + 7) = 2 + log2 (3x–1 + 1), then x =
(a) 1 (b) –1
(a) 0   (b)  1   (c)  2   (d)  1 or 2
(c) 2 (d) ­–2
52. If loga b = logb c = logc a, then:
log x log y log z
40. = = , find x2y2z2. (a) a > b c (b) a < b < c
l + m − 2m m + n − 2l n + l − 2m
(c) a = b = c (b) a < b ≤ c
(a) 2 (b) –1
1 2
(c) 4 (d) 1 53. If = – 2, then x =
log x 10 log a 10
x+ y 1 x y
41. If log = (log x + log y), then + = (a) a/2   (b)  a/100   (c)  a2/10   (d)  a2/100
5 2 y x
(a) 20 (b) 23 1 1
54. If a2 + b2 = c2, then + =
(c) 22 (d) 21 log c + a b log c − a b

 3x − 3 y  (a) 1   (b)  2   (c)  –1   (d) ­–2


42. If log (x + y) = log    , then log x ­­­­– log y =
 2  55. If log10 87.5 = 1.9421, then the number of digits in (875)10
(a) log 2 (b) log 3 is:
(c) log 5 (d) log 6 (a) 30   (b)  29   (c)  20   (d)  19
56. If log10 2 = 0.3010, log10 3 = 0.4771, then the number of
43. If log2 x + log4 x + log16 x = 21/4, then x = zeros between the decimal point and the first significant
(a) 8   (b)  4   (c)  2   (d)  16 figure in (0.0432)10 is:
(a) 10   (b)  13   (c)  14   (d)  15
44. If 0 < a x, the minimum value of loga x + logx a is:
(a) 1 (b) 2 1 1
57. If (4.2)x = (0.42)y = 100, then – =
(c) 3 (d) 5 x y
(a) 1   (b)  2   (c)  1/2   (d)  –1
log x log y log
45. If = = , then xyz = xa  × yb  × zc
b−c c−a a b log9 11 log3 11
58. – =
= xb+c × yc+a × za+b = log5 13 log 5 13
(a) 1 (b) 0 (a) 1 (b) –1
(c) 2 (d) None of these (c) 0 (d) None of these

Chapter_19.indd 6 6/5/2015 12:07:52 PM


Logarithms 19.7

log x log y log z (a) 3 (b) 3/2


59. If = = , then yz in terms of x is:
2 3 5 (c) 2 (d) 1
(a) x   (b)  x2   (c)  x3   (d)  x4 log x log y log z
1 1 72. If 2 2
= 2 2
= , then
x+ x− a + ab + b b + bc + c c + ca + a 2
2
60. If 4x + 22x – 1 = 3 2 +3 2 , then x =
xa–b × yb–c × zc–a =
(a) 1/2   (b)  3/2   (c)  5/2   (d)  1
(a) 0   (b)  –1   (c)  1   (d)  2
61. If log8 p = 2.5, log2 q = 5, then p in terms of q is:
73. If 3x–2 = 5 and log10 2 = 0.20103, log10 3 = 0.4771,
(a) 3 q (b) 2q then x =
(c) q (d) q/2 22187 22187
(a) 1 (b) 2
62. If 0 < a < 1, 0 < x < 1 and x < a, then loga x 47710 47710
(a) < 1   (b)  > 1   (c)  < 0   (d)  ≤ 1 22187
(c) 3 (d) None of these
47710
 1  1  1
63. log5 1 +  + log5 1 +  + log5 1 +  + ... 74. If log2 = 0.30103 and log3 = 0.4771, then number of digits
 5   6   7
in (648)5 is:
 1  (a) 12   (b)  13   (c)  14   (d)  15
+ log5 1 + 
 624 
log y log z
(a) 5   (b)  4   (c)  3   (d)  2 75. If log x = = , then x4 ×  y3 × z–2 =
2 5
64. If log10 2986 = 3.4751, then log10 0.02986 = (a) 2   (b)  10   (c)  1   (d)  0
(a) 1 .2986 (b) 2 .4751
76. log 27 + log 1000 + log8
(c) 0.34751 (d) None of these log120
65. If log (2a – 3b) = log a – log b, then a = (a) 1/2   (b)  1   (c)  3/2   (d)  2
2 x
3b 3b 10 log10
(a) (b) 77. For x > 0, if y = and x = ya, then a =
2b − 1 2b − 1 x 2

b2 3b 2 (a) 1   (b)  –1   (c)  0   (d)  2


(c) (d)
2b + 1 2b + 1 78. If x = 1004/3(1/2), y = log1/2(1/3), then:
1 1 x y (a) x > y   (b)  x < y   (c)  x = y   (d)  x ≥ y
66. If log(x – y) – log 5 – log x – log y = 0, then + =
2 2 y x 79. Let u = (log2x) 2 - 6 log2x + 12 where, x is a real number.
(a) 25   (b)  26   (c)  27   (d)  28 Then the equation xu = 256, has
(a) no solution for x.
67. If log x:3 = log y:4 = log z:5, then zx = (b) exactly one solution for x.
(a) 2y   (b)  y2   (c)  8y   (d)  4y (c) exactly two distinct solutions for x.
(d) exactly three distinct solutions for x.
68. If 3 + log5 x = 2 log25 y, then x =
[Based on CAT, 2010]
(a) y/125 (b) y/25
2 a 2 b2
(c) y /625 (d) 3 – y2/25 80. If log x (a − b) − log x (b/a ), find 2 + 2 .
b a
log 2 a log3 b log 4 c (a) 4   (b)  2   (c)  3   (d)  6
69. If = = and a1/2 × b1/3 × c1/4 = 24,
3 3 4 [Based on CAT, 2012]
then: 81. log2 [log7 (x2 − x + 37)] = 1, then what could be the value
(a) a = 24 (b) b = 81 of ‘x’?
(c) c = 64 (d) c = 256 (a) 3 (b) 5
log 2 x log 2 y log 2 z z (c) 4 (d) None of these
70. If = = and 3 4 = 1, then k = [Based on CAT, 1997]
3 4 5k x y
82. log6 216 6 is:
(a) 3   (b)  4   (c)  5   (d)  –5 3
(a) 3 (b)
3 + log10 343 2
71. = 7
1  49  1  1  (c) (d) None of these
2 + log   + log   2
2  4  3  125  [Based on CAT, 1994]

Chapter_19.indd 7 6/5/2015 12:07:55 PM


19.8 Chapter 19

84. The value of the expression:


83. If log7 log5 ( x + 5 + x ) = 0, find the value of x. 100
1
(a) 1 (b) 0 ∑
i = 2 log i 100!
is:
(c) 2 (d) None of these
(a) 0.01 (b) 0.1
[Based on CAT, 1994] (c) 1 (d) 10
(e) 100
[Based on XAT, 2014]

Answer Keys
Difficulty Level-1

1.  (a) 2.  (c) 3.  (d ) 4.  (c) 5.  (b) 6.  (b) 7.  (c) 8.  (a) 9.  (c) 10.  (d ) 11.  (d ) 12.  (d ) 13.  (b)
14.  (b) 15.  (a) 16.  (d ) 17.  (c) 18.  (b) 19.  (d ) 20.  (b) 21.  (c) 22.  (d ) 23.  (b) 24.  (a) 25.  (c) 26.  (b)
27.  (b) 28.  (b) 29.  (c) 30.  (d ) 31.  (a) 32.  (c) 33.  (d )

Difficulty Level-2

1.  (b) 2.  (d ) 3.  (c) 4.  (d ) 5.  (a) 6.  (a) 7.  (c) 8.  (c) 9.  (d ) 10.  (d ) 11.  (d ) 12.  (b) 13.  (c)
14.  (d ) 15.  (b) 16.  (a, b, c) 17.  (c) 18.  (c) 19.  (c) 20.  (c) 21.  (d ) 22.  (c) 23.  (b) 24.  (c) 25.  (b) 26.  (c)
27.  (b) 28.  (b) 29.  (c) 30.  (c) 31.  (a) 32.  (b) 33.  (c) 34.  (b) 35.  (b) 36.  (b) 37.  (d ) 38.  (b) 39.  (a)
40.  (d ) 41.  (b) 42.  (c) 43.  (a) 44.  (b) 45.  (a) 46.  (c) 47.  (d ) 48.  (a) 49.  (b) 50.  (c) 51.  (d ) 52.  (c)
53.  (d ) 54.  (b) 55.  (a) 56.  (b) 57.  (c) 58.  (c) 59.  (d ) 60.  (b) 61.  (a) 62.  (b) 63.  (b) 64.  (b) 65.  (a)
66.  (c) 67.  (b) 68.  (a) 69.  (d ) 70.  (c) 71.  (a) 72.  (c) 73.  (c) 74.  (d ) 75.  (c) 76.  (c) 77.  (b) 78.  (b)
79.  (b) 80.  (d ) 81.  (a) 82.  (c) 83.  (b) 84.  (c)­­­

Explanatory Answers

Difficulty Level-1

1.
(a) – x = (x + 1)2 ⇒ (x – 25) (x + 1) = 0
2
⇒ x + 3x + 1 = 0 ⇒ x = 25
−3 + 5 −3 − 5 3.
(d) log12 27 = a
\ x = ,
2 2
⇒ log12 33 = a
−3 + 5 a
But only satisfies the other condition of x + ⇒ log12 3 =
1 > 0. 2 3
3
log( x 2 + x) log( x + 1) ⇒ log3 12 =
2.
(c) – = 2 a
log 5 log 5
3
⇒ log3 4 + log3 3 =
log(x2 + x) – log (x + 1) = 2 log 5 a
 x2 + x  2 3−a
⇒ ⇒ 2 log3 2 =
log 
x + 1  = log 5 a
 
3−a
x2 + x ⇒ log3 2 = (1)
⇒ = 25 2a
x +1

Now, log6 16 = 4 log6 2 = A (let)
⇒ x2 – 24x – 25 = 0

Chapter_19.indd 8 6/5/2015 12:07:56 PM


19.9
Logarithms

A ⇒ log(2A2) – 2 log A = C
⇒ log6 2 = ⇒ log(2A2) – log A2 = C
4
4  2 A2 
⇒ log2 6 = ⇒ log  2  = C
A  A 
 
4− A ⇒ log 2 = C
⇒ log2 3 =
A (a) logx a, ax/2 and logb x are in GP, then
8.
A (ax/2)2 = (logx a) × (logb x)
⇒ log3 2 = (2) ⇒ ax = logb a
4− A
⇒ x log a = loga (logb a)
From Eqs. (1) and (2),
9.
(c) loga (ab) = x
3−a A 2a 4− A
= ⇒ =
2a 4 − A 3−a A log ab
⇒ =x
log a
2a 4− A
⇒ +1 = +1 log b
3−a A ⇒ =x–1

log a
2a + 3 − a 4− A+ A
⇒ = log a 1
3−a A ⇒ =
log b x −1

4(3 − a )
\ A = log ab log a + log b
(3 + a ) Now, logb (ab) = =
log b log b
1 1 k log a 1
4.
(c) loga b = , logb c = , logc a = = +1= +1
2 3 5 log b x −1
log b 1 log c 1 log a k 1+ x −1 x
⇒ = , = , = = =
log a 2 log b 3 log c 5 x −1 x −1

1 1 k 1 1
⇒ × × = 1 ⇒ k = 30 10.
(d) log8 x + log8 =
2 3 5 6 3
5.
(b) Let, log17 275 = log19 375  1 1
⇒ log8  x ×  =
log 275 log 375  6 3
⇒ =
log 17 log 19
 x 1
⇒ log8   =
275 375 6
  3
⇒ =
17 19 x
\ 16.18 < 19.74 ⇒ (8)1/3 =
6
Hence, log17 275 < log19 375 [Q if loga y = x, than (a)x = y]
1 x
6.
(b) log10 25 − 2 log10 3 + log10 18 ⇒ (23 )1/3 =
2 6
= log10 (25)1/2 − log10 (3) 2 + log10 18 ⇒ x = 12

= log10 5 − log10 9 + log10 18 1 1


11.
(d) log8 x + log8 =
5 6 3
= log10 + log10 18
9 1
log
5 × 18 log x 6 = 1
= log10 = log10 10 = 1 ⇒ +
9 log 8 log 8 3

(c) Given log(A2 + B2) – (log A + log B) = C
7. log x + log 1 − log 6 1
⇒ 3
=
If A = B, then log 2 3

Chapter_19.indd 9 6/5/2015 12:14:12 PM


19.10 Chapter 19

⇒ log x + log 1 – log 6 = log 2 log b


⇒ + 1 = x
log x + log 1 = log 6 + log 2 log a
log x = log 12
log b
\ x = 12 ⇒ = x – 1
log a
(d) x2 – 6x + 45 = 100
12.
x = –5, 11 log a 1
⇒ =
log b x −1
13.
(b) log10 125 + log10 8 = x
⇒ log10 (125 × 8) = x log ab log a
\ logbab = = 1+
⇒  10x = 1000 log b log b
\  x = 3
1 x
= 1 + =
 a 2 b2 c2  x −1 x –1
14.
(b) log  2  2  2  = log (1) = 0
b c a  22.
(d) Let, the number of bacteria in the beginning be P,
which doubles after time T.
9  15   35 
15.
(a) log   − log   + log   T
 14   16   24   10 
\ P 1 +  = 2P
9 16 35  100 
= log × × = log 1 = 0 [Here 1 unit of time = 5 minutes]
14 15 24
⇒ (1.1)T = 2
16.
(d) Characteristics = number of digits – 1 = 5 – 1 = 4
⇒ T log (1.1) = log 2
17.
(c) logb8 = 3 ⇒ 3logb2 = 3 ⇒ logb2 = 1
logab = log2b × loga2 = log2b × log32 × loga3 log 2
⇒ T =
log1.1
= 1 × log32 × 2 = 2log32 = log34
0.3010 3010
1 1 ⇒ T = =
18.
(b) log4928 = log 7 (7 × 4) = (1 + log 7 4) 0.0414 414
2 2
= 7.27 units
1 1 1
= + .2log 7 2 = + log 7 2 = 36.35 minutes
2 2 2
23.
(b) log3/2 3.375 = x
1 1 + 2m
= +m = x
2 2 3
⇒   = 3.375
2 2 2
(d) log1000(x2) =
19. log10 x = y
3 3 ⇒ (1.5)x = (1.5)3 ⇒ x = 3

1  24   52   34 
20.
(b) log 25 – 2 log 3 + log 18 24.
(a) 7 log  + 5log  3 + 3log  4
2  5 × 3   2 × 3   2 × 5 
     
1
= log 52 – 2 log 3 + log (2 × 3 × 3)
2 = 28 log 2 – 7 log 5 – 7 log 3 + 10 log 5 – 15 log 2 
= log 5 – 2 log 3 + log 2 + log 3 2 -5 log 3 + 12 log 3 – 12 log 2 – 3 log 5 = log 2

= log 5 + log 2 log 75/2 + log 55/2 − log 25/2


25.
(c)
= log (5 × 2) = log 10 = 1 log 17.5

21.
(c) logaab = x 5(log 7 + log 5 − log 2) 5
= =
log ab  35  2
⇒ = x 2log  
log a  2 

log a + log b 26.


(b) log10 tan40° × log10 tan41° ... log10 tan50°
⇒ =x
log a = 0, since log10 tan45° = 0

Chapter_19.indd 10 6/5/2015 12:07:59 PM


Logarithms 19.11

1 1 (d) 2log37 – 7log32 = 2log27 × log32 – 7log32


30.
27.
(b) loga  y = , loga z =
1 − log a x 1 − log a y = 7log32 – 7log32 = 0
1
1 − log a x
\ loga z =  1  =  30 
1−   − log a x 31.
(a) a + b = log3015 = log30   = 1 – log30 2
 1 − log a x   2 
1 ⇒ log30 2 = 1 – a – b.
⇒ – loga z = –1 +
log a x \ log30 8 = 3(1 – a – b)
1
⇒ = 1 – loga z
log a x p
32.
(c) log52 = ⇒ 2 = 5p/q = 2q = 5p
1 q
\ loga x =
1 − log a z ⇒ even number = odd number,
1 k  which is a contradiction.
⇒ x = z
= a (given)
a1− log a \ log52 is an irrational number.
1
\ k = 33. (d) We have
1 − log a z
log 27 9log16 64 log9 log 64 log 4
(b) loge 2 × 4 logb 5 = 4 × log10 2 × loge 10 = 4 loge 2
28. = × ×
log 4 2 log 27 log16 log 2
⇒ logb 5 = 1 ⇒ b = 5
2log3 6log 2 2log 2
1 = × ×
log5 7 log 7 5
3log3 4log 2 1 log2
29.
(c) 5 – (7log7 5 ) 2
log5 7 1 1 6
= 5 − = × × 4 = 4.
log 7 5 3 4
5
⇒ 5 log 57 log57 =0
−5

Difficulty Level-2

1.
(b) log10 x − log10 x = 2 logx10  n ( n +1)  n /2
m 2   m( n +1) 
1 2 = log  ( n −1) n  = log  ( n −1)  .
⇒ log10 x − log10 x =    n 
n
2 log10 x 2

log10 x 2 3. (c) If the length of the circumference of a circle equals the
⇒ =
2 log10 x perimeter of a regular polygon then
   Area of circle > Area of regular polygon
(log10x)2 = 4
   Also, if two regular polygons have the same
⇒ log10x = ±2 perimeter, then the regular polygon having larger no.
of sides will have area greater than that of the regular
1 polygon having less number of sides.
⇒ x = 10–2, 102 = , 100.
100
\ c > s > t.
 m2   m3 
2.
(d) log m + log   + log  2 
 n  n  4.
(d) Given logn48 = 4 logn2 + logn3 = a and,
   
logn 108 = 2.logn 2 + 3 logn 3 = b
 m4 
+ log  3  + ... + nth term Let, logn 2 = x and, logn 3 = y
n 
 
⇒ 4x + y = a (1)
 m ⋅ m 2 ⋅ m 2 ... m n   m(1+ 2 + 3+...+ n )  2x + 3y = b (2)
= log  2 3 n −1 
= log  (1+ 2 + 3+...+ n −1) 
 n ⋅ n ⋅ n ... n   n  2 × (2) – (1) gives

Chapter_19.indd 11 6/5/2015 12:08:00 PM


19.12 Chapter 19

6y – y = 2b – a  442 443 444 899 900 


⇒ log x  × × × ... × ×  =2
2b − a  441 442 443 898 899 
⇒ y =
5
900 900 30 10
⇒      log x = 2 ⇒ x2 = ⇒x= =
3a − b 441 441 21 7
Similarly, x =
10
(d) [log10 (5 log10 100)]2 = [log10 (5 × 2)]2
10.
logn 1296 = 4 (logn 2 + logn 3)
= [log10 10]2
 2b − a 3a − b 
= 4  + = 12 = 1
 5 10 
11.
(d)  log5 12 = log5 (3 × 4)
 a + 3b  2(a + 3b)
= 4 
10  = 5 = log5 3 + log5 4
 
= log5 3 + 2 1og5 2
n n
1
5. (a) ∑ = ∑ log a (2 n
) log10 3 2log10 2
n = 1 log 2n ( a ) n =1 = +
log10 5 log10 5
n
= ∑ n log a 2 =
log10 3

2 log10 2
n =1 log10 10  log10 2 log10 10  log10 2
n
n (n + 1) b 2a 2a  b
= log a 2 ∑n= 2
loga2 = 
1 a 1 a
=
1 a
n =1

a b 12.
(b) a = log8 225, b = log2 15
6.
(a) log + log = log (a + b)
b a 2 2
\ a = log 23 152 = s log215 =  b
3 3
a b
⇒ log  ×  = log (a + b)
b a 13.
(c) log x – 5 log 3 = – 2
\  a + b = 1
35
1 ⇒ log10 = – 2
7.
(c) log x (0.1) =  x
3
1 x
 = 10– 2
⇒ x 3
= 0.1 ⇒ x = 1000 243

243
8.
(c) log7 log5 ( x  5  x ) = 0 ⇒ x = ⇒ x = 2.43
100
⇒ 70 = log5 ( x  5  x )
(125) (625) (53 ) (54 )
⇒ log5 ( x  5  x ) = 1 14.
(d) log5 = log5
25 52
⇒ 51 = x  5  x
⇒ x = 4 = log557 – 2 = log5 55
= 5 log 55 = 5
9.
(d) The given equation
15.
(b) log10 (a2 – 15a) = 2
442
log  a2 – 15a = 102
1 441 log 442
= = = x 2
log 442 x log x 441 ⇒ a – 15a – 100 = 0
441
  (a – 20) (a + 5) = 0
442 443 444 ⇒ a = 20, – 5
log x + log x + log x + ...
441 442 443
16.
(a, b, c) Option (d) is wrong as the expression evaluate to
899 900 1 as in (c) and not zero. In all, options (a), (b) and (c)
+ log x + log x =2
898 899 are correct.

Chapter_19.indd 12 6/5/2015 12:08:01 PM


Logarithms 19.13

17.
(c) 2 log x = 5 log y + 3 23.
(b) Changing the base to 10, we get
2 5 log10 x
log x = log y + 3 log1010
log 10
x = = 2 log10 x
2 5
log x = log y + 3 log1010 3 log10 10
= log (y5 × 103) log10 x 3log10 x
log 3 100 x = 3
=
2 5 log10 100 2
⇒ x = 1000y
3
(c) Given, log4 log4 4a–b = 2 log4 ( a  b )  1
18. \ log10 x + 2log10 x + log10 x =
27
2
log4 (a – b) log4 4 = log 4 ( a  b ) 2  log 4 4 9
⇒     log10 x = 27
log 4 (a  b) = log 4 4( a  b ) 2 2

⇒  log10x = 6
(a – b) = 4( a  b ) 2
\ x = 106
⇒      a  b  4 a  4 b 1
1
log 2 x

3 a = 5 b 24.
(c) y = 2 log x 4 = 2log 4 x = 2 2

a 5 = 2log 2 x
= x
⇒ = .
b 3 2
\ x = y
19.
(c) logaa =1
log a log b log c
1010 25.
(b) = = =r
1010 b−c c−a a−b
= log10 10
\ a log a + b log b + c log c
1010 1010
= 1010 log1010 = 1010 = r[a(b – c) + b(c – a) + c(a – b)]
10
i.e., with each log10 one 10 is removed there are ⇒ log aa + log bb + log cc = 0
5 – 10s (including the ones in powers) and 4 – log10 ⇒ log(aa bb cc) = log 1
therefore, last will be log101010 = 10log1010 = 10.
⇒ aa bb cc = 1
20.
(c) log10 (log2 3) + log10 (log3 4) + … + log10
log1023 1024) 26.
(c) (logba logca – 1) + (logab . logcb – 1)
= log10 [(log2 3)(log3 4)(log4 5) … (log1023 1024)] + (logac logbc – 1) = 0
 log 3 log 4 log 5 log1024  log a log a log b log b log c log c
= log10  × × × ... × ⇒ . + . + . =3
 log 2 log 3 log 4 log1023  log b log c log a log c log a log b

 log a  ⇒ (loga)3 + (logb)3 + (logc)3 = 3 log a log b log c


 logb a = log b 
  ⇒ (loga + logb + logc) = 0
= log10 (log2 1024) = log10 log2 210 = log10 10 = 1 [Q If a3 + b3 + c3 – 3abc = 0 then a + b + c = 0 if a ≠ b ≠ c]
⇒ logabc = log 1 ⇒ abc = 1
1 1 1
21.
(d) Required sum = − + – ... to ∞ 1
2 3 4 2. log3 3
27. 32log9 3 3=
(b) = 2 3
x 2 x3
Now, loge(1 + x) = x – + – ...
2 3 28.
(b) log10 x − log10 x = 2 logx10
1 1
\ loge2 = 1 – + – ... to ∞ 1 2
2 3 ⇒ log10 x − log10 x =
2 log10 x
1 1 1
⇒ − + .. to ∞ = 1 – loge2 = 1 – ln 2 log10 x 2
2 3 4 ⇒ =
2 log10 x
22.
(c) Given: log2x + log2y ≥ 6 ⇒ (log10x)2 = 4 ⇒ log10x = ±2
⇒ log2(xy) ≥ 6 1
⇒ x = 10–2, 102 = , 100
⇒ xy ≥ 64 100

Chapter_19.indd 13 6/5/2015 12:08:02 PM


19.14 Chapter 19

29. (c) If the length of the circumference of a circle equals the log e 10 − log e 2
perimeter of a regular polygon, then, = 1 +
3 log e 2 − 3 log e 10
Area of circle > Area of regular polygon
log e 10 − log e 2
   Also, if two regular polygons have the same = 1 +
perimeter, then the regular polygon having larger − 3 (log e 10 − log e 2)
number of sides will have area greater than that of the
1 2
regular polygon having less number of sides. \ log3 (MN9)1/3 = 1 − =
3 3
\ c > s > t
1 2 1
(c) log (A2 + B2) – [log (A) + log (B)] = C
30. ⇒ ( MN 9 ) 3 = 3 3 = (9) 3
\ A = B ⇒ log (2A2) –­ 2 log A = C ⇒ MN9 = 9
⇒ log 2A2 – log A2 = C \ N9 =
9
M
2 A2
⇒ log = C ⇒ C = log 2
A2 35.
(b) log10x – log10 x = 2 logx 10

31.
(a) – x = (x + 1)2 1
⇒ log10x – log10x = 2 logx10
⇒ x2 + 3x + 1 = 0 2

−3 + 5 −3 − 5 1
\ x = , ⇒ log10 x = 2. logx10
2 2 2
⇒ log10 x = 4. logx10
−3 + 5
But only = satisfies the other condition of ⇒ log10x = logx104
2
x + 1 > 0. or, log10x = logx10000
Now putting the value of x = 10
32.
(b) log7 log5 ( x + 5 + x ) = 0
1 = 4, which is not possible
⇒ log5 ( x + 5 + x ) = 70 = 1 1
Putting the value of x =  , we get – 2 = – 2.
100
x + 5 + x = 51 = 5 Thus answer is (b).

⇒ 2 x = 0 \ x = 0 36.
(b) u = (log2 x)2 – 6(log2 x) + 12
Let, log2 x = p (1)
33.
(c) log2 [log7(x2 – x + 37)] = 1
2
⇒ u = p – 6p + 12
⇒ 2 = log7 (x2 – x + 37)
xu = 256 = (28)
⇒ 49 = x2 – x + 37
   Applying log to base 2 on both sides, we get u log2
or, x2 – x – 12 = 0 x = log2 28.
⇒ (x – 4)(x + 3) = 0. \ x = 4 ⇒ u log2x = 8 (2)

1 Dividing (2) by (1), we get


34.
(b) log3M + 3 log3N = 1 + log0.008 5
3 u = 8/p

log e 5 ⇒ 8/p = p2 – 6p + 12
⇒  log3M1/3 + log3N3 = 1 +
log e 0.008 ⇒ 8 – p3 – 6p2 + 12p = 0
log e 5
⇒  log3 (M. N9)1/3 = 1 + or, p3 – 6p2 + 12p – 8 = 0
8
log e
1000 ⇒ (p – 2)3 = 0 or, p = 2
log e 10 − log e 2 \ log2 x = 2 ⇒ 2 ⇒ x = 22 = 4
= 1+
log e 8 − log e 1000
Thus, we have exactly one solution.

Chapter_19.indd 14 6/5/2015 12:08:03 PM


Logarithms 19.15

1 1 44.
(b) 0 < a ≤ x; Min. value of loga x + logx a is 2 when we
37.
(d) log8x + log8 = put x = a
6 3
log x log y log z
 1 1  x 1 45.
(a) = = = k (say)
⇒ log8  x ×  = or, log8   = b−c c−a a−b
 6 3 6 3
⇒ log x = k (b – c), log y = k (c – a),
1/3 x x
⇒ (8) = ⇒  (23)1/3 = log z = k (a – b)
6 6
\ x = 12 ⇒ log x + log y + log z = 0 ⇒ xy z = 1.
Also, a log x + b log y + c log z = 0
38.
(b) log17275 = log19375
⇒ xa . yb . zc = 1.
log 275 log 375
⇒ = Again (b + c) log x + (c + a) log y + (a + b) log z = 0.
log17 log19
⇒ xb+c × yc+a × za+b = 1.
275 375
⇒ = \ xyz = xa × yb × zc = xb+c × yc+a × za+b = 1
17 19
Q 16.18 < 19.74 (c) xlog y–log z × ylog z – log x × zlog x–log y = k (say)
46.
\ log17275 < log19375 ⇒ (log y – log z) log x + (log z – log x) log y
+ (log x – log y) log z = log k = 0
39.
(a) yz(2 – x) = 2yz – xyz = 2 log4a 2a – log4a a
⇒ k = 1
 4a 2 
= log4a   = 1 47.
(d) 98 + x 2 − 12 x + 36 = 100
 a 
40.
(d) Each is equal to k ⇒ x 2 − 12 x + 36 = 2
x = k (l + m – 2n),
⇒ log  ⇒ x2 – 12x + 32 = 0
log y = k (m + n – 2l), ⇒ x = 8, 4
48. (a) x = loga bc
log z = k (n + l – 2m).
⇒ ax = bc
⇒ log xyz = k (0) ⇒ ax+1 = abc
⇒ xyz = e0 = 1 ⇒ x2y2z2 = 1 ⇒ a = (abc)1/x+1.
Similarly, b = (abc)1/y+1 and c = (abc)1/z+1
x+ y 1
41.
(b) l og   = [log x + log  y] 1
+
1
+
1
 5  2
\ abc = (abc) x +1 y +1 z +1

⇒ x + y = 5 xy ⇒ x2 + y2 = 23xy 1 1 1
⇒ 1 = + +
x y x +1 y +1 z +1
⇒ + = 23
y x ⇒ (x + 1) (y + 1) (z + 1) = (y + 1) (z + 1)
+ (x + 1) (z + 1) + (x + 1) (y + 1)
3x − 3 y ⇒ xyz = x + y + z + 2
42.
(c) x + y =
2 x x x
y x y
x 49.
(b) b = a ⇒ b = a , c = a z , d = a w
⇒ x = 5 y ⇒ =5
y
 x x x
  x x x
⇒ log  x – log y = log 5 loga (bcd) = log a  a y .a z .a w  = + +
y z w
1 1 21 1 1 1
43.
(a) log2 x + log 2 x + log 2 xs = = x  + + 
2 4 4  y z w
 1
 1 1 21 50.
(c) log10   = –log10 2 = – 0.3010
⇒ log 2x 1 + +  = 2
 2 4 4
⇒ log2x = 3 ⇒ x = 8 = 1 – 0.3010 – 1 = 1 .6990

Chapter_19.indd 15 6/5/2015 12:08:05 PM


19.16 I Chapter 19

(d) log2 (32x–2 + 7) = log 24 + log2 (3x–1 + 1)


51. 2
+1
⇒ 42 = 10 x (1)
[Q 2 = 2 log22 = log222]
y
⇒ 32x–2 + 7 = 4 (3x–1 + 1)  42 
  = 100
 100 
⇒ t2 + 7 = 4(t + 1), where, 3x–1 = t
⇒ (42)y = 102+2y
⇒ t2 – 4t + 3 = 0 ⇒ t = 1, 3
2
+2
When t = 1 ⇒ 3x–1 = 1 ⇒ x = 1 ⇒ 42 = 10 y (2)
x–1 1
When t = 3 ⇒ 3 =3 ⇒x=2 2 2 1 1 1
From (1) and (2), − =1⇒ − =
52.
(c) loga b = logb c = logc a = k (say) x y x y 2

⇒ b = ak, c = bk, a = ck log9 11 log3 11 log3 11 log3 11


58.
(c) – = – =0
⇒ k k k2
c = (a ) = a = (c ) = c k2 k k3 log5 13 log 5 13 2.log5 13 2.log5 13

⇒ k3 = 1 ⇒ k = 1.  \  a = b = c
log x log y log z
59.
(d) = = = k (say)
53.
(d) log10 x = 2 log10 a – 2 2 3 5
⇒ log10 x = 2 (log10 a – 1) ⇒ log x = 2k, log y = 3k, log z = 5k

a a2 ⇒ log yz = 3k + 5k = 8k; log x4 = 8k


⇒ log10 x = 2 log10   ⇒ x =
 10  100 \ log yz = log x4 ⇒ yz = x4
54.
(b) logb(c + a) + logb(c – a)
4x 3x
2 2 2 60.
(b) 4x + = + 3x. 3
= logb(c – a ) = logbb = 2 2 3
x
55.
(a) x = (875)10 = (87.5 × 10)10 3 4 4 8
⇒ 4x ×  = 3x ×  ⇒   =
2 3 3
  3 3
\ log10 x = 10(log1087.5 + 1)
x 3/2
= 10(1.9421 + 1) 4 4 3
⇒   =   ⇒x=
3 3 2
= 10(2.9421) = 29.421.
15
\ x = Antilog (29.421). 5
61.
(a) log8 p = ⇒ p = (8)5/2 = 2 2 = (25)3/2
\ Number of digits in x = 30. 2

10
log2 q = 5 ⇒ q = 25. \ p = q3/2
 432 
(b) x = (0.0432)10 = 
56. 
 10000  62.
(b) 0 < a < 1, 0 < x < 1 and x < a
10 ⇒ loga x > loga a ⇒ loga x > 1
 33.24 
=  4 
 10  6 7 8 625
  63.
(b) log5 + log5 + log5 + ... + log5
5 6 7 624
\ log10x = 10 (3log103 + 4log102 – 4)
 6 7 8 625   625 
= 10 (1.4313 + 1.2040 – 4) = log5  ...  = log5   =4
 5 6 7 624   5 
= 10 (–1.3647) = –13.647

= 14 .353  2986 
64.
(b) log10(0.02986) = log10  
 100000 
\ x = Antilog (.053)
= 3.4751 – 5 = –1.5249
\ Number of zeros between the decimal and the first
significant figure = 13. =  2.4751
a
57.
(c) (4.2)x = 100 65.
(a) 2a – 3b =
b
⇒ (42)x = 102+x ⇒ 2ab – 3b2 = a

Chapter_19.indd 16 6/5/2015 12:08:06 PM


Logarithms I 19.17

⇒ 3b2 = a(2b – 1) Similarly, yb–c = ek(b3–c3), zc–a = ek(c3–a3).


2
3b \ xa–b . yb–c . zc–a = e0 = 1
⇒ a =
2b − 1
73.
(c) 3x–2 = 5
(c) (x – y)2 = 25xy
66.  90 
⇒ 3x = 45 =  
⇒ x2 + y2 = 27xy  2 
x y ⇒ x log103 = log1090 – log102
⇒ + = 27
y x = 2 log103 + 1 – log102
log x log y log z ⇒ x(0.4771) = 1.65317
67.
(b) = = =k
3 4 5
165317 22187
⇒ log x = 3k; log y = 4k; log z = 5k. ⇒ x =
=3
47710 47710
⇒ log (zx) = log z + log x = 8k = 2 log y
74. (d) log (648)5 = 5 log (81 × 8) = 20 log3 + 15 log2
\ zx = y2
= 20(0.4771) + 15(0.30103)
68.
(a) 3 + log5 x = log5 y = 14.05745.
\ Number of digits in (648)5 is 15
⇒ log5(125x) = log5 y
log x log y log z
y 75.
(c) = = =k
⇒ x = 1 2 5
125
x = k, log y = 2k, log z = 5k.
⇒ log 
log 2 a log3 b log 4 c
69.
(d) = = =k \ log(x4 . y3 . z–2) = 4 log x + 3 log y – 2 log z = 0
3 3 4
x4 . y3 . z–2 = 1
⇒ 
⇒ a = 22k, b = 33k, c = 44k and,
a1/2 × b1/3 × c1/4 = 2k × 3k × 4k = 24 log 27 + log 1000 + log8
76.
(c)
⇒ 24k = 241 log120
⇒ k = 1. 3
(log 3 + log10 + log 4)
\ a = 4, b = 27, c = 256 2 3
= =
log 3 + log10 + log 4 2
z
70.
(c) = 1
x3 y 4 10log10 x 1 1
77.
(b) y = = = a = y–a ⇒ a = –1
x2 x y
⇒ log2 z – 3 log2 x – 4 log2 y = 0
78.
(b) x = log4/3 (1/2) = –log4/3 2 < 0
3.3 4
⇒ log2 z –
⋅ log 2 z − 4 ⋅ ⋅ log 2 z = 0
5k 5k and, y = log1/2 (1/3) = log2 3 > 0
9 16 ⇒ y > x

1– − = 0
5k 5k u
79. (b) x = 256
⇒ 5k – 25 = 0
Take log on base 2, u log2x = 8
⇒ k = 5
Let, log2x = p
3(1 + log10 7) 3(1 + log10 7) Then,
71.
(a) =
7 1 7
2 + log + log 2 + log   8
2 5  10  = p 2 − 6 p + 12
p
3(1 + log10 7)
= =3 p 3 − 6 p 2 + 12 p − 8 = 0
1 + log10 7
p 2 ( p − 2) − 4 p ( p − 2) + 4( p − 2)
2 2
72.
(c) Each ratio = k ⇒ logx = k(a + ab + b ) ( p 2 − 4 p + 4)( p − 2) = 0
⇒ (a – b)log x = k(a3 – b3) ( p − 2)3 = 0

⇒ log xa–b = k(a3 – b3) ⇒ xa–b = ek(a3–b3) p=2

Chapter_19.indd 17 6/5/2015 12:16:55 PM


19.18 Chapter 19

80. (d) We have 81. (a) We have,


b
log x ( a − b ) − log x ( a + b ) = log x log 2 [log 7 ( x 2 − x + 37)] = 1
a
a−b b ⇒ log 7 ( x 2 − x + 37) = (2)1
⇒ log x = log x
a+b a ⇒ ( x 2 − x + 37) = (7) 2
⇒ a (a − b) = b (a + b)
⇒ x 2 − x + 37 − 49 = 0
⇒ a 2 − ab = ab + b 2
⇒ x 2 − x − 12 = 0
⇒ a 2 − b 2 = 2ab
⇒ x = 4.
⇒ a 2 − 2ab − b 2 = 0
2
a a 82. (c) Let log6 216 6 = x, then 216 6 = (6) x
⇒   − 2  −1 = 0
b
  b
1 7
a 7
This is quadratic in . The product of the root is −1, ⇒ (6)3 × (6) 2 = (6) x ⇒ (6) 2 = (6) x ⇒ x=
b 2
a 1
i.e., if is a root, then − will also be a root.
b a 83. (b) log 7 log 5 ( x + 5 + x ) = 0
b ⇒ log 5 ( x + 5 + x ) = (7)0 = 1
Therefore,
⇒ ( x + 5 + x ) = ( 5) = 5
1
2
2 2
  2
 a   b   a   −1  2 x =0
  +  =  + a 
b a b  
  ∴x = 0
 b 
2
   100
1 100
 a  −1  
=  +    + 2 = 22 + 2 = 6
84. (c) ∑ log 100! = ∑ log
i=2 i=2
i
100 !
i
 b  a  
 
  b  = log100 ! 2 + log100 ! 3 + log100 ! 4 + ...., + log100 ! 100


( As the sum of the roots is 2 ).
= log100 ! 100! = 1

Chapter_19.indd 18 6/5/2015 12:08:13 PM


Stocks, Shares
CHAPTER

and Debentures 20
Introduction Sl. Par value Number of Rate of dividend
To start a big business or an industry, a large sum of money No. of a share Common Shares declared on a
is required. It may not be possible for one or two persons to Common Share
arrange for the requisite finance and expertise required for (i) `10 500 10% per annum
the project. So a number of individuals join hands to form a (ii) `10 800 5% semi-annually
company called a Joint Stock Company. It is a registered (iii) `100 1500 5% quarterly
body under the Companies Act. The persons who join
(iv) `10 2500 2% per month
together to form the company are called its Promoters. The
total amount of money required by the company is called Solution: (i) Annual dividend on one share
the Capital.
 10 
The promoters of the company issue a circular giving = 10% of `10 = `  × 10  = `1
the details of the project, its benefits and drawbacks; and  100 
invite the public to come forward and subscribe towards the Annual dividend on 500 shares
capital of the company. The company divides the required = `(500 × 1) = `500.
capital into small units of equal amount. Each unit is called
a share. Each person who purchases one or more shares (ii) Annual dividend on one share
of the company is called a shareholder of the company. = `(2 × 5)% of `10
The company issues a share certificate to each of its  10 
shareholders stating the number of shares alloted to the = ` × 10  = `1
 100 
person and the value of each share. The value of a share as
stated on the share certificate is called the nominal value \ Annual dividend on 800 shares
(face value or par value) of the share. = `(800 × 1) = `800.
When a company earns a profit during a financial year, (iii) Annual dividend on one share
a part of it is used in paying for working expenses, taxes,
interest on loans and keeping some part of it as reserve fund = (4 × 5)% of `100
for future expansion of the project, the remaining profit is  20 
= ` × 100  = `20
distributed among the shareholders. The distributed profit is  100 
called the dividend.
\ Annual dividend on 1500 shares
Dividends are declared annually, semi-annually,
quarterly as per regulations of the company. The dividend = `(1500 × 20) = `30000.
on a share is expressed as certain percentage of its face (iv) Annual dividend on one share
value which is printed on the share certificate. Sometimes = (12 × 2)% of `10
it is also expressed as a specified amount per share. For
example, we may say that dividend on a share is 12% of its  24 
= ` × 10  = `2.40
face value or the dividend is `2 per share.  100 
Illustration 1  Find the annual dividend paid in each of the \ Annual dividend on 2500 shares
following cases: = `(2500 × 2.40) = `6000.

Chapter_20.indd 1 6/5/2015 12:35:30 PM


20.2 Chapter 20

TYPES OF SHARES Illustration 2  Find the cost of purchasing 150 shares of a


company, each of par value `10, quoted at `16 each in the
The shares are generally of two types:
market from the original shareholder. Also, find the gain
(i) Preferred shares to the new shareholder if he sells each share at a premium
(ii) Common (ordinary) shares of `10
(i) Preferred shares   These shares get preference in Solution: Market value of share = `16
terms of payment of dividend and return of capital \ Market value of 150 shares
over ordinary shares. The rate of dividend for these = `(150 × 16) = `2400
shares is decided when they are issued and dividend Thus, the new shareholder spent `2400 for buying 150
to preferred shareholders is paid before any dividend shares. The new shareholder sold the shares at a premium
is paid to common shareholders. of `10.
(ii) Ordinary shares  Ordinary shareholders are paid \ Now, market value of a share
dividend only when profits are left after preferred = `(10 + 10) = `20
shareholders have been paid dividend at specified
The selling price of 150 shares at the new market value
rate. The rate of dividend on these shares is also not
fixed and depends upon the amount of available profit. = `(150 × 20) = `3000
\  Gain of the new shareholder in the transaction
FACE VALUE AND MARKET VALUE OF A SHARE = `(3000 – 2400) = `600.
The price at which the shares are initially issued by the Illustration 3  Raja buys 200 shares, each of par value `10
company to its shareholders is called the face value of a of a company which pays annual dividend of 15% at such
share (This is also called nominal or par value of a share). a price that he gets 12% on his investment. Find the market
In fact, this is that value of a share which is mentioned in the value of a share.
share certificate issued by the company to its shareholders. Solution: Par value of 200 shares
As other things, shares are also sold in (or purchased from)
= `(200 × 10) = `2000
the market. The value of a share quoted in the market is called
the market value of the share. The market value of a share keeps Dividend received by Raja
on changing according to its demand and supply changes.  2000 × 15 
= `  = `300
If the market value of a share is equal to the par value  100 
of the share, the share is said to be at par. If the market
Let the market value of 200 shares be `x. We have to
value of a share is more than its face (or par) value, the
find x such that 12% of x = 300,
share is said to be at premium. On other hand, if the market
value of a share is less than its face value, the share is said 12 100 × 300
i.e., × x = 300   \  x = = 2500
to be at discount (or below par). 100 12
For example If the market value of a `100 share is `130, it i.e., Market value of 200 shares = `2500
is said to be at 30% premium. Hence, the market value of one share = `12.50.
If the market value of a `100 share is `90, it is said to
be at 10% discount. If `100 share is quoted at 45 premium Stocks and Brokerage
then its market value is Stock
`(100 + 45) = `145
In the previous section, we have learnt about shares which
Every company declares dividend on the face value of can be sold and purchased by the public. The nominal value
its shares irrespective of the market value of the share. or face value of shares is fixed (usually `10 or `100) but
Notes: their market value varies.
Sometimes, joint stock companies or the government
The statement “32%, `100 shares at `125” means:
also raises loans from the market by issuing bonds or
(i) Face value of each share is `100 promisory notes. They promise to pay a fixed amount (called
(ii) Dividend on each share is 32% of the face value redemption value) on a future date and interest payments at
(iii) Market value of each share is `125 fixed periods until that time. The money paid to company or
(iv) An investment of `125 earns an annual income of government for buying such bonds is called stock.
`32.

Chapter_20.indd 2 6/5/2015 12:35:30 PM


Stocks, Shares and Debentures 20.3

The stocks are usually known by their rates of dividend. Illustration 5   Find the income on 10% stock of `25000
Thus by 9% stock we mean that the dividend on a `100 purchased at `120
stock is `9. Solution:  Face value of stock = `25000
If the market value of `100 stock, which yields a dividend
Income on `100 stock = `10
of `5, is `115, the stock is called 5% stock at 115. Similarly,
10% stock at 120 means that a stock of face value `100 gives  10 
Income on `1 stock = `  
a dividend of `10 and is available in the market of 120.  100 
Income on `25000 stock
Note:
 25000 × 10 
= ` 
There can be stocks in units different from `100, say `500,  100 
`1000, etc., but the phrase “8% stock at 90” can be used only = `2500
in case of that stock whose face value is `100. Dividend on
a stock is fixed (declared at the time of issue) whereas for a COMPUTATION OF INVESTMENT OR MARKET
share it varies with time. Usually, the date of maturity of the VALUE OF A STOCK
stock is fixed. In case the holder of the stock requires money
before the due date, he can sell his stock to some other person, If the face value of a stock is given, the market value of the
whereby his claim of interest is transferred to that person. stock can be found on the basis of market value of each unit
of stock.
Brokerage
Illustration 6   Find the investment required to purchase
The sale and purchase of stock is generally executed through `75000 of 10% stock at 95.
a stockbroker, who charges some money, called Brokerage
Solution: Market value of `100 stock = `95
from both the seller and purchaser. The brokerage is charged
either as some fixed amount on each unit of stock or as some \  Market value of `75000 stock
percentage of the market value of unit of stock.  95 
= ` × 75000  = `71250
Thus, the brokerage of `x means that x rupees are to  100 
be added or subtracted from the market value of the stock. \  An investment of `71250 is required to purchase
Similarly, brokerage 2% means that the brokerage equal to `75000 of 10% stock at 95
2% of the market value of a unit of stock and be added to (or
subtracted from) the market value of a unit of stock. Illustration 7   Find the investment required to get an
1
Notes: income of `4200 from 10 % stock at 80 (Brokerage: 2%).
2
(i) The brokerage is added to the market value when the  2 
Solution:  Brokerage = 2% of `80 = `  × 80  = `1.60
stock is purchased.  100 
(ii) The brokerage is subtracted from the market value \ Investment needed to buy `100 stock
when the stock is sold. 1
= `81.60 on which the income is `10
2
CALCULATION OF INCOME ON A STOCK
1
When the face value of the total stock is given, the income For income of `10 , the investment = `81.60
2
can be calculated on the assumption that the face value of For income of `4200, the investment
each unit of stock is `100. On the contrary, if the market
 2 
value of the total investment is given, the income can be = `  81.60 × × 4200  = `32640
calculated on the basis of the market value of a unit of stock.  21 

Illustration 4  Find the income from `2875 of 4% stock COMPUTATION OF GAIN OR LOSS IN THE SALE
Solution: By 4% stock, we mean a stock of `100 will fetch AND PURCHASE OF A STOCK
a dividend of `4 p.a. When the market is favourable to stock holders, i.e., they
Hence, the income from `2875 of 4% stock are likely to get better proceeds for their stock, they sell the
2875 × 4 stock and may reinvest the money so obtained in another
= = `115 stock which may give them more income.
100

Chapter_20.indd 3 6/5/2015 12:35:30 PM


20.4 Chapter 20

Illustration 8  Ram bought `12000 of 8% stock at 92 and period of time and at a fixed rate of interest by dividing
sold it when the price rose to 98. Find his total gain and the amount required into small parts. These small parts are
gain per cent called debentures.
Solution:  Investment made by Ram in buying `12000 of The debenture holders are creditors of the company
8% stock at 92 and do not have any right on the profits declared by the
 92  company. However, interest at fixed rate and fixed time
= ` 12000 × = `11040 is payable to debenture holders, irrespective of the fact
 100 
whether the company is running in profits or losses.
When the price rose to `98, Ram sold the stock, thus
Like shares, debentures can also be sold in or purchased
money realised from selling the stock
from the market. The terms used in case of shares, are also
 98  used with the same meaning in case of debentures. Thus
= ` 12000 × = `11760
 100  we use the terms ‘debentures at premium’, ‘debentures at
\  Gain realised in the transaction discount’ etc. Furthermore, the rules for calculating the
brokerage on debenture are also the same as those in case
= `(11760 – 11040) = `720
of shares.
(720 × 100) 12
\  Gain per cent = = 6 %
11040 23 DIFFERENCE BETWEEN SHARES
AND DEBENTURES
Change IN INCOME ON SALE OR REINVESTMENT Shares Debentures
(a) Share money forms a (a) Debentures are a mere
A person having one type of stock may sell it to buy another part of the capital of the debt.
which gives higher income. In such problems, the income in company. (b) Debenture holders are
two cases is calculated and change is found out. (b) Shareholders have right creditors of the company
on the profit declared by and do not have any right
Illustration 9  Ram invests `46500 in 6% stock at 93 and the company. on the profit declared by
sells the stock when its price rose to `95. He invests the the company.
sale proceeds in 9% stock at 95. Find the change in Ram’s (c) Shareholders may re- (c) Debenture holders re-
income ceive different dividend ceive interest at a fixed
 6  according as profit is rate.
Solution:  Income from first stock = `  × 46500 
 93  more or less.
           = `3000 Illustration 10  Find the income percent of a buyer on 8%
We have to find the amount realized on selling this debentures of face value `120 and available in the market
stock. at `180
Amount realized on selling `93 stock = `95 Solution: The market value of a debenture is `180
\ Amount realized on selling `46500 stock \ Income on `180 is `8
95
=` × 4650 = `47500  8  1
93 \ Income on `120 is `  × 120  = ` 5
 180  3
This amount is invested in 9% stock at 95
1
\ Income from 2nd stock \ Per cent income on the debenture is 5 %
3
 9 
= `  × 47500  = `4500 Illustration 11  Ram has 500 shares of par value `10 each
 95 
of a company and 500 debentures of par value `100 each.
Hence, increase in income The company pays a dividend of 8% on the shares and pays
= `(4500 – 3000) = `1500 an interest of 10% on its debentures. Find the total annual
income of Ram and the rate of return on her investments
DEBENTURES Solution: Annual dividend on 500 shares
Sometimes a running joint stock company may require more (500 × 10 × 8)
=`
capital for its further expansion. The company borrows the 100
required sum of money from the general public for a fixed = `400

Chapter_20.indd 4 6/5/2015 12:35:31 PM


Stocks, Shares and Debentures 20.5

Annual interest on 500 debentures Total investment of Ram


(500 × 100 × 10) = `(500 × 10 + 500 × 100) = `55000.
=`
100 Rate of return on Ram’s investment
= `5000  5400 
=  × 100  %
\  Total annual income of Ram  55000 
= `(5000 + 400) 108 9
= `5400 = %  or,   9 %
11 11

Practice Exercises

Difficulty Level-1
(Based on Memory)

1.
A man who owned 25% of the equity capital of a certain Arun drives twice as many miles as Balu. What is the ratio
company sold one-third of his holding last year and five- of the number of hours that Arun spends in driving to the
twelfths of the remaining this year. What part of the meeting to the number of the hours that Balu spends in
business does he now own ? driving to the meeting?
(a) 1/5 (b) 5/144 (a) 3:2 (b) 8:3
(c) 7/72 (d) 65/72 (c) 2:3 (d) 4:3
[Based on MAT, 2004] 7.
A man wants to secure an annual income of `1,500 by
2.
Arun has 800 shares of par value of `50 each, and 600 investing in 15% debentures of face value `100 each and
debentures of par value `100 each of a company. The available for `104 each. If the brokerage is 1%, then the
company pays an annual dividend of 6% the shares and sum of money he should invest is:
interest of 12% on the debentures. The rate of return of his (a) `19,642 (b) `10,784
investment is: (c) `10,504 (d) `15,000
(a) 8% (b) 9.6% 8.
A person invests `5508 in ‘4% stock at 102’. He afterwards
(c) 10.6% (d) 8.6% sells out at 105 and reinvest in ‘5% stock at 126’. What is
[Based on MAT, 2008] the change in his income?
3.
The capital of a company is made up of 50,000 preferred (a) `20 (b) `7
shares with dividend of 20% and 20,000 common shares, (c) `10 (d) `9
[Based on IIFT, 2005]
the par value of each type of share being `10. The company
had a total profit of `1,80,000 out of which `30,000 was 9.
Shyam, Gopal and Madhur are three partners in a business.
kept in reserve fund and the remaining distributed to Their capitals are respectively `4000, `8000 and `6000.
shareholders. Find the dividend per cent to the common Shyam gets 20% of total profit for managing the business.
share-holders. The remaining profit is divided among the three in the
(a) 20% (b) 24% ratio of their capitals. At the end of the year, the profit of
(c) 25% (d) 30% Shyam is `2200 less than the sum of the profit of Gopal
and Madhur. How much profit, Madhur will get?
4.
At what price should I buy a share, the value of which (a) `1600 (b) `2400
is `100, paying a dividend of 8%, so that my yield is (c) `3000 (d) `5000
11%? [Based on IIFT, 2010]
(a) `70 (b) `72.72
10.
A sum of `2236 is divided among A, B and C such that A
(c) `75 (d) `84
receives 25% more than C and C receives 25% less than
5. A person had deposited `13,200 in a bank which pays B. What is A’s share in the amount?
14% interest. He withdraws the money and invests in (a) `460 (b) `890
`100 stock at `110 which pays a dividend of 15% How (c) `780 (d) `1280
much does he gain or lose? [Based on Indian Overseas Bank PO, 2009]
(a) loses `48 (b) gains `48 11.
A sum of money is divided among A, B, C and D in the
(c) loses `132 (d) gains `132 ratio of 2:3:7:11, respectively. If the share of C is `2755
6. Arun and Balu drive separately for a meeting. Arun’s more than the share of A, then what is the total amount of
average driving speed is one-third greater than Balu’s and money of B and D together?

Chapter_20.indd 5 6/5/2015 12:35:31 PM


20.6 I Chapter 20

(a) `4408 (b) `5510 19.


Jatin invested `27,260 in buying `100 shares of a company
(c) `6612 (d) `7714 at `116 each. If the company paid 16% dividend at the end
of the year, find his income from the dividend.
12.
Mrudul invested an amount of `29500 in order to start a
(a) `3,560 (b) `2,760
business. Shalaka joined her 4 months later by investing
an amount of `33500. If the business earned a profit of (c) `3,760 (d) `3,660
`120575 at the end of two years, what was Mrudul’s share 20.
A company issued 50,000 shares of par value `10 each.
of the profit? If the total dividend declared by the company is `62,500,
(a) `60725 (b) `61950 find the rate of dividend paid by the company.
(c) `59250 (d) `58625
1 1
(a) 8 % (b) 12 %
13.
A man had two sons. To the elder he left five-elevenths 2 2
of his property, to the younger five-elevenths of the
remainder, the rest of the window. Find the share of the 3
(c) 12% (d) 13 %
sons if the widow gets `3600. 4
(a) `1200, `1000 1
21.
A company declared a semi-annual dividend of 7 %
(b) `6600, `2000 2
(c) `7500, `1000 Find the annual dividend of Chetan, owning 1,250 shares
(d) None of these of the company having a par value of `10 each.
[Based on NMAT, 2006] (a) `1,875 (b) `1,757
14.
Rahul spends 50% of his monthly income on household (c) `1,680 (d) `1,575
items, 20% of his monthly income on buying clothes, 5%
of his monthly income on medicines and the remaining 22.
A medicine company issued 1,25,000 shares of par value
amount of `11250 he saves. What is Rahul’s monthly `20 each. If the total dividend declared by the company is
income? `3,75,000, find the rate of dividend paid by the company.
(a) `38200 (b) `34000 (a) 15 % (b) 13%
(c) `41600 (d) `45000 (c) 10% (d) 14%

15.
Sonu invested 10% more than Mona. Mona invested 10% 23.
Seema had 50 preferred shares and 400 common shares of
less than Raghu. If the total sum of their investment is par value `100 each. If the dividend declared on preferred
`5780, how much amount did Raghu invest? shares is 10% per annum and a semi-annual dividend
of 7.5% is on common shares find the annual dividend
(a) `2010 (b) `2000
received by Seema.
(c) `2100 (d) `2210
(a) `7,500 (b) `6,500
16.
In a business partnership among A, B, C and D, the profit (c) `8,500 (d) `5,500
is shared as follows
24.
Find the annual dividend received by Sunil for his 200
A 's share B 's share's share 1
= = preferred shares and 1,000 common shares, both of par
B 's share C 's share's share 3 value `100 each if the dividend declared on a preferred
If the total profit is `4,00,000 the share of C is
1
(a) `1,12,500 share is 10% per annum and an annual dividend of 12
2
(b) `1,37,500 % on the common shares.
(c) `90,000 (a) `4,500 (b) `550
(d) `2,70,000 (c) `4,000 (d) `3,500
17.
A company declared an annual dividend of 10% Find
25.
A company issued 50,000 shares of par value `100 each. If
the annual dividend of Ram owning 1,500 shares of the
the total dividend declared by the company is `1,25,000,
company of par value `10 each.
out of which `50,000 have been kept in reserve fund and
(a) `1,400 (b) `1,500 the remaining is distributed as dividend, find the rate of
(c) `1,700 (d) `1,600 dividend paid by the company.
18.
A company declared an annual dividend of 10% Find the 3 1
annual dividend received by Anu owning 4,000 shares of (a) 2 % (b) 1 %
4 2
the company having a par value of `100 each.
(a) `45,000 (b) `40,000 1
(c) 1 % (d) 2%
(c) `50,000 (d) `60,000 4

M20_KHAT6981_C20.indd 6 6/16/2015 5:41:10 PM


Stocks, Shares and Debentures 20.7

26.
Find the annual dividend received by Nishita from 1,200 of 15% If the market value of a share of Company Y is
preferred shares and 3,000 common shares, both of par `40, find the number of shares of Company Y purchased
value `50 each, if the dividend paid on preferred shares by the man.
1 (a) 3,850 (b) 3,750
is 10% and semi-annual dividend of 3 % is declared on
2 (c) 3,700 (d) 3,800
common shares.
(a) `18,500 (b) `16,500 33.
The shares of a company of par value `10 each, are
(c) `14,500 (d) `14,800 available at 20% premium. Find the amount paid by the
buyer who wants to buy 2,500 shares. What would be the
27.
12500 shares, of par value `20 each, are purchased from gain of the buyer if he sells those shares at the rate of `20
Ram by Mohan at a price of `25 each. Find the amount per share?
required to purchase the shares. If Mohan further sells
(a) `25,000 (b) `30,000
the shares at a premium of `11 each, find his gain in the
transaction. (c) `20,000 (d) `22,000
(a) `75,000 (b) `85,000 34.
Find the income on 12% stock of `60,000 purchased at
(c) `70,000 (d) `65,000 `110.
(a) `7,200 (b) `7,500
28.
Mac buys 200 shares of par value `10 each, of a company,
which pays an annual dividend of 8% at such a price that (c) `7,400 (d) `8,200
he gets 10% on his investment. Find the market value of
one share. 1
35.
Find the income on 7 % stock of `20,000 purchased at
2
(a) `8 (b) `10
`120.
(c) `6 (d) `12
(a) `1,550 (b) `1,450
29.
Shyam purchased 12,000 shares of a company, of par (c) `1,500 (d) `1,600
value `10 each, paying an annual dividend of 15% at
such a price, that he gets 10% on his investment. Find the 36.
Find the income by investing `81,000 in 9% stock at 135.
market value of a share. (a) `5,500 (b) `6,400
(a) `25 (b) `15 (c) `5,400 (d) `6,000
(c) `20 (d) `14
1
37.
Find the income obtained by investing `90,000 in 7 %
30.
The capital of a company is made up of 50,000 preferred 2
shares with dividend of 20% and 20,000 common shares, 1
the par value of each type of share being `10. The company stock at 112 .
2
had a total profit of `1,80,000 out of which `30,000 were
kept in reserve fund and the remaining distributed to (a) `6,000 (b) `6,500
shareholders. Find the dividend per cent to the common (c) `7,500 (d) `7,000
shareholders.
(a) 24% (b) 20% 1
38.
A person buys 9 % stock of `72,000 at 144. Find his
(c) 25% (d) 30% 2
annual income.
31.
A company has issued 10,000 preferred shares and 50,000 (a) `6,640 (b) `6,840
common shares both of par value `100 each. The dividend
on a preferred share and a common share is 12% and (c) `6,900 (d) `7,240
17.6%, respectively. The company had a total profit of
1
15 lakhs rupees out of which some amount was kept in 39.
Mr Lal invested `92,000 in 9 % stock at 91 (Brokerage:
reserve fund and the remaining distributed as dividend. 2
Find the amount kept in reserve fund. Re 1). Find the annual income of Mr Lal from this
investment.
(a) `5 lakhs (b) `6 lakhs
(a) `9,000 (b) `9,500
(c) `6.5 lakhs (d) `5.5 lakhs
(c) `10,500 (d) `8,000
32.
A man sells 5,000 common shares of a Company X (each
of par value `10), which pays a dividend of 20%, at `30 1 1
40.
Raja invested `99,000 in 7 % stocks at 81 (Broker-
per share. He invests the sale proceeds in ordinary shares 2 2
of Company Y (each of par value `25) that pays a dividend age: `1). Find Raja’s annual income from his investment.

Chapter_20.indd 7 6/5/2015 12:35:32 PM


20.8 Chapter 20

(a) `9,500 (b) `10,000 49.


Sushma invested `2,45,000 in 7% stock at 98 and sold the
(c) `10,500 (d) `9,000 stock when its price rose to `100. She invested the sale
proceeds in 9% stock at 125. Find the change in income
1
41.
Ram invested `88,008 in 9 % stock at 112 (Brokerage: of Sushma.
2
(a) `600 (b) `400
`2). Find annual income of Ram from this investment.
(c) `500 (d) `650
(a) `6,334 (b) `6,874
(c) `7,334 (d) `6,534 50.
Anu invested `32,400 in 8% stock at 90. She sold out
`18,000 stock when the price rose to `95 and the remaining
42.
Find the investment required to purchase `1,25,000 of 8% stock at `98. She invested the total sale proceeds in 10%
stock at 92. 1
stock at 96 . Find the change in income of Anu.
(a) `1,15,000 (b) `1,20,000 2
(c) `1,05,000 (d) `1,25,000 (a) `750 (b) `720
(c) `760 (d) `740
43.
What investment will be required to purchase `90,000 of
8% stock at 110? 51.
A man invested `50,490 in 5% stock at 99 and sold it
(a) `88,000 (b) `99,000 when the price rose to `102. He invested the sale proceeds
(c) `88,500 (d) `9,950 in 8% stock at 96. Find the change in man’s income
(Brokerage: `3).
44.
Find the investment required to get an income of `1,938
(a) `1,485 (b) `1,585
1
from 9 % stock at 90 (Brokerage 1%). (c) `1,385 (d) `1,685
2
(a) `19,642.60 (b) `17,543.00 52.
A man invested `2,60,000 in 5% stock at 104. He sold the
(c) `18,543.60 (d) `18,600.60 stock when the price rose to `125 and invested the sale
proceeds in 6% stock. By doing this his income increased
45.
A man bought `20,000 of 5% stock at 90 and sold it when by `2,500. At what price did he purchase the second
3 stock?
its price rose to `93 . Find his gain per cent.
4
(a) `225 (b) `175
1 1
(a) 5 % (b) 4 % (c) `125 (d) `150
6 6
53.
Find the income per cent of a buyer on 5% debentures of
5 5
(c) 5 % (d) 4 % face value `95 and available in the market for `125.
6 6
(a) 4.8% (b) 5.8%
1 (c) 3.8% (d) 2.8%
46.
Meena bought `36,000 of 7 % stock at 92 and sold it
2
54.
Find the income per cent on 10% debentures of par value
3 `120 available in the market for `150.
when its price rose to `93 . Find her gain per cent.
4 (a) 9% (b) 8%
(a) 1.9% (b) 2.9% (c) 7% (d) 6%
(c) 2.3% (d) 1.4%
55.
Brij has 800 shares of par value `50 each and 600
47.
A man invested `27,600 in 4% stock at 92. He sold debentures of par value `100 each of a company. The
`20,000 stock when the price rose to `96, and sold the company pays an annual dividend of 6% on the shares and
remaining stock when the market value fell to `90. How interest of 12% on the debentures. Find the total annual
much did he gain or loss in the transaction? income of Brij and rate of return on his investment.
(a) Gain = `600 (b) Loss = `600 (a) `9,600, 9.6% (b) `8,000, 8%
(c) Loss = `650 (d) Gain = `650 (c) `10,600, 10.6% (d) `9,000, 8.6%
48.
A person invested `28,500 in 5% stock at 95. He sold
`15,000 stock when the price rose to `98 and sold 56.
A man bought 20 shares of `50 at 5 discount, the rate of
the remaining stock when the market value of the 3
discount being 4 % The rate of interest obtained is:
stock fell to `90. How much did he gain or loss in the 4
transaction? 3 1
(a) 4 % (b) 3 %
(a) Gain = `300 (b) Loss = `300 4 4
(c) Gain = `400 (d) Loss = `400 (c) 5.28% (d) 4.95%

Chapter_20.indd 8 6/5/2015 12:35:32 PM


Stocks, Shares and Debentures 20.9

57.
I sold out all the 100 shares of company A at `95 per share, 58.
Ram invests in 5% and 8% stocks buying them at x%
which is giving 10% dividend and then purchase shares premium and at a discount of x%, respectively. The total
of company B at a price of `114 per share. Company B is amount of premium paid is 70% of the total discount
giving a dividend of 15% In these transactions, how much received. The dividend from the 5% stock forms x% of
I gained/loss? (Assume face value of share of both the dividend from the 8% stock. Find x.
companies be `100).
(a) 34.5% (b) 35.37%
(a) `10 loss (b) `250 profit
(c) 43.75% (d) 56.8%
(c) `150 loss (d) `175 profit

Answer Keys
Difficulty Level-1

1. (c) 2. (b) 3. (c) 4. (b) 5. (a) 6. (b) 7. (c) 8. (d ) 9. (b) 10. (c) 11. (d ) 12. (b) 13. (d )
14. (d ) 15. (b) 16. (c) 17. (b) 18. (b) 19. (c) 20. (b) 21. (a) 22. (a) 23. (b) 24. (a) 25. (b) 26. (b)
27. (a) 28. (a) 29. (b) 30. (c) 31. (a) 32. (b) 33. (c) 34. (a) 35. (c) 36. (c) 37. (a) 38. (b) 39. (b)
40. (d ) 41. (c) 42. (a) 43. (b) 44. (c) 45. (b) 46. (a) 47. (a) 48. (b) 49. (c) 50. (b) 51. (b) 52. (c)
53. (c) 54. (b) 55. (a) 56. (c) 57. (b) 58. (c)

Explanatory Answers

Difficulty Level-1

1. (c) Let, the total capital of the company = x 4. (b) Given 11% of x = 8
\ The capital that the man held last year 800
⇒ x = = `72.72.
11
1 1 1 x x x
= x − of x = − = 5. (a) Interest earned on `13,200 at the rate of 14%
4 3 4 4 12 6
= `1,848
The capital that the man holds this year 13200
\ Number of shares purchased = = 120
x 5 x x 5x 7x 110
= − of = − = . Dividend earned by him on 120 shares at the rate of
6 12 6 6 72 72
`15 per share = `1,800
2.
(b) Total investment = 800 × 50 + 600 × 100 \ Net loss = `48.
= 40000 + 60000 = 100000 6.
(b) Arun’s speed = x miles/h, say
Annual return = 6% of 40000 + 12% of 60000 4x
\ Balu’s speed = miles/h
6 × 40000 12 × 60000 3
= + Suppose Arun drives 2x miles and Balu drives x miles
100 100
to the meeting.
= 2400 + 7200 = 9600
\ Arun drives 2x miles in 2 hours and Balu drives x
9600
= Rate of return = × 100% = 9.6% 3
100000 miles in hour.
4
3.
(c) Dividend for preferred share 3
⇒ Required ratio = 2 : = 8:3.
20 4
= 50000 × 10 × = `1,00,000 7. (c)
100
Rest dividend = 180000 – 100000 – 30000 8.
(d) Let, the CP of share be `100.
= `50,000 5508
Number of shares =
% of dividend for common share 102
50000 × 100 = 54 and value of share
= = 25%
20000 × 10 = 5400

Chapter_20.indd 9 6/5/2015 12:35:33 PM


20.10 Chapter 20

\ SP = 54 × 105 = 5670 708


Share of Mrudul = × 120575 = 61950.
5670 708 + 670
Now, share = = 45 and
126
value of share = 4500 5
13.
(d) First son gets =
Now, change in income 11
= 5400 × 4 % – 4500 × 5 % 5 6 30
Second son gets = × =
= 216 – 225 = – 9 = `9. 11 11 121
9.
(b) The ratio of the profits of Shyam, Gopal and Mathur is 5 30  36
2:4:3. Let, the annual profit be x. Widow gets = 1 −  +  = = 3600
 11 121  121
   Then, Shyam will get (0.2x) of this and the
remaining (0.8x) will be distributed in the ratio of So, total = 12100
their capitals. Hence, sons get `5500 and `3000, respectively.
7 2 14.
(d) Suppose monthly income is x
Given, (0.8 x)  (0.8 x)  0.2 x = 2200
9 9
(100 − 75)
So,  x = 9000 x× = 11250
100
0.8  9000
Thus, Mathur’s share = = 2400. 25
3 ⇒ x× = 11250
100
10.
(c) Suppose B got `x.
1
25 ⇒ x× = 11250
Amount to C = x – x × 4
100
⇒ x = 11250 × 4
100 x − 25 x x = `45000
=
100 Shortcut
75 x 3x 100
= =` × 11250 = `45000.
100 4 (100 − 50 − 20 − 5)
3 x 125 15 x 15.
(b) Suppose amount invested by Raghu = `x
So, the amount to A = × =
4 100 16 Amount invested by Mona
15 x 3x 9
:x: A:B:C = = x = 0.9x
16 4 10
= 15x:16x:12x Amount invested by Sonu
Sum of the ratio = 15x + 16x + 12x = 43x 9 110
2236 × 15 x = x× = 0.99x
\ Share of A = = `780. 10 100
43 x x + 0.9x + 0.99x = 5780
11.
(d) Suppose the amount of A, B, C and D are 2x, 3x, 7x, 2.89x = 5780
11x. 5780
x = = `2000.
Q 7x – 2x = 2755 2.89
\ 5x = 2755 1
16.
(c) A’s share = of B’s share
2755 3
x = = 551
5 1
B’s share = of C’s share
\ Total amount of B and D 3
= (3 + 11)x 1
C’s share = of D’s share
= 14 × 551 [x = 551] 3
= 7714. 1 1 
Q C’s share =  + + 1 + 3 
9 3 
12.
(b) Monthly investment by Mrudul
= `4,00,000
= 29500 × 24 = 708000
and, by Shalaka = 33500 × 20 = 670000  1 + 3 + 9 + 27 
C’s share = 
⇒ 
Ratio = 708000:670000  9 
= 708:670 = `4,00,000

Chapter_20.indd 10 6/5/2015 12:35:33 PM


Stocks, Shares and Debentures 20.11

9  15 
\ C’s share = × 4,00,000 =  × 10  = `1.50
40  100 
= `90,000. \ Annual dividend on 1,250 shares
17.
(b) Annual dividend on one share = `(1250 × 1.50) = `1,875.
= 10% of ` 10 22.
(a) Number of shares = 1,25,000
 10  Par value of a share = `20
= ` 10 ×  =`1
 100  \ Total par value of 1,25,000 shares
Annual dividend of Ram owning 1,500 shares = `(1250000 × 20) = `25,00,000
= (1500 ×1) = `1,500. Total dividend = `3,75,000
   Alternatively, we could have found the total par \ Rate of dividend paid by the company
value of 1,500 shares first and then find dividend at
10% of it as shown below:  375000 
=  × 100  % = 15%
 2500000 
Total par value of 1,500 shares
= `(1500 × 10) = `1,5000 23.
(b) Dividend on 50 preferred shares
\ Total annual dividend of Ram  10 
• = `  50 × 100 ×  = `500
 100 
 10 
= 15000 ×  = `1,500.
 100  Dividend on 400 common shares

18.
(b) Annual dividend on one share = 10% of ` 100  100 15 
= `  400 × × × 2  = `6,000
 100 2 
 10 
= `  × 100  = `10
 100  \ Total dividend received by Seema
\ Annual dividend on 4,000 shares = `(500 + 6000) = `6,500.
= `(4000 × 10) = `40,000. 24.
(a) Dividend on 200 preferred shares
19.
(c) Number of shares purchased by Jatin = 10% of `(200 × 100)
27260  10 
= = 235 = `  × 20000  = `2,000
116  100 
Face value of 235 shares Dividend on 1,000 common shares
= `(235 × 100) = `23,500 1
= 12 % of `(1000 × 100)
Annual income from 235 shares 2
= 16% of `23,500  25 / 2 
= `  × 100000 
 16   100 
= `  × 23500  = `3,760.
 100 
 25 
= `  × 1000  = `12,500
20.
(b) Number of shares = 50,000  2 
Par value of a share = `10 \ Total dividend received
\ Total par value of 50,000 shares = `5,00,000 = `( 2000 + 12500) = `4,500.
Total dividend = `62,500 25.
(b) The total dividend declared = `1,25,000
\ Rate of dividend paid by the company Amount kept in reserve fund = `50,000
Net amount paid as dividend to the shareholders
 62500  1
=  × 100  % = 12 % = `(125000 – 50000) = `75,000
 500000  2
Number of shares of par value `100 each
 1 = 50,000
21.
(a) Annual dividend on one share =  2 × 7  % Total par value of 50,000 shares
 2
i.e.,  15% of `10 = `(50000 × 100) = `50,00,000

Chapter_20.indd 11 6/5/2015 12:35:34 PM


20.12 I Chapter 20

Rate of dividend paid by the company 29.


(b) Par value of 12,000 shares = `(12000 × 10)

 75000  3 1           = `1,20,000


=  × 100  % = % = 1 %
 5000000  2 2 Dividend received by Shyam

 15 
26.
(b) Dividend on 1,200 preferred shares = `  × 120000 
 100 
= 10% of `(1200 × 50)
= `18,000
 10 
= `  × 1200 × 50  = `60,00 Let, the market value of 12000 shares be `x.
 100 
We have to find x such that 10% of x = 18000
Dividend on 3,000 common shares
10
⇒ × x = 18000
 1  100
=  3 × 2  % of `(3000 × 50)
 2  ⇒ x = 18000 × 10 = 180000

 7  i.e., Market value of 12,000 shares = `1,80,000.


= `  × 3000 × 50  = `10,500
 100  Hence, the market value of one share
\ Total dividend received by Nishita  180000 
= `(6000 + 10500) = `16,500.
= `   = `15.
 12000 
27.
(a) Market value of a share = `25 30.
(c) The total profit of the company = `1,80,000
\ Market value of 12,500 shares Amount kept in reserve fund = `30,000
= `(25 × 12500) = `3,12,500 \ Net amount paid as dividend to shareholders
  Thus, the amount required to purchase 12500
= `(180000 – 30000) = `1,50,000
shares = `312500
   Dividend paid by the company on 50,000 preferred
   Then, Mohan sells these shares at a premium of
shares
`11 each.
\ New market rate per shares  10 × 20 
= `  50000 ×  = `1,00,000
 10 
= `(20 + 11) = `31
\ Selling price of these shares \ Dividend to be paid to common shareholders
= `( 31 × 12500) = `3,87,500 = `(150000 – 100000) = `50,000
\ Gain = S.P. – C.P. Thus, dividend paid on a common share
= `(387500 – 312500)  50000 
= `75,000.
= `   = `2.50
 20000 
28.
(a) Par value of 200 shares = `(200 × 10) Hence, dividend per cent paid on a common share
= `2,000
 2.50 
 8  = `  × 100  % = 25%
Dividend received by Mac = `  × 2000   10 
 100 
= `160 31.
(a) 12% of `(10000 × 100)
Let, the market value of 200 shares be `x  12 
= `  × 10000 × 100  = `1,20,000
We have to find x such that 10% of x = 160  100 
10 Dividend on 50,000 common shares
⇒ × x = 160
100
= 17.6% of `(50000 × 100)
⇒ x = 160 × 10 = 1,600
i.e., Market value of 200 shares = `1,600  17.6 
= `  × 50000 × 100  = `880000
Hence, the market value of one share  100 
 1600  \ Total dividend paid = `(120000 + 880000)
= `   = `8.
 200  = `1000000 = `10 lakhs

Chapter_20.indd 12 6/5/2015 12:35:34 PM


Stocks, Shares and Debentures I 20.13

\ Amount kept in reserve fund  9 


\ Income on `81,000 is `  × 81000 
= `15 lakhs – 10 lakhs = `5 lakhs.  135 
32.
(b) Income of the man from 5,000 ordinary shares of = `5,400.
Company X, which pays a dividend of 20%
37.
(a) Here market value of the stock = `90,000
 5000 × 10 × 20  1
= `   = `10,000    By investing `112 , stock of par value `100 is
 100  2
available.
Selling price of a share of Company X = `30
1 1
\ Selling price of 5,000 shares of Company X \ Income on `112 is 7 %
2 2
= `(5000 × 30)
= `1,50,000  15 2 
\ Income on `90,000 is `  × × 90000 
   Now, the market value of a share of Company Y is  2 225 
given to be `40. = `6,000.
\ Number of shares of Company Y purchased by the 38.
(b) Face value of the stock = `72,000
man from `1,50,000
 72000 19 
 150000  \ Income on stock = `  ×  = `6,840.
=   = `3,750.  100 2
 40 
33.
(c) Par value of a share = `10 39.
(b) Market value of `100 stock
 120  = `(91 + 1) = `92
Market value of a share = ` 10 ×  = `12
 100  1
Income on `92 = `9
   The amount to be paid by the buyer to purchase 2
2,500 shares = `(2500 × 12) = `30,000
 19 19 
Gain of the shareholder on selling one share \ Income on `92,000 = `  × × 92000 
 2 92 
= `(20 – 12) = `8
\ Gain from selling 2,500 shares = `9,500.
= `(2500 × 8) = `20,000. 40.
(d) Market value of `100 stock
 1  1
34.
(a) Face value of the stock = `60000 = `  81 + 1 = `82
 2  2
Income on `100 stock = `12
Income on `82 = `7
 12 
Income on Re 1 stock = `  
 100   15 2 
\ Income on `99,000 = `  × × 99000 
 2 165 
 12 
Income on `60,000 = `  × 60000 
 100  = `9,000.
= `7,200. 41.
(c) Market value of `100 stock
35.
(c) Face value of the stock = `20,000 = `(112 + 2) = `114
1 1
Income on `100 stock = `7 Income on `114 = `9
2 2
 15 / 2   15 
Income on Re 1 stock = `   = ` 
 100   200  19 1
\ Income on `88,008 = ` × × 88008
2 114
 15 
Income on `20,000 stock = ` 
× 20000  = `7,334.
 200 
= `1,500. 42.
(a) Market value of `100 stock = `92
\ Market value of `1,25,000 stock
36.
(c) Here the market value of the stock
= `81,000  92 
= `  × 125000  = `1,15,000
   By investing `135, stock of par value `100 is  100 
available \ An investment of `1,15,000 is required to purchase
\ Income on `135 is `9 `1,25,000 of 8% stock at 92.

Chapter_20.indd 13 6/5/2015 12:35:35 PM


20.14 Chapter 20

43.
(b) Market value of `100 stock = `110
 630 
\
Gain per cent =  × 100  %
\ Market value of `90,000 stock  33120 
 110  = 1.9 (approx).
= `  × 90000  = `99,000
 100  47.
(a) Stock purchased by investing `27,600 in 4% stock at
\ An investment of `99,000 is required to purchase 92
`90,000 of 8% stock at 110.  27600 × 100 
= `   = `30,000
 92 
44.
(c) Brokerage = 1% `90 = `0.90
\ Investment needed to buy `100 stock
Money realized by selling `20,000 stock at market
value of `96
= `90.90 on which the income is `9%
 20000 × 96 
1 = `   = `19,200
For income of `9 , the investment  100 
2
Remaining stock = `(30000 – 20000) = `10,000
= `90.90
Money realized by selling `10,000 stock at `90
For income of `1,938, the investment
 90 
 90.90 × 2 
= ` 10000 ×  = `9,000
= `  × 1938   100 
 19  \ Total money realized by selling the whole stock
= `18,543.60. = `(19200 + 9000) = `28,200
45.
(b) Investment made by the man in buying `20,000 of 5% Money invested = `27,600
stock at 90 \ Gain = `(28200 – 27600) = `600.
 90  48.
(b) Stock purchased by investing `28,500 in 5% stock at
= `  × 20000 
 100  95
= `18,000  100 
= `  × 28500  = `30,000
 95 
3
   When the price rose to `93 , the man sold the Money realized by selling `15,000 stock market
4
value of `98
stock. Thus, money realized from selling the stock
 98 
= `  × 15000  = `14,700
 375 1   100 
= `  × × 20000  = `18,750
 4 100  Remaining stock = `( 30000 – 15000) = `15,000
\ Gain in the transaction Money realized by selling `15,000 stock at `90

= `(18750 – 18000) = `750  90 


= `  × 15000  = `13,500
 100 
 750  1 \  Total money realized
\ Gain per cent =  × 100  % = 4 %
 18000  6 = `(14700 + 13500) = `28,200
Money invested = `28,500
46.
(a) Investment made by Meena in buying `36,000 of
\ Loss = `(28500 – 28200) = `300.
1  92 
7 % stock at 92 = `  × 36000 
2  100   7 
49. (c) Income from first stock = `  × 245000 
 98 
= `33,120
= `17,500
3
   When the price rose to `93 , Meena sold the We have to find the amount realised on selling this
4
stock.
stock. Thus, money realized from selling the stock
Amount realized on selling `98 stock = `100
 375 1  \ Amount realized on selling `2,45,000 stock
= `  × × 36000  = `33,750
 4 100   100 
= `  × 245000  = `2,50,000
\ Gain in the transaction  98 
= `(33750 – 33120) = `630 This amount is invested in 9% stock at 125

Chapter_20.indd 14 6/5/2015 12:35:36 PM


Stocks, Shares and Debentures 20.15

\ Income from second stock  5 


52.
(c) Income on first stock = `  × 260000 
 9   104 
= `  × 250000  = `18,000
 125  = `12,500
Hence, increase in income Money realized by selling the stock when price
= `(18000 – 17500) = `500. rose to `125
50.
(b) Income from first stock  125 
= `  × 260000  = `3,12,500
 104 
 8 
= `  × 32400  = `2,880
 90  Income on second stock is `2,500 more than on the
first stock.
Amount of stock purchased by Anu
\ Income on second stock
 100  = `(12500 + 2500) = `15,000
= `  × 32400  = `36,000
 90  Let, `x be the market value of the second stock
Amount received by selling `18,000 stock at 95 312500 × 6 312500 × 6
\ = 15000 ⇒ x = = 125
 95  x 15000
= `  × 18000  = `17,100
 100  i.e.,  The man purchased the stock at `125.

Amount received by selling the remaining s 18,000 53.


(c) The market value of a debenture = `125
stock at 98 \ Income on `125 is `5
 98 
= `  × 18000  = `17,640  5  19
 100  \ Income on `95 is `  × 95  = `
 125  5
\ Total amount received \ Per cent income on the debentures is 3.8%
= `(17100 + 17640) = `34,740
54.
(b) The market value of a debenture = `150
The amount of `34,740 is invested in 10% stock at \ Income on `150 is `10
1
96 .  10 
2 \ Income on `120 = `  × 120  = `8
 150 
\ Income from this stock
\ Per cent income on the debentures = 8%
 2 
= ` 10 × × 34740  = `3,600 55.
(a) Annual dividend on 800 shares
 193 
Hence, change in income  800 × 50 × 6 
= `   = `2,400
= `(3600 – 2880) = `720.  100 
Annual interest on 600 debentures
51. (a) Purchase price of first stock
= `(99 + 3) = `102  600 × 100 × 12 
= `   = `7,200
\ Income on first stock  100 
 5  \ Total annual income of Brij
= `  × 50490  = `2,475
 102  = `(2400 + 7200) = `9,600
Sale price of stock = `(102 – 3) = `99 Total investment of Brij
\ Amount received by selling the first stock = `(800 × 50 + 600 × 100)
 99  = `(40000 + 60000)
= `  × 50490  = `49,005
 102  = `1,00,000
Purchase price of the second stock \ Rate of return
= `(96 + 3) = `99  9600 
=  × 100  % = 9.6%
\ Income on second stock  100000 
 8 
= `  × 49005  = `3,960 56.
(c) Face value = `(50 × 20) = `1,000
 99 
Hence, change in income  1000 × 19   95 
Dividend = `   = ` 
= `(3960 – 2475) = `1,485.  4 × 100   2 

Chapter_20.indd 15 6/5/2015 12:35:36 PM


20.16 Chapter 20

Investment = `(45 × 20) = `900 Let, ‘b’ stocks be bought at x% discount, i.e., at `(100 –
x) per stock.
 95 × 100 
Rate = `   = 5.28% Now, total premium paid = ax and total discount = bx
 2 × 900  \ ax = 70% of bx ⇒ ax = 0.7bx
10 a 7
57.
(b) Divdend from company A = × 100 × 100 \ =
100 b 10
= `1,000
Dividend from 5% stock = 5a
Dividend from company
Dividend from 8% stock = 8b
100 × 95 15
B = × 100 × 5a
114 100 \ Required percentage = × 100
8b
= 83.33 × 15 = `1,250
5 7
Hence, total gain = `250 = × × 100
8 10
58.
(c) Let, ‘a’ stocks be bought at x% premium, i.e., at `(100 +
x) per stock. = 43.75%

Chapter_20.indd 16 6/5/2015 1:24:39 PM


Discount: True
CHAPTER

and Banker’s 21
INTRODUCTION bank to pay `10000 to B after 3 months. Besides, 3 days
grace period is also added to this date (named as nominally
Suppose, a man buys a pen at a credit of one year for due date) of expiry of 3 months to arrive at a date called
`105 at 5% simple interest. If the money is to be paid
legally due date. Thus, if 4 April 2004 is the nominally due
immediately, he shall give `100. `100 is the present value date then 7 April 2004 will be legally due date. The amount
or present worth of `105 due 1 year hence. The sum due of `10000 is called the face value.
(`105) is called the amount and the reduction made in
consideration of making the immediate payment is called Now, suppose B needs the money of this bill earlier
true discount. than 7 April say 7 March. In such a case, B can approach
the banker or broker to pay him money against the bill.
Present Value Obviously, in such a situation, the money paid by the banker
will be less than the face value of the bill. Now suppose,
The present value or present worth of a sum of money due at the bill is presented to the banker on 7 March 2004, then
the end of a given time is that sum which with its interest for the banker will deduct the interest on the face value for
the given time at the given rate will amount to the sum due. the period 7 March 2004 to 7 April 2004 and this interest
is called the Banker’s Discount (B.D.) or Commercial
True Discount (T.D.) Discount.
The true discount is the difference between the sum due at Thus, Banker’s Discount is the simple interest on the
the end of a given time and its present worth. face value for the period from the date on which the bill was
Thus, T.D. = Amount (A) – Present worth (P.W.) discounted and the legally due date. The amount mentioned
in the bill is called the face value of the bill. It may be noted
In the above case, T.D. = `(105 – 100) = `5.
that banker’s discount is greater than true discount because
while the true discount is the interest on the present worth,
Note: banker’s discount is the interest on sum due.
The difference between the present worth and cash
1. Clearly, T.D. is the interest on P.W. and value of a bill is called the banker’s gain (B.G.) Thus, the
A = P.W. + T.D. interest on the bill value (or the face value) is called the
banker’s discount (B.D) and the difference between the
2. Interest is reckoned on P.W. and T.D. is reckoned on banker’s discount and true discount (T.D.) is called banker’s
amount. gain (B.G.)

Banker’s Discount We have the following results:

Suppose, a businessman A purchases goods worth `10000 Banker’s gain = Banker’s discount – True discount
from businessman B at a credit of say 3 months. Thus, B = Interest on sum due – Interest on present
prepares a bill, called the bill of exchange.. On receipt of worth
the goods, A gives an agreement and signs the bill accepting = Interest on (sum due – present worth)
that the money can be withdrawn from his account after
= Interest on true discount.
3 months of the date of the bill. Accordingly, A orders his

Chapter_21.indd 1 6/5/2015 12:47:46 PM


21.2 Chapter 21

soMe Basic forMulae

Illustration 4 The true discount on a certain sum of money


1. If rate = R% p.a. and time = T years, then due 2 years hence is `1800 and the simple interest on the
100 × A 100 × T.D. same sum is `2232. Find the sum
(i) P.W. = =
100 + R × T R ×T S.I. × T.D. 2232 × 1800
Solution: Sum (A) = =
S.I. − T.D. 2232 − 1800
Illustration 1 Find the present worth of `8700 due in
2232 × 1800
3 years at 15% per annum at simple interest = = `9300
432
Also, find the true discount.
100 × A 100 × 8700 (iv) S.I. – T.D. = S.I. on T.D.
Solution: P.W. = =
100 + R × T 100 + 15 × 3
Explanation
100 × 8700 A × R × T P.W. × R × T
= = `6000 S.I. – T.D. = −
145 100 100
T.D. = Amount – P.W. R ×T
= (A – P.W.) ×
= 8700 – 6000 = `2700 100
T.D. × R × T
P.W. × R × T A× R ×T = = S.I. on T.D
(ii) T.D. = = 100
100 100 + R × T Illustration 5 The discount on a certain sum due 4 years
hence is `100 but the interest on the same sum for the same
Illustration 2 Find the true discount and the sum due 15 period is `125. Find the sum and the rate per cent
months hence whose present value at 8% is `1000 Solution: We have,
P.W. × R × T 1000 × 8 × 15 S.I. on T.D. = S.I. – T.D.
Solution: T.D. = =
100 100 × 12 = 125 – 100 = `25.
= `100. 25 × 100 25 1
\ Rate (R) = = =6 %
Sum due = P.W. + T.D. 100 × 4 4 4
= 1000 + 100 = `1100 S.I. × T.D. 125 × 100
and, Sum (A) = =
Illustration 3 Find the true discount reckoning 3% p.a. S.I. − T.D. 125 − 100
simple interest on `1802 due in 2 year’s time = `500
A× R ×T
Solution: T.D. = (v) When the money is invested on compound
100 + R × T interest,
1802 × 3 × 2 A
= = `102 P.W. = T
100 + 3 × 2  R 
1 + 
 100 
S.I. × T.D.
(iii) Sum (A) = Illustration 6 Find the present worth of a bill of `3380 due
S.I. − T.D.
2 years hence at 4% compound interest. Also, calculate T.D
Explanation A 3380
Solution: P.W. = =
 T   R 
T
 4 
2

S.I. × T.D.  A× R ×  × T.D.  1 +   1 + 


 100   100   100 
= =A
S.I. − T.D.  T 
 T.D. × R ×  3380 × 25 × 25
 100  = = `3125
26 × 26
S.I. × T.D.
\ A= T.D. = A – P.W.
S.I. − T.D. = 3380 – 3125 = `255

Chapter_21.indd 2 6/5/2015 12:47:47 PM


Discount: True and Banker’s 21.3

Illustration 8 The banker’s discount and the true discount


2. B.D. = S.I. on the bill for unexpired time on a certain sum of money due 4 months hence are `48 and
3. B.G. = B.D. – T.D. `45, respectively. Find the sum and the rate of interest.
4. B.D. = T.D. + Interest on T.D. B.D. × T.D. 48 × 45 48 × 45
Solution: Sum = = =
B.D. × T.D. B.D. − T.D. 48 − 45 3
5. Sum Due =
B.D. − T.D. = `720
6. T.D. = P.W. × B.G. Now, the banker’s discount is simple interest on the
sum due for 4 months.
B.G. × 100
7. T.D. = 100 × 48 × 3
Rate × Time Rate of interest = = 20% p.a.
720 × 1
Amount × Rate × Time
8. T.D. = Illustration 9 Find the face value of 3 months bill when the
100 + (Rate × Time)
banker’s discount at 3% per annum is `18
Amount × Rate × Time
9. B.D. = Solution: B.D. = `18, Rate (R) = 3%,
100
1
B.D. × 100 Time (T) = years
10. Sum Due = 4
R ×T
B.D. × 100 18 × 100 × 4
11. Money paid by the banker = Amount – B.D. ∴ Face value = =
R×T 3 ×1
Illustration 7 A bill is drawn for `5050 on 12 June 2004 at = `2400
5 months credit. It is discounted on 3 September at 5% per
Illustration 10 The present worth of a bill due sometimes
annum. Find:
hence is `1500. Find the banker’s discount on the bill, if the
(i) Banker’s discount true discount is `75
(ii) Money received by the holder of the bill, and
Solution: T.D. = P.W. × B.G.
(iii) Banker’s gain.
Solution: Amount = `5050 ⇒ 75 = 1500 × B.G.
Date of drawing = 12 June, 2004 (for 5 months)
Date of maturing = 15 Nov, 2004 (including 3 days grace) ⇒ 75 × 75 = 1500 × B.G.
Date of discounting = 3 September, 2004 75 × 75 15
⇒ B.G. = =` or, `3.75
No. of days from 3 Sept. to 15 Nov. = 1500 4
Sept. Oct. Nov. B.D. = T.D. + B.G.
1
27 + 31 + 15 = 73 days = year = 75 + 3.75 = `78.75
5
Amount × Rate × Time Illustration 11 The banker’s gain on a bill due 1 year hence
∴ (i) Banker’s discount = at 12% per annum is `6. Find the true discount
100
5050 × 1 × 5 B.G. × 100 6 × 100
= = `50.50 Solution: T.D. = = = `50
100 × 5 R ×T 12 × 1
(ii) Amount received by the holder of the bill
= Amount – B.D. Illustration 12 If the true discount on a certain sum due 6
= 5050 – 50.50 = `4999.50 months hence at 6% is `36, what is the banker’s discount on
the same sum for the same time and at the same rate?
(iii) True discount on `5050
1 Solution: B.D. = T.D. + Interest on T.D.
5050 + × 5
Amount × Rate × Time 5 T.D. × R × T 36 × 6 × 6
= = = `50 = T.D. + = 36 +
100+(Rate × Time) 1
100 + × 5 100 100 × 12
5
Banker’s gain = B.D. – T.D. = `0.50. = 36 + 1.08 = `37.08

Chapter_21.indd 3 6/9/2015 2:56:22 PM


21.4 Chapter 21

Practice Exercises

Difficulty Level-1
(Based on Memory)

1. The true discount on a bill for `1260 due 6 months hence 9. If `21 in the true discount on `371 for a certain time, what
at 10% per annum is: is the true discount on the same sum for double that time,
(a) `60 (b) `160 the rate being the same in both cases?
(c) `80 (d) `260 (a) `39.00 (b) `35.75
2. If the discount on a certain sum in 2 years at a certain rate (c) `40.00 (d) `39.75
is `150 and the interest in 3 years is `240. Find the sum 10. The T.D. on a certain sum of money due 2 years hence is
and the rate % `1800 and the simple interest on the same sum is `2232.
1 1 Find the sum.
(a) `2400, 3 % (b) `2400, 4 %
3 3 (a) `9300 (b) `9350
(c) `9450 (d) `9400
1
(c) `2200, 5 % (d) None of these 11. The present worth of `220.50 due in 2 years reckoning
3
compound interest at 5% is:
3. If the true discount on `161 due 2 years 6 months hence is (a) `200 (b) `197.5
`21, find the rate of interest. (c) `202 (d) `192.25
1 1
(a) 2 % (b) 4 % 12. The T.D. on `936 due after a certain time at 8% is `36.
2 2
Money is due after:
(c) 5% (d) 6% (a) 6 months (b) 3 months
4. The present worth of `920 due at the end of 3 years at 5% (c) 1 years (d) 9 months
simple interest per annum is: 13. A man bought a motor-cycle for `32500 and sold it for
(a) `780 (b) `850 `35000, allowing the buyer 6 months credit. If the money
(c) `800 (d) `810 be worth 4% per annum, the gain per cent is:
1 9
5. If the simple interest on a certain sum due some years (a) 8 % (b) 7 %
hence at 6% is `180 and the discount at 5% on the same 7 13
sum is `140. Find the sum and the time. 5 2
(c) 7 % (d) 8 %
3 13 3
(a) `2100 and 1 years
7 14. Find the present worth of a bill of `3720 which is due 2
3 years hence at 12% compound interest, being compounded
(b) `2200 and 2 years
7 annually.
(a) `3100 (b) `3150
3
(c) `2000 and 2 years (c) `3125 (d) `3225
7

(d) None of these 15. The holder of a bill for `17850 nominally due on 21 May,
1991 received `357 less than the amount of the bill by
6. The banker’s gain on a certain sum due 9 months hence at having it discounted at 5% When was it doscounted?
4% is `2.25. The sum is: (a) 29 Dec. 1990 (b) 30 Dec. 1989
(a) `2575 (b) `2500 (c) 19 Dec. 1990 (d) None of these
(c) `2250 (d) `3250 16. The true discount on a certain bill due nine months hence
7. At a given rate per cent, the simple interest and the true at 4% simple interest is `150. Find the amount of the bill.
discount on a certain sum for a given time are `24 and (a) `5150 (b) `5250
`22, respectively. The sum is: (c) `4750 (d) `5650
(a) `264 (b) `220 17. A banker discounts a 4 months bill at 3% discount. If the
(c) `288 (d) `295 proceeds be invested so that nothing is lost, the rate p.c.
8. The present worth of a bill of `1764 due 2 years hence at should be:
5% compound interest is: (a) 3% (b) 4%
(a) `1650 (b) `1700 1
(c) 3 % (d) None of these
(c) `1600 (d) `1714 33

Chapter_21.indd 4 6/5/2015 12:47:49 PM


Discount: True and Banker’s 21.5

18. The difference between the simple interest and the true 27. What rate per cent does a man get for his money when in
discount on a certain sum of money for 2 years at 15% per discounting a bill due 10 months hence, he deducts 4% of
annum at simple interest is `45. Find the sum. the amount of the bill?
(a) `700 (b) `650 (a) 5% (b) 6%
(c) `675 (d) `625 (c) 8% (d) 4%
19. The present worth of a sum of money due 146 days hence 28. The discount on `5229 due in 1 year 9 months reckoning
at 5% is `400. The sum due is: compound interest at 5% is:
(a) `410 (b) `408 (a) `429.00 (b) `415.00
(c) `415 (d) `450 (c) `393.25 (d) None of these

20. The present worth of a bill due sometime hence is `1500. 29. A bill is discounted at 5% per annum. If banker’s discount
Find the banker’s discount on the bill, if the true discount be allowed, at what rate per cent must the proceeds be
is `75. invested, so that nothing may be lost?
(a) `78.75 (b) `77.75 19
(a) 5% (b) 4 %
20
(c) `82.75 (d) `76.75
5
(c) 5 % (d) 10%
21. If the simple interest on `2000 at 5% p.a. is equal to the 19
true discount on `2500 for the same time and at the same
rate, the time is: 30.
An article is listed at `65. A customer bought this article
for `56.16 with two successive discounts of which one is
1 10% The other discount of this discount scheme that was
(a) 4 years (b) 5 years
2 allowed by the shopkeeper is:
1 1 (a) 4% (b) 3%
(c) 7 years (d) 2 years
2 2 (c) 6% (d) 2.5%
22. `21 is the true discount on `371 for a certain time at [Based on MAT (Sept), 2010 (Dec), 2005)]
certain rate p.c. If the rate p.c. is kept same, true discount 31.
The simple interest and the true discount on a certain sum
on the same sum for double that time will be: and at a given rate are `25 and `20 respectively. The sum
(a) `44.38 (b) `39.75 is:
(c) `33.25 (d) None of these (a) `500 (b) `200
(c) `250 (d) `100
23. The true discount on a bill of `5450 due in 9 months is
[Based on MAT (Feb), 2009)]
`450. Find the rate of interest.
(a) 12% (b) 12.5% 32.
Two shopkeepers announce the same price of `700 for a
shirt. The first offers successive discounts of 30% and 6%
(c) 11.5% (d) 13.1%
while the second offers successive discounts of 20% and
24. If `10 be allowed as true discount on a bill of `110 due at 16% The shopkeeper that offers better discount is more
the end of a certain time, then the discount allowed on the of:
same sum due at the end of double the time is: (a) `22.40 (b) `16.80
(a) `20 (b) `21.81 (c) `9.80 (d) `36.40
(c) `22 (d) `18.33 [Based on MAT (May), 2009)]
33.
A cash payment that will settle a bill for 250 chairs at `50
25. A bill which being due at the end of 4 years is now worth
per chair less 20% and 15% with a further discount of 5%
1 on cash payment is:
`575 but if it is due 2 years hence it would now be
2 (a) `8075 (b) `7025
worth `620. The sum of the bill is:
(c) `8500 (d) None of these
(a) `695 (b) `725
[Based on MAT (Feb), 2008)]
(c) `713 (d) None of these
34.
A shopkeeper allows a discount of 10% on his goods. For
26. Find the present worth (P.W.) and the true discount cash payments, he further allows a discount of 20% Find
reckoning 6% per annum simple interest of `176 due in a single discount equivalent of the above offer.
20 months time. (a) 30% (b) 18%
(a) `160, `16 (b) `130, `46 (c) 28% (d) 15%
(c) `150, `26 (d) None of these [Based on MAT, 1999)]

Chapter_21.indd 5 6/5/2015 12:47:49 PM


21.6 I Chapter 21

35.
Successive discounts of 10%, 12% and 15% amount to a 43.
If on a marked price, the difference of selling prices with a
single discount of: discount of 30% and two successive discounts of 20% and
(a) 35.28% 10% is `72, then the marked price (in rupees) is:
(b) 36.68% (a) 3,600 (b) 3,000
(c) 37% (c) 2,500 (d) 2,400
(d) None of these 44.
Successive discounts of 10%, 20% and 30% is equivalent
[Based on MAT, 1999)] to a single discount of:
36. A tradesman gives 4% discount on the marked price and (a) 60% (b) 49.6%
gives 1 article free for buying every 15 articles and thus (c) 40.5% (d) 36%
gains 30% The marked price is above the cost price by:
45. The true discount on a bill of `2550 due after 3 months is
(a) 40% (b) 39% `50. Find the banker’s discount.
(c) 20% (d) 50% (a) `53 (b) `51
[Based on NMAT, 2005)] (c) `55 (d) `57
37. A discount series of p% and q% on an invoice is the same
as a single discount of: 46. A owes B `1350 due 3 months hence and B owes A
`1078 due 5 months hence. If they agree to settle their
 pq   pq  account right now at rate 5% p.a., A should pay B:
(a)  p + q + % (b)   p − q %
 100   100  1
(a) `277
 qp  3
(c) 100 –  p + q +  %
(d)  None of these (b) `288.25
 100 
[Based on NMAT, 2006)]
(c) `302
(d) None of these
38. A man buys a single apple for `25. If he were to buy a
dozen apples, he would have to pay a total amount of 47. What is the rate of interest when the P.W. of `1245 due 15
`250. What would be the approximate per cent discount months hence is `1200?
he would get on buying a dozen apples? (a) 3% (b) 4%
(a) 32 (b) 20 1
(c) 4 % (d) 5%
(c) 12 (d) 17 2
39. If the compound interest on a certain sum of money for 2 48. A has to pay `22 to B after 1 year. B asks A to pay `110 in
years at 4% is `45.90, the true discount on the same sum cash and defer the payment of `110 for 2 years. A agrees
of money due 2 years hence at 4% simple interest is: to it. Counting the rate of interest at 10% per annum in this
(a) `39.69 (b) `41.67 new mode of payment,
(c) `45.00 (d) `38.45 (a) there is no gain or loss to anyone
(b) A gains `7.34
40.
An article is marked 40% above the cost price and
(c) A loses `7.34
a discount of 30% is allowed. What is the gain or loss
percentage? (d) A gains `11
(a) 10% gain (b) 5% gain 49. The B.G. on a sum due 3 years hence at 10% is `180. The
(c) 2% loss (d) 12% loss B.D. is:
(a) `680 (b) `780
41.
The difference between a discount of 40% on `500 and
two successive discounts of 36%, 4% on the same amount (c) `580 (d) `480
is: 50. The banker’s gain on a certain sum due 2 years hence at
(a) `0 (b) `2 5% per annum is `8. The present worth is:
(c) `1.93 (d) `7.20 (a) `800 (b) `1600
42. `20 is the true discount on `260 due after a certain time. (c) `1200 (d) `880
What will be the true discount on the same sum due 51. The true discount on a certain sum of money due
after half of the former time, the rate of interest being 1
the same: 12 years hence is `150 and the simple interest on the
2
(a) `10 (b) `10.40 same sum for the same time and at the same rate is `200.
(c) `15.20 (d) `13 Interest rate per annum is:

M21_KHAT6981_C21.indd 6 6/16/2015 5:46:35 PM


Discount: True and Banker’s 21.7

(a) 10% (b) 12% including the sales tax, then what is, the marked price of
1 1 the item?
(c) 12 % (d) 8 %
2 2 (a) `630 (b) `700
[Based on MAT, 2011)]
52. `20 is the true discount on `260 due after a certain time.
(c) `780 (d) None of these
What will be the true discount on the same sum due after [Based on MAT, 2014)]
half of the former time, the rate of interest being the same? 54. An article is listed at `65. A customer bought this article
(a) `15.20 (b) `10.40 for `56.16 and got two successive discounts of which the
first one is 10% The other rate of discount of this scheme
(c) `10.83 (d) `13
that was allowed by the shopkeeper was:
[Based on MAT, 2011)]
(a) 3% (b) 4%
53. A shopkeeper allows a discount of 10% on the marked
(c) 6% (d) 2%
price of an item but charges a sales tax of 8% on the
discounted price. If the customer pays 680.40 as the price [Based on SNAP, 2012)]

Answer Keys
Difficulty Level-1

1. (a) 2. (a) 3. (d ) 4. (c) 5. (a) 6. (a) 7. (a) 8. (c) 9. (d ) 10. (a) 11. (a) 12. (a) 13. (b)
14. (c) 15. (a) 16. (a) 17. (c) 18. (b) 19. (b) 20. (a) 21. (b) 22. (b) 23. (a) 24. (d ) 25. (c) 26. (a)
27. (a) 28. (a) 29. (c) 30. (a) 31. (d ) 32. (c) 33. (a) 34. (c) 35. (d ) 36. (d ) 37. (d ) 38. (d ) 39. (b)
40. (c) 41. (d ) 42. (b) 43. (a) 44. (b) 45. (b) 46. (a) 47. (a) 48. (b) 49. (b) 50. (a) 51. (c) 52. (b)
53. (b) 54. (b)­

Explanatory Answers

Difficulty Level-1

1 A × 100 920 × 100


1260 × × 10 4. (c) Present worth = =`
1.
(a) T.D. = `  2 = `60. 100 + R × T 100 + 3 × 5
1
100 + × 10 920 × 100
2 =` = `800.
115
240 × 2
2.
(a) Interest for 2 years = = `160 5.
(a) B.D. or Simple Interest at 5%
3
Discount for 2 years = `150 180 × 5
= = `150
B.D. × T.D. 6
Sum due =
B.D. − T.D. Discount at 5% = `140
160 × 150 B.D × T.D.
= = `2400 Sum =
160 − 150 B.D. − T.D.
240 × 100 1
Rate of interest = =3 % 150 × 140
2400 × 3 3 = = `2100.
150 − 140
3.
(d) `21 is the interest on `(161 – 21) 180 × 100
Rate per cent =
or, `140 for 2 years 6 months 2100 × 6
21 × 100 3
\ Rate % = = 6% =1 years.
140 × 5/2 7

Chapter_21.indd 7 6/5/2015 12:47:50 PM


21.8 Chapter 21

6.
(a) B.G. is the interest on T.D. 13.
(b) S.P. of motorcycle = `35000
2.25 × 100 Gain = `35000 – `32500
\ T.D. = = `75
3
×4 = `2500
4
2500
B.D. = `75 + `2.25 = `77.25 \ Gain % = × 100%
32500
B.D. × T.D.
\ Sum due = 100 9
B.G. = %=7 %
13 13
77.25 × 75 A 3720
= ` = `2575. 14.
(c) P.W. = =`
2.25 T 2
 R   12 
7.
(a) Sum =
T.D. × S.I.
=
24 × 22
= `264. 1 +  1+ 
 100   100 
S.I. − T.D. 24 − 22
25 25
2 = `3720 × × = `3125.
 5  28 28
8.
(c) P.W. = `1764 ÷ 1 + 
 100  15.
(a) Clearly, S.I. on `17850 at 5% is `357
400  100 × 357  2
= `1764 × = `1600. \ Time = 
441  =  years = 146 days.
 17850 × 5  5
9.
(d) `21 is the interest on `(371 – 21) = `350    So, the bill is 146 days prior to 24 May, the legally
350 × no. of years × rate due date.
\ = 21
100 May April March Feb. Jan. Dec.
2100
⇒ No. of years × rate = = 6 24 + 30 + 31 + 28 + 31 + 2 = 146 days
350
So, the bill was discounted on 29 Dec. 1990.
\  Twice no. of years × rate = 12
100 150 × 100
Now, on `112, the T.D. is `12 16.
(a) P.W. = T.D. ×  =`
R ×T 9
×4
12 12
\ on `371, the T.D. = ` × 371
112 = `5,000
= `39.75. \ Amount of the bill = `5000 + `150
  = `5150.
Simple interest on the sum × (T.D.)
10.
(a) Sum = 1
Simple interest on the sum − (T.D.) 17.
(c) 4 months = year
3
 2232 × 1800  1
= `  \ Banker deducts `3 × = `1 from a bill of `100
 3
 2232 − 1800 
So, the banker pays `(100 – 1) = `99
 2232 × 1800  So, the bill holder loses Re. 1
= `  
 432  So, for investment Re. 1 should be interest on `99 for
4 months.
= `9300.
1 × 100 100 1
\ Rate = = =1 %
220.5 220.5 × 20 × 20 4 33 33
11.
(a) P.W. = = 99 ×
 5 
2 21 × 21 12
1 + 
 100  18.
(b) Let, the sum be `100
= `200. Time = 2 years
12.
(a) P.W. = `(936 – 36) = `900 Rate = 15% per annum

`36 is S.I. on `900  100 × 15 × 2 


S.I. = `   = `30
 100 
36 × 100 1
\ Time = = year
900 × 8 2  100 × R × T 
Time Discount = `  
= 6 months. 100 + ( R × T ) 

Chapter_21.indd 8 6/5/2015 12:50:17 PM


Discount: True and Banker’s 21.9

 100 × 15 × 2  23.
(a) Amount = `5450
  P.W. = Amount – T.D.
100 + (15 × 2) 
= ` 
= `5450 – `450
100 × 15 × 2  300
= ` 
130  = ` 13 . = `5000
 
The difference between S.I. and T.D. is   In other words, simple interest on `5000 for
9 months is `450
 300  90
`  30 −  =` S.I. × 100 450 × 100 × 4
 13  13 \ Rate = =
90 P × T 5000 × 3
If the difference in S.I. and T.D. is ` , the sum
= `100. 13
= 12% per annum.
If the difference in S.I. and T.D. is Re. 1, the sum = 24.
(d) S.I. on `(110 – 10) for a given time = ` 10
13 S.I. on `100 for double the time = `20
`100 × .
10 Sum = `(100 + 20) = `120
If the difference in S.I. and T.D. is `45, the sum = `100
13  20 
× × 45 = `650. T.D. on `110 = `  × 110  = `18.33.
90  120 
146 5 25.
(c) Let, rate p.c. be r %
19.
(b) T.D. = `400 × × = `8
365 100 Let, `x be the amount of the bill.
S.D. = `400 + `8 = `408. x × 100
Then, 575 =
100 + 4r
20.
(a) T.D. = P.W. × B.G.
i.e., 57500 + 2300r = 100x
75 = 1500 × B.G. \ x = 575 + 23r (1)
Squaring both the sides 100 x
and, 620 =
75 × 75 = 1500 × B.G. 5
100 + r
75 × 75 15 2
B.G. = ` =` = `3.75 \ 62000 + 1550r = 100x
1500 4
\ 6200 + 155r = 10x
B.D. = T.D. + B.G.
= 5750 + 230r [Using (1)]
= 75 + `3.75 = `78.75.
\ 75r = 450
21.
(b) T.D. = `500 \ r=6
Rate = 5% \  From 1 ⇒ x = 575 + 138
P.W. = `2000 = `713.
500 × 100 100 × 76
\ Time = = 5 years. 26.
(a) Present worth = = `160
2000 × 5 20
100 + 6 ×
12
22.
(b) P.W. of `371 = `(371 – 21) = `350
True discount = Amount – Present worth
Also, T.D. = Simple Interest on P.W. = `176 – `160
\ Simple interest on `350 for a certain period at = `16.
certain rate p.c. = `21
27.
(a) Let, the amount of the bill be `100
\ Simple Interest on `350 for double the period at Money deducted = `4
same rate p.c. = `42
Money received by holder of the bill
\ `42 is T.D. on `(350 + 42) = `392 for double the = `(100 – 4) = `96
period.
S.I. on `96 for 10 months = `4
\ T.D. on `371 for double the period and same rate
42 100 × 4 × 6
p.c. = × 371 = `39.75. Rate = = 5%
392 96 × 5

Chapter_21.indd 9 6/5/2015 12:47:52 PM


21.10 I Chapter 21

5229 Price after discount


28. (a) P.W. = 80 85 95
 5  3 5  = 12500 × × ×
1 + 1 + ⋅  100 100 100
 100  4 100 
20 80 = `8075
= 5229 × × = `4800
21 83 34.
(c) Quicker Method: Discount equivalent to two
successive discounts of x % and y % is given by
\ T.D. = `(5229 – 4800) = `429.
 x× y
29.
(c) Let, the sum be `100. Then, B.D. = `5 x + y − %
 100 
Proceeds = `(100 – 5) = `95 \ Required equivalent discount
\ `5 must be the interest on `95 for 1 year. 10 × 20
= 10 + 20 – = 30 – 2 = 28%
 100 × 5  5 100
So, rate =   =5 % 35.
(d) We know that discount equivalent to two sucessive
 95 × 1  19 discounts of x% and y% is given by
30.
(a) Let, the other discount be x%
 xy 
90 (100 − x ) x + y − %
\ 65 × × = 56.16  100 
100 100
Now, from the question,
56.16 × 100 × 100
⇒  100 – x =    Equivalent discount of two successive discounts
65 × 90
10% and 12%
⇒  100 – x = 96
 10 × 12 
⇒  x = 4% = 10 + 12 −  = 20.8%
 100 
31.
(d) Let, the amount be `x and rate is r %
Now, equivalent discounts of 20.8% and 15%
Then, for simple intrest
20.8 × 15
x×r = 20.8 + 15 –
= 25 100
100
⇒ x × r = 2500 (1) = 35.8 – 4.12 = 31.68%
For true discount 36.
(d) Let, C.P. of (15 + 1) articles = `1600
( x − 20) × r 1600 × 135
= 20 \ S.P. of 15 articles =
100 100
x × r − 20r 1600 × 135
⇒ = 20 S.P. of 1 article = = 144
100 100 × 15
⇒ xr – 20r = 2000 (2)
144 × 100
From Eqs. (1) and (2), we get Now, M.P. = = 150%
96
2500 – 20r = 2000
⇒ r = 25% \ M.P. above the C.P. = 50%
From Eq. (1) 37.
(d) Single discount series of p% and q%
x × 25 = 2500 ⇒ x = 100
 pq 
32.
(c) Selling price of first shopkeeper = p+q− %
 100 
70 94
= 700 × × = `460.60
100 100 38.
(d) Cost of one apple = `25
Selling price of second shopkeeper \ Cost of 12 apples = 25 × 12 = `300
80 84
= 700 × × = `470.40 Amount paid = `250
100 100
Required difference Discount = 300 – 250 = `50
= 470.40 – 460.60 = `9.80 50 × 100
% Discount =
300
33.
(a) Original price of 250 chairs
= 250 × 50 = `12500 = 17% (approx.)

Chapter_21.indd 10 6/5/2015 12:47:53 PM


Discount: True and Banker’s I 21.11

39.
(b) Let, the sum be `x. Then, Therefore,

 2  72 x 70 x
4  ` − = `72
45.90 = x 1 +  − 1 100 100
 100  
2x
⇒ = 72
 26 2   676 − 625  100
=   − 1 =   ⇒ 2x = 7200
 25    625 
7200
⇒ x = = `3600.
625 × 45.9 2
\ x = = `562.5
51 44.
(b) Single equivalent discount for successive discounts of
\ T.D. on `562.5 10% and 20%
 20 × 100 
562.5 × 4 × 2 4500 = 10 + 20 − %
= = = `41.67.  100 
100 + 4 × 2 108
= 28%
40.
(c) Let, the C.P. of the article be `100. Therefore, marked
Single equivalent discount for 28% and 30%
price = `140
S.P. = 70% of 140 = `98  28 × 30 
=  28 + 30 − 
100 − 98  100 
Loss% = × 100 = 2%
100 = 49.6%
41.
(d) Single equivalent discount for 36% and 4%
45.
(b) T.D. = `50
 36 + 4 − 36 × 4 
=   P.W. = `2550 – `50 = `2500
 100 
50 × 100 × 4
= (40 – 1.44)% = 38.56% Rate of Interest =
2500 × 1
Therefore, required difference = 8% per annum
= 1.44% of 500
 2550 1
1.44 B.D. = `  × 8 ×  = `51.
= × 500 = `7.20  100 4
100
42.
(b) S.I. on `240 for a given time = `20 1350 × 100 1350 × 400
S.I. on `240 for half the time = `10 46.
(a) P.W. of `1350 = =
3 405
100 + × 5
\ `10 is T.D. on `250 12
 10  4000
So, T.D. on `260 = `  × 260  = `10.40.
 250  =`
3
43.
(a) Let, the marked price be `x 1078 × 100 1078 × 1200
Therefore, in case 1, S.P. P.W. of `1078 = =
5 1225
100 + × 5
70 x 12
= `
100 = `1056
   Single discount equivalent to successive discounts
of 20% and 10%  4000 
\ A should pay B = `  − 1056 
 20 + 10 − 20 × 10   3 
=  %
 100 
832 1
=` = `277
 30 − 200  3 3
Ex =  %
 100 
47.
(a) T.D. = `1245 – `1200 = `45
= (30 – 2)% = 28%
P.W. = `1200
Hence S.P. in this case
72 x 45 × 100
= ` \ Rate % = = 3%
100 1200 × 15/12

Chapter_21.indd 11 6/5/2015 12:47:53 PM


21.12 Chapter 21

48.
(b) A has to pay the P.W. of `220 due 1 year hence, which In second conditon, rate is same but time is half,
is
thereforee we get
 100 × 220 
= `   = `200 T 25 1 25
100 + (10 × 1)  R = =
2 3 2 6
A actually pays = `[110 + P.W. of `110 due 2 years 260 × 25
hence]. Therefore, required TD = `
 25 
 100 × 110  100 +  × 6
= ` 110 +   6 
 100 + (8 × 2) 
260 × 25 × 6
= `192.66 =`
625 × 6
\ A gains = `[200 – 192.66] = `7.34.
260
=` = `10.40
B.G. × 100 180 × 100 25
49.
(b) T.D. = = `  = `600
R×T 10 × 3
53. (b) Let, the marked price on item = `100
\ B.D. = `(600 + 180) = `780.
Discount allowed = 10% = `10
B.G. × 100  8 × 100 
50.
(a) T.D. = = `  ∴  Discounted price of item = 100 – 10 = `90
Rate × Time  5× 2 
= `800. Sales tax = 8% on
90 × 8 72
2 ` 90 = = = ` 7.20
51. (c) We have, TD = `150, SI = `200 and t = 2 years 100 10
3
∴ Total paid amount = 90 + 7.20 = ` 97.20
8
i.e., years
3 When `97.20 is paid, then the marked price = `100
Therefore, from the formula,
TD × r × t 100
SI − TD = , we get and, `1 is paid, then the marked price = `
100 97.20
150 × r × 8 when `680.40 is paid, then the marked price
50 =
300 × 3
100 1 100
⇒ r= %, i.e., 12 % =` × 680.40
8 2 97.20

100 × 6804
52. (b) If time and rate be T and R respectively, then we have =` = ` 700
972
Amount × R × T
TD =
100 + ( R × T ) 54. (b) Price of the article after first discount,
260 × RT
⇒ 20 =  10 
100 + RT 65 −  65 ×  = `58.5
 100 
⇒ 100 + RT = 13RT
Therefore, the second discount
⇒ 12 RT = 100
100 25 58.5 − 56.16
⇒ RT = = = × 100 = 4%
12 3 58.5

Chapter_21.indd 12 6/5/2015 12:47:57 PM


CHAPTER

Binary Number System 22


INTRODUCTION Illustration 1 In the number (5342)x
A number system is nothing more than a code. For each The weight of 2 is x0
distinct quantity, there is an assigned symbol. The most The weight of 4 is x1
familiar number system is the decimal system which uses 10 The weight of 3 is x2
digits, that is, 0, 1, 2, 3, 4, 5, 6, 7, 8, 9. The main advantage
The weight of 5 is x3
of this system is its simplicity and long use. Most of the
ancient societies used this system. Even in our everyday The sum of all the digits multiplied by their respective
life, we use this system and is sometimes being taken as the weights is equal to the decimal equivalent of that number
natural way to count. Since this system uses 10 digits it is and gives the total amount represented by that number.
called a system to base 10. (5342)x = (5x3 + 3x2 + 4x + 2x0)10
A binary number system is a code that uses only two Illustration 2
basic symbols, that is, 0 and 1. This system is very useful
in computers. Since, in this system, only two symbols are 5 7 0 3 4 Number to the base 10,
there, it can be used in electronic industry using ‘on’ and 10 4
10 3
10 2
10 1
10 0
that is, decimal number
‘off’ positions of a switch denoted by the two digits 0 and 1. weights
Decimal Number System ∴ 5 × 104 + 7 × 103 + 0 × 102 + 3 × 10 + 4 × 100
Decimal number system used 10 digits, 0 through 9, that is, = Value represented or decimal equivalent
the digits 0, 1, 2, 3, 4, 5, 6, 7, 8, 9. Illustration 3
Binary Number System 1 1 0 0 1 Number to the base 2
4 3 2 1 0
Binary means two. The binary number system uses only 2 2 2 2 2 that is, binary number weights
two digits, 0 and 1.
∴ 1 × 24 + 1 × 23 + 0 × 22 + 0 × 21 + 1 × 20
Base or Radix = 16 + 8 + 1 = 25
The base or radix of a number system is equal to the = Decimal equivalent or value represented by
number of digits or symbols used in that number system. 110012
For example, decimal system uses 10 digits, so that base
of decimal system (that is, decimal numbers) is 10. Binary Decimal to Binary Conversion
numbers have base 2. Step 1: Divide the number by 2.
A subscript attached to a number indicates the base Step 2: Divide Quotient of Step 1 by 2.
of the number. For example, 1002 means binary 100. 10010
stands for decimal 100. Continue the process till we get quotient = 0
and remainder as 1.
Weights
Then, the remainders from down upwards
In any number to a given base, each digit, depending on its
written from left to right give the binary number.
position in the number has a weight in powers of the base.

Chapter 22.indd 1 6/5/2015 12:58:13 PM


22.2 Chapter 22

Illustration 4 Convert decimal 23 to binary Binary Subtraction


Solution: 2 23 Remainders 1. 0 – 0 = 0
2 11 1 2. 1 – 0 = 1
2 5 1 3. 1 – 1 = 0
2 2 1 4. 10 – 1 = 1
1 0 5. 0 – 1 = –1
0 1 (Complement of a binary number is the exact reverse of the
given number)
Reading the remainders upwards and writing from left Complement of 0 = 1
to right we get the binary equivalent of decimal 23 as 10111.
Complement of 1 = 0
That is, Binary 10111 is equivalent to decimal 23 or we can
write 101112 = 2310 For subtraction of binary number, the following method
known as one’s complement method is used.
Binary to Decimal Conversion
Subtraction of a lower number from a higher
Following steps are involved to convert a binary number. To determine which binary number is lower
number to its decimal equivalent and which is higher, it is advisable to find their decimal
Step 1: Write the binary number. equivalents.
Step 2: Write the weights 20, 21, 22, 23, ... under the Step I: Make the number of digits equal in both
binary digits starting from extreme right. the numbers.
Step 3: Cross out any weight under a zero, that is, Step II: Take the complement of the second
weights under zeros in the binary number number, that is, take the complement of
should be deleted. the number to be subtracted.
Step 4: Add the remaining weights. Step III: Add the complement obtained in Step
II to the first number. The carry over
obtained from this addition indicates that
Illustration 5 Convert binary 1101 to its decimal equivalent the answer shall be positive.
Solution: 1 1 0 1 Binary number
Step IV: This carry over is taken out and added to
23 22 21 20 weights the first digit on the right, that is, extreme
The weight 21 is under 0 so it can be deleted. Sum of right digit.
the remaining weights Step IV: The digits so obtained is the final answer.
= 23 + 22 + 20 = 8 + 4 = 1 = 13
\ Decimal equivalent of binary 1101 = 13
Illustration 7 Subtract 11 from 101
that is, 11012 = 1310
Solution: Now, 1012 = 4 + 1 = 510, 112 = 2 + 1 = 310. Clearly,
Binary Addition 11 is smaller than 101. Making the number of digits equal,
we write 11 as 011
In binary number system there are only 2 digits, that is, 0
and 1. In decimal system we carry 1 for every 10 whereas Complement of 011 = 100.
in binary system we carry 1 for every 2. Hence, rules of Adding 100 to 101, we get
addition are as under: 101
0+0 =0 100
0+1 =1
(1) 001 [Carry over is 1]
1+0 =1
1 + 1 = 10 Taking out the carry over and adding to extreme right
Illustration 6 Add 1010 to 10100 digit, we get
001
Solution: 10100 1
+ 1010 010
11110 \ The answer is 010 or 10

Chapter 22.indd 2 6/5/2015 12:58:14 PM


Binary Number System 22.3

Subtraction of a higher number from a lower number. 1001


0001 [There is no carry over]
Step I: Take the complement of the second number.
Step II: Add the complement obtained in Step I to 1010
the first number. In this case, there is no carry Complement of 1010 = 0101
over indicating that the answer is negative. \ The answer is –0101 or 101
Step III: Recomplement the digits obtained after
adding the complement of the second
number to the first number. Binary Multiplication
Step IV: Put a negative sign before the result Rules: 1 × 1 = 1, 1 × 0 = 0
obtained in Step IV.
Illustration 9 Multiply 1111 by 11
Illustration 8 Subtract 1110 from 1001
Solution: 1111
Solution: Now, 11102 = 8 + 4 + 2 = 1410;
1001 = 8 + 2 = 1010 11
Clearly, 11102 > 10012. 1111
Complement of 1110 = 0001 11111
———
Adding 0001 to 1001, we get 101101

Practice Exercises

Difficulty level-1
(BaseD on MeMory)

1. Find the binary equivalent of decimal 117. 10. Add 11010 to 11100:
(a) 1010101 (b) 1110101 (a) 110110 (b) 111110
(c) 1111101 (d) None of these (c) 110111 (d) None of these
2. Find the binary equivalent of decimal 52. 11. 111112 + 100012 + 10112 =
(a) 110100 (b) 111100 (a) 110111 (b) 111001
(c) Remainder (d) None of these (c) 111011 (d) None of these
3. Find the decimal equivalent of binary 1110101. 12. 110012 + 110112 + 111112 =
(a) 11010 (b) 11110 (a) 1010011 (b) 111011
(c) 11710 (d) None of these (c) 1110011 (d) None of these
4. Find the binary equivalent of decimal 235. 13. 112 + 1112 + 11112 + 111112 =
(a) 10101112 (b) 10101112 (a) 101010 (b) 111000
(c) 111010112 (d) None of these (c) 101100 (d) None of these
5. Find the binary equivalent of decimal 701. 14. 1112 + 1012 =
(a) 10101111012 (b) 10111011012 (a) 1111 (b) 10111
(c) 11101111012 (d) None of these (c) 1100 (d) None of these
6. Find the decimal equivalent of binary 101001. 15. 10002 + 11012 + 11112 =
(a) 31 (b) 41 (a) 100100 (b) 111100
(c) 51 (d) None of these (c) 101010 (d) None of these
7. Find the decimal equivalent of binary 10000010011. 16. 1112 + 1012 + 0112 =
(a) 1043 (b) 1023 (a) 1011 (b) 1111
(c) 1033 (d) None of these (c) 1101 (d) None of these
8. Find the decimal equivalent of binary 111011. 17. 1110002 – 110012 =
(a) 69 (b) 49 (a) 11111 (b) 10111
(c) 59 (d) None of these (c) 11011 (d) None of these
9. Add 1001 to 0101: 18. 100012 – 11112 =
(a) 1111 (b) 1110 (a) 101 (b) 11
(c) 1010 (d) None of these (c) 10 (d) None of these

Chapter 22.indd 3 6/5/2015 12:58:14 PM


22.4 Chapter 22

19. 1111012 – 101112 = 23. Multiply 101 by 11:


(a) 111110 (b) 100110 (a) 1111 (b) 1011
(c) 101110 (d) None of these (c) 1110 (d) None of these
20. 111112 – 100012 = 24. Multiply 101101 by 1101:
(a) 1010 (b) 1111
(a) 1111001001
(c) 1110 (d) None of these
(b) 1001101001
21. 1000012 – 111102 =
(a) 11 (b) 111 (c) 1001001001
(c) 10 (d) None of these (d) None of these
22. Multiply 1111 by 11: 25. Multiply 11001 by 101:
(a) 110101 (b) 101101 (a) 1111101 (b) 1110101
(c) 110100 (d) None of these (c) 1011101 (d) None of these

Difficulty Level-2
(Based on Memory)

1. Convert the number 1982 from bases 10 to 12. The result is: 2. Two numbers, 297B and 792B, belong to base B number
(a) 1182 system. If the first number is a factor of the second number
then the value of B is:
(b) 1912
(a) 11 (b) 12
(c) 1192 (c) 15 (d) 17
(d) 1292 (e) 19
[Based on CAT, 2000] [Based on XAT, 2014]

Answer Keys
Difficulty Level-1

1. (b) 2. (a) 3. (c) 4. (c) 5. (a) 6. (b) 7. (a) 8. (c) 9. (b) 10. (a) 11. (c) 12. (a) 13. (b)
14. (c ) 15. (a) 16. (b) 17. (a) 18. (c) 19. (b) 20. (c) 21. (a) 22. (b) 23. (a) 24. (c ) 25. (a)

Difficulty Level-2

1. (c) 2. (d)

Chapter 22.indd 4 6/15/2015 2:29:21 PM


Binary Number System 22.5

Explanatory Answers

Difficulty Level-1

1.
(b) 2 117 Remainder 5.
(a) 2 701 Remainder
2 58 1 2 350 1
2 29 0 2 175 0
2 14 1 2 87 1
2 7 0 2 43 1
2 3 1 2 21 1
1 1 2 10 1
0 1 2 5 0
2 2 1
\ The binary equivalent of decimal 117 is 1110101.
1 0
2.
(a) 0 1
2 52 Remainder
2 26 0 \ (701)10 = 10101111012.
2 13 0 6.
(b)
`2 6 1 1 0 1 0 0 1
2 3 0
25 24 23 22 21 20
1 1
Decimal equivalent
0 1
= 1 × 25 + 0 × 24 + 1 × 23 + 0 + 22 + 0 × 21 + 1 × 20
\ The binary equivalent of decimal 52 is 110100. = 25 + 23 + 1 × 20 = 32 = 32 + 8 + 1 = 41.

3.
(c) 7.
(a)
1 1 1 0 1 0 1 1 0 0 0 0 0 1 0 0 1 1
26 25 24 23 22 21 20 210 29 28 27 26 25 24 23 22 21 20
Delete the weights 23 and 21. Decimal equivalent
Adding the remaining weights, we get = 1 × 210 + 0 × 29 + 0 × 28 + 0 × 27
+ 0 × 26 + 0 × 25 + 1 × 24 + 0 × 23
26 + 25 + 24 + 22 + 20
+ 0 × 22 + 1 × 21 + 1 × 20
= 64 + 32 + 16 + 4 + 1
= 210 + 24 + 21 + 20 = 1043.
= 117
i.e., 11101012 = 11710. 8.
(c)
1 1 1 0 1 1
4.
(c) 2 235 25 24 23 22 21 20
Remainder
Decimal equivalent
2 117 1
= 1 × 25 + 1 × 24 + 1 × 23 + 0 × 22 + 1 × 21 + 1 × 20
2 58 1
= 25 + 24 + 23 + 21 + 20 = 59.
2 29 0
2 14 9.
(b) 0101
1
2 7 0 +1001
2 3 1 1110

1 1
10.
(a) 11100
0 1
+ 11010
\ 23510 = 111010112. 110110

Chapter 22.indd 5 6/5/2015 12:58:15 PM


22.6 Chapter 22

11.
(c) 11111    Since 110012 < 1110002, so we are to subtract a
10001 lower number from a higher number.
  Making the digits equal in the number to be
1011
subtracted, we get
111011
011001
12.
(a) Complement of 011001 = 100110.
11001
Adding 100110 to 111 000, we get
11011
11111 111000
1010011 100110
[1]011110
3
Column 1 : 1 + 1 + 1 = 3; = Quotient 1, Remainder 1 [1 in the [ ] is the 1 carried over]
2
Adding 1 to the extreme right digit in 011 110, we get
Column 2: 0 + 1 + 1 + 1 (carry from first column)
3 011110
= 3;
= Quotient 1 and Remainder 1 1
2
Column 3: 0 + 0 + 1 + 1 (carry from second column) 11111

2 \ 1110002 – 110012 = 11111.
= 2; = Quotient 1 and Remainder 0
2
Column 4: 1 + 1 + 1 (carry from column 3) (c) 100012 = 24 + 1 = 17,
18.
4 11112 = 23 + 22 + 21 + 1 = 15
= 4; = Quotient 2 and Remainder 0
2    Since 11112 < 100012, we are to subtract a lower
number from a higher number.
Column 5: 1 + 1 + 1 + 2 (carry from column 4)
  Making the digits equal in the number to be
= 5, 510 = 1012.
subtracted, we get
Note: 01111
Quotient in any column is carry for next column. Complement of 01111 is 10000
Adding 10000 to 10001, we get
13.
(b) 11
111 10001
1111 10000

11111 [1]00001

111000 Adding 1 to 1 in 00001, we get
14.
(c) 111 00001
101 1
1100 00010

\ 100012 – 11112 = 10.
15.
(a)1000
1101 19.
(b) Complement of 0101112 = 101000
1111 111101
100100 Now, + 101000
16. 111
(b) [1]100101
101 Adding 1 to the extreme right digit in 100101, we get

011
100101
1111
+ 1
17.
(a) 100110
1110002 = 32 + 16 + 8 = 56
110012 = 16 + 8 + 1 = 25 \ 1111012 – 101112 = 100110.

Chapter 22.indd 6 6/5/2015 12:58:15 PM


Binary Number System 22.7

20.
(c) Complement of 100012 = 01110. 23.
(a) 101
11111 11
Now, + 01110 101
[1]01101 01101 101
+ 1 1111
01110
\ 11 1112 – 100012 = 1110. 24. (c)       101101
21.
(a) Complement of 011110 = 100001 1101
100001 101101
Now, + 100001 0000000
[1]000010 10110100
000010 101101000
+ 1 1001001001
000011
25. (a)       11001
\ 1000012 – 111102 = 11.
101
22.
(b) 1111 11001
11 000000
11 11 1100100
1 11 1 1111101
10 11 0 1

Difficulty Level-2

1. (c) 12 1982 2. (e) 297 B = 192 × 2 + 19 × 9 + 7 = 900


12 165 − 2 792 B = 192 × 7 + 19 × 9 + 2 = 2700
12 13 − 9
Clearly 900, is a factor of 2700.
1−1
The required number is 1192.

Chapter 22.indd 7 6/5/2015 12:58:16 PM


This page is intentionally left blank

FM.indd 6 6/10/2015 2:36:16 PM


CHAPTER

Series 23
Introduction 1 1 1 1
(c) , , , , ...
Now-a-days questions on series are asked in almost every 4 12 36 100
competitive examination. These questions may involve 1 1 1 1
numbers only, letters (A, B, ...) only or a combination of (d) , , , , ...
5 30 180 1080
both.
(e) x, x2, x3,x4, ... (where x is any fixed real number),
Series are all geometric series. The ratio of any term in
(a) to the preceding term is 2. The corresponding
A series is a sequence of numbers. These numbers are
1 1
called terms of the sequence. All the terms of the sequence ratios in (b), (c), (d) and (e) are –2, , and x,
are arranged according to a certain predefined rule. After 3 6
respectively.
carefully studying the given series and finding the specific
pattern in which the terms are changing, it is possible to find 3. Series of squares, cubes and so on.  Simple powers
out the next term of the series. of natural numbers (squares, cubes, etc.) or their
combinations are sometimes used to form some series.
Number series Illustration 3
1. Arithmetic Series  An arithmetic series is one in
(a) 4, 9, 16, 25, 36, ...
which the difference between any two consecutive
terms is always the same and is called the common Each term in this series is a perfect square. The square roots
difference, that is, each successive number is obtained of the terms are 2, 3, 4, 5, 6, ... . Clearly, the square roots
by adding (or subtracting) a fixed number to the of the terms of the given series are forming an arithmetic
previous number. series with common difference 1. So, the next term of the
series will be (6 + 1)2, that is, 49.
Illustration 1  Consider the series: 1, 3, 5, 7, 9, ....
(b) 1, 27, 125, 343, ...
Here, 2nd term – Ist term = 3rd term – 2nd term
Each term in this series is a perfect cube. The cube roots
= 4th term – 3rd term = ... = 2
of its terms are 1, 3, 5, 7, ... clearly, the cube roots of the
Hence, 1, 3, 5, 7, ... is an arithmetic series. terms of the given series are forming an arithmetic series
2. Geometric series  A geometric series is one in which with common difference 2.
the ratio of any two consecutive terms is always the So, the next term of the series will be 93, that is, 729.
same and is called the common ratio, that is, each 1 4 9 16
successive number is obtained by multiplying (or (c) , , , , ...
8 27 64 125
dividing) a fixed number by the previous number.
In the above series, the numerators are squares of natural
Illustration 2  The series given below: number (n), while the denominators are cubes of (n + 1).
(a) 2, 4, 8, 16, 32, ... 25
So, the next term of the series will be .
(b) 3, –6, 12, –24, 48,... 216

Chapter 23.indd 1 6/5/2015 1:05:27 PM


23.2 Chapter 23

4. Arithmetic series of second order  We know Illustration 7  The series:2, 5, 17, 65, .... is a geometrico-
that in an arithmetic series, the difference of any arithmetic series as each successive term is obtained by first
two consecutive terms is always the same. This is multiplying the previous term by 4 and then subtracting 3
arithmetic series of first order. from it.
A series in which the difference between successive terms So, the next term of the series will be (65 × 4) –3, that
themselves form an arithmetic series is called an arithmetic is, 257.
series of second order. Again, note that the differences of successive numbers
in the above series are 3, 12, 48, ... which are forming a
Illustration 4  Consider the series 1, 3, 7, 13, ... geometric series.
The difference between successive terms of the above series 8. Double series  It consists of two series combined into
are 2, 4, 6, ... which form an arithmetic series with common a single series. The alternating terms of this series
difference 2. form an independent series.
So, the next term of the series will be (13 + 8), that is, 21. Illustration 8  Consider the series:
5. Arithmetic series of third order  A series in which 1, 2, 4, 6, 7, 18, 10, 54, ....
the difference between successive terms themselves Terms at odd places of the series: 1, 4, 7, 10, .... is an
form an arithmetic series of second order is called an arithmetic series.
arithmetic series of third order. Terms at even places of the series: 2, 6, 18, 54, .... is a
geometric series.
Illustration 5  Consider the series:2, 9, 17, 28, ... So, the next term of the series will be (10 + 3), that
The difference of successive terms of the above series is 7, is, 13.
8, 11, 16, ...
Finding the wrong term in a series
The difference of successive terms of the above series is
1, 3, 5, ... which forms an arithmetic series with common In such questions, a number series is given of which all
difference 2. others except one are similar in some respect. The one
term of the sequence does not follow the same pattern as
So, the next term of the series will be (28 + 16), i.e., 44.
is followed by the others. This one is the wrong term in the
In this manner, we can construct arithmetic series of higher series. To find the wrong term in a given series we must
order. study the given series carefully and find the pattern/rule in
6. Arithmetico-Geometric series  In this series, each which the terms are changing. After that, we should find
successive term is obtained by first adding a fixed which of the terms is not changing according that pattern/
number to the previous term and then multiplying it rule. Thus, the wrong term is found.
by another fixed number. Illustration 9 Find the wrong term in the given series: 5,
10, 17, 24, 37, 50, 65
Illustration 6  The series: 1, 9, 33, 105, ... is an
Solution: The terms of the series are in the following order:
arithmetico-geometric series as each successive term
is obtained by first adding 2 to the previous term and 22 + 1, 32 + 1, 42 + 1, 52 + 1, 62 + 1, 72 + 1, 82 + 1
multiplying it by 3. Clearly, fourth term of the series, that is, 24 should be
replaced by 26 so that all the terms of the series follow a
So, the next term of the series will be (105 + 2) × 3,
particular pattern. Thus, 24 is the wrong term in the given
that is, 321.
series.
It is important to note that the differences of successive
numbers in the above series are 8, 24, 72, ... which are Finding the missing term of the series
forming a geometric series.
In such questions, a number series is given in which a blank
7. Geometrico-Arithmetic series  In this series, each space or question mark is provided in place of any one term
successive term is obtained by first multiplying (or of the series. The term at the blank space follow the same
dividing) the previous term by a fixed number and pattern as followed by other terms. We are required to find
then adding (or subtracting) another fixed number. the missing term to replace the blank space or question mark.

Chapter 23.indd 2 6/5/2015 1:05:27 PM


23.3
Series

Illustration 10  Find the missing term in the given series: 3×3=9


49, 56, 64, 72, ?, 90, 100 3 × 9 = 27
Solution: The terms of the series are in the following order
9 × 27 = 243
72, 72 + 7, 82, 82 + 8, 92, 92 + 9, 102
27 × 243 = 6561
Clearly, fifth term in place of question mark will be 92,
that is, 81. Thus, the number in place of question mark will
be 27 × 243, that is, 6561.

Some special series 4. By Use of Digit Sum


1. Series of Date or Time Consider the series,
(a) Consider the series, 11, 13, 17, 25, 32, ?
3 – 2 – 2004, 13 ­– 2 – 2004, 23 – 2 – 2004,
5 – 3 – 2004, Here, 13 = 11 + (1 + 1)
Here, each successive date differs by 10 days. 17 = 13 + (1 + 3)
Since 2004 is a leap year, 5 – 3 – 2004 should be 25 = 17 + (1 + 7)
replaced by 4 – 3 – 2004.
32 = 25 + (2 + 5)
(b) Consider the series,
That is, next number = previous number + digit
3.35, 5.00, 6.25, 7.40, 9.15, 10.40 sum of pervious number.
Here, each successive time differs by 1 hr 25 min. Thus, the number in place of question mark will
Therefore, 7.40 should be replaced by 7.50. be 32 + (3 + 2) = 37.
2. Numbers Followed by Their L.C.M. or H.C.F.
(a) Consider the series, Alpha–Numeric Series
1, 2, 3, 6, 4, 5, 6, 60, 5, 6, 7, ....? Such series involve the use of both the letters of the
alphabet as well as the numbers. It is a two-line series.
1st part 2nd part 3rd part
One line is a number series while the other line is an
1, 2, 3, 6 4, 5, 6, 60 5, 6, 7 ? alphabet series. The terms of both the series follow the
same pattern/rule. One of these two series is completely
   Here, in each part fourth number is L.C.M. of first known. We have to find the required number of the
three numbers. Thus, the number in place of question incomplete series.
mark will be 210 ( L.C.M. of 5, 6, 7).
Illustration 11 2, 7, 17, 37, 77,
(b) Consider the series,
3, a, b, c, d,
8, 4, 4, 7, 8, 1, 3, 9, 3, 2, 1, ?
1st part 2nd part 3rd part 4th part 2 7 17 37 77
8, 4, 4 7, 8, 1 3, 9, 3 2, 1 ?

Here, in each part third number is H.C.F. of first


two numbers. Thus , the number in place of question × 2 + 3 × 2 + 3 × 2 + 3 ×2+3
mark will be 1 ( H.C.F. of 2, 1,).
\ a = 3 × 2 + 3 = 9
3. Numbers Followed by Their Product
b = 9 × 2 + 3 = 21
Consider the series,
1, 3, 3, 9, 27, 243, ? c = 21 × 2 + 3 = 45

Here, 1 × 3 = 3 d = 45 × 2 + 3 = 93

Chapter 23.indd 3 6/5/2015 1:05:27 PM


23.4 Chapter 23

Practice Exercises

Difficulty Level-1
(Based on Memory)

Directions (Q. 1 and 2): In each of these questions, one term in (a) 32 (b) 60
the given number series is wrong. Find out the wrong term. (c) 68 (d) 66
1. 8, 14, 26, 48, 98, 194, 386 [Based on MAT, 2000]
(a) 14 (b) 48 11.
(c) 98 (d) 194
[Based on MAT, 2004]
2. 11, 5, 20, 12, 40, 26, 74, 54
(a) 5 (b) 20
(c) 40 (d) 26
[Based on MAT, 2004]
Directions (Q. 3 to 5): In each of the following number series, two
(a) 240 (b) 195
terms have been put within brackets. Mark your answer as
(c) 84 (d) None of these
(a) If both the bracketed terms are right.
[Based on MAT, 2001]
(b) If the first bracketed term is right and the second is wrong.
(c) If the first bracketed term is wrong and the second is right. 12.
6, 15, 36, 75, ?
(d) If both the bracketed terms are wrong. (a) 231 (b) 138
(c) 214 (d) None of these
3. 4, 6, 10, (12), 16, (14), 22. [Based on MAT, 2004] [Based on MAT, 2001]
4. 3, 10, 29, (66), (127), 218. [Based on MAT, 2004] 13.
15, 45, ?, 405
5. (2), 5, (12), 25, 41, 61. [Based on MAT, 2004] (a) 90 (b) 75
Directions (Q. 6 and 7): What number will replace the question (c) 135 (d) 51
mark (?) in these questions: [Based on MAT, 2001]
14. Find the next number:
6. 3 8 10 2 ? 1
6 56 90 2 20 0 2743, 2198, 1729, 1332,?
(a) 0 (b) 3 (a) 1015 (b) 1001
(c) 5 (d) 7 (c) 999 (d) 317
[Based on MAT, 2004] [Based on FMS (Delhi), 2004]
7. 1 2 3 2 10 12 15. Find the next number:
2 5 12 10 16 13 –1, 3, –15, 105, –945, 10395,?
1 2 1 ? 10 24
(a) 145535 (b) 135135
(a) 5 (b) 11
(c) –145535 (d) –135135
(c) 13 (d) 8
[Based on FMS (Delhi), 2004]
[Based on MAT, 2004]
16. The first four numbers below form a series. Insert the
8. The missing number in the series 8, 24, 12, 36, 18, 54, —
missing number at the end of the series from the given
is:
options:
(a) 27 (b) 108
(c) 68 (d) 72 28, 33, 31, 36,...
[Based on MAT, 2002] (a) 32 (b) 34
9.
What is the next number in the series given below? (c) 38 (d) 40
2, 5, 9, 14, 20 [Based on Narsee Manjee Inst. of Man. Studies, 2003]
(a) 25 (b) 26 17.
Find the next number:
(c) 27 (d) 28 999, 730, 511, 344, 215, ?
[Based on MAT, 2000] (a) 103 (b) 104
10.
Complete the series: (c) 125 (d) 126
10, 18, 34, ??, 130, 258 [Based on FMS (Delhi), 2003]

Chapter 23.indd 4 6/5/2015 1:05:27 PM


23.5
Series

18.
Find the next number: 27.
24:126 :: 48:?
1, 3, 15, 105, 945,? (a) 433
(b) 192
(a) 9995 (b) 10395
(c) 240
(c) 956 (d) 19395
(d) 344
[Based on FMS (Delhi), 2003]
28.
1:8 :: 27:?
Directions (Q. 19 to 21): In each of these questions, a number
(a) 37 (b) 47
series is given. After the series, a number is given followed by (a),
(b), (c), (d) and (e). You have to complete the series starting with (c) 57 (d) 64
the number given following the sequence of the given series to
29.
Find the wrong number in the series:
answer these questions:
6, 9, 15, 22, 51, 99
19. 3 10 26 83 336
1683 (a) 99 (b) 51
7 (a) (b) (c) (d) (e) (c) 22 (d) 15
What will come in place of (b)?
30.
8, 15, 36, 99, 288, ...?
(a) 32 (b) 30
(a) 368
(c) 34 (d) 36
(b) 676
(e) None of these [Based on IRMA, 2002]
(c) 855
20.
4 3 4 7 1.5
23.5 (d) 908
6 (a) (b) (c) (d) (e)
31.
4, 196, 16, 169, ? , 144, 64
What will come in place of (c) ?
(a) 21 (b) 81
(a) 8.5 (b) 9.5
(c) 36 (d) 32
(c) 8 (d) 9
(e) None of these [Based on IRMA, 2002] 32.
Find out the questioned number. 6:5:: 8:?
(a) 2 (b) 4
21.
3 4 16 75 364 1945
(c) 6 (d) 10
1 (a) (b) (c) (d) (e)
What will come in place of (c)? 33.
5, 21, 69, 213, 645, __ ?
(a) 72 (2) 63 (a) 1670
(c) 66 (d) 69 (b) 1941
(e) None of these [Based on IRMA, 2002] (c) 720
(d) 1320
22. 48, 24, 72, 36, 108, ?
(a) 115 (b) 110 34.
121, 144, 289, 324, 529, 576, __ ?
(c) 121 (d) 54 (a) 961
[Based on MAT, 2008]
(b) 841
23.
7, 9, 13, 21, 37, ? (c) 900
(a) 58 (b) 63
(d) 729
(c) 69 (d) 72
35.
14, 19, 29, 49, 89, __ ?
24.
36, 28, 24, 22, ?
(a) 139 (b) 149
(a) 18 (b) 19
(c) 159 (d) 169
(c) 21 (d) 22
36.
34, 18, 10, ?
25.
0, 4, 18, 48, ?, 180
(a) 58 (b) 68 (a) 8 (b) 5
(c) 84 (d) 100 (c) 7 (d) 6

26.
987:IHG :: 654:? 37.
9, 8, 10, 16, 11, ?, 12, 64
(a) FDE (b) FED (a) 28 (b) 36
(c) EFD (d) DEF (c) 25 (d) 32

M23_KHAT6981_C23.indd 5 6/17/2015 12:16:30 PM


23.6 Chapter 23

38.
7, 8, 18, 57, ? 48. 12 12 18 36 90 270 ?
(a) 232 (a) 945 (b) 810
(b) 228 (c) 1080 (d) 1215
(c) 234 (e) None of these
(d) 226
49. 1015 508 255 129 66.5 ? 20.875
(e) None of these
(a) 34.50
39.
7, 11, 19, 35, ? (b) 35
(a) 71 (b) 69
(c) 35.30
(c) 65 (d) 73
(d) 35.75
(e) None of these
(e) None of these
40.
5, 11, 23, ?, 95
50. 8 9 20 63 256 1285 ?
(a) 45 (b) 49
(a) 6430
(c) 47 (d) 46
(b) 7450
(e) None of these
(c) 7716
41.
17, 22, 52, 165, ?
(d) 7746
(a) 648 (b) 468
(e) None of these
(c) 334 (d) 668
(e) None of these 51. 980 484 236 112 50 ? 3.5
(a) 25 (b) 17
42.
Find the value of x in the series 2, 6, 30, 210, x, 30030, ...
(a) 2310 (b) 1890 (c) 21 (d) 29
(c) 2520 (d) 2730 (e) None of these

52.
The first three numbers in a series are – 3, 0, 3, the 10th
Directions (Q. 43 to 47): In each of these questions, one term in number in the series will be:
the given number series is wrong. Find out the wrong term.
(a) 18 (b) 21
43. 142 119 100 83 65 59 52 (c) 24 (d) 27
(a) 65 (b) 100 [Based on MAT (Sept), 2007]
(c) 59 (d) 119
(e) None of these 53.
Four different integers form an increasing AP If one of
these numbers is equal to the sum of the squares of the
44. 8 12 24 46 72 108 152 other three numbers, then the numbers are:
(a) 12 (b) 24 (a) –2, –1, 0, 1
(c) 46 (d) 72 (b) 0, 1, 2, 3
(e) None of these (c) –1, 0, 1, 2
45. 13 25 40 57 79 103 130 (d) None of these
[Based on MAT (Dec), 2006]
(a) 25 (b) 40
(c) 57 (d) 79 54.
The missing number in the series 8, 24, 12, 36, 18, 54, ...
(e) None of these is:
46. 2 10 18 54 162 486 1458 (a) 27 (b) 108
(a) 18 (b) 54 (c) 68 (d) 72
(c) 162 (d) 10 [Based on MAT (May), 2002]
(e) None of these 55.
What is the next number in the series given below?
47. 850 600 550 500 475 462.5 456.25 53, 48, 50, 50, 47
(a) 600 (b) 550 (a) 51 (b) 46
(c) 500 (d) 462.5 (c) 53 (d) 52
(e) None of these [Based on MAT (Dec), 2000]

M23_KHAT6981_C23.indd 6 6/17/2015 10:38:25 AM


23.7
Series

56.
What is the next number in the series given below? 66.
The next three terms of the series 28204492188 are:
2, 5, 9, 14, 20 (a) 299
(a) 25 (b) 26 (b) 436
(c) 27 (d) 28 (c) 380
[Based on MAT (Dec), 2000] (d) 456
57.
In the following number series there is a wrong number. [Based on XAT, 2006]
Find out that number. 67.
The next three terms of the series 137153163 are:
56, 72, 90, 108, 132 (a) 786 (b) 524
(a) 72 (b) 132 (c) 127 (d) 611
(c) 108 (d) None of these [Based on XAT, 2006]
[Based on MAT, 1999] 68.
The next three terms of the series 13102164129 are:
58.
What should come in place of the question mark in the (a) 778 (b) 612
following series? (c) 542 (d) 388
0, 1, ?, 64, 325 [Based on XAT, 2006]
(a) 15 (b) 12 69.
The next three terms of the series 151620294 are:
(c) 36 (d) 32 (a) 436
[Based on MAT, 1999] (b) 570
59.
The next number of the sequence 5, 25, 50, 250, 500, ... is: (c) 287
(a) 2500 (b) 1250 (d) 698
[Based on XAT, 2006]
(c) 1000 (d) 250
[Based on MAT, 1999] Directions (Q. 70 to 79): In each of these questions, one term in
the given number series is wrong. Find out the wrong term.
60.
What is the next number in the series given be­low?
70. 484 240 120 57 26.5 11.25 3.625
2, 5, 9, 14, 20
(a) 240
(a) 25 (b) 26
(b) 120
(c) 27 (d) 28
(c) 57
[Based on MAT, 2000]
(d) 26.5
61.
What is the next number in the series given below?
(e) 11.25
53, 48, 50, 50, 47
71. 3 5 13 43 176 891 5353
(a) 51 (b) 46
(a) 5 (b) 13
(c) 53 (d) 52
[Based on MAT, 2000] (c) 43 (d) 176
(e) 891
62.
Insert the missing number:
4, – 8, 16, – 32, 64, (...) 72. 6 7 16 41 90 154 292
(a) 128 (b) –128 (a) 7 (b) 16
(c) 192 (d) –192 (c) 41 (d) 90
[Based on FMS (MS), 2006] (e) 154
63.
The next 3 terms of the series 121143165 are: 73. 5 7 16 57 244 1245 7506
(a) 789 (b) 246 (a) 7 (b) 16
(c) 187 (d) 209 (c) 57 (d) 244
[Based on XAT, 2006] (e) 1245
64.
The next three terms of the series 814265098 are: 74. 4 2.5 3.5 6.5 15.5 41.25 126.75
(a) 386 (b) 194 (a) 2.5 (b) 3.5
(c) 943 (d) 650 (c) 6.5 (d) 15.5
[Based on XAT, 2006] (e) 41.25
65.
The next three terms of the series 813213246 are: 75. 32 34 37 46 62 87 123
(a) 74 (b) 98 (a) 34 (b) 37
(c) 54 (d) 63 (c) 62 (d) 87
[Based on XAT, 2006] (e) 46

M23_KHAT6981_C23.indd 7 6/17/2015 10:38:26 AM


23.8 Chapter 23

76. 7 18 40 106 183 282 403 85. 7 20 46 98 202?


(a) 18 (b) 282 (a) 420
(c) 40 (d) 106 (b) 410
(e) 183 (c) 310
77. 850 843 829 808 788 745 703 (d) 320
(a) 843 (e) None of these
(b) 829 86. 210 209 213 186 202?
(c) 808 (a) 138
(d) 788 (b) 77
(e) 745 (c) 177
78. 33 321 465 537 590 600 (d) 327
(a) 321 (e) None of these
(b) 465 87. 27 38 71 126 203?
(c) 573 (a) 212 (b) 202
(d) 537 (c) 301 (d) 312
(e) 590 (e) None of these
79. 37 47 52 67 87 112 142 88. 435 354 282 219 165?
(a) 47 (b) 52 (a) 103
(c) 67 (d) 87 (b) 112
(e) 112 (c) 120
80. 586 587 586 581 570? 522 (d) 130
(a) 545 (b) 543 (e) None of these
(c) 551 (d) 557 89. 4 200 369 513 634?
(e) None of these (a) 788
81. 64 54 69 49 74 44 ? (b) 715
(a) 89 (b) 69 (c) 734
(c) 59 (d) 99 (d) 755
(e) None of these (e) None of these

82. 4000 2008 1012? 265 140.5 78.25 90.


8 11 17 47 128 371 1100
(a) 506 (a) 11 (b) 47
(b) 514 (c) 17 (d) 371
(c) 520 (e) 128
(d) 512 91.
1 5 13 31 61 125 253
(e) None of these (a) 1 (b) 5
(c) 31 (d) 61
83. 5 5 15 75? 4725 51975
(e) 125
(a) 520
(b) 450 92. 325 314 288 247 191?
(c) 525 (a) 126 (b) 116
(d) 300 (c) 130 (d) 120
(e) None of these (e) None of these
84. 52 26 26 39 78? 585 93. 45 46 70 141? 1061.5
(a) 195 (b) 156 (a) 353 (b) 353.5
(c) 234 (d) 117 (c) 352.5 (d) 352
(e) None of these (e) None of these

M23_KHAT6981_C23.indd 8 6/17/2015 10:38:26 AM


23.9
Series

94. 620 632 608 644 596? 105. 96 48 72 180 ? 2835


(a) 536 (b) 556 (a) 915 (b) 584
(c) 656 (d) 646 (c) 630 (d) 762
(e) None of these (e) None of these [Based on IRMA, 2006]
95. 15 25 40 65? 170 106. 83 71 154 ? 379 604
(a) 115 (b) 90 (a) 242
(c) 105 (d) 120 (b) 205
(e) None of these (c) 308
96. 120 320? 2070 5195 13007.5 (d) 196
(a) 800 (b) 920 (e) None of these [Based on IRMA, 2006]
(c) 850 (d) 900 107. 3 52 88 113 129 ?
(e) None of these (a) 128 (b) 142
97. 51 26 29 42 86 217.5 (c) 133 (d) 145
(a) 29 (b) 217.5 (d) None of these
(c) 86 (d) 42 108. 2 3 8 ? 112 565
(e) None of these [Based on IRMA, 2005] (a) 36 (b) 14
(c) 27 (d) 45
98. 4 7 24 196 5488 1075648
(d) None of these
(a) 24
(b) 1075648 109. 6 4 8 23 ? 385.25
(c) 196 (a) 84.5
(d) 5488 (b) 73
(e) None of these [Based on IRMA, 2005] (c) 78.5
(d) 82
99. 288 432 648 972 1456 2187 (e) None of these
(a) 2187 (b) 648 110. 8 64 216 512 ? 1728
(c) 1456 (d) 972 (a) 729
(e) None of these [Based on IRMA, 2005]
(b) 1331
100. 12 35 136 677 4010 28045 (c) 684
(a) 4010 (b) 677 (d) 1000
(c) 136 (d) 28045 (e) None of these
(e) None of these [Based on IRMA, 2005] 111. 5 11 32 108 444 ?
(a) 1780
101. 93 309 434 498 521 533
(b) 2230
(a) 533 (b) 498
(c) 434 (d) 309 (c) 1784
(e) None of these [Based on IRMA, 2005]
(d) 2225
(e) None of these
102. 33 376 ? 717 781 808 112. 9 11 15 ? 39 71
(a) 486 (b) 621
(a) 29 (b) 23
(c) 493 (d) 592
(c) 21 (d) 27
(e) None of these [Based on IRMA, 2006]
(e) None of these
103. 11 20 57 244 1115 ? 113. 7 8 12 21 ? 62
(a) 4371 (b) 6684 (a) 42 (b) 51
(c) 3246 (d) 5148 (c) 48 (d) 35
(e) None of these [Based on IRMA, 2006] (e) None of these
104. 8 96 515 2084 6267 ? 114. 5 6 16 57 244 ?
(a) 12542 (b) 9863 (a) 1225 (b) 992
(c) 10495 (d) 8916 (c) 964 (d) 1245
(e) None of these [Based on IRMA, 2006] (e) None of these

M23_KHAT6981_C23.indd 9 6/17/2015 10:38:26 AM


23.10 Chapter 23

115. 3 19 97 391 ? 2359 125. 7 13 ? 49 97


(a) 1084 (a) 27 (b) 25
(b) 1567 (c) 23 (d) 29
(c) 1177 (e) None of these
(d) 1958
126. 5 3 6 ? 64.75
(e) None of these
(a) 15 (b) 15.5
116. 848 422 208 100 45 ? (c) 17.5 (d) 17.25
(a) 16.5 (b) 18 (e) None of these
(c) 22.5 (d) 24
127. 12 12 18 45 180 1170?
(e) None of these
(a) 12285
117. 7.5 47.5 87.5 157.5 247.5 357.5 487.5
(b) 10530
(a) 357.5 (b) 87.5
(c) 11700
(c) 157.5 (d) 7.5
(d) 12870
(e) 47.5
(e) 7605
118. 1500 1581 1664 1749 1833 1925 2016 128. 444 467 513 582 674 789?
(a) 1581 (a) 950 (b) 904
(b) 1664 (c) 927 (d) 881
(c) 1833
(e) 973
(d) 1925
129. 1 16 81 256 625 1296?
(e) 1749
(a) 4096
119. 1331 2197 3375 4914 6859 9261 12167 (b) 2401
(a) 4914 (b) 6859 (c) 1764
(c) 9261 (d) 2197 (d) 3136
(e) 12167 (e) 6561
120. 13 16 21 27 39 52 69 130. 23 25 53 163 657 3291?
(a) 21 (b) 39 (a) 16461
(c) 27 (d) 52 (b) 13169
(e) 16 (c) 9877
121. 66 91 120 153 190 233 276 (d) 23045
(a) 120 (b) 233 (e) 19753
(c) 153 (d) 276 131. 13 13 65 585 7605 129285?
(e) 190
(a) 2456415
122. 2 8 26 ? 242 (b) 2235675
(a) 78 (b) 72 (c) 2980565
(c) 82 (d) 84 (d) 2714985
(e) None of these (e) 2197845

123. 3 4 12 ? 196 132. 649.6875 1299.375 866.25 346.5 99 22?


(a) 45 (b) 40 (a) 4 (b) 7
(c) 41 (d) 49 (c) 10 (d) 12
(e) None of these (e) None of these
124. 9 17 ? 65 129 133. 30 16 10 8 8 9?
(a) 32 (b) 24 (a) 12.75 (b) 13
(c) 35 (d) 33 (c) 14 (d) 10.5
(e) None of these (e) None of theses

M23_KHAT6981_C23.indd 10 6/17/2015 10:38:26 AM


23.11
Series I
134. 10 18 63 253 1137 5901? 143. 3 7 16 32 56 93 142
(a) 39754 (a) 56 (b) 16
(b) 35749 (c) 32 (d) 7
(c) 37594 (e) None of these
(d) 35794
144. 11 18 29 42 59 80 101
(e) None of these
(a) 42 (b) 18
135. 11 26 58 124 258 528?
(c) 29 (d) 59
(a) 1020
(e) None of these
(b) 1135
(c) 1285 145. 2 9 32 105 436 2159 13182
(d) 1340 (a) 436 (b) 2195
(e) None of these (c) 9 (d) 32
(e) None of these
136. 738 765 819 900 1008 1143?
(a) 1445 (b) 1565 146. 5 5 495 3465 17325 34650 51975
(c) 1305 (d) 1275 (a) 495
(e) None of these (b) 34650
137. 9050 5675 3478 2147 1418 1077 950 (c) 55
(a) 3478 (d) 17325
(b) 1418 (e) None of these
(c) 5675 147. 17 52 158 477 ? 4310
(d) 2147 (a) 1433
(e) 1077 (b) 1432
138. 7 12 40 222 1742 17390 208608 (c) 1435
(a) 7 (b) 12 (d) 1434
(c) 40 (d) 1742 (e) None of these
(e) 208608 148. 3 22 ? 673 2696 8093
139. 6 91 584 2935 11756 35277 70558 (a) 133
(a) 91 (b) 70558 (b) 155
(c) 584 (d) 2935 (c) 156
(e) 35277 (d) 134
140. 1 4 25 256 3125 46656 823543 (e) None of these
(a) 3125
149. 6 13 38 ? 532 2675
(b) 823543
(a) 129
(c) 46656
(b) 123
(d) 25
(c) 172
(e) 256
(d) 164
141. 8424 4212 2106 1051 526.5 263.25 131.625
(e) None of these
(a) 131.625
(b) 1051 150. 286 142 ? 34 16 7
(c) 4212 (a) 66 (b) 72
(d) 8424 (c) 64 (d) 74
(e) 263.25 (e) None of these
142. 4 5 12 38 160 805 4836 151. 17 9 ? 16.5 35 90
(a) 12 (b) 160 (a) 5 (b) 15
(c) 38 (d) 805 (c) 10 (d) 20
(e) None of these (e) None of these

M23_KHAT6981_C23.indd 11 6/17/2015 12:46:44 PM


23.12 I Chapter 23

152. 0 5 18 43 84 145 ? 161. 282 286 302 ? 402 502


(a) 220 (b) 240 (a) 366
(c) 260 (d) 280 (b) 318
(e) None of these (c) 326
153. 10 17 48 165 688 3475 ? (d) 338
(a) 27584 (e) None of these
(b) 25670 162. 2187 729 243 81 27 9 ?
(c) 21369 (a) 36 (b) 3
(d) 20892 (c) 18 (d) 6
(e) None of these (e) 12
154. 1 3 24 360 8640 302400 ? 163. 522 1235 2661 4800 7652 11217 ?
(a) 14525100 (a) 15495
(b) 154152000 (b) 16208
(c) 14515200 (c) 14782
(d) 15425100 (d) 16921
(e) None of these (e) 14069
155. 12 14 32 102 416 2090 ? 164. 51975 9450 2100 600 240 160 ?
(a) 15522 (a) 80 (b) 120
(b) 12552 (c) 320 (d) 240
(c) 13525 (e) 300
(d) 17552 165. 4 18 48 100 180 294 ?
(e) None of these (a) 416
156. 10 15 15 12.5 9.375 6.5625 ? (b) 480
(a) 4.375 (c) 512
(b) 3.2375 (d) 384
(c) 4.6275 (e) 448
(d) 3.575 166. 6 26 134 666 3334 16666 ?
(e) None of these (a) 84344
157. 15 25 40 130 ? 2560 (b) 83443
(a) 500 (b) 520 (c) 84434
(c) 490 (d) 480 (d) 83334
(e) None of these (e) 83344
158. 186 94 48 25 ? 7.75 167. 30 35 65 100 165 265 ?
(a) 13.5 (b) 14.8 (a) 270 (b) 520
(c) 12.5 (d) 14 (c) 430 (d) 395
(e) None of these (e) None of these
159. 124 112 176 420 1488 ? 168. 3 5 7 ? 13 17
(a) 8568 (b) 7140 (a) 9 (b) 10
(c) 5712 (d) 6150 (c) 11 (d) 8
(e) None of these (e) None of these
160. 384 381 372 345 264 ? 169. 16 17 15 18 14 ?
(a) 23 (b) 25 (a) 10 (b) 17
(c) 43 (d) 24 (c) 18 (4) 20
(e) None of these (e) None of these

M23_KHAT6981_C23.indd 12 6/17/2015 10:38:26 AM


23.13
Series I
170. 3125 256 ? 4 1 179. 18 27 49 84 132 (?)
(a) 27 (b) 128 (a) 190 (b) 183
(c) 64 (d) 32 (c) 180 (d) 193
(e) None of these (e) None of these
171. 2 3 6 18 108 ? 180. 33 43 65 99 145 (?)
(a) 126 (a) 201
(b) 1944 (b) 203
(c) 648 (c) 205
(d) 756 (d) 211
(e) None of these (e) None of these
172. 9 15 27 51 99? 181. 655 439 314 250 223 (?)
(a) 165 (a) 205
(b) 195 (b) 210
(c) 180 (c) 195
(d) 190 (d) 190
(e) None of these (e) None of these
173. 13 21 36 58 87 ? 182. 15 21 39 77 143 (?)
(a) 122 (a) 243
(b) 128 (b) 240
(c) 133 (c) 253
(d) 123 (d) 245
(e) None of these (e) None of these
174. 7 9 19 45 95 ? 183. 33 39 57 87 129 (?)
(a) 150 (b) 160 (a) 183
(c) 145 (d) 177 (b) 177
(e) None of these (c) 189
175. 14 15 23 32 96 ? (d) 199
(a) 121 (b) 124 (e) None of these
(c) 152 (d) 111 184. 15 19 83 119 631 (?)
(e) None of these (a) 731 (b) 693
176. 20 24 36 56 84 ? (c) 712 (d) 683
(a) 116 (b) 124 (e) None of these
(c) 120 (d) 128 185. 19 26 40 68 124 (?)
(e) None of these (a) 246 (b) 238
177. 117 389 525 593 627 (?) (c) 236 (d) 256
(a) 654 (e) None of these
(b) 640 186. 43 69 58 84 73 (?)
(c) 634 (a) 62 (b) 98
(d) 630 (c) 109 (d) 63
(e) None of these (e) None of these
178. 7 11 23 51 103 (?) 187. 2.5 4 ? 10 14.5 20 26.5
(a) 186 (b) 188 (a) 8 (b) 7.5
(c) 185 (d) 187 (c) 6 (d) 5.5
(e) None of these (e) None of these

M23_KHAT6981_C23.indd 13 6/17/2015 10:38:26 AM


23.14 I Chapter 23

188. 4 5 12 39 160 805 ? 196. 85 43 44 67.5 137 345


(a) 4836 125 (a) (b) (c) (d) (e)
(b) 3224 What will come in place of (c)?
(c) 5642 (a) 86 (b) 107.5
(d) 4030 (c) 112.5 (d) 97.5
(e) None of these (e) None of these
189. 8 108 189 253 302 ? 363 [Based on IRMA, 2009]
(a) 351 197. Consider the series (1), (2, 3) (4, 5, 6), (7, 8, 9, 10), …
(b) 327 Find the sum of numbers in fiftieth such bracket.
(c) 338 (a) 62525 (b) 1225
(d) 311 (c) 12525 (d) 13225
(e) None of these [Based on ATMA, 2005]

190. 248 217 188 165 ? 129 116 198. 1 7 49 343 (?)
(a) 144 (b) 136 (a) 16807 (b) 1227
(c) 134 (d) 146 (c) 2058 (d) 2401
(e) None of these (e) None of these
191. 3 15 39 75 123 183 ? 199. 13 20 39 78 145 (?)
(a) 255 (b) 218 (a) 234 (b) 244
(c) 243 (d) 225 (c) 236 (d) 248
(e) None of these (e) None of these
192. 12 30 120 460 1368 2730 200. 12 35 81 173 357 (?)
16 (a) (b) (c) (d) (e)
(a) 725 (b) 715
What will come in place of (d)?
(c) 726 (d) 736
(a) 1384 (b) 2642
(e) None of these
(c) 2808 (d) 1988
(e) None of these 201. 3 100 297 594 991 (?)
[Based on IRMA, 2009] (a) 1489
193. 154 462 231 693 346.5 1039.5 (b) 1479
276 (a) (b) (c) (d) (e) (c) 1478
What will come in place of (e)? (d) 1498
(a) 1746 (b) 621 (e) None of these
(c) 1242 (d) 983 202. 112 119 140 175 224 (?)
(e) None of these (a) 277 (b) 276
[Based on IRMA, 2009]
(c) 287 (d) 266
194. 7 91 1001 7007 35035 105105 (e) None of these
14.5 (a) (b) (c) (d) (e)
What will come in place of (c)? 203. 21 27 48 75 123 198 323
(a) 21132.5 (b) 14514.5 (a) 198 (b) 323
(c) 20020.5 (d) 13864.5 (c) 75 (d) 27
(e) None of these (e) None of these
[Based on IRMA, 2009] [Based on NMAT, 2008]

195. 582 574 601 537 662 446 204. 32 52 87 146.5 245.75 403.125 653.6875
2004 (a) (b) (c) (d) (e) (a) 52
What will come in place of (d)? (b) 87
(a) 2084 (b) 68 (c) 245.75
(c) 174 (d) 331 (d) 403.125
(e) None of these (e) None of these
[Based on IRMA, 2009] [Based on NMAT, 2008]

M23_KHAT6981_C23.indd 14 6/17/2015 10:38:27 AM


23.15
Series I
205. 16 17 32 99 392 1960 11784 214. 39 52 78 117 169 (?)
(a) 17 (b) 99 (a) 246
(c) 11784 (d) 1960 (b) 182
(c) 234
(e) None of these
(d) 256
[Based on NMAT, 2008]
(e) None of these
206. 5 9 18 34 60 95 144
215. 656 432 320 264 236 (?)
(a) 144 (b) 95 (a) 222
(c) 18 (d) 9 (b) 229
(e) None of these (c) 232
[Based on NMAT, 2008] (d) 223
207. 36 54 81 120.5 182.25 273.375 410.0625 (e) None of these
(a) 54 (b) 81 216. 62 87 187 412 812 (?)
(c) 120.5 (d) 273.375 (a) 1012
(e) None of these (b) 1437
[Based on NMAT, 2008] (c) 1337
208. 4 10 40 190 940 ? 23440 (d) 1457
(a) 4690 (e) None of these
(b) 2930 217. 7 8 24 105 361 (?)
(c) 5140 (a) 986
(d) 3680 (b) 617
(e) None of these (c) 486
(d) 1657
209. 4000 2008 1012 ? 265 140.5 78.25
(e) None of these
(a) 506
218. 9 62 ? 1854 7415 22244
(b) 514
(a) 433 (b) 309
(c) 520
(c) 406 (d) 371
(d) 512
(e) None of these
(e) None of these
219. 4 8 24 60 ? 224
210. 7 4 5 9 ? 52.5 160.5 (a) 178 (b) 96
(a) 32 (b) 16 (c) 109 (d) 141
(c) 14 (d) 20 (e) None of these
(e) None of these 220. 8000 1600 320 64 12.8 ?
211. 5 54 90 115 131 140 ? (a) 2.56 (b) 3.5
(a) 149 (b) 146 (c) 3.2 (d) 2.98
(c) 142 (d) 152 (e) None of these
(e) None of these 221. 6 9 15 27 51 ?
(a) 84 (b) 99
212. 6 42 ? 1260 5040 15120 30240
(c) 123 (d) 75
(a) 546
(e) None of these
(b) 424
222. 7 8 18 ? 232 1165
(c) 252
(a) 84 (b) 42
(d) 328
(c) 57 (d) 36
(e) None of these
(e) None of these
213. 13 16 22 33 51 (?) 223. 9 19 40 83 ? 345 696
(a) 89 (b) 78 (a) 162 (b) 170
(c) 102 (d) 69 (c) 175 (d) 166
(e) None of these (e) None of these

M23_KHAT6981_C23.indd 15 6/17/2015 10:38:27 AM


23.16 Chapter 23

Answer Keys
Difficulty Level-1

  1. (b) 2.  (c) 3.  (b) 4.  (b) 5.  (d) 6.  (c) 7.  (c) 8.  (a) 9.  (c) 10.  (d) 11.  (b) 12.  (b) 13.  (c)
  14.  (c) 15.  (d) 16.  (a) 17.  (d) 18.  (b) 19.  (c) 20.  (a) 21.  (b) 22.  (e) 23.  (c) 24.  (c) 25.  (d) 26.  (b)
  27.  (d) 28.  (d) 29.  (c) 30.  (c) 31.  (c) 32.  (c) 33.  (b) 34.  (d) 35.  (d) 36.  (d) 37.  (d) 38.  (a) 39.  (e)
  40.  (c) 41.  (d) 42.  (a) 43.  (a) 44.  (c) 45.  (c) 46.  (d) 47.  (a) 48.  (a) 49.  (d) 50.  (c) 51.  (e) 52.  (c)
  53.  (c) 54.  (a) 55.  (d) 56.  (c) 57.  (c) 58.  (a) 59.  (a) 60.  (c) 61.  (d) 62.  (b) 63.  (c) 64.  (b) 65.  (d)
  66.  (c) 67.  (c) 68.  (d) 69.  (b) 70.  (b) 71.  (d) 72.  (e) 73.  (a) 74.  (c) 75.  (a) 76.  (c) 77.  (d) 78.  (e)
  79.  (a) 80.  (c) 81.  (e) 82.  (b) 83.  (c) 84.  (a) 85.  (b) 86.  (b) 87.  (e) 88.  (c) 89.  (c) 90.  (c) 91.  (c)
  92.  (d) 93.  (b) 94.  (c) 95.  (c) 96.  (e) 97.  (a) 98.  (a) 99.  (c) 100.  (b) 101.  (e) 102.  (d) 103.  (b) 104.  (a)
105.  (c) 106.  (e) 107.  (e) 108.  (c) 109. (a) 110.  (d) 111.  (b) 112.  (b) 113.  (e) 114.  (d) 115.  (c) 116.  (a) 117.  (e)
118.  (c) 119.  (a) 120.  (c) 121.  (b) 122.  (e) 123.  (a) 124.  (d) 125.  (b) 126.  (c) 127.  (a) 128.  (c) 129.  (b) 130.  (e)
131.  (d) 132.  (a) 133.  (d) 134.  (b) 135.  (e) 136.  (c) 137.  (e) 138.  (d) 139.  (c) 140.  (d) 141.  (b) 142.  (c) 143.  (a)
144.  (e) 145.  (d) 146.  (b) 147.  (c) 148.  (d) 149.  (a) 150.  (e) 151.  (c) 152.  (e) 153.  (d) 154.  (c) 155.  (b) 156.  (a)
157.  (e) 158.  (a) 159.  (b) 160.  (e) 161.  (d) 162.  (b) 163.  (a) 164.  (c) 165.  (e) 166.  (d) 167.  (c) 168.  (b) 169.  (e)
170.  (a) 171.  (b) 172.  (b) 173.  (d) 174.  (d) 175.  (a) 176.  (c) 177.  (e) 178.  (d) 179.  (d) 180.  (b) 181.  (e) 182.  (e)
183.  (a) 184.  (a) 185.  (c) 186.  (e) 187.  (e) 188.  (a) 189.  (c) 190.  (d) 191.  (a) 192.  (c) 193.  (e) 194.  (a) 195.  (a)
196.  (d) 197.  (a) 198.  (d) 199.  (d) 200.  (a) 201.  (e) 202.  (c) 203.  (b) 204.  (c) 205.  (d) 206.  (e) 207.  (c) 208.  (a)
209.  (b) 210.  (d) 211.  (e) 212.  (c) 213.  (b) 214.  (c) 215.  (a) 216.  (b) 217.  (a) 218.  (d) 219.  (e) 220.  (a) 221.  (b)
222.  (c) 223.  (b)

Explanatory Answers

Difficulty Level-1

1.
(b) The sequence in the given series is + 6, + 12, + 24, + 10.
(d) The sequence in the given series is + 8, + 16, + 32,
48, + 96, + 192. Hence the number 48 must be replaced + 64, + 128.
by 50.
(b) (6 + 2 + 3 + 2)2 – 1 = 169 – 1 = 168
11.
2.
(c) Series I: 11 20 38 74
(5 + 1 + 2 + 3)2 – 1 = 121 – 1 = 120
The sequence in this series is × 2 – 2.
(3 + 5 + 4 + 2)2 – 1 = 196 – 1 = 195
Series II: 5 12 26 24
The sequence in this series is × 2 + 2. Hence the 12.
(b) The sequence in the given series is + 9, + 21, + 39,
number 40 must be replaced by 38. + 63. Therefore, the next number should be 138.
3.
(b) The sequence in the given series is + 2, + 4, + 2, + 4, 13.
(c) The sequence in the given series is × 3.
+ 2, + 4. Therefore, 14 must be replaced by 18. 14.
(c) The sequence in the given series is – 545, – 469, –
(b) The terms of the given series are 13 + 2, 23 + 2, 33 +
4. 397, – 333, because in the series 545, 469, 397, 333
2, 43 + 2, 53 + 2, 63 + 2. the sequence is – 76, – 72, – 64.
5.
(d) Here 2 must be replaced by 1 and 12 must be replaced 15.
(d) The sequence in the given series is
by 3. × (–3), × (–5) × (–7), × (–9), × (–11), × (–13).
Then, the sequence in the series will be + 4, + 8, + 16.
(a) The sequence in the given series is + 5, – 2, + 5, – 2.
12, + 16, + 20.
17.
(d) The terms of the given series are
6.
(c) Sum of the numbers in each column is a perfect
103 – 1, 93 + 1, 83 – 1, 73 + 1, 63 – 1, 53 + 1.
square.
18.
(b) The sequence in the given series is × 3, × 5, × 7, × 9,
7.
(c) Sum of the numbers in each column is a perfect
× 11.
square.
19. (c) The sequence in the given series is × 1 + 7, × 2 + 6, ×
8.
(a) The sequence in the given series is × 2 ÷ 2.
3 + 5, × 4 + 4, × 5 + 3.
9.
(c) The sequence in the given series is + 3, + 4, + 5, + 6, + 7. \ The new series formed must be 7, 14, 34, 107,...

Chapter 23.indd 16 6/5/2015 1:05:29 PM


23.17
Series

1 (c) 22 = 4, 42 = 16, 82 = 64
31.
20. (a) The sequence in the given series is × + 1, × 1 + 1, Consider the alternative term
× 1.5 + 1, × 2 + 1, × 2.5 + 1 2
22 = 4, 42 = 16, ? = ?, 82 = 64
The new series formed must be 6, 4, 5, 8.5, 18, 46.
Hence ? has to be replaced by (6)2 = 36
21. (b) The sequence in the given series is × 1 + 13, × 2 + 23,
× 3 + 33, × 4 + 43, × 5 + 53. 32.
(c) 6 – 5 = 1
\ The new series must be 1, 2, 12, 63, 316,... 8 – ? = 2
–? = 2 – 8
22.
(e) 48, 24, 72, 36, 108, ? ÷ 2, × 3 (series)
–? = – 6
48, 48 ÷ 2 = 24, 24 × 3 = 72, 72 ÷ 3 \ ? =6
= 36, 36 × 3 = 108
33.
(b) 5, 21, 69, 213, 645
108 ÷ 2 = 54
21 – 5 = 16
23.
(c) \ 16 × 3 = 48
69 – 21 = 48
and, 48 × 3 = 144
213 – 69 = 144
\ 144 × 3 = 432
24.
(c) 645 – 213 = 432
432 × 3 = 1296
Likewise,

? – 645 = 1296
25.
(d)
\ ? = 1296 + 645
? = 1941
34.
(d) 11 × 11 = 121
12 × 12 = 144
Difference = 17 – 12 = 5

17 × 17 = 289
26.
(b) 9 8 7 18 × 18 = 324
↓ ↓ ↓ Difference = 23 – 18 = 5
I H G 23 × 23 = 529

Likewise, 24 × 24 = 576
6 5 4 Likewise,
↓ ↓ ↓ ? – 24 = 5
F E D ? = 29

Hence, 29 × 29 = 729
(d) 52 – 1 = 24; 53 + 1 = 126 ::
27.
72 – 1 = 48; 73 + 1 = 344 35.
(d) 19 – 14 = 5
(d) (1)3 = 1; (2)3 = 8 ::
28. 29 – 19 = 10
(3)3 = 27; (4)3 = 64 49 – 29 = 20
29.
(c) 89 – 49 = 40
Likewise,
? – 89 = 80
? = 80 + 89
\ 22 should be replaced by 27.
? = 169
+7 +21 +63
30.
(c) 8 →15 
→ 36 
→ 36.
(d) 34, 18, 10, ?
+189 +567
34 − 18 =16 
99 
→ 288 → 855  16 ÷ 2 =
8
18 − 10 =8 
8÷2= 4
The difference between the consecutive term 10 − ? =4 
keeps on multiplying by 3.

Chapter 23.indd 17 6/5/2015 1:08:24 PM


23.18 Chapter 23

Therefore, – ? = 4 – 10 42.
(a)
⇒ = ? = –6
⇒ ? = 6

37.
(d)
43.
(a)


So, wrong number = 65
38.
(a) Correct number = 83 – 13 = 70
44.
(c)


So, wrong number = 46
Correct number = 24 × 1.5 = 36
45.
(c)

39.
(e)

So, wrong number = 57
Correct number = 40 + 18 = 58
46.
(d)


So, wrong number = 10
Correct number = 2 × 3 = 6
47.
(a)
40.
(c)


So, wrong number = 600
Correct number = 850 – 200 = 650
48.
(a)



41.
(d)
49.
(d)


50.
(c)

Chapter 23.indd 18 6/5/2015 1:05:30 PM


23.19
Series

51.
(e) 66.
(c) 2 8 20 44 92 188
2, (2 + 6), (8 + 12), (20 + 24), (44 + 48), (92 + 96)
Hence, the next term must be 188 + 192 = 380.

67.
(c) 1 3 10 21 64 129
52.
(c) a = – 3, d = 3
1, 3, 7, 15, 31, 63, …
\ T10 = a + (10 – 1) × d
T10 = – 3 + 9 × 3 = 24 1, (1 × 2) + 1, (3 × 2) + 1, (7 × 2) + 1, (15 × 2)
53.
(c) By hit and trial or common sense, + 1, (31 × 2) + 1
2 = (–l)2 + (0)2 + (1)2 Hence, the next term must be (63 × 2) + 1 = 127.

54.
(a) 8, 24, 12, 36, 18, 54 68.
(d) 13 102 164 129
1, 3, 10, 21, 64, 129, …
1, (1 × 2) + 1, (3 × 3) + 1, (10 × 2) + 1, (21 × 3)
Hence, 27 will come in the blank space.
+ 1, (64 × 2) + 1, (129 × 3) + 1
55.
(d) 53, 48, 50, 50, 47, ... Hence, the last term is (129 × 3) + 1 = 388.
The above series can be splitted into two series one
in ascending order and other in descending order 69.
(b) 15 16 20 294
15, 15 + 12, 16 + 22, 20 + 32, 29 + 42, 45 + 52
Hence, the next three terms must be 570.
Hence, 52 will be the next number.
70.
(b)
56.
(c)


Hence, the next number of the series will be 27.
57.
(c) The sequence in the given series is + 16, + 18, + 20, + Hence, wrong number is 120.
22. Hence the number 108 must be replaced by 110. 71.
(d)
58.
(a) The sequence in the given series is
× 1 + 1, × 2 + 2, × 3 +3, × 4 + 4, × 5 + 5.
59.
(a) The sequence in the given series is × 5, × 2, × 5, × 2, × 5.

60.
(c) The sequence in the given series is + 3, + 4, + 5, + 6,
Hence, wrong number is 176.
+ 7.
61.
(d) The numbers at even places form an AP, e.g., 48, 50, 72.
(e)
52, …
62.
(b) 64 × (–2) = –128
63.
(c) The given series is | 2 | 143 165
The series is of the form 121 + 143 + 165 + …
Hence, wrong number is 154.
It is in AP with common difference 22.
73.
(a)
Hence, the next term is 165 + 22 = 187
64.
(b) The given series is 8 14 26 50 98.
14 = (8 × 2) – 2
26 = (14 × 2) – 2
50 = (26 × 2) – 2 Hence wrong number is 7.
Hence, the next term must be (98 × 2) – 2 = 194.
74.
(c)
65.
(d) 8 13 21 32 46
The difference between the consecutive terms
forms a series of 5 + 8 + 11 + 14 + ...
8, (8 + 5), (13 + 8), (21 + 11), (32 + 14), ....
Hence, the next term must be 46 + 17 = 63. Hence wrong number is 6.5.

Chapter 23.indd 19 6/5/2015 1:05:31 PM


23.20 Chapter 23

75.
(a) 85.
(b)


So, the wrong number is 34 which must be 33. 86.
(b)

76.
(c)


87.
(e)
So, the wrong number is 40 which must be 51.

77.
(d)
88.
(c)

So, the wrong number is 788 which must be 780.

78.
(e)
89.
(c)


So, the wrong number is 590 which must be 591.

79.
(a)
90.
(c)


So, the wrong number is 47 which must be 42.
 Right number = 11 + 32
80.
(c)
= 11 + 9 = 20
Wrong number is 17.
91.
(c)

81.
(e)

Wrong number is 31.
82.
(b) Right number = 13 + 24 = 13 + 16 = 29.
92.
(d)


83.
(c)
93.
(b)


84.
(a)
94.
(c)

Chapter 23.indd 20 6/5/2015 1:05:32 PM


23.21
Series

95.
(c) 103.
(b) 11 × 2 – 2 = 20
20 × 3 – 3 = 57
57 × 4 – 4 = 224
224 × 5 – 5 = 1115
96.
(e) 1115 × 6 – 6 = 6690 – 6 = 6684

104.
(a) 8 × 6 + 48 = 96
96 × 5 + 35 = 515

1 1 515 × 4 + 24 = 2084
97.
(a) 51 ×
+ = 26 2084 × 3 + 15 = 6267
2 2
26 × 1 + 1 = 27 6267 × 2 + 8 = 12542
27 × 1.5 + 1.5 = 42
105.
(c) 96 × 0.5 = 48
Therefore, wrong number is 29.
48 × 1.5 = 72
98.
(a) The given pattern is
72 × 2.5 = 180
4 × 7 = 28
180 × 3.5 = 630
7 × 28 = 196
28 × 196 = 5488 106.
(e) 83 + 71 = 154
196 × 5488 = 1075648 71 + 154 = 225
The wrong number is 24.
107.
(e)
99.
(c) The given pattern is
3
288 × = 432
2

3
432 × = 648 So, 138 is the answer.
2
108.
(c)
3
648 × = 972
2
3
972 × = 1458
2
So, the answer is 27.
The wrong number is 1456.
109.
(a)
100.
(b) The given pattern is
12 × 3 – 12, 35 × 4 – 22, 136 × 5 – 32,
671 × 6 – 42, 4010 × 7 – 52

Hence, the wrong number is 677.
So, the answer is 84.5.
101.
(e) The given pattern is
93 + 63 = 309 110.
(d)
3
309 + 5 = 434
434 + 43 = 498
498 + 33 = 525
525 + 23 = 533 So, the answer is 1000.
Hence, the wrong number is 521.
111.
(b)
102.
(d) 33 376 ? 717 781 808
808 – 781 = 27 = 33
781 – 717 = 64 = 43
717 – 53 = 717 – 125 = 592 So, the answer is 2230.

Chapter 23.indd 21 6/5/2015 1:05:33 PM


23.22 Chapter 23

112.
(b) 120.
(c)


Right number = 21 + 7 = 28
So, 23 will come at the place of question mark (?).
So, wrong number = 27.
113.
(e) 121.
(b)


So, 37 will come at the place of question mark (?). Right number = 190 + 41 = 231
So, wrong number = 233.
114.
(d) 122.
(e)


So, 1245 will come at the place of question mark (?).
So, 80 will come at the place of question mark.
115.
(c) 123.
(a)



So, 1177 will come at the place of question mark (?).
Hence, the answer is 45.
116.
(a) 124.
(d)


So, 16.5 will come at the place of question mark (?). Hence, the answer is 33.
125.
(b)
117.
(e)


Hence, the answer is 25.
Right number = 7.5 + 30 = 37.5 126.
(c)
So, wrong number = 47.5.
118.
(c)

Hence, the answer is 17.5.
127.
(a)
Right number = 1836
So, wrong number = 1833.
119.
(a)
So, 12285 will come at the place of question mark.
128.
(c)

Right number = (17)3 = 4913


So, wrong number = 4914. So, 927 will come at the place of question mark.

Chapter 23.indd 22 6/5/2015 1:05:34 PM


23.23
Series

129.
(b) 1 16 81 256 625 1296 2401 17390 × 12 – 72 = 208608
↓ ↓ ↓ ↓ ↓ ↓ Hence, 1742 is wrong number.
2 4 6
(2)2 (3)4 (4)4 (5)4 (6)4 (7)4 Here, 2 = 2 × ; 8 = 4 × ; 18 = 6 ×
2 2 2
So, 2401 will come at the place of question mark.
8 10 12
130.
(e) 32 = 8 × ; 50 = 10 × ; 72 = 12 ×
2 2 2
139.
(c)

So, 19753 will come at the place of question mark.
131.
(d)
Hence, 584 is the wrong number.
140.
(d)

So, 2714985 will come at the place of question mark.
132.
(a)
Hence, 25 is the wrong number.
141.
(b)

So, 4 will come at the place of question mark.
133.
(d) Hence, 1051 is the wrong number.
142.
(c)


So, 10.5 will come at the place of question mark.
134.
(b) Hence, the wrong number is 38.
Right number = 12 × 3 + 3 = 36 + 3 = 39
143.
(a)

So, 35749 will come at the place of question mark.
135.
(e)
Hence, the wrong number is 56.
Right number = 32 + (5)2
So, 1070 will come at the place of question mark. = 32 + 25
= 57.
136.
(c)
144.
(e)


So, 1305 will come at the place of question mark.
137.
(e) Hence, the wrong number is 78.
Right number = 59 + 19 = 78.
145.
(d)

\ Hence, 1077 is wrong number.


138.
(d) 7 × 2 – 2 = 12
12 × 4 – 8 = 40 So, the wrong number is 32.
40 × 6 – 18 = 222 Right number = 9 × 2 + 2 × 6
222 × 8 – 32 = 1742 → 1744 = 18 + 12
1744 × 10 – 50 = 17390 = 30.

Chapter 23.indd 23 6/5/2015 1:05:35 PM


23.24 Chapter 23

146.
(b) 155.
(b)


So, the wrong number is 34650. 156.
(a)
Right number = 17325 × 3
= 51975.
147.
(c)

157.
(e)

? = 1435
148.
(d)
158.
(a)

? = 134
149.
(a)
159.
(b)



? = 70 160.
(e)
150.
(e)


161.
(d)
? = 129
151.
(c)

162.
(b)

? = 10
152.
(e)

So, 3 will come at the place of question mark.
163.
(a)


153.
(d) So, 15495 will come at the place of question mark.
164.
(c)

154.
(c)

So, 320 will come at the place of question mark.
165.
(e) 4 → 2 × 2
18 → 3 × 6
48 → 4 × 12

Chapter 23.indd 24 6/5/2015 1:05:36 PM


23.25
Series

100 → 5 × 20 +2 ↓
180 → 6 × 30 19 + (5)2 + 1 = 45
294 → 7 × 42 +2 ↓
448 → 8 × 56 45 + (7)2 + 1 = 95
So, 448 will come at the place of question mark. +2 ↓
95 + (9)2 + 1 = 177
166.
(d) 175.
(a)

So, 83334 will come at the place of question mark.


167.
(c) 176.
(c)



168.
(b) 177.
(e)



So, 10 will come at the place of question mark.
178.
(d)
169.
(e)



179.
(d)
So 19 will come at the place of question mark.
170.
(a)


So, 27 will come at the place of question mark. 180.
(b)
171.
(b)


So, 1944 will come at the place of question mark.
172.
(b) 181.
(e)


173.
(d) The pattern of series is

182.
(e)


174.
(d) The pattern of series is
7 + (1)2 + 1 = 9
+2 ↓
9 + (3)2 + 1 = 19

Chapter 23.indd 25 6/5/2015 1:05:37 PM


23.26 I Chapter 23

183.
(a) 460 × 3 – 3 × 4
183 1368 × 2 – 2 × 3
33 39 57 87 129 ? Thus, answer is (c).
~~~~~
+6 +18 +30 +42 +S4
193.
(e) 154 × 3 = 462
~~~~
.,' .,' 462 ÷ 2 = 231
184.
(a) 231 × 3 = 693
731 Thus, answer is (e).
15 19 83 119 631 ?
194.
(a) 7 × 13 = 91
~~~~~
+4 .... +36 H12 +100 91 × 11 = 1001
I I I I I 1001 × 7 = 7007
(2)' (0)' (6)' (')' (10)'
7007 × 5 = 35035
185.
(c) 35035 × 3 = 105105
236
Thus, answer is (a).
19 26 40 68 124 ?
~~~~~ 195.
(a) 582 – (2)3 = 574
+7 +14 +28 +56 +112
~~~~ 574 + (3)3 = 601
7'2 14)( 2 28 )(2 56 x 2
601 – (4)3 = 537
186.
(e) 537 + (5)3 = 662
662 – (6)3 = 446
+15 +15
99
I II I
43 69 58 84 73 ? Thus, answer is (a)
I II I 196.
(d) 85 × .5 + .5 = 43
+15 +1,
43 × 1 + 1 = 44
187.
(e) The sequence of the series is
2.5 4 6.5 10 14.5 20 26.5 44 × 1.5 + 1.5 = 67.5
~~~~~~ 67.5 × 2 + 2 = 137
+1.5 +2.5 +3.5 +4.5 +5.5 +6.5
Thus, answer is (d).
~~~~~
+1 +1 +1 +1 +1
197.
(a) Addition of 1, 2, 3, 4 will be 10 and 10 the last digit of
188.
(a) The sequence of the series is 4th term. What will be the 50th term?
4 5 12 39 160 805 4836
(50 + 1)
~~~~~~ 50th term = 50 – 49 = 1275 – 49 = 1226
)(1+1 )(2+2 )(3+3 ><4+4 )(5+5 )(6+6 2
189.
(c) The sequence of the series is We know that
8 108 189 253 302 338 363 n
Sn = {2a + (n − 1)d }
~~~~~~ 2
+102 +~ +g2 +72 -t# +5 2
50
190.
(d) The sequence of the series is Sn = {2 × 1276 + (50 − 1)1}
2
248 217 188 165 146 129 116
~~~~~~ Sn = 2501 × 25 = 62525
-31 -29 -23 -19 -17 -13
198.
(d) The sequence of the series is
In the above series numbers are decreasing by prime
1 7 49 343 2401
numbers.
~~~~
191.
(a) The sequence of the series is >4 >4 x7 .7

3 15 39 75 123 183 255 199.


(d) The sequence of the series is
~~~~~~
+12 +24 +36 +72 13 20 39 78 145 248
~~~~~
192.
(c) 12 × 6 – 6 × 7 +7 +l!J +39 +67 +103
~~~~
30 × 5 – 5 × 6 +12 +20 +28 +36
~~~
120 × 4 – 4 × 5
" "

Chapter 23.indd 26 6/5/2015 1:05:37 PM


Series I 23.27

200.
(a) The sequence of the series is 208.
(a)
12 35 81 173 357 725 4690
~~~~~
+23 +46 +92 +184 +368 4 10 40 190 940 1 23440
~~~~
., ., ., .,
201.
(e) 3 + 97 = 100
209.
(b) 4000 2008 1012 ? 265 140.5 78.25
+ 100 ↓
4000 – 2008 = 1992 ÷ 2 = 996
100 + 197 = 297
2008 – 1012 = 996 ÷ 2 = 498
+ 100 ↓
1012 – 514 = 498 ÷ 2 = 249
297 + 297 = 594
514 – 265 = 249 ÷ 2 = 124.5
+ 100 ↓
265 – 140.5 = 124.5 ÷ 2 = 62.25
594 + 397 = 991
210.
(d)
+ 100 ↓
20
991 + 497 = 1488
7 4 59? 52.5 160.5
202.
(c)
~~~~~~
112 119 140 175 224 287 ".S+.5 ><1+1 "LS+I.S ><2+1 ><2.S+2.5 ><3+3
~~ ~ ~ ~
211.
(e)
" '" '" +149
""
203.
(b) 144
21 27 48 75 123 198 323 5 54 90 115 131 140 1
I I

I + -->+-->
I I
+--++--+ +
I
-->
I
321 TT~TTT
(rr (6'1 (5'1 (4'1 (3)2 (2'1

204.
(c) 32, 52, 87, 146.5, 245.75, 403.125, 653.6875
Now, 32 × 1.5 + (2)2 = 52 212.
(c)
52 × 1.5 + (3)2 = 87 252
87 × 1.5 + (4)2 = 146.5 6 42 ? 1260 5040 15120 30240
146.5 × 1.5 + (5)2 = 244.75 ≠ 245.75 ~~~~~~
~ ~ ~ ~ ><3 ><2
205.
(d) 16, 17, 32, 99, 392, 1960, 11784
213.
(b)
16 × 1 + 1 = 17
17 × 2 – 2 = 32 78
32 × 3 + 3 = 99 13 16 22 33 50 (1)

99 × 4 – 4 = 392
L+U ~ l<:1!J l:<:I!J ~
L+U L+U L+U L+U
392 × 5 + 5 = 1965 ≠ 1960
214.
(c)
206.
(e) 5, 9, 18, 34, 60, 95, 144
5 + (2)2 = 9 234
9 + (3)2 = 18 39 52 78 117 159 (1)



18 + (4)2 = 34
34 + (5)2 = 59 ≠ 60
1 1 1 1 1 1
59 + (6)2 = 95
215.
(a)
207.
(c) 36, 54, 81, 120.5, 182.25, 273. 375, 410.0625
222
36
= 18 + 36 = 54 656 432 320 264 236 (?)
2 1-224+ 1-112+ ~ ~ ~

54 L+U L+U L+U L+U


= 27 + 54 = 81
2 216.
(b)
81 1437
= 40.5 + 81 = 121.5 ≠ 120.5 62 87 187 412 812 (1)
2
~ 1-100+ 1-225+ 1_+ 1-",+
121.5 L1U ~ LBU ~
= 60.75 + 121.5 = 182.25
2 ~~~

Chapter 23.indd 27 6/5/2015 1:05:38 PM


23.28 Chapter 23

217.
(a) 220.
(a)


Hence, the answer will be 2.56.
221.
(b)



218.
(d) Hence, the answer will be 99.
222.
(c)


Hence, the answer will be 371.

219.
(e) Hence, the answer will be 57.

223.
(b)


Hence, the answer will be 124.

Chapter 23.indd 28 6/5/2015 1:05:39 PM


CHAPTER

Clocks and Calendar 24


CLOCKS
The circumference of a dial of a clock (or watch) is divided (or long hand) indicates time in minutes. In an hour, the
into 60 equal parts called minute spaces. The clock has two hour hand covers 5 minutes spaces while the minute hand
hands—the hour hand and the minute hand. The hour hand covers 60 minutes spaces. Thus, in hour or 60 minutes, the
(or short hand) indicates time in hours and the minute hand minute hand gains 55 minutes spaces over the hour hand.

soMe Basic facts

1. In every hour: 3. (a) When the hands are coincident, the angle
(a) Both the hands coincide once. between them is 0°.
(b) The hands are straight (point in opposite (b) When the hands point in opposite directions,
directions) once. In this position, the hands the angle between them is 180°.
are 30 minutes spaces apart. (c) The hands are in the same straight line, when
(c) The hands are twice at right angles. In this they are coincident or opposite to each other. So,
position, the hands are 15 minutes spaces apart. the angle between the two hands is 0° or 180°.
2. The minute hand moves through 6° in each 4. The minute hand moves 12 times as fast as the
1º hour hand.
minute whereas the hour hand moves through
2 5. If a clock indicates 6.10, when the correct time
in each minute. Thus, in one minute, the minute is 6, it is said to be 10 minutes too fast. And if it

hand gains 5 than the hour hand. indicates 5.50, when the correct time is 6, it is said
2
to be 10 minutes too slow.

soMe useful sHort-cut MetHoDs

1. The two hands of the clock will be together  the minute hand gains 55 minutes spaces in 60 minutes,
60
 60 H  \ the minute hand will gain 5H minutes spaces in
between H and (H + 1) O’clock at   55
 11 
60 H
minutes past H O’clock. × 5H = minutes. Thus, the two hands of clock will
11
Explanation  60 H 
be together between H and (H + 1) O’clock at  
At H O’clock the minute hand is 5H min spaces behind the  11 
hour hand. minutes past H O’clock.

Chapter_24.indd 1 6/5/2015 6:17:05 PM


24.2 Chapter 24

Illustration 1 At what time between 5 and 6 O’clock are the


hands of a clock together? 3. The two hands of the clock will be in the same
straight line but not together between H and (H +
Solution: Here, H = 5. 1) O’clock at
60 H 60 300 3 12
\ = ×5= = 27 (5H – 30) minutes past H, when H > 6
11 11 11 11 11
3 12
\ Hands of a clock are together at 22 minutes past and, (5H + 30) minutes past H, when H < 6
11 11
5 O’clock
Illustration 3 Find at what time between 2 and 3 O’clock
2. The two hands of the clock will be at right angles will the hands of a clock be in the same straight line but not
together
12
between H and (H + 1) O’clock at (5H ± 15) Solution: Here, H = 2 < 6.
11
minutes past H O’clock. 12 12
\ (5H + 30) = (5 × 2 + 30)
11 11
Explanation 480 7
= , i.e., 43
At H O’clock, the minute hand will be 5H minutes spaces 11 11
behind the hour hand. When the two hands are at right angle, So, the hands will be in the same straight line but not
they are 15 minutes spaces apart. So there are two cases: 7
together at 43 minutes past 2 O’clock
11
Case I When the minute hand is 15 minutes spaces behind
the hour hand, to be in this position, the minute hand will
have to gain (5H – 15) minutes spaces over the hour hand. 4. Between H and (H + 1) O’clock, the two hands
12
of a clock are M minutes apart at (5H  M)
Case II When the minute hand is 15 minutes spaces ahead 11
of the hour hand, to be in this position, the minute hand will minutes past H O’clock.
have to gain (5H + 15) minutes spaces over the hour hand.
Combining the two cases, the minute hand will have to gain Explanation
(5H 15) minutes spaces over the hour hand.
At H O’clock, the two hands are 5H minutes spaces apart
Now, 55 minutes spaces are gained in 60 minutes.
\ (5H  15) minutes spaces are gained in Case I Minute hand is M minute spaces behind the hour
hand. In this case, the minute hand has to gain (5H – M)
60 12 minutes spaces over the hour hand.
(5H  15) = (5H  15) minutes
55 11
Case II Minute hand is M minute spaces ahead of
12 the hour hand. In this case, the minute hand has to gain
So, they are at right angle at (5H  15) minutes past
11 (5H + M) minute spaces over the hour hand.
H O’clock.
Combining the two cases, the minute hand has to gain
Illustration 2 At what time between 5 and 6 O’clock will (5H ± M) minute spaces over the hour hand.
the hands of a clock be at right angle?
Now, 55 minutes spaces are gained in 60 minutes
Solution: Here H = 5
\ (5H ± M) minutes spaces are gained in
12 12 10 7
\ (5H  15) = (5 × 5  15) = 10 and, 43 60 12
11 11 11 11 (5H ± M) = (5H ± M) minutes.
55 11
10
\ Hands of a clock are at right angle at 10 minutes
11 \ The hands will be M minutes apart at
7 12
past 5 and 43 minutes past 5. (5H ± M) minutes past H O’clock
11 11

Chapter_24.indd 2 6/5/2015 6:17:06 PM


Clocks and Calendar 24.3

Illustration 4 Find the time between 4 and 5 O’clock when Illustration 6 The minute hand of a clock overtakes the
the two hands of a clock are 4 minutes apart hour hand at intervals of 65 minutes. How much a day does
the clock gain or lose?
Solution: Here, H = 4 and M = 4.
12 12 Solution: Here, M = 65
\ (5H ± M) = (5 × 4 ± 4)
11 11 \ The clock gains or loses in a day by
2 5
= 26 and, 17  720  60 × 24 
11 11 =  − M  
 11  M 
2
\ The hands will be 4 minutes apart at 26
11  720  60 × 24 
5 =  − 65  
minutes past 4 and, 17 minutes past 4 O’clock  11  65 
11
5 12 × 24 1440
= × =
5. Angle between hands of a clock 11 13 143
(a) When the minute hand is behind the hour
10
hand, the angle between the two hands at M = 10 minutes
143
 M M
minutes past H O’clock = 30  H −  +
 5  2 10
degrees. Since the sign is +ve, the clock gains by 10 minutes
143
(b) When the minute hand is ahead of the hour
hand, the angle between the two hands, at
CALENDAR
M 
M minutes past H O’clock = 30  − H  In this section, we will mainly deal with finding the day of
 5 
M the week on a particular given date. The process of finding
– degree.
2 it depends upon the number of odd days, which are quite
different from the odd numbers. So, we should be familiar
Illustration 5 Find the angle between the two hands of a with odd days.
clock at 15 minutes past 4 O’clock
Odd Days
Solution: Here, H = 4 and M = 15
The days more than the complete number of weeks in a
\ The required angle
given period are called odd days.
 M M
= 30  H −  + degrees
 5  2 Ordinary Year

 15  15 An ordinary year has 365 days.


= 30  4 −  +
 5 2
Leap Year
75
= , i.e., 37.5° That year (except century) which is divisible by 4 is called
2 a leap year, whereas century is a leap year by itself when it
is divisible by 400.
6. The minute hand of a clock overtakes the hour For example, 1964, 1968, 1972, 1984, and so on, are
hand at intervals of M minutes of correct time. all leap years whereas 1986, 1990, 1994, 1998, and so on,
The clock gains or loses in a day by are not leap years.
 720  60 × 24  Further, the centuries 1200, 1600, 2000 and so on, are
=  − M   minutes all leap years as they are divisible by 400 whereas 900,
 11  M 
1300, 1500 and so on, are not leap years.

Chapter_24.indd 3 6/5/2015 6:17:06 PM


24.4 Chapter 24

soMe Basic facts

1. An ordinary year has 365 days, i.e., 52 weeks 9. The first day of a century must either be Monday,
and 1 odd day. Tuesday, Thursday or Saturday.
2. A leap year has 366 days, i.e., 52 weeks and 2 odd
days. Explanation
3. A century has 76 ordinary years and 24 leap years No. of odd days in first century = 5
\ Last day of first century is Friday
\ 100 years = 76 ordinary years + 24 leap years
No. of odd days in two centuries = 3
= 76 odd days + 24 × 2 odd days
\ Wednesday is the last day
= 124 odd days = 17 weeks + 5 days
No. of odd days in three centuries = 1
\ 100 years contain 5 odd days. \ Monday is the last day
4. 200 years contain 10 and therefore 3 odd days. No. of odd days in four centuries = 0
5. 300 years contain 15 and therefore 1 odd day. \ Last day is Sunday
6. 400 years contain (20 + 1) and therefore 1 odd day. Since the order is continually kept in successive cyles,
7. February in an ordinary year has no odd day, but the last day of a century cannot be Tuesday, Thursday or
in a leap year has one odd day. Saturday. So, the last day of a century should be either
8. Last day of a century cannot be either Tuesday, Sunday, Monday, Wednesday or Friday. Therefore, the first
Thursday or Saturday. day of a century must be either Monday, Tuesday, Thursday
or Saturday.

soMe useful sHort-cut MetHoDs

= 13 days = 1 week + 6 odd days


1. Working rule to find the day of the week on a
particular date when reference day is given: Hence, January 7, 2000 would be 6 days ahead of
Sunday, i.e., it was on Saturday
Step I Find the net number of odd days for the
period between the reference date and 2. Working Rule to find the day of the week on a
the given date (Exclude the reference day particular date when no reference day is given
but count the given date for counting the Step I Count the net number of odd days on the
number of net odd days). given date
Step II The day of the week on the particular date
Step II Write:
is equal to the number of net odd days
ahead of the reference day (if the reference Sunday for 0 odd day
day was before this date) but behind the Monday for 1 odd day
reference day (if this date was behind the Tuesday for 2 odd days
reference day).   
Illustration 7 January 11, 1997 was a Sunday. What day of Saturday for 6 odd days.
the week was on January 7, 2000?
Illustration 8 What day of the week was on June 5, 1999?
Solution: Total number of days between January 11, 1997
and January 7, 2000 Solution: June 5, 1999 means 1998 years + first five months
= (365 – 11) in 1997 + (365 days in 1998) up to May of 1999 + 5 days of June
+ (365 days in 1999) + (7 days in 2000) 1600 years have 0 odd day
= (50 weeks + 4 odd days) + (52 weeks + 1 odd day) 300 years have 1 odd day
+ (52 weeks + 1 odd day) + (7 odd days) 98 years have 24 leap years + 74 ordinary years

Chapter_24.indd 4 6/5/2015 6:17:07 PM


Clocks and Calendar 24.5

= (24 × 2) + (74 × 1) days = 16 days = 2 weeks + odd days


= 122 days = 17 weeks + 3 odd days Total number of odd days on June 5, 1999
Thus, 1998 years have 4 odd days. = (4 + 2) odd days = 6 odd days
January 1, 1999 to May 31, 1999 has Hence, June, 5 1999 was Saturday.
= (3 + 0 + 3 + 2 + 3 + 5) 2 odd days

Practice Exercises
Difficulty Level-1
(Based on Memory)

1. A watch is 1 minute slow at 1 p.m. on Tuesday and 2 minutes 9.


In an ordinary year which months begin on the same day
fast at 1 p.m. on Thursday. When did it show the correct time? of the week:
(a) 1.00 a.m. on Wednesday (a) Feb; Nov (b) Jan; Nov
(b) 5.00 a.m. on Wednesday (c) Feb; Oct (d) Jan; Sept
(c) 1.00 p.m. on Wednesday
(d) 5.00 p.m. on Wednesday 10.
If March 2, 1994 was on Wednesday, January 25, 1994
[Based on MAT, 2004] was on:
2. The minute and the hour hand of a watch meet every (a) Wednesday (b) Thursday
65 minutes. How much does the watch lose or gain time? (c) Tuesday (d) Monday
(a) 25 seconds (b) 27 seconds
11.
Calendar for 2000 will also serve for:
(c) 27.16 seconds (d) 30 seconds
[Based on MAT, 2003] (a) 2003 (b) 2006
3. February 1, 1984 was a Wednesday just like the February (c) 2007 (d) 2005
29, 1984. When will the calender show another February 12.
January 7, 1992 was Tuesday. Find the day of the week on
with a similar situation? the same date after 5 years, i.e., on January 7, 1997?
(a) 2000 (b) 1996
(a) Tuesday (b) Wednesday
(c) 2012 (d) 2004
(c) Saturday (d) Friday
4.
It was Wednesday on January 1, 2000. What would be the
day on January 1, 2001? My watch was 3 minutes slow at 5 p.m. Tuesday and it was
13.
(a) Monday (b) Wednesday 5 minutes fast at 11 p.m. Wednesday. When did it give
(c) Thursday (d) Friday correct time?
(a) Wednesday 4.15 a.m. (b) Wednesday 7.30 a.m.
5.
Out of the following four choices which does not show the
coinciding of the hour hand and minute hand? (c) Tuesday 3.45 p.m. (d) None of these
(a) 3.16.2 (b) 6.32.43 14. A clock gains 10 minutes in every 24 hrs. It is set right
(c) 9.59.05 (d) 5.27.16 on Monday at 8 a.m. What will be the correct time on the
following Wednesday, when the watch indicates 6 p.m.?
6. February 29, 2000 was a Tuesday. In this century, how
many times February 29, will fall on a Tuesday? (a) 5.36 p.m. (b) 5.40 p.m.
(a) 1 (b) 2 (c) 4.36 p.m. (d) None of these
(c) 3 (d) 4 15. If a clock takes 22 seconds to strike 12, how much time
[Based on IITTM, Gwalior, 2003] will it take to strike 6?
7. In a day how many times the minute-hand and hour-hand (a) 10 seconds (b) 12 seconds
make right angle between them? (c) 14 seconds (d) None of these
(a) 12 (b) 20 16. A clock strikes 4 taking 9 seconds. In order to strike 12 at
(c) 22 (d) 44 the same rate, the time taken is:
8. In a 366-day year, how many days occur 53 times? (a) 36 seconds (b) 27 seconds
(a) 2 (b) 1 (c) 30 seconds (d) 33 seconds
(c) 0 (d) 3 [Based on MAT (Feb), 2008]

Chapter_24.indd 5 6/5/2015 6:17:07 PM


24.6 Chapter 24

17.
The hands of a clock are 10 cm and 7 cm, respectively. A clock gains 15 minutes per day. It is set right at 12 noon.
18.
The difference between the distance traversed by their What time will the clock show at 4.00 a.m., the next day?
extremities in 3 days 5 h is:
(a) 4:10 a.m. (b) 4:15 a.m.
(a) 4552.67 cm (b) 4557.67 cm
(c) 4:30 a.m. (d) 5:00 a.m.
(c) 4555.67 cm (d) 4559.67 cm
[Based on MAT, 1998]
[Based on MAT (May), 2006]

Difficulty Level-2
(Based on Memory)

1. An astronomical clock has its dial divided into 24 (a) 9.54 a.m. (b) 9.44 p.m.
divisions instead of 12, and the small hand goes round (c) 9.46 a.m. (d) 9.44 a.m.
in 24 hrs. The large hand goes round once every hour. If 6.
My watch was 8 minutes behind at 8 p.m. on Sunday but
24th hour is noon, then when are the hands at right angles within a week at 8 p.m. on Wednesday it was 7 minutes
between 24 and 1? ahead of time. During this period at which time this watch
2 has shown the correct time?
(a) 23 minutes past 24
15 (a) Tuesday 10.24 a.m.
23 (b) Wednesday 9.16 p.m.
(b) 2 minutes past 24
15 (c) It cannot show the correct time during this period
15 (d) None of the these
(c) 15 minutes past 24
23 7.
What are the possible times when a clock shows 35º angle
11 between two hands between 3 p.m. and 4 p.m.?
(d) 15 minutes past 24
23 (a) 20 min 25 seconds (b) 25 min 20 seconds
2.
At what time are the hands of clock together between (c) 22 min 43 seconds (d) None of these
7 p.m. and 8 p.m.?
8. What is the area of the face of a clock described by its
(a) 7.45.54 (b) 7.36.27 minute hand between 9 a.m. and 9.35 a.m., if the minute
(c) 7.37.49 (d) 7.38.11 hand is 10 cm long?
3. A watch which gains uniformly, is 5 minutes slow at 1 1
(a) 157  cm2 (b) 183  cm2
8 o’clock in the morning on Sunday, and is 5 minutes 7 3
48 seconds fast at 8 p.m. on the following Sunday. When
was it correct? 2
(c) 36  cm2 (d) None of these
(a) 7:20 p.m. on Tuesday 3
(b) 9:20 p.m. on Wednesday 9. Between 5 and 6, a lady looked at her watch and mistaking
(c) 7:20 p.m. on Wednesday the hour hand for the minute hand, she thought that the
(d) 9:20 p.m. on Tuesday time was 57 minutes earlier than the correct time. The
[Based on FMS (Delhi), 2004] correct time was:
(a) 12 minutes past 5 (b) 24 minutes past 5
4.
The relative speed of minute-hand with respect to hour-
hand is: (c) 36 minutes past 5 (d) 48 minutes past 5

 1 º  11  Directions (Q. 10 and 11): Read the following information


(a)  5   per minutes (b)   degree per minutes
 2  12  carefully to answer these questions:
(c) 6° per minutes (d) Both (a) and (b)
A person had left his home at the age of about 14 years.
A clock loses 2 minutes in a hour and another clock gains
5. He remembers that the day was Monday. Since then, he has
2 minutes in every 2 hrs. Both these clocks are set been fasting on every Tuesday. Today he is celebrating his
correctly at a certain time on Sunday and both the clocks 60th birth anniversary in a five-star hotel with his friends. As
stop simultaneously on the next day with the time shown today is Tuesday, he is not taking anything except wine. At
being 9 a.m. and 10.06 a.m. What is the correct time at the end of the party, he discloses that it is his 2400th Tuesday
which they stopped? of fasting.

Chapter_24.indd 6 6/5/2015 6:17:07 PM


Clocks and Calendar I 24.7

10. Today is October 9, 2001. On which date had he left his be added to the time shown by the watch at a given time.
home? When the watch shows 9 a.m. on March 21, n equals:
(a) October 10, 1955 (b) October 9, 1955 14 1
(a) 14 (b) 14
(c) October 8, 1955 (d) None of these 23 14
101 83
11. He was born on: (c) 13 (d) 13
115 115
(a) Wednesday (b) Tuesday
[Based on FMS, 2010]
(c) Monday (d) Thursday
19.
A man on his way to dinner shortly after 6:00 p.m.
12. A watch which gains uniformly, is 5 minutes slow at observes that the hands of his watch form an angle of
8 O’clock in the morning on Sunday, and is 5 minutes 110°. Returning before 7:00 p.m. he notices that again the
48 seconds fast at 8 p.m. on the following Sunday. When hands of his watch form an angle of 110°. The number of
was it correct? minutes that he has been away is:
(a) 7.20 p.m. on Tuesday 2
(a) 36 (b) 40
(b) 9.20 p.m. on Wednesday 3
(c) 42 (d) 42.4
(c) 7.20 p.m. on Wednesday
[Based on FMS, 2010]
(d) 9.20 p.m. on Tuesday
20.
The times between 7 and 8 o’clock, correct to the nearest
13. A mechanical grandfather clock is at present showing minute, when the hands of a clock will form an angle of
7 hrs 40 minutes 6 seconds. Assuming that it loses 4 seconds 84 degrees are:
1 (a) 7:23 and 7:53 (b) 7:20 and 7:50
in every hour, what time will it show after exactly 6 hrs.
2 (c) 7:22 and 7:53 (d) 7:23 and 7:52
(a) 2 hrs 9 minutes 40 seconds [Based on FMS, 2011]
(b) 2 hrs 10 minutes 6 seconds 21.
Number of times the hands of a clock are in a straight line
(c) 14 hrs 9 minutes 34 seconds every day is:
(d) 14 h rs10 minutes 32 seconds (a) 44 (b) 24
14. Imagine that your watch was correct at noon, but then it (c) 42 (d) 22
began to lose 30 minutes each hour. It now shows 4 p.m., [Based on IIFT, 2005]
but it stopped 5 hrs ago. What is the correct time now? 22.
Three Vice Presidents (VPs) regularly visit the plant on
(a) 9.30 p.m. (b) 11 p.m. different days. Due to labour unrest, VP (HR) regularly
(c) 1 a.m. (d) 1.30 a.m. visits the plant after a gap of 2 days. VP (Operations)
regularly visits the plant after a gap 3 days. VP (Sales)
15.
Two clocks are set right at 10 a.m. One gains 20 seconds and regularly visits the plant after a gap of 5 days. The VPs
the other loses 40 seconds in 24 hrs. What will be the true time do not deviate, from their individual schedules. CEO of
when the first clock indicates 4 p.m. on the following day? the company meets the VPs when all the three VPs come
2521 1 to the plant together. CEO is on leave from January 5, to
(a) 3.59 p.m. (b) 3.31 p.m.
4321 471 January 28, 2012. Last time, CEO met the VPs on January
7 2521 3, 2012. When is the next time CEO will meet all the VPs?
(c) 3.59 p.m. (d) 3.57 p.m.
12 4321 (a) February 6, 2012 (b) February 7, 2012
16.
How much does a watch lose per day, if its hands coincide (c) February 8, 2012 (d) February 9, 2012
every 64 min? [Based on XAT, 2012]
(a) 96 minutes (b) 90 minutes 23.
At what time between 4 p.m. and 5 p.m. will the hands of
5 8 a clock coincide?
(c) 36 minutes (d) 32 minutes
11 11 9
(a) 21 minutes past 4 p.m.
[Based on FMS (MS), 2006] 11
17.
If March 1, 2006 was Wednesday, which day was it on 1
March 1, 2002? (b) 1 minutes past 4 p.m.
11
(a) Wednesday (b) Thursday
(c) Friday (d) Saturday 9
[Based on FMS, 2009]
(c) 21 minutes before 5 p.m.
11
1 1
18.
A watch loses 2 minutes per day. It is set right at 1 p.m. (d) minutes before 5 p.m.
2 11
on March 15. Let n be the positive correction, in min, to [Based on ATMA, 2008]

Chapter_24.indd 7 6/5/2015 6:17:08 PM


24.8 I Chapter 24

24.
In a clock having a circular scale of twelve hrs, when (a) 10 (b) 11
time changes from 7.45 a.m. to 7.47 a.m. by how many (c) 12 (d) 15
degrees the angle formed by the hour hand and minute [Based on XAT, 2010]
hand changes?

Answer Keys
Difficulty Level-1

1. (b) 2. (a) 3. (c) 4. (d ) 5. (c) 6. (c) 7. (c) 8. (a) 9. (a) 10. (c) 11. (d ) 12. (a) 13. (a)
14. (a) 15. (a) 16. (b) 17. (b) 18. (a)

Difficulty Level-2

1. (c) 2. (d ) 3. (c) 4. (d ) 5. (d ) 6. (a) 7. (c) 8. (b) 9. (b) 10. (a) 11. (d ) 12. (c) 13. (a)
14. (c) 15. (a) 16. (d ) 17. (c) 18. (a) 19. (b) 20. (a) 21. (a) 22. (c) 23. (a) 24. (b)­­­

Explanatory Answers

Difficulty Level-1

1. (b) Watch gains 3 minutes in 48 hrs. Therefore, after 16 7 × 4 = 28 years have to pass after 1984. Therefore it
hrs, it will show the correct time. will be the year 2012 when a February again has five
2. (a) In 60 minutes, the hour hand moves through an angle Wednesdays.
of 30º
4.
(d) Year 2000 is a leap year, hence there are two odd days
\  In 65 minutes, the hour hand will move through which will shift the whole calender of the year 2001
30 by 2 days.
an angle of × 65 = 32.5º
60
In 60 minutes, the minute hand moves through an 5. (c)
angle of 360º 6. (c) In 2028, 2056 and 2084, i.e., after every 28 years.
\  In 65 minutes, the minute hand moves through 7.
(c) Since between 2 a.m. and 3 a.m. (2 p.m. and 3 p.m.)
360 and 8 a.m. and 10 a.m. (8 p.m. and 10 p.m.) two hands
an angle of × 65 = 390º = 360º + 30º of a clock make 90º angle only 3 times in rest of the
60
each hour two hands make 90º angle 2 times.
⇒ Difference of the angles made by the hour hand
and the minute hand = 2.5 (In 65 minutes) 8. (a)
Now an angle of 30º is made by the minute hand in 9.
(a) In an ordinary year, Feb. has no odd day
5 minutes
\ Feb. and March begin on same day of week
5
\ An angle of 2.5º is made by the minute hand in Also we know that, November and March begin
30 on same day of the week.
2.5
minutes, i.e., 25 seconds.
× 2.5 minutes, i.e., 10.
(c) Number of days from January 25, 1994 to March 2,
6
1994 is
3.
(c) The month of February 1984 had five Wednesdays.
Only in a leap year this is possible. Seven leap years Jan Feb March
have to go by before this situation can occur again, 6 + 28 + 2 = 36
because in each of the leap years the 29th would
fall on a different weekday. Seven leap years means \ Number of odd days = 1

Chapter_24.indd 8 6/5/2015 6:17:08 PM


Clocks and Calendar 24.9

\ Day on January 25, 1994 is one day before the day Thus, the correct time on the following Wednesday
on March 2, 1994 will be 5.36 p.m.
But March 2, 1994 was on Wednesday 15.
(a) In order to hear 12 strikes, there are 11 intervals
\ January 25, 1994 was on Tuesday. (12 – 1) and time of each interval is uniform

11.
(d) Starting with 2000, count for number of odd days in 22
Hence, time to hear each strike is = 2 seconds
successive years till the sum is divisible by 7. 11
2000 2001 2002 2003 2004 Now, to hear six strikes, there are 6 – 1, i.e., 5 × 2 =
2 + 1 + 1 + 1 + 2 = 7 10 seconds

\ No. of odd days up to 2004 = 0 Hence, it will take 10 seconds for a clock to strike 6.

\ Calendar for 2000 will serve for 2005 also. 16.


(b) A clock strikes 4 taking 9 seconds.

12. (a) During the interval we have two leap years as 1992 9 × 12
\ Time taken to strike 12 = = 27seconds
and 1996 and it contains February of both these years 4
\ The interval has (5 + 2) = 7 odd days or 0 odd day 17.
(b) Distance traversed by the extremity of the minute-
Hence, January 7, 1997 was also Tuesday. 22
hand in one hour = 2 × × 10
13. (a) Time from 5 p.m. Tuesday to 11 p.m. Wednesday = 7
30 hrs Distance traversed by the extremity of the minute-
Clock gains 8 minutes in 30 hrs hand in 3 days and 5 hrs, i.e., in 77 hrs
30 22
\ It gains 3 minutes in × 3 hrs =2× × 10 × 77 = 22 × 220 = 4840 cm
8 7
= 11 hrs 15 minutes
\ Correct time is 11 hrs 15 minutes after 5 p.m. Distance traversed by the hour-hand in 12 hrs
= 4.15 a.m. on Wednesday. 22
= 2 × ×7=44 cm
7
14. (a) Total number of hrs from Monday at 8 a.m. to the
following Wednesday at 6 p.m. Distance traversed by the hour-hand in 77 hrs
24 × 2 + 10 = 58 hrs 44 11 × 77 847
= × 77 = = = 282.33 cm
24 hrs 10 minutes of this clock are the same as 12 3 30
24 hrs of a correct clock \ Required difference
145 = 4840 – 282.33 = 4557.67 cm
   hrs of the incorrect clock = 24 hrs of correct
6
clock 18.
(a) In 24 hrs a clock gains 15 minutes
\ In 16 hrs (12 noon to 4 a.m.) a clock gains
24 × 6
58 hrs of the incorrect clock = × 58 hrs of
145 15
= × 16 = 10 minutes
3 24
correct clock = 57 hrs of correct clock
5 \ Required time = 4:10 a.m.

Chapter_24.indd 9 6/5/2015 6:17:08 PM


24.10 Chapter 24

Difficulty Level-2

1.
(c) Here angle covered by the large hand in 60 minutes Since, second watch gains 1 min 1 hr so it must
= 360º and angle covered by the small hand in show 22 minutes extra than the correct time in 22 hrs
360° Hence, the correct time can be found by subtracting
60 minutes = = 15º. Required time when the
24 22 minutes from 10:06.
hands are at right angles between 24 and 1
6.
(a) In 72 hrs my watch gains (8 + 7) = 15 minutes.
60 To show the correct time watch must gain 8 minutes.
= × 90 minutes past 24
(360 − 15) Since the watch gains 15 minutes in 72 × 60 minutes.
60   Therefore, the watch will gain 8 minutes in
= × 90 minutes past 24
345
72 × 60 × 8
360 minutes = 72 × 60 × 8 = 38 hrs 24 minutes
= minutes past 24 15 15
23
90 − 35 55
15 7.
(c) = × 2 = 10 minutes
= 15 minutes past 24 5.5 11
23
So, the required time = 3:10:00
210 210 420 2
2.
(d) = ×2= = 38 minutes = 38 minutes 90 + 35 125 250 8
5.5 11 11 11 Again, = ×2= = 22 minutes
5.5 11 11 11
11 seconds
Therefore, required time = 7:38:11. = 22 minutes 43 seconds
3. (c) From 8 a.m on Sunday to 8 p.m on the following 7 22 7 1
Sunday, the watch will be 348 seconds fast (b) π × (10) 2 ×
8. = × 10 × 10 × = 183  cm2
12 7 12 3
\ There will be difference of 648 seconds in the
duration of 180 hrs 9.
(b)

\ Difference of 300 seconds (5 minutes), so that the 10.


(a)
watch shows the correct time will be there after 11.
(d)
180 250 1
× 300 hrs, i.e., hrs, i.e., 83 hrs, i.e., 12.
(c) From 8 a.m. on Sunday to 8 p.m. on the following
648 3 3 Sunday, the watch will be 348 seconds fast.
at 7.20 p.m. on Wednesday.
\ There will be difference of 648 seconds in the
duration of 180 hrs.
4.
(d) Both (a) and (b) are correct.
\ Difference of 300 seconds (5 minutes), so that
Relative speed = Speed of minute-hand – Speed of
the watch shows the correct time will be there after
hour-hand
180 250 1
× 300 hrs, i.e., hrs, i.e., 83 hrs, i.e., at
 1 °  1 ° 648 3 3
= 6º –   = 5  
2 2
7.20 p.m. on Wednesday.
13.
(a)
1 11
and 1 min – minutes = minutes
12 12 14.
(c)

5.
(d) Actually they create a difference of 3 minute per hour 15.
(a) From 10 a.m. to 4 p.m. on the following day = 30 hrs
and the two watches are showing a difference of 66 Now, 24 hrs 20 seconds of the first clock
minutes. Thus, they must have been corrected 22 hrs
= 24 hrs of the current clock
earlier
Now, the correct time can be found by comparing 24 × 180
\ 1 hour of the first clock = hrs
any one of the watch 4321

Chapter_24.indd 10 6/5/2015 6:17:09 PM


Clocks and Calendar 24.11

24 × 180 × 30 \ 6y – 180 – 0.5y = 110°


\ 30 hrs of the first clock = hrs
4321
\ 5.5y – 180 = 110°
24 × 180 × 30 2521 \ y ≈ 52.72 minutes ≈ 52 minutes 43 seconds
Now, hrs = 29 hrs 59 minutes
4321 4321
\ The man leaves at 06:12:43 p.m. and returns at
\ When the first clock indicates 4 p.m. on the follo­ 06:52:43 p.m.
2521
wing day the true time will be 3 hrs 59  minutes. \ He is away for 40 minutes.
4321

60 5 20.
(a) At 7 O’clock, the minute hand is 210° behind the hour
16.
(d) 60 minutes are gained in × 60 = 65 minutes hand.
55 11

But they are together after 64 minutes

5 5 16
Gain in 64 minutes = 65  64 = 1  mins
11 11 11

16  24  60 8
Gain in 24 hrs = = 32 mins.
11  64 11

17.
(c) Since in between 2002 to 2006, this is a one leap year
\ Total numbers = (2006 – 2002) + 1 = 5 The angle between the two hands will be 84° when
the minute hand gains (210° – 84°) = 126° or when it
\ Required day = Wednesday – 5 = Friday. gains (210° + 84°) = 294° with respect to hour hand.
The relative speed of the minute hand with respect to
18.
(a) When the actual time elapsed in 24 hrs = 24 × 60 = 1440
1
min, the time elapsed on the faulty watch = 1437.5 min hour hand = 5 ° per minutes.
From 1 p.m. on March 15 to 9 a.m. on March 21, the 2
time elapsed on the faulty watch = 140 hrs = 140 × 60
Thus, the time taken to gain 126° and 294° will be
= 8400 min.
\ The actual time elapsed 126 10 294 6
= = 22 minutes and, = 53 minutes
11 11 11 11
8400  1440 2 2
=
1437.5
Thus, the angle between the two hands will be 84° at
84  144  16 14 10 5
=  8414 22 minutes past 7 O’ clock and 53 minutes past
23 23 11 11
14 14 7 O’clock, i.e., approx 7:23 and, 7:53.
\ The correction n = 8414
 8400 
14 mins
23 23
21.
(a) The hands are in straight line twice in one hour
19.
(b) Let the time after 6 p.m. be x minutes.
In 12 hrs the hands of a clock are in straight line
The speed of the minute hand is 6° per minutes and (11 × 2) = 22 times.
the speed of the hour hand is 0.5° per minutes.
Hence, in one day it will be 44 times.
Initial distance between the hour and the minute
hands at 6:00 p.m. is 180°. 22.
(c) After January 3, they will be together on January 15,
January 27 and February 8, 2012
\ (180 + 0.5x) – (6x) = 110°
Since CEO is on leave up to January 28, he will
⇒ 180 – 5.5x = 110° meet the VPs on February 8, 2012
\ x ≈ 12.72 minutes ≈ 12 minutes 43 seconds
23.
(a) At 4 O’clock, the hour hand is at 4 and the minute
Let the time before 7 p.m. be y minutes hand is at 12 i.e., they are 20 minutes space apart.

Chapter_24.indd 11 6/5/2015 6:17:09 PM


24.12 Chapter 24

To be together, the minute hand have to gain An hour hand in 12 hrs make an angle of 360°
20 minutes over the hour hand
We know that 55 minutes are gained in 60 minutes 31
\ An hour hand in hrs make an angle of
4
60 240 9
\ 20 m are gained in × 20 = = 21 minutes
55 11 11  30 × 31  º
9   = 232.5′
i.e., 21 minutes past 4 p.m.  4 
11
\ Angle between hour hand and minute hand at
24.
(b) A minute hand in 60 minutes makes an angle of 360° 7:45 a.m. is 37.5. Likewise calculate for 7:47 and
\ A minute hand in 45 minutes makes an angle of calculate the difference.
(6 × 45) = 270.

Chapter_24.indd 12 6/5/2015 6:17:09 PM


CHAPTER

Polynomials 25
polynomial For example, in the polynomial 8x6 – 4x5 + 7x3 – 8x2
A function p (x) of the form + 3, the term with the highest power is x6. Hence, the degree
of the polynomial is 6.
p (x) = a0 + a1x + a2x2 + ... + anxn
A polynomial of degree 1 is called a linear polynomial.
where a0, a1, a2, ..., an are real numbers, an ≠ 0 and n is a
non-negative integer is called a polynomial in x over reals. It is of the form ax + b, a ≠ 0.

The real number a0, a1, ..., an are called the coefficients A polynomial of degree 2 is called a quadratic
of the polynomial. polynomial.

If a0, a1, a2, ..., an are all integers, we call it a It is of the form ax2 + bx + c, a ≠ 0.
polynomial over integers.
Division of a Polynomial by a Polynomial
If they are rational numbers, we call it a polynomial
over rationals. Let p (x) and f (x) be two polynomials and f (x) ≠ 0. Then, if
we can find polynomials q (x) and r (x), such that
Illustration 1
p (x) = f (x). q (x) + r (x),
(a) 4x2 + 7x – 8 is a polynomial over integers.
where degree r  (x) < degree f  (x), then we say that p  (x)
7 3 2 2 8 divided by f (x), gives q (x) as quotient and r (x) as remainder.
(b) x + x – x + 5 is a polynomial over rationals.
4 3 7 If the remainder r (x) is zero, we say that divisor f (x)
(c) 4x – 2
3x+ 5 is a polynomial over reals. is a factor of p (x) and we have
p (x) = f (x). q (x).
Monomial Illustration 2  Divide f (x) = 5x3 – 70x2 + 153x – 342 by
A polynomial having only one term is called a monomial. g (x) = x2 – 10x + 16. Find the quotient and the remainder
For example, 7, 2x, 8x3 are monomials. Solution:
5x – 20
Binomial
x2 – 10x + 16 5x3 – 70x2 + 153x – 342
A polynomial having two terms is called a binomial.
5x3 – 50x2 + 80x
For example, 2x + 3, 7x2 – 4x, x2 + 8 are binomials.
–  +   –
Trinomial   –20x2 + 73x – 342
A polynomial having three terms is called a trinomial.   –20x2 + 200x –320
For example, 7x2 – 3x + 8 is a trinomial. + –    +
–127x – 22
Degree of a Polynomial
\ Quotient = 5x – 20 and
The exponent in the term with the highest power is called
the degree of the polynomial. Remainder = –127x – 22.

Chapter_25.indd 1 6/5/2015 6:10:04 PM


25.2 Chapter 25

Illustration 3 Determine if (x – 1) is a factor of


p (x) = x3 – 3x2 + 4x + 2
soMe useful results anD forMulae

x2 – 2x + 2 1. (A + B)2 = A2 + B2 + 2AB
3 2
x–1 x – 3x + 4x + 2 2. (A – B)2 = A2 + B2 – 2AB = (A + B)2 – 4AB
x3 – x2 3. (A + B) (A – B) = A2 – B2
– + 4. (A + B)2 + (A – B)2 = 2 (A2 + B2)
– 2x2 + 4x 5. (A + B)2 – (A – B)2 = 4AB
– 2x2 + 2x 6. (A + B)3 = A3 + B3 + 3AB (A +B)
7. (A – B)3 = A3 – B3 – 3AB (A – B)
– +
8. A2 + B2 = (A + B)2 – 2AB
2x + 2
9. A3 + B3 = (A + B) (A2 + B2 – AB)
2x – 2 10. A3 – B3 = (A – B) (A2 + B2 + AB)
– + 11. (A + B + C)2 = A2 + B2 + C2 + 2 (AB + BC + CA)
4 12. (A3 + B3 + C3 –3 ABC
Since the remainder is not zero, (x – 1) is not a factor = (A + B + C) (A2 + B2 + C2 – AB – CA – BC)
of p (x). 13. A + B + C = 0 ⇒ A3 + B3 + C3 = 3ABC.
14. An – Bn is divisible by (A – B) for all values of n.
soMe Basic tHeoreMs 15. An – Bn is divisible by (A + B) only for even values
of n.
Factor Theorem 16. An + Bn is never divisible by (A – B).
Let p (x) be a polynomial of degree n > 0. If p (a) = 17. An + Bn is divisible by (A + B) only when n is odd.
0 for a real number a, then (x – a) is a factor of p (x).
Conversely, if (x – a) is a factor of p (x), then p (a)
= 0. a useful sHort-cut MetHoD

Illustration 4 Use factor theorem to determine if (x – 1) is When a polynomial f (x) is divided by x – a and
a factor of x8 – x7 + x6 – x5 + x4 – x + 1 x – b, the respective remainders are A and B. Then,
Solution: Let p (x) = x8 – x7 + x6 – x5 + x4 – x + 1 if the same polynomial is divided by (x – a) (x – b),
Then, p (1) = (1)8 – (1)7 + (1)6 – (1)5 the remainder will be
A− B Ba − Ab
+ (1)4 – 1 + 1 = 1 ≠ 0 x+ .
a −b a −b
Hence, (x( – 1) is not a factor of p(x
p(x)
( )
(x
Remainder Theorem Illustration 6 When a polynomial f (x) is divided by (x – 1)
and (x – 2), the respective remainders are 15 and 9. What is
Let p (x) be any polynomial of degree ≥ 1 and a any the remainder when it is divided by
number.
(x – 1) (x – 2)?
If p (x) is divided by x – a, the remainder is p (a).
A− B Ba − Ab
Solution: Remainder = x+
5 4 a −b a −b
Illustration 5 Let p (x) = x + 5x – 3x + 7 be divided by
(x – 1). Find the remainder 15 − 9 9(1) − 15(2)
= x+
Solution: Remainder = p (1) = (1)5 + 5 (1)4 –3 (1) + 7 1− 2 1− 2
= 10 = (–x + 21)

Chapter_25.indd 2 6/5/2015 6:10:06 PM


25.3
Polynomials

Practice Exercises

Difficulty Level-1
(Based on Memory)

1. If (a + b + 2c + 3d) (a – b – 2c + 3d) 10. Resolve into factors: 81x2y2 + 108xyz + 36z2.


= (a – b + 2c – 3d) × (a + b – 2c – 3d), (a) (6xy + 9z)2 (b) (9xy – 7z)2
then 2bc is equal to: (c) (9xy + 6z)2 (d) None of these
3 If a + b + c = 0, then the value of a2 (b + c) + b2 (c + a)
11.
(a) 3 ad (b)
2 + c2 (a + b) is:
3a (a) abc (b) 3 abc
(c) a2d2 (d)
2d (c) –3 abc (d) 0
[Based on MAT, 2003] [Based on MAT, 1999]
2.
What is the value of the following expression? If (x + 1) is a factor of 2x3 – ax2 – (2a – 3) x + 2, then the
12.
value of ‘a’ is:
(1 + x)(1 + x2)(1 + x4)(1 + x8)(1 – x)
(a) 3 (b) 2
(a) 1 + x (b) 1 – x16
16

(c) x16 – 1 (d) x8 + 1 (c) 3/2 (d) 1/2


[Based on MAT, 1999]
[Based on MAT, 2000]
x 2 − 36 x+6
13.
If P = 2 and Q = , then the value of P/Q is:
3. If x2 – 6x + a is divisible by x – 2, then a is equal to: x − 49 x +7
(a) 8 (b) 6
x−6 x−6
(c) 0 (d) None of these (a) (b)
x−7 x+7
4. If (x – 2) is a factor of the polynomial x3 – 2ax2 + ax – 1,
find the value of a. x−6 x+6
(c) (d)
(a) 5/6 (b) 7/6 x+6 x−7
(c) 11/6 (d) None of these [Based on MAT, 1999]
14.
What is the value of the following expression?
5. Divide the polynomial 4y3 – 3y2 + 2y – 4 by y + 2 and find (1 + x) (1 + x2) (1 + x4) (1 + x8) (1 – x)
the quotient and remainder.
(a) 1 + x16 (b) 1 – x16
(a) 4y2 – 11y + 24, – 52 (b) 6y2 – 13y + 36, – 64
(c) x16 – 1 (d) x8 + 1
(c) 4y2 + 13y – 24, + 52 (d) None of these
[Based on MAT, 2000]
6. Resolve into factors: 16 (x – y)2 – 9 (x + y)2.
15. Resolve into factors:
(a) (x – 5y) (5x – y) (b) (x + 7y) (7x + y) 9 (3x + 5y)2 – 12 (3x + 5y) (2x + 3y) + 4 (2x + 3y)2
(c) (x – 7y) (7x – y) (d) None of these (a) (7x + 9y)2 (b) (5x + 9y)2
2 2
7. Resolve into factors: 4x + 12xy + 9y – 8x – 12y. (c) (5x – 9y)2 (d) None of these
(a) (3x + 2y) (4x + 2y – 3) If x + 1/x = 3, then x3 + 1/x3 is equal to:
16.
(b) (2x + 3y) (2x + 3y – 4) (a) 9 (b) 18
(c) (2x – 3y) (2x + 3y + 4) (c) 27 (d) 6
(d) None of these [Based on IIFT, 2005]
17. Factorize: 45a3b + 5ab3 – 30a2b2
8. Resolve into factors:
(a) 5ab (5a – b)2 (b) 7ab (5a – b)2
16x2 – 72xy + 81y2 – 12x + 27y (c) 5ab (3a – b)2 (d) None of these
(a) (6x – 7y) (6x – 7y – 5) (b) (4x – 9y) (4x – 9y – 3)
18. Find the factors of (a – b)3 + (b – c)3 + (c – a)3
(c) (4x + 9y) (4x + 9y + 3) (d)  None of these
(a) 3(a + b) (b + c) (c + a)
9. Resolve into factors: (a + b)2 –14c (a + b) + 49c2. (b) 5(a – b) (b – c) (c – a)
(a) (a – b – 9c)3 (b) (a + b – 7c)2 (c) 3(a – b) (b – c) (c – a)
(c) (a + b + 9c)2 (d) None of these (d) None of these

Chapter_25.indd 3 6/5/2015 6:10:06 PM


25.4 Chapter 25

1 2 (a) 64 (b) 14
19. Factorize a2 + + 3 – 2a – (c) 8 (d) 2
a2 a
1 1
 1  1  22. If = 5, what will be the value of x2 + 2 ?
x +
(a)  a + − 1 a − + 1 x x
 a  a 
(a) 927 (b) 727
 1  1  (c) 527 (d) 627
(b)  a + − 1 a + + 1
 a  a 
1 1
23. If x += 3, the value of x6 + 6 is:
 1  1  x x
(c)  a + + 1 a + + 1 (a) 927 (b) 414
 a  a 
(c) 364 (d) 322
 1  1 
(d)  a + − 1 a + − 1  1  1
 a  a  24. If  x3 + 3  = 52, the value of x + is:
 x  x
1 1 (a) 4 (b) 3
20. If x += 2, find the value of x4 + 4 .
x x (c) 6 (d) 13
(a) 2 (b) 4 1 1 1
25. If x + = 1 and y + = 1, find the value of z +
(c) 6 (d) 8 y z x
1 1 (a) 2 (b) 1
21. If x + = 2, then x3 + 3 is equal to:
x x (c) 0 (d) 3

Difficulty Level-2
(Based on Memory)

1. (xn – an) is divisible by (x – a) Value of k for which (x – 1) is a factor (x3 – k), is:
5.
(a) For all values of n (a) – 1 (b) 1
(b) Only for even values of n
(c) 8 (d) – 8
(c) Only for odd values of n
[Based on FMS, 2005]
(d) Only for prime values of n
1/3 1/3 1/3
2. Which of the following expressions are exactly equal in 6.
If x y x 
0, then:
value? (a) x + y + z = 0 (b) (x + y + z)3 = 27 xyz
(a) (3x – y)2 – (5x2 – 2xy)
(c) x + y + z = 3 xyz (d) x3 + y3 + z3 = 0
(b) (2x – y)2
[Based on FMS, 2005]
(c) (2x + y)2 – 2xy
(d) (2x + 3y)2 – 8y (2x + y) If 3x3 – 9x2 + kx – 12 is divisible by x – 3, then it is also
7.
divisible by:
(a) (a) and (b) only (b) (a), (b) and (c) only
(a) 3x2 – 4 (b) 3x2 + 4
(c) (b) and (d) only (d) (a), (b) and (d) only
(c) 3x – 4 (d) 3x + 4
[Based on IRMA, 2002]
[Based on FMS, 2010]
3. Find the values of m and n in the polynomial 2x3 + mx2 +
nx – 14 such that (x – 1) and (x + 2) are its factors. If the expression ax2 + bx + c is equal to 4 when
8.
(a) m = 4, n = 5 (b) m = 9, n = 3 x = 0, leaves a remainder 4 when divided by x + 1 and a
(c) m = 6, n = 7 (d) None of these remainder 6 when divided by x + 2, then the values of a, b
and c are respectively:
4. Factorize (a – b + c)2 + (b – c + a)2 + 2(a – b + c)
(b + c – a). (a) 1, 1, 4 (b) 2, 2, 4
(a) 4a2 (b) 6a2 (c) 3, 3, 4 (d) 4, 4, 4
(c) 8a2 (d) None of these [Based on XAT, 2006]

Chapter_25.indd 4 6/5/2015 6:10:07 PM


25.5
Polynomials

The condition that x5 + 10x4 – 7x3 + 10ax + 5a2 will


9. 18. If x + y + z = 9 and xy + yz + zx = 23, the value of x3 + y3
contain x + 1 as a factor is: + z3 – 3xyz is:
(a) a = −169 (b) a = –2 (a) 108 (b) 207
2 (c) 669 (d) 729
(c) 5a – 10a + 16 = 0 (d) 5a2 – 10a – 16 = 0
[Based on XAT, 2006] 19. When (x3 – 2x2 + px – q) is divided by x2 – 2x – 3 the
remainder is (x – 6). The values of p and q are:
If x3 + 2x2 + ax + b is exactly divisible by x2 – 1, then the
10.
values of a and b are respectively: (a) p = –2, q = –6 (b) p = 2, q = –6
(a) 1 and 2 (b) 1 and 0 (c) p = –2, q = 6 (d) p = 2, q = 6
(c) –1 and –2 (d) 0 and 1 n n–1
20. Let f (x) = a0 x + a1 x + a2 xn–2 + ... + an–1 x + an,
[Based on XAT, 2006] where a0, a1, a2, ... , an are constants. If f (x) is divided by
If the polynomial x3 + px + q has three distinct roots, then
11. ax – b, the remainder is:
which of the following is a possible value of p?
(a) –1 (b) 0 b  −b 
(a) f    (b) f   
a  a 
(c) 1 (d) 2
[Based on XAT, 2007]
a  −a 
3 3
(c) f    (d) f   
x y x y b
   b 
12. If + = 6, find the value of 3 + 3 .
y x y x
21. If (x3/2 – xy1/2 + x1/2y – y3/2) is divided by (x1/2 – y1/2), the
(a) 176 (b) 198 quotient is:
(c) 184 (d) None of these (a) x + y (b) x – y
(c) x1/2 + y1/2 (d) x2 – y2
x2 + y 2 + z 2
13. If x + y + 2 = 0, what will be the value of ?
x 2 − yz 22.  When 4x3 – ax2 + bx – 4 is divided by x – 2 and x + 1, the
(a) 4 (b) 6 respective remainders are 20 and –13. Find the values of a
and b.
(c) 2 (d) 8
(a) a = 3, b = 2 (b) a = 5, b = 4
14. Which of the following must be equal to zero for all real (c) a = 7, b = 6 (d) a = 9 , b = 8
numbers x?
23. When a polynomial f (x) is divided by x – 3 and x + 6, the
I. x3 – x2 II. x0 III. x1 respective remainders are 7 and 22. What is the remainder
(a) II only (b) I only when f (x) is divided by (x – 3) (x + 6)?
(c) 1 and II only (d) None of these
−5 −7
[Based on NMAT, 2005] (a) x + 12 (b) x + 14
3 3
15. Factorize: (2x + 3y)2 + 2 (2x + 3y) (2x – 3y) + (2x – 3y)2
(a) 16x2 (b) 18x2 −5 −7
(c) x + 16 (d) x + 12
(c) 12x2 (d) None of these 3 3

16. Factors of a2 +
1
+ a will be: 24. If (x – 1) is a factor of Ax3 + Bx2 – 36x + 22 and 2B = 64A,
4 find A and B.
2
 1  1  1 (a) A = 4, B = 16 (b) A = 6, B = 24
(a)  a +  a −  (b)  a + 
 2  2  2 (c) A = 2, B = 12 (c) A = 8, B = 16
3
 1  1
(c)  a +  (d)  a +   × a
 2  2 25. Find the remainder when a 3 − 5a 2 + 7 a − 9 is divided
by a 2 + a − 6.
a 2 2 2
b c 
17. If a + b + c = 0, the value of  + + is:
 bc ca ab  (a) 19a − 31 (b) 19a − 38
 
(a) 1 (b) 0 (c) 19a − 49 (d) 19a − 45
(c) –1 (d) 3 [Based on CAT, 2009]

Chapter_25.indd 5 6/5/2015 6:10:10 PM


25.6 I Chapter 25

Answer Keys
Difficulty Level-1

1. (a) 2. (b) 3. (a) 4. (b) 5. (a) 6. (c) 7. (b) 8. (b) 9. (b) 10. (c) 11. (c) 12. (a) 13. (a)
14. (b) 15. (b) 16. (b) 17. (c) 18. (c) 19. (d ) 20. (a) 21. (d ) 22. (c) 23. (d ) 24. (a) 25. (b)

Difficulty Level-2

1. (a) 2. (d ) 3. (b) 4. (a) 5. (b) 6. (b) 7. (b) 8. (a) 9. (c) 10. (c) 11. (a) 12. (b) 13. (c)
14. (d ) 15. (a) 16. (b) 17. (d ) 18. (a) 19. (c) 20. (a) 21. (a) 22. (a) 23. (a) 24. (c) 25. (d )­­­­­­

Explanatory Answers

Difficulty Level-1

1. (a) Given expression 5.


(a)
⇒ (a + b) (a – b) – (a + b) (2c – 3d) 4y2 – 11y + 24
+ (2c + 3d) (a – b) – (2c + 3d) (2c – 3d) y + 2 4y3 – 3y2 + 2y – 4
= (a – b) (a + b) – (a – b) (2c + 3d) 4y3 + 8y2

+ (2c – 3d) (a + b) – (2c – 3d) (2c + 3d) –  –
⇒ (a + b) (2c – 3d) = (a – b) (2c + 3d)     11y2 + 2y – 4
– 
    11y2 – 22y
– 
⇒ 2ac – 3ad + 2bc – 3bd = 2ac + 3ad – 2bc – 3bd
   +  +
⇒ 4bc = 6ad
  24y – 4

2bc = 3ad.   24y + 48
   –  –
2.
(b) Given expression
     – 52
= (1 ­– x)(1 + x)(1 + x2)(1 + x4)(1 + x8)
\ Quotient = 4y2 – 11y + 24
2 2 4 8
= (1 – x )(1 + x )(1 + x )(1 + x ) Remainder = –52.
= (1 – x4)(1 + x4)(1 + x8)
(c) 16 (x – y)2 – 9 (x + y)2
6.
8 8 16
= (1 – x )(1 + x ) = 1 – x .
= [4 (x – y)]2 – [3 (x + y)]2
3.
(a) = [4 (x – y) –3 (x + y)] [4 (x – y) + 3 (x + y)]
= (4x – 4y – 3x – 3y) (4x – 4y + 3x + 3y)
4.
(b) Let, p (x) = x3 – 2ax2 + ax – 1
Since x – 2 is a factor of p (x), we must have p (2) = 0 = (x – 7y) (7x – y).

\ (2)3 –2a (2)2 + 2a – 1 = 0 (b) 4x2 + 12xy + 9y2 – 8x – 12y


7.
⇒ 8 – 8a + 2a – 1 = 0 = [(2x)2 + 2 (2x) (3y) + (3y)2] –4 (2x + 3y)

7 = (2x + 3y)2 –4 (2x + 3y)


⇒ –6a = –7 ⇒ a = .
6 = (2x + 3y) (2x + 3y – 4).

Chapter_25.indd 6 6/5/2015 6:10:11 PM


25.7
Polynomials

(b) 16x2 – 72xy + 81y2 – 12x + 27y


8. = 5ab [9a2 – 6ab + b2]
= (4x)2 –2 (4x) (9y) + (9y)2 –3 (4x – 9y) = 5ab [(3a)2 –2 (3a) (b) + (b)2]
2
= (4x – 9y) –3(4x – 9y) = 5ab [3a – b]2.
= (4x – 9y) (4x – 9y – 3). 18.
(c) Suppose, a – b = x, b – c = y, c – a = z
(b) (a + b)2 – 14c (a + b) + 49c2
9. \ (a – b) + (b – c) + (c – a) = x + y + z
= (a + b)2 –2 (a + b) ⋅ (7c) + (7c)2 ⇒ 0 = x + y + z
= (a + b – 7c)2. \ x + y = – z (1)
\ (x + y)3 = (–z)3
(c) 81x2y2 + 108xyz + 36z2
10.
or, x3 + y3 + 3xy (x + y) = – z3
= (9xy)2 + 2 (9xy) (6z) + (6z)2
or, x + z3 + z3 + 3xy (–z) = – z3
3
= (9xy + 6z)2.
[On substituting x + y = – z from eq. (1)]
11.
(c) If a + b + c = 0, then or, x3 + y3 – 3xyz = – z3
a3 + b3 + c3 = 3 abc or, x3 + y3 + z3 = 3xyz
\ a2(b + c) + b2(c + a) + c2 (a + b) \  (a – b)3 + (b – c)3 + (c – a)3
= a2 (–a) + b2(–b) + c2(–c) = 3(a – b) (b – c) (c – a)
= – a3 – b3 – c3 1 2
(d) a2 +
19. + 3 – 2a −
= –(a3 + b3 + c3) a2 a
= – 3abc.  1  2
=  a 2 + 2 + 2  – 2a − + 1
12.
(a) x = –1 satisfies the equation  a  a
2
2x3 – ax2 – (2a – 3)x + 2 = 0  1  1
=  a +  – 2 a +  + 1
⇒ a = 3  a  a
2  1 
P x − 36 x + 7 x−6 = x2 – 2x + 1 suppose a + a =
x
13.
(a) = 2 × =  
Q x − 49 x + 6 x −7
= (x – 1)2
14.
(b) Given expression 2
 1 
2 4
= (1 – x) (1 + x) (1 + x ) (1 + x ) (1 + x )8 =  a + − 1 .
 a 
= (1 – x2) (1 + x2) (1 + x4) (1 + x8) 1
2
 1
4 4
= (1 – x ) (1 + x ) (1 + x ) 8 20.
(a) x+ = 2 ⇒  x +  = (2)2
x  x
= (1 – x8) (1 + x8) = 1 – x16.
1 1
(b) 9 (3x + 5y)2 – 12 (3x + 5y) (2x + 3y) + 4 (2x + 3y)2
15. \ x2 + 2
+ 2x. = 4
x x
= [3 (3x + 5y)]2 –2 [3 (3x + 5y)] [2 (2x + 3y]
1
+ [2 (2x + 3y)]2 ⇒ x2 + + 2 = 4
2
x2
= [3 (3x + 5y) –2 (2x + 3y)]
1

= (9x + 15y – 4x – 6y)2 ⇒ x2 + = 2
x2
= (5x + 9y)2. 2
 2 1  2
 1
3
1 1 1 \  x + 2  = (2)
(b)  x +  = x3 + 3 + 3 ⋅ x
16. x+   x 
 x  x x x
1 1
1 ⇒ x4 + + 2x2. = 4
⇒   27 – 9 = x3 + x 4
x2
x3
1 1
\ x3 + = 18. ⇒ x4 + + 2 = 4
x3 x4
(c) 45a3b + 5ab3 – 30a2b2
17. 1
\ x4 + = 2.
= 5ab [9a2 + b2 – 6ab] x4

Chapter_25.indd 7 6/5/2015 6:10:12 PM


25.8 Chapter 25

1 1
21.
(d) x+ = 2 ⇒ x2 + = 7
x x2
3 3
 1  2 1  3
⇒  x +  = 23 ⇒
 x + 2  = 7
 x  x 

1  1 1  1 
⇒ x3 + 3
+ 3  x +  = 8 \ x6 + + 3  x 2 + 2  = 343
x  x x6  x 
1
⇒ x6 + 6 + 3 × 7 = 343
1 x
⇒ x3 + + 3 × 2 = 8
x3
1
\ x6 + = 343 – 21 = 322.
3 1 x6
⇒ x + = 2.
x3
3
 1  1   1
1 24. (a)  x +  =  x3 + 3  + 3  x + 
22.
(c) x+ = 5  x   x   x
x 3
 1  1 3 1
 1 
2 \  x +  − 3  x +  = x + x3 = 52
2  x  x
⇒  x+  = (5)
 x ⇒ 3
y – 3y = 52
1
1 1 where, y = x +
\ x+2 x + = 25 x
x x
i.e., y3 – 3y – 52 = 0
1 Clearly, y = 4, satisfies y3 – 3y – 52 = 0
\ 2+x+ = 25
x 1
\ x+ = 4.
1 x
⇒ x+ = 23 1 1 y −1
x 25.
(b) x + = 1 ⇒ x = 1 – =
y y y
2
 1 2
\  x +  = (23) 1 y
 x ⇒ =
x y −1
1
⇒ x2 + + 2 = 529
x2 1 1 1
and y+ = 1 ⇒ = 1–y ⇒ z =
z z 1− y
1
⇒ x2 + = 527.
x2 1 1 y 1 y
\ z+ = + = −
x 1− y y −1 1− y 1− y
2
 1 2
23.
(d)  x +  = 3 1− y
 x = = 1.
1− y

Difficulty Level-2

1.
(a) (b) = (2x – y)2
(c) = (2x + y)2 – 2xy
2.
(d) (a) = (3x – y)2 – (5x2 –­2xy)
= 4x2 + y2 + 2xy
2 2 2
= 9x + y – 6xy – 5x + 2xy
(d) = (2x + 3y)2 – 8y (2x + y)
2 2
= 4x + y – 4xy = 4x2 + 9y2 + 12xy – 16xy – 8y2
= (2x – y)2 = 4x2 + y2 – 4xy = (2x – y)2.

Chapter_25.indd 8 6/5/2015 6:10:13 PM


25.9
Polynomials I
3 2
3.
(b) Let, f (x) = 2x + mx + nx – 14 So, the equation will be
Since, x – 1 is a factor of f (x) 3x3 – 9x2 + 4x – 12 = 0
\ f (1) = 0 [By factor theorem] \ (x – 3) (3x2 + 4) = 0
⇒ 2 (1)3 + m (1)2 + n (1) – 14 = 0 So, the given equation is also divisible by (3x2 + 4).

⇒ 2 + m + n – 14 = 0
⇒ m + n = 12 (1) 8.
(a) Applying factor theorem and Remainder theorem
f (0) = 4
Since, x + 2, i.e., x – (–2) is a factor of f (x)
⇒ c = 4
\ f (–2) = 0 [By factor theorem]
f (–1) = 4
⇒ 2 (–2)3 + m (–2)2 + n (–2) – 14 = 0
⇒ a – b + c = 4
⇒ –16 + 4m – 2n – 24 = 0
⇒ 4m – 2n – 30 = 0 and, f (–2) = 6

⇒ 2m – n = 15 (2) ⇒ 4a – 2b + c = 6
Adding Eqs. (1) and (2), we get On solving, we get

3m = 27 ⇒ m = 9 a = 1, b = 1, c = 4.
Put, m = 9 in Eq. (1),
9.
(c) Using factor theorem
we get 9 + n = 12
x + 1 = 0 ⇒ x = –1
⇒ n = 3
Putting f (– 1) = 0, we get
(a) (a – b + c)2 + (b – c + a)2 + 2(a – b + c) (b + c – a)
4. 5a2 – 10a + 16 = 0
2
= (a –­ b + c) + 2 (a – b + c) (b + c – a)
+ (b – c + a)2   [rearranging] 10.
(c) Using factor theorem

= [(a – b + c) + (b – c + a)]2 = (2a)2 = 4a2 f (–1) = 0 and f (1) = 0


we get a = –1 and b = – 2.
5.
(b) Put x – 1 = 0 ⇒ x = 1
Putting the value in (a) Since, coefficient of x2 = 0. Sum of roots a + b + g
11.
x3 – k = 0 = 0.

\ k = l. This means at least one of the roots must be negative.

1 1 1
(b) If x 3 + y 3 + z 3 = 0, then
6. x y
12.
(b) + = 6
y x
1 1 1
(x + y + z) = 3x 3 × y 3 × x 3 3
 x y 3
⇒  +  = (6)
(If a + b + c = 0, then a3 + b3 + c3 = 3abc)  y x
⇒ (x + y + z)3 = 27 xyz.
x3 y3  x y
3 2 \ 3
+ 3
+ 3  +  = 216
7.
(b) 3x – 9x + kx – 12 is divisible by x – 3 y x  y x
\ f (3) = 0
3 2 x3 y3
⇒ 3 × 3 – 9 × 3 + 3k – 12 = 0 \ + + 3 × 6 = 216
y3 x3
⇒ 81 – 81 + 3k – 12 = 0
⇒ 3k = 12 x3 y3
\ + = 216 – 18 = 198.
⇒ k = 4 y3 x3

Chapter_25.indd 9 6/5/2015 6:10:13 PM


25.10 Chapter 25

(c) Q x + y + z = 0 ⇒ (x + y + z)2 = 0
13.
x3/2 − xy1/2 + x1/2 y − y 3/2
2 2 2
\ x + y + z + 2 (xy + yz + zx) = 0 \ = (x + y)
x1/2 − y1/2
2 2 2
\ x + y + z = –2 (xy + yz + zx)
(a) Let f (x) = 4x3 – ax2 + bx –4. When the expression f (x)
22.
= –2 [x (y + z) + yz]
is divided by x – 2, the remainder is
= –2 (x × –x + yz)
f (2) = 4 (2)3 –a (2)2 + b (2)­–4 = 20 (given)
(Q x + y + z = 0)
⇒ 2b – 4a + 28 = 20
= 2 (x2 – yz)
⇒ 2a – b = 4 (1)
x2 + y 2 + z 2
\ =2 Similarly, when the expression f (x) is divided by
x 2 − yz x – (–1), the remainder is
14. (d) None of the given expression will be zero by hit and f (–1) = 4 × (–1)3 –a (–1) + b (–1) – 4 = –13 (given)
trial method. ⇒ – 4 – a – b – 4 = – 13
⇒ a + b = 5 (2)
(a) (2x + 3y)2 + 2 (2x + 3y) (2x – 3y) + (2x – 3y)2
15.
= [2x + 3y) + (2x – 3y)]2 = (4x)2 = 16x2 Solving Eqs. (1) and (2), we get

2
a = 3, b = 2.
1 2 1 1
16.
(b) a + + a = a2 +   + 2.a   23.
(a) The function f (x) is not known
4 2 2
Here, a = 3, b = –6
2
 1
=  a +  . A = 7, B = 22.
 2
Required remainder
(d) a + b + c = 0 ⇒ a3 + b3 + c3 = 3abc
17.
A− B Ba − Ab
= x+
a3 b3 c3 a 2 b2 c2 a−b a−b
\ + + = 3  or   + + = 3.
abc abc abc bc ac ab
7 − 22 22 × 3 − 7 × (−6)
= x+
(a) x3 + y3 + z3 – 3xyz
18. 3 − (−6) 3 − (−6)
= (x+ y + z) (x2 + y2 + z2 – xy – yz – zx)
5
= (x + y + z) [(x + y + z)2 –3 (xy + yz + zx)] = – x + 12.
3
= 9 [(9)2 –3 (23)] = 9 [81 – 69]
(c) Since x – 1 is a factor of Ax3 + Bx2 – 36x + 22
24.
= 9 × 12 = 108.
19.
(c) On actual division, remainder is (p + 3) x – q \ A (1)3 + B (1)2 –36 (1) + 22 = 0

\ (p + 3) x – q = x – 6 ⇒ A + B = 14

⇒ p + 3 = 1 and q = 6 and, 2B = (26)A

⇒ p = –2, q = 6 ⇒ B = 6A
\ A = 2, B = 12.
b
20.
(a) ax – b = 0 ⇒ x = 25. (d) By direct division method,
a
(a 2 + a − 6)a 3 − 5a 2 + 7 a − 9(a − 6)
b
So, remainder = f    .
a ( −) a 3 + a 2 − 6a
−6a 2 + 13a − 9
(a) x3/2 – xy1/2 + x1/2 y – y 3/2
21. (−) − 6a 2 − 6a + 36
= x (x1/2 – y1/2) + y (x1/2 –­ y1/2) 19a − 45
= (x + y) (x1/2 – y1/2) ∴ Remainder = 19a − 45.

Chapter_25.indd 10 6/5/2015 6:10:15 PM


H.C.F. and L.C.M.
CHAPTER

of Polynomials 26
INTRODUCTION
G.C.D. by Factorization Method
We have already learnt in Chapter 2 how to find the
greatest common divisor (g.C.d.) or highest common Step 1 Resolve the given polynomials p (x) and
factor (H.C.F.) and least common multiple (L.C.M.) of q (x) in the complete factored form.
two integers. In this chapter, we will study how to find the
Step 2 Find the g.C.d. of the numerical factors of
g.C.d. and L.C.M. of polynomials which have integral
p (x) and q (x).
coefficients.
Step 3 Find the factors of highest degree common
Divisor to the two polynomials p (x) and q (x).
A polynomial d (x)) is said to be a divisor of polynomial Step 4 The product of all such common factors
p (x) if d (x) is a factor of p (x), i.e., p (x) can be written as and the g.C.d. of the numerical factors is
p (x) = d (x) q (x), the g.C.d. of the two given polynomials
where q (x) is a polynomial. p (x) and q (x).
For example, (x – 2) is a divisor of the polynomial Illustration 2 Find the g.C.d. of 4 + 9x – 9x2 and 9x2 – 24x
(x – 2)3 (x + 3). + 16
Solution: We have the factorization
Common Divisor
p(x) = 4 + 9x – 9x2 = – (9x2 – 9x – 4)
A polynomial d (x) is said to be a common divisor of the
polynomials p (x) and q (x), if d (x) is a factor of each of = – (9x2 – 12x + 3x – 4)
p (x) and q (x). = – (3x (3x – 4) + 1 (3x – 4)
For example, (x + 4) is a common divisor of the polynomials = – (3x + 1) (3x – 4)
(x + 4)3 (x – 2) (x + 3) and (x + 4) (x – 2)3 (x + 5). q(x) = 9x2 – 24x + 16 = (3x – 4)2
 g.C.d. of numerical factors = 1
G.C.D. (H.C.F) of Two Polynomials
and the highest degree common divisor = (3x – 4),
The g.C.d. of two polynomials p (x) and q (x) is the
common divisor which has highest degree among all \ required g.C.d. = (3x – 4).
common divisors and which has the highest degree term Illustration 3 Find the g.C.d. of 8 (x4 + x3 + x2) and 20
coefficient as positive. (x3 – 1)
Illustration l Find the g.C.d. of (3x – 2) (4x + 3); Solution: Here p (x) = 8 (x4 + x3 + x2) = 23. x2. (x2 + x + 1)
(3x – 2)2 (2x + 5) q (x) = 20 (x3 – 1) = 22. 5. (x – 1). (x2 + x + 1).
Solution: Here we find that (3x – 2) is a polynomial which  g.C.d. of numerical factors = 22
is a common divisor and has highest degree among all and the highest degree common divisor = x2 + x + 1,
common divisors. Further, the coefficient of the highest
degree term (3x) is 3 which is positive. Hence, (3x – 2) is \ required g.C.d. = 22 (x2 + x + 1)
the g.C.d. of the given polynomial. = 4 (x2 + x + 1).

Chapter_26.indd 1 6/5/2015 1:39:29 PM


26.2 Chapter 26

L.C.M. of Two Polynomials Illustration 4 Find the L.C.M. of the polynomials


We know that if a and b are two natural numbers, the product of (x + 2)2 (x – 1) (x + 4)2
a and b is equal to the product of their g.C.d. and L.C.M., i.e., and, (x + 4)3 (x + 2) (x + 7)
a × b = (g.C.d. or H.C.F. of a and b). Solution: We have, p (x) = (x + 2)2 (x – 1) (x + 4)2
(L.C.M. of a and b)
a×b q (x) = (x + 4)3 (x + 2) (x + 7)
or, L.C.M. of a and b =
G.C.D. of a and b Take the highest powers of factors common to both
Similarly, if p (x) and q (x) are two polynomials, then p (x) and q (x) and remaining terms for L.C.M.
p (x) × q (x) \ L.C.M. = (x + 4)3 (x + 2)2 (x – 1) (x + 7)
L.C.M. of p (x) and q (x) =
G.C.D. of p (x) and q (x)
Illustration 5 Find the L.C.M. of the polynomials
Thus, L.C.M. of two polynomials
(2x2 – 3x – 2) and (x3 – 4x2 + 4x)
Product of two polynomials
=
G.C.D. of the two polynomials Solution: We have, p (x) = 2x2 – 3x – 2
Note: = (x – 2) (2x + 1)
q (x) = x – 4x + 4x = x(x2 – 4x + 4) = x(x – 2)2
3 2
L.C.M. of two or more given polynomials is a polynomial
\ H.C.F. = (x – 2)
of smallest degree which is divided by each one of the given
polynomials. p (x) ⋅ q (x) x(x − 2)3 .(2 x + 1)
Hence, L.C.M. = =
L.C.M. by Factorization Method H.C.F. ( x − 2)
Step 1 Resolve the given polynomials p(x) and = x(x – 2)2 (2x + 1).
q(x) in the complete factored form. OR
Step 2 The required L.C.M. is the product of each Taking the highest powers of factors common to both p (x)
factor of p(x) and q(x) and if a factor is and q (x) and remaining terms for L.C.M., we have
common, we take that factor which has the
highest degree in p(x) or q(x). L.C.M. = x (x – 2)2 (2x + 1)

Practice Exercises

Difficulty level-1
(BASeD ON MeMORy)

1. Find the g.C.d. of 22x (x + 1)2; 36x2 (2x2 + 3x + 1). (a) 20a2b2c3 (b) 10ab2c
2
(a) 2x (x + 1) (b) 3x (x + 1) (c) 80ab c (d) 10ab2c2
(c) x (x + 1) (d) None of these 5. What will be the H.C.F. of (a – b) (a – 2b), (a – 2b)
(a – 3b) and (a – 3b) (a – 4b)?
2. Find the g.C.d. of 16 – 4x2; x2 + x – 6.
(a) (a – 2b) (b) a – 3b
(a) x – 2 (b) x – 3
(c) a – 4b (d) None of these
(c) x – 4 (d) None of these
6. If x + k is the H.C.F. of x2 + ax + b and x2 + cx + d, the
3. For what value of a, the g.C.d. of x2 – 2x – 24 and x2 – ax –
value of k is:
6 is (x – 6)?
(a) 7 (b) 5 b+d a+b
(a) (b)
a+c c+d
(c) 9 (d) None of these
a−b b−d
4. determine the H.C.F. of 50 a2b2c3, 80 ab3c2 and (c) (d)
c−d a−c
120a2b3c.

Chapter_26.indd 2 6/5/2015 1:39:30 PM


H.C.F. and L.C.M. of Polynomials 26.3

7. Find the g.c.d. of: 16. Find the g.c.d. of 4x4 + y4, 2x3 – xy2 – y3 and
(2x – 7) (3x + 4); (2x – 7)2 (x + 3). 2x2+ 2xy + y2.

(a) (2x – 5) (b) (2x – 9) (a) 2x2 + 2xy + y2



(c) (2x – 7) (d) None of these (b) 2x3 + 4xy + y2
(c) 3x2 + 2xy + y2
8. Find the g.c.d. of the polynomials (x2 – 9) (x – 3) and x2 +
(d) None of these
6x + 9.
(a) (x + 3) (b) (x + 5) 17. Find the g.c.d. of (x + 4)2 (x – 3)2 and (x – 1)
(c) (x – 3) (d) None of these (x + 4) (x – 3)2.
(a) (x + 3) (x + 9)2
9. The L.C.M. and H.C.F. of two polynomials P  (x) and
Q (x) are 36x3 (x + a) (x3 – a3) and x2 (x – a), respectively. (b) (x + 4) (x – 3)3
If P (x) = 4x2 (x2 – a2), find Q (x). (c) (x + 4) (x – 3)2

(a) 9x3 (x3 – a3) (d) None of these
3 3 3
(b) 6x (x – a )
18. Find the L.C.M. of the polynomials:
(c) 9x3(x3 + a3)
16 – 4x2; x2 + x – 6.
(d) None of these 2
(a) –4(x – 4) (x + 3)
3 3 4 2 2 4 3 2 2 3 4
10. The H.C.F. of x – y , x + x y + y and x y + x y + xy (b) 6(x2 – 4) (x + 4)
will be:
(c) 8(x2 – 6) (x + 3)
(a) x4 + x2y2 + y4 (b) x2 + xy + y2
(d) None of these
(c) x2 – xy + y2 (d) x(x + y)
19. Find the g.c.d. of x2 – 4 and x3 – 5x + 6.
11. If (x – 4) is the H.C.F. of x2 – x – 12 and (x2 – mx – 8), the
value of m is: (a) x – 3 (b) x – 2
(a) 0 (b) 1 (c) x + 4 (d) None of these
(c) 2 (d) 6
20. The H.C.F. (Highest Common Factor) of two polynomials
is (y – 7) and their L.C.M. is y3 – 10y2 + 11y + 70. If one
Find the g.c.d. of 3 + 13x – 30x2; 25x2 – 30x + 9.
12.
of the polynomials is y2 – 5y – 14, find the other.
(a) 7x – 4 (b) 5x – 3
(a) y2 – 12y + 35
(c) 6x – 5 (d) None of these
(b) y2 – 8y + 35
13. Find the L.C.M. of the polynomials: (c) y2 – 14y + 45
2 2 3
(x + 3) (x – 2) (x + 1) ; (x + 1) (x + 3) (x + 4). (d) None of these
(a) (x + 3) (x + 1)2 (x + 4)
(b) (x + 3)2 (x + 1) (x – 2) 21. If x – 4 is the g.c.d. of x2 – x – 12 and x2 – mx – 8, find
the value of m.
(c) (x + 3)2 (x + 1)3 (x – 2) (x + 4)
(a) 4 (b) 6
(d) None of these
(c) 2 (d) None of these
14. Find the L.C.M. of the polynomials:
2x2 – 3x – 2; x3 – 4x2 + 4x. 22. Find the g.c.d. of the polynomials:
2
(a) x(x – 2) (2x + 1) (x – 2)2 (x + 3) (x – 4); (x – 2) (x + 2) (x – 5).
(b) x (x – 2) (2x + 1)2 (a) (x – 4) (b) (x – 6)
(c) x (x – 2) (2x + 1) (c) (x – 2) (d) None of these
(d) None of these
23. For what value of a, the g.c.d. of x2 – 2x – 24 and x2 – ax –
3 2 3 6 is (x – 6)?
15. Find the g.c.d. of 8(x – x + x); 28 (x + 1).
(a) 6(x2 + x – 1) (b) 4(x2 – x + 1) (a) 7 (b) 5
2
(c) 8(x + 2x – 1) (d) None of these (c) 9 (d) None of these

Chapter_26.indd 3 6/5/2015 1:39:30 PM


26.4 Chapter 26

24. L.C.M. and H.C.F. of two polynomials p (x) and q (x) are 30. Find the L.C.M. of the polynomials:
36x3(x + a) (x3 – a3) and x2(x – a), respectively. If p (x) = 30x2 + 13x – 3; 25x2 – 30x + 9.
4x2 (x2 – a2), find q (x).
(a) – (5x – 3)2 (5x + 3) (6x – 1)
(a) 12x3 (x3 – a3)
(b) (5x – 3)2 (5x + 3) (6x – 1)
(b) 6x3 (x3 – a3)
(c) (5x + 3)2 (6x – 1)
(c) 9x3 (x3 – a3)
(d) None of these
(d) None of these
31. Find the g.c.d. of the polynomials 6x2 + 11x and 2x2 +
25. If (x – a) is the g.c.d. of x2 – x – 6 and x2 + 3x – 18, find x – 3.
the value of a.
(a) 4x + 5 (b) 2x – 3
(a) 3 (b) 6
(c) 2x + 3 (d) None of these
(c) 9 (d) None of these
32. H.C.F. of two expressions p and q is 1. Their L.C.M. is:
26. G.C.D. and L.C.M. of two polynomials p (x) and q (x) are
(a) (p + q) (b) (p – q)
x (x + a) and 12x2 (x + a) (x2 – a2), respectively. If p (x) =
4x (x + a)2, find q (x). 1
(c) pq (d)
pq
(a) 3x2 (x2 – a2)
(b) 5x2 (x3 – a3) 33. H.C.F. of (2x2 – 4x), (3x4 – 12x2) and (2x5 – 2x4 – 4x3)
is:
(c) 4x2 (x2 – a2)
(a) 2x (x + 2) (b) 2x (2 – x)
(d) None of these
(c) 2x (x – 2) (d) x(x – 2)
27. Find the g.c.d. of 8 (x4 – 16) and 12 (x3 – 8).
(a) 6(x – 2) (b) 4(x – 2) 34. The product of two non-zero expressions is (x + y + z)p3.
If their H.C.F. is p2, their L.C.M. is:
(c) 8(x – 2) (d) None of these
(a) (x + y)p (b) (y + 2)p
2
28. Find the L.C.M. of the polynomials (x + 3) (–6x + 5x + (c) (z + x)p (d) (x + y + z)p
4); (2x2 + 7x + 3) (x + 3).
(a) – (x + 3)2 (3x – 4) (2x + 1) 35. If (x – 1) is the H.C.F. of x2 – 1 and px2 – q (x + 1), then:

(b) (x + 3)2 (3x – 4) (2x + 1) (a) p = 2q (b) q = 2p


(c) 3p = 2q (d) 2p = 3q
(c) (x + 3)2 (3x + 4) (2x + 1)
(d) None of these 36. L.C.M. of (x2 – y2), (x3 – y3), (x3 – x2y – xy2 + y3) is:
29. Find the g.c.d. of the polynomials 36x2 – 49 and 6x2 – (a) (x + y) (x – y) (x2 + y2 + xy)
25x + 21. (b) (x + y) (x – y)2 (x2 + y2 + xy)
(a) 8x – 9 (b) 9x – 5 (c) (x + y) (x – y)2 (x2 + y2 – xy)
(c) 6x – 7 (d) None of these (d) (x + y)2 (x – y)2

Answer Keys
Difficulty Level-1

1. (a) 2. (a) 3. (b) 4. (b) 5. (d) 6. (d) 7. (c) 8. (d) 9. (a) 10. (b ) 11. (c) 12. (b) 13. (c)
14. (a) 15. (b) 16. (a) 17. (c) 18. (a) 19. (b) 20. (a) 21. (c) 22. (c) 23. (b) 24. (c) 25. (a) 26. (a)
27. (b) 28. (a) 29. (c) 30. (b ) 31. (c) 32. (c) 33. (d ) 34. (d) 35. (a) 36. (b)

Chapter_26.indd 4 6/5/2015 1:39:30 PM


H.C.F. and L.C.M. of Polynomials 26.5

Explanatory Answers

Difficulty Level-1

(a) Here, p (x) = 22x (x + 1)2


1. and, q (x) = x2 + 6x + 9
= x2 + 2x × 3 + 32 = (x + 3)2
q (x) = 36x2 (2x2 + 3x + 1)
The highest degree common divisor of the given
= 36x2(2x2 + 2x + x + 1) polynomials is (x + 3)
= 36x2[2x(x + 1) + 1(x + 1)] \ g.c.d. is (x + 3).

2
= 36x (2x + 1) (x + 1)
(a) L.C.M. = 36x3 (x + a) (x3 – a3)
9.
\ g.c.d. of numerical factors = 2
= 36x3 (x + a) (x – a) (x2 + a2 + ax)
and the highest degree common divisor = x (x + 1)
H.C.F. = x2 (x – a)
Hence, required g.c.D. = 2x (x + 1).
P (x) = 4x2 (x2 – a2) = 4x2 (x – a) (x + a)
2.
(a)
(L.C.M.)(H.C.F.)
(b) Obviously, (x – 6) divides both x2 – 2x – 24 and x2 –
3. \ Q (x) =
ax – 6 P( x)
So, x = 6 must make each polynomial zero
36 x3 ( x + a )( x − a )( x 2 + a 2 + ax) x 2 ( x − a )
\ (6)2 – 2 × 6 – 24 = (6)2 – 6a – 6 =
4 x 2 ( x − a )( x + a )
or, 6a = 30 or a = 5.
= 9x3 (x2 + a2 + ax) (x – a) = 9x3 (x3 – a3).
2 2 3 2 2 2 3
4.
(b) 50a b c = 5 × 2 × a × b × c
10.
(b) x3 – y3 = (x – y) (x2 + xy + y2)
3 2 4 3 3
80ab c = 5 × 2 × a × b × c
x4 + x2y2 + y4 = x4 + 2x2y2 + y4 – x2y2
120a2b3c = 3 × 5 × 23 × a2 × b3 × c
= (x2 + y2)2 – (xy)2
2
\ Required H.C.F. = 5 × 2 × a × b × c
= (x2 + xy + y2) (x2 – xy + y2)
= 10ab2c.
5.
(d) Since no factor is common to three given expressions, and, x3y2 + x2y3 + xy4
1 will be a common factor and hence H.C.F. = 1. = xy2 (x2 + xy + y2)
6.
(d) Since x + k is the H.C.F. it will divide each one of the \ Required H.C.F. = x2 + xy + y2.
given expressions. So, x = –k will make each one zero.
\ k2 – ak + b = 0, k2 – ck + d = 0 11.
(c) Clearly, x = 4 will make each one of the given
So, k2 – a k + b = k2 – ck + d expressions zero. So, 16 – 4m – 8 = 0 or, m = 2.

b−d 12.
(b) Here,
or, k = .
a−c
p (x) = 3 + 13x – 30x2 = 3 + 18x – 5x – 30x2
7.
(c) Here we find that (2x – 7) is a polynomial which is
a common divisor and has highest degree among = 3 (1 + 6x) – 5x (1+ 6x)
all common divisors. Further, the coefficient = (3 – 5x) (1 + 6x)
of the highest degree term (2x) is 2 which is
positive. Hence, (2x – 7) is the g.c.d. of the given = – (5x – 3) (1 + 6x)
polynomial.
q (x) = 25x2 – 30x + 9 = (5x – 3)2
8.
(d) Let, p (x) = (x2 – 9) (x – 3) = (x2 – 32) (x – 3) \ g.c.d. of numerical factors = 1 and highest degree
= (x + 3) (x – 3)2 of common divisor = (5x – 3).

Chapter_26.indd 5 6/5/2015 1:39:31 PM


26.6 I Chapter 26

13.
(c) p (x) = (x + 3)2 (x – 2) (x + 1)2 19.
(b) Let, p (x) = x2 – 4 = x2 – 22
q (x) = (x + 1)3 (x + 3) (x + 4) = (x + 2) (x – 2)
and, q (x) = x2 – 5x + 6 = x2 – 2x – 3x + 6
\ L.C.M. = (x + 3)2 (x + 1)3 (x – 2) (x + 4).
= x(x – 2) – 3 (x – 2) = (x – 2) (x – 3)
14.
(a) We have The highest degree common divisor is x – 2
\ g.c.d. of p (x) and q (x) is x – 2.
p (x) = 2x2 – 3x – 2 = 2x2 – 4x + x – 2
20.
(a) H.C.F. = (y – 7)
= 2x (x – 2) + 1 (x – 2)
L.C.M. = y3 – 10y2 + 11y + 70
= (2x + 1) (x – 2)
p (x) = y2 – 5y – 14
3 2
q (x) = x – 4x + 4x q (x) = ?
= x(x – 4x + 4) = x (x – 2) 2 2 L.C.M. of two polynomials

\ L.C.M. = x(x – 2)2 (2x + 1). Ist Polynomial × IInd Polynomial


=
H.C.F. of two polynomials
15.
(b) We have the factorization
p (x) = 8 (x3 – x2 + x) = 23 . x . (x2 – x + 1) \ L.C.M. =
p( x) q( x)
3
q (x) = 28 (x + 1) H.C.F.
= 22 . 7 . (x + 1) (x2 – x + 1)
( y 2 − 5 y − 14) × q ( x)
\ G.C.D. of numerical factors = 22 y3 – 10y2 + 11y + 70 =
( y − 7)
and the highest degree common divisor
= x2 – x + 1
( y − 7)( y 3 − 10 y 2 + 11 y + 70)
Therefore, required \ q (x) =
( y 2 − 5 y − 14)
g.c.d. = 22 (x2 – x + 1)
= 4 (x2 – x + 1). = (y – 7) (y – 5)

(a) 1st expression = (2x2 + y2)2 – (2xy)2


16. = y2 – 12y + 35.
= (2x2 + y2 + 2xy) (2x2 + y2 – 2xy) 21.
(c) H.C.F. = (x – 4)
2nd expression = (2x3 – 2y3) – y2 (x – y)
p (x) = x2 – x – 12 = (x – 4) (x + 3)
= 2 (x – y) (x2 + xy + y2) – y2 (x – y)
q (x) = x2 – mx – 8
= (x – y) (2x2 + 2xy + 2y2 – y2)
= (x – y) (2x2 + 2xy + y2) As (x – 4) is common in p  (x) and q  (x). Hence,
2
Hence, g.c.d. = 2x + 2xy + y . 2 x – 4 should be a factor of x2 – mx – 8
Thus, putting (x – 4) = 0 in q (x), we get (Remainder
17.
(c) Let, p (x) = (x + 4)2 (x – 3)2
2 theorem)
and, q (x) = (x – 1) (x + 4) (x – 3)
The highest degree common divisor is q (x) = x2 – mx – 8
(x + 4) (x – 3)2 q (4) = 42 – m × 4 – 8 = 0
\ The g.c.d. of given polynomial is ⇒ 16 – 4m – 8 = 0
(x + 4) (x – 3)2.
⇒ m = 2.
18.
(a) We have
p (x) = 16 – 4x2 = 4 (4 – x2)
22.
(c) Let, p (x) = (x – 2)2 (x + 3) (x – 4)
= 4 (2 – x) (2 + x) = – 4 (x – 2) (x + 2)
and, q (x) = (x – 2) (x + 2) (x – 5)
q (x) = x2 + x – 6 = x2 + 3x – 2x – 6
= x(x + 3) – 2 (x + 3) = (x + 3) (x – 2) the highest degree common divisor of the given
\ L.C.M. = –4 (x – 2) (x + 2) (x + 3) polynomials is x – 2.

= –4 (x2 – 4) (x + 3). \ The g.c.d. is x – 2.

Chapter_26.indd 6 6/5/2015 1:39:31 PM


H.C.F. and L.C.M. of Polynomials I 26.7

23.
(b) Here, p (x) = x2 – 2x – 24 q (x) = (2x2 + 7x + 3) (x + 3)
and, q (x) = x2 – ax – 6 = (2x + 1) (x + 3) (x + 3)

Since (x – 6) is the G.C.D. of p  (x) and q  (x), \ L.C.M. = – (x + 3)2 (3x – 4) (2x + 1).
(x – 6) is a factor of p (x) and q (x) both
29.
(c) p (x) = 36x2 – 49
⇒ p (6) = q (6)
= (6x)2 – (7)2 = (6x + 7) (6x – 7)
⇒ 36 – 2 × 6 – 24 = 36 – a × 6 – 6
q (x) = 6x2 – 25x + 21
⇒ a = 5.
= 6x2 – 18x – 7x + 21
24.
(c) We know that = 6x(x – 3) – 7(x – 3)

p  (x) × q (x) = L.C.M. × H.C.F. = (6x – 7) (x – 3)

4x2 (x2 – a2) × q (x) = 36 x3 (x + a) (x3 – a3) x2 (x –a) \ G.C.D. = (6x – 7).
5 2 2 3 3
⇒ q (x) =
36 x ( x − a )( x − a ) (b) 30x2 + 13x – 3 = 30x2 + 18x – 5x – 3
30.
2 2 2
4x (x − a )
= 6x (5x + 3) – 1 (5x + 3)
3 3 3
= 9x (x – a ). = (5x + 3) (6x – 1)

25.
(a) Let, p (x) = x2 – x – 6 q (x) = 25x2 – 30x + 9

and, q(x) = x2 + 3x – 18 = 25x2 – 15x – 15x + 9


= 5x (5x – 3) – 3 (5x – 3)
Since (x – a) is the G.C.D. of p (x) and q (x), (x – a)
is a divisor of p (x) and q (x) or (x – a) is a factor of = (5x – 3)2
p (x) and q (x) both. L.C.M. = (5x – 3)2 (5x + 3) (6x – 1).
⇒ p (a) = 0 and q (a) = 0 31.
(c) p (x) = 6x2 + 11x + 3
⇒ p (a) = q (a) = 6x2 + 9x + 2x + 3
⇒ a2 – a – 6 = a2 + 3a – 18 = 3x (2x + 3) + 1 (2x + 3)
⇒ 4a = 12 ⇒ a = 3. = (2x + 3) (3x + 1)
q (x) = 2x2 + x – 3
L.C.M. × H.C.F.
26.
(a) q (x) =
p( x) = 2x2 + 3x – 2x – 3
= (2x + 3) (x – 1)
12 x 2 ( x + a )( x 2 − a 2 ) x( x + a)
= \ G.C.D. = (2x + 3).
4 x( x + a) 2
Product of expressions
= 3x2 (x2 – a2). 32.
(c) L.C.M. =
H.C.F.
27.
(b) p (x) = 8 (x4 – 16) pq
= = pq.
= 4 × 2 (x + 4) (x + 2) (x – 2) 2 1

q (x) = 12 (x3 – 8) (d) 2x2 – 4x = 2x (x – 2)


33.
2
= 4 × 3 (x – 2) (x + 2x + 4)
3x4 – 12x2 = 3x2 (x2 – 4)
Hence, G.C.D. = 4 (x – 2).
= 3x2 (x – 2) (x + 2)
28. (x) = (x + 3) (–6x2 + 5x + 4)
(a) p  2x5 – 2x4 – 4x3 = 2x3 (x2 – x – 2)
2
= (x + 3) (–6x + 8x – 3x + 4) = 2x3 (x – 2) (x + 1)
= – (x + 3) (3x – 4) (2x + 1) \ H.C.F. = x (x – 2).

Chapter_26.indd 7 6/5/2015 1:39:31 PM


26.8 Chapter 26

36.
(b) x2 – y2 = (x – y) (x + y),
Product ( x + y + z ) p3
34.
(d) L.C.M. = =
H.C.F. p2 x3 – y3 = (x – y) (x2 + xy + y2),

= (x + y + z)p x3 – x2y – xy2 + y3 = x2 (x – y) – y2 (x – y)


= (x – y) (x2 – y2)
35.
(a) Since (x – 1) is the H.C.F., it will divide each one of
the given expressions. So, x = – 1 will make each one = (x – y)2 (x + y)
zero \ L.C.M. = (x – y)2 (x + y) (x2 + y2 + xy).
\ p × 12 – q (1 + 1) = 0 or p = 2q.

Chapter_26.indd 8 6/5/2015 1:39:31 PM


CHAPTER

Linear Equations 27
LINEAR EQUATION IN ONE VARIABLE From Eq. (1), we get y = 7 – x
A linear equation in one variable is an equation of the Substituting y = 7 – x in Eq. (2), we get
type ax + b = 0 or ax = c, where a, b, c are constants (real 3x – 2 (7 – x) = 11 ⇒ 3x – 14 + 2x = 11
numbers), a ≠ 0 and x is an unknown variable.
⇒ 5x = 25 ⇒ x = 5
The solution of the linear equation ax + b = 0 is Substituting this value of x in Eq. (1), we get
b b 5 + y = 7 ⇒ y = 7 – 5 or y = 2
x = – . We also say that – is the root of the linear
a a
Hence, x = 5, y = 2 is the required solution.
equation ax + b = 0.
For example, the equation 2x + 3 = 0 is a linear equation 2. Method of Elimination
3 Step 1 Multiply both the equations by such numbers
in one unknown variable x. Its solution or root is – .
2 so as to make the coefficients of one of the
two unknowns numerically the same.
LINEAR EQUATION IN TWO VARIABLES Step 2 Add or subtract the two questions to get
A linear equation in two variables is an equation of the type an equation containing only one unknown.
ax + by + c = 0 or ax + by = d, where a, b, c and d are Solve this equation to get the value of the
constants, a ≠ 0, b ≠ 0. unknown.
For example, 3x + 4y + 7 = 0 and 2x – 3y = 5 are linear Step 3 Substitute the value of the unknown in
equations in two variables x and y. either of the two original equations. By
solving that the value of the other unknown
Methods of Solving Two Simultaneous is obtained.
Linear Equations
Illustration 2 Solve: –6x + 5y = 2, –5x + 6y = 9
1. Method of Substitution
Solution: The given equations are
Step 1 Find the value of one variable, say y, in terms
–6x + 5y = 2 (1)
of the other, i.e., x from either equation.
–5x + 6y = 9 (2)
Step 2 Substitute the value of y so obtained in the
other equation. Thus, we get an equation in Multiply Eq. (1) by 6,
only one variable x. –36x + 30y = 12 (3)
Step 3 Solve this equation for x. Multiply Eq. (2) by 5,
Step 4 Substitute the value of x, thus obtained, in –25x + 30y = 45 (4)
step 1 and find the value of y. Subtract Eq. (4) from Eq. (3), we get
Illustration 1 Solve 2x + 3y = 7, 3x – y = 5 –11x = –33 or x = 3
Solution: The given equations are Substitute x = 3 in Eq. (1), we get
x+y=7 (1) –18 + 5y = 2 or y = 4
and 3x – 2y = 11 (2) Hence, x = 3 and y = 4 is the required solution.

Chapter_27.indd 1 6/5/2015 2:51:23 PM


27.2 Chapter 27

3. Short-cut Method a1 b c
Let the two equations be (b) If = 1 = 1 , the system is consistent
a2 b2 c2
a1x + b1y = c1 and has infinitely many solutions.
a2x + b2y = c2. Note: The graphs of such equations will have
The solution is written as coincident lines.
x y −1 a b c
= = (c) If 1 = 1 ≠ 1 , the system has no
b1c2 − b2c1 c1a2 − c2 a1 a1b2 − a2b1 a2 b2 c2
b1c2 − b2c1 c a −c a solution and is inconsistent.
i.e., x= – ,y=– 1 2 2 1 .
a1b2 − a2b1 a1b2 − a2b1 Note: The graphs of such equations will have parallel
lines.
Illustration 3 Solve 3x + 2y = – 25, – 2x – y = 10
Solution: The two equations are Illustration 4 For what values of k will the system of
equations kx + 2y = 5 and 3x + y = 1 have a unique solution?
3x + 2y = – 25
Solution: If the given system of equations has a unique
–2x – y = 10
solution,
The solution is given by
x y a1 b k 2
= ≠ 1 ⇒ ≠ ⇒k≠6
2 × 10 − (−1) × (−25) (−25) × (−2) − 3 × 10 a2 b2 3 1

−1 Hence, for k ≠ 6, the given system of equations will


= have a unique solution.
3 × (−1) − (−2) × 2
x y −1 Illustration 5 For what value of k, the system of equations
or, = =
−5 20 1 3x + 4y = 6 and 6x + 8y = k represent, coincident lines?
or, x = 5, y = –20 Solution: If the given system of equations represents
coincident lines
Consistent and Inconsistent Equations a1 b c 3 4 6
= 1 = 1 ⇒ = =
When a system of equations has a solution, the system a2 b2 c2 6 8 k
is called consistent. When a system of equations has no 6×8
solution, the system is called inconsistent. ⇒k= = 12
4

Test for Consistency Illustration 6 For what value of k the equations 9x + 4y =


9 and 7x + ky = 5 have no solution?
If we are given two linear equations
Solution: The given system of equations will have no
a1x + b1y = c1 and a2x + b2y = c2. Then,
solution
a1 b a1 b c 9 4 9
(a) If = 1 , the system will have exactly if = 1 ≠ 1 ⇒ = ≠
a2 b2 a2 b2 c2 7 k 5
one solution and will be consistent.
Note: The graphs of such equations will have 28
⇒ 9k = 28 or k =
intersecting lines. 9

Chapter_27.indd 2 6/5/2015 2:51:24 PM


Linear Equations 27.3

Practice Exercises

Difficulty Level-1
(Based on Memory)

1. Village A has a population of 6800, which is decreasing at 7.


There are 190 chairs, they are to be arranged in rectangular
the rate of 120 per year. Village B has a population of 4200, manner. Initially, when the chairs are arranged 5 more
which is increasing at the rate of 80 per year. In how many chairs are needed. When number of rows is increased by
years will the population of the two villages be equal? 2, 35 more chairs are needed, how many rows were there
(a) 9 (b) 11 initially?
(c) 13 (d) 16 (a) 17 (b) 13
[Based on MAT, 2004] (c) 15 (d) None of these

2. Ram and Mohan are friends. Each has some money. If 8. Neha buys 5 kg of rice and 7 kg of dal for `339 and Mani
Ram gives `30 to Mohan, then Mohan will have twice the buys 5 kg of dal and 7 kg of rice for `321. Find the cost of
money left with Ram. But, If Mohan gives `10 to Ram, rice and dal in `per kg.
then Ram will have thrice as much as is left with Mohan. (a) 23, 32 (b) 11, 44
How much money does each have? (c) 17, 38 (d) Data insufficient
(a) `62, `34 (b) `6, `2
(c) `170, `124 (d) `43, `26 9.
The students present in an auditorium are asked to stand in
[Based on MAT, 2003] rows. If there were 4 students extra in a row there would
be 4 less rows. However, if 2 students were less in a row,
3. In an examination, a student attempted 15 questions there would be 4 more rows. The number of students
correctly and secured 40 marks. If there were two types present in the auditorium is:
of questions (2 marks and 4 marks questions), how many
(a) 80 (b) 96
questions of 2 marks did he attempt correctly?
(c) 100 (d) 128
(a) 5 (b) 10
[Based on MAT (Feb), 2010]
(c) 20 (d) 40
[Based on MAT, 2003]
10.
The fuel indicator in a car shows one-fifth of the fuel tank
as full. When 22 more litres of fuel are poured into the
4. In a zoo, there are rabbits and pigeons. If their heads are tank, the indicator rests at the three-fourths of the full
counted, these are 90 while their legs are 224. Find the mark. Find the capacity of the fuel tank.
number of pigeons in the zoo.
(a) 25 litres (b) 35 litres
(a) 70 (b) 68 (c) 30 litres (d) 40 litres
(c) 72 (d) 22 [Based on MAT (Dec), 2008]
[Based on MAT, 2001]
11.
The highest score in an innings was two-ninths of the
5.
At the first stop on his route, a driver unloaded 2/5 of total score and the next highest was two-ninths of the
the packages in his van. After he unloaded another three remainder. These scores differ by 8 runs. What was the
packages at his next stop, ½ of the original number of total score in the innings?
packages remained. How many packages were in the van (a) 152 (b) 162
before the first delivery? (c) 142 (d) 132
(a) 25 (b) 10 [Based on MAT (May), 2008]
(c) 30 (d) 36 12.
At the first stop on his route, a driver unloaded 2/5 of
[Based on MAT, 2008] the packages in his van. After he unloaded another three
6.
There are two solutions of sulphuric acid in water. The packages at his next stop, half of the original number of
first is 40% strong and the second is 60% strong. The two packages remained. How many packages were in the van
solutions are mixed, 5 kg of pure water added and a 20% before the first delivery?
solutions is obtained. If instead, 5 kg of an 80% solution is (a) 25 (b) 10
added, a 70% solution is obtained, how much of the 40% (c) 30 (d) 36
solution and 60% solution were there initially? [Based on MAT (Feb), 2008]
(a) 2 kg; 1 kg (b) 1 kg; 1 kg 13.
The present population of a village is 5500. If the number
(c) 2 kg; 3 kg (d) 1 kg; 2kg of males increases by 11% and the number of females

Chapter_27.indd 3 6/5/2015 2:51:24 PM


27.4 Chapter 27

increases by 20% then the population will becomes 6330. yet another month, 500 units are consumed. The bill for
What is the present population of females in the village? that month would be:
(a) 3000 (b) 3500 (a) `1560 (b) `1680
(c) 2500 (d) 2000 (c) `1840 (d) `1950
[Based on MAT (Feb), 2011] [Based on MAT (Sept), 2009]
14.
Postcard costing 30 paise each and inland letters costing 20.
Krishan has some hens and some cows. If the total number
`1.50 each were purchased for `66. Total number of of animal heads are 59 and the total number of feet are
postcards and inland letters purchased was 60. If the 190, how many cows Krishan have?
number of postcards and inland letters is interchanged, (a) 36 (b) 32
then the cost is:
(c) 23 (d) Cannot be determined
(a) `42 (b) `30 [Based on MAT (May), 2009 (Feb), 2006]
(c) `21 (d) `60 21.
The cost of 3 chairs and 10 tables is `9856. What is the
[Based on MAT (Feb), 2011]
cost of 6 chairs and 20 tables?
15.
A lending library has a fixed charge for the first
(a) `17227 (b) `19712
three days and an additional charge for each day
thereafter. Sanchit paid `45 for a book kept for 7 days, (c) `19172 (d) Cannot be determined
while Karan paid `25 for the book he kept for 5 days. The [Based on MAT (May), 2009]
fixed charge and the charge for each extra day is: 22.
A manufacturer of a certain item can sell all he can
(a) `5 and `10 (b) `10 and `5 produce at the selling price of `60 each. It costs him `40
(c) `15 and `5 (d) `5 and `15 in materials and labour to produce each item and he has
[Based on MAT (Feb), 2011] overhead expenses of `3000 per week in order to operate
that plant. The number of units he should produce and sell
16.
There are two examination rooms A and B. If 10 in order to make a profit of at least `1000 per week is:
candidates are sent from A to B, the number of students
(a) 300 (b) 400
in each room is the same. If 20 candidates are sent from
B to A, the number of students in A is double the number (c) 250 (d) 200
of students in B. How many students are there in rooms A [Based on MAT (Feb), 2009]
and B respectively? 1
23.
A lamp post has half of its length in mud, of its length
(a) 100 and 80 (b) 120 and 60 3
1
(c) 80 and 100 (d) 140 and 60 in water and 3 m above the water. Find the total length
[Based on MAT (Dec), 2010 (May), 1999] 3
of the post.
(a) 20 m (b) 15 m
17.
The basic one-way railway fare for a child aged between
3 and 10 year costs half the regular fare for an adult plus a (c) 25 m (d) 30 m
reservation charge that is the same on the child’s ticket as on [Based on MAT (Feb), 2009]
the adults ticket. One reserved ticket for an adult costs `216 24.
There were 35 students in a hostel. If the number of students
and the cost of a reserved ticket for an adult and a child (aged increases by 7, the expenses of the mess increase by `42
between 3 and 10) costs `327. What is the basic fare for per day while the average expenditure per head diminishes
the journey for an adult? by `1. Find the original expenditure of the mess.
(a) `210 (b) `52.50 (a) `480 (b) `520
(c) `111 (d) `58.50 (c) `420 (d) `460
[Based on MAT (May), 2010] [Based on MAT (Dec), 2008]
25.
A group consisting of 25 teachers, 20 engineers, 18 doctors
18.
One-fourth of Nikhil’s money is equal to one-sixth of
and 12 salesmen visited a fair spent `1330 altogether. It
Yogesh’s money. If both together have `600, what is the
was found that 5 teachers spent as much as 4 engineers; 12
difference between their amounts?
engineers spent as much as 9 doctors and 6 doctors spent as
(a) `160 (b) `240 much as 8 salesmen. If every person in a professional group
(c) `200 (d) `120 spent the same amount, the amount spent by each engineer is:
[Based on MAT (Feb), 2010] (a) `18 (b) `17.50
19.
The electricity bill of a certain establishment is (c) `14 (d) `21
partly fixed and partly varies as the number of units [Based on MAT (Sept), 2008]
of electricity consumed. When in a certain month 26.
In a family, each daughter has the same number of brothers
540 units are consumed, the bill is `1800. In another as she has sisters and each son has twice as many sisters as
month, 620 units are consumed and the bill is `2040. In he has brothers. How many sons are there in the family?

Chapter_27.indd 4 6/5/2015 2:51:25 PM


Linear Equations 27.5

(a) 4 (b) 3 (a) 6 (b) 12


(c) 2 (d) 5 (c) 8 (d) None of these
[Based on MAT (Sept), 2008] [Based on MAT, 1999]
27. In a triangle ABC, ∠A = x°, ∠B = y° and ∠C = (y + 20)°. 34.
In a certain party, there was a bowl of rice for every two
If 4x – y = 10, then the triangle is: guests, a bowl of broth for every three of them and a bowl
(a) Right angled (b) Obtuse angled of meat for every four of them. If in all, there were 65
(c) Equilateral (d) None of these bowls of food, then how many guests were there in the
28. A body of 7300 troops is formed of 4 battalions so that party?
half of the first, two-thirds of the second, three-fourths of (a) 65 (b) 24
the third and four-fifths of the fourth are all composed of (c) 60 (d) 48
the same number of men. Find the same number. [Based on MAT, 1999]
(a) 1250 (b) 1200 35.
The sum of the two digits of a number is 15. If 9 be added
(c) 1300 (d) 1350 to the number, then the digits are reversed. The number is:
[Based on MAT (May), 2008] (a) 96 (b) 87
29.
A farmer has decided to build a wire fence along one (c) 78 (d) 69
straight side of his property. For this, he planned to place [Based on MAT, 1999]
several fence-posts at 6 m intervals, with posts fixed
36.
There are two examination rooms A and B. If 10 candidates
at both ends of the side. After he bought the posts and
are sent from room A to room B, the number of candidates
wire, he found that the number of posts he had bought
in each room is the same, while if 20 are sent from room
was 5 less than required. However, he discovered that the
B to room A, the number in room A becomes double the
number of posts he had bought would be just sufficient if
number in room B. The number of candidates in each
he spaced them 8 m apart. What is the length of the side of
room are respectively:
his property and how many posts did he buy?
(a) 100 m, 15 (b) 100 m, 16 (a) 80 and 100 (b) 100 and 80
(c) 120 m, 15 (d) 120 m, 16 (c) 80 and 120 (d) 100 and 60
[Based on MAT (Dec), 2006] [Based on MAT, 1999]
30.
A part of monthly expenses of a family is constant and the 37.
If two mixers and one TV cost `7,000, while two TVs and
remaining varies with the price of wheat. When the rate of one mixer cost `9,800, the value of one TV is:
wheat is `250 a quintal, the total monthly expenses of the (a) `2,800 (b) `2,100
family are `1000 and when it is `240 a quintal, the total (c) `4,200 (d) `4,800
monthly expenses are `980. Find the total monthly expenses
[Based on MAT, 2000]
of the family when the cost of wheat is `350 a quintal.
(a) `1000 (b) `1400 38.
The prices of a scooter and a television set are in the ratio
3:2. If a scooter costs `6,000 more than the television set,
(c) `1200 (d) `800
the price of the televi­sion set is:
[Based on MAT (May), 2006]
31.
A company received two shipments of ball bearings. In (a) `6,000 (b) `10,000
the first shipment, 1% of the ball bearings were defective. (c) `12,000 (d) `18,000
In the second shipment, which was twice as large as the [Based on MAT, 2000]
first, 4.5% of the ball bearings were defective. If the
39. Shyam had 85 currency notes in all, some of which
company received a total of 100 defective ball bearings,
were of `100 denomination and the remaining of `50
how many ball bearings were there in the first shipment?
denomination. The total amount of all these currency
(a) 990 (b) 2000 notes was `5000. How much amount in rupees did he
(c) 1000 (d) 3000 have in the denomination of `50?
[Based on MAT (May), 2006] (a) 3,500 (b) 70
32.
Two audio cassettes and three video cassettes cost `425,
(c) 15 (d) 1,500
and three audio cassettes and two video cassettes cost
`350. The price of a video cassette is: 40. The difference between a two-digit number and the
(a) `150 (b) `120 number obtained by interchanging the two digits of the
number is 9. What is the difference between the two digits
(c) `110 (d) `115
of the number?
[Based on MAT, 1998]
33.
In a group of buffaloes and ducks, the number of legs (a) 3 (b) 2
are 24 more than twice the number of heads. What is the (c) 1 (d) Cannot be determined
number of buffaloes in the group? [Based on NABARD Bank PO, 2009]

Chapter_27.indd 5 6/5/2015 2:51:25 PM


27.6 Chapter 27

41. Sushil planned to drive a distance of x Km. After driving 44. In a zoo, there are healthy deers and ducks. If the heads
120 Km, Sushil stopped for petrol. What fractional part of are counted, there are 180, while the legs are 448. What
the trip had Sushil covered when he stopped? will be the number of deers in the zoo?
(a) x/120 (b) x/(x + 120) (a) 156 (b) 68
(c) l/(x + 120) (d) None of these (c) 22 (d) 44
[Based on MAT, 2014]
42. The electricity bill of a certain establishment is partly
45. A bus starts from city X. The number of women in the bus
fixed and partly varies with the number of units of
is half of the number of men. In city Y, 10 men leave the
electricity consumed. When in a certain month 540 units
bus and five women enter. Now the number of men and
are consumed, the bill is `1800. In another month, 620
women is equal. In the beginning, how many passengers
units are consumed and the bill is `2040. In yet another
entered the bus?
month 500 units are consumed. The bill for that month
would be: (a) 15 (b) 30
(c) 36 (d) 45
(a) `1680 (b) `1560
(212)
(c) `184 (d) `1950 46. A teacher noticed a strange distribution of marks in the
[Based on MAT, 2013] exam. There were only three distinct scores : 6, 8 and
20. The mode of the distribution was 8. The sum of the
43. Two trains, 330 and 110 m long, are going in the same
scores of all the students was 504. The number of students
direction. The faster train takes one minute to pass
in the in most populated category was equal to the sum
the other completely. If they are moving in opposite
of the number of students with lowest score and twice
directions, they pass each other completely in 3 seconds.
the number of students with the highest score. The total
What is the speed of the faster train?
number of students in the class was:
(a) 42 m/s (b) 38 m/s (a) 50 (b) 51
(c) 46 m/s (d) 50 m/s (c) 53 (d) 57
[Based on MAT, 2013] [Based on XAT, 2014]

Difficulty Level-2
(Based on Memory)

1.
Which one of the following conditions must p, q and r plans, which one meets the machine availability
satisfy so that the following system of linear simultaneous constraints and maximises the profit?
equations has at least one solution, such that p + q + r ≠ 0? (a) Standard 75 bags, Deluxe 80 bags
x + 2y – 3z = p (b) Standard 100 bags, Deluxe 60 bags
2x + 5y – 11z = q (c) Standard 50 bags, Deluxe 100 bags
x – 2y + 7z = r (d) Standard 60 bags, Deluxe 90 bags
(a) 5p – 2q – r = 0 (b) 5p + 2q + r = 0 [Based on CAT, 2003]
(c) 5q + 2q – r = 0 (c) 5q – 2q + r = 0 3.
If x and y are integers then the equation:
[Based on CAT, 2003] 5x + 19y = 64 has
2.
A leather factory produces two kinds of bags, standard (a) No solution for x < 300 and y < 0
and deluxe. The profit margin is `20 on a standard bag (b) No solution for x > 250 and y > –100
and `30 on a deluxe bag. Every bag must be processed on (c) A solution for 250 < x < 300
machine A and on machine B. The processing times per (d) A solution for –59 < y < –56
bag on the two machines are as follows: [Based on CAT, 2004]
Time required (Hours/bag) 4.
If the demand for fertilizer product is given by p + 5q = 21
Machine A Machine B and the supply is determined by p – 2q = 7, where p and q
Standard Bag 4 6 denote the price of the commodity and q is the number of
units of fertilizer product supplied. If a man wants to buy q
Deluxe Bag 5 10
units of fertilizer product, then the amount paid by him is:
The total time available on machine A is 700 hours and on (a) `22 (b) `30
machine B is 1250 hrs. Among the following production (c) `32 (d) `24

Chapter_27.indd 6 6/5/2015 2:51:25 PM


Linear Equations 27.7

5. The quantity of fat in a kilogram of food A plus the 34


quantity of protein in a kilogram of food A is 100 g. The (a) (b) 34x
100x
quantity of protein in a kilogram of food A minus twice
the quantity of fat in a kilogram of food A is 10 g. How  34 
many grams of protein are there in a kilogram of food A? (c) x 1 −  (d) 44x
 100x 
(a) 30 (b) 45
12. A woman sells to the first customer half her stock and
(c) 50 (d) 70
half an apple, to the second customer she sells half her
6. Shyam had 85 currency notes in all, some of which remaining stock and half an apple, and so on to the third,
were of `100 denomination and the remaining of `50 and to a fourth customer. She finds that she has now 15
denomination. The total amount of all these currency apples left. How many apples did she have before she
notes was `5000. How much amount in rupees did he started selling?
have in the denomination of `50?
(a) 63 (b) 127
(a) 3500 (b) 70
(c) 240 (d) None of these
(c) 15 (d) 1500
[Based on Narsee Manjee Inst. of Man. Studies, 2003]
7.
A florist was asked to make a bouquet worth exactly 13. The equations 3x – 4y = 5 and 12x – 16y = 20 have:
`1000 with 100 sticks of roses of three colours— Pink, (a) No common solution
Yellow and Red. While Pink roses cost Re 0.50 a stick,
(b) Exactly one common solution
Red roses cost `10.00 per stick and Yellow roses cost
`50.00 per stick. How many Red roses did the florist use (c) Exactly two common solutions
in the bouquet? (d) More than two common solutions
(a) 1 [Based on FMS (Delhi), 2003]
(b) 5 14. There are two examination halls, P and Q. If 10 students
(c) 80 are sent from P to Q, then the number of students in each
hall is the same. If 20 students are sent from Q to P, then
(d) Several combinations are possible
the number of students in P is double of that in Q. The
[Based on FMS (Delhi), 2004]
number of students in halls P and Q respectively are:
8.
A lady went to the market with a few one-rupee notes and (a) 100, 80 (b) 80, 60
a few 20-paise coins. When she returned, she had as many
one-rupee notes as she originally had 20-paise coins and vice (c) 70, 50 (d) 60, 40
versa. She actually came back with about one-third of what [Based on FMS (Delhi), 2003]
she had gone with. How much did she spend in the market? 15.
To a proper fraction, when six is added to the numerator
(a) `14.40 (b) `14.70 and the denominator is increased by 50%, the ratio
(c) `15.50 (d) `17.40 becomes half and when the numerator is multiplied by
4 and denominator is reduced by 8, then the fraction
[Based on FMS (Delhi), 2004]
becomes 3. The fraction (simplified) is:
9. A student was asked to divide a number by 17/8. Instead,
he actually multiplied it by 17/8 and hence got 225 more 1 15
than the expected answer. What was the expected answer? (a) (b)
3 28
(a) 126 (b) 136
33
(c) 64 (d) None of these (c) (d) Cannot be determined
[Based on IIT Joint Man. Ent. Test, 2004]
52

10. A tells B “If you give me `400, then I shall have 25% 16. There are two numbers, such that they form the first and
more than what you have”. B tells A, “If you give me the last numbers of three numbers in an A.P. as well as
`200, then I shall have three-and-a-half times as much as that of three numbers in a G.P. Their sum is 10 while the
you have.” How much does B have? difference between the square of middle number in A.P.
and that in G.P. is 1. Find the numbers.
(a) `1200 (b) `900
(c) `800 (d) `1600 (a) 2, 8 (b) 3, 7
[Based on IIT Joint Man. Ent. Test, 2004] (c) 4, 6 (d) 5, 5

11.
In theater when price is `x per ticket for a show then only 17. A confused bank teller transposed the rupees and paise
60% of theater gets filled. But when its price is reduced when he cashed a cheque for Shailaja, giving her rupees
y instead of paise and paise instead of rupees. After buying
by , then whole theater gets filled and the revenue
100 a toffee for 50 paise, Sahilaja noticed that she was left
increased by 10%. Then y in terms of x is: with exactly three times as much as the amount of the

Chapter_27.indd 7 6/5/2015 2:51:25 PM


27.8 Chapter 27

cheque. Which of the following is a valid statement about (a) `14 (b) `18
the cheque amount? (c) `20 (d) None of these
(a) Over rupees 22 but less than rupees 23
24. Sum of two-third of a number and three-fourth of another
(b) Over rupees 18 but less than rupees 19 number is equal to 23. Also sum of one-fourth of the first
(c) Over rupees 4 but less than rupees 5 and one-fifth of the second number is 7. Find the greater
(d) Over rupees 13 but less than rupees 14 of the two numbers.
[Based on CAT, 2007] (a) 12 (b) 16
18. The pair of equations 3x+y = 81; 81x–y = 3 has: (c) 20 (d) 24
(a) No solution 25. A leather factory produces two kinds of bags, standard
1 7 and deluxe. The profit margin is `20 on a standard bag
(b) The solution x = 2 , y = 1 and `30 on a deluxe bag. Every bag must be processed on
2 8
machine A and on machine B. The processing times per
(c) The solution x = 2, y = 2
bag on the two machines are as follows:
1 7
(d) The solution x = 2 , y = 1 Time required (Hours/bag)
8 8 Machine A Machine B
19. Given x = a, y = b and z = c(a, b, c ≠ 0) satisfy the following
Standard Bag 4 6
system of linear equations
Deluxe Bag 5 10
x–y+z=0
x – 2y + 3z = 0 The total time available on machine A is 700 hours and on
machine B is 1250 hours. Among the following production
2x – 3y + 4z = 0
plans, which one meets the machine availability
Which of the following relations is valid? constraints and maximizes the profit?
(a) a:b:c: :2:1:1 (b) a:b:c: :1:1:2 (a) Standard 75 bags, Deluxe 80 bags
(c) a:b:c: :1:2:1 (d) a:b:c: :1:2:2 (b) Standard 100 bags, Deluxe 60 bags
20. The charge for sending a telegram is constant for the (c) Standard 50 bags, Deluxe 100 bags
first 10 or less words and an amount proportional to the (d) Standard 60 bags, Deluxe 90 bags
number of words exceeding 10. If the charge for sending 26. How can the relationship between x and y be best defined,
a 15 words telegram is `3 and that for a 20 words is `4.25, if values of x and y are as follows?
how much it cost to send a 35 words telegram? x 2 3 4 5 6
(a) `7 (b) `7.25 y 0 2 6 12 20
(c) `8 (d) `7.75 (a) y = 2x – 4 (b) y = x2 – 3x + 2
5x 7y (c) y = x – 4x (d) y = x2 – 4
2
21. Find the value of x and y, if + = 6 and
8 18 27. A man buys a certain quantity of apples, mangoes and
2(x – y) = –10. bananas. If the mangoes were to cost the same as apples,
(a) 4, 9 (b) 5, 7 he would have to forgo the bananas to buy the same
(c) 3, 12 (d) 10,4 number of mangoes as he had bought earlier (for the same
total amount). The amount spent by him on mangoes and
22. If the numerator and the denominator of a fraction are bananas together is 50% more than the amount spent on
2 apples. The total amount spent in the transaction is `140.
each increased by 4, the fraction becomes and when
1 The number of mangoes bought is the same as the number
numerator and denominator of the same fraction are each of bananas. If he wishes to buy the same number of apples
12 as well how much additional amount would have to be
decreased by 6, the fraction becomes . The sum of the spent by him?
1
numerator and the denominator is: (a) 56 (b) 140
(a) 11 (b) –11 (c) 28 (d) 42
(c) 25 (d) –25 28. In a certain party, there was a bowl of rice for every two
23. There is a chaat shop in a colony. Manu had 2 plates of guests, a bowl of broth for every three of them and a bowl
gol gappas, 5 plates of tikki and 4 plates of paapdi. His of meat for every four of them. If in all there were 65
bill amount was `80. Tanu had 4 plates of gol gappas, bowls of food, then how many guests were there in the
3 plates of tikki and 1 plate of paapdi. His bill amount party?
was `62. Ram had just one plate of tikki and one plate of (a) 65 (b) 24
pappdi. How much was the bill amount of Ram? (c) 60 (d) 48

Chapter_27.indd 8 6/5/2015 2:51:25 PM


Linear Equations 27.9

29.
A person buys 18 local tickets for `110. Each first class (a) 5 (b) 7
ticket costs `10 and each second class ticket costs `3. (c) 10 (d) 12
What will another lot of 18 tickets in which the number of [Based on FMS, 2011]
first class and second class tickets are interchanged cost?
37.
A boy read three-eighths of a book on one day and four-
(a) 112 (b) 118
fifths of the remainder on another day. If there were 30
(c) 121 (d) 124 pages unread, how many pages did the book contain?
[Based on SNAP, 2008]
(a) 600 (b) 300
30.
A sum of `312 was divided among 100 boys and girls in
(c) 240 (d) None of these
such a way that each boys gets `3.60 and each girls gets
[Based on XAT, 2006]
`2.40. The number of girls is:
(a) 40 (b) 60 38.
A lady went to the market with a few one-rupee notes and
a few 20-paise coins. When she returned, she had as many
(c) 35 (d) 65
one-rupee notes as she originally had 20-paise coins and
[Based on FMS (MS), 2006] vice versa. She actually came back with about one-third of
3x  y  1 2x  y  2 what she had gone with. How much did she spend in the
31.
The solution of the equations = =
3 5 market?
3x  2 y  1 (a) `14.40 (b) `14.70
is given by:
6 (c) `15.50 (d) `17.40
(a) x = 2, y = 1 (b) x = 1, y = 1 39.
A brokerage house offers 3 stock portfolios. Portfolio I
(c) x = – 1, y = – 1 (d) x = 1, y = 2 consists of 2 blocks of common stock and 1 municipal
[Based on FMS, 2005] bond. Portfolio II consists of 4 blocks of common stock, 2
municipal bonds and 3 blocks of preferred stock. Portfolio
32.
The condition for which the system of equations kx – y = III consists of 2 blocks of common stock, 2 municipal
2 and 6x – 2y = has a unique solution, if: bond and 3 blocks of preferred stock. A customer wants
(a) k = 3 (b) k ≠ 3 12 blocks of common stock, 6 municipal bonds and
(c) k ≠ 0 (d) k = 0 6 preferred stocks. How many portfolio III should be
[Based on FMS, 2006] offered?
Given, 2x = 8y +1 and 9y = 3x – 9; the value of x + y is:
33. (a) 1 (b) 2
(a) 18 (b) 21 (c) 3 (d) None of these
(c) 24 (d) 27 [Based on XAT, 2007]

[Based on FMS, 2010] 40.


Consider the system of linear equations
34.
Two numbers are such that their difference, their sum, and 2x + 3y + 4z = 16
their product are to one another as 1:7:24. The product of 4x + 4y + 5z = 26
the two numbers is: ax + by + cz = r
(a) 6 (b) 12 For r = 5 and a = 1 then the system of linear equation will
(c) 24 (d) 48 have infinite number of solutions, if c =
[Based on FMS, 2010] (a) 3/2 (b) 1
35.
Thirty-one magazines are arranged from left to right in (c) 1/2 (d) 0
order of increasing prices. The price of each magazine [Based on XAT, 2007]
differs by `2 from that of each adjacent magazine. For the 41.
In an examination there are 30 questions. 1 mark is given
price of the magazine at the extreme right a customer can for each correct answer and 0.25 is deducted for every
buy the middle magazine and an adjacent one. Then: incorrect answer. Ankur attempted all the questions and
(a) The adjacent magazine referred to is at the left of the scored 13.75. How many incorrect answers did he have?
middle magazine. (a) 10 (b) 11
(b) The middle magazine sells for `36. (c) 12 (d) None of these
(c) The most expensive magazine sells for `64 [Based on XAT, 2009]
(d) None of the above is correct 42.
A manufacturer produces two types of products– A and
[Based on FMS, 2010] B, which are subjected to two types operations, viz,
36.
In a group of cows and chickens, the number of legs was grinding and polishing. Each unit of product A takes 2 hrs
14 more than twice the number of heads. The number of of grinding and 3 hrs polishing whereas product B takes
cows was: 3 hrs of grinding and 2 hrs of polishing. The manufacturer

Chapter_27.indd 9 6/5/2015 2:51:26 PM


27.10 Chapter 27

has 10 grinders and 15 polishers. Each grinder operates 48. Anita had to do a multiplication. Instead of taking 35 as
for 12 hrs/day and each polisher 10 hrs/day. The profit one of the multipliers, she took 53. As a result, the product
margin per unit of A and B are `5 and `7, respectively. If went up by 540. What is the new product?
the manufacturer utilises all his resources for producing (a) 1050 (b) 540
these two types of items, what is the maximum profit that (c) 1040 (d) 1590
the manufacture can earn?
[Based on CAT, 2001]
(a) `280 (b) `294
49. In a four-digit number, the sum of the first two digits is
(c) `515 (d) `550 equal to that of the last two digits. The sum of the first and
[Based on XAT, 2010] last digits is equal to the third digit. Finally, the sum of
43.
Which one of the following conditions must a, b and c second and fourth digits is twice the sum of the other two
satisfy so that the following system of linear simultaneous digits. What is the third digit of the number:
equations has at least one solution, such that: (a) 5 (b) 8
a + b + c ≠ 0 (c) 1 (d) 4
m + 2n – 3r = a [Based on CAT, 2001]
2m + 6n – 11r = b 50. P, Q and R are three consecutive odd numbers in ascending
m – 2n + 7r = c order. If the value of three times P is 3 less than two times
(a) 5a + 2b + c = 0 (b) 5a + 2b – c = 0 R, find the value of R:
(c) 5a – 2b – c = 0 (d) 5a – 2b + c = 0 (a) 5 (b) 7
[Based on JMET, 2011] (c) 9 (d) 11
44. A tells B ‘If you give me `400, then I shall have 25% [Based on CAT, 1997]
more than what you have’. B tells A, ‘If you give me 7
51. A student instead of finding the value of of the number,
`200, then I shall have three-and-a-half times as much as 8
7
you have.’ How much does B have? found the value of of the number. If his answer differed
(a) `1,200 (b) `900 18
from the actual one by 770, find the number.
(c) `800 (d) `1,600
(a) 1584 (b) 2520
45. In a family of husband, wife and a daughter, the sum of
the husband’s age, twice the wife’s age and thrice the (c) 1728 (d) 1656
daughters age is 85; while the sum of twice the husband’s [Based on CAT, 1997]
age, four times the wife’s age and six times the daughter’s 52. Two positive integers differ by 4 and sum of their
age is 170. It is also given that the sum of five times the 10
reciprocals is . Then, one of the numbers is:
husband’s age, ten times the wife’s age and fifteen times 21
the daughter’s age equals 450. The number of possible (a) 3 (b) 1
solutions, in terms of the ages of the husband, wife and the
(c) 5 (d) 21
daughter, to this problem is:
[Based on CAT, 1995]
(a) 0 (b) 1
53. A shop stores x kg of rice. The first customer buys half of
(c) 2 (d) Infinitely many
this amount plus half a kg of rice. The second customer
[Based on JMET, 2006]
buys half of the remaining amount plus half a kg of rice.
46.
Nikhil’s mother asks him to buy 100 pieces of sweets worth Then, the third customer also buys half of the remaining
`100%. The sweet shop has 3 kinds of sweets, kajubarfi, amount plus half a kg of rice. Thereafter, no rice is left in
gulabjamun and sandesh. Kajubarfi costs `10 per piece, the shop. Which of the following best describes the value
gulabjamun costs `3 per piece and sandesh costs 50 paise of x?
per piece. If Nikhil decides to buy at least one sweet of (a) 2 ≤ x ≤ 6 (b) 5 ≤ x ≤ 8
each type, how many gulabjamuns should he buy?
(c) 9 ≤ x ≤ 12 (d) 11 ≤ x ≤ 14
(a) 1 (b) 2
[Based on CAT, 2008]
(c) 3 (d) 4
[Based on XAT, 2012] 54. A man buys a certain quantity of apples, mangoes and
bananas. If the mangoes were to cost as same as apples,
47. How many five digit positive integers that are divisible by
he would have to forego the bananas to buy the same
3 can be formed using digits 0, 1, 2, 3, 4 and 5, without
number of mangoes as he had bought earlier (for the same
any of the digits getting repeating?
total amount). The amount spent by him on mangoes and
(a) 15 (b) 96 bananas together is 50% more than the amount spent on
(c) 216 (d) 120 apple. The total amount spent in the transaction is `140.
[Based on CAT, 2010] The number of mangoes bought is the same as the number

Chapter_27.indd 10 6/5/2015 2:51:26 PM


Linear Equations 27.11

of bananas. If he wishes to buy the same number of apples 56. Amitabh picks a random integer between 1 and 999,
as well how much additional amount would have to be doubles it and gives the results to Sashi. Each time Sashi
spent by him? gets a number from Amitabh, he adds 50 to the number,
(a) 56 (b) 140 and gives the result back to Amitabh, who doubles the
(c) 28 (d) 42 number again. The first person, whose result is more
than 1000, loses the games. Let ‘X’ be the smallest initial
[Based on 2012]
number that results in a win for Amitabh. The sum of the
55. The sum of the possible values of x in the equation |x + 7| digits of ‘X’ is:
+ |x – 8| = 16 is:
(a) 3 (b) 5
(a) 0 (b) 1
(c) 2 (d) 3 (c) 7 (d) 9
(e) None of the above (e) None of these
[Based on XAT, 2014] [Based on XAT, 2014]

Answer Keys
Difficulty Level-1

1. (c) 2. (a) 3. (b) 4. (b) 5. (c) 6. (d) 7. (b) 8. (a) 9. (b) 10. (d ) 11. (b) 12. (c) 13. (c)
14. (a) 15. (a) 16. (a) 17. (a) 18. (d) 19. (b) 20. (a) 21. (b) 22. (d) 23. (a) 24. (c) 25. (b) 26. (b)
27. (a) 28. (b) 29. (d) 30. (c ) 31. (c) 32. (d) 33. (b ) 34. (c) 35. (c) 36. (b) 37. (c) 38. (c ) 39. (a)
40. (c ) 41. (d) 42. (a) 43. (a ) 44. (d) 45. (d) 46. (d)

Difficulty Level-2

1. (a) 2. (b) 3. (c) 4. (a) 5. (d) 6. (a) 7. (a) 8. (a) 9. (b) 10. (a) 11. (b) 12. (d) 13. (d)
14. (a ) 15. (d) 16. (c) 17. (c) 18. (d) 19. (c) 20. (c) 21. (a) 22. (c) 23. (a) 24. (c ) 25. (a) 26. (b)
27. (c) 28. (c) 29. (d) 30. (a) 31. (b ) 32. (b) 33. (d) 34. (d) 35. (a ) 36. (b) 37. (c ) 38. (a) 39. (d)
40. (c) 41. (d) 42. (b) 43. (c) 44. (a) 45. (a) 46. (a ) 47. (c) 48. (d) 49. (a) 50. (c) 51. (a) 52. (a)
53. (b) 54. (c ) 55. (b) 56. (c )

Chapter_27.indd 11 6/5/2015 2:51:27 PM


27.12 Chapter 27

Explanatory Answers

Difficulty Level-1

1.
(c) Let the population of the two villages A and B will be 7.
(b) Let x and y be the number of rows and columns
equal after x years. initially.
\ 6800 – 120x = 4200 + 80x Given xy – 5 = 190, (x + 2) y = 225
⇒ 200x = 2600 Solving, we get y = 15 and x = 13.
⇒ x = 13.
8.
(a) Let cost of rice be `x per kg and cost of dal be
2.
(a) Suppose Mohan has `y and Ram has `x y per kg.
\ 2 (x – 30) = y + 30 and x + 10 = 3 (y – 10)
⇒ 2x – y = 90 and x – 3y = –40 \ 5x + 7y = 339 (1)
⇒ 6x – 3y = 270 and x – 3y = –40 and, 7x + 5y = 321 (2)
⇒ 5x = 310 ⇒ x = 62, y = 34.
Adding Eqs. (1) and (2)
3.
(b) Suppose number of 2-mark questions = x 12x + 12y = 660 or x + y = 55 (3)
Suppose number of 4-mark questions = y
x + y = 15 ⇒ x = 10, y = 5. Subtracting Eqs. (2) from (1)

4.
(b) R + P = 90 and 4R + 2P = 224 –2x + 12y = 18, y – x = 9 (4)
\ R = 22, P = 68. Adding Eqs. (3) and (4) we get
5.
(c) Suppose there were x packages originally. 2y = 64 or y = 32, \ x = 23.
x 2x 5x − 4x x 9.
(b) Let there are r rows and x students in each row.
– = 3 ⇒ =3⇒ =3
2 5 10 10
(x + 4) × (r – 4) = x × r
x = 30.
⇒ x × r – 4x + 4r – 16 = x × r
6.
(d)
⇒ – 4x + 4r = 16 (1)
Sulphuric Acid: Quantity and, (x – 2) × (r + 4) = x × r
Mixture
Water taken out ⇒ x × r + 4x – 2r – 8 = x × r
First 40% 2:3 x kg ⇒ 4x – 2r = 8 (2)
Second 60% 3:2 y kg Adding Eqs. (1) and (2),
When mixed, we get 20% strong solution 2r = 24
2 3 ⇒ r = 12
x+ y 20 \ From Eq. (2), 4x – 24 = 8
⇒ 5 5 =
x+ y+5 100 ⇒ 4x = 32
⇒ 2x + 3y = x + y + 5 or x + 2y = 5 (1) ⇒ x = 8
\ Total number of students = 8 × 12 = 96.
Again on adding 5 kg of 80% solution, we get a
70% solution 10.
(d) Let the capacity of the fuel tank be x litres.

2 3 x 3
x+ y+4 Given + 22 = x
5 5 70 5 4
⇒ =
x+ y+5 100
3 1
\  −  x = 22
⇒ 4x + 6y + 40 = 7x + 7y + 35  4 5
⇒ 3x + y = 5 (2)
11
From (1) and (2), we get y = 2 and x = 1 ⇒ x = 22
20
\ Quantity of 40% solution = 1 kg and quantity of
60% solution = 2 kg. ⇒ x = 40 litres.

Chapter_27.indd 12 6/5/2015 2:51:27 PM


Linear Equations 27.13

11.
(b) Let the total score in the innings be x. Then, x – 10 = y + 10
2 ⇒ x = y + 20 (1)
Then, highest score = x
9 x + 20 = 2(y – 20)
2 2  ⇒ x = 2y – 60 (2)
Next highest score =  x − x
9 9  From Eqs. (1) and (2),

2 2 2  y + 20 = 2y – 60
\ x−  x − x  = 8
9 9 9  ⇒ y = 80
and, x = 100.
2 8×9
⇒ x−x+ x =
9 2 17.
(a) Let the basic fare be `x and reservation charges be
8×9×9 `y.
⇒ x= = 162.
2×2 Then, x + y = 216 (1)

12.
(c) Let the total number of packages be x. x
and, (x + y) + + y = 327
2 x 2
\ x + 3 =
5 2
x
1 ⇒ + y = 111 (2)
⇒ x = 3 2
10
\ x = 30. From Eqs. (1) and (2),

13.
(c) Let the present population of male and female be x
= 105 ⇒ x = `210.
x and y respectively. 2
\ x + y = 55000 (1)
18.
(d) Let the Nikhil money = N
111x 120 y
and, + = 6330 Let the Yogesh money = Y
100 100
⇒ 111x + 120y = 633000 (2) \ N + Y = 600 (1)

On solving Eqs. (1) and (2), we get N Y


and, = (2)
y = 2500. 4 6
2Y
14.
(a) Let number of postcards and inland letters be x and y From Eq. (2), N =
respectively. 3

\ 0.30x + 1.5y = 66 (1) 2Y


From Eq. (1), + Y = 600 ⇒ Y = `360
3
and, x + y = 60 (2)
\ N = 600 – 360 = `240
On solving Eqs. (1) and (2), we get
So, required difference = 360 – 240 = `120.
x = 20, y = 40
\ Required cost price = 0.30 × 40 + 1.5 × 20 19.
(b) Let per unit cost be `x and fixed cost be `y.
= 12 + 30 = `42. 540x + y =1800 (1)
15.
(a) Let fix charges of first three days be x and charges of 620x + y = 2040 (2)
each extra days be `y.
From Eqs. (1) and (2), x = `3
Then,
Then, y = `180
x + 4y = 45 (1)
\ 500x + y = 500 × 3 + 180 = `1680.
and, x + 2y = 25 (2)
Solving Eqs. (1) and (2), we get 20.
(a) A hen has 2 legs while a cow has 4 legs and each
x = `5 and y = `10. animal has one head.

16.
(a) Let number of students in rooms A and B be x and y, Let he has x number of cows and y number of hens.
respectively. Then, x + y = 59 (1)

Chapter_27.indd 13 6/5/2015 2:51:28 PM


27.14 Chapter 27

and, 4x + 2y = 190 (2) \ 76E = 1330


From Eqs. (1) and (2), ⇒ E = `17.50.
x = 36. 26.
(b) Let each son has 2x sisters and x brothers.
Then, the number of sons in the family = (x + 1)
21.
(b) 3 Chairs + 10 Tables = `9856
And number of daughters = 2x
2 × (3 Chairs + 10 Tables) = 2 × 9856
Now given, (2x – 1) = (x + 1)
⇒ 6 Chairs + 20 Tables = `19712. x=2
22.
(d) Let he should produce atleast x units. \ Number of sons in the family = 3.
Then, (60 – 40)x – 3000 = 1000 27.
(a) Given, x + y + (y + 20) = 180
⇒ 20 x = 4000 ⇒ x + 2y = 160 (1)
\ x = 200. and, 4x – y = 10 (2)
Solving Eqs. (1) and (2), we get y = 70, x = 20
23.
(a) Let the total length of lamp post be x m.
\ Angles of the triangle are 20°, 70°, 90°. Hence the
x x 10 D is right angled.
\ x = + +
2 3 3 28.
(b) Let 4 battalions be w, x, y and z.
x 10 1 2 3 4
⇒ = w = x= y = z
6 3 2 3 4 5
⇒ x = 20 m. 3 2 5
\ x = w,y= w,z= w
24.
(c) Let the average expenditure per student per day be `x. 4 3 8
Then, total expenditure per day = `35x Also, w + x + y + z = 7300
New total expenditure per day = `(35x + 42) 3 2 5
⇒ w+ w + w + w = 7300
New average expenditure = (x – 1) 4 3 8
24 w + 18w + 16 w + 15w
Given, ⇒ = 7300
24
35 x + 42
= x – 1 7300 × 24
42 ⇒ w = = 2400
73
\ 35x + 42 = 42x – 42
2400
⇒ 7x = 42 + 42 \ Same number of men = = 1200.
2
2 × 42 29.
(d) Let the length of the side of the property be x m and
⇒ x = = `12
7 y be the number of posts bought. When the space
\ Original expenditure of mess = 35 × 12 = `420. between posts is 8 m, the number of posts
x
25.
(b) 25T + 20E + 18D + 12S = 1330 (1) = + 1 = y (1)
8
Given, 5T = 4E When the space between posts is 6 m, the number of
4 posts
⇒ T = E x
5 = + 1 – y + 5 (2)
6
12E = 9D
From Eqs. (1) and (2),
12
⇒ D = E x x (3 − 4) x
9 − = –5 ⇒ = –5
6D = 8S 8 6 24

6 6 12 −x
⇒ S = D= × E=E ⇒ = –5
8 8 9 24
\ From Eq. (1), ⇒ x = 120 m, y = 16.
4 12 30.
(c) Let constant expenses of the family be E and they buy
25 × E + 20 E + 18 × E + 12E = 1330
5 9 x amount of wheat.

Chapter_27.indd 14 6/5/2015 2:51:28 PM


Linear Equations 27.15

Then, E + x × 250 = 1000 \ x – 10 = y + 10


and, E + x × 240 = 980 2(y – 20) = x + 20
= x – y = 20
\ x = 2
x – 2y = – 60
E = 500 ⇒ y = 80, x = 100.
Total expenses = 500 + 2 × 350 = `1200.
37.
(c) Suppose cost price of one mixer and one TV are
31.
(c) Let x number of ball bearings were there in first respectively `x and `y.
shipment. According to the question,
1 4.5 2x + y = 7000 (1)
Then, x× + 2x × = 100
100 100 x + 2y = 9800 (2)
⇒ x = 1000. Multiplying (2) by 2 and subtracting it from (1),
32.
(d) Suppose the prices of audio and video cassettes are –3y = – 12600
`x and `y respectively. or, y = `4200.

2x + 3y = 425 (1) 38.


(c)
Suppose price of scooter and television are
and, 3x + 2y = 350 (2) respectively `3x and `2x.
According to the question,
Multiplying (1) by 3, (2) by 2 and subtracting,
3x – 2x = 6000 or x = 6000
5y = 575 or y = `115. Hence, price of television = `2x
33.
(b) Let the no. of buffaloes be x and the no. of ducks be y. = `2 × 6000 = `12,000.
Now, total no. of legs = 4x + 2y 39.
(a) Let number of 100 rupee notes be x and number of 50
rupee notes be y.
Total no. of heads = x + y
\ x + y = 85
According to the question,
     100x + 50y = 5000
4x + 2y = 2(x + y) + 24
or, 4x + 2y = 2x + 2y + 24 ⇒ x = 15, y = 70
\ x = 12. \ The amount that we had in the denomination of
`50 = 50y = 50 × 70 = `3,500.
34.
(c) Suppose number of rice bowls = R
Suppose number of broth bowls = B 40.
(c) Suppose the number is 10x + y.
Suppose number of meat bowls = M (10x + y) – (10y + x) = 9
\ R + B + M = 65 (1) ⇒ 9x – 9y = 9
Also with respect to guests
\ Required difference x – y = 1.
2R = 3B = 4M (2)
From Eqs. (1) and (2), 120
41.
(d) of the planned distance.
B = 20, R = 30, M =15 x
\ Total number of guests 42. (a) Let the fixed amount be `x and varying amount be `y
= 2R = 3B = 4M = 60. per unit.
35.
(c) Let the number be 10x + y. We are given,
\ x + y = 15 x + 540y = 1800 (1)
and, 10x + y + 9 = 10y + x x + 620y = 2040 (2)
⇒ 9x – 9y = –9 On solving Eqs. (1) and (2), we get
⇒ x – y = –1 x = 180 and y = 3
∴  Amount of bill for 500 units = 180 + 500 × 3 =
\ x = 7, y = 8
`1680.
\ The given number is 78.
43. (a) Let the speed of faster train be x m/s and speed of
36.
(b) Suppose number of candidates in room A = x and slower train be y m/s.
number of candidates in room B = y. We are given,

Chapter_27.indd 15 6/5/2015 2:58:10 PM


27.16 Chapter 27

240 On solving Eqs. (1) and (2), we get x = 44 and y = 136


= 60 ∴  Number of deers = x = 44.
x− y
⇒ x − y = 4 (1) 45. (d) At X, let men = 2x and women = x
240 ∴ At Y, men = 2x − 10 and women = x + 5
and, =3 But it is given that at Y, Men = Women
x+ y
⇒ x + y = 42 (2) ⇒ 2x − 10 = x + 5  ⇒  x = 15
On solving Eq. (1) and Eq. (2), we get x = 42 m/s and ∴  Passengers at X = 3x = 45.
y = 38 m/s 46. (d) Let the number of students scoring 6,8 and 20 be x, y
∴  Speed of faster train = 42 m/s. and z respectively.
44. (d) Let number of deers = x So, 6x + 8y + 20z = 504
and numbers of ducks = y x + 2z = y
Total heads counted = 180
or, 7y + 4z = 252
∴  x + y = 180 (1)
By hit and trial we get y = 32 and z = 7
Total legs counted = 448
∴ 4 x + 2 y = 448 Therefore, x = 18
Therefore, total number of students = 32 + 7 + 18 = 57.
⇒ 2 x + y = 224 (2)

Difficulty Level-2

1.
(a) x + 2y – 3z = p Putting (d) in 5x + 19y = 64;
2x + 5y – 11z = q If –59 < y < –56 i.e., y = –58, –57, then x cannot be
x – 2y + 7z = r an integer.
Going through the options, we find that Hence, (d) is also wrong.
5 × (1) – 2 × (2) = (3) Therefore, (c) is true.
Thus, equation in choice (a) is satisfied. 4.
(a) Here the equations given are
p + 5q = 21 (1)
2.
(a) Let number of standard and deluxe bags produced by
companies be s and d respectively. and, p – 2q = 7 (2)
Profit P = 20s + 30d By solving the two equations, we get p = 11 and q = 2.
Since the man wants to buy q units, i.e., 2 units at
we have to find the maximum possible value of P for
the rate of p, i.e., 11, so amount paid by him = 11 × 2
which it meets the machine availability constraints.
= `22.
4s + 5d = 700 (1)
6s + 10d = 1250 (2) 5.
(d) Assume in a kilogram of food A the quantity of fat be
x gram and quantity of protein by y gram, then x + y =
Solving Eqs. (1) and (2), we get 100 and y – 2x = 10
s = 75, d = 80. ⇒ x = 30 gram and y = 70 gram
Aliter: The question can also be answered easily by Hence, the quantity of fat is 30 gram and the
substitution according to options. quantity of protein is 70 gram.
3.
(c) 5x + 19y = 64
64 − 5 x 6.
(a) Let number of 100-rupee notes be x and number of
y = (1) 50-rupee notes be y.
19
In Eq. (1) for every integral value of x, we get the \ x + y = 85 ⇒ x = 15, y = 70
corresponding value of y. \ The amount that we had in the denomination of
Now go through the options. `50 = 50y = 50 × 70 = `3500.
(a) is wrong because this is possible that x < 300 such
7.
(a) P + R + Y = 100 (1)
that y < 0.
(b) is also wrong because if x > 250 then y may be 0.50P + 10R + 50Y = 1000 (2)
greater than –100. From Eq. (1) R = 100 – (P + Y)

Chapter_27.indd 16 6/5/2015 2:51:30 PM


Linear Equations 27.17

⇒ 0.50P + 10 (100 – P – Y) + 50Y = 1000 12.


(d) Suppose she had x apples in the beginning.
or, 0.50P + 1000 – 10P – 10Y + 50Y = 1000 x 1 x +1
Sold to the first customer = + =
or, 9.5P = 40Y 2 2 2
P 40 80 x +1 1
or, = = Sold to the second customer = +
Y 9.5 19 4 2
\ The florist should use 2x + 6 x+3
= =
80 sticks of Pink = `40 8 4
19 sticks of Yellow = `950 x+3 1
1 stick of Red = `10 Sold to the third customer = +
8 2
100 sticks = `1000.
2x + 6 + 8 x + 7
8.
(a) Originally, the lady had, say, x one-rupee notes and y = =
20-paise coins 16 8
On returning, she had y one-rupee notes and x x+7 1
Sold to the fourth customer = +
20-paise coins 16 2
1 y x + 15
The balance was `  x +  =
3 5 16
 y  x  1 y  x + 1 x + 3 x + 7 x + 15 
\   x +  −  y +  =
 x +  (1) \ x− + + + = 15
 5  5  3 5  2 4 8 16 
x 13 15 x + 49 
⇒ = x−
y 7
⇒  = 15
 16 
If we take x = 13 and y = 7, then originally she had ⇒ x = 240 + 49 = 289.
`14.40 with her 13.
(d) The two lines coincide.
⇒ On returning, she had `9.60 14.
(a) Let the number of students in halls P and Q be x and y
\ The lady spent `4.80 or a multiple of `4.80, i.e., respectively.
`14.40, which is alternative (a). Hall P Hall Q
17 x 17 x 8 x x y
9.
(b) x × − = 225 ⇒ −
8 17/8 8 17 x – 10 y + 10

⇒ 225x = 136 × 225 \ x – 10 = y + 10


⇒ x = 136. ⇒ x – y = 20 (1)

10.
(a) Let A had `x and B had `y in the beginning. Hall P Hall Q
If B gives, `400 to A, then x + 20 y – 20

5 \ x + 20 = 2 (y – 20)
x + 400 = ( y − 400) ⇒ x – 2y = –60 (2)
4
Solving Eqs. (1) and (2), we get
⇒ 4x – 5y = –3600 (1)
x = 100, y = 80.
If A gives `200 to B, then
x
7 15.
(d) Let the fraction be
y + 200 = ( x − 200) y
2
⇒ 7x – 2y = 1800 (2) x+6 1
Then, =
1.5 y 2
Solving Eqs. (1) and (2), we get
\ 2x + 12 = 1.5y ⇒ 4x – 3y = –24
x = 600, y = 1200.
x×4
11.
(b) Let there be 100 seats in the theater then initial Further, = 3
y −8
revenue = 60x
⇒ 4x = 3y – 24 or 4x – 3y = –24
 y  But the equations are same, hence the exact answer
Given,   x − 100 = 1.1 × 60x ⇒ y = 34x.
 100  cannot be determined.

Chapter_27.indd 17 6/5/2015 2:51:31 PM


27.18 Chapter 27

16.
(c) Let the 2 numbers be a and b Putting x = y – z in Eq. (3)
\ The 3 numbers in A.P. are a, A, b and the 3 numbers 2(y – z) – 3y + 4z = 0
in G.P. are a, G, b ⇒ y = 2z
a+b ⇒ The system of equations has infinitely many
In an A.P., A = solutions
2
k k
In a G.P., G2 = ab If y = k, then z = and x = .
2 2
From the first condition, we get
k k
a + b = 10, (1) ⇒ a:b:c = :k:
2 2
From the second condition, we get
or, a:b:c = 1:2:1
A2 – G2 = 1 20.
(c) Let the constant cost be C, the variable cost be K and
2 number of words exceeding 10 be N.
a +b
\   – ab = 1 The total cost = C + KN
 2 
Now, C + 5K = 3 (1)
a 2 + b 2 + 2ab and, C + 10K = 4.25 (2)
⇒ – ab = 1
4 On solving Eqs. (1) and (2), we get
⇒ a2 + b2 + 2ab – 4ab = 4 C = 1.75 and K = 0.25 ⇒ C + 25K = 8.
⇒ a2 + b2 – 2ab = 4 5x 7y
21.
(a) We have, + = 6
2 8 18
\ (a – b) = 4  or a – b = ± 2
⇒ 45x + 28y = 432 (1)
If a – b = 2, then solving this equation and Eq. (1)
simultaneously, we get, Also, 2(x – y) = –10
⇒ 2x – 2y = –10
a = 6 and b = 4
\ 28x – 28y = –140 (2)
If a – b = –2, solving this equation and Eq. (1)
Solving Eq s. (1) and (2), we get
simultaneously, we get,
x = 4 and y = 9.
a = 4 and b = 6
22.
(c) We have, (n + 4)/(d + 4) = 2 and (n – 6)/(d – 6) = 12
\  The two numbers are 4 and 6.
Solving, n = 18 and d = 7. Hence, n + d = 25.
17.
(c) Let the original amount be `x and y paise and into
23.
(a) 2g + 5t + 4p = 80 (1)
changed amount = `y and x paise
4g + 3t + 1p = 62 (2)
3(100x + y) = 100y + x – 50
\ 2 × (1) – (2) = 7t + 7p = 98
299 x + 50
y = Hence, (t + p) = 14.
97
24.
(c) Let the two number be x and y
By putting the value of x from options we get (2)
option is correct. 2x 3y x 7
+ = 23: + = 7.
3 4 7 5
18.
(d) 3x+y = 81 and 81x–y = 3
25.
(a) Let number of standard and deluxe bags produced by
⇒ 3x+y = 34 and 34(x–y) = 3
companies be s and d respectively
1
or, x + y = 4 and x – y = Profit P = 20s + 30d
4
We have to find the maximum possible value of P
1 7 for which it meets the machine availability constraints
Solving, we get x = 2 , y = 1 .
8 8
4s + 5d = 700 (1)
19.
(c) x – y + z = 0 (1)
x – 2y + 3z = 0 (2) 6s + 10d = 1250 (2)
2x – 3y + 4z = 0 (3) Solving (1) and (2), we get
From Eq. (1), x = y – z. Putting x = y – z s = 75, d = 80
(y – z) – (y) + 3z = 0 Aliter: The question can also be answered easily by
⇒  y = 2z (4) substitution according to options.

Chapter_27.indd 18 6/5/2015 2:51:31 PM


Linear Equations I 27.19

(b) Putting x = 2, 3, 4, 5, 6 in y = x2 – 3x + 2 we get y = 0,


26. 3x  y  1 2 x  y  2 3x  2 y  1
2, 6, 12, 20. 31.
(b)  
3 5 6
27.
(c) Let the man buys x apples, y mangoes and z bananas
at the respective price of `A, `M, `B per piece. As it By option method, putting the value of x = 1 and y = 1.
is given that number of mangoes bought is same as
k −1
number of bananas, hence quantity assumed is same 32.
(b) \ ≠ 0 ⇒ k ≠ 3.
as y. Let the amount spent on apples be `P, then
amount spent on mangoes and bananas together is
I6 −2 I
1.5 p 33.
(d) 2x = 8y+1
Given, p + 1.5 p = 140 ⇒ 2x = (23)y+1
⇒ 2.5p = 140 ⇒ p = 56 ⇒ x = 3y + 3 (1)
\ Amount spent on apples is `56 and amount spent y
9 = 3 x–9
on mangoes and bananas together `84
⇒ (3 ) = 3x–9
2 y
Now, xA = 56, (1)
yB + yM = 84 ⇒ 2y = x – 9 (2)

⇒ y (B + M) = 84 (2) Solving Eq. (1) and (2) we get,


Given if mangoes cost the same as apples. no banana x = 21 and y = 6
can be bought
\ x + y = 21 + 6 = 27.
(x + y) A = 140
xA + yA = 140 34.
(d) Let the numbers be a and b
From (1)   yA = 140 – 56 = 84 (3)
\ a – b = x (1)
From (2) and (3)
\ a + b = 7x (2)
yB + yM + yA = 84 + 84 = 168
ab = 24x (3)
y (A + B + M) = 168
\ Additional amount required to be spent Solving Eqs. (1) and (2) we get,
= (168 – 140) = `28. a = 4x and b = 3x
28.
(c) Let the number of rice bowls be x, number of broth Substituting the values of a and b in Eq. (3) we get,
bowls be y x=2
and number of meat bowls be z \ ab = 24 × 2 = 48.
\ x + y + z = 65 (1)
and, 2x = 3y = 4z (2) 35.
(a) Let the price of the cheapest magazine, i.e., the one at
the extreme left be x.
From Eqs. (1) and (2) we get x = 30, y = 20, z = 15
Difference, between the prices of two adjacent
Therefore total number of guests magazines is `2
= 2R = 3B = 4M = 60.
\ Price of extreme right or the costliest magazine
29.
(d) Let there are x first class ticket will be x + 30 × 2 = x + 60
\ Total cost = 10x + (18 – x)3 Now, the price of the magazine in middle (the 16th
\ 10x + 54 – 3x = 110 position) = x + 15 × 2 = x + 30
⇒ 7x = 56 The price of the magazines, adjacent to the one in
⇒ x=8 the middle is x + 28 or x + 32 depending on whether
If first class and second class tickets are interchanged, it is on the left or right of the middle magazine
then total cost = 10 × 10 + 3 × 8 = 124. respectively.

30.
(a) x + y = 100 (1) Suppose x + 60 = x + 28 + x + 30
\ x + 60 = 2x + 58
3.60x + 2.40y = 312 (2)
\ x = 2
From Eqs. (1) and (2), we get And if x + 60 = x + 30 + x + 32
y = 40. \ 60 = x + 2

Chapter_27.indd 19 6/5/2015 2:51:32 PM


27.20 I Chapter 27

\ x = –2 (which is not possible) x + 2(y + z) = 6


So, the adjacent magazine is the one whose price is ⇒ x = 2
x + 28, i.e., one to the left of the middle magazine.
From Eqs. (1) and (3)
36.
(b) Let the number of cows and chickens = x and y,
respectively. 4 y + 2z =
8
Total number of heads = x + y y + z =2×2
Total number of legs = 4x + 2y 2y =4 ⇒ y =2

Given, 4x + 2y = 2 (x + y) + 14 Then, x = 2, y = 2, z = 0
2x = 14 ⇒ x = 7
\ Customer will buy,

So, number of cows = 7.
2 portfolio-I
37.
(c) Let total number of pages = x
3x  3x  4 7 x 2 portfolio-II
Total read page = + x − × =
8  8  5 8 0 portfolio-III
7x x 40.
(c) For r = 5 and a = 1, the line will have infinite solutions,
 Unread remain page = x  =
8 8 if it coincides with the other line, which happens when
x
\ = 30 1 1 1
8 b  0 and c  0, hence, c = .
2 2 2
⇒ x = 240.
38.
(a) Originally, the lady had, say, x one rupee notes and 41.
(d) Let Ankur solved x right questions
y 20 paise coins (30 − x)
On returning, she had y one rupee notes and x 20 \ His over all score = x −
4
paise coins
1 y ⇒ 4x – 30 + x = 13.75 × 4
The balance was `  x + 
3 5 ⇒ x = l7
 y  x 1 y \ Ankur did 13 wrong questions.
\  x +  −  y +  =  x +  (1)
 5  5 3 5
x 13 42.
(b) Total working time of grinding is 120 hours and of
⇒ =
y 7 Polishing is 150 hours
If we take x = 13 and y = 7, then originally she had Let x no. of items of A has been manufactured and
`14.40 with her y no. of B have been manufactured.
⇒ On returning, she had `9.60 \ 2x + 3y = 120
\ The lady spent `4.80 or a multiple of `4.80, i.e.,
\ 3x + 2y = 150
`14.40, which is alternative (a).
\ x = 42 and y = 12.
39.
(d) Common Municipal Preferred 43.
(c) Given that for a + b + c ≠ 0
I 2 1 × x
m + 2n – 3r = a
II 4 2 3 × y
2m + 6n – 11r = b
III 2 2 3 × z

m – 2n + 7r = c
Let customer buy’s I, II and III portfolio in number
of x, y and z respectively. For the above equations to have at least one
\ 2x + 4y + 2z = 12 (1) solution, the equations should not be inconsistent, i.e.,
the relation between the LHS of three equations must
x + 2y + 2z = 6 (2) be the same as the RHS of the three equations.
and, 3y + 3z = 6 (3) Going from the choices
From Eq. (3), 5a = 5m + 10n – 15r, 2b = 4m +12n – 22r
and, c = m – 2n + 7r
y + z = 2
By observation, we can see that,
From Eq. (2), 5a = 2b + c.

Chapter_27.indd 20 6/5/2015 2:51:32 PM


Linear Equations 27.21

44.
(a) Let A had `x and B had `y in the beginning. Case 2 If we do not use 3, then the arrangements
should take into account that 0 cannot be the first digit
If B gives, `400 to A, then
as a 5 digit number will not start with 0.
5 The first digit can be any of the 4 digits 1, 2, 4 or 5.
x + 400 = ( y − 400)
4 Then, the remaining 4 digits including 0 can be
⇒ 4x – 5y = –3600 (1) arranged in the other 4 places in 4! ways.
If A gives `200 to B, then So, there will be 4 × 4! number = 4 × 24 = 96
numbers.
7
y + 200 =
( x − 200) Combining Case 1 and 2, there are a total of 120 + 96
2
= 216 five-digit numbers divisible by 3 that be formed
⇒ 7x – 2y = 1800 (2)
using the digits 0 to 5.
Solving Eqs. (1) and (2), we get
48. (d) 53 x − 35 x = 540
x = 600, y = `1,200. ⇒ 18 x = 540 or, x = 30
45.
(a) Let the husband’s age be x. Therefore, new product = 53 × 30 = 1590.
Let the wife’s age be y. 49. (a) Let the first, second, third and fourth digits be a, b, c
Let the daughter’s age be z. and d, respectively
According to questions, Then,
x + 2y + 3z = 85 (1) a+b =c+d (1)
2x + 4y + 6z = 170 (2) a+d =c (2)
5x + 10y + 15z = 450 (3) b + d = 2(a + c) (3)
From Eq. (2), x + 2y + 3z = 85 From Eqs (1) and (2), a + b = a + 2d ⇒ b = 2d
From Eq. (3), x + 2y + 3z = 90 From Eq (3), 2d + d = 2 ( a + a + d )
From Eq. (1), x + 2y + 3z = 85
d
Hence, the above system of equation will give no ⇒ 3d = 2(2a + d ) ⇒ d = 4a or, a =
4
solution.
46.
(a) Let he buys K pieces of Kajuburfi, G pieces of d 5d
Now from Eq. (ii), a + d = +d = =c
Gulabjamun and S pieces of Sandesh. 4 4
K + G + S = 100 5
or, c =d
S 4
and, 10K + 3G + = 100 The value of d can be either 4 or 8. If d = 4, then c = 5.
2
If d = 8, then c = 10. But the value of c should be less
⇒ S = 100 – K – G
than 10. Hence, value of c would be 5.
and, S = 200 – 20K – 6G
50. (c) P, Q and R are three consecutive odd numbers, hence
or, 200 – 20K – 6G = 100 – K – G
or, 100 = 19K + 5G Q = P + 2 and, R = P + 4
Given: 3P = 2( P + 4) − 3
100 − 19 K
or, G = ⇒ P = 5.
5
Hence, R = 5 + 4 = 9.
⇒ K = 5 and G = 1
S = 100 – 5 – 1 = 94 7 7  126 − 56 
51. (a) x − x = 770 ⇒   x = 770
47. (c) There are six digits viz, 0, 1, 2, 3, 4 and 5. To form 8 18  144 
five-digit numbers we need exactly 5 digits. So, we ⇒ x = 1584.
should not be using one of the digits.
52. (a) Let one number be x, then second number will be
  The sum of all the digits 0, 1, 2, 3, 4 and 5 is 15. We (x + 4).
know that any number is divisible by 3 if and only if
1 1 10
the sum of its digits are divisible by 3. ∴ + =
x ( x + 4) 21
  Combining the two criteria that we use only 5 of
the 6 digits and pick them in such a way that the sum x + x + 4 10
or =
is divisible by 3, we should not use either 0 or 3 while x( x + 4) 21
forming the five digits numbers. 2 x + 4 10
⇒ =
Case 1 If we do not use 0, then the remaining 5 digits x( x + 4) 21
can be arranged in 5! ways = 120 numbers.
⇒ x = 3.

Chapter_27.indd 21 6/5/2015 2:51:36 PM


27.22 Chapter 27

53. (b) Quantity of rice left after the first transaction 55. (b) We have, | x + 7 | + | x − 8 | = 16
 x 1 x 1 Put x + 7 = 0 ⇒ x = −7
= x− +  = −
2 2 2 2 Also put x − 8 = 0 ⇒ x = 8
Quantity of rice left after second transaction
 x 1  x 1 1 x 3 −7 8
= − − − +  = −
2 2 4 4 2 4 4
Case-I: When x ≤ −7, then
Quantity of rice left after third transaction
| x + 7 | + | x − 8 |= 16
 x 3  x 3 1 x 7
= − − − +  = − ⇒ − x − 7 − x + 8 = 16
 4 4 8 8 2 8 8
⇒ x = −7.5
x 7
Given, − = 0 ⇒ x = 7. Case-II: When − 7 < x < 8, then
8 8
| x + 7 | + | x − 8 |= 16
54. (c) Let the man buys x apples, y mangoes and z bananas
at the respective price of `A, `M, `B, for each unit ⇒ x + 7 − x + 8 = 16
respectively ⇒ 15 = 16, which is not possible
As it is given that number of mangoes bought is same Case-II: When x ≥ 8, then
as the number as of bananas, hence quantity assumed | x + 7 | + | x − 8 | = 16
is same as y
⇒ x + 7 + x − 8 = 16
Let the amount spent on apples be `P
⇒ x = 8.5.
Therefore, the amount spent on mangoes and bananas
together is 1.5 P
Hence, sum of the possible values of x = -7.5 + 8.5 = 1.
Now, P + 1.5 P = 140 or, 2.5 P = 140 or P = 56
Amount spent on apples is ` 56 and the amount spent 56. (c) Smallest number = X
on mangoes and bananas together is `84
Step Amitabh Sashi
Again, x A = 56 (1)
1 2X 2X + 50
and, y B + y M = 84
2 4X + 100 4X + 150
or, y (B + M) = 84 (2)
3 8X + 300 8X + 350
If mangoes cost the same as apples, no bananas can be
bought. i.e., (x + y)A = 140 4 16X + 700 16X + 750
or xA + yA = 140 5 32X + 1500 32X + 1550
From (1), yA = 140 − 56 = 84 (3) Amitabh has to win and X is the least possible number
From (2) and (3), we have in the range 1-999
or, yB + yM + yA = 84 + 84 = 168 ∴  Step 4 has to be the last step
y (A + B + M) = 168 ⇒  16X + 170 > 1000
Additional amount required to be spent The least possible value of X = 16
= (168 − 140) = `28. Sum of the digit = 1 + 6 = 7.

Chapter_27.indd 22 6/5/2015 6:20:37 PM


CHAPTER

Quadratic Equations 28
INTRODUCTION ⇔ (2x + 1) (3x + 2) = 0
An equation of degree two is called a quadratic equation. 1 2
⇔ x = or, x = –
The general form of a quadratic equation is ax2 + bx + c = 0, 2 3
where a, b, c are real numbers, a ≠ 0 and x is a real variable. (ii) Using formula:
Some examples of quadratic equations are x2 + 4x + 3 = 0,
3x2 – 4x + 5 = 0 and 3x2 + 2x – 3 = 0. −1 ± (1) 2 − 4(2)( −1)
The roots are x =
2× 2
Roots of a Quadratic Equation −1 ± 9 −1 ± 3
= =
A root of the equation f (x) = 0 is that value of x which 4 4
makes f (x) = 0. In other words, x = a is said to be a root of
2 −4 1
f (x) = 0, where f (a) is the value of the polynomial f (x) at = , i.e., , –1
x = a and is obtained by replacing x by a in f (x). 4 4 2
For example, –1 is a root of the quadratic equation Using factorization:
x2 + 6x + 5 = 0 because (–1)2 + 6(–1) + 5 = 0. 2x2 + x – 1 = 0 ⇔ 2x2 + 2x – x – 1 = 0
⇔ 2x(x + 1) – 1(x + 1) = 0
Solution of a Quadratic Equation ⇔ (2x – 1) (x – 1) = 0
If there is a quadratic equation ax2 + bx + c = 0, a ≠ 0, the 1
⇔ x = or, x = –1
roots of this equation are 2
−b + b 2 − 4ac −b − b 2 − 4ac
and . Nature of Roots
2a 2a
A quadratic equation has exactly two roots may be real or
Illustration 1 Solve the following quadratic equations imaginary or coincident.
(i) 6x2 + x – 2 = 0 If ax2 + bx + c, a ≠ 0, then D = b2 – 4ac is called
(ii) 2x2 + x – 1 = 0 discriminant.
Solution: (i) Using formula:
1. If D > 0, then there are two distinct and real roots
−1 ± (1) 2 − 4(6)(−2) given by
The roots are x =
2×6
−b + b 2 − 4ac −b − b 2 − 4ac
−1 ± 49 6 −8 a= ,b= .
= = , 2a 2a
12 12 12
1 −2 2. If D = 0, then there is a repeated real root given by
i.e., ,
2 3 b
a=– i.e., roots are real and equal.
Using factorization: 2a
6x2 + x – 2 = 0 ⇔ 6x2 + 4x – 3x – 2 = 0 3. If D < 0, then there are no real roots.
⇔ 2x (3x + 2) – 1 (3x + 2) = 0

Chapter_28.indd 1 6/5/2015 3:47:01 PM


28.2 Chapter 28

2. ax2 + bx + c can be expressed as a product of two


Note: linear factors only when D ≥ 0
The roots are rational if D > 0 and D is a perfect square Illustration 5 For what value of k, the quadratic polynomial
whereas the roots are irrational if D > 0 but D is not a kx2 + 4x + 1 can be factorized into two real linear factors
perfect square.
Solution: D = (4)2 – 4 × k × 1 = 16 – 4k.
Illustration 2 Find the nature of the roots of the equations The given quadratic polynomial can be factorized into
real linear factors if D ≥ 0
(i) 2x2 + x – 1 = 0
i.e., 16 – 4k ≥ 0 or, – 4k ≥ – 6 or k ≤ 4
x2 + x + 1 = 0
(ii)
(iii) x2 + 5x + 5 = 0
Relation Between Roots and Coefficients
4 3
(iv) x2 – 2x + = 0 Let a, b be the roots of the equation,
3 4
ax2 + bx + c = 0
Solution: (i) D = (1)2 – 4 × 2 × (–1) = 9 > 0 Then, sum of the roots
Also, D is a perfect square. b coefficient of x
= a + b = – =–
So, the roots are real, distinct and rational. a coefficient of x 2
D = (1)2 – 4 × 1 × 1 = –3 < 0
(ii) and product of the roots
So, the roots are imaginary.
c constant term
D = (5)2 – 4 × 1 × 5 = 5 > 0
(iii) = ab = =
a coefficient of x 2
Also, D is not a perfect square.
So, the roots are real, distinct and irrational. Illustration 6  Find the sum and the product of the roots of
the quadratic equation 2x2 + 5 3 x + 6 = 0
4 3
D = (–2)2 – 4 ×
(iv) × =0
3 4 Solution: Here, a = 2, b = 5 3 , c = 6
So, the roots are real and equal. b 5 3
\ Sum of the roots = – =–
a 2
Illustration 3 For what value of k will the quadratic
equation kx2 – 2 5 x + 4 = 0 have real and equal roots c 6
Product of the roots = = =3
a 2
Solution: D = (–2 5 )2 – 4 × k × 4 = 20 – 16k
The given equation will have real and equal roots if Formation of a Quadratic Equation with Given Roots
D=0
If a, b are the roots of a quadratic equation the equation can
20 5 he written as
i.e., 20 – 16k = 0 or k = =
16 4
x2 – (a + b) x + ab = 0
Note: i.e., x2 – (sum of roots)x + product of roots = 0

1. If p + q is a root of a quadratic equation, then its other Illustration 7  Find the quadratic equation whose roots are
5 and –6
root is p – q.
Solution:   Sum of roots = 5 + (– 6) = – 1,
Illustration 4  If 2 + 3 is one root of a quadratic equation,    Product of roots = 5 × (– 6) = –30
find the other root \ The required quadratic equation is
Solution: The other root is 2 – 3 x2 – (–1)x + (– 30) = 0 i.e., x2 + x – 30 = 0

Chapter_28.indd 2 6/5/2015 3:47:01 PM


Quadratic Equations 28.3

Practice Exercises

Difficulty Level-1
(Based on Memory)

1. If a and b are the roots of the quardratic equation ax2 + bx If a, b are the roots of X2 – 8X + P = 0 and a2 + b2 = 40,
8.
α 2 β2 then the value of P is:
+ c = 0, then the value of + is:
β α (a) 8 (b) 10
(c) 12 (d) 14
2bc − a3 3abc − b3
(a) (b)
b 2c a 2c A man is 18X years old and his son is 2X2 years old. When
9.
he was 3X2 years old, his son was X + 4 years old. How
3abc − b 2 ab − b 2c old is he now?
(c) (d)
a3c 2b 2c (a) 68 years (b) 70 years
[Based on MAT, 2003] (c) 72 years (d) 74 years
2. If a, b are the two roots of a quadratic equation such that a 10. In a family, eleven times the number of children is greater
+ b = 24 and a – b = 8, then the quadratic equation having than twice the square of the number of children by 12.
a and b as its roots is, How many children are there?
(a) x2 + 2x + 8 = 0 (b) x2 – 4x + 8 = 0
(a) 3 (b) 4
(c) x2 – 24x + 128 = 0 (d) 2x2 + 8x + 9 = 0
(c) 2 (d) 5
[Based on MAT, 2003]
11. For what values of k, the equation
3.
One-fourth of a herd of cows is in the forest. Twice the
square root of the heard has gone to mountains and on the x2 + 2 (k – 4) x + 2k = 0 has equal roots?
remaining 15 are on the banks of a river. The total number (a) 8, 2 (b) 6, 4
of cows is: (c) 12, 2 (d) 10, 4
(a) 6 (b) 100 [Based on IIFT, 2003]
(c) 63 (d) 36 12. The number of quadratic equations which are unchanged
[Based on MAT, 2003] by squaring their roots is:
4. If the roots of the equation (a) 2 (b) 4
(c2 – ab)x2 – 2 (a2 –bc) x + (b2 – ac) = 0 (c) 5 (d) 6
for a ≠ 0 are real and equal, then the value of a3 + b3 + c3 is: [Based on FMS (Delhi), 2002]
(a) abc (b) 3abc 13. If a and b are the two roots of the equation 2x2 – 7x – 3 =
(c) zero (d) None of these 0, then find the value of (a + 2)(b + 2).
[Based on MAT, 2003] (a) 9 (b) –9.5
5. If 2x2 – 7xy + 3y2 = 0, then the value of x:y is: (c) 9.5 (d) 6
(a) 3:2 (b) 2:3 [Based on SCMHRD, 2002]
(c) 3:1 and 1:2 (d) 5:6 Given that a, g are the roots of the equation Ax2 – 4x + 1
14.
[Based on MAT, 2003] = 0 and b, d are the roots of the equation Bx2 – 6x + 1 = 0,
6. If a and b are the roots of the equation x2 + 2x – 1 = 0 and then the values of A and B, respectively, such that a, b, g
g and d are the roots of the equation x2 + 3x – 4 = 0, then and d are in H.P.
find the value of (a + g) (b + d) (a + d) (b + g). (a) –5, 9 (b) 3/2, 5
(a) –46 (b) –24 (c) 3, 8 (d) None of these
(c) 0 (d) –64
15.
A class decided to have a party for their class at a total cost
7.
Find the quadratic equation whose roots are a and b, of `720. Four students decided to stay out of the party. To
1 1 meet the expenses the remaining students have to increase
given that a + and b + are the roots of the equation their share by `9. What is the original cost per student?
β α
14x2 – 45x + 81 = 0. (a) `18 (b) `24
(a) 2x2 – 8x + 7 = 0 (b) 2x2 – 5x + 7 = 0 (c) `36 (d) `20
2
(c) 2x – 8x + 5 = 0 (d) 2x2 – 5x + 5 = 0 [Based on MAT (May), 2010]

Chapter_28.indd 3 6/5/2015 3:47:02 PM


28.4 Chapter 28

16.
Students of a class are made to stand in rows. If 4 students
13 + 3 13 − 3
are extra in each row, there would be 2 rows less. If 4 (a) (b)
students are less in each row, there would be 4 more rows. 2 2
The number of students in the class is: 15 + 3 15 − 3
(c) (d)
(a) 90 (b) 94 2 2
(c) 92 (d) 96 25. For what values of k, the equation x2 + 2 (k – 4) x + 2k = 0
[Based on MAT (Feb), 2006] has equal roots?
(a) 8, 2 (b) 6, 4
The solutions of the 25 − x 2 = x – 1 equation are:
17.
(c) 12, 2 (d) 10, 4
(a) x = 3 and x = 4 (b) x = 5 and x = 1
(c) x = –3 and x = 4 (d) x = 4 and x = –3 If the sum of the roots of the quadratic equation px2 + qx +
26.
[Based on MAT, 1999]
r = 0 is equal to the sum of the square of their reciprocals,
mark all the correct statements.
18.
The value of x satisfying the equation
(a) r/p, p/q and q/r are in AP
2 x + 3 + 2 x − 1 =is:
2 (b) p/r, q/p and r/q are in GP
(a) 3 (b) 2 (c) p/r, q/p and r/q are in HP
1 (d) p/r, q/p and r/q are in AP
(c) 1 (d) [Based on ITFT, 2006]
2
[Based on MAT, 1999] If one root of the equation ax2 + bx + c = 0 is double of the
27.
19.
One-fourth of a herd of cows is in the forest. Twice the other, then 2b2 is equal to:
square root of the heard has gone to mountains and on the (a) 9 ca (b) c 2 a
remaining 15 are on the banks of a river. The total number
(c) 2 3 ac (d) None of these
of cows is:
[Based on ITFT, 2008]
(a) 6 (b) 100
28. One of the roots of the equation x2 – x + 3m = 0 is double
(c) 63 (d) 36 of one of the roots of the equation x2 – x + m = 0. If m ≠ 0,
20.
A positive number when decreased by 4, is equal to 21 then find its value.
times the reciprocal of the number. The number is: (a) 1 (b) –1
(a) 3 (b) 5 (c) 2 (d) –2
(c) 7 (d) 9 29. If 4 is a solution of the equation x2 + 3x + k = 10, where k
[Based on MAT, 2000] is a constant, what is the other solution?
21.
For which value of k does the following pair of equations (a) –18 (b) –7
yield a unique solution for x such that the solution is (c) –28 (d) None of these
positive? [Based on ATMA, 2006]
x2 – y2 = 0 The coefficient of x in the equation x2 + px + p = 0 was
30.
(x – k)2 + y2 = 1 wrongly written as 17 in place of 13 and the roots thus found
(a) 2 (b) 0 were – 2 and – 15. The roots of the correct equation would be:
(a) –4, –9 (b) –3, –10
(c) 2 (d) − 2
(c) –3, –9 (d) –4, –10
The sum of all the roots of 4x3 – 8x2 – 63x – 9 = 0 is:
22. [Based on ATMA, 2006]
(a) 8 (b) 2 31. If the roots, x1 and x2, of the quadratic equation x2 – 2x +
(c) –8 (d) –2 c = 0 also satisfy the equation 7x2 – 4x1 = 47, then which
[Based on FMS, 2011] of the following is true?
(a) c = –15 (b) x1 = 5, x2 = 3
23.
The solution of 5 x  1  x  1 = 2 is:
(c) x1 = 4.5, x2 = –2.5 (d) None of these
(a) x = 1 (b) x = 2
2 If −2 3 is a root of the equation x2 + px – 6 = 0 and the
32.
(c) x = (d) x = 2, x = 1
3 [Based on FMS, 2011] equation x2 + px + q has equal roots, then the value of q is:
1 3
24. Let y = . What is the value of y? (a) (b) − 3
1 4
2+
1 4
3+ (c) (d) 3
1
2+ 3
3 + ... [Based on ATMA, 2008]

Chapter_28.indd 4 6/5/2015 3:47:02 PM


Quadratic Equations 28.5

At what value of x it will give the minimum value of x2 –


33. 39. If the roots of the equation (a 2 + b 2 ) x 2 + 2(b 2 + c 2 ) x + (b 2 +
4x + 7?
c 2 ) = 0 are real, which of the following must hold true?
(a) x ≥ 3 (b) x = 2
(c) x < 2 (d) x = 0 (a) c 2 ≥ a 2 (b) c 4 ≥ a 2 (b 2 + c 2 )
[Based on ATMA, 2008] (c) b 2 ≥ a 2 (d) a 4 ≤ b 2 (a 2 + c 2 )
34.
Which of the following equations has a root in common [Based on CAT, 2012]
with x2 – 6x + 5 = 0? 40. If p is a prime number and m is a positive integer, how
(a) x2 + 1 = 0 (b) x2 – 10x – 5 = 0 many solutions exist for the equation p 6 − p = (m 2 + m +
2
(c) x – 2x – 3 = 0 (d) 2x2 – 2 = 0 6)( p − 1)?
[Based on ATMA, 2008] (a) 0 (b) 1
(c) 2 (d) Infinite
35. If (x – 3) (2x + 1) = 0, then possible value of 2x + 1 are:
[Based on CAT, 2012]
(a) 0 only (b) 0 and 3
41. Three consecutive positive integers are raised to the first,
1 second and third powers, respectively and then added. The
(c) − and 3 (d) 0 and 7
2 sum so obtained is a perfect square whose square root equals
the total of the three original integers. Which of the following
36. If the roots of the quadratic equation y2 + My + N are N best describes the minimum, say m, of these three integers?
and M, then find the possible number of pairs of (M, N). (a) 1 ≤ m ≤ 3 (b) 4 ≤ m ≤ 6
(a) 0 (b) 1 (c) 7 ≤ m ≤ 9 (d) 10 ≤ m ≤ 12
(c) 2 (d) 3 (e) 13 ≤ m ≤ 15
[Based on, 2008]
[Based on CAT, 2009]
42. Two teams participating in a competition had to take a test
37. A manager is not used to work in the decimal system. She in a given time. Team B chose the easier test with 300
says that there are 100 employees in the office of which 2 questions, and teams A difficult test with 10% less
are men and 32 are women. Which number system does questions. Team A completed the test 3 hours before
the manager use? schedule while team B completed it 6 hours before
(a) 4 (b) 6 schedule. If team B answered 7 questions more than team A
per hour, how many questions did team A answer per hour?
(c) 8 (d) 16 [Based on MAT, 2013]
[Based on CAT, 2009]
(a) 15 (b) 18
38. The equation 2 x 2 + 2( p + 1) x + p = 0, where p is real, (c) 21 (d) 24
always has roots that are: 43. A polynomial ax 3 + bx 2 + cx + d intersects x-axis at 1
(a) Equal and −1 and y-axis at 2. The value of b is:
(b) Equal in magnitude but opposite in sign (a) –2 (b) 0
(c) Irrational (c) 1 (d) 2
(d) Real (e) Cannot be determined
[Based on CAT, 2010] [Based on XAT, 2014]

Difficulty Level-2
(Based on Memory)

Let p and q be the roots of the quadratic equation x2 –


1. (a) 10 (b) 7
(a – 2) x – a – 1 = 0. What is the minimum possible value (c) 6 (d) 12
of p2 + q2? [Based on CAT, 2004]
(a) 0 (b) 3
3.
The angry Arjun carried some arrows to fight Bheeshma
(c) 4 (d) 5
Pitamaha. Hiding behind Shikhandi with half the arrows,
[Based on CAT, 2003]
he perished the arrows thrown by Pitamaha on him and
2.
If both a and b belong to the set {1, 2, 3, 4} then the with six other arrows he killed the chariot driver (sarathi)
number of equations of the form ax2 + bx + 1 = 0 having of Pitamaha. Then with one arrow each, he knocked down
real roots is: the chariot, the flag on the chariot and the bow of Pitamaha

Chapter_28.indd 5 6/5/2015 3:47:06 PM


28.6 Chapter 28

and finally with one more than four times the square root 14. If a is an integer, then for how many integer values of n
of the arrows, he laid down the Pitamaha unconscious on can the quadratic equation x2 – (2a + 3) x + 4n = 0 have
the arrow bed. Assuming he used all the arrows with him, real and equal roots for x?
the total number of arrows with Arjun was: (a) 0 (b) 1
(a) 4 (b) 49 (c) 2 (d) 3
(c) 100 (d) 144
15. If p, r are positive and are in AP, the roots of quadratic
4.
If f (x, y, z) = sum of z terms of the sequence x, x + y, x + equation px2 + qx + r = 0 are real for:
2y ..., what is the value of z if f (1, 1, z) = 21? r p
(a)  − 7 ≥ 4 3 (b) −7 ≥4 3
(a) 3 (b) 4 p r
(c) 6 (d) 7 (c) all p and r (d) no p and r
5. If l, m, n are real and l = m, then the roots of the equations 16.
The sum of all the real roots of the equation
(l – m)x2 – 5(l + m)x – 2(l – m) = 0 are: 2
x − 2 + x − 2 − 2 = 0 is:
(a) Real and equal (b) Complex
(a) 2 (b) 3
(c) Real and unequal (d) None of these
(c) 4 (d) None of these
6. Which of the following expressions cannot be equal to
zero, when X2 – 2X = 3? Let a, b, c be real, if ax2 + bx + c = 0 has two real roots a,
17.
(a) X2 – 7X + 6 (b) X2 – 9 c b
2 b, where a < – 1 and b > 1, then the value of 1 + +
(c) X – 4X + 3 (d) X2 – 6X + 9 is: a a
[Based on SCMHRD Ent. Exam., 2003]
(a) less than zero (b) greater than zero
Directions (Q. 7 to 12): In each of these questions, two (c) equal to zero (d) equal to b2 – 4ac
equations I and II are given. You have to solve both the equations 18.
The values of a for which the quadratic equations (1 – 2a)
and give answer (a), if p < q (b), if p ≤ q; (c), if p = q, (d), if x2 – 6ax – 1 = 0 and ax2 – x + 1 = 0 have at least one root
p ≥ q and (e), if p > q. in common are:
I. p2 – 18p + 77 = 0
7. 1 2 1
(a) , (b) 0,
2 9 2
II. 3q2 – 25q + 28 = 0 [Based on IRMA, 2002]
2 1 2
(c) (d) 0, ,
I. 6q2 + q – 1 – 0
8. 9 2 9
II. 6p2 – 7p + 2 = 0 [Based on IRMA, 2002] If the roots of ax2 + bx + c = 0, a > 0, be each greater than
19.
unity, then:
I. 7p2 + 6p – 1 = 0
9.
(a) a + b + c = 0 (b) a + b + c > 0
II. 32q2 – 20q + 3 = 0 [Based on IRMA, 2002] (c) a + b + c < 0 (d) None of these
I. 4p2 = 9
10. 20.
If n is such that 36 ≤ n ≤ 72, then
2
II. 2q – 9q + 10 = 0 [Based on IRMA, 2002] n 2 + 2 n (n + 4) + 16
x= satisfies
n+4 n +4
I. 2p2 – 12p + 16 = 0
11.
(a) 20 < x < 54 (b) 23 < x < 58
II. q2 – 9q + 20 = 0 [Based on IRMA, 2002]
(c) 25 < x < 64 (d) 28 < x < 60
12. If is a root of the equation x2 + px + q = 0, where p and q 21. A telecom service provider engages male and female
are real, then (p, q) is: operators for answering 1000 calls per day. A male operator
(a) (2, 3) (b) (–2, 3) can handle 40 calls per day whereas a female operator can
(c) (4, 7) (d) (–4, 7) handle 50 calls per day. The male and the female operators
[Based on FMS (Delhi), 2002]
get a fixed wage of `250 and `300 per day respectively. In
addition, a male operator gets `15 per call he answers and a
13. A quadratic function f (x) attains a maximum of 3 at x = 1. female operator gets `10 per call she answers. To minimize
The value of the function at x = 0 is 1. What is the value of the total cost, how many male operators should the service
f (x) at x = 10? provider employ assuming he has to employ more than 7
(a) –110 (b) –180 of the 12 female operators available for the job?
(c) –105 (d) –159 (a) 15 (b) 14
[Based on CAT, 2007] (c) 12 (d) 10

Chapter_28.indd 6 6/5/2015 3:47:07 PM


Quadratic Equations 28.7

The inequality of p2 + 5 < 5p + 14 can be satisfied, if:


22.
x 2  bx m  1
(a) p ≤ 6, p = –1 (b) p = 6, p = –2 30. If  has roots which are numerically equal
ax  c m  1
(c) p ≤ 6, p ≤ 1 (d) p ≥ 6, p = 1 but of opposite signs, the value of m must be:
[Based on SNAP, 2008]
ab ab
23.
Each boy contributed rupees equal to the number of girls (a) (b)
ab ab
and each girl contributed rupees equal to the number of
boys in a class of 60 students. If the total contribution thus (c) c (d) 1/c
[Based on FMS, 2011]
collected is `1600, how many boys are there in the class?
31.
In solving a problem that reduces to a quadratic equation
(a) 30 (b) 25 one student makes a mistake only in the constant term of the
(c) 50 (d) Data inadequate equation and obtains 8 and 2 for the roots. Another student
[Based on FMS (MS), 2006] makes a mistake only in the coefficient of the first degree term
If the ratio between the roots of the equation lx2 + mx + n
24. and finds – 9 and – 1 for the roots. The correct equation is:
(a) x2 – 10x + 9 = 0 (b) x2 + 10x + 9 = 0
p q n 2
= 0 is p : q, then the value of + + is: (c) x – 10x +16 = 0 (d) x2 – 8x – 9 = 0
q p l [Based on FMS, 2011]
(a) 4 (b) 3 32.
The values of y which will satisfy the equations,
(c) 0 (d) –1 2x2 + 6x + 5y + 1 = 0
[Based on FMS, 2005] 2x + y + 3 = 0
may be found by solving:
25.
Vaidya and Vandana solved a quadratic equation. In solving
(a) x2 + 14y – 7 = 0 (b) y2 + 8y + 1 = 0
it, Vaidya made a mistake in the constant term and obtained 2
the roots as 6 and 2, while Vandana made a mistake in the (c) y + 10y – 7 = 0 (d) y2 + y – 12 = 0
coefficient of x only and obtained the roots as –7 and –1. [Based on FMS, 2011]
The correct roots of the equation are: x+a x+b
33. If the roots of the equation + = 1 are
(a) 6, 1 (b) 7, 2 x+a+c x+b+c
equal in magnitude but opposite in sign, then:
(c) 6, 2 (d) 7, 1
(a) c ≥ a (b) a ≥ c
[Based on FMS, 2006]
(c) a + b = 0 (d) a = b
If x is a number satisfying the equation 3 x  9  3 x  9
26. 34. If one root is the square of the other root in the equation x2 +
= 3, then x2 is between: px + q = 0, mark the correct relationship in the following
(a) 55 and 65 (b) 65 and 75 options.
(c) 75 and 85 (d) 85 and 95 (a) p3 – q(3p + i) + q2 = 0
[Based on FMS, 2010] (b) p3 – q(3p – 1) + q2 = 0
(c) p3 + q(3p – 1) + q2 = 0
27.
The number of real values of x satisfying the equation
2
(d) p3 – q(3p – 1) – q2 = 0
22 x  7 x  5  1 [Based on ITFT, 2006]
(a) 1 (b) 2 Find the root of the quadratic equation bx2 – 2ax + a = 0
35.
(c) 4 (d) More than 4 b a
(a) (b)
[Based on FMS, 2010] b  ab b  ab
28.
Which of the following sets of x-values satisfy the
inequality 2x2 + x < 6 a a
(c) (d)
3 3 a  ab a  ab
(a) 2  x  (b) x  or x   2
2 2 [Based on IIFT, 2010]
3 3 If the common factor of px2 + qx + r and qx2 + px + r is
36.
(c) x  (d)  x  2
2 2 (x + 2), then:
[Based on FMS, 2010] (a) p = q or p + q + r = 0 (b) p = r or p + q + r = 0
For what value (s) of k does the pair of equations y = x2
29. (c) q = r or p + q + r = 0 (d) p = q = r
and y = 3x + k have two identical solutions: [Based on XAT, 2006]
4 9 For which value of non-negative ‘a’ will the system x2 –
37.
(a)  (b)
9 4 y2 = 0, (x – a)2 + y2 = 1 have exactly three real solutions?
9 9 9 (a) – 2 (b) 1
(c)  (d) or 
4 4 4 (c) 2 (d) 2
[Based on FMS, 2010] [Based on XAT, 2007]

Chapter_28.indd 7 6/5/2015 3:47:07 PM


28.8 Chapter 28

If 0 < p < 1, then roots of the equation (1 – p)x2 + 4x + p =


38. (I) x2 + 22x + 115 = 0
0 are ...? (II) 2x2 + 44x + 115 = 0
(a) both 0 (b) imaginary (III) x2 – 22x – 115 = 0
(c) real and both positive (d) real and both negative (IV) x2 – 22x + 115 = 0
[Based on XAT, 2008] (a) I only (b) II only
39.
The number of possible real solutions of y in equation (c) III only (d) IV only
y2 – 2y cos x + 1 = 0 is ...? [Based on ATMA, 2008]
(a) 0 (b) 2 3 2
42. If a, b and c are roots of x – 6x + 11x – 6 = 0 and the roots
(c) 1 (d) 3 of the equation x3 – px2 + qx – r = 0 are a + b, b + c and c
[Based on XAT, 2008] + a, then r equals:
(a) 40 (b) 50
Let a and b be the roots of the quadratic equation x2 + 3x –
40. (c) 60 (d) 70
1 = 0. If Pn = an + bn for n ≥ 0, then, for n ≥ 2, Pn is equal
 [Based on JMET, 2009]
to:
(a) –3Pn–1 + Pn–2 (b) 3Pn–1 + Pn–2 43. A man covers a certain distance on a toy train. If the train
moved 4 Km/h faster, it would take 30 minutes less. If it
(c) –Pn–1 + 3Pn–2 (d) Pn–1 + 3Pn+1
moved 2 Km/h slower, it would have taken 20 minutes
[Based on XAT, 2009]
more. What is the distance covered?
115 (a) 65 Km (b) 60 Km
41.
If one of the roots of a quadratic equation is ,
11 + 6 (c) 70 Km (d) 75 Km
then the quadratic equation must be:
 [Based on MAT, 2013]

Answer Keys
Difficulty Level-1
1. (b) 2. (c) 3. (d) 4. (b) 5. (c) 6. (a) 7. (b) 8. (c) 9. (c) 10. (b) 11. (a) 12. (a) 13. (c)
14. (c ) 15. (c) 16. (d) 17. (d) 18. (d) 19. (d) 20. (c) 21. (c) 22. (b) 23. (a) 24. (d ) 25. (a) 26. (a, c)
27. (a) 28. (d) 29. (b) 30. (b) 31. (a) 32. (a) 33. (b) 34. (d) 35. (d ) 36. (c) 37. (b ) 38. (d) 39. (a)
40. (b) 41. (a) 42. (b) 43. (a)

Difficulty Level-2
1. (d) 2. (b) 3. (c) 4. (c) 5. (c) 6. (a) 7. (d) 8. (c) 9. (a) 10. (a) 11. (b) 12. (d) 13. (d)
14. (b ) 15. (b) 16. (c) 17. (c) 18. (c) 19. (b) 20. (c) 21. (d) 22. (c) 23. (d) 24. (a ) 25. (d) 26. (c)
27. (b) 28. (a) 29. (c) 30. (a) 31. (a) 32. (c) 33. (a) 34. (b) 35. (c ) 36. (a) 37. (b ) 38. (d) 39. (b)
40. (a) 41. (c) 42. (c) 43. (b)

Explanatory Answers

Difficulty Level-1

b c (α + β)(α 2 + β2 − αβ)
1.
(b) a + b = − , ab = =
a a αβ
a2 + b2 = (a + b)2 – 2ab
b2 2c b2 − 2 ac 2
 b   b − 2 ac c 
= −  =  − a   − 
a 2 a a2    a2 a 
=
α2
β 2
α3 + β3 c
\ + = a
β α αβ

Chapter_28.indd 8 6/5/2015 3:47:08 PM


Quadratic Equations 28.9

[Since g and d are the roots of the equation


−b (b2 − 3ac) a
= × x2 + 3x – 4 = 0]
a3 c 2 2
= (g + 3g – 4 – 5g + 3) (d + 3d – 4 – 5d + 3)
3abc − b3 = (–5g + 3) (–5d + 3)
= .
a2c = (25gd + 9 – 15 (d + g))
2.
(c) a + b = 24, a – b = 8 ⇒ a = 16, b = 8 = (–100 + 9 + 45) = – 46.
\ ab = 128
 1  1  45
\ Required equation is the one whose sum of the 7.
(b) Given that  α +  +  β +  =
 β  α  14
roots is 24 and product of the roots is 128
i.e., x2 – 24x + 128 = 0.  1  1  81
and,        α +  ×  β +  =
3.
(d) Suppose total number of cows = x  β  α  14
1 [(α + β) 2 − 2αβ] 45
 x of the cows are in forest, ⇒   (a + b) + =
4 αβ 14
2 x have gone to mountains and 15 are on the banks 1 81
of a river. ⇒ ab + + 2 =
αβ 14
1 Solve for ab and (a + b), we get
\ 2 x+ x + 15 = x
4 5 7
a + b = and ab =
3 2 2

⇒ 2 x − x + 15 = 0
4 Therefore, 2x2 – 5x + 7 = 0.
⇒ 8 x − 3 x + 60 = 0 8.
(c) We have, a + b = 8, ab = P
a + b2 = (a + b)2 – 2ab
2
⇒ 3( x )2 − 8 x − 60 = 0
= 64 – 2P = 40 (given)
⇒ 3( x )2 − 18 x + 10 x − 60 = 0 ⇒ P = 12.
⇒ 3 x ( x − 6) + 10 ( x − 6) = 0 9.
(c) By the condition given in question
10 3X 2 – 18X = X + 4 – 2X2
⇒ x = 6, x =– 2
3 ⇒ 5X – 19X – 4 = 0
⇒ x = 36. ⇒ (5X + 1) (X – 4) = 0
4.
(b) Discriminant 1
⇒ X = 4 or X = –
= [2(a2 – bc)]2 – 4 (c2 – ab) (b2 – ac) = 0 5
for the roots to be equal So, the present age of man = 18 × 4 = 72 years
\ a4 + b2c2 – 2a2bc – c2b2 + ac3 + ab3 – a2bc = 0 Note: Can be directly obtained from options.
⇒ a3 – 2abc + c3 + b3 – abc = 0 Man’s present age is given as 18X years.
⇒ a3 + b3 + c3 = 3abc. Therefore the answer should be divisible by 18 and
from options only 72 is divisible by 18.
x2 x
5.
(c) 2 −7 + 3 = 0 10.
(b) Let the number of children in the family be X.
y2 y
Given: 11X – 12 = 2X2 ⇒ 2X2 – 11X + 12 = 0
x 7 ± 49 − 24 Solving the quadratic equation, we get
⇒ =
y 2×2
3
7±5 1 X = or X = 4
= = 3, . 2
4 2 3
   Number of children can not be . Hence the
2
6.
(a) (a + g) (b + d) (a + d) (b + g)
number of children is 4.
= [(a + g) (b + g)] [(b + d) (a + d)]
(a) For any quadratic equation, ax2 + bx + c = 0, to have
11.
= [ab + g(a + b) + g2] [ab + d(a + b) + d2]
equal roots,
= (–1 – 2g + g2) (–1 – 2g + d2) b2 – 4ac = 0

Chapter_28.indd 9 6/5/2015 3:47:08 PM


28.10 Chapter 28

⇒ [2 (k – 4)]2 – 4 × 2k = 0 15.


(c) Let original cost per student be `x.
2
⇒ (k – 4) – 2k = 0 720
\ Total number of students =
⇒ k + 16 – 8k – 2k = 0 x
⇒ k2 – 10k + 16 = 0  720 
 − 4  × ( x + 9) = 720
⇒ k = 8, 2.  x 
(a) Let the equation be ax2 + bx + c = 0
12. 6480
⇒ 720 – 4 x + – 36 = 720
Suppose its roots are a and b. x
⇒ x2 + 9x – 1620 = 0
b
Then, a + b = – (1) ⇒ x2 + 45x – 36x – 1620 = 0
a
⇒ (x + 45) (x – 36) = 0
c ⇒ x = `36 (Q x ≠ 45)
ab = (2)
a
16.
(d) Let no. of rows be x and no. of students in each row
b be n.
Also, a2 + b2 = – (3) Then, total no. of students = xn
a
Again, (n + 4)(x – 2) = (n – 4)(x + 4) = xn
c
a2b2 = (4) ⇒ n = 12 and x = 8
a
\ No. of students = 12 × 8 = 96.
From (2) and (4), we get
⇒ ab = 1 or c = a 17.
(d) 25 − x 2 = x – 1
From (1), we get ⇒ 25 – x2 = (x – 1)2
b2 ⇒ 25 – x2 = x2 + 1 – 2x
a2 + b2 + 2ab =
a2 ⇒ 2x2 – 2x – 24 = 0
⇒ x2 – x – 12 = 0
b b2
⇒ − + 2 = 2 ⇒ (x – 4) (x + 3) = 0
a a
⇒ x = 4, x = –3.
⇒ b2 + ab – 2a2 = 0 1
18.
(d) x = satisfies the given equation.
2
− a ± a + 8a 2 2
or, b = = a, –2a. 19.
(d) Suppose total number of cows = x
2
1
7  x of the cows are in forest,
13.
(c) a + b = 4
2
−3 2 x have gone to mountains and 15 are on the banks
ab = of a river.
2
\ (a + 2) (b + 2) = ab + 2a + 2b + 4 1
\ 2 x+ x + 15 = x
= ab + 2 (a + b) + 4 4
3 7 3
= + 2× + 4 ⇒ 2 x− x + 15 = 0
2 2 4
−3 19 ⇒ 8 x − 3 x + 60 = 0
= + 11 = = 9.5.
2 2
⇒ 3( x ) 2 − 8 x − 60 = 0
14.
(c) Let us consider choice (a). When we put the values of
2
A and B respectively, we get the values of a, b, g and ⇒ 3( x ) − 18 x + 10 x − 60 = 0
d as – 1, 1/3, 1/5, 1/3, which are not in H.P. So, this
option is not correct. ⇒  3 x ( x − 6) + 10 ( x − 6) = 0
Now for our convenience we consider choice (c), 10
then by substituting the values of A and B, we get the ⇒ x = 6, x = −
value of a, b, g and d as 1, 1/2, 1/3 and 1/4 which are 3
in H.P. Hence, this could be the correct choice. ⇒ x = 36.

Chapter_28.indd 10 6/5/2015 3:47:09 PM


Quadratic Equations 28.11

20.
(c) Suppose the positive number is x. or, 2y2 + 7y = 3 + y or, 2y2 + 6y – 3 = 0
According to the question,
− 6 ± 36 + 24
1 \ y =
x – 4 = 21 – 4
x
or, x2 – 4x = 21 − 3 ± 15
=
or, x2 – 4x – 21 = 0 2
or, x(x – 7) + 3(x – 7) = 0 15 − 3
or, (x – 7)(x + 3) = 0 As the contained fraction is positive, y = .
2
\ x = 7 or x = –3
(a) For any quadractic equation, ax2 + bx + c = 0, to have
25.
Since x is positive equal roots,
\ x = 7. b2 – 4ac = 0 ⇒ [2 (k – 4)]2 – 4 × 2k = 0
21.
(c) y2 = x2 ⇒ (k – 4)2 – 2k = 0
2 2 ⇒ k + 16 – 8k – 2k = 0
2x – 2kx + k – 1 = 0
D = 0 ⇒ k2 – 10k + 16 = 0
⇒ 4k2 = 8k2 – 8 ⇒ 4k2 = 8 ⇒ k = 8, 2.
⇒ k = 2. 26.
(a, c) Option (a) is correct.
Coefficient of x 2 Option (b) is incorrect.
22.
(b) Sum of the roots = −
Coefficient of x3 Option (c) as P, Q, R in AP their reciprocals are in
HP.
(−8)
= =2. Option (d) is wrong as these terms in HP, not in AP.
4
Hence, Options (a) and (c) are correct.
23.
(a) 5 x − 1 + x − 1 = 2
27.
(a) Let the roots of the given equation are a and b.
5x − 1 = 2 – x − 1 Given, a = 2b (1)

Squaring both sides, Given equation be x2 + bx + c = 0
5x – 1 = 4 + x – 1 – 2 x − 1 b
Sum of the roots (a + b) = 
a
4x – 4 = – 2 x − 1 c
and product of roots (a × b) =
⇒ 2x – 2 = – x − 1 a
Squaring both sides, b
From a + b = 
4x2 + 4 – 8x = x – 1 a
4x2 – 9x + 5 = 0 ⇒ (4x – 5) (x – 1) = 0
b
5 2 b + b =  [by Eq. (1)]
x = or, x = 1 a
4
b b
5 ⇒ 3b =  ⇒b= 
x= does not satisfy the original equation. a 3a
4
1 b2
24.
(d) Since y = b2 = (2)
1 9a 2
2+
1 c
3+ Now, a × b =
1 a
2+
3 + ... c c
2b × b = ⇒ 2b2 =
1 a a
⇒ y =
1 c
2+ ⇒ b2 = (3)
3+ y 2a

3+ y b2 c
⇒ y = From Eqs. (2) and (3) 
6 + 2y + 1 9a 2 2a

Chapter_28.indd 11 6/5/2015 3:47:10 PM


28.12 Chapter 28

9a 2  c ⇒ 6 = 2 3 p
2b2 =  9ca
a 6
2
\ p =  3
\ 2b = 9ca. 2 3
Another given equation is
28.
(d) Let a, b be the roots of the equation 2
x + p x + q = 0
x2 – x + m = 0 ⇒ x2 + 3 x + q = 0 (2)
\ a + b = 1, ab = m (1)
Let 2a, g be the roots of the equation Also, given that Eq. (2) has equal roots let a be the
x2 – x + 3m = 0 root of Eq. (2)
\ 2a + g = 1, 2ag = 3m (2) b  3
Then, sum of roots = = =  3
m a 1
(1) ⇒ α + = 1
α i.e., (a + a) =  3
2
(2) ⇒ a – a + m = 0 (3)
 3
3m \ a =
(3)
⇒ 2α + = 1 2

c
⇒ 4a2 – 2a + 3m = 0 (4) and product of roots =
a
Equations (3) and (4)
q
⇒ m = 2a ⇒ a × a =
1
\ 2ag = 3m ⇒ g = 3
a2 = q
\ 2a + g = 1 ⇒ a = –1
2
a + b = 1 ⇒ b = 2   3
\ q =  [by Eq. (3)]
\ a = –1, b = 2, g = 3 ⇒ m = –­2.  2 
(b) Equation x2 + 3x + k = 0 puting x = 4
29. 3
\ q = .
16 + 12 + k = 0 4
k = –28 (b) Suppose y = x2 – 4x + 7
33.
By option method, put x = –7 dy
For minimum value of y, must be 0.
2 2
x + 3x + k = (–7) + 3(–7) – 28 dx
= 49 – 21 – 28 d 2 d ( x) d (7)
i.e., (x ) − 4 + =0
= 0 satisfy equation. dx dx dx
⇒ x = –7. ⇒ 2x – 4 + 0 = 0
⇒ 2x = 4 ⇒ x = 2.
(b) Equation x2 + px + q = 0
30.
If coefficient of x was wrong the product = 30 34.
(d) x2 – 6x + 5 = 0
2
\ Roots of correct equation are –3, –10 → 30. ⇒ x – 5x – x + 5 = 0
⇒ x (x – 5) – 1 (x – 5) = 0
31.
(a) We have, 7x2 – 4x1 = 47
⇒ (x – 5) (x – 1) = 0
and, x1 + x2 = 2
⇒ x = 5 or, 1
On solving, 11x1 = 55 or x1 = 5
\ Roots of given equation are 5 and 1.
\ x2 = – 3
Now, roots of equation (4)
\ c = – 15. i.e., 2x2 – 2 = 0
32.
(a) Given that 2 3 is a root of ⇒ 2x2 = 2, x2 = 1
x2 + p x – 6 = 0 (1) \ x = ± 1
\ 2 3 must satisfy Eq. (1) Which is common with root of Eq. (1).
Put x = 2 3 in Eq. (1), we have 35.
(d) Given, (x – 3) (2x + 1) = 0
( 2 3) 2  p ( 2 3)  6 = 0 Then, (x – 3) = 0
⇒ x = 3
⇒ 12 – 2 3 p – 6 = 0 and, (2x + 1) = 0

Chapter_28.indd 12 6/5/2015 3:47:11 PM


Quadratic Equations 28.13

If x = 3, then (2x + 1) = 7 Since, the three integers are positive, the value of ‘n’
\ Possible values of (2x + 1) are 0 and 7. cannot be equal to 1, therefore the value of n = 4 or m
=n-1=3
36. (c) As M and N are the roots of y2 + My + N = 0,
Hence, three consecutive integers are 3, 4 and 5
M + N = − M and MN = N
Hence, option (a) is the correct choice.
MN = N ⇒ N = 0 or M = 1 42. (b) Number of questions for team A = 300
If N = 0, then M = − M ⇒ M = 0 Also, number of questions for team B 90% of 300
If M = 1, then N = −2 M ⇒ N = −2 90 × 300
= = 270
That is two ( M , N ) pairs (0, 0) and (1, − 2) are 100
possible. Now, let the questions attempted per hour by A = x
37. (b) Let the required number system be N, then Then, questions attempted per hour by B = x + 7
(100)N = (24)N + (32)N We are given,
270 300
N2 = 2N + 4 + 3N + 2 = 5N + 6 +3= +6
x x+7
⇒  N2 − 5N − 6 = 0
⇒  (N + 1)(N − 6) = 0 270 300
= +3
So, N = 6 as −1 is not possible. x x+7
270 300 + 3 ( x + 7)
38. (d) We have, =
x x+7
a = 2, b = 2( p + 1) and c = p.
270 ( x + 7) = (300 + 3 x + 21) x
Therefore, the discriminant is
[2( p + 1)]2 − 4 ⋅ 2 ⋅ p 270 x + 1890 = 321x + 3 x 2
= 4( p + 1) 2 − 8 p 3 x 2 + 51x − 1890 = 0
2
= 4[( p + 1) − 2 p ] x 2 + 17 x − 630 = 0
2
= 4[( p + 2 p + 1) − 2 p ]
x 2 + 35 − 18 x − 630 = 0
= (4 p 2 + 1)

x( x + 35) − 18( x + 35) = 0
For any real value of p, 4( p 2 + 1) will always be
positive as p 2 cannot be negative for real p. ( x − 18)( x + 35) = 0
Hence, the roots of the quadratic equation are real. ⇒ x = 18, − 35
39. (a) Since the roots of the given equation are real, So, team A attempted 18 questions per h.
therefore
( ∴ Negative value of x is not permitted)
(2(b 2 + c 2 )) 2 − 4(a 2 + b 2 )(b 2 + c 2 ) ≥ 0
⇒ (b 2 + c 2 ) − (a 2 + b 2 ) ≥ 0 43. (a) ax 3 + bx 2 + cx + d intersect x-axis at 1 and −1
Hence when x = 1 or –1 then
⇒ c2 ≥ a2.
ax 3 + bx 2 + cx + d = 0
40. (b) We have, ∴ a + b + c + d = 0 (1)
6
p −p − a + b − c + d = 0 (2)
= m2 + m + 6
p −1 on adding (1) and (2), we get
5 4 3 2
⇒ p + p + p + p + p = m(m + 1) + 6 2 (b + d ) = 0
⇒ b+d =0
41. (a) Let the three consecutive positive integers be
Since ax 3 + bx 2 + cx + d intersects y -axis at 2
(n − 1), n and (n + 1)
Given, n + 1 + n 2 + (n + 1)3 = (3n) 2 Hence when x = 0, then ax 3 + bx 2 + cx + d = 0
⇒ n 3 + 4n 2 + 4n = 9n 2 ∴0 + d = 2
⇒ n 2 − 5n + 4 = 0 ⇒ d =2
⇒ n = 1 or n = 4
∴ b = −2.

Chapter_28.indd 13 6/5/2015 3:47:17 PM


28.14 Chapter 28

Difficulty Level-2

−{−(α − 2)} + 400 ⇒ x2 – 104x + 400 = 0 ⇒ (x – 4) (x – 100) = 0


1.
(d) Sum of the root = p + q = ⇒ x = 4 or x = 100. x has to be greater than 4 because
1
Arjun killed the chariot driver of Pitamaha with six
= a – 2 arrows. Hence, x = 100.
Product of the roots
4.
(c) f (1, 1, z) = 1 + 2 + 3 + ... + z terms
−(α + 1) z ( z + 1)
= pq = = – (a + 1) Given, = 21
1 2
⇒ z2 + z – 42 = 0
Now, p2 + q2 = (p + q)2 – 2pq
or, (z + 7) (z – 6) = 0
= (a – 2)2 – 2 (–1) (a + 1)
\ z = 6.
= a2 – 4a + 4 + 2a + 2
(c) Here, B2 – 4AC = 25 (l + m)2 + 8(l – m)2 > 0
5.
= a2 – 2a + 6
⇒ Roots are real and unequal.
We have to find the minimum possible value of
a2 – 2a + 6. 6.
(a)
X2 – 2X – 3 = (X – 3) (X + 1)
2
D = (–2) – 4 × 1 × 6 = –20 (a) ⇒ X2 – 7X + 6 = (X – 6) (X – 1)
and coefficient of a2 is +ve (b) ⇒ X2 – 9 = (X + 3) (X –­3)
(c) ⇒ X2 – 4X + 3 = (X – 3) (X – 1)
\ Rough diagram of a2 – 2a + 6 is
(d) ⇒ X2 – 6X + 9 = (X – 3)2.

7.
(d) p2 – 18p + 77 = 0
⇒ p = 11, p = 7
3q2 – 25q + 28 = 0
⇒ 3q2 – 21q – 4q + 28 = 0
⇒ 3q (q – 7) – 4 (q – 7) = 0

⇒ (3q – 4) (q – 7) = 0
−D −(−20) 20 4
\ minimum value = = = = 5. ⇒ q = , 7.
4a 4/1 4 3
(b) ax2 + bx + 1 = 0 has real roots if b2 – 4a ≥ 0
2. \ p ≥ q.

Corresponding (c) 6q2 + q – 1 – 0


8.
Value of a value of b for No. of ways ⇒ 6q2 + 3q – 2q – 1 = 0
which b2 – 4a ≥ 0 ⇒ 3q (2q + 1) – (2q + 1) = 0
1 2, 3, 4 3 ⇒ (3q – 1) (2q + 1) = 0
2 3, 4 2 1 −1
⇒ q = ,
3 4 1 3 2
4 4 1 6p2 – 7p + 2 = 0
Total no. of ways = 7. 2
⇒ 6p – 4p – 3p + 2 = 0
3.
(c) Let Arjun has x arrows. According to the given ⇒ 2p (3p – 2) – (3p – 2) = 0
1 1 2
condition x + 6 + 1 + 1 + 1 + 1 + 4√x = x ⇒ 8√x ⇒ p = , p =
2 2 3
– 20. Squaring both the sides, we get 64x = x2 – 40x \ p > q.

Chapter_28.indd 14 6/5/2015 3:47:17 PM


Quadratic Equations 28.15

9.
(a) 7p2 + 6p – 1 = 0 f (0) = 1 ⇒ c = 1, a = –2
2 f (x) = –2x2 + 4x + 1
⇒ 7p + 7p – p – 1 = 0
⇒ 7p (p + 1) – (p + 1) = 0 f (10) = –2(10)2 + 4(10) + 1
1 = –159.
⇒ p= , p = –1
7 14.
(b) Let the equal roots be m and m
32q2 – 20q + 3 = 0 ⇒ b2 – 4ac = 0
⇒ 2
32q – 12q – 8q + 3 = 0 ⇒ (2a + 3)2 = 4n+1
⇒ 4q (8q – 3) – 1 (8q – 3) = 0 ⇒ 2a + 3 = 2n+1 or, –(2n+1)
3 1 Only possible solution for (a, n) are (–1, –1) or
⇒ q = , (–2, –1) So, only one possible value of n exists.
8 4
\ p < q. 15.
(b) p, q r are in A.P.
p+r
3 q = [ p + r = 2p]
10.
(a) 2
4p = 9 ⇒ p = ± 2
2
For the real roots q2 – 4pr ≥ 0
2q2 – 9q + 10 = 0
2
⇒ 2q2 – 5q – 4q + 10 = 0  p+r
⇒   – 4pr
⇒ q (2q – 5) – 2 (2q –­5) = 0  2 
5
³ 0 ⇒ p2 + r2 – 14pr ≥ 0
⇒ q = ,2
2 2
 p  p
\ p < q. ⇒     – 14   + 1 ≥ 0
2 r
(b) 2p2 – 12p + 16 = 0
11. p 
2
⇒       − 7  ≥ 48
⇒ p2 – 6p + 8 = 0 r 
⇒ (p – 4) (p – 2) = 0 p
⇒ p = 4, 2
⇒       –7 ≥4 3.
r
q2 – 9q + 20 = 0
(c) The given equation is | x – 2 |2 + | x – 2 | = 0
16.
⇒ q = 5, 4
Let us assume | x – 2 | = m
\ p £ q.
then, m2 + m – 2 = 0
⇒ (m – 1) (m + 2) = 0
12.
(d) If 2 + i 3 is a root the equation
Only admissible value is
x2 + px + q = 0,
m = 1 [m ≠ –2 as m ≥ 0]
then its other root will be 2 − i 3 \ | x – 2 | = 1
⇒ x – 2 = 1 ⇒ x = 3
\ x2 + px + q = ( x − 2 − i 3)( x − 2 + i 3)
or, –(x – 2) = 1 ⇒ x = 1
= ( x − 2)2 − (i 3)2 Hence, x = 1, 3
= x2 + 4 – 4x + 3 = x2 – 4x + 7 \ Sum of the roots of equation = 1 + 3 = 4.
\ p = – 4, q = 7. 17.
(c) Assume some values of a, b conforming the basic
constraints of the problem.
13.
(d) f (x) = ax2 + bx + c e.g., a = –2, b = 8, then the equation becomes
f attains a maximum at x = 1 x2 – 6x – 16
f (x) = 0 ⇒ b = –5 and c = –16
−b
⇒ 2ax + b = 0 ⇒ x = = 1 ⇒ b = –2a c b
2a \ 1+ + = 1 – 16 + 6 = –9
max f (x) = 3 a a
a + b + c = 3 \ The value of the expression is negative, hence
c – a = 3 choice (a) is correct.

Chapter_28.indd 15 6/5/2015 3:47:18 PM


28.16 I Chapter 28

o Note:
Since a < 1 and b > 1
21.
(d) There are two equations to be formed


40 m + 50 f = 1000
250 m + 300 f + 40 × 15 m + 50 × 10 × f = A
c 850 m + 8000 f = A
\ ab < 1 ⇒ < 1
a m and f are the number of males and females A is
Further the product of any two numbers (n1, n2 amount paid by the employer.
≠ 0) is less then the sum of the number if any one of Then, the possible value of f = 8, 9, 10, 11, 12
them is negative.
If, f = 8
So,  ab < a + b (Q Here ab is negative)
m = 15
c b
\         - 1-1
a
<
a If f = 9, 10, 11 then m will not be an integer while
f = 12, then m will be 10.
c b
but is numerically greater than 22.
(c)
a a
c b 23.
(d) Let number of boys = x
⇒      1 + + < 0.
- 1-1
a a Girls = 60 – x
According to question,
1
18.
(c) For a = 0 or a = one of the quadratic equation 2x(60 – x) = 1600
2 2
x – 60x – 800 = 0
1
becomes linear, So, a ≠ 0, a ≠ . ⇒ On solving, we get two values of x but we cannot
2
determine which value is that of boys and which value
2 is that of girls, hence data is inadequate.
Hence, the only answer is a = .
9 (a) lx2 + mx + n = 0
24.
2
19.
(b) Let f (x) = ax + bx + c. Since 1 lies outside the roots Let us consider x2 + 4x + 4 = 0
of f (x) = 0. So,
p n
af (1) > 0 ⇒ f (1) > 0 ( a > 0) Then, = 1 and =4
q l
⇒ a + b + c > 0.
p q n
\   = 4.
1 1 4 
2
.r
(c) n + 2 n (n + 4) + 16
20. q p l

25.
(d) a + b = 6 + 2 = 8
= n 2 + 2n n + 8 n + 16 .r .r a × b = 7
= n.r.r
n ( n + 2) + 8(.r n + 2) \ a = 7 and b = 1.
= n + 8) (.r
(.r n + 2) 26.
(c) Let, a = x + 9 and b = x – 9
= (n.rn + 2)[(.r 3
n ) + (2) ]3
\ The given equation is,

= (.r
n + 2) (.rn + 2) (n − 2 n + 4) 1 1
a 3  b 3 = 27

n + 2.r
2
n (n + 4) + 16
\ x = Cubing both the sides we get,
n + 4.r n+4 1 1 1 1

(.rn + 2) (n − 2.r
2
n + 4) a  b  3a 3 b 3  a 3  b 3  = 3

=
(.rn + 2) 2 1 1
⇒ a  b  9a 3 b 3 = 27
= n − 2.rn+4
1 1
r
If n = 36 then x = 36 − 2 36 + 4 = 28
⇒ x  9  x  9  9( x  9) 3 ( x  9) 3 = 27

If n = 72 then x = 72 − 2r72 + 4 ≈ 59 1 1

⇒ 25 < x < 64. ⇒ 18  9( x  9) 3 ( x  9) 3 = 27


Chapter_28.indd 16 6/5/2015 3:47:19 PM


Quadratic Equations I 28.17

1 1 31.
(a) The student obtained the roots as 8 and 2, when he
⇒ 9( x  9) 3 (x  9) 3 =9 made a mistake only with the constant term, i.e., the
coefficient of x that he obtained was correct.
1 1
Thus, the correct sum of the roots = 10
⇒ ( x  9) 3 ( x  9) 3 = –1
In the same way, the correct value of the constant
⇒ (x + 9) (x – 9) = –1 term is the product of the roots =(–9)(–1) = 9
⇒ x2 – 81 = – 1 Thus, the quadratic equation = x2 – 10x + 9 = 0.
2
⇒ x = 80
(c) 2x2 + 6x + 5y + 1 = 0
32. (1)
⇒ 75 < x2 < 85.
2x + y + 3 = 0 (2)
2 x2  7 x 5 0 In the options, all the equations involved have
27.
(b) 2 = l = 2
only y in them. So, we take x in terms of y from one
⇒ 2x2 – 7x + 5 = 0
equation and substitute it in the other. From Eq. (2),
⇒ 2x2 – 5x – 2x + 5 = 0
 y + 3
⇒ x(2x – 5) – 1(2x – 5) = 0 x = −  
 2 
⇒ (2x – 5) (x – 1) = 0 Substituting the value of x in Eq. (1),
x = 1 or 5/2 2 ( y + 3) 2 6 ( y + 3)
So, there are two real values of x which satisfy the − + 5 y + 1 = 0
4 2
equation.
y2 + 6 y + 9
28.
(a) 2x2 + x < 6 − 3 ( y + 3) + 5 y + 1 = 0
2
⇒ 2x2 + x – 6 < 0
2
y2 + 6y + 9 + 4y – 16 = 0
⇒ 2x + 4x – 3x – 6 < 0
⇒ x2 + 10y – 7 = 0.
⇒ 2x (x + 2) – 3(x + 2) < 0
⇒ (2x – 3)(x + 2) < 0 33.
(a) a = –b, or a + b = 0
\ – 2 < x < 3/2. Use discriminant,  D = b2 – 4ac.
34.
(b) Since, one root is the square of the other root in
29.
(c) y = x2 and y = 3x + k
equation
\ x2 = 3x + k
x2 + px + q = 0
⇒ x2 – 3x – k = 0
\ p3 – q(3p – 1) + q2 = 0.
This equation has two identical solutions when the
discriminant of the equation is 0. (c) Given, bx2 – 2ax + a = 0
35.
\ (–3)2 – 4(– k) = 0 The roots are
⇒ 9 + 4k = 0 2a  4a 2  4ab
x =
\ k = – 9/4. 2b

30.
(a) a  a 2  ab a  a 2  ab
= and
b b
x 2  bx m 1
=
ax  c m 1 a  a 2  ab
Consider = , rationalise this
2
(x – bx)(m + 1) = (ax – c)(m – 1) b
x2m + x2 – bxm – bx = axm – ax – cm + c
a  a 2  ab a  a 2  ab a 2  (a 2  ab)
x2(m + 1) + x(a – am – bm – b) + cm – c = 0  =
b a  a 2  ab b (a  a 2  ab )
Since, the roots are numerically equal but opposite
in sign, so the sum of the roots will be zero. a
     
=
a – am – bm – b = 0
a  a 2  ab
– m(a + b) = b – a
ab a
m = .      
=
ab a  ab

Chapter_28.indd 17 6/5/2015 3:47:19 PM


28.18 Chapter 28

Hence, the roots are 115


41.
(c) Given one root is
a a 11 + 6
and
a  ab a  ab
115  11 − 6 
⇒ ×  
The most likely answer is option (c). 11 + 6  11 − 6 
36.
(a) Since x + 2 is a factor of both the polynomials.
115 (11 − 6)
Put, x + 2 = 0 ⇒ x = –2 ⇒ ⇒ 11 – 6
121 − 6
2
Let, f (x) = px + qx + r
\ Other root is = 11 + 6
and, g(x) = qx2 + px + r
\ Equation will be
\ f (–2) = 0 ⇒ p(–2)2 + q (–2) + r = 0
x2 – (11 – 6 + 11 + 6)x
⇒ 4p – 2q + r = 0 (1)
and, g (–2) = 0 ⇒ q (–2)2 + p(–2) + r = 0 + (11 – 6 ) (11 + 6)=0
⇒ x2 – 22x + (121 – 6) = 0
⇒ 4q – 2p + r = 0 (2)
⇒ x2 – 22x + 115 = 0
Adding Eqs. (1) and (2), we get p + q + r = 0
Subtracting Eq. (2) from Eq. (1), we get p = q (c) Roots of the equation x3 – 6x2 + 11x – 6 = 0 are 1, 2
42.
and 3
37.
(b) Given, x2 – y2 = 0
\ Roots of the equation x3 – px2 + qx – r are 3, 4
⇒ x2 = y2 (1)
and 5
and, (x – a)2 + y2 = 1 \ r = 60 [Q constant term be the product of roots].
⇒ (x – a)2 + x2 = 1
43. (b) Let the distance covered be d km and speed of train be
⇒ 2x2 – 2ax + a2 – 1 = 0
x Km/h
For x to be real, D > 0 We are given,
4a2 – 8(a2 – 1) > 0
d d 30
⇒ 4a2 – 8a2 + 8 > 0 − =
x x + 4 60
⇒ 4a2 < 8 d ( x + 4 − x) 1
⇒ =
⇒ a2 < 2 x( x + 4) 2
As a is non-negative, a = 1. ⇒ 8d = x 2 + 4 x (1)
38.
(d)     (1 – p)x2 + 4x + p = 0 d d 20
and − =
x − 2 x 60
b  b 2  4ac 4  42  4(1  p ) p
x= =  x − x + 2 1
2a 2(1  p ) ⇒ d =
 x ( x − 2)  3
\ Roots of the equation will be real and both
negative. ⇒ 6d = x 2 − 2 x (2)
(b) y2 – 2y cos x + 1 = 0
39.
From Eqs. (1) and (2), we get
Now, D = (– 2 cos x)2 – 4 = –4 sin2 x < 0
Hence, no real solution of y exists. x2 + 4x x2 − 2x
=
40.
(a) Here, a + b = –3 and a × b = –1 8 6
2 2
⇒ 6 x + 24 x = 8 x − 16 x
\ P2 = a2 + b2 = (a + b)2 – 2ab
⇒ 2 x 2 − 40 x = 0
= 9 – (–2) = 11
∴ 2 x( x − 20) = 0
Now go through options
∴ x = 20 Km/hh
For (a) –3Pn–1 + Pn–2 = Pn

⇒ P2 = –3(P1) + P0 (20) 2 − 2 × 20
Hence, distance covered =
= –3(a + b) + (a0 + b0) 6
= 60 Km.
= –3(–3) + 1 + 1 = 9 + 2 = 11.

Chapter_28.indd 18 6/5/2015 3:47:21 PM


CHAPTER

Progressions 29
Introduction (iii) 2, 6, 18, 54, ..., 1458
In this chapter, we will be concerned with the study of 1 1 1
(iv) 1, , , , ...
sequences, i.e., special types of functions whose domain is 2 4 8
the set N of natural numbers. We will study particular types of
(v) 1, 4, 9, 16, ...
sequences called arithmetic sequences, geometric sequences
and harmonic sequences and also their corresponding series. We observe that each term (except the first) in (i) is
Premiums on life insurance, fixed deposits in a bank, formed by adding 2 to the preceding term; each term in (ii)
loan instalments payments, disintegration or decay of is formed by subtracting 3 from the preceding term; each
radioactive materials and the like are some of the examples term in (iii) is formed by multiplying the preceding term
where the concept of sequence and series is used. by 3; each term in (iv) is formed by dividing the preceding
term by 2; each term in (v) is formed by squaring the next
Sequence natural number. Thus, each of (i) to (v) is a progression.
A sequence is a function whose domain is the set N of Moreover, (i) and (iii) are finite sequences, whereas (ii), (iv)
natural numbers and range, a subset of real numbers or and (v) are infinite sequences.
complex numbers. However, to define a sequence we need not always
A sequence whose range is a subset of real numbers is have an explicit formula for the nth term. For example, for
called a real sequence. Since we will be dealing with real the infinite sequence 2, 3, 5, 7, 11, 13, 17, ... of all positive
sequences only, we will use the term ‘sequence’ to denote a prime numbers, we may not be able to give an explicit
‘real sequence’. formula for the nth term.

Notation Series
The different terms of a sequence are usually denoted by By adding or subtracting the terms of a sequence, we
a1, a2, a3, ... or t1, t2, t3, ... The subscript (always a natural obtain a series. A series is finite or infinite according as the
number) denotes the position of the term in the sequence. number of terms in the corresponding sequence is finite or
The number occurring at the nth place of a sequence, i.e., tn infinite.
is called the general term of the sequence.
Illustration 2 The following
Note :
(i) 3 + 5 + 7 + 9 + ... + 21
A sequence is said to be finite or infinite (accordingly as
finite or infinite number of terms it has.) (ii) 8 + 5 + 2 + (–1) + ...
(iii) 2 + 6 + 18 + 54 + ... + 1458
Progressions
1 1 1
If the terms of a sequence follow certain pattern, then the (iv) 1 + + + + ...
sequence is called a progression. 2 4 8
Illustration 1  Consider the following sequences: (v) 1 + 4 + 9 + 16 + ...
(i) 3, 5, 7, 9, ..., 21 are the series corresponding to the above sequences, (i)
(ii) 8, 5, 2, ­­–1, –4, ... to (v).

Chapter_29.indd 1 6/5/2015 4:25:34 PM


29.2 Chapter 29

ArItHMetIc ProgreSSIon (A.P.) Illustration 5 A sequence < tn > is given by the formula
A sequence whose terms increase or decrease by a fixed tn = 10 – 3n. Prove that it is an A.P.
number is called an arithmetic progression. The fixed Solution: We have
number is called the common difference of the A.P. tn = 10 – 3n ⇒ tn+1 = 10 – 3(n + 1) = 7 – 3n.
In an A.P., we usually denote the first term by a, the \ tn+1 – tn = (7 – 3n) – (10 – 3n) = –3,
common difference by d and the nth term by tn. Clearly, d =
tn – tn–1. Thus, an A.P. can be written as which is independent of n and hence a constant.
Therefore, the given sequence < tn > is an A.P.
a, a + d, a + 2d, ..., a + (n – 1) d, ...
Illustration 6 Find the nth term and 19th term of the
Illustration 3 Consider the series: 1, 3, 5, 7, 9, ...
sequence 5, 2, –1 , –4, ...
Here 2nd term – 1st term = 3rd term – 2nd term = 4th
term – 3rd term = ... = 2 Solution: Clearly, the given sequence is an A.P. with a = 5
Hence, 1, 3, 5, 7, ... are in A.P. whose first term is 1 and and d = –3
common difference is 2 \ tn = a + (n – 1) d = 5 + (n – 1) (–3) = –3n + 8
Illustration 4 The series: 5, 3, 1, –1, –3, –5, –7, ... is in A.P. For the 19th term, putting n = 19, we get t19 = –3.19 +
whose first term is 5 and common difference is –2. 8 = –49
Sum of n terms of an A.P.
Notes: The sum of n terms of an A.P. with first term ‘a’ and
• A sequence t1, t2, t3, t4, ... will be in A.P. if t2 – t1 = common difference ‘d’ is given by
t3 – t2 = t4 – t3 = ..., i.e., tn – tn–1 = constant, for n ≥ 2. n
Sn = [2a + (n – 1)d]
• Three numbers a, b, c are in A.P. if and only if 2
b – a = c – b, i.e., if and only if a + c = 2b.
• Any three numbers in an A.P. can be taken as Notes:
a – d, a, a + d. Any four numbers in an A.P. can
be taken as a – 3d, a – d, a + d and a + 3d. • If Sn is the sum of n terms of an A.P. whose first
Similarly, five numbers in an A.P. can be taken as term is ‘a’ and last term is l, then
a – 2d, a – d, a, a + d and a + 2d. n
Sn = (a + l).
generAL terM oF An A.P. 2
Let a be the first term and d be the common difference • If common difference d, number of terms n and
of an A.P. Then, the A.P. is a, a + d, a + 2d, a + 3d,... . the last term l, are given then
We also observe that n
Sn = [2l – (n – 1) d]
t1, the first term, is a = a + (1 – 1) d; 2
t2, the second term, is a + d = a + (2 – 1) d; • tn = Sn – Sn–1.
t3, the third term, is a + 2d = a + (3 – 1) d;
t4, the fourth term, is a + 3d = a + (4 – 1) d; Illustration 7 Find the sum of the series
tn, the nth term, is a + (n – 1) d. .5 + .51 +. 52 + ... to 100 terms
Thus, the formula, tn = a + (n – 1) d gives the
Solution: The given series is an A.P. with first term, a = .5
general term of an A.P.
and common difference, d = . 51 – . 5 = . 01
Notes: \ Sum of 100 terms
• If an A.P. has n terms, then the nth term is called 100
the last term of A.P. and it is denoted by l. = [2 × .5 + (100 – 1) × .01]
2
Therefore, l = a + (n – 1) d.
= 50 (1 + 99 × .01) = 50 (1 +. 99)
• If a is the first term and d the common difference
of an A.P. having m terms, then nth term from the = 50 × 1.99 = 99.5
end is (m – n + 1)th term from the beginning. Illustration 8 Find the sum of 20 terms of an A.P., whose
\ nth term from the end = a + (m – n) d. first term is 3 and the last term is 57.

Chapter_29.indd 2 6/5/2015 4:25:35 PM


Progressions 29.3

Solution: We have, a = 3, l = 57, n = 20


On multiplying these, we get
n
\ Sn = (a + l), t2t3t4 ... tn = t1t2t3 ... tn–1rn–1 ⇒ tn = t1rn–1; but
2
t1 = a.
20
\ S20 = (3 + 57) = 600. \ General term = tn = arn–1.
2
Hence, the sum of 20 terms is 600 Thus, if a is the first term and r the common ratio of
a G.P. then the G.P. is a, ar, ar2, ..., arn–1 or a, ar,
geoMetrIc ProgreSSIon ar2, ... according as it is finite or infinite.
A sequence (finite or infinite) of non-zero numbers in which Cor. If the last term of a G.P. consisting of n terms is
every term, except the first one, bears a constant ratio denoted by l, then l = arn–1.
with its preceding term, is called a geometric progression,
Notes:
abbreviated as G.P.
• If a is the first term and r the common ratio of a
Illustration 9 The sequences given below:
finite G.P. consisting of m terms, then the nth term
(i) 2, 4, ,8, 16, 32, ... from the end is given by arm–n.
(ii) 3, –6, 12, –24, 48, ... • The nth term from the end of a G.P. with the last
1 1 1 1 1 term l and common ratio r is l/rn–1.
(iii) , , , , , ...
4 12 36 108 324 • Three numbers in G.P. can be taken as a/r, a, ar;
1 1 1 1 1 four numbers in G.P. can be taken as a/r3,
(iv) , , , , , ...
5 30 180 1080 6480 a/r, ar, ar3; five numbers in G.P. can be taken as
(v) x, x2, x3, x4, x5, ... (where x is any fixed real a/r2, a/r, a, ar, ar2, and so on...
number) • Three numbers a, b, c are in G.P. if and only if
are all geometric progressions. The ratio of any term in (i) b/a = c/b, i.e., if and only if b2 = ac.
to the preceding term is 2. The corresponding ratios in (ii),
1 1 Illustration 10 Find the nth term and 12th term of the
(iii), (iv) and (v) are –2, , , and x, respectively. The
3 6 sequence –6, 18, –54, ...
ratio of any term of a G.P. to the preceding term is called the
common ratio of the G.P. Thus, in the above examples, the Solution: The given sequence is a G.P. with a = –6 and
1 1 r = –3
common ratios are 2, –2, , and x, respectively
3 6 \ tn = arn–1 = (–6) (–3) n–1 = (–1)n × 6 × 3n–1
For the 12th term, putting n = 12, we get
Note: t12 = (–1)12 × 6 × 311 = 2 × 312.
In a G.P., any term may be obtained by multiplying
the preceding term by the common ratio of the G.P. Sum of n terms of a G.P.
Therefore, if any one term and the common ratio of a
G.P. be known, any term can be written out, i.e., the The sum of first n terms of a G.P. with first term a
G.P. is then completely known. and common ratio r is given by
In particular, if the first term and the common a (r n − 1)
Sn =
ratio are known, the G.P. is completely known. r −1
The first term and the common ratio of a G.P. are
Notes:
generally denoted by a and r, respectively.
(i) When r = 1
generAL terM oF g.P.
Sn = a + a + ... up to n terms = na
Let a be the first term and r ( ≠ 0) be the common
ratio of a G.P. Let t1, t2, t3, ... ,tn denote Ist, 2nd, 3rd, (ii) It l is the last term of the G.P., then
... , nth terms, respectively. Then, we have lr − a
Sn = ,r≠1
t2 = t1r, t3 = t2r, t4 = t3r, ..., tn = tn–1r r−a

Chapter_29.indd 3 6/5/2015 4:25:36 PM


29.4 Chapter 29

Sum of An Infinite G.P. When | r | < 1 nth Term of An H.P.


The sum of an infinite G.P. with first term a and nth term of H.P.
a
common ratio r is S∞ = ; when | r | < 1, i.e., 1
1− r =
–1<r<1 nth term of the corresponding A.P.

Notes:
Illustration 11 Find the sum of 8 terms and n terms of the
1
sequence 9, –3, 1, –1/3, ... • Three numbers a, b, c are in H.P. if and only if ,
a
Solution: The given sequence is a G.P. with a = 9 and r = 1
–1/3 , are in A.P., i.e.,
c
We know that 1 1 1 2ac
+ = 2 × or b = .
1 − (−1/3)8 1 − 1/ 38 27  1 a c b a+c
S8 = 9 =9 =  1− 8 
1 − (−1/3) 4/3 4  3 
• No term of H.P. can be zero.
8
27 3 − 1 1 6561 − 1 6560 1640 • There is no general formula for finding the sum to
= = = =
4 3 8 4 3 5 4 × 243 243 n terms of H.P.
• Reciprocals of terms of H.P. are in A.P. and then
1 − (−1/3) n 1 − (−1) n /3n properties of A.P. can be used.
Also, Sn = 9 =9
1 − (−1/3) 4/3

27 3n − (−1) n 3n − (−1) n Illustration 14 Find the 100th of the sequence


= =
4 3n 4 ⋅ 3n−3 1 1 1
1, , , , ...
Illustration 12 Find the sum of the infinite sequence 7, –1, 3 5 7
1 1 1 1 1
,– , ... Solution: The sequence 1, , , , ... is an H.P.
7 49 3 5 7
Solution: The given sequence is a G.P. with a = 7 and r = – Corresponding A.P. is 1, 3, 5, 7, ...
1 1 Now, for the corresponding A.P., first term a = 1, d = 2
, so | r | = − = <1
7 7 \ 100th term of the corresponding A.P.
7 7 49  a  = a + (100 – 1) d
\ S= = =  S = 
1 − (−1/7) 8/7 8  1 − r = 1 + (100 – 1) 2 = 199
1
Hence, the 100th term of the given sequence = .
HArMonIc ProgreSSIon 199
A sequence of non-zero numbers a1, a2, a3, ... is said to be a
harmonic progression (abbreviated as H.P.) if the sequence Some Special Sequences
1 1 1 1. The sum of first n natural numbers
, , , ... is an A.P.
a1 a2 a3 n (n + 1)
Sn = 1 + 2 + 3 + ... + n =
2
1 1 1
Illustration 13 The sequence 1, , , , ... is a H.P. The 2. The sum of squares of first n natural numbers Sn2 = 12
4 7 10
n (n + 1) (2n + 1)
sequence obtained by taking reciprocals of its corresponding + 22 + 32 + ... + n2 = .
6
terms, i.e., 1, 4, 7, 10, ... is an A.P.
3. The sum of cubes of first n natural numbers Sn3 = 13 +
1 1 1
A general H.P. is , , , ... 2
a a + d a + 2d  n (n + 1) 
23 + 33 + ... + n3 =  .
 2 

Chapter_29.indd 4 6/5/2015 4:25:38 PM


29.5
Progressions

Notes: n (2n 2 + 3n + 1)
=

6
If nth term of a sequences is
1 1 1
Tn = an3 + bn2 + cn + d = n3 + n2 + n

3 2 6
then the sum of n terms is given by
1 3 1 2 1 
Sn = STn = aSn3 + bSn2 + cSn + Sd, \ Sn = S   n + n + n 
3 2 6 
which can be evaluated using the above results.
1 1 1
= Sn3 + Sn2 + Sn

2 2 2
Illustration 15  Find 2 + 4 + 6 + ... + (2n) 2 3 2 6

Solution: nth term of the given series is (2n)2. Then, 1 n 2 (n + 1) 2 1 n(n + 1) (2n + 1)
Tn = 4n2 =
+
3 4 2 6
4n (n + 1) (2n + 1)
\ Sn = 4 S n2 = 1 n(n + 1)
6 +
6 2
2n (n + 1) (2n + 1) n(n + 1)
\ Sn = =
[n (n + 1) + 2n + 1 + 1]
3 12

Illustration 16  Sum the series 12 + (12 + 22) + (12 + 22 + n(n + 1) 2


=
(n + 3n + 2)
32) + ... to n terms 12
n(n + 1)
Solution: Here, Tn = (12 + 22 + 32 + ... n2) =
(n + 1) (n + 2)
12
n (n + 1) (2n + 1) n
= Sn2 =
=
(n + 1)2 (n + 2).
6 12

Practice Exercises

Difficulty Level-1
(Based on Memory)

1. If the sum of the 6th and the 15th elements of an arithmetic 3. If p, q, r, s are in harmonic progression and p > s, then:
progression is equal to the sum of the 7th, 10th and 12th
elements of the same progression, then which element of 1 1
(a) (b) q + r = p + s
the series should necessarily be equal to zero? ps qr
(a) 10th (b) 8th
1 1 1 1
(c) 1st (d) None of these (c) + = + (d) None of these
q r p s
[Based on MAT, 2003]
[Based on MAT, 2003]
2. The sum of the 6th and 15th elements of an arithmetic
4. Mohan ate half a pizza on Monday. He ate half of what
progression is equal to the sum of 7th, 10th and 12th
was left on Tuesday and so on. He followed this pattern
elements of the same progression. Which element of the
for one week. How much of the pizza would he have eaten
series should necessarily be equal to zero?
during the week?
(a) 10th (b) 8th (a) 99.22% (b) 95%
(c) 1st (d) None of these (c) 98.22% (d) 100%
[Based on MAT, 2003] [Based on MAT, 2003]

Chapter_29.indd 5 6/5/2015 4:25:38 PM


29.6 Chapter 29

5. If logxa, ax/2 and logbx are in GP, then x is: 14.


The series of positive integers is divided in the following
(a) loga(logba) (b) loga(logea) + loga(logeb) way
(c) –loga(logab) (d) loga(logeb) – loga(logea) 1 + (2 + 3) + (4 + 5 + 6) + (7 + 8 + 9 + 10) + ...
What will be the first term in the (n + 1)th group?
[Based on MAT, 2002]
6. A person pays `975 in monthly instalments, each monthly (n 2 − n + 2) (n 2 + n + 2)
(a) (b)
instalment being less than the former by `5. The amount 2 2
of the first instalment is `100. In what tune, will the entire
amount be paid? (2n 2 + 3) n(n + 1) 2
(c) (d)
(a) 12 months (b) 26 months 4n 4
(c) 15 months (d) 18 months
15. What is the least value of n such that
[Based on MAT, 2002]
(1 + 3 + 32 + ... + 3n) exceeds 2000?
7. Let Sn denote the sum of the first ‘n’ terms of an A.P.
S2n = 3Sn. Then, the ratio S3n/Sn is equal to: (a) 7 (b) 5
(a) 4 (b) 6 (c) 8 (d) 6
[Based on I.P. Univ., 2002]
(c) 8 (d) 10
[Based on MAT, 2002] 16. The sum of 12 terms of an A.P., whose first term is 4, is
256. What is the last term?
8.
Three numbers are in G.P. Their sum is 28 and product is
512. The numbers are: (a) 35 (b) 36
(a) 6, 9 and 13 (b) 4, 8 and 16 (c) 37 (d) 116/3
[Based on SCMHRD, 2002]
(c) 2, 8 and 18 (d) 2, 6 and 18
[Based on MAT, 1999] 17. The harmonic mean between two numbers is 4, their
9.
The sum of the series arithmetic mean is A and geometric mean is G. If 2A + G2
= 27, then the numbers are:
12 + 22 + 32 + 42 + ... + 152 is:
(a) 8, 2 (b) 8, 6
(a) 1080 (b) 1240 (c) 6, 3 (d) 6, 4
(c) 1460 (d) 1620
[Based on MAT, 1999] 18. Consider a geometric progression in which all terms
10.
If the nth term of an A.P. is 4n + 1, then the common are positive. If in it, any term is equal to the sum of the
difference is: next two following terms, then what is the value of the
common ratio?
(a) 3 (b) 4
(c) 5 (d) 6 1− 5
(a) (b) 1
[Based on MAT, 1999] 2
1 1 1 1 5 −1
11.
If + = + , then a, b, c form a/an: (c) (d) None of these
b−a b−c a c 2
(a) Arithmetic progression (b) Geometric progression
19. The first, sixth and 14th term of an A.P. are consecutive
(c) Harmonic progression (d) None of these. terms of a GP. The common ratio of the G.P. will be
[Based on MAT, 1999]
(a) 1.5 (b) 2
12. In G.P., the first term is 5 and the common ratio is 2. The (c) 3 (d) 2.5
eighth term is:
(a) 640 (b) 1280 20. The sum of all terms of an infinite geometric series is 6
and the sum of all terms of the infinite series formed by
(c) 256 (d) 160
squaring the terms of the previous series is 24. Find the
[Based on MAT, 2000]
second term of the first series.
13.
If the arithmetic mean of two numbers is 5 and geometric
mean is 4, then the numbers are 6 8
(a) (b)
(a) 4, 6 (b) 4, 7 25 25

(c) 3, 8 (d) 2, 8 24 576


(c) (d)
[Based on MAT, 2000] 25 25

Chapter_29.indd 6 6/5/2015 4:25:39 PM


29.7
Progressions

21. The average of 49th, 50th and 51st term of an arithmetic 29.
What is the eighth term of the sequence 1, 4, 9, 16, 25, …?
progression is equal to 49. What is the sum of the first 99 (a) 8 (b) 64
terms of this arithmetic progression?
(c) 128 (d) 200
(a) 4,851 (b) 4,950
[Based on MAT (Sept), 2003]
(c) 5,049 (d) Cannot be determined
30.
If the arithmetic mean of two numbers is 5 and geometric
22.
The number of two-digit numbers exactly divisible by 3 is: mean is 4, then the numbers are:
(a) 33 (b) 32 (a) 4, 6 (b) 4, 7
(c) 31 (d) 30 (c) 3, 8 (d) 2, 8
[Based on MAT (Feb), 2008] [Based on MAT (Dec), 2000]
23.
The natural numbers are divided into groups as (1), (2, 3), 31.
Two numbers A and B are such that their G.M. is 20 per cent
(4, 5, 6), (7, 8, 9, 10) and so on. The sum of the numbers less than their A.M. Find the ratio between the numbers.
in the 50th group is: (a) 3:2 (b) 4:1
(a) 1225 (b) 24505 (c) 2:1 (d) 3:1
(c) 62525 (d) 52650
[Based on MAT (Sept), 2007]  7
If log32, log3(2x – 5) and log3  2 x −  are in A.P., then
32.
 2
24.
A man arranges to pay off a debt of `3600 by 40 annual
instalments which are in AP. When 30 of the instalments the value of x is:
are paid he dies leaving one-third of the debt unpaid. The (a) 2 (b) 3
value of the 8th instalment is: (c) 4 (d) 5
(a) `35 (b) `50
33.
The interior angles of a polygon are in AP, the smallest
(c) `65 (d) None of these
angle is 120° and the common difference is 5. Then, the
[Based on MAT (Dec), 2006] number of sides of the polygon are:
25.
A club consists of members whose ages are in AP, the (a) 16 (b) 9
common difference being 3 months. If the youngest (c) 8 (d) 12
member of the club is just 7 years old and the sum of the
[Based on MAT (May), 1999]
ages of all the members is 250 year, then the number of
members in the club are: 34.
A man arranges to pay off a debt of `3600 in 40 annual
(a) 15 (b) 20 instalments which form an AP. When 30 of the instalments
are paid, he dies leaving one-third of the debt unpaid. Find
(c) 25 (d) 30
the value of the first instalment.
[Based on MAT (Feb), 2006]
(a) 55 (b) 53
26.
How many terms are there is an AP whose first and fifth (c) 51 (d) 49
terms are – 14 and 2 respectively and the sum of terms is 40?
[Based on MAT (May), 1999]
(a) 15 (b) 10
35.
A five-digit number divisible by 3 is to be formed using
(c) 5 (d) 20
numerical 0, 1, 2, 3, 4 and 5 without repetition. The total
[Based on MAT (Dec), 2007] number of ways this can be done is:
27.
In a geometric progression, the sum of the first and the last (a) 122 (b) 210
term is 66 and the product of the second and the last but (c) 216 (d) 217
one term is 128. Determine the first term of the series.
[Based on SNAP, 2010]
(a) 64 (b) 64 or 2
36.
The sum of all even natural numbers less than 100 is:
(c) 2 or 32 (d) 32
(a) 2450 (b) 2272
[Based on MAT (Feb), 2005]
(c) 2352 (d) 2468
A sequence is generated by the rule that the xth term is x2 +
28.
[Based on FMS, 2005]
1 for each positive integer x. In this sequence, for any value
x > 1, the value of (x + 1)th term less the value of xth term is 37.
The sum of all odd numbers between 100 and 200 is:
(a) 2x2 +1 (b) x2 + 1 (a) 6200 (b) 6500
(c) 2x + 1 (d) x + 2 (c) 7500 (d) 3750
[Based on MAT (Feb), 2005] [Based on FMS, 2006]

Chapter_29.indd 7 6/5/2015 4:25:39 PM


29.8 Chapter 29

If a, b, c are in G.P. and ax = by = cz, then:


38. 46.
The second term in a geometric infinite series is 2, whose
sum is 25/2. Then the fourth term of the series is:
1 1 2 1 1 2
(a) + = (b)   (a) 2/25 (b) 2/5
x z y x z y
(c) 4/25 (d) 4/5
1 1 2 1 1 2 [Based on IIFT, 2005]
(c)   (d)  
x y z x y z 47.
If three positive real numbers a, b and c (c > a) are
[Based on FMS, 2009] in Harmonic Progression, then log (a + c) + log
39.
The angles of a pentagon are in arithmetic progression. (a – 2b + c) is equal to:
One of the angles, in degrees, must be (a) 2 log (c – b) (b) 2 log (a – c)
(a) 108 (b) 90 (c) 2 1og (c – a) (d) log a + log b + log c
[Based on IIFT, 2008]
(c) 72 (d) 54
[Based on FMS, 2010] 48.
If the positive real numbers a, b and c are in Arithmetic
1 Progression, such that abc = 4, then minimum possible
If xk 1 = xk  for k = 1, 2, …, n – 1 and x1 = 1, find
40.
2 value of b is:
x1 + x2 + … + xn. 3 2

n3 n 1 2 (a) 2 2 (b) 2 3


(a) (b)
3 2 1
n2  n n 2  3n (c) 2 3 (d) None of these
(c) (d)
4 4 [Based on IIFT, 2008]
[Based on FMS, 2010] 49.
Find the sum of the following series.
41.
Three numbers a, b, c non-zero, form an arithmetic
2 3 6 11 18
progression. Increasing a by 1 or increasing c by 2 results      ...
in a geometric progression. Then b equals: 1! 2! 3! 4! 5!
(a) 16 (b) 14 (a) 3e – 1 (b) 3(e – 1)
(c) 12 (d) 10 (c) 3(e + 1) (d) 3e + 1
[Based on FMS, 2010] [Based on IIFT, 2010]
42.
If the sum of the first 10 terms of an arithmetic progression 50.
How many positive integers ‘n’ can we form using the
is four times the sum of the first five terms, then the ratio digits 3, 4, 4, 5, 6, 6, 7, if we want ‘n’ to exceed 60,00,000?
of the first term to the common difference is:
(a) 320 (b) 360
(a) 1:2 (b) 2:1
(c) 540 (d) 720
(c) 1:4 (d) 4:1
[Based on IIFT, 2010]
[Based on FMS, 2011]
1 2 1 2
43.   
The sum to infinity of ... 1 1 1 1
7 7 2 73 7 4 51. If + + + = 0, which of the following
x z x− y z− y
1 5 statements is true?
(a) (b)
24 48
y
1 (a) x, y, z are in HP of x, , z are in A.P.
(c) (d) None of these 2
16 [Based on FMS, 2011] (b) x, y, z are in AP or x, y, z are in H.P.
44.
How many two digit odd numbers can be formed from the y
digits 1, 2, 3, 4, 5 and 8, if repetition of digit is allowed? (c) x, , z are in HP or x, y, z are in G.P.
2
(a) 5 (b) 15
(d) x, y, z are in GP or x, y, z are in A.P.
(c) 35 (d) 18
[Based on JMET, 2006]
[Based on IIFT, 2005]
45.
The positive numbers x, y, z are in arithmetic progression. 52. The angles of a convex hexagon in degrees are integers
They are also in harmonic progression. Then: and in arithmetic progression. ∠M denote the largest of
these 6 angles. Then the maximum value that M can take
(a) They cannot be in geometric progression
is:
(b) x and 2 cannot be equal
(c) y, z, x cannot be in arithmetic progression (a) 125º (b) 150º
(d) None of the above (c) 175º (d) 179º
[Based on IIFT, 2005] [Based on JMET, 2006]

Chapter_29.indd 8 6/5/2015 4:25:39 PM


29.9
Progressions

53.
How many multiples of 7 are there between 33 and 329? (a) 8 (b) 9
(a) 43 (c) 10 (d) 11
(b) 35 [Based on ATMA, 2005]
(c) 329 1 1 1 1 1
55. + + + + + … equals
(d) 77 2 3 6 10 15
[Based on ATMA, 2008] (a) 2 (b) 3
54. In a regular polygon, each interior angle is 140º. The (c) 5 (d) ∞
number of sides in the polygon will be: [Based on JMET, 2006]

Difficulty Level-2
(Based on Memory)

1. The sum of 3rd and 15th elements of an arithmetic (a) 2/13 (b) 22/3
progression is equal to the sum of 6th, 11th and 13th (c) 21/4 (d) 23/4
elements of the same progression. Then which element of [Based on CAT, 2004]
the series should necessarily be equal to zero?
7. What is the sum of the series 1 + 2x + 4x2 + 7x3 + 10x4
(a) 1st (b) 9th
+ ... up to ∞? (Given 0 < x < 1)
(c) 12th (d) None of these
[Based on CAT, 2003] 1 − x(1 − x) x2 + 1
2. x x
If log32, log3 (2 – 5), log3 (2 – 7/2) are in arithmetic (a) (b)
(1 − x)3 (1 − x) 2
progression, then the value of x is equal to:
(a) 5 (b) 4 1 + x(1 − x)
(c) (d) None of these
(c) 2 (d) 3 (1 − x)3
[Based on CAT, 2003]
3.
The are 8436 steel balls, each with a radius of 1 centimetre, 8. A square S1 had dimensions 6 cm × 6 cm. Another square
stacked in a pile, with 1 ball on top, 3 balls in the second S2 is drawn by joining the mid-points of the sides of S1.
layer, 6 in the third layer, 10 in the fourth, and so on. The Square S3 is drawn joining the mid-points of S2 and so on.
number of horizontal layers in the pile is: What is the sum of area (S1) + area (S2) + ... ∞?
(a) 34 (b) 38 (a) 72 cm2 (b) 36 2 ( 2 – 1) cm2
(c) 36 (d) 32 (c) 14.4 cm2 (d) 36 2 cm2
[Based on CAT, 2003]
4.
Let T be the set of integers {3, 11, 19, 27, ... 451, 459, 9.
What is the sum of all two-digit numbers that give a
467} and S be a subset of T such that the sum of no two remainder of 3 when they are divided by 7?
elements of S is 470. The maximum possible number of (a) 666 (b) 676
elements in S is: (c) 683 (d) 777
(a) 32 (b) 28 [Based on CAT, 2004]
4 9 16 25
(c) 29 (d) 30 10.
The infinite sum 1 + + 2 + 3 + 4 + ... equals:
7 7 7 7
[Based on CAT, 2003]
5.
In a certain examination paper, there are n questions. For j 27 21
(a) (b)
= 1, 2, ... n, there are 2n–j students who answered j or more 14 13
questions wrongly. If the total number of wrong answers
is 4095, then the value of n is: 49 256
(c) (d)
(a) 12 (b) 11 27 147
[Based on CAT, 2004]
(c) 10 (d) 9
[Based on CAT, 2003] 11. The sum of the series 1 + 2 × 2 + 3 × 22 + 4 × 23 + 5 × 24
6.
If three positive real numbers x, y, z satisfy y – x + ... + 100 × 299 is:
= z –­ y and xyz = 4, then what is the minimum possible (a) 99 × 2100 (b) 99 × 2100 + 1
value of y? (c) 100 × 2100 (d) 100 × 2100 + 1

Chapter_29.indd 9 6/5/2015 4:25:40 PM


29.10 Chapter 29

12.
The value of 3 + 5 + 7 + ... + n terms
(13 + 23 + 33 + ... 153) – (1 + 2 + 3 + ... 15) is: 19. If = 7, then the value of n is:
5 + 8 + 11 + ... + 10 terms
(a) 14280 (b) 14400
(c) 12280 (d) 13280 (a) 35 (b) 36
(c) 37 (d) 40
13. f (a, b) is a series of which the first three terms are
(a + b)2, (a2 + b2) and (a – b)2. We add the first n terms of 20. If the sum of first n natural numbers is one-fifth of the sum
the series f (a, b) and call it S (a, b). If a = 7, b = 3, then of their squares, then n is:
find S (7, 3) for n = 20. (a) 5 (b) 6
(a) –5980 (b) 6000 (c) 7 (d) 8
(c) 6960 (d) None of these [Based on FMS (Delhi), 2002]
[Based on FMS (Delhi), 2004] 21. The number of ordered triplets of positive integers which
are solutions of the equation:
14. 30 trees are planted in a straight line at intervals of
5 m. To water them, the gardener needs to bring water for x + y + z = 100 is
each tree, separately from a well, which is 10 m from the (a) 4851 (b) 5081
first tree in line with the trees. How far will he have to (c) 6871 (d) 7081
walk in order to water all the trees beginning with the first [Based on FMS (Delhi), 2002]
tree? Assume that he starts from the well. 22. Three distinct numbers x, y, z form a geometric progression
(a) 4785 m (b) 4795 m in that order, and x + y, y + z, z + x form an arithmetic
(c) 4800 m (d) None of these progression in that order. Find the common ratio of the
[Based on FMS (Delhi), 2004] geometric progression.
15. f (x) + 2x; where x is an integer. If we arrange the value of (a) –2 (b) 2
f (x), for x = 25, 24, 23.... (continuously decreasing value (c) 0.5 (d) –0.5
of x), we get an Arithmetic Progression (A.P.) whose first [Based on SCMHRD, 2002]
term is 50. Find the maximum value of the sum of all the 23. The digits of a three-digit number form G.P. If 400 is
terms of the A.P. subtracted from it, then we get another three-digit number
(a) 600 (b) 625 whose digits form an arithmetic series. What is the sum of
(c) 650 (d) None of these these two numbers?
[Based on FMS (Delhi), 2004] (a) 1356 (b) 1648
16. If one of the roots of the equation (c) 1462 (d) 1000
3x3 + 11x2 + 12x + 4 = 0: [Based on SCMHRD, 2002]
is (–1), then all the three roots are in 24. Find the sum of all natural numbers not exceeding 1000,
(a) Arithmetic progression which are divisible by 4 but not by 8.
(b) Geometric progression (a) 62500 (b) 62800
(c) Harmonic progression (c) 64000 (d) 65600
(d) None of the above [Based on SCMHRD, 2002]
[Based on IITTM, Gwalior, 2003] 25. If a, b, c are in A.P. and x, y, z are in G.P., then the value of
17. A man has an apple orchard and he sells to his first xb–c yc–a za–b is:
customer half of all the apples plus half an apple; to the (a) 0 (b) 1
2nd customer he sells half of the rest plus half an apple, (c) xyz (d) xaybzc
and so on. To the seventh customer, he sells half of what
remains and another half an apple. And that is all he had. 26. If log 2, log (2x – 1) and log (2x + 3) (all to the base 10)
How many apples did the man start out with? be three consecutive terms of an Arithmetic Progression,
(a) 47 (b) 97 then the value of x is equal to:
(c) 127 (d) 137 (a) 0 (b) 1
(c) log25 (d) log102
1 2 3 4 [Based on REC Tiruchirapalli, 2002]
18. The value of 2 4 . 2 8 . 216 . 2 32 ... is equal to: 27.
Consider a sequence where the nth term, tn = n/(n + 2),
(a) 1 (b) 2 n = 1, 2, ... the value of t3 × t4 × t5 × … × t53 equals.
(a) 4/495 (b) 2/495
3 5 (c) 12/55 (d) 1/1485
(c) (d)
2 2 [Based on CAT, 2007]

Chapter_29.indd 10 6/5/2015 4:25:40 PM


29.11
Progressions

28. I open a book store with a certain number of books. On the 36.
There are two arithmetic progressions, A1 and A2, whose
first day, I sell 1 book; on the second day, I sell 2 books; first terms are 3 and 5 respectively and whose common
on the third day, I sell 3 books and so on. At the end of the differences are 6 and 8 respectively. How many terms of
month (30 days). I realise that I sold the same number of the series are common in the first n terms of A1 and A2, if
books with which I started. Find the number of books in the sum of the nth terms of A1 and A2 is equal to 6,000?
the beginning. (a) 103 (b) 107
(a) 365 (b) 420 (c) 109 (d) 113
(c) 465 (d) 501
37. Infinite circles are inscribed successively inside the upper
29. A contractor, who got the contract for building the flyover, half of circles, as shown in the figure below. If the radius
failed to construct the flyover in the specified time and 1
was supposed to pay `50,000 for the first day of extra of the largest circle is units, find the sum of area of all
π
time. This amount increased by `4,000 each day. If he
the circles formed in square units.
completes the flyover after one month of stipulated time,
he suffers a loss of 10% in the business. What is the
amount he received for making the flyover in crores of
rupee? (one month = 30 days):
(a) 3.1 (b) 3.24
(c) 3.46 (d) 3.68
Directions (Questions 30 to 34): Refer to the data below and
answer the questions that follow.
The starting term of an A.P. is equal to the starting term of a
G.P. The difference between the fifth term of the A.P. and the list
term of the A.P. is equal to the difference between the first term of
the G.P. and the 2nd term of the G.P.
(a) 2 (b) 1.5
30. The first term of the progression is: (c) 1.33 (d) 1
(a) 0 (b) 1
Given x = 1 /0 y + y – y – y3 + y4 + ... ∞ and z = 1 + y + y2
38. 2 2
(c) 2 (d) 3
+ y3 + y4 + ... ∞ (|y| < 1). Which of the following is true?
31. The common difference of the A.P. is: (a) Harmonic mean of x and y is 1
(a) 1 (b) 2 (b) Arithmetic mean of x and z is 1
(c) 3 (d) 4 (c) Harmonic mean of x and z is 1
32. The common ratio of the G.P. is: (d) None of these
(a) 1 (b) 3 39. Three numbers form an increasing geometric progression.
(c) 5 (d) 7 When the second number is doubled, the numbers form
an arithmetic progression. What is the ratio of the first
33. The sum of the first four terms of the A.P. is: number and the third number?
(a) 5 (b) 10 (a) 1:7 − 4 3 (b) 1:7 + 4 3
(c) 15 (d) 20
(c) 1:2 − 3 (d) Either (a) or (b)
34. The sum of the first four terms of G.P. is:
40. Twenty six men –A, B, C . . . Y and Z – running at the
(a) 120 (b) 126 respective speeds of a, b, c, . . . y and z are participating
(c) 150 (d) 156 in a 10 Km running race on a circular track of length 100
m. Their speeds are in arithmetic progression from a to z,
35. A boy throws a ball to the ground with force from a height in that order. If the time taken by Z to meet A, for the first
of 10 m. After hitting the ground for the first time it rises time after the start, is 20 seconds and the time taken by
to a height of 20 m. There after it rises only upto half the M to complete the race is 52 minutes and 5 seconds, then
prior height on hitting the ground. find the time taken for all the twenty six men to meet for
What is the total distance travelled by the ball till it comes the first time at the starting point. (All of them started the
to rest? race at same time and from the same point).
(a) 80 m (b) 40 m (a) 1,000 seconds (b) 500 seconds
(c) 50 m (d) 90 m (c) 225 seconds (d) 125 seconds

Chapter_29.indd 11 6/5/2015 4:25:40 PM


29.12 Chapter 29

41. The pth and the (p + 3)th term of an arithmetic progression 48.
The first two terms of a geometric progression add up to
are in the ratio p:p + 3. The sum of the first 3p terms of the 12. The sum of the third and the fourth terms is 48. If the
arithmetic progression and the sum of its first 4p terms are terms of the geometric progression are alternately positive
in the ratio 61:108. Find the value of p. and negative, then the first term is:
(a) 10 (b) 15 (a) –2 (b) –4
(c) 20 (d) 25 (c) –12 (d) 8
42.
If a1, a2, a3, ..., an are in A.P. where ai > 0 for all i, then the [Based on SNAP, 2009, 2010]
1 1 1 Given that (1 + 2 + 3 + … + 102) = 385, then the value
49. 2 2 2
value of + + ... + is: of (22 + 42 + 62 + … + 202) is equal to:
a1 + a2 a2 + a3 an −1 + an
(a) 770 (b) 1540
n −1 n +1 (c) 1155 (d) (385)2
(a) (b)
a1 + an a1 + an [Based on FMS (MS), 2006]
n 50.
From a group of boys and girls, 15 girls leave. There are
(c) (d) None of these then left two boys for each girl. After this 45 boys leave.
a1 − an
There are then 5 girls for each boy. The number of girls in
43.
A retailer has n stones by which he can measure (or
the beginning was:
weigh) all the quantities from 1 kg to 121 kg (in integers
only. e.g., 1 kg, 2 kg, 3 kg,) keeping these stones on either (a) 40 (b) 43
side of the balance. What is the minimum value of n? (c) 29 (d) None of these
(a) 3 (b) 4 [Based on FMS, 2011]

(c) 5 (d) 11 51.


The ratio between the number of sides of two regular
polygons is 1:2 and the ratio between their interior
44. In a certain examination paper, there are n questions. For angles is 2:3. The number of sides of these polygons are
i = 1, 2, ..., n, there are 2n – j students who answered j respectively:
or more questions wrongly. If the total number of wrong (a) 4, 8 (b) 5, 10
answers is 4,095, then the value of n is: (c) 6, 12 (d) 8, 16
(a) 12 (b) 11 [Based on IIFT, 2005]
(c) 10 (d) 9 52.
The inverse of the sum of the following series up to n
45. An intelligence agency forms a code of two distinct digits 3 3 7
selected from 0, 1, 2, ..., 9 such that the first digit of the code terms can be written as    ... :
4 36 144
is non zero. The code, handwritten on a slip, can however
potentially create confusion when read upside down — for (n  1) 2 n 2  2n
(a) (b)
example, the code 91 may appear as 16. How many codes 2
n  2n (n  1) 2
are there for which no such confusion can arise?
(a) 80 (b) 78 n 2  2n (n  1) 2
(c) (d)
(c) 71 (d) 69 (n  1) 2 n 2  2n
[Based on IIFT, 2006]
46. Let S1 be a square of side a. Another square S2 is formed
by joining the mid-points of the sides of S1. The same 53.
If H1, H2, H3, ..., Hn, are n Harmonic means between ‘a’
process is applied to S2 to form yet another square S3, and H  a Hn  b
and ‘b’ (≠ a), then value of 1  is equal to:
so on. If A1, A2, A3, ... be the areas and P1, P2, P3, ... be H1  a H n  b
the perimeters of S1, S2, S3, ..., respectively, then the ratio (a) n + 1 (b) 2n
P1 + P2 + P3 + ...
equals: (c) 2n + 3 (d) n – l
A1 + A2 + A3 + ... [Based on IIFT, 2008]
2(1 + 2) 2(2 − 2) Suppose a, b and c are in Arithmetic Progression and a2,
54.
(a) (b) b2 and c2 are in Geometric Progression. If a < b < c and a
a a
3
2(2 + 2) 2(1 + 2 2) + b + c = , then the value of a is equal to:
(c) (d) 2
a a
1 1
47.
The 288th term of the sequence a, b, b, c, c, c, d, d, d, (a) (b)
d, … is: 2 2 2 3
(a) u (b) v 1 1 1 1
(c)  (d) 
(c) w (d) x 2 3 2 2
[Based on SNAP, 2008] [Based on IIFT, 2008]

Chapter_29.indd 12 6/5/2015 4:25:41 PM


29.13
Progressions I
Sum of the series 12 – 22 + 32 – 42 + … + 20012 – 20022
55. (a) 26 (b) 32
+ 20032 is: (c) 34 (d) 40
(a) 2007006 (b) 1005004 [Based on XAT, 2011]
(c) 200506 (d) None of these 61.
A saint has a magic pot. He puts one gold ball of radius
[Based on IIFT, 2008] 1 mm daily inside it for 10 days. If the weight of the first
56.
A man arranged to pay off a debt of `3600 by 40 annual bail is 1 g and if the radius of a ball inside the pot doubles
instal­ments which are in Arithmetical Progression when every day, how much gold has the saint made due to his
30 of the instalments have been paid, he dies leaving one- magic pot?
third of the debt unpaid. The value of the 8th instalment
(230 − 69) (230 + 69)
is: (a) g (b) g
7 7
(a) `35 (b) `50
(c) `65 (d) None of these (230 – 71) (230 + 71)
(c) g (d) g
57.
Because of economic slowdown, a multinational company 7 7
curtailed some of the allowances of its employees. Rashid, [Based on JMET, 2011]
the marketing manager of the company whose monthly 62. If x, y and z are in harmonic progression, which of the
salary has been reduced to `42000 is unable to cut down following statement(s) is/are true?
on his expenditure. He finds that there is a deficit of
`2000 between his earnings and expenses in the first y (x + z ) z( x − y)
I.  x = x=
II.
month. This deficit, because of inflationary pressure, 2z y−z
will keep on increasing by `500 every month. Rashid
y−z
has a saving of `60000 which will be used to fill this III. x =
deficit. After his savings get exhausted, Rashid would x−z
start borrowing from his friends. How soon will he start (a) I only (b) I and II
borrowing? (c) II only (d) II and III
(a) 10th months (b) 11th months [Based on CAT, 2009]
(c) 12th months (d) 13th months 63. The set of natural numbers N is divided into subsets
[Based on IIFT, 2009] A1 = (1), A2 = (2, 3), A3 = (4, 5, 6), A4 = (7, 8, 9, 10) and
so on. What is the sum of the elements of the subset A50?
58.
In a green view apartment, the houses of a row are
(a) 42455 (b) 61250
numbered consecutively from 1 to 49. Assuming that
there is a value of ‘x’ such that the sum of the numbers of (c) 62525 (d) 65525
the houses preceding the house numbered ‘x’ is equal to [Based on CAT, 2009]
the sum of the numbers of the houses following it. Then, 64. The value with which Ram Kumar buys Bank’s cash
what will be the value of ‘x’? certificates every year exceeds the previous year’s
(a) 21 (b) 30 purchase by `300. After 20 years, he finds that the total
(c) 35 (d) 42 value of the certificates purchased by him is `83000. Find
the value of the certificate purchased by him in the 13th
[Based on IIFT, 2010]
year.
59.
The operation (x) is defined by
(a) `4900 (b) `6900
I. (1) = 2 (c) `1300 (d) None of these
II. (x + y) = (x)(y) [Based on CAT, 2010]
for all positive integers x and y 65. The sum of 3rd and 15th elements of an arithmetic
n progression is equal to the sum of 6th, 11th, and 13th
If ∑ ( x) = 1022, then n is equal to elements of the same progression. Then which element of
x =1 the series should necessarily be equal to zero?
(a) 8 (b) 9 (a) 1st (b) 9th
(c) 10 (d) 11 (c) 12th (d) None of these
[Based on XAT, 2010] [Based on CAT, 2010]

60.
In a list of 7 integers, one integer, denoted as x is unknown. 66. Let Sn denote the sum of the squares of the first n odd
The other six integers are 20, 4, 10, 4, 8 and 4. If the mean, natural numbers. If Sn = 533n, find the value of n.
median and mode of these seven integers are arranged in (a) 18 (b) 20
increasing order, they form an arithmetic progression. The (c) 24 (d) 30
sum of all possible wayes of x is: [Based on CAT, 2011]

M29_KHAT6981_C29.indd 13 6/17/2015 12:15:12 PM


29.14 Chapter 29

67. Consider a sequence S whose nth term Tn is defined as 72. What is the value of the following expression?
1 + 3/n, where n = 1, 2, …. Find the product of all the  1   1   1   1 
consecutive terms of S starting from the 4th term to the  2 + 2 + 2  + ... +  2 
 (2 − 1)   (4 − 1)   (6 − 1)   (20 − 1) 
60th term.
(a) 1980.55 (b) 1985.55 (a)
9
(b)
10
19 19
(c) 1990.55 (d) 1975.55
[Based on CAT, 2012] 10 11
(c) (d)
21 21
68. If (a 2 + b 2 ), (b 2 + c 2 ) and (a 2 + c 2 ) are in geometric [Based on CAT, 2000]
progression (G.P.), which of the following holds true?
73. Let un + 1 = 2un + 1, (n = 0, 1, 2, …) and u0 = 0. Then, u10
a4 − c4 a4 − c4 would be nearest to:
(a) b 2 − c 2 = 2 2
(b) b 2 − a 2 = 2
b +a b + c2 (a) 1023 (b) 2047
b4 − a 4 b4 − c4 (c) 4095 (d) 8195
(c) b 2 − c 2 = 2 2
(d) b 2 − a 2 = 2 [Based on CAT, 1993]
b +a b + a2
[Based on CAT, 2012] 74. A young girl counted in the following way on the fingers
of her left hand. She started calling the thumb 1, the index
69. If ax + bx + c = 0 and 2a, b and 2c are in arithmetic
2
finger 2, middle finger 3, ring finger 4, and little finger
progression (A.P.), which of the following are the roots of
5, then reversed the direction, calling the ring finger 6,
the equation?
middle finger 7, index finger 8, thumb 9 and then back to
(a) a, c (b) –a, −c the index finger for 10, middle finger for 11 and so on. She
a c c counted up to 1994. She ended on her:
(c) − , − (d) − , −1
2 2 a (a) thumb (b) index finger
[Based on CAT, 2012] (c) middle finger (d) ring finger
70. The sum of first ten terms of an AP is 155 and the sum of [Based on CAT, 1993]
first terms of a GP is 9. The first term of the AP is equal to 75. Balls are arranged in rows to form an equilateral triangle.
the common ratio of the GP and the first term of the GP is The first row consists of one ball, the second row consists
equal to the common difference of the AP. Which can be of two balls and so on. If 669 more balls are added, then
the AP as per the given conditions? all the balls can be arranged in the shape of a square and
(a) 2, 4, 6, 8 (b) 2, 5, 8, 11, each of the sides contains 8 balls less than each side of the
25 79 83 triangle had. The initial number of balls is:
(c) , , , (d) Both (b) and (c) (a) 1600 (b) 1500
2 6 6
[Based on CAT, 2013] (c) 1540 (d) 1690
71. If a1, a2, a3, . . . ,an be an AP and s1, s2 and s3 be the sum
of first n, 2n and 3n terms respectively, then S3−S2–S1 is 76. x, 17, 3 x − y 2 − 2, and 3x + y 2 − 30, are four consecutive
equal to (where a is the first term and d is the common terms of an increasing arithmetic sequence. The sum of
difference): the four numbers is divisible by:
(a) 3a– 2n–d (b) a(n + 2d) (a) 2 (b) 3
(c) 3a + 2nd (d) 2n2d (c) 5 (d) 7
[Based on CAT, 2013] [Based on XAT, 2014]

M29_KHAT6981_C29.indd 14 6/17/2015 12:50:26 PM


29.15
Progressions

Answer Keys
Difficulty Level-1

1. (b) 2. (b) 3. (c) 4. (a) 5. (a) 6. (c) 7. (b) 8. (b) 9. (b) 10. (b ) 11. (c) 12. (a) 13. (d)
14. (b) 15. (c) 16. (d) 17. (c) 18. (c) 19. (b) 20. (c) 21. (a) 22. (d) 23. (c) 24. (c) 25. (c) 26. (b)
27. (b) 28. (c) 29. (b) 30. (d ) 31. (b) 32. (b) 33. (b ) 34. (c) 35. (c) 36. (a) 37. (c) 38. (a ) 39. (a)
40. (d ) 41. (c) 42. (a) 43. (d ) 44. (d) 45. (c) 46. (a) 47. (c ) 48. (b ) 49. (b) 50. (c) 51. (a) 52. (c)
53. (a) 54. (b) 55. (b)

Difficulty Level-2

1. (c) 2. (d) 3. (c) 4. (d) 5. (a) 6. (b) 7. (a) 8. (a) 9. (b) 10. (c) 11. (b) 12. (a) 13. (a)
14. (b ) 15. (c) 16. (c) 17. (c) 18. (b) 19. (a) 20. (c) 21. (a) 22. (a) 23. (c) 24. (a ) 25. (b) 26. (c)
27. (b) 28. (c) 29. (b) 30. (b) 31. (a ) 32. (c) 33. (b) 34. (d) 35. (d ) 36. (b) 37. (c ) 38. (c) 39. (a)
40. (b) 41. (c) 42. (a) 43. (c) 44. (a) 45. (c) 46. (c ) 47. (d) 48. (c) 49. (b) 50. (a) 51. (a) 52. (d)
53. (b) 54. (d ) 55. (a) 56. (c ) 57. (d) 58. (c) 59. (b) 60. (d) 61. (c) 62. (b) 63. (c ) 64. (a) 65. (c)
66. (b) 67. (b) 68. (b) 69. (d) 70. (d) 71. (d) 72. (c) 73. (a) 74. (c) 75. (c) 76. (a )­

Explanatory Answers

Difficulty Level-1

1.
(b) Let a be the first term and d be the common ratio of an 1 1 1 1 1 1 1
A.P. 4.
(a) Mohan ate + + + + + + of the
2 4 8 16 32 64 128
\ (a + 5d) + (a + 14d) pizza during the week. That is,
= (a + 6d) + (a + 9d) + (a + 11d) 1 1 
7
1 −   
⇒ a + 7d = 0 2   2   1 127
⇒ 8th term = 0. = 1 − = = 99.22%
1 128 128
1−
2.
(b) Let a be the first term of the series in A.P. 2
Let d be its common difference.  1 1 
 Here
= a = ,r for the given G.P. of 7 terms 
\ (a + 5d) + (a + 14d)  2 2 
= (a + 6d) + (a + 9d) + (a + 11d) 5.
(a) Given statement
⇒ 2a + 19d = 3a + 26d ⇒ a = –7d ⇒ (ax/2)2 = (logb x) × (logx a) ⇒ ax = logb a
⇒ 8th term = a + 7d ⇒ –7d + 7d = 0. ⇒ x loga = loga [logb a] ⇒ x = loga [logb a].
3.
(c) p, q, r, s are in harmonical progression 6.
(c) Let n be the number of months in which all the
1 1 1 1 instalments can be paid
⇒ , , and are in A.P.
p q r s First instalment = `100
1 1 1 1 Last instalment = `5
⇒ − = −
q p s r Common Difference = –5
1 1 1 1 ⇒ Sum of the series with n terms whose first term is
⇒ + = + .
q r p s 100 or common difference is (–5) = 975

Chapter_29.indd 15 6/5/2015 4:25:48 PM


29.16 Chapter 29

n Put n = 15, we have, 12 + 22 + 32 + 42 + ... + 152


i.e.,  [2a + (n – 1) d] = 975 15(15 + 1)(30 + 1)
2 = = 1240.
6
n
i.e.,  [2 × 100 + (n – 1) (–5)] = 975 10.
(b) nth term = a + (n – 1)d = 4n + 1
2 where a = first term and d = common difference
i.e.,    n2 – 41n + 390 = 0 \ (a – d) + nd = 1 + 4n ⇒ a – d = 1, d = 4
i.e.,      n = 26 or n = 15 ⇒ a = 5.
For n = 15, total amount paid 1 1 1 1
11.
(c) − = −
15 b−a c a b−c
=  [2 × 100 + (15 – 1)(–5)]
2 2ac 1 1 1
⇒ b = ⇒ , , are in A.P.
15 a+c a b c
=  [200 – 70] = 975.
2 ⇒ a, b, c are in H.P.

n (a) nth term of a G.P. = arn–1


12.
7.
(b) Sn =
[ a + (n − 1) d ] where a = first term and r is the common ratio
2
[where a is the first term and d is \ 8th term = 5 × (2)7 = 5 × 128 = 640.
the common difference] 13.
(d) Let x, y be the numbers
n
S2n=
[ a + (n − 1) d ] x+y
2 \ = 5  and   xy = 4 ⇒ xy = 16
3n 2
S3n = [ a + (3n − 1) d ] \ x + y = 10, xy = 16
2
⇒ (x – y)2 = (x + y)2 – 4xy = 100 – 64 = 36
Given, S2n = 3Sn
⇒ x – y = 6
n  \ x = 8, y = 2.
⇒ n [a + 2nd – d] = 3  (a + nd − d ) 
2  14.
(b) Number of terms in the first n groups = 1 + 2 + 3 + ...
a n(n + 1) n2 + n
⇒ d = n = =
1+ n 2 2
So, first term is the (n + 1)th group
3n
[ a + 3nd − d ]
S3n n2 + n
\ = 2 = +1
Sn n 2
[ a + nd − d ]
2
n2 + n + 2
= .
 3na a  2
3 a + − 3n − 1
1 + n 1 + n  > 2000 ⇒ 3n > 4001 ⇒ n = 8.
=  = 6. 15.
(c)
3 −1
na a
a+ −
1+ n 1+ n n
2 16.
(d) Sn = [2 a + (n − 1) d ]
8.
(b) Let the three numbers be a, ar, ar , where r is the 2
common ratio. where a = 4, d = ?, n = 12
\ a + ar + ar2 = 28 and a3r3 = 512 12
\ ar = 8 ⇒ a + ar2 = 20 \ S12 = 256 = [2 × 4 + (12 − 1) × d ]
2
⇒ 8r2 – 20r + 8 = 0
104
1 ⇒ d =
⇒ r = 2,  r = 33
2
\ Last term = 12th term = T12
If r = 2, a = 4. Therefore, the three numbers are 4,
T12 = a + (n – 1) d
8, 16.
104
= 4 + (12 − 1) ×
9.
(b) The sum of the squares of the first n natural numbers is 33
n (n + 1)(2 n + 1) 104 116
= 4 + =.
6 3 3

Chapter_29.indd 16 6/5/2015 4:25:49 PM


29.17
Progressions

17.
(c) Let the numbers be a and b, then 23.
(c) Let, S = 1 + 2 + 4 + 7 + ... + Tn
2ab or, S = l + 2 + 4 + ... + Tn–1 + Tn
= 4 and a + b + ab = 27
a+b Subtracting, we get
⇒ 2(a + b) – ab = 0 and a + b + ab = 27 0 = 1 + [1 + 2 + 3 + ... (n – l)] – Tn
⇒ ab = 8 and a + b = 9 ⇒ Tn = l + 2 + 3 + ... + (n – 1) + 1
⇒ a = 6, b = 3 n(n − 1)
= +1
2
18.
(c) Let the 3 numbers in the geometric progression be a, \ First number of 50th term,
ar, ar2.
50 × 49
\ a = ar + ar2 = + 1 = 1226
2 2
\ r + r = 1
\ Sum of numbers of 50th term,
⇒ r2 + r – 1 = 0
= 1226 + 1227 + … upto 50th term
− 1 ± (1) 2 − 4(1) (−1) 50
\ r = = [2 × 1226 + (50 – 1) × 1]
2(1) 2
−1± 5 = 25 × 2501 = 62525.
=
2 24.
(c) Let the first instalment be ‘a’ and the common
Q All the numbers are positive, the ratio cannot be difference between any two consecutive instalments
negative be ‘d’.

−1− 5 Using the formula for the sum of an A.P.


\ is not possible
2 n
Sn = [2a + (n − 1)d ]
2
5 –1
\ r =
. We have,
2
19.
(b) As per the given information: (a + d) × (a + 13d) = 40
3600 = [2a + (40 – 1)d] = 20(2a + 39d)
(a + 5d)2 Solving, a = 3d 2
Now common ratio of G.P., (a + 5d)/(a + d) = 2. ⇒ 180 = 2a + 39d (1)
2
a a 30
20.
(c) = 6 and = 24 Again, 2400 = [2a + (30 – 1)d]
1− r 1 − r2 2
a = 15(2a + 29d)
Dividing, = 4
1+ r ⇒ 160 = 2a + 29d (2)
1 24 Solving Eqs. (1) and (2),
⇒ r = and a =
5 5 Therefore, 180 = 2a + 39 × 2
24 ⇒ 2a = 102 ⇒ a = 51
Hence, second term of the first series = a × r = .
25
Value of 8th instalment
21.
(a) Average of 49th, 50th and 51st term = 50th term = 49
= 51 + (8 –1) × 2 = 51 + 14
Hence, (a + 49d) = 49
= `65.
Sum of the first 99 terms = 49 terms before 50th
term + 50th term + 49 terms after 50th term = 99 × 25.
(c) Let n be the number of members in the club.
50th term = 99 × 49 = 4,851. n 3
Then, 250 =  2 × 7 + (n − 1) 
22.
(d) Required numbers are 12,15,18, …., 99 2 12 
This is an AP with a = 12 and d = 3 n 1 1
= 14 + n −
⇒ 250
\ Tn = a + (n – 1)d 2 4 4 
99 = 12 + (n – 1) × 3
99 – 12 n2 n
⇒ n – 1 = ⇒ 250 = 7 n + −
3 8 8
n = 29 + 1 = 30. ⇒ n = 25.

Chapter_29.indd 17 6/5/2015 4:25:49 PM


29.18 Chapter 29

26.
(b) T5 = a + (n – 1) × d Solving Eqs. (1) and (2),
2 = – 14 + 4d x = 8 and y = 2.
16 ( A + B)
d = =4 31.
(b) AB = 0.8 ×
4 2
n or, AB = 0.16 (A + B)2
\ Sn = [2a + (n – 1) × d]
2
Using option (b),
n we find that 4 = 0.16 (4 + 1)2 ⇒ 4 = 4.
⇒ 40 = [– 28 + (n – 1) × 4]
2
(b) Given 2 log3(2x – 5) = log32 + log3(2x – 7/2)
32.
⇒ 80 = – 28 n + 4n2 – 4n
⇒ (2x – 5)2 = 2[2x – 7/2]
⇒ 4n2 – 32n – 80 = 0
= 22x – 10.2x + 25 = 2.2x – 7
⇒ n2 – 8n – 20 = 0
⇒ 22x – 12.2x + 32 = 0
⇒ (n – 10) (n + 2) = 0
⇒ y2 – 12y + 32 = 0
\ n = 10. (Q n ≠ – 2)
⇒ (y – 8) (y – 4) = 0
27.
(b) Let the last term be n, then
or, y = 4, 8
a + arn–1 = 66 (1)
\ 2x = 32, 22 or, x = 3, 2
and, ar × arn–2 = 128
But, x = 2, gives 2x – 5 = –1
⇒ a2rn–1 = 128 (2)
\ x = 2 is impossible, so x = 3.
From Eqs. (1) and (2),
a (66 – a) = 128 33.
(b) Let the polygon has n sides.
2
⇒ a – 66a + 128 = 0 Given the smallest interior angle is 120°, hence the
greatest exterior angle will be (180° – 120°) = 60°.
⇒ a = 64, 2. We know sum of exterior angles of a polygon = 360°
(c) (x + 1)th term – xth term
28. 60 + 55 + 50 + ... = 360
= (x + 1)2 + 1 – (x2 + 1) {Common difference = –5}
= x2 + 2x + 1 + 1 – x2 – 1 = 2x + 1. n
\ [2a + (n − 1)d ] = 360
29.
(b) 1, 4, 9, 16, 25, 2
(1)2 (2)2 (3)2 (4)2 (5)2
n
Each term of the progression is the square of a [120 + (n − 1) × – 5] = 360
2
natural number
Hence, the eighth term of the sequence will be ⇒ n2 – 25n + 144 = 0
(8)2 = 64. ⇒ n = 9, 16
Number of sides cannot be 16
30.
(d) Let the two numbers be x and y Hence, n = 9.
Then, A.M.
34.
(c) Sum of 40 instalments S40 = 3600
x+ y
=5 = 20 (2a + 39d)
2
⇒ x + y = 10 ⇒ 2a + 39d = 180 (1)

and, GM, xy = 4
G.M. (1) Sum of 30 instalments S 30 = 2400 = 15(2a + 29d)

xy = 16 ⇒ 2a + 29d = 160 (2)

⇒ (x – y)2 = (x + y)2 – 4xy Solving Eqs. (1) and (2), we get


= 100 – 64 = 36 a = 51 and d = 2
or, x – y = 6 (2) \ The value of first instalment = `51.

Chapter_29.indd 18 6/5/2015 4:25:50 PM


29.19
Progressions

35.
(c) Using the digits 0, 1, 2, 3, 4 and 5, five-digit numbers n 1
divisible by 3, can be formed using the following =  2(1)  (n  1) 
2 2
combinations.
Case (i): 1, 2, 3, 4, 5 n n 1  n  n  3
= 2  = 
Total number of numbers formed using these digits 2  2 2  2  2 
= 5! = 120 n 2 + 3n
= .
Case (ii): 0, 1, 2, 4, 5 4
Total number of numbers formed using these digits 41.
(c) a, b, c form an A.P.
= 4 × 4 × 3 × 2 = 96 2b = a + c
Thus, total numbers = 120 + 96 = 216 Increasing a by 1 or c by 2 results in a G.P.
Hence, option (c). \ b2 = (a + 1)c (1)
and, b2 = a(c + 2) (2)
36.
(a) We know that \ (a + 1)c = a(c + 2)
Sum of even number = n (n + 1) \ ac + c = ac + 2a
Here, n = 49 \ c = 2a
\ Sum = 49 × 50 = 2450. Now, 2b = a + c
37.
(c) Sum between 100 to 200, \ 2b = a + 2a
50 3a
(101  199) Sn = \ b =
2 2
= 25 × 300 = 7500. Putting this in Eq. (1), we get

(a) Let ax = by = cz = k
38. 9a 2
= (a + 1) 2a
1 1 4
⇒ a = k x , b  k y 9a
\ = 2a + 2
1 4
and, c = kz \ 9a = 8a + 8
2 1 1
\ a = 8
Q 2
b = ac ⇒ k  y
kz kx 3a
\ b =
2
2 1 1

⇒ k y = k z x 38
= = 12.
2
2 1 1
⇒ = + .
y z x 42.
(a) Let the first term and the common difference of the
arithmetic progression be a and d, respectively. It is
39.
(a) The sum of interior angles of a pentagon = 540° given the sum of the first ten terms is equal to four
Let the angles of the pentagon be a – 2d, a – d, times the sum of the first five terms.
a, a + d, a + 2d 5
Sum of the first five terms = [2a  4d ]
\ a – 2d + a – d + a + a + d + a + 2d = 540 2
\ 5a = 5400 10
Sum of the first ten terms = [2a  9d ]
\ a = 108° 2
\ One of the angles must be 108°. 10 5 
Given, [2a  9d ] = 4  (2a  49d ) 
1 2  3 
40.
(d) xk 1 = xk 
2 2a + 9d = 2[2a + 4d]
\ x1, x2, x3, ... xn form an arithmetic progression with
d =2a
common difference d = 1/2
Thus, the ratio of the first term to the common
Q x1 = 1, first term a = 1
difference
n a 1
Sn = [2a  (n  1)d ] = = .
2 d 2

Chapter_29.indd 19 6/5/2015 4:25:51 PM


29.20 Chapter 29

1 2 1 2 Now, log (a + c) + log (a – 2b + c)


43.
(d) + + + + ...
7 7 2 73 7 4 = log [(a + c) × (a – 2b + c)]
1 1  2 2  = log [a2 – 2ab + ac + ac – 2bc + c2]
=  + 3 + ... +  2 + 2 + ...
7 7  7 7  = log [(a2 + c2 + 2ac) – 2ab – 2 bc]

2 = log [(a + c)2 – 2b(a + c)]


1
7 2
7 2a c
= + = log [(a + c)2 – 2 × × (a + c)]
1 1 ac
1− 2 1− 2
7 7 = log [(a + c)2 – 4ac]
7 2 = log [a – c]2
= +
48 48 We know that c > a
9 3 and, (a – c)2 = (c – a)2
= = .
48 16 \ log (c – a)2 ⇒ 2 log (c – a).
44.
(d) Number are 1, 2, 3, 4, 5, 8
48.
(b) a b c = 4 and a, b, c are in AP.
\ Total digit = 6 Then, value of b will be minimum when all three
Here repetition of digits is allowed are equal.
\ First place can be filled by any 6 ways, \ b3 = 4
and second place can be filled by 3 ways 1 2

6 3
Total number of ways = Pl × P1 = 6 × 3 = 18. ⇒ b = 4 3  2 3
45.
(c) Since, x, y, z are in A.P. and also in H.P. 49.
(b) Given series is
x+z 2 3 6 11 18
In A.P. y =      ...
2 1! 2! 3! 4! 5!

2xz So, the nth term of the given series is
In H.P. y =
x+z 2  (n  1) 2
tn =
2xz x+z n!
\ =
x+z 2 ∞ 2 (n − 1) 2 

2
⇒ (x + z) – 4 xz = 0
and, Sn = ∑  n! + n! 

n =1
\ x = z ∞ 2 n 2 − 2n + 1 
Hence y, z and x connot be in A.P. = ∑  n! + n!


n =1
46.
(a) ar = 2 ∞
2 n 2 1
⇒ a =
2 = ∑  n! + (n − 1)! − (n − 1)! + n!
r n =1

2 25 ∞
3 n −1+1 2 
Also,
r (1 − r )
=
2
= ∑  n! + −
(n − 1)! (n − 1)!
n =1
4 1
\ r = , ∞
3

 1 

 1 2 
5 5 = ∑  n! + ∑  (n − 2)! + ∑  (n − 1)! − (n − 1)!
2 3 =n 1=n 2 n =1
\ ×r T4 = ar3 =
r ∞
3 ∞
1 ∞
1
= 2r2 = 2 ×
1
or, 2 ×
16 = ∑ n! + ∑ (n − 2)! − ∑ (n − 1)!
n=
1 n−2 n=
1
25 25
2 32  1 1 
= or, . = 3 1 + + + ... + e − e
25 25  2! 3! 
47. (c) a, b, c are in H.P. = 3(e – 1)
2a c 1 1
\ b = (1) Remember that, e = 1 + + + ...
ac 2! 3!

Chapter_29.indd 20 6/5/2015 4:25:52 PM


29.21
Progressions

50.
(c) As per the given condition, number in the highest The largest angle should be a multiple of one of
position should be either 6 or 7, which can be done in the convex angles and less than 180º, hence it can
two ways. be 175º.
If the first digit is 6, the other digits can be arranged in 53.
(a) Given series be 35, 42, 49, ..., 329
Here, a = 35
6!
= = 360 ways d = 7
2!
If the first digit is 7, the other digits can be arranged in and, tn = 329, than n = ?
6! \ tn = a + (n – 1)d
= = 180 ways
2!  2! ⇒ 329 = 35 + (n – 1) × 7
Thus, the required possibilities for ⇒ 294 = (n – 1)7
n = 360 + 180 = 540 ways. 294
⇒ n – 1 = = 42
7
1 1 1 1
51.
(a) + + + = 0 \ n = 42 + 1 = 43.
x z−y z x− y
(2n − 4) × 90
x+z− y x+z− y 54.
(b) = 140
⇒ + = 0 n
x( z − y ) z( x − y)
18n – 36 = 14n
⇒ xz – xy + zx – zy = 0
\ n = 9.
2xz = y(x + z)
2 1 1 1 1 1  1 1 
= + 55.
(b) +  +  +  +  +…
y x z 1  3 6   10 15 
y
Hence, x, y, z are in H.P. and x, , z are in A.P. 1 1 1
2 = 1 + + + + ...
2 6 12
52.
(c) Let the angle be 3 1 1 
= 1 + + 2 + ...∞ 
(a – 5d) + (a – 3d) + (a – d) 2 2 2 
+ (a + d) + (a + 3d) + (a + 5d) = 720 3 1
2  1 3
[Sum of angles of hexagon = 720º] = 1 −  = × 2 =3.
 2 2
⇒ 6a = 720º ⇒ a = 120º

Difficulty Level-2

1.
(c) Let the first term be a and common difference d. \ 2log3 (2x – 5) = log32 + log3(2x – 7/2)
As we know, nth term of an AP ⇒ (2x – 5)2 = 2 × (x2 – 7/2)
= a + (n – 1) d Putting 2x = t
According to the question, ⇒ (t – 5)2 = 2 × (t – 7/2)
a + (3 – 1) d + a + (15 – 1) d ⇒ t2 – 10t + 25 = 2t – 7
= a + (6 – 1) d + a + (11 – 1) d + a + (13 – 1) d ⇒ t2 – 12t + 32 = 0 ⇒ t = 4, 8
⇒ 2a + 16 d = 3a + 27 d Now, 2x = 4
⇒ a = –11 d ⇒ x = 2 which is not possible because 2x – 5 is
negative.
we have to find the value of n such that a + (n – 1) 
d = 0. If, 2x = 8 ⇒ x = 3
\ 0 = –11d + (n – 1)  d ⇒ n = 12. \ x = 3.

3.
(c) Let there be total n layers of balls.
(d) log32, log3 (2x – 5), log3 (2x – 7/2) are in arithmetic
2.
progression, 1st layer → 1 ball

Chapter_29.indd 21 6/5/2015 4:25:52 PM


29.22 Chapter 29

2nd layer → 3 balls 6.


(b) Since y – x = z – y
3rd layer → 6 balls \ x, y and z are in AP.
4th layer → 10 balls Let x, y and z are (a – d), (a) and (a + d)
... ... ... ... ... ... ... ...
Again, xyz = 4
n (n + 1)
nth layer → balls ⇒ (a – d) a (a + d) = 4
2
⇒ a (a2 – d2) = 4
According to the question,
4 4
n (n + 1) ⇒ a2 – d2 = ⇒ d2 = a2 −
∑ 2
= 8436 a a
For minimum possible value of y, i.e., a, d should be
n2 n
⇒ ∑2 +
2
= 8436 equal to zero. That is,
4 4
n2 n a2 − = 0 ⇒ a2 = ⇒ a3 = 22
⇒ ∑ 2 + ∑ 2 = 8436 a a
⇒ a = (22)1/3 = 22/3.
1 n (n + 1)(2n + 1) 1 n (n + 1)
⇒ × + × = 8436
2 6 2 2 7.
(a) Let, S = 1 + 2x + 4x2 + 7x3 + 10x4 + ... (1)
1  n (n + 1)(2n + 1) n (n + 1)  Then,
⇒  +  = 8436
2 6 2  xS = x + 2x2 + 4x3 + 7x4 + ... (2)
Now, go through the options: From (1) and (2),
n = 36 satisfies S(1 – x) = 1 + x + 2x2 + 3x3 + 4x4 + ... (3)
\ Number of layers = 36. 2 3 4
xS(1 – x) = x + x + 2x + 3x + ... (4)
4.
(d) T = {3, 11, 19, 27, ..., 451, 459, 467} From (3) and (4),
Terms in set T are in A.P. S(1 – x)2 = 1 + x2 + x3 + x4 + ...
We have to find the number of elements in S (subset ⇒ S(1 – x)2 = 1 + x2(1 + x + x2 + x3 + ...)
of T) such that no two elements add up to 470. As we
x2
know, in a finite A.P. the sum of the terms equidistant ⇒ S(1 – x)2 = 1 +
from beginning and end is always the same. Sum of 1− x
first and last term = 3 + 467 = 470.
[1 – x(1 – x)]
467 − 3 \ S =
Number of terms = + 1 = 59 (1 – x)3
8
So, there are 29 pairs which give 470 as sum. 1 1 1
8.
(a) Area = 36 + 36 + 36 + 36 ...
3, 11, 19, 27, ..., 227, 235, 243, ?? 443, 451, 459, 467 2 4 8

So, the number of element such that sum of no two 36


= = 36 × 2 = 72
elements is 470 = 59 – 29 = 30. 1
1−
2
5.
(a) No. of students who answered one or more questions
wrongly = 2n–j = 72 cm2.

The value of j lies between 1 and n (including 1 and n) 9.


(b) The two-digit number is of the form 7n + 3
\ Total number of wrong answers First two-digit number will be for n = 1
= 2n–1 + 2n–2 + 2n–3 + ... + 21 + 20 i.e., 7 × 1 + 3 = 10
Last two-digit number will be for n = 13
According to the question,
i.e., 7 × 13 + 3 = 94
20 + 21 + ... + 2n–3 + 2n­–2 – 2n–1 = 4095
No. of teems = 13
1(2n − 1) 13
⇒ = 4095 ⇒ 2n – 1 = 4095 Sum of all 13 terms =
(10 + 94)
2 −1 2
⇒ 2n = 4095 + 1 = 40996 ⇒ n = 12. = 13 × 52 = 676.

Chapter_29.indd 22 6/5/2015 4:25:53 PM


29.23
Progressions

10.
(c) We have to find the sum of the series Given, n = 20
4 9 16 25
1 + + 2 + 3 + 4 + ... 20
7 7 7 7 \ S (a, b) = [2(a + b)2 + (20 − 1)(−2 ab)]
2
1
Putting = x we get = 10 [2a2 + 2b2 + 4ab – 38ab]
7
= 20 [a2 + b2 – 17ab]
1 + 22x + 32x2 + 42x3 + 52x4 + ...
\ S (7, 3) = 20 [49 + 9 – 357]
Let, S = 1 + 4x + 9x2 + 16x3 + 25x4 + ... = 20 × (–299) = –5980.
S.x = x + 4x2 + 9x3 + 16x4 + ... 14.
(b) To find the sum of the series:
S – S.x = 1 + 3x + 5x2 + 7x3 + 9x4 + ... 10 + 10 + 15 + 15 + (20 + 20) + ... + (150 + 150) + 155
x (S – Sx) = x + 3x2 + 5x3 + 7x4 + ... = 2 (10 + 15 + ... + 150) + 155
(S – Sx) – {x (S – Sx)}  29 
= 2  (2 × 10 + (29 − 1) × 5)  + 155
= 1 + 2x + 2x2 + 2x3 + ... + to ∞ 2 
2x = 29 (20 + 140) + 155
⇒ (1 – x)2 S = 1 + ; Since | x | < 1
1− x = 29 × 160 + 155
= 4640 + 155 = 4795.
1+ x
⇒ S = 3 15.
(c) Max. sum = S
(1 − x )
= 50 + 48 + 46 + ... + 6 + 4 + 2
We may use it as direct formula for solving this type 25
of problem. =  [2 × 50 + (25 – 1) (–2)]
2
1
Substituting x = we get 25
7 =  [100 – 48] = 25 × 26 = 650.
2
1
1+ 16.
(c) The roots of the given equation are –2, –1, –2/3.
7 8 × 243 49
S = 3
= = . These roots are neither in A.P. nor in G.P. These roots
 1 7 × 216 27
 1 −  1 3
 7 are in H.P. because − , − 1, − are in A.P. with
2 2
11. (b) Let S = 1 + 2 × 2 + 3 × 22 + 4 × 23 + ... + 100 × 299 common difference −
1
.
2
\ 2S = 1 × 2 + 2 × 2 + 3 × 2 + ... + 99 × 2 3 99 2

+ 100 × 2100 2
\ –2, –1, − are in H.P.
Subtracting, we get 3
–S = 1 + 1 × 2 + 1 × 22 + 1 × 23 + ... + 1 × 299 17.
(c) Let x be the original number of apples, then first
100 x 1 x +1
– 100 × 2
customer bought + = , the 2nd customer
1(2100 − 1) 2 2 2
= – 100 × 2100
2 −1 1 x +1 1 x +1
100 100
bought x−  + = 2 , the third customer
= 2 – 1 – 100 × 2 2 2  2 2

\ S = 100 × 2100 – 2100 + 1 = 99 × 2100 + 1 1 x +1 x +1 1 x +1


bought t x− −  + = 3 and the 7th
2 2 4  2 2
12.
(a) Given expression
x +1
customer bought , we thus have the following
2 27
15 × 16  15 × 16
=   − 2 = (120)2 ­– 120 x +1 x +1 x +1 x +1
 2  equation: + 2 + 3 + ... + 2 = x
2 2 2 2
= 120 × 119 = 14280. or, (x + 1)
2 2 2 2
13.
(a) (a + b) , (a + b ), (a – b) , ...
1 1 1 1 
This is a series in A.P. with common difference  + 2 + 3 + ... + 7  = x
(–2ab). 2 2 2 2 

Chapter_29.indd 23 6/5/2015 4:25:54 PM


29.24 Chapter 29

Computing the sum of the terms of the G.P. in the = 98 + 97 + 96 + ... + 1


parentheses, we get 98(98 + 1)
x 1 = = 4851.
= 1 − 7 ⇒ x = 27 – 1 = 127. 2
x +1 2
1
+
2 3
+ +
4
+ ... 22.
(a) x, y, z are in G.P.
(b) The given product = 2 4
18. 8 16 32
x + y, y + z, z + x are in A.P.
(1)
1 2 3 4 y z
Let, S = + + + + ... (1) \ Common ratio of the G.P. = = = r, say
4 8 16 32 x y
1 1 2 3 Also (1)
\ S = + + + ... (2)
2 8 16 32 ( x + y) + ( z + x )
⇒ y + z = ⇒ 2x = y + z
1 2
1 1 1 1 14
(1) – (2) S = + + + ... = = y y 2y 2
2 4 8 16 1 2 \ r= = = =
1− x ( y + z )/2 y+z z
2 1+
\ The given product = 21 = 2. y
2
⇒ r =
19.
(a) Sn = Sum of n terms of an A.P. 1+r
n ⇒ r2 + r – 2 = 0
= [2a + (n − 1) d ]
2 ⇒ (r – 1) (r + 2) = 0
where a = first term, d = common difference ⇒ r = –2 [r = 1 ⇒ x = y = z].
3 + 5 + 7 + ... + n terms
\ = 7 23.
(c) Let the digits of a three-digit number be x, y and z and
5 + 8 + 11 + ... + 10 terms
the number be 100z + 10y + x, where x, y, z are in G.P.
n
[2 × 3 + (n − 1) × 2] \ y2 = xz
⇒ 2 = 7 ⇒ xz must be a square number
10
[2 × 5 + (10 − 1) × 3] i.e., xz = 9, i.e., x = 1, z = 9
2
\ y = 3, so that x, y, z are in A.P.
n (2n + 4)
⇒ = 7 \ The number is 931.
370
⇒ 2n2 + 4n – 2590 = 0 ⇒ The other number will be 531 so that 1, 3 and 5 are
in A.P. Their sum = 931 + 531 = 1462.
⇒ n2 + 2n – 1295 = 0
2 24.
(a) 4 + 12 + 20 + 28 + ... + 996
⇒ n + 37n – 35n – 1295 = 0
⇒ n (n + 37) – 35 (n + 37) = 0 Tn = a + (n – 1) d
⇒ (n – 35) (n + 37) = 0 Here, a = 4, d = 8,
⇒ n = 35. Tn = 996
n (n + 1) \ 996 = 4 + (n – 1) × 8
20.
(c) Sum of the first n natural numbers =
2 ⇒ 8n – 8 + 4 = 996 ⇒ n = 125
Sum of the squares of the first n natural numbers
125
n (n + 1)(2 n + 1) \ S125 = [2 × 4 + (125 − 1) × 8]
= 2
6
n (n + 1) 1  n (n + 1)(2 n + 1)  125
\ =  = [8 × 125] = 62500.
2 5 6  2

⇒ 2n + 1 = 15 ⇒ n = 7.
25.
(b) x, y, z are in G.P. ⇒ y = xz
21.
(a) x can take the values 1, 2, ... 98.
For x = 1, y can take the values 1, 2, ..., 98
a+c
For x = 2, y can take the values 1, 2, ..., 97 a, b, c are in A.P. ⇒ b =
2
For x = 98, y can take the value 1
a+c c−a a +c
Since z is dependent on x and y, therefore the −c a−
required number of solutions \ The expression = x 2 ( xz ).z 2 2

Chapter_29.indd 24 6/5/2015 4:25:55 PM


29.25
Progressions

a+c c−a c−a a+c \ S30 = `3240000 = `32.4 lakhs


−c+ +a−
= x 2 2 ⋅ z 2 2 Loss in the business = 10%
= x0z0 = 1. \ Amount he received for making the flyover
(c) log2, log (2x – 1),
26. 3240000
x = = `3,24,00,000 = `3.24 crores.
log (2 + 3) are in A.P. 0.1
⇒ 2 [log (2x – 1)] = log 2 + log (2x + 3) For answers to Questions 30 to 34:
= log [2 × (2x + 3)] Let the first term of the progression be a.
⇒ log (2x – 1)2 = log [2x+1 + 6)]
Also, the common ratio be r and common
⇒ (2x – 1)2 = 2x+1 + 6 = 2x×2 + 6 difference be d.
x
Let, 2 = y Thus, ar – a = (a + 4d) – a
\ (y – 1)2 = 2y + 6
or, a(r – 1) = 4d (1)
⇒ y2 – 2y + 1 = 2y + 6
Also, a(r2 – 1) = 24d (2)
⇒ y2 – 4y – 5 = 0
Dividing (2) by (1) we get: r + 1 = 6
⇒ (y – 5) (y + 1) = 0
Thus, r = 5.
⇒ y = 5, –1.
Putting in (1), 4a = 4d a = d.
If y = 5 ⇒ 2x = 5
Also, a + 4d = 5, we get a = 1, d = 1
⇒ x log 2 = log 5
Thus, we have the following answers.
log 5
⇒ x = ⇒ x = log25. 30.
(b)
log 2
31.
(a)
n 32.
(c)
27.
(b) tn =
n+2 33.
(b)
3 5 4 5 53 34.
(d)
t3 = = , t4 = , t5 , ... t53 =
3+ 2 3 6 7 55 35.
(d) Distance travelled by the ball ground till it rises 20 m
and then comes back to the ground = 40 m
3 4 5 53
t3 × t4 × t5 × … × t53 = × × × ... × Next it rises 10 m
5 6 7 55
\ Distance from ground to top to ground = 20 m, this
2 continues
        = .
495 So, the series = 40 + 20 + 10 + ...
28.
(c) The first day, I sell 1 book, on the second day, I sell
 1
2 books and so on. This is an A.P. and for one month 40 ÷ 1 −  = 80 m
(i.e., 30 days), the number of books sold is same as the  2
sum of first n natural numbers But the ball was first thrown from a height of 10 m.
n(n + 1) 30 × 31 \ Total distance = 80 + 10 = 90 m.
= = = 465
2 2 36.
(b) Given the arithmetic progressions
The numbers of books in the beginning is 465. A1 → 3, 9, 15, 21 . . . .   and
A2 → 5, 13, 21, 29
29.
(b) The sum of money that the contractor was supposed
to pay for the period of one month over the stipulated We can see that the first term common between the
time is two series is 21. Since the common difference of A1
and A2 are 6 and 8 respectively, any two consecutive
30
S30 = [2 × 50000 + (30 – 1) 4000] terms common between A1 and A2 differ by L.C.M.
2  (6, 8) i.e., 24. So, the series of common terms also
form an arithmetic progression. The series is 21, 45,
 n  69, . . . say let us call it series A3.
 Sn= 2 [2a + (n − 1)d ] ,
   Given that
where, a = 50,000,  n = 30,  d = 4000 (tn of A1) + (tn of A2) = 6,000
\ S30 = 10[100000 + 29 × 4,000] ⇒ (3 + (n – 1) 6) + (5 + (n – 1) 8) = 6,000

Chapter_29.indd 25 6/5/2015 4:25:56 PM


29.26 Chapter 29

⇒ 14n – 6 = 6,000 a 1
s ⇒ 4a = + ar ⇒ 4 = + r
6000 r r
⇒ n = = 429
14 ⇒   r = 2 ± 3
t429 of A1 = (3 + (429 – 1) 6 = 2,571
Since the G.P. is increasing, r = 2 + 3
t429 of A2 = 5 + (429 – 1) 8 = 3,427
So, all the terms common to A1 and A2 will be less 1
⇒ = 2 − 3
than 2571 r
\ The number of terms common to A1 and A2 is   Hence, ratio of the first number and third number =
same as the number of terms in A3 less than or equal 1:r2 = 1:7 – 4 3.
to 2571
40.
(b) Given that the speeds a, b, c . . . z are in A.P
 2571 − 21   2550  \ Let a, b, c . . . be a, a + D, a + 2D, . . . a + 25D.
i.e., 
24  + 1 =  24  + 1 = 107.
    Given time taken by Z to meet A, for the first time is
20 sec, i.e.,
1
37.
(c) Radius of the largest circle = 100
π = 20 ⇒ D = 0.2 minutes/second
25∆ + a − a

⇒ Area of largest circle = 1 square unit
Time taken by M to complete the race at a speed of
Now, each subsequent circle’s radius is half the a + 12D, is = 52 minutes and 5 second
radius of previous circle. Therefore, areas would be
100 × 100
circle fourth. ⇒ = 3125
a + 12∆
\ Sum of areas of all the circles is S, where
⇒ 10,000 = (a + 2.4) (3125)
1 1
S = 1 + + ...infinite terms ⇒ a = 0.8 minutes/second
4 16
\ Time taken by all of them to meet for first time at
4 the starting point is
\ S = square units.
3  100 100 100 100 
LCM  , , , .... 
38. 2 3 4
(c) 1 – y + y – y + y ... ∞ (|y| < 1)  a b c z 
1  100 100 100 100 
⇒ x = = LCM  , , , .... 
1+ y  0.8 1.0 1.2 0.8 + 25(0.2) 
1 LCM (100, 100, ...100) 100
⇒ y = − 1 (1) = = = 500 seconds.
x HCF(0.8, 1.0, 1.2 ..., 5.8) 0.2
and, z = 1 + y + y2 + y3 + ... ∞  (|y| < 1) 41.
(c) Let the first term and the common ratio of the
arithmetic progression be a and d respectively.
1
⇒ z = a + ( p − 1) d p
1− y Given, =
a + ( p + 2) d p+3
1 ⇒ pa + 3a + d (p – 1) (p + 3) = pa + dp (p + 2)
⇒ y = 1 − (2)
z ⇒ 3a = 3d  ⇒  a = d
From (1) and (2)
 3p 
1 1 2   [2d + (3 p − 1) d ] 61
2
+ =
x 2 1 Also given,   =
 4p  108
⇒ x, 1, z are in H.P.   [2d + (4 p − 1) d ]
 2 
or, 1 is the H.M. of x and z. 3  3p + 1 61
⇒   = ⇒ p = 20.
a 4  4 p + 1 108
39.
(a) Three numbers = , a, ar
r 42.
(a) Go through options
Double of second number = 2a Let 1, 2, 3, 4, 5, ... be an A.P. then
a 1 1 1 1
Hence, , 2a and ar are in A.P. + + +
r a1 + a2 a2 + a3 a3 + a4 a4 + a5

Chapter_29.indd 26 6/5/2015 4:29:40 PM


29.27
Progressions

1 1 1 1 But these 16 two-digit codes include 69 and 96,


= + + + which create no confusion. Apart from these, 10,
1+ 2 2+ 3 3+ 4 4+ 5
60, 80 and 90 are such two digit codes as create no
= − ( 1− 2 − ) ( 2− 3 − ) ( 3− 4 − ) ( 4− 5 ) confusion because these codes are no expected.
Hence total number of two-digit codes which create
no confusion
5+ 1
= 5− 1 = 5 − 1× = 81 – 16 + 6 = 71.
5+ 1
5 −1 4 n –1 46.
(c) By the given condition in the problem,
= = = .
5+ 1 5+ 1 a1 + an Area and perimeter of S1 = a2, 4a
43.
(c) n = 5 a 2 4a
Area and perimeter of S2 = ,
The weight of each stone (in kg) is 1, 3, 9, 27, 81 2 2
1 kg = 1 kg
a 2 4a
2 kg = 3 – 1 = 2 kg Area and perimeter of S3 = ,
4 ( 2) 2
3 kg = 3 kg
4 kg = (3 + 1) = 4 kg a 2 4a
Area and perimeter of S4 = ,
5 kg = 9 – (3 + 1) = 5 kg 8 ( 2)3
6 kg = (9 – 3) = 6 kg Then, required ratio
4a 4a 4a
7 kg = (9 + 1) – 3 = 7 kg 4a + + 2
+ + ...
2 ( 2) ( 2)3
8 kg = (9 – 1) = 8 kg =
a2 a2 a2
9 kg = 9 kg a2 + + + + ...
2 4 8
10 kg = (9 + 1) = 10 kg and so on
1 1 1
Remember he is allowed to put the stones on either 4a 1 + + 2
+ ...
side of the balance. 2 ( 2) ( 2)3
=
1 1 1
44.
(a) There are 2 n – j
students who answer wrongly. by 5 nor a 2 1 + + + ...
2 4 8
by 2. For j = 1, 2, 3, ...., n, the number of students will
be a GP with base 2. Hence, 1 + 2 + 22 + ... + 2n – 1 = 1
4,095. Using the formula, we get 2n = 4095 + 1 4a 2
1 4a
⇒ n = 12. 1−
2 2 −1
= =
1 a2 × 2
45.
(c) The digits which create confusion are 0, 1, 6, 8, 9. a2
1
The total number of two-digit codes having distinct 1−
2
digits and first digit non-zero.
1st place 2nd place 2
4a
9 options 9 options 2 −1 2 2 ( 2 + 1) 2(2 + 2)
⇒ 2
= = .
(1 to 9) (one digit is already used a ×2 a a
out of 1 to 9 and 0 is
n(n + 1)th
included) 47.
(d) The last instance of nth letter is letter of
2
= 9 × 9 = 81 such codes
series is S23 = 276 th and S24 = 300 th. All terms from
Total number of two digit codes which can create 276 to 300 are 24th letters of the alphabet i.e., x.
confusion
48.
(c) Let ‘d’ be the first term and ‘r’ be the common
1st digit 2nd digit difference
4 options 4 options a + ar = 12 (1)
(1, 6, 8 or 9) (one digit is already used ar2 + ar3 = 48 (2)
out of 1, 6, 8 and 9 and 0 On dividing Eq. (2) by Eq. (1), we get
is also included)
ar 2 (1  r )
= 4 × 4 = 16 =
a (1  r )

Chapter_29.indd 27 6/5/2015 4:25:58 PM


29.28 Chapter 29

48 nab  ab  a 2  nab
= ⇒ r2 = 4 ⇒ r = ± 2 =
12 a  nb
and since terms are alternately positive and negative
common ratio is ‘–2’ n ab  ab  a 2  nab
and, H1 – a =
\ First term is a – 2a = 12 a  nb
⇒ a = –12.
H1  a 2 nb  ab  a 2
\ =
49.
(b) 385 × 4 = 1540. H1  a ab  a 2

50.
(a) Let the initial number of boys and girls in the group 2 nb  b  a
=
= B and G, respectively. ba
After 15 girls leave, there are two boys for each girl.
Hn  b 2na  a  b
B = 2 (G – 15) (1) Similarly, =
Hn  b ab
Then, 45 boys leave after which there are 5 girls
for each boy,
5 (B – 45) = (G – 15) H1  a H n  b 2nb  2na
\  =
5B – 225 = G – 15 H1  a H n  b ba

From Eq. (1),
= 2n.
B = 2G – 30
54.
(d) a, b, c are in A.P.
5 (2G – 30) – 225 = G – 15 \ 2b = a + c
⇒ 9G = 225 – 15 + 150 but it is given that
3
360 b + a + c =
G = = 40. 2
9
3
⇒ 2b + b =
2
(2n1 − 4) × 90°
1
n1 2 ⇒ b =
51.
(a) = [Q n2 = 2n1] 2
(2 × 2n1 − 4) × 90° 3 \ a + c = 1 (1)
2n1
Now, a2, b2, c2 are in G.P.
\ n1 = 4 and n2 = 8. \ b4 = a2 c2
⇒ b2 = – a c (\ a < b < c)
52.
(d) The inverse of the sum of the series
1
⇒ = – a c
4
3 3 7 (n + 1) 2
+ + + ... is 2 . 1
4 36 144 n + 2n ⇒ c =  (2)
4a

53.
(b) a, H1, H2 .... Hn, b are in H.P. From Eqs. (1) and (2),
1
(n  1)ab 1 – a = 
\ H1 = 4a
a  nb ⇒ 4a – 4a2 = – 1
⇒ 4a2 – 4a – 1 = 0
(n  1)ab
and, Hn =  4  16  16
an  b ⇒ a =
24

(n  1)ab  4  32
\ H1 + a = a ⇒ a =
a  nb 24

Chapter_29.indd 28 6/5/2015 4:25:58 PM


29.29
Progressions

4 4 2 [Q Expenditure every month increases by 500 rupees]


⇒ a = 
8 8 ⇒ n2 + 7n – 240 = 0
⇒ n = 12.38
1 1
⇒ a = − . \ Rashid after 13 months will start borrowing money
2 2
from his friends.
(a) 12 – 22 + 32 – 42 + … 20012 – 20022 + 20032
55. 58.
(c) From the given information, sum of the first (x – 1)
= (12 + 32 + ... 20012 + 20032) – (22 + 42 + 62 + ... 20022)
natural numbers
= (12 + 32 + ... 20032) – 22 [12 + 22 + 32 + ... 10012]
= sum of the natural numbers from (x + 1) to 49

1001  (1001  1) (2002  1)  ( x  1) ( x) 49  50 ( x) ( x  1)


= (12 + 32 + ... 20032) – 4 
 So, = –
 6 2 2 2

= (12 + 32 + ... 20032) – 4  



1001  1002  2003  2x2 = 49 × 50 ⇒ x = 35.
 6 
n
We know that 59.
(b) ∑ ( x) = (1) + (2) + (3) + (4) + ... + (n)
= [12 + 32 + 52 … 20032] x =1

2 2 2 2 2 2 2 2 = (1) + (1 + 1) + (1 + 2) + (1 + 3) + ... + (n)



= [1 + 2 + 3 + ... 2003 ] – [2 + 4 + 6 ... 2002 ]
= (1) + (1)(1) + (1)(2) + (1)(3) + ... + (n)

2003  2004  2007 4  1001  1002  2003 =2+2×2+2×2×2+2×2×2

= 
6 6 × 2 + ... + (n)

2003  2004  4007 4  1001  1002  2003 = 2 + 4 + 8 + 16 + ... (n)



\ =  Now, this is a GP with common ratio = 2.
6 6
4  1001  1002  2003 On equating this with 1022, we get n = 9.

6 60.
(d) The integers are 4, 4, 4, 8, 10, 20 and x
2003  2004  4007 4  2  1001  1002  2003 Consider (A) x < 4
= –
6 6
50 + x
Mean = , Median 4, Mode = 4
2004 ?
= [2003 × 4007 – 4 × 1001 × 2003]
6 If these are in AP, mean 4.
= 334 [8026021 — 8020012]
50  x
= 334 × 6009 So, = 4 ⇒ x = – 22
7
= 2007006.
Consider (B) 4 < x < 8
56.
(c) As per question, let first term = a and common
difference = d 50  x 54 58
i.e., < Mean <
30 7 7 7
2400 = [2a + (30 − 1) d ] (1)
2 Mean = x, Mode = 4
40 as these are in AP, x = 6, Mean = 8
3600 = [2a + (40 − 1) d ] (2)
2
Consider (C) 8 < x
To solve Eqs. (1) and (2)
a = 51 and d = 2 50  x 58
Mean = 
Hence, the value of 8th instalment 7 7

= `[51 + (7) × 2] Median = 8, Mode = 4


= `65. As these are in AP, Mean = 12 i.e., x = 34
So, x can be – 22, 6 or 34
57.
(d) Let Rashid’s savings will last till ‘n’ months
The sum of these is 18, which is not there among
n the option. In exam, we would have to make a decision.
\ [2 × 2000 + (n – 1) 500] = 60000
2 the negative value of – 22 seems to have been ignored.

Chapter_29.indd 29 6/5/2015 4:25:59 PM


29.30 I Chapter 29

In this case, the mean, median and mode are all equal 63. (c) A1 has 1 element, A2 has 2 elements, A3 has 3
to 4. As 18 is not there among the options, we have to elements,…, A49has 49 elements.
select 40 (6 + 34). Number of elements in A1, A2, A3,…A49 are all
61.
(c) Weight of a solid spherical ball is proportional to the combined.
cube of its radius. The radius and weights of the 10 Therefore,
bails on the day 10 are tabulated below.
49 × 50
1 + 2 + 3 +  + 49 =
Ball put on day Radius (mm) Weight (gm) 2
1 29 89 = 49 × 25 = 1225.
2 28
88
Thus, A50 = (1226,1227,...,1275).
3 27 87 Thus, sum of elements in A50 ,
   50
(1226 + 1275) = 25 × 2501
9 21 8 2
10 20 1 = 62525.

The total weight of the 10 balls on day 10 is 64. (a) Let the value of certificates purchased in the first year
be `a.
810  1 230  1
1 + 8 + 82 + 89 = = The difference between the values of the certificates is
7 3 `300 (d = 300).
The weight of the 10 balls before they were put in the As it follows arithmetic progression, the total value of
pot = 10 g certificates after 20 years is given by

\ The weight of the gold ‘made’ by the saint (g). n


S n = [2a + (n − 1)d ]
2
230 230 − 71 20
 10 = . ⇒ 83000 = [2a + (20 − 1)300]
7 7 2
⇒ 83000 = 10(2a + 5700)
62. (b) As x, y and z are in harmonic progression,
⇒ 2a + 5700 = 8300.
1 1 1
, and are in arithmetic progession.
x y z On simplifying, we get
1 1 1 1 a = `1300
∴ − = −
z y y x
The value of the certificates purchased by him in the
y−z x− y 13th year is
⇒ = .
yz xy
Multiplying both the sides by xyz , we get a + (n − 1)d
x( y − z ) = z ( x − y ) = 1300 + (13 − 1) × 300
z( x − y) = ` 4900.
x= .
y−z
65. (c) If we consider the third term to be x
It implies II is true.
The 15th term will be (x + 12d)
And,
6th term will be(x + 3d)
1 1 1 1 11th term will be (x + 8d) and 13th term will be (x +
− = −
z y y x 10d)
2 1 1 We are given
⇒ = +
y z x 2x + 12d= 3x + 21dor x+ 9d = 0
2 z+x x + 9d will be the 12th term.
⇒ =
y xz
66. (b) The sum of the squares of the first n odd natural
y( x + z) numbers = Sum of the squares of the (2n − 1) natural
⇒x=
2z numbers − Sum of the squares of the first (n − 1) even
It implies I is true. natural numbers

Chapter_29.indd 30 6/5/2015 4:26:02 PM


29.31
Progressions I
∴ Sn =
( 2n − 1)( 2c ) ( 4n − 1) − 4  ( n − 1) n ( 2n − 1)  So,
  n
6  6  [2a + (n − 1)d ] = 155
n ( 2n − 1) ( 2n + 1) 2
= 10
3 ⇒ (2a + 9d ) = 155
2
Ass S n = 533n,
⇒ 2a + 9d = 31 (1)
n ( 2n − 1) ( 2n + 1) Also, given that b + br = 9
= 533n
3 Since b = d and r = a
⇒ 4n 2 = 1600 So, d + ad = 9 (2)
⇒ n = 20. Solving (1) and (2), we get
25 2
67. (b) [29(2)] a = 2, and d = 3,
2 3
We have
So, the AP can be 2, 5, 8, 11,…
n+3 25 79 83
Tn = . Or, , , ,........
n 2 6 6
Therefore, T4T5T6 T58T59T60 71. (d) Let us consider an AP of 6 terms, say, 1, 2, 3, 4, 5 and 6.
Then, 3n = 6, 2n = 4, n = 2
So, Now S3 − S 2 − S1
 7   8   9   10   61   62   63  = (1 + 2 + 3 + 4 + 5 + 6) – (1 + 2 + 3 + 4) – (1 + 2)
             
 4   5   6   7   58   59   60  = 21 – 10 – 3 = 8
61.62.63 In the above AP, n = 2, d = 1, a = 1
= = 1985.55.
4.5.6 From the options, we get
2 n2 d = 2 × 22×d = 8
68. (b)
that is equal to S3 − S 2 − S1 ,
As (a 2 + b 2 ), (b 2 + c 2 ) and (a 2 + c 2 ) are in G.P., Therefore, (d) is the correct answer.
(b 2 + c 2 ) 2 = (a 2 + b 2 )(a 2 + c 2 ) 72. (c) Given expression
⇒ b 4 + c 4 + 2b 2c 2 = a 4 + a 2b 2 + a 2c 2 + b 2c 2 1 1 1 1
= + + + ... +
4 4 2 2 4 2 2 2 2 1.3 3.5 5.7 19.21
⇒ b +c +b c =a +a b +a c
1 1 11 1 11 1
⇒ b 2 (b 2 + c 2 ) + c 4 = a 2 (b 2 + c 2 ) + a 4 = 1 −  +  −  +  − 
2 3 23 5 25 7 
⇒ (b 2 − a 2 )(b 2 + c 2 ) = a 4 − c 4
1 1 1 
a4 − c4 +.... +  − 
∴ b2 − a 2 = . 2  19 21 
b2 + c2
1 1  1 20 20 10
= 1 −  = × = = .
69. (d) As 2a, b and 2c are in A.P., 2  21  2 21 42 21

2b = 2a + 2c 73. (a) U n +1 = 2U n + 1(n = 0, 1, 2,....)
⇒b=a+c
Put n = 0, U1 = 1
(a + c)
2
−b ± − 4ac n = 1, U 2 = 3
x=
2a n = 2, U 3 = 7
−b ± ( a − c ) n = 3, U 4 = 15
=
2a n = 4, U 5 = 31
−(a + c) ± (a − c) c Seeing the pattern it is clear that U n = 2n − 1
= = − or, − 1.
2a a
Heence, U10 = (2)10 − 1 = 1023.
70. (d) Let the A.P. be a, a + d, a + 2d,……..
74. (c) If the girl counts the way as given in the question, then
And the G.P. be b, br, br2, br3,….. counting serial for the thumb will be 1, 8, 17, 25, …..
Given : sum of 10 terms of A.P. = 155 Hence, number 1992 will also fall on thumb. Hence,

Chapter_29.indd 31 6/5/2015 4:26:08 PM


29.32 Chapter 29

number 1994 will end on her middle finger. 76. (a) Since
75. (c) 1600 + 669 = 2269 = Not a perfect square
x, 17, 3 x − y 2 − 2 and 3 x + y 2 − 30 are in A.P.
1500 + 669 = 2169 = Not a perfect square
∴17 − x = 3 x + y 2 − 30 − 3 x + y 2 + 2
1540 + 669 = 2209 = Square of 47
1690 + 669 = 2359 = Not a perfect square ⇒ x + 2 y 2 = 45 (1)
Thus, by adding 669 to each of the alternatives, we
note that 1540 + 669 = 2209, and this is the only Also, 17 − x = 3 x − y 2 − 2 − 17
square number. ⇒ 4 x − y 2 = 36 (2)
So each side of the square contains 2209 = 47 balls . Solving equations (1) and (2), we get x = 13
∴  Each side of the triangle will contain 55 balls Now, x + 17 + 3 x − y 2 − 2 + 3x + y 2 − 30
Thus the triangle will contain = 7 x − 15 = 7(13) − 15 = 76
55(55 + 1) Out of the given options, 76 is only divisible by 2.
1 + 2 + 3 + .................. + 55 = = 1540 balls.
2

Chapter_29.indd 32 6/5/2015 4:26:10 PM


CHAPTER

Set Theory 30
introduction table may also form a set, their common property being that
they form a collection of objects lying on the table.
The concept of set is fundamental in all branches of
mathematics. Sets are the most basic tools of mathematics Illustration 1 Some other examples of sets are:
which are extensively used in developing the foundations of (i) The set of numbers 1, 3, 5, 7, 9, 14.
relations and functions, logic theory, sequences and series, (ii) The set of vowels in the alphabets of English.
geometry, probability theory, etc. In fact, these days most of
the concepts and results in mathematics are expressed in the (iii) The set of rivers in India.
set theoretic language. (iv) The set of all planets.
The modern theory of sets was developed by the (v) The set of points on a circle.
German mathematician Georg Cantor (1845–1918AD). (vi) The set of mathematics books in your library.
In this chapter, we will study some basic definitions (vii) The set of even positive integers (i.e., 2, 4, 6, 8, ...).
and operations involving sets. We will also discuss the
applications of sets. (viii) The set of multiples of 4 (i.e., 4, 8, 12, ...).
(ix) The set of factors of 12. (i.e., 1, 2, 3, 4, 6, 12).
SET (x) The set of integers less than zero (i.e., –1, –2, –3, ...).

We observe that in nature, varieties of objects occur in


groups. These groups are given different names such as, Notations
a collection of books, a bunch of keys, a herd of cattle, an Sets are usually denoted by capital letters A, B, C, etc., and
aggregate of points, etc., depending on the characteristic their elements by small letters a, b, c, etc.
of objects they represent. In literal sense, all these works Let A be any set of objects and let ‘a’ be a member of
have the same meaning. (i.e., a group or a collection). In A, then we write a ∈ A and read it as ‘a belongs to A’ or ‘a is
mathematical language, we call this collection of objects, an element of A’ or ‘a is a member of A’. If a is not an object
a set. From the above examples, it can be seen that each of A, then we write a ∉ A and read it as ‘a does not belong
collection has a well-defined property (characteristic) of to A’ or ‘a is not an element of A’.
its own.
Thus, a set is a well-defined collection of objects.
REPRESENTATION OF SETS
When we say well defined, we mean that the objects follow
a given rule or rules. With the help of this rule, we will be There are two ways of expressing a set. These are
able to say whether any given object belongs to this set or 1. Tabular form or roster form.
not. For example, if we say that we have a collection of
2. Set-builder form or rule method.
short students in a class, this collection is not a set as ‘short
students’, is not well defined. However, if we say that we
have a collection of students whose height is less than 5 Tabular Form or Roster Form
feet, then it represents a set.
In this method, we list all the members of the set separating
It is not necessary that a set may consist of same type them by means of commas and enclosing them in curly
of objects, For example, a book, a cup and a plate lying on a brackets {}.

Chapter_30.indd 1 6/5/2015 3:59:46 PM


30.2 Chapter 30

Illustration 2 Let A be the set consisting of the numbers 1, Illustration 6 Each of the following sets is an infinite set:
3, 4 and 5, then we write A = {1, 3, 4, 5} (i) the set of all natural numbers = {1, 2, 3, 4, ...).
Notes: (ii) the set of all prime numbers = {2, 3, 5, 7, ...).
(iii) the set of all points on a given line.
• The order of writing the elements of a set is immaterial.
(iv) the set of all lines in a given plane.
For example, {1, 3, 5}, {3, 1, 5}, {5, 3, 1} all denote the
(v) {x | x e R and 0 < x < 1}.
same set.
• An element of a set is not written more than once. Thus, EMPTY SET (OR NULL SET)
the set {1, 5, 1, 3, 4, 1, 4, 5} must be written as {1, 3, 4, 5}.
The set which contains no element is called the empty set or
the null set or void set.
Set Builder Form or Rule Method
The symbol for the empty set or the null set is f. Thus,
In this method, instead of listing all elements of a set, we f = {}, since there is no element in the empty set.
write the set by some special property or properties satisfied
The empty set is a finite set.
by all its elements and write it as
Since any object x which is not equal to itself does not
A = {x : P (x)} or, A = {x | x has the property P (x)}
exist, the set A = {x : x ≠ x} is the empty set f.
and read it as ‘A is the set of all elements x such that x has
A set which is not empty, i.e., which has at least one
the property P’. The symbol ‘:’ or ‘|’ stands for ‘such that’.
element is called a non-empty set or a non-void set.
Illustration 3 Let A be the set consisting of the elements 2, Illustration 7
3, 4, 5, 6, 7, 8, 9, 10. Then, the set A can be written as A =
{x : 2 ≤ x ≤ 10 and x e N} (i) The set of natural numbers less than 1 is an empty
set.
(ii) The set of odd numbers divisible by 2 is a null set.
FINITE AND INFINITE SETS
(iii) {x | x ∈ Z and x2 = 2} = f, because there is no integer
Finite Set whose square is 2.
A set having no element or a definite number of elements is (iv) {x | x ∈ R and x2 = –1} = f, because the square of a
called a finite set. Thus, in a finite set, either there is nothing real number is never negative.
to be counted or the number of elements can be counted, (v) {x | x ∈ N, 4 < x < 5} is the empty set.
one by one, with the counting process coming to an end.
(vi) {x | x ∈ Z, – 1 < x < 0} is the null set.
Illustration 4 Each of the following sets is a finite set: The empty set should not be confused with the set {0}.
(i) A = the set of prime numbers less than 10 It is the set containing one element, namely 0.
= {2, 3, 5, 7};
Singleton
(ii) B = the set of vowels in English alphabets
= {a, e, i, o, u}; A set containing only one element is called a singleton.
(iii) C = {x | x is divisor of 50}. Illustration 8
(i) The set {0} is a singleton since it has only one
Cardinal Number of a Finite Set element 0.
The number of distinct elements in a finite set S is called the (ii) The set of even prime numbers is the set {2} which
cardinal number of S and is denoted by n(S). is a singleton.
(iii) {x | x is an integer and –1 < x < 1} = {0} is a singleton.
Illustration 5 If A = {2, 4, 6, 8} then n(A) = 4
Infinite Set Equal Sets
A set having unlimited number of elements is called an Two sets A and B are said to be equal if they have the same
infinite set. Thus, in an infinite set, if the elements are elements and we write A = B. Thus, A = B if every element
counted one by one, the counting process never comes to of A is an element of B and every element of B is an element
an end. of A.

Chapter_30.indd 2 6/5/2015 3:59:46 PM


Set Theory 30.3

In symbols, A = B iff x ∈ A ⇒ x ∈ B and x ∈ B (iii) The set of real numbers is a subset of the set of
⇒ x ∈ A. To indicate that two sets A and B are not equal, we complex numbers. The set of rational numbers
will write A ≠ B. is a subset of the set of real numbers. The set of
integers is a subset of the set of rational numbers.
Illustration 9 Finally, the set of natural numbers is a subset of the
(i) If A = {2, 3, 4} and B = {x | 1 < x < 5, x ∈ N} then set of integers. Symbolically,
A = B. N ⊆ Z ⊆ Q ⊆ R ⊆ C.
(ii) If A = the set of letters in the word ‘WOLF’ and
B = the set of letters in the word ‘FOLLOW’ then Notes:
A = B as each = {W, O, L, F}, remembering that in
a set the repetition of elements is meaningless and • If we are to prove that A ⊆ B, then we should prove
order of elements is immaterial. that x ∈ A ⇒ x ∈ B. Symbolically,
A ⊆ B if and only if x ∈ A ⇒ x ∈ B.
Equivalent Sets
• If we are to prove that A B, then we should prove
Two finite sets A and B are said to be equivalent if they have that there exists at least one element x such that
the same number of elements, i.e., if we can find a one­-to- x ∈ A but x ∉ B. Symbolically, A B if and only if
one correspondence between the elements of the two sets. there exists x ∈ A such that x ∉ B.
The symbol ‘ ~ ’ is used to denote equivalence. Thus,
A ~ B is read as ‘A is equivalent to B’. Two finite sets A and Proper Subsets of a Set
B are equivalent if n(A) = n(B), i.e., if they have the same A set B is said to be a proper subset of the set A if every
cardinal number. element of set B is an element of A whereas every element
Equivalent sets have the same number of elements, of A is not an element of B.
not necessarily the same elements. The elements in two We write it as B ⊂ A and read it as ‘B is a proper subset
equivalent sets may or may not be the same. Thus, equal of A’.
sets are always equivalent but equivalent sets may or may Thus, B is a proper subset of A if every element of B is
not be equal. an element of A and there is at least one element in A which
is not in B.
Illustration 10
Illustration 12
(i) If A = {1, 2, 3} and B = {2, 4, 6) then A ~ B.
(ii) If A = {a, b, c, d} and B = {p, q, r, s} then A ~ B. (i) If A = {1, 2, 5} and B = {1, 2, 3, 4, 5}. Then A is a
proper subset of B.
(iii) If A = {3, 5, 7, 9} and B = {9, 7, 5, 3} then A ~ B.
(ii) The set N of all natural numbers is a proper subset of
Also, since A and B have same elements, \ A = B. the set Z of all integers because every natural number
is an integer, i.e., N ⊂ Z but every integer need not be
SubSet of a set a natural number, i.e., N ≠ Z.
If A and B are any two sets, then B is called a subset of A if
every element of B is also an element of A. Symbolically, Note:
we write it as B ⊆ A or A ⊇ B
(i) B ⊆ A is read as B is contained in A or B is a subset  If we are to prove that B ⊂ A, then we should prove that
B ⊆ A and there exists an element of A which is not in B.
of .
Symbolically, B ⊂ A if and only if B ⊆ A and there exists x
A ⊇ B is read as A contains B or A is super set of B.
(ii) ∈ A such that x ∉ B.

Illustration 11 Power Set


(i) The set A = {2, 4, 6} is a subset of B = {1, 2, 3, 4, Elements of a set can also be some sets. Such sets are called
5, 6}, since each number 2, 4 and 6 belonging to A, set of sets. For example, the set {f, {1}, {2}, {3, 4}} is a set
also belongs to B. whose elements are the sets f, {1}, {2}, {3, 4}.
(ii) The set A = {1, 3, 5} is not a subset of B = {1, 2, The set of all the subsets of a given set A is called the
3, 4} since 5 ∈ A but 5 ∉ B. power set of A and is denoted by P(A).

Chapter_30.indd 3 6/5/2015 3:59:46 PM


30.4 Chapter 30

Illustration 13 2. If A ⊂ B and A ≠ B, then A and B can be represented by


(i) If A = {a}, then P(A) = {f, A}. either of the diagrams [Fig. (b) and Fig. (c)].
(ii) If B = {2, 5}, then P(B) = {f, {2}, {5}, B}.
(iii) If S = {a, b, c}, then P(S) = {f, {a}, {b}, {c}, {a, b},
{a, c}, {b, c}, S}.

Notes:

• Every set is subset of itself.


(b)            (c)
 • Empty set is the subset of every set.
• If a set has n elements, then the number of its 3. If the sets A and B are not comparable, then neither of A
subsets is 2n. or B is a subset of the other. This fact can be represented
by either of the diagrams [Fig. (d) and Fig (e)].
Comparable Sets
If two sets A and B are such that either A ⊂ B or B ⊂ A, then
A and B are said to be comparable sets. If neither A ⊂ B nor
B ⊂ A, then A and B are said to be non-comparable sets.
Illustration 14
(i) If A = {1, 3, 5} and B = {1, 2, 3, 4, 5}, then A and
B are comparable sets because A ⊂ B.
(ii) If A = B, then A and B are comparable sets (d)             (e)

Universal Set 4. If A = {1, 2, 3, 4} and B = {5, 6, 7}, then A and B


are disjoint. These can be illustrated by Venn diagram
If in any discussion on set theory, all the given sets are given in Fig. (f).
subsets of a set U, then the set U is called the universal set.
Illustration 15 Let A = {2, 4, 6}, B = {1, 3, 5}, C = {3, 5, 7,
11}, D = {2, 4, 8, 16} and U = {1, 2, 3, 4, 5, 6, 7, 8, 10, 11,
16} be the given sets. Here the sets A, B, C, D are subsets of
the set U. Hence U can be taken as the universal set.

Venn Diagrams
In order to visualize and illustrate any property or theorem (f )
relating to universal sets, their subsets and certain operations on
Complement of a Set
sets, Venn, a British mathematician developed what are called
Venn diagrams. He represented a universal set by interior of a Let A ⊂ U (i.e., A is a proper subset of universal set U).
rectangle and other sets or subsets by interiors of circles. Evidently, U consists of all the elements of A together with
some elements which are not in A. Let us now constitute
Examples of Certain Relationships Between Sets by another set consisting of all the elements of U not in A.
Venn Diagrams Naturally, it will form another proper subset of U. We call
this subset the complement of the subset A in U and denote
1. If U be a set of letters of English alphabets and A, a set
it by A′ or by Ac i.e., Ac = {x : x ∈ U, x ∉ A}.
of vowels, then A ⊂ U. This relationship is illustrated
by Fig. (a). Thus, the complement of a given set is a set which
contains all those members of the universal set that do not
belong to the given set.
Illustration of A′ by Venn Diagram
Let A be a subset of the universal set U. The shaded area in
figure below represents the set A′ which consists of those
(a) elements of U which are not in A.

Chapter_30.indd 4 6/5/2015 3:59:47 PM


Set Theory 30.5

Illustration 17
Illustration 16
(i) Let A = {1, 2, 3, 4}, B = {2, 3, 6, 7, 9}, then,
(i) If the universal set is {a, b, c, d} and A ∪ B = {1, 2, 3, 4, 6, 7, 9}.
A = {a, b, d} then A′ = {c}. (ii) If A = O (set of odd natural numbers), B = E (set of
(ii) If the universal set U = {1, 2, 3, 4, 5, 6} even natural numbers), then,
and A = {2, 4, 6}, then A′ = {1, 3, 5}. A ∪ B = N.
(iii) If A is the set of rational numbers and B the set of
(iii) If U = N and A = O (the set of odd natural numbers),
irrational numbers, then A ∪ B = R.
then A′= E (the set of even natural numbers).
(iv) If A = {x : x2 = 4, x ∈ I} = {2, –2}, B = { y : y2 = 9,
(iv) If U = I, A = N,
y ∈ I} = {3, –3}, then,
then, A′ = {0, –1, –2, –3 ...}.
A ∪ B = {–3, –2, 2, 3}.
(v) If U = {1, 2, 3, 4}, A = {1, 2, 3, 4},
(v) If A = {x : 1 < x < 5, x ∈ N} = {2, 3, 4}, B = {y : 3
then A′ = f. < y < 7, y ∈ N} = {4, 5, 6}, then, A ∪ B = {2, 3, 4,
5, 6}.
Notes:

(i) Since A ⊂ A, we get A′ = f.


Notes:
(ii) (A′ )′ = A, i.e., complement of the complement of
a set is the set itself. From the definition of the union of two sets A and B, it is
clear that
Operations on Sets · x ∈ A ∪ B ⇔ x ∈ A or x ∈ B
(a)  Union of Sets · x Ï A ∪ B ⇔ x Ï A and x ∉ B
Let A and B be two given sets. Then the union of A and B · A ⊆ A ∪ B and B Í A ∪ B.
is the set of all those elements which belong to either A or (b) Intersection of Sets
B or both.
Let A and B be two given sets. Then the intersection of A
The union of A and B is denoted by A ∪ B and is read and B is the set of elements which belong to both A and
as A union B. The symbol ∪ stands for union. It is evident B. In other words, the intersection of A and B is the set of
that union is ‘either, or’ idea. Symbolically, common members of A and B.
A ∪ B = {x : either x ∈ A or x ∈ B}. The intersection of A and B is denoted by A ∩ B
and is read as A intersection B. The symbol ∩ stands for
intersection.
Note:
It is evident that intersection is an ‘and’ idea.
The union set contains all the elements of A and B, except Symbolically,
that the common elements of both A and B are exhibited A ∩ B = {x : x ∈ A and x ∈ B}.
only once.

Illustration of A ∪ B by Venn Diagram


Note:
Let A and B be any two sets contained in a universal
set U. Then A ∪ B is indicated by the shaded area From the definition of the intersection of two sets A and B,
in the figure below. it is clear that

Chapter_30.indd 5 6/9/2015 3:40:20 PM


30.6 Chapter 30

• x ∈ A ∩ B ⇔ x ∈ A and x ∈ B Illustration 19
• x ∉ A ∩ B ⇔ x ∉ A or x ∉ B (i) If A = {1, 2, 3, 4}, B = {2, 4, 7, 9}, then,
• A ∩ B ⊆ A and A ∩ B ⊆ B. A – B = {1, 3} and, B – A = {7, 9}.
Let A and B be any two sets contained in the universal Hence, A – B ≠ B – A.
set U. Then A ∩ B is indicated by the shaded area, as shown
in the figure below. (ii) If A = {12, 15, 17, 20, 21},
B = {12, 14, 16, 18, 21}
and, C = {15, 17, 18, 22}, then,
A – B = {15, 17, 20}
B – C = {12, 14, 16, 21}
C – A = {18, 22}
Illustration 18 B – A = {14, 16, 18}
(i) If A = {1, 2, 3, 6, 9, 18}, A – A = f.
and, B = {1, 2, 3, 4, 6, 8, 12, 24},
Illustration of A – B by Venn Diagrams
then, A ∩ B = {1, 2, 3, 6}.
(ii) If A is the set of odd natural numbers and B is the In the four cases shown by the diagrams below, A – B is
set of even natural numbers, then A ∩ B = f. given by shaded area.
[Intersection of two disjoint sets is empty set]
(iii) If A and B are sets of points on two distinct
concentric circles, then,
A ∩ B = f.
(iv) If A = {x : 1 < x < 6, x ∈ N } = {2, 3, 4, 5},
B = { y : 2 < y < 9, y ∈ N}
= {3, 4, 5, 6, 7, 8} 
then, A ∩ B = {3, 4, 5}.
Disjoint Sets
If A ∩ B = f, then A and B are said to be disjoint sets. For
example, let A = {2, 4, 6, 8} and B = {1, 3, 5, 7}. Then,
A and B are disjoint sets because there is no element
which is common to both A and B. The disjoint sets can
be represented by Venn diagram as shown in the figure below.

Applications of Sets
1. If a set S has only a finite number of elements, we
denote by n(S) the number of elements of S.
Illustration 20 If U = {1, 2, 3, 4, 5}, then n(U) = 5
2. For any two sets A and B, with finite number of
(c) Difference of Sets  elements, we have the following formula:
Let A and B be two given sets. The difference of sets A and n(A ∪ B) = n(A) + n(B) – n(A ∩ B).
B is the set of elements which are in A but not in B. It is 3. If A and B are disjoint sets, then
written as A – B and read as A difference B. Symbolically, n(A ∪ B) = n(A) + n(B).
A – B = {x : x ∈ A and x ∉ B}. Illustration 21 X and Y are two sets such that
Similarly, B – A = {x : x ∈ B and x ∉ A}. n(X) = 17, n(Y) = 23, n(X ∪ Y) = 38,
Caution: In general, A – B ≠ B ­– A. find n(X ∩ Y).

Chapter_30.indd 6 6/5/2015 3:59:47 PM


Set Theory 30.7

Solution: n(X) = 17, n(Y) = 23, n(X ∪ Y) = 38, Illustration 22 Suppose, A = {2, 4, 6} and B = {x, y}
n(X ∩ Y) = ? then,
Now, n (X ∪ Y) = n (X) + n (Y) – n (X ∩ Y) A × B = {(2, x}, (4, x), (6, x), (2, y), (4, y), (6, y)}
Then, 38 = 17 + 23 – n(X Ç Y) B × A = {(x, 2), (x, 4), (x, 6), (y, 2), (y, 4), (y, 6)}
⇒ n(X ∩ Y) = 17 + 23 – 38 = 2
Thus, we note that if A ≠ B, then A × B ≠ B × A.
Ordered Pair
Let A and B be two non-empty sets. If a ∈ A and b ∈ B, an Illustration 23 Let A = {1, 2, 3} and B = f. Then, A × B
element of the form (a, b) is called an ordered pair, where = f, as there will be no ordered pair belonging to A × B.
‘a’ is regarded as ‘the first element’ and ‘b’ as the second Thus, we note that A × B = f if A or B or both of A and B
element. It is evident from the definition that are empty sets
(i) (a, b) ≠ (b, a)
Illustration 24 Let n(A) represents the number of elements
(ii) (a, b) = (c, d) if and only if a = c and b = d. in set A. In Illustration 22, we can see that n(A) = 3, n(B)
Equality of two ordered pairs. Two ordered pairs = 2 and n(A × B) = 6. Thus, we note that n(A × B = n(A) ×
(a, b) and (c, d) are said to be equal if and only if a = c n (B)
and b = d. The ordered pairs (2, 4) and (2, 4) are equal In other works, if a set A has m elements and a set B
while the ordered pairs (2, 4) and (4, 2) are different. The has n elements, then A × B has mn elements. Further, it may
distinction between the set {2, 4} and the ordered pair be noted that n(A × B) = n(B × A). This implies that A × B
(2, 4) must be noted carefully. We have {2, 4} = {4, 2} and B × A are equivalent sets.
but (2, 4) ≠ (4, 2).
Illustration 25 If there are three sets A, B, C and a Î A, b
Cartesian product of sets
∈ B, c ∈ C, we form an ordered triplet (a, b, c). The set of
Let A and B be two non-empty sets. The cartesian product all ordered triplets (a, b, c) is called the cartesian product of
of A and B is denoted by A × B (read as ‘A cross B’) and is the sets A, B, C. That is,
defined as the set of all ordered pairs (a, b), where a e A and
b e B. Symbolically, A × B × C = {(a, b, c): a ∈ A, b ∈ B, c ∈ C}

A × B = {(a, b): a e A and b e B}

Practice Exercises

Difficulty Level-1
(Based on Memory)

1. 73% people take atleast one drink of tea, coffee, milk. opted Business. If there are 220 students, how many of
52% take tea, 38% take coffee, 10% take milk and 7% them opted for both Psychology and Business?
take all three. How many people take atleast two drinks? (a) 60 (b) 100
(a) 20% (b) 23% (c) 77 (d) 35
(c) 13% (d) 25% [Based on MAT, 2003]
2. There are 19 hockey players in a club. On a particular day 4. In a group of 15 women, 7 have nose studs, 8 have ear
14 were wearing the prescribed hockey shirts, while 11 rings and 3 have neither. How many of these have both
were wearing the prescribed hockey pants. None of them nose studs and ear rings?
was without hockey pant or hockey shirt. How many of (a) 0 (b) 2
them were in complete hockey uniform? (c) 3 (d) 7
(a) 8 (b) 6 [Based on MAT, 2002]
(c) 9 (d) 7 5.
In a science talent examination, 50% of the candidates fail
[Based on MAT, 2004] in Mathematics and 50% fail in Physics. If 20% fail in
3. All the students of a batch opted Psychology, Business, both these subjects, then the percentage who pass in both
or both. 73% of the students opted Psychology and 62% Mathematics and Physics is:

Chapter_30.indd 7 6/5/2015 3:59:48 PM


30.8 Chapter 30

(a) 0% (b) 20% flavoured, 90% of the rest are squashed. What percentage
(c) 25% (d) 50% of the bonbons is vanilla flavoured that are not squashed?
[Based on MAT, 1999] (a) 9% (b) 5%
6.
In a survey, it was found that 65% of the people watched (c) 1% (d) 10%
news on TV, 40% read in newspaper, 25% read newspaper [Based on MAT (May), 2010]
and watched TV. What percentage of people neither 14.
Out of 80 students in a class, 25 are studying Commerce,
watched TV nor read newspaper? 15 Mathematics and 13 Physics. 3 are studying
(a) 0% (b) 5% Commerce and Mathematics, 4 are studying Mathematics
(c) 10% (d) 20% and Physics and 2 are studying Commerce and Physics.
[Based on MAT, 2000] 1 student is studying all the three subjects together. How
7. In a class, 20 opted for Physics, 17 for Maths, 5 for both and many students are not studying any of the three subjects?
10 for other subjects. The class contains how many students? (a) 35 (b) 40
(a) 35 (b) 42 (c) 20 (d) 15
(c) 52 (d) 60 [Based on MAT (Feb), 2010]
[Based on MAT, 2000] 15.
A survey shows that 63% of the Britishers like cheese,
8.
In a community of 175 pesons, 40 read the Times, 50 whereas 76% like apples. If x % of the Britishers like both
read the Samachar and 100 do not read any. How many cheese and apples, then find the value of x.
persons read both the papers? (a) 40 ≤ x ≤ 60 (b) 35 ≤ x ≤ 65
(a) 10 (b) 15 (c) 39 ≤ x ≤ 63 (d) None of these
(c) 20 (d) 25 [Based on MAT (Feb), 2009]
[Based on MAT, 2000] 16.
In a coaching institute, 40 students are selected in banking
9. In a survey among 80 people, 50 people like arrange exam coaching, 30 students selected in staff selection
marriage and 70 people like love marriage. What is the exam coaching and 20 students are selected in both the
minimum and maximum number of people who like both examinations coaching. How many students are there in
the marriages, respectively? the institute?
(a) 40, 45 (b) 40, 50 (a) 45 (b) 55
(c) 30, 40 (d) Cannot be determined (c) 50 (d) 40
[Based on MAT (Dec), 2007]
10. Out of 450 students in a school, 193 students read Science
Today, 200 students read Junior Statesman, while 80 17.
In a class, 50 students play Cricket, 20 students play
students read neither. How many students read both the Football and 10 students play both Cricket and Football.
magazines? How many play at least one of these two games?
(a) 137 (b) 80 (a) 10 (b) 80
(c) 57 (d) 23 (c) 50 (d) 60
[Based on IITTM, Gwalior, 2003] [Based on MAT (Dec), 2007]
18.
Out of a total of 120 musicians in a club, 5% can play
11. In a class consisting of 100 students, 20 know English and all the three instruments—guitar, violin and flute. It so
20 do not know Hindi and 10 know neither English nor happens that the number of musicians who can play any
Hindi. The number of students knowing both Hindi and two and only two of the above instruments is 30. The
English is: number of musicians who can play the guitar alone is 40.
(a) 5 (b) 10 What is the total number of those who can play violin
(c) 15 (d) 20 alone or flute alone?
[Based on FMS (Delhi), 2003] (a) 30 (b) 44
12.
In an examination, 30% and 35% students, respectively, (c) 38 (d) 45
failed in History and Geography while 27% students failed [Based on MAT (Feb), 2006]
in both the subjects. If the umber of students passing the 19.
In an examination, 35% of the candidates failed in one
examination is 248, then find the total number of students subject and 42% failed in another sub­ject, while 15%
who appeared in the examination. failed in both the subjects. If 2500 candidates appeared
(a) 425 (b) 400 at the examination, how many candidates passed in either
(c) 380 (d) 725 subject but not in both?
[Based on MAT (Dec), 2010] (a) 325 (b) 1175
13.
In a chocolate store, there are vanilla and chocolate (c) 2125 (d) None of these
flavour bonbons only. 10% of the bonbons are chocolate [Based on MAT, 1999]

Chapter_30.indd 8 6/5/2015 3:59:48 PM


Set Theory 30.9

20.
In a science talent examination, 50% of the candidates fail 28. In a certain group of 36 people, only 18 are wearing
in Mathematics and 50% fail in Phys­ics. If 20% fail in hats and only 24 are wearing sweaters. If six people are
both these subjects then the percentage who pass in both wearing neither a hat nor a sweater, then how many people
Mathematics and Physics is: are wearing both a hat and a sweater?
(a) 0% (b) 20% (a) 30 (b) 22
(c) 25% (d) 50% (c) 12 (d) 8
[Based on MAT, 1999]
29. In a party, 70 guests were to be served tea or coffee after
21.
In a survey, it was found that 65% of the people watched dinner. There were 52 guests who preferred tea while 37
news on TV, 40% read in newspaper, 25% read newspaper preferred coffee. Each of the guests liked one or the other
and watched TV. What percent­ age of people neither beverage. How many guests liked both tea and coffee?
watched TV nor read newspaper? (a) 15 (b) 18
(a) 0% (b) 5% (c) 19 (d) 33
(c) 10% (d) 20%
30. In a class of 80 children, 35% children can play only
[Based on MAT, 2000]
cricket, 45% children can play only table tennis and the
22.
In a class, 20 opted for Physics, 17 for Maths, 5 for both remaining children can play both the games. In all, how
and 10 for other subjects. The class con­tains how many many children can play cricket?
students? (a) 55 (b) 44
(a) 35 (b) 42 (c) 36 (d) 28
(c) 52 (d) 60 31. A survey on a sample of 25 new cars being sold at a
[Based on MAT, 2000] local auto dealer was conducted to see which of the
23.
In a community of 75 persons, 40 read the Times, 50 three popular options-air-conditioning, radio and power
read the Samachar and 100 do not read any. How many windows-were already installed.
persons read both the papers? The survey found:
(a) 10 (b) 15 15 had air-conditioning.
(c) 20 (d) 25 2 had air-conditioning and power windows.
[Based on MAT, 2000] 12 had radios.
24.
In an examination, 52% of the candidates failed in English, 6 had air-conditioning and radios but no power windows.
42% in Mathematics and 17% in both. The number of 11 had power windows.
those who passed in both the subjects is: 4 had radio and power windows.
(a) 83% (b) 23% 3 had all three options.
(c) 64% (d) 55.5% What is the number of cars that had none of the options?
[Based on FMS, 2005]
(a) 4 (b) 3
25.
In a group of 52 persons, 16 drink tea but not coffee and
(c) 1 (d) 2
33 drink tea. How many drink coffee but not tea?
[Based on 2012]
(a) 3 (b) 7
32. The shaded region in the following Venn Diagram
(c) 17 (d) 19 represents:
[Based on FMS, 2006]
R
26.
How many subsets of {1, 2, 3, ..., 11} contain at least one =
5

12
=1

even integer?
AC

4 6 2
(a) 1900 (b) 1964 3
2 1
(c) 1984 (d) 2048
[Based on IIFT, 2010] 5

27. In a class of 50 students, 23 speak English, 15 speak Hindi PW = 11


and 18 speak Punjabi. 3 speak only English and Hindi, 6
speak only Hindi and Punjabi and 6 speak only English (a) A ∪ (B ∪ C)
and Punjabi. If 9 can speak only English, then how many (b) A∪ (B ∩ C)
students speak all the three languages? (c) A ∩ (B ∪ C)
(a) 1 (b) 2 (d) None of these
(c) 3 (d) 5 [Based on 2012]

Chapter_30.indd 9 6/5/2015 3:59:48 PM


30.10 Chapter 30

Difficulty Level-2
(Based on Memory)

1. 55% people like to travel by bus, 37% people like to travel of employees who are both engineers and MBAs twice
by auto and 25% people like to travel by taxi. Also, 30% that of the employees who are only MBAs. How many
people like to travel by at least two of bus, taxi and auto, employees are neither engineer B. Tech. nor MBAs?
while 6% people like to travel by all the three, then how (a) 24 (b) 38
many people like to travel by at least one of the three (c) 36 (d) Cannot be determined
vehicles?
(a) 94% (b) 93% 6. One Hundred Twenty-five (125) aliens descended on a set
of film as Extra Terrestrial Beings. 40 had two noses, 30
(c) 87% (d) 81%
had three legs, 20 had four ears, 10 had two noses and
three legs, 12 had three legs and four ears, 5 had two noses
2. In a class of 100 students, there are 60 students who
and four ears and 3 had all the three unusual features. How
study Mathematics, 55 students who study Physics and
many were there without any of these unusual features?
45 students who study Chemistry. The number of students
who study both Mathematics and Physics is 25, the number (a) 5 (b) 35
of students who study both Physics and Chemistry is 18 (c) 80 (d) None of these
and the number of students who study both Mathematics [Based on FMS (Delhi), 2004]
and Chemistry is 23. If certain number of students did not 7. Out of a total 85 children playing badminton or table
study either Mathematics or Physics or Chemistry, then tennis or both, total number of girls in the group is 70%
find the minimum possible number of students who study of the total number of boys in the group. The number of
only one subject i.e., either Mathematics or Physics or boys playing only badminton is 50% of the number of
Chemistry. boys and the total number of boys playing badminton is
(a) 18 (b) 35 60% of the total number of boys. The number of children
(c) 28 (d) 40 playing only table tennis is 40% of the total number of
children and a total of 12 children play badminton and
3. A is the set of first ten consecutive natural numbers. Find table tennis both. What is the number of girls playing only
the number of ways in which a subset B can be formed out badminton?
of set A, such that the sum of all the elements in B is odd. (a) 17 (b) 14
(a) 352 (b) 507 (c) 16 (d) Data inadequate
(c) 320 (d) 512
8. How many different subsets of the set {10, 14, 17, 24} are
4.
A survey on a sample of 25 new cars being sold at a there that contain an odd number of elements?
local auto dealer was conducted to see which of the three (a) 8 (b) 6
popular options — air conditioning, radio and power (c) 3 (d) 10
windows — were already installed. The survey found:
15 had air conditioning. 9. In a survey of brand preference for toothpastes, 82% of
the population (number of people covered for the survey)
2 had air conditioning and power windows but no radios.
liked at least one of the brands : I, II and III. 40% of those
12 had radio. asked liked brand I, 25% liked brand II and 35% liked
6 had air conditioning and radio but no power windows. brand III. If 5% of those asked, showed liking for all the
11 had radio. three brands, then what percentage of those asked liked
more than one of the three brands?
4 had radio and power windows.
3 had all three options. (a) 13 (b) 10
(c) 8 (d) 5
What is the number of cars that had none of the options?
[Based on SCMHRD Ent. Exam., 2003]
(a) 4 (b) 3
(c) 1 (d) 2 Directions (Q. 10 to 12): Use the following information.
[Based on CAT, 2004] Eighty five children went to amusement park where they
5.
In a group of 80 employees, the number of employees could ride on the Merry-go-round, Roller Coaster and Ferris
who are engineers is twice that of the employees who are Wheel. It was known that 20 of them took all three rides, and 55
MBAs. The number of employees who are not engineers is of them took at least two of the three rides. Each ride costs `1, and
32 and that of those who are not MBAs is 56. The number the total receipts of the amusement park were `145.

Chapter_30.indd 10 6/5/2015 3:59:48 PM


Set Theory 30.11

10. How many children took exactly one ride? 15.


The number of people (in lakhs) who read only one
(a) 5 (b) 10 newspaper is:
(c) 15 (d) 20 (a) 4.7 (b) 11.9
(c) 17.4 (d) 23.4
11. How many children did not try any of the rides? [Based on SNAP, 2009]
(a) 5 (b) 10
(c) 15 (d) 20 16.
A dinner party is to be fixed for a group of 100 persons. In
this party, 50 persons do not prefer fish, 60 prefer chicken
12. Out of 100 families in the neighborhood, 45 own radios, and 10 do not prefer either chicken or fish. The number of
75 have TVs, 25 have VCRs. Only 10 families have all persons who prefer both fish and chicken is:
three and each VCR owner also has a TV. If 25 families (a) 20 (b) 30
have radio only, how many have only TV? (c) 40 (d) 10
(a) 30 (b) 35 [Based on FMS, 2005]
(c) 40 (d) 45
17.
Let X = {a, b, c} and Y = {l, m}.
13. Consider the set S = {2, 3, 4, …, 2n + 1}, where n is a Consider the following four subsets of X × Y.
positive integer larger than 2007. Define X as the average F1 = {(a, l), (a, m), (b, l), (c, m)}
of the odd integers in S and Y as the average of the even
integers in S. What is the value of X – Y? F2 = {(a, l), (b, l), (c, l)}

1 n +1 F3 = {(a, l), (b, m), (c, m)}


(a) n (b)
2 2n F4 = {(a, l), (b, m)}
(c) 2008 (d) 1 Which one, amongst the choices given below, is a
[Based on CAT, 2007] representation of functions from X to Y?
Directions (Q. 14-15): Based on the information given below: (a) F1, F2 and F3 (b) F2 and F3
(c) F2, F3 and F4 (d) F3 and F4
The Venn diagram given below shows the estimated [Based on XAT, 2008]
readership of 3 daily newspapers (X, Y and Z) in a city. The total
18.
There are 240 second year students in a B-School.
readership and advertising cost for each of these papers is as below
The Finance area offers 3 electives in the second year.
These are Financial Derivatives, Behavioural Finance,
Readership Advertising cost and Security Analysis. Four students have taken all the
Newspaper
(lakhs) ( ` per sq cm) three electives, and 48 students have taken Financial
X 8.7 6000 Derivatives. There are twice as many students who
Y 9.1 6500 study Financial Derivatives and Security Analysis
but not Behavioural Finance, as those who study both
Z 5.6 5000
Financial Derivatives and Behavioural Finance but not
The total population of the city is estimated to be 14 Security Analysis, and 4 times as many who study all
million. The common readership (in lakhs) is indicated in the three. 124 students study Security Analysis. There
the given Venn diagram are 59 students who could not muster courage to take
up any of these subjects. The group of students who
study both Financial Derivatives and Security Analysis
but not Behavioural Finance, is exactly the same as the
group made up of students who study both Behavioural
Finance and Security Analysis. How many students study
Behavioural Finance only?
(a) 29 (b) 30
(c) 32 (d) 35

[Based on XAT, 2011]

14.
The number of people (in lakhs) who read at least one 19. How many different subsets of the set {10, 14, 17, 24} are
newspaper is: there that contain an odd number of elements?
(a) 4.7 (b) 11.9 (a) 8 (b) 6
(c) 17.4 (d) 23.4 (c) 3 (d) 10
[Based on SNAP, 2009] [Based on NMAT, 2005]

Chapter_30.indd 11 6/5/2015 3:59:48 PM


30.12 Chapter 30

20. Which of the following statements is false for the sets A, 25. If 5% of people watched DD and CNN, 10% watched DD
B and C, where: and BBC, then what percentage of people watched BBC
A = {x | x is letter of the word ‘BOWL’} and CNN only?
B = {x | x is a letter of the work ‘ELBOW’} (a) 2% (b) 5%
C = {x | x is a letter of the word ‘BELLOW’} (c) 8.5% (d) Cannot be determined
(a) A ⊂ B (b) B ⊃ C
26. Referring to the previous question, what percentage of
(c) B = C (d) B is a proper subset of C
people watched all the three channels?
Directions (Q. 21 to 24): Answer the questions based on the (a) 3.5% (b) 0%
following information.
(c) 8.5% (d) Cannot be determined
A and B are two sets (e.g., A = mothers, B = women). The
elements that could belong to both the sets (e.g., women who 27. What is the maximum percentage of people who can
are mothers) is given by the set C = A . B. The elements which watch all the three channels?
could belong to either A or B, or both is indicated by the set D = (a) 12.5% (b) 8.5%
A ∪ B. A set that does not contain any element is known as a null
(c) 15% (d) Data insufficient
set, represented by f (for example, if none of the women in the set
B is a mother, then C = A. B is a null set, or C = f) Directions (Q. 28 to 31): Answer the questions based on the
Let ‘V’ signify the set of all vertebrates; ‘M’ the set of all following information.
manmals; ‘D’ dogs, ‘F’ fish; ‘A’ alsatian and ‘P’, a dog named Pluto.
In a locality, there are five small cities: A, B, C, D and E. The
21. If P . A = f and P ∪ A = D, then which of the following is distances of these cities from each other are as follows
true?
AB = 2 Km, AC = 2 Km, AD > 2 Km, AE > 3 Km, BC = 2
(a) Pluto and alsatians are dogs
Km;  BD = 4 Km, BE = 3 Km, CD = 2 Km, CE = 3 Km, DE > 3 Km.
(b) Pluto is an alsatian
(c) Pluto is not an alsatian 28. If a ration shop is to be set up within 3 Km of each city,
how many ration shops will be required?
(d) D is null set
(a) 1 (b) 2
22. If Z = (P. D) ∪ M, then (c) 3 (d) 4
(a) The elements of Z consist of Pluto, the dog or any
29. If a ration shop is to be set up within 3 Km of each city,
other mammal
how many ration shops will be required?
(b) Z implies any dog or mammal
(a) 2 (b) 3
(c) Z implies Pluto or any dog that is a mammal (c) 4 (d) 5
(d) Z is a null set
30. In a locality, two-thirds of the people have cable TV, one-
23. If Y = F(D.V), is not a null set, it implies that fifth have VCR, and one tenth have both. What is the
(a) All fish are vertebrates fraction of people having either cable TV or VCR?
(b) All dogs are vertebrates (a) 19/30 (b) 2/3
(c) Some fish are dogs (c) 17/30 (d) 23/30
(d) None of the above 31. In an examination, 35% of the candidates failed in one
24. Given that X = M.D is such that X = D, which of the subject and 42% failed in another subject. While 15%
following is true? failed in both the subjects. If 2,500 candidates appeared
at the examination, how many students passed in either
(a) All dogs are mammals subject but not in both?
(b) Some dogs are mammals (a) 325 (b) 1,175
(c) X = f (c) 2,125 (d) None of these
(d) All mammals are dogs
Directions (Q. 32 to 34): Answer the questions based on the
Directions (Q. 25 to 27): Answer the questions based on the following information.
following information. Ghosh Babu is staying at Ghosh Housing Society, Aghosh
A survey of 200 people in a community who watched at least colony, Dighospur, Kolkata. In Ghosh Housing Society, 6 persons
one of the three channels—BBC, CNN and DD showed that 80% of read daily the Ganashakti and 4 persons read the Anand Bazar
the people watched DD, 22% watched BBC, and 15% watched CNN. Patrika. In his colony, there is no person who reads both. Total

Chapter_30.indd 12 6/5/2015 3:59:49 PM


Set Theory 30.13

number of persons who read these two newspapers in Aghosh colony 36. Out of a total of 120 musicians in a club, 5% can play all the
and Dighospur is 52 and 200 respectively. Number of persons who three instruments-guitar, violin and flute. It so happens that
read Ganashakti in Aghosh colony and Dighospur is 33 and 121 the number of musicians who can play any two and only two
respectively, while the persons who read the Anand Bazar Patrika of the above instruments is 30. The number of musicians
in Aghosh colony and Dighospur are 32 and 117 respectively. who can play the guitar alone is 40. What is the total number
of those who can play violin alone or flute alone?
32. The number of persons in Aghosh colony who read only
(a) 30 (b) 44
one newspaper is:
(c) 38 (d) 45
(a) 29 (b) 19
(c) 39 (d) 20 37. While preparing the progress reports of the students, the
class teacher found that 70% of the students passed in
33. The number of persons in Aghosh colony who read both Hindi, 80% passed in English and only 65% passed in
the newspapers is: both the subjects. Find out the percentage of students who
(a) 13 (b) 9 failed in both the subjects.
(c) 38 (d) 14 (a) 15% (b) 20%
34. Number of persons in Dighospur who read only (c) 30% (d) 35%
Ganashakti is: [Based on CAT, 2012]
(a) 121 (b) 83 38. S is a set given by S = {1, 2, 3,…, 4n}, where n is a
(c) 79 (d) 127 natural number. S is partitioned into n disjoint subsets
35. There are 50 students admitted to a nursery class. Some A1 , A2 , A3 ,...., An each containing four elements. It is
students can speak only English and some can speak only given that in each of these subsets there is one element,
Hindi. Ten students can speak both English and Hindi. which is the arithmetic mean of the other three elements
If the number of students who can speak English is 21, of the subsets, which of the following statements is
then how many students can speak Hindi, how many then true?
can speak only Hindi and how many can speak only (a) n≠ 1 and n≠ 2
English? (b) n≠ 1 but can be equal to 2
(a) 39, 29, 11, respectively (c) n≠ 2 but can be equal to 1
(b) 28 18 and 22, respectively (d) It is possible to satisfy the requirement for n = 1 as
(c) 37, 27 and 13, respectively well as for n = 2
(d) 21, 11 and 29, respectively [Based on CAT, 2012]

Answer Keys
Difficulty Level-1

1. (a) 2. (b) 3. (c) 4. (c) 5. (b) 6. (d) 7. (b) 8. (b) 9. (b) 10. (d ) 11. (b) 12. (b) 13. (a)
14. (a) 15. (c) 16. (c) 17. (d) 18. (b) 19. (b) 20. (b) 21. (d) 22. (b) 23. (b) 24. (b) 25. (d) 26. (c)
27. (d) 28. (c) 29. (c) 30. (b ) 31. (d) 32. (c)

Difficulty Level-2

1. (d) 2. (c) 3. (d) 4. (d) 5. (a) 6. (d) 7. (b) 8. (a) 9. (b) 10. (c) 11. (c) 12. (c) 13. (d)
14. (c ) 15. (b) 16. (a) 17. (c) 18. (a) 19. (a) 20. (d) 21. (c) 22. (a) 23. (c) 24. (a ) 25. (a) 26. (d)
27. (c) 28. (a) 29. (a) 30. (b) 31. (b ) 32. (c) 33. (a) 34. (b) 35. (a ) 36. (b) 37. (a ) 38. (b)

Chapter_30.indd 13 6/5/2015 3:59:49 PM


30.14 Chapter 30

Explanatory Answers

Difficulty Level-1

1.
(a) Let T denotes the number of people taking tea, C 6.
(d) 80% of the people either watched TV or read
denotes the number of people taking coffee and M newspaper
denotes the number of people taking milk. We know
that
n (T ∪ C ∪ M) = n (T) + n (C) + n (M) – n (T ∩ C)
– n (C ∩ M) – n (T ∩ M)
+ n (T ∩ C ∩ M)
Here, there were 100 people in all, then

73 = 52 + 38 + 10 – n (T ∩ C)
– n (C ∩ M) – n (T ∩ M) + 7 \ 20% neither watched TV nor read newspaper.
⇒ n (T ∩ C) + m (C ∩ M) + n (T ∩ M) = 34
7.
(b)
\ n (T ∩ C) + n (C ∩ M) + n (T ∩ M)
– 2n (T ∩ C ∩ M) = 34 – 2 × 7 = 20.
2.
(b) n (P ∪ S) = n (P) + n (S) – n (P ∩ S)
⇒ 19 = 11 + 14 – n (P ∩ S)
⇒ n (P ∩ S) = 6.

3.
(c)
\ Total number of students in the class
= 15 + 5 + 12 + 10 = 42.
8.
(b)


\ 77 students out of 220 opted Psychology and
Business both.
4.
(c)

\ (40 – x) + x + (50 – x) = 75
⇒ x = 15.

9.
(b) For the minium value of x:people who like only
5.
(b) \ Number of students passed in both the subjects arrange marriage and only love marriage must be
greater


= 100 – (30 + 20 + 30) = 20.

Chapter_30.indd 14 6/5/2015 3:59:50 PM


Set Theory 30.15

\ x = (70 + 50) – 80 = 40 Total students studying at least one subject


For the maximum value of x: (50 – x) and (70 – x) = 21 + 1 + 2 + 1 + 9 + 3 + 8 = 45
must not be negative, therefore max. possible value of
x is 50. Students who are not studying any of the three subjects

10.
(d) n (SR ∪ JS) = n (ST) + n (JS) – n (ST ∩ JS) = 80 – 45 = 35.
⇒ (450 – 80) = 193 + 200 – n (ST ∩ JS) 15.
(c) Minimum possible value of
⇒ n (ST ∩ JS) = 393 – 370 = 23.
x = 63 + 76 – 100 = 39
11.
(b)
Maximum possible value of x = 63
So, value of x lies between 39 and 63.
i.e., 39 ≤ x ≤ 63.

16.
(c)


12.
(b)



\ Total number of students = 20 + 20 + 10 = 50.
Percentage of students failed = 38
\ Percentage of students passed = 62 17.
(d) Number of students who play Cricket = 50
Let total number of students be x.
Number of students who play Football = 20
62% of x = 248
Number of students who play both games =10
100
\ x = 248 × = 400. Number of students who play only Cricket
62
13.
(a) Let there are 100 bonbons = 50 – 10 = 40

10 bonbons are chocolate flavoured Number of students who play only Football
90 bonbons are vanilla flavoured = 20 – 10 = 10
81 bonbons are squashed \ Number of students who play at least one game

So, 9 bonbons of vanilla flavoured are not squashed = 40 +10 + 10 = 60.
\ Required percentage = 9% 18.
(b)

14.
(a)


\ Required number of musicians
= 120 – (40 + 6 + 30)
= 44.

Chapter_30.indd 15 6/5/2015 3:59:50 PM


30.16 Chapter 30

19.
(b) The percentage of candidates who passed in both \ Total number of students in the class
the subjects = 100 – (35 + 42 – 15) = 38%
= 15 + 5 + 12 + 10 = 42.
The percentage of candidates who passed in one
subject = (100 – 35) = 65% 23.
(b) Let x persons read both the papers
The percentage of candidates who passed in Now,
another subject = (100 – 42) = 58%
Now, the percentage of candidates who passed in
one subject only = (65 – 38) = 27%
The percentage of candidates who passed in
another subject only (58 – 38) = 20%
\ Total percentage of candidates who passed in
either subject but not in both
= (27 + 20) = 47%
\ total no. of candidates who passed in either subject
2500 × 47 \ (40 – x) + x + (50 – x) = 75
but not in both = = 1175
100 ⇒ x = 15.
20.
(b) 24.
(b)



\ Number of students passed in both the sub­jects.
= 100 – (30 + 20 + 30) = 20. n(A ∪ B) = n(A) + n(B) – (A ∩ B)
= 52 + 42 – 17
21.
(d)
= 77
\ Pass candidates = 100 – 77 = 23%

25.
(d)

(40 + 25 + 25) = 80% of the people wither watched 52 – 33 = 19


TV or read newspaper
\ n(A ∪ B) = n(A) + n(B) – n(A ∩ B)
\ 20% neither watched TV nor read newspaper.
⇒ 52 = 33 + n(B) – n(A ∩ B)
22.
(b)
⇒ n(B) – n(A ∩ B) = 52 – 33 = 19.

26.
(c) Given set = {1, 2, 3, ..., 11}

The Total number of subsets = 211

The number of subset which contain no even


number = 26

Chapter_30.indd 16 6/5/2015 3:59:51 PM


Set Theory 30.17

The number of subsets which contain at least one 29.


(c)
even number
= 211 – 26 = 2048 – 64
= 1984.
27.
(d) Given b + c + g + d = 23 (1)

19 Guests like Tea and Coffee both.
30.
(b)

a + b + g + f = 15 (2)

e + f + g + d = 18 (3)
31. (d) Total number of cars = 25
From all the four equations

b = 3, f = 6, d = 6, c = 9 and g = 5. A B

28.
(c) Number of people wearing a hat or a sweater or both
= 36 – 6 = 30

Total of cars in figures circles = 23


∴  2 cars had none of the options.
32. (c) Let the regions among A, B and C be distributed by a,
b, c, d, e, f as in the Venn diagram shown below:

A a b c B
\ Number of people wearing both the hat and the
d e
sweater = 12
f
OR, C

n (H) + n (S) – n (H ∩ S) = n (H È S)


Form the figure, B ∩ C = b + c + d + e + f and A = a +
⇒ 18 + 24 – n (H ∩ S) = 30 b+d
A ∩ (B ∪ C) = b + d, which is the shaded region in the
⇒ n (H ∩ S) = 12. given figure.

Chapter_30.indd 17 6/5/2015 3:59:51 PM


30.18 Chapter 30

Difficulty Level-2

1.
(d) Let A, B and C denote the sets of people preferring Subset B must contain odd number and any number
bus, auto and taxi, respectively. of even numbers
We know that Required number of ways
n (A ∪ B ∪ C) = n (A) + n (B) + n (C) = [5C1 + 5C3 + 5C5] × [5C0 + 5C1 + 5C2 + 5C3
– n (A ∩ B) – n (B ∩ C) – n (A ∩ C) + 5C4 + 5C5]
+ n (A ∩ B ∩ C) = 16 × 32 = 512.
Hence, if there were 100 people in all, then
4.
(d) Let, A = cars having air-conditioning
n (A ∪ B ∪ C) = 55 + 37 + 25 – (30 + 6 + 6) + 6
= 81 B = cars having radio
i.e., there are 81 people who prefer atleast one of three C = cars having power windows
vehicles. From the given information
2.
(c) Let the number of students who study all the three n (A) = 15
subjects be ‘x’. n (A ∩ B′ ∩ C) = 2
n (B) = 12
n (A ∩ B ∩ C′) = 6
n (C) = 11
n (B ∩ C) = 4
n (A ∩ B ∩ C) = 3
Also, n (A ∩ B′ ∩ C) = 2
⇒ n (A ∩ C) – n (A ∩ B ∩ C) = 2
Number of students who study both Mathematics ⇒ n (A ∩ C) = 2 + 3 = 5
and Physics but not Chemistry = 25 – x. And, n (A ∩ B ∩ C′ ) = 6
Number of students who study both Mathematics ⇒ n (A ∩ B) – n (A Ç B ∩ C) = 6
and Chemistry but not Physics = 23 – x. ⇒ n (A ∩ B) = 9
Number of students study both Physics and Now, n (A ∪ B ∪ C)
Chemistry but not Mathematics = 18 – x. = n (A) + n (B) + n (C) – n (A ∩ B) – n (B ∩ C)
We can represent the situation by the above Venn – n (A ∩ C) + n (A ∩ B ∩ C)
diagram.
⇒ n (A ∪ B ∪ C)
Taking the union of all sets, we get the number of
students who study at least one subject, as (94 + x). = 15 + 12 + 11 – 9 – 4 – 5 + 3 = 23
Obviously, (94 + x) < 1000. \ No. of cars having none of the options
As it is mentioned, that certain number of students = 25 – 23 = 2.
did not study any of the subjects, we can conclude that 5.
(a) Let a be the number of engineers only c be the number
0 ≤ x ≤ 5. of MBAs only
So, minimum value of x under the given condition
is 0.
\ Minimum possible number of students who study
only one subject = 12 + 12 + 4 = 28.

3.
(d) Since there are an equal number of odd and even b be the number of employees who are both engineers
numbers in the set A, there will be an equal number and MBAs and d be the number of employees who are
of subsets of A which add up to even or odd numbers. neither engineer nor MBA
Total subsets (including null set) = 210
\ a + b + c + d = 80 (1)
Half of this = 512
(a + b) = 2(b + c)
Alternative Solution
⇒ (a – b) = 2c (2)
A = {1, 2, 3, .......10}, which has 5 even and 5 odd
numbers. c + d = 32 (3)

Chapter_30.indd 18 6/5/2015 3:59:51 PM


Set Theory 30.19

a + d = 56 (4) Total children = 85


b = 2c (5) Boys = x
70 x 7
From Eq. (2) and (5), we get Girls = = x
100 10
a = 2b (6)
7
From Eq. (6), we get b = 16 Boys : Girls = x : x = 10:7
10
\ a = 32 (from eq. 6) 10
\ Boys = × 85 = 50, Girls = 35
and, c = 8 (from eq. 5) 17
and, d = 24 50
Boys (playing only Badminton) = × 50 = 25
Hence 24 employees are neither engineers nor MBAs. 100

6.
(d) Let x be the number of aliens who were without any of 60
Boys (playing Badminton) = × 50 = 30
the given three unusual features 100
\ Boys (playing B and TT) = 30 – 25 = 5
Children (playing B and TT) = 12
\ Girls (playing B and TT) = 12 – 5 = 7
40
Children (playing only TT) = × 85 = 34
100
Boys (only TT playing) = 50 – 30 = 20
Girls (playing only TT) = 34 – 20 = 14
Girls (playing only B) = 35 – 7 – 14 = 14.
8. (a) Required subsets will contain 1 element each or 3
elements
Number of subsets containing 1 element = 4
\ From the figure, we have
Q {10}, {14}, {17}, {24}
a + b + c + d + e + f + g + x = 125 (1) Number of subsets containing 3 elements
Also, = Number of groups containing 3 nos. out of four
= 4C3 = 4
a + d + g + e = 40
\ Total required number of subsets = 4 + 4 = 8.
c + d + f + g = 30 9.
(b) Given,
b + e + f + g = 20 a + b + c + d + e + f = 77 (1)
a + b + c = 35 (2)
 d + g = 20 Given d + b + e = 20 (3)
f + c + e = 30 (4)
 g + f = 12

 g + e = 5

 g = 3

⇒ g = 3, d = 7, e = 2, f = 9, b = 6, c = 11, a = 28

\ (1) ⇒ x = 59.

7.
(b)

Adding (2) and (4), we get


a + b + c + (d + e + f) + b + c + e = 82 (5)
⇒ b + c + e = 5
⇒ b + c + e + 5 = 10.

Chapter_30.indd 19 6/5/2015 3:59:52 PM


30.20 I Chapter 30

10.
(c) 14.
(c) No. of people who read atleast one newspaper is = no.
11.
(c) Children who took at least two of the three rides = (x of people who read one newspaper + no. of people
+ y + z + 20) = 55 who read two newspapers + no. of people who read
Hence, children who took only two rides (x + y + z) = three newspapers.
55 – 20 = 35
15.
(b) No. of people who read only one newspaper
Receipt from those who took all the three rides
= 20 × 3 = `60 = x + y + z
M R
= 4.7 + 4.6 + 2.6 = 11.9.

16.
(a) Let total persons be 100
10 do not prefer either fish or chicken.
[x = persons who like both]
\ 90 = 50 + 60 – x
⇒ x = 20.

F 17.
(c) X = {a, b, c} and Y = {l, m}
Receipt from those who took only two rides
F1 = {(a, l), (a, m), (b, l), (c, m)}
= 35 × 2 = `70
Total money received from both = `130. Since F2 = {(a, l), (b, l), (c, l)}
total collection was `145 therefore, balance amount F3 = {(a, l), (b, m), (c, m)}
of `15 was collected from those who took the single
ride only = 15 F4 = {(a, l), (b, m)}
\ Number of children who took one or more rides = From given statement, it is clear that F2, F3, and F4
(55 + 15) = 70 is a representation of functions from X to Y.
Hence, number of those who did not try a single ride
= (85 – 70) = 15. 18.
(a) Let X be the number of students, who have taken
12.
(c) Number of families having all the three = 10 Financial Derivates and Behavioral Finance, but not
Security Analysis
Number of families having VCR = 25

FD BF
(48)

SA
It is given that each VCR owner also has TV. It
means that number of families who own VCR only
is 0 and number of families who own radio and VCR 2x = 4 × 4
only is also zero.
8
\ Numbers of families who own TV only X = = 16 and, 2x – 4 = 12
2x
= 75 – (10 + 10 + 15) = 40.
13.
(d) S = {2, 3, 4, … 2n + 1); S = 2n f + 8 + 4 + 16 = 48
3 + 5 + ... + (2n + 1) and, 5 + 16 = 4 + 12 = 124
X =
n f = 20 and, s = 92
2 + 4 + ... + 2n
Y = 20 + 92 + b + 16 + 8 + 12 + 4
n
1 + 1 + 1... + (n times) n n = 240 – 59
X – Y = = = 1.
n n b = 29.

Chapter_30.indd 20 6/5/2015 3:59:52 PM


Set Theory I 30.21

19.
(a) Odd number of elements means that there can be 27.
(c) Shaded portion shows the area which represents
either 1 or 3 elements. Hence, number of ways that people who watch all the three channels. Now to
this happens is 4C1 + 4C3 = 4 + 4 = 8. calculate

20.
(d)

21.
(c) P. A = f; P ∪ A = D
P ∩ A = f means no alsatians are Pluto or Pluto is 22% 15%
not an alsatian where dogs are composed of alsatian or
Pluto or both.
22.
(a) Z = (P. D) ∪ M,  Z = (P ∩ D) ∪ M
P ∩ D means Pluto, the dog
DD
P ∩ D ∪ M means Pluto, the dog or any other mammal.
23.
(c) Y = F ∩ (D ∩ V) is not a null set means some F’s are the maximum value we have to assume that those who
D’s and some D’s are V’s. That means some fish are watch two channels is 0% and also those who watch
dogs. CNN only is also 0%

24.
(a) The shaded region is represented by the relationship
X = M.D. Now, it is given that M ∪ D = D. It means
that the D is a subset of M. Hence, all the dogs are
mammals.
M D

65

Hence it is clear that the maximum percentage of


people who can watch all the three channels is that
M.D
component (CNN) which has the least percentage of
all the three. Hence, option (c) is our answer.
25.
(a) The shaded portion represents the area which shows
people who do not watch DD channel. Since 80% 28.
(a) In case shop is to be set up within 3 Km of each city,
watch DD hence 20% do not watch DD. Let those then one shop will be required to cater to cities with a
who watch BBC and CNN only be x% then maximum distance of 6 Km between them.
29.
(a) Since the maximum distance between two cities is 5
Km and we need one shop for a distance of 8 Km
hence to serve cities having distance 5 Km between
them one more shop will be needed.
Hence total number of shops required is 2.

30.
(b) Fraction of people who watch cable only

2 1  17
 −  =
 3 10  30
DD
Cable VCR
12 + 10 – x = 20
⇒ x = 2
Hence, 2% watch BBC and CNN only.

26.
(d) Since percentage of those who watch BBC and CNN
or those who watch DD only this question can not be
answered. and fraction of people who watch VCR only

Chapter_30.indd 21 6/5/2015 3:59:52 PM


30.22 Chapter 30

1 1 1 34.
(b) Number of persons reading both the papers in
 −  = s Dighospur = 117 + 121 = 200 = 38
 5 10  10

Therefore, fraction of people who watch either


17 1 20 2
cable or VCR = + = = .
30 10 30 3

31.
(b)

\ Number or persons reading Ganashakti only


= 121 – 38 = 83.

35.
(a)

Therefore % of those who passed in either of two


but not in both = 20 + 27 = 47%
\ Required number = 2500 × 47% = 1,175.

32.
(c) Number of persons who read Ganashakti only =
(33 – 13) = 20
36.
(b)


Number of persons who read Anand Bazar Patrika 37.
(a)
only = (32 – 13) = 19
Hence, number of persons who read only one
newspaper = (20 + 19) = 39.

33.
(a) Number of persons reading both the papers in Aghosh
colony 33 + 32 – 52 = 13

38. (b) For n = 1, the requirement cannot be satisfied.


But it is possible for n = 2
The partitioned subsets are {2, 3, 4, 7} and {1, 5, 6, 8},
where 4 and 5 are the arithmetic means respectively.

Chapter_30.indd 22 6/5/2015 3:59:53 PM


Permutations and
CHAPTER

Combinations 31
Introduction 30! 30 × 29 × 28!
Solution: (i) =
We often come across questions such as the following: 28! 28!
1. In how many ways can four bottles be arranged in a row? = 30 × 29 = 870
2. In how many ways can five students be seated at a
round table? 9! 9 × 8 × 7 × 6 × 5!
(ii) = = 504
3. In how many ways can a group of five people be 5!3! 5! × 3× 2
selected out of a gathering of ten people?
4. In how many ways can 5 maps be selected out of 8 and 12! − 10! 12 × 11× 10! − 10!
(iii) =
displayed in a row? 9! 9!

Answers to these questions and many other important 10!


and more difficult ones can often be given without actually = [132 – 1]
9!
writing down all the different possibilities. In this chapter,
we will study some basic principles of the art of counting = 10 × 131
without counting which will enable us to answer such = 1310
questions in an elegant manner. 1 1 1 4×5 5 1
(iv)   + + = + +
Factorial Notation 3! 4! 5! 3! × 4 × 5 4! × 5 5!
The continued product of first n natural numbers is called n 20 5 1
factorial or factorial n and is denoted by or n! = + +
5! 5! 5!
Thus, n or n! = 1 × 2 × 3 × 4 ... (n – 1)n
= n ×(n – 1) × (n – 2) ... 3 × 2 × 1 26 13
= =
(in reverse order) 5! 60
Notes:
Illustration 2  Convert into factorials:
1. When n is a negative integer or a fraction, n! is not (i) 4.5.6.7.8.9.10.11.   (ii) 2.4.6.8.10.
defined. Thus, n! is defined only for positive integers. Solution: (i) 4 × 5 × 6 × 7 × 8 × 9 × 10 × 11
2. According to the above definition, 0! makes no sense.
However, we define 0! = 1. 1.2.3.4.5.6.7.8.9.10.11
=
3. n! = n (n – 1)! 1.2.3.
4. (2n)! = 2n × n! [1 × 3 × 5 × 7 ... (2n – 1)]. 11!
=
Illustration 1  Evaluate 3!
30! 9! (ii) 2 × 4 × 6 × 8 × 10
(i) (ii)
28! 5!3! = (2 × 1) × (2 × 2) × (2 × 3) × (2 × 4) × (2.5)
12! − 10! 1 1 1 = (2 × 2 × 2 × 2 × 2) × (1 × 2 × 3 × 4 × 5)
(iii) (iv) + +
9! 3! 4! 5! = 25 × 5!

Chapter_31.indd 1 6/5/2015 4:48:06 PM


31.2 Chapter 31

Fundamental Principle of Counting choices for returning to Agra, he can perform the journey in
Multiplication Principle  If an operation can be performed 6 × 5 ways, that is, 30 ways.
in ‘m’ different ways; following which a second operation Addition Principle
can be performed in ‘n’ different ways, then the two operations
If an operation can be performed in ‘m’ different ways
in succession can be performed in m × n different ways.
and another operation, which is independent of the first
Illustration 3  How many numbers of two digits can be operation, can be performed in ‘n’ different ways, then either
formed out of the digits 1, 2, 3, 4, no digit being repeated? of the two operations can be performed in (m + n) ways.
Solution: The first digit can be any one of the four digits
Note:
1, 2, 3, 4, that is the first digit can be chosen in four ways.
Having chosen the first digit, we are left with three digits The above two principles can be extended for any finite
from which the second digit can be chosen. Therefore, number of operations.
the possible ways of choosing the two digits are 12 ways
Illustration 6  Suppose there are 5 gates to a stadium, 2
of choosing both the digits. Thus, 12 numbers can be formed.
on one side and 3 on the other. Sohan has to go out of the
Since the first digit can be chosen in four ways and for stadium. he can go out from any one of the 5 gates. Thus,
each choice of the first digit there are three ways of choosing the number of ways in which he can go out is 5. Hence,
the second digit, therefore, there are 4 × 3 the work of going out through the gates on one side will be
done in 2 ways and the work of going out through the gates
on other side will be done in 3 ways. The work of going out
will be done when Sohan goes out from side I or side II.
Thus the work of going out can be done in (2 + 3) = 5 ways.

Note:

Addition theorem of counting is also true for more than two


opeartions.
Illustration 4  Anu wishes to buy a birthday card for her
brother Manu and send it by post. Five different types of Permutation
cards are available at the card shop, and four different types Each of the different arrangements which can be made by
of postage stamps are available at the post office. In how taking some or all of given number of things or objects at a
many ways can she choose the card and the stamp? time is called a permutation.
Solution:  She can choose the card in five ways. For each
choice of the card she has four choices for the stamp. Note:
Therefore, there are 5 × 4 ways, that is, 20 ways of choosing
the card and the stamp. Permutation of things means arrangement of things. The
word arrangement is used if order of things is taken into
Illustration 5  Mohan wishes to go from Agra to Chennai account. Thus, if order of different things changes, then
by train and return from Chennai to Delhi by air. There are their arrangement also changes.
six different trains from Agra to Chennai and five different
flights from Chennai to Agra. In how many ways can he Notation
perform the journey? Let r and n be positive integers such that 1 ≤ r ≤ n. Then,
Solution:  Since he can choose any one of the six trains the number of permutations of n different things, taken r at
for going to Chennai, and for each such choice he has five a time, is denoted by the symbol nPr or P, (n, r).

Chapter_31.indd 2 6/5/2015 4:48:06 PM


Permutations and Combinations 31.3

soMe Basic results

Illustration 9 In how many ways can 5 apples be distributed


n! among 4 boys, there being no restriction to the number of
1. nPr or, P(n, r) = = n (n – 1) (n – 2) apples each boy may get?
(n − r )!
Solution: The required number of ways = 45
...[n – (r + 1)], 0 ≤ r ≤ n.
Illustration 7 Evaluate the following: 4. Permutations Under Restrictions
(i) P(6, 4), (ii) P(15, 3), (iii) P(30, 2). (a) Number of permutations of n different things,
Solution: (i) We have taken r at a time, when a particular thing is to
6! 6.5.4.3.2.1 be always included in each arrangement, is
P(6, 4) = = = 360
(6 − 4)! 2.1 r × n–1Pr–1.
(ii) We have (b) Number of permutations of n different things,
15! 15! taken r at a time, when s particular things are
P(15, 3) = = to be always included in each arrangement, is
(15 − 3)! 12!
s! (r – (s – 1)) × n–sPr–s.
(15 ⋅14 ⋅13) (12!)
= = 2730 (c) Number of permutations of n different things,
12!
taken r at a time, when a particular thing is
30! 30! never taken in each arrangement, is
(iii) P(30, 2) = =
(30 − 2)! 28! n–1
Pr.
(30.29) (28!) (d) Number of permutations of n different things,
=
28! taken all at a time, when m specifi ed things
= 870 always come together, is m! × (n – m + 1)!
(e) Number of permutations of n different things,
2. The number of permutations of n things, taken taken all at a time, when m specifi ed things
all at a time, out of which p are alike and are of never come together, is n! – m!× (n – m + 1)!.
one type, q are alike and are of second type and
n! 5. Circular Permutations
rest are all different = .
p! q! (a) Number of circular arrangements (permutations)
of n different things = (n – 1)!.
Illustration 8 There are 5 red, 4 white and 3 blue marbles
in a bag. They are drawn one by one and arranged in a row.
Illustration 10 In how many ways can eight people be
Assuming that all the 12 marbles are drawn, determine the
seated at a round table?
number of different arrangements.
Solution: Required number of ways = (8 – 1)!
Solution: Here, n = 12, p1 = 5, p2 = 4 and, p3 = 3
= 7! = 5040
\ The required number of different arrangements
n! 12!
= = (b) Number of circular arrangements
p1 ! p2 ! p3 ! 5!4!3!
(permutations) of n different things when
12 × 11× 10 × 9 × 8 × 7 × 6 11× 10 × 9 × 8 × 7 clockwise and anticlockwise arrangements
= = are not different, that is, when observation
4 × 3 × 2 × 1× 3 × 2 × 1 2
= 990 × 4 × 7 = 27720 1
can be made from both sides = (n − 1)! .
2
3. The number of permutations of n different things
taken r at a time when each thing may be repeated
Illustration 11 Find the number of ways in which n
any number of times is nr.
different beads can be arranged to form a necklace.

Chapter_31.indd 3 6/5/2015 4:48:07 PM


31.4 Chapter 31

Solution: Required number of arrangements


n
1 1 Cr n − r +1
= (5 – 1)! = × 4! = 12 5. n
= .
2 2 Cr −1 r
Combination
6. If n is even then the greatest value of nCr is nCn/2.
Each of the different groups or selections which can be made
by taking some or all of a number of things (irrespective of 7. If n is odd then the greatest value of nCr is
order) is called a combination. n
C n +1 or, n
C n −1 .
2 2
Note:
r decreasing numbers starting with n
Combination of things means selection of things. Obviously, 8. nCr =
r increasing numbers starting with 1
in selection of things order of things has no importance.
Thus, with the change of order of things selection of things n (n − 1) (n − 2) ...(n − r + 1)
does not change. =
1× 2 × 3... r.
Notations The number of combinations of n different
things taken r at a time is denoted by nCr or, C (n, r). 9. nC0 + nC1 + nC2 + ... + nCn = 2n.
n! 10. nC0 + nC2 + nC4 + ... = nC1 + nC3 + nC5 + ...
Thus, nCr = (0 ≤ r ≤ n)
r ! (n − r )! = 2n–1.
n
Pr 11. Number of combinations of n different things
=
r! taken r at a time
n (n − 1) (n − 2) ... (n − r + 1) (a) when p particular things are always included
=
r (r − 1) (r − 2) ... 3 ⋅ 2 ⋅1 = n–pCr–p
If r > n, then, nCr = 0. (b) when p particular things are never included
Illustration 12 Evaluate: = n–pCr
(i) 11C3 (ii) 10C8 (iii) 100C98 (c) when p particular things are not together in
11! 11! any selection = nCr – n–pCr–p.
Solution: (i) C3 =
11
=
3! (11 − 3)! 3!8!
Illustration 13 In how many ways can 5 members forming
11 × 10 × 9 × 8! a committee out of 10 be selected so that
= = 165
3 × 2 × 8! (i) two particular members must be included.
10! 10 × 9 × 8! (ii) two particular members must not be included.
(ii) 10
C8 = = = 45
8! 2! 8! × 2
Solution: (i) When two particular members are included,
100 100! then, we have to select 5 – 2 = 3 members out of 10 – 2 = 8
(iii) C98 =
98! 2!
\ The required no. of ways
100 × 99 × 98! 8! 8× 7× 6
= = 4950 = C (8, 3) = = = 56
98! × 2 3!5! 6
(ii) When 2 particular members are not included, then,
Key Points to reMeMBer we have to select 5 members out of 10 – 2 = 8
1. nCr = nCn – r \ The required no. of ways
2. nC0 = nCn = 1, nC1 = n 8!
= C (8, 5) =
3. If nCx = nCy then either x = y 5!3!
or, y = n – x that is, x + y = n
8× 7× 6
4. nCr + nCr–1 = n+1Cr = = 56
6

Chapter_31.indd 4 6/5/2015 4:48:09 PM


Permutations and Combinations 31.5

12. (a) Number of selections of r consecutive things (f) If out of (p + q + r + t) things, p are alike
out of n things in a row = n – r + 1. of one kind, q are alike of second kind, r are
alike of third kind and t are different, then the
(b) Number of selections of r consecutive things
total number of selections is
out of n things along a circle
(p + 1) (q + 1) (r + 1) 2t – 1.
{n, when r < n
1, when r = n
. (g) The number of ways of selecting some or all
out of p + q + r items where p are alike of one
13. (a) Number of selections of zero or more things kind, q are alike of second kind and rest are
out of n different things alike of third kind is [(p + 1) (q + 1) (r + 1)] – 1.
n
C0 + nC1 + nC2 + ... + nCn = 2n 14. (a) Number of ways of dividing m + n different
things in two groups containing m and n
(b) Number of combinations of n different things
things, respectively (m ≠ n):
selecting at least one of them is
n (m + n)!
C1 + nC2 + ... + nCn = 2n – 1. m+n
Cm = .
m! n!
(c) Number of selections of r things (r ≤ n) out of
n identical things is 1. (b) Number of ways of dividing m + n + p
(d) Number of selections of zero or more things different things in three groups containing
out of n identical things = n + 1. m, n and p things, respectively (m ≠ n ≠ p):
(e) Number of selections of one or more things (m + n + p )!
.
out of n identical things = n. m! n! p !

soMe useful sHort-cut MetHoDs

1. The number of triangles which can be formed by 3. If there are ‘m’ horizontal lines and ‘n’ vertical
joining the angular points of a polygon of n sides lines then the number of different rectangles
n(n − 1)(n − 2) formed are given by (mC2 × nC2).
as vertices are .
6 Illustration 16 In a chess board, there are 9 vertical and 9
Illustration 14 Find the number of triangles formed by horizontal lines. Find the number of rectangles formed in
joining the vertices of an octagon. the chess board.
Solution: The required number of triangles Solution: The require number of rectangles
= 9C2 × 9C2 = 36 × 36 = 1296
n(n − 1)(n − 2)
=
6 4. These are ‘n’ points in a plane out of which ‘m’
points are collinear. The number of triangles formed
8(8 − 1)(8 − 2) 8× 7 × 6
= = = 56 by the points as vertices are given by nC3 – mC3.
6 6
Illustration 17 There are 14 points in a plane out of which
2. The number of diagonals which can be formed 4 are collinear. Fine the number of triangles formed by the
by joining the vertices of a polygon of n sides are points as vertices.
n(n − 3) Solution: The required number of triangles
.
2 = 14C3 – 4C3 = 364 – 4 = 360
Illustration 15 How many diagonals are there in a decagon? 5. There are ‘n’ points in a plane out of which ‘m’
Solution: The required number of diagonals points are collinear. The number of straight lines
formed by joining them are given by
n(n − 3) 10(10 − 3) 10 × 7
= = = = 35 (nC2 – mC2 + 1).
2 2 2

Chapter_31.indd 5 6/5/2015 4:48:10 PM


31.6 Chapter 31

Illustration 18 There are 10 points in a plane out of which


There are 10 points in a plane out of which
5 are collinear. Find the number of straight lines formed by 8. There are n points in a plane and no points are
joining them. collinear, then the number of straight lines that
can be drawn using these ‘n’ points are given by
Solution: The required number of straight lines
n(n − 1)
= nC2 – mC2 + 1 .
2
= 10C2 – 5C2 + 1 = 45 – 10 + 1 = 36
Illustration 21 How many straight lines can be drawn with
6. If there are ‘n’ points in a plane and no three points 18 points on a plane of which no points are collinear?
are collinear, then the number of triangles formed
Solution: The required number of straight lines
n(n − 1)(n − 2)
with ‘n’ points are given by . 18(18 − 1) 18 × 17 n(n − 2)
6 = = = = 153
2 2 2
Illustration 19 Find the number of triangles that can be
9. In a party, every person shakes hands with every
formed with 14 points in a plane of which no three points
other person. If there was a total of H handshakes
are collinear.
in the party, then the number of persons ‘n’ who
Solution: The required number of triangles were present in the party can be calculated from
n(n − 1)(n − 2) 14 × 13 × 12 the equation:
= = = 364 n(n − 1)
6 6 = H.
2
7. The number of quadrilaterals that can be formed
by joining the vertices of a polygon of n sides are Illustration 22 In a party every person shakes hands with
n(n − 1)(n − 2)(n − 3) every other person. If there was a total of 105 handshakes
given by , where n >3. in the party, fi nd the number of persons who were present
24
in the party.
Illustration 20 Find the number of quadrilaterals that can Solution: Let ‘n’ be the number of persons present in the
be formed by joining the vertices of a septagon. party.
Solution: The required number of quadrilaterals We have the equation
n(n − 1)(n − 2)(n − 3) n(n − 1)
= =H
24 2
7(7 − 1)(7 − 2)(7 − 3) n(n − 1)
= ⇒ = 105
24 2
7 × 6× 5× 4 ⇒ n(n – 1) = 15 × (15 – 1)
= = 35
24 ⇒ n = 15

Practice Exercises

Difficulty level-1
(BaseD on MeMory)

1. There are 6 boxes numbered 1, 2, ... 6. Each box is to 2. A graph may be defi ned as a set of points connected by
be fi lled up either with a red or a green ball in such a lines called edges. Every edge connects a pair of points.
way that at least 1 box contains a green ball and the boxes Thus, a triangle is a graph with 3 edges and 3 points.
containing green balls are consecutively numbered. The The degree of a point is the number of edges connected
total number of ways in which this can be done is: to it. For example, a triangle is a graph with three points
(a) 5 (b) 21 of degree 2 each. Consider a graph with 12 points. It is
possible to reach any point from any other point through
(c) 33 (d) 60
a sequence of edges. The number of edges, e, in the graph
[Based on CAT, 2003] must satisfy the condition:

Chapter_31.indd 6 6/5/2015 4:48:11 PM


Permutations and Combinations 31.7

(a) 11 ≤ e ≤ 66 (b) 10 ≤ e ≤ 66 The value of 10C4 + 10C5 is:


10.
(c) 11 ≤ e ≤ 65 (d) 0 ≤ e ≤ 11 (a) 462 (b) 466
[Based on CAT, 2003] (c) 469 (d) 465
[Based on MAT, 2005]
3. The letters of the word PROMISE are arranged so that 11.
In the figure below, how many paths are there from A to X
no two of the vowels should come together. Find the total if the only ways to move are up and to the right?
number of arrangements.
(a) 49 (b) 1440
(c) 7 (d) 1898
[Based on MAT, 2003]
4. In an examination paper there are two groups, each
containing 4 questions. A candidate is required to attempt
(a) 4 (b) 5
5 questions but not more than 3 questions from any group.
In how many ways can 5 questions be selected? (c) 6 (d) 9
[Based on MAT (Dec), 2006]
(a) 24 (b) 48
12.
All letters of the word ‘AGAIN’ are permuted in all
(c) 96 (d) None of these
possible ways and the words so formed (with or without
[Based on MAT, 2002]
meaning) are written as in a dictionary. Then, the 50th
5. A box contains 10 balls out of which 3 are red and the word is:
rest are blue. In how many ways can a random sample (a) NAAGI (b) NAA1G
of 6 balls be drawn from the bag so that at the most 2 red
(c) IAANG (d) INAGA
balls are included in the sample and no sample has all the
6 balls of the same colour? [Based on MAT (Feb), 2011]

(a) 105 (b) 168 13.


An examination paper contains 8 questions of which 4
have 3 possible answers each, 3 have 2 possible answers
(c) 189 (d) 120
each and the remaining one question has 5 possible
[Based on MAT, 2002] answers. The total number of possible answers to all the
6.
A cricket team of 11 players is to be formed from 20 questions is:
players including 6 bowlers and 3 wicketkeepers. The (a) 1278 (b) 1728
number of ways in which a team can be formed having (c) 1306 (d) 3240
exactly 4 bowlers and 2 wicketkeepers is: [Based on MAT (Feb), 2011]
(a) 20790 (b) 6930 14.
There are ten steps in a staircase and a person has to take
(c) 10790 (d) 360 those steps. At every step, the person has got a choice of
[Based on MAT, 2002] taking one step or two steps or three steps. The number of
7. Three boys and three girls are to be seated around a table ways in which a person can take those steps is:
in a circle. Among them the boy X does not want any girl (a) 310 (b) 39
8
neighbour and the girl Y does not want any boy neighbour. (c) 3 (d) None of these
How many such arrangements are possible? [Based on MAT (Feb), 2011]
(a) 5 (b) 6 15.
What is the number of six-digit telephone numbers in
(c) 4 (d) 2 a city if atleast one of their digits is repeated and zero
[Based on MAT, 2002]
cannot initiate, the number?
(a) 763920 (b) 453621
8.
In a hockey championship, three were 153 matches
played. Every two teams played one match with (c) 145698 (d) 781243
each other. The number of teams participating in the [Based on MAT (Dec), 2010]
championship is: 16.
Eight balls of different colours are to be placed in three
(a) 18 (b) 19 boxes of different sizes. Each box can hold all the eight
balls. What is the number of ways the balls can be placed
(c) 17 (d) 16
so that no box remains empty?
[Based on MAT, 2002]
(a) 7968 (b) 6796
9.
Seven points lie on a circle. How many chords can be
(c) 3652 (d) 846720
drawn by joining these points?
[Based on MAT (Dec), 2010]
(a) 22 (b) 21
17.
There are 5 different green dyes, 4 different blue dyes and
(c) 23 (d) 24 3 different red dyes. How many combinations of dyes can
[Based on MAT, 2005] be chosen taking atleast one green and one blue dye?

Chapter_31.indd 7 6/5/2015 4:48:11 PM


31.8 Chapter 31

(a) 3720 (b) 1253 (a) 104 (b) 10!


(c) 2546 (d) 2373 (c) 94 (d) 9!
[Based on MAT (Dec), 2010] [Based on MAT (Sept), 2009]
18.
What is the number of ways in which 5 identical balls can 25.
A number lock on a suitcase has 3 wheels each labelled
be distributed among 10 identical boxes, if not more than with 10 digits from 0 to 9. If opening of the lock is a
one ball can go into a box? particular sequence of three digits with no repeats, how
(a) 230 (b) 204 many such sequences will be possible?
(c) 250 (d) None of these (a) 720 (b) 760
[Based on MAT (Dec), 2010] (c) 680 (d) 780
19.
Three boys and three girls are to be seated around a [Based on MAT (Sept), 2009]
table in a circle. Among the boys, X does not want any 26.
There are 6 multiple choice questions in an examination.
girl adjacent to him and the girl Y does not want any How many sequences of answers are possible, if the first
boy adjacent to her. How many such arrangements are three questions have 4 choices each and the next three
possible? have 5 each?
(a) 8 (b) 4 (a) 4000 (b) 5000
(c) 6 (d) 2 (c) 6000 (d) 8000
[Based on MAT (Sept), 2010 (Dec), 2009] [Based on MAT (May), 2009]
20. In a monthly test, the teacher decides that there will be 27.
The average age of 3 children in a family is 20% of the
three questions; one each from Exercises 7, 8 and 9 of average age of the father and the eldest child. The total
the text book. There are 12 questions in Exercise 7, 18 in age of the mother and the youngest child is 39 years. If the
Exercise 8 and 9 in Exercise 9. In how many ways can the father’s age is 26 years, what is the age of the second child?
three questions be selected? (a) 20 years (b) 18 years
(a) 1944 (b) 2094
(c) 15 years (d) Cannot be determined
(c) 1894 (d) 2194
[Based on MAT (May), 2009]
[Based on MAT (Sept), 2010]
28.
The number of ways in which a committee of 3 ladies
21.
If there are 12 persons in a party and each of them and 4 gentlemen can be appointed out of 8 ladies and 7
shakes hands with each other, how many handshakes gentlemen, if Mrs X refuses to serve in a committee of
happen in the party? which Mr Y is a member, is
(a) 77 (b) 66 (a) 1540 (b) 1960
(c) 44 (d) 55
(c) 3240 (d) None of these
[Based on MAT (Sept), 2010]
[Based on MAT (Feb), 2009]
22.
There are 6 boxes numbered 1, 2, ..., 6. Each box is to 29.
A student is to answer 10 out of 13 questions in an
be filled up either with a red or a green ball in such a examination such that he must choose atleast four from
way that atleast 1 box contains a green ball and the boxes the first five questions. The number of choices available
containing green balls are consecutively numbered. The to him is:
total number of ways in which this can be done is:
(a) 140 (b) 196
(a) 33 (b) 21
(c) 280 (d) 346
(c) 5 (d) 60
[Based on MAT (May), 2010] [Based on MAT (May), 2008]
30.
The number of ways in which a team of eleven players
23.
A man has 5 friends and his wife has 4 friends. They
can be selected from 22 players including 2 of them and
want to invite either of their friends, one or more to
excluding 4 of them is
a party. In how many ways can they do so?
(a) 16C11 (b) 16C5
(a) 9 (b) 18
(c) 31 (d) 46 (c) 16C9 (d) 20C9
[Based on MAT (Feb), 2010] [Based on MAT (May), 2008]
31.
An examination paper contains 8 questions of which 4
24.
Chanda Negi got a 4-digit pass code (which is formed
have 3 possible answers each, 3 have 2 possible answers
out of the digits 0, 1, 2, 3, 4, 5, 6, 7, 8, 9) of her ATM
each and the remaining 1 question has 5 possible answers.
card from SBI Bank. But after the 50th day, she lost
The total number of possible answers to all the questions is:
the pass code and also forgot the number. How many
maximum number of trials she many have to take to get (a) 2880 (b) 78
the right number? (0 can be the beginning of the code (c) 94 (d) 3240
number.) [Based on MAT (May), 2008]

Chapter_31.indd 8 6/5/2015 4:48:11 PM


Permutations and Combinations 31.9

32.
The number of ways in which a committee of 5 can be 39.
The number of ways in which 6 men and 5 women can
chosen from 10 candidates so as to exclude the youngest, dine at a round table, if no two women are to sit together
if it includes the oldest is: is given by:
(a) 178 (b) 196 (a) 6! × 5! (b) 5! × 4!
(c) 292 (d) None of these (c) 30 (d) 7! × 5!
[Based on MAT (Feb), 2008] [Based on MAT (Feb), 2006]
33.
11 persons decide to spend an afternoon in two groups. A 40.
A number of points are marked on a plane and are connected
group of them decides to go to a theatre and the remaining pairwise by a line segment. If the total number of line
decide to play tennis. In how many ways can the group for segments is 10, how many points are marked on the plane?
tennis be formed, if there must be atleast four persons in (a) 14 (b) 10
each group?
(c) 5 (d) 9
(a) 1682 (b) 1584 [Based on MAT, 1999]
(c) 1884 (d) 1782 41.
There are 6 parallel vertical lines and 7 parallel horizontal
[Based on MAT (Dec), 2007] lines. These two groups of parallel lines intersect each
34.
A committee of 5 is to be formed from a group of 12 other. How many parallelograms will be formed?
students consisting of 8 boys and 4 girls. In how many (a) 294 (b) 42
ways can the committee be formed if it consists of exactly
(c) 315 (d) None of these
3 boys and 2 girls?
[Based on MAT, 1999]
(a) 436 (b) 336
42.
How many numbers between 1000 and 10,000 contain the
(c) 548 (d) 356 digits 1, 3, 5 and 7?
[Based on MAT (Dec), 2007] (a) 16 (b) 24
35.
In how many ways can a student choose a programme
(c) 12 (d) None of these
of 5 courses, if 9 courses are available and 2 courses are
[Based on MAT, 1999]
compulsory for every student?
43.
From among 36 teachers in a school, one prin­cipal and
(a) 45 ways (b) 55 ways
one vice-principal are to be appointed. In how many ways
(c) 35 ways (d) 65 ways can this be done?
[Based on MAT (Dec), 2007]
(a) 1260 (b) 1250
36.
A family consists of a grandfather, 5 sons and daughters
(c) 1240 (d) 1800
and 8 grandchildren. They are to be seated in a row for
dinner. The grandchildren wish to occupy the 4 seats at [Based on MAT, 1999]
each end and the grandfather refuses to have a grandchild 44.
A boy has 3 library tickets and 8 books of his interest in the
on either side of him. The number of ways in which the library. Of these 8, he does not want to borrow Chemistry
family can be made to sit is: part II unless Chem­istry part I is also borrowed. In how
many ways can he choose the three books to be borrowed?
(a) 11360 (b) 11520
(a) 56 (b) 27
(c) 21530 (d) None of these
[Based on MAT (Dec), 2006] (c) 26 (d) 41
37.
A father with 8 children takes 3 children at a time to the [Based on MAT, 1999]
zoological garden, as often as he can without taking the 45.
The number of 3-digit numbers exactly divisible by 5 is:
same 3 children together more than once. Then (a) 181 (b) 180
(a) number of times he will go to the zoological garden is 56 (c) 179 (d) 199
(b) number of times each child will go to the zoological [Based on MAT, 1999]
garden is 21 46.
In how many ways can six different rings be worn on four
(c) number of times a particular child will not go to the fingers of one hand?
zoological garden is 35 (a) 10 (b) 12
(d) All of the above (c) 15 (d) 16
[Based on MAT (Dec), 2006] [Based on MAT, 2000]
38.
There are 4 candidates for the post of a lecturer in 47.
How many arrangements can be formed out of the letters
Mathematics and one is to be selected by votes of 5 men. of the word EXAMINATION so that vowels always
The number of ways in which the votes can be given is: occupy odd places?
(a) 1048 (b) 1072 (a) 72000 (b) 86400
(c) 1024 (d) None of these (c) 10800 (d) 64000
[Based on MAT (Feb), 2006] [Based on SNAP, 2007]

Chapter_31.indd 9 6/5/2015 4:48:11 PM


31.10 Chapter 31

48.
There are 10 stations on a railway line. The number of (a) 4 (b) 3
different journey tickets that are required by the authorities (c) 2 (d) 6
is:
[Based on NMAT, 2005]
(a) 101 (b) 90
56.
A cultural committee of 6 is to be formed from 7 men
(c) 81 (d) 10
and 4 women. In how many ways the committee can be
[Based on SNAP, 2008] formed with at least 2 women in the committee?
49.
In how many ways can four letters of the word ‘SERIES’ (a) 210   (b)  140   (c)  300   (d) 371
be arranged?
[Based on ATMA, 2006]
(a) 24 (b) 42
(c) 84 (d) 102 57.
The highest power of 10 that will divide 100! is:
[Based on IIFT, 2010] (a)
2  (b) 24  (c) 4  (d) 25
50.
Amit has 11 friends: 7 boys and 4 girls. In how many [Based on ATMA, 2008]
ways, can Amit invite them, if there have to be exactly 4 58. The total number of possible proper three-digit integers
boys in the invitees? that can be formed using 0, 1, 3, 4 and 5 without repetition
(a) 560 (b) 450 such that they are divisible by 5 are:
(c) 650 (d) 820 (a) 20 (b) 21
[Based on JMET, 2011] (c) 22 (d) 24
Directions (Q. Nos. 51 to 52) Read the following information and [Based on JMET, 2009]
answer the questions. 59. There are 6 equally spaced points A, B, C, D, E and F
   Out of 6 ruling and 5 opposition party members, 4 are to be marked on a circle with radius R. How many convex
selected for a delegation. pentagons of distinctly different areas can be drawn using
these points as vertices?
51.
In how many ways can this be done so as to include
(a) 6P5 (b) 6
exactly one ruling party member?
(a) 50 (b) 80 (c) 55 (d) None of these
(c) 45 (d) 60 60. Find the number of ways in which ten different flowers
can be strung together to make a garland in such a way
[Based on JMET, 2011]
that three particular flowers are always together?
52.
In how many ways can this be done so as to include at
(a) 30240 (b) 30420
least one opposition member?
(c) 23400 (d) None of these
(a) 300 (b) 315
[Based on MAT, 2012]
(c) 415 (d) 410
[Based on JMET, 2011]
61. A committee of 3 experts is to be selected our of a
panel of 7 persons. Three of them are engineers, three
Directions (Q. 53–54): Answer the questions based on the are managers and one is both engineer and manager. In
following information. how many ways can the committee be selected, if it must
   On a shelf there are 3 books on Psychology, 3 books on have atleast one engineer and one manager?
Management and 4 books on Economics. (a) 33  (b) 22  (c) 11  (d) 66
[Based on MAT, 2012]
53. In how many ways can the books be arranged, if the books
on only Management are to be arranged together? 62. A cricket team of 11 players is to be formed from a pool of
16 players that includes 4 bowlers and 2 wicket keepers.
(a) 62540 (b) 320 In how many different ways can a team be formed, so that
(c) 40320 (d) None of these the team has atleast 3 bowlers and one wicket keeper?
[Based on IRMA, 2008] (a) 2472 (b) 2274
54. In how many ways can all the books be arranged at (c) 2427 (d) 1236
random? [Based on MAT, 2012]
(a) 3628800 (b) 41520 63. A student is to answer 10 out of 13 questions in an
(c) 30020 (d) 4164840 examination such that he must choose atleast 4 from the first
[Based on IRMA, 2008]
five questions. The number of choices available to him is:
55. How many different ways can 2 students be seated in a (a) 346 (b) 140
row of 4 desks, so that there is always at least one empty (c) 196 (d) 280
desk between the students? [Based on MAT, 2013]

Chapter_31.indd 10 6/5/2015 4:48:12 PM


Permutations and Combinations 31.11

64. In how many ways can 10 boys a1, a2, a3, … a10 can be 71. A family consists of a grandfather, 5 sons and daughters
seated in a row such that a2 always sits before a5 and a5 and 8 grandchildren. They are to be seated in a row for
always sits before a7? dinner. The grandfather wishes to occupy the 4 seats at
9! each end and the grandfather refuses to have a grandchild
(a) (b) 10 p3
3 on either side of him. The number of ways in which the
family can be made to sit is:
(c) 10 C3 × 90 (d) None of these
[Based on MAT, 2013] (a) 1136 (b) 11520
65. In a tennis tournament, eight players must be divided into (c) 21530 (d) None of these
four teams of two players each. The number of different [Based on SNAP, 2013]
ways in which this can be done is: 72. The number of ways in which a committee of 3 ladies
(a) 120 (b) 35 and 4 gentlemen can be appointed out of 8 ladies and 7
(c) 30 (d) 105 gentlemen, if Mrs. X refuses to serve in a committee of
[Based on MAT, 2013]
which Mr. Y is a member, is:
(a) 1,540 (b) 1,960
66. Two circles touch each other internally. The sum of
their areas is 116π cm2 and distance between their (c) 3,240 (d) None of these
[Based on SNAP 2012]
centres is 6 cm. Find the radii of the circles.
(a) 10 cm, 4 cm 73. In a football championship, 153 matches were played.
(b) 11 cm, 4 cm Every two teams played one match with each other. The
number of teams, participating in the championship were:
(c) 9 cm, 5 cm
(d) 10 cm, 5 cm (a) 18 (b) 14
[Based on MAT, 2014] (c) 16 (d) 22
67. There is a number lock with four rings. How many (Based on MAT 2011)
attempts at the maximum would have to be made before
74. In a certain city, all telephone numbers have six digits, the
getting the right number?
first two digits always being 41 or 42 or 46 or 62 or 64.
(a) 104 (b) 255 The number of telephone numbers having all six digits
(c) 104 − 1 (d) 256 distinct is:
[Based on MAT, 2014]
(a) 9200 (b) 7200
68. There are 10 persons P1, P2, . . . , P9, P10. Out of these 10
persons, 5 persons are to be arranged in a line such that in (c) 8400 (d) 1200
each arrangement P1 must occur whereas P4 and P5 do not (Based on MAT 2011)
occur. The number of such possible arrangements are: 75. There are six teachers. Out of them, two are primary
(a) 7C4 × 5! teachers and two are secondary teachers. They are to stand
(b) 9C5 × 5! in a row, so as the primary teachers, middle teachers and
(c) 8C5 × 5! secondary teachers are always in a set. The number of
ways in which they can do so, is:
(d) 9C5 × 4!
[Based on MAT, 2014] (a) 52 (b) 48
69. There are five boys and three girls who are sitting together (c) 34 (d) 34
to discuss a management problem at a round table. In how (Based on MAT 2011)
many ways can they sit around the table so that no two 76. A candidate is required to answer 7 questions out of
girls are together? 12 questions which are divided into two groups each
(a) 1200 (b) 1400 containing 6 questions. He is not permitted to attempt
(c) 1420 (d) 1440 more than 5 questions from each group. The number of
[Based on SNAP, 2013] ways in which he can choose the 7 questions is:
70. The number of ways in which a committee of 3 ladies and (a) 780 (b) 640
4 gentlemen can be appointed from a meeting consisting (c) 820 (d) 720
of 8 ladies and 7 gentlemen, if Mrs. X refuses to serve in (Based on MAT (Feb), 2012)
a committee if Mr. Y is its member, is:
77. If 20 lines are drawn in a plane such that no two of them
(a) 1960
are parallel and no three are concurrent, the number of
(b) 3240 points in which they intersect each other is:
(c) 1540 (a) 220 (b) 190
(d) None of these (c) 250 (d) 120
[Based on SNAP, 2013] (Based on MAT (Feb), 2012)

Chapter_31.indd 11 6/5/2015 4:48:13 PM


31.12 Chapter 31

78. A boy has 3 library tickets and 8 books of his interest are 79. What is the maximum sum of the terms in the arithmetic
there in the library. Out of these 8, he does not want to progression 25, 24½, 24, … ?
borrow Chemistry Part II, unless Chemistry Part I is also (a) 637½
borrowed. The number of ways in which he can choose (b) 625
the three books to be borrowed is:
(c) 662½
(a) 61 (b) 32
(d) 650
(c) 51 (d) None of these (Based on SNAP 2013)
(Based on MAT (Feb), 2012)

Difficulty Level-2
(Based on Memory)

1. How many numbers greater than a million can be formed of the class is 5 feet. What is the difference between the
by using the digits 7, 4, 6 and 0 if 4 has to be used twice, maximum and minimum number of handshakes that can
6 has to be used thrice and the rest only once? happen in such a class?

7!  40 C2 
(a) 720 (b) (a)  − 20  (b) 361
3!  2 
 
(c) 360 (d) None of these
(c) 40C2 – 40 (d) 40C2
2.
From a group of 8 students, 3 students have to be selected.
8.
8 lines in a set are parallel to each other, where the
Mr X is not selected unless Mr Y is also selected. In how
distance between any two adjacent parallel lines is 1 cm.
many ways can the 3 students be selected?
6 parallel lines in another set are there where the distance
(a) 41 (b) 21 between any two adjacent lines is also 1 cm. These 6 lines
(c) 20 (d) None of these of the second set intersect the lines of the previous set
3.
A box contains 5 white balls, and 7 red balls. In how many to form a number of parallelograms. How many of the
ways can 3 balls be drawn from the box if at least one parallelograms thus formed are not rhombuses?
black ball is to be included in the draw? (a) 385 (b) 365
(a) 396 (b) 180 (c) 350 (d) 335
(c) 20 (d) 596
9. 18 guests have to sit, half on each side of a long table.
4.
The total number of numbers from 1,000 to 9,999 (both Four particular guests desire to sit on a particular side,
inclusive) that do not have four different digits is: three others on the other side. The number of ways in
which the seating arrangement can be made is
(a) 1,236 (b) 2,325
(c) 3,574 (d) 4,464 11!9! 11!9!9!
(a) (b)
5!6! 5!6!
5. What is the remainder left after dividing 1! + 2! + 3! + ...
+ 100! by 7? (c) 11! 4! 3! (d) None of these
(a) 0 (b) 5 10. Five persons A, B, C, D and E occupy seats in a row such
(c) 21 (d) 14 that A and B sit next to each other. In how many possible
[Based on CAT, 2003] ways can these five people sit?
6.
There are 5 boys and 3 girls in a family. They are (a) 24 (b) 48
photographed in groups of 2 boys and one girl. The (c) 72 (d) None of these
number of different photographs will be: [Based on IIT Joint Man. Ent. Test, 2004]
(a) 360 (b) 180
11. What is the number of ways in which triangles are formed
(c) 30 (d) 60 in an octagon with each triangle having the centre point
of the octagon as one vertex and vertices of octagon as
7.
There are 40 students in a class. A student is allowed to
the other?
shake hand only once with a student who is taller than
him or equal in height to him. He cannot shake hand (a) 25 (b) 28
with somebody who is shorter than him. Average height (c) 30 (d) 32

Chapter_31.indd 12 6/5/2015 4:48:13 PM


Permutations and Combinations 31.13

Directions (Q. 12 and 13):  Refer to the data and answer the 20. How many four-digit numbers, each divisible by 4 can
questions that follow: Five couples, Bansals, Kansals, Singhals, be formed using the digits 1, 2, 3, 4 and 5, repetitions of
Sharmas and Khannas go for dinner. They want to sit around a digits being allowed in any number?
round table. (a) 100 (b) 150
12. If the couples want to sit together, number of ways in (c) 125 (d) 75
which they can sit around the table is: [Based on SCMHRD, 2002]
10 9! 21. How many 6 digit numbers can be formed using the digit
(a) 5 (b) 5
2 2 2 two times and the digit 4 four times?
(c) 4! 25 (d) 9! 25 (a) 16 (b) 15

13. From a group of 7 men and 6 women 5 people are to be (c) 18 (d) 64
selected to form a committee so that at least 3 men are 22. Mr Greg Chappel has to select the Indian team for the
there on the committee. In how many ways can it be done? finals among 8 bowlers, 6 batsmen and 3 wicketkeepers.
(a) 756 (b) 735 How many possible combinations does Greg has at his
(c) 564 (d) 645 disposal if he has to choose 11 players consisting of 5
bowlers, 5 batsmen and a wicketkeeper?
[Based on IRMA, 2002]
(a) 504 ways
14.
In how many different ways can the letters of the word
TRAINER be arranged so that the vowels always come (b) 1,008 ways
together? (c) 108 ways
(a) 1440 (b) 120 (d) 1,196 ways
(c) 720 (d) 360
23. How many four-letter computer passwords can be
[Based on IRMA, 2002]
formed using only the symmetric letters (no repetition
15. A polygon has 44 diagonals, then the number of its sides are: allowed)?
(a) 11 (b) 9 (a) 7920 (b) 330
(c) 7 (d) 5 (c) 14640 (d) 419430
[Based on FMS (Delhi), 2002]
[Based on CAT, 2002]
16. Ten different letters of an alphabet are given. Words with
five letters are formed from these given letters. Then the 24. How many three-letter computer passwords can be
number of words which have at least one letter repeated is: formed (no repetition allowed) with at least one symmetric
letter?
(a) 19670 (b) 39758
(a) 990 (b) 2730
(c) 69760 (d) 99748
(c) 12870 (d) 1560000
[Based on FMS (Delhi), 2002]
[Based on CAT, 2002]
17. A five-digit number divisible by 3 is to be formed using
the numerals 0, 1, 2, 3, 4 and 5, without repetition. The 25. In how many ways is it possible to choose a white square
total number of ways in which this can be done is: and a black square on a chess board so that the squares
must not lie in the same row or column?
(a) 211 (b) 216
(a) 56 (b) 896
(c) 221 (d) 311
(c) 60 (d) 768
[Based on FMS (Delhi), 2002] [Based on CAT, 2002]
18. How many parallelograms will be formed if 7 parallel
26.
There are 5 red, 6 black and 4 white balls in a box.
horizontal lines intersect 6 parallel vertical lines?
First ball drawn is kept back after noting down its
(a) 42 (b) 294 colour. In how many ways one can draw the second ball
(c) 315 (d) None of these having different colour than that of the previously selected
[Based on I.P. Univ., 2002] ball?
(a) 154 ways (b) 120 ways
1 × 3 × 5 ... (2n − 1)
19. Which of the following is equal to ? (c) 148 ways (d) 89 ways
2 × 4 × 6 ... (2n)
(a) (2n)! ÷ (2n(n!))2 (b) (2n)! ÷ n! 27. The figure below shows the network connecting cities
A, B, C, D, E and F. The arrows indicate permissible
(c) (2n – 1) ÷ (n – 1)! (d) 2n direction of travel. What is the number of distinct paths
[Based on SCMHRD, 2002] from A to F?

Chapter_31.indd 13 6/5/2015 4:48:13 PM


31.14 Chapter 31

35.
The Vice-Chancellor of University of Delhi decided to
form a committee to look into the feasibility of introduction
of semester systems at the undergraduate level in the
University. 5 members from the Executive Council and 7
members of the Academic Council were found suitable for
the job. In how many ways can the Vice-Chancellor form
a committee of 6 members such that at least 4 members of
the committee belong to the Academic Council?
(a) 462 (b) 422
(a) 9 (b) 10 (c) 412 (d) 442
(c) 11 (d) None of these [Based on FMS, 2009]
[Based on CAT, 2001] 36. How many three-digit numbers are there, such that if one
28. In how many ways seven girls and six boys can sit around of the digits is 8, the following digit is 9?
a round table so that no two boys sit together? (a) 666 (b) 665
(a) (6!)2 (b) 6! × 7! (c) 17 (d) 19
(c) (7!)2 (d) 5! × 6! 37. A football team of 11 players is posing for a photograph
along with its coach. The football players are standing in
29. There are 6 tasks and 6 persons. Task 1 cannot be assigned two rows in groups of six, one behind the other. The coach
either to person 1 or to person 2, task 2 must be assigned to and the vice captain stand together in the centre in the
either person 3 or person 4. Every person is to be assigned first row while the captain stands behind the vice captain.
one task. In how many way can the assignment be done? The goalkeeper stands in the corner, while exactly two out
(a) 144 (b) 180 of three defenders stand next to each other. In how many
(c) 192 (d) 360 ways can it be done?
[Based on CAT, 2006] (a) 18,720 (b) 34,560
7! 13333
(c) 95,040 (d) 1,29,600
30. Find the remainder when  (6!) is divided by 13.
  38. Eight people, A, B, C, D, E, F, G and H are sitting around
(a) 1 (b) 5 a circular table numbered 1 to 8 in a clockwise order.
(c) 8 (d) None of these A and F sit together. B and D never sit adjacent to each
other. H and G sit opposite each other. What is the total
31.
In how many ways can the letters of the word ABACUS be number of ways in which these people can be seated?
rearranged such that the vowels always appear together?
(a) 1,280 ways (b) 2,560 ways
(a) 6!/2! (b) 3! × 3!
(c) 5,040 ways (d) None of these
(c) (3! × 3!)/2! (d) (4! × 3!)/2!
[Based on SNAP, 2009] 39. Seven friends go to an electronics shop and buy items
worth `500, `600, `800, `1500, `1200, `1600 and
32.
A five-digit number divisible by 3 is to be formed using
`1800. They are carrying only 100-rupee notes. If they are
numerical 0, 1, 2, 3, 4 and 5 without repetition. The total
allowed to borrow and lend money from one another and
number of ways this can be done is:
the total money they have is exactly the amount required
(a) 122 (b) 210 to buy these items, what is the total number of different
(c) 216 (d) 217 ways they can contribute to the total money?
[Based on SNAP, 2009] (a) 48,63,51,200 (b) 5,37,32,00,880
33.
There are 10 stations on a railway line. The number of (c) 6,38,41,68,200 (d) None of these
different journey tickets that are required by the authorities is:
n n
(a) 92 (b) 90 Pr
(c) 91 (d) None of these
40.
The value of ∑ r!
is:
r =1
[Based on SNAP, 2010]
(a) 2 (b) 2n – 1
n
34.
In how many ways can the letters of the word ABACUS be
(c) 2n–1 (d) 2n + l
rearranged such that the vowels always appear together?
[Based on IIFT, 2007]
6!
(a) (b) 3! × 3! n+2 n−2
2! 41.
If C8 : P4 = 57:16, then n is equal to:
3!  3! 4!  3! (a) 20 (b) 22
(c) (d)
2! 2! (c) 15 (d) None of these
[Based on SNAP, 2010] [Based on ITFT, 2008]

Chapter_31.indd 14 6/5/2015 4:48:14 PM


Permutations and Combinations 31.15

42.
The number of ways in which a mixed double tennis game • If a match ends in a draw, both the teams get 1/2 point.
can be arranged amongst 9 married couples if no husband    After the league was over, the teams were ranked
and wife play in the same game is: according to the points that they earned at the end of
(a) 1514 (b) 1512 the tournament. Analysis of the points table revealed
(c) 3024 (d) None of these the following
[Based on ITFT, 2008] • Exactly half of the points earned by each team were
43.
While packing for a business trip Mr. Debashis has packed earned in games against the ten teams which finished
3 pairs of shoes, 4 pants, 3 half-pants, 6 shirts, 3 sweater at the bottom of the table.
and 2 jackets.The outfit is defined as consisting of a pair • Each of the bottom ten teams earned half of their total
of shoes, a choice of ‘lower wear’ (either a pant or a half- points against the other nine teams in the bottom ten.
pant), a choice of ‘upper wear’ (it could be a shirt or a
sweater or both) and finally he may or may not choose to How many teams participated in the league?
wear a jacket. How many different outfits are possible? (a) 16 (b) 18
(a) 567 (b) 1821 (c) 19 (d) 25
(c) 743 (d) None of these [Based on XAT, 2011]
[Based on ITFT, 2008] 48.
Suppose the English alphabet letters A, B, C, ..., Z are
44.
If F (x, n) be the number of ways of distributing ‘x’ toys to denoted by the remainders obtained on dividing the
‘n’ children so that each child receives at the most 2 toys, numbers 20, 21, 22, …, 225, respectively, by 29, then the
then F (4, 3) = letter ‘K’ would be denoted by:
(a) 2 (b) 3 (a) 6 (b) 7
(c) 4 (d) 6 (c) 8 (d) 9
[Based on XAT, 2008] [Based on JMET, 2011]
45.
Let X be a four-digit number with exactly three consecu- 49. A committee is to be formed from amongst 9 boys and
tive digits being same and it is a multiple of 9. How many 6 girls. In how many ways can the boys and girls divide
such X’s are possible? themselves into groups of three so that no group has more
(a) 12 (b) 16 than 2 girls and no group has all boys?
(c) 19 (d) None of these (a) 2025 × 6! (b) 45 × 9!
[Based on XAT, 2009]
46.
In the country of twenty, there are exactly twenty cities and 15
(c) × 9! × 3! (d) None of these
there is exactly one direct road between any two cities. No 4
two direct roads have an overlapping road segment. After
the election dates are announced, candidates from their 50. Everyday 5 visitors enter a residential complex and park
respective cities start visiting the other cities. Following their cars in one of the 8 adjacent spaces available. How
are the rules that the election commission has laid down many different arrangements of these 5 cars are possible
for the candidates, provided all the cars are always together?
• Each candidate must visit each of the other cities (a) 4! (b) 5 × 4!
exactly once. (c) 4 × 5! (d) 5!
• Each candidate must use only the direct roads between
two cities for going from one city to another. 51. How may zeroes are there at the end of 125! – 124! –
• The candidate must return to his own city at the end of 123!?
the campaign. (a) 27 (b) 28
• No direct road between two cities would be used by (c) 29 (d) 30
more than one candidate.
The maximum possible number of candidates is: 52. A pack of 52 playing cards comprises four suites of 13
(a) 5 (b) 6 cards each. In each suite, there are cards of 13 distinct
face values — from 1 to 13. In how many ways can a
(c) 7 (d) 9
pack of 52 playing cards be distributed equally among
[Based on XAT, 2011]
four persons so that each person receives cards of all the
47.
The football league of a certain country is played possible face values?
according to the following rules
• Each team plays exactly one game against each of the (a) 52C13 (b) (24)13
other teams.
52!
• The winning team of each game is awarded 1 point (c) (d) (13!)4
and the losing team gets 0 point. (13!) 4

Chapter_31.indd 15 6/5/2015 4:48:14 PM


31.16 Chapter 31

53. Let n! = 1 × 2 × 3 × . . . × n for integer n ≥ 1. If p = 1! + (2 60.


Eight chairs are numbered from 1 to 8. Two women
+ 2!) + (3 × 3!) + . . . + (10 + 10!), then p + 2 when divided and three men wish to occupy one chair each. First, the
by 11! leaves a remainder of women chose the chairs from amongst the chairs marked
(a) 10 (b) 0 1 to 4. Then the men selected the chairs from amongst
the remaining, marked 5 to 8. The number of possible
(c) 7 (d) 1 arrangements is:
54. If there are 10 positive real numbers n1 < n2 < n3 ... < n10 (a) 6C3 × 4C4 (b) 4P2 × 4P3
... How many triplets of these numbers (n1, n2, n3), (n2,
(c) 4C3 × 4P3 (d) 4C2 × 4C3
n3, n4) ... can be generated such that in each triplet the first
number is always less than the second number, and the 61. A box contains 10 balls out of which 3 are red and the
second number is always less than the third number: rest are blue. In how many ways can a random sample
(a) 45 (b) 90 of 6 balls be drawn from the bag so that at the most 2 red
balls are included in the sample and no sample has all the
(c) 120 (d) 180
6 balls of the same colour?
55. Boxes numbered 1, 2, 3, 4 and 5 are kept in a row, and
they are to be filled with either a red or a blue ball, such (a) 105 (b) 168
that no two adjacent boxes can be filled with blue balls. (c) 189 (d) 120
Then, how many different arrangements are possible, 62. How many integers, greater than 999 but not greater
given that all balls of a given colour are exactly identical than 4000, can be formed with the digits 0, 1, 2, 3 and 4,
in all respects? repetition of digits is allowed?
(a) 8 (b) 10 (a) 499 (b) 500
(c) 15 (d) 22 (c) 375 (d) 376
56. A, B, C, and D are four towns, any three of which are non- [Based on CAT, 2010]
collinear. Then, the number of ways to construct three 63. Rajat draws a 10 × 10 grid on the ground such that there
roads each joining a pair of towns so that the roads do not are 100 identical squares numbered from 1 to 100. If he
form a triangle is: has to place two identical stones on any two separate
(a) 7 (b) 8 squares in the grid, how many distinct ways are possible?
(c) 9 (d) 24 (a) 2475 (b) 4950
(c) 9900 (d) 1000
57. An intelligence agency forms a code of two distinct digits
[Based on CAT, 2011]
selected from 0, 1, 2, ..., 9 such that the first digit of code
is non zero. The code, handwritten on a slip, can however 64. Vaibhav wrote a certain number of positive prime numbers
potentially create confusion when read upside down, for on a piece of paper. Vikram wrote down the product of
example, the code 91 may appear as 16. How many codes all the possible triplets among those numbers. For every
are there for which no such confusion can arise? pair of numbers written by Vikram, Vishal wrote down the
corresponding GCD. If 90 out of the total numbers written
(a) 80 (b) 78
by Vishal were prime, how many numbers did Vaibhav
(c) 71 (d) 69 write?
58. For a scholarship, at the most n candidates out of 2n + 1 can (a) 6 (b) 8
be selected. If the number of different ways of selection (c) 10 (d) Cannot be determined
of at least one candidate is 63, the maximum number of [Based on CAT, 2011]
candidates that can be selected for the scholarship is:
65. Letter of the word ‘ATTRACT’ are written on cards and
(a) 3 (b) 4
are kept on a table. Manish is asked to lift three cards at
(c) 6 (d) 5 a time, write all possible combinations of the three letters
59. In a certain laboratory, chemicals are identified by a on a piece of paper and then replace the three cards. The
colour-coding system. There are 20 different chemicals. exercise ends when all possible combinations of letters
Each one is coded with either a single colour or a unique are exhausted. Then, he is asked to strike out all words
two-colour pair. If the order of colours in the pairs does not in his list that look the same when seen in a mirror. How
matter, what is the minimum number of different colours many words is he left with?
needed to code all 20 chemicals with either a single colour (a) 40
or a unique pair of colours? (b) 20
(a) 7 (b) 6 (c) 30
(c) 5 (d) 20 (d) None of these
[Based on NMAT, 2005] [Based on CAT, 2012]

Chapter_31.indd 16 6/5/2015 4:48:14 PM


Permutations and Combinations 31.17

66. A student is asked to form numbers between 3000 and (a) 449
9000 with digits 2, 3, 5, 7 and 9. If no digits is to be (b) 500
repeated, in how many ways can the student do so? (c) 375
(a) 24 (b) 120 (d) 376
(c) 60 (d) 72 [Based on CAT, 2008]
[Based on CAT, 2012] 70. A company CEO invited nine persons for a business
67. In a chess tournament held this year in Kolkata, there were meeting, where the host will be seated at a circular table.
only two women participants among all the members that How many different arrangements are possible, if two
participated in the tournament. Every participant played invitees X and Y be seated on either side of the host CEO?
two games with each other participant. The number of (a) 10080
games that men played between themselves proved to
exceed by 66, as compared to the number of games the (b) 10800
men played with women. How many participants were (c) 9200
there in the tournament? (d) 4600
(a) 156 (b) 13 [Based on MAT, 2012]
(c) 610 (d) 108 71. Read the following instruction carefully and answer the
[Based on CAT, 2013] question that follows:
68. ABC is a three-digit number in which A > 0. The value of
13
1 x
Expression ∑ can also be written as
ABC is equal to the sum of the factorials of its three digits. n =1 n 13!
What is the value of B? What would be remainder if x is divided by 11?
(a) 9 (b) 7 (a) 2
(c) 4 (d) 2 (b) 4
[Based on CAT, 1997] (c) 7
69. How many integers which are greater than 999 but not (d) 9
greater than 4000, can be formed with the digits 0, 1, 2, 3 (e) None of the above
and 4, if repetition of digits is allowed?
[Based on XAT, 2014]

Answer Keys
Difficulty Level-1

1. (b) 2. (a) 3. (b) 4. (b) 5. (b) 6. (a) 7. (c) 8. (a) 9. (b) 10. (a ) 11. (c) 12. (b) 13. (d)
14. (c) 15. (a) 16. (d) 17. (a) 18. (d) 19. (b) 20. (a) 21. (b) 22. (b) 23. (d) 24. (a) 25. (a) 26. (d)
27. (d) 28. (a) 29. (b) 30. (c ) 31. (d) 32. (b) 33. (b ) 34. (b) 35. (c) 36. (d) 37. (d) 38. (d ) 39. (a)
40. (c ) 41. (c) 42. (b) 43. (a ) 44. (d) 45. (b) 46. (c) 47. (c ) 48. (b ) 49. (d) 50. (a) 51. (d) 52. (b)
53. (d) 54. (a) 55. (d) 56. (d) 57. (b) 58. (b) 59. (b) 60. (d) 61. (a) 62. (a) 63. (b) 64. (a) 65. (b)
66. (a) 67. (c) 68. (a) 69. (d) 70. (d) 71. (b) 72. (a) 73. (a) 74. (c) 75. (b) 76. (a) 77. (b) 78. (d)
79. (a)

Difficulty Level-2

1. (c) 2. (a) 3. (d) 4. (d) 5. (b) 6. (b) 7. (d) 8. (d) 9. (b) 10. (b) 11. (b) 12. (c) 13. (a)
14. (d ) 15. (a) 16. (c) 17. (b) 18. (c) 19. (a) 20. (c) 21. (b) 22. (b) 23. (a) 24. (c ) 25. (d) 26. (c)
27. (b) 28. (b) 29. (a) 30. (a) 31. (d ) 32. (c) 33. (b) 34. (d) 35. (a ) 36. (b) 37. (d ) 38. (d) 39. (d)
40. (b) 41. (d) 42. (b) 43. (d) 44. (d) 45. (d) 46. (d ) 47. (d) 48. (d) 49. (c) 50. (c) 51. (c) 52. (b)
53. (d) 54. (c ) 55. (d) 56. (d ) 57. (d) 58. (a) 59. (b) 60. (b) 61. (b) 62. (d) 63. (b ) 64. (a) 65. (a)
66. (d) 67. (b) 68. (c) 69. (d) 70. (a) 71. (d)

Chapter_31.indd 17 6/5/2015 4:48:15 PM


31.18 Chapter 31

Explanatory Answers

Difficulty Level-1

1.
(b) The number of ways in which 1 green ball can be 6.
(a) There are 6 bowlers, 3 wicket keepers and 11 batsmen
put = 6 in all. The number of ways in which a team of 4
The number of ways in which two green balls can bowlers, 2 wicketkeepers and 5 batsmen can be chosen.
be put such that the boxes are consecutive = 5 (i.e., = 6C4 × 3C2 × 11C5
(1, 2), (2, 3), (3, 4), (4, 5), (5, 6)) = 6C2 × 3C1 × 11C5
Similarly, the number of ways in which three green 6 × 5 3 11 × 10 × 9 × 8 × 7
balls can be put = 4 (i.e., (1, 2, 3), (2, 3, 4), (3, 4, 5), = × × = 20790.
2×1 1 5× 4× 3× 2×1
(4, 5, 6))
... ... ... ... ... ... ... 7.
(c) Four
and so on.
\ Total number of ways of doing this
= 6 + 5 + 4 + 3 + 2 + 1 = 21.
2. (a) Since every edge connects a pair of points, the given
12 points have to be joined using lines
We may have minimum number of edges if all the
12 points are collinear
No. of edges in this particular case = 12 – 1 = 11
Maximum number of edges are possible when
all the 12 points are non-collinear. In this particular
case, number of edges is equal to number of different
straight lines that can be formed using 12 points,
12 × 11 18 × 17
which is equal to 12C2 = = 66 (a) 18C2 =
8. = 153.
2 2

Therefore, following inequality holds for e: 7! 7 × 6 × 5!


(b) 7C2 =
9. = = 21.
11 ≤ e ≤ 66. 2! 5! 5! × 2!

3.
(b) The four constants can be written in 4! ways, i.e., 24 (a) 10C4 + 10C5 = 11C5 = 462.
10.
ways. The three vowels can be written in 3! ways, 11.
(c) There are six ways
i.e., 6 ways. Since no two vowels can come together,
therefore vowels can be inserted in any three places
out of the five places available, such as, P R M
S, i.e., in 5C3 ways, i.e., 10 ways. Total number of
arrangements required = 24 × 6 × 10 = 1440.

(b) 4C3 × 4C2 + 4C2 × 4C3 = 4 × 6 + 4 × 6 = 48.


4.

5.
(b) The possible ways are as follows:
AHGFX
(i) 1 red ball out of the three and 5 blue balls out of the
seven AHEFX
(ii) 2 red balls out of the three and 4 blue balls out of AHEDX
the seven
ABEFX

\ Total number of ways in which a random sample
of six balls can be drwan ABEDX
= 3C1 × 7C5 + 3C2 × 7C4 = 168. ABCDX

Chapter_31.indd 18 6/5/2015 4:48:15 PM


Permutations and Combinations 31.19

12.
(b) Word ‘AGAIN’ has letters ‘A, A, G, I, N’. 18.
(d) Out of 10 boxes we have to choose only 5 boxes
because the balls are identical and the boxes are also
When letter start with A, then number of ways
identical
= 4! = 24
10!
\ Required number of ways = 10C5 = = 252.
When letter start with G, then number of ways (5!) 2
4!
= = 12 19.
(b) Required number of ways = 2 × 2 = 4
2!

When letter start with I, then number of ways


4!
= = 12
2!
The next 49th word is NAAGI and the 50th word
is NAAIG.

(d) \ Required number of ways = 34 × 23 × 51


13.

       = 81 × 8 × 5 = 3240.
[Since B1 and B2 interchange their position, also and
G2 interchange their position.]
14.
(c) Required number of ways = 38.
(a) Required number of ways = 12C1 × 18C1 × 9C1
20.
15.
(a) The total six digit telephone numbers in which zero
cannot initiate the number = 12 × 18 × 9
= 9 × 10 × 10 × 10 × 10 × 10 = 1944.
= 900000
(b) Required number of handshakes = 12C2
21.
Total-six digit telephone numbers in which zero
cannot initiate the number and no digit is repeated 12 × 11
= = 66.
2 ×1
= 9 × 9 × 8 × 7 × 6 × 5
22.
(b)
= 136080
1. If one green ball in a box, then number of ways = 6
\ Required six-digit telephone numbers if atleast 2. If two green balls in a box, then number of ways = 5
one digit is repeated = 900000 – 136080 = 763920.
3. If three green balls in a box, then number of ways = 4
16.
(d) Here balls are of different colours and boxes are of 4. If four green balls in a box, then number of ways = 3
different sizes 5. If five green balls in a box, then number of ways = 2
Let, n = 8, r = 3 6. If six green balls in a box, then number of ways = 1
\ Total number of ways = 6 + 5 + 4 + 3 + 2 + 1 = 21.
\ Required number of ways
23.
(d) A man invite the friends
n! × n–1Cr–1 = 8! × 7C2
= 5C1 + 5C2 + 5C3 + 5C4 + 5C5
= 40320 × 21
= 25 – 1 = 31 ways
 = 846720.
His wife invite the friends
17.
(a) The total number of combinations which can be = 4C1 + 4C2 + 4C3 + 4C4
formed of 5 different green dyes, taking one or more = 24 – 1 = 15 ways
of them is 25 – 1 = 31
\ Total number of ways = 31 + 15 = 46.
Similarly, by taking one or more of 4 different blue
(a) \ Required number of ways = 104.
24.
dyes 24 – 1 = 15 and 3 different red dyes is 23 = 8
25.
(a) In a number lock system, we can consider 0 as the
\ The required number of combinations beginning
= 31 × 15 × 18 = 3720. \ Required number of ways = 10 × 9 × 8 = 720.

Chapter_31.indd 19 6/5/2015 4:48:16 PM


31.20 Chapter 31

(d) Total number of ways = 43 × 53


26. Case II: If we do not include the oldest, then number
= 64 × 125 9×8×7×6
of ways = 9 C5 = = 126
4 × 3 × 2 ×1
= 8000.
\ Total number of ways = 70 + 126 = 196.
27.
(d) Let the age’s of three children be x1, x2 and x3.
33.
(b) Case I: When 4 persons in tennis and rest in theatre
x + x2 + x3 20  26 + x3  Number of ways = 11C4 × 7C7
Then, 1 =  
3 100  3 
11 × 10 × 9 × 8
 = × 1 = 330
x1 + x2 + x3 26 + x3 4× 3× 2
⇒   = (1)
3 10 Case II: When 5 persons in tennis and rest in theatre,
Also,    M + x1 = 39 (2) number of ways = 11C5 × 6C6

From Eqs. (1) and (2), we connot determine the 11 × 10 × 9 × 8 × 7


= × 1 = 462
value of x2. 5 × 4 × 3 × 2 ×1
Case III: When 6 persons in tennis and rest in theatre
28.
(a) Case I: If Mr Y is a member, then Mrs X is refuse to
serve a member. Number of ways = 11C6 × 5C5 = 462

Number of ways = 7C3 × 6C3 = 35 × 20 Case IV: When 7 persons in tennis and rest in theatre.

= 700 Number of ways = 11C7 × 4C4 = 330


\ Total number of ways = 2(330 + 462)
Case II: If Mr Y is not a member, then Mrs X may be
a member, then = 2(792) = 1584.

number of ways = 8C3 × 6C4 34.


(b) \ Required number of ways

= 56 × 15 = 8C3 × 4C2 = 56 × 6 = 336

= 840
35.
(c) Required number of ways
\ Total number of ways = 700 + 840 = 9–2C5–2 = 7C3
= 1540 ways.
7×6×5
= = 35 ways.
29.
(b) Case I: Choose four questions from first five questions 3× 2

= 5C4 × 8C6 36.


(d) Total number of seats
= 5 × 28 = 140 = 1 grandfather + 5 sons and daughters
Case II: Choose five questions from first five + 8 grandchildren
questions = 5C5 × 8C5 = 1 × 56 = 56 = 14
\ Total number of ways = 140 + 56 = 196. The grandchildren can occupy the 4 seats on either
side of the table in 8P4 × 4! = 8!
(c) Required number of ways = 22–4–2C11–2
30. The grandfather can occupy a seat in 4 ways (i.e.,
= 16C9. S6, S7, S8, S9)
And, the remaining seats can be occupied in 5! = 5
31.
(d) Total possible answers = 34 × 23 × 51 × 4 × 3 × 2 = 120 ways (5 seat for sons and daughters)
= 81 × 8 × 5 = 3240. Hence, the total number of required ways
32.
(b) Case I: If it includes the oldest, then we exclude the = 8! × 480 = 19353600.
youngest. The number of ways 37.
(d) The number of times the father would go to the
= 10–1–1C5–1 = 8C4 zoological garden = Number of ways of selection of 3
children taken at a time
8×7×6×5 8 × 7 × 6! × 5!
= = 70 =
= 8
C3 = 56
4 × 3 × 2 ×1 5! × 3 × 2

Chapter_31.indd 20 6/5/2015 4:48:16 PM


Permutations and Combinations 31.21

Number of times a child will go to the zoological Now three books can be chosen in the following ways
garden = Number of times he is accompanied by two He borrows chemistry part II or he does not borrow
other 1 × 7C2 = 21 chemistry part II
⇒ Number of times a child will not go to the If chemistry part II is borrowed then chemistry
zoological garden = 56 – 21 = 35. part I will also be borrowed. Hence the third book can
be chosen out of remaining 6 books in 6 ways
38.
(d) Each man gives the votes for any of the four
If chemistry part II is not borrowed then three
candidates
books are chosen in 7C3 ways i.e., 35 ways
\ Total number of ways = 4 × 4 × 4 × 4 × 4 \ Required no. of ways = 6 + 35 = 41.
= 1024. 45. (b) 100, 105, 110, … 995
39.
(a) Firstly we fix the alternate position of men in a round The above numbers are in AP.
table is (6 – 1)! ways = 5! We have the following formula,
In out of six positions 5 women can be seated in tn = a + (n – 1)d
6P = 6! Here we have to find n and
5

Required no. of ways = 6! × 5! tn = 995, a = 100 and d = 5 are given


Now,
40.
(c) Let the no. of points be n 995 = 100 + (n – 1)5
According to the question, 900
\ n = = 180.
n
C2 = 10 5

n 6×5
or, = 10 (c) Required no. of ways = 6 C4 =
46. = 15 ways.
2 n−2 2 ×1

47.
(c) Number of arrangements
n(n − 1) 6! 5!
or, =0 = × = 180 × 60 = 10800
2 2! × 2! 2!
or, n2 – n = 20 48.
(b) 45 tickets from one side and 45 from the opposite
or, n = 5, –4 side.

\ n = 5 (Q –4 is not possible) 49.


(d) The given word = SERIES, this contains 2S, 2E and
rest are distinct
41.
(c) If there are ‘m’ horizontal and ‘n’ vertical lines, then The number of ways of selecting the 4 L and the
the no. of different rectangles formed are given by number of arrangements are as follows:
m
C2 × n C2 . 1. 4 L are distinct = S, E, R, I = 4! = 24
2. 2 L are same and rest are distinct
Here, m = 7 and, n = 6
= SSRI, SSRE, SSIE, EERI, EERS and EEIS
\ Required no. of parallelograms = 7 C2 × 6C2 4!
= × 6 = 72
2!
7 6 3. Two are same of one kind and two are same of the
= ×
2 7−2 2 6−2
other kind
4!
= SSEE = 6
7×6×6×5 2!  2!
= = 315.
2×2 Total ways = 24 + 72 + 6 = 102.
42.
(b) With 1, 3, 5, 7; 4! = 24 numbers can be formed 50.
(a) Case 1 = Number of ways of inviting friends, when no
between 1,000 and 10,000, i.e., all four-digit numbers. girls is invited = 7C4 × 4C6 = 35 ways
43. (a) One principal can be appointed in 36 ways Case 2 = Number of ways of inviting friends, when 1
One vice-principal appointed in remaining 35 ways girl is invited = 7C4 × 4C1 = 140 ways
\ Total no. of ways = 36 × 35 = 1260. Case 3 = Number of ways of inviting the friends,
44. (d) It is clear that out of these eight books two books are when two girls are invited
of chemistry = 7C4 × 4C2 = 210 ways

Chapter_31.indd 21 6/5/2015 4:48:16 PM


31.22 Chapter 31

Case 4 = Number of ways of inviting the friends, In any n!, number, of 10 = number of 5
when three girls are invited \ Higher power of 10 that will divide 100! is
= 7C4 × 4C3 = 140 ways 100  100 
Case 5 = Number of ways of inviting the friends,  5  +  2  = 20 + 4 = 24.
  5 
when four girls are invited
58. (b) Number divisible by 5 will end with either ‘0’ or ‘5’
= 7C4 × 4C4 = 35 ways Total numbers ending with ‘0’ = 4 × 3 × 1 = 12
So, total ways = 35 + 140 + 210 + 140 + 35 Total of numbers ending with ‘5’ = 3 × 3 × 1 = 9
= 560 ways. \ Total such numbers = 21.
51.
(d) Number of ways of selecting exactly one ruling party 59. (b) Six pentagons ABCDE, ABCDF, ABCEF, ABDEF,
member ACDEF and BCDEF can be formed. All these six
pentagons will have the same area.
= 6C1 = 6 ways
60. (d) Since, 3 particular flowers are always together. So,
Number of ways of selecting remaining 3 members
considering these 3 flowers as 1, total number of
from opposition party
flowers is 8. These 8 flowers can be arranged in 8!
= 5C3 = 10 ways ways. Remaining 3 flowers (which we considered as
1) can be arranged in 3! ways. Hence, total number of
Total ways = 6 × 10 = 60 ways.
ways = 8! × 3! = 241920.
52.
(b) Selecting one opposition party member 61. (a) Required number of ways
= 5C1 × 6C3 = 5 × 20 = 100 ways
=3 C13C11C1 + 3 C13C2 + 3 C23C1 + 3 C21C1 + 3 C21C1
Selecting two opposition party members = 9 + 9 + 9 + 3 + 3 = 33
= 5C2 × 6C2 =10 × 15 = 150 ways 62. (a) From 4 bowlers, 2 wicket keepers and 10 players,
Selecting three opposition party members total number of ways of required selection
= 4C32C110C7 + 4C4 2C110C6 + 4C32C210C6 + 4C4 2C210C5
= 5C3 × 6C1 = 10 × 6 = 60 ways
= 4 × 2 × 120 + 1 × 2 × 210 + 4 × 1 × 201 + 1 × 1 × 252
Selecting all four opposition party members
= 960 + 420 + 840 + 252
= 5C4 = 5 ways
= 2472.
Total ways = 100 + 150 + 60 + 5 = 315 ways.
63. (b) Student can choose 4 questions from first 5 questions,
53. (d) Suppose total management book be 1 unit, then there the number of ways = 5C4 = 5
are total 8 books, which are arrange in 8! ways Similarly, student can choose 6 questions, form
Also, 3 management books are arranged in 3! ways remaining 8 questions, then number of ways 8C6
\ Number of ways = 3! × 8! 8× 7
= 28
 = 6 × 40320 2
 = 241920. ∴ Total number of choices available = 5 × 28 = 140.
64. (a) There are ten places, where the boys can be seated.
54. (a) Total number of book = 10 Now, a2 cannot come at 9th and 10th positions
\ Number of ways arrangement = 10! otherwise condition is not fulfilled
 = 3628800. ∴  Total number of ways the boys can be arranged in
55.
(d) X and Y are two students and they 8 places = 8!
X _ _ Y = 2 ways can be arranged in the given ways Similarly for a5, he cannot be seated in first place and at
_ X _ Y = 1 the 10th place, then possible ways of arrangement = 8!
Y _ X _ = 1 Now, a7 cannot be seated on 1st and 2nd places, then
possible number of ways = 8!
Y _ _ X = 2 ways
Total no. of ways = 2 + 1 + 1 + 2 = 6 ways. ∴  Total number of ways
= 8!+ 8!+ 8!
56.
(d) Number of ways
7 = 3 × 8!
C4 × 4C2 + 7C3 × 4C3 × 7C4 + 4C1 = 371.
3 × 3 × 8! 9 × 8! 9!
= = =
57. (b) 100! = 1 × 2 × 3 × 4 × 5 × ... × 100 3 3 3
also, 10 = 2 × 5 Q now

Chapter_31.indd 22 6/5/2015 4:48:18 PM


Permutations and Combinations 31.23

65. (b) According to the question, 4 teams have to chosen 69. (d) We have no girls together, let us first arrange the 5
form 8 players boys and after that we can arrange the girls in the
Now, number of ways for choosing 4 teams form 8 space between the boys
8× 7× 6×5 Number of ways of arranging the boys around a circle
players = 8C4 = = 70
4 × 3 × 2 ×1 = [5 − 1]! = 24
Now, as there have to 2 players in each team, then Number of ways of arranging the girls would be by
70 70 placing them in the 5 spaces that are formed between
possible number of ways are = = 35 ways.
2! 2 the boys. This can be done in 5 P3 ways = 60 ways
66. (a) Let the radius of smaller circle be r cm and radius of Total arrangements = 24 × 60 = 1440.
larger circle = R cm.
70. (d) 3 ladies can be appointed out of 7 ladies as 1
is not includdid in 7C3 ways
Also 3 gentlemen can be appointed out of 6 gentlemen
6 cm

as 1 is already member in 6C3 ways


Hence, the required number of ways
= 7C3 × 6C3 = 700
Difference between the area = 116π
p R2 + p r 2 =
116p
71. (d) Total no. of seats = 1 grandfather
R2 + r 2 =
116 (1)
+ 5 sons and daughters + 8 grand children
Also, the difference between their centres = 6 cm The grandchildren can occupy the 4 seats on
i.e.,  R – r = 6
either side of table in 4! = 24 ways
or,  R = (6 + r)(2)
The grandfather can occupy a seat in (5 − 1) = 4
From Eqs. (1) and (2),
ways (4 gaps between 5 sons and daughters)
(6 + r ) 2 + r 2 = 116
And the remaning seat can be occupied in
⇒ 36 + r 2 + 12r + r 2 = 116 5! = 5 × 4 × 3 × 2 = 120 ways
⇒ 2r 2 + 12r + 36 − 106 = 0 (5 seats for sons and daughters)
⇒ 2r 2 + 12r − 80 = 0 Hence, the total number of required ways
⇒ r 2 + 6r − 40 = 0 = 8!× 480 = 193536.
⇒ r 2 + 10r − 4r − 40 = 0 72. (a) Case I: When Y is a member.
⇒ (r + 10)(r − 4) = 0 Then no. of ways = 6C3 × 7C3 = 20 × 35 = 700
∴ r = −10, 4 (radius cannot be negative) Case II: When Y is not a member.
∴ Radius of smaller circle = 4 cm Then, no. of ways = 6C4 × 8C3 = 15 × 56 = 840
∴  Total no. of ways = 700 + 840 = 1540.
and Radius of larger circle = 4 + 6 = 10 cm.
73. (a) Let the number of teams participating be n. Then, we
have
67. (c) The lock contains 4 rings, so any place can have
n(n − 1)
number from 0 to 9 = 153
2
∴  Total ways to getting a code = 104
⇒ n(n − 1) = 153 × 2
∴  So, maximum attempts before getting the right
number is 1 less than total ways i.e., 104 - 1. ⇒ n(n − 1) = 18 × 17
68. (a) There are total of 10 persons of which P1 is always ⇒ n = 18.

selected and hence P4 and P5 are not be included in 74. (c) When the first two digits of the number is 41, then
the arrangement remaining four digits can be arranged in 8P4 ways
Therefore, persons left = 10 - 3 = 7 (as P1 is already Similarly, for 42, 46, 62, and 64, the remaining four
selected and P4 and P5 are not selected) digits can be arranged in 8P4 ways
∴  To select 4 persons from 7 persons total ways Hence, total number of numbers having all six digits
= 7C4 distinct = 5 × 8P4
Also, these 5 persons can be arranged in 5! ways 8!
= 5× = 5 × 8 × 7 × 6 × 5 = 8400.
∴  Required ways of arrangement = 7C4 × 5! 4!

Chapter_31.indd 23 6/5/2015 4:48:22 PM


31.24 Chapter 31

75. (b) The three pairs of teachers can be arranged in 3! 20 × 19


ways but every pair can be arranged in 2! ways 77. (b) Required number of points = 20C2 = = 190.
2 ×1
Thus, required number of ways
78. (d) From 8 books, he can choose 2 book out of 6 and 1
= 6C2 6C5 + 6C36C4 + 6C4 + 6C3 + 6C56C2 book out of 2 Chemistry books

6 6 6 6
= 2( C2 C5number
76. (a) Required + C3 C4of
) ways
∴  Total number of ways
= 26(6 C62 6C1 + 66C366C2 )
= C2 C5 + C3 C4 + 6C4 + 6C3 + 6C56C2
= 6C2 6C2 = 15 × 2 = 30.
 6×5 6×5× 4 6×5 
= 22(6 C2 6C
= × 6 ++ 6C 6C )× 
 2 × 1 5 3 ×32 ×41 2 × 1 
6
= 22((90C+2 300
=
6 6
C1 )+ C3 C2 ) 6
79. (a) The maximun sum would occur when we take the
= 2 × 390
6 × 5= 780. 6 × 5 × 4
6×5 sum of all the positive terms of the series. The
= 2 ×6+ ×  series 25, 24.5, 24, 233.5, 23,...,1, 0.5, 0 has
 2 ×1 3 × 2 ×1 2 ×1 
51 terms. The sum of the series would be given
= 2(90 + 300)
by n × average = 51 × 12.5 = 637.5.
= 2 × 390 = 780.

Difficulty Level-2

1. (c) The given digits are 4, 6, 0, 6, 7, 4, 6 which are 7 in = (6C1 × 12C2) + (6C2 × 12C1) + 6C3
all. So, numbers greater than a million can be formed
 12 × 11   6 × 5  6×5× 4
by using all the digits =  6 × + × 12  +
 2   2  6
So, 4 occurs twice, 6 occurs thrice while 0 and 7
occur once = 396 + 180 + 20 = 596.


\ Total number of arrangements 4. (d) Total number of numbers from 1000 to 9999 = 9000
7! 7×6×5× 4 Number of numbers having 4 different digits
= = = 420
3! 2! 2  = 9 × 9P3 = 4536
But these also include the numbers starting with ⇒ Required number = 90000 – 4536 = 4,464.
0 which is not greater than a million. So, numbers
5.
(b) 7! + 8! + 9! + ... + 100! is divisible by 7
starting with 0 have the remaining places filled with
the remaining digits 6! + 5! + 4! + 3! + 2! + 1!
6! 6×5× 4 = 720 + 120 + 24 + 6 + 2 + 1 = 873
Number of such numbers = = = 60
2! 3! 2 873, when divided by 7, leaves a remainder of 5.
\ Numbers greater than a million = 420 – 60 = 360. 6.
(b) The number of ways of forming the groups
= 5C2 × 3C1 = 30
2.
(a) If Mr Y is selected, then 2 more students have to be
selected out of remaining 7 Numbers of each group can be arranged among
members themselves in 3! ways = 6 ways
7×6
Number of ways = 7C2 = = 21 ways Therefore, the number of photographs
2
If Mr Y is not selected, then Mr X cannot be = 30 × 6 = 180.
selected. Then, 3 students have to be selected out of 6 7.
(d) If all are of equal height, number of handshakes
students, which can be done in = 40C2
6 6×5× 4 If all are of different heights, number of handshakes
C3 ways = = 20 ways
3× 2 =0
\ Required number of ways = 21 + 20 = 41 ways. Difference = 40C2 – 0 = 40C2.
3. (d) Number of black balls = 6 8. (d) Number of parallelograms = 8C2 × 6C2 = 420
Number of non-black balls = 12 Rhombuses of side 1 cm = (6 – 1) × (8 – 1) = 35
Number of ways of drawing at least 1 black ball = Rhombuses of side 2 cm = (6 – 2) × (8 – 2) = 24
Number of ways of drawing 1 black ball + Number of Rhombuses of side 3 cm = (6 – 3) × (8 – 3) = 15
ways of drawing 2 black balls + Number of ways of
Rhombuses of side 4 cm = (6 – 4) × (8 – 4) = 8
drawing 3 black balls

Chapter_31.indd 24 6/5/2015 4:48:24 PM


Permutations and Combinations 31.25

Rhombuses of side 5 cm = (6 – 5) × (8 – 5) = 3 ⇒ n2 – 11n + 8n – 88 = 0


Total = 85 ⇒ n (n – 11) + 8 (n – 11) = 0
Thus, parallelograms which are not rhombuses ⇒ (n – 11) (n + 8) = 0
= 420 – 85 = 335. ⇒ n = 11.
9.
(b) Total number of ways of sitting of 4 particular guests 16. (c) The total number of words that can be formed with
on a paticular side = 9P4 =
9! five letters out of the ten given letters = 105 = 100000
5! The total number of words that can be formed with
Similarly, total number of ways of sitting of 3 five distinct letters
9! = 10 × 9 × 8 × 7 × 6 = 30240
another guests on the other side = 9P3 =
6! \ The total number of words in which atleast one
After these total two operations, the remaining letter is repeated = 100000 – 30240 = 69760.
11 guests can sit on remaining 11 seats in 11!
17. (b) All permutations formed with 1, 2, 3, 4, 5 (sum = 15)
ways. Hence, total number of seating arrangements
will be divisible by 3
9! 9! There are 5! = 120 such permutations. Such
= × × 11!
5! 6! numbers can also be formed using 0 and 1, 2, 4, 5.
10. (b) 4! × 2 ways, i.e., 24 × 2 = 48 ways. There are 4 × 4! such numbers, i.e., 96. (Factor of 4 for
11. (b) Octagon is 8 sided polygon and a triangle has 3 four positions of 0 and 4! for different permutations of
vertices these four numbers)
Of these, one vertex of a triangle is selected as the \ Total of such numbers = 120 + 96 = 216
centre of the octagon in 1C1 way (c) 7C2 × 6C2 = 315.
18.
And, the other 2 vertices of a triangle can be
selected from any of the 8 sides of the octagon in 8C2 [1 × 3 × 5 ... (2n − 1)][2 × 4 × 6 ... 2n]
19.
(a)
ways, [2 × 4 × 6 ... 2n]2
\ The total number of ways = 1C1 × 8C2 (2n)!
8×7 = n
= 1. = 28. [2 (1, 2, 3, ... n)]2
2 ×1
(2n)!
12. (c) If the couples want to sit together, there are 5 pairs = .
[2n × n!]2
5 pairs can sit around a table in (5–1)! ways
20.
(c) The four-digit number formed from the digits 1, 2, 3,
Each couple can sit together in 2 ways 4, 5 will be divisible by 4 if the last two digits are 12
\ Number of ways is (4!)25 or 24 or 32 or 44 or 52. The first two can be chosen in
= 5 × 4 = 20 ways and also as 11, 22, 33, 44, 55, i.e.,
(a) 7C3 × 6C2 + 7C4 × 6C1 + 7C5 × 6C0
13.
25 ways in all. Hence these 125 four-digit numbers
= 7C3 × 6C2 + 7C3 × 6 + 7C2 × 1 are divisible by 4.
7×6×5 6×5 7×6×5 7× 6 6!
= × + ×6+ 21.
(b) = 15.
3 × 2 ×1 2 ×1 3 × 2 ×1 2×1 2! × 4!
= 525 + 210 + 21 = 756.
(b) 5 bowlers can be selected out of 8 cm 8C5 ways =
22.
14. (d) Treating (AIE), i.e., all the vowels together as one
letter. Therefore, the word capital, i.e., TRNR (AIE) 8× 7× 6
= 56 ways
5! 3 × 2 ×1
can be arranged in ways. (Q R is repeated)
2 5 batsman can be selected out of 6 in 6C5 ways = 6
Since (AIE) can also be arranged in 3! ways, ways
therefore, required number of ways
One wicket keeper can be selected in 3C1 = 3 ways
5! 120 × 6
= × 3! = = 360. Hence, total number of ways = 3 × 6 × 56
2 2
= 1,008 ways.
15. (a) Let the number of sides be n
n 23.
(a) 11 × 10 × 9 × 8 = 7920.
\ C2 – n = 44, n > 0
n (n − 1) 24.
(c) Total number of passwords using all alphabets total
⇒ − n = 44 number of passwords using no symmetric alphabets
2
⇒ n2 – 3n – 88 = 0 = (26 × 25 × 24) – (15 × 14 × 13) = 12870.

Chapter_31.indd 25 6/5/2015 4:48:25 PM


31.26 Chapter 31

25.
(d) A block square can be chosen in 32 ways. Once a 33. (b) From a certain station, there will be a ticket for each
black square is there, you cannot choose the 8 white of the other 9 stations and there are 10 stations on the
squares in its row or column, so the number of white railway line.
squares available = 24. \ The number of different journey tickets
No. of ways = 32 × 24 = 768. = 9 × 10 = 90
Hence, option (b).
26. (c) One can draw red ball in first draw in 5 ways and
black or white ball in second draw in 6 + 4 = 10 ways, 34. (d) The three vowels in ABACUS are A, A and U. These
black ball in first draw in 6 and red or white ball in 3!
three can be arranged among themselves in =3
second draw in 9 ways and white ball in first draw ways 2!
in 4 and red or black ball in second draw in 11 ways. As the three vowels are to appear together, we
Hence, total number of ways consider them as one entity. Thus we have four letters;
= 5 × 10 + 6 × 9 + 4 × 11 = 148. (AAU), B, C and S to be arranged. This can be done in
27. (b) The maximum routes from A to F are listed below. 4! ways
4!  3!
\ Required number of ways =
2
Hence, option (d).
35. (a) 7C4 × 5C2 + 7C5 × 5C1 + 7C6
= 350 + 105 + 7 = 462.
36. (b) (a) From 890 to 899 (except 898) we have 8 in the
hundreds place and 9 in tens place, i.e., 9 three-
digit numbers.
(b) With 8 in the tens place and 9 in the units place we
(1) ABDF (2) ACEF (3) ABF have 8 three-digit numbers (189, 289, etc except
(4) ABEF (5) ACDF (6) BCDEF 889)
(c) If 8 is not any one of the three-digits, we have
(7) ACDEF (8) ABDEF (9) ABCDF
(1 to 9 except 8, 0 to 9 in the hundreds, tens and
(10) ABCEF. the units digits) i.e.,
28.
(b) Seven girls can sit around a circular table in (7 – 1)! 8 × 9 × 9 = 648 three-digit numbers
ways. Now six gaps can be chosen in 7C6 ways in \ Total is 9 + 8 + 648 = 665.
which boys can fit in then the boys can be arranged in 37. (d) The vice captain and the coach can be arranged in
6! ways. the middle position of the first row in 2 ways. The
Hence, total number of ways = 6! 7C6 × 6! = 6! × 7! position of the captain can be fixed thus
Two defenders out of 3 can be selected in 3C2 ways
29.
(a) Total number of ways
3 captain 2
= 2 × 3 × 4 × 3 × 2 × 1
= 144 ways. ab c
30. (a) 6! = 720, which when divided by 13 gives a remainder 4 e coach vice captain d 1
5. The remainder when given expression is divided
by 13 is the remainder obtained when (5)7!×13333 is The goalkeeper can be placed at positions 1, 2, 3 or 4.
divided by 13. Suppose he is at position 3
(25)7!×13333)/2 = (26 – 1)even number = 26K + 1  Two defenders out of three can be placed at
positions (a, b) or (2, c) or (4, e) or (1, d) and rest of
So, remainder when (25)(7!×13333)/2 is divided by the players can be placed at 6 positions in 6! ways.
13 is 1. The two defenders can be arranged among themselves
31.
(d) Number of ways of arranging vowels 3!/2! in 2 ways

Now, take vowels as a unit, then total no. of Thus, the total number of ways of arranging the
arrangement will be = 4! × 3!/ 2! team and coach with the goalkeeper at position 3 is
32.
(c) No. of ways of forming numbers without ‘o’ 2 × 3C2 × 2 × 4 × 6!
Suppose the goalkeeper is at position 1
= 5 × 4 × 3 × 2 × 1 = 120
No. of ways of forming numbers with ‘o’ Two defenders can be placed at positions (3, a) or
= 4 × 4 × 3 × 2 × 1 = 96 (a, b) or (c, 2) or (4, e)
[This time only 0, 1, 2, 4 and 5 digits will be considered If they are placed at (c, 2) or (4, e), rest of the
so as to make the number divisible by 3] players can be arranged in 6! ways

Chapter_31.indd 26 6/5/2015 4:48:25 PM


Permutations and Combinations 31.27


Thus total number of arrangements would be 2 × 86 × 85 × 84 × 83 × 82 × 81
3 =
C2 × 2 × 2 × 6!
6 × 5 × 4 × 3 × 2 ×1
If the two defenders are placed at (3, a) or (a, b),
then the third defender cannot be placed at b (or 3) = 47,01,55,077
as exactly two defenders can be placed together. The
third defender would have to be placed at one of the Note: 
positions from c, 2 4, e or d, i.e., in 5 ways. Thus,
86 × 85 × ... × 81 has only one zero at the end and 1
if the goal keeper were at position 1, the number
× 2 × ... × 6 will also have only one zero at the end.
of arrangements would be (2 × 3C2 × 2 × 2 × 6!) +
Hence, the answer will not end with 0.
(2 × 3C2 × 2 × 2 × 5 × 5!)
n n n
The number of arrangements would be same if the Pr n!
goalkeeper were at positions 2 or 4
40.
(b) ∑ r!
= ∑ r !(n − r )!
r =1 r =1
\ Total number of possible arrangements would be n
2 × 3C2 × 2 × 4 × 6! + 3 × [(2 × 3C2 × 2 × 2 × 6!) + (2
× 3C2 × 2 × 2 × 5 × 5!)]
= ∑ n Cr
r =1
= 2 × 3C2 × 2[4 × 6! + 3 × 2 × 6! + 3 × 2 × 5 × 5!)] = C1  nC2  nC3  ...  nCn 1  nCn
n
(1)

= 12[2880 + 4320 + 3600] = 1,29,600. We know that,
38.
(d) A can be seated in 8 ways and F in 2 ways n
C0  nC1  nC2  nC3  ...  nCn 1  nCn = 2n
\ A and F can together be seated in 8 × 2 = 16 ways ⇒ n C1  nC2  nC3  ...  nCn 1  nC1
Now, B and D never sit together
= 2n − nC0 =2n − 1.
For B and D:
n2
Case 1: B sits adjacent to A or F 41.
(d) C8 : n  2 P4 = 57:16

Note D cannot be adjacent to F or A as then G and H n  2! n  2!


⇒ : = 57:16
cannot sit opposite each other 8!(n  2  8)! (n  2  4)!
\ D can sit in 3 places. G can sit in 2 places and H n  2! n  6! 57
has only 1 seat ⇒  =
8!(n  6)! (n  2)! 16

\ 2 × 3 × 2 = 12 ways
n  2! 57
Case 2: B sits opposite A or F ⇒ =
8! (n  2)! 16

D can sit in 4 ways, G in 2 ways, H in 1 way 57
⇒ (n – 1) n(n + 1) (n + 2) = 8! 
\ Number of ways = 2 × 4 × 2 = 16 16
Case 3: B is neither opposite nor adjacent to A or F. B ⇒ (n – 1) n(n + 1) (n + 2) = 18 × 19 × 20 × 21
can sit in 2 ways. D can sit in 2 ways, G in 2 ways, H
in one way On solving, we get
\ n = 19.
\ Number of ways = 2 × 2 × 2 = 8
42. (b) In a mixed double of tennis game there is a pair of
The remaining 2 people can be seated in 2 ways man and woman on each ride
\ Total number of ways So, total 2 men and 2 women are in each game
= 8 × 2 × (12 + 16 +8) × 2 × 1 = 1,152. There are 9 men and 9 women
Ways to select 2 men = 9C2
39.
(d) Let the number of 100—rupee notes carried by them
be and x7 Wives of these 2 men cannot play in same game
So ways to select 2 women = 7C2
⇒ 100(x1 + x2 + x3 + x4 + x5 + x6 + x7)
In a game these 4 persons can be paired by 2 types
= 500 + 600 + 800 + 1500 + 1200 + 1600 + 1800
\ Total number of ways = 9C2 × 7C2 × 2
⇒ x1 + x2 + x3 + x4 + x5 + x6 and x7 = `80
98 76
The total number of solutions to the equation =  2
2 1 2 1
= 80+7–1C7–1 = 1512.
86! 43.
(d) Shoes can be selected = 3 ways
=
80!6! Lower wear can be selected = (4 + 3) ways = 7 ways

Chapter_31.indd 27 6/5/2015 4:48:26 PM


31.28 Chapter 31

Upper wear can be selected = (6 + 3+ 9 × 2) = 27 ways


210 1024
Jacket can be selected = 2 way Letter ‘K’ is represented by = 
29 29
He may not wear jacket = 1 ways = 3 ways
= 9 (Remainder)
\ Total possible outputs = 3 × 7 × 27 × 3
      = 1701 ways.
49.
(c) One group has 2 girls and others have 1 girl each. Total
43 number of ways in which 5 groups can be formed
44. (d) F (4, 3) = = 6.
2
45. (d) Let x be represented as pppq, then 3p + q must be a = 6C2 × 9 × 4 × 8C2 × 3 × 6C2 × 2 × 4C2
multiple of 9 8×7 6×5 4×3
i.e., (1116), (6111), (2223), (3222), (3339), (9333) = 6C2 × 9 × 4! × × ×
2 2 2
(4446), (6444), (5553), (3555),
(6669), (9666), (7776), (6777), (8883), (3888) 15 × 9! × 4! 15
= = × 9! × 3!
(9990), (9000), (3330) (6660) 16 4
Hence, there are 20 cases possible. 50.
(c) Since all the 5 cars must be adjacent, this can be done
in 4 ways
46. (d) If we consider the distance between any two cities as
paths, there were a total of 20C2 = 190 paths Further the 5 cars can be arranged amongst
themselves in 5! ways
Now, each candidate needed to visit all the cities
and then come back to the city he started from i.e., Hence, the answer is 4 × 5!.
each candidate needed to take 20 paths
51.
(c) 125! – 124! – 123! = (125 × 124!) – 124! – 123!
Let the maximum number of candidates be n
= 124 × 124! – 123!
Now, 20n < 190
= (1242 – 1) × 123!
n < 9.5
Now, last digit of (1242 – 1) is 5 and 123! has 28
Since, n is an integer, the maximum value of n is 9. zeroes. Hence, there are 29 zeroes in all.
47.
(d) There are 10 teams in the bottom group and say n 52.
(b) There are 4 cards of each denomination in a pack of 52
teams in the top group. The bottom group gets 45 cards. Each set of four cards of the same denomination
points (there are 45 matches and 1 point per match) can be distributed among 4 persons in 4! ways. Hence,
playing amongst themselves. Therefore, they should the required number of ways is (4!)13 = (24)13.
get 45 points from their matches against the top group 53.
(d) If P = 1! = 1
i.e., 45 out of the 10n points. The top group get nC2
  Then, P + 2 = 3, when divided by 2! remainder will
points from the matches among themselves. They also
be 1
get, 10n – 45 points against the bottom group, which
is half their total points. If     P = 1! + 2 × 2! = 5
n Then, P + 2 = 7 when divided by 3! remainder is still 1
\ C2 = 10n – 45
Hence,   P = 1! + (2 × 2!) + (3 + 3!) . . . + (10 + 10!)
⇒ n(n + 1) = 20n – 90
When divided by 11! leaves remainder 1
⇒ n2 – 21n + 90 =0
Alternative method:
⇒ (n – 6) (n – 15) =0 P = 1 + 2.2! + 3.3! + . . . + 10.10!
If n = 6, the top group would get nC2 + 10n – 45 = (2 – 1) 1! + (3 – 1) 2! + (4 – 1) 3! + ... (11 – 1) 10!
= nC2 + 10(6) – 45 = 30 points, or an average of 5 = 2! – 1! + 3! – 2! + . . . 11! – 10! = 11! + 1
points per team, while the bottom group would get (45 Hence, the remainder is 1.
+ 45)/10 or an average of 9. This is not possible
\ n = 15 54.
(c) Three numbers can be selected and arranged out of
10 !
The total number of teams is 10 + 15 or 25. 10 numbers in 10P3 ways = = 10 × 9 × 8. Now
7!
48.
(d) this arrangement is restricted to a given condition that
first number is always less than the second number
A B C D E F G H I J K and second number is always less than the third
0 1 2 3 4 5 6 7 8 9 10 number. Thus, three numbers can be arranged among
2 2 2 2 2 2 2 2 2 2 2
themselves in 31 ways.

Chapter_31.indd 28 6/5/2015 4:48:26 PM


Permutations and Combinations I 31.29

Hence, required number of arrangements 60.


(b)
10 × 9 × 8 61. (b) The possible ways are as follows:
= = 120 ways.
3× 2 (i) 1 red ball out of the three and 5 blue balls out of
the seven
55. (d) Total number of ways of filling the 5 boxes numbered
as (1, 2, 3, 4 and 5) with either blue or red balls = 25 = (ii) 2 red balls out of the three and 4 blue balls out of
32. Two adjacent boxes with blue can be obtained in the seven
4 ways, i.e., (12), (23), (34) and (45). Three adjacent \ Total number of ways in which a random sample
boxes with blue can be obtained in 3 ways, i.e., (123), of six balls can be drawn
(234) and (345). Four adjacent boxes with blue can be = 3C1 × 7C5 + 3C2 × 7C4 = 168.
obtained in 2 ways, i.e., (1234) and (2345). And five 62. (d) The smallest number in the series is 1000, a four digit
boxes with blue can be got in 1 way. Hence, the total number
number of ways of filling the boxes such that adjacent
The largest number in the series is 4000, the only
boxes have blue = (4 + 3 + 2 + 1) = 10
4-digit number to start with 4
Hence, the number of ways of filling up the boxes
The left most digit (thousands place) of each of the
such that no two adjacent boxes have blue = 32 – 10
four digit numbers other than 4000 can take one of the
= 22.
3 values-1 or 2 or 3
56.
(d) To construct 2 roads, three towns can be selected out The next three digits (hundreds, tens and units place)
of 4 in 4 × 3 × 2 = 24 ways. Now, if third road goes can take any of the 5 values 0 or 1or 2 or 3 or 4
from the third town to the first town, a triangle is
Hence, there are 3 × 5 × 5 × 5 or 375 numbers from
formed, and if it goes to the fourth town, a triangle is
1000 to 3999.
not formed. So, there are 24 ways to form a triangle
and 24 ways of avoiding triangle. Including 4000, there will be 376 such numbers.
63. (b) Two identical stones are to be placed on any two
57.
(d) The available digits are 0, 1, 2, ...9. The first digit can separate squares in the grid
be chosen in 9 ways (0 not acceptable), the second
∴  r = 2
digit can be accepted in 9 ways (digit repretition not
allowed). Thus the code can be made in 9 × 9 = 81 Total number of squares = 100
ways ∴  Total number of stones (n) = 100
The number of ways of selecting 2 things out of 100 is
n
Cr = 100C2 = 4950.
~--il--~---+~ 3 possibloways 64. (a) Suppose Vaibhav wrote m prime numbers
Vikram wrote down n = mC3 numbers of the form
4 possible ways pipjpk, where pi, pj and pk are the numbers written by
Vaibhav
Now, there are only 4 digits which can create Vishal wrote down n(n - 1)/2 instances of some
confusion 1, 6, 8, 9. The same can be given in the numbers
following ways Some of these were 1 (and hence not prime)
Total number of ways confusion can arise Some were of the form pi and others were of the form
= 4 × 3 = 12 pipj (and hence not prime)
Thus, required answer = 81 – 12 = 69. Each of the prime numbers (of vaibhav)were written
58. (a) At least one candidate out of (2n + 1) candidates can down by vishal a certain number of times.
be selected in (2n + 1 – 1) ways Consider one particular number, say, pi. Among the
\ 22n + 1 – 1 = 63 other m − 3 numbers, the number of ways of choosing
⇒ 22n + 1 = 64 = (2)6 2 is m −3C2
⇒ n = 2.5. Among the remaining m − 3 numbers, the number of
Since n cannot be a fraction. Hence, n = 3. ways of choosing 2 is m −3 C2
59.
(b) Each one is coded with either a single colour or unique
two-colour pair. Therefore, total number of ways But in the product ( m −1 C2 )( m − 3 C2 ) , each such pair has
been counted twice. Therefore, the number of distinct
= n + nC2
pair is,
\ Minimum number of different colours needed to
code all 20 chemicals will be 6 ( m −1 C2 )( m − 3 C2 )
\ 6 + 6C2 = 6 + 15 = 21. 2

Chapter_31.indd 29 6/5/2015 4:48:28 PM


31.30 I Chapter 31

Vishal writes down so many numbers for each of the But -6 is not possible as n > 0
m primes of Vaibhav ∴  Total number of participants = 11 + 2 = 13.
∴  Number of instances of primes that vishal writes are
68. (c) Given, 100A + 10B +C = A! + B! + C!
m( m −1 C2 )( m − 3 C2 ) From the options, value of B will be less than 7, because
2 7! = 5040 which is a 4 digit number. Hence, B can be
either 4 or 2. Again value of A and C has to be any of
m (m − 1)(m − 2) (m − 3)(m − 4)
⇒ × × = 90 6! = 720 or = 144 because any other combination will
2 2 2 produce a three digit number. Hence, required number
⇒ m(m − 1)(m − 2)(m − 3)(m − 4) = (90)(8) will be 145 as 1! + 4! + 5!= 1 + 24 + 120 = 145.
⇒ m(m − 1)(m − 2)(m − 3)(m − 4) = (6)(5)(4)(3)(2) 69. (d) The number required is greater than 999, but less than
⇒ m = 6. and equal to 4000
Now out of given digits, 0, 1, 2, 3, and 4 form a
65. (a) There are 4 cases, number greater than 999 and less the 4000.
I:  Number of words using 3 Ts = 1 The digit at thousands place can be selected in 3 ways
II:  Number of words using 2 Ts = 3 × 3 = 9 (As 0 and 4 cannot be taken)
As the third letter may be anyone of A, C and R and The digit at hundreds place can be selected in 5 ways
can be placed in any one of the positions, e.g., CTT,
The digit at tens place can be selected in 5 ways
TCT or TTC
The digit at units can be selected in 5 ways
III.  Number of words using 2 As = 3 × 3 = 9
IV. Number of words with all 3 letters distinct ∴  Total required number of ways
= 3 × 5 × 5 × 5 = 375
= 4 P=
3 24
Since, 4000 is also one of the required number.
∴  Total number of words = 24 + 9 + 9 + 1 = 43 Therefore, total number of ways = 375 + 1 = 376.
Since the 3 words TTT, ATA and TAT look the same in 70. (a) Total number of persons seating at the circular table
a mirror, therefore they are striked out = 9 + 1 = 10
Thus, the number of words left = 43 − 3 = 40 words. Considering CEO, X and Y, remaining 7 persons can
66. (d) The first digit can be chosen from 3, 5 and 7 in 3 ways. be seated in 7! Ways. X and Y can sit on either side of
the host CEO by 2 ways. Hence, total number of ways
After choosing the first digit, the remaining three
= 7! × 2 = 10080.
digits can be chosen from the remaining 4 numbers in
13
1 x
4
P3 = 24 ways 71. (d) ∑ =
n=1 n 13!
∴  Total number of ways = 3 × 24 = 72. 1 1 1 1 1 1 x
⇒ + + + ... + + + =
67. (b) Let the total number of women participants be x 1 2 3 11 12 13 13!
As every participant played two games with each 13! 13! 13! 13! 13! 13!
∴x = + + + ... + + +
other, therefore, the total number of games played 1 2 3 11 2 3
among men is
All the terms in x are divisible by 11 expect 13!/11
n n! 13!
2 × C2 = 2 × = n(n − 1) = 1.2.3.4....10.12.13
2!(n − 2) 11

Number of games played with each women = 2n According to Wilson theorem,
Since, each women must have played two games with When, (p − 1)! Is divided by p, remainder will be −1
each men
 ( p − 1)! 
∴  Total matches played by woman = 2 × 2n = 4n i.e., rem   = −1
 p 
Now, according to the question,
13!
n (n − 1) – 4n = 66 Now , = 1.2.3.4 ... 10.12.13 = 10!(122)(13)
11
⇒  n2– n – 4n = 66
 13! 
⇒  n2 – 5n – 66 = 0   10!(12)(13)
⇒  (n − 11) (n + 6) = 0 ∴ rem  11  = = (−1).1.2 = −2 = 9.
 11  11
⇒  n = 11, −6  

Chapter_31.indd 30 6/5/2015 4:48:32 PM


CHAPTER

Probability 32
Introduction 3. The number of combinations of n objects taken r at a
time (r ≤ n) is denoted by C(n, r) or nCr and is defined
The word probability or chance is very frequently used in
as
day-to-day life. For example, we generally say, ‘He may
come today’ or ‘probably it may rain tomorrow’ or ‘most n n! n (n − 1) (n − 2) ... to r factors
Cr = =
probably he will get through the examination’. All these r !(n − r )! 1 × 2 × 3 ... r.
phrases involve an element of uncertainty and probability
is a concept which measures these uncertainties. The 5× 4 × 3
Illustration 1  5C3 = = 10, nC0 = 1 and nCn = 1
probability when defined in simplest way is the chance of 1× 2 × 3
occurring of a certain event when expressed quantitatively,
i.e., probability is a quantitative measure of the certainty. n
If r > , then it is better to simplify nCr as nCn – r
2
The probability has its origin in the problems dealing
with games of chance such as gambling, coin tossing, die 52
C50 = 52C52–50 = 52C2
Illustration 2
throwing and playing cards. In all these cases, the outcome
52 × 51
of a trial is uncertain. These days probability is widely used = 2 × 1
in business and economics in the field of predictions for
future. = 26.51 = 1326
The following remarks may be important for learning When, r > n, nCr = 0
this chapter on probability.
Some Important Terms and Concepts
1. Die:  A die is a small cube used in games of chance. On
its six faces, dots are marked as Random Experiment or Trial:  The performance of an
experiment is called a trial. An experiment is characterized
. .. ... : : : . : MM by the property that its observations under a given set of
Plural of die is dice. The outcome of throwing circumstances do not always lead to the same observed
(or tossing) a die is the number of dots on its uppermost outcome but rather to the different outcomes. If in an
face. An ace on a die means one dot. experiment, all the possible outcomes are known in advance
and none of the outcomes can be predicted with certainty,
2. Cards:  A pack (or deck) of playing cards has 52 cards, then such an experiment is called a random experiment.
divided into four suits: For example, tossing a coin or throwing a die are
(i) Spades gqde () (ii) Clubs fpM+h () random experiments.
(iii) Hearts iku () (iv) Diamonds b±V () Event:  The possible outcomes of a trial are called events.
Each suit has 13 cards, nine cards numbered 2 to 10, an Events are generally denoted by capital letters A, B, C and
Ace (bDdk), a King (ckn'kkg), Queen (csxe) and a Jack or so on.
Knave (xqyke). Spades and Clubs are black-faced cards
while Hearts and Diamonds are red-faced cards. The Illustration 3  (i) When a coin is tossed the outcome of
Aces, Kings, Queens and Jacks are called face cards getting a head or a tail is an event
and other cards are called number cards. The Kings, (ii) When a die is thrown the outcome of getting 1 or 2
Queens and Jacks are called court cards. or 3 or 4 or 5 or 6 is an event

Chapter_32.indd 1 6/5/2015 5:22:49 PM


32.2 Chapter 32

Sample Space:  The set of all possible outcomes of an Thus, in this case, there are 36(=62) ordered pairs.
experiment is called a sample space. We generally denote Hence, the number of exhaustive cases in the simultaneous
it by S. throw of two dice is 36
(iv) Three dice are thrown, the number of exhaustive
Illustration 4  (i) When a coin is tossed, S = {H, T} where
cases is 63, i.e., 216
H = head, T = tail
(ii) When a die is thrown, S = {1, 2, 3, 4, 5, 6} Algebra of Events
(iii) When two coins are tossed simultaneously, If A and B are two events associated with sample space S,
then
S = {HH, HT, TH, TT}
(i) A ∪ B is the event that either A or B or both occur.
Equally Likely Events:  Events are said to be equally likely (ii) A ∩ B is the event that A and B both occur
if there is no reason to expect any one in preference to other. simultaneously.
Thus, equally likely events mean outcome is as likely to
occur as any other outcome. A is the event that A does not occur.
(iii)
A ∩ B is an event of non-occurrence of both A
(iv)
Illustration 5  In throwing a die, all the six faces (1, 2, 3,
4, 5, 6) are equally likely to occur and B, i.e., none of the events A and B occurs.
Illustration 9  In a single throw of a die, let A be the event
Simple and Compound Events of getting an even number and B be the event of getting a
In the case of simple events, we consider the probability of number greater than 2. Then,
happening or non-happening of single events. A = {1, 3, 5}, B = {3, 4, 5, 6}
Illustration 6  We might be interested in finding out the \    A ∪ B = {1, 3, 4, 5, 6}
probability of drawing an ace from a pack of cards. A ∪ B is the event of getting an odd number or a number
In the case of compound events, we consider the joint greater than 2
occurrence of two or more events. A ∩ B = {3, 5}
Illustration 7  If from a bag, containing 8 red and 5 green A ∩ B is the event of getting an odd number greater than 2.
balls, two successive draws of 2 balls are made, we will A = {2, 4, 6} [Those elements of S which are not in A.]
be finding out the probability of getting 2 red balls in the
first draw and 2 green balls in the second draw. We are thus A is the event of not getting an odd number, i.e., getting an
dealing with a compound event. even number.
B = {1, 2}
Exhaustive Events:  It is the total number of all possible
outcomes of any trial. B is the event of not getting a number greater that 2, i.e.,
getting a number less than or equal to 2.
Illustration 8  (i) When a coin is tossed, either head or tail
may turn up and therefore, there are two exhaustive cases. A ∩ B = {2}
 (ii) There are six exhaustive cases or events in
A ∩ B is the event of neither getting an odd number
throwing a die.
nor a number greater than 2
(iii) If two dice are thrown simultaneously, the possible
outcomes are Mutually Exclusive Events
(1, 1) (2, 1) (3, 1) (4, 1) (5, 1) (6, 1) In an experiment, if the occurrence of an event precludes or
(1, 2) (2, 2) (3, 2) (4, 2) (5, 2) (6, 2) rules out the happening of all the other events in the same
experiment.
(1, 3) (2, 3) (3, 3) (4, 3) (5, 3) (6, 3)
(1, 4) (2, 4) (3, 4) (4, 4) (5, 4) (6, 4) Illustration 10  (i) When a coin is tossed either head or
tail will appear. Head and tail cannot appear simultaneously.
(1, 5) (2, 5) (3, 5) (4, 5) (5, 5) (6, 5)
Therefore, occurrence of a head or a tail are two mutually
(1, 6) (2, 6) (3, 6) (4, 6) (5, 6) (6, 6) exclusive events.

Chapter_32.indd 2 6/5/2015 5:22:50 PM


32.3
Probability

(ii) In throwing a die, all the 6 faces numbered 1 to 6 are Similarly,


mutually exclusive since if any one of these faces comes, B = [(1, 2), (2, 1), (1, 5), (5, 1), (3, 3), (2, 4), (4, 2),
the possibility of others in the same trial, is ruled out.
(3, 6), (6, 3), (4, 5), (5, 4), (6, 6)]
Note: C = [(1, 1), (2, 1), (1, 2)] D = [(6, 6)].
A and B are mutually exclusive events ⇔ A ∩ B = f, i.e., A We find that A ∩ B = [(1, 5), (2, 4), (3, 3), (4, 2), (5, 1),
and B are disjoint sets. (6, 6)] ≠ f
Illustration 11  (i) If the random experiment is ‘a die is Thus, A and B are not mutually exclusive.
thrown’ and A, B are the events, A:the number is less than
Similarly, A ∩ C ¹ f, A ∩ D ¹ f, B ∩ C ¹ f, B ∩ D
3; B:the number is more than 4, then A = {1, 2}, B = {5, 6}
¹ f, C ∩ D = f. Thus, C and D are mutually exclusive.
A ∩ B = f, thus A and B are mutually exclusive events.
(ii) If the random experiment is ‘a card is drawn from Probability of an Event
a well-shuffled pack of cards’ and A, B are the events The probability of an event is defined in the following two
A: the card is Black; B: the card is an ace. ways:
Since a black card can be an ace, A ∩ B ≠ f, thus A and (i) Mathematical (or a priori) definition
B are not mutually exclusive events.
(ii) Statistical (or empirical) definition
Mutually Exclusive and Exhaustive Events
Mathematical Definition of Probability:   Probability of
Events E1, E2, ..., En are called mutually exclusive and an event A, denoted as P(A), is defined as
n Number of cases favourable to A
exhaustive if E1 È E2 È ... È En = S, i.e.,  Ei = S and P(A) =
i =1 Number of possible outcomes
Ei Ç Ej = f for all i ¹ j. Thus, if an event A can happen in m ways and fails
For example, in a single throw of a die, let A be the (does not happen) in n ways and each of m + n ways is
event of getting an even number and B be event of getting equally likely to occur then the probability of happening of
odd numbers, then the event A (also called success of A) is given by
A = {2, 4, 6}, B = {1, 3, 5} m
P(A) =
A ∩ B = f, A ∪ B = {1, 2, 3, 4, 5, 6} = S m+n
\  A and B are mutually exclusive and exhaustive events. and that the probability of non-occurrence of the A (also
Illustration 12 Two dices are thrown and the sum of the called its failure) is given by
numbers which come up on the dice noted. Let us consider n
the following events: P (not A) or P ( A) =
m+n
A  :  ‘the sum is even’
If the probability of the happening of a certain event
B  :  ‘the sum is a multiple of 3’
is denoted by p and that of not happening by q, then
C  :  ‘the sum is less than 4’
m n
D  :  ‘the sum is greater than 11’ p+q= + =1
m+n m+n
Which pairs of these events are mutually exclusive?
Here, p, q are non-negative and cannot exceed unity,
Solution: There are 6 × 6 = 36 elements in the sample space
i.e., 0 ≤ p ≤ 1 and 0 ≤ q ≤ 1
(Refer to Example 2).
A is the event “the sum is even”. It means we have to When, p = 1, then the event is certain to occur.
consider those ordered pairs (x, y) in which x + y is When, p = 0, then the event is impossible. For example,
even. Thus, the probability of throwing eight with a single die is zero.
A = [(1, 1), (2, 2) (1, 3), (1, 5), (2, 4), (2, 6), (3, 1), Probability as defined above is sometimes called
      (3, 3), (3, 5), (4, 2), (4, 4), (4, 6) Priori Probability, i.e., it is determined before hand, that
   (5, 1), (5, 3), (5, 5), (6, 2), (6, 4), (6, 6)]. is, before the actual trials are made.

Chapter_32.indd 3 6/5/2015 5:22:50 PM


32.4 Chapter 32

Illustration 13  A coin is tossed once. What are all possible Illustration 16  What are the odds in favour of getting a ‘3’
outcomes? What is the probability of the coin coming up in a throw of a die? What are the odds against getting a ‘3’?
‘tails’? Solution: There is only one outcome favourable to the
Solution: The coin can come up either “heads” (H) or event “getting” a 3, the other five outcomes, namely, 1, 2, 4,
“tails” (T). Thus, the set S of all possible outcomes is S = 5, 6 are unfavourable. Thus,
{H, T} Odds in favour of getting a ‘3’
1 Number of favourable outcomes
\ P(T ) = =
2 Number of unfavourable outcomes
Illustration 14  What is the probability of getting an even 1
=  or, 1 to 5
number in a single throw of a die? 5
Solution: Clearly, a die can fall with any of its faces upper Odd against getting a ‘3’
most. The number on each of the faces is, therefore, a Number of unfavourable outcomes
=
possible outcome. Thus, there are total 6 outcomes. Since Number of favourable outcomes
there are 3 even numbers on the die, namely, 2, 4 and 6, 5
= or, 5 to 1.
3 1 1
P(even number) = =
6 2 Illustration 17  If the odds in favour of an event are 4 to 5,
Illustration 15  What is the probability of drawing a ‘king’ find the probability that it will occur.
from a well-shuffled deck of 52 cards? 4
Solution: The odds in favour of the event are . Thus,
Solution: Well-shuffled ensures equally likely outcomes. 5
There are 4 kings in a deck. Thus, P( A) 4
= , i.e., 4[1– P(A)] = 5P(A),
4 1 1 − P( A) 5
P(a king) = –
52 13 4
i.e., P(A) =
9
Odds of an Event 4
The probability that it will occur = .
Suppose, there are m outcomes favourable to a certain 9
event and n outcomes unfavourable to the event in a sample
Fundamental Theorems on Probability
space, then
odds in favour of the event Theorem 1  In a random experiment, if S is the sample
space and E is an event, then
Number of favourable outcomes m
= = (i) P(E) ≥ 0    (ii) P(f) = 0    (iii) P(S) = 1.
Number of unfavourable outcomes n
Remarks: It follows from above results that
and odds against the event
(i) probability of occurrence of an event is always
Number of unfavourable outcomes n non-negative;
= = .
Number of favourable outcome m (ii) probability of occurrence of an impossible event
If odds in favour of an event A are a:b, then the is 0;
a (iii) probability of occurrence of a sure event is 1.
probability of happening of event A = P(A) = and
a+b Theorem 2  If E and F are mutually exclusive events, then
b (i) P(E ∩ F) = 0 and
probability of not happening of event A = P( A) = .
a+b (ii) P(E ∪ F) = P(E) + P(F).
If odds against happening of an event A are a:b, then Notes:
b
probability of happening of event A = P(A) = and
a+b 1. For mutually exclusive events E and F, we have
probability of not happening of event P(E or F) = P(E ∪ F) = P(E) + P(F).
a 2. If E1, E2, ..., Ek are mutually exclusive events, then
A = P ( A) = .
a+b P(E1 È E1 È, ... , È Ek) = P(E1) + P(E2) + ... + P(Ek).

Chapter_32.indd 4 6/5/2015 5:22:50 PM


32.5
Probability

Theorem 3  If E and F are two mutually exclusive and (ii) P(two of heart or two of diamond)
exhaustive events, then P(E) + P(F) = 1. = P(two of heart) + P(two of diamond)
Theorem 4  Let E be any event and be its complementary 1 1 2 1
event, then = 1 – P(E). = + = =
52 52 52 26
Theorem 5   For any two events E and F,
Illustration 19  Find the probability of getting a sum of 7 or
P(E – F) = P(E) – P(E ∩ F). 11 in a simultaneous throw of two dice.
Theorem 6  (Addition Theorem). For any two events Solution: When two dice are thrown we have observed that
E and F , there are 36 possible outcomes. Now, we can have a sum
P(E ∪ F) = P(E) + P(F) – P(E ∩ F) of 7 as
Notes: 1 + 6 = 7, 2 + 5 = 7, 3 + 4 = 7, 4 + 3 = 7, 5 + 2 = 7, 6 + 1
=7
1. We may express the above results as Thus, the six favourable cases are (1, 6), (2, 5), (3, 4),
P (E or F) = P (E) + (F) – P (E and F) (4, 3), (5, 2), (6, 1)
2. If E and F are mutually exclusive, then 6 1
\ P(a sum of 7) = =
P(E ∩ F) = 0 and so P(E ∪ F) = P(E) + P(F). 36 6
Again, the favourable cases of getting a sum of 11 are
Theorem 7  If E1 and E2 be two events such that E1 ⊆ E2,
(5, 6), (6, 5)
then prove that P(E1) ≤ P(E2).
2 1
Theorem 8  If E is an event associated with a random \ P(a sum of 11) = =
36 18
experiment, then 0 ≤ P (E) ≤ 1.
Since the events of getting ‘a sum of 7’ or ‘a sum of 11’
Theorem 9. For any three events E, F, G are mutually exclusive:
P(E ∪ F ∪ G) = P(E) + P(F) + P(G) – P(E ∩ F) \ P(a sum of 7 or 11)
– P(F ∩ G) – P(E ∩G) + P(E ∩ F ∩ G) = P(a sum of 7) + P(a sum of 11)
Illustration 18 A card is drawn at random from a well- 1 1 4 2
= + = = .
shuffled pack of 52 cards. Find the probability of getting 6 18 18 9
(i) a jack or a queen or a king.
Illustration 20  From a well-shuffled pack of 52 cards, a
(ii) a two of heart or diamond. card is drawn at random, find the probability that it is either
Solution: (i) In a pack of 52 cards, we have a heart or a queen.
4 jacks, 4 queens and 4 kings. Solution: A:Getting a heart card  B:Getting a queen card
Now, clearly a jack and a queen and a king are mutually 13 4 1
exclusive events. P(A) = , P(B) = , P(A ∩ B) =
52 52 52
4
C 4 1
Also, P(a jack) = 52 1 = = Required probability = P(A ∪ B) = P(A) + P(B) – P(A ∩ B)
C1 52 13
13 4 1 16 4
4
C1 4 1 = + − = = .
P(a queen) = 52 = = 52 52 52 52 13
C1 52 13
4
C1 INDEPENDENT EVENTS
4 1
P(a king) = = =
52
C1 52 13 Two event A and B are said to be independent if the
\ By the addition theorem of Probability, occurrence (or non-occurrence of one does not affect the
probability of the occurrence (and hence non-occurrence)
P(a jack or a queen or a king) of the other.
= P(a jack) + P(a queen) + P(a king) Illustration 21  In the simultaneous throw of two coins,
1 1 1 3 ‘getting a head’ on first coin and ‘getting a tail on the second
= + + =
13 13 13 13 coin are independent events.

Chapter_32.indd 5 6/5/2015 5:22:51 PM


32.6 Chapter 32

Illustration 22  When a card is drawn from a pack of well- (i) only one of them will be selected.
shuffled cards and replaced before the second card is drawn, (ii) none of them be selected.
the result of second draw is independent of first draw. We now
Solution: Let A:Arun is s selected B:Tarun is selected.
state, without proof, the theorem which gives the probabilities
of simultaneous occurrence of the independent events. 1 1
Then,   P(A) = and P(B) =
3 5
Theorem 10  If A and B are two independent events, then Clearly, ‘A’ and ‘not B’ are independent also ‘not A’ and
P(A and B) = P(A). P(B) ‘not B’ are independent, ‘B’ and ‘not A’ are independent.
Illustration 23  Two dice are thrown. Find the probability (i) P (only one of them will be selected)
of getting an odd number on the one die and a multiple of = P (A and not B or, B and not A)
three on the other. = P (A) P (not B) + P (B) P (not A)
Solution: Since the events of ‘getting an odd number’ on 1 1 1 1
one die and the event of getting a multiple of three on the = 1 −  + 1 − 
3 5  5 3
other are independent events,
        P(A and B) = P (A) × P(B)(1) 1 4 1 2 4 2
× × × = +
=
3 1 3 5 5 3 15 12
Now, P(A) = P(an odd number ) = = [There are 6 2
6 2 = =
three odd numbers 1, 3, 5] and P(B) = P(a multiple of 3) = 15 5
2 1 P (only one of them be selected)
(ii)
= [Multiples of 3 are 3 and 6]
6 3 = P (not A and not B)
1 1 1 = P(not A) × P(not B)
\  From (1), required probability = × = .
2 3 6
 1  1
= 1 −  × 1 − 
Illustration 24  Arun and Tarun appear for an interview  3  5
for two vacancies. The probability of Arun’s selection is
2 4 8
one-third and that of Tarun’s selection is one-fifth. Find the = × = .
probability that 3 5 15

Practice Exercises

Difficulty Level-1
(Based on Memory)

1. Suppose six coins are flipped. Then the probability of probability of heads showing on 50 coins is equal to that
getting at least one tail is: of heads on 51 coins; then the value of p is:
71 53 1 49
(a) (b) (a) (b)
72 54 2 101
63 1 50 51
(c) (d) (c) (d)
64 12 101 101
[Based on MAT, 2002]
[Based on MAT, 1999]
2.
If events A and B are independent and P(A) = 0.15,
P(A ∪ B) = 0.45, then, P(B) = 4.
The probability that a marksman will hit a target is given
6 6 as one-fifth. Then, his probability of atleast one hit in 10
(a) (b) shots is:
13 17
10
6 6 1 4
(c) (d) (a) (b) 1 –  
19 23 610 5
[Based on MAT, 2005]
1 1
(c) 1 – (d) 1 –
3.
One hundered identical coins each with probability 510 519
p of showing up heads are tossed. If 0 < p < 1 and the
[Based on MAT, 2005]

Chapter_32.indd 6 6/5/2015 5:22:52 PM


32.7
Probability

5.
Two dice are tossed. The probability that the total score is 12.
Two events A and B have probabilities 0.25 and 0.50
a prime number is: respectively. The probability that both A and B occur
1 5 simultaneously is 0.12. Then, the probability that neither
(a) (b)
6 12 A nor B occurs is:
1 7 (a) 0.38 (b) 0.13
(c) (d)
2 9 (c) 0.63 (d) 0.37
[Based on MAT, 2000]
[Based on MAT (Feb), 2011]
6.
Four different objects 1, 2, 3, 4 are distributed at random 13. A has 3 shares in a lottery containing 3 prizes and
in four places marked 1, 2, 3, 4. What is the probability 9 blanks. B has two shares in a lottery containing
that none of the objects occupy the place corresponding to 2 prizes and 6 blanks. Compare their chances of success.
its number?
(a) 145:362 (b) 952:715
(a) 17/24 (b) 3/8
(c) 123:213 (d) 145:716
(c) 1/2 (d) 5/8
[Based on MAT (Feb), 2011]
[Based on MAT, 2001]
7. Three students try to solve a problem independently with 14.
A classroom has 3 electric lamps. From a collection of
a probability of solving it as 1/3, 2/5, 5/12, respectively. 10 electric bulbs of which 6 are good, 3 are selected at
What is the probability that the problem is solved? random and put in the lamps. Find the probability that all
lamps are burning.
(a) 1/18 (b) 12/30
(a) 1/8 (b) 1/4
(c) 23/30 (d) 1/2
[Based on IIT Joint Man. Ent. Test, 2004] (c) 1/2 (d) 1/6
[Based on MAT (Dec), 2010]
8. If the probability of rain on any given day in Pune city is
50% then what is the probability that it rains on exactly 3 15.
The odds in favour of A winning a game of badminton
days in a five-day period? against B are 5:2 If three games are to be played, what are
(a) 8/125 (b) 5/16 the odds in favour of A’s winning aleast one game?
(c) 8/25 (d) 2/25 (a) 425:5 (b) 365:1
[Based on SCMHRD Ent. Exam., 2003] (c) 335:8 (d) None of these
[Based on MAT (Dec), 2010]
9.
A Chartered Accountant applies for a job in two firms X
and Y. The probability of his being selected in firm X is 16.
A and B play a game where each is asked to select a
0.7, and being rejected at Y is 0.5 and the probability of his number from 1 to 16. If the two numbers match, both of
application being rejected is 0.6. What is the probability them win a prize. Find the probability that they will not
that he will be selected in one of the firm? win a prize in a single trial.
(a) 0.8 (b) 0.2 (a) 15/16 (b) 12/17
(c) 0.4 (d) 0.7 (c) 14/15 (d) 12/13
[Based on MAT, 2008] [Based on MAT (Dec), 2010]
10.
There are two boxes A and B. Box A has 5 oranges and 6 17.
The probability that a computer company will get a
apples in it and box B contains 3 apples and 4 oranges in it. computer hardware contract is 2/3 and the probability that
A fruit is taken from A and placed in B, after which a fruit is it will not get a software contract is 5/9. If the probability
then transfered from B to A. What is the probability that the of getting atleast one contract is 4/5, what is the probability
configuration of boxes does not change due to the transfers? that it will get both the contracts?
(a) 49/88 (b) 45/88 (a) 14/45 (b) 17/45
(c) 25/44 (d) None of these
(c) 16/45 (d) 11/45
[Based on MAT (Feb), 2011]
[Based on MAT (Dec), 2010]
11.
Eight chits are numbered 1 to 8. Three are drawn one
18.
A speaks truth in 75% of cases and B in 80% of cases. A
by one with replacement. The probability that the least
and B agree in a statement .What is the probability that the
number on any selected chit is 6, is:
statement is true?
3 3
3 3 (a) 12/13 (b) 11/15
(a) 1 −   (b)  
4 4 (c) 14/17 (d) 17/29
3 3 [Based on MAT (Sept), 2010]
 3  3
(c)   (d) 1 −  
8 8 19.
There are 10 persons, including A and B who stand in
[Based on MAT (Feb), 2011] the form of a circle. If the arrangement of the persons is

Chapter_32.indd 7 6/5/2015 5:22:52 PM


32.8 Chapter 32

at random, then the probability that there are exactly 3 (a) 3/12 (b) 2/11
persons between A and B is: (c) 1/6 (d) 4/9
(a) 1/9 (b) 7/9 [Based on MAT (Dec), 2009]
(c) 2/9 (d) 1/3
27.
There are 100 students in a college class of which 36
[Based on MAT (May), 2010]
are boys studying Statistics and 13 girls are not studying
20.
The odds in favour of A winning a game against B is Statistics. If there are 55 girls in all, the probability that a
4:3. If three games are to be played to decide the overall boy picked up at random is not studying Statistics is:
winner, the odds in favour of A winning atleast once is: (a) 1/5 (b) 2/5
(a) 343:27 (b) 316:27 (c) 3/5 (d) 4/5
(c) 343:316 (d) None of these [Based on MAT (Dec), 2009]
[Based on MAT (May), 2010]
28.
A can hit a target 4 times in 5 shots, B hits 3 times in 4
21.
Cards each marked with the numbers 1, 2, 3, ..., 10 are shots and C hits thrice in 3 shots, they fire together. Find
placed in a box and mixed throughly. One card is drawn the probability that atleast two shots hit the target.
at random from the box. Find the probability of getting a
(a) 13/30 (b) 5/6
prime number.
(c) 11/40 (d) None of these
(a) 2/5 (b) 4/9
[Based on MAT (Sept), 2009]
(c) 5/9 (d) 1/6
[Based on MAT (May), 2010] 29.
The odds that A speaks the truth are 3:2 and the odds that
B speaks the truth are 5:3. In what per cent of cases are
22.
Namrata want to visit four cities A, B, C and D on an they likely to agree each other on an identical point?
official trip. The probability that she visits A just before
(a) 47.5% (b) 37.5%
B is:
(c) 63.5% (d) None of these
(a) 1/2 (b) 1/12
[Based on MAT (Sept), 2009]
(c) 1/6 (d) 1/4
[Based on MAT (Feb), 2010] 30.
In a class of 25 students with Roll No. 1 to 25. A student
is picked up at random to answer a question. Find the
23.
A lot of 12 bulbs contains 4 defective bulbs. Three
probability that the roll number of the selected student is
bulbs are drawn at random from the lot, one after the
either multiple of 5 or 7.
other. The probability that all three are non defective is:
(a) 6/25 (b) 4/25
(a) 14/55 (b) 8/12
(c) 8/25 (d) 7/25
(c) 1/27 (d) None of these
[Based on MAT (Sept), 2009]
[Based on MAT (Feb), 2010]
24.
Three riflemen take one shot each at the same target. The 31.
A bag contains 5 white and 3 black balls and 4 are
probability of the first rifleman hitting the target is 0.4, the successively drawn out and not replaced; what is the
probability of the second rifleman hitting the target is 0.5 probability they are alternately of different colours?
and the probability of the third rifleman hitting the target 1 1
is 0.8. The probability that exactly two of them hit the (a) (b)
7 14
target, is:
(a) 0.54 (b) 0.44 3 2
(c) (d)
(c) 0.32 (d) 0.52 14 7
[Based on MAT (Dec), 2009] [Based on MAT (May), 2009]

25.
100 students appeared for two examinations. 60 passed 1 1
the first, 50 passed the second and 30 passed both. The 32.
The probabilities that three man hit a target are , and
6 4
probability that a student selected at random has failed in 1
both examinations is: respectively. If only one hits the target, what is the
3
(a) 0.3 (b) 0.2 probability that it was the first man?
(c) 0.4 (d) 0.1
11 21
[Based on MAT (Dec), 2009] (a) (b)
57 57
26.
A and B stand in a ring with 11 other persons. If the
12 2
arrangement of the 13 persons is at random, then the (c) (d)
probability that there are exactly 3 persons between A and 67 9
B is: [Based on MAT (May), 2009]

Chapter_32.indd 8 6/5/2015 5:22:52 PM


32.9
Probability

33.
A bag contains 100 tickets numbered 1, 2, 3, ..., 100. If a 40.
A fair coin is tossed a fixed number of times. If the
ticket is drawn out of it at random, what is the probability probability of getting 4 heads equals the probability of
that the ticket drawn has the digit 2 appearing on it? getting 7 heads, then the probability of getting 2 heads is:
(a) 21/100 (b) 19/100 (a) 1/1024 (b) 55/2048
(c) 32/100 (d) 23/100 (c) 3/4096 (d) None of these
[Based on MAT (Feb), 2009] [Based on MAT (Feb), 2008]
34.
A box contains 5 brown and 4 white socks. A man takes 41.
An ordinary cube has four blank faces, one face marked 2
out two socks. The probability that they are of the same and another marked 3, then the probability of obtained 12
colour is: in 5 throws is:
(a) 1/6 (b) 5/108 (a) 5/1944 (b) 5/1296
(c) 5/18 (d) 4/9 (c) 5/2592 (d) None of these
[Based on MAT (Feb), 2009] [Based on MAT (Feb), 2008]
35.
A fair coin is tossed repeatedly. If head appears on the first 42.
There are three events A, B and C, one of which must and
four tosses, then the probability of appearance of tail on only one can happen. The odds are 8 to 3 against A, 5 to 2
the fifth toss is: against B. Find the odds against C.
(a) 1/2 (b) 1/7 (a) 43:34 (b) 43:77
(c) 3/7 (d) 2/3 (c) 34:43 (d) 77:43
[Based on MAT (Feb), 2009] [Based on MAT (Dec), 2007]
36.
A car is parked by an owner amongst 25 cars in a 43. A card is drawn at random from a well-shuffled pack of 52
row, not at either end. On his return, he finds that cards. What is the probability of getting a two of hearts or
exactly 15 places are still occupied. The probability a two of diamonds?
that both the neighbouring parking places are empty 3 2
is: (a) (b)
26 17
(a) 91/276 (b) 15/164
1 4
(c) 15/92 (d) 91/164 (c) (d)
[Based on MAT (May), 2008] 26 13
[Based on MAT (Dec), 2007]
37.
A committee consists of 9 experts taken from three
institutions A, B and C of which 2 are from A, 3 from B 44.
An instrument manufactured by a company consists
and 4 from C. If three experts resign, then the probability of two parts A and B in manufacturing part A, 9 out of
that they belong to different institutions is: 100 are likely to be defective and in manufacturing part
B, 5 out of 100 are likely to be defective. Calculate the
(a) 1/729 (b) 1/24
probability that both the instruments will not be defective.
(c) 1/21 (d) 2/7
(a) 0.91 (b) 0.86
[Based on MAT (May), 2008]
(c) 0.95 (d) 0.83
38.
A Chartered Accountant applied for a job in two firms X [Based on MAT (Dec), 2007]
and Y. The ability of his being selected in firm X is 0.7
and being rejected at Y is 0.5 and the probability of atleast 45.
India plays two matches each with West Indies and
one of his applications being rejected is 0.6. What is the Australia. In any match the probabilities of India getting
probability that he will be selected atleast one of the points 0, 1, 2 are 0.45, 0.05 and 0.50 respectively.
firms? Assuming that outcomes are independent the probability
of India getting atleast 7 points is:
(a) 0.8 (b) 0.2
(a) 0.8750 (b) 0.06875
(c) 0.4 (d) 0.7
[Based on MAT (May), 2008] (c) 0.0875 (d) 0.0250
[Based on MAT (Dec), 2007]
39.
The probability that a contractor will get a plumbing
contract is 2/3 and the probability that he will not get an 46.
Three persons work independently on a problem. If the
electric contract is 5/9. If the probability of getting atleast respective probabilities that they will solve it are one-
one contract is 4/5, what is the probability that he will get third, one-fourth and one-fifth, then the probability that
both? none can solve it is:
(a) 31/45 (b) 8/45 (a) 1/5 (b) 1/3
(c) 14/45 (d) None of these (c) 2/5 (d) None of these
[Based on MAT (May), 2008] [Based on MAT (Dec), 2007]

Chapter_32.indd 9 6/5/2015 5:22:53 PM


32.10 Chapter 32

47.
A class consists of 100 students; 25 of them are girls and 53.
With the same data as for the previous question, what is
75 boys; 20 of them are rich and the remaining poor; 40 of the probability that the next program will run correctly
them are fair-complexioned. The probability of selecting after the third run but not earlier?
a fair complexioned rich girl is: (a) 9/10 (b) 3/10
(a) 0.05 (b) 0.04 (c) 7/20 (d) 1/10
(c) 0.02 (d) 0.08 [Based on MAT, 1997]
[Based on MAT (May), 2007]
54.
Course materials are sent to students by a dis­tance teaching
48.
Four boys and three girls stand in queue for an interview. institution. The probability that they will send a wrong
The probability that they will stand in alternate positions programme’s study mate­rial is 1/5. There is a probability
is: of 3/4 that the package is damaged in transit, and there is
(a) 1/34 (b) 1/35 a probability of 1/3 that there is a short shipment. What is
the probability that the complete material for the course
(c) 1/17 (d) 1/68 arrives without any dam­age in transit?
[Based on MAT (Dec), 2006]
(a) 4/5 (b) 8/60
49.
A and B play a game where each is asked to select a (c) 8/15 (d) 4/20
number from 1 to 5. If the two numbers match, both of [Based on MAT, 1997]
them win a prize. The probability that they will not win a
prize in a single trial is: 55.
With the same data as for the previous question, what is
the probability that the same student on two successive
(a) 1/25 (b) 24/25
occasions gets the wrong study material?
(c) 2/25 (d) None of these
(a) 1/25 (b) 1/5
[Based on MAT (Dec), 2006]
(c) 4/25 (d) 3/25
50.
Three groups A, B and C are contesting for a position in [Based on MAT, 1997]
the Board of Directors of a company. The probabilities of
their winning are 0.5, 0.3 and 0.2 respectively. If the group 56.
What is the probability of getting a sum as 6 when
A wins, then the probability of introducing a new product two dice are thrown simultaneously?
is 0.7 and the corresponding probabilities for group B and 5 35
C are 0.6 and 0.5 respectively. The probability that the (a) (b)
36 66
new product will be introduced is:
(a) 0.52 (b) 0.74 1 3
(c) (d)
(c) 0.63 (d) None of these 6 8
[Based on MAT, 1998]
[Based on MAT (May), 2006]
57.
Eight coins are tossed simultaneously. The probability of
51.
An article manufactured by a company consists of two getting at least 6 heads is:
parts A and B. In the process of manufacture of part A, 9
1 37
out of 100 are likely to be defective. (a) (b)
13 256
Similarly, 5 out of 100 are likely to be defective in the
process of manufacture of part B. The probability that the 25
(c) (d) None of these
assembled part will not be defective is: 57
(a) 0.8645 (b) 0.9645 [Based on MAT, 1998]
(c) 0.6243 (d) None of these 58.
Two cards are drawn together from a pack of 52 playing
[Based on MAT (May), 2006] cards at random. What is the probability that both are
kings?
52.
A programmer noted the results of attempting to run
20 programs. The results showed that 2 pro­grams ran 1 25
(a) (b)
correctly in the first attempt, 7 ran correctly in the 13 57
second attempt, 5 ran correctly in the third attempt, 4 ran
correctly in the fourth attempt and 2 ran correctly in the 37
(c) (d) None of these
fifth attempt. What is the probability that his next program 257
will run correctly on the third run? [Based on MAT, 1998]
(a) 1/4 (b) 1/3 59.
Three light bulbs are selected at random from 20 bulbs
(c) 1/6 (d) 1/5 of which 5 are defective. The probability that none of the
[Based on MAT, 1997] bulbs that is picked up is defec­tive is:

Chapter_32.indd 10 6/5/2015 5:22:53 PM


32.11
Probability

4 140 65.
A coin is tossed 5 times. What is the probability that head
(a) (b) appears an odd number of times?
7 228
(a) 2/5 (b) 1/5
137 91
(c) (d) (c) 1/2 (d) 4/25
228 228
[Based on MAT, 1998]
[Based on MAT, 1998]
66.
A bag contains 5 white, 7 red and 8 black balls. If 4 balls
60.
Two cards are drawn together from a pack of 52 cards are drawn one-by-one with replacement, what is the
(a set of traditional playing cards) at ran­ dom. The probability that all are white?
probability that one is a spade and the other is a heart is:
(a) 1/256 (b) 1/16
13 3 (c) 4/20 (d) 4/8
(a) (b)
102 20 [Based on MAT, 1998]
47 29 67.
Atal can hit a target 3 times in 6 shots, Bhola can hit the
(c) (d)
100 34 target 2 times in 6 shots and Chandra can hit the target
[Based on MAT, 1998] 4 times in 4 shots. What is the probability that at least 2
shots hit the target?
61.
A bag has 4 red and 5 black balls. A second bag has 3 red
and 7 black balls. One ball is drawn from the first bag and (a) 1/2 (b) 2/3
two from the second. The probability that there are two (c) 1/3 (d) 11/18
black balls and a red ball is: [Based on MAT, 1998]
14 11 68.
A bag contains 3 white balls and 2 black balls.
(a) (b)
45 45 Another bag contains 2 white balls and 4 black balls. A
bag and a ball are picked at random. The probability that
7 9
(c) (d) the ball will be white is:
15 54
7 7 5 7
[Based on MAT, 1998] (a)    (b)    (c)    (d) 
11 30 11 15
62.
Three boxes contain 6 red, 4 black; 4 red, 6 black and [Based on MAT, 1999]
5 red, 5 black balls respectively. One of these boxes is
69.
There are 6 positive and 8 negative numbers. Four numbers
selected at random and a ball is drawn from it. If the ball
are chosen at random and multiplied. The probability that
drawn is red, then the probability that it is drawn from the
the product is a positive number is:
first box is:
500 503
3 27 (a) (b)
(a) (b) 1001 1001
4 83
505 101
15 2 (c) (d)
(c) (d) 1001 1001
59 5 [Based on MAT, 1999]
[Based on MAT, 1998]
70.
One hundred identical coins each with probabil­ ity
63.
A dice is thrown 6 times. If ‘getting an odd num­ber’ is p of showing up heads are tossed. If 0 < p < 1 and the
‘success’. The probability of 5 successes is: probability of heads on 50 coins is equal to that of heads
1 3 on 51 coins, then the value of p is:
(a) (b)
10 32 1 49
(a) (b)
5 25 2 101
(c) (d)
6 36 50 51
(c) (d)
[Based on MAT, 1998] 101 101
[Based on MAT, 1999]
64. In the West Indies, there is a 3-match one-day International
tournament between West Indies and India. At the end of 71.
The odds against certain event are 5:2 and the odds favour
every match, either a team wins or loses. There is no draw. of another independent event are 6:5. The probability that
Find the probability that India wins the series by winning at least one of the event will happen is:
at least 2 consecutive matches. 12 25
(a) (b)
5 77 77
(a) 1 (b)
8 52 65
(c) (d)
3 1 77 77
(c) (d)
8 2 [Based on IIFT, 2005]

Chapter_32.indd 11 6/5/2015 5:22:54 PM


32.12 Chapter 32

72.
Three letters are drawn from the alphabet of 26 letters 79. The letters B, G, I, N and R are rearranged to form the
without replacement. The probability that they appear in word ‘Bring’. Find its probability.
alphabetical order is: 1 1
(a) 3C1/26C3 (b) 1/26C3 (a) (b) 4
120 5
(c) 1/3 (d) 1/6
1 5
[Based on IIFT, 2005] (c) (d) × 42
24 5
73.
A and B are independent events such that p (A) = p (A ∪ B)
= 0.4. Then, p (B) equals: 80.
There are three events A, B and C, one of which must and
1 only can happen. If the odds are 8:3 against A, 5:2 against
(a) (b) 0.7 B, the odds against C must be:
7
1 (a) 13:7 (b) 3:2
(c) 0.12 (d)
8 (c) 43:34 (d) 43:77
[Based on IIFT, 2005]
74.
A letter is taken out at random from Assistant and [Based on ATMA, 2008]
another taken out from Statistics. The probability that 81. A bag contains 4 five rupee coins, 3 two rupee coins and
they are the same letter is: 3 one rupee coins. If 6 coins are drawn from the bag at
(a) 1/45 (b) 13/90 random. What are the odds in favour of the draw yielding
maximum amount?
(c) 19/90 (d) None of the above
(a) 1:70 (b) 1:69
75. If the chances that the electricity goes off for a particular (c) 69:70 (d) 70:1
day is 50% then what is the probability that in a week it
[Based on MAT, 2012]
will go off exactly for three days?
8 5 82. A point is selected at random from the interior of a circle.
(a) (b)
15 16 The probability that the point is closer to the center then
the boundary of the circle is:
35
(c) (d) None of these (a) 3/4 (b) 1/2
128
(c) 1/4 (d) None of these
76.
If the chance that a vessel arrives safely at a port is 9/10 [Based on MAT, 2013]
then what is the chance that out of 5 vesels expected at
least 4 will arrive safely? 83. In a compitition A, B and C are participating. The
(14  94 ) (15  95 ) probability that A wins is twice that of B, the probability
(a) 5
(b) that B wins is twice that of C. The probability that A loses
10 104
is:
(14  93 ) (14  96 ) (a) 1/7 (b) 2/7
(c) 4
(d)
10 105 (c) 4/7 (d) 3/7
[Based on JMET, 2011] [Based on MAT, 2013]

77.
There are four hotels in a town. If 3 men check into the 84. It has been found that if A and B play a game 12 times A
hotels in a day then what is the probability that each wins 6 times, B wins 4 times and they draw twice. A and B
checks into a different hotel? take part in a series of 3 games. The probability that they
will win alternately is:
6 1
(a) (b) (a) 2/12 (b) 5/36
7 8
(c) 1/6 (d) 3/4
3 5 [Based on MAT, 2013]
(c) (d)
8 9
[Based on JMET, 2011] 85. Ten cars are parked in a row, what is the probability that
there are exactly five cars between the particular two cars:
78.
If 5% of the electric bulbs manufactured by a company are
defective then what is the probability that in a sample of 8×5 4 × 5!× 2!
(a) (b)
100 bulbs none is defective? 10! 10!
(a) e–3 (b) e–5 8 × 8! 16 × 8!
(c) (d)
(c) e5 (d) e 10 10!
[Based on JMET, 2011] [Based on MAT, 2013]

Chapter_32.indd 12 6/5/2015 5:22:55 PM


32.13
Probability

86. There are two bags, one of which contains 3 black and 4 (a)
1
(b)
1
white balls, while the other contains 4 black and 3 white 4 2
balls. A dice is cast. If the face 1 or 3 turns up, a ball 2 1
is taken from the first bag and if any other face turns up (c) (d)
3 3
a ball is chosen from the second bag. The probability of [Based on MAT, 2014]
choosing a black ball is: 90. A person draws a card from a pack of 52, replaces it and
11 10 shuffles it. He continues doing it until he draws a heart.
(a) (b)
21 21 What is the probability that he has to make 3 trials?
9 3
12 9 (a) (b)
(c) (d) 64 64
21 21
[Based on MAT, 2013] 5 1
(c) (d)
87. Out of 20 consecutive positive integers, two are chosen at 64 64
[Based on MAT, 2014]
random. The probability that their sum is odd is:
91. The probability that a contractor will get a plumbing
10 1 contract is 2/3 and the probability that he will not get an
(a) (b)
19 20 electric contract is 5/9. If the probability of getting at least
19 9 one contract is 4/5. What is the probability that he will get
(c) (d) both?
20 19
[Based on MAT, 2013] (a) 31/45 (b) 8/45
88. A fair coin is tossed repeatedly. If head appears on the first (c) 14/45 (d) None of these
four tosses, then the probability of appearance of tail on [Based on MAT, 2012]
the fifth toss is:
1 1
92. A candidate is selected for interview for three posts. For
(a) (b) the first post, there are 5 candidates, for the second there
2 7 are 8 and for third there are 7. What are the chances for
3 2 him getting at least one post?
(c) (d)
7 3 1 3
[Based on MAT, 2013] (a) (b)
5 5
89. Two fair dices are thrown. Given that, the sum of the dice 2 4
is less than or equal to 4, find the probability that only one (c) (d)
5 5
dice shows two. [Based on ATMA, 2008]

Difficulty Level-2
(Based on Memory)

1. A subscriber was dialing a telephone number. When he 1


n
 n − 1
n
forgot the last 3 digits and remembering that they were (a)   (b)  
n  n 
different, he dialled at random. What is the probability
that he dialled the right number? n n
 n − 1 1
3 3 (c) 1 –   (d) 1 –  
(a) (b)  n  n
10 720
1 3.
Two buckets contain black and white balls. The first has
(c) (d) Cannot be determined five black and three white and the second one has 4 white
720
and 4 black balls. A ball is picked from the first bucket
2. To open a lock, a key is taken out of a collection of n keys and put into the second bucket. Next a ball from the
at random. If the lock is not opened with this key, it is second bucket is picked and put into the first bucket. Find
put back into the collection and another key is tried. The the probability of the event that after these actions the
process is repeated again and again. It is given that with composition of first bucket has become that of the second
only one key in the collection, the lock can be opened. one and the composition of the second bucket has become
The probability that the lock will open in n trials is: that of first bucket.

Chapter_32.indd 13 6/5/2015 5:23:02 PM


32.14 Chapter 32

(a) 12/19 (b) 5/18 (a) 9:14 (b) 16:7


(c) 4/9 (d) 7/18 (c) 10:7 (d) 12:11

4.
If n positive integers are taken at random and multiplied 11.
A binary number is made up of 8 digits. Suppose that the
together, then the probability that the last digit of the probability of an incorrect digit appearing is p and that
product be 2, 4, 6 or 8 is: the errors in different digits are independent of each other.
4n + 2n 4n × 2n Then the probability of forming an incorrect number is:
(a) (b)
5 n
5n (a) p8 (b) p/8
(c) (1 – p8) (d) 1 – (1 – p)8
4n − 2n 4 n × 5n
(c) (d)
5n 2n 12. A special lottery is to be held to select a student who
will live in the only deluxe room in a hostel. There are
5.
India plays two matches each with the West Indies and 100 years-III, 150 year-II, and 500 years-I students who
Australia. In any match the probability of India getting points applied. Each year-III’s name is place in the lottery 3
0, 1 and 2 are 0.45, 0.05 and 0.50, respectively. Assuming times; each year-II’s name, 2 times; and each year-I’s
that the outcomes are independent, the probability of India name, 1 time. What is the probability that a year-III’s
getting at least 7 points is: name will be chosen?
(a) 0.8750 (b) 0.0875 (a) 1/8 (b) 2/9
(c) 0.0625 (d) 0.0250 (c) 2/7 (d) 3/8
6.
What is the probability of drawing 2 black balls randomly [Based on SNAP, 2007]
and in succession from a bag containing 5 black balls and 13.
A man who is firing at a distant target has 10% chance of
3 white balls if the first ball drawn is replaced before the hitting the target in one shot. The number of times he must
second draw is made? fire at the target to have about 50% chance of hitting the
25 23 target is (given log2 = 0.3010 and log3 = 0.4771):
(a) (b)
64 64 (a) 11 (b) 9
25 23 (c) 7 (d) 5
(c) (d)
56 56
14. In the above question, if in the 3 matches, one match ends
7. The odds against a certain event are 5:2, and the odds in in a draw, then what is the probability that India wins the
favour of another event independent of the former are 6:5. series by winning 2 consecutive matches?
The chance that at least one of the events will happen is: 1
(a) 1 (b)
25 52 2
(a) (b)
77 77 1 1
(c) 1 (d) None of the above (c) (d)
4 6
8. The probability that an event A happens in one trial of
an experiment is 0.4. Three independent trials of the 15. In a right triangle, the probability of one of the angles to
experiment are formed. The probability that the event A be 60º is. Then what is the probability of atleast one of the
happens at least once is: angles to be of 30º degrees?
(a) 0.934 (b) 0.784
1
(c) 0.548 (d) 0.343 (a) (b) 1
4
[Based on FMS (Delhi), 2002]
1
9. Thirty days are in September, April, June and November. (c) (d) Can’t be said
2
Some months are of thirty one days. A month is chosen at
random. Then its probability of having exactly three days
less than maximum of 31 is: 16. A solid cube of side 4 cm is painted on all sides. Then
the cube is cut to form, cubes of side 1 cm. If a cube is
(a) 15/16 (b) 1
selected at random, what is the probability that none of its
(c) 3/48 (d) None of these sides are painted?
[Based on SNAP, 2007]
1 3
10.
Ram has 3 shares in a lottery in which there are 3 prizes (a) (b)
8 4
and 6 blanks. Mohan has 1 share in lottery in which there
is 1 prize and 2 blanks. What is the ratio of Ram’s chance 1 7
(c) (d)
of success to Mohan’s chance of success? 2 8

Chapter_32.indd 14 6/5/2015 5:23:02 PM


32.15
Probability

17.
In a factory, each day the expected number of accidents 24.
While investigating the case of recent blasts in Delhi,
is related to the number of overtime hour by a linear the Delhi Police submitted two evidences E1 and E2
equation. Suppose that on one day there were 1000 suggesting the involvement of a suspect in the crime to a
overtime hours logged and 8 accidents reported and on local court. The court wants to decide whether the suspect
another day there were 400 overtime hours logged and 5 is guilty (G) on the basis of pieces of evidence E1 and
accidents. What is the expected number of accidents when E2. Suppose for both the evidences E1 and E2 the court
no overtime hours are logged? determines the probability of guilt P(G | E1) and P(G | E2)
(a) 2 (b) 3 to be 0.60 and 0.70, respectively. What is the probability
(c) 4 (d) 5 of guilt on the basis of both the evidences E1 and E2, i.e.,
[Based on SNAP, 2007] P(G | E1, E2)?
18.
Thirty days are in September, April, June and November. (a) 0.42 (b) 0.60
Some months are of thirty one days. A month is chosen at (c) 0.65 (d) 0.78
random. Then its probability of having exactly three days [Based on FMS, 2009]
less than maximum of 31 is: 25. Three of the men participating in a race are A, B and C.
(a) 15/16 (b) 1 1
(c) 3/48 (d) None of these The probability of A winning the race is , the probability
6
[Based on SNAP, 2007]
2 4
19. If all the angles of a triangle are integers. What is the for B winning the race is and for C is if dead heat
5 13
probability that an isosceles triangle is equilateral? among them is possible, what is the probability that only
1 1 one of the three winning the race?
(a) (b)
59 60 177 59
(a) (b)
1 1 380 130
(c) (d)
89 90 39 134
(c) (d)
20.
A dice is rolled three times and sum of three numbers 157 390
appearing on the uppermost face is 15. The chance that
the first roll was four is: 26.
In a pizza stall, Ajay and Mohan, being the lucky
(a) 2/5 (b) 1/5 customers, were given the option of drawing tickets
from a pot containing x number of tickets for the knife
(c) 1/6 (d) None of these
throwing show and y number of tickets for the talking doll
[Based on SNAP, 2009]
show. Both Ajay and Mohan being excited about the knife
21.
A five-digit number is formed by using the digits 1, 2, 3, 4 throwing show, start drawing tickets from the pot until
and 5 without repetitions. What is the probability that the they get one for the show, replacing any drawn ticket for
number is divisible by 4? the talking dolls show in the pot. Ajay draws the ticket
(a) 1/5 (b) 5/6 first, followed by Mohan. Given this, mark all the correct
(c) 4/5 (d) None of these options.
[Based on SNAP, 2009] (a) If the probability of Ajay first getting a ticket for the
knife throwing show is four times that for Mohan, the
22.
A dice is rolled three times and sum of three numbers ratio between x and y is 3:1
appearing on the uppermost face is 15. The chance that
the first roll was a four is: (b) If the probability of Ajay first getting a ticket for the
2 1 knife throwing show is five times that for Mohan, the
(a) (b) ratio between y and x is 1:4
5 5
(c) If the probability of Ajay first getting a ticket for the
1
(c) (d) None of these knife throwing show is two times that for Mohan, the
6 ratio between x and y is 1:1
[Based on SNAP, 2010]
(d) If the probability of Mohan first getting a ticket for
23.
A five-digit number is formed by using the digits 1, 2, 3, 4 the knife throwing show is six times that for Ajay, the
and 5 without repetitions. What is the probability that the ratio between x and y is 5:1
number is divisible by 4?
[Based on ITFT, 2006]
1 5
(a) (b)
5 6 27.
A medical clinic tests blood for certain disease from
4 which approximately one person in a hundred suffers.
(c) (d) None of these People come to the clinic in group of 50. The operator of
5
[Based on SNAP, 2010] the clinic wonders whether he can increase the efficiency

Chapter_32.indd 15 6/5/2015 5:23:03 PM


32.16 Chapter 32

of the testing procedure by conducting pooled tests. In the for a Government project. The probability that at least one
pooled test, the operator would pool the 50 blood samples of the junior professors would get selected is:
and test them altogether. If the polled test was negative, he (a) 5/6 (b) 2/3
could pronounce the whole group healthy. If not, he could
(c) 1/5 (d) 1/6
then test each person’s blood individually. The expected
[Based on XAT, 2007]
number of tests the operator will have to perform if he
pools the blood samples are:
32.
The supervisor of a packaging unit of a milk plant is being
(a) 47 (b) 25 pressurised to finish the job closer to the distribution time,
(c) 21 (d) None of these thus giving the production staff more leeway to cater to
[Based on ITFT, 2008] last minute demand. He has the option of running the unit
at normal speed or at 110% of normal ‘fast speed’. He
28.
The game of ‘chuck-a-luck’ is played at carnivals in some estimates that he will be able to run at the higher speed 60%
parts of Europe. Its rules are as follows:if you pick a of time. The packet is twice as likely to be damaged at the
number from 1 to 6 and the operator rolls three dice. If higher speed which would mean temporarily stopping the
the number you picked comes up on all three dice, the process. If a packet on a randomly selected packaging runs
operator pays you € 3; if it comes up on two dice, you are has probability of 0.112 of damage, what is the probability
paid € 2; and if it comes up on just one die, you are paid that the packet will not be damaged at normal speed?
€ 1. Only if the number you picked does not comes up at (a) 0.81 (b) 0.93
all, you pay the operator € 1. The probability that you will
win money playing in this game is: (c) 0.75 (d) 0.60
[Based on XAT, 2010]
(a) 0.52 (b) 0.753
(c) 0.42 (d) None of these 33.
The chance of India winning a cricket match against
Australia is one-sixth. What is the minimum number of
[Based on ITFT, 2008]
matches India should play against Australia so that there
is a fair chance of winning at least one match?
29. Events X, Y and Z are mutually exclusive events such that
(a) 3 (b) 4
3a + 1 1− a 1 − 2a (c) 5 (d) 6
P (X) = , P (Y) = and P (Z) = . The
3 4 2 [Based on XAT, 2010]
set of possible values of a are in the interval: 34.
There are four machines in a factory. At exactly 8 p.m.
when the mechanic is about to leave the factory, he is
1 2 
(a)  ,  informed that two of the four machines are not working
3 3 properly. The mechanic is in a hurry and decides that he
1 2 will identify the two faulty machines before going home
(b)  , and repair them next morning. It takes him 20 minutes to
2 3 
walk to the bus stop. The last bus leaves at 8:32 p.m. If it
1 1  takes 6 minutes to identify whether a machine is defective
(c)  ,  or not and if he decides to check the machines at random,
3 2
what is the probability that the mechanic will be able to
(d) [0, 1] catch the last bus?
(a) 0 (b) 1/6
30.
Sun Life Insurance Company issues standard, preferred (c) 1/4 (d) 1/3
and ultra-preferred policies. Among the company’s [Based on XAT, 2011]
policy holders of a certain age, 50% are standard with
a probability of 0.01 of dying in the next year, 30% are 35.
The scheduling officer for a local police department is
preferred with a probability of 0.008 of dying in the next trying to schedule additional patrol units in each of two
year and 20% are ultra-preferred with a probability of neighbourhoods—Southern and Northern. She knows
0.007 of dying in the next year. If a policy holder of that that on any given day, the probabilities of major crimes
age dies in the next year, what is the probability of the and minor crimes being committed in the Northern
deceased being a preferred policy holder? neighbourhood were 0.418 and 0.612, respectively, and
(a) 0.1591 (b) 0.2727 that the corresponding probabilities in the Southern
neighbourhood were 0.355 and 0.520. Assuming that all
(c) 0.375 (d) None of these crime occur independent of each other and likewise that
[Based on IIFT, 2010] crime in the two neighbourhoods are independent of each
31.
A management institute has six senior professors and four other, what is the probability that no crime of either type
junior professors. Three professors are selected at random is committed in either neighbourhood on any given day?

Chapter_32.indd 16 6/5/2015 5:23:03 PM


32.17
Probability

(a) 0.069 (b) 0.225 41. Badri has 9 pairs of dark blue socks and 9 pairs of black
(c) 0.690 (d) 0.775 socks. He keeps them all in the same bag. If he picks out
[Based on XAT, 2011] three socks at random, then what is the probability that he
will get a matching pair?
36. In a game of cards assume King to be greatest card and
(a) (29C29C1)/18C3
Ace to be the least value card and the other cards’ value
increases pinearly from Ace to King. Ram and Shyam are (b) (9C39C1)/18C3
playing a game in which Ram picks a card first and then (c) 1
Shyam picks a card. If the value of Shyam’s card is greater
than the value of Ram’s card then Shyam wins else he (d) None of these
loses. What is the probability of Shyam winning the bet.
Does he have greater probability if he plays first instead of 42. In his wardrobe, Timothy has 3 trousers. One of them
Ram? is black the second blue, and the third brown. In his
wardrobe, he also has 4 shirts. One of them is black and
(a) 0.47, No (b) 0.53, Yes the other 3 are white. He opens his wardrobe in the dark
(c) 0.53, No (d) 0.47, Yes and picks out one shirt trouser pair without examining the
colour. What is the likelihood that neither the shirt nor the
37. A blind man lives in an apartment containing 2 rooms. trouser is black?
Each day before going to work he enters any one room
(a) 1/12 (b) 1/6
randomly, picks up a bag and leaves home. One of
the rooms contains 3 blue, 4 green and 5 red bags and the (c) 1/4 (d) 1/2
other contains 2 blue, 1 green and 3 red bags. What is the
probability that he takes a green bag to work? 43.
I forgot the last digit of a 7-digit telephone number. If
I randomly dial the final 3 digits after correctly dialling
1 1
(a) (b) the first four, then what is the chance of dialling the
2 4 correct number?
1 1 1
(c) (d) (a)
3 6 1001
1
38. 7 Indians, 4 Americans and 2 Germans are to be seated (b)
on 13 chairs for a photograph. If a photograph is clicked, 990
what is the probability that in the photo no two Indians are 1
together? (d)
999
7!4!2! 7!6! 1
(a) (b) (d)
13! 13! 1000
7! 7
(c) (d)
13! 13 44. There are 6 positive and 8 negative numbers. Four numbers
are chosen at random and multiplied. The probability that
39. A number is selected at random from all the four digit the product is a positive number is:
natural numbers. What is the probability that it is a perfect
500
square? (a)
1001
16 17
(a) (b) 503
2250 2250 (b)
1001
9 17
(c) (d) 505
9000 2500 (c)
1001
40. Two people agree to meet on January 9, 2005 between 101
(d)
6.00 p.m. and 7.00 p.m., with the understanding that each 1001
will wait no longer than 20 minutes for the other. What is
the probability that they will meet? 45. A programmer noted the results of attempting to run
5 7 20 programs. The results showed that 2 programs
(a) (b) ran correctly in the first attempt, 7 ran correctly in the
9 9
second attempt, 5 ran correctly in the third attempt, 4 ran
2 4 correctly in the fourth attempt, 2 ran correctly in the fifth
(c) (d)
9 9 attempt.

Chapter_32.indd 17 6/5/2015 5:23:04 PM


32.18 Chapter 32

What is the probability that his next program will run (a) 0.20 (b) 0.25
correctly on the third run? (c) 0.30 (d) 0.33
1 1 [Based on ATMA, 2006]
(a) (b)
4 3
50.
In a charity show tickets numbered consecutively
1 1
(c) (d) from 101 through 350 are placed in a box. What is the
6 5 probability that a ticket selected at random (blindly) will
have a number with a hundreds digit of 2?
46. Course materials are sent to students by a distance (a) 0.285 (b) 0.40
teaching institution. The probability that they will send a
wrong programme’s study material is one-fifth. 100 99
(c) (d)
There is a probability of three-fourths that the package 249 250
is damaged in transit, and there is a probability of one- [Based on ATMA, 2006]
third that there is a short shipment. What is the probability
that the complete material for the course arrives without 51.
A group of 2n boys and 2n girls is divided at random into
any damage in transit? two equal batches. The probability that each batch will
4 8 have equal number of boys and girls is:
(a) (b)
5 60 1 1
(a) n (b) n
8 4 2 4
(c) (d)
15 20 ( 2 n Cn ) 2
(c) 4n
(d) 2n Cn
C2 n
47.
A set A is containing n elements. A subset P of A is chosen [Based on ATMA, 2008]
at random. The set is reconstructed by replacing the
elements of P. A subset of A is again chosen at random.
The probability that P and Q have no common elements 52. In ∆LMN, LO is the median and the bisector of ÐMLN. If
is: LO = 3 cm and LM = 5 cm, calculate the area of ∆LMN.
n (a) 12 cm2 (b) 10 cm2
3
(a) 5n (b)   (c) 4 cm2 (d) 6 cm2
4
n [Based on CAT, 2009]
3
(c)   (d) 2n
5 53. A garland is to be made from six different flowers and a
large pendant that has two different faces. In how many
48. There are three similar boxes, containing (i) 6 black and 4 ways, can the garland be made?
white balls; (ii) 3 black and 7 white balls and (iii) 5 black (a) 240 (b) 600
and 5 white balls, respectively. If you choose one of the (c) 720 (d) None of these
three boxes at random and from that particular box pick [Based on CAT, 2009]
up a ball at random and find that to be black, what is the
probability that the ball was picked up from the second
box? 54. There are four boxes. Each box contains two balls: one
red and one blue. You draw one ball from each of the four
3  14 
(a)   (b)   boxes. What is the probability of drawing at least one red
 14   30  ball?
1 1
 7  7 (a) (b)
(c)   (d)   2 4
 30   14 
1 15
[Based on JMET, 2009] (c) (d)
16 16
[Based on CAT, 2010]
49.
Set A = {2, 3, 4, 5}
Set B = {4, 5, 6, 7, 8}
55. A shopkeeper received a pack of 15 pens, out of which
Two integers will be randomly selected from the sets 4 were defective. The shopkeeper decided to examine
above, one integer from set A and one integer from set B. every pen one by one selecting a pen at random. The pen
What is the probability that the sum of the two integers examined are not put back. What is the probability that the
will equal 9? 9th one that had been examined is the last defective pen?

Chapter_32.indd 18 6/5/2015 5:23:05 PM


32.19
Probability

11 16 (a) 0.2 (b) 0.8


(a) (b)
195 195 (c) 0.4 (d) 0.7
8 17 [Based on MAT, 2012]
(c) (c)
195 195 60. There are two bags, one of which contains 3 black and 4
[Based on CAT, 2013]
white balls, while the other contains 4 black and 3 white
56. Varun throws two unbiased dice together and gets a sum balls. A dice is cast, if the face 1 or 3 turns up, a ball is
of 7. If his friend Tarun, then throws the same two dice, taken from the first bag and if any other face turns up,
what is the probability that the sum is less than 7? a ball is chosen from the second bag. The probability of
1 7 choosing a black ball is:
(a) (b)
6 12 (a) 10/21 (b) 11/21
(c) 12/21 (d) 9/21
1 5
(c) (d) [Based on MAT, 2012]
2 12
[Based on CAT, 2012]
61. A can hit a target 4 times in 5 shots, B can hit 3 times in
57. In a factory where toys are manufactured, machines A, 4 shots and C can hit twice in 3 shots. When they fire
B and C produce 25%, 35% and 40% of the total toys, together, what is the probability that atleast two shots hit
respectively. Of their output, 5%, 4% and 2% respectively, the target?
are defective toys. If a toy drawn at random is found to be (a) 13/30 (b) 1/3
defective, what is the probability that it is manufactured
(c) 5/6 (d) None of these
on machine B?
[Based on MAT, 2012]
17 28
(a) (b)
69 69 62. In a defective 6 faced die with number 1 to 6 inscribed, the
probability of getting an odd umber is twice the probability
35
(c) (d) None of these of getting an even number. Find the probability of getting
69 a two-digit prime number on adding 2 successive throws
[Based on MAT, 2012]
of the die:
58. A and B alternately throw a pair of dice. A wins if he
throws 6 before B throws 7; and B wins if he throws 7 2 4
(a) (b)
before A throws 6. What are their respective chances of 81 81
winning, if A throws the dice first? 1 1
(c) (d)
13 31 30 31 3 9
(a) , (b) ,
16 16 61 61 [Based on MAT, 2013]

(c)
31 41 38 23
, (d) ,
63. The probability that a randomly chosen positive divisor
61 61 61 61 of 1029 is an integer multiple of 1023 is: a2/b2, then ‘b – a’
[Based on MAT, 2012] would be:
(a) 8 (b) 15
59. A chartered Accountant applies for a job in two firms X (c) 21 (d) 23
and Y. The probability of his being selected in firm X is 0.7 [Based on XAT, 2014]
and being rejected at Y is 0.5 and the probability of atleast
one of his applications being rejected is 0.6. What is the
probability that he will be selected in one of the firns?

Chapter_32.indd 19 6/5/2015 5:23:10 PM


32.20 Chapter 32

Answer Keys
Difficulty Level-1

1. (c) 2. (b) 3. (d) 4. (b) 5. (b) 6. (c) 7. (c) 8. (b) 9. (b) 10. (a) 11. (c) 12. (d) 13. (b)
14. (d ) 15. (c) 16. (a) 17. (a) 18. (a) 19. (c) 20. (b) 21. (a) 22. (d) 23. (a) 24. (b ) 25. (b) 26. (c)
27. (a) 28. (d) 29. (d) 30. (c) 31. (a ) 32. (d) 33. (b) 34. (d) 35. (a ) 36. (c) 37. (d ) 38. (a) 39. (c)
40. (b) 41. (c) 42. (a) 43. (b) 44. (b) 45. (c) 46. (c ) 47. (c) 48. (b) 49. (b) 50. (c) 51. (a) 52. (a)
53. (b) 54. (b ) 55. (a) 56. (a ) 57. (d) 58. (d) 59. (d) 60. (a) 61. (c) 62. (d) 63. (b ) 64. (c) 65. (c)
66. (a) 67. (b) 68. (d) 69. (c) 70. (d) 71. (c) 72. (d) 73. (a) 74. (c) 75. (c) 76. (a ) 77. (c) 78. (b )
79. (a) 80. (c) 81. (c) 82. (b) 83. (d) 84. (b) 85. (c) 86. (a) 87. (a) 88. (a) 89. (d ) 90. (a) 91. (c )
­ 92. (c)

Difficulty Level-2

1. (c) 2. (c) 3. (b) 4. (c) 5. (b) 6. (a) 7. (b) 8. (b) 9. (d) 10. (b) 11. (d) 12. (d) 13. (c)
14. (d ) 15. (c) 16. (a) 17. (b) 18. (d) 19. (c) 20. (b) 21. (a) 22. (b) 23. (a) 24. (a ) 25. (b) 26. (a,b,c)
27. (c) 28. (c) 29. (c) 30. (b) 31. (a ) 32. (b) 33. (b) 34. (d) 35. (a ) 36. (d) 37. (b ) 38. (b) 39. (b)
40. (d) 41. (c) 42. (d) 43. (d) 44. (c) 45. (a) 46. (b ) 47. (a) 48. (a) 49. (a) 50. (b) 51. (c) 52. (a)
53. (c) 54. (d ) 55. (c) 56. (d ) 57. (d) 58. (b) 59. (b) 60. (b) 61. (c) 62. (b) 63. (d )

Explanatory Answers

Difficulty Level-1

1. (c) Total number of events that would occur by flipping 1 1 4


six coins = 26 = 64 4.
(b) P(A) = , P ( A) = 1 – =
5 5 5
1
Probability that no tail occurs = The probability that he will not hit the target in 10
64
10
\ Probability of occuring at least one tail 4
shots is  
1 63 5
==1− . So, probability that atleast once target will be hit
64 64
10
2.
(b) P(A ∩ B) = P(A) × P(B) = 0.15 × P(B) 4
= 1 –   .
P(A ∪ B) = P(A) + P(B) – P(A ∩ B) 5
0.45 = 0.15 + P(B) – (0.15) × P(B) 5.
(b) The event ‘Total score is a prime number when two
= 0.15 + P(B) (1 – 0.15) dice are tossed’ occurs in the following 15 ways:
(1, 1), (1, 2), (1, 4), (1, 6), (2, 1), (2, 3), (2, 5), (3, 2),
= 0.15 + 0.85 P(B)
(3, 4), (4, 1), (4, 3), (5, 2), (5, 6), (6, 1), (6, 5).
\ 0.85 P(B) = 0.45 – 0.15 = 0.30
15 5
0.30 30 6 \ Required probability = = .
\ P(B) = = = . 36 12
0.85 85 17
6.
(c) Let the four places be 1  2  3  4
3. (d) Let x be the number of coins showing heads.
Now object i cannot occupy the place i (A)
\ Prob. when x = 50 = Prob. when x = 51
Suppose Object 2 occupies the place 1. Then other
⇒ 100C50 × p50 × (1 – p)50 = 100C51 × p51 × (1 – p)49 placements can be done in 6 ways as follows:
51 (1) 2 1 3 4
⇒    p = . (2) 2 1 4 3

You might also like